100% found this document useful (4 votes)
2K views1,192 pages

@StudyPivot Plancess Maths 11 PDF

Uploaded by

Jatin Singla
Copyright
© © All Rights Reserved
We take content rights seriously. If you suspect this is your content, claim it here.
Available Formats
Download as PDF, TXT or read online on Scribd
100% found this document useful (4 votes)
2K views1,192 pages

@StudyPivot Plancess Maths 11 PDF

Uploaded by

Jatin Singla
Copyright
© © All Rights Reserved
We take content rights seriously. If you suspect this is your content, claim it here.
Available Formats
Download as PDF, TXT or read online on Scribd
You are on page 1/ 1192

2017-18 100 &

op kers
Class 11 T
By E ran culty
-JE Fa r
IIT enior emie .
S fP r es
o titut
Ins

MATHEMATICS
FOR JEE MAIN & ADVANCED
SECOND
EDITION

Exhaustive Theory
(Now Revised)

Formula Sheet
9000+ Problems
based on latest JEE pattern

2500 + 1000 (New) Problems


of previous 35 years of
AIEEE (JEE Main) and IIT-JEE (JEE Adv)

5000+Illustrations and Solved Examples


Detailed Solutions
of all problems available

Topic Covered Plancess Concepts


Tips & Tricks, Facts, Notes, Misconceptions,
Basic Mathematics Key Take Aways, Problem Solving Tactics

PlancEssential
Questions recommended for revision
1. B A S I C M AT H E M AT I C S

NUMBER SYSTEM

Imaginary
Complex Number
x  0, y  0
(z = x + iy)

Pure Imaginary
x  0, y  0

Pure Real (R) Definition : The union of the


(Real Number) sets of rational numbers and
Definition : Number that can be x  0, y  0 irrational numbers
written as a ratio of two integers

The set of Rational numbers Definition : Real


includes all decimals that have number that are
either a finite number of decimal Rational irrational not rational
places or that repeat in the same Number (Q) Number (Qc)
pattern of digits. e.g. ,2, 3
Fractions
Make up : Includes integers, e.g. 3/4, 0.2
whole number, Natural numbers
and these number are written Integers (I or Z) e.g. -1, 0, 4
as the ratio of two integers.

More eg. 3/5, -1/7 e.g. 1/2 and -3/1

Also known as
counting number

Positive Negative Whole Even Odd


I0
Integer Integer Number (W) Number Number

Natural
Number (N) Zero is a special number, it doesn’t
All the integers quite obey all the same laws as
excluding zero other numbers i.e. you cannot
divide by zero

Prime Composite
Number Number

Figure 1.1: The number system


1 . 2 | Basic Mathematics

1. NUMBER SYSTEM
(a) Natural Numbers: The counting numbers 1, 2, 3, 4, ……. are called natural numbers. The set of natural
numbers is denoted by N.
N = {1, 2, 3, 4, ……..} N is also denoted by I’ or Z’
(b) Whole Numbers: Natural numbers including zero are called whole numbers. The set of whole numbers is
denoted by W.
Thus W = {0, 1, 2, ……}
(c) Integers: The numbers ….. -3, -2, -1, 0, 1, 2, 3 …….. are called integers and the set of integers is denoted by I
or Z.
Thus I (or Z) = {……-3, -2, -1, 0, 1, 2, 3……..}
(a) Set of negative integers is denoted by I¯ and consists of {……., -3, -2, -1}
(b) Set of non-negative integers is denoted by W.
(c) Set of non-positive integers {……., -3, -2, -1, 0}
(d) Even integers: Integers which are divisible by 2 are called even integers. e.g. 0, ±2, ± 4, ………
(e) Odd integers: Integers which are not divisible by 2 are called odd integers. e.g. ±1, ±3, ±5, ±7
(f) Prime numbers: A natural number (except unity) is said to be a prime number if it is exactly divisible by unity
and itself only. e.g. 2, 3, 5, 7, 11, 13, 17, 19, 23, 29, 31, ……
(g) Composite numbers: Natural numbers which are not prime (except unity) are called composite numbers.
(h) Co-prime numbers: Two natural numbers (not necessarily prime) are said to be co-prime, if their H.C.F.
(Highest common factor) is one. e.g. (1, 2), (1, 3), (3, 4), (3, 10), (3, 8), (4, 9), (5, 6), (7, 8) etc. These numbers are
also called as relatively prime numbers.
(i) Twin prime numbers: If the difference between two prime numbers is two, then the numbers are called twin
prime numbers. e.g. {3, 5}, {5, 7}, {11, 13}, {17, 19}, {29, 31}
( j) Rational numbers: All the numbers that can be represented in the form p/q, where p and q are integers and
1 22
q ≠ 0, are called rational numbers and their set is denoted by Q. e.g. , 2, 0, -5, , 2.5, 0.3333 …….. etc. Thus
2 7
p 
Q =  : p,q ∈ I and q ≠ 0  . It may be noted that every integer is a rational number since it can be written as
q 
p/1. The decimal part of rational numbers is either terminating or recurring.
(k) Irrational numbers: There are real numbers which cannot be expressed in p/q form. These numbers are
called irrational numbers and their set is denoted by QC or Q (i.e. complementary set of Q). The decimal part
of irrational numbers is neither terminating nor recurring e.g. 2 , 1 + 3 , π etc.
(l) Real numbers: The complete set of rational and irrational numbers is the set of real numbers and is denoted
by R. Thus R = Q ∪ QC.
(m) Complex numbers: A number of the form a + ib is called a complex number, where a, b ∈ R and i = −1 . A
complex number is usually denoted by ‘z’ and a set of complex numbers is denoted by C.

PLANCESS CONCEPTS

•• Zero is neither positive nor negative but zero is non-negative and non-positive.
•• ‘1’ is neither prime nor composite
•• ‘2’ is the only even prime number
•• ‘4’ is the smallest composite number
M a them a tics | 1.3

•• Two distinct prime numbers are always co-prime but the converse need not be true.
•• Consecutive natural numbers are always co-prime numbers.
e ≈ 2.71 is called Napier’s constant and π ≈ 3.14. And both are irrational.

Vaibhav Gupta (JEE 2009, AIR 22)

2. RATIO AND PROPORTION

2.1 Ratio
(a) If A and B are two quantities of the same kind, then their ratio is A : B; which may be denoted by the fraction
A
(this may be an integer or fraction)
B
a ma na
(b) A ratio may be represented in a number of ways e.g.= = = ……… where m, n, …….. are non-zero
b mb nb
numbers.
(c) To compare two or more ratios, reduce them to their common denominator.

2.2 Proportion
a c
When two ratios are equal, then the four quantities composing them are said to be proportional. If = , then it
is written as a : b = c : d or a : b :: c : d b d

(a) ‘a’ and ‘d’ are known as extremes and ‘b’ and ‘c’ are known as means.
(b) An important property of proportion; product of extremes = product of means.
(c) If a: b = c : d, then b : a = d : c (invertendo)

(d) If a: b = c : d, then a : c = b : d (alternando)


a+b c+d
(e) If a: b = c : d, then = (componendo)
b d
a−b c −d
(f) If a: b = c : d, then = (dividendo)
b d
a+b c+d
(g) If a: b = c : d, then = (componendo and dividendo)
a−b c −d

3. DEFINITION OF INDICES
If ‘a’ is any non-zero real or imaginary number and ‘m’ is a positive integer, then am = a.a.a…...a (m times). Here ‘a’
is called the base and m is the index, power, or exponent.

Law of indices:

(a) a0 =1 , (a ≠ 0)
1
(b) a-m = , (a ≠ 0)
am
(c) am+n = am .an , where m and n are real numbers
am
(d) am–n = , where m and n are real numbers
an
1 . 4 | Basic Mathematics

(v) (am)n = amn


q p
(vi) ap/q = a

4. SOME IMPORTANT IDENTITIES


(a) (a + b)2 = a2 + 2ab + b2 = (a – b)2 + 4ab

(b) (a - b)2 = a2 - 2ab + b2 = (a + b)2 - 4ab

(c) a2 - b2 = (a + b) (a - b)

(d) (a + b)3 = a3 + b3 + 3ab (a + b)

(e) (a – b)3 = a3 – b3 – 3ab (a – b)

(f) a3 + b3 = (a + b)3 -3ab (a + b) = (a + b) (a2 + b2 – ab)

(g) a3 – b3 = (a – b)3 + 3ab (a – b) = (a – b) (a2 + b2 + ab)


1 1 1
(h) (a + b + c)2 = a2 + b2 + c2 + 2ab + 2bc + 2ca = a2 + b2 + c2 + 2abc  + + 
a b c
1
(i) a2 + b2 + c2 – ab – bc – ca = [(a – b)2 + (b – c)2 + (c – a)2]
2
( j) a3 + b3 + c3 – 3abc = (a + b + c) (a2 + b2 + c2 – ab – bc – ca)
1
= (a + b + c) [(a – b)2 + (b – c)2 + (c – a)2]
2
(k) a4 – b4 = (a + b) (a – b) (a2 + b2)
(l) a4 + a2 + 1 = (a2 + 1)2 - a2 = (1 + a + a2) (1 – a + a2)

5. SURDS
3 4 3
Any root of an arithmetical number which cannot be completely found is called surd. E.g. 2, 5, 7 etc. are all
surds.

(a) Pure Surd: A surd which consists of purely an irrational number expressed as n where a ≠ xn (x∈I) is called
3 5
a pure surd. e.g. 7, 5 etc.
3 5
(b) Mixed surd: A pure surd when multiplied with a rational number becomes a mixed surd. e.g. 2 3,4 5,2
3 etc.
3 3 3
A mixed surd can be written as a pure surd. e.g. 2 × 3 = 3× 8 = 24 , 2 5 = 20
4 3 6
(c) Order of Surd: The order of a surd is indicated by the number denoting the roots i.e. 2, 5, 7 are surds
of the 4th, 3rd and 6th order respectively.
5 3 3
(d) Simple Surd: Surds consisting of one term only are called simple surds. E.g. 2, 3, a2bc etc. are simple
surds or Monomial surds.

(e) Compound Surd: An expression consisting of two or more simple surds connected by (+) or (-) sign is called
a compound surd. E.g. 5 2 +4 3, 3 + 2, 3‒ 5.
M a them a tics | 1.5

6 LOGARITHM

6.1 Introduction
( 45.5)
2
2
It is very lengthy and time consuming to find the value of 5 0.0000165 , or finding number of
(3.2) ( 6.5)
2 2

digits in 312, 28. John Napier (1550-1617 AD) invented logarithm (in 1614 AD) to solve such problems. The word
“Logarithm” was formed by two Greek words, ‘logos’ which means ‘ratio’, and ‘arithmos’ meaning ‘number’. Henry
Briggs (1556-1630 AD) introduced common logarithm. He published logarithm in 1624 AD.
In its simplest form, a logarithm answers the question, “How many of one number do we multiply to get another
number?”

Illustration 1: How many 2s do we multiply to get 8?  (JEE MAIN)

Sol: 2 × 2 × 2 = 8, So we needed to multiply 3 of the 2s to get 8. So the logarithm of 8 to the base 2, written as
log2 ( 8 ) is 3.

6.1.1 How to Write it


We would write “the number of 2s you need to multiply to get 8 is 3” as
2
 2 ×2 =
× 8 ↔ log2 ( 8 ) = 3
3

Base

So these two things are the same.


The number we are multiplying is called the “base”, so we would say “The logarithm of 8 with the base 2 is 3”.
Or “log base 2 of 8 is 3” or “the base-2 log of 8 is 3”

6.1.2 Exponents
Exponents and Logarithms are related, let’s find out how…

The exponent says the times


exponent to use the number in a
multiplication.
23
In this example : 23 = 2 x 2 x 2 = 8

base (2 is used 3 times in a multiplication


to get 8)

Figure 1.2

So a logarithm answers a question like this: 2? = 8


In this way

?
exponent
2 = 8
3
2 = 8 log2(8) = 3
log2(8) = 3 base
(a) (b)
Figure 1.3
1 . 6 | Basic Mathematics

So the logarithm answer the question: The general case is


x
a = y

loga(y) = x
Figure 1.4

6.2 Definition of Logrithm


If ax = N, then x is called the logarithm of N to the base a. It is also designated as loga N.
So loga N =x; ax = N, a > 0, a ≠ 1 and N > 0

Note:
(a) The logarithm of a number is unique i.e. No number can have two different log to a given base.
loga N
(b) From the definition of the logarithm of the number to a given base ‘a’. a = N, a > 0, a ≠ 1 and N > 0 is
known as the fundamental logarithmic identity.
(c) The base of log can be any positive number other than 1, but basically two bases are mostly used. They are
10 and e (= 2.718 approx.)
Logarithm of a number to the base 10 are named as common logarithm, whereas the logarithm of numbers to the
base e are called as Natural or Napierian logarithm.

6.2.1 Common Logarithm: Base 10


Many a times, the logarithm is written without a base, like this, log (100)
This usually means that the base is really 10.
It is called “common logarithm”. Engineers love to use it.

6.2.2 Natural Logarithms: Base “e”


Another base that is often used is e (Euler’s Number) which is approximately 2.71828.
This is called a “natural logarithm”. Mathematicians use this one quite often.
But There is Some Confusion …….!
Mathematicians use “log” (instead of “ l n ”) to mean the natural logarithm. This can lead to confusion:

Example Engineer Thinks Mathematician Thinks


log(50) log10(50) loge(50) Confusion

ln (50) loge(50) loge(50) No confusion

log10 (50) log10 (50 log10 (50) No confusion

So make sure that when you read “log” that you know what base they mean.
Note: Since NCERT assumed log x to be loge x, for JEE Main and Advanced this convention is to be used.

6.3 Properties of Logarithm


Let M and N be arbitrary positive numbers such that a > 0, a ≠ 1, b > 0, b ≠ 1 and x, y are real numbers. Then,
(a) loga (M × N) = logaM + logaN (Product rule)
Proof: Let loga M = x and loga N = y
Then from the basic definition of logarithm, M = ax and N = ay
M a them a tics | 1.7

⇒ (M × N) = ax + y ⇒ loga (M × N) = x + y
⇒ loga(M × N) = loga M + logaN

(b) loga (M/N) = logaM – logaN (Division rule)


Proof: Let loga M = x and loga N = y
M = ax and N = ay
⇒ M/N = ax – y ⇒ loga (M/N) = x – y = loga M – loga N

(c) loga Mx = x loga.M (Power rule)


Proof: Let loga Mx = y … (i)
⇒ Mx = ay ⇒ (Mx)1/x = (ay)1/x
⇒ M = ay/x ⇒ logaM = y/x
⇒ x loga M = y  … (ii)
From (i) and (ii), we can say that loga Mx = x logaM

1
(d) log x M = loga M (x ≠ 0) (Power rule for base)
a x
Proof: Let log x M = y … (i)
a

⇒ M = axy ⇒ M1/x = ay ⇒ logaM1/x = logaay

1
logaM = y … (ii)
x
1
Using (i) and (ii) log x M = loga M
a x
logc a log a
(e) logb a = (c > 0, c ≠ 1) = (Base changing theorem)
logc b log b

Proof: Let logc a = x and logc b = y

⇒ a = cx and b = cy ⇒ a1/x = c and b1/y = c ⇒ a1/x = b1/y ⇒ (a1/x)x = (b1/y)x ⇒ a = bx/y


x logc a log a
⇒ logb a = = =
y logc b log b
This is the most important property of logarithms and applies to most of the problems. Here, the base can be taken
as any positive real number except unity.
log2 5 log10 5 log1/2 5
E.g. log3 5 = = =
log2 3 log10 3 log1/2 3

Note: log π and log 3 are reciprocals of each other.


3 𝜋

The following properties can be deduced using base changing theorem.


1 loga a 1
(a) logb a = ; Proof: logb a = =
loga b loga b loga b

log a log b log c log a


(b) logba . logcb . logdc = logda Proof: logba . logcb . log
= c= . . = = logd a
d
log b log c log d log d

loge c loge a log c.log a log c log a loge a loge N


(c) a =c =a e alogeec = (a
Proof: = e ) e c ( a = N)
1
(i) (logba . logab = 1 ⇒ logba = (ii) e xl n a = ax
loga b
1 . 8 | Basic Mathematics

5
Illustration 2: What is logarithm of 32 4 to the base 2 2  (JEE MAIN)
3
4 as 25 41/5 = ( 2 )
5 27/5
Sol: Here we can write 32 and 2 2 as 22 and then by using the formulae logaMx = x loga .M
1
and log x M = loga M we can solve it.
a x
log2 2
325=
4 log
2
3/2 

(2
5
)
41/5= log
2
3/2 

(2)
27/5
=
2 27
3 5
log2=
18
2 = 3.6
5
   

Illustration 3: Prove that, log4/3 (1.3) = 1  (JEE MAIN)

4
Sol: By solving we get 1.3 = , and use the formula logaa = 1 .
3
log4/3 1.3 = 1
Let x = 1.333 … … (i)
10x = 13.3333 … ... (ii)
From Equation (i) and (ii), we get
So 9x = 12 ⇒ x = 12/9, x = 4/3;
Now log4/3 1 / 3 = log4/3 (4/3) = 1

Illustration 4: If N = n! (n ∈ N, n ≥ 2) then lim [(log2N)–1 + (log3N)–1 + … + (logn N)–1] is (JEE ADVANCED)
N→ ∞

1
Sol: Here by using logab = we can write given expansion as logN2 + logN3 + …… + logNn and then by using
logb a
loga (M.N) = logaM + logaN and N = n! we can solve this.

(log2N)–1 + (log3N)–1 + …… + (lognN)–1 = logN2 + logN3 + …… + logNn = logn(2.3 …. N) = logN N = 1.

Illustration 5: If log x2 – log 2x = 3 log 3 –log 6 then x equals  (JEE ADVANCED)

(M.N)
Sol: By using loga = logaM + logaN and logaMx = x loga .M we can easily solve above problem.
Clearly x > 0. Then the given equation can be written as 2 log x – log 2 – log x = 3 log 3 – log 2 – log 3

⇒ log x = 2 log 3 ⇒ x = 9

Illustration 6: Prove that, log2− 3 (2 + 3) = –1 (JEE ADVANCED)

1
Sol: By multiplying and dividing by 2 + 3 to 2 − 3 we will get 2 + 3 = . Therefore by using log1/N N = –1
we can easily prove this. 2 − 3
1
(2 − 3 ) (2 − 3 ) = −1
−1
⇒ log2− 3
⇒ log 2− 3
⇒ −1.log2− 3
2− 3

Illustration 7: Prove that, log5 5 5 5.......∞ = 1 (JEE ADVANCED)

Sol: Here 5 5 5.......∞ can be represented as y = 5y where y = 5 5 5.......∞ . Hence, by obtaining the value
of y we can prove this.

Let y = 5 5 5.......∞
M a them a tics | 1.9

y= 5y ⇒ y2 = 5y or y2 – 5y = 0
y(y – 5) = 0 ⇒ y = 0, y = 5
y = 0 is not possible because log is not defined for zero.
∴ log55 = 1

Illustration 8: Prove that, log2.25 ( 0.4 ) = –1 (JEE MAIN)

Sol: As similar to illustration 3 we can solve it by using log1/N N = –1.

x = 0.4444..... … (i)
10x = 4.4444..... … (ii)
Equ (ii) – Equ (i)
So 9x = 4 ⇒ x = 4/9
225 9 4
Also, 2.25
= = ; ( )
log2.25 0.4 = log 9    = −1
100 4  9
4  

log6 18 log6 3
Illustration 9: Find the value of 2 .3  (JEE MAIN)

(M.N)
Sol: We can solve above problem by using loga = logaM + logaN and a
loge c loge a
=c step by step.

log6 18 log 3 log (6×3) log 3 1 +log 3 log 3 log6 3 log6 3 loge c loge a
2 (3) 6 = 2=
6 .3 6 2=
6 .3 6 2·2 ·3 ( a =c )
log 2 log 3 log6 2 +log6 3 log (6)
2·(3)
= 6 ·(3) 6 2(3) = 2(3) =
6 2·(3)
= 6

(
Illustration 10: Find the value of, logsec α cos3 α where α ∈ (0, π/2) ) (JEE MAIN)

( )
Sol: Consider logsec α cos3 α =x . Therefore by using formula = y
y loga x ⇔ a= x we can write Cos3α = (sec α)x .
Hence by solving this we will get the value of x.
Let logsec α cos3 α =x
x
 1 
Cos α = (sec α) ⇒ (cos α) = 
3 x
 ⇒ (cos α) = (cos α) ⇒ x = –3
3 3 –x

 cos α 

Illustration 11: If k ∈ N, such that log2x + log4x + log8x = logkx and ∀ x ∈ R’  (JEE ADVANCED)
If k = (a)1/b then find the value of a + b; a ∈ N, b ∈ N and b is a prime number.
logc a loga
Sol: By using log
= ba = we can obtain the value of k and then by comparing it to k = (a)1/b we can
logc b logb
obtain value of a + b.

logx logx logx logx logx  1 1 1  logx logx  11  logx  11 1 1 


Given, + + =⇒  + + = ⇒  = ⇒ logx  − =0
log2 2log2 3log2 logk log2  1 2 3  logk log2  6  logk  6 log2 logk 

11 1 1 11 log2 11
Also, − =0 ⇒ = ⇒ =logk 2
6 log2 logk 6 logk 6
11 1 1
So 2 = k 6 ; 26 11 =
k ⇒ 26 ( ) 11 k ⇒ ( 64 )11 =
= k

Comparing by k = (a)1/b ⇒ a = 64 and b = 11 ⇒ a + b = 64 + 11 = 75


1 . 1 0 | Basic Mathematics

6.4 Logarithmic Equation


While solving logarithmic equation, we tend to simplify the equation. Solving the equation after simplification may
give some roots which do not define all the terms in the initial equation. Thus, while solving an equation involving
logarithmic function, we must take care of all the terms involving logarithm.
Let a = log (x) and b = log (x + 2)
In general, a + b = log (x) + log (x + 2) = log [x (x + 2)]
If we take, x = –3, a and b both are not defined, but a + b will be defined.
as a + b = log [(–3) (–3 + 2)] = log (3)
Here, the problem lies in the definition of a and b. a and b is not defined here, so addition of a and b i.e. a + b will
not be defined.
Note: A similar situation might arise while solving logarithmic equations. To avoid or to reject extraneous roots we
have to define the logarithm.

Illustration 12: Solve log48 + log4 (x + 3) – log4 (x –1) = 2  (JEE MAIN)


M
(M.N)
Sol: As we know loga = logaM + logaN , loga   = logaM – logaN and = y
y loga x ⇔ a= x . By using
N
these formulae we can solve the problem above.
log48 + log4 (x + 3) –log4(x – 1) = 2

8 ( x + 3) 8 ( x + 3)
⇒ log4 2⇒
= 42 ⇒ x + 3 = 2x – 2 ⇒ x = 5
=
x −1 x −1
Also for x = 5 all terms of the equation are defined.

Illustration 13: Solve log (–x) = 2 log (x + 1) (JEE MAIN)

Sol: Here it’s given that log (–x) = 2 log (x + 1). Therefore by using the formula logaMx = x loga .M . We can evaluate
the value of x.
By definition, x < 0 and x + 1 > 0 ⇒ –1 < x < 0
Now log (–x) = 2 log (x + 1) ⇒ –x = (x + 1)2 ⇒ x2 + 3x + 1 = 0

−3 + 5 −3 − 5 −3 + 5
⇒x= , (rejected). Hence, x = is the only solution.
2 2 2

Illustration 14: Find the number of solutions to the equation log2 (x + 5) = 6–x.  (JEE MAIN)
y y = 2c
Sol: By using the formula =y loga x ⇔ a= x , we can write given the equation as y=x+5
x + 5 = 2 . Hence, by checking the number of intersections made by the graph of
6–x 10
y = x + 5 and y = 26 –x we will obtain the number of solutions. 5
Here, x + 5 = 2 6–x

-5 5
Now graph of y = x + 5 and y = 26 –x intersects only once. Hence, there is only one
solution.

PLANCESS CONCEPTS

Always check your answer by putting it back in the equation; sometimes answer might not be in the
domain of logarithm.
Shrikant Nagori (JEE 2009, AIR 7)
M a them a ti cs | 1.11

6.5 Graph of Logarithmic Function


y y

(i) x (ii) x
(0, 0) (1, 0) (0, 0) (1, 0)

y = loga x  a > 1 y = loga x  0 < a < 1

Figure 1.6 Figure 1.7

If the number and the base are on the same side of unity, then the logarithm is positive, and if the number and the
base are on different side of unity then the logarithm is negative.

Illustration 15: Which of the following numbers are positive/negative?  (JEE MAIN)
(i) log27 (ii) log1/23 (iii) log1/3 (1/5) (iv) log4 3 (v) log2 9

Sol: By observing whether the Number and Base are on the same side of unity or not we can say whether the
numbers are positive or negative.
(i) Let log27 = x (number and base are on the same side of unity) ⇒ x > 0
(ii) Let log1/23 = x (number and base are on the same side of unity) ⇒ x < 0
(iii) Let log1/3(1/5) = x (number and base are on the same side of unity) ⇒ x > 0
(iv) Let log43 = x (number and base are on the same side of unity) ⇒ x > 0
(v) Let (log29) = x (number and base are on the same side of unity) ⇒ x > 0

6.6 Characteristic and Mantissa


(1) Given a number N, Logarithm can be
(a) log10N
= Integer   +    Fraction
↓ ↓
Characterstic Mantissa

(b) The mantissa part of the log of a number is always kept non-negative, it ranges from [0, 1]
(c) If the characteristic of log10N is C then the number of digits in N is (C + 1)
(d) If the characteristic of log10N is (–C) then there exist (C – 1) number of zeros after decimal point of N.

Illustration 16: Let x = (0.15)20. Find the characteristic and mantissa of the logarithm of x to the base 10. Assume
log10 2 = 0.301 and log10 3 = 0.477. (JEE ADVANCED)

Sol: Simply by applying log on both sides and using various logarithm formulas we can solve the above illustration.
 15   10 
logx = log(0.15)20 = 20 log   = 20[log 15 – 2] = 20[log3 + log5 – 2] = 20[log3 + 1 – log 2 – 2] log10 5 = log10 
 100   2
= 20 [–1 + log3 – log2] = – 20 × 0.824 = – 16.48 = 17.52
Hence, characteristic = – 17 and mantissa = 0.52

Illustration 17: Find the number of digits in the following: (i) 2100 (ii) 310 (JEE ADVANCED)
Sol: By considering x = 2100 and 310 respectively and applying log on both sides we can solve the problems given above.
(i) Let, x = 2100
log10X = log102100 = 100 log10 2 = 100 × 0.3010 = 30.10
1 . 1 2 | Basic Mathematics

Characteristic = 30, Mantissa = 0.10


Number of digits before decimal = C + 1 = 30 + 1 = 31
(ii) Let, X = 310
log10x = 10 log 3 = 10 × 0.4771 = 4.771
C = 4, M = 0.771
Number of digits before decimal = C + 1 = 4 + 1 = 5
Note: Let y = log (N) when 0 < N < 1
If N lies between 0 and 1, then the characteristic is negative
N = 1/10, log10N = log10 (1/10) = –1, C = –1, M = 0
N = 0.01, log11N = log10(10)–2 = – 2, C = –2, M = 0
N = 0.001, log10N = log10 10–3 = –3, C = –3, M = 0
No. of zeros after decimal = |–3| –1 = 2
N = 0.002, log10N = log (2 × 10–3) = log2 + log10–3 = 0.03010 + (–3) = – 0.3010 = – 2.699
C = – 3, M = 0.3010
Number of zeros after decimal = magnitude of the characteristic –1 = |C| –1 = |–3| –1 = 2

Illustration 18: Find the number of zeros after decimal before a significant figure in
(i) 3–50 (ii) 2–100 (iii) 7–100 (JEE ADVANCED)
Sol: Similar to the illustration above, we can solve these too.

(i) N = 3–50
log10 N = log10 3–50 = - 50 log103 = – 50 × (0.4771) ⇒ log10 N = – 23.855
Now to find the characteristic and mantissa many would say that (c = –23, m = –0.855) (which is wrong) because
mantissa is always non-negative.
log10N = – 23.855 = –23 –1 + 1 – 0.855 = – 24 + 0.145
C = –24, M = 0.145. Number of zeroes after decimal = |–24| –1 = 23 or |–24 + 1| = 23

(ii) N = 2–100
log10 N = – 100 log2 = –30.10 = –30 –0.10 = –31 + 0.90. Number of zeroes after decimal = |–31| –1 = 30 or |–31 + C| = 30

(iii) N = 7–100
log10N = –100 log 7 = –100 × 0.8451 = –84.51 = – 84–1 + 0.49 = – 85 + 0.49
C = –85, M = 0.49. Number of zeroes after decimal = |–85| –1 = 84 or |–85 + 1| = 84

Illustration 19: Find the number of positive integers which have the characteristic 2, when base of log is 6.
 (JEE ADVANCED)
Sol: If any number x has the characteristic a, when base of log is b then x = b . By using the given condition we can
a

solve the problem above.


x = 62 = 36 ; log6x = log662 = 2 log66 = 2
The smallest natural number which has characteristic 3 with base 6 is 63
x = 63 = 216 ; log6x = log663 = 3
Hence x = 215 will give characteristic 2.
M a them a ti cs | 1.13

Natural numbers ranging from 36 to 215 will give characteristic 2, when taken log with base 6.
Number of positive integers = 215 – 35 = 180

6.7 Algebraic Inequalities


(a) If a < b and b < c ⇒ a < c
a c
(b) If < ⇒ ad < bc, if b and d are of same sign. ⇒ ad > bc if b and d are of opposite sign.
b d
(c) If a > b then, aλ > bλ if λ > 0; aλ < bλ if λ < 0

6.8 Logarithmic Inequalities


If the base is less than one, then the inequality will change. If base is greater than one, then inequality will remain
the same.
loga x < α ⇒ 0 < x < aα 
 if a > 1
loga x < loga y ⇒ 0 < x < y 

loga x < α ⇒ x > aα


 if 0 < a < 1
loga x < loga y ⇒ x > y > 0 

x2 −2x
Illustration 20: Solve (1 / 2 ) <1/4 (JEE MAIN)
x2 −2x
Sol: Here we can write the given equation as (1 / 2 ) < (1 / 2 ) and then by comparing powers on both side we
2

can solve this.


x2 −2x
We have (1 / 2 ) < (1 / 2 ) . It means x2 –2x > 2
2

( (
⇒ x− 1+ 3 ) ) ( x − (1 − 3 ) ) > 0 ⇒ x > 1 + 3 or x < 1 – 3 ⇒ x ∈ (–∞, 1 – 3 ) ∪ (1+ 3 , ∞)

1 + 5x
Illustration 21: Solve ≥ 0 . (JEE MAIN)
7− x + 97

( )
Sol: Simply by multiplying 7− x − 72 on both sides and solving we will get the result. - + -
x
g(x) =
1 −5
7− x − 7
( )( )
≤ 0 . Now 1 − 5x 7− x − 7 ≤ 0 ; 5x – 1 = 0 ⇒ x = 0; 7–x –7 = 0 ⇒ x = –1 
-1 0

Figure 1.8
g(x) behavior on the number line. Hence, from above, x ∈ (–∞, -1) ∪ [0, ∞)

6.9 Modulus Function


Definition: Modulus of a number. The modulus of a number is denoted by |a|
y
a, if a ≥ 0  2
|a| =   Also, a = | a | ; Eg: y = |x|
x-

−a, if a < 0 
x

x
Basic properties of modulus O
Figure 1.9
(A) |ab| = |a| |b|

a |a|
(B) = where b ≠ 0
b |b|
1 . 1 4 | Basic Mathematics

(C) |a + b| ≤ |a| + |b|

(D) |a – b| ≥ |a| – |b| equality holds if ab ≥ 0

Using triangle inequality


If a > 0
(i) |x| = a ⇒ x = ± a
(ii) |x| = –a ⇒ No solution
(iii) |x| > a ⇒ x < – a or x > a
(iv) |x| < a ⇒ – a < x < a
(v) |x| > –a ⇒ x ∈ R
(vi) |x| < – a ⇒ No solution
(vii) a < |x| < b ⇒ x ∈ (–b, –a) ∪ (a, b) where a, b ∈ R +

Illustration 22: Solve for x, |x –2| = 3 (JEE MAIN)

Sol: The above illustration can be solved by taking two cases; the first one is by taking x – 2 as greater than 0 and
second one is by taking x - 2.
Case-I: When x – 2 ≥ 0 ⇒ x ≥ 2 ... (i)
Since x – 2 is non negative, the modulus can simply be removed. x – 2 = 3; x = 5
We had taken x ≥ 2 and we got x = 5 hence this result satisfy the initial condition ⇒ x = 5

Case-II: When x – 2 < 0 ⇒ x < 2; Since x – 2 is negative, the modulus will open with a –ve sign.
-(x – 2) = 3; -x + 2 = 3 ⇒ x = – 1 Since x < 2 Hence x = –1, 5

Illustration 23: Solve for x, |x + 3| + |x –2| = 11 (JEE ADVANCED)

Sol: As x + 3 = 0 ⇒ x = –3 and x – 2 = 0 ⇒ x = 2. Therefore  III II I


we can solve it by using the modulus inequality. -3 2
Case-I: For x ≥ 2, x + 3 > 0, x –2 > 0 ; x + 3 + x – 2 = 11 ⇒ 2x = 10 ⇒ x = 5 Figure 1.10

Case-II: For –3 ≤ x < 2, x + 3 ≥ 0, x –2 < 0 ; |x + 3| + |x – 2| = 11


⇒ x + 3 – x + 2 = 11 ⇒ 5 = 11 is impossible ⇒ Hence, No value of x

Case-III: For x < –3


x + 3 < 0, x – 2 < 0 ; |x + 3| + |x – 2| = 11 ⇒ – (x + 3) – (x –2) = 11
⇒ -x – 3 – x + 2 = 11 ⇒ – 2x = 12 ⇒ x = – 6, since x < –3
Hence, to satisfy the initial condition, combining all we get x = –6, 5

Illustration 24: Solve for x, x |x| = 4  (JEE MAIN)


Sol: Here we can solve this problem by using two case, first one for x > 0 and the other one is for x < 0.
Case-I: For x > 0 ; x.x = 4
x2 = 4 ⇒ x = ± 2 but x > 0, hence x = 2 (–2 rejected)
Case-II: For x < 0 ; x(–x) = 4
x2 = – 4 no solution ; Hence, the only solution is x = 2
M a them a ti cs | 1.15

Illustration 25: Solve for x, |x –3| + 2 |x + 1| = 4 (JEE ADVANCED)

Sol: As x –3 = 0 ⇒ x = 3 ; and x + 1 = 0 ⇒ x = –1. 


Therefore by applying the cases X ≥ 3, –1 ≤ x < 3 and x < –1 we can solve this.  III II I

Mark the points on number line -1 3

Case-I: For x ≥ 3 Figure 1.11

(x – 3) is non-negative (x + 1) is also positive


⇒ (x – 3) + 2 (x + 1) = 4 ⇒ 3x = 5 ⇒ x = 5/3 ⇒ x = 5/3 is discarded, since x should be > 3
Case-II: For –1 ≤ x < 3 ; x – 3 is –ve, x + 1 is positive
⇒ -(x – 3) + 2 (x + 1) = 4 ⇒ –x + 3 + 2x + 2 = 4 ⇒ x = –1 satisfies the initial condition
Case-III: x < –1
⇒ –(x – 3) –2(x + 1) = 4 ; –3x = 3 ⇒ x = –1 ⇒ Does not satisfy x < 1 Hence, solution is x = – 1 from case-II.

6.10 Exponential and Logarithm Series

6.10.1 The Number ‘e’


1 1 1
The sum of the series 1 + + + …………………… + ∞ is denoted by the number e
1! 2! 3!
n
 1
i.e. e = lim  1 + 
n→∞  n

(i) The number e lies between 2 and 3. Approximate value of e = 2.718281828.


(ii) e is an irrational number.

6.10.2 Some Standard Deduction from Exponential Series

x x 2 x3 xn
((i)i) e=x 1+ + +
1! 2! 3!
+ ....... + .......∞
n!

( −1) xn + .......∞
n
x x 2 x3
(ii) e = 1 –
–x
+ − + ....... (Replace x by –x)
1! 2! 3! n!

1 1 1
(iii) e = 1 + + + + .......∞ (Substituting x = 1 in (i))
1! 2! 3!

1 1 1
(iv) e–1 = 1 – + − + .......∞ (Substituting x = – 1 in (i))
1! 2! 3!

ex + e− x x2 x 4 x6
(v) =1 + + + + .......∞
2 2! 4! 6!

ex − e− x x3 x5
(vi) =x + + + .......∞
2 3! 5!

x2 x3
(vii) ax = 1+ x (ln a) + (ln a)2 + (ln a)3 + ………………; (a > 0) , where ln a = loge(a)
2! 3!
1 . 1 6 | Basic Mathematics

6.10.3 Logarithmic Series


If –1 < x ≤ 1
x 2 x3 x 4
(i) ln (1 + x) = x – + − + ......∞
2 3 4

x 2 x3 x 4
(ii) ln (1 – x) = – x – − − + ......∞
2 3 4
1 + x   x3 x5 
(iii) ln (x + 1) – ln (1 – x) = ln   = 2  x + 3 + 5 + ..... 
1 − x   
 x2 x 4 x6 
(iv) ln (1 + x) + ln (1 – x) = ln (1 – x2) = – 2  + + + ...... 
 2 4 6 
 

6.11 Antilogarithm
The positive number n is called the antilogarithm of a number m if m = log n. If n is the antilogarithm of m, we
write n = antilog m. For example
(i) log (100) = 2 ⇒ antilog 2 = 100
(ii) log (431.5) = 2.6350 ⇒ antilog (2.6350) = 431.5
(iii) log (0.1257) = 1.0993 ⇒ antilog ( 1.0993 ) = 0.1257

6.12 To find the Antilog of a Number


Step I: Determine whether the decimal part of the given number is positive or negative. If it is negative make it
positive by adding 1 to the decimal part and by subtracting 1 from the integral part. For, example, in – 2.5983
-2.5983 = – 2 – 0.5983 = – 2 – 1 + 1 – 0.5983 = – 3 + 0.4017 = 3.4017
Step II: In the antilogarithm, look into the row containing the first two digits in the decimal part of the given
number.
Step III: In the row obtained in step II, look at the number in the column headed by the third digit in the decimal
part.
Step IV: In the row chosen in step III, move in the column of mean differences and look at the number in the
column headed by the fourth digit in the decimal part. Add this number obtained in step III.
Step V: Obtain the integral part (characteristic) of the given number.
If the characteristic is positive and is equal to n, then insert decimal point after (n + 1) digits in the number obtained
in step IV.

Illustration 26: Find the antilogarithm of each of the following: (JEE MAIN)
(i) 2.7523 (ii) 0.7523 (iii) 2.7523 (iv) 3.7523

Sol: By using log table and following the above mentioned steps we can find the algorithms of above values.
(i) The mantissa of 2.7523 is positive and is equal to 0.7523.
Now, look into the row starting 0.75. In this row, look at the number in the column headed by 2. The number
is 5649. Now in the same row move in the column of mean differences and look at the number in the column
headed by 3. The number there is 4. Add this number to 5649 to get 5653. The characteristic is 2. So, the
decimal point is put after 3 digits to get 565.3
(ii) Proceeding as above, we have antilog (0.7523) = 5.653.
M a them a ti cs | 1.17

(iii) In this case, the characteristic is 2 , i.e., – 2. So, we write one zero on the digit side of the decimal point. Hence,
antilog ( 2 .7523) = 0.05653
(iv) Proceeding as above, antilog ( 3 .7523) = 0.005653

PROBLEM-SOLVING TACTICS
(a) The main thing to remember about surds and working them out is that it is about manipulation. Changing
and manipulating the equation so that you get the desired result. Rationalizing the denominator is all about
manipulating the algebra expression.
(b) Strategy for Solving Equations containing Logarithmic and Non-Logarithmic Expressions:
(i) Collect all logarithmic expressions on one side of the equation and all constants on the other side.
(ii) Use the Rules of Logarithms to rewrite the logarithmic expressions as the logarithm of a single quantity
with coefficient of 1.
(iii) Rewrite the logarithmic equation as an equivalent exponential equation.
(iv) Solve for the variable.
(v) Check each solution in the original equation, rejecting apparent solutions that produce any logarithm of
a negative number or the logarithm of 0. Usually, a visual check suffices!

Note: The logarithm of 0 is undefined

(c) Logarithmic series

x 2 x3 x 4
(i) ln (1 + x) = x – + − + ......∞
2 3 4

x 2 x3 x 4
(ii) ln (1 – x) = – x – − − + ......∞
2 3 4

1 + x   x3 x5 
(iii) ln (x + 1) – ln (1 – x) = ln   = 2  x + + + ...... 
1 − x  3 5 
 

 x2 x 4 x6 
(iv) ln (1 + x) + ln (1 – x) = ln (1 – x2) = – 2  + + + ...... 
 2 4 6 
 

FORMULAE SHEET
(a) Laws of indices

1
(i) a0 =1 , (a ≠ 0) (ii) a-m = , (a ≠ 0)
am

(iii) a
m+n
= am .an , where m and n are real numbers am
(iv) am–n =
an
1 . 1 8 | Basic Mathematics

( )
n q
(v) a
m
= amn (vi) a q
= ap

n
a an
( )
n
(vii) ab = anbn (viii)   =
b bn

(b) Some Important Identities

(i) (a + b)2 = a2 + 2ab + b2 = (a – b)2 + 4ab

(ii) (a - b)2 = a2 - 2ab + b2 = (a + b)2 - 4ab

(iii) a2 - b2 = (a + b) (a - b)

(iv) (a + b)3 = a3 + b3 + 3ab (a + b)

(v) (a – b)3 = a3 – b3 – 3ab (a – b)

(vi) a3 + b3 = (a + b)3 -3ab (a + b) = (a + b) (a2 + b2 – ab)

(vii) a3 – b3 = (a – b)3 + 3ab (a – b) = (a – b) (a2 + b2 + ab)

1 1 1
(viii) (a + b + c)2 = a2 + b2 + c2 + 2ab + 2bc + 2ca = a2 + b2 + c2 + 2abc  + + 
a b c

1
(ix) a2 + b2 + c2 – ab – bc – ca = [(a – b)2 + (b – c)2 + (c – a)2]
2

(x) a3 + b3 + c3 – 3abc = (a + b + c) (a2 + b2 + c2 – ab – bc – ca)

(xi) a4 – b4 = (a + b) (a – b) (a2 + b2)

(xii) a4 + a2 + 1 = (a2 + 1)2 - a2 = (1 + a + a2) (1 – a + a2)

(c) Laws of Surds

1
n
(i) a = an (ii) n
ab
= n
a×nb

a na
( a)
n
n
(iii) n = (iv) =a
b nb

( a)
m
(v) 
 m n a  = mn a (vi) n
= am
n

 

(d) Logarithm formulas

M
(i) loga =
M.N ( ) logaM + logaN (ii) loga   = logaM – logaN
N

y
(
(iii) y= loga x ⇔ a = x a, x > 0,a ≠ 1 ) (iv) logaMx = x loga .M
M a them a ti cs | 1.19

1 logc a loga
(v) log x M
=
a x
loga M ( x ≠ 0) (vi) logb a= =
logc b logb
( c > 0, c ≠ 1)
1 (viii) logb a . logcb . logdc = logda
(vii) logab =
logb a

loge c loge a
(ix) a =c (x) logb a . logab = 1

(xi) exln a = ax (xii) loga1 = 0 and logaa = 1

(e) Exponential series

( −1) xn + ......∞
n
x x 2 x3 xn x x 2 x3
(i) ex =1 +
(i) + + + ...... + + ......∞ (ii) e
(ii)
−x
=1 − + − + ...... +
1! 2! 3! n! 1! 2! 3! n!
1 1 1 1 1 1
(iii) e = 1 + + + + .......∞ (iv) e–1 = 1 – + − + .......∞
1! 2! 3! 1! 2! 3!

ex + e− x x2 x 4 x6 ex − e− x x3 x5
(v) =1 + + + + .......∞ (vi) =x + + + .......∞
2 2! 4! 6! 2 3! 5!

x2
(vii) ax =1+x (ln a)+ (ln a)2 +.. (a > 0)
2!

Solved Examples

JEE Main/Boards Example 2: Using logarithm, find the value of


6.45 × 981.4
3
Example 1: Evaluate 72.3 , if log10 72.3 = 1.8591 Sol: Consider x = 6.45 × 981.4 and then apply log on both
sides and solve by using loga = (M.N) logaM + logaN
Sol: Here consider x = 3 72.3 . Now by applying log on and log table.
both sides and solving using logarithm formula we will Then, log10 x = log10 (6.45 × 981.4)
3
get value of 72.3 . = log10 6.45 + log10 981.4

Let x = 3
72.3 , Then, log x = log (72.3)1/3 = 0.8096 + 2.9919 (using log table)

1 ∴ x = antilog (3.8015) = 6331 (using antilog table)


⇒ log10 x = log10 72.3 ... (i)
3

(
log 72.3 log 0.723 × 102
= ) Example 3: Find minimum value of x satisfying
|x –3| + 2 |x + 1| = 4
2
=⇒ log 0.723 + log 10
= 1.8591 =
+ 2 1.8591  ... (ii)
1 Sol: Similar to illustration 25.
⇒ log10 x = × 1.8591
3 Case-I: When x < –1
⇒ log10 x = 0.6197 ; ⇒ x=antilog (0.6197)
–1 (x –3) –2 (x + 1) = 4
⇒ x = 4.166 (using antilog table)
⇒ – x + 3 – 2x – 2 = 4 ; ⇒ –3x + 1 = 4
⇒ 3x = –3 ; ⇒ x = – 1
1 . 2 0 | Basic Mathematics

 x < – 1 so, x = –1 is not possible log10 x =12log35 − log10 100 =12


log10 7 + log10 5 − 2 =12
Case-II: When –1 ≤ x < 3
log10 7 + log10 10 − log10 2 − 2
⇒ – (x – 3) + 2 (x + 1) = 4 ⇒ – x + 3 + 2x + 2 = 4

⇒ x +5 =4 ⇒ x =− 1 ; So, x = - 1 is a solution. = 12[.8451 + 1 – .3010 –2] = 12 [.5441 –1]

Case-III: When x ≥ 3 is taken, (x – 3) +2 (x + 1) = 4 log10x = – 12 + 6.5292

⇒ 3x –1 = 4 ⇒ x = 5/3 ⇒ Therefore, no solution log10 x =


−12 + 6 + 0.5292 =
−6 + 0.5292 =
6.5292

Result x = –1 is the only solution. So x = 10−6.10−5212


Hence the number of zeros after the decimal = 5
Example 4: Let
log3 N = α1 + β1 ,log5 N = α2 + β2 ,log7 N = α3 + β3 Example 6: Find the number of zeros in, 2−40
where α1 , α2 , α3 ∈ I and β1 , β2 , β3 ∈ [0,1) then
Sol: Consider 2−40 = x and solve as in illustration 5.
(i) Find number of integral values of N if α1 = 4 and α1 = 2
1
(ii) Find the largest integral value of N if α1 = 5, α2 = 3, x= = 2−40
240
α3 = 2
log10 x =−40log10 2 =
−40 .3010  =
−12.0400
(iii) Difference of largest and smallest integral values
( 12 − 0.04 ) + 1 − 1 ⇒ log10 x =−13 + 0.96
log10 x =−
Sol: Here by using = y loga x ⇔ a= x we can obtain
y
10−13.100.96
⇒x=
values of N. After that by drawing a number line we will
get the required answer.  Number of zeros = 12
25 81 125 243
4 +β1 2 +β
=(i) N 3= and N 5 2
4 5 2 3
x 312 × 28
Example 7: Find the number of digits for=
=N [3=
,3 ) and N [5 ,5 ]
Sol: By applying log10 on both sides and then using a
N = [81, 243) and N = [25, 125)
log table we can solve the problem above.
So [81, 125] is the common part hence the no. of integral
values of N are 125 – 81 = 44 log10 x 12log10 3 + 8log10 2
=
5 +β1
(i) N 3=
= ,N 35 =
 )
,36 ,N 243,729 ) ( 0.4771) + 8 ( 0.3010 ) =
5.7252 + 2.4080

=
3+β2
N 5= ,N 53=
 )
,54 ,N 125,625 ) log=
10 x 8.1332 ⇒
= x (10 )108 0.1332

= ,73 ],N  49,343)


2 +β
3 ,N [72=
N 7= No. of digits = 8 + 1 = 9

x(
log x −2 )
49 125 243 343 625 729 Example 8: Solve x =9
1
Sol: Here, by using log x M = loga M we can solve the
problem above.
a x

x(
log x −2 )
Common part is [243, 343]. So largest integral value = 342 x = 9 ⇒ x2logx (x-2) = 9
2
logx ( x −2 )
(b) Difference of largest and smallest values ⇒ x = 9 where x > 0, x ≠ 1
= 342 – 243 = 99
⇒ ( x − 2) =
2
9 ; ⇒ x − 2 =±3
⇒ x = – 1, x = 5
Example 5: Find the number of zeros in, x = (0.35)12,
Given log10(7) = 0.8451, log10(2) = 0.3010 But x = –1 is rejected as x should be greater than 0.

Sol: By applying log10 on both sides and using logarithm


formulae we will get the result.
2
Example 9: log3 log1/2 (
x − 3 log1/2 x + 5 =
2 )
 35 
log10 x = 12log10  
 100 
2
Sol: log3 log1/2 (
x − 3 log1/2 x + 5 =
2 )
M a them a ti cs | 1.21

2
⇒ log1/2 x − 3log1/2 x + 5 =9 ; Hence substitute it in the above equation and solve
using the logarithm formula.
Let log1/2 ( x ) = t ⇒ t2 − 3t − 4 =
0
Given that
(t – 4) (t + 1) = 0 ⇒ t = 4, t = –1 ( )
log3x + 7 9 + 12x + 3x2 + log2x +3 6x2 + 23x + 21 =
4 ( )
⇒ log1/2 x =
4,log1/2 x =
−1
log3x + 7 ( 2x + 3) + log2x +3 ( 2x + 3)( 3x + 7 )  =
2
4 Let
x= 1/16, x = 2
1
log3x + 7 ( 2x + 3) =
A ; 2A + 1 + =4
( )
2
1 − 2 log10 x2 A
Example 10: Solve =1
⇒ 2A2 − 3A + 1 =0 ; 2A2 –2A –A +1 = 0
log10 x − 2 (log10 x )
2

⇒ 2A (A –1) –1 (A –1) = 0; A = 1/2, A = 1


Sol: Simply by putting log10 x = t we can solve the
problem above. ⇒ log3x + 7 ( 2x + 3) =
1/2

( )
2
1 − 2 log10 x2 1
For A= ,2x + 3= 3x + 7
= 1 , Let log10 x = t 2
log10 x − 2 (log10 x )
2

⇒ 4x2 + 9 + 12x = 3x + 7 ; 4x2 + 9x + 2 =0


1 − 2 ( 2t )
2

⇒ = 1 ⇒ 1 − 8t2 = t − 2t2 ⇒ 4x2 + 8x + x + 2 =0 ⇒ 4x ( x + 2 ) + 1 ( x + 2 ) =


0
2
t − 2t
−1
⇒ 6t2 + 3t − 2t − 1 = 0 ⇒ 3t ( 2t + 1 ) − 1 ( 2t + 1 ) = 0 ⇒x = , x = −2=
; For A 1,log3x=
+ 7 2x + 3 1
4
1 ⇒ 2x + 3 = 3x + 7
t = 1/3, t = –1/2 ⇒ log x = 1/3, log x = –
2 ⇒ x = –4 also 2x + 3 > 0, 3x + 7 > 0
1/3 −1/2
=x 10
= , x 10 ⇒ x > –3/2, x > – 7/3
−1
log2 x −log10 x ⇒ x= (-4 and -2 will be rejected)
1 10 1 log10 x −1 4
Example 11   = .5
5 125
3
(log2 x )2 + log2 x − 54 
 
am Example 2: Solve, ( x ) 4   = 2
Sol: By using am–n = we can evaluate the problem
above. an
Sol: By taking logx on both sides and solving we will
log2 x −log10 x
 1  10 (log x −1 ) −3 log x −log2 x
get the result.
  = 5 10 ∴5 5log x − 4
=
5
  Taking log on both sides to the base x
⇒ logx − log2 x = logx − 4 ⇒ log2 x =
4 3
(log2 x )2 +(log2 x ) − 54
logx ( x )  4 = logx ( 2)
x = 102, x = 10–2
3 5 1
( log2 x ) + (log2 x ) − =logx 2
2

4 4 2
JEE Advanced/Boards 3 2 5 1
Let log2 x = t ; t +t− =
4 4 2t
Example 1: Solve,
3t3 + 4t2 − 5t =2 ⇒ 3t3 + 4t2 − 5t − 2 =0
(
log3x + 7 9 + 12x + 4x 2
) + log (6x 2
+ 23x + 21 =
4 )
( )
2x +3
⇒ 3t3 + 3t2 − 6t + t2 + t − 2 =0 ⇒ ( 3t + 1 ) t2 + t − 2 =0
2
Sol: Here 6x + 23x + 21 1
⇒ ( 3t + 1 )( t + 2 )( t − 1 ) =
0 ; ⇒ t = 1, −2, −
(
( 2x + 3) (3x + 7 ) and 9 + 12x + 4x2 = ( 2x + 3) .
= ) 2

Putting t = log2 x
3
1 . 2 2 | Basic Mathematics

1 4 ± 16 − 20
log2 x = 1 ⇒ x = 2 ; log2 x =−2 ⇒ x = =⇒x ,x ∉R
4 2
1 / (2)
1/3
log2 x =−1 / 3 ⇒ x = which is not possible

log3 x2 −2logx 9 −6
Case-II: (4 – x) < 0 or x > 4 then (x + 1) = (x – 1) (x – 4)
Example 3: Solve x − 1 ( x − 1)
=
⇒ x + 1 = x2 − 5x + 4 ⇒ x2 − 6x + 3 =0
Sol: As ax is defined for a > 0 so (x –1) > 0. Therefore by
taking log on both side we can solve it. 6 ± 24
⇒ x2 − 6x + 3 =0 ⇒x=
Now taking log on both sides 2
6±2 6
(log x
3
2
)
− 2logx 9 log ( x − 1=
) log ( x − 1) ⇒x=
2
⇒ x =3± 6

   x > 4 is taken, hence x = 3 + 6


 2log x − 2 − 1  log ( x − 1 ) =
0
2
 log 2 x 
 3 
Example 5: If the sum of all solutions of the equation
Either log (x – 1) = 0 ⇒ x = 2
( )
2
0 is ( a)
 x log10 3  − 3log12 x − 2 = log b
( )
where b and c
Let log3x = t 
(2t – 4/t – 7) = 0 ⇒ 2t2 – 4 – 7t = 0 arerelatively prime and a, b c ∈ N then (a + b + c) = ?

⇒ 2t2 – 8t + t – 4 = 0 ⇒ 2t(t – 4) + 1 (t – 4) = 0 log10 x


Sol: Here by putting 3 = t and solving we will get
t = 4, t = –1/2 the result.

( )
2
log3 x = 4 or log3 x = −1 / 2  3 log10 x  − mlog10 x − 2 =
( )  0
 
(=
3 ) or x ( 3 )
4 −1/2
=x
log10 x
Let e 3 = t then
x = 81, x = 1/ 3
⇒ t − t − 2 =0 ; ⇒ t2 − 2t + t − 2 =
2
0
1
For x = log (x – 1) is not defined, so x = 2 or x = 81.
3 ⇒ t ( t − 2) + 1 ( t − 2) =
0 ⇒ ( t + 1 )( t − 2 ) =
0
;
log10 x
Example 4: Solve, Case-I:
⇒ Case −1 ; ⇒ t =−1 & t =2 ⇒ 3
−I: t = −1
=
Exponential value cannot be negative
( )
log4 x2 − 1 − log4 ( x −
= 1) log4
2
(4 − x)
2

Case-II: t = 2; 3
log10 x
=2
M Taking log3 both side
Sol: By using formula loga   = logaM – logaN and
N
log3 ( 3 )
log10 x log3 2
using modulus inequalities we can solve the problem = log3 2 ⇒ log10=
x log3 2 ⇒=
x 10
above.
Comparing by ( a)
logb c
we get
log4
(x 2
−1 ) =log 4 − x  x2 =x a = 10 , b = 3, c = 2
4
 
( x − 1)
2

∴ a + b + c = 10 + 3 + 2 = 15

⇒ log4
( x − 1)( x + 1) =
log4 4 − x
( x − 1)
2
Example 6: Find the number of zeros after decimal
before a significant digit in ( 9 / 8 )
−100
.
So we have
( x + 1=) 4−x −100
( x − 1) 9
Sol: By putting x =   and applying log10 on both
8
or (x + 1) = (x – 1) |4 – x| side we will get the result.
−100
Case-I: 4 – x > 0 or x < 4 then (x + 1) = (x – 1) (4 – x) 9
Let x =  
⇒ x + 1 = 4x − x2 − 4 + x ⇒ x2 − 4x + 5 = 0 8
M a them a ti cs | 1.23

⇒ log10 x =
−100 log10 9 − log10 8  Example 9: Solve, log ( x / 4 ) = 15
2
x
log2 − 1
⇒ log10 x =
−100 2log30 3 − 3log10 2 8

⇒ log10 x =−100 ( 2 × 0.4771 − 3 × 0.3010 ) Sol: Simply by putting log2 (x) = t and using basic
logarithmic formula we can solve the problem above.
−100 0.9542 − 0.9030  =
= −100 0.0512 =
−5.12
15 15
log30 x = ( −5 − 0.12 ) + 1 − 1 log2 ( x / 4 ) = ⇒ (log2 x − 2 ) =
x
log2 − 1
( 2 − 3) − 1
log x
log10 x = 6.88 ⇒ x = 10-6 ×100.88 8
Let log2 (x) = t
∴ Number of zeros before any significant digits = 5
15
⇒ t−2 = ⇒ t2 - 6t +8 = 15
( (
Example 7: Solve log4 2log3 1 + log2 (1 + 3log2 x ) =
1/2 )) t−4

y
⇒ t2 − 6t − 7 =0 ⇒ (t –7) (t + 1) = 0
Sol: Here by using =
y loga x ⇔ a= x we can solve it.
⇒ t = 7, t = – 1 ⇒ log2x = 7 and log2x = -1
( (
log4 2log3 1 + log3 ( 3log3 x ) )) = 1/2
⇒ x = 27 and x = 2–1
⇒ 2log3 (1 + log2 (1 + 3log3 x ) ) = 2

⇒ log3 (1 + log2 (1 + 3log3 x ) ) = 1 Example 10: Solve, ( )


log2 2x2 log4 (16x ) = log4 x3

⇒ 1 + log2 (1 + 3log3 x ) = 3 ⇒ log2 (1 + 3log 3 x ) = 2


(M.N)
Sol: By using loga = logaM + logaN we can
⇒ 1 + 3log3 x = 4 ⇒ 3log3 x = 3 ⇒ log3 x = 1 ⇒ x =
3 reduce the given

(1 + 2log2 x )  2 + 12 log2 x  =
3
 
equation to log2 x .
Example 8: Solve log0.5x x − 7log16x x3 + 40log4x 4 x =
0  2

loga After that putting log2 x = t we will get the result.


Sol: By using logb a = we can reduce the given
logb

equation to
log2 x
− +
7log2 x3 40log2 4 x
0 and
=
( )
log2 2x2 log4 (16x ) = log4 x3

then log2 0.5x log2 16x log2 4x


(1 + 2log2 x )  2 + 12 log2 x  =
3
 
⇒ log2 x
by putting log2 x = t we can solve it.  2 
Let log2 x = t Let log2 x = t
t 7 ( 3t ) 10t t 21 t 10 t
(1 + 2t )  2 + 2t  =
3
⇒ − + 0⇒
= − + 0
=  
−1 + t 4 + t 2 + t t −1 t + 4 t + 2 ⇒ t
 2

 ( t + 4 )( t + 2 ) − 21 ( t − 1 )( t + 2 ) + 10 ( t − 1 )( t + 4 )   4 + t  9t2 9t2
⇒ (1 + 2t )   = ⇒ ( 2t + 1 )( t + 4 ) =
⇒ t  = 0
( t − 1)( t + 4 )( t + 2)  2  4 2
 

 t2 + 6t + 8 − 21t2 − 21 t + 42 + 10t2 + 30 t − 40  ( )
⇒ 2. 2t2 + 9t + 4 = 9t2
⇒ t = 0
 ( t − 1)( t + 4 )( t + 2)  ⇒ 5t2 − 18t − 8 = 0 ⇒ 5t2 − 20t + 2t − 8 =0

 −10 t2 + 15 t + 10  ⇒ 5t ( t − 4 ) + 2 ( t − 4 ) =
0 ; t =
−2 / 5,t =
4
⇒ t =0
 ( t − 1 )( t + 4 )( t + 2 ) 
But t ≠ 4 ⇒ x =6 and logx = -2/5 is Not Possible

1 2 2
⇒ t = 0, − , 2 ∴ log2 x = 0 ⇒ x = 1 ∴ t=− ⇒ log2 x =− ∴ x=2−2/5
2 5 5
1 1
log2 x =− ⇒ x = and log2 x = 2 ⇒ x = 4
2 2
1 . 2 4 | Basic Mathematics

JEE Main/Boards

Exercise 1 1 1 1 
Q.5 1 − log5
=  log + logx + log5 
3 2 3 
Q.1 Solve
1  1  1 1  1
(i) log16 32 Q.6 logx − log  x − =
 log  x +  − log  x + 
2  2  2 2  8
(ii) log8 16 log10 x + 7
log10 x +1
Q.7 x 4 = 10
(iii) log1/3 (1 / 9 )
log210 x +log10 x2 −2
(iv) log2 3
(1728 )  log10 x 
Q.8  = log10 x

(v) log2 cos 45º  2 

(vi) log2 (log2 4 ) Q.9 log2 x − log2 8x + 1 =0

(vii) log3 ( tan30º )


Q.10 log1/3 x − 3 log1/3 x + 2 =0

Q.2 Prove the following


( )
2
logb x logb x
Q.11 a − 5a +6 =0
(i) log5 5 5 5 − ∞ =1

(ii) log0.125 ( 8 ) = −1 (
Q.12 log4 x2 − 1 − log4 ( x=
− 1)
2
) 
log4 

(4 − x)
2 

( )
(iii) log1.5 0.6 = −1
Q.13 2log3
x −3
+1 =log3
x −3
x−7 x −1
(iv) log2.25 ( 0.4 ) = −1

(v) log10 ( 0.9 ) = 0 ( )


Q.14 logx 9x2 log32 x = 4

Q.3 Find the no. of digits in Q.15 log0.5x x2 + 14log16x x2 + 40log4x x =


0

(i) 2100 (ii) 310


2
(
Q.16 log3 log1/2 x − 3log1/2 x + 5 =
2 )
Q.4 Solve
15
(i) logx −1 3 = 2 Q.17 log2 ( x / 4 ) =
x
log2 − 1
(
(ii) log3 3x − 8 =2 − x ) 1
8
Q.18 log10 (5x − 4 ) + log10 x + 1 = 2 + log10 0.18
2
(
(iii) log5− x x2 − 2x + 65 =
2 )
(iv) log3 ( x + 1 ) + log3 ( x + 3) =
1 Q.19 log
= 10 x
2
log10 (5x − 4 )

2log10 x
(v) x = 10.x2 1 1
Q.20 log2 ( x −=
2) − log1/8 3x − 5
log10 x +5 6 3
5 + log10 x
(vi) x 3 = 10

log x Q.21
log10 ( x +1 +1 ) =3
(vii) x 3 =9
log10 ( 3
x − 40 )
M a them a ti cs | 1.25

1 1 log 3 log x
Q.22 1 − log10 ( 2x=
− 1) log ( x − 9 ) Q.4 If 5x 2 + 3 2 = 162 then logarithm of x to the
2 2 10 base 4 has the value equal to
(A) 2 (B) 1 (C) –1 (D) 3/2
( )
Q.23 log10 3x2 + 7 − log10 ( 3x − 2 ) =
1
log2 x +log10 x3 +3 2
Q.5 (x) 10 =
 1 
Q.24  1 +  log10 3 + log10 2= log10 27 − 31/x
 2x 
( ) 1

1
x +1 –1 x +1 +1
1 where x1> x2> x3, then
Q.25 2 + x log10 x ( x + 2 ) + 1
log x + 3log10=
2 10 2
(A) x1 + x3 =
2x2 (B) x1 .x3 = x2

( )
Q.26 log2 4 x + 1 =x + log2 2x +3 − 6 ( ) (C) x2 =
2x1 x2
(D) x1−1 + x1−1 =
x3−1
x1 + x2
Q.27 log 5 (4 x
)
− 6 − log 5 (2 x
−2 =
2 ) Q.6 Let x = 2
log3
and y = 3
log2
where base of the
xlog10 4 logarithm is 10, then which one of the following holds
(
Q.28 log10 3x − 24 − x =
1
2 + log10 16 −
4
) 2
good?
(A) 2x < y (B) 2y < x (C) 3x = 2y (D) y = x
Q.29 log10 (log10 x ) + log10 log10 x − 3 =
0 ( 4
)
Q.7 Number of real solution(s) of the equation
x
Q.30 log3 9= ( )
+ 9 log3 3x 28 − 2.3x ( ) x −3
3x2 −10x +3
1 is-
=

(A) Exactly four (B) Exactly three


Exercise 2 (C) Exactly two (D) Exactly one

Single Correct Choice Type Q.8 If x1 and x2 are the roots of the equation
log2010 x
2010x = x2 , then find the cyphers at the end
1 1 1 of the product (x1x2)
Q.1 + +
log bc
abc log ac
abc log ab
abc
(A) 1 (B) 3 (C) 2 (D) 4
has the value equal to
Q.9 Let x = 2 or x = 3 satisfy the equation, log4 (x2 + bx
(A) 1/2 (B) 1 (C) 2 (D) 4 + c) = 1. Then find the value of |bc|.
(A) 50 (B) 60 (C) 40 (D) 55
Q.2 The equation, log2 2x2 + log2 x.x ( ) logx (log2 x +1 )

1 −3log1/2 (log2 x )
+ log4 2x 4 + 2 1 has
=
2
(A) Exactly one real solution (B) Two real solutions
(C) 3 Real solutions (D) No solution

Q.3 Number of zeros after decimal before a significant


figure in (75)–10 is:
(Use log10 2 = 0.301 and log10 3= 0.477)

(A) 20 (B) 19 (C) 18 (D) None


1 . 2 6 | Basic Mathematics

JEE Advanced/Boards

Exercise 1 log10 ( x − 3) 1
=
Q.1 Let A denotes the value of
Q.11 (a) Solve for x,
( 2
log10 x − 21 ) 2

    (b) log (log x) + log (logx3 –2) = 0; where base of log is


( ab ) − 4 ( a + b )  ( ab ) − 4 ( a + b ) 
2 2
 ab +  ab − 10 everywhere
log10   + log10  
 2   2  (c) logx2.log2x2 = log4x2
   
(d) 5logx + 5xlog5 = 3 (a > 0); where base of log is a
when a = 43 and b = 57 and B denotes the value of the

expression  2 6  .  3 6  . Find the value of (A.B).


log 18 log 3
Q.12 Solve the system of equations:
  
loga x loga (xyz) = 48
Q.2 Simplify: loga y loga (xyz) = 12

4 3
(a) log1/3 729 9−1.27−4/3 (b)
(
log b log b N ) loga z loga (xyz) = 84
log b a
a
Q.13 Let ‘L’ denotes the antilog of 0.4 to the base 1024.
and ‘M’ denotes the nuber of digits in 610 (Given log102 =
Q.3 (a) Which is smaller? 2 or (logπ 2 + log2 π )
0.3010, log103 = 0.4771) and ‘N’ denotes the number of
(b) Prove that log3 5 and log2 7 are both irrational. positive integers which have the characteristic 2, when
base of the logarithm is 6. Find the value of LMN.
Q.4 Find the square of the sum of the roots of the
equation log3x · log4x · log5x = log3x · log4x + log4x · log5x + Q.14 Prove the identity.
log5x · log3x.
logaN . logbN+ logbN . logcN + logcN . loga

Q.5 Find the value of the expression loga Nlogb Nlogc N


N=
2 3 logabc N
+
log4 ( 2000 ) log5 ( 2000 )
6 6
10
Q.15 If x, y > 0, logyx + logxy = and xy = 144, then
1 3
1 x+y
log5 9 log 6
3
 2  = N where N is a natural number, find the vaue
Q.6 Simplify:
81 +3
409


( 7) log25
− (125 ) 5 
log 6


2
of N.
 
log1/5 (1/2 ) 4 1
Q.7 Simplify: 5 + log + log1/2 Q.16 If log102 = 0.0310, log103 = 0.4771. Find the
2
7+ 3 10 + 2 21 number of integers in:
(a) 5200
2
Q.8 Given that log2 a = s, log4 b = s and log 2 8 = 2

2 5 s3 + 1 c (b) 615
a b
Write log2 as function of ‘s’ (a, b, c > 0) (c ≠ 1). (c) The number of zeros after the decimal in 3–100.
C4
log2 24 log2 192
Q.9 Prove that − =3 Q.17 log5120 + (x – 3) –2 log5 (1 – 5x –3) = – log5 (2 – 5x–4)
log96 2 log12 2

Q.18 logx+1 (x2 + x – 6)2 = 4


Q.10 Prove that ax – by = 0 wher x = loga b and

=y logb a , a > 0, b > 0 & b ≠ 1 . Q.19 x + log10 (1 + 2x) = x log105 + log106


M a them a ti cs | 1.27

Q.20 If ‘x’ and ‘y’ are real numbers such that,  1/3 1/3 
 
log10 ( 2x − 3x=
x log  x  + log  3   log x3 =
) log10 x + log10 y, find y +
 3 3 x 
x  3 2
  
 
Q.21 If a = log12 18 and b = log24 54 then find the value
of ab + 5 (a – b) Q.31 Let a = (log7 81 )(log6561 625 )(log125 216 )(log1296 2401 )

b denotes the sum of the roots of the equation


Q.22 Find the value of log3x if following is true
= ( 2x )
log2 x log2
x x and c denotes the sum of all natural
( ) log (3x ) = log x
log9 9x 4
3 3
3
solution of the equation |x + 1| + |x – 4| = 7.
Find the value of (a + b) ÷ c.
Q.23 Positive numbers x, y and z satisfy xyz = 10 and81

(log10 x) (log10 yz) + (log10 y) (log10 z) = 468. Find the


value of (log10x)2 + (log10y)2 + (log10z)2. Exercise 2
Single Correct Choice Type
Q.24 Find the sum of all solutions of the equation

3
(log9 x )2 − 9 log x + 5 =3 3 Q.1 Number of ordered pair(s) satisfying simultaneously,
9
2 x+ y
the system of equations, 2 = 256 and
Q.25 Let a, b, c, d are positive integers such that loga
log10 xy – log10 1.5 = 1, is:
b = 3/2 and logc d = 5/4. If (a – c) = 9, find the value of
(b – d). (A) Zero (B) Exactly one
(C) Exactly two (D) More than two
Q.26 Find the product of the positive roots of the

( 2008 ) ( x )
log2008 x
equation = x2 Q.2 Let ABC be a triangle right angled at C. The value of
logb + c a + logc −b a
(b + c ≠, c – b ≠ 1) equals
Q.27 Find x satisfying the equation logb + c a.logc −b a

2  4 2  4  2  2  (A) 1 (B) 2 (C) 3 (D) ½


log10  1 +  + log10  1 =
−  2log10  − 1
 x   x + 4   x − 1 
Q.3 Let B, C, P and L be positive real number such that
log (B · L) + log (B · P) = 2; log (P · L) + log (P · C) = 3; log
Q.28 Solve: log3 ( x+ x −1 ) (C · B) + log (C · L) = 4. The value of the product (BCPL)
equals (base of the log is 10)
(
= log9 4 x − 3 + 4 x − 1
) (A) 102 (B) 103 (C) 104 (D) 109

Q.29 Prove that


Q.4 If the equation
(
log12 log8 (log4 x ) ) = 0 has a
 
4 4 4 4
 loga ab +logb ab − loga b/a +log a/b  loga b ( (
log5 log4 logy (log2 x ) ))
2 
solution for ‘x’ when c < y < b, y ≠ a, where ‘b’ is as
2 if b ≥ a > 1
=  log b large as possible and ‘c’ is an small as possible, then the
2 a if1 < b < a value of (a + b + c) is equal to

(A) 18 (B) 19 (C) 20 (D) 21


Q.30 Find the value of x satisfying the equation

log 3x 1/3 + log 3x 1/3  log x3


 3 ( ) x( )  3
1 . 2 8 | Basic Mathematics

Q.5 For N > 1, the product


(lnx )
2
− 3lnx + 3
1 1 1 1 Q.13 If < 1 , then x belongs to:
. . . simplifies to ln x − 1
log2 N logN 8 log32 N logN 128
(A) (0, e) (B) (1, e) (C) (1, 2e) (D) (0, 3e)
3 3 3 5
(A) (B) (C) (D)
7 7 n2 5 n2 21
Multiple Correct Choice Type

( )
( ) 
2
3log2 −2log log103 + log log106
1 + 2log3 2
Q.6 Let N = 10 where 
Q.14 The number N = + log26 2
(1 + log3 2)
2
base of the logarithm is 10. The characteristics of the
logarithm of N to the base 3, is equal to
when simplified reduces to-
(A) 2 (B) 3 (C) 4 (D) 5
(A) A prime number
(B) An irrational number
10 + 2 10 − 2
Q.7 If x
= = and y , then the value (C) A real number is less than log3π
2 2
(
of log2 x2 + xy + y 2 , is equal to ) (D) A real which is greater than log76

(A) 0 (B) 2 (C) 3 (D) 4 Q.15 The value of x satisfying the equation,
22x ‒ 8.2x = ‒ 12 is
5 3 5 3 log3 1 3
Q.8 The sum + + − is equal to (A) 1 + (B) log6 (C) 1 + log (D) 1
4 2 4 2 log2 2 2
π π π π
(A) tan (B) cot (C) sec (D) sin x x
3 3 3 3
Q.16 If  5 2 − 7  + 6  5 2 + 7  =
7,
   
Q.9 Suppose that x < 0. Which of the following is equal then the value of x can be equal to-

( x − 2)
2
to 2x − (A) 0 (B) log 36
(5 2 −7 )
(A) x – 2 (B) 3x – 2 (C) 3x + 2 (D) – 3x + 2 −2
(C) (D) log 6
(
log6 5 2 + 7 ) 5 2 −7

Q.10 Solution set of the inequality


3x ( 0.333..... ) ≤ (1 / 27 ) is:
x −3 x

Assertion Reasoning Type


(A) [3/2, 5] (B) ( −∞ ,3 / 2
(A) Statement-I is true, statement-II is true and
(C) ( 2,∞ ) (D) None of these statement-II is correct explanation for statement-I.

2x +1
−3 (B) Statement-I is true, statement-II is true and
 1  1− x  1  statement-II is NOT the correct explanation for
Q.11 Solution set of the inequality   >  is-
5 5 statement-I.
(A) ( −∞ , −2 ) ∪ (1, ∞ ) (B) (1, 4)
(C) Statement-I is true, statement-II is false
(C) ( −∞ ,1 ) ∪ ( 2, ∞ ) (D) None of these
(D) Statement-I is false, statement-II is true

Q.12 The set of all x satisfying the equation


Q.17 Statement-I: logx cos ( 2πx ) is a meaningful
2
log3 x2 + (log3 x ) −10 1 quantity only if x ∈ (0, ¼) ∪ (3/4, 1).
= is-
x2
Because
 1   1  1 
(A) {1, 9} (B) 9,  (C) 1, 4,  (D) 1,9, 
 81   81   81 
M a them a ti cs | 1.29

Statement-I: If the number N > 0 and the base of the Comprehension Type
logarithm b (greater than zero not equal to 1) both lie
on the same side of unity then logbN > 0 and if they lie Paragraph 1: Equations of the form (i) f (logax) = 0,
on different side of unit then logb N < 0. a > 0, a ≠ 1 and (ii) g (logxA) = 0, A > 0, then Eq. (i) is
equivalent to f(t) = 0, where t = logax. If t1, t2, t3, ……… tk

(
1 log2 ( x − 1 )
Q.18 Statement-I: log2 2 17 − 2x =+ ) are the roots of f(t) = 0, then logax = t, loga x = t2, …… ,
x = tk and eq. (ii) is equivalent to f(y) = 0, where y
has a solution. logx A. If yf , y2, y3, …., yk are the root of f(y) = 0, then
because logx A = y1, logx, A = y2, ……. , logx A = yk.

Statement-II: Change of base in logarithms is possible. On the basis of above information, answer the following
questions.
Q.19 Consider the following statements
Q.20 The number of solution of the equation
log 5  x3 +1 
2
Statement-I: The equation 5   1 has two
−x = log3x 10 − 6log2x 10 + 11logx 10 − 6 =0 is:
distinct real solutions.
(A) 0 (B) 1 (C) 2 (D) 3
Because.
loga N
Statement-II: a = N when a > 0, a ≠ 1 and N > 0.

Match the Columns

Q.21

Column-I Column-II
(A) The value of x for which the radical product (p) 4

is equal to 13, is not greater than


3 x − 7x + 4x − 1 2x + 4x − 1 3 x + 7x + 4x − 1

P (x) (q) 7
(B) Let P(x) = x7 – 3x5 + x3 –7x2 + 5 and Q (x) = x – 2. The remainder of is not smaller than
Q(x)

(C) Given a right triangle with side of length a, b and c and area equal to a2 + b2 – c2. The ratio of the (r) 10
larger to the smaller leg of the triangle is

(D) If a, b and c ∈ N such ( 3


)(
4 + 2 − 2 a3 4 + b 3 2 + c = )
20 Then the value of (a + b – c), is not
(s) 17

equal to

Q.22

Column I Column II

(A) The expression


= x log2 log9 6 + 6 + 6 + .........∞ simplifies to (p) An integer

(B) The number N = 2( 2


log 3.log3 4.log4 5.....log99 100 )
simplifies to (q) A prime

1 1 1
(C) The expression + − simplifies to (r) A natural
log5 3 log6 3 log10 3

(D) The number N = simplifies to (s) A composite


2 + 5 − 6 − 3 5 + 14 − 6 5
1 . 3 0 | Basic Mathematics

PlancEssential Questions
JEE Main/Boards JEE Advanced/Boards
Exercise 1 Exercise 1
Q.2 Q.3 Q.15 Q.6 Q.12 Q.14
Q.25 Q.26 Q.16 Q.23

Exercise 2 Exercise 2
Q.3 Q.5 Q.9 Q.4 Q.6 Q.11
Q.10 Q.15 Q.17

Answer Key

JEE Main/Boards
Exercise 1
5 4 1 1
Q.1 (i) (ii) (iii) 2 (iv) 6 (v) – (vi) 1 (vii) –
4 3 2 2

Q.3 (i) 31 (ii) 5


3 +1 1− 3
1
Q.4 (i) 1 + 3 (ii) 2 (iii) –5 (iv) 0 (v) 10 2 ,10 2 (vi) ,1000
105
2 2
(vii) 3 , 3−

24
Q.5 Q.6 1 Q.7 10–4, 10 Q.8 10-3, 10, 102
51/3
loga b,3loga b
Q.9 2, 16 Q.10 1/3, (1/3)4 Q.11 2 Q.12 3+ 6

 17   17 
 –1 +   –1– 
 5   5 
Q.13 – 5 Q.14. 3, 1/9 Q.15 2 , 2 Q.16 1/16, 2

41
Q.17 27, 2–1 Q.18 8, – Q.19 4, 1 Q.20 3
5
1 1
Q.21 48 Q.22 13 Q.23 1, 9 Q.24 ,
4 2
Q.25 98 Q.26 0 Q.27 2 Q.28 3

Q.29 (10)–1/4, (10) Q.30 (–1), 2


M a them a ti cs | 1.31

Exercise 2
Single Correct Choice Type

Q.1 B Q.2 D Q.3 C Q.4 D Q.5 B Q.6 D

Q.7 B Q.8 C Q.9 A

JEE Advanced/Boards
Exercise 1

Q.1 12 Q.2 (a) 1 (b) logb N Q.3 (a) 2 Q.4 (61)2

Q.5 1/6 Q.6 1 Q.7 6 Q.8 2s + 10s2 – 3 (s3 + 1)


2
Q.11 (a) 5 (b) 10 (c) 2± (d) 2–log5a Q.12 (a4, a, a7) or (a–4, a–1, a–7) Q.13 23040

Q.15 507 Q.16 (a) 140 (b) 12 (c) 47 Q.17 -0.410 Q.18 1
5+3 5
Q.19 1 Q.20 4/9 Q.21 1 Q.22
10
Q.23 5625 Q.24 2196 Q.25 93 Q.26 (2008)2

Q.27 2, 6 Q.28 [0, 1] ∪ {4} Q.30 [1/3, 3] – {1} Q.31 1

Exercise 2
Single Correct Choice Type

Q.1 C Q.2 B Q.3 B Q.4 B Q.5 D Q.6 B

Q.7 C Q.8 A Q.9 D Q.10 D Q.11 B Q.12 D

Q.13 A

Multiple Correct Choice Type

Q.14 C, D Q.15 A, D Q.16 A , B, C, D

Assertion Reasoning Type

Q.17 D Q.18 B Q.19 B

Comprehenstion Type

Q.20 D

Match the Columns

Q.21 A → q, r, s; B → p, q, r, s; C → p; D → r Q.22 A → p; B → p, r, s; C → p, r; D → p, q, r
1 . 3 2 | Basic Mathematics

Solutions
let 5 5 5..........∞ =x
⇒ 5 x = x ⇒ 5 = x1/2 ⇒ x = 25
JEE Main/Boards log=
5 x log5=
25 log
= 55 1

3
Exercise 1 log20.5 ⇒ log0.5
2
(ii)log=
0.125 8
3
2
⇒ log1/2 ⇒ log2 −1 ⇒ − 1.log22 = −1
( 2)
Sol 1: (i) log1632 = log 25
24

we know log n ym =
m
logx y (iii) log ( 0.6 )
x n
2
5 5 = 0.6 0.6666.............
=
⇒ log 4 25 = log2 2 = 3
2 4 4
( 2) =
log ( 3 )
−1
4 4 4 ⇒ log3/2 3 −1
=
(ii) log816 = log 3 24 = log2 2 = (1) = 2 3/2
2 3 3 3
(iii) log1/3(1/9) = log1/3(1/3)2 = 2 log1/3(1/3) = 2.(1) = 2 ( )
(iv) log2.25 0.4

(iv) log2 (1728) = log2 (2 3 )6 4 225 9


3 3 ⇒ 0.4 =0.444....... = ⇒ 2.25 = =
9 100 4
= 6 log2 2 3 = 6(1) = 6
( 4 9 )= (9 4)
−1
3
⇒ log9/ log9/ = 1
1 1 1 4 4
(v) log2 cos45° = log2 1 = log2 (2)– 2 =– log22 = –
2 2
(vi) log2 (log24)
2
( )
(v) log10 0.9

0.9 =
0.99999 =
1 ⇒ log10 1 =
0
 22 
⇒ log2  log2 
 
Sol 3: We have to find out no. of digits in
⇒ log2 ( 2 log22 ) (i) 2100 = x (Assume)
⇒ log22  logaa = 1
⇒ log10x = log10 2100 = 100 log10 2 = 100(0.3010) = 30.10
⇒1
= 1030 (10 )
0.103
⇒ x 1030.103
=
(vii) log3 (tan 30°)
Total no. of digit = 30 + 1 = 31
1
 tan 30° = (ii) x = 310
3
 −1  ⇒ log10x = log10310 = 10 log103 = 10(0.47712) = 4.7712
 1 
⇒ log3  3 
2
⇒ log3    
 3   ⇒ x = 104.7712 = 104 × 100.7712
 
−1 Total no. of digits = 4 + 1 = 5
⇒ log33
2
−1 Sol 4: (i) logx–1 3 = 2 ( x ≠ 1, 2)

2
1
logx–13 = 1 ⇒ logx–131/2 = 1
2

Sol 2: (i) log5 5 5 5..........∞ 31/2 = x – 1 ⇒ x = 1 + 3

Let
let 5 5 5..........∞ =x (ii) log3(3x – 8) = 2 – x
1/2
⇒ 5 x = x ⇒ 5 = x1/2 ⇒ x = 25 ⇒ (3x – 8) = (3)2–x= 32 . 3–x = 9.3–x ⇒ 3x – 9.3–x = 8
log=
55
x log55=
25 log
= 55
5 1
M a them a ti cs | 1.33

Assume 3x = y So from equation (i)


9 (1 + 3 ) (1– 3 )
⇒y– = 8 ⇒ y2 – 9 = 8y ⇒ y2 – 8y – 9 = 0 1± 3
y log10x = ⇒ x = 10 2 and 10 2
2
8 ± 82 + 4(1)(9) 8 ± 64 + 36 8 ± 100 log10 x +5
⇒y= ⇒y= = 5 +log 10 x
2(1) 2 2 (vi) x 3 = 10
⇒ y = 4 ± 5 = 9, –1
take logarithm (base 10) on both side
Terefore, 3x = 9 ⇒ 3x = 32 ⇒ x = 2  log10 x +5 
log10  x 3  = log 105+log x
10
3x = – 1 ⇒ no solution  
 
Hence x = 2  log10 x + 5 
⇒ 
3
(5 + log10 x ) log10 10
 log10 x =
 
(iii) log5–x(x2 – 2x + 65) = 2
 5 + log10 x 
⇒ x2 – 2x + 65 = (5 – x)2 = x2 + 52 – 2(5)x ⇒ 
3
(5 + log10 x ) . (1)
 log10 x =
 
⇒ – 2x + 65 = 25 – 10x ⇒ 10x – 2x = 25 – 65 = – 40
⇒ log10x = 1(3) = 3 ⇒ x = 103 = 1000
40
⇒ 8x = – 40 ⇒ x = – = –5
8 2nd solution ⇒ 5 + logx = 0
(iv) log3(x + 1) + log3(x + 3) = 1
⇒ log10x = – 5 ⇒ x = 10–5
⇒ log3[(x + 1) . (x + 3)] = 1 log3 x
(vii) x =9
⇒ ( x + 1 )( x + 3) =
3 ⇒ x + x + 3x + 3(1) = 3
2
Take logarithm (base 3) in both sides
⇒ x + 4x = 0 ⇒ x(x + 4) = 0 ⇒ x = 0, – 4
2 logx
log3[ x 3 ]= log3 9 = log3 32 = 2log3 3
But at x = – 4, equation is
⇒ (log3 x)2 = 2 ⇒ |log3x| = 21/2 ⇒ log3 x = ± 2
log3(–4 + 1) + log3(–4 + 3) =1 2
⇒x= 3 , 3– 2

1 1 1 
Sol 5: 1 − log10 =
5  log10 + log10 x + log10 5 
It can’t be –ve so x ≠ – 4 ⇒ x = 0 3 2 3 
(v) x2 logx = 10 x2 1 1
3 (1 − log10 5 ) = log10 + log10 x + log10 5
2 3
Take logarithms is both sides
1
3 − log10 5 = log10 + log10 51/3 + log10 x
log10(x ) = log10 10x
2 logx 2 2
1
2 log10x (log10x) = log1010 + log10x2 ⇒ 3 log10 53 + log10
= + log10 51/3 + log10 x
2
2log10x(log10 x) = 1 + 2log10 x  1 
⇒ 3 log10 53 × × 51/3  + log10 x
=
 2 
Assume log10 x = y ... (i)
 3+ 1  1  
⇒ 2y(y) = 1 + 2y ⇒ 2y2 = 1 + 2y ⇒ 2y2 – 2y – 1 = 0 ⇒ log10 3 − log10 5 3 ×   
x=

  2  
2 ± 22 – 4(2)(–1)
⇒y=
2(2) log10 x =log10 103 − log10 510/3 × 2−1 ( )
2± 4+8 2±2 3 1± 3
y= = =
4 4 2
1 . 3 4 | Basic Mathematics

 103  53 × 23 ⇒ x2 = 4x2 − 2x − 1 ⇒ 3 x2 − 2x − 1 =0
log
= 10   log10
 510/32–1  510/3 × 2–1
  2 ± 4 + 12 2± 4
⇒x= ⇒x=
9–10 6 6
= log10[ 5 3 23+1 ] = log10 [5–1/3 24] 1 1
x= 1, − x= 1, −
24 24 3 at 3
log
= 10 x log10 = ⇒x
51/3 51/3 2log ( −2 ) =
log ( 4 )

Which is not possible ⇒ x =


1
1  1  1
Sol 6: log10 x – log10  x –  = log10   x + 
2  2  2
log10 x + 7
log10 x +1
1  1 Sol 7: x 4 = 10
– log10  x + 
2  8
Take logarithm on both sides
 1  1  1
2log10 x – log10  x –  = 2log10   x +  – log10  x +   log10 x + 7 
 2  2  8
log10  x 4
 (
 = log 10log10 x +1
 10 )
2  
 1  1  1
log10 x2 – log10  x –  = log10   x +  – log10  x + 
 2  2  8  log10 x + 7 
⇒   (log10 x ) =(log10 x + 1) log10 10
 4 
 1 
2
  
 x2   x +   y +7
 2  ⇒ Assume log10 x = y ⇒   (y) = y + 1
⇒ log10   = log    4 
x–1   x + 1  
  ⇒ y2 + 7y = 4(y + 1) = 4y + 4
 2  8 
 
⇒ y2 + 7y – 4y – 4 = 0 ⇒ y2 + 3y – 4 = 0
 1 
2
   x +  
 2  2  = 0 ⇒ (y + 4)(y – 1) = 0 ⇒ y = – 4 and + 1
⇒ log10  x  – log10  
 
x–1   x + 1  

⇒ log10 x = – 4 or 1
 
 2  8 
 
Hence x = 10–4 or 10
 
 1 
2 x+ log102 x +log10 x2 –2
⇒ log10  x × 8  =0  log10 x 
 2 Sol 8:   = log10 x
  x – 1   1   2 
 2 x + 2 
    log102 x +log10 x2 –2

 
(
⇒ log10 x1/2 ) = log10 x1/2
 1
   x+1  x2  x + 
 x2   8   8 ⇒ log102x + log10 x2 – 2 = 1 or log10 x1/2 = 1
⇒     = 1 ⇒ =1
x–1
2  2 1  1
   1   x –  x +  ⇒ log102x + 2log10x – 2 = 1; log10 x = 2 ⇒ x = 102
 2  x + 2   4  2
  
⇒ log2x + 2log x - 2=0
 1  1  1 Assume that log x = y
⇒ x2  x + 8  =  x2 –   x + 
   4  2 ⇒ y2 + 2y – 2 = 1 ⇒ y2 + 2y – 2 – 1 = 0

x x2 x 1 1 ⇒ (y + 3)(y – 1) = 0
⇒ x3 + = x3 + – –  
8 2 4 4 2 y = – 3 or y = 1
2 2
x x x 1 log10 x = – 3 or log10 x = 1
⇒ x3 + = x3 + − −
8 2 4 8
⇒ x = 10–3 or 101 ⇒ x = 10–3, 10, 102
M a them a ti cs | 1.35

Sol 9: 3 log2 x – log2 8x + 1 = 0 x +1 x +1


⇒ = (4 – x)2 ⇒ = |4 – x|
(x – 1) (x – 1)
⇒ 3 log2 x = log2 23x – 1 ⇒ 3 log2 x = 2 + log2 x
Case-I: When 4 – x ≥ 0
Assume that log2 x= y
x +1
⇒ = 4 – x ⇒ (x + 1) = (4 – x)(x – 1)
⇒3 y =2+y x –1
⇒ x + 1 = 4x – 4 – x2 + x ⇒ x2 – 4x – x + x + 1 + 4 = 0
Square on both sides
4 ± 42 – 4(5)(1)
⇒ (3 y )2 = (2 + y)2 ⇒ 9y = 22 + y2 + 2(2)(y) ⇒ x2 – 4x + 5 = 0 ⇒ x =
2(1)
⇒ 9y = 4 + y2 + 4y ⇒ y2 – 5y + 4 = 0
4 ± 16 – 20 4 ± –4
⇒x= = (no solution)
⇒ (y – 4)(y – 1) = 0 ⇒ y = 4 or y = 1 2 2

Case-II: When 4 – x < 0


⇒ log2 x = 4 or log2 x = 1
x +1
⇒ = x – 4 ⇒ x + 1 = (x – 1)(x – 4) = x2 + 4 – x – 4x
⇒ x = 24 or x = 21 ⇒ x = 16 or 2 x –1
⇒ x2 – 4x – x – x + 4 – 1 = 0 ⇒ x2 – 6x + 3 = 0
Sol 10: log1/3 x – 3 log1/3 x + 2 = 0
6 ± 62 – 4(3)(1) 6 ± 36 – 12
⇒x= ⇒x=
log1/3 x + 2 = 3 log1/3 x 2(1) 2

Assume that log1/3 x = y  ....... (i) 6 ± 24


= =3± 6
2
⇒ y + 2 = 3 y ⇒ y = 4 or y = 1 [Refer above solution]
But x > 4
log1/3 x = 4 or log1/3 x = 1 6
So, x = 3 +
4 1
1 1 1 1
⇒ x =   or x =   ⇒x= or
Sol 13: 2log3 x – 3 + 1 = log3 x – 3
3 3 81 3
x–7 x –1
2

( ) log3  x – 3  + log3 3 = log3 x – 3


2
log x log x
Sol 11: a b −5x b +6 =0  
x–7 x –1
Assume that x = by
 2 
log x log3  (x – 3) × 3 = log3 x – 3
⇒ ( ay )2 – 5( a b ) + 6 = 0 ⇒ a – 5a + 6 = 0
2y y
2 x –1
 (x – 7) 
⇒(ay – 3)(ay – 2) = 0 ⇒ ay = 2, 3
3(x – 3)2 (x – 3)
⇒ = ⇒ 3(x – 3)(x – 1) = (x – 7)2
y = loga 2, loga 3 ∴x =
log b log b
2 a ,3 a (x – 7) 2 (x – 1)

⇒ 3x2 + 9 – 3x – 9x = x2 – 14x + 49
2
Sol 12: log4(x – 1) – log4(x – 1) = log4( (4 – x) )
2 2
⇒ 2x2 + 2x – 40 = 0 ⇒ x2 + x – 20 = 0
 2  ⇒ (x + 5)(x – 4) = 0
log4  x – 1  = log4( (4 – x)2 )
 (x – 1)2  x = – 5 or x = 4
 
4–3
x2 – 1 At x = 4, equation is 2log3  4 – 3  + 1 = log
= (4 – x) 2   4 –7
4–7
(x – 1)2
4–3 +1
= ⇒ –ve which is not possible
(x – 1)(x + 1) 4–7 –3
⇒ = (4 – x)2 ; x ≠ 1,
(x – 1)2 Hence x ≠ 4, x = – 5
1 . 3 6 | Basic Mathematics

Sol 14: logx (9x2) log32 x = 4 ⇒ (logx32 x2)(log3 x)2 = 4  12   12 


(2)
2
2− − 41  −  2 − 2 1 + 
 5   5 
2 ⇒y=– ⇒y=
 log x 
⇒ 2[logx 3x]  e  = 4 2 (1 ) 2
 loge 3 
5 + 12
loge n 2−2
We know that logm n = 5 17
loge m ⇒y= ⇒y=–1±
2 5
2
 log x  Now, we have log2 x = y
⇒ 2[logx 3 + logx x]  e  = 4
 loge 3 
⇒x= 2
( –1+ 17/5 ) or 2
( –1– 17/5 )
2
 log 3   loge x 
⇒  e + 1   =2
 loge x   loge 3  2
Sol 16: log3[ log1/2 x – 3log1/2x + 5] = 2

loge 3 (loge x)2 Assume that log1/2 x = y


⇒ × + (log3 x)2 = 2
loge x (loge 3)2
⇒ log3[y2 – 3y + 5] = 2 ⇒ y2 – 3y + 5 = 9
⇒ log3 x + (log3x)2 = 2
⇒ y2 – 3y – 4 = 0 ⇒ (y – 4)(y + 1) = 0
Assume that log3 x = y
⇒ y = 4 or y = – 1 ⇒ log1/2 x = 4 or log1/2x = – 1
⇒ y2 + y = 2 ⇒ y2 + y – 2 = 0 ⇒ (y + 2)(y – 1) = 0
4 –1
⇒ y = – 2 or y = 1 1 1
⇒ x =   or x =  
Now, we have log2 x = – 2 or log3 x = 1 2 2
1
⇒ x = 3–2 or x = 3+1 ⇒x= or x = 2.
16
1
Hence, x = or x = 3
9 15
Sol 17: log2(x/4) =
x
log2 – 1
Sol 15: log0.5x x2 + 14 log16x x2 + 40 log4x x = 0 8
15
log2 x2 14log2 x2 40log2 x ⇒ log2x - log24 =
+ + =0 log2 x − log2 8 – 1
log2 (0.5x) log2 (16x) log2 (4x)
⇒ Assume that log2x = y
Assume that log2 x = y
15 15
y–2= =
2y 28y 20y y −3–1 y–4
⇒ + + =0
log2 .2−1 + y log2 24 + y log2 22 + y ⇒ (y – 2)(y – 4) = 15 ⇒ y2 – 6y + 8 = 15

y 14y 10y ⇒ y2 – 6y – 7 = 0 ⇒ (y – 7)(y + 1) = 0


⇒ + + =0
y –1 y+4 y+2 y = 7 or y = – 1
Now, we have log2 x = 7 or log2 x = – 1
 1 14 10 
y = 0 or  + +  =0 Hence x = 27or x = 2–1
 y –1 y + 4 y +2

⇒ log2 x = y ⇒ x = 2y = 2° = 1 1
Sol 18: log10(5x – 4) + log10 x + 1 = 2 + log100.18
2
or (y + 4)(y + 2) + 14(y – 1)(y + 2) + 10(y – 1)(y + 4) = 0
⇒ log10(5x – 4) + 2log10 x + 1 = 2[2 + log100.18]
⇒ y2 + 8 + 6y + 14y2 – 28 + 14y + 10y2 – 40 + 30y = 0
⇒ 25y2 + 50y – 60 = 0 ⇒ log10(5x – 4) + log10(x + 1) = 4 + 2log100.18
60 12
⇒ y2 + 2y – = 0 ⇒ y2 + 2y – =0 ⇒ log10[(5x – 4)(x + 1)] = 4 + log10(0.18)2
25 5
M a them a ti cs | 1.37

⇒ log10[(5x – 4)(x +1)] = log10[104 × (0.18)2] log10 ( x + 1 + 1) log10 ( x + 1 + 1)


Sol 21: =3 ⇒ =3
⇒ (5x – 4)(x + 1) = 104(0.0324) = 324 log10 (x – 40)1/3 1
log (x – 40)
⇒ 5x2 + x – 4 = 324 ⇒ 5x2 + x – 328 = 0 3 10
log10( x + 1 + 1) = log10(x – 40)
–1 ± (1)2 – 4(5)(–328)
x=
(10) x + 1 + 1 = x – 40 ⇒ x + 1 = x – 41
–1 ± 1 + 20(328) –1 ± 6561 On squaring both sides
x=– =
10 10
x + 1 = (x – 41)2 = x2 + 412 – 2(41)x
–1 ± 81 41
x= = 8, − ,
10 5 ⇒ x2 – 82x – x + 412 – 1 = 0 ⇒ x2 – 83x + 1680 = 0
Also it is clear that x > 4 / 5
( 83)
− 4 (1680 )(1 ) 83 ± 169 83 ± 13
2

−41 x=
83 ± = = 48, 35
=
∴x = is rejected 2 (1 ) 2 2
5
Now, for x = 35
Sol 19: log10x2 = log10(5x – 4) log10 35 + 1 + 1
The given equations yields = 3 –5
⇒ x2 = 5x – 4 ⇒ x2 – 5x + 4 = 0 log10 3 35 – 40
⇒ (x – 4)(x – 1) = 0 ⇒ x – 4 = 0or x – 1 = 0 Wich is not a possible solutions
Hence, x = 4, 1 Hence x ≠ 35 and x = 48

1 1 1 1
Sol 20: log2(x – 2)– = log1/8 3x – 5 Sol 22: 1 – log10(2x – 1) = log10(x – 9)
6 3 2 2
⇒ 2 – log10(2x – 1) = log10(x – 9)
1 1
⇒ log2(x – 2)– = log –3 3x – 5
6 3 2 ⇒ log10(x – 9) + log10(2x – 1) = 2

1 1 1 ⇒ log10(x – 9)(2x – 1) = log10102


⇒ log2(x – 2) – = – log2 3x – 5
6 3 3
⇒ (x – 9)(2x – 1) = 100
1
⇒ log2(x – 2) – 1 = – log2 3x – 5 ⇒ 2x2 – 18x – x + 9 = 100 ⇒ 2x2 – 19x – 91 = 0
2

⇒ log2(x – 2) + 2log2 3x – 5 = 2 19 ± 192 – 4(2)(–91) 19 ± 1089 7


x= = =13, –
2(2) 4 2
⇒ log2(x – 2) + log2(3x – 5) = log2 22
7
⇒ (x – 2)(3x – 5) = 4 ⇒ 3x2 + 10 – 6x – 5x = 4 But x = – is not in the domain hence x = 13
2
⇒ 3x2 – 11x + 6 = 0
Sol 23: log10(3x2 + 7) – log10(3x – 2) = 1
11 ± 121 − 4 ( 3)( 6 ) 11 ± 121 − 72
x =
2 (3) 6  2 
log10  3x + 7  = 1 = log1010
 3x – 2 
 
11 ± 49 2
⇒x= = 3, 3x2 + 7
6 3 = 10 and 3x2 + 7 = 10(3x – 2)
3x – 2
2 1 1  1
At x = , eq. ⇒ log2  – 2  –
3 6 3  3 ⇒ 3x2 + 7 = 30x – 20 ⇒ 3x2 – 30x + 27 = 0
⇒ x2 – 10x + 9 = 0 ⇒ (x – 1)(x - 9) = 0
= log1/8 (2)
3 –5 = –3 (Not a possible solution)
3 Hence x = 9, 1
So x = 3
1 . 3 8 | Basic Mathematics

 1  1
Sol 24:  1 +  log103 + log102 = log10(27 – 31/x) 2X = 1 or 2x = – (not valid)
2x 7
 
⇒x=
0 and so, x = 0 is only solution.
⇒ log10 3(1+1/2x ) + log102 = log10(27 – 31/x)

⇒ log102× (3)1+1/2x = log10(27–31/x) Sol 27: log (4x – 6) – log (2x – 2) = 2


5 5

⇒ 2 × 31+1/2x = 27 – 31/x  4x – 6  4x – 6
log  x  =2⇒ x =5
Assume that 3 1/x
=y 5 2 –2 2 –2
 
⇒ 2×3× y = 27 – y Assume that 2x = y

On squaring both sides, we get y2 – 6


⇒ = 5 ⇒ y2 – 6 = 5(y – 2) = 5y – 10
y–2
⇒ 22 × 32 × y = (27 – y)2 ⇒ 36y = 272 + y2 – 2(27)y
⇒ y2 – 54y – 36y + 272 = 0 ⇒ y2 – 90y + 272 = 0 ⇒ y2 – 5y – 6 + 10 = y2 – 5y + 4 = 0

(y - 81)(y - 9) = 0 ⇒ y = 81, 9 ⇒ (y – 4)(y – 1) = 0


1 1 1 1 1 y = 4 or y = 1
∴ x= ⇒x= or = ,
log3 y log3 81 log3 9 4 2 ⇒ 2x = 4 or 2x = 1
1 ⇒ x = 2 or x = 0
Clearly 31/x < 27 ∴ x >
3
x = 0 does not satisfy the equation, hence x = 2
So x = 1/4 is not valid

1 xlog10 4
1 Sol 28: log10(3x – 24–x) = 2 + log1016 –
Sol 25: log10 x + 3 log10=
2 + x log10 x ( x + 2) + 1 4 2
2
1 xlog22
log10 x + 6 log10
= 2 + x 2log10 x ( x + 2) + 2 ⇒ log10(3x–24–x)=log10102+ log1024 –
4 2
⇒ log10 x + log10(2 + x)3 – log10[x(x + 2)] = 2 4 x × 2 log10 2
⇒ log10(3X–24–X)=log10100+ log102–
 x(2 + x)3  4 2
⇒ log10   = log102
 x(x + 2)  ⇒ log10(3x – 24–x) = log10[100 × 2] – log2x

⇒ (2 + x)2 = 100 ⇒ 2 + x = ± 100


⇒ log10(3x – 24 –x) = log10 (200)
 100 – 2 = 98 2x
x  2 4
200
 –100 – 2 = –102 ⇒ 3x – = ⇒ 3x.2x – 24 = 200
x x
2 2
Here, x = – 102 does not satisfy the equation
Hence x = 98 ⇒ 6x = 200 + 24 ⇒ 216 = 63 ⇒ x = 3

Sol 29: log10(log10 x) + log10(log10 x4 – 3) = 0


Sol 26: log2(4x + 1) = x + log2(2x+3 – 6)
log10[(log10 x)(log10 x4 – 3)] = 0
log2(4x + 1) = log22x + log2(2x+3 – 6)
⇒ log2(4x + 1) = log2[2x[2x+3 – 6]] ⇒ (log10 x)(log10x4 – 3) = 1

⇒ 4x + 1 = 2x[2x23 – 6] (log10 x)(4log10 x – 3) = 1


Assume that 2x = y Assume that log10 x = y
⇒ y + 1 = y(8y – 6) ⇒ y + 1 = 8y – 6y
2 2 2
⇒ y(4y – 3) = 1; 4y2 – 3y = 1
⇒ 7y2 – 6y – 1 = 0 ⇒ (y – 1)(7y + 1) = 0 ⇒ 4y2 – 3y – 1 = 0 ⇒ (y – 1)(4y + 1) = 0
1 1
y = 1or y = – ⇒ y = 1 or y = –
7 4
M a them a ti cs | 1.39

1 1
log10 x = 1 or log10x = – ⇒ 1 + 2log2x + (log2x + 1)(log2x) +
1 4 4

⇒ x = 10 or x = 10 4 log2 (log2 x)3
+ log2x + (2) =1
1

for x ± 10 4 given log function is not defined. 1
⇒ 1 + 2log2x + (log2x)(log2x + 1) +
4
Hence, x = 10 + log2x + (log2x)3 = 1

Sol 30: log3(9x + 9) = log33x(28 – 2.3x) Assume log2x = y


1
⇒ 9x + 9 = 3x(28 – 2 . 3x) ⇒ 2y + y(y + 1) + + y + y3 = 0
4
Assume that 3x = y 1
⇒ y3 + 4y + y2 + =0
So 9 = (3 ) = (3 ) = y
x 2 x x 2 2 4
d 3 1
⇒ y2 + 9 = y(28 – 2y) ⇒ y2 + 9 = 28y – 2y2 Differential of equation is [y + 4y + y2 + ] = 0
dy 4
⇒ 3y2 – 28y + 9 = 0 ⇒ (3y - 1)(y - 9)= 0
–2 ± 22 – 4(4)(3)
1 ⇒ 3y2 + 4 + 2y = 0 ⇒ y = –
This gives y = 9, 2(3)
3
–2 ± –48 + 4
Hence, x = 2, –1 y=
6
No solution so there is no minima and maximum
Exercise 2 1
At y = 0 ⇒ f(y) = 0 + 0 + 0 + >0
4
Single Correct Choice Type 1
y = – 1, f (y) = (–1)3 + 4(–1) + (–1)2 +
4
1 1 1 1 1 15
Sol 1: (B) + + ⇒ – 1– 4 +1 + =–4+ =– <0
log bc
abc log ac
abc log ab
abc 4 4 4
It mean f(y) is zero some where –1 < y < 0
log10 bc log10 ac log10 ab
= + + So log2x < 0
log10 abc log10 abc log10 abc
But in equation (original) log2x should be positive so
there is no solution
log10 bc + log10 ac + log10 ab
=
log10 abc Sol 3: (C) x = (75)–10
 3
log10 bc ac ab log10 abc log10x = log10(75)–10 = – 10log1075 = −10 log10  100 × 
= = = 1  4
log10 abc log10 abc
= – 10[log1010 + + log103 – log102 ]
2 2

logx (log2 x +1) = – 10[2 + 0.477 – 2 (0.301)] = –18 75


Sol 2: (D) log2(2x2) + log2x. x
x 10−18.75
⇒= = 10−19 ×10−0.25
1 –3log1/2 (log2 x)
+ log42x4 + 2 =1
2 Number of zeros = 18
log (log x +1)
⇒ log2(2x2) + (log2x) (x) x 2
log2 3 log2 x
Sol 4: (D) 5x + 3 = 162
1 –3log1/2 (log2 x)
+ log4 41/2x4 + 2 =1 y log2 3 log2 2y
2 Assume x = 2y ⇒ 5.2 +3 162
=
logx (log2 x +1) log2 3y
⇒ 1 + 2log2x + (log2x) (x) ⇒ 5.2 + 3
y log2 2
= 162 ⇒ 5.3y + 3y = 6.3y = 162

1 4 162
3log (log x) 3y = = 27 = 33
+ log44 + log4x + 2 2 2 = 1 6
4 2
1 . 4 0 | Basic Mathematics

y = 3; x = 2y = 23 = 8 Sol 8: (C) x1 and x2 are roots of the equation


3
log4x = log48 = log4(4)3/2 = 2010 x
log2010 x
= x2
2
Assume that x = (2010)y
log2 x +log10 x3 +3
Sol 5: (B) (x) 10 y log2010 (2010)y
⇒ (2010)1/2 (2010) = (2010)2y
2
= = B (Assume) ⇒ (2010)1/2 (2010)y = (2010)2y
2
1 1

x +1 –1 x +1 +1 1 1
⇒ y2 + = 2y ⇒ y2 – 2y + =0
2 2 2 2
B= = 
1 1 x +1 +1 – x +1 +1
– 2 ± 22 – 4(1)(1 / 2) 2± 2 1
x +1 –1 x +1 +1 ⇒y= = = 1±
( x + 1 – 1)( x + 1 + 1) 2 2 2
1 1
1– 1+
B = ( ( x + 1 )2 – (1)2 = x + 1 – 1 = x x1x2 = (2010) 2 (2010) 2 = (2010)2 = (201 × 10)2
log2 x +3log10 x +3
So (x) 10 =x ⇒x=1 No. of zeros in x1x2 = 2

Or ⇒ Assume log10x = y
Sol 9: (A) Given that x = 2 or x = 3 satisfy the equation
⇒ y + 3y + 3 = 1 ⇒ y + 3y + 2 = 0
2 2
log4(x2 + bx + c) = 1 = log44
⇒ (y + 2)(y + 1) = 0 ⇒ x2 + bx + c – 4 = 0
y = – 2 or y = – 1 ⇒ b = 2 + 3 = 5 and c – 4 = 2 . 3 ⇒ c = 10

log10x = – 2 or log10x = – 1 bc = 10(– 5) = – 50


|bc| = 50
x = 10–2, 10–1

x1, x2, x3 = 1, 10–1, 10–2

x1 . x3 = 1.10–2 = (10–1)2 = (x2)2 JEE Advanced/Boards

Sol 6: (D) x = 2log3, y = 3log2 Exercise 1


x = 2log3 = 3log2 = y log6 18 log6 3
Sol 1: B = ( 2 ). (3 )
logn m logn a
As a =m log6 (6×3) log6 3 log6 6 +log6 3 log6 3
⇒B= 2 . 3 ⇒B= 2 . 3
2 –10x +3 1 +log6 3 log6 3 log6 3 log6 3
Sol 7: (B) | x – 3 |3x =1;x≠3 ⇒B= 2 3 =2× 2 . 3
log6 3
Or if |x – 3| = 1 ⇒ B = 2{6} = 2.3 = 6
2 2
⇒ x = 2 or 4 is solution A = log10 ab + (ab) – 4(a + b) + log10 ab – (ab) – 4(a + b)
2 2
If x – 3 ≠ 0 then 3x2 – 10x + 3 = 0 is another soln
 2 2 
A = log10  ab + (ab) – 4(a + b) × ab – (ab) – 4(a + b) 
3x – 10x + 3 = 0 ⇒ (3x 1)(x 3) = 0
2
 2 2 
 
+1
x = + 3 or =  2 2 2/2 
3 = log10  (ab) – ((ab) – 4(a + b)) 
1  4 
But x ≠ 3; so, x =
3
 2 2  4(a + b)
total solution ⇒ x =
1
, 2, 4 = log10  (ab) – (ab) + 4(a + b)  = log10
3 4 4
 
M a them a ti cs | 1.41

= log10(a + b) = log10(43 + 57) = log10100 = 2 = (log105)y2 + (log103)y2 + (log104)y2


⇒ A = 2 and B = 6 y3 = y2[log105 + log103 + log104]
Hence, AB = 12 ⇒ y3 = y2[log10(3.4.5)] = y2 log10 60
⇒ y = 0 or y = log1060
4 3
Sol 2: (a) log1/3 729 9 .27–1 –4/3
⇒ log10 x = 0 or y =log10x =log1060

= log1/3
4 3
729 3–2.3–4 ⇒ x = 1 or x = 60
Sum of roots = 1 + 60 = 61
4 4
= log1/3 729.3–2 = log1/3 81 = log1/33 = 1 Square of sum of roots = (61)2=3721

logb (logb N)
logb a 2 3
(b) a = ax say Sol 5: +
6
log4 (2000) log5 (2000)6
logb (logb N)
x= = loga(logb N) 2 3
logb a + 6log (2000)
6log4 (2000) 5
loga (logb N)
So ax = a = logbN
1  2 3 
=  + 
6  log (4 × 5 ) log (5 × 4 ) 
2 3 3 2
Sol 3: (a) logp2 + log2p  4 5 

1  2 3 
log2 log π log2 =  + 
⇒ + Assume that = x (+ve always) 6  log 42 + log 53 log 53 + log 42 
logπ log2 logπ  4 4 5 5 

1 1  2 3 
(2 < π < 10) ⇒ x + = c (Assume) =  + 
x 6  2 + 3log4 5 3 + 2log5 4 
c ± c2 – 4  
x2 – cx + 1 = 0 ⇒ x =
2  
1  2 3 
For x to be real c2 – 4 ≥ 0 = +
6  3log10 5 2log10 4 
2 + 3+ 
c2 ≥ 4 ⇒ c ≥ 2 ⇒ c = 2 ⇒ x = 1  log10 4 log10 5 
For all other value c > 2 (Not Possible)
1  2log10 4 3log10 5 
Here, logp2 + log2π is greater than 2 =  + 
6  2log10 4 + 3log10 5 3log10 5 + 2log10 4 
(b) For log35 and log27
Assume that log35 is rational ∴ log35= = a ⇒ 5 = 3a 1  2log10 4 + 3log10 5  1
=  
6  2log10 4 + 3log10 5  6
This is not possible when a is rational ∴ a is irrational
Similarly, log27 = b assuming b is rational gives 7 = 2b 3
1
Which is not possible, so b is irrational. log5 9 log
6
3  2 
81 +3  ( 7 )log25 7 – (125)log25 6 
Sol.6
409  
Sol 4: log3x . log4x . log5x = log3x.log4x + log4x. log5x +  
log5x log3x 9
2log9 5 3log3 6
+3  3
log 6 
=  ( 7 )2log7 25 – (25) 2 25 
Assume that log10x = y 409  
 
log10 x.log10 x.log10 x log9 52 log3 ( 6 )3
⇒ 9 +3 log7 25 log25 63/2
log10 3 log10 4 log10 5 = [7 – 25 ]
409
log10 xlog10 x log10 xlog10 x log10 x.log10 x
= + + ⇒ y3 52 + ( 6 )3 (52 )2 – (63/2 )2
log10 3log10 4 log10 4.log10 5 log10 5.log10 3 = [25 – 63/2] =
409 409
1 . 4 2 | Basic Mathematics

(25)2 – 63 409 Sol 10: We have to prove that


= = =1
409 409
ax – by = 0, where x= loga b

1
4 and y = logb a ⇒ x2 = logab
log1/5  
 2  + log
1
Sol 7: (5) 2
+ log1/2 1
7+ 3 10 + 2 21 y2 = logba ⇒ y2 = ⇒ x2y2 = 1
2
x
2 2
log 2  4  1 xy = 1 (x, y > 0) now ax – by = (b y )x – (ax )y
=5 5 + log 1   + log2–1
22
 7+ 3 10 + 2 21 ⇒ (bxy)y – (axy)x ⇒ by – ax ⇒ ax – by = by – ax = – (ax – by)

 4 
2 ⇒ ax – by + ax – by = 0 ⇒ 2(ax – by) = 0 ⇒ ax – by = 0
= 2 + log2   + log2 10 + 2 21
 7+ 3
log10 (x – 3) 1
2 Sol 11: (a) =
 4  16 16 2
log10 (x – 21) 2
=  = =
 7+ 3 7+3+2 7 3 10 + 2 21
⇒ 2log10(x – 3) = log10(x2 – 21)
16 (x – 3)2
= 2 + log2 (10 + 2 21 ) ⇒ log10(x – 3)2 log10(x2 – 21) = 0 ⇒ log10 =0
10 + 2 21 (x2 – 21)
= 2+ log2 24 = 2 + 4 = 6 (x – 3)2
⇒ = 1 ⇒ x2 + 32 – 2(3)x = x2 – 21
x2 – 21
30
Sol 8: log2 a = s ⇒ a = 2s ⇒ 9 – 6x = – 21 ⇒ 6x = 9 +21 ⇒ x = =5
6
2 2
log4 b = s2 ⇒ b = 4s = (2)2s (b) log(log x) + log(log x3– 2) = 0

1 2 ⇒ log[log x(log x3–2)] = 0 ⇒ (log x)(log x3 – 2) = 1


2 s3 +1
and log 2 8 = ⇒ 82 = c ⇒ (log x)(3 log x – 2) = 1 Assume that log x = y
c s3 + 1
s3 +1 3(s3 +1)
⇒ y(3y – 2) = 1 ⇒ 3y2 – 2y – 1 = 0
3/2 2
⇒c= (2 ) ; c= 2 4 1
⇒ 3y(y – 1) + 1(y – 1) = 0 ⇒ y = – or y = 1
3
2 2 1
a2b5 (2s )2 (22s )5 22s 210s 2 –3(s3 +1)) 1 –
Then = = = (2)(2s +10s ⇒ log10x = – or log10 x = 1 ⇒ x = (10) 3 or x = 101
c4 4 3 +1) 3
 (s3 +1)  23(s
 23 4  At x = 10–1/3 equation does not satisfy
 
 
  Hence, x = 10
2 5
⇒ log2 a b = (2s + 10s2 – 3(s3 + 1)) (c) logx 2 . log2x2 = log4x 2
c4
1 1 1
⇒ . =
log2 x log2 2x log2 4x
log2 24 log2 192
Sol 9: –
log96 2 log12 2 ⇒ log222+logx = (log2x)(log22+log2x)
1
⇒ We know that logm n = Assume log2x = y
logn m
⇒ 2 + y = y(1 + y) ⇒ 2 + y = y2 + y
⇒ (log296)(log224) – (log2192)(log212)
⇒ y2 = 2 ⇒ y = ± 2
Where, log2 24 = log2 12× 2= log2 12 + log2 2
log x = ± 2
⇒ log2 96(log2 12 + log2 2) – log2(96×2) log2 12
log2x = + 2 or log2x = – 2
⇒ log2 96. log2 12 + log2 96 − log2 96. log2 12 − log2 12
x = (2) 2
or x = 2– 2

( 3
)
⇒ log2 2 × 12 − log2 12 ⇒ 3 + log2 12 − log2 12 = 13
(d) 5
loga x
+ 5x
loga 5
= 3, (a > 0)
M a them a ti cs | 1.43

Assume x = ay No. of digits = 7 + 1 = 8 ∴ M = 8


loga ay y loga 5 loga 5y ⇒ log6 62 = 2 (characteristic 2)
⇒5 + 5a = 3 ⇒ 5y + 5a
1 ⇒ log6 63 = 3 (characteristic 3)
3
= 5 +5.5 = 6.5 = 3 ⇒ 5 =
y y y
= = 2–1 y
6 2 Total no. of positive integers which have the characteristic
Take logarithm (base 5) both side 2(between 62 and 63) = 63 – 62
= 216 – 36 = 180
⇒ log55y = log52–1 ⇒ y = log5 2–1
LMN = 16 × 8 × 180 = 23040
log 2–1
So x = a = y
a 5
– log5 a Sol 14: loga N.logb N + logb N.logcN + logc N.loga N … (i)
⇒x= 2
loga N.logb N.logc N
=
Sol 12: loga x loga(xyz) = 48 … (i) logabc N

loga y loga(xyz) = 12 … (ii) log y


We know that logx y =
logx
loga z loga(xyz) = 84 … (iii)
So, in equation (i) at R.H.S, we have
When sum of all equation is taken
logN logN logN
. .
loga(xyz)[loga x + loga y + loga z] loga logb logc (logN)2 logabc
= =
logN (loga)logb(logc)
= 48 + 12 + 84 = 144 = 122 logabc

(loga(xyz))(loga(xyz)) = 122 logN2 (loga + logb + logc)


=
loga logb logc
(loga xyz)2 = 122 ⇒ loga xyz = 12 (±1)
(logN)(logN) logN logN logN logN
In equation = + +
logb logc loga logc loga logb
(i) loga x (±12) = 48
= logaN logbN + logaN logcN + logbNlogcN
loga x = ±4 ⇒ x = a , a 4 –4
R.H.S. = L.H.S.
(ii) loga y(±12) = 12
10
Sol 15: x, y> 0 and logy x + logx y =
loga y = ±1 ⇒ y = a, a –1 3
log12 x log12 y 10
⇒ + =
(iii) loga z(±12) = 84 log12 y log12 x 3

loga z = ±7 ⇒ z = a7, a–7 log12 x


Assume that =a
log12 y
(x, y, z) = (a , a, a ) or (a ,a , a )
4 7 –4 –1 –7

1 10
⇒a+ = ⇒ 3a2 – 10a+ 3 = 0
Sol 13: Given a 3
1
L = antilog of 0.4 to the base 1024 ⇒ (3a 1)(a 3) = 0 ⇒ a = 3,  
3
⇒ L = (1024)0.4 = (210)0.4 = 24 = 16 log12 x
So = 3 ⇒ add + 1 both side
L = 16 log12 y
And M is the number of digits in 610
log12 x logx + log y
⇒ log10 610 = 10log10 6 ⇒ 10[0.7761] = 7.761 +1=3+1=4 ⇒ =4
log12 y log12 y
⇒ 6 = 1010 7.761
= 10 .10 7 0.761
1 . 4 4 | Basic Mathematics

log12 (xy) log12 122 2 ⇒


120 × 5x–3 × (2 – 5x– 4 )
=1
⇒ = =4 ⇒ =4
log12 y log y log12 y (1 – 5x–3 )2

2 1 120  5x  2(x −3)


log12 y = = ⇒ y = 121/2 ⇒ 5x 2 – 4
 = 12 + 5 – 2(5x–3)
4 2 53  5 
1 1 3
144 – 2– Assume that 5x = y
So x = =144 × 12 2 = 12 2 = 12 2
y
120  y  2× y
x+y ⇒ y 2 –  =1+y 5 –
2 –6

= N 5×5×5  25 × 25  53
2
(x + y)2 Multiply by 56
⇒ = N ⇒ x2 + y2 + 2xy = 4N
22 ⇒ 53 ×120y[2 – y 5–4] = 56 + y2 – 2x53y
⇒ (123/2)2 + (121/2) + 2(144) = 4 N 2
⇒ 53 × 240y – 120y = 56 + y2 – 2 × 53y
⇒ 12 + 12 + 2 × 144 = 4 N
3 5

⇒ 53 × 240y – 24y2 = 56 + y2 – 2 × 53y


4N = 2028 ⇒ N = 2028 ⇒ N = 507
4
54 × 48y – 25y2 = 56 – 10 × 52y
Sol 16: (a) log102 = 0.3010, log103 = 0.4771 Divide by 52 on the both side
⇒ 5200 = x (Assume) 52 × 48y – y2 = 54 – 10y

log10 x = log10 5200 = 200log105 ⇒ y2 – y(10 + 52 × 48) + 54 = 0

=200log10 10 =200(log1010–log102) ⇒ y2 –1210y + 625 = 0


2
= 200(1 – 0.3010) = 200(0.699) = 139.8 1210 ± (1210)2 – 4(1)(625)
⇒y=
2
⇒ x = 10139 × 100.8 1210 ± 1208.96
⇒y=
2
no. of digits in x = 139 + 1 = 140
y = 0.51675 or y = 1209. 48 (Rejected)
(b) x = 615 ⇒ log10x = log10 615 = 15 log10 6
5x = y = 0.51675 ⇒ x = log5 y
= 15(log2 + log3) =15 × (0.778) =11.67 Hence, x = – 0.410

∴ x = 1011.67 = 1011 100.67


Sol 18: Given that logx+1 (x2 + x – 6)2 = 4
No. of digits in x = 11 + 1 = 12
⇒ (x2 + x – 6)2 = (x + 1)4 ⇒ (x2 + x – 6) = ± (x + 1)2
(c) Number of zeros after the decimal in 3–100 = (x) When +ve case is taken → x2 + x – 6 = (x + 1)2
(Assume)
(and x2 + x – 6 ≥ 0)
log x = log3 –100
= – 100 log103 = – 100(0.4771) = – 47.71
x2 + x – 6 = x2 + 1 + 2x
So x = 10–47.71 = 10–47 × 10–0.71
x=–6–1=–7
∴ No. of zeros = 47
In the given equation, base is x + 1 = – 7 + 1 = – 6
which is negative
Sol 17: log5 120 + (x – 3) – 2log5(1 – 5x–3) = – log5(2– 5x–4)
So x ≠ – 7
⇒ log5120+(x–3)–log5(1–5x–3)2 + log5(2 – 5x–4) = 0 When -ve case is taken → x2 + x – 6 < 0

x–3 ⇒ x2 + x – 6 = – (x + 1)2 ⇒ x2 + x – 6 = – x2 – 1 – 2x
⇒ log5 120 × 5 × (2 – 5x– 4 ) =0
(1 – 5x–3 )2 ⇒ 2x2 + 3x – 5 = 0 ⇒ (2x + 5)(x – 1) = 0
M a them a ti cs | 1.45

5 Dividing E.q. (i) and (ii), we get


x=– or x = 1
2 (a – 2)(1 – 3b) = (1 – 2a)(b – 3)
5 ⇒ 2a(b – 3) + (a – 2)(1 – 3b) = b – 3
x=– also does not satisfy equation
2
⇒ 2ab – 6a + a – 3ab – 2 + 6b = b – 3
So x = 1
⇒ – ab – 5a + 5b + 1 = 0 ⇒ 5(b – a) – ab + 1 = 0

Sol 19: Given that x + log10(1 + 2x) = xlog105 + log106 ⇒ 5(a – b) + ab = 1

⇒ log1010x+log10(1+2x)=log105x+log106
Sol 22: log9 (9x 4 )log3 (3x) = log3 x3
⇒ log10[10 (1 + 2 )] = log10[5 6]
x x x

⇒ 10x (1 + 2x) = 6 5x ⇒ 10x + 20x = 5x 6


⇒ (1 + 4log3 x ) 1 + log3 x  = 3logx

Divide by 5x on the both the sides Assume that log3x = y


x x x
10 20 6 ×5 ⇒ (1 + 4y)(1 + y) = (3y)2 = 9y2
⇒ + = =6
x x x
5 5 5
⇒ 1 + 4y2 + 4y + y = 9y2 ⇒ 5y2 – 5y – 1 = 0
⇒ 2 + 4 = 6 ⇒ 2 + 22x = 6
x x x

Assume that 2x = y 5 ± 52 – 4(–1)(5) 5 ± 25 + 20


⇒y= =
2(5) 10
⇒ y + y2 = 6 ⇒ y2 + y – 6 = 0

⇒ (y – 2)(y + 3) = 0 ⇒ y = – 3 or y = 2 5 ± 45 5 ± 32 × 5 5±3 5
y= = =
10 10 10
⇒ 2x = –3 or 2x = 2 ⇒ 2x = - 3 is not possible so, 2x = 2
In equation (i) log3x > 0
Therefore, the real solution ⇒ x = 1
5+3 5
Hence, y =
Sol 20: 2log10(2y – 3x) = log10x + log10y 10
x
We have to find   Sol 23: Given that xyz = 1081
y
⇒ log10(2y – 3x)2 = log10(xy) ⇒ 4y2 – 12xy + 9x2 = xy (log10x)(log10yz) + (log10y)(log10z)= 468

Let x = ky We know that (a + b + c)2

⇒ 4y2 – 12ky2 + 9k2 y2 = ky2 ⇒ 9k2 – 13k + 4 = 0 = a2 + b2 + c2 + 2ab + 2bc + 2ca


4 = a2 + b2 + c2 + 2a(b + c) + 2bc  … (i)
⇒ (9k – 4)(k 1) = 0 ⇒ k = 1,
9
⇒ log10x(log10 y + log z) + (log10 y) (log10 z) = 468
If k = 1 ⇒ x = y ⇒ 2y – 3x is –ve
Assume that log10x = a, log10 y = b and log10 z = c
x 4
∴ = ⇒ a(b + c) + bc = 468
y 9
From equation (i)
Sol 21: We have a = log1218 and b = log2454
2a(b+c) + 2bc = (a+b+c)2 – (a2+b2+c2)
log2 18 2log2 3 + 1
⇒ a= = ⇒ 2a(b + c) + 2bc = 2 × 468 = 936
log2 12 2 + log2 3
⇒ (a + b + c)2 – (a2 + b2 + c2) = 936
⇒ (a – 2)log23 = 1 – 2a  ... (i)
⇒ a + b + c = log10x + log10y + log10z
log2 54 3log2 3 +1 = log10xyz = log101081 = 81
Similarly b = =
log2 24 3 + log2 3
⇒ 812 – (a2 + b2 + c2) = 936
⇒ (b – 3)log23 = 1 – 3b  ... (ii)
1 . 4 6 | Basic Mathematics

a2 + b2 + c2 = 812 – 936 = 5625 x+4  x  2 – x +1


2
log10   + log2  2
 = 2 log10  
⇒ (log10 x)2+ (log10 y)2 + (log10 z)2 = 5625  x  10  x + 4   x –1 
2
1  1
Sol 24: Sum of all solution of equation We know log10 = – log10x. So  log10  = (log10 x)2
9
x  x
(log9 x)2 – log9 x +5
⇒ [3] 2 =3 3
2 x+4 2 x+4 2 3−x
9 ⇒ log10   + log10   = 2 log10  
(log9 x)2 – log9 x +5  x   x   x –1
⇒ (3) 2 = (3)3/2
2 x+4 2 3– x
⇒ (log9x)2 –
9
log9 x + 5 =
3 log10   = log10  
2 2  x   x –1

Assume that log9 x = y x+4 3–x x 3– x


So = or = 
x x –1 x + 4  x –1
9 3 9 3 9 7
⇒ y2 – y + 5 = ⇒ y2 – y + 5 – = y2 – y + =0
2 2 2 2 2 2 x2 + 4x – x – 4 =3x – x2 or x2 –x =3x+12–x2– 4x
⇒ 2y2 – 9y + 7 = 0 ⇒ 2x2–4=0 or 2x2 = 12 ⇒ x2 = 2 or x2 = 6
7
⇒ (2y – 7)(y – 1) = 0 ⇒ y = ; y=1 x = ± 2 or x = ± 6
2
7
log9x = log9 x = 1 x= 2 and – 6 do not satisfy equation
2
⇒ x = (9)7/2 = 37; x = 9 So x = 2, 6
Sum of solution = 37 + 9 = 2196
Sol 28: log3( x + | x − 1 | )=log9(4 x –3 + 4 | x − 1 | )
Sol 25: a, b, c, d > 0
1
3 5 log3 ( )
x+ | x −1 | =
2
(
log3 4 x + −3 + 4 | x − 1 | )
 loga b = and logc d = , a – c = 9
2 4
log10 b 3 log10 d 5 ⇒ 2log3( x +| x –1|)=log3(4 x –3+4| x –1|)
= ; =
log10 a 2 log10 c 4
⇒ log3( x +| x –1|)2=log3(4 x –3+4|( x –1|)
2log10 b = 3 log10 a
⇒ ( x + | x – 1|)2 = (4 x – 3 + 4 | x – 1|)
4 log10 d = 5 log10 c
3 5
x+( x –1)2+2 x | x – 1| = 4 x – 3 + 4| x –1|
b= a2 ,d= c4
(i) Assume ( x – 1) < 0
∴ a should be perfect square and c should be perfect
power of 4 ⇒ | x – 1| = 1 – x
Let a = 25, c = 16
⇒ x + x + 1 – 2 x + 2 x (1 – x ) = 4 x – 3 + 4(1 – x)
∴ b = (5)3 = 125 ⇒ d = (16)5/4 = 32 ∴ b – d = 93
⇒1+2x – 2 x + 2 x – 2x = 4 x – 3 + 4 – 4 x
Sol 26: Refer Sol 11 of Ex 2 JEE Main
1 = 1 always correct

Sol 27: So x – 1 < 0 and x > 0


2  4 2  4  2  2 
log10 1 +  + log10 1 –  =2 log10  – 1 x<1
 x   x + 4   x − 1 
⇒ x ∈ (0, 1) and if x – 1 ≥ 0, x >0
2 x + 4 2 x + 4 – 4 2  2 – (x – 1) 
log10   + log10   =2 log10   x + x + 1 – 2 x + 2 x ( x – 1) = 4 x – 3 + 4 ( x – 1)
 x   x+4   x −1 
M a them a ti cs | 1.47

⇒ 2x + 1 – 2 x + 2x – 2 x =4 x – 3 – 4 + 4 x Sol 30: [log3 (3x)1/3 + logx (3x)1/3 ]log3 x3 +


⇒ 4x + 1 + 7 – 4 x = 8 x ⇒ 4x – 12 x + 8 = 0  1 1
  x 3  3 3  3
⇒ x – 3 x + 2 = 0 ⇒ ( x – 2)( x –1) = 0 log3  3  + logx  x   log3 x
     
⇒ x – 2 = 0 or x – 1 = 0 ⇒ x = 4 or x = 1  
Assume that
Put condition was ⇒ x –1≥0
1 1 
So x = [0, 1] ∪ {4} A=  log3 (3x) + logx (3x) log3 x
3

 3 3 

Sol 29: 3
⇒ (log3 x + 1) + (logx 3 + 1) log3 x
 4 4 4 4 
 loga ab +logb ab – loga b/a +logb a/b . loga b 3
2  = 2x
A= (2log3 x + (log3 x)2 + 1)
 1 
 (loga (a × b) + logb (a × b)) 
 4
⇒x=  log b A = |log3x + 1|
 a
1
 – (loga ba–1 + logb ab –1 ) 
 4  x 1 1 3 
And B =   log3  +  logx   log3 x3
 33 3 x 
1  1 + loga b + 1 + logb a 
x=   loga b
2   3
 – –1 + log b – 1 +log a
b  ⇒ log x – 1 + logx 3 – 1 log3 x
3 3
a

  B= ((log3 x)2 – 2log3 x + 1)


1  2loga b + 1 + (loga b )
2

x=  
2 
– –2loga b + (loga b ) + 1 
2
  B= (log3 x – 1)2 = |log3x – 1|

1 A + B =2 ⇒ |log3x +1| + |log3x – 1| =2


We know loga b =
logb a
log3x ≥ 1 ⇒ x ≥ 3
1   A + B ⇒ log3x + 1 + log3x – 1 = 2log3x = 2
(loga b – 1)
2 2
x=  (1 + loga b) – 
2   log3x = 1 ⇒ x = 3
1 x ≥ 3 and x = 3 ⇒ x = 3
x= (|1+ loga b| – |loga b – 1|)
2 1
If log3x < 1 and log3x + 1 > 0 ⇒ x < 3 and x >
3
When loga b ≥ 1 ⇒ b ≥ a > 1
A + B ⇒ log3x + 1 – (log3x – 1)
1 1
x = (1+logab – logab +1) = ×2 = 1 = log3x + 1 – log3x + 1 = 2 = 2(always)
2 2
so 2x = 21 = 2 (when b ≥ a > 1) 1 
So x ∈  , 3 
When loga b < 1 3 
1
⇒ b < a, a, b > 1 log3x ≤ – 1 ⇒ x ≤
3
1 A + B = – (log3x + 1) – (log3x – 1)
⇒x= [1 + logab – (1 – logab)]
2
= – log3x –1 – log3x + 1 = – 2log3x = 2
1 1
x = [1 + logab + loga b] = 2logab 1
2 2 ⇒ log3x = – 1 ⇒ x = 3–1 =
3
x = loga b
1 1 1
log b
x≥ and x = ⇒x=
2x = 2 a (if 1 < b < a) 3 3 3
1 . 4 8 | Basic Mathematics

1  |x – 4| = x – 4
So x =  ,3 – {1}
3 
Eq. → x + 1 + x – 4 = 2x – 3 = 7
x ≠ 1 because base can’t be 1
7+3 10
⇒ 2x = = 10?2x = 10 ⇒ x = =5
1 2
Sol 31: a = (log781)(log6561625)(log125216)(log12962401) If – 1 < x < 4

⇒ a = (log734) (log 8 54 ) (log 3 63 ) (log 74 ) ⇒ |x + 1| → 1 + x


3 5 64

4 3  4  |x – 4| → 4 – x
⇒ a = 4(log73) (log35)(log56)     log67
8 3  4  ⇒ 1 + x + 4 – x= 5 ≠ 7
2log10 3 log10 5 log10 6 log10 7 So no solution for this region → x = 5 and – 2
⇒ a = 2
log10 7 log10 3 log10 5 log10 6 But – 2 is not natural no.
⇒ and b = sum of roots of the equation So c = 5

x
log2 x
= (2x)
log2 x a+b=2+3=5
5
log2 x log2 x1/2 (a + b) ÷ c = =1
x = (2x) 5

Take logarithm (base x) both sides


1/2 Exercise 2
logx xlog2 x = logx (2x)log2 x
Single Correct Choice Type
(log2x)(1) = log2 x1/2[logx(2x)]
1 x+ y
log2x = log2x(logx 2 + 1) Sol 1: (C) 2 =256 and log10 xy –log101.5 = 1
2
x+ y
log2x = 0 ⇒ x = 1 or 2 = logx 2 + 1 ⇒ 2 = 256 = 28

logx 2 = 1 ⇒ x = 2 ⇒ x + y =8 … (i)

x1 + x2 = 1 + 2 = 3 and log10 xy = 1 + log101.5 = log1010 + log101.5

b=3
log10 xy = log10(10 × 1.5) = log1015
and c = sum of all natural solution of equation
⇒ xy = 15 ⇒ xy = 152 = 225
|x + 1| + |x – 4| = 7

I II III | x– y|= ( x + y )2 – 4 xy
 
-1 4
= 82 – 4 × 15 = 64 – 60
If x < – 1 → |x + 1| = – 1 – x
| x– y|= 4 =2
|x – 4| = 4 – x
x+ y =8
⇒ eq.→ – 1 –x + 4 –x = 3 – 2x = 7
4 ⇒ If x > y ⇒ (x, y) = (25, 9)
⇒ 2x = 3– 7 = – 4 ⇒ x = – =–2
2 ⇒ If x < y ⇒ (x, y) = (9, 25)
If x > 4 → |x + 1| = x + 1
M a them a ti cs | 1.49

Sol 2: (B) log16 y ≠ 1 ⇒ y ≠ 16


A
log4(logy16) > 0

c logy16 > 1 ⇒ y < 16


b
logy 16 > 0

⇒ a = 2, b = 16, c = 1
C B
a
a + b + c = 2 + 16 + 1 = 19
⇒c =a +b
2 2 2
⇒c –b =a
2 2 2

1 1 1 1 n2 nN 5n2 nN


loga a loga a Sol 5: (D) . = . . = . . .
+ log2 N logN 8 log32 N logN 128 nN 3n2 nN 7n2
logb + c a + logc–b a loga (b + c ) loga ( c – b )
=
logb + c a .logc–b a loga a loga a log2 logN 5log2 logN 5
= ⋅ ⋅ ⋅ =
loga (b + c ) loga ( c – b ) logN 3log2 logN 7log2 21

= (loga ( c – b ) + loga (b + c ) ) = loga(c2–b2) = 2


Sol 6: (B) N = 10p ; p = log10 8 − log10 9 + 2log10 6

Sol 3: (B) B, C, P, and L are positive number  8.36 


=p log
=   log10 32
 9 
∴ log(B.L) + log(B.P) = 2; log(P.L) + log(P.C) = 3
log 32
∴ N 10 =
= 10
32
and log(C.B) + log(C.L) = 4
Hence characteristics of log332 is 3
Adding all the above equations, we have
Sol 7: (C) log2  ( x + y ) − xy 
2
log[B.L.B.P.P.L.P.C.C.B.C.L] = 2 + 3 + 4 = 9
 
log(BCPL)3 = 9 ⇒ 3logBCPL = 9 10 − 2
But x + y= 2; xy
= = 2
9 4
⇒ log BCPL = =3
3 log2 (10 − 2=
) log2=
8 3
∴ BCPL = 103
5 3 5 3 5 25 3 5
log12 (log8 (log4 x)) Sol 8: (A) Let x = + + − ⇒ x2 = +2 − = +2
Sol 4: (B) =0 4 2 4 2 2 16 2 2
log5 (log4 (logy (log2 x)))
5 3 5 3 5 25 3 5 1
c < y < b, y ≠ a + + − x= ⇒ x2 = +2 − = + 2. = 3
4 2 4 2 2 16 2 2 4
where ‘b’ is as large as possible and ’c’ is as small as π
possible. ⇒ x= 3 = tan
3
⇒ log12(log8(log4x)) = 0 ⇒ log8(log4x) = 1 = log88
Sol 9: (D) y = |2x – |x – 2||=|2x – (2 – x)|=|3x – 2| as x < 0
log4 x = 8 ⇒ x = 48 = 22×8 = 216
Hence y = 2 –3x
and log5(log4(logy(log2x))) ≠ 0
x
⇒ log5(log4(logy(log2 216))) ≠ 0  1 
Sol 10: (D) 3x(0.333 ….)(x – 3) ≤  
 27 
⇒ log5(log4(logy16)) ≠ 0, y ≠ 1 x–3
1 x 3x
1  1
⇒ log4(logy 16) ≠ 1 ⇒ logy16 ≠ 4 ⇒ 3x  3  ≤   =  
  3
3  3
1 1 1
⇒ log y ≠ ⇒ log2y ≠ ⇒ log2y ≠ 1 ⇒ y ≠ 2 3x
1
24 4 4 4 ⇒3 3 x –(x–3)
=3 .3
x 3–x
≤  
3
log4(logy16) ≠ 0 ⇒ logy 16 ≠ 1
1 . 5 0 | Basic Mathematics

1
3x Multiple Correct Choice Type
3 = 27 ≤   =3–3x
3

3
1 + 2log3 2
3 ≤ – 3x ⇒ –x ≥ 1 ⇒ x ≤ –1 Sol 14: (C, D) N = + log26 2
(1 + log3 2)2
x ∈ [–∞, –1]
2
1 + 2log3 2  log3 2 
N= + 
2x +1
–3
(1 + log3 2)2  log 6 
 1  1–x 1  3 
Sol 11: (B)   >  
5 5
Assume that log32 = y
2x + 1
<–3
1–x 1 + 2y y2
⇒N= +
2x + 1 < – 3(1 –x) = – 3 + 3 x (if (1 –x) > 0) (1 + y)2 (log3 2 + log3 3)2

⇒ 2x + 1 < – 3 + 3x ⇒ 3x – 2x > 1 + 3 = 4 1 + 2y y2 y 2 + 2y + 1
⇒N= + =
⇒ x > 4 ⇒ x > 4 and x < 1 which implies no solution (1 + y)2 (1 + y)2 (1 + y)2
2x + 1 (1 + y)2
If x > 1 ⇒ 1 – x < 0 ⇒ <–3 ⇒N= =1
1–x
(1 + y)2
2x + 1
⇒ > – 3(1 – x) = 3x – 3
1 And π = 3.147 > 3 and 7 > 6

⇒ 3x – 2x < 1 + 3 = 4 ⇒ x < 4 and x > 1 ⇒ x ∈ (1, 4) So, log3 π > 1 and log7 6 < 1

log3 x2 +(log3 x)2 –10 1 Sol 15: (A, D) 22x –8.2x = – 12


Sol 12: (D) x = = x–2
2
x
Assume that 2x = y
⇒ log3x2 + (log3x)2 – 10 = – 2
⇒ y2 – 8y = – 12 ⇒ (y – 6)(y – 2) = 0 ⇒ y = 6 or y = 2
Assume log3x = y → 2y + y2 – 10 = – 2
⇒ y2 + 2y – 10 + 2 = y2 + 2y – 8 = 0 2x = 6 ; 2x = 21
x log102 = log106 = log10(2 × 3)
⇒ (y + 4)(y – 2) = 0 ⇒ y = –4 or y = 2

1 log10 2 + log10 3 log10 3


x = 3–4 = ;x=9 x= 1+
= ;x=
1
81 log10 2 log10 2
 1
x = 1, 9,  x x
81
  Sol 16: (A, B, C, D)  5 2 – 7  + 6  5 2 + 7  = 7
   
(l n) 2 - 3 l nx + 3 Assume x = log y
Sol 13: (A) <1
l nx - 1 5 2 –7

If ln x – 1 > 0 ⇒ ln x > 1 ⇒ x > e log y log y


⇒  5 2 – 7  +6  5 2 + 7 
5 2 –7 5 2 –7
=7
⇒ (ln x)2 – 3ln x + 3 < 1[(ln(x)) –1]    

Assume ln x = y 5 2 +7
5 2 –7 = 5 2 –7×
⇒ y – 3y + 3 < y – 1 ⇒ y – 3y – y + 3 + 1 < 0
2 2
5 2 +7
⇒ y2 – 4y + 4 < 0 ⇒ (y – 2)2 < 0 always false 50 – 49 –1
= =  5 2 + 7 
So if ln x < 1 ⇒ x < e and x > 0 5 2 +7  

y2 – 3y + 3 > (y – 1) ⇒ y2 – 3y – y + 3 + 1 > 0 − log y


⇒ y + 6  5 2 + 7 
5 2 +7
= 7 ⇒ y + 6y–1 = 7
y2 – 4y + 4 > 0 ⇒ (y – 2)2 > 0 always true  

So, x ∈ (0, e) ⇒ y2 + 6 = 7y ⇒ y2 – 7y + 6 = 0
M a them a ti cs | 1.51

⇒ (y – 6)(y –1) = 0 which gives y = 6 or y = 1


⇒ 17 – 2x = (x – 1)

x = log 6 or x = log 1= 0 Squaring both sides


5 2–7 5 2–7

⇒ 17 – 2x = (x – 1)2 = x2 – 2x + 1
⇒ x = log 6 = 2log(5 6 = log(5 36
(5 2 –7)1/2 2 –7) 2 –7)
⇒ 17 = x2 + 1 ⇒ x2 = 16⇒ x = ± 4
2 –2 ⇒ x ≠ – 4 does not satisfy equation in statement-I
x= =
log6 (5 2 – 7) log6 (5 2 + 7) So x = 4. x has a solution
Statement-II

Assertion Reasoning Type “Change of base in logarithm is possible” which is true


but not the correct explanation for statement-I.
Sol 17: (D) Statement-I
log5 (x3 +1)
/2 Sol 19: (B) Statement-I: 5 – x2 = 1 have two
distinct real solutions.
loga N
cos -ve cos +ve Statement-II: a = N when a > 0 a ≠ 1, N > 0
 2, 0 log5 x3 +1
⇒5 – x2 = 1
cos -ve cos +ve
log5 (x3 +1)
[5 = x3 + 1] from statement-II
3/2
⇒ x3 + 1 – x2 = 1 ⇒ x3 – x2 = 0

logx cos(2πx) is a meaningful quantity only if ⇒ x3 = x2 ⇒ x = 0 or 1


Statement-I is true and II is true and II is not the correct
 1 3 
x ∈  0,  ∪  ,1  explanation for statement -I.
 4 4 
π
cos 2px > 0 ⇒ > 2px > 0 Comprehension Type
2
1
> x > 0 and x ≠ 1 , x > 0 Paragraph 1:
4
3π 3 Sol 20: (D) log3x 10 – 6 log2x 10 + 11 logx 10 – 6 = 0
< 2px < 2p ⇒ < x < 1
2 4
Assume that logx 10 = y
 1 3  ⇒ y3 – 6y2 + 11y – 6 = 0
So x ∈  0,  ∪  , 1 
 4 4 
f(y) = y3 – 6y2 + 11y – 6
But also logx cos(2px) > 0 = logx 1
df(y)
cos 2px > 1 which is never possible = 3y2 – 12y + 11 → 0
dy
So statement-I is false
12 ± 122 – 4 × 3 × 11 12 ± 12
⇒y= =
Statement-II If the number N > O and the base of the 2(3) 6
logarithm b(greater than zero not equal to)
There is maxima and minima at
Both lie on the same side of unity than logb N > 0 and
if they lie on the different side of unity then logb N < 0 12 ± 12 6 2 2 1
y= =2± = 2± = 2±
statement-II is true 6 6 6 3
1
Sol 18: (B) Statement-I At y = 2+
3
1
log2(2 17 – 2x ) = 1 + log2(x –1) has a solution y – 6y + 11y – 6 is negative and at y = 2 –
3 2
,
3
⇒ 1 + log2( 17 – 2x ) = 1 + log2(x – 1) Equation y3 – 6y2 + 11y – 6 is positive
1 . 5 2 | Basic Mathematics

So there is total 3 solutions for this equation Area of triangle is

f(y) Area = a2 + b2 – c2

Also, we have b2 = a2 + c2
y
1 ac
So area = a2 + (a2 + c2) – c2 = ×a×c=
2 2
ac ac a
⇒ 2a2 = ⇒4= =
2 a2 c
c
⇒ ratio = =4
Match the Columns a
(D) a, b, c ∈ N
Sol 21: A → q, r, s; B → p, q, r, s; C → p; D → r
∴ ((4)1/3 + (2)1/3 – 2)(a(4)1/3+b(2)1/3+c)=20
(A)
= (22/3+21/3–2)(a22/3 + b21/3+c)=20
3 x – 7x + 4x – 1 2x + 4x – 1
⇒ a(24/3 + 2 – 2.22/3) + b[23/3 + 22/3 – 2.21/3]
+ c(22/3+21/3–23/3)=20
3 x + 7x + 4x – 1 = 13
⇒ 21/3(2a – 2b + c) + 23/3(a + b – c)
(3 x – 7x + 4x – 1 )(3 x + 7x + 4x – 1 )  + 22/3(–2a + b + c)= 20
20
⇒a+b–c= = 10
( 2x + 4x – 1 ) 2
2
(3 x ) ( )
2
= –  7x + 4x – 1  2x + 4x – 1
  Sol 22: A → p; B → p, r, s; C → p, r; D → p, q, r

= (9x − 7x − 4x − 1 )(2x + 4x − 1 ) (A) x = log2 log9 6 + 6 + 6 + ......∞

Assume that x = log2 log9y


= (2x − 4x − 1 )(2x + 4x − 1 )
⇒y= 6 + 6 + 6 + ......∞ = 6+y
( 2x ) − ( 4x − 1) =
2
( 4x )
2
= 13 ⇒ − 4x − 1 =
13
⇒ y2 = 6+ y ⇒ y2 – 6 – y = 0
14
⇒ ( 2x − 1 ) = 13 ⇒ x = = 7
2 ⇒(y – 3)(y + 2) = 0 ⇒ y = 3 or y = – 2, y ≠ – 2
(B) P (x) = x7 – 3x5 + x3 – 7x2 + 5 ∴ y=3
Q (x) = x – 2 x = log2 log93 = log2 log9(9)1/2
P(x)
Remainder ⇒ x = log2  1  = log2 2–1 = – 1
Q(x)  
2
Q (x) = 0 at x = 2
⇒ x = – 1 is an integer
So P(2) = 2 – 3(2) + 2 – 7(2) + 5 = 17
7 5 3 2
(log2 3.log3 4.log4 5......log99 100)
(B) N = 2
(C)
N = 2x (Assume)
a b
log3 log 4 log100 log100
⇒ x= . … = =log2100
log2 log3 log99 log2
c log2 100
N= 2 = 100
M a them a ti cs | 1.53

N = 100 which is a composite, integer, natural number


1 1 1
(C) +
log5 3 log6 3 log10 3

log5 log6 log10  log5 + log6 – log10 


⇒ + – = 
log3 log3 log3  log3 

log(5 × 6 ÷ 10) log3


⇒ = =1
log3 log3

⇒ 1 is natural and integer number

(D) N = 2 + 5 – 6 – 3 5 + 14 – 6 5

N = 2 + 5 – 6 – 3 5 + (3 – 5)2

N= 2 + 5 – 6 – 3 5 + (– 5 + 3) = 2+ 5 – 9– 4 5

N= 2 + 5 – ( 5)2 + (2)2 – 2(2) 5

N= 2 + 5 – ( 5 – 2)2 = 2 + 5 – 5 + 2 = 4 = 2

2 is natural prime and an integer.


2017-18 100 &
op kers
Class 11 T
By E ran ulty
-JE Fac
IIT enior emier
S f Pr tes.
o titu
Ins

MATHEMATICS
FOR JEE MAIN & ADVANCED
SECOND
EDITION

Exhaustive Theory
(Now Revised)

Formula Sheet
9000+ Problems
based on latest JEE pattern

2500 + 1000 (New) Problems


of previous 35 years of
AIEEE (JEE Main) and IIT-JEE (JEE Adv)

5000+Illustrations and Solved Examples


Detailed Solutions
of all problems available

Topic Covered Plancess Concepts


Tips & Tricks, Facts, Notes, Misconceptions,
Quadratic Equations Key Take Aways, Problem Solving Tactics
and Inequalities PlancEssential
Questions recommended for revision
2. QUADRATIC EQUATIONS
AND INEQUALITIES

1. QUADRATIC POLYNOMIAL
Quadratic polynomial: A Polynomial of degree 2 in one variable of the type f ( x ) = ax2 + bx + c where a, b, c, ∈ R
and a ≠ 0 is called a quadratic polynomial. ‘a’ is called the leading coefficient and ‘c’ is called the absolute term of
f (x). If a = 0, then y = bx + c is called a linear polynomial and if=a 0, b ≠ 0 & c = 0 then y = bx is called an odd
linear polynomial since f ( y ) + f ( − y ) =
0
Standard appearance of a polynomial of degree n is f ( x =
) anXn + an−1 X x −1 + an−2 Xn−2 + .... + a1 X + a0
Where an ≠ 0 & an , an−1 ,....a0 ∈ R ; n = 0, 1, 2…
When the Highest exponent is 3 → It is a cubic polynomial
When the Highest exponent 4 → It is a biquadratic polynomial
For different values of a, b, and c there can be 6 different graphs of y = ax2 + bx + c
y y
y
a>0 a>0
a>0 D<0
D>0 D=0

x1 x2
O x O x1 x2 x O x

Figure 2.1 Figure 2.2 Figure 2.3

y y
x y x
O O x O
x1 x2
a<0 a<0
a<0
D>0 D<0
D=0

Figure 2.4 Figure 2.5 Figure 2.6

Figure 1 and figure 4 ⇒ x1 , x2 are the zeros of the polynomial


Figure 2 ⇒ zeros of the polynomial coincide, i.e ax2 + bc + c is a perfect square; y ≥ 0 ∀ x ∈ R
Figure 3 ⇒ polynomial has no real zeros, i.e the quantity ax2 + bx + c > 0 for every x ∈ R
Figure 5 ⇒ zeros of the polynomial coincide, i.e ax2 + bx + c is a perfect square; y ≤ 0 ∀ x ∈ R
Figure 6 ⇒ polynomial has no real zeros; ax2 + bx + c < 0, ∀ x ∈ R < 0
2 . 2 | Quadratic Equations and Inequalities

2. QUADRATIC EQUATION
A quadratic polynomial expression equated to zero becomes a quadratic equation and the values of x which satisfy
the equation are called roots/ zeros of the Quadratic Equation.
General form: ax2 + bx + c =0
Where a, b, c, ∈ R and a ≠ 0 , the numbers a, b and c are called the coefficients of the equation.
a is called the leading coefficient, b is called the middle coefficient and c is called the constant term.
e.g 3x2 + x + 5 = 0, − x2 + 7x + 5 = 0, x2 + x = 0, x2 = 0

2.1 Roots of an Equation


The values of variables satisfying the given equation are called its roots.
In other words, x = α is a root of the equation f(x), if f ( α ) =0 . The real roots of an equation f(x) = 0 are the
x-coordinates of the points where the curve y = f(x) intersect the x-axis. e.g. x2 − 3x + 2 = 0 . At x = 1 & 2 the
equation becomes zero.
Note: A Polynomial can be rewritten as given below
y =a ( x − r1 )( x − r2 ) .... ( x − rn )
The factors like ( x − r1 ) are called linear factors, because they describe a line when you plot them.

2.2 Dividing Polynomials


Dividing polynomials: When 13 is divided by 5, we get a quotient 2 and a remainder 3.
Another way to represent this example is : 13 = 2 × 5 + 3
The division of polynomials is similar to this numerical example. If we divide a polynomial by (x – r), we obtain a
result of the form:
F(x) =(x – r) q(x) + R, where q(x) is the quotient and R is the remainder.

Illustration 1: Divide 3x2 + 5x − 8 by (x – 2) (JEE MAIN)

Sol: Similar to division of numbers, we can write the given


3x +11

x − 2)3 x2 + 5x − 8
3x2 − 6x
11x − 8
11x − 22
14

( x − 2) q(x) + R .
polynomial as 3x2 + 5x − 8 =
Thus, we can conclude that 3x2 + 5x − 8 = ( x − 2 )( 3x + 11 ) + 14
Where the quotient q ( x=
) 3x + 11 and the remainder R = 14.
M a them a tics | 2.3

3. REMAINDER AND FACTOR THEOREM

3.1 Remainder Theorem


Consider f ( x ) =
(x − r ) q(x) + R
Note that if we take x = r, the expression becomes
f (r ) =
(r − r ) q (r ) + R ; ⇒ f (r ) =
R
This leads us to the Remainder Theorem which states:
If a polynomial f ( x ) is divided by (x – r) and a remainder R is obtained, then f (r ) = R .

Illustration 2: Use the remainder theorem to find the remainder when f ( x ) = 3x2 + 5x − 8 is divided by (x – 2)
 (JEE MAIN)

Sol: Use Remainder theorem. Put x = 2 in f(x). Since we are dividing f ( x ) = 3x2 + 5x − 8 by (x – 2), we consider x = 2.
Hence, the remainder R is given by
R= f ( 2=
) 3 ( 2 ) + 5 ( 2 ) − 8= 14
2

This is the same remainder we arrived at with the preceding method.

Illustration 3: By using the remainder theorem, determine the remainder when 3 ( x ) − x2 − 20x + 5 is divided by
3

(x + 4) (JEE MAIN)

Sol: As in Illustration 2, we can solve this problem by taking r = - 4


f(x) = 3 ( x ) − x2 − 20x + 5 .
3

Therefore the remainder R = f( −4) =3 ( −4 ) − ( −4 ) − 20 ( −4 ) + 5 =−192 − 16 + 80 + 5 =−123


3 2

3.2 Factor Theorem


The Factor Theorem states:
If the remainder f (r )= R= 0 , then (x – r) is a factor of f ( x ) .
The Factor Theorem is powerful because it can be used to calculate the roots of polynomial equations having
degree more than 2.

(
Illustration 4: Find the remainder R by long division and by the Remainder Theorem 2x 4 − 10x2 + 30x − 60 ÷ ( x + 4 ) . )
 (JEE MAIN)
3 2
Sol: We can find the remainder in the given division problem by using 2x - 8x + 22x - 58
4 3 2
the long division method, i.e. similar to number division and also by x+4 2x + 0x - 10x + 30x - 60
the Remainder theorem, i.e. R = f(r). 4 4
-2x - 8x
Now using the Remainder Theorem: f ( x ) = 2x − 10x + 30x − 60
4 2
3 2
-8x - 10x
Remainder = f ( −4 ) = 2 ( −4 ) − 10 ( −4 ) + 30 ( −4 ) − 60 = 172
4 2
3 2
8x + 32x
This is the same answer we achieved by the long division method.
2
22x + 30x
2
-22x - 88x
-58x - 60
58x + 232
172
2 . 4 | Quadratic Equations and Inequalities

Illustration 5: Use the factor theorem to decide if (x – 2) is a factor of f ( x ) = x5 − 2x 4 + 3x3 − 6x2 − 4x + 8 .


 (JEE MAIN)
Sol: We know that (x – r) will be a factor of f(x) if f(r) = 0. Therefore, by using this condition we can decide whether
(x – 2) is a factor of the given polynomial or not.
f ( x ) = x5 − 2x 4 + 3x3 − 6x2 − 4x + 8
f ( 2=
) ( 2) − 2 ( 2 ) + 3 ( 2 ) − 6 ( 2 ) − 4 ( 2 ) + 8= 0
5 4 3 2

Since f ( 2 ) = 0 , we can conclude that (x – 2) is a factor.

Illustration 6: If x is a real number such that x3 + 4x =


8 , then find the value of the expression x7 + 64x2 . 
 (JEE MAIN)
Sol: Represent x7 + 64x2 as a product of x3 + 4x − 8 =0 and some other polynomial + constant term. The value of
the expression will be equal to the constant term.
Given x3 + 4x − 8 =
0;
Now y =x7 + 64x2 =x 4 (x3 + 4 x − 8) − 4 x5 + 8x 4 + 64x2

= ( )
−4x2 x3 + 4x − 8 + 16x3 + 64x = (
16 x3 + 4x − 8 + 128 =
128 )
3
Alter: x + 4x =
8
x7 + 64x2
Now divide x 7 + 64 x2 by x3 + 4x − 8 ⇒
x 3 +4x−8
Here, after division, the remainder will be the value of the expression x7 + 64x2 .
Thus, after dividing, the value is 128.

Illustration 7: A cubic polynomial P(x) contains only terms of the odd degree. When P (x) is divided by (x – 3), then
( )
the remainder is 6. If P(x) is divided by x2 − 9 , then the remainder is g(x). Find the value of g(2).  (JEE MAIN)

Sol: Let p(x) = ax3 + bx , and use Remainder theorem to get the value of g(2).
Let p(x) = ax3 + bx ; By remainder theorem P(3) =6
P(3) = 3(b + 9a) = 6 ; 9a + b = 2  …..(i)

( 2
)
P(x) = x − 9 ax + (b + 9a) x
Given that the remainder is g(x) when P(x) is divided by x2 − 9 ( )
∴ g(x) =+
(b 9a) x
From (i) (b + 9a) = 2 ∴ g(x) =
2x ∴ g ( 2 ) =
4

4. METHODS OF SOLVING QUADRATIC EQUATIONS


There are two methods to solve a Quadratic equation
(i) Graphical (absolute) (ii) Algebraic

Algebraic Method
ax2 + bx + c =0 ; Divide by a
2
bx c  b  b2 c b2 − 4ac
x2 + + =0 ⇒  x +  = − =
a a  2a  4a2 a 4a2
M a them a tics | 2.5

b b2 − 4ac −b ± b2 − 4ac
x+ =
± ; ⇒ x =
2a 2a 2a

b2 − 4ac =
D (Discriminant)
−b + D −b − D −b c
α= ,β = ; α + β = , α.β =
2a 2a a a
ax2 + bx + c= 0 ⇒ x2 − ( α + β ) x + ( α.β )= 0

5. NATURE OF ROOTS
Given the Quadratic Equation ax2 + bx + c =0 , where a, b, c, ∈ R and a ≠ 0
Discriminant: D = b2 − 4ac

D<0 D=0 D>0


Roots are imaginary & Roots are real and equal D is a perfect square then D is not a perfect square then
are given by and are given by roots are rational and different, roots are real and distinct and
-b/2a. provided are of the form
α + i β , α − iβ a, b, c, ∈ Q
P+ q &p- q,
provided a, b, c, ∈ Q

For the quadratic equation ax2 + bx + c =0 …(i)


(i) If a, b, c, ∈ R and a ≠ 0, then
(a) If D < 0, then equation (i) has non-real complex roots.
(b) If D > 0, then equation (i) has real and distinct roots, namely
−b + D −b − D
= α = ,β
2a 2a
2
c a ( x − α )( x − β ) 
And then ax + bx += ….(ii)
b
c a ( x − α ) …(iii)
2
(c) If D = 0, then equation (i) has real and equal roots. α = β = − and then ax2 + bx +=
2a
(ii) If a, b, c ∈ Q and D is a perfect square of a rational number, then the roots are rational numbers, and in case
D is not a perfect square then the roots are irrational.
(iii) If a, b, c ∈ R and p + iq is one root of equation (i) (and q ≠ 0) then the other must be the conjugate p – iq
and vice-versa. (p,q x2 R and i2 = –1).
If a, b, c ∈ Q and p +. q . is one root of equation (i) then the other
must be the conjugate
p - q and vice-versa (where p is a rational and q is an irrational
surd).
(iv) If exactly 1 root of Quadratic Equation is 0 then the product of roots
=0
⇒ c=0
∴ The equation becomes y = ax2 + bx = 0 the graph of which
passes through the origin as shown in Fig 2.7. Figure 2.7
(v) If both roots of quadratic equation are 0 then S = 0 & P = 0 where
S = sum and P = Product
2 . 6 | Quadratic Equations and Inequalities

b c
⇒ = 0 & = 0 ∴b = c = 0 ∴y =ax2
a a

(vi) If exactly one root is infinity ax2 + bx + c =0


1 a b
x= , then + +c =0 ; cy 2 + by + a =0 must have exactly one root 0

y y2 y
a
∴ P=0 ⇒ = 0 ⇒ a = 0 ; c≠0 ⇒ Y = bx + c
c

PLANCESS CONCEPTS

Very important conditions


• If y = ax2 + bx + cC is positive for all real values of x then a > 0 & D < 0
• If y = ax2 + bx + cC is negative for all real values of x then a < 0 & D < 0
1 a b
• If both roots are infinite for the equation ax2 + bx + c =0 ; x = ⇒ + +c =0
y y 2 y
b a
cy 2 + by + a =0 − = 0, = 0 ∴
= a 0,b
= 0 & c ≠ 0
c c
Vaibhav Gupta (JEE 2009 AIR 54)

5.1 Roots in Particular Cases


For the quadratic equation ax2 + bx + c =0
(a) If b = 0, ac < 0 ⇒ Roots are of equal magnitude but of opposite sign;
(b) If c = 0 ⇒ One root is zero, the other is –b/a;
(c) If b = c = 0 ⇒ Both roots are zero;
(d) If a = c ⇒ The roots are reciprocal to each other;
 a > 0 ;c < 0 
(e) If   ⇒ The roots are of opposite signs;
a < 0; c > 0
(f) If the sign of a = sign of b × sign of c ⇒ the root of greater magnitude is negative;
(g) If a + b + c = 0 ⇒ one root is 1 and the other is c/a;
(h) If a = b = c =0 then the equation will become an identity and will be satisfied by every value of x.

π
Illustration 8: Form a quadratic equation with rational coefficients having cos2 as one of its roots. (JEE MAIN)
8

(
Sol: If the coefficients are rational, then the irrational roots occur in conjugate pairs. Hence if one root is α + β )
( )
then other one will be α − β , Therefore, by using the formula x − ( sum of roots ) x + (product of roots ) =
2
0 we
can obtain the required equation.

π 1 π 1 π 1 1 
cos2 = × 2cos2 = 1 + cos  = 1 + 
8 2 8 2 4 2 2
1 1  1
Thus, the other root is 1 −  also Sum of roots = 1 and Product of roots =
2 2 8
M a them a tics | 2.7

Hence, the quadratic equation is 8x2 − 8x + 1 =0

1
Illustration 9: Find the quadratic equation with rational coefficients when one root is . (JEE MAIN)
( 2+ 5 )
Sol: Similar to Illustration 8.
If the coefficients are rational, then the irrational roots occur in conjugate pairs. Given that if one root is
1 1
α= = 5 − 2 , then the other root is β = = − 5 −2
(2 + 5) (2 − 5)
Sum of the roots α + β = − 4 and product of roots α β = -1. Thus, the required equation is x2 + 4x − 1 =0.

Illustration 10: If cos θ , sin φ , sin θ are in G.P then check the nature of the roots of x2 + 2 cot φ x+1 = 0? 
 (JEE MAIN)

Sol : As cos θ,sin φ and sin θ are in G.P., so we have sin2=


φ cos θ.sin θ . By calculating the discriminant (D), we can
check nature of roots.
We have sin2=
φ cos θ sin θ θ (as cos θ , sin φ , sin θ are in GP)
D = 4 cot2 φ - 4

 cos2 ϕ − sin2 φ  4(1 − 2sin2 φ) 4(1 − 2sin θ cos θ)  2(sin θ − cos θ)  2


= 4  =  = =   ≥0
 sin2 φ  sin2 φ sin2 φ  sin φ 

Hence the roots are real

Illustration 11: Form a quadratic equation with real coefficients when one root is 3 – 2i.  (JEE MAIN)

Sol: Since the complex roots always occur in pairs, so the other root is 3 + 2i. Therefore, by obtaining the sum and
the product of the roots, we can form the required quadratic equation.
The sum of the roots is
6 . The product of the root is ( 3 + 2i ) × ( 3 − 2 i ) = 9 − 4 i2 = 9 + 4 = 13
(3 + 2i) + (3 − 2i) =
Hence, the equation is x2 − Sx + P =0
⇒ x2 − 6x + 13 =
0

Illustration 12: If p, q and r are positive rational numbers such that p>q>r and the quadratic equation (p + q – 2r)
x2 + (q + r – 2p)x + (r + p – 2q) = 0 has a root in (-1, 0) then find the nature of the roots of px2 + 2qx + r =0
 (JEE ADVANCED)

Sol : In this problem, the sum of all coefficients is zero. Therefore one root is 1 and the other root is .
 r + p − 2q  r + p − 2q
  . which also lies in (-1, 0). Hence, by solving − 1 < < 0 we can obtain the nature of roots of
 p + q − 2r  p + q − 2r

px2 + 2qx + r =0.

(p + q – 2r) x2 + (q + r – 2p)x + (r + p – 2q) = 0


r + p − 2q
 One root is 1 & other lies in (-1, 0) ⇒ -1< <0 and p > q > r, Then p+q-2r > 0
p + q − 2r
r +p
r + p - 2q< 0 ⇒ r + p < 2q ⇒ <2
q
2 . 8 | Quadratic Equations and Inequalities

r2 + p2+2pr<4q2 ⇒ 4pr<4q2 ⇒ q2 > pr [ q2 > pr]


Hence D > 0, so the equation px2 + 2qx + r =0 has real & distinct roots.

Illustration 13: Consider the quadratic polynomial f ( x ) =x2 − px + q where f ( x ) = 0 has prime roots. If p + q = 11
and =a p2 + q2 , then find the value of f ( a) where a is an odd positive integer. (JEE ADVANCED)
2
Sol: Here f(x) = x − px + q, hence by considering α and β as its root and using the formulae for sum and product
of roots and the given conditions, we get the values of f(a).
f(x) = x2 − px + q
Given α and β are prime
α + β =p  ... (i);
αβ = q  ... (ii)
Given p + q = 11 ⇒ α + β + αβ = 11
⇒ ( α + 1 )( β + 1 ) =12 ; α = 2, β= 3 are the only primes that solve this equation.
∴ f(x) = (x – 2)(x – 3) = x2 − 5x + 6
∴ p = 5, q = 6 ⇒ a = p2 + q2 = 25 + 36 = 51; f(51) = (51 – 2)(51 – 3) = 49 × 48 = 3422

−2x2 + 4x + 3 and the line y =


Illustration 14: Find the maximum vertical distance 'd' between the parabola y =
x – 2 through the bounded region in the figure. (JEE MAIN)

Sol: In this problem, the maximum vertical distance d means the value of y.
The vertical distance is given by
y
d = −2x2 + 4x + 3 − ( x − 2 ) =
−2x2 + 3x + 5 ,
4
which is a parabola which opens downwards. 3
2
Its maximum value is the y-coordinate of -0.5 1
O x
-1
−b −3 3 -2 0.5 1 1.5 2 -2.5
the vertex which has x-coordinate equal to
= = .
2a 2 ( −2 ) 4
2
3 3 −9 18 40 49 Figure 2.8
Then y =−2   + 3   + 5 = + + =
4
  4
  8 8 8 8

Illustration 15: y = ax2 + bx + cC has no real roots. Prove that c(a+b+c) > 0. What can you say about expression
c(a – b + c)? (JEE ADVANCED)

∴ f (x1 )f (x2 ) > 0


Sol: Since there are no real roots, y will always be either positive or negative. Therefore

f(0)f(1) > 0 ⇒ c(a+ b + c) > 0 ; similarly f (0) f( −1) > 0 ⇒ c (a − b + c) > 0

Illustration 16: α , β are roots of the equation f ( x ) = x2 − 2x + 5 = 0 , then form a quadratic equation whose roots
are α3 + α2 − α + 22 & β3 + 4β2 − 7β + 35 . (JEE MAIN)

Sol: As α , β are roots of the equation ff ( xx) = = xx22 −− 2x


2x ++55 = = 00, ff ( α
α) , and f(β) will be 0. Therefore, by obtaining the
3 2 3 2
values of α + α − α + 22 and β + 4β − 7β + 35 we can form the required equation using sum and product
method.
From the given equation α2 − 2α + 5 = 0 and β2 − 2β + 5 = 0
M a them a tics | 2.9

We find α3 + α2 − α + 22 = α(α2 − 2α + 5) + 3α2 − 6α + 22 = 3(α2 − 2α + 5) + 7 = 7

( ) ( )
Similarly β3 + 4β2 − 7β + 35 = β β2 − 2β + 5 + 6β2 − 12β + 35 = 6 β2 − 2β + 5 + 5 =5
2
D1 : D2 Equation is x − 12x + 35 =
0

dy d2 y
y ax2 + bx + c > 0 ∀ x ∈ R , then prove that polynomial z =y +
Illustration 17: If= + will also be greater
dx dx2
than 0. (JEE ADVANCED)

Sol: In this problem, the given equation =y ax2 + bx + c > 0 ∀ x ∈ R means a > 0 & b2 − 4ac < 0 . Hence, by
dy d2 y
substituting y in z =y + + and solving we will get the result.
dx dx2
Since, y>0 ⇒ a > 0 & b2 − 4ac < 0
Z = ax2 + bx + c + 2ax + b + 2a = ax2 + (b + 2a) x + b + c + 2a
Again, as a > 0 & b2 − 4ac < 0
α (b + 2a) – 4a(b+ c +2a) = b2 − 4ac − 4a2 < 0
2
D=
For the new expression since D < 0 and a > 0, it is always positive.

Illustration 18: If a Quadratic equation (QE) is formed from y 2 = 4ax &=


y mx + c and has equal roots, then find
the relation between c, a & m. (JEE MAIN)

Sol: By solving these two equations, we get the quadratic equation; and as it has equal roots, hence D = 0.
(mx + c ) 4ax ; m2 x2 + 2 ( cm − 2a) x + c2 =
2
= 0
Given that the roots are equal. So, D = 0 ⇒ 4 ( cm − 2a) ⇒
2
= 4c2m2 ⇒ 4a2= 4acm
a
a=cm ⇒ c = ;
m
This is a condition for the line y = mx + c to be a tangent to the curve y 2 = 4ax .

2c
Illustration 19: Prove that the roots of the equation ax2 + bx + c =0 are given by  (JEE MAIN)
−b  b2 − 4ac

− b ± b2 − 4ac
Sol: We know that the roots of the quadratic equation ax2 + bx + c =0 are found by xx =
= . Therefore,
2a
in multiplying and dividing by − b  b2 − 4ac we can prove the above problem.
ax2 + bx + c =0
2 2
2b c  b  b  c
⇒ x + x += 0 ⇒ x + =    −
a a  2a   2a  a

 b 
2
b2 − 4ac  b   b2 − 4ac 
⇒ x +  = ⇒ x −  = ± 
 2a  4a2  2a   2a 
 

−b ± b2 − 4ac −b ± b2 − 4ac −b  b2 − 4ac


=⇒x = ⇒x ×
2a 2a −b  b2 − 4ac

=⇒x
( −b )
=
2
(
− b2 − 4ac
⇒x
) 2c
 2
2a  −b  b − 4ac  −b ± b2 − 4ac
 
2 . 1 0 | Quadratic Equations and Inequalities

 7 7 
Illustration 20: Let f ( x ) = ax + bx + a which satisfies the equation f  x +  = f  − x  and the equation
2
 4 4 
f ( x=
) 7x + a has only one solution. Find the value of (a + b). (JEE ADVANCED)

) 7x + a has only one solution, i.e. D = 0 and f  x + 74  = f  74 − x  . Hence, by solving these two equations
   
Sol: As f ( x=
   
simultaneously we will get the values of a and b.
Given
f ( x ) = ax2 + bx + a  …(i)
 7 7 
f  x +  = f  − x   …(ii)
 4 4 
and given that f ( x=
) 7x + a  …(iii)
has only one solution. Now using (i) and (ii).
2 2
 7  7 7  7   49 7   7  49 7  7 
a a  − x  + b  − x  + a ⇒ a  x2 +
a x +  + b  x +  + = + x + b x + =
 a + x2 − x  + b  − x 
 4  4 4  4   16 2   4  16 2  4 
⇒ 7ax + 2bx = 0 ; (7a + 2b)x = 0  … (iv)
f ( x ) = 7x + a has only one solution, i.e., D is equals to zero.

ax 2 + (b − 7 ) x= 0 ⇒ D = (b− 7)2 − 4a × 0 ⇒ D = x2 (b − 7 ) = 0; b = 7
2
ax2 + bx + a = 7x + a ⇒
Using equation (iv), a = –2, Then a + b = 5

Illustration 21: If the equation 2x2 + 4xy + 7y 2 − 12x − 2y + t =0 where t is a parameter that has exactly one real
solution of the form (x, y). Find the value of (x + y).  (JEE ADVANCED)

Sol: As the given equation has exactly one real solution, hence D = 0.
2x2 + 4x ( y − 3) + 7y 2 − 2y + t =0
D=0 (for one solution)

( ) (
⇒ 16 ( y − 3) − 8 7y 2 − 2y + t = 0 ⇒ 2 ( y − 3 ) − 7y 2 − 2y + t = 0
2 2
)
( ) ( )
⇒ 2 y 2 − 6y + 9 − 7y 2 − 2y + t = 0 ⇒ − 5y 2 − 10y + 18 − t = 0

⇒ 5y 2 + 10y + t − 18 =0

Again D = 0 (for one solution) ⇒ 100 – 20(t – 18) = 0


⇒ 5 – t + 18 = 0 ; For t = 23, 5y 2 + 10y + 5 =0
b2 − 4ac =
0 ⇒ b2 =
4ac
For y = −1 ; 2x2 − 16x + 32 =
0 ∴ x = 4 ⇒ x + y = 3

6. GRAPHICAL APPROACH
 b
2
D 
2 
Let y = ax + bx + c ; y = a  x +  − a, b and c are real coefficients. …(i)
 2a  4a2 

 −b −D  −b
Equation (i) represents a parabola with vertex  ,  and axis of the parabola is x =
 2a 4a  2a
M a them a ti cs | 2.11

If a > 0, the parabola opens upward, while if a < 0, the parabola opens downward.
The parabola intersects the x-axis at points corresponding to the roots of ax2 + bx + c =0 . If this equation has
(a) D > 0 the parabola intersects x – axis at two real and distinct points.
−b
(b) D = 0 the parabola meets x-axis at x =
2a
(c) D < 0 then;
If a > 0, parabola completely lies above x-axis.
If a < 0 parabola completely lies below x-axis.

Some Important Cases: If f ( x )= ax2 + bx + c= 0 and α , β are the roots of f (x)

1. If a> 0 and D > 0, y

( )
then f x > 0 ∀ ∈ ( − ∞ , α ) ∪ ( β, ∞ ) a>0
2 D>0
(where α <β , and are the roots of ax + bx + c =0)

  x

Figure 2.9: Roots are real & distinct

2.
( ) ( ) (
If a < 0 and D > 0 then f x < 0 ∀ x ∈ − ∞ , α ∪ β, ∞ ) y

where β > α
a<0
D>0
 
O x

Figure 2.10: Roots are real & distinct

3. If a>0 and D = 0 then α = β y


f (x) > 0 ∀ x : x ≠ α
a>0
f ( α ) =0 D=0

O  x

Figure 2.11: Roots are real & equal


4. 2
If a< 0 and D = 0 then px + qx + r =0 and y  x
f (x) < 0 ∀ x ≠ α O
a<0
f ( α ) =0 D=0

Figure 2.12: Roots are real & equal


5.
( )
If a> 0 and D < 0 then f x > 0 ∀ x ∈ R y
a>0
D<0

O x

Figure 2.13: Roots are complex


2 . 1 2 | Quadratic Equations and Inequalities

6. y
( )
If a< 0 and D < 0 then f x < 0 ∀ x ∈ R
O
x

a>0
D<0

Figure 2.14: Roots are complex

Illustration 22: The graph of a quadratic polynomial y = ax2 + bx + c is as shown in the figure below. Comment on
the sign of the following quantities.  (JEE MAIN)
(A) b – c (B) bc (C) c – a (D) ab2
y
Sol: Here a < 0; 

b c
− < 0 ⇒ b < 0; < 0 ⇒ c > 0. As b – c = (–ve) – (+ve); it must be negative;
a a x
O
Also, bc = (–ve)(+ve); this must be negative;
1 Figure 2.15
Then, β + = (–ve) (+ve); the product must be negative; finally,
α
c – a = (+ve) – (–ve), it must be positive.

Illustration 23: Suppose the graph of a quadratic polynomial 2x


Y y=
2
y = x + px + q is situated so that it has two arcs lying between the
rays y = x and y = 2x, x > 0. These two arcs are projected onto the
x-axis yielding segments SL and SR , with SR to the right of SL . Find y=x

the difference of the length (SR)-(SL)  (JEE MAIN)


x3 x1 x2 x4 X
Sol: Let the roots of x2 + px + q =x be x1 and x2 and the roots of SR
SL
x2 + px + q =2x be x3 and x4.
Figure 2.16
S=
R x 4 − x2 and S=
L x1 − x3 ⇒ SR − SL = x 4 + x3 − x1 − x2 .

{
∴ l ( SR ) − l ( SL ) = − (p − 2 ) − − (p − 1 )  =1
  }

7. THEORY OF EQUATIONS
Consider α , β, γ the roots of ax3 + bx2 + cx + d =0 ; then

d a ( x − α )( x − β )( x − γ )
ax3 + bx2 + cx +=

( )
+ d a x2 − ( α + β ) ( x + αβ )( x − γ )
ax3 + bx2 + cx=
3
+ d a ( x − x ( α + β + γ ) + x ( αβ + βγ + γα ) − αβγ )
2
ax + bx + cx = 3 2

x + x + x + ≡ ( x − x ( α + β + γ ) + x ( γ + βγ + γα ) − αβγ )
3 b c 2 d 3 2
a a a
−b coefficient of x2 c coefficient of x
Comparing them, α + β + γ = ⇒− , αβ + βγ + γα= ⇒
a coefficient of x 3 a coefficient of x3
M a them a ti cs | 2.13

d constant term x
αβγ = − ⇒ −
a coefficient of x2

Similarly for ax 4 + bx3 + cx2 + dx + e =0 ;


−b c d e
∑α = a
; ∑ αβ = a
; ∑ αβγ = −
a
; αβ γ δ =
a

PLANCESS CONCEPTS

As a general rule
a0 Xn + a1 Xn−1 + a2 Xn−2 + a3 Xn−3 + .... + an =
0 has roots X1 , X 2 , X3 ......Xn
−a coefficient of Xn−1 a2 coefficient of X n−2
∑ X1 = 1 =
− , ∑ X1 X=
2
=
a0 coefficient of Xn a0 coefficient of X n
a coefficient of Xn−3 constant term an
( −1)
n
∑ X1 X 2 X 3 =
− 3
a
=

coefficient of Xn
, X1 X 2 X3 ....xn =
coefficient of X n
= ( −1) n
a0
0

Vaibhav Krishan (JEE 2009 AIR 22)

0, x1 & x2 are the roots. Find the value of ( ax1 + b ) + ( ax2 + b )


−3 −3
Illustration 24: For ax2 + bx + c =

(JEE MAIN)

−b c
Sol: As x1 and x2 are the roots of equation ax2 + bx + c = 0, hence, x1 + x2 = and x1 x2 = .
a a
1 1
Therefore, by substituting this we will get the result +
( ax1 + b ) ( ax2 + b )
3 3

−d c
Now αβγ
= , αβ + βγ + =
λα
a a

( x1 + x21) − 3x+1x2 (x11+ x2 ) = −3b + b3 = b 3 − 3abc


3
1 1 1 1 x13 + x32
⇒ + − + = = ⇒
( ax1 + b −) c3 ( ax2 + b ) a c a c
3 3 2 2 3 3
( ax1 + b )
3
( ax2 + b )
3
−a3 x32 −a3 x13 a3 x13 x32 a3c3

Illustration 25: If the two roots of cubic equation x3 + px2 + qx + r = 0 are equal in magnitude but opposite in sign,
find the relation between p, q, and r.  (JEE MAIN)

Sol: Considering α, - α and β to be the roots and using the formula for the sum and product of roots, we can solve
above problem.
Let us assume the roots are α, - α and β
Then, α − α + β = −p ⇒ β = -p
−α2 − αp + αp = q ⇒ α2 = −q ; −α2β = −r ⇒ pq =
r

Illustration 26: If the roots of a quadratic equation ( a − b ) x2 + (b − c ) x + ( c − a ) =


0 are equal then prove
2a = (b +c) (JEE MAIN)
c−a
Sol: In this problem, the sum of all the coefficients is 0, hence its roots are 1 and . Therefore, by using the
a−b
2 . 1 4 | Quadratic Equations and Inequalities

product of roots formula we can prove the above problem.


As x = 1 is a root of the equation (since sum of all coefficients is 0)
∴ The other root is also 1
c−a
∴ Product = 1 = ; ∴ a – b = c – a ∴ 2a = b + c
a−b

2 1 1
Illustration 27: If the roots of p (q – r) x2 + q(r – p)x + r(p – q) = 0 has equal roots, prove that = + 
q p r
 (JEE MAIN)
Sol: This problem can be solved in the manner shown in the previous illustration.
One root is 1 ; ∴ other root is 1
rp − rq
∴ Product = 1 = ; ∴ pq – pr = rp – rq
pq − pr
2 p+r 1 1
∴ q(p + r) = 2rp ∴ = = +
q pr r p

0 then find the value of ∑ ( α − β ) 


2
Illustration 28: If α , β, γ are the roots of cubic x3 + qx + r = (JEE MAIN)

Sol: As we know, if α , β and γ are the roots of cubic equation ax3 + bx2 + cx + d =0
−b c −d
then α + β += γ , αβ + βγ + λα
= and αβγ = . Therefore, by using these formulae we can solve the above
illustration. a a a

α + β + γ =0 ; ∑ αβ = q ; αβγ = −r

( α + β + γ ) = 0 ⇒ α2 + β2 + γ2 = −2 ( αβ + βγ + γα ) ⇒ ∑ α2 = −2∑ αβ
2

Now ∑ ( α − β ) = ∑ ( α2 + β2 − 2αβ ) = 2 ( ∑ α2 − ∑ αβ ) = −6∑ αβ = −6q


2

Illustration 29: Form the cubic equation whose roots are greater by unity than the roots of x3 − 5x2 + 6x − 3 =0
 (JEE ADVANCED)

Sol: By using x3 − x2 ( ∑ α ) + x ( ∑ α1β1 ) − α1 β1 γ1 = 0 we can form cubic equation.


Here α1 = α + 1 β1 = β + 1 γ1 = γ + 1 and α , β and γ are the roots of x3 − 5x2 + 6x − 3 =0.

γ 5; ∑ αβ= 6; αβγ= 3
α+β+ =

Let the roots of the new equation be α1 , β1 , γ1

∴ The equation is x3 − x2 ( ∑ α ) + x ( ∑ α1β1 ) − α1 β1 γ1 = 0

α1 = α + 1 β1 = β + 1 γ1 = γ + 1

∑ α1 = α + β + γ + 3 = 8

∑ α1β1 = α1β1 + α1 γ1 + β1 γ1 = ( α + 1 )( β + 1 ) + ( β + 1 )( γ + 1 ) + ( γ + 1 )( α + 1 ) = 19

α1β1 γ1 = ( α + 1)(β + 1)( γ + 1) = 15

∴ x3 − 8x2 + 19x − 15 =
0
M a them a ti cs | 2.15

Alternate Method
y = x + 1 ⇒ x = y – 1 Put (y – 1) in given equation

⇒ ( y − 1) − 5 ( y − 1) + 6 ( y − 1) − 3 =
3 2
0 ⇒ y 3 − 1 − 3y 2 + 3y − 5y 2 − 5 + 10y + 6y − 6 − 3 = 0 ⇒ y 3 − 8y 2 + 19y − 15 = 0

Illustration 30: Find the sum of the squares and the sum of the cubes of the roots of x3 − ax2 + bx − c =0 
 (JEE ADVANCED)
Sol: Similar to the previous problem.

∑ α= a ; ∑ αβ
= b; ∑ αβγ= c;

(α + β + γ )
2
∴ α2 + β2 + γ 2 = − 2 ∑ α β = a2 − 2b

α3 + β3 + γ3 = (α3 + β3 + γ3 − 3αβγ ) + 3αβγ

= ( α + β + γ ) ( α2 + β2 + γ2 − α β −βγ − γα ) +=
3αβγ ( a) ( a2 − 2b − b=
) + 3c (a) (a2 − 3b) + 3c
π 
Illustration 31: If α , β, γ & δ are the roots of equation tan  + x  =
3tan3 x , then find the value of ∑ tan α
 4  (JEE ADVANCED)
1 + tanx  3 tan x − tan3 x 
Sol: Here, = 3  , therefore by putting tan x = y and solving we will get the result.
1 − tanx  1 − 3 tan2 x 
 

The given equation is:


1 + tan x  3 tan x − tan3 x 
=3  ; Let tanx =
y⇒
1 + y 3 3y − y
=
3
( )
1 − tan x  2  1−y 1 − 3y 2
 1 − 3 tan x 
⇒ 1 − 3y 2 + y − 3y 3 = 9y − 3y 3 − 9y 2 + 3y 4 ( y ≠ 1 )

⇒ 3y 4 − 6y 2 + 8y − 1= 0 ; ∑ y1= 0 ⇒ ∑ tan α= 0

Illustration 32: Find the number of quadratic equations with real roots remain unchanged even after squaring
their roots.  (JEE ADVANCED)

Sol: As given α β = α2 β2 and α2 + β2 = α + β , therefore by solving it we will get the values of α and β .
α β = α2 β2  …..(i)

and α2 + β2 = α + β  …..(ii)

Hence, αβ(1 − αβ) = 0 ⇒ α = 0 or β = 0 or αβ = 1

If α =0 then from (ii), β = 0 or β = 1 ⇒ roots are (0, 0) or (0,1)

If β =0 then, α =0 or α =1 ⇒ roots are (0,0) or (1,0)


2
1 1 1  1 1
If β = then α2 + = α+ ⇒ α +  − 2 = α +
α α 2 α  α α
Hence t − t − 2 = 0 ⇒ ( t − 2 )( t + 1 ) = 0 ⇒ t = 2 or t = −1
2

If t = 2 ⇒ α = 1 and β = 1, if t = –1 roots are imaginary ω or ω2


∴ The number of quadratic equations is one.
2 . 1 6 | Quadratic Equations and Inequalities

PLANCESS CONCEPTS

The relation between Roots and Coefficients.


If the roots of a quadratic equation ax2 + bx + c = 0 (a ≠ 0) are α and β then:

b2 − 4ac ± D
( α − β) = ± ( α + β)
2
• − 4αβ =± =
a a

b2 − 2ac
(α + β)
2
• α2 + β2 = − 2αβ =
a2
b b2 − 4ac
• α2 − β2 = ( α + β )( α − β ) = ±
a2

• α3 + β3 = ( α + β ) − 3αβ ( α + β ) −
2 (
b b2 − 3ac )
a3
(b2 − ac) b2 − 4ac
( α − β) + 3αβ ( α − β ) = ( α − β ) ( α + β ) − 4αβ + 3αβ  = ±
3 2
• α3 − β3 =
 a3

 b2 − 2ac  c2
• α 4 + β4 = (α 2
)
+ β2 − 2α2β2 = 
 a2




−2
a2

±b(b2 − 2ac) b2 − 4ac


• α 4 −β4 =(α2 − β2 )(α2 + β2 ) =
a4

• α2 + αβ + β2 = ( α + β)
2
− αβ = (b 2
)
− ac / a2

• α2β + β2 α = αβ ( α + β ) = −bc / a2

2 2
α β α 4 + β4 (α2 + β2 )2 − 2α2β2
•   +  = = =(b2 − 2ac / ac)2 − 2
β α 2 2 2 2
    α β α β
Nitish Jhawar (JEE 2009 AIR 7)

8. TRANSFORMATION OF EQUATIONS
We now list some of the rules to form an equation whose roots are given in terms of the roots of another equation.
Let the given equation be a0 xn + a1 xn−1 + ....an−1 x + an =
0 …. (i)

x
Rule 1: To form an equation whose roots are k(≠0) times the roots of the equation, replace x by .
k
Rule 2: To form an equation whose roots are the negatives of the roots in the equation, replace x by –x.
In rule 1, y = kx Hence x = y/k. Now replace x by y/k and form the equation. We can do the same thing for the
other rules.
Alternatively, change the sign of the coefficients of Xn−1 , Xn−3 , Xn−5 ,.... etc. in (i).
Rule 3: To form an equation whose roots are k more than the roots of the equation, replace x by x – k.
Rule 4: To form an equation whose roots are reciprocals of the roots of the equation, replace x by
1
( x ≠ 0 ) and then multiply both sides by xn .
x
M a them a ti cs | 2.17

Rule 5: To form an equation whose roots are the square of the roots of the equation in (1) proceed as follows:
Step 1 Replace x by x in (1)
Step 2 Collect all the terms involving x on one side.
Step 3 Square both the sides and simplify.
( α + β )( αβ ) ± αβ ( α + β ) − 4 ( αβ )
2 3

For instance, to form an equation whose roots are the squares of the roots of
2
replace x by x to obtain.

x x + 2x − x + 2 =0 ⇒ x ( x − 1 ) =−2 ( x + 1 )

x ( x − 1 ) = 4 ( x + 1 ) or x3 − 6x2 − 7x − 4 =
2 2
Squaring both sides, we get 0

Rule 6: To form an equation whose roots are the cubes of the roots of the equation, proceed as follows:
Step 1 Replace x by x1/3
Step 2 Collect all the terms involving x1/3 and x2/3 on one side.
Step 3 Cube both the sides and simplify.

9. CONDITION FOR MORE THAN 2 ROOTS


To find the condition that a quadratic equation has more than 2 roots.
ax2 + bx + c =0 Let α , β, γ be the roots of the equation
aα 2 + bα + c = 0  ... (i)
2
aβ + bβ + c =0  ... (ii)
aγ 2 + bγ + c =0  ... (iii)
2 2
Subtract (ii) from (i) a(α − β ) + b(α − β) = 0 ; (α − β)(a(α + β) + b) = 0
⇒ a(α + β) + b = 0  α ≠ β  …(iv)
Subtract (iii) from (ii) ⇒ a ( β + γ ) + b = 0  ...(v)
Subtract (i) from (iii) ⇒ a( γ + α ) + b = 0
Subtract (v) from (iv) ⇒ a( γ + β − β − α ) = 0 or a( γ − α ) = 0 ⇒ a =0
Keeping a = 0 in (iv) ; b = 0 and c = 0 ⇒ It is an identity

( )
Illustration 33: If a2 − 1 x2 + ( a − 1 ) x + a2 − 4a + 3 =0 is an identity in x, then find the value of a. (JEE MAIN)

Sol: The given relation is satisfied for all real values of x, so all the coefficients must be zero.
a2 − 1 =0 ⇒ a =±1 

a − 1 = 0 ⇒ a = 1  Common value a is 1

a2 − 4a + 3 = 0 ⇒ 1,3

(
Illustration 34: If the equation a ( x − 1 ) + b x2 − 3x + 2 + x − a2 =
2
)
0 is satisfied for all x ∈ R, find all possible
ordered pairs (a, b).  (JEE ADVANCED)

Sol: Similar to illustration 33, we can solve this illustration by taking all coefficients to be equal to zero.

( )
a ( x − 1 ) + b x2 − 3x + 2 + x − a2 =
2
0
2 . 1 8 | Quadratic Equations and Inequalities

⇒ ( a + b ) x2 − ( 2a + 3b − 1 ) x + 2b − a2 + a =0

Since the equation is satisfied for all α , it becomes an identity

Coeff. of x2 = 0 Coeff. of x = 0 Constant term = 0


a+b=0 2a + 3b – 1 = 0 2b – a² + a = 0 ; 2 – a² + a = 0

a = –b …..(i) using (i) ; ⇒ –2b + 3b = 1 ; a² – a – 2 = 0

⇒ b=1 ⇒ (a + 1) (a – 2) = 0 ; a = –1, 2

But from (i) a = –b ⇒ only a = –1 is the possible solution. Hence (a, b) = (–1, 1)

10. SOLVING INEQUALITIES

10.1 Intervals
Given E(x) = (x – a)(x – b)(x – c)(x – d) ≥ 0
To find the solution set of the above inequality we have to check the intervals in which E(x) is greater/less than zero.

Intervals

Closed interval Open interval Open-closed interval Closed-open interval

(a) Closed Interval: The set of all values of x, which lies between a & b and is also equal to a & b is known as a
closed interval, i.e. if a ≤ x ≤ b then it is denoted by x ∈ [a, b].
(b) Open Interval: The set of all values of x, which lies between a & b but equal to a & b is known as an open
interval, i.e. if a < x < b then it is denoted by a ∈ (a, b)
(c) Open-Closed Interval: The set of all values of x, which lies between a & b, equal to b, but not equal to a is
known as an open-closed interval, i.e. if a < x ≤ b then it is denoted by x ∈ (a, b].
(d) Closed-open Interval: The set of all values of x, which lies between a & b, equal to a but not equal to b is
called a closed-open interval, i.e. if a ≤ x < b, then it is denoted by x ∈ [a, b).
Note: (i) x ≥ a ⇒ [a, ∞ ) (ii) x > a ⇒ (a, ∞ )
(iii) x ≤ a ⇒ (- ∞ , a) (iv) x < a ⇒ (- ∞ , a)

10.2 Some Basic Properties of Intervals


(a) In an inequality, any number can be added or subtracted from both sides of inequality.
(b) Terms can be shifted from one side to the other side of the inequality. The sign of inequality does not change.
(c) If we multiply both sides of the inequality by a non-zero positive number, then the sign of inequality does not
change. But if we multiply both sides of the inequality by a non-zero negative number then the sign of the
inequality does get changed.
(d) In the inequality, if the sign of an expression is not known then it cannot be cross multiplied. Similarly, without
knowing the sign of an expression, division is not possible.
x−2
(i) > 1 ⇒ x − 2 > x − 5 (Not valid because we don’t know the sign of the expression)
x −5
x−2
> 1 ⇒ ( x − 2 ) > ( x − 5 ) (valid because (x − 5)2 is always positive)
2
(ii)
( x − 5)
2
M a them a ti cs | 2.19

10.3 Solution of the Inequality


(a) Write all the terms present in the inequality as their linear factors in standard form i.e. x ± a.
(b) If the inequality contains quadratic expressions, f(x) = ax² + bx + c; then first check the discriminant
(D = b² – 4ac)
(i) If D > 0, then the expression can be written as f(x) = a (x - α)(x - β). Where α and β are
−b ± b2 − 4ac
given by α, β =
2a
−b
(ii) If D = 0, then the expression can be written as f(x) = a ( x − α ) , where α =
2
.
2a
(iii) If D < 0 & if
 > 0, then f(x) > 0 ∀ X ∈ R and the expression will be cross multiplied and the sign of the inequality
• a
will not change.
 < 0, then f(x) < 0 ∀ X ∈ R and the expression will be cross multiplied and the sign of the inequality
• a
will change.
• If the expression (say 'f' ) is cancelled from the same side of the inequality, then cancel it and write
f ≠ 0 e.g.,

(i)
( x − 2)( x − 3) > 1 ⇒ (x − 3)
> 1 iff x − 2 ≠ 0
( x − 2)( x − 5) (x − 5)

( x − 5) ( x − 8 ) ≥ 0
2

(ii) ⇒ ( x − 5) ( x − 8 ) ≥ 0 iff x − 5 ≠ 0
( x − 5)
( x − a1 ) 1 ( x − a2 ) 2 .... ( x − a) n
k k k

(iii) Let f(x) =


( x − b1 ) 1 ( x − b2 ) 2 .... ( x − b ) n
r r r

Where k1 , k 2 ……………. kn & r1 , r2 ……….. rn ∈ N and a1 , a2 , ……….. an & b1 , b2 ………. bn are fixed real numbers.
The points where the numerator becomes zero are called zeros or roots of the function and points where the
denominator becomes zero are called poles of the function. Find poles and zeros of the function f(x). The
corresponding zeros are a1 , a2 , ……….. an and poles are b1 , b2 ………. bn . Mark the poles and zeros on the real
numbers line. If there are n poles & n zeros the entire number line is divided into ‘n+1’ intervals. For f(x), a number
line is divided into ‘2n+1’ intervals.
Place a positive sign in the right-most interval and then alternate the sign in the neighboring interval if the pole or
zero dividing the two interval has appeared an odd number of times. If the pole or zero dividing the interval has
appeared an even number of times then retain the sign in the neighboring interval. The solution of f(x) > 0 is the
union of all the intervals in which the plus sign is placed, and the solution of f(x) < 0 is the union of all the intervals
in which minus sign is placed. This method is known as the WAVY CURVE method.

Now we shall discuss the various types of inequalities.


Type I: Inequalities involving non-repeating linear factors (x – 1) (x – 2) ≥ 0

1st condition
( x − 1) > 0 ⇒ x > 1 x ≥ 2

( x − 2 ) > 0 ⇒ x > 2 
x − 1 < 0 ⇒ x < 1
2nd condition  x ≤1
x − 2<0⇒ x <2 

∴ x ∈ ( −∞ ,1] ∪ [2, ∞ )
2 . 2 0 | Quadratic Equations and Inequalities

 12 
Illustration 35: (x – 3) (x+1)  x −  < 0, find range of x  (JEE MAIN)
 7 
Sol:Comparing all brackets separately with 0, we can find the range of values for x.
12  12 
x < −1 and < x < 3 ; ∴ x ∈ ( −∞ , −1 ) ∪  ,3 
7  7 
Type II: Inequalities involving repeating linear factors

( x − 1 ) ( x + 2 ) ( x − 3) ≤ 0
2 3

( x + 1) ( x + 2) ( x + 2)( x − 3) ≤ 0
2 2

⇒ (x+2)(x – 3) < 0 ; x ∈ [-2, 3]

Illustration 36: Find the greatest integer satisfying the equation.

( x + 1 ) ( x − 3) ( x − 5 ) ( x − 4 ) ( x − 2 )
101 2 11 200 555
<0 (JEE MAIN)

Sol: Comparing all brackets separately with 0, we can find the greatest integer.

{ } − ( x − 2) − 2 =0
2
The inequality − ( x − 2 )

⇒ (x + 1)(x – 5)(x – 2) < 0

x = -1, 3 , 5, 4, 2

x ∈ ( −∞ , −1 ) U (2, 3) U (3, 4) U (4, 5) .

Type III: Inequalities expressed in rational form.

Illustration 37:
( x − 1)( x + 2) ≥ 0  (JEE MAIN)
( x + 3)( x − 4 )
( x + a)( x + b ) ≥ 0 then x + c x + d ≠ 0 , and x + a x + b =
Sol: If ( )( ) ( )( ) 0
( x + c )( x + d)
Hence, x ≠ − 3, 4 & x = 1, -2 ; x ∈ ( −∞ , −3) U [-2, 1] U (4, ∞ )

x2 (x + 1)
Illustration 38: <0  (JEE MAIN)
(x − 3)3
Sol: Similar to the illustration above.
x +1
< 0 x ≠ 3, -1, 0 ; x ∈ (–1, 0) U (0, 3)
x −3

x2 − 1
Illustration 39: ≥ 1 (JEE MAIN)
x2 − 7x + 12

Sol: First reduce the given inequalities in rational form and then solve it in the manner similar to the illustration
above.
x2 − 1
≥1
( x − 4 )( x − 3)
M a them a ti cs | 2.21


( x + 1)( x − 1) ≥ 1 ⇒ x2 − 1 − 1 ≥ 0
( x − 4 )( x − 3) x2 − 7x + 12

x2 − 1 − x2 + 7x − 12 7x − 13
∴ ≥ 0 ∴ ≥0
( x − 4 )( x − 3) ( x − 4 )( x − 3)
 13 
∴ x ≠ 3, 4 ; x ∈  ,3  ∪ (4, ∞ )
7 
Type IV: Double inequality

3x2 − 7x + 8
Illustration 40: 1 < ≤2  (JEE ADVANCED)
x2 + 1

Sol: Here 3x2 − 7x + 8 > x2 + 1 therefore if D < 0 & if a > 0, then f(x) > 0 and always positive for all real x.
3x2 − 7x + 8 > x2 + 1 ⇒ 2x2 − 7x + 7 > 0 ;
D= b2 − 4ac = 49 – 56 = -7
∴ D < 0 & a > 0 ∴ always positive for all real x
3x2 − 7x + 8 ≤ 2x2 + 2 ⇒ x2 − 7x + 6 ≤ 0 ⇒ ( x − 1 )( x − 6 ) ≤ 0
x ∈ [1, 6] ; x ∈ [1, 6] ∩ R

Type V: Inequalities involving biquadrate expressions

Illustration 41: (x 2
)(
+ 3x + 1 x2 + 3x − 3 ≥ 5  ) (JEE ADVANCED)

Sol: Using x2 + 3x =
y ,we can solve this problem
Let x2 + 3x =
y ∴( y + 1 )( y − 3 ) ≥ 5

y 2 − 2y − 8 ≥ 0 ∴ (y – 4) (y + 2) ≥ 0

∴ (x + 4) (x – 1)(x + 2)(x + 1) ≥ 0 ⇒ x ∈ ( −∞ , −4  ∪  −2, −1  ∪ 1, ∞ )

11. CONDITION FOR COMMON ROOTS


Consider that two quadratic equations are a1 x2 + b1 x + c1 =
0 and a2 x2 + b2 x + c2 =
0
(i) One root is common b1 c1 a1 b1

Let α, be the common root. then α satisfies  b2 c2 a2 b2


2
a1α + b1α + c1 = 0
a3 α3 + b2 α + c2 = 0

α2 α 1
By cross multiplication method,
= =
b1c2 − b2c1 − ( a1c2 − c1a2 ) a1b2 − b1a2

α2 α 1
= =
b1c2 − b2c1 c1a2 − a1c2 a1b2 − b1a2
b c − b2c1
α2 = 1 2  ... (i)
a1b2 − b1a2
2 . 2 2 | Quadratic Equations and Inequalities

c1a2 − a1c2
α=
a1b2 − b1a2  … (ii)

Divide (1)/(2)
b1c2 − b2c1
α=  … (iii)
c1a2 − c2a1
equating (i) and (ii) ; ( c1 a2 − c2 a1 ) =( a1 b2 − b1 a2 ) (b1 c2 − b2 c1 ) is the condition for a common root.
2

a1 b1 c1
(ii) If both roots are common, then = =
a2 b2 c2

Illustration 42: Determine the values of m for which the equations 3x2 + 4mx + 2 =0 and 2x2 + 3x − 2 =0 may
have a common root.  (JEE MAIN)

Sol: Consider α to be the common root of the given equations. Then, α must satisfy both the equations. Therefore,
by using a multiplication method we can solve this problem.
3α2 + 4mα + 2 = 0 ; 2α2 + 3α − 2 = 0
Using the cross multiplication method, we have 
(–6 – 4)2 = (9 – 8m)(– 8m – 6)
⇒ 50 = (8m – 9)(4m +3) ⇒ 32m2 − 12m − 77 =
0
⇒ 32m2 − 56m + 44 =
0 ⇒ 8m(4m – 7)+11(4m – 7) = 0
11 7
⇒ (8m + 11)(4m – 7) = 0 ⇒ m= − ,
8 4

Illustration 43: The equation ax2 + bx + c and y ≥ 0 have two roots common, Find the value of (a + b).
 (JEE ADVANCED)
Sol: We can reduce x3 − 2x2 + 2x − 1 =
2
(
0 to (x – 1) x2 − x + 1 = )
0 as the given equations have two common roots,
therefore − ω and − ω are the common roots (as both roots of a quadratic equation are either real or non-real).

( )
We have x3 − 2x2 + 2x − 1 = 0 ⇒ ( x − 1 ) x2 − x + 1 = 0
−1 + 3i
⇒ x = 1 or x = − ω , −ω2 , where ω =
2
Since ax2 + bx + a = 0 and x3 − 2x2 + 2x − 1 =0 have two roots in common, therefore −ω and −ω2 are the common
roots (as both roots of a quadratic equation are either real or non-real), also −ω is a root of ax2 + bx + a =0 . Hence.

( )
a 1 + ω2 − b ω = 0 ⇒ a ( −ω) − b ω = 0 (as 1 + w+ ω2 = 0)
⇒a+b=0

12. MAXIMUM AND MINIMUM VALUE OF A QUADRATIC EQUATION


−b
y = ax2 + bx + c attains its minimum or maximum value at x = according to a > 0 or a < 0
2a
MAXIMUM value case
−D  −D 
When a < 0 then ymax = i.e. y ∈ −∞ , 
4a  4a 
M a them a ti cs | 2.23

MINIMUM value case


−D  −D 
When a > 0 then ymin = i.e. y ∈  , ∞ 
4a  4a 
y
-D
4a y

a<0
a<0

-b
  2a 
x
O -b x
2a -D
4a
Figure 2.17
Figure 2.18

PLANCESS CONCEPTS

If α is a repeated root, i.e., the two roots are α , α of the equation f(x) = 0, then α will be a root of the
df
derived equation f’(x) = 0 where f’(x) =
dx
If α is a repeated root common in f(x) = 0 and φ (x) = 0, then α is a common root both in f’(x) = 0 and
φ ‘(x) = 0.
Shrikant Nagori (JEE 2009 AIR 30)

x
Illustration 44: Find the range of y = 2
 (JEE ADVANCED)
x − 5x + 9

Sol: Here as x ∈ R therefore D ≥ 0 . Hence, by solving these inequalities we can find the required range.
x = yx2 − 5yx + 9y; yx2 − 5yx + 9y − x =.
0
yx2 − (5y + 1 ) x + 9y =
0;  x ∈ R D≥0
∴ (5y + 1 ) − 36 y ≥ 0 ;
2 2
∴ 25 y + 1 + 10y − 36 y 2 ≥ 0
2

∴ − 11 y 2 + 10y + 1 ≥ 0 11y 2 − 11y + y − 1 ≤ 0

(11y +1)(y – 1) ≤ 0 ;  −1 
y ∈  ,1
 11 
x
∴ ( x − 3) = 0
2
Putting the end points in the eq. 1= ; x2 − 6x + 9= 0
2
x − 5x + 9
−1 x
If D < 0, then 1 would be open, i.e. excluded; =
11 x2 − 5x + 9

( )
⇒ − x2 − 5x + 9= 11x ; 9 0 ; ∴ ( x + 3)= 0 ; ∴
∴ x2 + 6x +=
2 −1
11
remains closed

x 1
Alternative Solution:
= y =
x2 − 5x + 9  9
x −5 + 
 x
Apply the concept of Arithmetic mean > Geometric mean for the values for x and 9/x
2 . 2 4 | Quadratic Equations and Inequalities

 9
x + 
 x 9
We have ≥ x*
2 x
9 9
Thus x + ≥ 6 for x ≥ 0 and x + ≤ 6 for x < 0
x x
Since the term is in the denominator if we consider its maximum value, we will get the minimum value of y and
vice versa.
1
The maximum value of y will be = 1 and
6 −5
1 −1
The minimum value of will be = .
−6 −5 11
 −1 
Thus the range of y is  ,1 
 11 

x2 + 2x − 3
Illustration 45: Find range of y =  (JEE ADVANCED)
x2 + 2x − 8

Sol: Similar to the preceding problem, by taking b2 ‒ 4ac ≥ 0 we can solve it.
x2 + 2x ‒ 3 = yx2 + 2xy ‒ 8y
(y ‒ 1)x2 + (2y ‒ 2)x ‒ (8y ‒ 3) = 0 ; b2 ‒ 4ac ≥ 0
∴ (2y ‒ 2)2 + (4)(8y ‒ 3)(y ‒1) ≥ 0 ⇒ 4y2 + 4 ‒ 8y + 4(8y2 ‒ 3y ‒ 8y + 3) ≥ 0
4y2 + 4 ‒ 8y ‒ 44y ‒ 32y2 + 12 ≥ 0 ⇒ 36y2 ‒ 52y + 16 ≥ 0
 4
∴ 9y2 ‒ 13y + 4 ≥ 0 ⇒ (y – 1) (9y – 4) ≥ 0 ; ∴ y ∈  − ∞ ,  ∪ (1, ∞ )
 9
To verify if the bracket is open or closed, apply the end points in the equation,
4 4 x2 + 2x − 3
Check for y = ; =
9 9 x2 + 2x − 8

4x2 + 8x − 32 = 9x2 + 18x − 27 ; ∴ 5x2 + 10x + 5 =0


∴ x2 + 2x + 1 =0 ; ∴ ( x + 1) =
2
0
4
∴ x =−1 ∴ is closed
9
x2 + 2x − 3
Check for 1, 1=
x2 + 2x − 8

∴ x2 + 2x − 3 = x2 + 2x − 8 Since no value of x can be found, 1 is open

ax2 − 7x + 5
Illustration 46: Find the limits of ‘a’ such that y = is capable of all the values of ‘x’ being a real
quantity.  5x2 − 7x + a (JEE ADVANCED)

Sol: Similar to Illustration 45.

5yx2 − 7xy + ay = ax2 − 7x + 5

(5y − a) x2 − 7x ( y − 1) + ay − 5 =0 ;
( )
D = 49 y 2 + 1 − 2y − 4 ( ay − 5 )(5y − a) ; D≥0
M a them a ti cs | 2.25

49y2 ‒ 49 ‒ 98y ‒ 20ay2 + 100y + 4a2y ‒ 20 a ≥ 0

y2 (49 ‒ 20a) + y(2 + 4a2) + 49 ‒ 20 a ≥ 0

(2 + 4a2)2 ‒ [2(49 ‒ 20a)]2 ≤ 0 ; ∴ (2 + 4a2 + 2(49 ‒ 20a))(2 + 4a2 ‒ a2(49 ‒ 20a))

(1 + 2a2 + 49 ‒ 20a)(1 + 2a2 ‒ 49 + 20 a) ≤ 0

(a2 ‒ 10a + 25) (a2 + 10 a ‒ 24) ≤ 0 ; ∴ (a + 12)(a ‒ 2) ≤ 0

13. LOCATION OF ROOTS

Let f(x) = ax2 + bx + c; a,b,c ∈  a is not equal to 0 and α , β be the roots of f ( x ) = 0


()
Type I: If both the roots of a quadratic equation f x = 0 are greater than a specified number, say ‘d’, then

a>0
 
D>0
d -b x-axis
b
2a a<0
2a D>0
d   X-axis
Figure 2.20
Figure 2.19


d b x-axis a>0
2a D=0
a<0 -b
D=0 2a
d = x-axis

Figure 2.22
Figure 2.21

−b
(i) D ≥ 0 (ii) f(x) > 0 (iii) >d
2a
Type II: If both the roots are less than a specified number, say ‘d’, then

a>0 

D>0
b d x-axis
b 2a a<0
2a D>0
  d x-axis
Figure 2.24
Figure 2.23
2 . 2 6 | Quadratic Equations and Inequalities

=
-b d x=axis
2a a>0
D=0
a<0 -b
D=0 2a
= d x-axis

Figure 2.25 Figure 2.26

−b
(i) D ≥ 0 (ii) f(x) > 0 (iii) <d
2a

2 2
Illustration 47: If both the roots of the quadratic equation x + x 4 − 2k + k − 3k − 1 =( )
0 are less than 3, then find
the range of values of k.  (JEE MAIN)
−b
Sol: Here both the roots of the given equation is less than 3, hence, D ≥ 0 , < 3 and f (3) > 0.
2a

() (
The equation is f x = x2 + x 4 − 2k + k 2 − 3k − 1= 0 )
D ≥ 0 …(i)
3
−b
d
< 3 ….(ii)
2a
f (3) > 0 ….(iii)

(i) D ≥ 0 ⇒ (4 ‒ 2k)2 ‒ 4(k2 ‒ 3k ‒ 1) ≥ 0

⇒ (k2 ‒ 4k + 4) ‒(k2 ‒ 3k ‒ 1) ≥ 0 5

⇒ –k + 5 ≥ 0 ⇒ k–5 ≤ 0 ;

(
k ∈ −∞ ,5
4 5
−4 ( 4 − 2k )
(ii) < 3 ; k – 2 < 3; k < 5
2
()
(iii) f 3 > 0 ⇒ 9 + 3(4 – 2k) + k2 – 3k – 1 > 0

⇒ k2 ‒ 9k + 20 > 0 ⇒ (k – 4)(k – 5) > 0

k ∈ (‒∞, 4) ∪ (5, ∞) ; Combining all values we get k ∈ (‒∞, 4)


Type III: A real number d lies between the
()
roots of f x = 0 or both the roots lie on either side of a fixed number say ‘d’ then af(d)<0, and D>0.

a>0
a<0
b
2a d
  
d -b x-axis  x-axis
2a

Figure 2.27 Figure 2.28


M a them a ti cs | 2.27

Type IV: Exactly one root lies in the interval (d, e) when d < e, then f(d)⋅f(e)<0

a>0
D>0  
b d b e x-axis
2a e 2a
d   x-axis a<0
D>0
Figure 2.29
Figure 2.30

a>0
D>0   e
b
2a d b d x-axis
e x-axis 2a a<0
 
D>0

Figure 2.31
Figure 2.32

()
Type V: If both the roots of f x = 0 are confined between real numbers‘d’ and ‘e’, where d < e. Then

b
(i) D ≥ 0, (ii) f(d)f(e) >0, (iii) d < − < e.
2a

d = e
d   e
b x-axis
2a b
a<0 2a a<0
D=0 D<0

Figure 2.33 Figure 2.34

a>0
D>0
b a>0
2a D=0
d   e x-axis -b
2a
= d x-axis
Figure 2.35
Figure 2.36

Type VI: One root is greater than e and the other root is less than ‘d’.

a>0 

D>0
b d -b e x-axis
d 2a e 2a a<0
  x-axis D>0

Figure 2.37 Figure 2.38


2 . 2 8 | Quadratic Equations and Inequalities

14. QUADRATIC EXPRESSION IN TWO VARIABLES


The general quadratic expression ax2 + 2hxy + by2 + 2gx + 2fy + c can be factorized into two linear factors. The
corresponding quadratic equation is in two variables
ax2 + 2hxy + by2 + 2gx + 2fy + c = 0

or ax2 + 2(hx + g)x + by2 + 2fy + c = 0 …(i)

∴x =
−2 (hy + g) ± 4 (hy + g) − 4a by 2 + 2fy + c
2

⇒x=
( )
− (hy + g) ± h2 y 2 + g2 + 2ghy − 2afy − ac − aby 2
2a a
⇒ ax + hy + g =± h2 y 2 + g2 + 2ghy − aby 2 − 2afy − ac  …..(ii)

At this point, the expression (i) can be resolved into two linear factors if

(h2
)
− ab y 2 + 2 ( gh − af ) y + g2 − ac is a perfect square and h2 ‒ ab > 0.

( )
But h2 − ab y 2 + 2 ( gh − af ) y + g2 − ac will be a perfect square if D = 0

⇒ g2h2 + a2 f 2 − 2afgh − h2g2 + abg2 + ach2 − a2bc =


0 and h2 − ab > 0

⇒ abc + 2fgh − af 2 − bg2 − ch2 =


0 and h2 − ab > 0
This is the required condition. The condition that this expression may be resolved into two linear rational factors is
a h g
=
∆ h=b f 0
g f c

⇒ abc + 2fgh − af 2 − bg2 − ch2 =


0 and h2 − ab > 0
This expression is called the discriminant of the above quadratic expression.

Illustration 48: If the equation x2 + 16y2 ‒ 3x + 2 = 0 is satisfied by real values of x & y, then prove that
 −1 1 
x ∈[1, 2], y ∈  ,   (JEE MAIN)
 8 8

Sol: For real values of x and y, D ≥ 0 . Solve this by taking the x term and the y term constant one by one.
x2 − 3x + 16y 2 + 2 =
0 ; D ≥ 0 as x ∈ R

(
⇒ 9 − 4 16y 2 + 2 ≥ 0 ; ) ⇒ 9 – 64y2 – 8 ≥ 0

∴ 64y 2 − 1 ≤ 0
 −1 1 
⇒ (8y – 1)(8y + 1) ≤ 0 ∴ y∈ , 
 8 8

To find the range of x, in 16y 2 + x2 − 3x + 2 =0 D>0

(
Hence, -64 x2 − 3x + 2 ≥ 0 )
Solving this, we get x ∈ [1, 2]
M a them a ti cs | 2.29

Illustration 49: Show that in the equation x2 ‒ 3xy + 2y2 ‒ 2x ‒ 3y ‒ 35 ‒ 0, for every real value of x there is a real
value of y. (JEE MAIN)

−b + b2 − 4ac 3y + 2 + y 2 + 24y + 144


Sol: By using the formula x = we will get x = .
2a 2
Here, the quadratic equation in y is a perfect square.

(
x2 − x ( 3y + 2 ) + 2y 2 − 3y − 35 =
0 )
 2
Now, x = 3y + 2 + 2 quadratic in y . As the quadratic equation in y is a perfect square  y + 12  .

( ) 
∴ The relation between x & y is a linear equation which is a straight line.
∴∀ x ∈ R , y is a real value.

( ) (
Illustration 50: If a1 x2 + b1 x + c1 y + a2 x2 + b2 x + c2 = )
0 find the condition that x is a rational function of y
 (JEE ADVANCED)

Sol: For x is a rational function of y, its discriminant will be greater than or equal to zero, i.e. D ≥ 0.
x − ( a1 y + a2 ) + x (b1 y + b2 ) + ( c1 y + c2 ) =
2
0

− (b1 y + b2 ) ±
(b1 y + b2 ) − 4 ( a 1 y + a2 )( c1 y + c2 )
2

x=
2 ( a 1 y + a2 )

For the above relation to exist (b1 y + b2 ) − 4 ( a 1 y + a2 )( c1 y + c2 ) ≥ 0


2

( ) (
⇒ b12 − 4a1c1 y 2 + 2 (b1b2 − 2a1c2 − 2a2c1 ) y + b22 − 4a2c2 ≥ 0 )
⇒ b12 − 4a1c1 > 0 and D ≤ 0
Solving this will result in a relation for which x is a rational function of y.

15. NUMBER OF ROOTS OF A POLYNOMIAL EQUATION


(a) If f(x) is an increasing function in [a, b], then f(x) = 0 will have at most one root in [a, b].
(b) Let f(x) = 0 be a polynomial equation. a, b are two real numbers. Then f(x) =0 will have at least one real root
or an odd number of real roots in (a, b) if f(a) and f(b) (a < b) are of opposite signs.

A(a, f(a))

A(a,f(a))
P (b,0) (b,0)
(a,0) (a,0)
B(b,f(B))
B(b,f(b))
One real root Odd number of real roots

Figure 2.39
2 . 3 0 | Quadratic Equations and Inequalities

But if f(a) and f(b) are of the same sign, then either f(x) = 0 have one real root or an even number of real roots
in (a, b)
B(b,f(b))
A(a,f(a)) B(b,f(b))
A(a,f(a))
P R
Q S
(a,0) (b,0)

No real root Even number of real roots


Figure 2.40

(c) If the equation f(x) = 0 has two real roots a and b, then f’(x) = 0 will have at least one real root lying between
a and b (using Rolle’s theorem).

PLANCESS CONCEPTS

Descartes’ rule of sign for the roots of a polynomial


Rule 1: The maximum number of positive real roots of a polynomial equation

f ( x )= a0 xn + a1 xn−1 + a2 xn−2 + .... + an−1 x + an= 0 is the number of changes of the sign of coefficients from
positive to negative and negative to positive. For instance, in the equation x3 + 3x2 + 7x − 11 =
0 the
sign of the coefficients are +++- as there is just one change of sign, the number of positive roots of
x3 + 3x2 + 7x − 11 = 0 is at most 1.

Rule 2 : The maximum number of negative roots of the polynomial equation f(x) = 0 is the number of
changes from positive to negative and negative to positive in the sign of the coefficient of the equation
f(–x) = 0.
Shivam Agarwal (JEE 2009 AIR 27)

PROBLEM-SOLVING TACTICS

Some hints for solving polynomial equations:


a+b
(a) To solve an equation of the form ( x − a) + ( x − b ) =
4 4
A ; Put y = x −
2
In general to solve an equation of the form ( x − a)
2n
+ ( x − b)
2n
A , where
= n∈ Z + , put y = x − a + b
2
(b) To solve an equation of the form, a0 f ( x ) + a1 ( f ( x ) ) + a2 =
0 we put ( f ( x ) ) = y and solve a0 y + a1 y + a2 =
2n n n 2
0
to obtain its roots y1 and y 2 .
solve, ( f ( x ) ) 1 and ( f ( x ) )
n n
Finally, to obtain the solution of (1) we = y= y2

( )( ) (
(c) An equation of the form ax2 + bx + c1 ax2 + bx + c2 ..... ax2 + bx + cn = )
A . Where c1, c2, ......cn, A ∈ R , can be
solved by putting ax + bx = y.
2
M a them a ti cs | 2.31

(d) An equation of the form (x – a)(x – b)(x – c)(x – d) = ⇒ Awhere ab = cd, can be reduced to a product of two
ab
quadratic polynomials by putting y = x + .
2
(e) An equation of the form (x – a) (x – b)(x – c)(x – d) = A where a < b < c < d, b – a = d – c can be solved by a

( x − a) + ( x − b ) + ( x − c ) + ( x − d) = x − 1
change of variable y =
4 4
( a + b + c + d)
(f) A polynomial f(x, y) is said to be symmetric if f(x, y) = f(y, x) ∀ x, y. A symmetric polynomial can be represented
as a function of x + y and xy.

Solving equations reducible to quadratic


(a) To solve an equation of the type ax 4 + bx2 + c =0 , put x2 = y .
(b) To solve an equation of the type a (p ( x ) ) + bp ( x ) + c =
2
0 (p(x) is an expression of x), put p(x) = y.

b
(c) To solve an equation of the type ap x + () p(x)
+c =0 where p(x) is an expression of x, put p(x)= y

This reduces the equation to ay 2 + cy + b =0

1
(d) To solve an equation of the form a  x2 + 1  + b  x + 1  + c =
   
0 , put x + =y
2
 x   x x
 1   1 1
and to solve a  x2 + + b x −  + c =0 , put x − =y
2  x x
 x   

(e) To solve a reciprocal equation of the type ax 4 + bx3 + cx2 + bx + a= 0, a ≠ 0,


2 1
we divide the equation by d y to obtain a  x2 + 1  + b  x + 1  + c =
   
0 , and then put x + =y
2
dx 2
 x   x x

(f) To solve an equation of the type (x + a)(x + b)(x + c)(x + d) + k = 0 where a+b =c+d, put x2 +(a+ b)x = y

(g) To solve an equation of the type ax + b = cx + d or ax2 + bx + c = dx + e , square both the sides.

(h) To solve an equation of the type = ax + b ± cx + d =


e , proceed as follows.

Step 1: Transfer one of the radical to the other side and square both the sides.
Step 2: Keep the expression with radical sign on one side and transfer the remaining expression on the other side
Step 3: Now solve as in 7 above.

FORMULAE SHEET

(a) A quadratic equation is represented as : ax2 + bx + =


c 0, a ≠ 0
−b ± D
(b) Roots of quadratic equation: x = , where D(discriminant) = b2 − 4ac
2a

(c) Nature of roots: (i) D > 0 ⇒ roots are real and distinct (unequal)

(ii) D = 0 ⇒ roots are real and equal (coincident)

(iii) D < 0 ⇒ roots are imaginary and unequal


2 . 3 2 | Quadratic Equations and Inequalities

( ) ( ) ( )( )
(d) The roots α + iβ , α − iβ and α + β , α − β are the conjugate pair of each other.

(e) Sum and Product of roots : If α and β are the roots of a quadratic equation, then

−b Coefficient of x c constant term


(i) S = α + β = = (ii) P = αβ = =
a Coefficient of x 2 a Coefficient of x2

(
(f) Equation in the form of roots: x2 − α + β x + α. β = 0 ) ( )
(g) In equation ax2 + bx + c= 0, a ≠ 0 If
(i) b = 0 ⇒ roots are of equal magnitude but of opposite sign.

(ii) c = 0 ⇒ one root is zero and other is –b/a

(iii) b = c = 0 ⇒ both roots are zero.

(iv) a = c ⇒ roots are reciprocal to each other.

(v) a > 0, c < 0 or a < 0, c > 0 ⇒ roots are of opposite signs.

(vi) a > 0, b > 0, c > 0 or a < 0, b < 0, c < 0 ⇒ both roots are –ve.

(vii) a > 0, b < 0, c > 0 or a < 0,b > 0, c < 0 ⇒ both roots are +ve.

2 2
(h) The equations a1 x + b1 x + c1 =
0 and a2 x + b2 x + c2 =
0 have

b1c2 − b2c1 c1a2 − c2a1


(i) One common root if =
c1a2 − c2a1 a1b2 − a2b1

1 a b1 c1
(ii) Both roots common if = =
a2 b2 c2

 b
2
D 
2  
(i) In equation ax + bx +=
c a x +  −
 2a  4a2 
 
4ac − b2 −b
(i) If a>0, the equation has minimum value = at x= and there is no maximum value.
4a 2a
4ac − b2 −b
(ii) If a < 0, the equation has maximum value at x = and there is no minimum value.
4a 2a
( j) For cubic equation ax3 + bx2 + cx + d =0,
−b
(i) α+β+ γ =
a
c
(ii) αβ + βγ + λα =
a
−d
(iii) αβγ = … where α , β, γ are its roots.
a
M a them a ti cs | 2.33

Solved Examples

JEE Main/Boards 2− 5
=
Example 1: For what values of ‘m’ does the quadratic (2 + 5 )(2 − 5 )
equation (1 + m) x2 – 2(1 + 3m)x + (1 + 8m) = 0 have
equal roots? = 5 −2
Then the other root, x2 + px + q = x will be −2 − 5 ,
Sol: The roots are equal if discriminant (D)= 0.
α + β = ‒4 and αβ = ‒1
(
4(1 + 3m)2 – 4(1 + m)(1 + 8m) = 0 ⇒ 4 m2 − 3m =
0 )
Thus, the required quadratic equation is :
⇒ m = 0, 3
x2 − ( α + β ) x + αβ = 0 Or , x2 + 4x − 1 =0
Example 2: When pr = 2(q + s), where p, q, r, s are real
numbers, show that at least one of the equations x2 + Example 4: The quadratic equations x2 ‒ ax + b = 0 and
px + q and x2 + rx +s = 0 has real roots.
x2 ‒ px + q = 0 have a common root and the second
ap
Sol: For at least one of the given equation has equal roots, show that b + q = .
2
equations to have real roots means one of
Sol: By considering α and β to be the roots of eq. (i) and
their discriminant must be non negative. α to be the common root, we can solve the problem by
using the sum and product of roots formulae.
The given equations are
The given quadratic equations are
f(α ) =0 + px + q = 0  … (i)
x2 − ax + b =0 ... (i)
f(α ) =0 + rx + s = 0  … (ii)
x2 − px + q =0 ... (ii)
consider D1 and D2 be the discriminants of equations (i)
and (ii) respectively, Consider α and β to be the roots of eq. (i) and α to be
the common root.
D1 + D2 = p2 ‒ 4q + r2 ‒4s
From (i) α + β = a, α = b
= p2 + r2 ‒ 4(q + s)
From (ii) 2α = p, α2 = q
= p + r ‒ 2pr
2 2
ap
∴ b + q = αβ + α2 = α ( α + β ) =
= (p ‒ r) ≥ 0 [ p and r are real]
2
2
 At least one of D1 and D2 must be non negative.
Example 5: If α and αn are the roots of the
Hence, at least one of the given equation has real roots. quadratic equation ax2 + bx + c = 0, then show that
1 1

Example 3: Find the quadratic equation where one of ( ) acn n+1


( )
+ anc n+1 +b =0.

1 Sol: By using the sum and product of roots formulae


the roots is .
2+ 5 we can prove this.
Sol: If one root is (α + β ) then other one will be
Given that α and αn are the roots.
(α − β ) . c
1 ∴ α.αn =
given α = a 1
2+ 5  c  n+1
⇒ α = 
Multiplying the numerator and denominator by a
2 − 5 , we get −b
And α + αn =
a
2 . 3 4 | Quadratic Equations and Inequalities

1
 c  n+1  c  n+1 −b
⇒  +  =
1
∴α +β =
−1
2
3 2
and 4α2 + 2α − 1 = 04α − 3α= 4α + 2α − 1 ( )  α − 12 
a a a
1 1 (  1 
) 1
4α3 − 3α= 4α2 + 2α − 1  α −  −  α +  = β
Or ( ) can n+1
( )
+ cna n+1 +b =0.  2  2

Hence 1 3
−  α + 4α= β− 3α is the other root.
2  2
Example 6: x2 + ax + bc = 0 and x + bx + ca = 0 have
a non zero common root and a ≠ b, then show that the 1 1 1
Example 9: the roots of + = are equal in
other roots are roots of the equation, x2 + cx + ab = 0, x +p x +q r
c ≠ 0. magnitude, but opposite in sign, show that p+ q = 2r
p2 + q2
Sol: By considering α to be the common root of and the product of the roots = −
2
the equations and β , γ to be the other roots of the
equations respectively, and then by using the sum and Sol: By considering α and − α as the roots of the given
product of roots formulae we can prove this. equation and then by using the sum and product of
roots formulae we can solve it.
Further, α + β = −a and αβ = bc ;
1 1 1
+ =  ….(i)
α + γ = −b , α. γ = ca x +p x +q r
2α + β + γ = − ( a + b ) and α2βγ = abc2 … (i) ⇒ (x + q + x +p)r = x2 + (p + q)x + pq

∴ β + γ = c − 2c = −c  (ii) … (ii) ⇒ x2 + (p + q – 2r)x + pq – r(p + q) = 0

and c2βγ = c2ba Since, its roots are equal in magnitude but opposite in
sign
∴ βγ = ab … (iii)
consider roots are α , − α.
From equation (ii) and (iii),
∴ α − α= p + q − 2r
β and γ are the roots of the equation x2 + cx + ab =
0
⇒ p+q=2r
Product of roots = pq – r(p + q)
Example 7: Solve for x when
(p + q)
2
p2 + q2
log10= (
log10 x log ) 2
x 
x: x >1 = pq –
2
= −
2
 

Sol: By using the formula Example 10: If α , β are the roots of x + px + q = 0. 2


log10 a
logaMx = x loga .M and logb a =
log10 b
we can Prove that
α
β
2 2
(
is a root of qx + 2q − p x + q =0 )
solve this problem. Sol: For
α
β
(
to be a root of qx2 + 2q − p2 x + q = 0 )
log10 ( log
=10 x log
= )2 x x
log10 x 1
=
log10 x2 2
it must satisfy thegiven equation. Hence by using sum
and product of roots formula, we can find out the value
1 α
of .
Let y = log10 x ; then = log10 y ; β
2
As α1β1γ1 are the roots of x2 + px + q =0
1 1
⇒ == log y ∴ y 10 and thus x  1010
=
2 2 10 α + β = –p and αβ = q
α
We need to show that is a root of
Example 8: If α is a root of the equation 4x2 + 2x ‒ 1= 0, β
then prove that 4α3 − 3α is the other root.
(
ax2 + 2q − p2 x + q =)
0

Sol: Consider α, β to be the two roots of the given That means


equation 4x2 + 2x − 1 =0 , therefore, by solving this we α2
can get the result. q
β2
(
+ 2q − p2 ) αβ + q =0
M a them a ti cs | 2.35

(
i.e., qα2 + 2q − p2 αβ + qβ2 = 0 ) Sol: Consider y ∈ R and also that given as x ∈ R. Hence,
ax2 + x − 2
(
i.e., q α2 + 2αβ + β2 − p2 αβ = 0 ) the discriminant of y =
than or equal to zero. a + x − 2x2
must be greater

i.e., q ( α + β ) − p αβ = 0
2 2
Let y ∈ R; then,
2 2
i.e., p q − p q =
0 which is obviously true. ax2 + x − 2
y= for some x ∈ R
a + x − 2x2
Example 11: Find the value of ‘a’ for which (a + 2y) x2 + (1 – y)x – 2 – ay = 0
(
3x2 + 2 a2 + 1 x + a2 − 3a + 2 = )
0 possesses roots with
∴ (1 – y)2 + 4(a +2y)(2 + ay) ≥ 0 ; ∀y ∈R
opposite signs.

Sol: Roots of the given equation are of opposite sign,


( )
Or ( 8a + 1 ) y 2 + 4a2 + 14 y + 8a + 1 ≥ 0

hence, their product is negative and the discriminant is ∀ y ∈ R ∴ 8a + 1 > 0 and


positive.
∴ Product of roots is negative (4a2 + 14)2 ‒ 4(8a+1)2 ≤ 0

a2 − 3a + 2 1

3
<0 Or a > −
8
( )
and a2 − 4a + 3 ( a + 2 ) ≤ 0
= (a – 2) (a – 1) < 0 and a ∈ (1, 2) And D > 0 1
Or a > − and (a – 3)(a – 1) < 0
( 2
)
4 a + 1 − 4.3 a − 3a + 2 > 0 ( 3
) 8
i.e. a ∈ [1, 3]
This equation will always hold true for a ∈ (1, 2)

Example 2: Find the value of x if


Example 12: If x is real, find the range of the
2x + 5 + | x2 + 4x +3| = 0
x2 + 14x + 9
quadratic expression .
x2 + 2x + 3 Sol: For 2x + 5 + | x2 + 4x +3| = 0, 2x + 5 must be less
x2 + 14x + 9 than or equal to zero. And whether x2 + 4x +3 will be
Sol: By considering = y and as x is real its positive or negative depends on the value of x.
x2 + 2x + 3
discriminant must be greater than or equal to zero. ⇒ 2x + 5 + | x2 + 4x + 3| = 0
2
x + 14x + 9
Let =y Case -I When x ≤ –3 or x ≥ -1
x2 + 2x + 3
⇒ x2 + 14x + 9= x2 y + 2xy + 3y x2 + 4x + 3 + 2x + 5 = 0

⇒ x2 (1 – y) + 2x(7 – y)+ 3 (3 – y) = 0 (x + 2)(x + 4) = 0 ; ⇒ x = –4

Hence, D ≥ 0 Case-II –3 < x < -1

4(7 – y) β – 12(1 – y)(3 – y) y2 0 x2 + 4x + 3 = 2x + 5 ; x2 + 2x – 2 = 0

– 2y γ – 2y + 40 y3 0 −1 − 3
⇒x=
2
⇒ y 2 + y − 20 ≤ 0
⇒ ≤ ⇒ ≤ ≤ Example 3: Solve the equation 2|x +1| − 2x = 2x − 1 + 1
(y + 5)(y – 4) 0 –5 y 4
Sol: By taking the conditions as x ≥ 0 and x ≤ 0 we can
solve this problem.
JEE Advanced/Boards
2x − 1 if x ≥ 0
x 
2 −1 =
Example 1: Prove that y =
2
ax + x − 2
2
 x
− 2 − 1 ( ) if x < 0
a + x − 2x
Case-I x ≥ 0
takes all real values for x ∈ R only if a ∈ [1, 3]
2 . 3 6 | Quadratic Equations and Inequalities

2|x +1| − 2x = 2x − 1 + 1 3 3
Case II : Let < x < 1 ; 2x – < x2
8 4
This is true ∀ x ≥ 0
Or 4 x2 – 8x + 3 > 0
Case-II x < 0 ; 2|x +1| − 2x =1 − 2x + 1
3 1
(2x – 3)(2x – 1)> 0; ∴ x ∈  , 
2|x +1| = 2 ; |x + 1| = 1 ; x = –2
8 2

Example 4: For what values of a are the roots of the Example 6: Solve the equation
(
2
)
equation a + 1 x − 3ax + 4a (
= 0 a ≠ − 1 real and less ) (2x 2
− 3x + 1 ) (2x 2
)9x2
+ 5x + 1 =
than 1?

Sol: Here the roots of the given equation have to be Sol: This problem is solved by dividing both sides by x2
real and less than 1, therefore D ≥ 0 ; f(1).(a + 1) > 0 1
and taking y = 2x +
and the x-coordinate of the vertex < 1 . x
Let f(x) = (a + 1) x2 − 3ax + 4a ( )(
2x2 − 3x + 1 2x2 + 5x + 1 = 9x2  ) ... (i)
Clearly, x = 0 does not satisfy (i), Therefore, we can
D ≥ 0 ; f(1).(a + 1) > 0 and x-coordinate of vertex < 1
rewrite equation (i) as
16
D ≥ 0 ⇒− ≤ a ≤ 0  … (i)  1  1
9  ... (ii)
7  2x − 3 +   2x + 5 +  =
 x  x
(a + 1)f(1) > 0 ⇒ (2a + 1) (a + 1) > 0 1
∴ (y – 3)(y + 5) = 9 where y = 2x +
1 x
⇒ a < –1 or a > −  ... (ii)
2 Or y2 + 2y ‒ 24 = 0
16 −1    ⇒ (y + 6)(y – 4) = 0 ⇒ y = 4, –6
By (i) & (ii) a ∈  − , −1  ∪  ,0   … (iii)
 7   2 
1
Since x coordinate of vertex x < 1, we have When y = –6, 2x + = –6
x
 −1  ⇒ 2 x2 + 6x + 1 = 0
Combined with (iii) we get: a ∈  ,0 
 2 
−6 ± 36 − 8 −3 ± 7
⇒ x= =
Example 5: Find all the values of x satisfying the 4 2
 3 1
inequality ⇒  2x −  > 2 .
When y = 4, 2x + =4
 4 x
Sol: First, we can reduce the given inequality as ⇒ 2 x2 – 4x + 1 =
 3
logx  2x −  > logx x2 . Then, by applying each case of ⇒ x=
4 ± 16 − 8 −2 ± 7
=
 4 4 2
3 −−33±± 77 −−22±± 22
x > 1 and < x < 1 we can solve this problem. Thus, the solutions are x = , .
8 22 22
 3 3
logx  2x −  > 2 (∴ x ≠ 1 and x > ) Example 7: If α and β are the roots of the equation a
 4 8
x2 + bx + c = 0, then find the equation whose roots are,
⇒ logx  2x − 3  > logx x2  … (i) α2 +
α
2
+ββ2,
2 1
+
1
?
 4 α2 β2
3
Case I : Let x > 1 ; 2x − > x2
4 Sol: Using the sum and product of roots formulae, we
can get the value of α and β and then by using
Or 4 ⇒ – 8x + 3 < 0
x2 − ( sum of roots ) x + (product of roots ) =
0
 1  3  3
Or 4  x −   x −  < 0 ∴ x ∈  1, 
 2  2  2 we can arrive at the required equation.

Let S be the sum and P be the product of the roots


M a them a ti cs | 2.37

α2 + β2 ,
1
+
1 a2 a
2 2 Hence – 11 + a = 0, a = 0 or a = 24
α β 9 3
Since a ≠ 0, a = 24
α2 + β2  b2 − 2ac   b2 − 2ac 
2 2
As S = α + β += (
 a2
+
  c2

 )  x2 − 11x + 24 =
∴ the common factor of 
0
( αβ )
2
    2
x − 14x + 48 =0
 a2 + c2 
2
(
= b − 2ac  2 2 
 ac 
 
) is clearly x – 8 or the common root is x = 8.
Note: A shorter method is in eliminating a from both
Now the product of the roots will be expressions

1 2x2 − 22x + 2a 2


α2 + β2
P==
( )  b − 2ac  c2

2
× 2
 ; x – 8x = 0
x2 − 14x + 2a 
⇒ x (x – 8) = 0
α2β2  a2  a
 
∴ x ≠ 0, ∴ (x – 8)
Hence equation is

( )( ) ( )
2
( acx )
2
− b2 − 2ac a2 + c2 x + b2 − 2ac 0
= Example 10: α and β are the roots of
a x2 + bx+ c= 0 and γ , δ be the roots of
Example 8: If an are the roots of a x2 + bx + c = 0 p x2 +qx + r = 0; . If α , β, γ , δ are
2
and γ , δ the roots of x + mx + n = 0 , then find the
equation whose roots are α γ + β δ and α δ + β γ ? in A.P., then find the ratio of their Discriminants.

Sol: In the method similar to example 8. Sol: As α , β, γ , δ are in A.P., hence, β − α = δ − γ , by


squaring both side and substituting their values we will
Here S = α γ +βδ + α δ +β γ ( ) ( ) get the result.
Consider D1 and D2 be their discriminants respectively
= α ( γ + δ ) + β ( γ + δ ) = ( α + β )( γ + δ )
−b c
We have α +
= β ,α
= β
a a
 −b   −m  bm
= =    … (i)
−q c
 a    a and γ +=
δ , γ=
δ
p a
Also P = ( α γ + βδ )( αδ + βγ ) Since, α , β, γ , δ are in A.P.

= (α 2
)
+ β2 γδ + αβ γ 2 + δ2 ( ) … (ii) ⇒β − α = δ − γ ; ( β − α ) = ( δ − γ )
2 2

(β + α ) ( γ + δ)
2 2
= b2 n  + m2ac − 4 acn  / a2 2 − 4αβ = − 4 γδ
b2 4c q2 4r
Hence, from x2 – Sx + P = 0 ⇒ − = −
a2 a p2 p
bm
2 b2n  + m2ac − 4acn
x − x+ 0
=
a a2 2 b2 − 4ac q2 − 4qr
⇒ =
a2 p2
Example 9: The expression x2 ‒ 11x + a = 0 and
D1 D2 D1 a2
x2 ‒ 14x + 2a = 0 must have a common factor and a ≠ = ⇒ =
0, Find the common factor and then the common root. a2 p2 D2 p2

Sol: Here consider (x ‒ α) to be the common factor then p a b


Example 11: The equation= + has two
x = α becomes the root of the corresponding equation. 2x x + c x − c
Hence, by substituting x = α in both the equations and equal roots and c ≠ 0 , then find the possible values of p?
solving we will get the result.
Sol: For equal roots discriminant(D) must be zero.
∴α2 − 11α + a= 0, α2 − 14 α + 2a= 0
a p a b
Subtracting 3α − a= 0 ⇒ α = As given= +
3 2x x + c x − c
2 . 3 8 | Quadratic Equations and Inequalities

p (a+ b) x + c(b− a) −b ± D
⇒ = Therefore by putting x2 + 2x = y and using x =
2x x 2 − c2 2a
we can solve this.
⇒ p(x2‒ c2) = 2(a + b)x2 ‒ 2c(a ‒ b)x
Put x2 + 2x = y  … (i)
⇒ (2a + 2b – p) x2 – 2c(a – b)x + p c2 = 0
(y – 80)(y – 24) = 660
For this equation to have equal roots
4c2 ( a − b ) − 4pc2 ( 2a + 2b − p ) =
2
0 ⇒ y2 ‒ 104y + 1920 ‒ 660 = 0

⇒ y2 ‒ 104y + 1920 = 0
⇒ ( a − b ) − 2p ( a + b ) + p2 =
2
0
⇒ (y – 90)(y – 14) = 0 ⇒ y = 90 or 14
⇒ p22 − 2p ( a + b ) =− (a − b)
2 2

When y = 90 (i) gives x2 + 2x − 90 =


0
⇒ p2 − 2p ( a + b ) + ( a + b ) = ( a + b ) − ( a − b )
2 2 22 2 2 2

− 2 ± 42 − 4 × ( − 90 )
[p ‒ (a + b)] = 4ab 2
x= =−1 ± 94
2
⇒ p – (a + b) = ± 2 ab
When y = 14, (i) gives x2 + 2x – 14 = 0

( )
2
⇒ p=a+b±2 ab = a± b
− 2 ± 42 − 4 × ( − 14 )
x= =−1 ± 3 2
2
Example 12: Solve (x + 10)(x – 4)(x – 8)(x + 6)= 660
x
The solutions are: −1 ± 3 2 & −1 ± 94
2
Sol: By multiplying (x + 10)(x – 8)(x – 4)(x + 6) we get x − 5x + 9
( x2 + 2x – 80)( x2 + 2x – 24) = 660.

JEE Main/Boards

Exercise 1 Q.6 Determine the values of m for which the equations


3 x2 + 4mx + 2 = 0 and 2 x2 + 3x – 2 = 0 may have a
Q.1 If the sum of the roots of the equation px2+qx+r=0 common root.

be equal to the sum of their squares, show that


Q.7 If α and β be the roots of the equation x2 ‒ px + q = 0,
2pr = pq + q2
1 1
find the equation whose roots are α + and β + .
Q.2 Show that the roots of the equation β α
(a + b)
2 2
x −2 a −b ( 2 2
) x + (a − b) 2
0 are equal.
= Q.8 Solve for x :
4x
2
≥1
x +3
Q.3 Find the value of m, for which the equation 5 x2 –
4x + 2 + m (4 x2 – 2x – 1) = 0 has Q.9 If c, d are the roots of the equation (x – a)(x – b) – k =0
show that a, b are the roots of the equation (x – c)(x – d)
(i) equal roots + k = 0.
(ii) product of the roots as 2
Q.10 Find the real values of x which satisfy x2 ‒ 3x + 2 > 0
(iii) The sum of the roots as 6
and x2 ‒ 3x ‒ 4 ≤ 0.
Q.4 If one root of the equation 5 x2 + 13x + k =0 be
Q.11 Let a, b, c, be real numbers with a ≠ 0 and let α , β
reciprocal of the other, find k.
2
be the roots of the equation a x2 + bx +c = 0. Express
Q.5 If the difference of the roots of x – px + q = 0 is
unity, then prove that p2 ‒ 4q = 1 the roots of a3x2 + abcx + c3 = 0 in terms of α, β.
M a them a ti cs | 2.39

Q.12 If a and b are integers and the roots of equation Q.24 Let α be a root of the equation a x2 + bx + c = 0
x2 + ax + b =0 are rational, show that they will be and β be a root of the equation – ax2 + bx + c = 0. Show
integers. a
that there exists a root of the equation x2 + bx + c = 0
2
Q.13 For what values of m, can the following expression
be split as product of two linear factors?
(
that lies between α and β α , β ≠ 0 . )
(i) 3x2 − xy − 2y 2 + mx + y + 1 Q.25 Let a, b and c be integers with a > 1, and let p be a
prime number. Show that if ax2 + bx + c is equal to p for
(ii) 6 x2 − 7xy − 3y 2 + mx + 17y − 20 two distinct integral values of x, then it cannot be equal
to “2p” for any integral value of x. (a ≠ p).
x2 − 2x + 4
Q.14 Prove that the expression lies between Q.26 For a ≤ 0, determine all real roots of the equation:
x2 + 2x + 4
1
and 3 for all real values of x. x2 − 2a x − a − 3a2 =0.
3

Q.15 Find all the values of a for which the roots of the Q.27 Find the values of a for which the inequality
equation (1 + a) x+ ‒ 3ax + 4a = 0 exceed unity. x2 + ax + a2 + 6a < 0 is satisfied for all x ∈ (1, 2).

Q.16 If P(x) = a x2 + bx + c and Q(x) = ‒ax2 + bx + c Q.28 If the roots of 2x3 + x2 − 7 = 0 are α , β and
where ac ≠ 0, show that the equation P(x). Q(x) = 0 has f(x) = x + x(4 − 2k) + k − 3k − 1 = 0 ,
2 2

at least two real roots.


α β
find the value of ∑  +  .
Q.17 If roots of the equation ax2 + 2bx + c = 0 be α and β α
β and those of the equation A x2 + 2Bx + C = 0 be α + Q.29 Find all values of k for which the inequality (x – 3k)
k and β + k, prove that: (x – k – 3)< 0 is satisfied for all x in the interval [1, 3].
2
b2 − aca
= 
B − AC  A 
2
Exercise 2
(15 + 4 14 ) + (15 − 4 14 ) =
t t
Q.18 Solve for x: 30 Single Correct Choice Type
where t = x2 − 2| x | . Q.1 If a2 + b2 + c2 = 1 then ab + bc + ca lies in the
interval (a, b, c, ∈ R)
Q.19 Show that (x – 2)(x - 3) – 8 (x – 1)(x – 3) + 9 (x – 1)
(x – 2) = 2 x2 is an identity. 1
(C)  − 1 ,1  (D)  −1, 1 
    
(A)  ,2 (B) [–1, 2 ]
2   2   2
Q.20 For which values of a does the equation
2
 2   2  Q.2 If P(x) = ax2 + bx + c and Q(x) = – ax2 + dx + c,
(1 + a)  2x  − 3a  2x  + 4a =
0 have real roots? where ac ≠ 0, then P(x). Q(x) = 0 has
 x +1  x +1
(A) Exactly one real root
( )
Q.21 If one root of the equation l − m x + lx + 1 =0 be 2
(B) At least two real roots
9
double of the other and if l be real, show that m ≤ . (C) Exactly are real roots
8
2
(D) All four are real roots
Q.22 If a x2 + 2bx + c = 0 and a1 x + 2b1 x + c1 =
0
have a common root and a , b , c are in A.P show Q.3 If α and β be the roots of the equation x2 + 3x + 1 =0
a1 b1 c1 2 2
that a1 , b1 , c1 are in G.P.  α   β 
then the value of   +  is equal to
1 + β  α +1
Q.23 If the ratio of the roots of the equation a x2 + bx
+ c = 0 be equal to that of the roots of the equation (A) 15 (B) 18 (C) 21 (D) None of these
2
b  ca
a1x2 + 2b1x +c1 = 0, prove that   =

 b1  c1a1
2 . 4 0 | Quadratic Equations and Inequalities

Q.4 Let a > 0, b > 0 & c > 0. Then both the roots of the
equation a x2 + bx + c = 0
( ) ( ) (
(A) − ∞ , 0 ∪ 6, ∞ (B) − ∞ , 0  ∪ 6, ∞ ( )
(A) Are real & negative (C) ( − ∞ , 0  ∪ 6, ∞ ) (D) (0, 6)
(B) Have negative real parts
Q.12 If α , β are roots of the equation
(C) Are rational numbers
x2 − 2mx + m2 − 1 =0 then the number of integral
(D) None values of m for which α , β ∈ (–2, 4) is
(A) 0 (B) 1 (C) 2 (D) All of these
Q.5 The equation x2 + bx + c = 0 has distinct roots. If 2
is subtracted from each root, the results are reciprocals
(
of the original roots. The value of b2 + c2 + bc equals ) Q.13 If x be the real number such that x3 + 4x =
the value of the expression x7 + 64x2 is
8 then

(A) 7 (B) 9 (C) 10 (D) 11


(A) 124 (B) 125 (C) 128 (D) 132
Q.6 If a, b, c are real numbers satisfying the condition
a + b + c = 0 then the roots of the quadratic equation Q.14 If a and b are positive integers and each of the
3ax2 + 5bx + 7c = 0 are: equations x2 + ax + 2b = 0 and x2 + 2bx + a = 0 has
real roots, then the smallest possible value of (a + b) is
(A) Positive (B) Real & distinct
(A) 3 (B) 4 (C) 5 (D) 6
(C) Negative (D) Imaginary

Q.15 Let ‘a’ be a real number. Number of real roots of


Q.7 If one solution of the equation x3 − 2x2 + ax + 10 =0
is the additive inverse of another, then which one of the
( )(
the equation x2 + ax + 1 3x2 + ax − 3 =0 is )
following inequalities is true ? (A) At least two (B) At most two
(A) –40 < a < –30 (B) –30 < a < –20 (C) Exactly two (D) All four
(C) –20 < a < –10 (D) –10 < a < 0
Q.16 Let f(x) = x2 + ax + b . If the maximum and the
Q.8 The sum of the roots of the equation minimum values of f(x) are 3 and 2 respectively for 0 ≤
x ≤ 2, then the possible ordered pair (s) of (a, b) is/are
( ) (
(x + 1) = 2 log2 2x + 3 − 2 log4 1980 − 2− x is )  3
(C)  − 5 , 3  (D)  − 5 ,2 
    
(A) (–2, 3) (B)  − ,2 
(A) 3954 (B) log2 11  2   2   2 
(C) log2 3954 (D) Indeterminate

Q.9 The quadratic equation x2 – 1088x + 295680 = 0 Previous Years’ Questions


has two positive integral roots whose greatest common
divisor is 16. The least common multiple of the two Q.1 The smallest value of k, for which both the roots
roots is of the equation x2 − 8kx + 16 k 2 − k + 1 = ( 0 are real, )
(A) 18240 (B) 18480 distinct and have values at least 4, is ……….. (2009)
(C) 18960 (D) 19240
Q.2 Find the set of all x for which
2 2x 1
Q.10 If x is real and 4 y + 4xy + x + 6 = 0, then the >  (1987)
complete set of values of x for which y is real is 2x2 + 5x + 2 x + 1

(A) x ≤ –2 or x ≥ 3 (B) x ≤ 2 or x ≥ 3
Q.3 Let a, b, c be real numbers with a ≠ 0 and let α , β be
(C) x ≤ –3 or x ≥ 2 (D) –3 ≤x≤ 2 the roots of the equation ax2 + bx + c = 0 . Express the
roots of a3 x2 + abcx + c3 =0 in terms of α , β .  (2001)
Q.11 If exactly one root of the quadratic equation
()
f x = 0 – (a + 1)x + 2a = 0 lies in the interval (0, 3) Q.4 If α , β are the roots of ax2 + bx + c = 0 , (a ≠ 0)
then the set of values ‘a’ is given by and α + δ, β + δ are the roots of Ax + Bx + C =
2
0 , (A ≠
0) for some constant δ , then
M a them a ti cs | 2.41

b2 − 4ac B2 − 4AC Q.9 Let α, β be the roots of the equation


prove that =  (2000)
a 2
A 2
x2 − px + r =0 and α , 2 β be the roots of the equation
2
Q.5 Let a, b, c, be real. If ax2 + bx + c =0 has two roots
x2 − qx + r =0 . Then, the value of r is  (2007)
α and β , where α < –1 and β > 1, then 2
c b
(A)
9
(p − q) ( 2q − p ) (B) 92 ( q − p ) ( 2p − q)
show that 1 + + < 0  (1995)
a a
2 2
(C)
9
( q − 2p ) ( 2q − p ) (D) ( 2p − q) ( 2q − p )
9
y
a<0
y=ax2+bx+c Q.10 If one root is square of the other root of the

-1 1 equation x2 + px + q =0 , then the relation between p


 0 
x
y and q is  (2004)

a>0
3
( ) 2
(A) p − 3 3p − 1 q + q =0
(B) p − q ( 3p + 1 ) + q =
3 2
2
y=ax +bx+c 0
x
(C) p + q ( 3p − 1 ) + q =
 3 2
-1 0 1  0
(D) p + q ( 3p + 1 ) + q =
3 2
0

Assertion Reasoning Type (


Q. 11 For all ‘x’, x2 + 2ax + 10 − 3a > 0 , then the )
interval in which ‘a’ lies is  (2004)
For the following questions, choose the correct answer
(A) a < –5 (B) –5 < a < 2
from the codes (a). (b), (c) and (d) defined as follows.
(C) a > 5 (D) 2 < a < 5
(A) Statement-I is true, statement-II is true and
statement-II is correct explanation for statement-I
Q.12 The set of all real numbers x for which
(B) Statement-I is true, statement-II is true and
statement-II is NOT the correct explanation for x2 − x + 2 + a > 0 is  (2002)
statement-I.
(C) Statement-I is true, statement-II is false.
( ) ( )
(A) − ∞ , − 2 ∪ 2, ∞ (
(B) − ∞ , − 2 ∪ ) ( 2, ∞ )
(C) ( − ∞ , − 1 ) ∪ (1, ∞ ) (D) ( 2, ∞ )
(D) Statement-I is false, statement-II is true.

Q.6 Let a, b, c, p, q be the real numbers. Suppose f k 2 ( ) Q.13 The number of solutions of log4 x −= (
1 log2 x − 3 ) ( )
are the roots of the equation is  (2001)

x2 + 2px + q =0 and 1 (A) 3 (B) 1 (C) 2 (D) 0


α, are the roots of the equation
β 2
ax2 + 2bx + c =0, where β ∉ {−1, 0 , 1} . 
( )
Q.14 If α and β α < β are the roots of the equation
(
Statement-I : p − q b − ac ≥ 0 and
2
)( 2
) x2 + bx + c =0 where c < 0 < b, then  (2000)

Statement-II : b ≠ pa or c ≠ qa  (2008) (A) 0 < α < β (B) α < 0 < β < | α |


(C) α < β < 0 (D) α < 0 | α | < β
Q.7 The sum of all real roots of the equation
2
0 is ……
x−2 + x−2 −2 = (1997) Q.15 The equation x + 1 − x − 1= 4x − 1 has (1997)
(A) No solution (B) One solution
Q.8 A value of b for which the equations x2 + bx − 1 =0,
x2 + x + b =0 have one root in common is  (2011) (C) Two solution (D) More than two solution

(A) − 2 (B) −i 3 (C) i 5 (D) 2


2 . 4 2 | Quadratic Equations and Inequalities

Q.16 The quadratic equations x2 – 6x + a = 0 and x2 – cx Q.19 Let α and β be the roots of equation px2 + qx + r = 0,
+ 6 = 0 have one root in common. The other roots of 1 1
the first and second equations are integers in the ratio p ≠ 0. If p, q, r are in A.P. and + =4 , then the value
α β
4 : 3. Then the common root is (2008) of |α – β| is (2014)

(A) 1 (B) 4 (C) 3 (D) 2


34 2 13
(A) (B)
9 9
Q.17 If the roots of the equation bx2 + cx + a =0 be
imaginary, then for all real values of x, the expression 61 2 17
(C) (D)
3b2 x2 +6bcx + 2c2 is  (2009) 9 9
(A) Greater than 4ab (B) Less than 4ab
(C) Greater than – 4ab (D) Less than – 4ab Q.20 If the equations x2 + 2x + 3 = 0 and ax2 + bx + c = 0,
a,b,c R, have a common root, then a : b : c is(2013)
Q.18 Let α and β be the roots of equation x2 – 6x – 2 = 0. (A) 1 : 2 : 3 (B) 3 : 2 : 1
a − 2a8 (C) 1 : 3 : 2 (D) 3 : 1 : 2
If an = αn ‒ βn, for , then the value of 10 is equal
2a9
to : (2015)
(A) 6 (B) – 6 (C) 3 (D) –3

JEE Advanced/Boards

Exercise 1 2
Q.4 When y + my + 2 is divided by (y – 1) then
the quotient is f(y) and the remainder is R1. When
Q.1 A quadratic polynomial y 2 + my + 2 is divided by (y + 1) then quotient is
g(y) and the remainder is R2. If R1 = R2, find the value
f ( x ) = x2 + ax + b is formed with one of its zeros of m.
4+3 3
being where a and b are integers Also, Q.5 Find the value of m for which the quadratic
2+ 3
0 and x2 − 14x + 2m =
equations x2 − 11x + m = 0 may
g ( x ) = x 4 + 2x3 − 10 x2 + 4x − 10 is a biquadrate have common root.

4+3 3
polynomial such that g  =  c 3 + d where c Q.6 The quadratic polynomial P(x) = ax2 + bx + C
 2+ 3 
  has two different zeroes including –2. The quadratic
polynomial Q ( x ) = ax2 + cx + b has two different
and d are also integers. Find the values of a, b, c and d.
zeroes including 3. If α and β be the other zeroes of P(x)
α
and Q(x) respectively then find the value of .
Q.2 Find the range of values of a, such β
Instructions for Q.7 and Q.8
ax2 + 2 ( a + 1 ) x + 9a + 4
that f(x) = is always negative. Let α , β , γ be distinct real numbers such
x2 − 8x + 32
2
that a α + b α + c= ( sin θ) α2 + ( cos θ) α
Q.3 Let a, b, be arbitrary real numbers. Find the smallest
natural number ‘b’ for which the equation
a β2 + b β + c= ( sin θ) β2 + ( cos θ) β
x2 + 2 ( a + b ) x + ( a − b + 8 ) =0 has unequal real roots
a γ 2 + b γ + c= ( sin θ) γ2 + ( cos θ) γ
for all a ∈ R. (where a, b, c, ∈ R.)
M a them a ti cs | 2.43

( )
Q.7 log|x + 6| 2 .log2 x2 − x − 2 ≥ 1 ( ) Q.14 If the range of m, so that the equations


( x + 2mx + 7m − 12) = 0
2

Q.8 If

V1= sin θ ˆi + cos θ ˆj makes an angle π /3 with ( 4x − 4mx + 5m − 6 ) =
2
0

V2 = ˆi + ˆj + 2 kˆ then find the number of values of have two distinct real roots, is (a, b) then find (a + b).

θ ∈ 0, 2 π .
Q.15 Match the column

Q.9 (a) If α , β are the roots of the quadratic equation


Column I Column II
ax2 + bx + c = 0 then which of the following expressions
in α , β will denote the symmetric functions of roots. (A) Let α and β be the roots of a (p) 0
quadratic equation
Give proper reasoning.
4x2 − (5p + 1 ) x + 5p =
0
( ) 2
(i) f α , β = α − β
if β = 1 + α
(ii) f ( α , β ) = α2β + αβ2
Then the integral value of p, is
α
(iii) f ( α , β ) =ln
β (B) Integers laying in the range of (q) 1
(iv) f ( α=
, β ) cos ( α − β ) the expression
x2 − 3x + 4
( )
(b) If a,β are the roots of the equation x − px + q =0, 2 y=
x2 + 3x + 4
is (are)
then find the quadratic equation the roots of which are

(α )( α )&α
3 2
2
− β2 3
− β3 β − α2 β3 . (C) Positive integral values of x (r) 2
satisfying
x +1 x +5
Q.10 Find the product of the real roots of the ≥ , is (are)
x −1 x +1
equation x2 + 18x + 30= 2 x2 + 18 x + 45
(D) The value of expression (s) 3
x2 + ax + 4 2π 4π 4π 4
Q.11 Let f(x) = is defined for all real, then sin sin + sin
x2 + bx + 4
7 7 7
, is
find the number of possible ordered pairs
8π 8 π 2π
sin + sin sin
7 7 7
(a – b) (where a, b, ∈ I).

Q.12 If the equation 9x2 − 12ax + 4 − a2 =0 has a unique Q.16 Find the product of uncommon real roots of the
root in (0, 1) then find the number of integers in the two polynomials
range of a.
P ( x ) = x 4 + 2x3 − 8x2 − 6x + 15 and

Q.13 (a) Find all real numbers x such that. Q ( x ) = x3 + 4x2 − x − 10 .


1 1
 1 2  1 2
 x −  + 1 −  = x Q.17 Solve the following where x ∈ R .
 x   x
6
 1 6 1 (a) ( x − 1 ) x2 − 4x + 3 + 2x2 + 3x − 5 =0
x +  − x − 6 − 2
x x
(b) Find the minimum value of  for
3
x>0  1 3 1 (b) 3 x2 + 4x + 2 = 5x − 4
x +  +x + 3
 x x
(c) x3 + 1 + x2 − x − 2 =0

(d) 2( x + 2) − 2x +1 − 1= 2x +1 + 1
2 . 4 4 | Quadratic Equations and Inequalities

(e) For a ≤ 0, determine all real roots of the equation Q.25 Find the complete set of real values of ‘a’
2
x − 2a x − a − 3a =
0. 2 for which both roots of the quadratic equation
(a2
) (
0 lie on
− 6a + 5 x2 − a2 + 2a x + 6a − a2 − 8 = )
Q.18 (a) Let α , β and γ are the roots of the cubic either side of the origin.
x3 − 3x2 + 1 =0 . Find a cubic whose
Solve the inequality.
α β γ
roots are , and . 2
α−2 β−2 γ −2  x5 
( )
4
Q.26 log2 x  log1  − 20 log2 x + 148 < 0
Hence or otherwise find the value of ( α − 2 ) ( β − 2 ) ( γ − 2 ) .  4
 2 
(b) If α , β , γ are roots of the cubic 2011
Q.27 (log 100 x ) + (log 10 x ) + log x ≤ 14
2 2
x3 + 2x2 + 1 =0 , then find
(i) ( α β )
−1
+ (β γ )
−1
+ (γ α)
−1
(ii) α −2 + β−2 + γ −1
( )
Q.28 log1/2 x + 1 > log2 2 − x ( )
Q.19 If the range of parameter t in the interval (0, 2π),
satisfying (
Q.29 log1/5 2x2 + 5x + 1 < 0 )
( −2x 2
+ 5x − 10 )
( sin t ) x2 + 2 (1 + sint ) x + 9 sin t + 4
Exercise 2
for all real value of x is (a, b), then a + b =
kπ . ( )
Find the value of k. Single Correct Choice Type

Q.1 Let r1, r2 and r3 be the solutions of equation


Q.20 Find all numbers p for each of which the least
value of the quadratic trinomial 0 then the value of
x3 − 2x2 + 4x + 5074 =

4x2 − 4px + p2 − 2p + 2 on the interval 0 ≤ x ≤ 2 is (r1 + 2) (r2 + 2) (r3 + 2) is


equal to 3.
(A) 5050 (B) 5066 (C) –5050 (D) –5066

() 2
Q.21 Let P x = x + bx + c where b and c are
integers. If P(x) is a factor of both x 4 + 6x2 + 25 and Q.2 For every x ∈ R , the polynomial x8 − x5 + x2 − x + 1 is
3x 4 + 4x2 + 28x + 5 . Find the value of P(1). (A) Positive
(B) Never positive
Q.22 If α , β are the roots of the equation,
(C) Positive as well as negative
x2 − 2x − a2 + 1 =0 and γ , δ are the roots of the
(D) Negative
2
(
equation, x − 2 a + 1 x + a a − 1 = )
0 such that ( )
α , β ∈( γ , δ ) then find the value of ‘a’.
Q.3 If the equation a(x ‒ 1)2 + b(x2 ‒3x + 2) + x ‒ a2 = 0 is
satisfied for all x ∈ R then the number of ordered pairs
Q.23 Let A denotes the set of values of x for which
of (a, b) can be
x+2
≤ 0 and B denotes the set of values of x for (A) 0 (B) 1 (C) 2 (D) Infinite
x−4
2
which x − ax − 4 ≤ 0 . If B is the subset of A, then find
the number of possible integral values of a. Q.4 The inequality The inequality y(‒1)≥ ‒4, y(1) ≤ 0 and
y(3)≥5 are known to hold for y = ax2+ bx+ c then the
least value of ‘a’ is :
Q.24 The quadratic ax2 + bx − c = 0 has two
different roots including the root ‒2. The equation (A) – 1/4 (B) –1/3 (C) 1/4 (D) 1/8
ax2 + cx + b =0 has two different roots including the
root 3. The absolute value of the product of the four
roots of the equation expressed in lowest rational is
p
  . Find (p+ q).
q
M a them a ti cs | 2.45

4λ 2 − 2 λ2 Q.11 The absolute term in P(x) has the value equal to


Q.5 If x = 2
and y = 2
where
1+λ 1+ λ 5 −1 5 −1 5 +1 1
(A) (B) (C) (D)
4 16 16 16
λ is a real parameter, and x2 ‒ xy + y2 lies between
[a, b] then (a + b) is
Assertion Reasoning Type
(A) 8 (B) 10 (C) 13 (D) 25
(A) Statement-I is true, statement-II is true and
statement-II is correct explanation for statement-I
Multiple Correct Choice Type
(B) Statement-I is true, statement-II is true and
Q.6 If the quadratic equations x + abx + c = 0 2 statement-II is NOT the correct explanation for statement-I.
and x2 + acx + b = 0 have a common root then the (C) Statement-I is true, statement-II is false.
equation containing their other roots is/are:
(D) Statement-I is false, statement-II is true.
2
( ) 2
(A) x + a b + c x − a bc =
0
Q.12 Consider a cubic function
(B) x − a (b + c ) x + a bc =
2 2
0
f ( x ) = ax3 + bx + c where a, b, c ∈ R.
(C) a (b + c ) x − (b + c ) x + abc =
2
0
Statement-I: f(x) can not have 3 non - negative real roots.
(D) a (b + c ) x + (b + c ) x − abc =
2
0
Statement-II: Sum of roots is equal to zero.
Q.7 If one of the roots of the equation 4x2 ‒ 15x + 4p = 0
is the square of the other, then the value of p is Q.13 Consider two quadratic functions

(A) 125/64 (B) –27/8 (C) –125/8 (D) 27/8 f ( x ) = ax2 + ax + ( a + b ) and g(x) = ax2 + 3ax + 3a + b,
where a and b non-zero real numbers having same sign.
Q.8 For the quadratic polynomial f(x) = 4x ‒ 8kx + k, 2
Statement-I: Graphs of the both y = f(x) and y = g(x)
the statements which hold good are either completely lie above x-axis or lie completely
(A) There is only one integral k for which f(x) is non below x-axis ∀ x ∈R .because
negative ∀ x ∈ R Statement-II: If discriminant of f(x), D < 0, then y = f(x)
(B) for k < 0 the number zero lies between the zeros of ∀x ∈ R is of same sign and f(x+1) will also be of same
the polynomial. sign as that of f(x) . ∀x ∈ R
(C) f(x) = 0 has two distinct solution in (0, 1) for k ∈
(1/4, 4/7) Match the Columns
(D) Minimum value of y ∀ k ∈R is k (1+ 12k) ( )
Q.14 It is given that α , β β ≥ α are the roots of the
() 2
equation if f x = ax + bx + c . Also a f(t) > 0.
Q.9 The roots of the quadratic equation x2 ‒ 30x + b = 0
are positive and one of them is the square of the other. Match the condition given in column I with their
If the roots are r and s with r > s then corresponding conclusions given in column II.

(A) b + r – s = 145 (B) b + r + s = 50


Column I Column II
(C) b – r – s = 100 (D) b – r + s = 105
(A) a > 0 and b² > 4ac (p) t≠α

Comprehension Type (B) a > 0 and b² = 4ac (q) no solution

Consider the polynomia (C) a < 0 and b² > 4ac (r) α<t<β
(D) a < 0 and b² = 4ac (s) t < α or r > β
P ( x ) = ( x − cos 36° )( x − cos 84° )( x − cos156° )

Q.10 The coefficient of x2 is


1 5 −1
(A) 0 (B) 1 (C) − (D)
2 2
2 . 4 6 | Quadratic Equations and Inequalities

Q.15 Match the conditions on column I with the (C) Suppose a cubic polynomial f(x) = (r) 277
intervals in column II. x3+px2+qx+72 is divisible by both
() 2 2
Let f x = x − 2px + p − 1 , then x2+ax+b and x2+bx+a (where a, b, p,
q are cubic polynomial and a ≠ b).
The sum of the squares of the roots
Column I Column II
of the cubic polynomial, is
(A) Both the roots of f(x) = 0 are (p) (–1, ∞)
(s) 298
less than 4, if p ∈ R
(B) Both the roots of f(x) = 0 are (q) (–∞, 3)
greater than –2 if p ∈ R
Previous Years’ Questions
(C) exactly one root of f(x) = 0 (r) (0, 2)
lie in (–2, 4), if p ∈ R Q.1 Let (x, y, z) be points with integer coordinates
(D) 1 lies between the roots of (s) (–3, –1) U (3,5) satisfying the system of homogeneous equations
f(x) = 0, if p ∈ R 3x − y − z = 0, − 3x + z = 0, − 3x + 2y + z = 0 . Then the
number of such points for which x2 + y2 + z2 ≤ 100 is
Q.16 ……… (2009)

Column I Column II Q.2 If x2 ‒ 10ax ‒ 11b = 0 have roots c and d.


x2 ‒ 10cx ‒ 11d = 0 have roots a and b, then find
(A) The minimum value of (p) 2
a + b + c + d.  (2006)
6
 1  6 1 
x +  − x + 6  − 2
 x  x  for x>0
2
( ) (
Q.3 If x + a − b x + 1 − a − b = )
0 where a, b, ∈ R, then
 1
3
1 find the values of a for which equation has real and
3
x +  + x + 3 unequal roots for all values of b.  (2003)
 x x

(B) The integral values of the parameters c (q) 4 Q.4 Let -1 ≤ p < 1. Show that the equation
for which the inequality 4x3 ‒ 3x ‒ p=0 has a unique root in the interval [1/2, 1]
and identify it,  (2001)
 7
1 + log2  2x2 + 2x +  ≥ log2 (cx2 + c)
2

has at least one solution is
2
()
Q.5 Let f x = Ax + Bx + C where, A, B, C, are real
numbers. Prove that if f(x) is an integer whenever x is an
(C) Let P(x) = x2+bx+c, where b and c (r) 6 integer, then the numbers 2A, A + B and C are all integers.
are integers. If P(x) is a factor of both Conversely prove that if the numbers 2A, A + B and C are
x4+6x2+25 and 3x4+4x2+28x5, then the all integers, then f(x) is an integer whenever x is an integer.
value of P(1) equals  (1998)
(s) 8
Q.6 Find the set of all solution of the equation
y
2 − 2y −1 − 1= 2y −1 + 1  (1997)
Q.17

Column I Column II
logequation
Q.7 Solve x in the following
2
(
( 2x +3) 6x + 23x + 21 )
(A) α, β are the roots of the equation K (p) 146 (
log( 2x +3) 6x2 + 23x + 21 = ) (
4 − log(3x + 7 ) 4x2 + 12x + 9 )
(x2-x) + x + 5 = 0. If K1 & K2 are the
two values of K for which the roots
=
(
4 − log(3x + 7 ) 4x2 + 12x + 9 ) (1987)

α, β are Connected by the relation Passage Based Questions


(α / β) + (β / α) = 4/5. The value of
Read the following passage and answer the questions.
(K1/K2)+(K2/K1) equals.
(B) If the range of the function (q) 254 Paragraph 1: If a continuous f defined on the real line
x2 + ax + b R, assumes positive and negative values in R, then the
f(x) = is [-5, 4],
x2 + 2x + 3 equation f(x) = 0 has a root in R. For example, If it is
Then, the value of a2 + b2 equals to known that a continuous function f on R is positive at
some point and its minimum value is negative.
M a them a ti cs | 2.47

Then the equation f(x) = 0 has a root in R. Consider Q.15 Let p and q be real numbers such that p ≠ 0, p³ ≠
()
f=x kex − x for all real x where k is real constant. (2007) q and p³ ≠ –q. If α and β are non-zero complex numbers
3 3
satisfying α +β = –p and α + β = q , then a quadratic
x
Q.8 The line y = x meets y = ke for k ≤ 0 at α β
equation having and as its roots is  (2010)
(A) No point (B) One point β α

(C) Two point (D) More than two points ( ) ( ) (


(A) p3 + q x2 − p3 + 2q x + p3 + q =
0 )
Q.9 The positive value of k for which kex − x =0 has ( ) ( ) (
(B) p3 + q x2 − p3 − 2q x + p3 + q =
0 )
only one root is 

(A)
1
(B) 1 (C) e (D) loge 2 ( ) ( ) (
(C) p3 − q x2 − 5p3 − 2q x + p3 − q =
0 )
e

Q.10 For k > 0, the set of all values of k for which


( ) ( ) (
(D) p3 − q x2 − 5p3 + 2q x + p3 − q =
0 )
kex − x =0 has two distinct root, is 
Q.16 If a, b, c, are the sides of a triangle ABC such that
1
(B)  1 , 1  (C)  1 , ∞ 
    
(A)  0,  (D) (0, 1)
 e e  e  x2 − 2 ( a + b + c ) x + 3 λ ( ab + bc + ca) =0 has real roots,
then  (2006)
Q.11 Consider the polynomial f(x) = 1 + 2x + 3x + 4x . 2 3 4 5
(A) λ < (B) λ <
Let s be the sum of all distinct real roots of f(x) and let 3 3
t = |s| . The real numbers s lies in the interval  (2010)
(C) λ ∈  4 , 5  (D) λ ∈  1 , 5 
   
 1   3  3 3 3 3
(A)  − ,0  (B)  −11, − 
 4   4
Q.17 If b > a, then the equation (x – a)(x – b) – 1 = 0 has
(C)  − 3 , − 1 
 
(D)  0, 1 
   (2000)
 4 2   4  (A) Both roots in (a, b)

Q.12 The area bounded by the curve y = f(x) and the


( )
(b) Both roots in − ∞ ,a
lines x= 0, y = 0 and x = t, lies in the interval (2010) (C) Both roots in (b + ∞ )
(D) One root in ( − ∞ ,a ) and the other in (b, ∞ )
(A)  3 , 3  (B)  21 , 11 
   
4   64 16 
Q.18 If the roots of the equation x2 − 2ax + a2 + a − 3 =0
 21 
(C) (9, 10) (D)  0,  are real and less than 3, then (1999)
 64 
(A) a < 2 (B) 2 ≤ a ≤ 3 (C) 3 < a ≤ 4 (D) a > 4
Q.13 The function f’(x) is  (2010)
Q.19 Let f(x) be a quadratic expression which is positive
 1  1 
(A) Increasing in  −t, −  and Decreasing  − , − t  for all real values of x. If g(x) = f(x) + f’(x) + f’’(x), then
 4  4  for any real x  (1990)
(B) Decreasing in  −t, − 1  and Increasing  − 1 , − t 
   
(A) g(x) < 0 (B) g(x) > 0 (C) g(x) = 0 (D) g(x) ≥ 0
 4  4 
(C) Increasing in (–t, t)
Q.20 Let α, β be the roots of the equation x2 − px + r = 0
(D) Decreasing in (–t, t) and be the roots of the equation x2 − qx + r = 0. Then
the value of r is (2007)
Q.14 Let α and β be the roots of x2 ‒ 6x ‒ 2 = 0, with 2 2
n n (A) (p − q)(2q − p) (B) (q − p)(2p − q)
α >β. If an = α − β for n ≥ 1, then the value of 9 9
a10 − 2a8
is. (2011) 2 2
2a9 (C) (q − 2p)(2q − p) (D) (2p − q)(2q − p)
9 9
(A) 1 (B) 2 (C) 3 (D) 4
2 . 4 8 | Quadratic Equations and Inequalities

Q.21 Let a, b, c, p, q be real numbers. Suppose α, β are Q.22 Let b = 6, with a and c satisfying (E). If α and β are
1 the roots of the quadratic equation ax2 + bx + c = 0,
the roots of the equation x2 + 2px + q = 0 and α, are ∞ n
β  1 1
the roots of the equation ax2 + 2bx + c = 0, where β2 ∉
then ∑  α + β  is (2011)
n= 0  
{−1, 0, 1}.
6
(A) 6 (B) 7 (C) (D) ∞
Statement-I: (p2 − q) (b2 − ac) ≥ 0 and 7
Statement-II: b ≠ pa or c ≠ qa (2008)
Q.23 A value of b for which the equations
(A) Statement-I is True, statement-II is True; statement-II x2 + bx - 1 = 0
is a correct explanation for statement-I
x2 + x + b = 0,
(B) Statement-I is True, statement-II is True; statement-II
is NOT a correct explanation for statement-I. have one root in common is (2011)

(C) Statement-I is True, statement-II is False (A) − 2 (B) ‒i 3


(D) Statement-I is False, statement-II is True (C) i 5 (D) 2

PlancEssential Questions
JEE Main/Boards JEE Advanced/Boards

Exercise 1 Exercise 1
Q. 12 Q. 15 Q. 18 Q. 7 Q. 10 Q. 13
Q. 20 Q. 22 Q. 24 Q. 17 Q. 19 Q. 22
Q. 25 Q. 30
Exercise 2
Exercise 2
Q. 3 Q. 8 Q. 9
Q. 5 Q. 9 Q. 12
Q. 11 Q. 14 Q. 17
Q. 16
Previous Years’ Questions
Previous Years’ Questions
Q. 2 Q. 5 Q. 6
Q. 5 Q. 6 Q. 8
Q. 15
Q. 13
M a them a ti cs | 2.49

Answer Key

JEE Main/Boards

Exercise 1
 −16 
−6 −8 −13 Q.15 a ∈  , − 1
Q.3 (i)m = , 1 (ii) m = (iii) m =  7 
5 9 11
Q.4 K = 5 Q.18 x =± 1, ± 1 + 2 ( )
−11 7
Q.6 m = , Q.20 0
8 4

Q.7 qx2 − p ( q + 1 ) x + ( q + 1 ) =
2
0 (
Q.26 −a 1 + 6 ,a 1 + 2) ( )
Q.8 1 ≤ x ≤ 3
Q.27 −7 − 3 5 ≤ a ≤ − 4 + 2 3
Q.10 −1 ≤ x < 1 or 2 < x ≤ 4 2

Q.28 –3
Q.11 α2 β, α β2
 1
−7 98 Q.29 k ∈  0, 
Q.13 (i) 4, (ii) 7,  3
2 3

Exercise 2

Single Correct Choice Type

Q.1 C Q.2 B Q.3 B Q.4 B Q.5 A Q.6 C


Q.7 D Q.8 B Q.9 B Q.10 A Q.11 B Q.12 D
Q.13 C Q.14 D Q.15 A Q.16 B

Previous Years’ Questions


 2 1
Q.1 k = 2 Q.2 x ∈ ( −2, − 1 ) ∪  − , −  Q.3 x = α2 β, αβ2 Q.6 B Q.7 4
 3 2

Q.8 B Q.9 D Q.10 A Q.11 B Q.12 B Q.13 B

Q.14 B Q.15 A Q.16 D Q.17 C Q.18 C Q.19 B

Q.20 A

JEE Advanced/Boards

Exercise 1
Q.2 a∈ − ∞ , − 1  ;
 
Q.1 a = 2, b = –11, c = 4, d = –1 Q.3 5
 2
Q.4 0 Q.5 0 or 24 Q.6 11

Q.7 x < − 7, − 5 < x ≤ − 2 , x ≥ 4 Q.8 3


2 . 5 0 | Quadratic Equations and Inequalities

Q.9 (a) (ii) and (iv); (b) x2 − p(p 4 − 5p2q+ 5q2 )x + p2 q2 (p2 − 4q)(p2 − q) =
0 Q.10 20

5 +1
Q.11 135 Q.12 10 Q.13 (a) x = ; (b) (a) ymin = 6
2
Q.14 6 Q.15 (A) S; (B) Q,R,S,T (C) R, S ; (D) P Q.16 6

Q.17 (a) x = 1 ; (b) x = 2 or 5 ; (c) x = –1 or 1 (d) x ≥ − 1 or x = –3 ; (e) =


x (1 − 2 ) a or ( )
6 −1 a

Q.18 (a) 3y
3
− 9y 2 − 3y + 1 =0 ; ( α − 2 )( α − 2 )( γ − 2 ) =3 ; (b) (i) 2 ; (ii) – 4 Q.19 3

Q.22 a∈ − 1 , 1 
a= 1 − 2 or 5 + 10  
Q.20 Q.21 P(1) = 4
 4 

Q.23 3 Q.24 115 ( ) ( ) ( )


Q.25 − ∞ , − 2 ∪ 0,1 ∪ 2, 4 ∪ 5, ∞
 1 1 1
Q.26 x ∈ ,  ∪ ( 8, 16 ) Q.27 9
≤ x ≤ 10
 16 8  10
1− 5 1+ 5
Q.28 −1 < x < or < x < 2
2 2

( ) (
Q.29 − ∞ , − 2.5 ∪ 0 , ∞ )

Exercise 2
Single Correct Choice Type

Q.1 C Q.2 A Q.3 B Q.4 D Q.5 A

Multiple Correct Choice Type

Q.6 B, D Q.7 C, D Q.8 A,B,C Q.9 A, D

Comprehenstion Type

Q.10 A Q.11 B

Assertion Reasoining Type

Q.12 D Q.13 A

Match the Columns

Q.14 A → p,s; B → p, s; C → p, s; D → p, s Q.15 A → q; B → p; C → s; D → r

Q.16 A → r; B → p, q, r, s; C → q Q.17 A → q; B → r; C → p

Previous Years’ Questions


1
Q.1 7 Q.2 1210 Q.3 a > l { }
Q.6 y ∈ −1 ∪ 1, ∞ ) Q.7 −
4
Q.8 B Q.9 A Q.10 A Q.11 C Q.12 A Q.13 B

Q.14 C Q.15 B Q.16 A Q.17 D Q.18 A Q.19 B

Q.20 D Q.21 B Q.22 B Q.23 B


M a them a ti cs | 2.51

Solutions

Exercise 1 −8
⇒m=
9
Sol 1: Equation px2 + qx + r =0.
−q (iii) Sum of roots is 6
The sum of roots of a quadratic equation is: .
p −b (4 + 2m)
c −q ⇒ =6⇒ =6 ⇒ 22m =−26
Let roots be = 1 ⇒ r1 + r2 = a 5 + 4m
a p
−13
⇒m=
Given that:- r12 + r22 = r1 + r2 ⇒ (r1 + r2 )2 − 2r1r2 = r1 + r2 11

+r Sol 4: Eq. is 5x2 + 13x + k =0


Product of roots is
= = r1 r2
p 1
2 2
One root is reciprocal of other ⇒ r1 =
 −q  2r −q q 2r −q r2
⇒  − = ⇒ − = c
 p  p p p p
2
p ∴ =1 ⇒ k =5
a
⇒ q2 − 2pr =−qp ⇒ q2 + pq =2pr
Sol 5: Difference of roots is 1

Sol 2: Equation 1 ∴ (r1 − r2 )2 =


⇒ r1 − r2 = 1
(a + b)2 x2 − 2(a2 − b2 )x + (a − b)2 =
0
⇒ (r1 + r2 )2 − 4r1 r2 =
1
2
For an eq. ax + bx + c =0, if roots are equal then b2 =
4ac b2 4c
⇒ −= 1 → (1) ... (i)
a2 a 
∴ for above eq.
eq. is x2 − px + q =0
( )
2
D =  −2 a2 − b2  − 4 ( a + b ) ( a − b )
2 2
 
∴ Putting in eq. (i)
= 4(a2 − b2 )2 − 4[(a + b)(a − b)]2
( −p)2
− 4q =1 ⇒ p2 − 4q =1
= 4(a2 − b2 )2 − 4(a2 − b2 )2 = 0 1

Hence the roots are equal. Sol 6: Equations an 3x2 + 4mx + 2 =0


and 2x2 + 3x − 2 =0.
2 2
Sol 3: Eq. is 5x − 4x + 2 + m(4x − 2x − 1) =0 Let the common root be α

⇒ (5 + 4m)x2 − (4 + 2m)x + (2 − m) =
0 ⇒ 3α2 + 4mα + 2 = 0  … (i)
and 2α2 + 3α − 2 = 0  … (ii)
(i) If the eq. has equal roots then b2 − 4ac =
0
Solving equation (ii) we get
⇒ [ −(4 + 2m)]2 − 4(5 + 4m)(2 − m) =0
2α2 + 4α − α − 2= 0 ⇒ (2α − 1)(α + 2)= 0
2
⇒ 4m + 16m + 16 − 4( −4m + 3m + 10) =
0 2 1
∴α = or α = − 2
2
20m2 + 4m − 24 =
0
3 1
5m2 + m − 6 =0 + 2m + 2 =0 (Putting α = )
4 2
11
0 ⇒ m = 1 or m = −6 / 5 .
⇒ (m − 1)(5m + 6) = m= −
8
(ii) Product of roots is 2 and 3 × 4 − 8m + 2 =0 (Putting α = −2 )
c (2 − m) 14 7
⇒ 2
=⇒ =2 ⇒ 9m =−8 ⇒m= =
a 5 + 4m 8 4
2 . 5 2 | Quadratic Equations and Inequalities

Sol 7: α & β are roots of the equation x2 ‒ px + q = 0 ∴ x ∈ ( −∞ ,1) ∪ (2, ∞ )

⇒ α + β = p and αβ = q In the second equation, we have


1 1 x2 − 4x + x − 4 ≤ 0 ; ⇒ (x + 1)(x − 4) ≤ 0
The equations whose roots are α + and β + is
β α
∴ x ∈ [ −1, 4]
  1    1 
 x −  α +  x −  β +  = 0 1
  β    α  m−n =1−
x
 1 1  1  1 ∴ The values of x which satisfies both the equations
⇒ x2 −  α + β + +  x +  α +  β +  = 0
 α β  β  α
= −[( −∞ ,1) ∪ (2, ∞ )] ∩ [ −1, 4] ⇒ x ∈ [ −1,1) ∪ (2, 4]
2  ( α + β)   1 
x −  ( α + β) +  x +  αβ + 2 + =0
 αβ   αβ 
Sol 11: ax2 + bx + c =0 (α and β are roots of this eq.)
 p  1 
⇒ x2 − p +  x + q + + 2 =
0 ⇒ α+β =
−b
& αβ =
c
 q  q  a a

∴ Eq. is qx2 − (pq + p)x + (q2 + 2q + 1) =


0 Given eq. a3 x2 + abcx + c3 =
0

Which is qx2 − p(q + 1)x + (q + 1)2 =


0 ⇒ Let the roots be r & s
−abc −b c
r + s= = ×
4x a 3 a a
Sol 8: ≥1
x2 + 3 = (α + β) × αβ = α2β + αβ2
Since x2 + 3 is positive, we can directly take it to other c3
side. ⇒ rs =
a3 (
= α3β3 )
⇒ 4x ≥ x2 + 3 ∴ We can see here that r = α2β and s = αβ2
2 ∴ The given equation will become
⇒ x − 4x + 3 ≤ 0
⇒ (x − 1)(x − 3) ≤≤=00 (x − α2β)(x − αβ2 ) = 0

The critical points are 1,3


Sol 12: a and b are integer
+ – +
Roots of x2 + ax + b =0 are rational
1 3 Let the roots be α and β putting α in eq.
Hence solution is [1,3] α2 + aα = −b
Sol 9: (x − a)(x − b) − k =0 and c and d are the roots of α(α + a) = −b
the equation a is an integer and b is an integer
The equation with root c and d is (x − c)(x − d) =
0
∴ α has to be an integer
∴(x − c)(x − d) = (x − a)(x − b) − k 2 2
Sol 13: An equation ax + 2hxy + by + 2gx + 2fx + c =0
∴(x − a)(x − b) = (x − c)(x − d) + k can be factorized into two linear factors
⇒ a and b are roots of equation (x − c)(x − d) + k =0 a h g
If h b f = 0 and h2 − ab > 0 .
Sol 10: x2 − 3x + 2 > 0 and x2 − 3x − 4 ≤ 0 g f c

From the first equation, we can write (x − 1)(x − 2) > 0 (i) The expression is 3x2 − xy − 2y 2 + mx + y + 1
+ – + −1 m 1
h= , a = 3, b = –2, g = , f = and c = 1
1 2 2 2 2
M a them a ti cs | 2.53

And h2 − ab > 0 which is 1 + 6 > 0 true. ⇒ 4y 2 + 8y + 4 − 16y 2 + 32y − 16 ≥ 0


4
1 m ⇒ 12y 2 − 40y + 12 ≤ 0 , ⇒ 3y 2 − 10y + 3 ≤ 0
3 −
2 2
⇒ (3y − 1)(y − 3) ≤ 0
1 1
And − −2 = 0
2 2 1 
∴y ∈  ,3
m 1 3 
1
2 2
Sol 15: (1 + a)x2 − 3ax + 4a =
0
 1  1  1 m m 1 
⇒ 3  −2 −  +  − −  +  − + m  =0
 4 2 2 4  2  4  Let f ( x ) =( a + 1 ) x2 − 3ax + 4a and d = 1.

22 (m + 2 ) m ( 4m − 1 ) The roots exceed unity


⇒ − − + 0
=
4 8 8

⇒ −54 − m − 2 + 4m2 − m =0 ⇒ 4m2 − 2m − 56 =


0, ⇒
1
2 or
2
0 , ⇒ 2m − 8m + 7m − 28 =
0 1
⇒ 2m − m − 28 =

−7
⇒ ( 2m + 7 )(m − 4 ) =
0, ⇒ m = ,4 The conditions are
2
(ii) 6x2 − 7xy − 3y 2 + mx + 17y − 20 (i) D ≥ 0

−7 m 17 (i) 9a2 − 16a(1 + a) ≥ 0


⇒ a = 6, h = , b = −3 ⇒ g = , f= , c = −20
2 2 2
⇒ 9a2 − 16a − 16a2 ≥ 0 , ⇒ 7a2 + 16a ≤ 0
2
 −7   −16 
And h2 − ab > 0 which is   + 18 > 0 True. a(7a + 16) ≤ 0 a ∈  ,0 
 2   7 
−7 m
6 (ii) a f (d) > 0
2 2
−7 17 Note that this a is the co-efficient of x2 and not to be
and −3 0
=
2 2 confused with 'a
m 17 ⇒ (1 + a)(1 + a − 3a + 4a) > 0
−20
2 2
⇒ (1 + a)(2a + 1) > 0
 289  7  17m  m  −119 3m   −1 
⇒ 6  60 −  +  70 − +  + =0 ∴ a ∈ ( −∞ , −1) ∪  , ∞ 
 4  2 4  2 4 2   2 
⇒ 12 ( −49 ) + 7 ( 280 − 17m) + m ( −119 + 6m) =
0 −b 3a
(iii) >d⇒ >1
2a 2(1 + a)
6m2 − 238 m + 1272 =
0
3a a−2
⇒ −1 > 0 ⇒ >0
98 2(1 + a) (a + 1)
∴m =
7, are solutions of this equation of this
3
equation + – +
–1 –2
2
x − 2x + 4
Sol 14: =y ∴ a ∈ ( −∞ , −1) ∪ (2, ∞ )
x2 + 2x + 4
So taking intersection to all 3 solutions
x2 (1 − y) − x(2 + 2y) + 4(1 − y) =
0
 −16 
a∈  , −1
Since x is real ∴ b2 − 4ac ≥ 0  7 
⇒ (2 + 2y)2 − 16(1 − y)2 ≥ 0
2 . 5 4 | Quadratic Equations and Inequalities

Sol 16: P(x) = ax2 + bx + c and Q(x) =


−ax2 + bx + c When x > 0

P(x).Q(x) = 0 x2 − 2 x =x2 − 2x

⇒ (ax2 + bx + c)( −ax2 + bx + c) =


0 ⇒ x2 − 2x − 1 =0 or x2 − 2x + 1 =0
⇒ D of P(x) = b2 – 4ac
2+2 2
=⇒x = or x 1
⇒ D of Q(x) = b + 4ac 2
2

Clearly both cannot be less than zero at the same time. When x < 0

Hence the equation has at least 2 real roots x2 − 2 x =x2 + 2x

Sol 17: We have ax2 + 2bx + c =0 ⇒ x2 + 2x − 1 =0 or x2 + 2x + 1 =0

−2b c −2 − 2 2
⇒ α=
+β ,=
αβ ⇒x= or x = −1
a a 2
2
For equation Ax + 2Bx + c =0 ( ) (
The values of x are −1, −1 − 2 ,1, 1 + 2 )
−2B b B Sol 19:
⇒ (α + β) + 2k = , ⇒k = −
A a A LHS = (x − 2)(x − 3) − 8(x − 1)(x − 3) + 9(x − 1)(x − 2)
C C
Also, ( α + k )( β + k ) = , ⇒ k 2 + (α + β)k + αβ = = x2 − 5x + 6 − 8x2 + 32x − 24 + 9x2 − 27x + 18 = 2x2
A A
2
 b B   −2b  b B  c C Which is always equal to RHS no matter what the value
⇒ −  +  −  + =
 a A   a  a A  a A of x

 b B  b B  c C ∴ The equation is an identify


⇒  −  − −  + =
 a A  a A  a A 2
 x2   x2 
B2 b2 c C B2 C b2 c Sol 20: (1 + a)   − 3a   + 4a =
0
 x2 + 1   x2 + 1 
⇒ − + = ⇒ − = −    
A2 a2 a A A2 A a2 a x2
Let y = ⇒ x2 (1 − y) − y = 0
B2 − AC b2 − ac b2 − ac  a 
2 x2 + 1
∴ =⇒ = 
A2 a2 B2 − AC  A  Since x is real, ⇒ 4y(1 − y) ≥ 0

∴ y ∈ [0, 1]
( ) ( )
t t
Sol 18: We have 15 + 4 14 + 15 − 4 14 30
=
∴ The given equation becomes (1 + a)y 2 − 3a(y) + 4a =
0
t x2 − 2 x
and =
where the roots of equation should be between (0 & 1)
( )
t
Let 15 + 4 14 y
=
0 1
1 or
⇒y+ =30 0 1
y
30 ± 896
⇒ y 2 − 30y + 1 =0 ⇒y= These conditions should be satisfied
2
2 (i) D≥0
y 15 ±
⇒= 224 ⇒ =
y 15 ± 4 14
2 ∴ 9a2 − 16a(a + 1) ≥ 0 ⇒ 7a2 + 16a ≤ 0

(15 + 4 14 )
t
15 ± 4 14
=  −16 
∴a∈  ,0 
∴ t =1 or t =−1  7 
M a them a ti cs | 2.55

af(d) > 0 & af(e) > 0 Sol 22: From condition of common root

(ii) (1 + a)f(0) ≥ 0 ⇒ (1 + a)4a ≥ 0 (ca1 − ac1 )2 = (2bc1 − 2b1c)(2ab1 − 2a1b)


2
∴ a ∈ ( −∞ , −1] u [0, ∞ )  c a b c   a b 
2
(a1c1 )  − =  4b1c1  −  a1b1  −  …(i)
and (1 + a)f(1) > 0 ⇒ (1 + a)(2a + 1) > 0  c1 a1   b1 c1   a1 b1 
a b c
1 , , are in AP
∴ a ∈ ( −∞ , −1] ∪ [ − , ∞ ) a1 b1 c1
2
−b Let the difference be d.
(iii) d ≤ < e as the range is from [0,1]
2a
 c a
3a  − 

0≤ ≤1 b a c b c a1   … (ii)
2(1 + a) ∴ − = − =d = 1
b1 a1 c1 b1 2
3a 3a
⇒ ≥0& −1 < 0
2(1 + a) 2(1 + a) Using (i) and (ii)

a−2 ∴ (a1c1 )2 × 4d=


2
4a1c1b12 × d2
⇒ <0
2(a + 1)
∴ b12 =
a1c1
+ – +
a1, b1, c1 are in G.P.
-1 0
a ∈ ( −∞ , −1)u(0, ∞ )
α α1 β β1
Sol 23: = ⇒ =
+ – + β β1 α α1
a∈
-1 2
2 2
α β α1 β1 α2 + β2 α1 + β1
a ∈ ( −1,2) + = + ⇒ =
β α β1 α1 αβ α1β1
∴ a ∈ (0,2)
2
α2 + β2 α12 + β12 (α + β)2 (α1 + β1 )
Taking intersection at all 3 possibilities a=0 is the only ⇒ = +2 +2 ⇒ =
αβ α1β1 αβ α1β1
possible solution.

( −b / a) ( −b1 / a1 )
2 2
2
Sol 21: (l − m)x + lx + 1 =0 ⇒ =
c/a c1 / a1
Let one root be α: other root = 2α
2
l l b2 b12 b  ca

⇒ α += ⇒=
α ⇒ = ⇒   =
m−l 3(m − l) ac a1c1  b1  c1a1

l
2α2 = (Product of roots) Sol 24: α is root of equation ax2 + bc + c =0
l−m
2
⇒ aα 2 + bα + c = 0
2l 1
⇒ =
9(m − l) 2 (l − m) Similarly −aβ2 + bβ + c = 0
2 ax2
⇒ 2l= 9(l − m) Let f(x) = + bx + c
2
2
⇒ 2l − (9)l + 9m =
0
aα 2 −aα2
f(=
α) + bα =
+c
2 2
The roots are real ⇒ b2 − 4ac ≥ 0
aβ2 3aβ2
9 f(=
β) + bβ +=
c
81 − 8 × 9m ≥ 0 ⇒ m ≤ 2 2
8
−3a2 2 2
∴ f(α ). f(
= β) α β <0
4
2 . 5 6 | Quadratic Equations and Inequalities

∴ By mean value theorem, there exists a root of f(x) Sol 27: x2 + ax + a2 + 6a < 0 is satisfied for all x ∈ (1, 2)
between α and β
∴ (1 and 2 exists between the roots)

2
Sol 25: Given, ax + bx + c − p =0 for two distinct α & β ∴ Bring condition for the given case.

f(1) × 1 < 0 & f(2) × 1 < 0


∴ α and β are root of eqn
ax2 + bx + (c − p) =
0 ∴ a2 + 7a + 1 < 0 & a2 + 8a + 4 < 0

−b c −p −7 − 3 5
∴α + β = & αβ = Solving we get ≤ a ≤ −4 + 2 3
a a 2
To prove ax2 + bx + c − 2p ≠ 0 for any integral value of x,
let us assume these exist integer R satisfying Sol 28: Given that, 2x3 + x2 − 7 =0
−1 −7
ax2 + bx + c − 2p =0 For roots, α + β=
+γ γα 0 & αβγ =
αβ + βα +=
2 2
⇒ ak 2 + bk + c − 2p =0 α β
For ∑  + 
k 2 + bk c − p p β α
or + =
a a a α α  β β  γ γ 
=  + + 1 +  + + 1 +  + + 1 − 3
p β γ  α γ  α β 
or (k − α )(k − β) ==an integer
a
p
Since p is a prime number ⇒ is an integer if a=p or
∑ αβ ∑ αβ ∑ αβ
a = + + − 3 =0 − 3 =−3
a=1 but a > 1 ∴ a = p βγ αγ γβ
⇒ (k – α) (k – β) = 1
Sol 29: (x − 3k)(x − (k + 3)) < 0
∴ either k – α = –1 and k – β = –1

⇒ α = β (not possible)
1 3
∴ There is contradiction
k+3 3k

Sol 26: We are given that a ≤ 0 and


⇒ f(1) < 0 andf(3) < 0
x2 − 2a x − a − 3a2 =0
(using condition to given are)
for x > a
(1 − 3k)(1 − (k + 3)) < 0 and ( 3 − 3k )( −k ) < 0
Equation becomes x2 − 2ax − a2 =
0
 1
2a ± 8a2 ∴ k ∈  −2,  & k ∈(0,1)
∴x =  3
2
 1
= a ± a 2 but since x > a & a < 0 ∴ k ∈ 0, 
 3
( )
∴ a 1 − 2 is the only solution

For x < a
Exercise 2
2 2
Eqn becomes x + 2ax − 5a =
0
Single Correct Choice Type
−2a ± 24a2
⇒ x= =−a±a 6
2 Sol 1: (C) Given that a2 + b2 + c2 = 1
But since x < a We know that

∴a ( )
6 − 1 is the only possible solution. (a + b +c)2 = a2 + b2 + c2 + 2 (ab + bc + ca)
⇒ (a +b +c)2 = 1 + 2 (ab + bc + ca)
M a them a ti cs | 2.57

(a + b + c )2 − 1 1
⇒ ab + bc + ca = …(i) Sol 5: (A) α − 2 =
2 α
Also, 2(a2 + b2 + c2) – 2ab – 2bc –2ac ⇒ α2 − 2α − 1 = 0 ⇒ b = −2 , c = −1

= (a – b)2 + (b – c)2 + (c – a)2 ∴ b2 + c2 + bc =( −2)2 + ( −1)2 + ( −2 )( −1 ) =7


Now, (a – b)2 + (b – c)2 + (c – a)2 > 0
∴ ab +bc + ca < a2 + b2 + c2 < 1 Sol 6: (C) a + b + c = 0
−1 D of eq. = 25b2 − 84ac= 25(a + c)2 − 84ac
Here, min (ab + bc + ac) =
2
= 25c2 − 34ac + 25a2
Max (ab + bc + ac) = 1
  c 2 c 
= a  25   − 34   + 25 
2
Sol 2: (B) P(x) = ax2 + bx + c  a a 
 
D(P) = b2 ‒ 4ac D2 of this eq. < 0
2
If D(P) < 0 ⇒ 4ac > b
∴ The eq. is always positive when a ≠ 0
If D(Q) < 0 ⇒ 4ac < –d ⇒ D(P) > 0 2

∴ At least one of P and Q is real. Sol 7: (D) One root is α ∴ The other root = −α
Let third root = β
∴ P(x) & Q(x) = 0 has atleast 2 real roots
⇒ α − α + β= 2 ⇒ β= 2
2
Sol 3: (B) Given that x + 3x + 1 =0
Putting this value in the given equation
For roots, α + β = −3 αβ = 1
23 − 23 + 2a + 10 =0 ⇒ a =−5
2 2
 α   β  [α(α + 1)]2 + [β(β + 1)]2 ∴ a ∈ ( −10,0)
  +  = 2
1 + β  α +1 (α + 1) (β + 1)2

( (α ) Sol 8: (B) x=
+ 1 log2 (2x + 3)2 − 2log(1980 − 2− x )
2
2
+ α ) + (β2 + β) − 2αβ(α + 1)(β + 1)
=
(αβ + α + β + 1)2 (2x + 3)2
⇒ 2x +1 =
1980 − 2− x
2
(α + β)2 − 2αβ + (α + β) − 2αβ(αβ + α + β + 1)
=  ⇒ 1980 × 2 × 2x − 2= (2x + 3)2
(αβ + α + β + 1)2
Let 2x = t
[9 − 2 − 3]2 − 2 × 1( −3 + 2) ⇒ t2 + 6t + 11 + 1980 × 2t =
0
= = 16 + 2 = 18
2
(1 − 3 + 1)
Now 2α × 2β =
11

⇒ 2α+β= 11 ⇒ α + β= log2 11
Sol 4: (B) ax2 + bx + c =0
a > 0, b > 0 & c > 0 Sol 9: (B) Product of H.C.F. & L.C.M. of two numbers =
product of the nos
−b ± b2 − 4ac
⇒α=
2a ∴ 16 × LCM =
295680

If α is real ⇒ b2 − 4ac < b 295680


∴ LCM
= = 18480
16
∴ α is negative

If b2 − 4ac < 0 then real part of α is always negative

∴ The roots have negative real parts


2 . 5 8 | Quadratic Equations and Inequalities

Sol 10: (A) Given that 4y 2 + 4xy + x + 6 =0 Sol 13: (C) (x3 + 4x)2 = 82

y = real ⇒ b2 − 4ac ≥ 0 ⇒ x6 + 16x2 + 8x4 = 64;


Mulitiply both sides by x
⇒ 16x2 − 16(x + 6) ≥ 0
⇒ x7 + 16x3 + 8x4 = 64;
x2 − x − 6 ≥ 0 ⇒ (x − 3)(x + 2) ≥ 0
Add 16x3 in both sides
∴x ≤ −2 or x ≥ 3 ⇒ x7 + 8x5 + 32x3 = 16x3 + 64;

Sol 11: (B) If exactly one root lies in (0, 3) (as interval ⇒ x7 + 8x2(x3 + 4x) = 16(x3 + 4x);
is open) ⇒ x7 + 8x2 × 8 = 16 × 8;
⇒ f(0)f(3) < 0 ⇒ x7 + 64x2 = 128

∴ 2a(6 − a) < 0
Sol 14: (D) Given equations have real roots so,
⇒ a ∈ ( −∞ ,0) ∪ (6, ∞ )
a2 − 8b ≥ 0 ⇒ a2 ≥ 8b and 4b2 − 4a ≥ 0 ∴ ⇒ b2 ≥ a
Now we check at boundaries
⇒ b 4 ≥ a2 ≥ 8b
At a = 0 ⇒ x – x = 0
2

⇒ b≥2 & a≥4


∴ Other root = 1 which lies in (0, 3)
Hence, (a + b)min = 2 + 4 = 6
∴ Now at a = 6, ⇒ f(x) = x2 − 7x + 12 = 0

⇒ x = 3, 4 Sol 15: (A) (x 2 + ax + 1)(3x2 + ax − 3) =


0
No root lies in (0, 3) D=
1 a2 − 4

∴ a ∈ ( −∞ ,0] ∪ (6, ∞ ) D= a2 + 36
2

D2 is always > 0
Sol 12: (D) x2 − 2mx + m2 − 1 =0
∴ The equation has atleast two real roots.
Since both roots lies between (–2, 4)
−b
⇒ D ≥ 0 af(d) > 0 & af(e) > 0 and d < <e Sol 16: (B) f(x) = x2 + ax + b
2a
2 2
(i) ⇒ 4m − 4(m − 1) ≥ 0 For X ∈ [0, 2]

4 ≥ 0 ....true f(x)max = 3 and f(x)min = 2


f(0) = b = 2 ... (i)
(ii) 1. f( −2) > 0
⇒ (4 + 4m + m2 − 1) > 0 f(2) = 4 + 2a + b = 3 ... (ii)
By solving (i) and (ii)
⇒ m ∈ ( −∞ , −3) ∪ ( −1, ∞ )
3
a= − ; b=2
(iii) 1.f(4) > 0 2

⇒ (16 − 8m + m2 − 1) > 0
Previous Years’ Questions
⇒ m ∈ ( −∞ ,3) ∪ (5, ∞ )

⇒− 2 <
−b
< 4 ⇒ −2 < m < 4
(
Sol 1: (i) Given x2 − 8kx + 16 k 2 − k + 1 =)
0
2a
D
Now, = 64 {k − (k − k + 1=
2 2
)} 64 (k − 1 ) > 0
Combining all the above three conditions, we get
∴ m ∈ ( −1,3) ∴k > 1

b 8k
∴ Integral values of m are 0, 1, 2 (ii) − >4⇒ > 4⇒k >1
2a 2
M a them a ti cs | 2.59

(iii) f(4) ≥ 0
( )
⇒ x x − α2β − αβ2 x + α2β = 0 ( )
( 2
)
16 – 32k + 16 k − k + 1 ≥ 0 ⇒ k − 3k + 2 ≥ 0 2
a3 x2 + abcx + c3 =
0
⇒ (k – 2)(k – 1) ≥ 0
⇒ x = α2β, α β2 which is the required answer.
⇒ k ≤ 1 or k ≥ 2 Hence, k = 2.
b c
2x 1 Sol 4: Since α + β = − , αβ = and
Sol 2: Given > a a
2x2 + 5x − 2 x +1
B C
α+δ+β+δ = − , ( α + δ )( β + δ ) =
2x 1 A A
⇒ − >0;
( 2x + 1 ) ( x + 2) ( x + 1 ) Now, α − β = ( α + δ ) − (β + δ )
( ) (
a1 x2 + b1 x + c1 y + a2 x2 + b2 x + c2 =
0 ) 2
⇒ ( α − β ) = ( α + δ ) − ( β + δ ) 
2

x ( a1 y + a2 ) + x (b1 y + b2 ) + ( c1 y + c2 ) =
2
0
2
⇒ ( α + β ) − 4αβ= ( α + δ ) − ( β + δ )  − 4 ( α + δ ) . ( β + δ )
2
− ( 3x + 2 )
⇒ >0
( 2x + 1)( x + 1)( x + 2)  b 4c  B 
2
4C
2
⇒ −  − = −  −
 a a  A A
Using number line rule
b2 4c B2 4C b2 − 4ac B2 − 4AC
 2 1 ⇒ − = − ⇒ =
∴ x ∈ ( −2, − 1 ) ∪  − , −  a2 a A2 A a2 A2
 3 2
Sol 5: From figure it is clear that if a > 0, then f(– 1) < 0
Sol 3: Since ax2 + bx + c =0 in terms of α , β . and f(1) < 0, if a < 0, f(–1) > 0 and f(1) > 0. In both cases,
⇒ α + β = −b / a and α β =c / a af(–1)< 0 and af(–1) < 0
⇒ a (a – b + c) < 0 and a (a + b + c) < 0
Now, a3 x2 + abc x + c3 =
0 ………(i)
On dividing by a2, we get
On dividing the equation by c2 , we get
b c b c
a3 2 abcx c3 1− + < 0 and 1 + + < 0
x + + 0
= a a a a
c2 c2 c2
On combining both, we get
2
 ax   ax 
⇒ a  + b   + c =0 y
 c   c 
a<0
2
y=ax +bx+c

ax
⇒ = α , β are the roots -1 1
c  0 
x
y
c c a>0
⇒ x = α , β are the roots
a a 2
y=ax +bx+c

⇒ x = α β α , α ββ are the roots  -1 0 1 

⇒ x = α2 β, α β2 are the roots


b c b c
Alternate solution 3
1± + < 0 ⇒ 1+ + < 0
b c c a a a a
2
Divide the Eq. (i) by a3 , we get x + . x +   =
0
a a a
Sol 6: (B) Given x2 + 2px + q =0
⇒ x − ( α + β ) . ( αβ ) x + ( αβ ) = 0
2 3

∴ α + β = − 2p  … (i)
⇒ x − α βx − αβ x + ( αβ ) = 0
2 2 2 3
αβ = q … (ii)

2 . 6 0 | Quadratic Equations and Inequalities

and ax2 + 2bx + c = 0 ⇒ (x – 1)(x – 4) = 0


1 −2b
∴ α+ =−  ... (iii) ⇒ x = 1, 4(4 is rejected)
β a
α c ⇒ x = 1 … (ii)
and = … (iv)
β a Hence, the sum of the roots is 3 + 1 = 4

(
Now, p2 − q b2 − ac )( ) Alternate solution
 1
2  2
  α + β 2 
 α+ β  Given x − 2 + x − 2 − 2 =0
 α
 −  a2
=   − αβ  
 −2 

  −2 
 
β

( ) (
⇒ x − 2 + 2 + x − 2 −1 =
0 )

   ∴ x − 2 =−2, 1 (neglecting – 2)

α2 2 1
2 ⇒ x−2 =
1
=
16
( α − β)  α −  ≥ 0
β
 ⇒ x = 3, 1
∴ Statement-I is true. ⇒ Sum of roots = 4
a
Again now pa = − (α + β) Sol 8: (B) If a1 x2 + b1 x + c1 =
0 and a2 x2 + b2 x + c2 =
0
2
Have a common real root, then
a 1
and b = −  α +  Since, pa ≠ b
⇒ ( a1c2 − a2c1 ) = (b1c2 − b2c1 )( a1b2 − a2b1 )
2
2 β

1 x2 + bx − 1 =0 
⇒ α + ≠ α + β ⇒ β2 ≠ 1, β ≠ {−1, 0, 1} ∴
2
 have a common root.
β x + x +b = 0 

which is correct. Similarly, if c ≠ qa ⇒ (1 + b ) =


2
(b 2
)
+ 1 (1 − b )
α  1
⇒a ≠ a αβ; ⇒ α  β −  ≠ 0 ⇒ α ≠ 0 ⇒ b2 + 2b + 1 = b2 − b3 + 1 − b
β  β
⇒ b3 + 3b =
0
1
and β − ≠ 0 ⇒ β ≠ {−1,0,1}
β (
∴ b b2 + 3 =
0 )
Statement-II is true.
⇒ b = 0, ± 3 i
2
Sol 7: Given, x − 2 + x − 2 − 2 =0
Sol 9: (D) The equation x2 − px + r = 0 has roots α , β
Case I when x ≥ 2 and the equation
α
⇒ (x − 2)2 + (x − 2) − 2 =0 x2 − qx + r =0 has roots , 2 β .
2
α
⇒ x2 + 4 – 4x + x – 2 – 2 = 0 ⇒ r = αβ and α + β = p , and + 2 β =q
2
⇒ x2 – 3x = 0 ⇒ x(x – 3)= 0
2q − p 2 ( 2p − q)
⇒β= and α =
⇒ x = 0, 3 (0 is rejected) 3 3
2
⇒ x = 3 … (i) ⇒ α β= r =
9
( 2q − p ) ( 2p − q)
Case II when x < 2.
Sol 10: (A) Let the roots of x2 + px + q =0 be α and α2.
{ } − ( x − 2) − 2 =0
2
⇒ − ( x − 2)
⇒ α + α2 = −p ; and α3 =q
⇒ ( x − 2) − x + 2 − 2 =
2
0 ⇒ x2 + 4 – 4x – x = 0
⇒ α ( α + 1 ) = −p
⇒ x2 – 4x – 1 (x – 4) = 0 ⇒ x(x – 4) – 1(x – 4) = 0
M a them a ti cs | 2.61

{ }
⇒ α3 α3 + 1 + 3 α ( α + 1 ) = −p3 (cubing both sides) ⇒ x = 5 [∴ x = 2 make log (x – 3) undefined].
Hence, one solution exits.
⇒ q (q + 1 – 3p) = −p3

⇒ p3 − ( 3p − 1 ) q + q2 =
0 Sol 14: (B) Given c < 0 < b
Since α + β = −b  ... (i)
Sol 11: (B) As we know ax2 + bx + c > 0 for all x ∈R , if
a > 0 and D < 0 and α β =c  ... (ii)

Given equation is From Eq. (ii), c < 0 ⇒ αβ < 0

x2 + 2ax + (10 − 3a) > 0, ∀ x ∈ R Now, ⇒ Either α is – ve, β is + ve or α is + ve,

⇒ 4a2 − 4 (10 − 3a) < 0 Or β is – ve

(
⇒ 4 a2 + 3a − 10 < 0 ) From Eq. (i), b > 0 ⇒ − b < 0 ⇒ α + β < 0

⇒ (a + 5) (a – 2) < 0 ⇒ The sum is negative.

⇒ a ∈ (–5,2) ⇒ Modulus of negative quantity is > modulus of


positive quantity but α < β is given.
Sol 12: (B) Given x2 − x + 2 + x > 0 ... (i) Therefore, it is clear that α is negative and β is positive

and modulus of α is greater than
Case I when x + 2 ≥ 0
Modulus of
∴ x2 − x − 2 + x > 0
β⇒α<0<β< α
⇒ x2 − 2 > 0
Note: This question is not on the theory of interval in
⇒ x < − 2 or x > 2
which root lie, which appears looking at
⇒ x ∈ x  −2, − 2 ∪
 ) ( 2, ∞ ) ... (ii) First sight. It is new type and first time asked in the
paper. It is important for future. The actual
Case II when x + 2 < 0
Type is interval in which parameter lie.
∴ x2 + x + 2 + x > 0

⇒ x2 + 2x + 2 > 0 Sol 15: (A) Since x + 1 − x − 1= 4x − 1


⇒ ( x + 1) + 1 > 0
2
⇒ ( x + 1 ) + ( x − 1 ) − 2 x2 − 1 = 4x − 1
Which is true for all x. 2 2
2x 2 x2 − 1 ⇒ 1 + 4x − 4x= 4x − 4
⇒ 1 −=
∴ x ≤ − 2 or x ∈ ( − ∞ , − 2 )  ... (iii) 5
⇒ 4x = 5 ⇒x=
From Eqs. (ii) and (iii), we get 4
But it does not satisfy the given equation.
(
x ∈ − ∞, − 2 ∪ ) ( 2, ∞ ) Hence, no solution exists.

Sol 13: (B) Given Sol 16: (D) Let α and 4β be roots of x2 – 6x + a = 0 and
α, 3β be the roots of x2 – cx + 6 = 0, then
log4 ( x −=
1 ) log2 ( x −=
3) log ( x − 3)
41/2
α + 4β = 6 and 4αβ = a
⇒ log4 ( x=
− 1 ) 2log4 ( x − 3)
α + 3β = c and 3αβ = 6.
⇒ log4 ( x −=
1 ) log4 ( x − 3)
2
We get αβ = 2 ⇒ a = 8
⇒ ( x − 3) =x − 1 ⇒ x2 − 7x + 10 =
2
0 So the first equation is x2 – 6x + 8 = 0 ⇒ x = 2, 4
⇒ ( x − 2) ( x − 5) =
0 ⇒ x = 2 or x = 5 If α = 2 and 4β = 4 then 3β = 3
2 . 6 2 | Quadratic Equations and Inequalities

If α = 4 and 4β = 2, then 3β = 3/2 (non-integer) 2


 −q  4r q2 − 4pr
∴ Common root is x = 2. =   −=
 p  p |p|

Sol 17: (C) bx2 + cx + a =0


16r 2 + 36r 2 2 13
Roots are imaginary ⇒ c -4ab < 0 ⇒ c < 4ab=
2 2 =
| −9r | 9
⇒ c2 > -4ab
3b2 x2 +6bcx + 2c2 Sol 20: (A) x2 + 2x + 3 = 0  ... (i)

since 3b2 >0 ax + bx + c = 0 


2
... (ii)
Since equation (i) has imaginary roots
Given expression has minimum value
So equation (ii) will also have both roots same as (i).
4(3b2 )(2c2 ) − 36b2c2
Minimum value = a b c
4(3b2 ) Thus, = = ⇒ a =λ ,b =λ 2 ,c =λ3
1 2 3
12b2c2
=− = −c2 > −4ab Hence 1 : 2 : 3
2
12b

Sol 18: (C) x2 – 6x – 2 = 0


an = αn − βn JEE Advanced/Boards
a10 − 2a8 α10 − β10 − 2(α8 − β8 ) Exercise 1
=
2a9 2(α9 − β9 )
Sol 1: f(x) = x2 + ax + b
8 2 8 2 9 9
α (α − 2) − β (β − 2) 6α − 6β α+β 6
= = = = 3
9 9
2(α − β ) 9 9
2(α − β ) 2 2 One root is
2+ 3
4+3 3
= (
4+3 3 2− 3 )( ) =−1 + 2 3
∴ The other root is −1 − 2 3

Sol 19: (B)  p, q, r are in AP Sum of roots = –a = – 2

2q = p + r  ... (i) ⇒a=2


b
1 1 Product = = ( −1 + 2 3)( −1 − 2 3) =
1 − 12 =
−11
Also, + =4 1
α β
∴ g(x) = x 4 + 2x3 − 10x2 + 4x − 10
α+β
⇒ 4
= = x 4 + 2x3 − 11x2 + x2 + 2x − 11 + 1 + 2x
αβ
= x2 f(x) + f(x) + 2x + 1
q

p 4+3 3
= =
r
4⇒q=−4r
g
 2+ 3 
 
(
 = x2 × 0 + 0 + 2 −1 + 2 3 = )
+1 4 3 −1
p ….(ii)
From (i) ∴ c = 4 & d = −1

2(–4r) = p + r ax2 + 2(a + 1)x + 9a + 4


Sol 2: f(x) =
p = – 9r x2 − 8x + 32

q = – 4r x2 − 8x + 32 is always positive as a > 0 & b2 − 4ac < 0


r=r ∴ For f(x) to be always negative

Now | α − β=| (α + β)2 − 4αβ ax2 + 2(a + 1)x + (9a + 4) < 0 for all x

⇒ a < 0 & b2 − 4ac < 0


M a them a ti cs | 2.63

2
∴ 2(a + 1) − 4a(9a + 4) < 0 Q(x) = ax2 + cx + b ( β & 3 are roots)

4(a2 + 2a + 1) − 36a2 − 16a < 0 −b c


−2
⇒ α= &−
= 2α
a a
⇒ 32a2 + 8a − 4 > 0 ⇒ 8a2 + 2a − 1 > 0
−c b
and=
β+3 & 3β
=
⇒ 8a2 + 4a − 2a − 1 > 0 ⇒ (4a − 1)(2a + 1) > 0 a a
 −1   1  ∴ 3β= 2 − α and 3 + β = 2α
∴ a ∈  −∞ ,  ∪  , ∞  but a is a < 0
 2  4 
1 3 11 α
⇒ β= and α = 2 − = ⇒ = 11
 −1  7 7 7 β
∴ a ∈  −∞ , 
 2 
Sol 7: (log|x + 6| 2)log2 (x2 − x − 2) ≥ 1
2
Sol 3: x + 2(a + b)x + (a − b + 8) =0
⇒ log|x + 6| (x2 − x − 2) ≥ 1
Since the equation has unequal roots
| x + 6 |≠ 1 ⇒ x ≠ −5, −7
b2 − 4ac > 0
When | x + 6 | > 1 ⇒ x ∈ ( −∞ , −7) ∪ ( −5, ∞ )
⇒ 4(a + b)2 − 4(a − b + 8) > 0
x2 − x − 2 > | x + 6 |
⇒ a2 + 2ab + b2 − a + b − 8 > 0 when x ∈ ( −5, ∞ )
2 2
a + (2b − 1)a + (b + b − 8) > 0 ⇒ x2 − x − 2 > x + 6 ⇒ x2 − 2x − 8 > 0
Now the quadratic in a always > 0 ⇒ x ∈ ( −∞ , −2) ∪ (4, ∞ )
Discriminant should be less than 0 ∴ x ∈ ( −5, −2) ∪ (4, ∞ )
2 2
∴ (2b − 1) − 4(b + b − 8) < 0 when x ∈ ( −∞ , −7)
−4b + 1 − 4b + 32 < 0 ⇒ x2 − x − 2 > −x − 6 ⇒ x2 + 4 > 0
33
⇒ b> ⇒ x ∈ ( −∞ , −7)
8
∴ The smallest natural number for b is 5 when x ∈ ( −7, −5)

x2 − x − 2 ≤ | x + 6 |
Sol 4: When y 2 + my + 2 is divided by (y − 1) the
x2 − 2x − 8 ≤ 0 when x ∈ ( −5, −6)
remainder = f(1) = 1 + m + 2 = 3 + m
Similarly R 2 =g( −1) =3 − m ⇒ x ∈ ( −2, +4) ⇒ no possible value of x

if R1 = R 2 ⇒ m = 0 When x ∈ ( −7, −5)

x2 − x − 2 ≤ −x − 6 ⇒ x2 + 4 ≤ 0 ⇒ not possible
Sol 5: x2 − 11x + m =
0 and x2 − 14x + 2m =
0
∴ x ∈ ( −7, −∞ ) ∪ ( −5, −2) ∪ (4, ∞ )
Let α be the common root

2 2 Sol 8: V1= sin θˆi + cos θˆj
Let α − 11α + m = 0 and α − 14α + 2m = 0

∴ 3α − m = 0 ⇒ α =
m
. Substituting V2 = ˆi + ˆj + 2 kˆ
3  
angel between V1 & V2 =α = π / 3
2 m2 8m
⇒ m − 11m + m =
0 ⇒ − 0
=
9 3 9 3 sin θ + cos θ
cos α =
for m = 0, 24 the equations have common roots.
1×2

⇒ sin θ + cos θ = 1
2
Sol 6: p(x) = ax + bx + c ( α & − 2 are roots) 1×2 2
2 . 6 4 | Quadratic Equations and Inequalities

⇒ sin θ + cos θ =1 ⇒ x ∈ ( −∞ , −15) ∪ ( −3, ∞ ) and also x2 + 18x + 30 > 0


π ∴ The product of the real roots = 20
The value of θ ∈ [0,2π] are 0, ,2π
2
∴ No. of values of θ are 3
x2 + ax + 4
Sol 11: f(x) =
x2 + bx + 16
Sol 9: (a) A function is symmetric it when we replace α
by β & β by α the function remains same for f(x) > 0 both x2 + ax + 4 > 0 & x2 + bx + 16 > 0
(i) f(β, α ) = β2 − α ≠ f(α , β) (not symmetric) ⇒ D ≤ 0 for first and D < 0 for second eqn denominator
2 2 2 2
(ii) f(β, α ) = β α + βα =α β + β α = f(α , β) can’t be 0.

β n α a ∈ [ −4, 4] & b ∈ ( −8,8)


(iii) f(β, α ) = n =− ≠ f(α , β)
α β
∴ The possibly integral solution of (a, b) are 9× 15 =
135
(not symmetric)

(iv) f(β, α ) = cos(β − α ) = cos(α − β) = f(α , β) Sol 12: f ( 0 ) .f (1 ) < 0


f(x) = 9x – 12ax + 4 – a2
∴ Symmetric
f(0) = 4 – a2
2
(b) α & β are roots of x − px + q
f(1) = 13 – 12a – a2
β p& αβ
⇒ α += = q
f(0)f(1) = (a – 2)(a + 2)(a + 13)(a – 1) < 0
⇒ R1 = (α2 − β2 )(α3 − β3 )
a ∈ (-13, -2) ∪ (1, 2)

⇒ [(α + β)2 − 4αβ](α + β)(α + β2 ) − αβ) Number of integers = 10

⇒ (p2 − 4q) p (p2 − q) 1/2 1/2


 1  1
Sol 13: (a)  x −  + 1 −  x
= ….. (i)
⇒ R 2 = α2β2 (α + β) = q2p  x  x

x −1
∴ R1 + R 2 = q2p + p(p2 − 4q)(p2 − q) and = x (factorizing)
1/2 1/2
 1  1
= p(p 4 − 5p2q + 5q2 ) x −  − 1 − 
 x  x

R 2 q2p2 (p2 − 4q)(p2 − q)


R1 += 1 1
Let x −= m & 1 −= n
x x
The equation is x2 − (R1 + R 2 )x + R1R 2 =
0
m+1
m1/2 =
2
2 2
Sol 10: x + 18x + 30 = 2 x + 18x + 45 = (m + 1)2
∴ 4m

Let x2 + 18x + 30 =
t ∴ (m− 1)2 =0 ⇒ m =1

⇒= t2 4(t + 15)
t 2 t + 15 ⇒ = 1 1± 5
∴x− = 1 ⇒ x2 − x − 1 = 0 ⇒ x =
x 2
⇒ t2 − 4t − 60 =
0 ⇒ (t − 10)(t + 6) =
0 now in equation (i) LHS >0 ∴ x > 0

∴ t = 10 or t = −6 1+ 5
∴ x= only possible solution
2
⇒ x2 + 18x + 20= 0 ; d > 0
3
 1 1
or x2 + 18x + 36= 0 ; d > 0 (b) Let  x + = m & x3 += n
 x x3
But also x2 + 18x + 45 > 0
M a them a ti cs | 2.65

6 2
 1  3 1  5p 5p + 1
⇒ α(1 + α ) =
x +  − x + 3  4 & 2a +1 = 4
x  x  m2 − n2
⇒  =

3
1  3 1  m+n 5p − 3
or α =
x +  + x + 3  8
 x  x 
5p − 3 5(p + 1) 5
3 ⇒ × = p
 1  1   1 8 8 4
= m − n =  x +  −  x3 +  = 3  x + 
 x  
3
x   x
5p2 − 14p − 3 =0
1
The minimum value of x + 2
= 5p2 − 15p + p − 3 =0
x
−1
∴ f(x)min = 3 × 2 = 6 ⇒ p 3=
= or p
5
p = 3 is the integral value
Sol 14: Given that
X2 + 2mx + 7m – 12 = 0 …. (i) x2 − 3x + 4
(b) y =
4x2 – 4mx + 5m – 6 = 0 x2 + 3x + 4
2
For equation (i), D > 0 ⇒ x (y − 1) + 3x(y + 1) + 4(y − 1) =
0

(2m)2 – 4(7m - 12) > 0 x ∈R


⇒ 4m – 28 m + 48 > 0
2
∴ 9(y + 1)2 − 16(y − 1)2 > 0
28 ± (28)2 − 4 × 4 × 48 ⇒ −7y 2 + 50y − 7 > 0
⇒m=
8
28 ± 784 − 768 ⇒ 7y 2 − 50y + 7 < 0
=
8 1 
⇒ y ∈  ,7 
=
28 ± 4
= 4, 3 7 
8
Integers lying in range are 1,2,3,4, or option Q R S T are
For equation (i), D > 0 correct.
16m2 – 4 × 4 × (5m - 6) > 0 x +1 x +5
(c) ≥
⇒ 16m2 – 16(5m - 6) > 0 x −1 x +1

⇒ 16m2 – 80m – 96 > 0 x +1  x +5


⇒ − ≥0
x −1  x +1
80 ± (80)2 − 4 × 16 × 96
⇒m=
32 x2 + 2x + 1 − x2 − 4x + 5
21 19 ⇒ ≥0
⇒ m =, (x − 1)(x + 1)
8 8
19 2(3 − x)
Minimum value of m = ≥0
8 (x − 1)(x + 1)

Maximum value of m = 4 + - + -

19 -1 1 3
Then, a + b= +4
8
x ≠ 1 as x − 1 is in denominator the positive integral
19 + 32 51 values of x are 2 & 3 Ans (R ) (S)
= =
8 8
2π 4π 4π 8π 8π 2π
(d) sin sin + sin sin + sin sin f ( say )
=
2
Sol 15: (a) 4x − (5p + 1)x + 5p = 4 7 7 7 7 7
0
2π 4π 8π
β= 1 + α Let =A = B and =C
7 7 7
2 . 6 6 | Quadratic Equations and Inequalities

  2π 4 π   6π   −4 π  
 cos  −  − cos   + cos   −x2 + 4x − 3 + 2x + 5 =0 ⇒ x2 − 6x − 2 =0
1  7 7   7   7 
f= 6 ± 40
2  12π   6π   10π   ⇒ x= = 3 ± 10
 − cos   + cos   − cos    2

  7   7   7  
which doesn’t belong to (1,3)

1  2π   4π   12π   10π   ∴ x=
1 is the only solution
 cos   + cos   − cos   − cos  
2  7   7   7   7  (b) 3 |x2+4x+2|=5x−4

 2π   2π   12π  Case I: x2+4x+2 > 0


cos =  cos  2π − =  cos  
 7  7   7  3(x2+4x+2)=5x − 4

 4π   4π   10π  ⇒ 3x2+12x+6=5x − 4
cos  =  cos  2π − =  cos  
 7   7   7  ⇒ 3x2+7x+10=0
∴ f =0 −7 ± 49 − 120
⇒ x⇒=
6
Sol 16: x 4 + 2x3 − 8x2 − 6x + 15 =
p(x) Which indicates x is imaginary here. So, this is not
acceptable.
Q(x) = x3 + 4x2 − x − 10 Case II: x2+4x+2 < 0
By trial one root of Q(x) = −2 −3(x2+4x+2) = 5x − 4

∴ Q(x) = (x + 2)(x2 + 2x − 5) ⇒ 3x2+12x+6= −5x + 4

∴ The root of x2 + 2x − 5 should satisfy p(x) x2 + 2x − 5 ⇒ 3x2+17x+2 = 0


has irrational roots and since −17 ± 289 − 24 −17 ± 265
= ⇒x =  ...(i)
Irrational root exist in pairs 6 6
Also, x2+4x+2 < 0
x2 + 2x − 5 should be a factor of p(x)

∴ p(x) = x 4 + 2x3 − 5x2 − 3x2 − 6x + 15  (  )  (


x − 2 − 2  x − 2 − 2  < 0
 ) ... (ii)

= x2 (x2 + 2x − 5) − 3(x2 + 2x − 5) = (x2 − 3)(x2 + 2x − 5) x will be the union of Eq. (i) and Eq. (ii)
(c) For x ≥ −1
The uncommon real roots are
x=
3 ,x =
− 3 &x =
−2 x3 + x 2 − x − 1 =0

∴ Product = 6 x = 1 & x = −1 are the solutions

(x + 1)(x2 − 0x − 1)
2 2
Sol 17: (a) (x − 1) x − 4x + 3 + 2x + 3x − 5 =0
(x + 1)(x2 + 0x − 1) ⇒ (x +1)2 (x – 1) = 0
2 2
(x − 1) x − 4x + 3 + 2x + 5x − 2x − 5 =0
for x < ‒ 1

(x − 1) x2 − 4x + 3 + (x − 1)(2x + 5) =
0 ∴ –x3 – 1 + x2 – x – 2 = 0

∴ x = 1 is one solution and x2 − 4x + 3 + (2x + 5) =


0 x3 − x 2 + x + 3 =0

When x ∈( −∞ ,1) ∪ (3, ∞ ) (x + 1)(x2 − 2x + 3) =


0

x = ‒ 1 is only solution
x2 − 2x + 8 =0
D < 0 so not possible x = –1, 1

When x ∈(1,3) (d) Same as Example 4 of Solved Examples JEE Advanced.


M a them a ti cs | 2.67

Sol 18: Given that x3 − 3x2 + 1 =0 Sol 20: Minimum value of quadratic occurs at

⇒ α + β + γ = 3 , αβ + βγ + γα = 0 , αβγ = −1 −b 4p p
x
= = =
2a 8 2
Now we have (α − 2)(β − 2)( γ − 2)
When xmin ∈ [0,2]
= (αβ − 2α − 2β + 4)( γ − 2)
∴ f(x)
= min f(x
= min ) 3
= αβγ − 2(αβ + βγ + γα ) + 4(α + β + γ ) − 8
⇒ p ∈ [0, 4]
=−1 + 12 − 8 =3
⇒ p2 − 2p2 + p2 − 2p + 2 =3 ⇒ 2p = −1
Similarly we can find
−1
 α  β  γ   α β   α  ⇒ p= not true
   , ∑ × , ∑  2
 α − 2  β − 2  λ − 2   α − 2 β − 2  α −2
when xmin < 0 ⇒ p<0
 α  β  λ  αβγ −1
 =    =
 α − 2   β − 2   λ − 2  (α − 2)(β − 2)( γ − 2) 3

α β 3αβγ − 2(αβ + βγ + γα ) −3
∑α−2×β−2 =(α − 2)(β − 2)( γ − 2)
=
3
= −1 Xmin

α 4(α + β + γ ) + 3(αβγ ) − 4( ∑ αβ) 12 − 3


∑=
α−2 (α − 2)(β − 2)( γ − 2)
= = 3
3
f(0)<+(2)
Xmin
( −2x2 + 5x − 10)
Sol 19: >0 o
2
2
(sint)x + 2(1 + sint)x + sint + 4
The above expansion is always < 0 as D < 0
⇒ fmin occurs at x = 0
2
∴ (sint)x + 2(1 + sint)x + 9 sint + 4 < 0 ∴ f(0) = p2 − 2p + 2 = 3
For all x ⇒ p2 − 2p − 1 =0
⇒ sint < 0
2± 8
2 p= = 1 ± 2 but p < 0
and 4(1 + sint) − 4 sin t + (9 sint + 4) < 0 2

⇒ −32sin2 t − 8sint + 4 < 0 ⇒ p= 1 − 2


2
⇒ 8sin t + 2sint − 1 > 0 When xmin > 2 ⇒ p > 4
2
⇒ 8sin t + 4 sint − 2sint − 1 > 0 ⇒ fmin occurs at x = 2
⇒ 4 sint(2sint + 1) − 1(2sint + 1) > 0

⇒ ( 4 sin t − 1 ) ( 2sin t + 1 ) > 0 Xmin 2


 −1   1  2
⇒ sint ∈  −1,  ∪  ,1
 2  4 

but sin t < 0 f(2) = 16 − 8p + p2 − 2p + 2 = 3


 −1   4 π 5π 
⇒ sint ∈  −1,  ⇒ t ∈  ,  10 ± 40
 2   3 3  ⇒ p2 − 10p + 15 =
0 ⇒p =
2

a+b = =3⇒K =3 p=5± 10
3
But p > 0 ⇒ p =
5 + 10
2 . 6 8 | Quadratic Equations and Inequalities

Sol 21: Since p(x) is a factor of q(x) =x 4 + 6x2 + 25 and For product of root for (i) and (ii), we can
c b
r(x) = 3x 4 + 4x2 + 28x + 5 , then p(x) will also be a factor –2α = , 3β =
a a
of its linear combination.
We can write here
2
Now, r(x) - 3q(x) = x − 2x + 5
α – 2 = ‒3β and β + 3 = 2α
2
∴ p(x) = x − 2x + 5
Solving these two equations
α – 2 = ‒3 (2α - 3)
Sol 22: f(x) = x2 − 2x − a2 + 1 =(x − 1)2 − a2
⇒ α – 2 = ‒6α + 9
= (x − 1 − a)(x − 1 + a) 11
⇒ 7α = 11 ⇒α=
7
∴ α = a + 1 &β = 1 − a
 11 
Now g(α ) < 0 & g(β) < 0 Therefore, for β, β = 2   -3
7
∴ (a + 1)2 − 2(a + 1)(a + 1) + a2 − a < 0
22 1
= − 3=
−1 7 7
⇒ −a2 − 2a − 1 + a2 − a < 0 ⇒ a >
3 Absolute product of four roots
2 2
and (1 − a) − 2(a + 1)(1 − a) + a − a < 0
1 11 3 −2 66
= =
2
∴ 4a − 3a − 1 < 0 7 7 1 1 49
 1  Therefore, (p + q) = 66 + 49 = 115
⇒ (4a + 1)(a − 1) < 0 ⇒ a ∈  − ,1 
 4 
Sol 25: For origin to lie between the roots.
x+2
Sol 23: ≤ 0 ⇒ x ∈ [ −2, 4)
x−4 af(0) < 0

x2 − ax − 4 ≤ 0 ⇒ (a2 − 6a + 5)(6a − a2 − 8) < 0


 a − a2 + 16 a + a2 + 16  ⇒ (a − 5)(a − 1)(a − 2)(a − 4) > 0
⇒ x∈ , 
 2 2 
  + - + +
1 2 4 5
a + a2 + 16
⇒ < 4 ⇒ a2 + 16 < (a − 8)2 a ∈ ( −∞ ,1) ∪ (2, 4) ∪ (5, ∞ )
2
Also a2 + 2a ≥ 0 ⇒ a(a + 2) ≥ 0
⇒ a2 + 16 < a2 − 16a + 64 ⇒ a < 3
⇒ a ∈ ( −∞ , −2] ∪ [0, ∞ )
a − a2 + 16 2
and ≥ −2 ⇒ a − a + 16 ≥ −4 ∴ a ∈ ( −∞ , −2] ∪ [0,1) ∪ (2, 4) ∪ (5, ∞ )
2
2
(a + 4)2 ≥ a2 + 16 ⇒ a ≥ 0 4
 x5 
Sol 26: (log2 x) −  log1/2  − 20log2 x + 148 < 0
 4
∴ The possible integral values of a are 0, 1, 2
⇒ (log2 x)4 − (5log2 x − 2)2 − 20log2 x + 148 < 0

Sol 24: Given equations are Let log2x = t


ax + bx - c = 0
2
(i) ⇒ t 4 − (25t2 − 20t + 4) − 20t + 148 < 0
ax2 + cx + b = 0 (ii) ⇒ t 4 − 25t2 + 144 < 0
For sum of roots for (i) and (ii), we can ⇒ (t2 − 16)(t2 − 9) < 0
−b −c
α–2= ,β+3= ⇒ (t − 3)(t + 3)(t − 4)(t + 4) < 0
a a
M a them a ti cs | 2.69

+ - + + The equation we have is x3 − 2x2 + 4x + 5074 =


0
-4 -3 3 4
We can write r1 + r2 + r3 =
2 , Σ r1 r2 =
+4 , r1 r2 r3 = −5074
⇒ t ∈ ( −4, −3) ∪ (3, 4) ∴ f =−5074 + 4 × 2 + 4 × 2 + 8 = −5050
 1 1
∴ x ∈  ,  ∪ (8,16) Method 2: (We have to find the product of roots of a
 16 8  cubic whose roots are α + 2, β + 2, γ + 2
2 2 ⇒ α + 2 = x ∴ α= (x − 2) Substituting we get
Sol 27: (log100 x) + (log10x) + logx ≤ 14
(x − 2)3 − 2(x − 2)2 + 4(x − 2) + 5074
2 2
⇒ (2 + logx) + (1 + logx) + logx ≤ 14 The constant term = 5050 ∴ Product = –5050
2
⇒ 2(logx) + 7logx + 5 ≤ 14
Sol 2: (A) We are given that" after x ∈ R and the
2
⇒ 2(logx) + 7logx − 9 ≤ 0 polynomial x8 − x5 + x2 − x + 1

⇒ (logx − 1)(2logx + 9) ≤ 0 When |x| < 1

9 ∴ f(x) = x8 + (x2 − x5 ) + (1 − x) > 0


⇒ − ≤ logx ≤ 1 ⇒ 10−9/2 ≤ x ≤ 10
2 as x2 − x5 > 0 & (1 − x) > 0
When | x | ≥ 1
Sol 28: log1/2 (x + 1) > log2 (2 − x)
f(x)= (x8 − x5 ) + (x2 − x) + 1 > 0
⇒ log2 (2 − x) + log2 (x + 1) < 0 as x8 − x5 > 0 & x2 − x > 0
⇒ log2 (x + 1)(2 − x) < 0 ⇒ (x + 1)(2 − x) < 1 ∴ f(x) is always positive.

⇒ x2 − x − 1 > 0 Sol 3: (B) a(x2 − 2x + 1) + b(x2 − 3x + 2) + x − a2 =0

 1 − 5  1 + 5  ⇒ (a + b)x2 + (1 − 2a − 3b)x + a + 2b − a2 =0
x ∈  −∞ , ∪ ,∞
 2   2 
  Since this is satisfied by all x
Also x + 1 > 0 ⇒ x > –1 and x < 2 ⇒ a+b =0 , 2a + 3b =
1

 ⇒ b = 1 & a = −1
1 − 5  1 + 5 
∴ x ∈  −1, ∪ ,2 
 2   2  also a + 2b − a2 =
0
 
Which is satisfied by (-1, 1)
2
Sol 29: log1/5 (2x + 5x + 1) < 0

⇒ 2x2 + 5x + 1 > 1 Sol 4: (D) y( −1) ≥ −4


⇒ a − b + c ≥ −4  ... (i)
x(2x + 5) > 0
y(1) ≤ 0 ⇒ a + b + c ≤ 0  ... (ii)
 −5 
⇒ x ∈  −∞ ,  ∪ (0, ∞ )
 2  y(3) ≥ 5 ⇒ 9a + 3b + c ≥ 5  ... (iii)
From (i) and (iii)
12a + 4c ≥ −7  ... (iv)
Exercise 2 Equation can be written as

Single Correct Choice Type −a − b − c ≥ 0  ... (v)


∴ From (iv) and (i)
Sol 1: (C) Let (r1 + 2)(r2 + 2)(r3 + 2) =
f
2a + 2c ≥ −4 ⇒ a + c ≥ −2  ... (vi)
= (r1 r2 + 2(r1 + r2 ) + 4)(r3 + 2)
From (v) and (vi)
= r1 r2 r3 + 4(r1 + r2 + r3 ) + 2(r1 r2 + r2 r3 + r1 r3 ) + 8 1
8a ≥ 1 ⇒ a ≥
8
2 . 7 0 | Quadratic Equations and Inequalities

4λ 2 − 2λ2
5: (A) x
Sol = = ,y
1 + λ2 1 + λ2
Let =
λ tan θ 8k ± 64k 2 − 16k k
α= k
=± k2 −
8 4
⇒ x = 2sin2θ & y = 2cos2θ
k k
f = x2 − xy + y 2 α = k − k2 − , β = k + k2 −
4 4
4 4 sin2θ cos2θ =
=− 4 − 2sin 4θ
 2 k 
∴ f lies between 2 and 6 or f ∈ [2,6] If k < 0 then α < 0, β > 0  k − > −k 
 4 
∴a = 2 & b = 6 ∴a+b = 8  
 p 
Let −k =p  ⇒ p2 + > p 
Multiple Correct Choice Type  4 
 

Sol 6: (B, D) x2 + abx=


+ c 0 & x2 + acx =
+ b 0 have a (c) α1β ∈ (0,1)
common roots lets say α k 1
⇒ α2 + abα +=
c 0 & α2 + acα +=
b 0 (i) D ≥ 0 ⇒ k 2 − ≥ 0 ⇒ k > ,k < 0
4 4
1 β ac = & γ ab
∴ α == , (ii) af(0) > 0 ⇒ 4(k) ≥ 0 ⇒ k > 0
a
The other eqn is x2 − a(b + c)x + a2bc =
0
4
af(1) > 0 ⇒ 4(4 − 7k) > 0 ⇒ k <
7
Sol 7: (C, D) Given α, a2 are root of the equation
8k 1 4
4x2 − 15x + 4p =
0 (iii) 0 < < 1 ⇒ 0 < k < 1 ⇒ k ∈ , c
8 4 7
15
⇒ α + α2 = f(n) min = 4(k)2 − 8k 2 + k
4  … (i)

−b
⇒ α3 =p  … (ii) at n= = k − 4k 2
2a
From equation (i)
Sol 9: (A, D) α , α2 (α > 0) are roots of x2 − 30x + b =0
4α2 + 4α − 15 = 0 ⇒ 4α2 + 10α − 6α − 15 = 0
−5 3 α + α2 =30 ; α3 =b
⇒=
α ,=
α
2 2
α2 + α − 30 = 0
−125 27
=⇒p = or p (α + 6)(α − 5) = 0 ⇒ α = −6 , α = 5
8 8
α 5 ( α > 0)
=
Sol 8: (A, B, C) f(n) = 4n2 − 8kn + k , f(n) ≥ 0
α2 =25

=r 25,
= s 5,=
b 125

b+r −s =145

b+r +s =155
2
⇒ 4n − 8kn + k ≥ 0 ⇒∆ ≤ 0 b − r − s =95
⇒ 64k 2 − 16k ≤ 0 ⇒ (2k + 1)(2k − 1) ≤ 0 b −r + s =105

 −1 1 
⇒ k ∈ , 
 2 2 Comprehension Type

∴ k=0 is the only integral solution (x − cos360 )(x − cos840 )(x − cos1560 )
Sol 10: (A) p(x) =

(b) Roots of the equation f(n) = 0 are co efficient of x2 is −(cos360 + cos840 + cos1560 )
M a them a ti cs | 2.71

cos360 + 2cos(360 )cos1200 =


= 0

(B) a > 0 & b2 = 4ac ∴ f(x) =


Sol 11: (B) Absolute term = − cos360 cos840 cos1560  

−1 af(t) > 0 at t < α , af(x) = & t ≠ α


(cos360 )(cos2400 + cos720 )
2 ∴ t < α or t > α c = β & t ≠ α
−1  −1 
cos360  + cos720  2
2  2  (C) a < 0 and b > 4ac
∴ af(t) > 0 for
 −1   5 + 1   5 − 1 − 2 
=    
 2   4   4 
 t < α or t > β t ≠ α , β
 −1   1 
=    5=
 2   16 
−3−2 5 ( )
5 −1 .
16
(D) a < 0 &b2 =
4ac

∴ af(t) > 0 for

Assertion Reasoning Type t < α or t > β( = α ) &

Sol 12: (D) f(x) = ax3 + bx + c sum of three roots =0 t≠α


sum is zero only when atleast one of them is negative
or all roots are zero.
α = β = γ = 0 is one set to prove assertion as false. Sol 15: f(x) = x2 − 2px + p2 − 1

(A) Both roots of f(x) = 0 are less then 4


Sol 13: (A) f(x)= ax2 + ax + (a + b)  …(i)
−b
∴ af(4) > 0 & <4
2
g(x) = ax + 3ax + 3a +=
b f(x + 1)  ….(ii) 2a
2p
D (f ) = −3a2 − 4ab
a2 − 4a(a + b) = ∴ 1 × (16 − 8p + p2 − 1) > 0 & <4
p
since a and b are of same signs, f is either always ⇒ (p − 3) or (p − 5) > 0 & P < 4  ... (i)
positive or always negative depending on a.
P < 3 or p > 5  …(ii)
Since g = f(x + 1)
From (i) and (ii) p ∈ ( −∞ ,3)
∴ g(x) will just shift the group of f to 1 unit left. There
will be no change along y-axis (B) Both roots are greeter then -2

∴ Statement-II is correct explanation of statement-I. −b


∴ af( −2) > 0 & > −2
2a
2p
Match the Columns ⇒ 1(4 + 4,p + p2 − 1) > 0 , > −2 ⇒ p > −2
2a

Sol 14: ax2 + =


bx + c f(x), af(t) > 0 ∴ (p + 1)(p + 3) > 0, p > − 2

p < −3 or p > −1 & p > −2


(A) a > 0 & b2 > 4ac ∴ f(x) =
  ∴ p ∈ ( −1, ∞ )

(C) Exactly one root lies between (–2, 4)


af(x) =
  ⇒ f( −2)f(4) < 0 ⇒ (4 + 4p + p2 − 1)(16 − 8p + p2 − 1) < 0

⇒ (p + 1)(p + 3)(p − 3)(p − 5) < 0


af(t) > 0 at t < α or t > β
+ - + +
&t≠α
-

–3 –1 3 5
2 . 7 2 | Quadratic Equations and Inequalities

∴ p ∈ ( −3, −1) ∪ (3,5) Sol 17: (A) K(x2) + (1 – K)x + 5 = 0

(D) 1 lies between the root α β 4


Given, + =
∴ af(1) < 0 β α 5

⇒ 1(1 − 2p + p2 − 1) < 0 ⇒ p(p − 2) < 0 α2 + β2 4 (α + β)2 − 2αβ 4


= ⇒ =
αβ 5 αβ 5
⇒ P ∈ ( 0, 2 )
(1 − K)2 / K 2 − 2 × 5 / K 4
∴ =
Sol 16: (A) +5 / K 5
6 6 2
(1 − K )
2
 1  6 1   1  3 1 
 x +  −  x + 6 + 2 x +  − x + 3  ⇒ − 10 =
4
 x   x =  x  x  K
3 3
 1 3 1  1  3 1 
x +  + x + 3 x +  + x + 3  K 2 − 2K + 1 − 10K =4K
 x  x  x  x 

 1
3
1 ⇒ K 2 − 16K + 1 =0
Let  x + =
 m & x3 + = n
 x x3 K1 K 2 (K1 + K 2 )2 − 2K1K 2 (16)2 − 2 × 1
∴= + = = 254
m2 − n2  1 
3
1  K 2 K1 K1K 2 1
= = m − n = x +  −  x3 + 3 
m+n  x  x  x2 + ax + b
(B) y =
1  1 x2 + 2x + 3
= 3x + 3 × = 3 x + 
x  x
(y − 1)x2 + (2y − a)x + (3y − b) =
0
x +1
The minimum value of =2 (2y − a)2 − 4(y − 1)(3y − b) ≥ 0
x
∴ f(x)min = 6 ⇒ 4y 2 + a2 − 4ay − 4(3y 2 − (b + 3)y + b) ≥ 0

(B) We want atleast one solution ∴ we want to eliminate ⇒ 8y 2 + 4(a − b − 3)y + 4b − a2 ≤ 0


the cases when these is no solution
2y 2 + (a − b − 3)y + 4b − a2 ≤ 0
∴ All c except when
Now −5 & 4 are solution of equation
 7
1 + log2  2x2 + 2x +  < log2 (cx2 + c)
 2 On solving we get a2 + b2 =
277
For all x (C) f(x) = x3 + px2 + qx + 72
⇒ 4x2 + 4x + 7 ≤ (x2 + c) for all x x2 + ax + b & x2 + bx + a
⇒ (c − 4)x2 − 4x + (c − 7) > 0 Have a common root α
∴c>4 &D<0 ⇒ α 2 + aα + b = 0
⇒ 16 − 4(c − 4)(c − 7) < 0 α 2 + bα + a = 0
∴ c ∈ ( −∞ ,3) ∪ (8, ∞ ) ⇒ α =1 common root
Taking intersection Sum of roots = β + α = −a
∴ The given expansion is not true for any x when ⇒ β = −(a + 1)
c ∈ (8, ∞ )
γ = −(b + 1)
∴ For c ∈ (0,8] the given expansion is true for atleast
one x. ⇒ −(a + 1) =b

[cx2 + c > 0 ⇒ c > 0] ∴ a+b =


1
M a them a ti cs | 2.73

Product of roots = –72 ∴ a + b + c + d = 10c + 10a


∴ ab × 1 =72 = 10(c + a) = 1210
a(1 − a) =−72
Sol 3: Given x2 + ( a − b ) x + (1 − a − b ) =
0 has real and
a2 − a − 72 =0 unequal roots
a2 − 9a + 8a − 72 =
0 ⇒ D>0
⇒ ( a − b ) − 4 (1 )(1 − a − b ) > 0
2
∴ a = 9 or a = −8

In either case b = –8 or b = 9 ⇒ a2 + b2 − 2ab − 4 + 4a + 4b > 0

Sum of squares of roots = a2 + b2 + (1)2 Now, to find values of ‘a’ for which equation has unequal
real roots for all values of b.
= 81 + 1 + 64 = 146
i.e, above equation is true for all b.

Previous Years’ Questions ( )


or b2 + b ( 4 − 2a) a2 + 4a − 4 > 0 is true for all b.

∴ Discriminate, D < 0
Sol 1: Given 3x − y − z =0 … (i)
−3x + 2y + z =0  … (ii)
(
⇒ ( 4 − 2a) − 4 a2 + 4a − 4 < 0
2

2 2
)
⇒ 16 − 16 a + 4a − 4a − 16a + 16 < 0
and −3x + z =0  … (iii)
⇒ − 32a + 32 < 0 ⇒ a > 1
On adding Eqs. (i) and (ii), we get y = 0 So,

3x = z Now, x2 + y 2 + z 2 ≤ 100 Sol 4: Let f ( x ) = 4x3 − 3x − p

⇒ x2 + ( 3x ) + 0 ≤ 100
2 3
1 1 1
Now, f   = 4   − 3   − p
2
  2
  2
⇒ 10 x2 ≤ 100; ⇒ x2 ≤ 10
4 3
X = –3, – 2, – 1, 0, 1, 2, 3 = − − p =−1 (1 + p )
8 2
So, number of such 7 points are possible
f(1) = 4(1)³ – 3(1) – p = 1 – p

Sol 2: Here a +b =10c and c +d =10a 1


⇒ f   .f (1 ) =− (1 + p )(1 − p )
⇒ ( a − c ) + (b − d=
2
) 10 ( c − a)
= (p + 1)(p – 1) = p² – 1
⇒ (b − d=
) 11 ( c − a) … (i)

Which is ≤ 0, ∀ p ∈  −1, 1 .
Since ‘c’ is the root of x2 ‒ 10ax ‒ 11b = 0
1 
⇒ c2 − 10 ac − 11b =
0 ………(ii) ∴ f(x) has at least one root in  ,1
2 
Similarly, ‘a’ is the root of Now, f’(x) = x2 − 3
x2 − 10cx − 11d =
0 = 3(2x – 1)(2x + 1)
⇒ a2 − 10 ca − 11d =
0 ... (iii) 3  1  1 1 
=  x −   x +  > 0 in  ,1
4  2  2  2 
On subtracting Eq.(iv) from Eq. (ii) we get
⇒ f(x) is an increasing function in  1 ,1
( )
c2 − a2 = 11 (b − d)

... (iv) 2 
∴ ( c + a)( c − a) = 11 × 11 ( c − a) 1 
Therefore, f(x) has exactly one root in  ,1 for any p
[from Eq. (i)] ⇒ c +a = 121 ∈ [–1, 1]. 2 
2 . 7 4 | Quadratic Equations and Inequalities

Now let x = cos θ


∴ 22 =
2.2y −1
1   π
∴ x ∈  , 1 ⇒ θ∈ 0,  ⇒ 2y − 2.2y −1 =
0
2   3
From Eq. (i), ⇒ 2y − 2y =
0 true for all y > 1  ... (iii)
4 cos2 θ − 3 cos θ= p ⇒ cos3 θ= p ∴ From Eqs. (i), (ii), and (iii), we get y ∈ {−1} ∪ 1, ∞ )
⇒ 3 θ =cos−1 p
1
Sol 7: Given, (
log( 2x +3) 6x2 + 23x + 21 )
⇒ θ = cos−1 p
3 (
log( 2x +3) 6x2 + 23x + 21 ) = (
4 − log(3x + 7 ) 4x2 + 12x + 9 )
1 
⇒ cos θ =cos  cos−1 p 
 3 
=
⇒ log( 2x +(3x (
4 − log + 7 ) 4x2 + 12x + 9
3) (
)
2x + 3)( 3x + 7 ) =4 − log(3x + 7 ) ( 2x + 3)
2

1  ⇒ 1 + log( 2x +3) ( 3x + 7 ) = 4 − log(3x + 7 ) ( 2x + 3)


cos  cos−1 p  s
⇒x=
 3 
2
Put log 2x +3 ( 3x + 7 ) = y ∴ y + −3= 0
( ) y
Sol 5: Suppose f ( x ) = Ax2 + Bx + C is an integer
whenever x is an integer. ⇒ y 2 − 3y + 2 = 0 ⇒ ( y − 1 )( y − 2 ) = 0
⇒ y = 1 or y = 2
∴ f(0), f(1), f(–1) are integers.

⇒ C, A + B + C, A – B + C are integers. 1 or log( 2x +3) ( 3x + 7 ) =


log( 2x +3) ( 3x + 7 ) = 2

⇒ 3x + 7 = 2x + 3 or ( 3x + 7 ) = ( 2x + 3)
2
⇒ C, A + B, A – B are integers.

⇒ C, A + B, (A + B) – (A – B ) = 2A are integers. ⇒ x = –4 or 3x + 7= 4x2 + 12x + 9

Conversely suppose 2A, A + B and C are integers. ⇒ x = –4 or 4x2 + 9x + 2 =0

Let n be any integer. We have, ⇒ x = –4 or (4x + 1)(x + 2) = 0

 n (n − 1 )  ∴ x =−2, − 4, − 1 / 4  … (i)
f (n) = An2 =
+ Bn + C 2A   + ( A + B)n + C
 2  But log exists only when, 6x2 + 23x + 21 > 0 ,

Since, n is an integer, n (n – 1)/2 is an integers. Also 2A, 4x2 + 12x + 9 > 0 ,


A + B and C are integers. 2x + 3 > 0 and 3x + 7 > 0
We get f(n) is an integer for all integer n. 3
⇒x>−  … (ii)
2
y
Sol 6: Given 2 − 2y −1 − 1= 2y −1 + 1 1
∴ x =− is the only solution.
4
Case I when y ∈( − ∞ , 0

( )
∴ 2− y + 2y −1 − 1 = 2y −1 + 1
Sol 8: (B) Let y = x intersect the curve y = kex at exactly
one point when k ≤ 0.
⇒ 2− y = 2 ⇒ y = −1 ∈ ( − ∞ ,0  … (i)

Sol 9: (A) Let f (=
x ) kex − x
Case II when y ∈ ( 0, 1
f ' ( x ) =kex − 1 =0 ; ⇒ x =−ln k
( )
∴ 2y + 2y −1 − 1 = 2y −1 + 1
f '' ( x ) =
kex ; ∴ f " ( x )  1>0
=
⇒ 2 =2 ⇒ y =1 ∈ ( 0, 1
y
… (ii) x = −1n k

Hence, f(–ln k ) = 1 + ln k
Case III when y ∈ (1, ∞ )
For one root of given equation
M a them a ti cs | 2.75

1 2 2
(∴ α is root of x − 6x − 2 = 0 ⇒ α − 2 = 6α )
1 + ln k = 0 ; ⇒ k =
e
2 2
( Also, β is root of x − 6x − 2 = 0 ⇒β − 2 = 6β )
Sol 10: (A) For two distinct roots, 1 + ln k < 0 (k > 0)
1  1
a10 − 2a10
=
α
8
( 6α=
) − β8 ( 6β ) (
= 3
9
6 α −β
9
)
ln k < –1 , k < ; Hence, k ∈  0, 
e  e
2a9 (
2 α 9 − β9 ) (
9 α 9 − β9 )
Sol 11: (C) Given f ( x ) = 4x2 + 3x3 + 2x + 1
α2 + β2
Sol 15: (B) Sum of roots = and product = 1
( )
f ' ( x ) =2 6x2 + 3x + 1 ; D =9 − 24 < 0 αβ
Given, α + β = −p and α3 + β3 = q
Hence, f(x) = 0 has only one real root.
 1 3 4
f −  = 1 −1 + − > 0
(
⇒ ( α + β ) α2 − αβ + β2 = q )
 2 4 8
−q
∴ α2 + β2 − αβ = .…(i)
 3 6 27 108 p
f  −  =1 − + − 
and ( α + β ) = p2
2
 4  4 16 64

64 − 96 + 108 − 108 ⇒ α2 + β2 + 2αβ = p2  .…(ii)


<0 f(x) changes its sign in
64
From Eqs.(i) and (ii), we get
 3 1  3 1
 − , −  , hence f(x) = 0 has a root in − , − .
 4 2  4 2 p3 − q p3 + q
α2 + β2 = and α β =
3p 3p

(p − 2q) + 1 =0
1/2 t 3/ 4
f ( x ) dx < ∫ f ( x ) dx < ∫ f ( x ) dx
3
Sol 12: (A) ∫ ∴ Required equation x 2

0 0 0
(p + q)
3

∫ f ( x ) dx = ∫ (1 + 2x + 3x )
2 3
Now, + 4x dx

2
=x + x + x + x ; 3 4 ( ) (
⇒ p3 + q x2 − p3 − 2q x + p3 + q =
0 ) ( )
1/2 3/ 4
15 3 530 Sol 16: (A) Since, roots are real therefore D ≥ 0
⇒ ∫ f ( x ) dx = >
16 4 ∫ f ( x=
) dx 256
<3
0
⇒ 4 ( a + b + c ) − 12 λ ( ab + bc + ca) ≥ 0
0 2

⇒ ( a + b + c ) − 3 λ ( ab + bc + ca) ≥ 0
2
Sol 13: (B) Figure is self explanatory

⇒ a2 + b2 + c2 ≥ ( ab + bc + ca) ( 3λ − 2 )

a2 + b2 + c2
⇒ 3λ − 2 ≤  … (i)
ab + bc + ca
-3 -1 1 3
4 2 2 4 b2 + c2 − a2
Also
= , cos A <1
2bc
⇒ b2 + c2 − a2 < 2bc
a10 − 2a8 α10 − β10 − 2α8 + 2β8
Sol 14: (C) =
2a9 (
2 α9 − β9 ) Similarly, c2 + a2 − b2 < 2c a and a2 + b2 − c2 < 2 a b

=
(
α8 α2 − 2 − β8 β2 − 2)
( ) ⇒ a2 + b2 − c2 < 2 ( ab + bc + ca)

2(α − β )
9 9
a2 + b2 + c2
⇒ <2  … (ii)
ab + bc + ca
2 . 7 6 | Quadratic Equations and Inequalities

∴ From Eqs. (i) and (ii), we get 3 λ − 2 < 2 From Eqs. (i), (ii), (iii) and (iv), we get

⇒λ<
4 a ∈ ( −4, 2 )
3
Note: There is correction in answer a < 2 should be –4
< a < 2.

  3 Sol 19: (B) Let f ( x )= ax2 + bx + c > 0 for all x ∈ R


⇒ a > 0 and b2 − 4ac < 0  ... (i)
Sol 17: (D) From graph it is clear that one of the roots of
∴ g(x) = f(x) + f’(x) + f’’(x)
⇒ g ( x )= ax2 + bx + c + 2ax + b + 2a

⇒ g ( x )= ax2 + x (b + 2a)( c + b + 2a)


y=(x-a)(x-1)-1

(b + 2a) − 4a ( c + b + 2a)
2
 a Whose discriminant =
b 
1 =b + 4a + 4ab − 4ac − 4ab − 8a2 =b2 − 4a2 − 4ac
2 2

(x – a)(x – b) – 1 = 0 lies in (‒∞, a) and other lies in ( )


= b2 − 4ac − 4a2 < 0 [from Eq. (i)]
(b, ∞). Therefore,(d) is the answer. ∴ g(x) > 0 for all x, as a > 0 and discriminate < 0.
Thus, g(x) > 0 for all x ∈ R .
Sol 18: (A) Let f ( x ) = x2 − 2ax + a2 + a − 3
Since, both roots are less than 3. Sol 20: (D) The equation x2 − px + r = 0 has roots
(α, β) and the equation
⇒ α < 3, β < 3 ⇒ Sum, S = α + β < 6
α 
x2 − qx + r = 0 has roots,  , β 
α+β 2α 2 
⇒ <3;⇒ <3
2 2
α
⇒ r = αβ and α + β = p and + 2β =q
⇒ a < 3 … (i) 2
Again, product of roots P = α β 2q − p 2(2p − q)
⇒ β= and α =
⇒ p < 9; ⇒ α β < 9 3 3
2
⇒ a2 + a − 3 < 9 ⇒ a2 + a − 12 < 0 ⇒ αβ = r = (2p − q)(2q − p)
9
⇒ (a – 3)(a + 4) < 0

⇒ –4 < a < 3  ... (ii) Sol 21: (B) Suppose roots are imaginary then β = α
1 1
Again, D =B2 − 4AC ≥ 0 and = α ⇒ β = not possible
β β

(
⇒ ( −2a) − 4.1 a2 + a − 3 ≥ 0
2
) ⇒ Roots are real ⇒ (p2 − q) (b2 − ac) ≥ 0

⇒ 4a2 − 4a2 − 4a + 12 ≥ 0 ⇒ − 4a + 12 ≥ 0 ⇒ Statement-I is correct.

⇒a≤0  … (iii) −2b 1 α c


= α + and = , α + β = −2p, αβ = q
a β β a
Again, a f(3) > 0
If β = 1, then α = q ⇒ c = qa(not possible)
⇒ 1 ( 3) − 2a ( 3) + a2 + a − 3 > 0
2
  −2b −2b
+1
also α = 2p
⇒ −= b ap (not possible)
⇒=
2
⇒ 9 − 6a + a2 + a − 3 > 0 ⇒ a − 5a + 6 > 0 a a

⇒ Statement-II is correct but it is not the correct


⇒ (a – 2)(a – 3) > 0
explanation.
∴ a ∈ ( − ∞ ,2 ) ∪ ( 3, ∞ ) … (iv)

M a them a ti cs | 2.77

Sol 22: (B) ax2 + bx + c = 0 ⇒ x2 + 6x – 7 = 0


⇒ α = 1, β = – 7

∞ ∞ n n
 1 1 1 1 
∑  α + β  = ∑ 1 − 7  = 7
= 
n 0=  n 0 

Sol 23: (B) x2 + bx - 1 = 0


x2 + x + b = 0  … (i)
Common root is
(b ‒ 1) x ‒ 1 ‒ b = 0
b +1
⇒ x =
b −1
This value of x satisfies equation (i)

(b + 1) + b + 1 + b = 0 ⇒ b =
2

⇒ 3i − 3i,0
(b − 1) b − 1
2
2017-18 100 &
op kers
Class 11 T
By E ran culty
-JE Fa r
IIT enior emie .
S fP r es
o titut
Ins

MATHEMATICS
FOR JEE MAIN & ADVANCED
SECOND
EDITION

Exhaustive Theory
(Now Revised)

Formula Sheet
9000+ Problems
based on latest JEE pattern

2500 + 1000 (New) Problems


of previous 35 years of
AIEEE (JEE Main) and IIT-JEE (JEE Adv)

5000+Illustrations and Solved Examples


Detailed Solutions
of all problems available

Topic Covered Plancess Concepts


Tips & Tricks, Facts, Notes, Misconceptions,
Sequences and Key Take Aways, Problem Solving Tactics
Series
PlancEssential
Questions recommended for revision
3. SEQUENCES AND
SERIES

1. SEQUENCE

1.1 Introduction
A sequence can be defined as an ordered collection of things (usually numbers) or a set of numbers arranged one
after another. Sometimes, sequence is also referred as progression. The numbers a1, a2 ,a3 .....an are known as terms
or elements of the sequence. The subscript is the set of positive integers 1, 2, 3..... that indicates the position of the
term in the sequence. Tn is used to denote the nth term.
Some examples of a sequence are as follows:
0, 7, 26......................., 1, 4, 7, 10......................., 2, 4, 6, 8…………………..

Note: The minimum number terms in a sequence should be 3.

Sequence

3, 5, 7, 9, .....

three dots means


4rd term goes on forever (infinite)
3rd term
1 st term 2nd term

(”term’’’’, element’’ or ‘’ member’’’’ mean the same thing)


Figure 3.1

1.2 Finite and Infinite Sequences


A sequence containing a finite number of terms is called a finite sequence. If the sequence contains a infinite
number of terms, it is known as an infinite sequence. It is infinite in the sense that it never ends. Examples of infinite
and finite sequences are as follows:
{1, 2, 3, 4......} is an infinite sequence
{20, 25, 30, 35....} is an infinite sequence
{1, 3, 5, 7} is the sequence of the first 4 odd numbers, which is a finite sequence
3 . 2 | Sequences and Series

1.3 Rule
A sequence usually has a rule, on the basis of which the terms in the sequence are built up. With the help of this
rule, we can find any term involved in the sequence. For example, the sequence {3, 5, 7, 9} starts at the number 3
and jumps 2 every time.

+2 +2 +2 +2

0 1 2 3 4 5 6 7 8 9 9

Figure 3.2

As a Formula:
Saying ‘start at the number 3 and jump 2 every time’ is fine, but it does not help to calculate the 10th term or 100th
term or nth term. Hence, we want a formula for the sequence with “n” in it (where n is any term number). What
would the rule for {3, 5, 7, 9.......} be? First, we can see the sequence goes up 2 every time; hence, we can guess that
the rule will be something like ‘2 times n’ (where ‘n’ is the term number). Let us test it out.
n Test Rule Term
1 2n = 2 × 1 = 2 3
2 2n = 2 × 2 = 4 5
3 2n = 2 × 3 = 6 7
That nearly worked! But it is less by 1 every time. Let us try changing it to 2n+1.
n Test Rule Term
1 2n + 1 = 2 × 1 + 1 = 3 3
2 2n + 1 = 2 × 2 + 1 = 5 5
3 2n + 1 = 2 × 3 + 1 = 7 7

That Works: Therefore, instead of saying ‘starts at the number 3 and jumps 2 every time,’ we write the expression
2n + 1. We can now calculate, e.g. the 100th term as 2 × 100 + 1 = 201.

1.4 Notation
The notation Tn is used to represent the general term of the sequence. Here, the position of the term in the
sequence is represented by n. To mention for the ‘5’th term, just write T5.
Thus, the rule for {3, 5, 7, 9…} can be written as the following equation: Tn = 2n + 1.
To calculate the 10th term, we can write T10 = 2n + 1 = 2 × 10 + 1 = 21

Illustration 1: Find out the first 4 terms of the sequence, {Tn} = {–1/n}n.  (JEE MAIN)

Sol: By substituting n = 1, 2, 3 and 4 in{Tn} = {–1/n}n, we will get the first 4 terms of given sequence.
T1 = (–1/1)1 = –1
T2 = (–1/2)2 = 1/4
T3 = (–1/3)3 = –1/27
T4 = (–1/4)4 = 1/256
⇒ {Tn} = {–1, ¼,–1/27, 1/256 ...}
M a them a tics | 3.3

Illustration 2: Write the sequence whose nth term is (i) 2n and (ii) log(nx). (JEE MAIN)

Sol: By substituting n = 1, 2, 3…….., we will get the sequence.


(i) nth term = 2n (ii) nth term = an = log(nx)
a1 = 21, a2 = 22....... a1=log(x)
a2 = log(2x), an = log(nx)
Sequence ⇒ 21, 22,.........2n Sequence ⇒ log(x), log(2x),................log(nx)

2. SERIES
Series is something that we get from a given sequence by adding all the terms. If we have a sequence as
T1, T2, …. ,Tn, then the series that we get from this sequence is T1 + T2 +….+Tn. Sn is used to represent the sum of n
terms. Hence, Sn = T1 + T2 +…. +Tn

3. SIGMA AND PI NOTATIONS

3.1 Sigma Notation


The meaning of the symbol Σ (sigma) is summation. To find the sum of any sequence, the symbol Σ (sigma) is used
before its nth term. For example:
9 n
(i) ∑ n =1+2+3+.........+9 (ii) ∑ ra or ∑ na =1 +2 +3 +........+ n
a a a a

n=1 r =1
5
i+1 1+1 2+1 3+1 4 +1 5+1
(iii) ∑ 2i + 4 = 2 × 1 + 4 + 2 × 2 + 4 + 2 × 3 + 4 + 2 × 4 + 4 + 2 × 5 + 4
i=l

Properties of Σ (Sigma)

k k k
(i) ∑ a = a + a + a… (k times) = ka, where a is a constant. (ii) ∑ ai = a∑ i , where a is a constant.
i=l =i l =i l
n n n in jn jn in
(iii) ∑ (ar ± br =)  ∑ ar ± ∑ br (iv) ∑ ∑ aia j = ∑ ∑ aia j
=r l =r l =r l =i i0=j j0 =j j0=i i0

3.2 Pi Notation
The symbol ∏ denotes the product of similar terms. For example:
6 k
(i) ∏ n = 1 × 2 × 3 × 4 × 5 × 6 (ii) ∏ nm = 1m × 2m × 3m × 4m × ............. × km
n=1 n=1

k
(iii) ∏ n = 1 × 2 × 3 × ........ × k = k!
n=1

4. ARITHMETIC PROGRESSION
The sequence in which the successive terms maintain a constant difference is known as an arithmetic progression
(AP). Consider the following sequences:
a, a + d, a + 2d, a + 3d
T1,    T2,     T3,  T4
3 . 4 | Sequences and Series

 T2 – T1 = T3 – T2 = T4 – T3 = constant (common difference)


The given sequence is an example of AP. The set of natural numbers is also an example of AP.

4.1 General Term


General term (nth term) of an AP is given by Tn = a + (n – 1) d, where a is the first term of the sequence and d is the
common difference of the sequence.
Note:
(i) General term is also denoted by  (last term).
(ii) n (number of terms) always belongs to the set of natural numbers.
(iii) Common difference can be zero, + ve or – ve.
If d > 0 ⇒ increasing AP and the sequence tends to +∞
If d < 0 ⇒ decreasing AP and the sequence tends to −∞
If d = 0 ⇒ constant AP (all the terms remain same)
(iv) The nth term from end is (m – n + 1) term from the beginning, where m is the total number of terms and is given
by the following expression:
Tm-n+1 =Tm − (n–1) d

PLANCESS CONCEPTS

• If the mth term is n and the nth term is m, then the (m + n)th term is 0.
• If m times the mth term is equal to n times the nth term, then the (m + n)th term is 0.
Vaibhav Krishnan (JEE 2009, AIR 54)

Illustration 3: If the 5th term of an AP is 17 and its 7th term is 15, then find the 22th term. (JEE MAIN)

Sol: Using the formula Tn = a + (n – 1) d, we can solve above problem.


Given a + 4d = 17 and a + 6d = 15
⇒ 2d = −2 ⇒ d = −1, a = 21
∴ T22 = 21 − 21 = 0

Illustration 4: If 11 times the 11th term of an AP is equal to 9 times the 9th term, then find the 20th term. (JEE MAIN)

Sol: By solving 11 (a + 10d) = 9 (a + 8d), we will get the value of a and d.


∴ 2a = –38 d ⇒ a = –19d
∴ 20th term = a + 19d = 0

Illustration 5: Check whether the sequences given below are AP or not.  (JEE MAIN)
(i) Tn = n2 (ii) Tn = an + b

Sol: By taking the difference of two consecutive terms, we can check whether the sequences are in AP or not.
(i) Tn = n2; Tn–1 = (n – 1)2
M a them a tics | 3.5

Difference = Tn – Tn–1 = n2 – (n – 1)2 = n2 – (n2 – 2n +1) = 2n – 1


This difference varies with respect to the term. Hence, the sequence is not an AP.
(ii) Tn = an + b; Tn – 1 = a (n – 1) + b
Difference = (an + b) – (a (n – 1) + b) = a (constant)
Hence, the sequence is an AP.

3 1 1
Illustration 6: The 2nd, 31st and the last term of an AP are given as 7 , and −6 , respectively. Find the first term
4 2 2
and the number of terms. (JEE MAIN)

Sol: Using Tn = a + (n – 1) d, we can get the first term and common difference. Suppose a be the first term and d
be the common difference of the AP.
3 31
Given, T2 = 7 ⇒ a+ d =  .... (i)
4 4
1 1
T31 = ⇒ a + 30d =  .... (ii)
2 2
1 31 29 −1
Subtracting (i) from (ii), we get 29d = – =– ⇒ d=
2 4 4 4

1 31 31 1 32
Putting the value of d in (i), we get a – = ⇒ a= + = = 8
4 4 4 4 4
13
Suppose the number of terms be n, so that Tn = –
2
13  1 13
i.e. a + (n – 1) d = – ⇒ 8 + (n – 1) –  = –
2  4  2
⇒ 32 – n + 1 = – 26 ⇒ n = 59
Hence, the first term = 8 and the number of terms = 59.

Illustration 7: Prove that the square roots of three unequal prime numbers cannot be three terms of an AP.
 (JEE ADVANCED)

Sol: Here by Considering p, q, r to be the lth, µth and vth terms of an AP and solving them using Tn = a + (n – 1)
d, we prove the problem.

If possible let p , q, r be the three terms of an AP. a, a +d, a + 2d........., where p ≠ q ≠ r and they are prime numbers.
Let them be the l , µ and vth terms, respectively.
th th

∴ p = a + (λ – 1) d
q = a + (µ – 1) d
r = a + (ν – 1) d
∴ p – r = (λ – µ) d
Also, q – r = (µ – v) d


p– q
=
λ–µ
or
( p – q )( q+ r ) = λ–µ
q– r µ−v
( q – r )( q+ r) µ− v

λ–µ λ–µ
or pq + pr – q – qr = (q – r) or pq + pr – qr =
q+ (q − r) = rational number
µ–v µ−v
3 . 6 | Sequences and Series

Since p, q, r are unequal primes, pq, pr and qr are unequal pure irrational numbers. Thus, LHS is irrational,
but irrational ≠ rational.
Hence, the problem is proved.

Illustration 8: If x, y and z are real numbers satisfying the equation 25 (9x2 + y2) + 9z2 – 15 (5xy + yz + 3zx) = 0, then
prove that x, y and z are in AP.  (JEE ADVANCED)

Sol: By solving the equation 25 (9x2 + y2) + 9z2 – 15 (5xy + yz + 3zx) = 0, we can prove that x, y and z are in AP.
We have
(15x)2 + (5y)2 + (3z)2 – (15x) (5y) – (5y) (3z) – (3z) (15x) = 0
⇒ (15x – 5y)2 + (5y – 3z)2 + (3z – 15x)2 =0
⇒ 15x – 5y = 0, 5y – 3z = 0, 3z – 15x = 0
x y y
⇒ 15x = 5y = 3z ⇒ = = (= k say)
1 3 5
∴ x = k, y = 3k, z = 5k
Thus, x, y and z are in AP.

Illustration 9: Let a1, a2, a3, ...., an be in AP, where a1 = 0 and the common difference ≠ 0. Show that

a3 a4 a5 an 1 1 1  a a
+ + + ........ – a2  +
a
+ ....... +  = n–1 + 2 (JEE ADVANCED)
a2 a3 a4 an–1  2 a3 an–2  a2 an–1


Sol: Given a1 = 0 and d = a2 – a1 = a2 – 0 = a2

By solving LHS and RHS separately, we can solve the problem.

a3 – a2 a4 – a2 a5 – a2 an–1 – a2 an
LHS
= + + + ... + +
a2 a3 a4 an–2 an–1

a2 a3 a4 an–2 an
= + + + ...... + +
a2 a3 a4 an–2 an–1

an a1 + (n – 1)d n–1
= (n–3) + = (n – 3) + (n – 3) +
= {a1 =
0}
an–1 a1 + (n – 2)d n–2
a1 + (n – 2)d a2 1 1 n–1
RHS = + = (n – 2) + = (n – 3) + 1 + = (n – 3) +
a2 a1 + (n – 2)d n–2 n–2 n–2

∴LHS = RHS

PLANCESS CONCEPTS

A sequence obtained by multiplication or division of corresponding terms of two APs may not be in AP
For example, let the first AP be 2, 4, 6, 8,................ and the second AP be 1, 2, 3, 4, 5.......
Multiplying these two, we get 2,8,18,32, ........., which is clearly not an AP
Vaibhav Gupta (JEE 2009, AIR 54)
M a them a tics | 3.7

4.2 Series of an AP
Series of an AP can be obtained as
Sn = a + (a + d) + (a + 2d) ...... [a+(n – 1)d]
Sn = [a + (n – 1)d] + [a+(n – 2)d] ...... + a (writing in the reverse order)
∴ 2Sn = n(2a + (n –1) d)
n n
∴ Sum to n terms, Sn =(2a + (n – 1)d) =( T1 + Tn )
2 2

Illustration 10: Find the sum of the first 19 terms of an AP when a4 + a8 + a12 + a16 = 224. (JEE MAIN)
19
Sol: We need to find out the sum of the first 19 terms of an AP, i.e. (2a + 18d), and we can represent the given
2
equation as (a + 3d) + (a + 7d) + (a + 11d) + (a + 15d) = 224.
Given (a + 3d) + (a + 7d) + (a + 11d) + (a +15d) = 224
⇒ 4a + 36d = 224 ⇒ a + 9d = 56
19 19
Sum of the first 19 terms ⇒ S = (2a + 18d) = × 2 × 56 =1064
2 2

7n + 1
Illustration 11: The sum of n terms of two arithmetic progressions is in the ratio of . Find ratio of the 11th
4n + 27
terms? (JEE MAIN)
n
Sol: Since we know the sum of n terms, i.e. Sn = 2a + (n – 1 ) d , we can write the equation as
2
n
(2a1 + (n – 1)d1 )
7n + 1
= 2 . Hence, by putting n = 11 in this equation, we can obtain the ratio of the 11th terms.
4n + 27 n
(2a + (n – 1)d2 )
2 2
n−1
a1 +  d
7n + 1  2  1
=
4n + 27 n−1
a1 +   d2
 2 

a1 + 10d1 n–1
We want the ratio of . Hence, = 10 ⇒ n = 21
a2 + 10d2 2

a1 + 10d2 148
⇒ =
a2 + 10d2 111

Illustration 12: In an AP of n terms, prove that the sum of the kth term from the beginning and the kth term from
the end is independent of k and equal to the sum of the first and last terms. (JEE MAIN)

Sol: Using the formula Tk = a + (k – 1)d and Tn−k +1= [a + (n – k)d] , we can obtain the kth term from the beginning
and end, respectively, and after that by adding these values we can prove the given problem.
Suppose a be the first term and d be the common difference of the AP.

∴ kth term from the beginning = Tk = a + (k – 1)d  ... (i)

Let l be the last term of the AP and l = a + (n – 1) d


3 . 8 | Sequences and Series

The kth term from the end of the given AP is the (n – k + 1)th term from the beginning.

∴ Tn−k +1= [a + (n – k)d]  ...(ii)

Adding (i) and (ii), we get

∴ The required sum = Tk + Tn−k +1 = [a + (k – 1)d] + a + (n – k)d 2a + (k – 1 + n – k) d

= 2a + (n – 1) d .... (iii), which is independent of k


Moreover, the sum of the first and last terms = a + l = a + [a + (n – 1) d] = 2a + (n – 1) d. ...(iv)
Thus, the sum of the first and last terms is independent of k and (3) = (4). Hence proved.

Illustration 13: If a1, a2, a3 ,..., an is an AP of non-zero terms , then prove that (JEE ADVANCED)

1 1 1 n–1
+ + ..... + =
a1a2 a2a3 an–1an a1an

Sol: By considering a as the first term and d as the common difference, we can write an as a + (n – 1) d, where n =
1, 2, 3,… n.
1 1 1 1 1 1
+ + ..... + = + + ......
a1a2 a2a3 an–1an a(a + d) (a + d)(a + 2d) [a + (n – 2)d][a + (n − 1)d]

1 1 1  1 1 1  1 1 1 
=  – +  –  + ... +  – 
d  a a + d  d  a + d a + 2d  d  a + (n – 2)d a + (n – 1)d 

1 1 1  a + (n − 1)d – a (n − 1)d n−1 n−1


=  − = = = =
d  a a + (n− 1)d  ad(a + (n − 1)d) ad(a + (n − 1)d) a(a + (n − 1)d) a1an

PLANCESS CONCEPTS

Facts:
• If each term of an AP is increased, decreased, multiplied or divided by the same non-zero number, the
resulting sequence is also an AP.
• The sum of the two terms of an AP equidistant from the beginning and end is constant and is equal
to the sum of the first and last terms.
a1 + an = a2 + an – 1 = a3 + an – 2 = ....
Vaibhav Krishnan (JEE 2009, AIR 54

Illustration 14: Split 69 into three parts such that they are in AP and the product of the two smaller parts is 483.
 (JEE MAIN)
Sol: By considering the three parts as a – d, a, and a + d and using the given conditions, we can solve the given
problem.
Sum of the three terms = 69 ⇒ (a – d) + a + (a + d) = 69
⇒ 3a = 69 ⇒ a = 23 ..... (i)
M a them a tics | 3.9

Product of the two smaller parts = 483 ⇒ a (a – d) = 483


⇒ 23(23 – d) = 483 ⇒ 23 – d = 21 ⇒ d = 23 – 21 = 2
Hence, the three parts are 21, 23 and 25.

Illustration 15: Divide 32 into four parts that are in AP such that the ratio of the product of extremes to the product
of mean is 7: 15. (JEE MAIN)

Sol: We can consider the four parts as (a – 3d), (a – d), (a + d) and (a + 3d).
Sum of the four parts = 32
⇒ (a – 3d) + (a – d) + (a + d) + (a + 3d) = 32 ⇒ 4a = 32 ⇒ a = 8

(a – 3d)(a + 3d) 7 a2 – 9d2 7 64 – 9d2 7


And = ⇒ = ⇒ =
(a – d)(a + d) 15 a2 – d2 15 64 – d 2 15

⇒ 128 d2 = 512 ⇒ d2 = 4 ⇒ d = ± 2

Thus, the required parts are 2, 6, 10 and 14.

b+c c+a a+b 1 1 1


Illustration 16: If a + b + c ≠ 0 and , , are in AP, then prove that , , are also in AP.
a b c a b c
 (JEE ADVANCED)
b+c c+a a+b
Sol: Here , , are in AP; therefore, by adding 1 to each term and then by dividing each term by
a b c
b+c c+a a+b
a + b +c, we will get the required result , , are in AP
a b c

b +c  c+a  a+b 
Adding 1 to each term, find that  + 1 , + 1 , + 1  are in AP
 a   b   c 
a+b+c c+a+b a+b+c
i.e. , , are in AP
a b c
1 1 1
Dividing each term by a + b + c, we find that , , are in AP
a b c

 1 1 1 1 1 1 
Illustration 17: If a  +  ,b  +  ,c  +  are in AP, then prove that a, b, c are in AP. (JEE ADVANCED)
b c a c a b

 abc 
Sol: By adding 1 and then multiplying by   to each term, we will get the result.
 ab + bc + ac 

 1 1 1 1 1 1  b + c a+ c b + a
a  +  , b  +  , c  +  are in AP ⇒ a  , b  , c  are in AP
b c a c a b  bc   ac   ab 

ab + ac ab + bc bc + ac
Adding 1, we find that + 1, + 1, + 1 are in AP
bc ac ab

ab + ac + bc ab + bc + ac bc + ac + ab
⇒ , , are in AP
bc ac ab

 abc 
Multiplying by   to all the terms, we find that a, b, c are in AP
 ab + bc + ac 
3 . 1 0 | Sequences and Series

4.3 Arithmetic Mean


The arithmetic mean (AM) A of any two numbers a and b is given by the equation (a + b)/2. Please note that the
sequence a, A, b is in AP. If a1, a2, ...., an are n numbers, the (AM) A, of these numbers is given by:
1
A= (a + a2 + ..... + an )
n 1

Inserting ‘n’ AMs between ‘a’ and ‘b’


Suppose A1, A2, A3, ......., An be the n means between a and b. Thus, a, A1, A2 ,...., An, b is an AP and b is the (n + 2)th term.
b–a
Thus, b = a + (n + 1 )d ⇒ d =
n+1
Now,
A=
1 a + d
A=
2 a + 2d
:
A=
n a + nd
n
 n(n + 1)   n(n + 1)   b − a 
∑ Ai = na + (1 + 2 + 3 + ..... + n)d = na + 
2
=d na +   
i=1    2  n + 1 
n a+b
= nA where, Α =
[2a + b – a] =
2 2

Note: The sum of the n AMs inserted between a and b is equal to n times A.M. between them.

Illustration 18: Insert 20 AMs between the numbers 4 and 67. (JEE MAIN)
b–a
Sol: Given, a = 4 and b = 67; therefore by using the formula d= , we can solve it.
n+1
67 – 4
d= =3
20 + 1
A1 = a + d ⇒ A1 = 7
A2 = a + 2d ⇒ A2 = 10
A3 = a + 3d ⇒ A3 = 13
A20 = a + 20 d ⇒ A20 = 63

Thus, between 4 and 67, 20 AMs are 7, 10, 13, 16, ...., 63.

an + bn
Illustration 19: If is the A.M. between a and b, find the value of n.  (JEE ADVANCED)
an–1 + bn–1
a+b an + bn
Sol: Since the A.M. between a and b = , we can obtain the value of n by equating this to .
2 an–1 + bn–1
an + bn a+b
⇒ = [Given] ⇒ 2an + 2bn = an + abn–1 + an–1 b + bn
a n–1
+b n–1 2

⇒ an – an–1 b = abn–1 –bn ⇒ an–1(a – b) = bn–1 (a – b)

a
n–1
a
0   a 0 
⇒ a n–1
=b n–1
[ a ≠ b] ⇒   = 1=      = 1 
b b  b 
 
⇒ n – 1 = 0 ⇒ n = 1
M a them a ti cs | 3.11

Illustration 20: Between 1 and 31, m arithmetic means are inserted in such a way that the ratio of the 7th and
(m – 1)th means is 5: 9. Calculate the value of m. (JEE MAIN)

Sol: AMs inserted between 1 and 31 are in AP. Thus, by considering d to be the common difference of AP and
obtaining the 7th and (m – 1)th means we can solve the problem.
Suppose A1, A2, A3, A4,...... Am be the m AMs between 1 and 31.
Thus, 1, A1, A2,....Am, 31 are in AP
The total number of terms is m + 2 and Tm+2 = 31
30
1 + (m + 2 – 1) d = 31 ⇒(m + 1) d = 30 ⇒ d =
m+1

30 m + 1 + 210 m + 211
A7 = T8 = a + 7d = 1 +=
7× =
m+1 m+1 m+1

30 m + 1 + 30m – 30 31m – 29
Am–1 = Tm = 1 + (m –1) d = 1 + (m – 1) × = =
m+1 m+1 m+1

A7 (m + 211) / (m + 1) m + 211 5
= = = [Given]
Am–1 (31m − 29) / (m + 1) 31m – 29 9

m + 211 5
⇒ = ⇒ 9m +1899 = 155m – 145
31m – 29 9
2044
⇒ 146m = 2044 ⇒ m= = 14 ;Thus, m = 14
146

Illustration 21: Gate receipts at the show of “Baghbaan” amounted to Rs 9500 on the first night and showed a
drop of Rs 250 every succeeding night. If the operational expenses of the show are Rs 2000 a day, find out on which
night the show ceases to be profitable? (JEE MAIN)

Sol: Here, a = 9500 and d = –250. The show ceases to be profitable on the night when the receipts are just Rs 2000.
Thus, by considering that it will happen at nth night and using Tn = a + (n – 1) d, we can solve this problem.
We have the cost of gate receipt on the first night (a) = 9500
Common difference (d) = –250
Suppose, it happens on the nth night, then
2000 = 9500 + (n – 1) (–250) ⇒ 2000 – 9500 = – 250n + 250
7750
⇒ –7500 – 250 = – 250n ⇒ –7750 = – 250n ⇒ n= = 31
250

PLANCESS CONCEPTS

(a) If the sum of n terms Sn is given, then the general term Tn = Sn – Sn–1, where Sn–1 is sum of (n – 1) terms of AP.
(b) In a series, if Sn is a quadratic function of n or Tn is a linear function of n, the series is an AP.
(i) If Tn = an + b, the series so formed is an AP and its common difference is a.
(ii) If Sn = an2 + bn + c, the series so formed is an AP and its common difference is 2a.
3 . 1 2 | Sequences and Series

PLANCESS CONCEPTS

(c) If in a finite AP, the number of terms is odd, then its middle term is the A.M. between the first term and
last term and its sum is equal to the product of the middle term and number of terms.
(d) It is found that the sum of infinite terms of an AP is ∞, if d > 0 and –∞, if d < 0.
(e) If for an AP, the pth term is q and the qth term is p, then the mth term is = p + q – m.
(f) If for an AP, the sum of p terms is q and sum of q terms is p, then the sum of (p + q) terms is – (p + q).
(g) If for an AP, the sum of p terms is equal to the sum of q terms, then the sum of (p +q) terms is zero.

Sn fn Tn f(2n – 1)
(h) If for different APs, = , then = .
S'n φn T'n φ(2n – 1)
n+1
A  +B
TnAn + B S  2 
(i) If for two APs, = , then we find that n = .
T'n Cn + D S'n n+1
C  + D
 2 
Shrikant Nagori (JEE 2009, AIR 54)

An Important Property of AP: A sequence is said to be an AP if the sum of its n terms is of the form An2 + Bn,
where A and B are constants. Thus, the common difference of the AP is 2A.
Proof: Suppose, a and d be the first term and common difference of AP, respectively, and Sn be the sum of n terms.
n
Sn
= [2a + (n – 1)d]
2
n2 n d 2  d
⇒ Sn =
an + d – d =
 n +  a – n
2 2 2
   2

d d
⇒ Sn = An2 + Bn, where A = and B = a –
2 2

Hence, the sum of n terms of an AP is of the form An2 + Bn.


Conversely, suppose the sum Sn of n terms of a sequence a1, a2, a3, ..........an..... is of the form
An2 + Bn. Then, we have to prove that the sequence is an AP.
We have Sn = An2 – Bn
⇒ Sn – 1 = A(n – 1)2 + B(n – 1) [On replacing n by n = 1]
Now, An = Sn – Sn–1
⇒ An = {An2 + Bn} – {A(n–1)2 – B(n–1)} = 2An + (B – A)
⇒ An + 1 = 2A (n+1) + (B – A) [On replacing n by (n + 1)]
∴ An + 1 – An = {2A(n + 1) +B – A} – {2An + (B – A)} = 2A
Since An + 1 – An = 2A for all n ∈ N, the sequence is an AP with a common difference 2A.
For example, if Sn = 3n2 + 2n, we can say that it is the sum of the n terms of an AP with a common difference of 6.
M a them a ti cs | 3.13

5. GEOMETRIC PROGRESSION
A sequence of non-zero numbers is called a geometric progression (GP) if the ratio of successive terms is constant.
In general, G.P. is written in the following form: a, ar, ar2, ....., arn–1,....
where a is the first term and r is the common ratio

5.1 General Term


If a is the first term and r is the common ratio, then Tn = arn–1.

Illustration 22: The 5th, 8th and 11th terms of a G.P. are given as p, q and, s respectively. Prove that q2 = ps.
 (JEE MAIN)
Sol: By using Tn = arn – 1 and solving it, we can prove the problem.
Given, T5 = p, T8 = q, T11 = s  ....(i)
Now, T5 = ar5–1 = ar4 ⇒ ar4 = p  .... (ii) [Using (i)]
T8 = ar 8–1
= ar ⇒
7
ar =q 
7
... (iii) [Using (i)]
T11 = ar11–1 = ar10 ⇒ ar10 = s  ..... (iv)
On squaring (iii), we get
q2 = a2 r14 = a⋅a.r4⋅r10 = (ar4) (ar10)
⇒ q2 = ps  [Using (ii) and (iv)] proved.

5.2 Series of GP
Let us suppose Sn = a + ar + ar2 + ...... +arn–1  ... (i)
Multiplying ‘r’ on both the sides of (i) and shifting the RHS terms by one place, we get
Snr = 0 + ar + ar2 + ...... +arn ... (ii)
By subtracting (ii) from (i), we get
Sn (1 – r) = a – arn = a (1 – rn)
a(1 – rn )
Sn = , where r ≠ 1
1–r
 a(rn – 1)   Tn+1 – a 
by ⇒ Sn =
Thus, the sum of the first n terms of a G.P. is given= 
 r – 1   r – 1 
   
And Sn = na, when r = 1

Note: If r = 1, then the sequence is of both AP and GP, and its sum is equal to na, i.e. Sn = na.
If |r| < 1, the nth term of G.P. converges to zero and the sum becomes finite.

a(rn – 1)
The sum to infinite terms of G.P. = lim Sn = lim
n→∞ n→∞ r – 1
a
As |r| < 1 rn → 0 as n → ∞ ∴ S∞ =
1–r

5.3 Geometric Mean


If a, b and c are three positive numbers in GP, then b is called the geometrical mean (GM) between a and c, and
b2 = ac. If a and b are two real numbers of the same sign and G is the G.M. between them, G2 = ab.
3 . 1 4 | Sequences and Series

Note: If a and b are two number of opposite signs, then the G.M. between them does not exist.
To Insert ‘n’ GMs Between a and b: If a and b are two positive numbers and we have to insert n GMs, G1, G2, ………,
Gn, between the two numbers ‘a’ and ‘b’ then a, G1, G2, ……….., Gn, b will be in GP. The series consists of (n + 2) terms
and the last term is b and the first term is a.
1
 b  n+1
Thus, b = arn + 2 – 1 ⇒ b = arn + 1 ⇒ r =  
a
⇒ G1 = ar, G2 = ar …….Gn = ar or Gn = ayn+1 . byn+1 = (ab)yn+1
2 n

Note: The product of n GMs inserted between ‘a’ and ‘b’ is equal to the nth power of the single G.M. between ‘a’
and ‘b,’ i.e.
n
∏ Gr = (G)n, where G = ab (GM between a and b)
r =1

5.4 Relation between A.M. and GM


For any two non-negative number A.M. ≥ G.M.

Proof: Let two non-negative numbers be a and b .

( ) ≥0⇒a–2 a+b
2
Now, we can write a– b ab + b ≥ 0 ⇒ a + b ≥ 2 ab ⇒ ≥ ab ⇒ A.M. ≥ GM
2
Note: (i) Equality for AM, G.M. (i.e. A.M. = GM) exists when a = b.
(ii) Since A.M. ≥ GM; (AM)min = GM; (GM)max = AM

x y z
Illustration 23: If x, y and z have the same sign, then prove that + + ≥ 3 . (JEE ADVANCED)
y z x
x y z
Sol: As we know that A.M. ≥ G.M., therefore by obtaining A.M. and G.M. of , and we can prove the problem.
y z x
x y z
Let = x1; = x2; = x3
y z x
x y z
+ +
x1 + x2 + x3 y z x
∴ ≥ (x1 x2 x3 )1/3 ⇒ ≥1
3 3

Hence proved.

an+1 + bn+1
Illustration 24: Calculate the values of n so that may be the G.M. between a and b.(JEE ADVANCED)
an + bn
an+1 + bn+1
Sol: We know that the G.M. between a and b = ab , but here G.M. between a and b is .
an + bn
an+1 + bn+1
⇒ = ab ⇒ an + 1 + bn + 1 = (an + bn) (ab)1/2
an + bn
1 1 1 1 1 1 1 1
n+ n+ n+ n+
⇒ an + 1 + bn + 1 = a 2 ⋅ b 2 + a2 ⋅ b 2 ⇒ an+1 – a 2 ⋅ b 2 = a2 ⋅ b 2 – bn+1

n+
1  1 1 
n+ 
1 1 1 
n+
1
n+
1
⇒ a 2  a2 – b 2  = b 2  a2 – b 2  ⇒ a 2 = b 2
   
   
1
n+ 0
a 2 a 1 1
⇒   = 1 =   ⇒ n+ = 0 ⇒ n= –
b b 2 2
M a them a ti cs | 3.15

Illustration 25: Find the sum to n terms for the series 9 + 99 + 999 …… n. (JEE ADVANCED)

Sol: The given series can be written as S = (10 – 1)+ (102 – 1) + (103 – 1) ….. +(10n − 1).
a(1 – rn )
Thus, by using Sn = , we can find out the required sum.
1–r
10(1 – 10n )
∴ S = (10 + 102 + 103 +….+ 10n) – n; S = –n
1 – 10

Illustration 26: If a1, a2 and a3 are in G.P. with a common ratio r (r > 0 and a > 0), then values of r for which inequality
9a1 + 5a3 > 14a2 hold good are? (JEE ADVANCED)
a
Sol: Since a1 = a = a, a3 = ar, by substituting these values to the given inequality we will get the result.
r 2
a 9a
a1 = , a = a, a3 = ar Now, + 5ar > 14a
r 2 r
⇒ 5ar2 – 14ar + 9a > 0 ⇒ 5r2 – 14r + 9 > 0
⇒ 5r2 – 5r – 9r + 9 > 0 ⇒ 5r(r – 1) – 9 (r – 1) > 0
 5
⇒ (5r – 9) (r – 1) > 0 ⇒ r ∈ R −  1, 
 9
PLANCESS CONCEPTS

• The product of n geometric means between a and (1/a) is 1.


• Let the first term of a G.P. be negative; if r > 1, then it is a decreasing G.P. and if 0< r < 1, then it is an
increasing GP.
a a a a
• If a1, a2, a3,…., an are in AP, a 1 ,a 2 ,a 3 ,.....,a n will be in G.P. whose common ratio is ad.
Nitish Jhawar JEE 2009, AIR 54

Illustration 27: On a certain date, the height of a plant is 1.6 m. If the height increases by 5 cm in the following year and
if the increase in each year is half of that in the preceding years, show that the height of the plant will never be 1.7 m.
 (JEE MAIN)

Sol: Here, the sum of the increases in the height of the plant in the first, second, third, … year is equal to (1.7 – 1.6) m
= 0.1 m = 10 cm.
5 5
According to the question, increases in the height of the plant in the first, second, third, … year are 5, , , … cm,
respectively. 2 4

Let it reach the height of 1.7 m (i.e. increases [1.7 – 1.6] m = 0.1 m = 10 cm).
5 5
Therefore, the sum of 5, , , …to n terms = 10
2 4
 1 
5 1 – n 
 2  a(1 – rn )
⇒ = 10 [ a = 5, r = ½, Sn = ]
1 1–r
1–
2
 1  1 1
⇒ 10  1 –  = 10 ⇒ 1 – =1⇒ = 0 , which does not hold for any n. Thus, the plant will never reach the
n
 n
2  2 2n
height of 1.7 m.
3 . 1 6 | Sequences and Series

Illustration 28: A manufacturer reckons that the value of a machine (price = Rs 15,625) will depreciate each year
by 20%. Calculate the estimated value at the end of 5 years.  (JEE MAIN)

Sol: Here the value of the machine after 5 years= ar5, where a= 15,625. We will obtain the value of r using the given
condition.
The present value of the machine = Rs 15,625
80
The value of the machine in the next year = Rs 15,625 ×
100

80 80
The value of the machine after 2 years = Rs 15,625 × ×
100 100

The values of the machine in the present year, after 1 year and after 2 years are

80 80 80
Rs 15,625, Rs 15,625 × and Rs 15,625 × × , respectively
100 100 100
These values form a GP.
80 4
Here, the first term is Rs 15,625 and the common ratio is ,i.e. .
100 5
5
4 15625 × 1024
Thus, thee value of the machine after 5 years = ar = Rs 15,625 ×   =
5
= 1024 × 5 = Rs 5120
5 625 × 5

5.5 Properties of GP
(a) If each term of a G.P. is multiplied or divided by the same non-zero quantity, then the resulting sequence is
also a GP.
(b) If in a finite GP, the number of terms is odd, then its middle term is the G.M. of the first and last terms.
b c
(c) If a, b and c are in GP, then = ⇒ b2 =ac (which is the condition of GP).
a b
1
(d) The reciprocals of the terms of a given G.P. also give a G.P. with a common ratio of .
r
Proof: Let a1, a2, a3, a4,....., an,...... be the terms of a G.P. with the common ratio r.
an+1
Then, = r for all n ∈ N ... (i)
an
The sequence formed by the reciprocals of the terms of the above G.P. is given by

1 1 1 1
, , ,....., ,.....
a1 a2 a3 an

1 / an+1 an 1
Now, = =  [Using (i)]
1 / an an+1 r
Hence, the new sequence is also a G.P. with the common ratio 1/r.
(e) If each term of a G.P. is raised to the same power (say k), then the resulting sequence also forms a G.P. with
the common ratio as rk.

Proof: Let a1, a2, a3, a4 ,...., an.... be the terms of a G.P. with the common ratio r.
an+1
Then, = r for all n ∈ N (i)
an
Let k be a non-zero real number. Consider the sequence. a1k ,ak2 ,ak3 ,.......,ank ,....
M a them a ti cs | 3.17

k
akn+1  an+1 
Here,
= =  rk for all n ∈ d [Using (i)]
akn a
 n 

Thus, a1k ,ak2 ,ak3 ,......,ank ,..... is a G.P. with a common ratio rk.

(f) In a GP, the product of the terms equidistant from the beginning and the end is always the same and it is equal
to the product of the first and last terms (only for finite GP).
Proof: Let a1, a2, a3 ,....., an be a finite G.P. with the common ratio r. Then,
kth term from the beginning = ak = a1rk–1
kth term from the end = (n – k + 1)th term from the beginning
an–k +1 = a1rn–k
=
∴ (kth term from the beginning) (kth term from the end)= akan–k+1
= a1rk-1a1rn–k =a12 rn–1 =a1.a1rn–1= a1an for all k = 2,3,....,n –1
Thus, the product of the terms equidistant from the beginning and the end is always the same and is equal to
the product of the first and last terms.
(g) If the terms of a G.P. are chosen at regular intervals, the new sequence so formed also forms a G.P. with the
common ratio as rp , where p is the size of interval.
For example:
2, 4, 8, 16, 32, 64, 128,..... (GP, where r = 2)
4, 16, 64 ..... (also a GP, where r = 4)
(h) If a1, a2, a3 ,...., ar..... is a G.P. of non-zero, non-negative terms, then log a1, log a2,...., log an,..... is an AP and vice
versa.
(i) If T1, T2, T3... and t1, t2, t3 are two GPs, T1t1, T2t2, T3t3.... is also in GP.

Proof: Let the two GPs be T1, T2, ...., Tn,... with the common ratio R
Tn+1
⇒ = R  ... (i)
Tn

and t1, t2,....,tn,.... with common the ratio r

tn+1
⇒ =r .... (ii)
tn
 T  tn–1 
Multiplying each term of the sequence (i) by the corresponding term of (ii), we get  n+1 
  = Rr
 Tn  tn 
Thus, the resulting sequence is also in G.P. with the common ratio Rr.
( j) The resulting sequence thus formed by dividing the terms of a G.P. by the corresponding terms of another
G.P. is also a GP.
Proof: Let the two GPs be T1,T2,....,Tn,.... with the common ratio R
Tn+1
⇒ = R  ... (i)
Tn

and t1,t2,....,tn.... with the common ratio r

tn+1
⇒ = r  ...(ii)
tn
Dividing each term of the sequence (i) by the corresponding term of (ii), we get
3 . 1 8 | Sequences and Series

Tn+1
tn+1 R 
=    [Using (i) and (ii)]
Tn r
tn
R 
Thus, the resulting sequence is also in G.P. with the common ratio   .
r

Illustration 29: If sum of infinite terms of G.P. is 15 and sum of squares of infinite terms of G.P. is 45, then find GP.
 (JEE MAIN)
a
Sol: As the sum of infinite terms S∞ = , therefore by using this formula we can obtain the value of the common
ratio. 1–r

a
= 15
1–r
a2 225(1 – r)(1 – r)
Now, a2, a2r2, a2r4,……; = 45 ∴ = 45
1–r 2 (1 – r)(1 + r)

225− 225r = 45 + 45r; 180 = 270 r ∴ r = 2/3

Illustration 30: If x = 1 + a + a2 + .... ∞, y = 1 + b + b2+.... ∞ and |a| < 1, |b| < 1, then prove that
xy
1 + ab + a2b2 + .... =  (JEE MAIN)
x + y –1
a
Sol: By using the formula S∞ = , we can solve problem.
1–r
1
x = 1 + a + a2 + .... to ∞ = ( |a| < 1)
1–a
1 1 x –1
⇒ 1–a= ⇒ a=1– ⇒ a=  ... (i)
x x x
1
Also, y = 1 + b + b2 +...... to ∞ = ( |b| < 1)
1–b
1 1 y –1
⇒ 1–b= ⇒ b =1– ⇒ b =  ... (ii)
y y y
1
∴ 1 + ab + a2b2 + ..... to ∞ = ( |a| < 1, |b| < 1 ⇒ |ab| < 1)
1 – ab
1 xy xy
= [Using (i) and (ii)] = =
x –1 y –1 xy – xy + x + y – 1 x + y – 1
1– .
x y
Hence proved.

Illustration 31: If S1, S2, S3, ……., SP denote the sum of an infinite G.P. whose first terms are 1, 2, 3, …… , p, respectively
1 1 1 1 p(p + 3)
and whose common ratios are , , ,……., , respectively, show that S1 + S2 + S3 + ……..+ Sp = .
2 3 4 (p + 1) 2
 (JEE ADVANCED)
a
Sol: By using S∞ = we can obtain S1, S2, S3, ……., SP and after that by adding them we can prove the given
equation. 1–r
M a them a ti cs | 3.19

1 1
For S1, we have a = 1, r= ∴ S1 = =2
2 1
1–
2
1 2
For S2, we have a = 2, r= ∴ S2 = =3
3 1
1–
3
1 3
For S3, we have a = 3, r= ∴ S3 = =4
4 1
1–
4

1 p
For Sp, we have a = p, r= ∴ Sp = =p+1
p +1 1
1–
p +1

Adding all these, we get S1 + S2 + S3 + ………+ Sp = 2 + 3 + 4 + …….. + (p + 1)

p p p(p + 3)
= [2 + (p + 1)] = [p + 3] =
2 2 2
Hence proved.

6. ARITHMETIC GEOMETRIC PROGRESSION


A series formed by multiplying the corresponding terms of AP and G.P. is called arithmetic geometric progression
(AGP).
Let a = first term of AP, b = first term of GP, d = common difference and r = common ratio of GP, then
AP: a, a + d, a + 2d, a+ 3d, ….., a + (n – 1) d
GP: b, br, br2, br3,….., brn–1
AGP: ab, (a + d) br, (a + 2d) br2 …. (a + (n – 1) d) brn–1 (Standard appearance of AGP)
The general term (nth term) of an AGP is given as Tn = [a + (n – 1)d] brn–1.

6.1 Series of AGP


To find the sum of n terms of an AGP, we suppose that its sum as Sn and then multiply both the sides by the
common ratio of the corresponding G.P. and then subtract as in the following way. Thus, we get a G.P. whose sum
can be easily obtained.
Sn = ab + (a + d) br + (a + 2d) br2 + ….. + (a + (n– 1)d) brn–1  …(i)
rSn = 0 + abr + (a + d) br + …… + (a + (n – 1)d) br 
2 n
…(ii)
After subtraction, we get
Sn(1 – r) = ab + [dbr + dbr2+…..+ up to (n – 1) terms] – [ (a + (n – 1)d)brn ]

dbr(1 – rn–1 )
Sn(1 – r) = ab + − (a + (n – 1)d)brn
1–r

ab dbr(1 – rn–1 ) (a + (n – 1)d)brn


Sn = + − . This is the sum of n terms of AGP
1–r (1 – r)2 1–r

For an infinite AGP, as n → ∞, then rn → 0 (Q|r| < 1)

ab dbr
⇒ S∞ = +
1 – r (1 – r)2
3 . 2 0 | Sequences and Series

Illustration 32: If |x| < 1, then find the sum S = 1 + 2x + 3x2 + 4x3 ….. + ∞. (JEE MAIN)

Sol: The sum can be found out by calculating the value of Sx – S.


Sx = x + 2x2 + 3x3 + 4x3+……∞

1 1
S(1 – x) = 1+ x + x2 + x3 + ….. +∞; S(1 – x) = ⇒S =
(1 – x) (1 – x)2

Illustration 33: If |x| < 1, then find the sum S = 1 + 3x + 6x2 + 10x3 + ……∞.  (JEE ADVANCED)

Sol: Similar to above illustration.


S = 1 + 3x + 6x2 + 10x3 + ……∞
Sx = x + 3x2 + 6x3 ……….∞
S(1–x) = 1 + 2x + 3x2 + 4x3 ……….. ∞
S(x)(1–x) = x + 2x2 + 3x3 …….. ∞

S (1–x)2 = 1+ x + x2 + …… ∞

1 1
S(1 – x)2 = S=
1–x (1 – x)3

7. MISCELLANEOUS SEQUENCES

Type 1: Some Standard Results


n
n(n + 1)
(i) Sum of the first n natural numbers = ∑r = 2
r =1
n
(ii) Sum of the first n odd natural numbers = ∑ (2r – 1) = n2
r =1
n
(iii) Sum of the first n even natural numbers = ∑=
2r n(n + 1)
r =1
n
n(n + 1)(2n + 1)
(iv) Sum of the squares of the first n natural numbers = ∑ r2 = 6
r =1
n
n(n + 1)(2n + 1)
Proof: ∑ n2 = 6
n=1

Consider (x + 1)3 = x3 + 1 + 3x2 + 3x


(x + 1)3 – x3 = 3x2 + 3x + 1
Put x = 1, 2, 3…. n
23 – 13 = 3.12 +3.1 + 1
33 – 23 = 3.22 +3.2 + 1
(n + 1)3 – n3 = 3n2 + 3.n + 1
Adding all, we get
⇒ (n + 1)3 – 1 = 3(12 + 22 +32+….+n2 ) + 3 (1 + 2 + …. +n) + n
3n(n + 1) 3n(n + 1)
⇒ (n + 1)3 – 1 = 3Σn2 + 3 + n ⇒ 3Σn2 = (n + 1)3 – 1 – –n
2 2
M a them a ti cs | 3.21

3n(n + 1)
⇒ 3Σn2 = n3 + 1 + 3n2 + 3n – 1 – –n
2
3n(n + 1)
⇒ 3Σn2 = n3 + 3n2 + 2n –
2
2n3 + 6n2 + 4n – 3n2 – 3n 2n3 + 3n2 + n
⇒ 3Σn2 = ⇒ 3Σn2 =
2 2
2n3 + 3n2 + n2 + n 2n2 (n + 1) + n(n + 1)
⇒ 3Σn2 = ⇒ 3Σn2 =
2 2

n(n + 1) × (2n + 1) 2 n(n + 1) × (2n + 1)


⇒ 3Σn2 = ⇒ Σn =
2 6
n 2
 n(n + 1) 
(v) Sum of the cubes of first n natural numbers ∑ r =  
3

r =1  2 
Proof: Consider (x + 1)4 – x4 = 4x3 + 6x2 + 4x + 1
Put x = 1, 2, 3…… n
24 – 14 = 4⋅13 + 6⋅12 + 4⋅1 + 1
34 – 24 = 4⋅23 + 6⋅22 + 4⋅2 + 1
44 – 34 = 4⋅32 + 6⋅22 + 4⋅3 + 1
:
(n + 1)4 – n4 = 4⋅n3 + 6⋅n2 + 4⋅n + 1
Adding all, we get
(n + 1)4 – 14 = 4(13 + 23 + ….. + n3) + 6(12 + 22+ ….+ n2) + 4(1 + 2 + 3….. +n) +n

 n(n – 1)(2n + 1)    n + 1 
= 4Σn3 + 6   + 4 n  + n
 6    2 
On simplification, we get
2
  n + 1 
Σn =  n 
3

  2 
(vi) Sum of the fourth powers of the first n natural numbers (Σn4)

n(n + 1)(2n + 1)(3n2 + 3n – 1)


Σn4 = 14 + 24 + ….. +n4 ; Σn4 =
30

[The result can be proved in the same manner as done for Σn3]
n i j
Illustration 34: Find the value of ∑∑ ∑ (1) .  (JEE MAIN)
=i 1 =j 1 =
k 1

n n
n(n + 1) n(n + 1)(2n + 1)
Sol: Using the formula ∑r = 2
and ∑ r2 = 6
, we can solve the problem.
r =1 r =1
n i j
Let S = ∑∑ ∑ (1)
=i 1 =j 1 =
k 1

n i n
i(i + 1) 1 1  n(n + 1)(2n + 1) n(n + 1)  n(n + 1)(2n + 4) n(n + 1)(n + 2)
S= ∑∑ ( j)= ∑ 2
==
2
[Σn2 + Σn]
2

6
= +
2 
 =
12 6

=i 1 =j 1 =i 1
3 . 2 2 | Sequences and Series

Illustration 35: Find the sum of 1.2.3 + 2.3.4 + 3.4.5…… n terms.  (JEE MAIN)

Sol: The given series is in the form of Tn = n(n + 1) (n + 2) = n3 + 3n2 + 2n.

n 2 n n
n+1 n(n + 1)(2n + 1) n(n + 1)
Therefore, by using ∑ r3 = n2  2  ∑ r2 = 6
and ∑r = 2
, we can solve the problem.
r =1   r =1 r =1

Tn = n(n + 1) (n + 2) = n (n2 + 3n + 2)
Tn = n3 + 3n2 + 2n

2 n+1
3n (n + 1 ) (2n + 1)
2

ΣTn = Σn3 + 3Σn2 + 2Σn = n   + + n(n + 1)


 2  6

Type 2: Using Method of Difference: If T1, T2, T3, T2, T4 T5…. is a sequence whose terms are sometimes in AP and
sometimes in GP, then for such series we first compute their nth term and then compute the sum to n terms using
sigma notation.

Illustration 36: Find Sn = 6 + 13 + 22 ……… + Tn. (JEE ADVANCED)

Sol: By calculating [Sn + (– Sn)], we will get Tn. After that we will obtain Σ Tn and thus we will get the result.
Σn = 6 + 13 + 22 ……… Tn
– Σn = – 6 – 13 ……… – Tn–1 – Tn
⇒ 0 = 6 + (7 + 9 +11 ….. (Tn – Tn–1)) – Tn
⇒ Tn = 6 + (7 + 9 + 11…. (Tn – Tn–1)) = 6 + (n – 1) (7 + n – 2) = 6 + (n – 1) (n + 5)
⇒ Tn = 6 +n2 + 4n – 5 = n2 + 4n + 1

n(n + 1)(n + 1)
Σn = Σn2 + 4Σn + n = + 2n (n +1) + n
6

 1  1 1 
Illustration 37: Find S = 1 +  1 +  +  1 + +  + ...... n terms. (JEE ADVANCED)
 3  3 32 

1 1 1
Sol: Given, Tn = 1 + + + ..... + ; therefore by obtaining ΣTn, we will get the result.
3 32 3n−1

 1  1 1
S = 1 +  1 +  +  1 + +  .......
 3  3 9

 1 
31 – n 
1 1 1  3 
Tn = 1 + + + ..... + =
3 32
3 −1
n 2

3n 3 1 3n 3  1 1 1  3 3 1 
ΣTn = – =
Σ –  + ......  =  n –  1 –  
2 2 3n 2 23 3 2 n
3  2 2 3n  

Type 3: Splitting the nth term as a difference of two: Here, S is a series in which each term is composed of the
reciprocal of the product of r factors in an AP.
M a them a ti cs | 3.23

1 1 1
Illustration 38: Find the sum of n terms of the series + + + .....  (JEE ADVANCED)
1 ⋅ 2 ⋅ 3 ⋅ 4 2 ⋅ 3 ⋅ 4 ⋅5 3 ⋅ 4 ⋅5 ⋅ 6

1
Sol: Here nth term of the series will be Tn = .
n(n + 1)(n + 2)(n + 3)

1
By considering Sn = c – λ, where λ = , we will get the result.
3(n + 1)(n + 2)(n + 3)

1
First calculate the nth term, Tn =
n(n + 1)(n + 2)(n + 3)

Now, let the sum of the above series be given by:


Sn = c – λ, where λ is obtained by replacing the first factor by (last factor – first factor)  ... (i)
Hence,
1 First factor = n 
λ=   ... (ii)
3(n + 1)(n + 2)(n + 3) Last factor= n + 3 
1
Using (ii) ⇒ Sn = c – ... (iii)
3(n + 1)(n + 2)(n + 3) 

To calculate ‘c’, put n = 1 in (iii)


1 1 1
S1 = c – ⇒ =c– ⇒ c = 1/18
1⋅2⋅3⋅ 4 1⋅2⋅3⋅ 4 3⋅2⋅3⋅ 4

Put the value of ‘c’ in (iii)


1 1 1 1 1 
Sn = – , S =  – 
18 3(n + 1)(n + 2)(n + 3) n 3  6 (n + 1)(n + 2)(n + 3) 

1 1
Remark ⇒ If we want to calculate S∞, then n → ∞, → 0 ⇒ S∞ =
(n + 1)(n + 2)(n + 3) 18

Note: The above method is applicable only when the series looks like as follows:

1 1 1
+ + + ....
a(a + d)(a + 2d) (a + d)(a + 2d)(a+ 3d) (a + 2d)(a + 3d)(a + 4d)

Type 4: Vn Method: This is method of resolving the nth term into partial fraction and summation by telescopic
cancellation. First, find the nth term of the series and try to create a denominator part in the numerator by using
partial fraction whenever the series is in the form of fraction or Tn is in the form of fraction.
For example, let us suppose a summation where the nth term is like the following:
2
Tn =
2
n –1
1 1
Using the partial fraction, we can write the nth term as Tn = –
n–1 n+1
Now, when we find the summation, there will be telescopic cancellation and thus we will get the sum of the given
series.

Type 5: Dealing with Sn4: This technique is valid for Σn2 and Σn3. In this type, there is a series in which each term
is composed of factors in an AP, i.e. factors of several terms being in AP.
3 . 2 4 | Sequences and Series

1 1
Tn = [(n + 1)(n + 2)(n + 3)(n + 4)[n– (n – 1)]] = (n(n + 1)(n + 2)(n+ 3)(n + 4) – (n – 1)(n + 1)(n + 2)(n + 3)(n + 4))
5 5
1
T1 = (1 ⋅ 2 ⋅ 3 ⋅ 4 ⋅ 5 − 0)
5
1
T2 = (2 ⋅ 3 ⋅ 4 ⋅ 5 ⋅ 6 – 1 ⋅ 2 ⋅ 3 ⋅ 4 ⋅ 5)
5
1
T3 = (3 ⋅ 4 ⋅ 5 ⋅ 6 ⋅ 7 – 2 ⋅ 3 ⋅ 4 ⋅ 5 ⋅ 6)
5
1
Tn = = (n(n + 1)(n + 2)(n+ 3)(n + 4) – (n – 1)(n + 1)(n + 2)(n + 3)(n + 4))
5

Adding all, we have

1
Sn =
5
(n(n + 1)(n + 2)(n + 3)(n + 4))
Note: This method will be applicable only when the series looks like the following:

a(a + d) ( a+ 2d) + (a + d) (a + 2d) (a + 3d) + (a + 2d) (a + 3d) (a + 4d)+….+ up to n term, where a = first term and
d = common difference

PLANCESS CONCEPTS

1 1 1 1 n
• + + + .... + =
1.2 2.3 3.4 n(n + 1) n + 1

1 1 1 1 1
+ + .... + =–
1.2.3 2.3.4 n(n + 1)(n + 2) 4 2(n + 1)(n + 2)

1 1 1 1  1 1 
+ + ..... + =  – 
a1a2 ....ar a2a3 ....ar +1 anan+1 ....an+r –1 (r – 1)(a2 – a1 )  a1a2 ....ar –1 an+1an+ 2 ....an+r –1 

1
• a1a2….ar + a2a3….ar+1 +…..+ anan+1….an+r–1 = [ a a ….a – a0a1a2….an]
(r + 1)(a2 – a1 ) n n+1 n+r

Where a1a2….an are in AP and a0 = a1 – d

Shivam Agarwal JEE 2009, AIR 54

8. HARMONIC PROGRESSION
A sequence will be in harmonic progression (HP) if the reciprocals of its terms are in AP, e.g. if a1, a2, a3, ……….
1 1 1
are in HP, then , , ……. are in AP. For every AP, there will be a corresponding HP, and the standard H.P. is
a1 a2 a3
1 1 1 1
, , + ............ + .
a a + d a + 2d a + (n – 1)d

The terms of a harmonic series are the outcomes of an AP.


M a them a ti cs | 3.25

Note:
(i) 0 cannot be a term of H.P. because ∞ is not a term of AP, but ∞ can be a term of HP.
(ii) There is no general formula for finding the sum to n terms of HP.
1 1 1
(iii) If a, b and c are in HP, then , , are in AP.
a b c
2 1 1 2ac
∴ = + ⇒ b=
b a c a+c

⇒ a, b and c are in HP

1 1 1 1 a–b b–c a a–b


Moreover, – = – i.e. = ; i.e. =
b a c b ab bc c b–c

1 1 3
Illustration 39: If the 3rd, 6th and last terms of a H.P. are , , , then find the number of terms.  (JEE MAIN)
3 5 203
1
Sol: If nth term of a H.P. is , then the nth term of the corresponding AP will be a. Thus, by using
a
Tn = a + (n – 1) d, we will get the result.
Let a be the first term and d be the common difference of the corresponding AP.
1
If the 3rd term of H.P. = ; then the 3rd term of the corresponding AP = 3
3
⇒ a + 2d = 3  ….. (i)
1
If the 6th term of H.P. = ; then the 6th term of the corresponding AP = 5
5
⇒ a + 5d = 5 ….. (ii)
2 5
From (i) and (ii), we get d = ⇒ a =
3 3
3 203
If the nth term of H.P. = ; then nth term of AP =
203 3
203 5 2 203
a + (n – 1) d = ; + (n – 1) =
3 3 3 3
5 + 2n – 2 = 203; n = 100

Illustration 40: If a1, a2, ……….an are in H.P. then the expression a1a2 + a2a3 + …….+ an – 1an is equal to. 
 (JEE ADVANCED)
1 1 1 1 1 1 1 1 1
Sol: As , ,...... are in AP, taking = – = – ... – = d, we can obtain the values of a1a2, a2a3 and
a1 a2 an a2 a1 a3 a2 an an–1
so on.
a1, a2, ……. an are in HP
1 1 1
, ,...... are in AP
a1 a2 an

1 1 1 1 1 1
⇒ = – = – ... – = d (say)
a2 a1 a3 a2 an an–1

1 1 1
⇒ a1a2 = (a1 – a2 ) , a2a3 = (a2 – a3 ) ,……., an–1an = (an – 1 – an)
d d d
3 . 2 6 | Sequences and Series

1 1
Hence, a1a2 + a2a3 + ….+an – 1an = [a1 – a2 + a2 – a3 + …..+an – 1 – an] = (a1 – an)
d d
1 1 a1 – an
But = + (n – 1) d ⇒ = (n – 1) d
an a1 ana1

∴ a1a2 + a2a3 + ……. + an –1 an = (n – 1)a1an

8.1 Harmonic Mean


If a, b and c are in HP, then the middle term is called the harmonic mean (HM) between them. If H is the HM
2ac
between a and b, then a, H, c are in H.P. and H = .
a+c
To Insert n HMs Between a and b
Let H1, H2, ………., Hn be the n HMs between a and b.
Thus, a, H1, H2, ……….., Hn b are in HP.
1 1 1 1 1
, , .......... , are in AP.
a H1 H2 Hn b

1 1 1 1 a–b
⇒ = + (n + 1)d; – = (n + 1) d; d =
b a b a ab(n + 1)

1 1 1 1
⇒ = + d ⇒ = + 2d
H1 a H2 a

1 1 1 1
⇒ = + 3d ⇒ = + nd
H3 a Hn a

Adding all, we get


n
1 n d(n)(n + 1) n n(n + 1) (a – b) 1 a – b  n n(a + b) 1
∑H =
a
+
2
=
a
+
2 ab(n + 1)
=n  +
a 2ab
=
2ab
[2b + a – b]=
2ab
= n
H
i=1 i  

Note: The sum of the reciprocals of all the n HMs between a and b is equal to n times the reciprocal of the single
HM between a and b.
100
1 1
For example, between 1 and if 100 HMs are inserted, then ∑ = 5050.
100 i=1 Hi

8.2 Sum of the Reciprocal of ‘n’ Harmonic Means

n(a + b)
The sum of reciprocal of n harmonic means =
2ab

To Insert n Harmonic Means Between a and b

a, H1, H2, H3 …………Hn, b → H.P.

1 1 1 1 1 1
, , , ........... , → A.P.
a H1 H2 H3 Hn b

1 1 a–b
= + (n + 1) d ⇒ (n + 1) d =
b a ab
M a them a ti cs | 3.27

a–b
d=
(n + 1)ab

1 1 1 1
Illustration 41: Find the sum of + + ......... .
H1 H2 H3 Hn
n
1 1 1 1 1
Sol: Using =
Hn a
+ nd , we can obtain the values of ,
H1 H2
and so on. Then, by obtaining the value of ∑H , we
n=1 n
will get the result.

1 1 1 1
= + d = + 2d
H1 a H2 a
n
1 1 1 n n(n + 1)
=
Hn a
+ nd ⇒ ∑H= a
+
2
d
n=1 i

n n(n + 1) × (a – b) n (2b + a – b) n
= + = = (a+ b)
a 2(n + 1)ab a 2b 2ab

(i) For 3 numbers a, b and c, HM is defined as the reciprocals of the mean of the reciprocals of a, b and c, i.e. means
11 1 1 3
of reciprocal =  + +  ; HM =
3a b c  1 1 1
 + + 
a b c
(ii) If a1, a2, a3, ………., an are n numbers, then

 a + a2 + a3 + a3 + ........ + an 
AM =  | 1 | 
 n 
GM = (a1 a2 a3 …….an)1/n
 
 
n
HM =  
 1 1 1 1 
 + + + ....... 
 a1 a2 a2 an 

Illustration 42: If a2, b2, c2 are in AP, then show that b + c, c + a, a + b are in HP.

Sol: Given that a2, b2 and c2 are in AP. Thus, by adding ab + ac + bc to each term and then dividing each term by
(a + b)(b + c)(c + a), we will get the result.
By adding ab + ac + bc to each term, we find that a2 + ab + ac + bc, b2 + ba + bc + ac, c2 + ca + cb + ab are in AP, i.e.
(a + b)(a + c), (b + c)(b + a), (c + a)(c + b) are in AP
∴ Dividing each terms by (a + b)(b + c)(c + a), we find that
1 1 1
, , are in AP, i.e.
b+c c+a a+b

b + c, c + a, a + b are in HP

H1 + a Hn + b
Illustration 43: If H1, H2, ……., Hn are n harmonic means between a and b (≠ a), then find the value of + .
H1 – a Hn – b
1 1 1 1 1
Sol: As a, H1, H2, ....., Hn, b are in HP, , , ....... , are in AP. By considering d as the common difference of this
a H1 H2 Hn b
3 . 2 8 | Sequences and Series

AP and using Tn = a + (n – 1) d we can solve this problem.

1 1 1 1
= + (n + 1)d and – = (n – 1)d
b a Hn H1

H1 + a 1 / a + 1 / H1 1 / a + 1 / H1
Now, = =
H1 – a 1 / a – 1 / H1 –d

Hn + b 1 / b + 1 / Hn 1 / b + 1 / Hn
and = =
Hn – b 1 / b – 1 / Hn d

H1 + a Hn + b 1 / a + 1 / H1 1 / b + 1 / Hn 1  1 1   1 1 
∴ + = + =  –  +  –   = 2n
H1 – a Hn – b –d d d  b a   Hn H1  

9. RELATION BETWEEN AM, G.M. AND HM

If a and b are two positive numbers, then it can be shown that A ≥ G ≥ H and A, G, H are in GP, i.e. G2 = AH.

a+b 2ab
Proof: Given that, A = , G = ab and H =
2 a+b
a+b
∴ A–G= – ab
2

( a – b)2
⇒ A–G= ≥0
2

⇒ A ≥ G ……(i)
2ab
G–H= ab –
a+b
 a + b – 2 ab  ab
⇒ G–H= ab   ⇒ G–H= ( a – b )2 ≥ 0
 a+b  a+b
 
⇒ G ≥ H ……(ii)
Using (i) and (ii), we find that
A ≥ G ≥ H
Please note that the equality holds only when a = b.
Proof of G2 = AH
a+ b 2ab
Proof: A = ,G= ab and H =
2 a+b
Now, AH = ab = G2 ⇒ A, G & H are in G.P.
A G
Moreover, = ; ∴ A ≥ G ⇒ G ≥ H
G H
Therefore, A ≥ G ≥ H; in fact, RMS ≥ A.M. ≥ G.M. ≥ HM (where RMS is root mean square).
M a them a ti cs | 3.29

PLANCESS CONCEPTS

• If a and b are two positive quantities, then AM, G.M. and HM are always in GP, i.e. only for two numbers.
• If there are three numbers. then AM, G.M. and HM are in G.P. only when the three numbers. are in GP.
For example, 2, 4, 8 → GP
14 24
GM = 4; A.M. = ; HM =
3 7
• For two positive numbers, it has been shown that A ≥ G ≥ H, equality holding for equal numbers.
• For n non-zero positive numbers, it has been shown that A ≥ G ≥ H, equality holding when all the
numbers are equal.
Girish Chandani (JEE 2009, AIR 54)

Illustration 44: If a, b and c are unequal positive numbers in HP, then prove that
a+b c+b
+ > 4.  (JEE ADVANCED)
2a – b 2c – b
2 1 1
Sol: As a, b and c are in HP, therefore = + . Thus, by substituting this to LHS, we can prove the given problem.
b a c
1 1 1 1 1 1 1 1
+ + + +
LHS = b a + b c = b a + b c , using (i)
2 1 2 1 1 1
– –
b a b c c a

c c a a a+c a c
=+ + += + + .
b a b c b c a
a c
+
a c a c
Now, A.M. > G.M. ⇒ c a > . or + > 2.
2 c a c a

(=
a + c) (a − c)
2 2
a+c a+c
= = +2
b 2ac 2ac 2ac
a+c
a+ca c
∴ LHS =  + > 2 + 2 = 4
b c a

10. PROPERTIES OF AM, G.M. AND HM


(i) The equation with a and b as its roots is x2 – 2Ax + G2 = 0

Proof: The equation with a and b as its roots is x2 – (a + b) x + ab = 0


a+b
⇒ x2 – 2Ax + G2 = 0 (∴ A = ,G= ab )
2
(ii) If A, G and H are the arithmetic, geometric and harmonic means, respectively, between three given numbers a,
3G3
b and c, then the equation having a, b, c as its roots is x3 –3 Ax2 + x – G3 =0
H
3 . 3 0 | Sequences and Series

1 1 1
a+b+c + +
1
Proof: As given, A = , G = (abc)1/3 and = a b c
3 H 3

3G3
⇒ a + b + c = 3A, abc = G3 and = ab + bc + ca
H

The equation having a, b and c as its roots is x3 – (a + b + c) x2 + (ab +bc +ca) x – abc = 0

3G3
⇒ x3 – 3Ax2 + x – G3 = 0
H

Illustration 45: The harmonic means between two numbers is given as 4, their A.M. is A, and G.M. is G, satisfy the
relation 2A + G2 = 27. Determine the two numbers.  (JEE ADVANCED)

Sol: Let a and b be the two numbers and H = 4 be the harmonic mean between them. Therefore, by using A.M. =
a+b
and G.M. = ab , we can obtain the values of a and b.
2
H = 4 (given)
As A, G and H are in GP, therefore G2 = AH ⇒ G2 = 4A
Also, 2A + G2 = 27 (given; ∴G2 = 4A)
∴ 6A = 27
9 a+b 9
⇒ A= ⇒ = ⇒ a+b=9
2 2 2
We have, G2 = 4A and A = 9/2 ⇒ G2 = 18 ⇒ ab = 18
The quadratic equation having a and b as its roots is x2 – (a + b) x + ab = 0 or, x2 – 9x +18 = 0
⇒ x = 3, 6
Thus, the two numbers are 3 and 6.

Illustration 46: If 2a + b + 3c = 1 and a > 0, b > 0, c > 0, then find the greatest value of a4b2c2 and obtain the
corresponding values of a, b and c.  (JEE ADVANCED)
Sol: Since there is a4,. take four equal parts of 2a; as there is b2, take two equal parts of b; as there is c2, take two
equal parts of 3c. Since A.M. ≥ G.M., obtaining A.M. and G.M. of these numbers will help in solving this illustration.

2a 2a 2a 2a b b 3c 3c
Let us consider the positive numbers , , , , , , , .
4 4 4 4 2 2 2 2

2a 2a 2a 2a b b 3c 3c
+ + + + + + +
For the numbers, A = 4 4 4 4 2 2 2 2 = 2a + b + 3c = 1
4+2+2 8 8
(∴ 2a + b + 3c = 1)
1 1
 2a 2a 2a 2a b b 3c 3c  8  1 1 1 8
G =  . . . . . . .  =  . . .32 a4b2c2 
 4 4 4 4 2 2 2 2  24 22 22 
1
1  32 4 2 2  8
∴ A ≥ G ⇒ ≥ a b c 
8  28 

1 32 28 1
or ≥ a4b2c2 or ≥ a4b2c2 or ≥ a4 b2 c2 .
8 8 2 8 8
8 2 3 .8 9.4
M a them a ti cs | 3.31

1
Hence, the greatest value of a4b2c2 =
9.48
It has been found that when the equality holds, the greatest value takes place.
We know that A = G when all the numbers are equal, i.e.
2a b 3c
= = ⇒ a = b = 3c
4 2 2
a b c
∴ = = = k ∴ a = 3k , b = 3k, c = k
3 3 1

∴ 2a + b + 3c = 1 ⇒ 6k + 3k + 3k = 1

1 3 3 1 1 1 1
∴ k= ∴ a= ,b= , c = , i.e. a = , b = , c =
12 12 12 12 4 4 12

Arithmetic Mean of the mth power


Suppose a1, a2, …..,an be n positive real numbers (not all equal) and let m be a real number, then
m
a1m + a2m + .....anm  a + a2 + .....an 
>  1  , if m ∈ R –[0, 1]
n  n 
m
a1m + a2m + .....anm  a + a2 + .....an 
If m ∈ (0, 1), then <  1 
n  n 
m
a1m + a2m + .....anm  a + a2 + .....an 
Thus, if m ∈ {0, 1}, then =  1 
n  n 

PROBLEM-SOLVING TACTICS

(a) When looking for a pattern in a sequence or series, writing out several terms will help you see the pattern, do
not simplify directly. If you do this way, it is often easier to spot the pattern (if you leave terms as products,
sums, etc.).
(b) If each term of an AP is multiplied by (or divided by a non-zero) fixed constant C, the resulting sequence is
 1 
also an AP, with a common difference C times  or times  the previous.
 c 
(c) Tips for AP problems
(i) When the number of terms are three, then we take the terms as a – d, a, a + d;
Five terms as a - 2d, a - d, a, a + d, a + 2d
Here, we take the middle term as ‘a’ and common difference as ‘d’.
(ii) When the number of terms is even, then we take:
Four terms as a – 3d, a – d, a + d, a + 3d;
Six terms as a – 5d, a – 3d, a – d, a + d, a + 3d, a + 5d
Here, we take ‘a – d’ and ‘a + d’ as the middle terms and common difference as ‘2d’.
(iii) If the number of terms in an AP is even, then take the number of terms as 2n and if odd then take it as
(2n + 1).
3 . 3 2 | Sequences and Series

(d) Tips for G.P. problems


a a
(i) When the number of terms is odd, then we take three terms as a/r, a, ar; five terms as , ,a,ar,ar 2 .
2 r
r
Here, we take the middle term as ‘a’ and common ratio as ‘r.’

a a a a a
(ii) When the number of terms is even, then we take four terms as, ,ar,ar3; six terms as , ,ar,ar3 ,ar5.
r 3 r
r5 r 3 r
'a'
Here, we take and ‘ar’ as the middle terms and common ratio as ‘r2.’
r

(e) Tips for H.P. problems


1 1 1
For three terms, we take as , ,
a–d a a+d

1 1 1 1
For four terms, we take as , , ,
a – 3d a – d a + d a + 3d

1 1 1 1 1
For five terms, we take as , , , ,
a – 2d a – d a a + d a + 2d

FORMULAE SHEET

Arithmetic Progression: Here, a, d, A and Sn represent the first term, common difference, A.M. and sum of the
numbers, respectively, and Tn stands for the nth term.

n
1. Tn = a + (n – 1) d 4. Sn = 2a + (n – 1 ) d
2 

2. Tn =
Tn–1 + Tn+1
5. A = 
( a1 + a2 + ….. + an )
2 n
Insertion of n arithmetic means between a and b is An
n
3.
Sn =
2
( a + Tn ) 6. n (b – a)
=a+
n+1

Geometric Progression: Here, a, r, Sn and G represent the first term, common ratio, sum of the terms and
G.M., respectively, and Tn stands for the nth term.

a(rn – 1)
1. Tn = a.rn–1 4. Sn =
r– 1

Tn = Tn−1 . Tn+1 a
2. 5. S∞ = (for – 1 < r < 1)
1–r

Insertion of n geometric means between a and b is


Tn+1 – a G1 = ar, G2 = ar2 …….Gn = arn or Gn = b/r, where
3. Sn =
r– 1 6.
1
 b  n+1
r=  
a
M a them a ti cs | 3.33

Arithmetic Geometric Progression: Here, a = the first term of AP, b = the first term of GP, d = common dif-
ference and r = common ratio of GP.

1. Sn = ab + (a + d)br + (a + 2d)br2 + (a + 3d)br3 + …..

2. n–1 a + (n – 1)d brn


Sn = ab + dbr (1 – r ) – 
1–r (1 – r)2 1–r

3. ab dbr
S∞ = + (for – 1 < r < 1)
1 – r (1 – r)2

Harmonic Progression
1 1 1 1
1. an = , where a = and d = −
a + (n− 1)d a1 a2 a1

1 1 1 1 1 
2. =  + + ... + 
H n  a1 a2 an 

3. Insertion of n harmonic means between a and b


1 1 a−b
= +
H1 a (n+ 1)ab

1 1 2(a − b)  1 1 n(a − b) 
= + and so on ⇒  = + 
H2 a (n+ 1)ab  Hn a (n+ 1)ab 

n
n(n + 1)
1. The sum of n natural numbers
∑r = 2
r =1

n
2. The sum of n odd natural numbers
∑ (2r – 1) = n2
r =1

n
3. The sum of n even natural numbers
∑ 2r
= n(n + 1)
r =1

n
n(n + 1)(2n + 1)
4. The sum of squares of n natural numbers
∑ r2 = 6
r =1

n 2
 n(n + 1) 
5. The sum of cubes of n natural numbers ∑ r3 =  2 
r =1  
3 . 3 4 | Sequences and Series

Solved Examples

JEE Main/Boards n ( 2a + (n – 1)d)


Sn =
2
Example 1: Find the rth term if the pth term of an AP is
q and the qth term is p . 20
S20 = [2 × 99 + (20 – 1)(–4)]
2
Sol: Using Tn = a + (n – 1) d, we can obtain the pth, qth = 10[198 + 19 × (–4)] = 10(198 – 76) = 1220
and rth terms.
Let the initial term and common difference of the given Example 4: Find out the G.P. if the fifth and second
AP be a and d, respectively. terms of a G.P. are 81 and 24, respectively.
As given,
Sol: We know that in GP, the nth term is given by
q = a + (p – 1) d  … (i) Tn = a.rn – 1. Thus, by using this formula, we can find the GP.
p = a + (q – 1) d  … (ii) Given, T5 = 81 and T2 = 24
Subtracting (i) by (ii), we find that ∴ 81 = ar4 … (i)
q – p = (p – q) d and 24 = ar … (ii)
∴d=–1 Dividing (i) by (ii), we get
3
Putting d = – 1 in (i), we get 81 27 3 3
= r3 ⇒ r3 = ⇒ r3 =   ⇒ r =
a = q+ p – 1 24 8 2 2

∴ tr = a +(r – 1)d Substituting the value of r in (ii), we get, a = 16

= (q + p – 1) – r+ 1 = p + q – r Thus, the required G.P. is 16, 24, 36, 54 ,….

Example 2: Find out the number of terms in a given AP Example 5: If the sum of four numbers in AP is 50 and
20, 25, 30, 35, …. 100. the greatest of them is four times the least, then find
the numbers.
Sol: We know that Tn = a + (n – 1) d.
Sol: Let the four numbers in AP be a, a +d, a + 2d, a
Given, a = 20, d = 5 and Tn = 100. Therefore, by solving +3d with d > 0.
the equation, we will get the number of terms.
As given, sum of the numbers is 50.
Let the number of terms be n.
a + (a + d) + (a + 2d) + (a + 3d) = 50
Given, Tn = 100, a = 20, d = 5
∴ 4a + 6d = 50
Tn = a + (n – 1) d
⇒ 2a + 3d =
25 ...(i) and
⇒ 100 = 20 + (n – 1) 5 ⇒ 80 = (n – 1) 5
a + 3d = 4a
⇒ 16 = (n – 1) ⇒ n = 17
⇒ 3d = 3a
∴d = a
Example 3: Solve the following series:
∴ Equation (i) becomes 5a = 25
99 + 95 + 91 +87 + …. to 20 terms
Thus, a = 5 = d
n
Sol: Using Sn = 2a + (n – 1 ) d , we can solve the given Therefore, the four number are 5, 10, 15 and 20.
problem. 2

We know that the terms of the given series are in AP. Example 6: If S1, S2, S3,….., SP are the sum of p infinite
Given, geometric progression whose first terms are 1, 2, 3,….,
D = – 4, a = 99 and n = 20 1 1 1 1
p and whose common ratios are , , ,.... ,
2 3 4 p +1
M a them a ti cs | 3.35

p(p + 3) n–1
respectively, then prove that S1 + S2 +….+ Sp = . a1 + d
2 ⇒ 2 1 = 5n + 4  … (i)
a n–1 9n + 6
Sol: As we know S∞ = , therefore by using this a2 + d2
1–r 2
formula we can obtain the value of S1, S2 , ….Sp. a + 12d1
The ratio of the 13th terms is 1 [which is
obtained from (i) with n = 25] a2 + 12d2
a
We know that S∞ =
1–r
a1 + 12d1 129
1 2 ∴ =
∴ S1 = = 2 ; S2 = =3 a2 + 12d2 231
1 1
1– 1–
2 3
p Example 9: If the 7th and 8th terms of an H.P. are 8 and
Sp = = p +1
1 7, respectively, then find its 15th term.
1–
p +1 1
p p Sol: We know that tn = . Therefore, by using
S1 + S2 +….+Sp = [2 × 2 + (p – 1)1] = [p + 3] a + (n− 1)d
2 2
this formula we can solve the given problem.
Example 7: Solve the series 1 +2⋅2+ 3⋅22 + 4⋅23 + ….. Given, T7 = 8 =and T8 = 7
+100⋅299. 1
∴ = 8 ⇒ 8 a + 48 d − 1 = 0 ..(i)
Sol: Let S = 1 + 2⋅2 + 3⋅2 + 4⋅2 + ….+ 100⋅2 .
2 3 99
a + 6d
Therefore, by multiplying 2 on both the sides and then 1
= 7 ⇒ 7 a + 49 d − 1 = 0 ..(ii)
taking the difference, we can solve the given problem. a + 7d
Given,
By solving these two equations, we find that d = a
S = 1 + 2⋅2 + 3⋅22 + 4⋅23 + ….+ 100⋅299  … (i)
1
Multiplying 2 on both the sides, ∴ From eq.(i), we get 8
=
7a
2S= 1⋅2+2⋅22+3⋅23+…..+99⋅299+100⋅2100  … (ii) 1
⇒a=d=
56
Subtracting (ii) from (i), we get
1 56
–S =1 + 1⋅2 + 1⋅22 +1⋅23+……+1⋅299 - 100⋅2100 ∴ T15 = =
a + 14d 15
1 – 2100
–S= − 100 ⋅ 2100 ;
1–2 Example 10: Suppose x y and z are positive real
numbers, which are different from 1.
⇒ S = 99⋅2100 + 1
If x18 = y21 = z28, then show that 3, 3logy(x). 3logz(y) and
Example 8: If (5n + 4) : (9n + 6) is the ratio of the sums 7 logx(z) are in AP.
of the nth terms of two APs, then find out the ratio of
Sol: By applying log on x18 = y21 = z28, we can find the
their 13th terms.
values of logy x , logz y and logx z .
Sol: Let a1 and a2 be the first terms of the two APs and Given, x18 = y21 = z28
d1 and d2 be their respective common difference.
Taking log, we find that
n
Applying Sn = 2a + (n – 1 ) d , we can solve the given
2 18 log x = 21 log y = 28 log z
problem.
logx 7
Given, log
= y x =
log y 6
n 7
[2a + (n – 1)d1 ] ⇒ 3logyx =  … (i)
2 1 5n + 4 2
=
n 9n + 6 log y 4
[2a2 + (n – 1)d2 ] logzy = = ; 3logzy = 4 … (ii)
2 logz 3
3 . 3 6 | Sequences and Series

logz 9 or (1 + d) (1 + 2d) (1 + 3d) + 15 = 0


log
=xz =
logx 14 or (d + 2) (6d2 + 5d + 8) = 0
9 ⇒ d=–2
⇒ 7logx z =  … (ii)
2
The other factors do not give any real solution.
7 9
The numbers 3, , 4 and are in AP with common
2 2 ∴ x = y–1, y = z–3, z = x1/3 or x = y–1 = z3
1
difference = .
2
Example 13: There are four numbers of which the
∴ 3, logyx, 3logzy and 7logxz are in AP.
first three are in G.P. and the last there are in AP, with a
y
common difference of 6. If the first number and the last
Example 11: If x= a = b z c and if a, b and c are number are equal, then find the numbers.
positive and in GP, then prove that x, y and z are in AP.
Sol: Let the four numbers be a, a – 2d, a – d, a, where
Sol: This problem can be solved by taking log on d=6
x y z
a
= b
= c. a, a – 12, a – 6 are in GP.
a 1/x
=b 1/y
=c 1/z
⇒ a(a – 6) = (a – 12)2
loga logb logc ⇒ a2 – 6a = a2 – 24a + 144
⇒ = = = k
x y z ⇒ 18a = 144
⇒ log a = kx, log b = ky, log c = kz  … (i)
⇒ a=8
a, b and c are in GP
The numbers are 8, – 4, 2 and 8.
⇒ b2 =ac
∴ 2log b = log a + log c Example 14: a, b and c are the pth, qth and rth terms
of both an AP and a GP, respectively, then prove that
⇒ 2ky = kx + kz by (i)
ab–c. bc–a. ca–b = 1 (both progressions have the same first
⇒ 2y = x + z term.)
⇒ x, y and z are in AP.
Sol: By using formula Tn = a + (n – 1) d and Tn = a.rn–1,
we can obtain the pth, qth and rth terms of both an AP
Example 12: Determine the relation between x, y and z and a GP.
if 1, logyx, logzy, – 15logxz are in AP.
Tp = a = a1 + (p – 1) d1 =a1(r1)p–1  … (i)
Sol: By considering the common difference as d and Tq = b = a1 + (q – 1)d1 = a1(r1)q–1  … (ii)
obtaining its value by logyx = 1 + d and logzy = 1 + 2d,
we can determine the required relation. Tr = c = a1 + (r – 1)d1 = a1(r1)r-1  … (iii)

Suppose d be the common difference of the given AP, From (i), (ii), (iii)
then a – b = (p - q) d1
logyx = 1 + d ⇒ x = y 1+d
 … (i) b – c = (q - r) d1
logzy = 1 + 2d ⇒ y = z1+2d  … (ii) c – a = (r - p) d1
15logxz =– (1 + 3d) Therefore, ab–c. bc–a. ca–b
1 +3d
= (a1r1p–1)b–c (a1r1q–1)c–a (a1r1r–1)a–b
⇒ z=x –15  … (iii)
Elimination y and z from equations (i), (ii) and (iii), we = a1b–c+c-a+a–b.r1(p–1)(b–c)+(q–1)(c–a)+(r–1)(a-b)
get (p −1)( q−r )d1 +( q−1)(r −p )d1 +(r −1)(p −q)d1
= a10 . r1
(1–d)(1 + 2d)(1 +3d)
x= x –15 0 0
= a=
1 .r1 1

(1 + d)(1 + 2d)(1 + 3d)


∴ 1=
–15
M a them a ti cs | 3.37

JEE Advanced/Boards Sol: We can write x – y = p as

Example 1: If the sum of first n terms of three arithmetic ( x+ y )( x− y =


p. )
progressions are S1, S2 and S3, the first term of each
Thus, by following this method we can represent
being 1 and the common differences being 1, 2 and 3,
difference of a1, a2, a3, ……., an.
respectively, then prove that S1 + S3 = 2S2
a1, a2, a3, …….., an are in AP.
n
Sol: Using Sn = 2a + (n – 1 ) d , we can get the values
2  a2 – a1 = a3 – a2 = ……….= an – an–1 = d (say)
of S1, S2 and S3.
Given, a = 1, d1 = 1, d2 = 2, d3 = 3
⇒ ( a2 + a1 )( a2 – a1 )
S1 =
n
2
[2a + (n – 1) d1] = ( a3 + a2 )( a – a ) .........
3 2

= ( a )( a – a ) = d
n n
= [2 × 1 + (n – 1) 1] = [1 + n] an + n–1 n n–1
2 2

= ( a – a ),
n n 1 1 1
S2 = [2a + (n – 1)d2] = [2 × 1 + (n – 1) 2] = n2 ⇒ 2 1
2 2 a2 + a d a + a2
1 3
n n
S3 = [2a + (n – 1) d3] = [2 × 1 + (n – 1) 3]
2 2 a3 – a2
= , ………
n d
= [3n – 1]
2
1 an – an–1
n =
S1 + S3 = [1 + n + 3n – 1] = 2n2 = 2S an + an–1 d
2
1 1
Example 2: Calculate the sum to n terms of the series: LHS= + +…………+
a1 + a2 a2 + a3
8 +88 + 888 + ……..

Sol: We can solve this problem by taking 8 as common


from given series and applying various operations.
=
1
d
( an – a1 )
Let Sn = 8 + 88 + 888 +……. to n terms = 8 [1 + 11 + an – a1
111 + …….]  d=
n–1
8 8
= [9 + 99 + 999 + …….] = [(10 – 1) + (100 – 1) an – a1 n–1
9 9 ⇒ LHS = (n – 1)= = RHS
+(1000 – 1) + ……… to n terms] an – a1 an + a1

8 8 8
= [10 + 100 + 1000 + … + to n terms] – n= Example 4: A series of natural numbers is divided into
9 9 9
groups: (1); (2, 3, 4); (5, 6, 7, 8, 9) and so on. Prove that
(10n – 1) 8n the sum of the numbers in the nth group is (n – 1)3 + n3.

9 9
8 Sol: In this problem, the last term of each group is the
= [10n+1 – 9n – 10] square of the corresponding number of the group.
81
Thus, the first term of the nth group is (n – 1)2 + 1 = n2
n
Example 3: If a1, a2, a3, ……., an are in AP, where ai > 0 for – 2n + 2. Hence, by using Sn = 2a + (n – 1 ) d , we can
all I, then show that 2
solve the problem.
1 1
+ + ........ +
a1 + a2 a2 + a3 The number of terms in the first group = 1
The number of terms in the second group = 3
1 n–1
= The number of terms in the third group = 5
an–1 + an a1 + an
∴ The number of terms in the nth group = 2n–1
3 . 3 8 | Sequences and Series

The common difference of the numbers in the nth group 3


S1 =
=1 8
2n – 1 3 11
The required sum = [2(n2 – 2n + 2) + (2n – 2) 1] From (i), S∞= 1 + =
2 8 8
2n – 1
= [2n2 – 2n + 2] = (2n – 1)[n2 – n + 1]
2 Example 6: If n ∈ N and n > 1, then prove that
= 2n3 – 3n2 + 3n – 1 = n3 + (n – 1)3 n–1
(a) nn ≥ 1.3.5 ………(2n – 1) (b) 2n ≥ 1 + n 2 2

Example 5: Find the sum of the series


Sol: (a) Use the inequality A.M. ≥ GM.
1 3 5
1+ + + + ….. to ∞ . 1 + 2 + 22 + .... + 2n–1
5 52
53 (b) By solving ≥ (1.2.22 …….2n–1)1/n,
n
Sol: We can write given series as S∞ = 1 + S1, where S1 = we can prove the given equation.
1 3 5 1
+ + + …… to ∞ Thus, by multiplying on both 1 + 3 + 5 + ...... + (2n – 1) 1/n
5 52 53 5 (a)∴ ≥ 1.3.5 …. ( 2n – 1 ) 
n
the sides and subtracting, we can obtain the required n
sum. 1 + (2n – 1) 1/n
⇒ 2 ≥ 1.3.5 …. ( 2n – 1 ) 
1 3 5 n
Let S∞ = 1 + + + + …… to ∞
5 5 2
53 1/n
⇒n =1.3.5 …. ( 2n – 1 ) 
∴ S∞ = 1 + S1  ………..(i)  
Where
1.3.5 …. ( 2n – 1 )
∴ nn =
1 1 3 5
∴ S = + + + ………. to ∞ ....(ii)
5 1 52 53 54 1 + 2 + 22 + .... + 2n–1
(b) ≥ (1.2.22 …….2n–1)1/n
Subtracting, (ii) from (i) n
1/n
2n – 1 1  2 
n(n–1)
4 1 1 1 
∴ S = +2  + + .......to ∞ ⇒ × ≥ 2 
5 1 5 52
53  2–1 n  
 
1
1 52 1 2 1 3 2n – 1
= +2 = +  = , ⇒ ≥ 2n−1/2
5 1 5 4  5  10 n
1–
5
⇒ 2n ≥ n 2n–1 + 1

JEE Main/Boards

Exercise 1 Q.3 Sum of n terms of the series,


(i) 0.7 + 0.77 + 0.777 + …. (ii) 6 + 66 + 666 + ….
Q.1 In a G.P. sum of n terms is 364. First term is 1 and
common ratio is 3. Find n.
Q.4 If a, b, c are in A.P., prove that
(i) b + c, c + a, a + b are also in A.P.
Q.2 The sum of an infinite geometric progression is 2
and the sum of the geometric progression made from 1 1 1
(ii) , , are also in A.P.
the cubes of this infinite series in 24. Then find the series. bc ca ab

(iii) a2(b + c), b2(c + a), c2(a + b) are also in A.P.


M a them a ti cs | 3.39

 1 1 1 1 1 1  (ii) The numbers 2, a, b are consecutive terms of an A.P.


(iv ) a  +  ,b  +  ,c  +  are also in A.P. and
b c c a a b
(iii) The numbers b, c, 18 are consecutive terms of a G.P.
Q.5 The sum of three numbers in A.P. is 12 and the sum
of their cubes is 288. Find the numbers. Q.18 If a > 0, b > 0 and c > 0, prove that:
1 1 1
Q.6 Find the sum of the integers between 1 and 200 (a + b + c)  + +  ≥ 9
a b c
which are
(i) Multiple of 3 (ii) Multiple of 7 Q.19 If A1, A2, G1, G2, ; and H1 ,H2 be two A.M.’s, G.M.’s
(iii) Multiple of 3 and 7 and H.M’s between two numbers, then prove that:
G1G2 A1 + A2
=
Q.7 The sum of first n terms of two A.P.’s are in the ratio H1H2 H1 + H2
(3n – 3): (5n + 21). Find the ratio of their 24th terms.
Q.20 Find the coefficient of x99 and x98 in the polynomial:
Q.8 If the p term of an A.P. is x and q term is y, show
th th
(x – 1) (x – 2) (x – 3) … (x – 100).
p+q x – y
that the sum first (p + q) terms is x + y + 
2  p – q
Q.21 The interior angles of a polygon are in A.P. The
a b c smallest angle is 120º and the common difference is 5º.
Q.9 If a, b, c are in H.P. prove that , , are Find the number of sides of the polygon
in H.P. b+c c+a a+b

Q.22 A number consists of three digits in G.P.. The sum


Q.10 Find the sum of n terms of the series of the digits at units and hundreds place exceeds twice
4 7 10 the digit at tens place by 1 and the sum of the digits at
1+ + + + ...... . tens and hundreds place is two third of the sum of the
5 52 53
digits at tens and units place. Find the number.
Q.11 Let a, b, c, d, e be five real numbers such that a, b,
c are in A.P.; b, c, d are in G.P.; c, d, e are in H.P. If a = 2 Q.23 25 trees are planted in a straight line at intervals
and e = 18, find all possible values of b, c and d. of 5 meters. To water them the gardener must bring
water for each tree separately from a well 10 meters
from the first tree in line with the trees. How far he will
Q.12 Find the sum of first n terms of the series:
have to cover in order to water all the tree beginning
1⋅2⋅3 + 2⋅3⋅4 + 3⋅4⋅5 + ……
with the first if he starts from the well.

Q.13 Find the sum of first 2n terms of the series: Q.24 Natural numbers have been grouped in the
12+ 2 + 32 + 4 + 52 + 6 + …. following way 1 ; (2, 3) ; (4, 5, 6); (7, 8, 9, 10) ; ……
Show that the sum of the numbers in the nth group is
Q.14 The H.M of two numbers is 4 and their A.M. (A)
n(n2 + 1)
and G.M. (G) satisfy the relation 2A + G2 = 27. Find the .
numbers. 2

Q.25 In three series of GP’s, the corresponding


Q.15 Find the sum of first 10 terms of the series: numbers in G.P. are subtracted and the difference of
(33 – 23) + (53 – 43) + (73 – 63) + ….. the numbers are also found to be in G.P. Prove that the
three sequences have the same common ratio.
Q.16 Find the sum of first 20 terms of the series: 1⋅32 +
Q.26 If a1, a2, a3 ,…. Are in A.P such that
2⋅52 + 3⋅72 + ….
ai ≠ 0, show that
1 1 1 n
Q.17 Find three numbers a, b, c between 2 and 18 such S= + + ..... + =
a1a2 a2a3 anan+1 a1an+1
that:
(i) Their sum is 25. Also evaluate lim S .
a→∞
3 . 4 0 | Sequences and Series

Q.27 If 9 arithmetic means and 9 harmonic means be 1 1 1


1 1 1
6 (A) , (B) 2, (C) , (D) ,2
inserted between 2 and 3, prove that A+ = 5 , where 2 4 3 2 3 2
H
A is any arithmetic mean and H the corresponding Q.6 If x ∈ R, the numbers (51+x + 51 – x), a/2 (25x + 25–x)
harmonic mean. form an A.P. then ‘a’ must lie in the interval
(A) [1, 5] (B) [2, 5] (C) [5, 12] (D) [12, ∞]
Q.28 If x + y + z = 1 and x, y, z are positive numbers,
show that (1 – x) (1 – y) (1 – z) ≥ 8 xyz.
Q.7 If the sum of the first 11 terms of an arithmetical
progression equals that of the first 19 terms, then the
Q.29 Show that any positive integral power (greater than sum of its first 30 terms, is
1) of a positive integer m, is the sum of m consecutive
odd positive integers. Find the first odd integer for (A) Equal to 0 (B) Equal to – 1
mr(r > 1). (C) Equal to 1 (D) Non unique

Q.8 Let s1, s2, s3 ….. and t1,t2,t3 …. are two arithmetic
Exercise 2 10 15
sequence such that s1 = t1 ≠ 0; s2 = 2t2 and ∑ si = ∑ ti .
=i 1 =i 1
Single Correct Choice Type s2 – s1
Then the value of is
t2 – t1
Q.1 If a, b, c are distinct positive real in H.P., then the
(A) 8/3 (B) 3/2 (C) 19/8 (D) 2
b+a b+c
value of the expression, + is equal to
b–a b–c
Q.9 Let an, n ∈ I be the nth term an A.P. with common
(A) 1 (B) 2 (C) 3 (D) 4 difference ‘d’ and all whose terms are non-zero. If n
approaches infinity, then the sum
Q.2 The sum of infinity of the series 1 1 1
+ + ..... + will approach
1 1 1 a1a2 a2a3 anan+1
+ + + ........ is equal to
1 1+2 1+2+3
1 2 1
(A) (B) (C) (D) a1d
(A) 2 (B) 5/2 (C) 3 (D) None a1d a1d 2a1d

Q.3 Along a road lies an odd number of stones placed Q.10 The sum of the first three terms of an increasing
at intervals of 10 m. These stones have to be assembled G.P. is 21 and the sum of their squares is 189. Then the
around the middle stone. A person can carry only one sum of its first n term is
stone at a time. A man carried out the job starting with
 1 
the stone in the middle, carrying stones in succession, (A) 3(2n – 1) (B) 12  1 – 
thereby covering a distance of 4.8 km. Then the number  2n 
of stones is  1 
(C) 6  1 –  (D) 6(2n – 1)
(A) 15 (B) 29 (C) 31 (D) 35  2n 


 n 
Q.4 If S = 12 +32 +52 + …. + (99)2 then the value of the Q.11 The sum ∑  n4 + 4  is equal to
sum 22 + 42 +62 +….. + (100)2 is n=1  

(A) S + 2550 (B) 2S (C) 4S (D) S +5050 (A) 1/4 (B) 1/3 (C) 3/8 (D) 1/2

Q.5 In an A.P. with first term and the common difference Q.12 If a ≠ 1 and (ln a2) + (ln a2)2 + (ln a2)3 + ….. = 3 [lna
d (a, d ≠ 0), the ratio ‘S’ of the sum of the first n terms to + (ln a)2 + (ln a)3 + (ln a)4 + ……], then ‘a’ is equal to
sum of n terms succeeding them does not depend on
n. Then the ratio "a/d" and the ratio ‘ρ’, respectively are (A) e1/5 (B) e1/2 (C) 3e1/2 (D) e1/4
M a them a ti cs | 3.41

Previous Years’ Questions Q.8 If the sum of first n terms of an AP is cn2, then the
sum of squares of these n terms is  (2009)
Q.1 If a, b, c d and p are distinct real numbers such that n(4n2 – 1)c2 n(4n2 + 1)c2
(A) (B)
(a2 + b2 + c2)p2 – 2(ab + bc + cd)p + (b2 + c2 + d2) ≤ 0, 6 3
then a, b, c, d (1987) n(4n2 – 1)c2 n(4n2 + 1)c2
(C) (D)
(A) Are in A.P. (B) Are in G.P. 3 6

(C) Are in H.P. (D) Satisfy ab = cd


Q.9 The first two terms of a geometric progression add
up to 12. The sum of the third and the fourth terms
Q.2 Sum of the first n terms of the series is 48. If the terms of the geometric progression are
1 3 7 15 alternately positive and negative, then the first term is
+ + + + .... is equal to
2 4 8 16  (2008)

(A) 2n – n – 1 (B) 1 – 2–n (A) –4 (B) –12 (C) 12 (D) 4

(C) n + 2–n – 1 (D) 2n + 1


Q.10 The sum to the infinity of the series is
2 6 10 14 (2009)
Q.3 If x > 1, y > 1, z > 1 are in G.P. then 1+ + + + + ....... 
3 32 33 34
1 1 1
, , are in  (1998) (A) 2 (B) 3 (C) 4 (D) 6
1 + lnx 1 + ln y 1 + lnz

(A) AP (B) H.P. (C) G.P. (D) None Q.11 If m is the A. M. of two distinct real numbers l
and n(l, n > 1) and G1, G2 and G3 are three geometric
means between l and n, then G14 + 2G24 + G34 equals:
Q.4 If a, b, c, d are positive real number such that
 (2015)
a + b + c + d = 2, then M = (a + b) (c +d) satisfies the
relation  (2000) (A) 4 l2 mm (B) 4 lm2 n (C) 4 lmn2 (D) 4 l2m2n2
(A) 0 < M ≤ 1 (B) 1 ≤ M ≤ 2
Q.12 The sum of first 9 terms of the series is
(C) 2 ≤ M ≤ 3 (D) 3 ≤ M ≤ 4
13 13 + 23 13 + 23 + 33
+ + + .......  (2015)
Q.5 Let the positive numbers a, b, c, d be in A.P. then 1 1+ 3 1+ 3 + 5
abc, abd, acd, bcd are  (2001) (A) 71 (B) 96 (C) 142 (D) 192
(A) not in AP/GP/HP (B) in AP
Q.13 If the 2nd, 5th and 9th terms of a non-constant
(C) in GP (D) in HP
A.P. are in G.P., then the common ratio of this G.P. is:
 (2016)
Q.6 Suppose a, b, c are in AP and a2, b2, c2 are in G.P. 4 7 8
3 (A) (B) 1 (C) (D)
If a < b < c and a + b + c = , then the value of a is 3 3 5
 2 (2002)
1 1 Q.14 If the sum of the first ten terms of the series is 16 m
(A) (B) 5
2 2 2 3 2
 3  2   1
2 2 2
2  4
then m is equal to 1  +  2  +  3  + 4 +  4  + .......
1 1 1 1  5  5  5  5
(C) – (D) –  (2016)
2 3 2 2
(A) 101 (B) 100 (C) 99 (D) 102
Q.7 An infinite G.P. has first term x and sum 5, then x
belongs to  (2004) Q.15 Three positive numbers form an increasing G.P. If
the middle term in this G.P. is doubled, the new numbers
(A) x < – 10 (B) – 10 < x < 0
are in A.P. Then the common ratio of the G.P. is (2014)
(C) 0 < x < 10 (D) x > 10
(A) 2 − 3 (B) 2 + 3 (C) 2 + 3 (D) 3 + 2
3 . 4 2 | Sequences and Series

Assertion Reasoning Type Q.19 The real number k for which the equation, 2x3 +
3x + k = 0 has two distinct real roots in [0, 1]  (2013)
Q.16 Statement-I: The sum of the series 1 + (1 + 2 + 4) + (A) Lies between 1 and 2
(4 + 6 + 9) + (9 + 12 + 16) + ...... + (361 + 380 + 400) is 8000.
n (B) Lies between 2 and 3
Statement-II: ∑ (k3 − (k − 1)3 ) =
n3 for any natural
(C) Lies between -1 and 0
number n.  k =1 (2012)
(A) Statement-I is false, statement-II is true (D) Does not exist.

(B) Statement-I is true, statement-II is true; statement-II


is a correct explanation for statement-I Q.20 If the equations x2 + 2x + 3 = 0 and ax2 + bx +
c = 0, a,b,c ∈ R, have a common root, then a : b : c is
(C) Statement-I is true, statement-II is true; statement-II  (2013)
is not a correct explanation for statement-I (A) 1 : 2 : 3 (B) 3 : 2 : 1 (C) 1 : 3 : 2 (D) 3 : 1 : 2
(D) Statement-I is true, statement-II is false 
Q.21 The sum of first 20 terms of the sequence 0.7,
Q.17 Statement-I: The sum of the series 1 + (1 + 2 + 4) + 0.77, 0.777,....., is  (2013)
(4 + 6 + 9) + (9 + 12 + 16) + ...... + (361 + 380 + 400) is 8000. 7 7
n (A) (179 − 10−20 ) (B) (99 − 10−20 )
81 9
Statement-II: ∑ (k3 − (k − 1)3 ) =
n3 for any natural
number n.  k =1 7 7
(C) (179 + 10−20 ) (D) (99 + 10−20 )
81 9
Q.18 If 100 times the 100th term of an AP with non zero
Q.22 If x, y, z are in A.P. and tan-1x, tan-1y and tan-1z
common difference equals the 50 times its 50th term,
are also in A.P., then  (2013)
then the 150th term of this AP is  (2012)
(A) x = y = z (B) 2x = 3y = 6z
(A) –150 (B) 150 times its 50th term
(C) 6x = 3y = 2z (D) 6x = 4y = 3z
(C) 150 (D) Zero

JEE Advanced/Boards

Exercise 1 (i) Prove that a, c, e are in GP


(ii) Prove that e = (2b – a)2/a
Q.1 (i) The harmonic mean of two numbers is 4. The
arithmetic mean A & the geometric mean G satisfy the (iii) If a = 2 & e = 18, find all possible values of b, c, d.
relation 2A + G2 = 27. Find the two numbers.
Q.5 Let a1 and a2 be two real values of α for which
(ii) The A.M. of two numbers exceeds their G.M. by 15 the numbers 2a2, α4, 24 taken in that order form an
and HM by 27. Find the numbers. arithmetic progression. If b1 and b2 are two real values
of β for which the numbers 1, b2, 6 – b2 taken in that
Q.2 If the 10th term of an H.P. is 21 and 21st term of the order form a geometric progression, then find the value
same H.P. is 10, then find the 210th term. of (α12 + α22 + β12 + β22 ) .

Q.3 If sinx, sin22x and cosx.sin4x form an increasing Q.6 Two distinct, real infinite geometric series each
geometric sequence, then find the numerical value have a sum of 1 and have the same second term. The
of cos2x. Also find the common ratio of geometric third term of one of the series is 18. If the second term
sequence. m–n
of both the series can be written in the form ,
p
Q.4 If a, b, c, d, e be 5 numbers such that a, b, c are in where m, n and p are positive integers, and m is not
AP; b, c, d are in G.P. & c, d, e are in H.P. then, divisible by the square of any prime, find the value of
100m + 10n+p.
M a them a ti cs | 3.43

99
5100 a– b
Q.7 Let S = ∑ (25)n + 5100 . Find [s]. where Tn is the nth term of series then T22 =
4
(a,
n=1 b ∈ N). Find the value of (a + b).
Where [y] denotes largest integer less than or equal to y.
Q.17 Given a three digit number whose digits are
Q.8 Given that the cubic ax – ax + 9bx – b = 0 (a ≠ 0)
3 2 three successive terms of a G.P. If we subtract 792
has all three positive roots. Find the harmonic mean of form it, we get a number written by the same digits
the roots independent of a and b, hence deduce that in the reverse order. Now if we subtract four from the
the root are all equal. Find also the minimum value of hundred’s digit of the initial number and leave the
(a + b), if a and b ∈ N. other digits unchanged, we get a number whose digits
are successive terms of an A.P. Find the number.
Q.9 A computer solved several problems in succession.
π
The time it took the computer to solve each successive Q.18 For 0 < θ < , let S(θ) = 1 + (1 + sinθ) cos θ +
4
problem was the same number of times smaller than
(1 + sinθ + sin2θ) cos2θ + ….. ∞.
the time it took to solve the preceding problem. How
many problems were suggested to the computer if it  2 –1  π
Then find the value of  S  .
spent 63.5 min to solve all the problems except for the  2   4 

first, 127 min to solve all the problems except for the
last one, an 31.5 min to solve all the problems except  π  π  x 
for the first two? Q.19 If tan  – x  ,tan ,tan  + x  in order are
 12  12  12 
three consecutive terms of a G.P., then sum of all the
Q.10 The sequence a1, a2, a3, …… a98 satisfies the relation
solutions in [0, 314] is kπ. Find the value of k.
an+1 = an +1 for 1,2,3,…. 97 and has the sum equal to
49
4949. Evaluate ∑ a2k . Exercise 2
k =1

Q.11 Let a and b be positive integers. The value of xyz Single Correct Choice Type
343
is 55 or , according as a, x, y, z, b are in arithmetic
55 Q.1 The arithmetic mean of the nine numbers in the
progression or harmonic progression resp.. Find the give set {9, 99,999,……999999999} is a 9 digit number
value of (a2 +b2). N, all whose digits are distinct. The number N does not
contain the digit
Q.12 If the roots of 10x3 – cx2 – 54x – 27= 0 are in (A) 0 (B) 2 (C) 5 (D) 9
harmonic progression, then find c and all the roots.
360  
1
Q.13 If a, b, c be in G.P. & logca, logbc, logab be in AP,
Q.2 ∑   is the ratio of two relative
k =1  k k + 1 + (k + 1 k ) 
then find the common difference of the AP if logac = 4.
prime positive integers m and n. The value of (m + n)
is equal to
Q.14 The first term of a geometric progression is equal
to b – 2, then third term is b + 6, and the arithmetic (A) 43 (B) 41 (C) 39 (D) 37
mean of the first and third term to the second term is in 100
k
the ratio 5: 3. Find the positive integral value of b. Q.3 The sum ∑ k 4 + k2 + 1 is equal to
k =1

Q.15 In a G.P. the ratio of the sum of the first eleven 4950
(A) (B) 5050
terms to the sum of the last eleven terms is 1/8 and the 10101 10101
ratio of the sum of all the terms without the first nine to
the sum of all the terms with out the last nine is 2. Find 5151
(C) (D) None of these
the number of terms in the GP. 10101

Q.4 A circle of radius r is inscribed in a square. The mid


Q.16 If sum of first n terms of an AP (having positive
point of sides of the square have been connected by
terms) is given by Sn=(1+2Tn) (1 – Tn)
line segment and a new square resulted. The sides of
3 . 4 4 | Sequences and Series

these square were also connected by segments so that Q.9 If the roots of the equation x3 + px2 + qx – 1 = 0
a new square was obtained and so on, then the radius form an increasing GP, where p and q are real, then
of the circle inscribed in the nth square is
(A) p + q = 0
 1–n   3–3n  (B) p ∈ (–3, ∞)
(A) 2 2  r (B) 2 2  r
    (C) One of the roots is unity

 n  5 – 3n  (D) One root is smaller than 1


(C) 2 2  r (D) 2 2  r
   
  Q.10 If the triplets log a, log b, log c and (log a – log
2b), (log 2b – log3c), (log 3c – loga) are in arithmetic
Assertion Reasoning Type progression then

(A) Statement-I is true, statement-II is true and (A) 18 (a + b + c)2 = 18(a2 + b2 +c2) + ab
statment-II is correct explanation for statment-I. (B) a, b, c are in GP
(B) Statement-I is true, statement-II is true and (C) a, 2b, 2c are in HP
statment-II is NOT the correct explanation for statment-I.
(D) a, b, c can be the lengths of the sides of a triangle.
(C) Statement-I is true, statement -II is false.
(Assume all logarithmic terms to be defined)
(D) Statement-I is false, statement-II is true

Q.11 x1, x2 are the roots of the equation


Q.5 Statement-I: If 27 abc ≥ (a + b + c)3 and 3a + 4b + 5c x2 – 3x + A = 0; x3, x4 are roots of the equation x2 – 12x +
1 1 1 B = 0, such that x1, x2, x3, x4 form an increasing G.P. then
= 12 then + + =,10 where a, b, c are positive
2 3
a b c5 (A) A = 2 (B) B = 32
real numbers.
(C) x1 + x3 = 5 (D) x2 + x4 = 10
Statement-II: For positive real numbers A.M. ≥ G.M.

Multiple Correct Choice Type Previous Years’ Question


Q.6 Let a1, a2, a3 .... and b1, b2, b3 .... be arithmetic Q.1 If the first and the (2n – 1)th term of an AP, G.P.
progressions such that a1 = 25, b = 75 and a100 + b100 = and H.P. are equal and their nth terms are a, b, and c
100. Then, respectively, then  (1988)
(A) The difference between successive terms in progression (A) a = b = c (B) a ≥ b ≥ c
‘a’ is opposite of the difference in progression ‘b’.
(C) a + c =b (D) ac – b2 = 0
(B) an + bn = 100, for any n.
(C) (a1 + b1), (a2 + b2), (a3 + b3),... are in AP Q.2 Let S1, S2…. be squares such that for each n ≥ 1 the
100
length of a side of Sn equals the length of a diagonal of
(D) ∑ (ar + br ) =
10000 Sn+1. If the length of a side of S1 is 10 cm, then for which
r =1 of the following values of n is the area of Sn less than
1 sq. cm ? (1999)
Q.7 If sin (x – y), sin x and sin (x + y) are in H.P. then the (A) 7 (B) 8 (C) 9 (D) 10
y
value of sin x.sec =
2 Q.3 Let Sk, k = 1,2,….., 100, denotes the sum of the
(A) 2 (B) 21/2 (C) – 2 (D) –21/2 k –1
infinite geometric series whose first term is
k!
1
Q.8 The sum of the first three terms of the G.P. in which and the common ratio is . Then the value of
k
the difference between the second and the first term is 1002 100 2
6 and the difference between the fourth and the third + ∑ | (k – 3k + 1)Sk | is  (2010)
100! k =1
term 54, is
(A) 39 (B) –10.5 (C) 27 (D) –27
M a them a ti cs | 3.45

Q.4 Let a1, a2, a3,….., a11 be real numbers satisfying Q.9 The sum V1 + V2 + ...... + Vn is

a1 = 15, 27 – 2a2 > 0 and ak = 2ak–1 –a(k–2), for k = 3, 1


(A) n(n + 1)(3n2 – n + 1)
12
a12 + a22 + ..... + a11
2
4,…., 11. If = 90 , then the value of
11 1
a1 + a2 + ....a11 (B) n(n + 1)(3n2 + n + 2)
is equal to (2010) 12
11
1
(C) n(2n2 – n + 1)
Q.5 Le a1, a2, a3,…. a100 be an arithmetic progression with 2
p
1
a1 = 3 and Sp = ∑ ai ,1 ≤ p ≤ 100 . For any integer n (D) (2n3 – 2n + 3)
i=1 3
Sm
with 1 ≤ n ≤ 20, let m = 5n. If does not depend on
Sn Q.10 Tr is always
n, then a2 is ……. (2011)
(A) An odd number (B) An even number
Paragraph 1: Let A1, G1, H1 denote the arithmetic, (C) A prime number (D) A composite number
geometric and harmonic means, respectively, of two
distinct positive numbers. For n > 2, let An–1 and Hn–1 Q.11 Which one of the following is a correct statement?
have arithmetic, geometric and harmonic means as
An, Gn, Hn respectively. (A) Q1, Q2, Q3....... are in A.P. with common difference 5.
(B) Q1, Q2, Q3....... are in A.P. with common difference 6.
Q.6 Which one of the following statements is correct? (C) Q1, Q2, Q3....... are in A.P. with common difference 11.
(A) G1 > G2 > G3 >...... (D) Q1 = Q2 = Q3 =.......
(B) G1 < G2 < G3 < ......
n−1
n
n n
(C) G1 = G2 = G3 =...... Q.12 Let Sn = ∑
2 2
and Tn = ∑ n2 + kn + k 2
k =1 n + kn + k k =0
(D) G1< G3 < G5 <...... and G2 > G4 > G6 >......
for n = 1, 2, 3, …. Then,  (2008)
Q.7 Which one of the following statements is correct?
π π
(A) A1 > A2 > A3 > ...... (A) Sn < (B) Sn >
3 3 3 3
(B) A1 < A2 < A3 < ...... π π
(C) Tn < (D) Tn >
(C) A1 > A3 > A5 >...... and A2 < A4 < A6 < ...... 3 3 3 3
(D) A1 < A3 < A5 <...... and A2 > A4 > A6 >......
Q.13 Suppose four distinct positive numbers a1, a2, a3,
Q.8 Which one of the following statements is correct? a4 are in G.P. Let b1 = a1, b2 = b1 + a2, b3 = b2 + a3 and
(A) H1 > H2 > H3 > ...... b4 = b3 + a4.  (2008)

(B) H1 < H2 < H3 < ...... Statement-I: The numbers b1, b2, b3, b4 are neither in
A.P. Nor in G.P.
(C) H1 > H3 > H5 >...... and H2 < H4 < H6 <......
Statement-II: The numbers b1, b2, b3, b4 are in H.P.
(D) H1 < H3 < H5 <...... and H2 > H4 > H6 >......
(A) Statement-I is true, statement-II is true; statement-II
is a correct explanation for statement-I
Paragraph 2: Let Vr denote the sum of the first ‘r’ terms
of an arithmetic progression (A.P.), whose first term is ‘r’ (B) Statement-I is true, statement-II is true; statement-II
and the common difference is (2r – 1). Let Tr = Vr + 1 – is not a correct explanation for statement-I.
Vr – 2 and Qr = Tr + 1 – Tr for r = 1, 2, ... (2007) (C) Statement-I is true, statement-II is false
(D) Statement-I is false, statement-II is true
3 . 4 6 | Sequences and Series

Q.14 If the sum of first n terms of an A.P. is cn2, then the Q.18 Let a1, a2, a3, ….. be in harmonic progression with
sum of squares of these n terms is  (2009) a1 = 5 and a20 = 25. The least positive integer n for
2 2 2 2 which an< 0 is (2012)
(A) n(4n − 1)c (B) n(4n + 1)c
6 3 (A) 22 (B) 23 (C) 24 (D) 25

4n k(k +1)
n(4n2 − 1)c2 n(4n2 + 1)c2
(C)
3
(D)
6
Q.19 Let S=
n ∑ ( −1) 2 k2. Then Sn can take value(s)
 k =1 (2013)
Q.15 Let Sk, k = 1, 2,...., 100, denote the sum of the (A) 1056 (B) 1088 (C) 1120 (D) 1332
k −1
infinite geometric series whose first term is b
1 k! Q.20 Let a, b, c be positive integers such that is an
a
and the common ratio is . Then the value of
2 100 k integer. If a, b, c are in geometric progression and the
100
+ ∑ (k 2 − 3k + 1)Sk is  (2010) arithmetic mean of a, b, c is b + 2, then the value of
100! k =1
a2 + a − 14 is ________  (2014)
a +1
Q.16 Let a1, a2, a3, …, a11 be real numbers satisfying
a1 = 15, 27 − 2a2 > 0 and ak = 2ak−1 − ak−2 for k
a12 + a22 + .... + a11
2
Q.21 Suppose that all the terms of an arithmetic
= 3, 4, …,11. If = 90 , then the value of
11 progression (A.P.) are natural numbers. If the ratio
a1 + a2 + .... + a11 of the sum of the first seven terms to the sum of the
is equal to  (2010)
11 first eleven terms is 6 : 11 and the seventh term lies in
between 130 and 140, then the common difference of
Q.17 Let b = 6, with a and c satisfying (E). If α and β are this A.P. is  (2015)
the roots of the quadratic equation ax2 + bx + c = 0, then
n

 1 1 Q.22 Let bi > 1 for i = 1, 2, …, 101. Suppose loge b1 loge
∑  α + β  is  (2011) b2, …, loge b101 are in Arithmetic Progression (A.P.) with
n= 0   the common difference loge 2. Suppose a1, a2, …, a101
6
(A) 6 (B) 7 (C) (D) ∞ are in A.P. such that a1 = b1 and a51 = b51. If t = b1 + b2 +
7
…+ b51 and s = a1 + a2 + … + a51, then  (2016)
(A) s > t and a101 > b101 (B) s > t and a101 < b101
(C) s < t and a101 > b101 (D) s < t and a101 < b101
M a them a ti cs | 3.47

PlancEssential Questions
JEE Main/Boards JEE Advanced/Boards
Exercise 1 Exercise 1
Q.3 Q.11 Q.14 Q.6 Q.9 Q.12
Q.17 Q.21 Q.25 Q.15 Q.17
Q.27

Exercise 2 Exercise 2
Q.2 Q.4 Q.10 Q.1 Q.4 Q.5 Q.12
Q.13
Previous Years’ Questions
Previous Years’ Questions Q.1 Q.3 Q.4
Q.2 Q.5 Q.8 Q.6 Q.7 Q.8

Answer Key

JEE Main/Boards Q.12 1 n(n + 1)(n + 2)(n + 3)


4
Exercise 1 1
Q.13 n(4n2 + 3n + 2)
3
Q.1 n = 6 Q.14 6,3
3 3 3
Q.2 3, − , , − ,.....is the series. Q.15 4960
2 4 8

7n 7   1 n  Q.16 188090
Q.3 (a) – 1–    ; (b) 2 [10n+1 – 9n – 10]
9 81   10   27 Q.17 a = 5, b = 8, c = 12.
 
Q.5 (2, 4, 6) or (6, 4, 2) 1
Q.20 –5050, [(5050)2 – 338350]
2
Q.6 (i) 6633 (ii) 2842 (iii) 945
Q.21 9
Q.7 69: 128
Q.22 469
35 3 (3n− 2)
Q.10 − − Q.23 3370 m
16 16(5 ) 4(5n−1 )
n − 2
1
Q.26
Q.11 c = 6, b = 4, d = 9; b = – 2, c = – 6, d = – 18 a1 (a2 – a1 )
3 . 4 8 | Sequences and Series

Exercise 2
Single Correct Choice Type

Q.1 B Q.2 A Q.3 C Q.4 D Q.5 C Q.6 D


Q.7 A Q.8 C Q.9 A Q.10 A Q.11 C Q.12 D

Previous Years’ Questions


Q.1 B Q.2 C Q.3 B Q.4 A Q.5 D Q.6 D
Q.7 C Q.8 C Q.9 B Q.10 B Q.11 B Q.12 B
Q.13 A Q.14 A Q.15 B Q.16 B Q.17 D Q.18 D
Q.19 A Q.20 C Q.21 A

JEE Advanced/Boards

Exercise 1
5 –1
Q.1 (i) 6, 3 ; (ii) 120, 30 Q.2 1 Q.3 ; 2
2
Q.4 (iii) b = 4, c = 6, d = 9 or b = – 2, c = – 6, d = – 18 Q.5 12 Q.6 518 Q.7 49

Q.8 28 Q.9 8 problems, 127.5 minutes Q.10 2499 Q.11 50

Q.12 C = 9; (3, –3/2, – 3/5) Q.13 13/4 Q.14 3 Q.15 n = 38

Q.16 6 Q.17 931 Q.18 2 Q.19 4950

Exercise 2
Single Correct Choice Type

Q.1 A Q.2 D Q.3 B Q.4 A

Assertion Reasoning Type

Q.5 D

Multiple Correct Choice Type

Q.6 A, B, C, D Q.7 B, C Q.8 A, B Q.9 A, C, D Q.10 B, D Q.11 A, B, C, D

Previous Years’ Questions


Q.1 A, B, D Q.2 B, C, D Q.3 4 Q.4 0 Q.5 3 or 9 Q.6 C
Q.7 A Q.8 B Q.9 B Q.10 D Q.11 B Q.12 A, D
Q.13 C Q.14 C Q.15 3 Q.16 0 Q.17 B Q.18 D
Q.19 A, D Q.20 6 Q.21 9 Q.22 B
M a them a ti cs | 3.49

Solutions

JEE Main/Boards = 7[0.1 + 0.11 + 0.111 + …] = 7 [0.9 + 0.99 + 0.999 …]


9
Exercise 1 7
= [1 – 0.1 + 1 – 0.01 + 1 – 0.001 …]
9

Sol 1: Sum of n terms is 364 = 7 [n – (0.1 + 0.01 + 0.001 …)]


9
a + ar + ar2 ………….. arn – 1 = 364
 0.1(1 – (0.1)n )  7  1 
a(rn − 1) = 7 n – = n – (1 – (0.1)n )
= 364 1 – 0.1 
9
r –1 9   9  

given r = 3, a = 1 7n 7   1 
n
= – 1–  
(3n – 1) 9 81   10  
⇒ = 364 ⇒3n – 1 = 728  
(3 – 1)
(b) 6 + 66 + 666 = 6[1 + 11 + 111 …]
⇒3n = 729
6 2
= [9 + 99 + 999 + ……] = [10 – 1 + 100 – 1 + 1000 – 1 … ]
⇒ n=6 9 3

2  10(10n – 1)  2 10 2n
a =  – n = × (10n – 1) –
Sol 2: Sum of infinite G.P. is 2 ⇒ =2 3  10 – 1  3 9 3
–r + 1
⇒a = – 2(r – 1)
Series is a, ar, ar2 ……. (|r| < 1) =
2
27
(
10n+1 – 10 –)2n × 9
3× 9
=
2  n+1
27 
10 – 9n – 10 

⇒a3, (ar)3, (ar2)3 …………….. …(2)
First term of this infinite series is a3 and ratio is r3 Sol 4: a, b, c are in AP
3
a (i) b + c, c + a, a + b are also in AP
Hence sum of this infinite series is 3
–r + 1
a, b, c are in AP ⇒ 2b = a + c
a3
Given = 24 ⇒b–a=c–b
–r 3 + 1
8(r − 1)3 (r – 1)2 ⇒ a–b =b–c
= 24⇒ =3
( – r + 1)
3 2
(r + r + 1)
Difference between term of given AP = a – b, b – c
which are equal by equation (i)
⇒r2 + 1 – 2r = 3r2 + 3r + 3
Hence b + c, c + a, a + b is an AP
⇒2r2 + 5r + 2 = 0 ⇒2r2 + 4r + r + 2 = 0
1 1 1
⇒(2r + 1) (r + 2) = 0 (ii) , , are also in AP
bc ac ab
1 (b – a) (c – b)
⇒ r = – 2, r = – Common difference = ,
2 cab abc
1 By equation (i) b – a = c – b ie difference between terms
|r| < 1 ⇒ r = – ⇒ a = +3
2 is same
3 3 3
Series is 3, − , , − ,..... Hence the given series is in AP
2 4 8
(iii) a2(b + c), b2(c + a), c2(a + b)
Sol 3: (a) Sum upto n terms Difference = b 2
c + b2a – a2b – a2c , c
2
a + c2b – b2c – b2a

Sn = 0.7 + 0.77 + 0.777 ……….… n terms d1 d2
3 . 5 0 | Sequences and Series

d1 = c(b2 – a2) + ab(b – a) = (ca + ab + cb) (b – a) (iii) Multiple of 3 and 7


= (ca + ab + bc) (c – b) [from eq.(i) 21, 42, 63 … 189 ⇒ n = 9
d2 = a(c2 – b2) + bc (c – b) = (ac + ab + bc) (c – b) 9 9
Sum = [21 + 189] = . 210 = 945
d1 = d2 2 2

Hence given series is an AP Sol 7: Sum of first n terms of 2 AP’s are in ratio
1 1 1 1 1 1  3n – 3
(iv) a  +  , b  +  , c  +  =
b c c a a b 5n + 21

1 b a 1 c b ⇒ Let the AP be a1, a1 + d1 …


⇒ d1 = (b – a) + – ⇒ d2 = (c – b) + –
c a b a b c
2nd AP be a2 , a2 + d2 …
b – a (b – a)(b + a)
d1 = + n
c ab [2a + (n – 1)d1 ]
2 1 3n – 3
=
c – b (c – b)(c + b) 1 1 1  n 5n + 21
d2 = + = (b – a)  + +  [2a + (n – 1)d1 ]
a bc c a b 2 2
1 1 1  1 1 1  (n – 1)
= (b – a)  + +  = (c – b)  + +  a1 + d1
 c a b  c a b 2 3n – 3
⇒ = .....(i)
(n – 1)d2 5n + 21 
From eq (i) ⇒ d1 = d2
n a2 +
2
Hence given series is also an AP a + 23d1
Ratio of 24th term well be 1
a2 + 23d2
Sol 5: Sum of first 3 numbers in AP is 12
Putting n = 47 in equation (i), we well get desired ratio
Let a – r, a, a + r be the first 3 numbers
a1 + 23d1 3(47) – 3 138 69
3a = 12 ⇒ a = 4 = = =
a2 + 23d2 5(47) + 21 256 128
⇒ (a – r)3 + a3 + (a + r)3 = 288
⇒ 2a3 + 6ar2 + a3 = 288 ⇒ 3a3 + 6ar2 = 288 Sol 8: given

⇒ 6 × 4 × r2 = 288 – 3(43) ⇒ 24r2 = 96 a + (p – 1)d = x  …(i)

⇒r=±2 a + (q – 1)d = y …(ii)

So numbers are, 4 – 2, 4, 4 + 2 = (2, 4, 6) (for r = 2) sum of first (p + q) terms

(4 + 2, 4, 4 – 2) for (r = – 2) p+q
= [2a + (p + q – 1)d] …(iii)
2 
⇒ (6, 4, 2)
Subtracting (ii) from (i)

Sol 6: (i) Sum of integers between 1 & 200 which are (p – q)d = x – y
multiple of 3 x–y
⇒d= and putting this value in equation (i)
3, 6, 9, … 198 ⇒ n = 66 p–q
n (p – 1)(x – y)
Hence sum = [a + l] a+ =x
2 p–q
(66)
= [3 + 198] = 33 [201] = 6633 (px – py – x + y) –qx + py + x – y
2 ⇒a= x– =
p–q p–q
(ii) Multiple of 7
7, 14, 21 … 196 ⇒ n = 28 Putting values of a and d in equation (iii)

n 28 p + q  2x – 2y – 2qx + 2py + (p + q – 1)(x – y) 


Now sum = [a + l] = [7 + 196] = 14[203] = 2842 Sp+q =  
2 2 2  p–q 
M a them a ti cs | 3.51

p + q  x – y – qx + py + px – qy  4sn  3n – 2  15 (5n–1 – 1)
=   =1–  n  +
2  p–q  5  5  4 5n

p+q (p – q) (x – y)  p + q  x – y 5 (3n – 2) 75  1 1 
= (x + y) + = x + y +  Sn = – +  – 
2  (p – q) p – q  2  p – q 4 4.5n–1 16  5 5n 

5× 4 15 3 –(3n – 2)
Sol 9: a, b, c are in HP = + - n–2
4×4 16 16 × 5 4 × 5n–1
2 1 1 2ac
i.e = + ⇒b= …(i)
b a c a+c  35 3 –(3n – 2)
= –
2a1c1 16 16(5n–2 ) 4(5n–1 )
[where a1 & c1 are 1st & 3rd terms of given series]
a1 + c1
2ac Sol 11: a, b, c are in AP
(a + b)(b + c) 2ac 2ac ⇒ 2b = a + c  …(i)
= = =
a c a2
+ ab + c 2
+ bc a2
+ c 2
+ b(a + c)
+ b, c, d in GP
b+c a+b
c2 = bd …(ii)
2ac
= (from equation (i) c, d, e are in HP
a2 + c2 + (2ac)
2ce
d= …(ii)
2ac b c + e 
= = = b1
(a + c)(a + c) a+c
Given that a = 2, e = 18
2a1c1
Middle term of given series, hence = b1 ie given (2b – 2)2
a1 + c1 We have (2b – 2) = c from (i) and = d from (ii)
Series is H.P b

(2b – 2)2 2 × (2b – 2)18


4 7 10 and also = from (iii)
Sol 10: 1 + + + b (2b – 2) + 18
5 52 53
36b
Sum of first n terms ⇒ (2b – 2) = ⇒ (b – 1) (b + 8) = 9b
2b + 16
1
4 10 1 + (n – 1)3
7
Sn = ++ + …  …(i)
50 5 53 5n–1 ⇒ b2 + 7b – 8 = 9b ⇒ b2 – 2b – 8=0
52
Sn 1 4 7 10 1 + (n – 1)3 ⇒ b = 4, – 2
= + + + …  …(ii)
5 5 5.5 5.52
5.53
5n ⇒ c = 6, – 6
Subtracting (ii) from (i) ⇒ d = 9, – 18
b, c, d = [4, 6, 9] and [–2, –6, – 18]
3 3 3  3n – 2 
=1+ + + …–  
5 52
53
 5n 
Sol 12: Sn = 1.2.3 + 2.3.4 + 3.4.5 …
(3n – 2) 1 1 1  Tn = n(n + 1) (n + 2) = n (n2 + 3n + 2)
=1– + 3  + ...
n 5 2 n–2 
5  5 5  = n3 + 3n2 + 2n = n3 + 3n2 + 2n
 1  Sn = STn = Sn3 + 3Sn2 + 2Sn
 1 − n–1  
(3n – 2) 1 5 
+ 3. 
2
=1–  n(n + 1)  (n)(n + 1)(2n + 1) n(n + 1)
5 n 5  1  =  +3 +2
1 –    2  6 2
 5 

n(n + 1)  n(n + 1) 3(2n + 1) 


 3n – 2  3 (5n–1 – 1) × 5 =  + + 2
=1–   + 2  2 3 
 5n  5 5n–1 × 4
3 . 5 2 | Sequences and Series

n(n + 1)  n2 + n + 4n + 2 + 4  – 23(1 + 23 + 33 … 103) + 1 – 1


=  
2  2  = 13 + 23 + 33 + 43 … 213 – 24 (1 + 23 … 103) – 1
2 2
n(n + 1) 2 n(n + 1)(n + 2)(n + 3)  21 × (21 + 1)  4  (10)(10 + 1) 
= (n + 5n + 6) = =   – 2   –1
4 4  2   2 
= (21 . 11)2 – 16(5 . 11)2 – 1
Sol 13: Sn = 12 + 2 + 32 + 4 + 52 + 6 = 112(21 . 21 – 16 . 25) – 1
(first 2n numbers) = 121 × 41 – 1 = 4961 – 1 = 4960
1 + 3 + 5 … n terms + 2 + 4 + 6… n terms
2 2 2

= 2(1 + 2 + 3 … + n) + 12 + 32 + 52 … Sol 16: Sn = 1.32 + 2.52 + 3.72 + …

(n + 1) Tn = n(2n + 1)2 = 4n3 + n + 4n2


= 2n + 12 + 22 + 32 … (2n – 1)2 – 22
2 Sn = STn = 4Sn3 + Sn + 4Sn2
– 42 – 62 … (2n – 2)2 2
 n(n + 1)  n(n + 1) n(n + 1)(2n + 1)
(2n – 1)(2n – 1 + 1)(4n – 2 + 1) =4  + + 4
= n(n + 1) +  2  2 6
6
– 22{12 + 22 + … (n – 1)2}
n(n + 1) 2
= n2(n + 1)2 + + n(n + 1) (2n +1)
2 3
(2n – 1)n(4n – 1)
= n(n + 1) +
3  1 4n 2 
= n(n + 1) n2 + n + + + 
(n – 1)n(2n – 1)  2 3 3
– 22
6
 7n 7 
(2n – 1)(n)(4n – 1) = n(n + 1) n2 + + 
= n(n +1) +  3 6
3
22 (n – 1)n(2n – 1)  7 7
– S20 = 20 × 21 202 + × 20 + 
6  3 6
n
=n(n+1)+ [(2n–1)(4n–1)–2(n – 1)(2n – 1)] 420
3 = 2400 + 280 + 7  = 70(2687) = 188090
6 
n
= n(n + 1) + (2n –1) [2n + 1]
3

= n(n +1) +
( 4n – 1) n = n [4n +3n + 2]
2
2
Sol 17: a, b, c ∈ (2, 18)
a + b + c = 25 …(i)
3 3
2a = 2 + b  …(ii)
2ab c2 = 18 b  …(iii)
Sol 14: HM of 2 numbers is 4 ie =4
a+b
b = 2a – 2
ab
= 2a + 2b …(i)
 c = 25 – a – 2a + 2 = 27 – 3a
a+b ⇒ (27 – 3a)2 = 18(2a – 2)
AM = and G.M. = ab
2
⇒ (9 – a)2 = 4(a – 1) ⇒ a2 + 81 – 18a = 4a – 4
We have 2A + G2 = 27 ⇒ a + b + ab = 27
⇒ a2 – 22a + 85 = 0
a + b + 2a + 2b = 27 [from (i)]
a = 17, 5
⇒ a + b = 9 and ab = 18
b = 32, 8
[a = 6, b = 3]; [a = 3, b = 6]
c = 24, 12
Sol 15: (33 – 23) + (53 – 43) + (73 – 63) + … Numbers are (5, 8, 12)

= 33 + 53 + 73 + … + 213
M a them a ti cs | 3.53

Sol 18: a > 0, b > 0, c > 0 Sol 21: Given that,for a polygon of "n" sides, we have
1 1 1 α = 120º; d = 5
To prove (a + b + c)  + +  ≥ 9
a b c Sum of interior angle
We know that A.M. ≥ G.M. ≥ HM (n – 2) 180º = a + (n – 1)d
Therefore, A.M. ≥ HM n n
= 2(120) + (n – 1)5 = 5n + 235
For 3 numbers a, b, c 2 2

3 5n2 + 235n = 360n – 720


a+b+c
AM = , HM =
3 1 1 1 ⇒ 5n2 – 125n + 720 = 0
+ +
a b c ⇒ n2 – 25n + 144 = 0 ⇒ n = 16, 9
a+b+c 3
⇒ ≥ If n = 16, then interior angle will be greater than 180º.
3 1 1 1
+ + Hence the answer is 9.
a b c

1 1 1 Sol 22: Let the number be a, b, c


⇒ (a + b + c)  + +  ≥ 9
a b c
2
b2 = ac, a + c = 2b + 1, b + a = (b + c)
3
Sol 19: Let the 2 number be a, b
⇒ a = 2b + 1 – c
a, A1, A2, b are in AP, (a, G1, G2, b) are in GP, 2
⇒ 3b + 1 – c = (b + c)
(a, H1, H2, b) are in HP 3
A1 + A2 = a + b  …(i) ⇒ 2b + 2c = 9b + 3 – 3c

G1 G2 = ab  …(ii) ⇒ 7b = 5c – 3

By properties of respective series c=


7b + 3
⇒ a = 2b + 1 –
( 7b + 3)
5 5
H1H2 ab
= 3b + 2 (3b + 2)(7b + 3)
H1 + H2 a+b a= ⇒ b2 =
5 25
⇒ 25b2 = 21b2 + 23b + 6 ⇒ 4b2 – 23b – 6 = 0
H1H2 G1G2
=
H1 + H2 A1 + A2 ⇒ 4b2 – 24b + b – 6 = 0
⇒ b = 6, c = 9
G1G2 A1 + A2
= Hence proved ⇒ a = 4 Number is 469
H1H2 H1 + H2

Sol 20: (x –1) (x – 2) (x – 3) … (x – 100) 1 2 3 24 25


Sol 23:
–b 10 m
Coefficient of x99 = 5m 5m
a
We can see that a = 1 Sn = 10 + 10 + 15 + 15 + 20 + 20
100 × 101
and b = 1 + 2 + 3 … 100 = = 5050 … 85 + 85 + … + 125 + 125 + 130
2
∴ Coefficient of x99 = −5050 = 2[10 + 15 + 20 … 130] – 130

Coefficient of x98 = 10[2 + 3 + 4 … 26] – 130


= 1 × 2 + 1 × 3 + .... + 1 × 100 + 2 × 3 + 2 × 4
 26 × 27 
= 1 × 2 + 1 × 3 + .... + 1 × 100 + 2 × 3 + 2 × 4 +.... + 99 × 100 = 10  – 1 – 130 = 10 × 350 – 130
+.... + 99 × 100  2 
=
1
2 { (
(1 + 2 + .... + 100 ) − 1 + 2 + 100
2 2 2
)}2 = 3500 – 130 = 3370 m

2{
1
(5050 ) − 338350}
2
=
3 . 5 4 | Sequences and Series

Sol 24: Number of elements in nth group = n 1 1 1 1


, … , in AP
n(n – 1) + 2 2 HM1 HM9 3
First number in the group will be 1
2 1 –1
⇒ + 10d = ⇒d=
n 2 3 60
Sn = [n(n – 1) + 2 + (n – 1)1]
2 60
n 2 n HMn =
= [n – n + 2 + n – 1] = [n2 + 1] 30 – n
2 2
6 n 6
A+ =2+ + (30 – n)
H 10 60
Sol 25: Let the 3 number in G.P. be a, ar, ar2 & other 3
numbers be a1, a1r1, a1r12 n n
=2+ +3– =5
10 10
(a1r1 – ar)2 = (a1r12 – ar2) (a1 – a)
Hence proved
a12r12 + a2r2 – 2aa1rr1 = a12r12 – aa1r2 – aa1r2 + a2r2
2rr1 = r2+r12
Sol 28: x + y + z = 1
⇒ (r1 – r)2 = 0
For x, y, z. A.M. ≥ HM
⇒ r = r1
a1r1 – ar (a1 – a)r
Ratio for third G.P. = = =r x+y+z 3
a1 – a (a1 – a) ⇒ ≥
3 1 1 1
+ +
Hence ratio of the three G.P. is same x y z

1 1 1 1 1 1
Sol 26: S = + … ⇒ + + ≥9
a1a2 a2a3 anan+1 x y z

1  a2 – a1 a3 – a2  Therefore, xy + yz + zx – 9xyz ≥ 0
=  + ...  Hence proved.
a2 – a1  a1a2 a2a3 

As a2 – a1 = a3 – a2 = … an+1 – an Sol 29: We must prove that for some m and p;


1 1 1 1 1 1 1  m
=  – + – ... + –  Mp = [2a + (m –1)2], for some odd a
a2 – a1  a1 a2 a2 a3 an an+1  2
= m[a + m – 1]
1 1 1  an+1 – a1
=  –  = Let us prove this by induction
a2 – a1  a1 an+1  (a2 – a1 )a1an+1
Taking, P = 2
nd n m2 = m[a+m –1] ⇒ a = 1 is the required a.
= =
d(a1an+1 ) a1an+1 mp+1 = mp.m
= m[a + m – 1] m = m[ma + m2 – m]
n n
S= = = m[ma + m2 – 2m + 1 + m – 1]
a1an+1 a1 (a1 + nd)
= m [ma + (m – 1)2 + m – 1]
n 1 1
=lim S lim
= l= im = m [A + m – 1]
n→∞ a (a + nd) a
 a  1d
n→∞ n→∞
1 1 a1  d+ 1 
 n We must prove that A is odd.
A is odd
Sol 27: 2, AM1, AM2 … AM9, 3
For even m, ma is even and (m – 1)2 is odd ⇒ A is odd
2, HM1, HM2 … HM9, 3 for odd m, ma is odd and (m – 1)2 is even ⇒ A is odd
Suppose we take AMn and HMn ∴ By induction hypothesis,
1
⇒ 2 + 10d = 3 ⇒ d = m
10 Mp = [2a + (m –1)2], with odd a.
2
n
AMn = 2 + nd = 2 + Hence proved
10
M a them a ti cs | 3.55

Exercise 2 n
S2 = [2a + 2nd + (n – 1)d]
2
Single Correct Choice Type s1 2a + (n – 1)d
=
s2 2a + 2nd + (n – 1)d
2ac
Sol 1: (B) b = [a, b, c are in HP] …(i) d
a+c 2 + (n – 1)
⇒ a =s
2ac 2ac d
+a +c 2 + (3n – 1)
b+a b+c a+c
= + = + +c
a a
b–a b–c 2ac 2ac
–c –c d d
a+c a+c ⇒ 2 + (n – 1) = 2s + (3n – 1) s
a a
3ac + a2 3ac + c2 3c + a (3a + c) d
= + = + ⇒ 2s – 2 = [n – 3ns + s – 1]
ac – a 2
ac – c 2 c–a a–c a
a
3c + a (3a + c) 2c – 2a (2s – 2) = n (1 – 3s) + s – 1
= – = =2 d
c–a c–a c–a
This is independent of n ie coefficient of n will be zero
1 1 1 1 a 1
Sol 2: (A) Given summation is, + + ⇒s= ; ⇒ =
1 1+2 1+2+3 3 d 2
2 1 1 
… 
Tn = ⇒ Tn = 2  –  Sol 6: (D) a = 51+x + 51 –x + 25x + 25–x
n(n + 1) n n + 1 
5
1 1 1 1 1 1  ⇒ a = 5.5x + + (5x)2 + (5–x)2
Sn = STn = 2  – + – + ...  = 2 5x
1 2 2 3 3 ∞  5 1
Let 5x = t ⇒ a = 5t + + t2 + 2
t t
5 2 1
5t + t + 2
Sol 3: (C) t t ≥ 1 hence a ≥ 10 + 2
≥ 5;
• • • • • • • 2 2

←10m→ ←middle→ n stones ∴ a ≥ 12

n stones
Sol 7: (A) S11 = S19
⇒ 2[20 + 40 + 60 …n terms] = 4800
11 19
[2a + 10d] = [2a + 18d]
⇒ 120 = 1 + 2 … n terms 2 2
n(n + 1) a –29
⇒ 120 = ⇒ n2 + n = 240 16a = – 232 d ⇒ =
2 d 2
⇒ n = 15 30  29d 
S30 = [2a + 29d] = 30 a + =0
Total no of stone = 2n+1 = 31 [C] 2  2 

Sol 4: (D) S = 12 + 32 + 52 … 992 Sol 8: (C) S2 = 2t2


S1 = 22 + 42 + 62 … (100)2 S1 + ds = 2(t1 + dt)
S1 – S = 2 – 1 + 4 – 3 + 6 – 5 …
2 2 2 2 2 2
ds = t1 + 2dt
= 2+1+4+3+6+5+ … +100+99 10 15
⇒ [2s1 + 9ds] = [2t1 + 14dt ]
100 2 2
= [1 + 100] = 50 [101] = 5050
2 ⇒ 18ds = 2t1 + 42dt ⇒ 18ds = 2ds – 4dt + 42dt

Sol 5: (C) a, a + d, … a + (n – 1)d, a + nd, … a + (2n – 1) d ⇒ 16ds = 38dt


n ds 19
S1 = [2a + (n – 1)d] =
2 dt 8
3 . 5 6 | Sequences and Series

Sol 9: (A) The given expression is equal to Previous Years’ Questions


1 1 1 1 1 1
( − + − + ...) =
d a1 a2 a2 a3 a1d Sol 1: (B) Here, (a2 + b2 + c2) p2 – 2(ab + bc + cd)p +
(b2 + c2 +d2) ≤ 0

Sol 10: (A) a + ar + ar2 = 21 …(i) ⇒ (a2p2 – 2abp + b2) + (b2p2 – 2bcp + c2) +
(c2p2 – 2cdp + d2) ≤ 0
a2 + a2r2 + a2r4 = 189 …(ii)
⇒ (ap – b)2 + (bp – c)2 + (cp – d)2 ≤ 0
Squaring equation (i) & then dividing by (ii)
(Since, sum of squares is never less than zero )
2 2 2
a (1 + r + r ) 441 ⇒ Each of the squares is zero
⇒ =
2 2
a (1 + r + r ) 4 189
∴(ap – b)2 = (bp – c)2 = (cp – d)2 = 0
(1 + r + r 2 )(1 + r + r 2 ) 441 b c d
⇒ = ⇒p= = =
2
(1 + r + r )(1 – r + r ) 2 189 a b c
\a, b, c are in G.P.
⇒ 2r2 – 5r + 2 = 0
1 Sol 2: (C) Sum of the n terms of the series
⇒ r = 2, ⇒ a = 3, 12
2
1 3 7 15
(2n – 1) + + + + .... upto n terms can be written as
GP is 3, 6, 12 … Sn = 3 = 3(2n – 1) Hence, (A) is 2 4 8 16
the correct choice 2 – 1
 1  1  1  1 
1 –  + 1 –  + 1 –  + 1 –  .... upto n terms
 2   4   8   16 

n 1
Sol 11: (C) ∑ n4 + 4 =
 2 4  1 1 1 
n=1
nn + 2 + 4 – 4  = n –  + + + .... + nterms 
 n  2 4 8 
1 1 
1 1 1 – n 
=∑ = ∑  2  2  2 2 
 2
2  = n–  = n + 2–n – 1
n  n +  – 4  n n + – 2 n + + 2 1
 n   n  n  1–
  2
n Sol 3: (B) Let the common ratio of the G.P. be r. Then,
= ∑ (n2 + 2 – 2n)(n2 + 2n + 2)
Y = xr and z = xr2
1  1 1  ⇒ ln y = lnx + ln r and ln z = ln x + 2 ln r
= ∑ – 
4  n2 – 2n + 2 n2 + 2n + 2  Let A = 1 + ln x, D = ln r
1 1
1 ∞  1 1  Then, = ,
= ∑  – 
4 n=1  (n − 1) + 1 (n + 1) + 1 
2 2 1 + lnx A
1 1 1
= =
1 1 ∞  1 1  3 1 + ln y 1 + lnx + lnr A + D
= 1 + + ∑  –  =

4 2 n=1  (n + 1) + 1 (n + 1) + 1   8
2 2
1 1 1
and
= =
1 + lnz 1 + lnx + 2lnr A + 2D
Sol 12: (D) ln a2 + (ln a2)2 + (ln a2)3 …..
1 1 1
Therefore, , , are in H.P.
= 3 {ln a + (ln a)2 + (ln a)3 + …. } 1 + lnx 1 + ln y 1 + lnz
2lna 3lna
⇒ =
1 – 2lna 1 – lna Sol 4: (A) Since A.M. ≥ G.M., then
⇒ 2 – 2 ln a = 3 – 6 ln a
(a + b) + (c + d)
≥ (a + b)(c + d)
⇒ 1 = 4 ln a 2
⇒ a = e1/4 ⇒M ≤ 1
M a them a ti cs | 3.57

Also,(a + b) + (c + d) > 0( a, b, c, d > 0) Sol 8: (C) Let Sn = cn2


∴0 < M ≤ 1 Sn–1 = c (n – 1)2 = cn2 + c – 2cn
∴Tn = 2cn – c ( Tn = Sn – Sn – 1)
Sol 5: (D) Since a, b, c, d are in A.P.
T2n = (2cn – c)2 = 4c2n2 + c2 – 4c2n
a b c d
⇒ , , , are in AP ∴ Sum = STn2
abcd abcd abcd abcd
4c2n(n + 1)(2n + 1)
1 1 1 1 = + nc2 – 2c2n(n + 1)
⇒ , , , are in A.P. 6
bcd cda abd abc
⇒ bcd, cda, abd, abc are in HP. 2c2n(n + 1)(2n + 1) + 3nc2 – 6c2n(n + 1)
=
3
⇒ abc, abd, cda, bcd are in HP.
nc2 (4n2 + 6n + 2 + 3 – 6n – 6) nc2 (4n2 – 1)
= =
Sol 6: (D) Since a, b, c are in AP. 3 3

Leta = A – D, b = A, c = A + D
Sol 9: (B) Let a, ar, ar2, …
3
Given, a + b + c = a + ar = 12  … (i)
2
3 3 1 ar + ar = 48 
2 3
… (ii)
⇒ (A – D) + A + (A + D) = ⇒ 3A = ⇒ A = Dividing (ii) by (i), we have
2 2 2
1 1 1 ar 2 (1 + r)
∴ The numbers are – D, , + D =4
2 2 2 a(r + 1)
2 2
1  1 1  ⇒ r2 = 4 if r ≠ – 1
Also,  – D  , ,  + D  are in GP.
2  4 2  ∴r=–2
Also, a = – 12 (using (i)).
2 2 2 2
1 1  1  1 1 
=
⇒    – D  +D ⇒ =  – D2  2 6 10 14
4 2  2  16  4  Sol 10: (B) Let S =1 + + + + + .......
 ...(1)
... (i)
3 32 33 34
1 1 1 1 1 1 2 6 10
⇒ – D2 = ± ⇒ D2 = ⇒ D =± S= + + + + ....... ...(2)... (ii)
4 4 2 2 3 3 32 33 34

1 1 1 1 Dividing (i) and (ii)


⇒ a= + or −
2 2 2 2
 1 1 4 4 4
1 1 S 1 −  =1 + + + + + .......
So, out of the given values, a = – is the right  3  3 32
33
34
choice 2 2
2 4 4 1 1 
S = + 1 + + + ...... 
3 3 3  3 3
2 2

Sol 7: (C) We know that, the sum of infinite term of G.P. is
 
 a 2 4 4 1  4 4 3 4 2 6
 , | r |< 1 ⇒ S= +  = + = + =
S∞ = 1 – r 3 3 32  1  3 32 2 3 3 2
 ∞,  1 − 
 | r |≤ 1  3
x 2 6
S∞
∴= = 5 (thus |r| < 1) ⇒ S= ⇒S= 3
1–r 3 3
x 5–x  +n
or 1 – r = ⇒r = exists only when |r| < 1
5 5 Sol 11: (B) M =
2
5–x  , G1 , G2 , G3 , n are in G.P.
i.e., –1 < <1
5 1
or –10 < – x < 0⇒ 0 < x < 10  n 4
r= 

3 . 5 8 | Sequences and Series

1 1 3
 n 4  n 2  n 4
=G1  =
  G2 =
  G3    Sol 15: (B) a, ar, ar2 → G.P.
  
a, 2ar, ar2 → A.P.
G14 + 2G24 + G34
2 × 2ar = a + ar2
2 3
n n n
=  4 × + 2 4 × +  4 × 4r = 1 + r2
2
  3
⇒ r2 – 4r + 1 = 0
=3n + 22n2 + n3
4 ± 16 − 4
= n(2 + 2n + n2 ) r= = 2± 3
2
= n( + n)2
= 4m2n r= 2 + 3

n2 (n + 1)2 r = 2 − 3 is rejected

Sol 12: (B) Tn =


4 ∴ (r > 1)
n2 G.P. is increasing.
1
Tn
= (n + 1)2
4 Sol 16: (B) Statement-I has 20 terms whose sum is 8000
1 2 and statement-II is true and supporting statement-I.
Tn= [n + 2n + 1]
4
∴ kth bracket is (k – 1)2 + k(k – 1) + k2 = 3k2 – 3k + 1.
n
Sn = ∑ Tn
n=1 Sol 17: (D) 100(T100) = 50(T50) ⇒ 2[a + 99d] = a + 49d
1  n(n + 1)(2n + 1)  ⇒ a + 149d = 0 ⇒ T150 = 0
=Sn  + n(n + 1) + n
4 6 
n=9 Sol 18: (D) f(x) = 2x3 + 3x + k
1  9 × 10 × 19  1 384 f’(x) = 6x2 + 3 > 0 ∀x ∈R
S=
9  + 9 × 10 + 9=
 [285 + 90 + 9]
= = 96
4 6  4 4 ⇒ f(x) is strictly increasing function
⇒ f(x) = 0 has only one real root, so two roots are not
Sol 13: (A) a + d, a + 4d, a + 8d → G.P possible
∴ (a + 4d)2 = a2 + 9ad + 8d2
Sol 19: (A) x2 + 2x + 3 = 0  ... (i)
⇒ 8d2 = ad ⇒ a = 8d
ax2 + bx + c = 0  ... (ii)
∴ 9d, 12d, 16d → G.P.
Since equation (i) has imaginary roots
12 4
r
Common ratio= =
9 3 So equation (ii) will also have both roots same as (i).
Thus
Sol 14: (A)
a b c
= = ⇒ a =λ ,b =λ
2 ,c =λ
3
2 2
 8   12   16   20   24 
2 2 2 1 2 3
  +   +   +   +   + ....... Hence 1 : 2 : 3
5  5   5   5   5 

82 122 162 202 242 7 77 777


+ + + + + ...... Sol 20: (C) + + + ....... + up to 20 terms
52 52 52 52 52 10 100 103
10
(4n + 4)2 1 16 10 2
Tn =
2
5= 52 n
; Sn
= ∑16(n =
∑ (n + 2n + 1)
+ 1)2
25 n 1
1
= 7 +
11 111
+

+ ....... + up to 20 terms 
10 100 10
1= 3

16 10 × 11× 21 2 × 10 × 11  16 16 7  9 99 999 
= + + 10  = × 505 = m ⇒ m = =101 + + + ....... + up to 20 terms 
 9 10 100 1000
25  6 2  25 5 
⇒ m = 101
M a them a ti cs | 3.59

7  1  1   1   G2
= 1 −  + 1 − 2  + 1 − 3  + ....... + up to 20 terms  HM =
AM
(we know this)
9  10   10   10  
⇒ (A – 27) (A) = (A – 15)2
 20 
 1   1    ⇒ 27A = 225 – 30 A
1−  
7  10   10      20  
A = 75 …(i)
= 20 −    = = 7 20 − 1 1 −  1   
 
9 1  9 9   10    G.M. = 60 …(ii)
 1 −    
10
  H.M. = 48
 

7 179 1  1   7 
20  a + b = 150 using (i) and (ii)
= +   = 179 + (10)−20 
9 9 9  10   81   ab = 3600
 
a = 120
b = 30
Sol 21: (A) 2y = x + z
2 tan-1 y = tan-1 x + tan-1 (z) Sol 2: H10 = 21, H21 = 10
 2y   x+z  1 1 1
tan−1   = tan−1   = + 9d =
 1− y2   1 − xz  H10 H1 21
 
x+z x+z 1 1 1
= = + 20d =
1− y 2 1 − xz H10 H1 10
11 1
⇒ 11d = ⇒d=
⇒ y2 = xz or x + z = 0 ⇒ x = y = z 210 210
1 9 10
+ =
H1 210 210
JEE Advanced/Boards 1 1
=
H1 210
Exercise 1 1 1 1 209 210
= + 209d = + = =1
H210 H1 210 210 210
Sol 1: (i) Let 2 numbers be a, b
2ab ab Sol 3: sin x, sin22x, cos x sin 4x are in GP.
Given H.M. = =4⇒a+b=
a+b 2
sin42x = sin x cos x sin 4x
a+b
We have A.M. = and G.M. = ab (2 sin x cos x)4= sin x cos x 2 sin 2x cos 2x
2
(G.M.)2 = ab  … (i) 16 sin4 x cos4 x = 4 sin2 x cos2 x cos 2x

2A.M. = a + b … (ii) 4 sin2x cos2x = cos 2x

2A + G2 = 27 Sin22x = cos 2x

a + b + ab = 27 using (i) and (ii) 1 – cos2 2x = cos 2x

3ab cos2 2x + cos 2x – 1 = 0


= 27 ⇒ ab = 18
2 –1 ± 5
cos 2x =
2
a+b=9
–1 – 5
⇒ a, b = 3, 6 ∴ cos θ can never be equal to i.e
2
–1 + 5
(ii) A.M. = G.M. + 15 = H.M. + 27 ∴ cos 2x =
2
a+b 2ab sin2 2x 4 sin2 x cos2 x
= + 27 = ab + 15 Common ratio = =
2 a+b sinx sinx
3 . 6 0 | Sequences and Series

= 4 cos2 x sin x = 2 cos x sin 2x a2


= 1 ⇒ a2 = 1 – r2
1 – r2
1 + cos2x
= 2 1 – cos2 2x a1r1 = a2r2 ⇒ (1 – r1)r1 = (1 – r2)r2
2
⇒ r1 – r2 = (r1 – r2)(r1 + r2)
(6 – 2 5)  5 –1
= 2 1– 1+  ⇒ r1 + r2 = 1
4  2 
 
a1r12 = 1/8 ⇒ (1 – r1)r12 = 1/8
5 −1 5 +1 = 2 . 4
= 2 . = 2
2 2 4 ⇒ (2r1 – 1)(4r12 – 2r1 – 1) = 0

If r1 = ½ then r1 = r2
Sol 4: a, b, c, d, e be 5 numbers ⇒ 4r12 – 2r1 – 1 = 0
a b c in AP, b c d in GP, c d e in HP
1± 5
2ce
⇒ r1 =
2b = a + c ,c2 = bd, d = , Let b be b 4
c+e
c be br, d be br2
1− 5
If r1 = then, r2 > 1.
2bre br 2 4
br2 = ⇒ br2 + er = 2e ⇒ e =
br + e 2–r 1+ 5
⇒ r1 =
a = 2b – br = b(2 – r) 4
  1 + 5  1 + 5 
ae = b2r2 = c2 hence a,c,e are in GP …(i) ∴ a1r1 = (1 – r1)r1 =  1 −   
  4  4 
(2b – a)2 c2 b2r 2 br 2
(ii) ⇒ = = = =e
a a b(2 – r) 2–r    5 −1 m −n
=  3 − 5  1 + 5  = =
Hence proved.  4  4  8 p

(iii) a = 2 e = 18 ⇒ c = ± 6 ∴ 100m + 10n +p = 500 + 10 + 8 = 518


bc, d, e = (4, 6, 9); (–2, –6, –18)
⇒ b = 4, – 2 ⇒d = 9, – 18
99
5100
Sol 7: S = ∑ 25n + 5100
n=1

Sol 5: 2α2, α4, 24 form A.P. 5100


T1 =
α4 = α2 + 12 …(i) 52 + 5100
α4 – α2 = 12 5100 5100 52
T99 = = =
⇒ α = 2, – 2 (α1, α2 = 2, – 2 52×99 + 5100 5200 52 + 5100
+ 5100
(β2)2 = 1(6 – β2) 52
T1 + Tn = 1
β4 + β2 = 6
S = T1 + T2 + … + T99 = 1 + 1 … T50
β4 + β2 – 6 = 0
5100
β4 + 3β2 – 2β2 – 6 = 0 = 49 + T50 = 49 + = 49+1/2
5100 + 5100
β2 = 2 [S] = 49
β1, β2 = 2,– 2
α1 + α2 + β12 + β22 = 4 + 4 + 2 + 2 = 12
2 2 Sol 8: ax3 – ax2 + 9bx – b = 0
3 3αβγ
HM roots = =
Sol 6: 2G.P.s 1 1 1 αβ + βγ + γα
α + β + γ =1 α + β + γ
a1
= 1 ⇒ a1 = 1 – r1
1 – r1 9b1
αβ + βγ + γα =
a1
M a them a ti cs | 3.61

b1 127 S
αβγ = 127 – =S–
a1 r r n–1

1  1  63.5 
H.M .= 127  1 –  = 32r – r – 31.5 
3 r r
  
α+β+ γ
≥ H.M.(abc) from (v)
3
α+β+ γ 1 127
≥ 127 – = 63.5 r – 63.5
3 3 r
⇒r=2
α + β + γ ≥1  …(i)
∴ s = 64
Its given α + β + γ =1
s 1
Equality of equation (i) holds only if α = β = γ =
r n–1 2
1
i.e all the roots are
3 2n–2 = 64
3
b 1 ⇒n=8
= α3 =  
a 3
b = 27a Sol 10: an+1 = an +1 for n = 1 … 97

b + a = 28a ⇒ a2 = a1 + 1

∴ a is an integer, min (a + b) = 28 ⇒ a3 = a2 + 1 = a1 + 2
⇒ a4 = a1 + 3
Sol 9: Let time taken to solve 1st problem be S time to an = a1 + (n – 1)
S 98
solve second problem will be ⇒ a1 + a2 … a98 = 4949 = [2a1 + 97.1]
r 2
S S
+… + = 63.5 …(i) 101 = 2a1 + 97 ⇒ a1 = 2
r rn–1
s s Now, we can write here Sa2k
Sn = 127 = S + … …(ii)
r r n–2
 = a2 + a4 + a6 … a98 = a1 + 1 + a1 + 3 … a1 + 97
s s 49
31.5 = +… …(iii) = [2a1 + 2 + 48 × 2] = 49[a1 + 49] = 49 × 51
2 n–1
r r  2
S = 2499
= 32 …(iv)
r
S 343
127 + = 63.5 + S Sol 11: xyz = 55 or acc to a, x, y, z, b in AP/HP
rn–1 55
For a, x, y, z, b in AP
S
63.5 + =S x = a + d; y = a + 2d z = a + 3d
rn–1
b–a
S S  1 b = a + 4d ⇒ d =
– = 63.5  1 –  4
r r n
 r
(a + d) (a + 2d) (a + 3d) = 55  …(i)
s 63.5 For a, x, y, z, b in HP
32 – = 63.5 –
rn r
1 1 1 1 1 1 1
63.5 S = + dH ; = + 4dH ⇒ dH =  – 
= 31.5 …(v) x a b a 4 b a 
r rn
1 1
 1  = + 2dH
S  1 – n–1  y a
r 
127 = 
 1 1 1
1 –  = + 3dH
 r z a
3 . 6 2 | Sequences and Series

1 1  1  1  55 Sol 13: We have b2 = ac


=  + dH   + 2dH   + 3dH  = 343  …(ii)
xyz a  a  a  Also, logca, logbc and logab are in AP
Equation (i) can be written as We can write logab = logca + (3 - 1)d

 b – a  (b – a)2   (b – a)3  log b − logc a


a +  a +  a +  =55 ⇒d= a
 4  4  4  2

loga ac − logc a
(3a + b)(2a + 2b)(a + 3b) =
= 55 2
64
1 + 3 loga c
Equation (ii) can be written as =
4
 3 1   1 1   1 3  55 Given that logac = 4
 +   +   +  =
 4a 4b   2a 2b   4a 4b  343
1+ 3× 4 13
∴d = =
(3b + a)(2b + 2a)(b + 3a) 55 4 4
=
2 2
64a b ab 343
Sol 14: a = b – 2
⇒ a3b3 = 343
ar2 = b + 6
⇒ ab = 7
a & b are integers a + ar 2 5
=
2ar 3
i.e a = 1 , b = 7or a = 7, b = 1
2b + 4 5
i.e a2 + b2 = 50 =
2ar 3
3
Sol 12: 10x3 – cx2 – 54x – 27 = 0 (b + 2) = ar
5
Let α, b, g be the roots
9
(b + 2)2 = (b + 6) (b – 2)
c 25
α+β+g= …(i)
10 
54 ⇒ 9(b2 + 4 + 4b) = 25 (b2 + 4b – 12)
aβ + by + gα = – …(ii)
10  16b2 + 64b – 336 = 0
27
abg = …(iii) b2 + 4b – 21 = 0
10 
α β & g are in harmonic progression b2 + 7b – 3b – 21 = 0
2αγ b = 7, 3 ⇒ +ve integral value of b is 3.
i.e β =
α+γ
bα + bg = 2αg ….(iv) S1–11 1
Sol 15: = … (i)
Sn–10–n 8
Putting this in equation (iii)
S10–n
β = – 3/2 this in equation (iv) =2 … (ii)
S(n–8 ) –n
 –3  –3.6
(α + γ )   =
2
  10
S1 – 11 = a
(r 11
–1 )S = ar 9
(r n–9
–1 )
10 – n
12
r –1 r –1
α+ γ =
5
–3 S(n – 10) – n = ar n–11
(r 11
–1 )s =a
(r n–9
–1 )
⇒ α = 3; γ = (n–8)–n
5 r –1 r –1
–3 –3
The 3 roots are 3, , Putting these values in equation (i) and equation (ii)
2 5
1 1
C 12 3 9 =
=α+β+y= − = ⇒C=9 r n–11 8
10 5 2 10
M a them a ti cs | 3.63

1 ⇒ a(a – 8) = (a – 6)2 ⇒ a = 9, r = 3
r =2⇒r=
9
29
So the number is 931
⇒ rn – 11 = 23
n–11 Sol 18: S(θ) = 1 + (1 + sin θ) cos θ
⇒ 2 9 = 23
+ (1 + sin θ + sin2θ) cos2 θ … ∞
⇒ n = 11 + 27 = 38 = 1 + cos θ + cos2 θ ...
+ sin θ (cos θ + cos2 θ ...) + sin2 θ
Sol 16: Sn = (1 + 2Tn) (1 – Tn)
1 sin θ sin2 θ cos2 θ
n = + cos θ +
Sn = [2a + (n – 1)d] 1 – cos θ 1 – cos θ 1 – cos θ
2
= [1 + 2a + (n – 1) 2d] [1 – a – (n – 1)d] 1
= [1+ sin θ cos θ + sin2θ cos2θ …]
1 – cos θ
Tn = a + (n – 1)d
1
Sn = 1 + Tn – 2Tn2 S(θ) =
(1 – sin θ cos θ)(1 – cos θ)
S1 = 1 + T1 – 2T12 = T1
1 1  1  2 2
S π  = 1 –  =
T1 =
2  
4

 1 –
1 

2 ( 2 –1 )
 2
S2 = 1 + T2 – 2T2 = T1 + T2 2

1
1– = 2T22  π  π  π 
2 Sol 19: tan  – x  , tan , tan  + x  are in GP
 12  12  12 
 2 –1 2– 2 4– 2
  = T22 = = π  π   π 
 2 2  4 4 tan2 = tan  – x  tan  + x 
  12  12   12 
⇒ a = 4 and b = 2
 π   π  π
sin  + x  sin  – x  cos2x – cos
a+b=6  12   12  6 π
= ⇒ = tan2
 π   π  π 12
cos  + x  cos  – x  cos2x + cos
Sol 17: Let number be abc (a > b > c)  12   12  6
a a
a = a, b = ,c= π π π π π 
r r2 cos tan2 + cos cos tan2 + 1
6 12 6 = 6  12 
10a a a 10a − a cos 2x =
⇒ 100a + + – 100 – = 792 2 π 2 π
r 2 2 r 1 – tan 1 − tan
r r 12 12
99a a
⇒ 99a – = 792 ⇒ a – =8  2 π  2 π   
r 2
r2  sin   + cos     
π  12
  12
  3 1
cos  =1
∴ New number = 100 ( a − 4 ) + 10b + c =
6  2 π  2 π 
= 2   π 
 cos   − sin     cos   
2a a   12   12     6 
⇒ 2b = a – 4 + c ⇒ =a–4+
r r2
2a 3 2
⇒ =a–4+a–8 = × 1
=
r 2 3
2a a a ∴ cos 2x = 1
⇒ 2a – 12 = ⇒a= +6= +8
r r r2
a
⇒r=
a–6 π/2

 a 
2 O π
a
⇒   =
 a – 6  a – 8
3 . 6 4 | Sequences and Series

Solutions are O, π, 2π, 3π … 99π 1  1002 + 100  5050


=   =
99 2  10101  10101
[2π + 98π] = 50 π . 99 = 4950 π
2

K = 4950
Sol 4: (A)

r
a
Exercise 2
a/2
Single Correct Choice Type
Circle inscribed in 1st circle = r = a/2
Sol 1: (A) A.M. = 9 + 99 … 999999999/9
a
⇒ 9[1 + 11 + 111 + …. 111111111]/9 In 2nd circle = r1 =
2 2
= 123456789 a 1
In 3rd circle = this is G.P. with common ratio
This does not contain 0 2×2 2
n–1  1–n 
n–1
a 1  1 2  
360
1 m ar =   = r  = r2 2  [A]
Sol 2: (D) Given ∑ =
n
2 2  2
k =1 k k + 1 + (k + 1) k

360
1  1 k +1 – k  Assertion Reasoning Type
∑ =  
k =1 k + 1 k  k + k + 1 k + 1 – k 
Sol 5: (D) Statement-I: If (a + b + c)3 ≤ 27 abc
360
k +1 – k 3a + 4b + 5c = 12
∑ =
k =1 k +1 k
Statement-II: ⇒ A.M. ≥ G.M. (True)
360  1 1  We Know A.M. ≥ G.M.
∑ = ∑ – 
k =1  k k + 1 For three numbers a + b + c
360
1 1 1 1 1 a+b+c
∴ ∑ Tk =–
2
+
2

3
…- ⇒
3
≥ (abc)1/3
k =1 1 361
1 18 m ⇒ (a + b + c)3 ≥ 27 abc
=1– = = (given)
19 19 n Given (a + b + c)3 ≤ 27 abc
⇒ m + n = 18 + 19 = 37 ⇒ a = b = c & (a + b + c)3 = 27 abc
3a + 4a + 5a = 12 ⇒ a = b = c = 1
100
k k
Sol 3: (B) ∑ 4
= ∑ (k 2 + 1)2 – k 2 1
+
1
+
1
=
1
+
1
+
1
= 3 ≠ 10
k =1 k + k2 + 1 a 2 3
b b 5 1 1 1
k
= ∑ (k 2 + 1 + k)(k 2 + 1– k) So statement-II is false.

k  1 1  Multiple Correct Choice Type


= ∑ 2k  k 2 + k + 1 + k 2 – k + 1
 
Sol 6: (A, B, C, D) a1 = 25, b1 = 75, a100 + b100 = 100
1 1 1 
= ∑ 2  k 2 – k + 1 – k 2 + k + 1 a1 + 99da + b1 + 99db = 100
 
⇒ da + db = 0
1 1 1 1 1 1  ⇒da = – dB
= ∑  – + – ... 
2  1 3 3 7 100 + 100 + 1 
2
(a) Hence a is correct
M a them a ti cs | 3.65

(b) an + bn = a1 + (n – 1)da + b1 + (n – 1)dB a2


+ a2r + a2 = q
= a1 + b1 + (n – 1) (da + dB) = a1 + b1 = 100 r

Hence, correct a3 = 1 ⇒ a = 1

(c) (a1 + b1) (a2 + b2) (a3 + b3) …  1


 r + 1 +  = – p … (i)
 r
= 100, 100, 100 is in AP
100 100 1 
 + r + 1 = q ⇒ q = – p ⇒ p + q =
0
(d) ∑ (ar + br ) = ∑ (100) = 10000  r 
r =1 r =1
1
r +1+
r ≥ 1 [AM ≥ GM]
Sol 7: (B, C) sin (x – y), sin x, sin (x + y) are in HP 3
1
2sin(x – y)sin(x + y) r+1+ ≥3
sin x = r
sin(x – y) + sin(x + y)
p∈ (– ∞, 3) [B is incorrect]
⇒ 2 sin2x cosy = 2cos2y – 2cos2x
one root (a) is unity
⇒ sin2x (cos y – 1) = cos2 y – 1 1
one root is & other is r, so if 1 root is greater than 1
⇒ sin2x = 1 + cos y = 2 cos2y/2 r
and other less than [ACD]
y
⇒ sin x sec   = 2
2 a 2b
Sol 10: (B, D) log a, log b, log c, log , log ,
2b 3c
Sol 8: (A, B) Given series is a ar ar2 ar3… 3c
log are in AP
Given that ar – a = 6 and ar3 – ar2 = 54 a
a1, a2, a3, a4, a5, a6
⇒ ar2(r – 1) = 54 ⇒ a ( r – 1) = 6
Dividing these (2) 2a5 = a4 + a6

r2 = 9 4b2 3c
=
r=±3 9c 2 2b

a = 3, for r = 3 b 3 3c
= ⇒b=
ar = 9 c 2 2

ar2 = 27 3b 9c
a= =
2 4
a=3
15c
sum = 39 [A] a+b= >c
4
–3 13c 3c
for r = – 3; a = a+c= 4 > 2 (b)
2
9 –27 5c 9c
ar = ; ar2 = b+c= > (a)
2 2 2 4
–3 9 27 27 –21 Hence a, b, c can form ∆
∴ sum = + – =3– =
2 2 2 2 2 log b – log a = log c – log b
2log b = log a + log c
Sol 9: (A, C, D) x3 + px2 + qx – 1
b2 = ac ie a, b, c are in GP[B]
Roots form increasing GP
a = a, b = ar, c = ar2
a
Roots be , a, ar 18 (a + b + c)2 – 18a2 – 18b2 – 18c2
r
a = 18 (2ab + 2bc + 2ac)
+ a + ar = – p
r = 36 (ab + bc +ac) > ab so A is incorrect
3 . 6 6 | Sequences and Series

Sol 11: (A, B, C, D) x2 – 3x + A = 0  1 


n−1
⇒ an = 10   (Qa1 = 10 given)
x1 + x2 = 3  2
x1x2 = A 2(n–1)
 1 
⇒ an2 = 100  
x2 – 12x + B = 0
 2
x3 x4 = B 100
⇒ ≤ 1 ( an2 ≤ 1given )
n–1
x3 + x4 = 12 2
x1 = a ; x2 = ar ; x3 = ar ; x4 = ar
2 3
⇒ 100 ≤ 2n–1
a2r = A This is possible for n ≥ 8. So (b), (c), (d) are the answer.
a(1 + r) = 3
a2r5 = B k −1
k! = 1
ar2(1 + r) = 12 Sol 3: (4) We have Sk =
1 (k – 1)!
1–
r=±2 k
a = 1, – 3 Now, (k2 – 3k + 1) Sk = {(k – 2) (k – 1) – 1} × Sk
A = a2 r = 1 × 2 = 2
1 1 100
a2r5 = 25 = 32 = B = –
(k – 3)! (k – 1)!
⇒ ∑ | (k 2 – 3k + 1)Sk |
k =1
x1 + x3 = a(1 + r2) = 5
x2 + x4 = ar (1 + r2) = 2.5 = 10
 1 1  1002
= 1+1+ 2 –  + = 4–
 99! 98!  100!

Previous Years’ Questions 1002 100 2


⇒ + ∑ | (k – 3k + 1)Sk | =
4
100! k =1
Sol 1: (A, B, D) Since, first and (2n – 1)the terms are
equal. Let first term be x and (2n – 1)th term by y. whose Sol 4: (0) ak = 2ak–1 – ak –2
middle term is tn.
⇒ a1, a2,…., a11 are in AP
x+y
Thus in arithmetic progression ; tn = =a
2
a12 + a22 +.... + a11
2
11a2 +35×11d2 +10ad
∴ = = 90
In geometric progression : xy =b 11 11

2xy ⇒ 225 + 35d2 +150d = 90


In harmonic progression ; tn = =c
x+y ⇒ 35d2 + 150d + 135 = 0
9
⇒ b2 = ac and a ≥ b ≥ c (using A.M. ≥ G.M. ≥ HM) ⇒ d = – 3, –
7
Here, equality holds (ie, a = b = c) only if all terms are 27 9
same. Given a2 < ∴ d = – 3 and d ≠ –
2 7
a1 + a2 + ..... + a11 11
Sol 2: (B, C, D) Let an denotes the length of side of the ⇒ = [30 –=
10 × 3] 0
square Sn. 11 2

We are given an = length of diagonal of Sn+1.


Sol 5: a1, a2, a3, …. A100 is an A.P.
an
⇒ an = 2an+1 ⇒ an+1 = p
2 a1 = 3, Sp = ∑ a1 , 1 ≤ p ≤ 100
i =1
This show that a1, a2, a3…. Form a G.P. with common
5n
ratio 1 / 2
n–1
Sm S5n 2
(
6 + (5n − 1) d )
 1  = =
Sn Sn n
Therefore, an = a1 
 2

2
( 6 − d + nd)
M a them a ti cs | 3.67

Sm is independent of n of 6 – d = 0 ⇒ d = 6 Sol 11: (B) Since,


a1 = 3 Tr = 3r2 + 2r – 1
a2 = 3 + 6 = 9 ∴Tr+1 = 3(r + 1)2 +2 (r +1) –1
a2 = 9 ∴Qr = Tr + 1 – Tr = 3[2r + 1] + 2[1]
Qr = 6r + 5
Sol 6: (C) Let a and b are two numbers. Then,
a+b 2ab n
1 1
=A1 = ;G1 ab ;H1
= Sol 12: (A, D) Sn < lim Sn = lim ∑ ⋅
3 a+b
k =1 n 1 + k / n + (k / n)
n→∞ n→∞ 2

An–1 + Hn–1 1 dx π
=An =
2
,Gn An–1Hn–1 , = ∫=
0
1 + x + x2 3 3
2An–1Hn–1
Hn =
An–1 + Hn–1 π n−1 n
Now, Tn > as h f(kh) > 1f(x)dx > h f(kh)
3 3 0 ∑ ∫ ∑
Clearly, G1 = G2 = G3 = ….. = = k 0= k 1
ab

Sol 7: (A) A2 is A.M. of A1 and H1 and A1 > H1 Sol 13: (C) b1 = a1, b2 = a1 + a2, b3 = a1 + a2 + a3,
b4 = a1 + a2 + a3 + a4
⇒ A1 > A2 > H1
Hence b1, b2, b3, b4 are neither in A.P. nor in G.P. nor in
A3 is A.M. of A2 and H2 and A2 > H2
H.P.
⇒ A2 > A3 > A4
::: Sol 14: (C) tn = c {n2 − (n − 1)2} = c (2n − 1)
∴ A1 > A2 > A3 >……. tn2 c2 (4n2 − 4n + 1)
⇒=

n
Sol 8: (B) As above A1 > H2 > H1, A2 > H3 > H2  4n(n + 1)(2n + 1) 4n(n + 1) 
⇒ ∑ tn2 c2 
= − + n
∴ H1 < H2 < H2 < …… n=1  6 2 

c2n
r 1 3 2
Sol 9: (B) Here, Vr= [2r + (r – 1)(2r – 1)] =(2r – r + r)=
6
{4(n + 1)(2n + 1) − 12(n + 1) + 6}
2 2
1 c2n c2
∴ ΣV
=r
2
[2Σr3 – Σr 2 + Σr] =
3
{
4n2 + 6n + 2 − 6n − 6=
+3
3
n(4n2 − 1) }
1   n(n + 1)  n(n + 1)(2n + 1) n(n + 1) 
2
2   – +  100
2  2  6 2 
  Sol 15: (3) ∑ (k 2 − 3k + 1)Sk
k =2
n(n + 1)
= [3n(n + 1) – (2n + 1) + 3] for k = 2 |k2 - 3k + 1) Sk| = 1
12
1
= n(n + 1)(3n2 + n + 2) 100
k −1 k −1 +1
12 ∑ (k − 2)! − (k − 1)!
k =3
1 1
Sol 10: (D) Vr +1 – Vr = (r + 1)3 – r3 – [(r + 1)2 – r 2 ] + (1) 100
2 2 1 1 1 1
= 3r2 + 2r – 1
∑ (k − 3)! + (k − 2)! − (k − 2)! − (k − 1)!
k =3

∴ Tr = 3r + 2r – 1= (r + 1) (3r – 1)
2
100
 1 1 
Which is a composite number. ∑  (k − 3)! − (k − 1)! 
k =3  
3 . 6 8 | Sequences and Series

 1 1 1 1 1  1 1 1 1 (n−1)
S =+
1 1 −  +  −  +  −  +  −  +  −  + =
 2!   1! 3!   2! 4!   3! 5!   4! 6! 
∑ ((4r + 4)2 + (4r + 3)2 − (4r + 2)2 − (4r + 1)2 )
r =0
 1 1   1 1   1 1   1 1 
....  − + − + − + −  (n−1) (n−1)
94! 96! 95! 97! 96! 98! 97! 99!
        =
∑ (2(8r + 6) + 2(8r=
+ 4)) ∑ (32r + 20)
1 1 r =0 r =0
2
=− −
98! 99!
1056 for n = 8
= 16(n – 1)n + 20n = 4n(4n + 1) = 
1002 1 1 1332 for n = 9
E
∴= +3− −
100! 98! 99.98!
b c
1002 100 1002 100 Sol 20: (6) = = (int eger)
= +3− = +3− = 3 a b
100! 99! 100.99! 99!
b2
b2 = ac ⇒ c =
Sol 16: (0) ak = 2ak−1 − ak−2 ⇒ a1, a2, …, a11 are in A.P. a
a+b+c
a2 + a22 + .... + a11
2
11a2 + 35 × 11d2 + 10ad = b+2
∴ 1 = = 90 3
11 11
a + b + c = 3b + 6 ⇒ a − 2b + c = 6
⇒ 225 + 35d2 + 150d = 90
35d2 + 150d + 135 = 0 ⇒ d = − 3, − 9/7 b2 2b b2 6
a − 2b + =6 ⇒ 1 − + =
27 a a a2 a
Given a2 < ∴ d = − 3 and d ≠ − 9/7 ⇒
2 2
b  6
a1 + a2 + .... + a11 11  − 1  = ⇒ a = 6only
= 30 − 10 × 3
=  0
11 2 a  a

Sol 17: (B) ax2 + bx + c = 0 ⇒ x2 + 6x – 7 = 0 Sol 21: (9) Let seventh term be ‘a’ and common
difference be ‘d’
⇒ α = 1, β = – 7
n n
S7 6
∞ ∞ Given, = ⇒ a = 15d
 1 1 1 1  S11 11
∑  α + β  = ∑ 1− 7  = 7
= 
n 0=  n 0  Hence, 130 < 15d < 140
⇒ d=9
Sol 18: (D) Corresponding A.P.
Sol 22: (B) log (b2) - log (b1) = log (2)
1 1
,.......................... (20th term) b2
5 25 ⇒ =2 ⇒ b1, b2, …. are in G.P. with common ratio 2
b1
1 1 1  −4  4
= + 19d ⇒d=  = − ∴ t = b1 + 2b1 + …… + 250 b1 = b1 (251 - 1)
25 5 19  25  19 × 25
S = a1 + a2 + ….. + a51
an < 0
51 51 51
= (a + a ) = (b + b ) = b (1 + 250 )
1 4 2 1 51 2 1 2 2 1
− × (n − 1) < 0
5 19 × 25  51   53 
− t b1  + 51 × 249 − 251 =
S= + 1  b1  + 249 × 47  ⇒ S > t
19 × 5  2   2 
< n−1
4
⇒S>t
n > 24.75 b101 = 2100 b1
4n k(k +1) a101 = a1 + 100 d = 2 (a1 + 50d) - a1
Sol 19: (A, D) S
=n ∑ (−1) 2 k2
= 2a51 - a1 = 2b51 - b1 = (2 × 251 - 1) b1 = (251 - 1) b1
k =1
∴ b101 > a101
2017-18 100 &
op kers
Class 11 T
By E ran culty
-JE Fa r
IIT enior emie .
S fP r es
o titut
Ins

MATHEMATICS
FOR JEE MAIN & ADVANCED
SECOND
EDITION

Exhaustive Theory
(Now Revised)

Formula Sheet
9000+ Problems
based on latest JEE pattern

2500 + 1000 (New) Problems


of previous 35 years of
AIEEE (JEE Main) and IIT-JEE (JEE Adv)

5000+Illustrations and Solved Examples


Detailed Solutions
of all problems available

Topic Covered Plancess Concepts


Tips & Tricks, Facts, Notes, Misconceptions,
Binomial Theorem Key Take Aways, Problem Solving Tactics

PlancEssential
Questions recommended for revision
4. BINOMIAL THEOREM

MATHEMATICAL INDUCTION
The technique of Induction is used to prove mathematical theorems. A variety of statements can be proved using
this method. Mathematically, if we show that a statement is true for some integer value, say n = 0, and then we
prove that the statement is true for some integer k+1 if it is true for the integer k (k is greater than or equal to 0),
then we can conclude that it is true for all integers greater than or equal to 0.
The solution in mathematical induction consists of the following steps:
Step 1: Write the statement to be proved as P(n) where n is the variable.
Step 2: Show that P(n) is true for the starting value of n equal to 0(say).
Step 3: Assuming that P(k) is true for some k greater than the starting value of n, prove that P(k+1) is also true.
Step 4: Once P(k+1) has been proved to be true, we say that the statement is true for all values of the variable.
The following illustrations will help to understand the technique better.

Illustration 1: Prove that 1+2+3+...+n=n(n+1)/2 for all n, n is natural. (JEE MAIN)

Sol: Clearly, the statement P(n) is true for n = 1. Assuming P(k) to be true, add (k+1) on both sides of the statement.
P(n):1+2+3+...+n=n(n+1)/2
Clearly, P(1) is true as 1=1.2/2.
Let P(k) be true. That is, let 1+2+3+...+k be equal to k(k+1)/2
Now, we have to show that P(k+1) is true, or that
1+2+3+...+(k+1)=(k+1)(k+2)/2.
L.H.S = 1+2+3+...+(k+1)
= 1+2+3+...+k+(k+1) = k(k+1)/2 + (k+1) (As P(k) is true)
= (k+1) (k/2+1) = (k+1)(k+2)/2
= R.H.S

Illustration 2: Prove that (n+1)! >2n for all n>1. (JEE MAIN)

Sol: For n = 2, the given statement is true. Now assume the statement to be true for n = m and multiply (m+2) on
both sides.
4 . 2 | Binomial Theorem

Let (n+1)! >2n  … (i)


Putting n=2 in eq. (i), we get,
3! > 22
3! > 4
Since this is true,
Therefore the equation holds true for n=2.
Assume that equation holds true for n=m,
(m+1)! > 2m  … (ii)
Now, we have to prove that this equation holds true for n=m+1, i.e. (m+2)! > 2m+1 .
From equation 2, (m+1)! > 2m .
Multiply above equation by m+2
(m+2)! > 2m (m+2)
> 2m+1 + 2m.m
> 2m+1.
Hence proved.

Illustration 3: Prove that n2 + n is even for all natural numbers n. (JEE MAIN)

Sol: Consider P(n) = n2 + n. It can written as a product of two consecutive natural numbers. Use this fact to prove
the question.
Consider that P(n) n2 + n is even, P(1) is true as 12 + 1= 2 is an even number.
Consider P(k) be true,
To prove : P(k + 1) is true.
P(k + 1) states that (k + 1)2 + (k + 1) is even.
Now, (k + 1)2 + (k + 1) = k2 + 2k + 1 + k + 1 = k2 + k + 2k + 2
As P(k) is true, hence k2 + k is an even number and can be written as 2 λ , where λ is sum of natural number.
∴ 2 λ + 2k + 2 ⇒ 2( λ + k + 1) = a multiple of 2.
Thus, (k + 1)2 + (k + 1) is an even number.
Hence, P(n) is true for all n, where n is a natural number.

Illustration 4: Prove that exactly one among n+10, n+12 and n+14 is divisible by 3, considering n is always an
natural number. (JEE MAIN)

Sol: We can observe here that


For n = 1,
n+10 = 11
n+12 = 13
n+14 = 15
Exactly one i.e 15 is divisible by 3.
Let us assume that that for n = m exactly one out of n+10, n+12, n+14 is divisible by 3
M a them a tics | 4.3

Without the loss of generality consider for n=m, m+10 was divisible by 3
Therefore, m+10 = 3k
m+12 = 3k+2
m+14 = 3k+4
We need to prove that for n=m+1 , exactly one among them is divisible by 3. Putting m+1 in place of n, we get
(m+1)+10 = m+11 = 3k + 1 (not divisible by 3)
(m+1)+12 = m+13 = 3k+3 = 3(k+1) (divisible by 3)
(m+1)+14 = m+15 = 3k+5 (not divisible by 3)
Therefore, for n=m+1 also exactly one among the three, n+10, n+12 and n+14 is divisible by 3.
Similarly we can prove that exactly one among three of these is divisible by 3 by considering cases when n+12 = 3k
and n+14 = 3k.

BINOMIAL THEOREM

1. INTRODUCTION TO BINOMIAL THEOREM

1.1 Introduction
Consider two numbers a and b, then

(a + b)
2
=a2 + 2ab + b2

(a + b)
3
=( a + b )( a + b )
2
(
=( a + b ) a2 + 2ab + b2 ) =a3 + 3a2b + 3ab2 + b3

(a + b) =( a + b ) ( a + b ) ( )( )
4 2 2
= a2 + 2ab + b2 a2 + 2ab + b2 =a4 + 4a3b + 6a2b2 + 4ab3 + b 4

As the power increases, the expansion becomes lengthy, difficult to remember and tedious to calculate. A binomial
expression that has been raised to a very large power (or degree), can be easily calculated with the help of Binomial
Theorem.

1.2 Binomial Expression


A binomial expression is an algebraic expression which contains two dissimilar terms.
1
For example: x + y, a2 + b2 , 3 − x, x2 + 1 + etc.
3
x3 + 1

1.3 Binomial Theorem


Let n be any natural number and x, a be any real number, then

( x +=
a)
n n
C0 xna0 + n C1 xn−1a1 + n C2 xn−2a2 + ...... + n Cr xn−r ar + ...... + n Cn−1 x1an−1 + n Cn x0 an
n
n!
i.e. ( x + a) =
n
∑ n Cr xn−r ar where n Cr = r! (n − r ) !
0

and the co-efficients nC0, nC1, nC2, ……………… and nCn are known as binomial coefficient.
4 . 4 | Binomial Theorem

PLANCESS CONCEPTS
n
(a) The total number of terms in the expansion of (x + a)n = ∑ n Cr xn−r ar , is (n + 1).
r =0
(b) The sum of the indices of x and a in each term is n.
(c) n
C0, nC1, nC2, . . . , nCn are called binomial coefficients and also represented by C0, C1, C2 and so on.
n n
(i) Cx= C y ⇒ x= y or x + y= n (ii) n Cr = n Cn−r
n +1
(iii) n Cr + n Cr −1 = n
Cr (iv) = Cr n / (n − r ) . n−1 Cr

Vaibhav Gupta (JEE 2009, AIR 22)

Illustration 5: Expand the following binomials


4
 3x3 
(i) (x – 2)5 (ii) 1 −   (JEE MAIN)
 2 

Sol: By using formula of binomial expansion.

( x − 2) = 5 C0 x5 + 5 C1 x 4 ( −2 ) + 5 C2 x3 ( −2 ) + 5 C3 x2 ( −2 ) + 5 C 4 x ( −2 ) + 5 C5 ( −2 )
5 1 2 3 4 5
(i)

= x5 – 10x4 + 40x3 – 80x2 + 80x – 32


4 2 3 4
 3x3   3x3  4  3x3   3x3   3x3 
(ii)  1 −  = 4 C0 + 4 C1  −  + C2  −  + 4 C3  −  + 4 C4  − 
 2   2   2   2   2 
        
27 6 27 9 81 12
= 1 – 6x3 + x − x + x
2 2 16

2. DEDUCTIONS FROM BINOMIAL THEOREM

2.1 Results of Binomial Theorem

D-1 On replacing a by –a, in the expansion of (x + a)n , we get

( x=
− a) C0 xna0 − n C1 xn−1a1 + n C2 xn−2 .a2 − ..... + ( −1 ) Cr xn−r ar + ...... + ( −1 )
n n r n n n
Cn x0 an
n
i.e. ( x − a) = ∑ ( −1 )
n r n
Cr xn−r ar
r =0

Therefore, the terms in (x – a)n are alternatively positive and negative, and the sign of the last term is positive or
negative depending on whether n is even or odd.

D-2 Putting x = 1 and a = x in the expansion of (x + a)n, we get

(1 + x ) =
n n
C0 + n C1 x + n C2 x2 + ...... + n Cr xr + .... + n Cn xn
n
⇒ (1 + x ) =
n
∑ n Cr xr
r =0

This is the expansion of (1 + x)n in ascending powers of x.


M a them a tics | 4.5

D-3 Putting a = 1 in the expansion of (x + a)n , we get


n
C0 xn + n C1 xn−1 + n C2 xn−2 + .... + n Cr xn−r + ....... + n Cn−1 x + n Cn ⇒ (1 + x ) =
n
(x + 1)
=n n
∑ n Cr xn−r
r =0

This is the expansion of (1 + x) in descending powers of x.


n

D-4 Putting x = 1 and a = –x in the expansion of (x + a)n, we get

(1 − x )= C0 − n C1 x + n C2 x2 − n C3 x3 + ..... + ( −1 ) Cr xr + ..... + ( −1 )
n n r n n n
Cn xn

D-5 From the above expansions, we can also deduce the following

( x + a) + (=
x − a)
n n
2  n C0 xna0 + n C2 xn−2a2 + ..... 
 

and ( x + a) −=
( x − a) 2  n C1 xn−1a1 + n C3 xn−3 a3 + .....
n n

PLANCESS CONCEPTS

{({x( x++aa) ) ++( x( x−−aa) )}}and


If n is odd then
nn
and{({x( x++aa) ) −−( x( x−−aa) )}} both have the same number of terms equal
and
nn nn nn

 where as if n is even, then {( x + a) + ( x − a) } has  + 1  terms.


n+1 n n n 
to 
 2   2 
Nikhil Khandelwal (JEE 2009, AIR 94)

2.2 Properties of Binomial Coefficients

Using binomial expansion, we have

(1 + x )
n n
= C0 + n C1 x + n C2 x2 + ....... + n Cr xr + ...... + n Cn xn

Also, (1 + x )=
n n
C0 xn + n C1 xn−1 + n C2 xn−2 + ....... + n Cr xn−r + ...... + n Cn−1 x + n Cn

Let us represent the binomial coefficients n C0 , n C1 , n C2 ,......., n Cn−1 , n Cn by C0 ,C1 ,C2 ,.......,Cn−1 ,Cn respectively. Then
the above expansions become
n
(1 + x ) =C0 + C1 x + C2 x2 + ...... + Cn xn i.e. (1 + x ) =∑ Cr xr
n n

r =0
n
Also, (1 + x ) = C0 xn + C1 xn−1 + C2 xn−2 + ...... + Cr xn−r + ...... + Cn−1 x + Cn i.e. (1 + x ) =
n n
∑ Cr xn−r
r =0
The binomial coefficients C0, C1, C2,……..Cn–1, and Cn posses the following properties:

Property-I In the expansion of (1 + x)n, the coefficients of terms equidistant from the beginning and the end
are equal.

Property-II The sum of the binomial coefficients in the expansion of (1 + x)n is 2n.
n
i.e. C0 + C1 + C2 + ……. + Cn = 2n or, ∑ n Cr = 2n .
r =0
4 . 6 | Binomial Theorem

Property-III The sum of the coefficient of the odd terms in the expansion of (1 + x)n is equal to the sum of the
coefficient of the even terms and each is equal to 2n–1.
i.e. C0 + C2 + C 4 + ........ = C1 + C3 + C5 + ....... = 2n−1

n n n n − 1 n− 2
Property-IV Cr =⋅ n−1 Cr −1 =⋅ ⋅ Cr −2 and so on.
r r r −1

C0 − C1 + C2 − C3 + C 4 − ...... + ( −1 ) Cn =0
n
Property-V
n
∑ ( −1 )
r n
i.e. Cr =
0
r =0

PLANCESS CONCEPTS
n n+1
Cr Cr +1
(a) (n+1)
Cr = Cr + Cr – 1
n n
(b) r Cr = n
n n –1
Cr – 1 (c) r + 1 =
n+1

(
(d) When n is even, ( x + a) + ( x − a) = 2 xn + n C2 xn−2a2 + n C 4 xn− 4 a4 + ..... + n Cnan
n n
)
(
When n is odd, ( x + a) + ( x − a) = 2 xn + n C2 xn−2a2 + ..... + n Cn−1 x an−1
n n
)
When n is even ( x + a) − (=x − a) 2( C x )
n n n n−1
1 a + n C3 xn−3a3 + ..... + n Cn−1 x an−1

When n is odd ( x + a) − (=x − a) 2( C x )


n n n n−1
1 a + n C3 xn−3a3 + ..... + n Cn an

Saurabh Gupta (JEE 2010, AIR 443)

Illustration 6: If (1 + x)n = C0 + C1 x + C2 x2 + ........... + Cn xn , then show that  (JEE MAIN)

(i) C0 + 4C1 + 42 C2 + ........... + 4n Cn =


5n

(ii) C0 + 2C1 + 3C2 + ........... + (n + 1)C


=n 2n−1 (n + 2)

C1 C2 C3 Cn
1
(iii) C0 − + − + ......... + ( −1)n =
2 3 4 n+1 n+1

Sol: By using properties of binomial coefficients and methods of summation, differentiation, and integration we
can easily prove given equations.
(i) (1 + x)n = C0 + C1 x + C2 x2 + .......... + Cn xn

Putting x=4, we have C0 + 4C1 + 42 C2 + ........... + 4n Cn =


5n

(ii) C0 + 2C1 + 3C2 + ........... + (n + 1)C


=n 2n−1 (n + 2)

Method 1: By Summations
rth term in the series is given by (r + 1 ) . n Cr
n
Therefore, L.H.S = n C0 + 2.n C1 + 3.n C2 + ........ + (n + 1). n C
=n ∑ (r + 1). n Cr
r =0
M a them a tics | 4.7

n n n n
= ∑ r. n Cr + ∑ n Cr = n∑ n−1 Cr −1 + ∑ n Cr = n.2n−1 + 2=
n
2n−1 (n + 2) = R.H.S
=r 0=r 0 =r 0 =r 0

Method 2: By Differentiation

(1 + x)n = C0 + C1 x + C2 x2 + ....... + Cn xn
3
Multiplying x on both sides, x(1 + x)n = C0 x + C1 x2 + C2 x + ....... + Cn xn+1

On differentiating, we have (1 + x ) + xn (1 + x ) = C0 + 2.C1 x + 3.C2 x2 + ...... + (n + 1 ) Cn xn


n n−1

Putting x = 1, we get C0 + 2.C1 + 3.C2 + ...... + (n + 1 ) Cn = 2 n + n.2 n−1

C0 + 2.C1 + 3.C2 + ...... + (n + 1 ) =


Cn 2n−1 (n + 2 )

C1 C2 C3 Cn 1
(iii) C0 − + − + ......... + ( −1)n =
2 3 4 n+1 n+1
Method 1: By Summations
n
Cr
rth term in the series is given by ( −1 ) .
r

r +1
n
C1 C2 C3 Cn n C
+ ...... + ( −1 ) .
n
∑ ( −1 ) . r + 1r
r
Therefore, L.H.S. = C0 − + − =
2 3 4 n+1 r =0

1 n  n+1 n  1  n+1
C3 − ........ + ( −1 ) . n+1 Cn+1 
n
( −1 ) n+1 Cr +1
r
= ∑ using
n+1
=
. Cr = Cr +1  C1 − n+1 C2 + n+1
n + 1 r =0 r +1 n + 1 
 
 

Adding and subtracting the term n+1 C0 , we have

1  n+1
C1 − n+1 C2 + ........ + ( −1 ) . n+1 Cn+1 + C0 
n+1 n n+1
= − C0 +
n + 1  

1
as  − n+1 C0 + C1 − n+1 C2 + ........ + ( −1 ) . n+1 Cn+1 =0  = R.H.S.
n+1 n
=  
n+1

Method 2: By Integration
(1 + x)n = C0 + C1x + C2x2 + ……… + Cnxn.
On integrating both sides within the limits –1 to 0, we have
0 0

∫ (1 ) dx ∫ ( C0 + )
n
+ x= C1 x + C2 x2 + ……… + Cn xn dx
−1 −1
0
 1 + x n+1 
 ( )  =  x2 x3 xn+1 
0

⇒ C0 x + C1 + C2 + ...... + Cn 
 n+1   2 3 n + 1 
  −1 −1

1  C C n+1 Cn  C1 C2 n Cn 1
⇒ − 0 = 0 −  −C0 + 1 − 2 + ...... + ( −1 )  ⇒ C0 − + + ...... + ( −1 ) =
n+1  2 3 n + 1 2 3 n+1 n+1

Illustration 7: If (1 + x ) = C0 + C 1 x + C2 x2 + ...... + Cn xn , then prove that


n

(i) C20 + C12 + C22 + ...... + Cn2 =


2n
Cn
4 . 8 | Binomial Theorem

2n 2n
(ii) C0 C2 + C1C3 + C2C 4 + ...... + Cn−2Cn = Cn−2 or Cn+ 2

(iii) 1 . C20 + 3. C12 + 5 . C22 + ...... + ( 2n + 1 ) =


. Cn2 . 2n . 2n−1 Cn + 2n Cn (JEE ADVANCED)


Sol: In the expansion of (1+x)2n, (i) and (ii) can be proved by comparing the coefficients of xn and xn-2 respectively.
The third equation can be proved by two methods - the method of summation and the methods of differentiation.

(i) (1 + x ) =C0 + C1 x + C2 x2 + ........ + Cn xn 


n
….(i)

Also, ( x + 1 ) = C0 xn + C1 xn−1 + C2 xn−2 + ........ + Cn x0 


n
…. (ii)

Multiplying equation (i) and (ii)

(1 + x )
2n
(
= C0 + C1 x + C2 x2 + ...... + Cn xn ) (C x 0
n
+ C1 xn−1 + ....... + Cn x0  ) …. (iii)

On comparing the coefficients of xn both sides, we have


2n
⇒ Cn = C02 + C12 + C22 + ....... + Cn2 Hence, Proved.

(ii) From (iii), on comparing the coefficients of xn – 2 or xn + 2, we have


2n 2n
C0 C1 + C1C3 + C2C 4 + ....... + Cn−2Cn = Cn−2 or Cn+ 2

(iii) 1 . C20 + 3. C12 + 5 . C22 + ...... + ( 2n + 1 ) =


. Cn2 . 2n . 2n−1 Cn + 2n Cn

Method 1: By Summation
rth term in the series is given by ( 2r + 1 ) n Cr2
n
L.H.S. = 1.C20 + 3.C12 + 5.C22 + ...... + ( 2n + 1 ) Cn2
= ∑ ( 2r + 1 ) n Cr2
r =0
n n n
∑ 2.r. ( n Cr ) + ∑ ( n Cr )= 2∑ .n. n−1 Cr −1 n Cr + 2n Cn
2 2
=
=r 0 =r 0=r 1

(1 + x )
n n
= C0 + n C1 x + n C2 x2 + ....... + n Cn xn  …. (i)

( x + 1=
)
n−1 n−1
C0 xn−1 + n−1
C1 xn−2 + ....... + n−1
Cn−1 x0  …. (ii)

Multiplying (i) and (ii) and comparing coefficients of xn, we have


2n−1 n−1
Cn
= C0 . n C1 + n−1
C1 . n C2 + ....... + n−1
Cn−1 . n Cn
n
i.e. ∑ n−1 Cr −1 n Cr = 2n−1 Cn
r =1

Hence, required summation is 2n. 2n–1Cn + 2nCn

Method 2: By Differentiation

(1 + x )
n
2
= C0 + C1 x2 + C2 x 4 + C3 x6 + ...... + Cn x2n

Multiplying x on both sides

( )
n
x 1 + x2 = C0 x + C1 x3 + C2 x5 + ...... + Cn x2n+1
M a them a tics | 4.9

Differentiating both sides

( ) ( )
n−1 n
x.n 1 + x2 .2x + 1 + x2 = C0 + 3.C1 x2 + 5.C2 x 4 + ...... + ( 2n + 1 ) Cn x2n  …. (i)

(x )
n
2
+ 1 = C0 x2n + C1 x2n−2 + C2 x2n− 4 + ....... + Cn  …. (ii)

On multiplying (i) and (ii), we have

( ) ( ) = (C )( )
2n−1 2n
2nx2 1 + x2 + 1 + x2 0 + 3C1 x2 + 5C2 x 4 + ..... + ( 2n + 1 ) Cn x2n C0 x2n + C1 x2n−2 + ...... + Cn

Comparing coefficient of x2n,

2n.2n−1 Cn−1 +2n Cn = C20 + 3C12 + 5C22 + ..... + ( 2n + 1 ) Cn2

∴ C20 + 3C12 + 5C22 + ..... + ( 2n + 1=


) Cn2 2n.2n−1 Cn +2n Cn

Illustration 8: If (1 + x ) = C0 + C1 x + C2 x2 + ...... + Cn xn ,
n

2n!
Prove that C0 Cr + C1Cr +1 + C2Cr + 2 + ........ + Cn−r Cn = (JEE MAIN)
(n − r ) ! (n + r ) ! 

Sol: Clearly the differences of lower suffixes of binomial coefficients in each term is r.
By using properties of binomial coefficients we can easily prove given equations.
(1 + x )
n
Given =C0 + C1 x + C2 x2 + ........ + Cn−r xn−r + ......... + Cn xn  …. (i)

Now ( x + 1 ) = C0 xn + C1 xn−1 + C2 xn−2 + ........ + Cr xn−r + Cr +1 xn−r −1 + ......... + Cn 


n
….(ii)

Multiplying (i) and (ii), we get

( x + 1)
2n
= (C 0 + C1 x + C2 x2 + ..... + Cn−r xn−r + ......... + Cn xn )
(
× C0 xn + C1 xn−1 + C2 xn−2 + ..... + Cr xn−r + Cr +1 xn−r −1 + Cr + 2 xn−r −2 + ...... + Cn  ) …. (iii)

2n 2n!
Now coefficient of xn–r on L.H.S. of (iii) = Cn−r =
(n − r ) ! (n + r ) !
and coefficient of xn–r on R.H.S. of (iii) = C0 Cr + C1Cr +1 + C2Cr + 2 + ........ + Cn−r Cn

But (iii) is an identity, therefore, of xn–r in R.H.S. = Coefficient of xn–r in L.H.S.


2n!
⇒ C0 Cr + C1Cr +1 + C2Cr + 2 + ........ + Cn−r Cn =
( ) ! (n + r ) !
n − r

Hence, Proved.

Illustration 9: Prove that n C0 ⋅ 2n Cr − n C1 ⋅ 2n−2 Cr + ....... =


2 2n−r . n Cr −n if r > n and 0 if r < n.  (JEE MAIN)
n
Sol: By comparing coefficient of xr in L.H.S. and R.H.S. in the expansion of (1 + x ) − 1  we can prove it.
2

n
 
 1 + x 2 − 1= n C 1 + x 2n − n C 1 + x 2n−2 + n C 1 + x 2n− 4 + ... 
( )  0( ) 1( ) 2( ) …. (i)

Coefficient of xr in R.H.S. = n C0 . 2n Cr − n C1 .2n−2 Cr + .............


 … (ii)
L.H.S. = [(1 + x)2 –1]n =[2x + x2]n = xn (2 + x)n
4 . 1 0 | Binomial Theorem

∴ Coefficient of xr in xn (2 + x)n
= Coefficient of xr–n in (2 + x)n = nCr–n 22n–r if r > n
= 0 if r < n (Since lower suffix cannot be negative)
But (i) is an identity, therefore coefficient of xr in R.H.S. = coefficient of xr in L.H.S.

Hence n C0 . 2n Cr − n C1 .2n−2 Cr + ...... = n Cr −n 22n−r if r > n

= 0 if r < n.

Illustration 10: Show that C0 . 2n Cn − C1 .2n−1 Cn + C2 .2n−2 Cn − C3 .2n−3 Cn + ...... + ( −1 ) Cn .n Cn =1


n
(JEE ADVANCED)

Sol: Observe the pattern in the terms on the LHS. The first term C0 . 2n Cn is the co-efficient of xn in the expansion

of C0 (1 + x ) . Similarly, C1 . 2n−1 Cn is the co-efficient of xn in C0 (1 + x )


2n 2n
and so on. On adding all the coefficients

of xn we can prove the given equation.

Note that C0 . 2n Cn − C1 .2n−1 Cn + C2 .2n−2 Cn − C3 .2n−3 Cn + ...... + ( −1 ) nCn .n Cn

= Coefficient of xn in C0 (1 + x ) − C1 (1 + x ) + C2 (1 + x ) − C3 (1 + x ) + .... ( −1 ) Cn (1 + x ) 


2n 2n−1 2n−2 2n−3 n n

 

= Coefficient of xn in (1 + x ) C0 (1 + x ) − C1 (1 + x ) + C2 (1 + x ) − C3 (1 + x ) + ..... ( −1 ) Cn 


n n n−1 n− 2 n−3 n

 
n
= Coefficient of xn in (1 + x ) (1 + x ) − 1 
n

= Coefficient of xn in (1 + x ) ( x )
n n

= Coefficient of the constant terms in (1 + x)n = 1

3. TERMS IN BINOMIAL EXPANSION

3.1 General Term in Binomial Expansion

We have, ( x +=
a)
n n
C0 xna0 + n C1 xn−1 a1 + n C2 xn−2 x2 + ...... + n Cr xn−r ar + ...... + n Cn x0 an

(r+1)th term is given by n Cr xn−r ar


Thus, if Tr +1 denotes the (r+1)th term, then Tr +1 = n Cr xn−r ar
This is called the general term of the binomial expansion.
(a) The general term in the expansion of (x – a)n, is given by Tr+1 = (–1)r.nCrxn-rar
(b) The general term in the expansion of (1 + x)n, is given by Tr + 1 = nCr xr
(c) The general term in the expansion of (1 – x)n , is given by Tr + 1 = (–1)r nCrxr
(d) In the binomial expansion of (x + a)n, the rth term from the end is ((n + 1) – r + 1)th term i.e. (n –r + 2)th term
from the beginning.

Illustration 11: The number of dissimilar terms in the expansion of (1 – 3x + 3x2 – x3)20 is (JEE MAIN)

Sol: As we know that number of dissimilar terms in the expansion of (1 – x)n is n+1. Rewrite the given expression
in the form of (1 – x)n.
M a them a ti cs | 4.11

(1 – 3x + 3x2 – x)20 = [(1 – x)3]20 = (1 – x)60


Therefore number of dissimilar terms in the expansion of (1 – 3x + 3x2 – x3)20 is 61.

9
 4x 5 
Illustration 12: Find (i) 28 term of (5x + 8y) (ii) 7 term of 
th
−  
30 th
(JEE MAIN)
 5 2x 
Sol: Here in this problem, by using Tr +1 = n Cr xn−r ar we can easily obtain (r+1)th term of given expansion.
(i) 28th term of (5x + 8y)30
30!
( 5x ) . ( 8y )
3 27
T28 = T27 + 1 = 30C27 (5x)30–27 (8y)27 =
3!.27!
9
 4x 5 
(ii) 7th term of  − 
 5 2x 
9 −6 6 3 6
 4x   5  9!  4x   5  10500
T7 = T6+1 = 9 C6  =  −  =    
5
   2x  3!6! 5
   2x x3

Illustration 13: Find the number of rational terms in the expansion of (91/4 + 81/6)1000. (JEE ADVANCED)

Sol: In this problem, by using Tr +1 = n Cr xn−r ar we can easily obtain (r+1)th term of given expansion and after that by
using the conditions of rational number we can obtain number of rational terms.
The general term in the expansion of (91/4 + 81/6)1000 is
1000 −r
 1 1000 −r r

( )
r
1000
Tr +1 Cr  9 4 
= 81/6 1000
Cr 3 2 22
 
 
1000 − r r
Tr +1 will be rational if the power of 3 and 2 are integers. It means and must be integers.
2 2
Therefore the possible set of values of r is {0, 2, 4... … 1000}. Hence, number of rational terms is 501.

3.2 Middle Term in Binomial Expansion

(a) If n is even, then the number of terms in the expansion i.e. (n + 1) is odd, therefore, there will be only one
th th
n+2 n 
middle term which is   term i.e.  + 1  term.
 2   2 
th n n
n 
So middle term =  + 1  term i.e. T n 
= n C n x 2 a2
2   +1 
2
2 

(b) If n is odd, then the number of terms in the expansion i.e. (n+1) is even, therefore there will be two middle
terms which are
th th n+1 n−1 n−1 n+1
n+1 n+3
=  and   term i.e. T n+1  = n C n−1  x 2 a 2 and T n+3  = n C n+1  x 2 a 2

 2   2         
 2   2   2   2 
4 . 1 2 | Binomial Theorem

PLANCESS CONCEPTS

•• When there are two middle terms in the expansion then their binomial coefficients are equal.
•• Binomial coefficient of middle term is the greatest Binomial coefficient.
Neeraj Toshniwal (JEE 2009, AIR 21)

14 9
 x2   a3 
Illustration 14: Find the middle term(s) in the expansion of (i)  1 −  (ii)  3a −   (JEE MAIN)
 2   6 
 
th th th
n  n+1 n+3
Sol: By using appropriate formula of finding middle term(s) i.e.  + 1  when n is even and   and  
 2   2   2 
when n is odd, we can obtain the middle terms of given expansion.
14 th
 x2   14 
(i)  1 −  Since, n is even, therefore middle term is  + 1 term.
 2   2 

7
14
 x2  429 14
∴ T8 = Cc7  −  =
14
− x
 2  16
 
9
 a3 
(ii)  3a − 
 6 

th th
9 +1 9 +1 
Since, n is odd therefore, the middle terms are   and  + 1 .
 2   2 
4 5
 a3  189 17  a3  21
C 4 ( 3a) C5 ( 3a)
9 9−4 9 9 −5
∴ T5
=  −=  a and T6 = −  = − a19 .
 6 8  6 16
   

3.3 Determining a Particular Term


n
 1 
In the expansion of  x α ±  , if xm occurs in Tr + 1, then r is given by
 xβ 
nα − m
nα − r ( α + β ) =m ⇒ r =
α+β
Thus in above expansion if constant term i.e. the term independent of x, occurs in Tr + 1 then r is determined by

nα − r ( α + β ) = 0 ⇒ r =
α+β

9
4 3 
Illustration 15: The term independent of x in the expansion of  x2 −  is (JEE MAIN)
3 2x 

Sol: By using the result proved above i.e. r = , we can obtain the term independent of x. Here, α and β are
α+β
n
 1 
obtained by comparing given expansion to  x α ±  .
 xβ 
9 n
4 3   1 
On comparing  x2 −  with  x α ±  , we get α= 2, β= 1, n= 9
3 2x   xβ 
M a them a ti cs | 4.13

9 (2)
i.e. r = = 6 ∴ (6 + 1) = 7th term is independent of x.
2+1

15
 2
Illustration 16: The ratio of the coefficient of x15 to the term independent of x in  x2 +  is (JEE MAIN)
 x

Sol: Here in this problem, by using standard formulas of finding general term and term independent of x we can
obtain the required ratio.
r

( ) 2
15 −r
15 2 15 30 −3r r
General term in the expansion is Tr +1 = Cr x   i.e., Cr x .2
x
For x15, 30 – 3r = 15 ⇒ 3r = 15 ⇒ r=5
5

( ) 2
15 −5
15
∴ T6 = T5+1 = C5 x2 15 15 5
  i.e., C5 x .2
x
 
∴ Coefficient of x15 is 15C5 25 (r = 5)

For the constant term 30 – 3r = 0 ⇒ r = 10.


10

( ) 2
15 −10
15
T11 T=
∴= 10 +1 C10 x2 15
  i.e., C10 2
10
x
 
∴ Coefficient of constant term is 15C10210.

Hence, the required ratio is 1 : 32.


9
 1 
Illustration 17: The term independent of x in the expansion of  6 x −  is equal to  (JEE MAIN)
3
 x
Sol: By using the formula Tr +1 = n Cr xn−r ar we can solve it.
r 9 −r r  9 −3r 

( )  1 
9 −r −  
= 9 Cr ( −1 ) x
r
Cr ( −1 ) x
9 6 9 r 6 
=Tr +1 Cr x − 3  6 3=
 x
9 − 3r
⇒ 0 ⇒r =
= 3 ∴ T4 =T3+1= – 9 C3
6

Illustration 18: If the second, third and fourth terms in the expansion of (b+a)n are 135, 30 and 10/3 respectively,
then n is equal to (JEE MAIN)

Sol: In this problem, by using the formula of finding general term we will get the equation of given terms and by
taking ratios of these terms we can get the value of n.
T2 = nC1 abn–1 = 135  ...(i)
T3 = nC2 a2bn–2 = 30  ...(ii)
10
T4 = nC3 a3bn–3 =  ...(iii)
3
On dividing (i) by (ii), we get
n
C1abn−1 135 n b 9
= ⇒ = …(iv)
n 2 n− 2
C2 a b 30 n(n − 1) a 2
2
b 9
∴ =
a 4
(n − 1)  ...(v)
4 . 1 4 | Binomial Theorem

Dividing (ii) and (iii), we get


n (n − 1 )
2 b 30 × 3 3 b
= . = 9 ⇒ 9 
= ...(vi)
n (n − 1 )(n − 2 ) a 10 (n− 2) a
3.2
Eliminating a and b from (v) and (vi) ⇒ n = 5

Illustration 19: If a, b, c and d are the coefficients of any four consecutive terms in the expansion of (1+x)n, n being
a c 2b
positive integer, show that + = (JEE MAIN)
a + b c + d b + c 

Sol: Consider four consecutive terms and use n Cr −1 + n Cr =


n+1
Cr .
The (r + 1)th term is Tr+1 = nCrxr
∴ The coefficient of term Tr + 1 =nCr
∴ Now take four consecutive terms as (r – 1)th, rth, (r + 1)th and (r + 2)th
n n n n
∴ We get
= a C=
r −2 , b C=
r −1 , c Cr , d
= Cr +1
n
a +=
b Cr −2 + n Cr=
−1
n+1
Cr −1
n
b +=
c Cr −1 + n =
Cr n+1
Cr
n
c + d= Cr + n Cr +1= n+1
Cr +1


a
=
n
Cr −2
=
n!
×
(r − 1)! (n − r + 2)=! r − 1
a+b n+1
Cr −1 (r − 2 ) ! (n − r + 2 ) ! (n + 1 ) ! n+1

b
n
Cr −1 n! r! (n − r + 1 ) ! r
= = × =
b+c n +1
Cr (r − 1 ) ! (n − r + 2 ) ! (n + 1 ) ! n + 1
c
n
=r =
C n!
×
(r + 1 ) ! (n − r ) ! =
r +1
c + d n+1 C
r +1
n! ( )
n − r ! ( )
n + 1 ! n +1

a c r −1 r +1 2r  r  2b
∴ + = + = = 2 =
a+b c + d n+1 n+1 n+1 n+1 b + c

3.4 Finding a Term from the End of Expansion


In the expansion of (x + a)n, (r + 1)th term from end = (n – r + 1) th term from beginning i.e. Tr +1 (E ) = Tn−r +1 (B )

∴ Tr (E ) =
Tn−r + 2 (B )

Illustration 20: The 4th term from the end in the expansion of (2x – 1/x2)10 is (JEE MAIN)

Sol: By using Tr (E) = Tn-r+2(B) we will get the fourth term from the end in the given expansion.
7
3 1 
Required term = T10 − 4 + 2 = T8 = 10
C7 ( 2x )  −  = – 960 x–11
2
 x 
M a them a ti cs | 4.15

3.5 Greatest Term in the Expansion


Let Tr+1 and Tr be (r+1)th and rth terms respectively in the expansion of (x+a)n. Then, Tr +1 = n Cr xn−r ar and

Tr = n Cr −1 xn−r +1ar −1 .


Tr +1
= r
n
C xn−r ar
=
n!
x
(r − 1 ) ! (n − r + 1 ) ! . a =
n−r +1 a
.
Tr n
Cr −1 xn−r +1ar −1 (n − r ) !r! n! x r x

Tr +1 n−r +1 a  n + 1   a
Now, Tr +1 > = < Tr ⇒ > = < 1 ⇒ . >=< 1 ⇒   − 1 > = < 1
Tr r x  r  
 x

n+1 x n+1  x n+1


⇒ −1 > =< ⇒ > = < 1 +  ⇒ >=< r
r a r  a x
1+
a
 
 n+1 
Thus, Tr +1 > = < Tr according as   > = < r  ...(i)
1 + x 
 
 a
Now, two cases arise

n+1 n+1
Case-I: When is an integer Let = m , Then, from (i), we have
x x
1+ 1+
a a
Tr +1 > Tr , for =
r 1,2,3,.....(m − 1)  ….(ii)

Tr +1 T=
= r , for r m …(iii)

and, Tr +1 < Tr , for r =


m + 1,....n  ….(iv)

∴ T2 > T1 , T3 > T2 , T4 > T3 ,......Tm > Tm−1 [From (ii)]

Tm + 1 = Tm [From (iii)]

and, Tm+ 2 < Tm+1 , Tm+3 < Tm+ 2 , Tn+1 < Tn [From (iv)]

⇒ T1 < T2 < ..... < Tm−1 < Tm= Tm+1 > Tm+ 2 .... > Tn

This shows that mth and (m + 1)th terms are greatest terms.

n + 1 
Case-II: When   = m . Then, from (i), we have
 1 + ax 

Tr +1 > Tr for r =
1,2,.....,m  ….(v)

and Tr +1 < Tr for r =m + 1,m + 2,.....,n  ….(vi)

∴ T2 > T1 , T3 > T2 ,......, Tm+1 > Tm [From (v)]

and, Tm+ 2 < Tm+1 , Tm+3 < Tm+ 2 ,......, Tn+1 < Tn [From (vi)]

⇒ T1 < T2 < T3 < ..... < Tm < Tm+1 > Tm+ 2 > Tm+3 ...... < Tn+1
4 . 1 6 | Binomial Theorem

⇒ (m + 1)th term is the greatest term.

Following algorithm may be used to find the greatest term in a binomial expansion.

3.6 Algorithm to Find Greatest Term

Step I: From the given expansion, get Tr+1 and Tr


Tr +1
Step II: Find
Tr
Tr +1
Step III: Put >1
Tr

Step IV: Simplify the inequality obtained in step III, and write it in the form of either r < m or r > m.
Step V: If m is an integer, then mth and (m+1)th terms are the greatest terms and they are equal.
If m is not an integer, then ([m]+1)th term is the greatest term, where [m] means the integral part of m.

3.7 Greatest Coefficient

Case-I When n is even, we have

nC
r = ×
n! (r + 1 ) ! (n − r − 1 ) ! =
r +1
 ….(i)
nC (n − r )!r! n! n−r
r +1
n n n
Now, for 0 ≤ r ≤ − 1 ⇒ 1 ≤ r + 1 ≤ and + 1 < n − r ≤ n
2 2 2
n
r +1 Cr
⇒ < 1 [Using (i)] ⇒ < 1 ⇒ nCr < n Cr +1
n−r n
Cr +1
n 
Putting
= r 0,1,2,......,  − 1  , we get n C0 < n C1 , n C1 < n C2 , n C2 < n C3 ..... < n C n < n C n
2  2
−1
2

⇒ n C0 < n C1 < n C2 < ..... < n C n < n C n  ...(ii)


−1
2 2
n n
Since Cn−r = Cr

∴ n=
C0 n
Cn , n=
C1 n
Cn−1 , n=
C2 n
Cn−2 ,......, n C n < nCn
−1
2 2

Substituting these values in (ii), we get


 ...(iii)
n
Cn < n Cn−1 < n Cn−2 < ..... < n C n < nCn
+1
2 2

From (ii) and (iii), we refer that the maximum value of nCr is nCn/2.

Case-II When n is odd


n
Cr r +1
We have, =  ...(i)
n
Cr +1 n−r

n−3 n−1 n−1


Now, 0 ≤ r < ⇒ 0 < r +1 < and ≤ n−r ≤ n
2 2 2
M a them a ti cs | 4.17

n
r +1 Cr n
⇒ <1⇒ < 1 [Using (i)] ⇒ Cr < nCr +1
n−1 n
Cr +1

Putting r = 0, 1, 2, ……., n − 3
2
We get n C0 < n C1 , n C1 < n C2 , n C2 < n C3 ,....., < n C n−3 < n C n−1 =
n
C n+1
2 2 2
n n n n n n n
⇒ C0 < C1 < C2 < C3 < ..... < C n−3 < C n−1 =C n−1
2 2 2  ...(ii)
Since nCn–r = nCr. Therefore,

∴ n=
C0 n
Cn , n=
C1 n
Cn−1 , n=
C2 n
Cn−2 ,....., n C n=
−1
n
C n+1  …(iii)
2 2

From (ii) and (iii), it follows that the maximum value of nCr is n C n−1 = n C n+1
2 2

1
Illustration 21: Find the numerically greatest term in the expansion of (3 – 4x)15, when x = . (JEE MAIN)
4
Sol: Follow the algorithm mentioned above.
Let rth and (r + 1) th be two consecutive terms in the expansion of (3 – 4x)15
Tr +1 > Tr

15 −(r −1 )
( ) ( −4x )
15 r r −1
Cr 315−r −4x > 15
Cr −1 3

(15) ! −4x > 3. (15!) 1


⇒ 5. (16 − r ) > 3r ⇒ 16 − r > 3r
(15 − r ) !r! (16 − r ) ! (r − 1) ! 5

⇒ 4r < 16 ⇒r<4

Hence, we have T1 < T2 < T3 < T4 .


Similarly, if we simplify Tr +1 = Tr , we get r=4.
Therefore the numerically greatest term is T4 and T5 .

4. APPLICATION OF BINOMIAL THEOREM

4.1 Divisibility Test

Illustration 22: Show that 72n + 7 is divisible by 8, where n is a positive integer.  (JEE MAIN)

Sol: Write 72n + 7 in the form of 8λ + c, where c is a constant. If c = 0 then we can conclude that 72n + 7 is divisible
by 8.
2n
72n + 7 = (8 - 1)2n + 7 = C0 82n − 2n
C1 .82n−1 + 2n
C2 .82n−2 − ...... + 2n
C2n + 7

= 82n.2n C0 − 82n−1.2n C1 + ...... − 8.2n C2n−1 + 8 = 8λ where λ is a positive integer

Hence, 72n + 7 is divisible by 8.


4 . 1 8 | Binomial Theorem

Illustration 23: Prove that 1399 ‒ 1957 is divisible by 162.  (JEE ADVANCED)

Sol: Reduce 1399 − 1957 into the form of 162λ + C using binomial expansion and If C = 0 then 1399 − 1957 is
divisible by 162.
Let the given number be called S. Hence, S = 1399 –1957 = (1 + 3×4)99 – (1 + 9×2)57

{
S = 1+ 99
C1 . ( 3 × 4 ) + 99
C2 . ( 3 × 4 ) +
2 99
C3 . ( 3 × 4 ) + ...... +
3 99
C99 . ( 3 × 4 )
99
}
 – {1 + C1 . ( 9 × 2 ) + 57 C2 . ( 9 × 2 ) + 57 C3 . ( 9 × 2 ) + ...... + 57 C57 . ( 9 × 2 ) }
57 2 3 57

S = 1+ { 99
( ) } {
C1 . ( 3 × 4 ) + 34 × 2 k1 − 1 + 57 C1 . ( 9 × 2 ) + 34 × 2 k 2 ( ) }
All terms like { 99
C1 . ( 3 × 4 ) , 99 C2 . ( 3 × 4 ) ,......, 99 C99 . ( 3 × 4 )
2 3 99
} and
{ 57
C2 . ( 9 × 2 ) , 57 C3 . ( 9 × 2 ) ,......, 57 C57 . ( 9 × 2 )
2 3 57
} have a common factor of (3 .2 = 162) . 4

Hence they can be written as (34.2) k1 and (34.2) k2 respectively, where k1 and k2 are integers.

Therefore, S = 1 + 99
C0 . ( 3 × 4 ) − 1 − 57 C1 . ( 9 × 2 ) + (162 )(k1 − k 2 )

{
= (1188 − 1026 ) + 162 × (k1 − k 2 ) = (162 × some integer) }
Hence the given number S is exactly divisible by 162.

4.2 Finding Remainder

Illustration 24: What is the remainder when 52015 is divisible by 13.  (JEE MAIN)

Sol: In this problem, we can obtain required remainder by reducing 52015 into the form of 13 λ +a, where λ and a
are integers.
52015 = 5.52014 = 5.(25)1007

5  1007 C0 ( 26 ) − 1007 C1 ( 26 ) + ...... + 1007 C1006 ( 26 ) − 1007 C1007 ( 26 ) 


1007 1006 1 0
( 26 − 1)
1007
= 5=  

5  1007 C0 ( 26 ) − 1007 C1 ( 26 ) + ...... + 1007 C1006 ( 26 ) − 1


1007 1006 1
=
 

5  1007 C0 ( 26 ) − 1007 C1 ( 26 ) + ...... + 1007 C1006 ( 26 ) − 13 + 60


1007 1006 1
=  

= 13 (k ) + 52 + 8 = 13 × (some integer) + 8.

4.3 Finding Digits of a Number

Illustration 25: Find the last two digits of the number (13)10. (JEE MAIN)

Sol: Write (13)10 in the form of ( x − 1 ) , such that x is a multiple of 10. Then using expansion formula we will get
n

last two digits.

(170 −=
1) C0 (170 ) − 5 C1 . (170 ) + ....... + 5 C 4 (170 ) − 5 C5 (170 )
5 5 5 4 1 0
(13)10 = (169)
= 5

M a them a ti cs | 4.19

C0 (170 ) − 5 C1 . (170 ) + ... + 5 C3 (170 ) + 5 × 170 − 1 = A multiple of 100 + 849


5 5 4 2

=

Therefore, the last two digits are 49

Illustration 26: Find the last three digits of 13256. (JEE MAIN)

Sol: Similar to above problem..


We have 132 = 169 = 170 – 1

(13 =
)
128
(170 − 1)
128
132
Now, = 2

C0 (170 ) − 128 C1 . (170 ) + 128 C2 . (170 ) − ... + 128 C126 (170 ) − 128 C127 (170 ) + 1
128 128 127 126 2
=

= 1000 m + (128) (170) (10794) + 1 (where m is a positive integer)

= 1000 m + 234877440 + 1 = 1000m+234877441

Thus, the last three digits of 13256 are 441.

4.4 Relation between Two Numbers

Illustration 27: Which number is smaller (1.01)1000000 or 10,000  (JEE MAIN)

Sol: By reducing (1.01)1000000 into the form of (1 + 0.01 ) and solve it by using expansion formula we can obtain the
n

value of (1.01)1000000 .
(1.01)1000000 = (1 + 0.01)1000000
1 + 1000000 C1 ( 0.01 ) + 1000000 C2 ( 0.01 ) + 1000000 C3 ( 0.01 ) + ...
2 3
=
1 + 1000000 × (0.01) + some positive terms
=
= 1 + 10000 + some positive terms
Hence 10,000 < (1.01)1000000.

5. MULTINOMIAL THEOREM
Using binomial theorem, we have
n
( x + a)
n
= ∑ n Cr xn−r ar , n∈N
r =0
n
n! n! s r
= ∑ (n − r ) !r! xn−r ar = ∑ xa, wheres= n − r
=r 0 =r +s n r!s!

Let us now consider the expansion of ( x1 + x2 + x3 )


n

n n  k 
( x1 =
+ x 2 + x3 ) ( x2 + x3 ) ∑ (n− n! k!
n k
∑ n Ck x1n−k= k)! k!
x1n−k  ∑

xk −p xp 
(k − p)! p! 2 3 
=k 0 =k 0=p 0 
n k
n! n!
= ∑ ∑ xn−k xk −p xp
(n− k)! (k − p)! p! 1 2 3
= ∑ xr xq xp where, k - p = q, n - k = r.
r! q! p! 1 2 3
=k 0=p 0 p + q+r =n
4 . 2 0 | Binomial Theorem

And so on, if we want to generalize for n terms, we get


n!
( x1 + x2 + ..... + xk )
n r r r
= ∑ x11 x22 .......xkk
r1 +r2 +... +rk n r1 !r2 !.....rk !
=

n!
Therefore, general term in the expansion of ( x1 + x2 + ..... + xk ) is
n r r r r
x 1 x 2 x 3 .......xkk
r1 !r2 !r3 !.....rk ! 1 2 3

The number of terms is equal to the number of non-negative integral solution of the equation r1 + r2 +………..+ rk = n,
because each solution of this equation gives a term in the above expansion. The number of such solutions is n + k –1Ck –1.

Number of terms for the following expansions


n! r s t
(x + y + z)
n
(a) =∑ x y z The above expansion has n+3−1
C3−1 = n+ 2
C2 terms.
r + s + t =n r!s!t!
n!
( x + y + z + u)
n
(b) = ∑ p!q!r!s!
xp y q zr us . There are n+ 4 −1
C 4 −1 = n+ 3
C3 term in the above.
p + q+r + s =n

PLANCESS CONCEPTS

The greatest coefficient in the expansion of x1 + x2 + ...... + xnm is ( ) n!


r
, where q and r are
( q!)  ( q + 1 ) !
m−r
the quotient and remainder respectively when n is divided by m.  

Aman Gour (JEE 2012, AIR 230)

6. BINOMIAL THEOREM FOR ANY INDEX


Let n be a rational number and x be a real number such that |x| < 1, then

n(n − 1) 2 n(n − 1)(n − 2)...(n − r + 1) r


(1 + x )
n
= 1 + nx + x + ... + x + ... + terms upto ∞
2! r!
n (n − 1 )(n − 2 ) ......... (n − r + 1 )
The general term in the expansion of (1 + x)n is xr and is represented by Tr+1.
r!

PLANCESS CONCEPTS

The above result is also true for complex x, n.


B Rajiv Reddy (JEE 2012, AIR 11)

Illustration 28: If x is very large and n is a negative integer or a proper fraction, then an approximate value of
n
1 + x 
  is equal to___________ (JEE MAIN)
 x 

1
Sol: Since x is very large therefore will be very small. Neglect the terms containing three and higher powers of
x n
1 1 + x 
in the expansion to obtain the approximate value of   .
x  x 
M a them a ti cs | 4.21

n n (n − 1 )  1 
n 2
 1
1 +  = 1 + +   + ...... Since x is very large, we can ignore terms after the 2nd term.
 x x 1.2  x 

(1 − 3x ) + (1 − x )
1/2 5/3

Illustration 29: If is approximately equal to a + bx for small values of x, then (a, b) is equals
to.  4−x (JEE MAIN)

(1 − 3x ) + (1 − x )
1/2 5/3

Sol: Calculate the value of and equate it to a + bx.


4−x
Using the binomial expansion for any rational index, we have

 1 1 11   5 521
1 + ( −3x ) +  −  ( −3x ) + ......  + 1 + ( −x ) + ( −x ) + ......
2 2

(1 − 3x ) + (1 − x )
1/2 5/3
2 2  2  2   3 3 3 2 
=
1/2  2 
 x 1 x  1 11 x 
2 1 −  2 1 +  −  +  −   −  + ........ 
 4  2 4 2 22 4 
 

 19 53 2 
1 − 12 x + 144 x − .......... 
 = 35
= 1− x + .......
 x 1 2  24
1 − 8 − 8 x − ....... 
 
35 35
Neglecting the higher powers of x, ⇒ a + bx = 1 − x ⇒ a =1,b =−
24 24

Illustration 30: Find the coefficient of a3b2c4d in the expansion of (a – b – c + d)10  (JEE ADVANCED)

Sol: Expand (a – b – c + d)10 using multinomial theorem and by using coefficient property we can obtain the
required result.
Using multinomial theorem, we have
(10 ) !
( a − b − c + d) ( a) 1 ( −b ) 2 ( −c ) 3 ( d) 4
10 r r r r
= ∑
10 r1 !r2 !r3 !r4
r1 +r2 +r3 +r4 = !

We want to get coefficient of a3b2c4, this implies that r1 = 3, r2 = 2, r3 = 4, r4 = 1


(10 ) !
( −1) ( −1)
2 4
∴ Coefficient of a3b2c4d is 12600
=
3!2! 4!

11
 5
Illustration 31: In the expansion of  1 + x +  find the term independent of x.  (JEE ADVANCED)
 x
11
 5
Sol: By expanding  1 + x +  using multinomial theorem and obtaining the coefficient of x0 we will get the term
independent of x.  x

 5
11
(11) ! r 5
3 r

∑ r !r !r ! (1) 1 ( x ) 2  x 
r
1 + x +  =
 x r1 + r2 + r3 =
11 1 2 3  

The exponent 11 is to be divided in such a way that we get x0. Therefore, possible set of values of (r1, r2, r3) are
(11, 0, 0), (9, 1, 1), (7, 2, 2) (5, 3, 3), (3, 4, 4), (1, 5, 5) Hence the required term is
4 . 2 2 | Binomial Theorem

(11) ! 50 + (11) ! 51 + (11) ! 52 + (11) ! 53 + (11) ! 54 + (11) ! 55


(11) ! ( ) 9!1!1! 7!2!2! 5!3!3! 3! 4! 4! 1!5!5!

1+
=
(11) ! . 2! 51 + (11) ! . 4! 52 + (11) ! . 6! 53 + (11) ! . 8! 54 + (11) ! . (10 ) ! 55
9!2! 1!1! 7! 4! 2!2! 5!6! 3!3! 3!8! 4! 4! 1!10! 5!5!

1 + 11 C2 × 2 C1 × 51 + 11 C 4 × 4 C2 × 52 + 11 C6 × 6 C3 × 53 + 11 C8 × 8 C 4 × 54 + 11 C10 × 10 C5 × 55
=
5
1 + ∑ 11 C2r . 2r Cr × 5r
=
r =1

PROBLEM-SOLVING TACTICS
Summation of series involving binomial coefficients
For (1 + x ) = n C0 + n C1 x + n C2 x2 + ...... + n Cn xn , the binomial coefficients are n C0 , n C1 , n C2 ,... , n Cn . A number of
n

series may be formed with these coefficients figuring in the terms of a series.
Some standard series of the binomial coefficients are as follows:
n
(a) By putting x = 1, we get C0 + n C1 + n C2 + ...... + n Cn =
2n …(i)

C0 − n C1 + n C2 − ...... + ( −1 ) . n Cn =0 ...(ii)
n n
(b) By putting x =-1, we get
n
(c) On adding (i) and (ii), we get C0 + n C2 + n C 4 + ...... =
2n−1 …(iii)

(d) On subtracting (ii) from (i), we get n C1 + n C3 + n C5 + ...... =


2n−1 …(iv)

(e)  2n C0 + 2n
C1 + 2n
C2 + ...... + 2n
Cn−1 + 2n
22n−1
Cn =

Proof: From the expansion of (1 + x ) , we get


2n 2n 2n 2n 2n 2n
C0 + C1 + C2 + ...... + C2n−1 + 22n
C2n =


⇒2 ( 2n
C0 + 2n
C1 + 2n
C2 + ....... + 2n
Cn−1 +) 2n
Cn =2n
2= [ 2n C0 2n
C2n , 2n C1
= 2n
C2n−1 and so on. ]

(f)  2n+1 C0 + 2n+1


C1 + 2n+1
C2 + ....... + 2n+1
22n
Cn =

Proof: (as above)

(g) Sum of the first half of n C0 + n C1 + ... + n Cn = Sum of the last half of n C0 + n C1 + ... + n Cn = 2n–1

(h) Bino-geometric series: n C0 + n C1 x + n C2 x2 + ....... + n Cn xn =(1 + x )


n

(i) Bino-arithmetic series: a n C0 + ( a + d) n C1 + ( a + 2d) n C2 + ....... + ( a + nd) n Cn

Consider an AP -a, (a+d), (a+2d), … , (a+nd)


Sequence of Binomial Co-efficient - n C0 , n C1 , n C2 ,......., n Cn
A bino-arithmetic series is nothing but the sum of the products of corresponding terms of the sequences. It
can be added in two ways.
(i) By elimination of r in the multiplier of binomial coefficient from the (r+1)th term of the series

(By using r. C n
r = nn−1Cr −1 )
( )
n
(ii) By differentiating the expansion of xd 1 + xd .
M a them a ti cs | 4.23

n n n n
C0 C1 C2 Cn
(j) Bino-harmonic series: + + + ...... +
a a+d a + 2d a + nd
1 1 1 1
Consider an HP - , , ,... ,
a a + d a + 2d a + nd

Sequence of Binomial Co-efficient - n C0 , n C1 , n C2 ,......., n Cn

It is obtained by the sum of the products of corresponding terms of the sequences. Such series are calculated
in two ways :
(i) By elimination of r in the multiplier of binomial coefficient from the (r + 1)th term of the series
 1 n 1 n+1 
 By using Cr = Cr +1 
 r + 1 n + 1 

(ii) By integrating suitable expansion.


For explanation see illustration 2

(k) Bino-binomial series: n C0 . n Cr + n C1 . n Cr +1 + n C2 . n Cr + 2 + ....... + n Cn−r . n Cr


m
or, C0 . n Cr + m C1 . n Cr −1 + m C2 . n Cr −2 + ..... + m Cr . n C0

As the name suggests such series are obtained by multiplying two binomial expansion, one involving the first
factors as coefficient and the other involving the second factors as coefficient. They can be calculated by equating
coefficients of a suitable power on both sides.
For explanation see illustration 4

FORMULAE SHEET

Binomial theorem for any positive integral index:


n
( x + a)=
n n
C0 xn + n C1 xn−1a + n C2 xn−2a2 + ........ + n Cr xn−r ar + ...... + n Cna=
n
∑ n Cr xn−r ar
r =0
(a) General term – Tr +1 = n Cr xn−r ar is the (r + 1)th term from beginning.
(b) (m + 1)th term from the end = (n – m + 1)th from beginning = Tn–m+1
(c) Middle term
th
n 
(i) If n is even then middle term =  + 1  term
2 
th th
n+1 n+3
(ii) If n is odd then middle term =   and  
 2   2 
Binomial coefficient of middle term is the greatest binomial coefficient.

To determine a particular term in the given expansion:


th
 1 
Let the given expansion be  x α ±  , if xn occurs in Tr+1 (r + 1)th term then r is given by n α − r ( α + β ) =m and for
 xβ 
x0 ,n α − r ( α + β ) = 0
4 . 2 4 | Binomial Theorem

Properties of Binomial coefficients:

For the sake of convenience the coefficients n C0 , n C1 , n C2 ...... n Cr ....... n Cn are usually denoted by C0 ,C1 ,......Cr .......Cn
respectively.

2n
C0 + C1 + C2 + ....... + Cn =

C0 − C1 + C2 − C3 + ....... + Cn =
0

C0 + C2 + C 4 + ....... = C1 + C3 + C5 + ......... = 2n−1

n n n−1 n n − 1 n− 2
Cr= Cr −1= ⋅ Cr −2 and so on........
r r r −1

2n 2n!
Cn+r =
(n − r ) ! (n + r ) !
n
Cr + n Cr −1 =
n+1
Cr

C1 + 2C2 + 3C3 + ....... + n Cn =


n.2n−1

C1 − 2C2 + 3C3 ....... =


0

C0 + 2C1 + 3C2 + ....... + (n + 1 ) Cn = (n + 2 ) 2n−1

C20 + C12 + C22 + ....... + Cn2=


( 2n)=! 2n
Cn
(n!)
2

 0, if n is odd
C20 − C12 + C22 − C32 + ..... = n/2 n
( −1) Cn/2 , if n is even

Note: 2n+1
C0 + 2n+1
C1 + ....... + 2n+1
Cn
= 2n+1
Cn+1 + 2n+1
Cn+ 2 + ...... + 2n+1
C2n
= +1 22n

; C − C1 + C2 − C3 ...... + ( ) n =
C1 C2 Cn 2n+1 − 1 −1 n C 1
C0 + + + ...... + =
2 3 n+1 n+1 0
2 3 4 n+1 n+1
(a) Greatest term:

(i) If
(n + 1 ) a ∈ Z
(integer) then the expansion has two greatest terms. These are kth and (k + 1)th where x
x+a
and a are +ve real numbers.

(ii) If
(n + 1 ) a ∉ Z  (n + 1 ) a 
then the expansion has only one greatest term. This is (K + 1)th term k =  
x+a  x + a 
denotes greatest integer less than or equal to x}

(b) Multinomial theorem:


n!
Generalized ( x1 + x2 + ...... + xk ) =
n r r r
∑ r !r !......r !
x11 x22 .....xkk
r1 +r2 +.....rk n 1 2
= k

(c) Total no. of terms in the expansion ( x1 + x2 + ......xn ) is


m m+n−1
Cn−1
M a them a ti cs | 4.25

Solved Examples

JEE Main/Boards The total number of terms in the expansion are 8.


7 + 1 th 7 + 3 th
10 Therefore and i.e. 4th and 5th terms are the
 c3  1 2 2
Example 1: Find the coefficient of in  + y 3

( )
7 −3  1 
y2  y2  two middle terms. 4th term = 7 C3 2x2 −
   
 x
Sol: By using formula of finding general term we can 7!
= − 16x8 −3 =−560x5
1 3! 4!
easily get coefficient of . 4

( )
7 − 4  −1 
y2 7 2 2
and 5 term = C 4 2x   = 280x
th

In the binomial expansion, (r + 1)th term is  x 


n−r Hence the two middle terms are –560x5 and 280x2.
 c3 
Cr ( y )   : n 10
n r
Tr +1 =
 y2 
  Example 4: The coefficient of (r – 1)th , rth and (r + 1)th
10 −r
 1  term in the expansion of (x + 1)n are in the ratio 1:3:5.
( )
10 −r
Cr ( y ) c3
10 r
⇒ Tr +1 =  2 Find n and r.
y 
 
= 10
Cr c30 −3r y 3r −20  ...(i) Sol: In this problem, by using the formula of general
term we will get the equation of given terms and by
∴ 3r − 20 =−2;r =6 taking ratios of these terms we can get the value of n
and r.
∴ 7th term will contain y–2 and from (i) the coefficient of
y–2 is = 210 c12 Coefficient of (r – 1)th term is n Cr −2
Coefficient of rth term is n Cr −1
Example 2: Use Binomial theorem to find the value of Coefficient of (r + 1)th term is n Cr
(10.1)5.
Coefficient are in ratio of 1 : 3 : 5
Sol: After reducing (10.1)5 into the form of (10 + 0.1)n n
Cr −2 1
n
Cr −1 3
we can use binomial expansion to get required result. = and =
n
Cr −1 3
Cr n 5
(10.1)5 = (10 + 0.1)5
r −1 1 r 3
or = and =
(10 ) + 5 C1 (10 ) ( 0.1) + 5 C2 (10 ) ( 0.1)
5 4 3 2
= n−r + 2 3 n−r +1 5

+ 5 C3 (10 ) ( 0.1 ) + 5 C 4 10 ( 0.1 ) + ( 0.1 )


2 3 4 5
i.e. n – 4r + 5 = 0 and 3n – 8r + 3 = 0

5
( )
=(10 ) + 5 103 + 10 (10 ) ( 0.01 ) + 10 (10 )
3 2 Solving both we get n = 7 & r = 3

( 0.001) + 5 (10 )( 0.0001) + ( 0.00001) Example 5: Find the remainder when 2710 + 751 is
divided by 10
= 100000 + 5000 + 100 + 1 +0.005 + 0.00001
Sol: We can obtain the remainder by reducing 2710 +
= 105101.00501 751 into the form of 10 λ +a, where λ is any integer and
a is an integer less than 10.
Example 3: Find the middle term(s) in the expansion of We have 2710 = 330 = 915 = (10 – 1)15
7
 2 1 751 = 7.750 = 7.(49)25 = 7 (50 –1)25
 2x −  .
 x 2710 = 10m1 …(i)

Sol: Since n = 7 is a odd number. Therefore, find the 751= 7(50 –1)25 = 10m2 – 7 ...(ii)

nn++11 th nn++33th Adding (i) and (ii)


th
thand
and th
thterm.
22 22 2710 + 751 = (10m1 – 1) +(10m2 –7) = 10m1 + 10m2 – 8
4 . 2 6 | Binomial Theorem

= 10m1 + 10m2 – 10 + 2 (ii) a0 − a1 + a2 − a3 + ...... + a2n =


1
Thus, the remainder is 2 when 2710 + 751 is divided by 10. 3n−1
(iii) a0 + a3 + a6 + ...... =

Example 6: If A be the sum of odd numbered terms and Sol: By putting x = 1, -1, and ω, ω2
B the sum of even numbered terms in the expansion of Respectively in the expansion of (1 + x + x2)n we will get
(x + a)n prove that A2 – B2 = (x2 – a2)n the result.

Sol: Do it yourself. Given (1 + x + x2)n

( x + a) =
n n
C0 xn + n C1 xn−1a =a0 + a1 x + a2 x2 + ...... + a2n x2n  ….(i)

(i) Putting x = 1, we get


x + n C2 xn−2a2 + .... + n Cnan = A + B
3n = a0 + a1 + a2 + ..... + a2n ..…(A)
When A= n C0 xn + n C2 xn−2a2 + n C 4 xn− 4 a4 + ....
(ii) Putting x = –1 in (i), we get
B = n C1 xn−1a + n C3 xn−3a3 + n C5 xn−5a5 + ....
1 = a0 − a1 + a2 − a3 ..... + a2n
∴ ( x − a) = A − B , A2 − B2 = ( A − B )( A + B )
n
(iii) Putting x = ω, ω2 successively in (i), we get
( ) 0= a0 + a1 ω + a2 ω2 + a3
n
=( x − a) ( x + a) = x2 − a2
n n

+a4 ω + a5 ω2 + ..... + a2nω2n …..(B) 0= a0 + a1 ω2 + a2 ω + a3


Example 7: If Cr denotes the binomial coefficient n Cr , +a4 ω2 + a5 ω + a6 + ..... + a2nω4n ...(C)
prove that :
2n! Adding (A), (B) and (C) we have
C20 + C12 + ...... + Cn2 = .
(n!) 3n= 3 ( a0 + a3 + a6 + ...... )
2

Sol: Multiply the expansion of (x+1)n and (1+x)n and 3n−1
∴ a0 + a3 + a6 + ....... =
compare the coefficients of xn on both sides.

Example 9: If (1 + x ) = C 0 + C1 x +
n
We know that (1 + x ) =
n n
C0 + n C1 x

C2 x2 + C3 x3 + ....... + Cn xn
+ n C2 x2 + .... + n Cn−1 xn−1 + n Cn xn
then prove that C12 + 2C22 + 3C32 + ...... + nCn2 =
( 2n − 1)!
( x + 1) =
n n n n n−1
C0 x + C1 x
( )
2
(n − 1 ) !
n n− 2 n n
+ C2 x + ..... + Cn−1 x + Cn
Sol: Expanding (1 + x ) and ( x + 1 ) and multiplying
n n

Multiplying these equations side by side, we get these two expansion and comparing the coefficient of
xn–1 we will prove above equation.
(1 + x ) ( x + 1 )
n n
= (C 0 + C1 x + C2 x2 + .... + Cn−1 xn−1 + Cn xn ) Given (1 + x ) =C0 + C1 x +
n

(
× C0 xn + C1 xn−1 + C2 xn−2 + .... + Cn−1 x + n Cn )
C2 x2 + C3 x3 + ..... + Cn xn
Coefficient of xn on R.H.S. is equal to Differentiating both sides w. r. t. to x, we get
C20 + C12 + C22 + .... + Cn2−1 + Cn2
n (1 + x )
n−1
=0 + C1 + 2C2 x +3C3 x2 + ..... + nCn xn−1
2n 2n!
Coefficient of xn in L.H.S. is Cn = . ⇒ n (1 + x )
n−1
=C1 + 2C2 x
n!n!
This proves the required identity. +3C2 x2 + ..... + nCn xn−1 
….(i)

( )
n
and ( x + 1 ) = C0 xn + C1 xn−1 + C2 xn−2
n
Example 8: If 1 + x + x2 = a0 + a1 x + a2 x2 +...... + a2n x2n
show that
n
+C3 xn−3 + C 4 xn− 4 + ....... + Cn  .…(ii)
(i) a0 + a1 + a2 + ...... + a2n =
3
M a them a ti cs | 4.27

Multiplying (i) and (ii), we get +....= 3 ( a0 + a3 + a6 + .... )

n (1 + x )
2n−1
= (C 1 + 2C2 x + 3C3 x2 + ..... + nCn xn−1 ) 3n−1
∴ a0 + a3 + a6 + ...... =

(
× C0 xn + C1 xn−1 + C2 xn−2 + C3 xn−3 + .... + Cn .. ( iii) .…(iii) ) From (i) + (ii) × w2 (iii) × w,+we get,

3n + 0 × ω2 + 0 × ω
Now, coefficient of xn–1 on R.H.S.

= C12 + 2C22 + 3C32 + .... + nCn2 and coefficient of x n−1


on
(
= a0 1 + ω2 + ω + a1 1 + ω3 + ω3 ) ( )
+ a (1 + ω
2
4
+ ω ) + a (1 + ω
5
3
5
+ω )
7

L.H.S. = n. 2n−1 Cn−1 + a (1 + ω


4
6
+ ω ) + ...
9

= n=
( 2n − 1) ! ( 2n − 1)! =
( 2n − 1)! ∴ 3n= 3 ( a1 + a4 + a7 + ...... )
(n − 1) !n! (n − 1 ) ! (n − 1 ) !
( (n − 1 ) !) 
 2
Because coefficient of each is

But (iii) is an identity, therefore the coefficient of xn−1 in ω + ω2 0, using


1 += = w23 = 1
ω
R.H.S. = coefficient of xn−1 in R.H.S.
3n−1
∴ a1 + a4 + a7 + ........ =

⇒ C12 + 2C22 + 3C32 + .... + nCn2 =


( 2n − 1) ! Again, from (i) + (ii) ω + (iii) × ω3, we get
( )
2
(n − 1 ) ! (
= 3n= a0 1 + ω + ω2 + a1 1 + ω2 + ω4 ) ( )
Example 10: Find the numerically greatest term in the ( )
...... 3 ( a2 + a5 + a8 + ........ )
+a2 1 + ω3 + ω3 + =
expansion of (3 – 5x)15 when x = 1/5.

Sol: Follow the algorithm for the greatest term. Example 12: Sum the series
C1 C2 Cn
Using standard notations w.r.t. (x + a)n C0 + + + ...... +
2 3 n+1
n+1 16
= =4
Sol: Expanding (1 + x ) integrating it from 0 to 1 or by
n
x 3
1+
a 1 + ( −1 ) using summation method we will get result.
n+1 C n+1
1
Sum = ∑ r −1
= ∑ n + 1 . n+1 Cr
T4 and T5 are numerically equal to each other and are r
=r 1 = r 1
greater than any other term.
=
1
n+1
( n+1
C0 + n+1 C1 + ...... + n+1 Cr +1 − n+1 C0 )
Example 11: If (1 + x + x ) = a0 + a1x + a2x + a3x ... + a2nx
2 2 3 2n

Then show that =


1
n+1
2n+1 − 1( )
a0 + a3 + a6 + ...... = a1 + a4 + a7 + ..... = 3n−1 .
Alternative method
Sol: By Putting x = 1 w, w respectively in the given
2

(1 + x )
n
equation and adding these values we can prove it. = C0 + C1 + C2 x2 + ..... + Cn xn

3n = a0 + a1 + a2 + a3 + a4 + ......  …(i) Integrating both sides w.r.t. x from 0 to 1


0= a0 + a1 ω + a2 ω2 + a3 ω3 + a4 ω4 + ......  ….(ii) 1 1

∫ (1 + x ) dx= ∫ ( C0 + C1 x + ...... + Cnx ) dx


n n
2
Because 1 + ω + ω = 0 0 0

0 = a0 + a1 ω2 + a2 ω4 + a3 ω6 + a4 ω8 + ......  …(iii) 2n+1 − 1 C C C


= C0 + 1 + 2 + ..... + n
n+1 2 3 n+1
Adding these

(
3n 3 ( a0 ) + a1 1 + ω + ω2 + a2
= )
(1 + ω 2
+ω 4
) + a (1 + ω
3
3
+ ω6 )
4 . 2 8 | Binomial Theorem

Example 13: Find the last three digits of 2726. = [ 26 − 6 C1 × 25 × x (1 − 3x ) + 6 C2 24

Sol: By reducing 2726 into the form ( 730 − 1 ) and using


n
×x2 (1 − 3x ) − 6 C3 23 × x3 (1 − 3x )
2 3

simple binomial expansion we will get required digits.


+ 6 C 4 22 × x 4 (1 − 3x ) − 2 × 6 C5
4
We have 272 = 729.
×x5 (1 − 3x ) + 6 C6 ×x6 (1 − 3x ) ]
5 6
Now 2726 = (729)13 = (730 –1)13

C0 ( 730 ) − 13 C1 ( 730 ) + 13 C2 ( 730 )


13 12 11
= 13 The term in x4 will come only from the three terms, viz.

−..... − 13
C10 ( 730 ) −
3 13
C12 ( 730 )
2 (
(a) 6 C2 × 24 × x2 (1 − 3x ) = 15 × 16x2 1 − 6x + 9x2
2
)
∴ The term in x4 is (15) (16) (9x4)
− 13 C12 ( 730 ) + 1 (b) − 6 C3 23 × x3 (1 − 3x )
3

(13)(12) =−20 × 8 × x3 1 − 9x + 27x2 − 27x3 


(14 ) − (13)( 730 ) + 1
2
1000m +
=  
2
Where m is a positive integer ∴ The term in x4 is −20 × ( −9 ) × ( 8 ) x 4

(c) 6 C 4 22 x 4 (1 − 3x ) = 15 × 4x 4 (1 − 4 × 3x + ....... )
4
= 1000 m + 15288 – 9490 + 1
= 1000 m + 5799
∴ The term in x4 is 15 × 4 × x 4
Thus, the last three digits of 17 256
are 799.
∴ The total term in x4 is

JEE Advanced/Boards 15 × 16 × 9 + 20 × 8 × 9 + 15 × 4  × x 4

= 2160 + 1440 + 60  x 4 = 3660x 4


Example 1: Find the coefficient of x4 in the expansion of
(i) (1 + x + x2 + x3)11 ∴ The coefficient of x4 is 3660.
(ii) (2 – x + 3x2)6 n
Example 2: Show that ∑ r (n − r ) Cr2 =
n2 . 2n−2 Cn
r =0
Sol: By expanding given equation using expansion
(1 + x )
n
formula we can get the coefficient x4 . Sol: By expanding and differentiating and
( x + 1)
n
(i) 1 + x + x + x = (1 + x) + x (1 + x) = (1 + x) (1 + x )
2 3 2 2
and then multiplying these expansion we can
prove given equations by comparing coefficient of xn-2
( ) = (1 + x ) (1 + x )
11 11 11
∴ 1 + x + x 2 + x3 2
on both side.

=+(1 C x + C x + C x + C x
11
1
11
2
2 11
3
3 11
4
4
+ ....... ) We have

(1 + x )
n
(1 + C x + C x + ......)
11
1
2 11
2
4 =C0 + C1 x + C2 x + ....... + Cn xn  ….(i)

To find term in x4 from the product of two brackets on Differentiating both side w.r.t x, we get
the right-hand-side, consider the following products
n (1 + x )
n−1
= C1 + 2C2 x + 3C3 x2 + .... + n Cn xn … (ii)
terms as 
(i) can also the be written as
1 × 11 C2 x 4 + 11 C2 x2 × 11 C1 x2 + 11 C 4 x 4
(1 + x ) =( x + 1 ) =C0 xn
n n

=  11 C2 + 11
C2 × 11
C1 + 11
C4  x 4
 
+C1 xn−1 + C2 xn−2 + ...... +Cn−1 x + Cn  ....(iii)
55 + 605 + 330  x 4 =
990x 4
Differentiating both sides w.r.t. x, we get
∴ The coefficient of x4 is 990.
n (1 + x=
) nC0 xn−1 + (n − 1)
n−1

( )
6 6
(ii) 2 − x + 3x2 = 2 − x (1 − 3x ) 
C1 xn−2 + (n − 2 ) C2 xn−3 + ..... + Cn−1  ….(iv)
M a them a ti cs | 4.29

Multiplying (ii) and (iv), we have 1


n
∫ (1 + x) dx =
n (1 + x ) ( x + 1) = n (1 + x )
2 n−1 n−1 2 2n−2
0

1
= [C1 + 2C2 + 3C3 x2 + ..... + r Cn xn−1 ] 2
+ C3 x3 + ....Cn xn ) dx
∫ (C0 + C1 x + C2 x + C2 x
0
x [nC0 xn−1 + (n − 1)C1 xn−2 + (n − 2)
1
 1 + x n+1 
C2 x n−3
+ ...... + Cn−2 x + Cn−1 ]  .…(v) ( )  =C x + C x2 +
 n + 1  0 1
2
 0
The coefficient of xn–2 on the LHS of (v) is
1
x3 xn+1 
n . 2 2n−2
Cn−2 = n . 2 2n−2
Cn C2 + ....... + Cn 
3 n + 1 
0
The coefficient of xn–2 on the RHS of (v) is 2 n+1
−1 C C C
= C0 + 1 + 2 + ....... + n
1. (n − 1 ) C12 + 2. (n − 2 ) C22 + ..... + (n − 1 ) .1Cn2−1 n+1 2 3 n+1

n−1 n (ii) Integrating both sides of equation (i) within limits


= ∑ r (n − r ) Cr2 = ∑ r (n − r ) Cr2 0 to 2.
=r 0=r 0
2 2

∫ (1 + x ) = ∫ ( C0 + C1 x + C2 x )
n n
( )
2
+ C3 x3 + .....Cn xn dx
Hence, r n − r Cr2 ∑( ) n
= 2 2n−2
Cn
0 0
r =0
2
 1 + x n+1 
or  ( )  
= C0 x + C1
x2
+ C2
x3
+ .... + Cn
xn+1 
2

Example 3: Prove that  n + 1  



 2 3 n + 1 
 0 0
C1 C2 Cn 2n+1 − 1
(i) C0 + + + ...... + =
2 3 n+1 n+1
3n+1 − 1 C C C
or = C0 .2 + 22. + 1 + 23. 2 + ....... + 2n+1. n
C1 C2 n+1 2 3 n+1
(ii) 2.C0 + 22. + 23.
+ ......
3 2
(iii) Integrating both sides of equation (i) within limits
C 3n+1 − 1 –1 to 0,
+2n+1. n =
n+1 n+1
0 0

∫ (1 + x ) ∫ ( C0 + C1 x + C2 x )
n 2
1 1 1 dx= + C3 x3 + .....Cn xn dx
(iii) C0 − C1 + C2 − C3 + ......
2 3 4 −1 −1

n Cn 1 0
+ ( −1 )  1 + x n+1 
( )
= 2 3 n+1
n+1 n+1  = C x + C x + C x + ....... + C x |0
 n + 1  0 1
2 2
3 n
n + 1 −1
(iv) C0 C1 C2  −1
+ + + ......
1.2 2.3 3.4 1  C C C 
Cn 2n+ 2 − n − 3 − 0 = 0 −  −C0 + 1 − 2 + ..... + ( −1)n+1 n 
+ = n+1  2 3 n + 1
(n + 1) . (n + 2) (n + 1)(n + 2)
1 C C n Cn
C2 C4 2n = C0 − 1 + 2 + ....... + ( −1 )
(v) C0 + + + ...... = n+1 2 3 n+1
3 5 n+1
n
Ck
(iv) General term of L.H.S =
Sol: Expand (1 + x ) and integrate it within the limit 0
n
(k + 1)(k + 2)
to 1, 0 to 2, -1 to 0 and -1 to 1 respectively to prove
these equations n+1
Ck +1  nC n−1
Cr −1  n+ 2
Ck + 2
= = =  r = 
(1 + x )
n
=C0 + C1 x + C2 x2 + C3 x3 + ....... + Cn xn  ….(i)
(n + 1) (k + 2)  n

r  (n + 1 )(n + 2 )
∴ The sum of terms on L.H.S.
(i) Integrating both sides of equation (i) within limits 0
n+ 2 n
to 1, we get n C 1
= ∑ (n + 1)(kn++2 2 )
= ⋅ ∑ n+ 2 Ck + 2
k =0 (n + 1)(n + 2) k =0
4 . 3 0 | Binomial Theorem

1 2n+ 2 − n+ 2 C − n+ 2 C   k. n C = n. n−1 C 
=  k k +1 
(n + 1)(n + 2)  0 1

1 2n+ 2 − n − 3 = 2n−2 n2 − n + 2n


2n=
+2
− 1 − (n + 2 )   
( )( )
n + 1 n + 2   (n + 1)(n + 2) n
n (n + 1 ) 2n−2
∴ ∑ k 2 n Ck =
(v) Integrating both sides of equation (i) within limits k =1

–1 to 1, we get
(ii) To prove ( C0 + C1 )( C1 + C2 )( C2 + C3 ) ......
1

∫ (1 + x )
n
dx =
C0 C1 ......Cn−1 (n + 1 )
n
−1
( Cn−1 + Cn ) =
n!
1

∫ ( C0 + C1 x + C2 x )
2
+ C3 x3 + ...... + C n xn dx Multiply and divide L.H.S. by C0 C1C2 .......Cn−1 ; then,
−1

1  C 
 1 + x n+1  L.H.S. = C0 C1C2 .......Cn−1  1 + 1
( ) 1
2 3 n+1 
 = C x + C x + C x + ....... + C x  C0 
 n + 1  0 1
2 2
3 n
n+1
 −1 −1  C2   Cn 
 1 +  ........  1 + 
2n+1 − 0  C C C   C C   C1   Cn−1 
= C0 + 1 + 2 + ....... + n  −  −C0 + 1 − 2 + ...... 
n+1  2 3 n+1  2 3 
n
Cr n−r +1
On using = we have,
2 n+1 C C  n
Cr −1 r
= 2 C0 + 2 + 4 + ....... 
n+1  3 5 
L.H.S. = C0 C1C2 .....Cn−1
2n C C
⇒ = C0 + 2 + 4 + .....
n+1 3 5  C1  C2   Cn 
 1 +   1 +  ......  1 + 
 C0  C1   Cn−1 
Example 4: Prove, by binomial expansion, that
 1 + n  1 + n   n + 1 
n = C0 C1C2 ...Cn−1 (1 + n)    ...  
(i) ∑k 2
⋅ Ck = n (n + 1 ) 2
n n− 2
 2  3   n 
k =1

C0 C1 ...Cn−1 (1 + n) C0 C1 .......Cn−1 (n + 1 )
n n
C0 C1 ........Cn−1 (n + 1 )
n
n
== = R.H.S.
(ii) ∏ ( Ck −1 + Ck ) =
n!
1.2.3........n n!
k =1

Sol: Expanding (1 + x ) and differentiating it twice we


n
Example 5: If (1 + x ) =C0 + C1 x + C2 x2
n

will prove given equation (i) and by multiplying and


dividing by C0 C1C2 .......Cn−1 in L.H.S. of equation (ii) we +C3 x3 + ....... + Cn xn
can prove it.
∑ ∑ ( Ci + C j )
2
Then find the value of
(i) Now (1 + x ) =C0 + C1 x + C2 x +C3 x + ....... + Cn x
n 2 3 n
0 ≤i < j≤n

Differentiating twice w.r.t. x, we get Sol: By using summation and coefficients properties
we can prove given equations.
n (n − 1 )(1 + x )
n− 2
= 2.C2 + 3.2.C3 x
∑ ∑ ( Ci + C j )
2

+4.3C 4 x + ...... + n (n − 1 ) Cn x
2 n− 2
0 ≤i < j≤n

= ( C0 + C1 ) + ( C0 + C2 ) + ...... +
2 2
Substituting x = 1, we get
n
n (n − 1 ) 2n−2 = ∑ k (k − 1 )( Ck ) ( C0 + Cn ) + ( C1 + C2 )
2 2
k =1
+ ..... + ..... +
n
∴ ∑ k2
k =1
( )( C ) =
n (n − 1 ) 2
n
k
n− 2
+ n.2n−1
( C1 + Cn ) + ( C2 + C3 )
2 2
+ .... + .... + ( C2 + Cn )
2
M a them a ti cs | 4.31

+........ + (Cn−1 + Cn )2 Replacing n by 1, 2, 3, 4, ……. n–1 successively in the


above reduction formula, we get
(
= n C20 + C12 + C22 + ..... + Cn2 + 2∑ ) ∑ Ci ⋅ C j
0 ≤i < j≤n 3n 3 (n − 1 ) 3 (n − 2 ) 3
In = ; ... I  …(ii)
The square of the sum of n terms is given by 3n + 1 3n − 2 3n − 5 4 0
1 1
( C0 + C1 + C2 + C3 + ...... + Cn )
2
( )
0
= 3
∫ 1−x
But I0 = ∫ dx =
dx = 1
0 0
= ( C20 + C12 + C22 + ...... + Cn2 ) + 2∑ 0 ≤i < j≤n
∑ Ci ⋅ C j Hence, from (ii),

∴ 2∑ ∑ Ci ⋅ C j 3n n!
In =
0 ≤i < j≤n
(3n + 1)(3n − 2)(3n − 5) ......7.4
= ( C0 + C1 + C2 + C3 + ...... + ) − C20 + C12 + ..... + Cn2 

2
 ( ) Using (i)

(2 )
2
= n
− 2n
Cn C0

C1
+
C2

C3
+...... +
( −1) n Cn =
3n n!
1 4 7 10 3n + 1 1.4.7.....(3n + 1)
∑ ( Ci + C j=
)
2
∴∑ n. 2n C  + 22n − 2n
Cn  ,
 n  
0 ≤i < j≤n
Example 7: Prove that
(n − 1) 2n Cn + 22n
=
n (n − 1 ) n (n − 1 ) ......2.1
1 n
C0 + C1 C2 +..... + C
m! (m + 1 ) ! (m − 2 ) ! (m + n) ! n
Example 6: Show that

C0 C1 C2 C3 ( −1 ) n Cn 3n n! =
(m + n + 1)(m + n + 2) ...... (m + 2n)
1

4
+
7

10
+ ..... + 3
2n + 1
=
1.4.7....... ( 3n + 1 ) (m + n) !

( ) using binomial expansion


n
Sol: As (1 + x ) . (1 + x ) =(1 + x )
m+n n m+ 2n
Sol: By expanding 1 − x3 and expanding
this by using expansion formula and equating the
and integrating it within a limit 0 to 1 we will prove coefficient of xn we can prove given equation.
given equation.

(1 − x ) 3
n
=C0 − C1 x + C2 x 3 6
⇒ ( m+n
C0 + m+n
C1 x + m+n
C2 x2 + .... + m +n
Cm+n xm+n )
−C3 x9 + C 4 x12 + .... + ( −1 ) Cn x3n
n
(
× C0 + C1 x + C2 x2 + ..... + Cn xn = ) (1 + x )
m+ 2n

Integrating both sides between limits 0 and 1, we get Equating the coefficients of xn on both sides, we find

1
( ) −1 n
Cn m+n
Cn .C0 + m+n
Cn−1 .C1 + m+n
Cn−2 .C2
∫ (1 − x ) dx =
C 3 C C n
1 2 3
C − + − + ..... + 0 …(i)
0
4 7 10 3n + 1
m+n m+ 2n
+..... + C0 .Cn = Cn
1

∫ (1 − x ) dx
n
(m + n) ! C (m + n) ! C
3
In
Also =
0 ⇒ +
m!n! 0
(n − 1 ) ! (m + 1 ) ! 1
1 1

( )  ( ) ( −3x ) .x dx
n n−1
(m + n) ! C + .... + (m + n) ! C =(m + 2n) !
2
=  x 1 − x3 − ∫ n 1 − x3 ;
 0 +
(n − 2 ) ! (m + 2 ) ! 2 (m + n) ! n (m + n) !n!
0

1
( )
n−1
= 3n∫ x3 1 − x3 dx
0
Dividing both sides by (m + n)!/n! we find
1 n (n − 1 ) n (n − 1 ) .....2.1
( )( ) 1 n
n−1
= 3n∫ x3 − 1 + 1 1 − x3 dx C0 + C1 + C2 +...... + C
0
m! (m + 1 ) ! (m + 2 ) ! (m + n) ! n
3n
= 3nIn−1 − 3nIn ; (1 + 3n) In =
3nIn−1 ∴ In = I
3n + 1 n−1
4 . 3 2 | Binomial Theorem

=
(m + 2n) ! (m + n + 1)(m + n + 2) ..... (m + 2n) = Coefficient of xn in

(m + n) ! (m + n) ! (m + n) !  1 + x n + nx 1 + x n−1  x + 1 n
( ) ( )  ( )
= Coefficient of xn in
Example 8: Find the sum of the following series
 1 + x 2n + nx 1 + x 2n=
( ) ( ) 
−1 2n
1 + 2 2 1 + 2 + 3 2 …Upto n term Cn + n. 2n−1 Cn−1
C12 +
S= C + C3 +
2 2 3

Sol: In this problem, first obtain the rth term and then = ( )  1 + n
2n !  2n  n
 = Cn  1 + 
by using binomial expansion and coefficient property n!n!  2  2
we can get required sum.
The rth term of the given series ⇒ 2C12 + 3C22 + 4C32 + ...... + (n + 1 ) Cn2

1 + 2 + ...... + r 2 r (r + 1) 2 1  n
=
r
Cr =
2r
C=
r
2
(r + 1) Cr2 = 2n
Cn  1 +  − 1
 2
 C0 = 1

1 1 1
∴ S = (1 + 1 ) C12 + ( 2 + 1 ) C22 + ⇒S
1  2n  n 
2 2 2 =  Cn  1 +  − 1
2  2 

(3 + 1) C32 + .... + 12 (n + 1) Cn2


Example 9: If n be a positive integer, then prove that

( ) is an odd integer. If f be
n
We know that
the integral part I of 5 + 2 6
C0 + C1 x + C2 x2 + .... + Cn xn = (1 + x )
n

the fractional part of (5 + 2 6 ) prove that I =


1 n
– f.
1−f
⇒ C0 x + C1 x2 + C2 x3 + .... + Cn xn+1 = x (1 + x )
n

Differentiating both sides w.r.t. x we get Sol: By using expansion formula we can expand the
given binomial and separating its integral and fractional
C0 + 2C1 x + 3C2 x2 + 4C3 x3 part we can prove given equations.

( )
n
+.... + (n + 1 ) Cn xn = (1 + x )
n
Let P =
5+2 6 I+f
=

+nx (1 + x ) ( )
n−1
 …(i) Or f 5n + C1 5n−1 2 6
I +=
Also
+C 5 ( 2 6 ) + ...... + C ( 2 6 ) 
2 n
n− 2
2 3 2 n ...(i)
1 1 1
C0 + C1   + C2   + C3  
0 < 5 − 2 6 < 1 ⇒ 0 < (5 − 2 6 ) < 1
x x x n

n n
1  1
Let (5 − 2 6 ) = f’, where 0 < f’ < 1.
n
+.... + Cn   =  1 +   …(ii)
x
   x

Now, C20 + 2C12 + 3C22 + 4C32 + .... + (n + 1 ) Cn2 f=' 5 − C 5 ( 2 6 )


n
1
n−1

= Coefficient of constant term in


+C 5 ( 2 6 ) − C 5 ( 2 6 ) + .... 
2 3
n− 2 n−3
2 3 ...(ii)
C + 2C x + 3C x2 + 4C x3 + ..... + (n + 1 ) C xn  ×
 0 1 2 3 n 
Adding (i) and (ii) I + f + f' =
 1 
( ) ( )
2 n  2 4 
1 1 2 5n + n C2 5n−2 2 6 + n C 4 5n−1 2 6 .... 
C0 + C1   + C 2   + ..... + Cn   
 x x  x    

Or I + f + f’ = even integer
= Coefficient of constant term in
Now 0 ≤ f < 1 and 0 < f’ < 1.
 1 + x n + nx 1 + x n−1  1 + 1 / x n
( ) ( )  ( ) ∴0 < f + f' < 2
M a them a ti cs | 4.33

∴ f + f ' =1 and ∴ I is an odd integer Again, from (i) + (ii) w + (iii) × w3, we get

(5 + 2 6 ) , ( ) ( )
n
Now I + f = = 3n= a0 1 + ω + ω2 + a1 1 + ω2 + ω4

(5 − 2 6 ) ( )
n
1 − f ⇒ ( I + f )(1 − f ) =
f' =
= 1 ...... 3 ( a2 + a5 + a8 + ........ )
+a2 1 + ω3 + ω3 + =

1 1
∴ (I + f ) = ∴=I −f Example 11: Find the
1−f 1−f
(i) Last digit
Example 10: If 1 + x + x ( 2
)= a0 + a1 x (ii) Last two digits and
+a2 x2 + a3 x3 .... + a2n x2n (iii) Last three digits of 17256.

Sol: By reducing 17256 into the form ( x − 1 ) and using


n
Then show that
simple binomial expansion we will get required digits.
a0 + a3 + a6 + ...... = a1 + a4 + a7 + ..... = 3n−1 .
Since
Sol: By using properties of binomial coefficients and
(17 =
)
128
( 289=
) ( 290 − 1)
256 2 128 128
cube root unity 1, ω , ω2 we can prove given problem. 17=

The rth term of the given series


C0 (190 ) − 128 C1 ( 290 )
128 127

= 17256 128

Putting x = 1, ω, ω2 , where ω is a non real cube root of


unity. + 128 C2 ( 290 )
126
− ... − 128 C125 ( 290 )
3

3n = a0 + a1 + a2 + a3 + a4 + ......  …(i)
+ 128 C126 ( 290 ) − 128 C127 ( 290 ) + 1
2
2 3 4
0= a0 + a1 ω + a2 ω + a3 ω + a4 ω + ......  ...(ii)

[ 128 C0 ( 290 ) − 128 C1 ( 290 )


128 127
Because 1 + ω + ω2 = 0

0 = a0 + a1 ω2 + a2 ω4 + a3 ω6 + a4 ω8 + ...... …(iii)
+ 128 C2 ( 290 ) − ...... − 128 C125 ( 290 ) ]
126 3

Adding these
+ 128 C126 ( 290 ) − 128 C127 ( 290 ) + 1
2
(
3n 3 ( a0 ) + a1 1 + ω + ω2 + a2
= )
(1 + ω 2
)
+ ω4 + a3 1 + ω3 + ω6 ( ) 1000m + 128 C2 ( 290 ) − 128 C1 ( 290 ) + 1 (m ∈ I + )
=
2

+....= 3 ( a0 + a3 + a6 + .... ) (128 )(127 )


( 290 )
2
= 1000m + − 128 × 290 + 1
n−1 2
∴ a0 + a3 + a6 + ...... =
3
= 1000 m + (128) (127) (290) (145) – 128 × 290 + 1
From (i) + (ii) × w2 + (iii) × w,
= 1000 m + (128) (290) (127 × 145 –1) + 1
n 2
we get, 3 + 0 × ω + 0 × ω = 1000 m + (128) (290) (18414) + 1
( 2
) (
= a0 1 + ω + ω + a1 1 + ω + ω 2
)3
= 1000 (m + 683527) + 681
Hence last three digits of 17256 must be 681. As result
+ a (1 + ω
2
4
+ ω ) + a (1 + ω
5
3
5
+ω )7
last two digits of 17256 or 81 and last digit of 17256 is 1.

+ a (1 + ω
4
6
+ ω ) + ...
9
32
Example 12: If 3232 is divided by 7, then find the
∴ 3 = 3 ( a1 + a4 + a7 + ...... )
n
remainder
Because coefficient of each is Sol: Here in this problem, we can obtain required
2 2 32
1 +=
ω+ω 0, using
= ω 1 remainder by reducing 3232 into the form of 7 λ +a,
3n−1
∴ a1 + a4 + a7 + ........ = where λ is any integer and a is an integer less than 7.
4 . 3 4 | Binomial Theorem

We have 32 = 25 = 4.( 5m+1 C0 ( 7 )


5m+1
+ 5m+1 C1 ( 7 )
5m

( )
32
∴ ( 32 ) = 2160 ; ( 32 ) = (3 − 1)
32 32 160
= 25
+ 5m+1 C2 ( 7 )
5m−1
+ .... +
160 160 160 159 160 160
= C0 3 − C1 3 + .... + C159 3 + C160 + 1 5m+1
C5m 7 + 5m+1 C5m+1 )
= 3 3 ( 159
− 160
C1 3 158
+ .... − 160
)
C159 + 1
= 4[7{ 5m+1 C0 − 75m + 5m+1 C1 75m−1
+
= 3m + 1, m ∈ I
+ 5m+1 C2 75m−2 + .... + 5m+1 C5m } + 1]
3232 3m+1 5( 3m+1 ) 15m+5
Now, 32
= 32
= 2= 2
4 7n + 1 ,
= n ∈ I+ = 28 n + 4
2( ).22 = 4. 8
32
( )
3 5m+1 5m+1
∴ 3232 = 32
This show that where 3232 is divided by 7, then
= 4. ( 7 + 1 )
5m+1 remainder is 4.

JEE Main/Boards

Exercise 1
n
 1
Q.9 Given that the 4th term in the expansion of  px + 
 x
5
( )
5
Q.1 Expand x2 + 2a by binomial theorem. is , find n and p.
2

Q.2 Expand ( a + b ) − ( a − b ) . Hence find the value of


6 6
Q. 10 If in the expansion of (1 + x)m (1 – x)n the coefficient
of x and x2 are 3 and –6 respectively then find m.
( ) −( )
6 6
2 +1 2 −1 .
Q.11 If the coefficients of ar −1 ,ar ,ar +1 in the
Q.3 Show that (101 )
50
> (100 )
50
+ ( 99 )
50 binomial expansion of (1 +a)n are in A.P., prove that
n2 − n ( 4r + 1 ) + 4r 2 − 2 =0.
Q.4 If x > 1 and the third term in the expansion of
5 Q.12 If n be a positive integer, then prove that
1 log10 x 
 +x  is 1000, find the value of x. 62n − 35n − 1 is divisible by 1225.
x 
Q.13 Using binomial theorem, show that 34n+1 + 16n − 3
Q.5 Find the sum of rational terms in the expansion of
is divisible by 256 if n is a positive integer.
( )
10
2 + 31/5 .
Q.14 If a1, a2, a3 and a4 be any four consecutive
7 coefficients in the expansion of (1 + x)n, prove that
 1
Q.6 Find the middle term in the expansion of  2x2 −  a1 a3 2a2
 x + =
a1 + a2 a3 + a4 a2 + a3
Q.7 Find the middle term in the expansion of

(1 − 2x + x ) .
n
2 Q.15 If 3 consecutive coefficients in the expansion of
(1 + x)n are in the ratio 6 : 33 : 110, find n and r.

Q.8 Show that the greatest coefficient in the expansion


Q.16 If a, b, c be the three consecutive coefficients in
1.3.5.... ( 2n − 1 ) .2n
2n
 1 the expansion of a power of (1 + x), prove that the index
of  x +  is .
 x n! 2ac + b ( a + c )
of the power is
b2 − ac
M a them a ti cs | 4.35

11 Q.2 If the coefficients of x7 & x8 in the expansion of


 1
Q.17 Expand  x −  , y ≠ 0 n
y  x
 2 +  are equal, then the value of n is:
 3
Q.18 Expand (1 – x + x2)4
(A) 15 (B) 45 (C) 55 (D) 56
Q.19 Which number is larger, (1.2)4000 or 800?
Q.3 The coefficient of x49 in the expansion of ( x − 1 )
Q.20 If in the expansion of (1 + x)n, the coefficients of
 1  1   1 
14th, 15th and 16th terms are in A.P., find n.  x −   x − 2  ..  x − 49  is equal to
 2   2   2 

Q.21 If three consecutive coefficient in the expansion  1 


of (1 + x)n be 165, 330 and 462, find n and the position (A) –2  1 −  (B) + ve coefficient of x
of the coefficient.  250 
 1 
(C) – ve coefficient of x (D) – 2  1 − 
Q.22 Find the greatest term in the expansion of;  249 
2 Q.4 The last digit of (3P + 2) is
(7 – 5x)11, where x =
3
8 (A) 1 (B) 2 (C) 4 (D) 5
 1
Q.23 Find the coefficient of x–1 in (1 + 3x2 + x4)  1 + 
 x Where P = 34n and n ∈ N
n
Q.24 Find the value of k so that the term  1
10 Q.5 The sum of the binomial coefficient of 2x + 
 k  x  
independent of x in  x +  of 405. is equal to 256. The constant term in the expansion is :
 x2 
(A) 1120 (B) 2110 (C) 1210 (D) None
Q.25 If A be the sum of odd terms and B the sum of
even terms in the expansion of (x + a)n, prove that 10
x 3 
Q.6 The coefficient of x in  − 
4
is
( )
2 A 2 + B2 = ( x + a )
2n
+ ( x − a)
2n
 2 x2 
405 504 450 405
(A) (B) (C) (D)
Q.26 Find the coefficient of x in the expansion of
40 256 259 263 512

(1 + 2x + x )
27
2
Q.7 If (11)27 + (21)27 when divided by 16 leaves the
remainder
9
3 1 
Q.27 Find the term independent of x in  x2 −  . (A) 0 (B) 1 (C) 2 (D) 14
2 3x 

Q.28 If (1 + ax)n = 1 + 8x + 24x2 + ….. Find a and n. Q.8 Last three digits of the number N = 7100 – 3100 are
(A) 100 (B) 300 (C) 500 (D) 000

Exercise 2 Q.9 The last two digits of the number 3400 are:
(A) 81 (B) 43 (C) 29 (D) 01
Single Correct Choice Type

( )
25
( )
15
Q.1 Given that the term of the expansion x1/3 − x −1/2 Q.10 If 1 + x + x2 = a0 + a1 x + a2 x2 + .. + a50 .x50

which does not contain x is 5 m where m ∈ N , then m = then a0 + a2 + a4 + .... + a50 is:

(A) 1100 (B) 1010 (C) 1001 (D) None (A) Even
(B) Odd and of the form 3n
(C) Odd and of the form (3n –1)
(D) Odd and of the form (3n+1)
4 . 3 6 | Binomial Theorem

Q.11 The sum of the series  9 


10
2
Q.18 In the expansion of x +   the term which
(1 2
) ( ) ( )
+ 1 .1! + 22 + 1 .2! + 32 + 1 .3! + .... + n2 + 1 .n! is ( ) does not contain x is :  43 
10 10 10
(A) C0 (B) C7 (C) C4 (D) None of these
(A) (n + 1). (n + 2)! (B) n.(n + 1)!
(C) (n + 1). (n + 1)! (D) None of these
Q.19 If the 6th term in the expansion of the binomial
8
 1 2 
Q.12 Let Pm stand for Pm . Then the expression 1.P1 +  8/3 + x log10 x  is 5600, then x equals to
n

x 
2.P2 +3.P3 + ..... + n.Pn =
(A) 5 (B) 8 (C) 10 (D) 100
(A) (n + 1)! –1 (B) (n + 1)! + 1
(C) (n + 1)! (D) None of these
(
Q.20 (1 + x ) 1 + x + x2 ) (1 + x + x 2
)
+ x3 .....

Q.13 The expression


(1 + x + x 2
+ .... + x100 ) when written in the ascending
  
7

7
1   1 + 4x + 1  −  1 − 4x + 1   power of x then the highest exponent of x is______.
4x + 1   2   2 
 
 (A) 4950 (B) 5050 (C) 5150 (D) None of these
is a polynomial in x of degree
( )
n
(A) 7 (B) 5 (C) 4 (D) 3 Q.21 Let 5 + 2 6 p + f where n ∈ N and p ∈ N and
=

0 < f < 1 then the value of, f2 – f + pf – p is


n
 a 
Q.14 If the second term of the expansion a1/13 +  (A) A natural number (B) A negative integer
n
C3 
 a−1 

is 14a5/2 then the value of is (C) A prime number (D) Are irrational number
n
C2
Q.22 Number of rational terms in the expansion of
(A) 4 (B) 3 (C) 12 (D) 6

( )
100
2+43 is-
(
Q.15 If (1 + x ) 1 + x + x2 )
(A) 25 (B) 26 (C) 27 (D) 28
(1 + x + x 2
+ x ) ..... (1 + x + x
3 2
+ x3 + .... + xn )
Q.23 The greatest value of the term independent of x in
= a0 + a1 x + a2 x2 + a3 x3 + .... + am xm 10
 cos θ 
m the expansion of  x sin θ +  is
Then ∑ ar has the value equal to  x 
10
r =0
10 10 C5
(A) C5 (B) 25 (C) 25. C5 (D)
(A) n! (B) (n + 1)! 25
(C) (n –1)! (D) None of these
(1 + x − 3x )
2145
Q.24 If 2
=a0 + a1 x + a2 x2 + .... then
Q.16 In the expansion of (1 + x) if the coefficient of 43 a0 − a1 + a2 − a3 + .... end with
the (2r + 1)th and the (r + 2)th terms are equal, the value
of r is : (A) 1 (B) 3 (C) 7 (D) 9

(A) 12 (B) 13 (C) 14 (D) 15


Q.25 Coefficient of x6 in the binomial expansion
9
Q.17 The positive value of a so that the coefficient of x5  4x2 3 
 −  is
 a 
10  3 2x 
is equal to that of x in the expansion of  x2 + 
15
is 
 x3 
(A) 2438 (B) 2688 (C) 2868 (D) None
1 1
(A) (B) (C) 1 (D) 2 3
2 3 3
M a them a ti cs | 4.37

Q.26 The expression Q.4 For 2 ≤ r ≤ n, nCr + 2n Cr −1 +n Cr −2


5 5

( ) ( )
1/2   1/2 
3 3 Is equal to (2000)
x + x − 1  + x − x − 1 
   
n+1
is a polynomial of degree (A) Cr −1 (B) 2n+1 Cr +1
(A) 5 (B) 6 (C) 7 (D) 8
(C) 2n+ 2 Cr (D) n+ 2
Cr

( )
Q.27 Given 1 − 2x + 5x2 − 10x3 (1 + x ) = 1 + a1x + a2x2
n

Q.5 Let Tn denotes the number of triangles which can


+ ..... and that a12 = 2a2 then the value of n is be formed using the vertices of a regular polygon of n
sides. If Tn+1 − Tn =
21 , then n equals  (2001)
(A) 6 (B) 2 (C) 5 (D) 3
(A) 5 (B) 7 (C) 6 (D) 4
Q.28 The sum of the series
aC0 + ( a + b ) C1 + ( a + 2b ) C2 + .... + ( a + nb ) Cn Q.6 If n−1
Cr
= (k 2
−3 ) n
Cr +1 , then k belongs to  (2004)

is where Cr denotes combinatorial coefficient in the


expansion of (1 + x)n, n ∈ N
(A) ( −∞ , −2 (B)  −2, − 3 ∪
 ) ( 3,2

(A) (a + 2nb)2n (B) (2a + nb)2n


(C) (a + nb)2n–1 (D) (2a + nb)2n–1
(C)  − 3, 3  (D)
  ( 3, ∞ 

30
Q.7 C030 C10 − 30
C1 30C11 + ...... 30
C20 30C30 is equal to
Previous Years’ Questions  (2005)
30 60 30 65
Q.1 Given positive integers r > 1, n > 2 and the (A) C11 (B) C10 (C) C10 (D) C55
coefficient of (3r)th and (r + 2)th terms in the binomial
expansion of (1 + x)2n are equal. Then (1980) Q.8 For r = 0, 1, ....., let Ar, Br and Cr denote, respectively, the
(A) n = 2r (B) n = 2r + 1 coefficient of xr in the expansions of (1 + x)10, (1 + x)20 and
10
(C) n = 3r (D) None of these (1 + x)30. Then ∑ Ar (B10Br − C10 Ar ) is equal to  (2010)
r =1

Q.2 If Cr stands for nCr, then the sum of the series (A) B10 – C10 2
(B) A10 B10 (
− C10 A10 )
n n (C) 0 (D) C10 − B10
2  ! !
2 2 . C20 − 2C12 + 3C22 − .... + ( −1 ) (n + 1 ) Cn2 
n

n!  
Q.9 If the coefficients of x3 and x 4 in the expansion
Where n is an even positive integer, is equal to (1986)
( )
of 1 + ax + bx2 (1 − 2x ) in powers of x are both zero,
18

(A) ( −1 ) (n + 2 ) (B) ( −1 ) (n + 1 )
n/2 n
then (a, b) is equal to:  (2014)

 251   251 
(C) ( −1 ) (n + 1 )
n/2
(D) None of these (A)  16,  (B)  14, 
 3   3 

Q.3 The expression  272   272 


(C)  14,  (D)  16, 
5 5  3   3 

( ) ( )
1/2   1/2 
3 3
x + x − 1  + x − x − 1 
   
Q.10 The sum of coefficients of integral powers of x in
is a polynomial of degree  (1992)

(A) 5 (B) 6 (C) 7 (D) 8


4 . 3 8 | Binomial Theorem

( )
50
the binomial expansion of 1 − 2 x is:  (2015) Q.11 If the number of terms in the expansion of
n
 2 4
(A)
2
(
1 50
3 + 1 (B)
2
)
1 50
3 ( )  1 − + 2  , x ≠ 0 , is 28, then the sum of the
 x x 
coefficients of all the terms in this expansion, is:(2016)
(C) (
1 50
2
3 − 1 (D))1 50
2
2 +1 ( )
(A) 64 (B) 2187 (C) 243 (D) 729

JEE Advanced/Boards

Exercise 1 Q.5 Let a and b be the coefficient of x3 in (1 + x + 2x2 +


3x3)4 and (1 + x + 2x2 + 3x3+ 4x4)4 respectively. Find the
value of (a – b).
Q.1 Let f ( x ) = 1 − x + x2 − x3 ..... + x16 − x17

= a0 + a1 (1 + x ) + a2 (1 + x ) + .... + a Q.6 Prove that the ratio of the coefficient of x10 in


(1 + x )
2 17
,
17 10
 2
Find the value of a2. (1 – x2)10 & the term independent of x in  x − 
 x
is 1 : 32.
Q.2 (a) Find the term independent of x in the expansion of
10
Q.7 Find the coefficient of
8
 x 3 1 
(i)  +  (ii)  x1/3 + x −1/5  (a) x2y3z4 in the expansion of (ax – by + cz)9.
 3 2x2  2 
(b) a2b3c4d in the expansion of (a – b – c + d)10.
(b) Find the value of x for which the fourth term in the
2 n
expansion, q+1 q+1 q+1
Q.8 Given Sn =1 + +  + .... +   ,
 2 log 
8 2  2   2 
4 x + 44 1
 55 5 +  is 336. n+1 n+1 n+1
 3
log5 2x −1 + 7  q ≠ 1 , prove that C1 + C2 .S1 + C3 .S2
 5 
n+1
+.... + 2n.Sn .
Cn+1 .Sn =
Q.3 Find the coefficients:
11 Q.9 Find numerically the greatest term in the expansion of
 1 
(i) x7 in  ax2 +  3
 bx  (i) (2 + 3x)9 when x =
2
 1 
11 1
(ii) (3 – 5x)15 when x =
(ii) x −7 in  ax −  5
 bx2 
Q.10 Given that
(iii) Find the relation between a and b, so that these
(1 + x + x )
n
coefficients are equal. 2
= a0 + a1 x + a2 x2 + .... + a2n x2n ,

Find the values of :


Q.4 (a) If the coefficients of the rth, (r + 1)th & (r + 2)th
terms in the expansion of (1 + x)14 are in AP, find r. (i) a0 + a1 + a2 + .... + a2n ;
(b) If the coefficients of 2 , 3 & 4 terms in the
nd rd th
(ii) a0 − a1 + a2 − a3 .... + a2n ;
expansion of (1 + x)2n are in AP, show that 2n2– 9n + 7 = 0.
(iii) a20 − a12 + a22 − a32 + .... + a22n
M a them a ti cs | 4.39

Q.11 For which positive values of x is the fourth term in Paragraph for questions. 22 and 23
the expansion of (5 + 3x)10 is the greatest.
A path of length n is a sequence of points
x 2
n
( x1 , y1 ) , ( x2 , y 2 ) ,....., ( xn , yn ) with integer coordinates
Q.12 Find the index n of the binomial  +  if the
5 5 such that for all i between 1 and n – 1 both inclusive,
9 term of the expansion has numerically the greatest
th either xi+1= xi + 1 and y i+1 = y i (in which case we say
coefficient (n ∈ N) . the ith step is rightward) or xi+1= xi and y i+1= y i + 1 (in
which case we say that the ith step is upward).
Q.13 Find the number of divisors of the number This path is said to start at (x1, y1) and end at ( xn , yn ) .
2000 Let P (a, b), for a and b non negative integers, denotes
N
= C1 + 2. 2000 C2 + 3. 2000 C3 + .... + 2000. 2000 C2000
the number of paths that start at (0, 0) and end at (a, b)
10
Q.14 Find number of different dissimilar terms in the
sum
Q.22 The value of ∑ P (i,10 − i) , is
i=0

( ) ( )
2011 2010
(1 + x )
2012 2 3
+ 1+x + 1+x (A) 1024 (B) 512 (C) 256 (D) 128

Q.15 Find the term independent of x in the expansion Q.23 Number of ordered pairs (i, j) where i ≠ j for which
9 P (i, 100 – i) = P( j, 100 – j), is
 3x2 1 
(
of 1 + x + 2x3 
 2
−  .)
3x  (A) 50 (B) 99 (C) 100 (D) 101

( )
2n+1
k  n n Q.24 If 6 6 + 14 = N & F be the fractional part of
Q.16 Let f(n) = ∑∑   . Find the total number of
k rr 
=r 0= N, prove that NF = 20 2n+1
(n ∈ N) .
divisors of f (11).
( )
5
Q.25 Let P = 2 + 3 and f = P – [P], where [P] denotes
i 11 11
Q.17 Find the sum ∑∑   .  f2 
=j 0=i j  j 
the greatest integer function. Find the value of  .
1 − f 
n  
[Note :   = n Cr ]
r  Q.26 If C0, C1, C2, …. , Cn are the combinatorial
coefficients in the expansion of (1 + x)n , n ∈ N then
n+ 4
( ) . (1 + x ) prove the following:
2 n
Q.18 Let 1 + x2 ∑ ak .xk . If a1, a2 and a3 are
=
k =0 n.2n−1
(a) C1 + 2C2 + 3C3 + .... + n.Cn =
in AP, find n.
(b) C0 + 2C1 + 3C2 + .... + (n + 1)Cn =(n + 2) 2n−1
n
Q.19 Prove that ∑ n Ck sink x.cos (n − k ) x =
2n−1 sinnx . (c) C0 + 3C1 + 5C2 + .... + (2n + 1)C =
n (n + 1) 2n
K =0
(d) ( C0 + C1 )( C1 + C2 )( C2 + C3 ) ....(Cn−1 + Cn )
Q.20 Find the sum of the roots (real or complex) of the
C0 .C1 .C2 ....Cn−1 (n + 1 )
n
2001
1 
equation x2001 +  − x  =0. =
2  n!
(n + 1)( 2n)!
2n
(e) 1.C20 + 3.C12 + 5.C22 + .... + ( 2n + 1) Cn2 =n!n!
Q.21 If for n ∈ N, ∑ (−1)k (2n Ck )2 =
A , then what will
k =0
2n
Q.27 Let I denotes the integral part and F the proper

(3 + 5 )
n
be the value of ∑ (−1) k
(k − 2n) ( Ck ) ? 2n 2
fractional part of where n ∈ N and if ρ
k =0
denotes the rational part and σ the irrational part of
the same, show that
4 . 4 0 | Binomial Theorem

1 1 100 100
=
ρ
2
(I + 1) and =
σ
2
(I + 2F − 1) Q.31 Let ∑ ∑ ( C12 + Cs2 + C=
r Cs ) m ( 2n
)
Cn + 2p
=r 0=s 0

Where m, n and p are even natural numbers and Cr


Q.28 Prove that represents the coefficient of xr in the expansion of
C1 2C2 3C3 n.Cn n (n + 1 ) (1 + x)100. Find the value of (m + n + p).
(a) + + + .... + =
C0 C1 C2 Cn−1 2
Q.32 The expressions 1 + x, 1 + x +x2, 1 + x + x2 + x3, .....
n+1
C1 C2 Cn 2 −1 1 + x + x2 + ..... + xn are multiplied together and the terms
(b) C0 + + + .... + =
2 3 n+1 n+1 of the product thus obtained are arranged in increasing
powers of x in the form of a0 + a1x + a2x2 + ......, then
22.C1 23.C2 24 .C3
(c) 2.C0 + + + (a) How many terms are there in the product.
2 3 4
(b) Show that the coefficients of the terms in the product,
2n+1 ⋅ Cn 3n+1 − 1 equidistant from the beginning and end are equal.
+.... + =
n+1 n+1
(c) Show that the sum of the odd coefficients = the sum
C1 C2 Cn 1 (n + 1 ) !
− .... + ( −1 )
n
(d) C0 − + = of the even coefficients =
2 3 n+1 n+1 2

Q.29 Prove the following identities using the theory


Q.33 Let S1
= ∑
= ∑ CiC j ,S2
0 ≤ i < j ≤100
of permutation where C0, C1, C2, .... , Cn are the
combinatorial coefficients in the expansion of (1 + x ) ,
n
∑ ∑ CiC j and S3 = ∑ ∑ CiC j
0 ≤ j <i ≤100 0 ≤ i = j ≤ 100
n ∈ N , the prove the following :
Where Cr represents coefficient of xr in the binomial
(a) C0 C1 + C1C2 + C2C3 + .... + expansion of (1 + x)100.

2n! If S1 + S2 + S3 = ab where a, b ∈ N then find the least


Cn−1Cn = value of (a + b).
( ) ! (n − 1 ) !
n + 1

(b) C0 Cr + C1Cr +1 + C2Cr + 2 + .... + Cn−r Cn = 2n!


(n − r ) ! (n + r ) ! Exercise 2
n− 2 ( 2n) !
(c) ∑ ( n Cr . n Cr +2 ) = Single Correct Choice Type
r =0 (n − 2 ) ! (n + 2 ) !
Q.1 In the binomial (21/3 + 3–1/3), if the ratio of the
(d) 100
C10 + 5. 100
C11 + 10. 100
C12 + 10. seventh term from the beginning of the expansion to
the seventh term from its end is 1/6, then n =
100
C13 + 5. 100 C14 + 100 C15 =
105
C90 (A) 6 (B) 9 (C) 12 (D) 15

Q.30 If a0, a1, a2,..... be the coefficients in the expansion Q.2 The remainder, when (1523 + 2323) is divided by 19, is
of (1 + x + x2)n in ascending powers of x, then prove
(A) 4 (B) 15 (C) 0 (D) 18
that :
(i) a0 a1 − a1a2 + a2a3 − .... =
0 Q.3 The value of 4 {Cn
1 + 4. n C2 + 42. n C3 + .... + 4n−1 is}
(ii) a0 a2 − a1a3 + a2a4 − .... + a2n−2a2n = an+1 or an−1
(A) 0 (B) 5n + 1 (C) 5n (D) 5n – 1
(iii) E=
1 E=
2 E=
3 3n−1 ;
Q.4 If n be a positive integer such that n ≥ 3 ,then the
Where E1 = a0 + a3 + a6 + ....;E2 = a1 + a4 + a7
value of the sum to n terms of the series
+..... & E3 = a2 + a5 + a8 + .... (n − 1 ) (n − 1)(n − 2)
1.n −
1!
(n − 1 ) + 2!
M a them a ti cs | 4.41

(n − 1)(n − 2)(n − 3) Q.10 The expression


(n − 2 ) − 3!
(n − 3) + ..... is 5 5

( ) ( )
1/2   1/2 
3 3
x + x − 1  + x − x − 1 
(A) 0 (B) 1    

(C) –1 (D) None of these Is a polynomial of degree


(A) 5 (B) 6 (C) 7 (D) 8
Q.5 If the 6 term in the expansion of the binomial
th

8
 1 2  Q.11 Value of the expression C20 + C12 + C22 + .... + Cn2 is
is
 8/3 + x log10 x  is 5600, then x equals to-
x  (A) 22n ‒ 1 (B) 2n (2nCn)
(A) 5 (B) 8 (C) 10 (D) 100 (C) 2nCn (D) None of these

Q.6 Coefficient of α t in the expansion of,


Q.12 The sum of the series
(α + p) + (α + p) ( α + q) +
m−1 m− 2
aC0 + ( a + b ) C1 + ( a + 2b ) C2 + ..... + ( a + nb ) Cn is

(α + p) ( α + q) + ..... + ( α + q)
m −3 2 m−1
where Cr denotes combinatorial coefficient in the
expansion of (1 + x)n, n ∈ N
Where α ≠ −q and p ≠ q is:
(A) (a + 2nb)2n (B) (2a + nb)2n

(A)
m
(
Ct pt − qt ) (B)
m
(
Ct pm−t − qm−t ) (C) (a + nb)2n–1 (D) (2a + nb)2n–1
p−q p−q

(C)
m
(
Ct pt + qt ) (D)
m
(
Ct pm−t + qm−t ) Previous Years’ Questions
p−q p−q
( 1) n.Cn 
n
Q.1 Prove that C12 − 2.C22 + C32 − ..... − 2n.C22n =−
( )
2145
Q.7 If 1 + x − 3x2 = a0 + a1 x + a2 x2 + ...  (1979)
then a0 − a1 + a − a3 + .... end with Q.2 Given,
2
(A) 1 (B) 3 (C) 7 (D) 9 Sn = 1 + q + q2 + .... + qn
2 n
Q.8 Coefficient of x6 in the binomial expansion q+1 q+1 q+1
Sn =
1+ +  +.... +   ,q ≠ 1
9 2  2   2 
 4x2 3 
 −  is n+1 n+1 n+1
 3 2x  Prove that C1 + C2S1 + C3S2

n+1 n
(A) 2438 (B) 2688 +.... + Cn+1Sn =
2 Sn  (1984)

(C) 2868 (D) None of these


Q.3 Find the sum of the series
n  1 3r 7r 15r 
∑ ( −1 )
r n
Q.9 The term independent of ‘x’ in the expansion of Cr  + + + ...upto m terms 
r 2r 3r 4r
18 r =0  2 2 2 2 
 1 
 9x −  , x > 0, is α times the corresponding  (1985)
 3 x
2n 2n
∑ ar ( x − 2=
) ∑ br ( x − 3)
r r
binomial coefficient. Then ‘ α ’ is: Q.4 If and ak = 1 for all
=r 0=r 0
1 1 2n+1
(A) 3 (B) (C) – (D) 1 k ≥ n , then show that bn = Cn+1  (1992)
3 3
4 . 4 2 | Binomial Theorem

Q.5 Let n be a positive integer and Q.8 The coefficients of three consecutive terms of

(1 + x + x ) (1 + x )
n n+5
2
= a0 + a1 x + .... + a2n x2n . are in the ratio 5 : 10 : 14. Then n =__(2013)

Show that a20 − a12 + .... + a22n =


an  (1994) Q.9 Coefficient of x11 in the expansion of

(1 + x ) (1 + x ) (1 + x )
4 7 12
2 3 4
is (2014)
Q.6 Prove that  (2003)
2k nC0nCk − 2k −1 nC1n−1Ck −1 + 2k −2 (A) 1051 (B) 1106 (C) 1113 (D) 1120
n
C2n−2Ck −2 − ...... + ( −1)k nCkn−k C0 =nCk
Q.10 The coefficient of x9 in the expansion of

(1 + x ) (1 + x2 )(1 + x3 ) .... (1 + x100 ) is  (2015)


Q.7 For r = 0, 1, …, 10, let Ar ,Br and Cr denote,

respectively, the coefficient of xr in the expansions of −1 + 3i


Q.11 Let z = , where i= −1 , and r,s∈{1,2,3} .
10 2
(1 + x ) , (1 + x ) and (1 + x ) . Then ∑ (B10Br − C10 Ar )
10 20 30

 −z r z 2s 
r =1
Let P = 
( ) and I be the identity matrix of
is equal to  (2010)
 z 2s zr 

(A) B10-C10 2
(
(B) A10 B10 − C10 A10 ) order 2. Then the total number of ordered pairs (r, s) for
(C) 0 (D) C10-B10 which P2 = −I is (2016)

PlancEssential Questions
JEE Main/Boards JEE Advanced/Boards
Exercise 1 Exercise 1
Q. 3 Q. 16 Q. 19 Q. 23 Q. 14 Q. 23 Q. 26 Q. 31
Q. 28 Q. 32 Q. 34 Q. 34 Q. 35

Exercise 2 Exercise 2
Q. 7 Q. 13 Q. 15 Q. 21 Q. 2 Q. 4 Q. 12
Q. 22 Q. 25 Q. 29
Previous Years’ Questions
Previous Years’ Questions Q. 3 Q. 4
Q. 2 Q. 3 Q. 5 Q. 6
Q. 8
M a them a ti cs | 4.43

Answer Key

JEE Main/Boards
Exercise 1
Q.1 x10 + 10x8 a + 40x6 a2 + 80x 4 a3 + 80x2a4 + 32a5 Q.2 4ab 3a4 + 10a2b2 + 3b 4⇒
 , 140 2
 
2n!
( −1 ) xn
n
Q.4 100 Q.5 41 Q.6 280x2 Q.7
n!n!
1
Q.9 n = 6, p = Q.10 m = 12 Q.15 n = 12, r = 1 Q.20 34, 23
2
Q.23 232 Q.24 k = ± 3
Q.21 11, T3+1, T3+2, T3+3 Q.26 54C14
7
Q.27 Q.28 n = 4, a = 2
18

Exercise 2

Single Correct Choice Type

Q.1 C Q.2 C Q.3 A Q.4 D Q.5 A Q.6 A

Q.7 A Q.8 D Q.9 D Q.10 A Q.11 B Q.12 A

Q.13 D Q.14 A Q.15 B Q.16 C Q.17 A Q.18 C

Q.19 C Q.20 B Q.21 B Q.22 B Q.23 D Q.24 B

Q.25 B Q.26 C Q.27 A Q.28 D

Previous Years’ Questions


Q.1 A Q.2 A Q.3 C Q.4 D Q.5 B Q.6 B

Q.7 C Q.8 D Q.9 D Q.10 A Q.11 D

JEE Advanced/Boards
Exercise 1
5
Q.1 816 Q.2 (a) (i) (ii) T6 = 7, (b) x = 0 or 1
12
11 a6 11 a5
Q.3 (i) C5 (ii) C6 (iii) ab = 1 Q.4 (a) r = 5 or 9
b5 b6
Q.5 0 Q.7 (a) – 1260 a2b3c4; (b) – 12600
7.313
Q.9 (i) T7 = (ii) 455 × 312 Q.10 (i) 3n (ii) 1, (iii) an
3
4 . 4 4 | Binomial Theorem

5 20
Q.11 <x< Q.12 n = 12 Q.13 8016 Q.14 4023
8 21

17
Q.15 Q.16 24 Q.17 4095 Q.18 n = 2 or 3 or 4
54
Q.20 500 Q.22 A Q.23 C Q.25 722

n2 + n + 2 n(n + 1) (n + 1)!
Q.31 502 Q.32 (a) , (b) a0 = a , (c) Q.33 66
2 2 2

Exercise 2

Single Correct Choice Type

Q.1 B Q.2 C Q.3 D Q.4 A Q.5 C Q.6 B

Q.7 B Q.8 B Q.9 D Q.10 C Q.11 C Q.12 D

Previous Years’ Questions

2mn − 1
Q.3 Q.9 C
(
2mn 2n − 1 )

Solutions

JEE Main/Boards + 6C4a2b4–6C5a4b5+6C0b6)

= 2[6a5b+20a3b3+6ab5]=4ab[3a4 +10a2b2 + 3b4]


Exercise 1
For finding the value, put a = 2 b = 1
Sol 1: (x + 2a)
2 5
(12 + 2013
∴ 2 (12 20 + )3)
= C0 (x ) + C1 (x ) (2a) + C2(x )
5 2 5 5 2 5–1 1 5 2 5–2
⇒ 140 2
(2a)3+5C3(x2)5–3(2a)3+5C4(x2)5–4

(2a)4+5C5(x2)5–5(2a)5 Sol 3: (101)50 > (100)50 +(99)50


(100+1)50> (100)50+(100-1)50
=x10+5x8(2a)+10x6(2a)2+10x4(2a)3+5x2(2a)4 +(2a)5
= (100+1)50-(100-1)50>10050
= x10+10x8a+40x6a2+80x4a3 +80x2a4+32a5
Both binomial will cancel every odd terms of each
Sol 2: (a+b) – (a–b)
6 6 others rest of the even terms are.

C0a6+6C1a5b+6C2a4b2+6C3a3b3
6 = 2[50C1(100)49 + 50C3(100)47]+ 50C5(100)45+

+ 6C4a2b4+6C5ab5+6C6b6 ……+50C49100]

-(6C0a6–6C1a5b+6C2a4b2–6C3a3b3 = 100(100)49+2[50C3(100)47 +……. + 50C49(100)]


M a them a ti cs | 4.45

= (100)50 + 2[ 50 C3 (100)47 + ... + 50


C 47 (100)] > 10050 Sol 7: (1-2x+x2)n = (1-2x+x2)n

Which is always true =(–1+x)2n = (x-1)2n


2n
So (101)50-(99)50 > (100)50 Middle term=
= Tn+1 Cn (x)2n−n ( −1)n

= (101)50 > 10050 – (99)50 2n! n


= x ( −1)n
n!n!
Sol 4: x > 1 2n
 1
5 4 Sol 8:  x + 
1 x log x  7  1  x
 +
10
 and =
T4 C3 (2 x2 )3 ⋅  =  280y 2 2n
x  x
  Greatest coefficient = Cn
5 −2

(x )
1 log10 x
2 2n! 2n!
T3 T=
= 5
C2   = 1000 = =
2 +1
x n!(2n − n)! n!n!

2n(2n − 1)(2n − 2)(2n − 3)(2n − 4).....3.2.1


 1  2log x
( log x2
= 10   x 10 = 1000 ⇒ x 10 = 100x3
 x3 
) =
n!(n(n − 1)(n − 2)(n − 3)..........3.2.1)

Assume x = 10 y 2n [n(n − 1)(n − 2)(n − 3)1]1.3.5.7........(2n − 1)


y 2 =
⇒ 10 =
y log
10 (10 ) 100(10
= y 3
) 102+3y n!(n(n − 1)......3.2.1)
2y(log10 10 y ) 2y 2
⇒ 10 = 10
= 102+3y 2n1.3.5.......(2n − 1)
=
⇒ 2y2=2+3y ⇒ 2y2-3y-2=0 n!
n
⇒ (y-2)(2y+1)=0  1
Sol 9: =  px + 
1  x
⇒ y=2 or y = −
2 5
Given = 4th term =
1 2
⇒ x=102 or x = 10−1/2 ⇒ x=100 or x =
10 3
n n−3  1 
5
But x>1 so x=100 T4 T=
= 3+1 C3 (px) =
x 2
5
Sol 5: ( 2 + 31/5 )10 ⇒ n C3pn−3 xn−3+3( −1) = x0
2
For rational number 5
⇒ n = 6 ⇒ 6 C3p6 −3 x0 =
y
( 2) → y= 2n,n ∈ N,0 2
6 × 4 ×5 3 5
(31/5 )z → z= 5n,n ∈ N,0 ⇒ p =
1.2.3 2
3
Rational terms 1 1 1
⇒ p3= =   ⇒ p = and n = 6
10
8 2 2
C0 ( 2)10 + 10 C10 ( 2)0 (31/5 )10 =25+32=32+9=41
Sol 10: (1+x)m (1-x)n
7
 1 = (mC0x0+mC1x1+mC2x2+……. +mCmxm)
Sol 6:  2x2 − 
 x
(nC0+nC1(–x)+nC2(-x)2+…. . nC2(-x)n)
Middle terms are T4 = T3+1 and T5 = T4 +1
terms of x= mC0 nC1 (-1) + nC0 mC1
4
7  1 7 × 6 ×5 2 3 1
T4 +1
= C 4 (2x2 )7 − 4 =
−  (2x ) = (–n)+m=m–n=3 (given)  …. (i)
 x  1.2.3 x4
terms of
x6
= 35 × 8 × = 280x2
x4 x2= mC0 nC2(–1)2+ mC2 nC0+mC1nC1(–1)
4 . 4 6 | Binomial Theorem

n × (n − 1) m(m − 1) = (35+1)n –35n–1


= 1. 1+ 1 + m( −n)
1.2 1.2 = n
C0 35n + nC1 35n−1 + ....... + nCn−2 352
n2 − n m2 − m
= + − mn =
−6  …. (ii) + n Cn−1 35 + n Cn 350 − 35n − 1
2 2
In equation (i) m–n=3 ⇒ n = (m-3) And 1225 = 352
Put the values of n in eq. (ii) so each term is a multiple of 352 and is divisible by
1225
(m − 3)2 − (m − 3) m2 − m
+ −m(m − 3) =−6
2 2
Sol 13: 34n+1 +16n–3is divisible by 256
m + 3 – 3(2)(m) – m + 3 + m – m – 2m + 6m = 12
2 2 2 2
8
256
= 2= 44
2m2 – 6m + 9 + 3 – 2m – 2m2 + 6m = – 12 12 − 2m =
−12
= 34n+1+16n–3
⇒ 2m = 24 ⇒ m = 12 and n = 9
= 3. 34n +16n–3

Sol 11: Coefficient of ar–1, ar, ar+1 in the binomial = 3[4–1]4n+16n–3


expansion of (1+a)n are in A. P. so
= 3[ 4nC0 4 4n + 4nC1 4 4n−1 ( −1) + ..... +
r −1 n r −1
Terms of a = T=
r Cr −1 (a) 4n
C 4n−2 (4)4n− 4n+ 2 − 4nC 4n−1 (4)4n− 4n+1 + 4nC 4n ] +16n − 3
r n r
Terms of=
a T=
r +1 Cr a
= all terms which is multiple of 4 4 is divisible by 256.
r +1 n r +1
Terms of a= T=
r +2 Cr +1a So rest of the terms

Coefficients of Tr , Tr +1 , Tr + 2 are in A. P. so = 3[ − 4n C 4n−2 (4)2 − 4nC 4n−1 (4)1 + 1] + 16n − 3


n
Cr −1 + nCr +1 =
2 nCr 4n(4n − 1) 2
3[ × 4 − 4n × 4 + 1] + 16n − 3
1.2
n! n! n!
+ =2 = 128n2. 3–128n=128(3n2–1)
(r − 1)!(n − r + 1)! (r + 1)!(n − r − 1)! r!(n − r)!
= 128n(3n-1) and (3n–1) is always even
1

(r − 1)!(n − r + 1)(n − r)(n − r − 1)! so 128n(3n − 1) = 128 × 2x +1 (assume), x ∈ N

1 2 = 256 × 2x , which is divisible by 256


+ =
(r + 1)r(r − 1)!(n − r − 1)! r(r − 1)!(n − r)(n − r − 1)!
Sol 14: a1, a2, a3 and a4 are any four consecutive
1 1 2 coefficients in the expansion of (1+x)n
⇒ + =
(n − r)(n − r + 1) r(r + 1) r(n − r)
a1= nCr, a2=nCr+1
r(r + 1) + (n − r)(n − r + 1) 2
⇒ = a3=nCr+2, a4=nCr+3
r(r + 1)(n − r)(n − r + 1) r(n − r)
L. H. S.
⇒ r2 + r + n2 – nr + n – nr + r2 – r = 2(r + 1)(n – r + 1)
a1 a3
= +
⇒ 2r2 + n2 – 2nr + n = 2rn – 2r2 + 2r + 2n – 2r + 2 a1 + a2 a3 + a4
n n
Cr Cr + 2
⇒ n2 + 4r2 – 4rn – n – 2 = 0 = +
n n n
Cr + Cr +1 Cr + 2 + nCr +3
⇒ n2 – n(4r + 1) + 4r2 – 2 = 0
n n
Cr Cr + 2
= +
n+1 n+1
Sol 12: n is a positive integer Cr +1 Cr +3

⇒ 62n–35n–1 = (62 )n − 35n − 1 = (36)n − 35n − 1 n!(r + 1)!(n− r)! n!(r + 3)!(n− r − 2)!
= +
(n+ 1)!(r)!(n− r)! (n+ 1)!(r + 2)!(n− r − 2)!
M a them a ti cs | 4.47

(r + 1) (r + 3) 2r + 4 2(r + 2) From (i) and (ii)


= + = =
(n+ 1) (n+ 1) n+1 n+1
an − b bn − b − b − 2c
=
n!(r + 2)!(n− r − 1)! n
Cr +1 a+b b+c
= 2 = 2
(n + 1)!(r + 1)!(n− r − 1)! n+1
Cr + 2 ⇒ abn – b2 + acn – bc = abn – 2ab – 2ac + b2n – 2b2 – 2bc
n
Cr +1 2a2 ⇒ n(ac − b2 ) =−ab − 2ac − bc
= 2 =
n
Cr + 2 + nCr +1 a2 + a3
ab + 2ac + bc 2ac + b(a + c)
=⇒n =
b2 − ac b2 − ac
Sol 15: 3 consecutive coefficients in the expansion of
(1 + x)n are in the ratio 6 : 33 : 110
11
= Tr+1 : Tr+2 : Tr+3  1
Sol 17:  x −  , y ≠ 0
 y
= nCr : nCr+1 : nCr+2 2
11  1  1
n! n! n! = C0 x11 + 11C1 x11−1  −  + 11C2 x11−2 − 
= : :  y  y
r!n − r! r + 1!n − r − 1! r + 2!n − r − 2!
1 1 1  1
3
 1
11
= : : + 11C3 x11−3  −  +  + 11C11  − 
(n − r)(n − r − 1) (r + 1)(n − r − 1) (r + 1)(r + 2)
 y   y
= 6 : 33 : 110

6 2 Sol 18: (1–x+x2)4= ((1–x)+x2)4


(r + 1)(n − r − 1) r +1 2
⇒ = = ⇒ =
(n − r)(n − r − 1) 33 11 n − r 11 = 4C0(1–x)4+4C1(1–x)3x2 + 4C2(1–x)2(x2)2

⇒ 11r + 11 = 2n − 2r ⇒ 2n–13r–11=0 …. (i) +4C3(1–x)(x2)3+4C4(x2)4

= (1–x)4+4x2(1–x)3+6(1+x2–2x)x4+4x6(1–x)+x8
(r + 1)(r + 2) 33 3 r+2 3
And = = ⇒ =
(r + 1)(n − r − 1) 110 10 n − r − 1 10 = (4C0–4C1x+4C2x2–4C3x3+4C4x4)

⇒ 10r + 20 = 3n – 3r – 3 ⇒ 3n – 13r – 23 = 0 …. (ii) + 4x2 ( 3C0 − 3C1 x + 3C2 x2 − 3C3 x3 )

Subtracting equation (i) from (ii), we get n = 12 +(6 + 6x2 − 12x)x 4 + 4x6 − 4x7 + x8
Putting n = 12 in equation (i) = 1 − 4x + 6x2 − 4x3 + x 4 + 4x2 − 12x3
13r = 2n – 11 = 2(12) – 11 = 24 – 11 = 13 ⇒ r = 1
+12x 4 − 4x5 + 6x 4 + 6x6 − 12x5 + 4x6 − 4x7 + x8
So terms are Tr +1 , Tr + 2 , Tr +3
=1 − 4x + 10x2 − 16x3 + 19x 4 − 16x5 + 10x6 − 4x7 + x8

Sol 16: a, b, c are three consecutive coefficients in the


expansion of power (say n) of (1 + x)n Sol 19: (1.2)4000 = (1+0. 2)4000

So a = nCr, b = nCr+1, c = nCr+2 = 4000C0(0. 2)0+4000C1(0. 2)1+4000C2(0. 2)2 +…….

a (r + 1) = 1+4000(0. 2)+………= 1+800+…. . =801+…….


= ⇒ an − ar = br + b
b (n − r)
So (1.2)4000 is greater than 800
an − b
⇒r=  ……(i)
a+b Sol 20: For (1+x)n
b (r + 2) T14,T15 and T16 are in A. P.
=
c (n − r − 1)
n
⇒ T14 + T16=2T15 ⇒ C13 + nC15 =
2 nC14
⇒ bn − br − b = cr + 2c
⇒ nC13+nC15=2 nC14
bn − b − 2c
⇒r = ……. (ii)
b+c 
4 . 4 8 | Binomial Theorem

n! n! n! So greatest is 4+n.
+ 2
= 3
13!(n − 13)! 15!(n − 15)! 14!(n − 14)! 11  2
| T4 | | T=
= 3+1 | C3 (7)11−3  5 × 
 3
1 1 2
+ =
(n − 13)(n − 14) 15 × 14 14(n − 14) 11 × 10 × 9 8 3 23 11 3 4 8
= ×7 5 × = 25 7
1.2.3 33 9
15 × 14 + (n − 13)(n − 14) 2
=
15 × 14(n − 13)(n − 14) 14(n − 14) 8
2  14
Sol 23: (1 + 3x + x )  1 + 
210 + n2 + 182 – n(13 + 14) = 2 × 15(n – 13)  x
n2 + 392 – 27n = 30n – 390 For Coefficient of x −1
n – 57n + 782 = 0
8 1 1 1
= (1) C1   + 3x2 8 C3 + x 4 8 C5
2
57 ± 57 − 4(1)(782) 57 ± 121 57 ± 11 x
  x 3
x5
n = =
2(1) 2 2
8 3 × 8 × 7 × 6 −1 8 × 7 × 6 −1
n = 34, 23 = + ×x + x
x 1×2×3 1.2.3

Sol 21: Given 1 232


= [8 + 168 + 56] =
x x
n
Cr=165; nCr+1=330; nCr+2=462
n Coefficient of x −1 = 232
Cr 165 1
= =
n
Cr +1 330 2
10
 k 

n!(r + 1)!(n − r − 1)! 1
= Sol 24:  x + 2 
 x 
r!(n − r )!n! 2
r 10 −r
 k  10
(r + 1) 1 10
Tr +1 = Cr ( x ) 10 −r
 2 = Cr (x) 2 kr x −2r
⇒ =
(n − r) 2 x 
Tr +1 is independent of x
⇒ 2r+2=n–r
⇒ 3r=n–2 …. (i) 10 − r 10
So − 2r = r = 2
0 ⇒ 10 - 5r = 0 ⇒=
2 5
n
Cr + 2 462 7 10
= = So Coefficient is = C2k 2
n
Cr +1 330 5
10 × 9 2
(r + 1)!n!(n − r − 1)! n − r − 1 7 = 405 ⇒ k 2 = 9 ⇒ k = ±3
×k =
= = 1× 2
n!(r + 2)!(n − r − 2)! r+2 5

⇒ 5n – 5r – 5 = 7r + 14 Sol 25: (x + a)n

⇒ 12r = 5n –19 …. (ii) A = Sum of odd terms

From eq (i) and (ii) B = Sum of even terms

4(n2) = 5n19 ⇒ n = 11 (ii) 2(A2+B2)=(x+a)2n+(x–a)2n

So 3r = 11 – 2 = 9 ⇒ r = 9/3 = 3 A = nC0xn+nC2xn–2a2+…. nCnx0an


Position of coefficients are T3+1 , T3+ 2 , T3+3 B = nC1xx-1a+nC3xn–3a3+……+nCn–1+xan–1
2 2(A2+B2) = (A+B)2 +(A–B)2 = (x+a)2n+(x–a)2n
Sol 22: (7–5x)11, x =
3
L. H. S. = R. H. S.
n+1 11 + 1 12 12
= = = = 3.87
x  7×3 21 3.1
1+ 1+  1 +
a 5×2  10
M a them a ti cs | 4.49

Sol 26: (1+2x+x2)27 = ((1+x)2)27 = (1+x)54 n


 x
Sol 2: (C) In the expansion  2 +  the coefficients of
Tr+1=54Crxr x & x are equal
7 8  3 

Coefficient of x 40 ⇒r=40 1


7 8
n n− 7 n n− 8  1 
C7 (2)   = C8 (2)  
Coefficient = 54C40=54C54–40= 54C14 3 3
6 1
9
= ⇒n − 7 =48 ⇒ n = 48 + 7 = 55
3 1  (n − 7) 8
Sol 27:  x2 − 
 2 3x 
9 −r r  1  1   1 
3   1  Sol 3: (A) (x − 1)  x −   x − 2  ......  x − 49 
=Tr +1 9
Cr  x2  −   2  2   2 
2   3x 
Max power of x = 50
For independence of x Coefficient of x49
2(9 − r) − r = 18 − 2r − r = 18 − 3r =0 1 1 1 1
= −1 − − − −
Coefficient r = 6 2 22 23 249
9 −6 6  1 
50
9 3  1 1 −   
T= T= C6   − 
r +1 6 +1
2  3

=  2  = 
−2 1 −
1 
 
 1−1   250 
9 33 1 9 × 8 × 7 (1) 7×3 7  2 
= C3 × = × = =  
3
2 3 6 1.2.3 3
23 3
2.33 18

Sol 28: (1+ax)n =1+8x+24x2+…… Sol 4: (D) (3P+2)


4n
n
C0 + nC1ax + nC2(ax)2 + …. . = 1 + 8x + 24x2 + …… P=34n, n ∈ N = 33 +2
So nC1a = 8 and nC2a2 = 24 30=1,31=3, 32=9, 33=27,34=81
n(n − 1) 2 Last digit = 1,3,9,7
na = 8 and a = 24
2
Last digit of 3x repeat after every power of 4 so 34n last
a2n2 – na2 = 48 ⇒ (8)2 – 8a = 48 ⇒ 64 – 8a = 48 digit =1
⇒a=2⇒n=4
31=3
31+2=5
Exercise 2 So last digit of 33
4n
+ 2 is 5

Single Correct Choice Type  1


n
Sol 5: (A)  2x + 
 x
Sol 1: (C) (x1/3 –x–1/2)15
Sum of binomial coefficient= 2n=256
Tr+1= 15Cr (x1/3)15–r(–x–1/2)r
2n=28⇒ n=8
15 − r r
x
Power of= −= 0 for x0 Constant term =
3 2
4
2(15–r) –3r=0 8 4 1 8 × 7 × 6 × 5 4 4−4
C 4 (2x)
= .  ×2 x = 1120
x 1.2.3.4
30 – 2r – 3r = 0 ⇒ 5r = 30 ⇒ r = 30/5 = 6
10
Coefficient Tr +1 = 15C6 ×1 = 5005 x 3 
Sol 6: (A)  − 
5m = 5005 ⇒ m =1001  2 x2 
10 −r r
10 x  3 
=Tr +1 Cr   − 2 
2  x 
4 . 5 0 | Binomial Theorem

Power of x=10–r–2(r)=4 (given) 325 + 1= 2(a0 + a2 + a4 + ..... + a50 )


⇒ 10 – 3r = 4 ⇒ 3r = 10 – 4 = 6 ⇒ r = 6/3 = 2 1 25
a0 + a2 + a4 + ..... + a50
= (3 + 1)
10 −2 2
10 1 1
Coefficient
= Tr +1 C2   ( −3)2 = (4 − 1)25 + 1
2 2
10 × 9 −8 2 5 × 9 × 9 405 1
=
1.2
=2 3 =
28 256
=
2
( 25C0 425 − 25C1 424 + .... + 25C24 4 − 1) + 1
= 2 m always even. ( divisible by 2)
Sol 7: (A) 1127+2127
= (16–5)27+(16+5)27 Sol 11: (B) (12+1)1!+(22+1)2!+(32+1)3!+……+(n2+1)n!
= 2  27 C0 1627 + ...... + 27 C26 16  Tn = (n2 + 1) n! = n(n + 1)! – (n – 1) n!
 
Sn = n(n+1)!
= 2. 16k = 32k
Always divisible by 16 Sol 12: (A) Pm → nPm
1P1 + 2P2 + 3P3 + …… + n. Pn
Sol 8: (D) N=700-3100
N = (5 + 2)100 − (5 − 2)200 = 1. n + 2. n(n – 1) + 3n(n – 1)(n – 2)

+ 4n(n – 1)(n – 2)(n – 3) + …. + n. n!


=N 2  100 C1 599.2 + ...... + 100 C99 5.299 
  Add (+1 and ‒1)
N = [ 100 C1 597.100 + 103 100 C3 594 = 1 + nC1 + 2 nC2 2!+ 3 nC3 3! +4 nC 4 4!+ ...n nCnn!− 1
n
+...... + 100 C99 10.299 ] = −1 + 1 + ∑ i nCi (i)!
i=0
=N 1000.[10.597 + .... + 299 ] n
= −1 + 1∑ iPi
Integer i=0

Last three digits = 000 When 1 + 1P1 + 2P2 + 3P3 +.... + nPn = (n + 1)!
= −1 + 1(n + 1)!− 1
Sol 9: (D) 3400 = (32)200
= (n + 1)!− 1
(9)200
= (10 − 1)200
200
= C0 10200 − 200 C1 10199 +.... 200 C199 101 + 200 C200 .1 Sol 13: (D)
= 10m + 1 (m∈N)  
7
  
7
1   1 + 4x + 1  −  1 − 4x + 1  
Last 2 digits are 01 4x + 1   2   2  
 
1.2
Sol 10: (A) (1+x+x2)25 = a0 + a1x + …. + a50x50 =  7 C 4x + 1 + 7 C ( 4x + 1)3
27
4x + 1  1 3

x=1
+...... + 7 C7 ( 4x + 1)7 
325
= a0 + a1 + a2 + ..... + a50 …(i) 

= 2−6  7 C1 + 7 C3 (4x + 1) + 7 C5
x = –1 

(1 – 1 + 1)25 = 1 (4x + 1)2 + 7 C7 (4x + 1)3 



= a0 – a1+ a2 – a3 + …… +a50 …. (ii) ⇒ Max. power of x = 3

Sum of both eqn.


M a them a ti cs | 4.51

Sol 14: (A)  2 a 


10

n Sol 18: (C)  x + 3 


 1/13   x 
a
( ) =(a )
n n
a +  = a1/13 + a1+1/2 1/13
+ a3/2
  Power of x for term
 a−1 
r
 a 
T2
= n
C1 (a1/13 )n−1 + (a3/2
= )1 14a5/2 Tr +1 = 10 Cr (x2 )10 −r  
3
x 
n−1 3
+ Power of x = 2(10 − r) − 3r = 20 − 5r =
0
⇒ n a 13 2 = 14a5/2 ⇒ n = 14
= r 20
⇒ 5r = 20= = /5 4
14
C3 14 − 3 + 1 12
= = = 4 T4 +1 = 10 C 4 binomial coefficient
14
C2 3 3
8
 1 
Sol 15: (B) (1+x)(1+x+x )(1+x+x +x ) 2 2 3 Sol 19: (C)  + x2 log10 x 
8/3
x 
…. . (1+x+…. . +xn) 8 −5
 1 
( x log x )
5
8 2
2 m T6 T=
= 5 +1 C5   10 = 5600
= a0 + a1 x + a2 x + ....... + am x  x8/3 

( )
m
8×7×6 3
∑ ar = a0 + a1 + a2 + ...... + am ⇒
1.2.3
× x −8/3 x10 (log10 x)5 =
5600
r =0

At x = 1 x −8 +10 (log10 x)5 = 100 ⇒ x2 (log10 x)5 = 100

= 2.3.4.5.6.....(n + 1) = (n+1)! Assume x = 10 y

So 102y (log10 10 y )5 = 102 ⇒ 102y −2 y5=1


Sol 16: (C) (1+x)43
⇒ y = 1 ⇒ x = 10
Given T2r +1 = Tr + 2
43 43 43
= C2r Cr +1
= C 43−(r +1) Sol 20: (B) (1+x)(1+x+x2)(1+x+x2+x3)
42 …. . (1+x+…. . x100)
r
⇒ 2r = 43 − r − 1= 42 − r ⇒ 3r = 42 ⇒= = 14
3
Highest power of x =1+2+3+…. . +100
10
 a  100(100 + 1)
Sol 17: (A)  x2 +  = = 50×101=5050
 x3  2

Coefficient of x5 is equal to that of x15 Sol 21: (B) (5 + 2 6 )n =


p+f
r
 a  p [(5 + 2 6 )n ] – f
=
Tr +1 = 10 Cr (x2 )10 −r  
 x3 
f 2 − f + pf − p = f(f − 1) + p(f − 1) = (f − 1)(f + p)
Power of x= 2(10 − r) − 3r = 20–5r n
 1 
Assume F = (5 − 2 6 )n =
 
20 – 5r = 5 ⇒ r = 3 5 + 2 6 
20 – 5r = 15 ⇒ r = 1 0 < f <1, 0 <F <1

T3+1 = T1+1 F + f + p = (5 + 2 6 )n +(5 − 2 6)n =integer = 2I

10
C3a3 = 10 C1a F + f = 2I − p = Integer

0 < F + f < 2 ⇒ F + f =⇒
1 F=1–f
10 × 9 × 8 2
a = 10
1.2.3 (F)(f + p) = (5 − 2 6)n (5 + 2 6)n = – 1
1 1 1
a2 = ⇒a
= =
12 12 2 3
4 . 5 2 | Binomial Theorem

Sol 22: (B) ( 2 + 4 3)100 = (21/2 + 31/ 4 )100 210 × 34


= 9×2×7× = 21 × 27 =
2688
L. C. M. of 2 and 4 =4 35 × 24

Total terms = n + 1= 100 + 1= 101


Sol 26: (C) [x+(x3–1)1/2]5+[x(x3–1)1/2]5
( ) (3 )
100 − 4n 4n
100
T rational = C 4n 21/2 1/ 4
= 2[ 5C0 x5 + 5C2 x5−2 (x3 − 1) + 5C 4 x5− 4 (x3 − 1)2 ]
⇒ 0 ≤ 100 − 4n ≤ 100
Max power of x = 7
⇒ 0 ≤ n ≤ 25 n ∈ N
n = {0,1,2,3.....25} Sol 27: (A) (1–2x+5x2–10x3)(1+x)n
Total number for n = 26 = 1+a1x–1a2x2+…… and a12=2a2

10 Coefficient of x=a1=nC1–2=n–2
 cos θ 
Sol 23: (D)  x sin θ +  n(n − 1)
 x  Co-efficient of x2=a2=5+nC2–2nC1= 5 + − 2n
2
r
10  cos θ  n2 − n − 4n n2 − 5n
Tr +1
= Cr (x sin θ)10 −r   = 5+ 5+
=
 x  2 2

Power of x = 10 − r + r( −1) = 10 − 2r =
0 (given)  10 + n2 − 5n 
a12 = 2a2 ⇒ (n − 2)2 =
2 
⇒r=5  2 
⇒ n2+4–4n=10+n2–5n ⇒ n=6
Tr +1= 10
C5 (sin θ)5 (cos θ)5 = 10 C5 (sin θ cos θ)5
5
10  sin2θ  Sol 28: (D) aC0+(a+b)C1+(a+2b)C2+…. +(a+nb)Cn
= C5   . Max value when sin2θ =1
 2  a(C0+C1+C2+….. +Cn)
10
C5 +b(C1+2C2+……+nCn)
∴ Max. value =
25
= a2n+b[n2n–1] = 2n–1[2a+nb]
Sol 24: (B) (1+x–3x2)2145=a0+a1x+a2x2+
At x= – 1 Previous Years’ Questions
(1 − 1 − 3)2145 =−(3)2145
Sol 1: (A) In the expansion
= a0 − a1 + a2 − a3 + .......
(1 + x ) C3r −1 ( x )
2n 2n 3r −1
,t3r =
2145 2144 1072
L. H. S. = 3 = 3.3 = 3[9]
Cr +1 ( x )
2n r +1
Even power of 9 ends with 1. Hence 32145 ends with 3. tr + 2 =

9 Since, binomial coefficient of t3r and tr+2 are equal.


 4x2 3 
Sol 25: (B)  − 
 3 2x  ⇒ 2n 2n
C3r −1 = Cr +1

 4x2 
9 −r r ⇒ 3r − 1 = r + 1 or 2n = (3r − 1) + (r + 1)
9  3 
=Tr +1 Cr   − 
 3   2x  2r 2 or=
⇒= 2n 4r
 

Power of x= 2(9 − r) + ( −1)r = 18 − 3r =


6 ⇒
= r 1 or
= n 2r

⇒ 3r = 18 − 6 = 12 ⇒ r = 4 But r > 1,

4
9− 4
 −3 
4
9×8×7×6 4  3
5 4 ∴ We take n = 2r
9
Coefficient C4     =    
3 2
  1.2.3.4  3   2 
M a them a ti cs | 4.53

Sol 2: (A) We have Cn2 − 2C12 + 3C22 − 4C32 n


Cr +n Cr −1 =n+1 Cr

+.... + ( −1 ) (n + 1 ) Cn2
n
∴ n+1Cr +n+1 Cr −1 = n+ 2Cr

= {C20 − C12 + C22 − C32 + .... + ( −1 ) Cn2 }


n
n  n   n 
Sol 5: (B)   + 2  + 
r  r − 1 r − 2
−{C12 − 2C22 + 3C32 − .... + ( −1 ) nCn2 }
n

 n   n    n   n  
n! n
−1 n n! =
  +    +  + 
( −1) − ( −1 ) 2
n/2
= ⋅
2 n n  r   r − 1    r − 1   r − 2  
n n
 ! !  ! !
2 2 2 2 n + 1 n + 1 n + 2
= + = 
 r  r −1  r 
( −1)  n n! n   1 + n2 
n/2  
=
  According to given condition, Tn = n C3
 ! !
2
  2
and Tn+1 − Tn =
21
n n
2  ! ! n+1
2 2 ⇒ C3 − n C3 =
21
∴   
n! 1 1

6
( n + 1 )(n)(n − 1 ) − n (n − 1 )(n − 2 ) =
6
21
{C20 − 2C12 + 3C22 − .... + ( −1 ) (n + 1 ) Cn2 }
r

n (n − 1 )
( n + 1 ) − ( n − 2 )  =
n n
2  ! !

6   21
 2   2  −1 n/2 n! (n + 2 ) =
( ) ( −1 ) (n + 2 ) n (n − 1)
n/2
=
n! n n 2 ⇒ 21 ⇒ n (n − 1 ) =
= 42
 ! ! 6
2 2
⇒n=7

Sol 3: (C) We know that ( a + b ) + ( a − b )


5 5

Sol 6: (B) Given, n−1


Cr
= (k 2
)
− 3 n Cr +1
=5 C0 a5 + 5 C1a4b + 5 C2a3b2

5 2 3 5 4 5 5 5 5 5 4
⇒ n−1
(
Cr =k 2 − 3 ) r +n 1 n−1
Cr
+ C3a b + C 4 ab + C5b + C0 a − C1a b
r +1
⇒ k2 − 3 =
n
+ 5 C2a3b2 − 5 C3a2b3 + 5 C 4 ab 4 − 5 C5b5
r +1
(Since,n ≥ r ⇒ ≤ 1 and n,r > 0)
n
=2 a5 + 10a3b2 + 5ab 4 
  ⇒ 0 < k2 − 3 ≤ 1
5 5

( ) ( )
1/2   1/2 
∴  x + x3 − 1 3
 + x − x − 1  ⇒ 3 < k2 ≤ 4
   

(
2
= 2  x5 + 10x3 x3 − 1 + 5x x3 − 1  ) ( )
⇒ k ∈  −2, − 3 ∪
 ) ( 3,2

 
 30   30   30   30 
Therefore, the given expression is a polynomial of Sol 7: (C) Let     −    
 0   10   1   11 
degree 7.
 30   30   30   30 
Sol 4: (D) n Cr + 2 nCr −1 +n Cr −2 +     − .... +    
 2   12   20   30 

(
= n Cr +n Cr −1 + ) ( n
Cr −1 +n Cr −2 ) ∴A
= 30
C0 . 30 C10 − 30
C1 . 30 C11

We know that
+ 30 C2 . 30 C12 − .... + 30
C20 . 30 C30
4 . 5 4 | Binomial Theorem

= Coefficient of x20 in (1 + x)30.(1 – x)30 So, sum of coefficient Integral powers of x

= Coefficient of x20 in (1 – x2)30 S= 50


C0 +50 C2 .22 +50 C 4 .24 + .... +50 C50 .250
30
( )
r
Now,
∑ ( −1)
r 30
= Coefficient of x20 in Cr x2
r =0
(1 + x )
50
1 +50 C1 x +50 C2 x2 +50 C3 x3 +50 C 4 x 4
=
∴ For coefficient of x clearly 2r = 20 ⇒ r = 10 20

+..... +50 C50 x50


30
Put (r = 10) = C10
Put x = 2, - 2
Sol 8: (D) Ar = Coefficient of x in (1 + x) = Cr r 10 10
350 =
1 +50 C1 .2 +50 C2 .22 +50 C3 .23
Br = Coefficient of x in (1 + x) = Cr r 2n 20  ….(i)
+50 C 4 .24 + .... +50 C50 .220
Cr = Coefficient of xr in (1 + x)30 = 30Cr
1= 1 −50 C1 .2 +50 C2 .22 −50 C3 .23  ….(ii)
10 10 10
∴ ∑ Ar (B10Br =
− C10 Ar ) ∑ ArB10Br − ∑ Ar C10 Ar +50 C 4 24 − .... +50 C50 .250
r =1 =r 1 =r 1

10 10 (i) + (ii)
∑ 10
Cr 20 C10 20 Cr − ∑ 10 Cr 30 C10 10 Cr
r 1 =r 1 350 + 1= 2 1 +50 C2 .22 +50 C 4 .24 + .... +50 C50 .250 
 
10 10
350 + 1
= ∑ 10 C10 −r ⋅ 20 C10 20 Cr − ∑ 10 C10 −r 30 C10 10 Cr ∴ =1 +50 C2 .22 +50 C 4 .24 + .... +50 C50 .250
r 1 =r 1 2
10 10
20
C10 ∑ 10 C10 −r . 20 Cr − 30 C10 ∑ 10 C10 −r 10 Cr
(=
n + 1 )(n + 2 )
28
=r 1 =r 1 Sol 11: (D) Number
= of terms
2
= 20
C10 ( 30
)
C10 − 1 − 30 C10 ( 20
C10 − 1 ) ⇒n=
6
n
30 20 a1 a2 a2n  2 4 
= C10 − C10 =C10 − B10 ∴ a0 + + + .... + = 1 − + 
x x2 x2n  x x2 

Put x = 1, n = 6,
Sol 9: (D) 1 + ax + bx2( ) a0 + a1 + a2 + .... + a2n = 36 = 729
1 −18 C 2x +18 C 2x −18 C 2x +18 C 2x 4
2( ) 3( ) 4( ) 
2 3
 1

Coefficient of x3 is
JEE Advanced/Boards
−18 C3 23 + a ( ) ( 18
C2 × 4 − b ) ( 18
C1 × 2 =
0  ) …(i)

Coefficient of x4 is
Exercise 1
18
( ) (
C 4 24 + a −18 C3 × 23 +18 C2b22 =
0 ) ….(ii) Sol 1: f(x)=1–x+x2–x3+……x16–x17

or solving both these equation = a0+a1(1+x)+a2(1+x)2 +……. +a17(1+x)17

a = 16 and b = 272/3. Differentiating both sides

–1 + 2x ... –17x16 = a1 + 2a2(1 + x) + … + 17a17(1 + x)16


Sol 10: (A) Again differentiating

(1 − 2 x ) =C − C (2 x ) ( )
50 1 2
50 50
+50 C2 2 x 2 – 6x + … = 2a2 + 6a3(1 + x) + …
0 1

− C (2 x ) + C (2 x )
50
3
50
4 Putting x = –1
3 4
M a them a ti cs | 4.55

⇒ 2 + 6 + 12 + 20 + ……. + 17 × 16 = 2a2 log5 ( 4 x + 44 )2/5


a1 = 5 = (4 x + 44)1/5
= ((4 x + 44)1/2 )2/5
2a2 = 1. 2 + 2. 3 + 3. 4……. + 16. 17
1 1
=a2 = = (2x −1 + 7)−1/3
Tn n(n + 1)
Tn for 1. 2+2. 3+3. 4 is = log5 (2 x −1 + 7)1/3
(2 x −1
+ 7)1/3
5
16 16 16
∑ Tn ∑ n2 + ∑ n
8
2a2
= = T4 = C3 (4 x + 44)5/5 (2x −1 + 7)−3/3
=i 1 =i 1 =i 1

8×7×6
(2(16) + 1)16(16 + 1) 16(16 + 1) = × (4 x + 44)(2x −1 + 7)=
−1
336
= + = 1632 1.2.3
6 2
4 x + 44 336
⇒ a2 = 816 ⇒ = = 6
2 x −1
+ 7 8x7
10
 x 3 ⇒ 4 x + 44 = 6 × 2x −1 + 6 × 7 = 3.2x + 42
Sol 2: (a) (i)  + 
 3 2x2 
  (2x)2–3(2)x+44–42=0
10 −r r
10
 x  3 Assume 2x = y
Tr +1 = Cr    2
 3  2x 
    y2 – 3y + 2 = 0 ⇒ (y – 2)(y – 1) = 0
10 −r 10 −r r
 1 
10
−2r  3 ⇒ y = 1 or y = 2 ⇒ x = 0 or x = 1
= Cr   x 2  
 3  2 
  11
 2 1 
For term independent of x- Sol 3:  ax + 
 bx 
10 − r r
0 ⇒ 10 − r − 4r =0 ⇒ r = 2
− 2r =  1 
2 Tr +1 = 11Cr (ax2 )11−r  
10 −2 2  bx 
10  1   3
So=
T3 T= C2     x 2(11 − r) + r( −1)
Power of = = 7 (given)
2 +1
 3  2 
 
⇒ 22–2r–r=7 ⇒ r = 5
10 × 9 1 3 5
= × × = 1
5
2 3 4 4 12 Coefficient T5+1 = C5 (a) 11 11 −5 11
C5a6b−5
  =
b
8
1  11
(ii)  x1/3 + x −1/5   1 
2 (ii)  ax − 
  bx2 

8 −r
1  r
Tr +1 = Cr  x1/3 
8
(x −1/5 )r 11 11 −r  1 
2  =Tr +1 Cr (ax) − 2 
 bx 
8 −r r
Power of x = − =0 for independence Power of x = 11 – r – 2r = –7 (given)
3 5
⇒ 5(8 – r) – 3r = 0 ⇒ 40 – 5r – 3r = 0 ⇒ r = 5 ⇒ r=6
6
8 −5  1 11
8 1  8×7×6 1 
3 Coefficient T=
r +1 T=
6 +1
11 11 −6
C6 a −  = C6 a5b−6
T5+=
1 T=
6 C5  (x1/3 ) .=
(x )–1/5 5
×  = 7  b
2  1.2.3 2
(iii) Given that both coefficient are equal
8
 2 log 4 x + 44

5 1 ⇒ 11
C5a6b−5 = 11C6 a5b−6 ⇒ ab = 1
(b)  55 + 
 3
log5 2x −1 + 7 
 5 
Sol 4: (a) (1+x)14
= (a1+a2)8 assume
Coefficients Tr=T(r-1)+1= 14Cr–1
8
T4 T=
= 3+1 C3 (a1 )8 −3 (a2 )3
4 . 5 6 | Binomial Theorem

Tr+1= 14Cr; T(r+1)+1=14Cr+1 So coefficient is= (1)r 10Cr = 10C5


10
Its given that they are in A. P. so. Tr+Tr+2=2Tr+1  2
And in  x − 
 x
14
Cr–1+14Cr+1=2 14Cr
r
 2
=Tr +1 10 Cr (x)10 −r  − 
14! 14! 14!  x
+ 2
=
(r − 1)!(14 − r + 1)! (r + 1)!(14 − r − 1)! r!(14 − r)! Power of x = 10 − r − r =0

1 1 2 0 ⇒ r =5
⇒ 10 − 2r =
⇒ + =
(15 − r)(14 − r) (r + 1)r r(14 − r)
10
Coefficient = C5 ( −2)5 =
− 10 C5 25
r(r + 1) + (15 − r)(14 − r) 2
⇒ = 10
C5 1 1
r(r + 1)(14 − r)(15 − r) r(14 − r) Ratio of both coefficients = = =
10
C5 25
25 32
⇒ r2+r+210–14r–15r+r2=2(r+1)(15–r)

⇒ 2r2–28r+210=30r–2r2+30–2r Sol 7: (a) (ax − by + cz)9


9! r r r
⇒ 4r2–56r+180=0 ⇒ r2–14r+45=0 General
= term (ax) 1 ( −by) 2 (cz) 3
r1 !r2 !r3 !
⇒ (r–9)(r–5)=0 ⇒ r=9 or r=5
r1 + r2 + r3 = 9
(b) (1+x) 2n
For coefficient of x2y3z4 so ⇒ r1=2, r2=3, r3=4
Coefficients T2=T1+1=2nC1=2n
9!
=
So Coefficient × a2 ⋅ b3 c 4
2n 2n(2n − 1) 2!3! 4!
T3 T=
= 2 +1 C2 = = n(2n1)
1.2
= −1260 a2 ⋅ b3 ⋅ c 4
2n(2n − 1)(2n − 2) n(2n − 1)(2n − 2)
T4 = 2nC3 = = (b) (a–b–c+d)10
1.2.3 3
10! r r r r
They all are in A. P. General Term= (a) 1 ( −b) 2 ( −c) 3 (d) 4
r1 !r2 !r3 !r4 !
So, T2 + T4 =
2T3 r1+r2+r3+r4=10

n(2n − 1)(2n − 2) It given that r1=2, r2=3, r3=4, r4=1


2n + = 2n(2n − 1)
3
10!
⇒ 3+(n–1)(2n–1)=3(2n–1)=6n–3 Coefficient ( −1)3 ( −1)4
2!3! 4!1!
⇒ 3+2n2–2n–n+1=6n–3 10 × 9 × 8 × 7 × 6 × 5
= − = – 12600
⇒ 2n –9n+7=0
2 2!3!

Sol 8: sn= 1 + q + q2 + …… + qn =
Sol 5: a = Coefficient of x3 in (1+x+2x2+3x3)4
2 n
b = Coefficient of x3 in (1+x+2x2+3x3+4x4)4 q + 1  (q + 1)  q+1
Sn =
1+ +  +... +   ,q ≠ 1
4x4 has no effect on the coefficient of x3.
2  2   2 

Hence a = b = n+1C1+n+1C2 s1+n+1C3s2


∴a‒b=0 +……+ n+1Cn+1sn

Constant term
Sol 6: (1–x2)10
T= 10
Cr ( −x2 )r
n+1
C1+n+1C2+……+n+1Cn+1 = 2n+1–1
r +1

Given that 2r=10 ⇒ r=5


M a them a ti cs | 4.57

2 n
1 1 1 Compare x → −x in
In Sn constant term = 1 + +   + .....  
2 2 2 (x2+x+1) → (x2–x+1) a0 – a1x + a2x2 – a3x3 + ….. + a2x x2x
n+1
1
1−  n+1 [(1+x+x2)(x2–x+1)]n
 2  1 (2n+1 − 1)
= = 2 1 −  =
1  2 2n = a20 x2n − a12 x2n − a3 x2n + a4 ⋅ x2n − ..... + a2n x2n
1−
2
 (2x +1 − 1)  ∴ For x = 1
So (2n+1 − 1) =
(2n ),  
 2n  a20 − a12 − a32 + .... + a2n
2
3n
=
 
n+1
C1 + n+1C2S1 + n+1C3Sn =
2n Sn
Sol 11: (5+3x)10
We can prove this with other terms also.
T4=10C3(5)10–3(3x)3
3
Sol 9: (i) (2 + 3x)9 , x = . Now we have =10C35733x3
10
C3 5733 x3 is the greatest term
2
n +1 9 +1 10 n +1 10 + 1
= = So, =
a 2 × 2 4 a 5
1+ 1+ 1+ 1+ 1+
x 3×3 9 x 3x

10 10 For greatest term to be T4


= = = 6.944
1 + 0.44 1.44
10 + 1
= 3< <4
Greatest terms is 5
6
1+
9×8×7 3 6 3 3x
T7=T6+1=9C6(2)9–6(3x)6= 2 ×3   33x
1.2.3 2 3< <4
3x + 5
32.7.36.36 7.313
= = 3(3x+5)<33x<4(3x+5)
1.2.3 2
1 9x+15<33x<12x+20
(ii) (3–5x)15 When x =
5
Solving each inequality separately we get
n +1 15 + 1 16 16
= = = = 4
a 3×5 1 + 3 4 9x + 15 < 33x
1+ 1+
x 5 ×1 ⇒ 24x > 15
So T4 and T4+1 are same greatest term. T4=15C4(3)15–4 15
(–5x)4 ⇒x>
24
4
15 × 14 × 13 × 12 11  −5  5
= 3   = 455. 312 ⇒x>
1.2.3.4  5  8
Also, 12x + 20 > 334
Sol 10: (i) (1+x+x2)n=a0++a1x+……. +a2nx2n
20
(i) at x=1 ⇒x<
21
a0+a1+a2+a3+……. +a2n=(1+1+1)n=3n 5 20
∴ <x<
8 21
(ii) at x=–1 ⇒ [1–1+(–1)2]n=1
n
⇒1=a0–a1+a2–a3+……. +a2n x 2
Sol 12: In the expansion of  +  , we have
5 5
(iii) (1+x+x2)n(x2–x+1)2n n− 8 8
x 2
T9 = nC8    
=(a0x2n–a1x2n–1+……. )(a0–a1x+…. . ) 5 5
4 . 5 8 | Binomial Theorem

Coefficient = n C8 (5)8 −n 285−8 = n


C8 5−n28 Where (+1) for constant term. And [x] is a singularity
function. [1. 35] = 1
Which is greatest coefficient
= 6034 – 1005 – 671 -335 = 4023
n+1
8< < 9 assume x=1 for find
x
1+  3x2 1 
9
a 3
Sol 15: (1 + x + 2x )  − 
 2 3x 

n+1
8< < 9 greatest coefficient
1×5 For independent terms
1+
5×2  3x2 1 
9
0
Coefficient of x in  −  = A0
n+1 n+1  2 3x 
8< <9 = 8< <9 
1 3
1+ 9
2 2  3x2 1 
Coefficient of x in  −  = A1
–1
 2 3x 
3 3 
8× < n+1 < 9×
2 2 9
 3x2 1 
27 Coefficient of x–3 in  −  = A2
12 < n + 1 <  2 3x 
2 
9 −r r
9
 3x2   1 
22 25 =Tr +1 Cr   − 
17 < n < − 1= = 12.5  2   3x 
2 2  

11 < n < 12.5 Power of x = 2(9–r) –r = 18 − 2r − r = 18 − 3r


There is only one natural no. in region i.e., 12 18
x0 ⇒ 18 − 3r = r
0 ⇒= = 6
3
9 −5 6
Sol 13: N=2000C1+2. 2000C2+3. 9 3  1
T
=6 T5=
+1 A=
0 C6   − 
2000
C3+……+20002000C2000 2  3
7
N=n. 2n–1 here n = 2000 A0 =
18
⇒ N= 2000 x2n–1 For x −1 = 18 − 3r =−1
⇒ N = 2 × (2 × 5)3 × 2n−1 = 23 × 532n 3r = 18 + 1 = 19
⇒ N=2×(2×5)3×2n–1=23×532n r=19/3 not natural no.
N = 2n+353 So A1 = 0
Number of divisors = (n + 3 + 1)(3 + 1) For x–3
= (2000 + 4)(4) = 2004 × 4 = 8016 21
18–3r=–3 ⇒ 3r=18+3=21 ⇒ =
r = 7
3
9 −7 7
Sol 14: (1+x)2012+(1+x2)2011+(1+x3)2010 9 3  1 −1
So A2
= C7   −  =
Number of different dissimilar terms 2  3  27
9
 3x2 1 
= 2012+2011+2010 (no. of terms which is common in So coefficient of x0 in (1 + x + 2x3 )  − 
 2 3x 

(1 + x)2012 and (1+x2) 2011
7  1  21 − 2(2) 17
of terms which is similar (1 + x)2012 and (1 + x3 )2010 ) = + 2 ×=
−  =
18  27  54 54
(no. of terms which is similar in (1 + x2 )2011 + (1 + x3 )2010 )

 2011   2012 + 1   1005 


= 2012 + 2011 + 2010 −  –  −   +1
 2   3   3 
M a them a ti cs | 4.59

n n
Sol 16: f (n) = ∑∑ k Cr x3 ⇒ 2 nC1 + nC3 =
a3
r −0 k −r
n(n − 1)(n − 2)
11 11
2n + a3
= …. (iii)
1.2.3 
For f (11 ) = ∑∑ Cr k

r −0 k −r It’s given that a1, a2, a3 are in A.P


0 1 2 11
= ( C0 + C0 + C0 + ....... + C0 ) 2a2 = a1 + a3
n(n− 1) (n− 2)
+ 1C1 + 2C1 + ......... + 1C1 4 + n(n‒1) = n + 2 +
6
+ 2C2 + 3C2 + ........ + 11C2 :::: :::: :::: 6n + 12n + n(n− 1) (n− 2)
=
10
C10 + 11C10 6

11 24 + 6n(n‒1) = 18n + n(n‒1) (n‒2)


C11
11 Solving this we get, n = 2 or 3 or 4
= C0 + ....... + 11C11 + 10 C0 + ...... + 10 C10 ::

+ 1C1 + 0 C0 n

11 10 9 1 0
Sol 19: ∑ nCk sin kx. cos (n-k) x = 2n−1 sin nx
= 2 +2 + 2 + ....... + 2 + 2 k =0
n

=
211+1 − 1
= 4095 = 4095 = 51.32 71. 131
L. H. S. = ∑ nCk sinkx cos(n − k)x
2 −1 k =0

No. of divisors = (1+1). (2+1). (1+1)(1+1) We know that 2 sin A cos B = sin(A+B) +sin (A–B)

= 2×3×4 = 24  A + B = kx + (n – k)x = kx + nx – kx = nx

A – B = kx – (n – k )k = kx – nx + kx = 2kx – nx
11 11
Sol 17: ∑∑ i C j So, ∑
n
1n
Ck [sinnx + sin(2kx − nx)]
j−0 i− j
k =0 2

= 0
C0 + ( 1
C0 +1 C1 + ) ( 2
C0 + .... +2 C2 ) =∑
n
1n n
1
Ck sinnx + ∑ nCk sin(2kx − nx)
+ ( 3
C0 + .... +3 C3 + ..... +) ( 11
)
C0 +11 C1 + .... +11 C11=k 2
0= k 02

n
= 20 + 21 + .... + 211 1 1
= sinnx
2
∑ nCk + 2
k =0
= 212 − 1
[ nC0 sin( −nx) + ...... + nCn sin(nx)]
n+ 4
∑ ak .xk
Sol 18: (1 + x2 ).(1 + x)n =
1
k =0 = sinnx.2n + 0 = (sinnx)2n−1 = 2n−1 sinnx
a1, a2 and a3 are in A.P 2
2001
(1+x4+2x2)(nC0+nC1x+nC2x2+nC3x3+……nCnxn) 2001 1 
Sol 20: x +  − x 0
=
= a0+a1x+a2x2+a3x3+…. 2 

Compare terms of xr in both side  1


2001
= x2001 +  2001C0   + ..... + 2001C1999
x1 ⇒ L. H. S. = nC1=n  2
 

R. H. S. = a1 1
2
1 
1999
  ( −x) + 2001C2000   ( −x)2000 
⇒ n=a1  …. (i) 2 2 

0
x2⇒2nC0+nC2=a2 2001 1
= x + C2001   ( −x)2001
n(n − 1) 2
2+ a2
= ……. (ii)
2 
4 . 6 0 | Binomial Theorem

2001 ( −x)1999 Sol 23: (C) P(i,100–i) = P( j,100–j)


= x + ...... + 2001C1999
4 100
Ci = 100 C j and i ≠ j
100
( −x)2000 Ci = 100 C100 − j
2001
+ C2000 − x2001
2 I = 100 – j
= Now maximum pointer of x = 2000 i+j = 100

Coefficient of x2001−1 i, j ∈ N,0


Sum of all solution is =
2000
Coefficient of x
(0,100)(1,99)……. (99,1)(100,1)
2001 1
C1999 ×
4 2001 × 2000 × 1 1 Total no. of ordered pairs
= = × = 500
2001 1 1.2 × 2001 × 1 2
C2000 × (i,j) = 100
2

Sol 24: (6 6 + 14)2n+1 =


I +F
Sol 21: Let
2n (assume)
S = ∑ ( −1) (k − 2n) ( Ck ) 
k 2n 2
…(i) 2
(6 6 ) − (14)2
k =0 (6 6 − 14) =
2n 6 6 + 14
⇒ S= ∑ (−1)2n − k (2n − k) (2n C2n − k )2 =
20
k =0
6 6 + 14
Writing the terms in S in the reverse order, we get
2n =I [(6 6 + 14)2n+1 ] = 0 < F < 1
S
= ∑ (−1)k k (2n Ck )2  …(ii)
k =0
=e (6 6 − 14)2n+1 = 0 < e < 1
Adding (i) and (ii) we get
2n e (6 6 + 14)2n+1
I + F −=
2n ∑ ( −1) ( Ck ) =
2S = − 2nA k 2n 2

k =0 - (6 6 − 14)2n+1
⇒ S = ‒ nA
= 2( nC1 (6 6 )2n+1 14

10
+ nC3 (6 6 )2n−11−3 143 + ......)
Sol 22: (A) ( ∑ P(i,10 − i)
i=0
= 2K (K is const. integer)
y
0 ≤F−e <1

F – e=2K–I= Integer

F – e = 0 = e = F = F= e= (6 6 − 14)2n+1
(20)2n+1
F=
(6 6 + 14)2n+1
x
(I + F)F= (6 6 + 14)2n+1

P(0,10)+P(1,9)+……+P(10,0) 202n+1
10 × 9 (6 6 + 14)2n+1
= 1 + (9 + 1)(1) + + ....
2
202n+1
(I + F)F =
10
= C0 − 10 C1 + 10 C2 + 10 C3 + ...... + 10 C10

= 210 = 1024
M a them a ti cs | 4.61

Sol 25: P= (2 + 3)5 (b) Sum of eq. (i) and (ii)

f = P – [P] n2n−1 + (1 + x)n = C1 + 2C2 + ..... + nCn


22 − ( 3)2 1 +C0 + C1 x + C2 x2 + ..... + Cn xn
2=
− 3 =
2+ 3 2+ 3
At x = 1
0 < 2 − 3 < 1
2n−1(n + 2) = C0 + 2C1
0 < f < 1
5
+3C2 + ...... + (n + 1)Cn
 1  1
3)5 
(2 + = = (c) Eq. (1) + 2 × eq. (2)
2− 3 F
At x = 1
[P] + f + f = (2 + 3)5 + (2 − 3)5
2n + n2n = C0 + C1 (1 × 2 + 1)
= 2[ 5C0 25 + 5C2 25−2 ( 3)2
+C2 (2 × 2 + 1) + ....... + Cn (2n + 1)
+....... + 5C 4 25− 4 ( 3)4 ]
2n−1(n + 2) = C0 + 2C1
 5× 4 3 
2 25 + × 2 × 3 + 5 × 2 × 32  + 7C3 + ........ + (2n + 1)Cn
 2 
(d) (C0 + C1 )(C1 + C2 )......(Cn−1 + Cn ) =
f+f = Integer
0≤f+f <2 C0 C1C2 .Cn−1 (n + 1)3n
∴ f+f = 1 n!

f = 1 − f = (2 − 3)5 Multiply and divide L. H. S. by


C0C1C2C3……. Cn–1
f = 1 − (2 3)5
 C   Cn 
f2 f 2 − 12 + 1 (f − 1)−1(f + 1) 1 = C0 C1C2 ........Cn−1  1 + 1  ......  1 + 
= = +
 C0   Cn−1 
1− f 1− f (1 − f) (1 − f)
n
1 Cr n−r +1
= −(f + 1) + On using = = L. H. S.
f
n
Cr −1 r
f =1 − f =f − 2 =−(f + 1)
 1 + n  1 + n  1 +n
1 C0 C1C2 .....Cn−1 (1 + n)    ….  
= f −2+  2  3   n 
f
C0 C1C2 .....Cn−1 (n + 1)n
= (2 + 3)5 − 2 + (2 − 3)5 =
n!
= 2[32+15×24+5×2×32]–2
(e) 1C20 + 3C12 + 5.C22 + ..... +
= 724 – 2 = 722
(n + 1)(2n)!
(2n + 1)Cn2 =
Sol 26: (1+x)n=nC0+C1+ n!n!

C2x2+C3x3+…. . +Cnxn  … (i) We know that (part (C))

(a) Differentiating at both sides C0+3C1+5C2+……+

n(1 + x)n−1 = C1 + 2C2 x + .... + 3 C3 x2n (2n+1)Cn+(n+1)2n

x =1 C0 + 3C1 + 5C2 x2 + .......


n−1  … (ii) C0+3C1+5C2x2+……
Put n ⋅ 2 =C1 + 2C2 + ....... + nCn
4 . 6 2 | Binomial Theorem

+(2n+1)Cnxn Sol 28:

= (n+1)(1+x)n = (n + 1)(1 + x)n C1 2C2 3C3 nCn n(n + 1)


(a) + + + ...... + =
Multiply with C0 C1 C2 Cn−1 2

n
C0 xn + C1 xn−1 + ...... + Cn =(x + 1)n Cr n−r +1
We know that =
n
Cr −1 r
= and compare xn and coefficient
n
r Cr
C0 + 3C12 + 5C22 + ...... + (2n + 1)Cn2 ⇒ = (n − r + 1)
n
Cr −1
= Coefficient of xn in (n+1) (1+x)n+n
L. H. S. = (n − 1 + 1) + (n − 2 + 1)
= (n+1)2nCn =
+(n − 3 + 1) + .......(n − n + 1)
L. H. S. = R. H. S.
n2 + n − (1 + 2 + 3 + ....n)

Sol 27: I = [(3 5)n ] = n2 + n −


n(n + 1)
2
I + F = (3 + 5)n
n2 + n n(n + 1)
= =
P = rational part 2 2
σ = irrational part 22 C1 23 C2
(b) 2C0 + +
9 − ( 5)2 4 2 3
3=
− 5 = n+1 n+1
3+ 5 3+ 5 2 Cn 3 −1
+ =
n+1 n+1
0 < 3− 5 <1
(c) In equation (i) from above que.
F= (3 − 5)n
x=2
n n
I + F + F = (3 + 5) + (3 − 5) 22 C1 2n+1 Cn 3n+1 − 1
2C0 + + ....... + =
=2 ( n
C0 3n + nC2 3n−2 ( 5)2 + ..... ) 2 n+1 n+1

Rational part (d) In eq. (i) x = −1

0<F+F<2 (0)n+1 − 1 C C
= C0 ( −1) + 1 − 2
n+1 2 3
F+F is 1 only integer between0and 2
1 Cn C1 C2
I + 1 = 2F ==
P (I + 1) +...... + ( −1)n+1 = C0 − + + .......
2 n+1 2 3

I + F − F = (3 + 5)n − (3 − 5)n Cn 1
+( −1)n =
n+1 n+1
I +F +F −F −F =2( nC1 3n−1
Sol 29: (a) In equation (ii) compare coefficient of xn−1
( 5)1 + nC3 3n−3 ( 5)3 + ........)
2n
I + 2F − (F + F) =2σ Cn−1 = C0 C1 + C1C2 + ....... + Cn−1Cn

I + 2F − 1 = 2σ 2n 2n!
Cn−1 =
(n − 1)!(n + 1)!
1
=
σ (I + 2F − 1)
2  2n–(n–1) = n+1

L. H. S. = R. H. S.
M a them a ti cs | 4.63

(b) In some equ. (ii) compare coefficient of xn-r So a0 = a2n


2n
Cn−r = C0 Cr + ........ + Cn−r Cn  …. (ii) a1 = a2n–1

2n! an–1 = an+1


2n
Cn−r =
(n + r)!(n − r)! So, a0a1+a2a3+a4a5+…. . +
L. H. S. = R. H. S. = a2na2n–1 + a2n–2a2n–3
n− 2
2n! +……. . + a1a2+a3a4
(c) ∑ ( nCr nCr +2 ) = (n − 2)!(n + 2)!
r =0
a0a1–a1a2+a2a3……=0
In equ. (iii) if r = 2
(ii) (1–x+x2)n
2n
= Cn−2= C0 C2 + C1C3 + ...... + Cn−2Cn
= a0–a1x+a2x2–a3x3 +…. .
2n 2n!
= Cn−2 = ….(iii) (1 + x + x2 )n (1 − x + x2 )n = (a0 − a1x + ...)
(n − 2)!(n + 2)! 

L. H. S. = R. H. S. (a0 x2n + a1 x2n−1 + ......)

(d) 100
C10 + 5. 100 C11 + 10. 100 C12 (1 + x2 +=
x 4 )n a0 a2 x2n−2 − a1a3 x2n−2 + ....

+10. 100 C13 + 5. 100 C14 + 100 C18 Compare x2n−2 coefficient
an+1 = an−1 = a0 a2 − a1a3 + ...
105 105 105 105!
= C90
= C105
= −90 C15
=
90!15! ( in (1 + x + x2 )n x → x2
2(n−1)
100! 5100! 10 × 100! So Coefficient of x = a= n−1 an+1 )
L. H. S. = + +
90!10! 11!89! 12!88! (iii) (1 + x + x2 )n = a0 + a1 x + ..... + a2n x2n

10 × 1001 5 × 100! 100! Put x = 1


+ + +
13!87! 14!80! 15!85!
3n = aa + a1 + a2 + .... + a2n …. 1

 15 × 14 × 13 × 12 × 11 x=w
100!  +
 90!15! 0= a0 + a1 ω + a2 ω2 + a3 + ....a2nω2n . . 2

5 × 90 × 15 × 14 × 13 × 12 10 × 90 × 89 × 15 × 14 × 13 x = ω2
+
15!90 15!90!
0= a0 + a1 ω2 + a2 ω + .... + a2nω4n …3
10 × 15 × 14 × 90 × 89 × 88 5.90 × 89.8887
+ + A +B + C =
70!15! 15!90!
3n = 3(a0 + a3 + a6 + ....)
90.89.88.87.86 
+ 
90!15!  a0 + a3 + a6 + ......3n−1 … (i)

x(1 + x + x2 )n = a0 x + a1 x2 + ...... + a2n x2n+1 …. A
100 × 101 × 102 × 103 × 104 × 105 = 105!
= 90!15!
90!15! x = ω = 0= a0 ω + a1 ω2 + a2 + ..... + a2nω2n+1 …B

= 105
C15
= 105
C90 x = ω2 = 0= a0 ω2 + a1 ω + a2 + ..... + a2nω4n+ 2 . . C

A,B,C,= 3n = 3(a2 + a5 + a8 + .....)


Sol 30: (i) (1+x+x2)n = a0+a1x + a2x2+……+a2nx2n
3n−1
a2 + a5 + a8 = … (ii)

(1+x+x ) = (x +x+1)
2 n 2 n
Sum as above
4 . 6 4 | Binomial Theorem

x2 (1 + x + x2 )n =a0 x2 (xn + xn−1 + ..... + x2 + x + 1)


x2(1+x+x2)n = a0x2 +a1 x3 + ..... + a2nx2n+1 an(n+1)
So, a0 =
2
x=w=
1
0 = a0 ω2 + a1 ω3 + ..... + a2nω2n+1 …. B2
Or if x =
y
x = ω2 = 0 = a0 ω4 + a1 + ..... + a2n …. C2 n(n + 1)
(y − )
2
A + B2 + C2 (a1 + a4 + a7 + .....)
(y+1)(y2+y+1)…. . (yn+y2+y+1)
3n
= = 3n−1 … (iii) n(n + 1) n(n + 1)
3  = a0 y + ....... + a
2 2
From (i), (ii) and (iii)
n(n + 1)
a0 = a
E E=
= 2 E=
3 3n−1 2
(c) Odd coefficient = a1+a3+a5+……
100 100
Sol 31: ∑∑ (Cr2 + C2s + Cr=
Cs ) m( 2nCn ) + 2P At x = 1
=r 0=s 0

M, n and p are even natural number 2. 3. 4. . . …. . (n+1) = (n+1)!

(1+x)100 = a0 a1 + a2 + ....... + an(n+1)


2
Cr = coefficients of xr in (1 + x)100
x = –1 = a0–a1+a2–a3=0
100
= a0 + a2 + a4 + ..... = a1 + a3 + a5 + ......
= ∑ [Cr2 × 101 + C02 + C12 + ..... + C100
2

r =0
Assume P = Q
+Cr (C0 + C1 + ..... + C100 )]
(n + 1)!
P + Q = 2P = 2Q = (n + i)! =P = Q =
  2
= 101  ∑ Cr2  + 101( 2nCn ) + ∑ Cr(2n )
 
 
Sol 33: S=
1 ∑< ∑ CiC j.
= 101 2nCn + 101 2nCn + 2n (2n ) 0 ≤i j≤100

= 202 2nCn+2100+100 S=
2 ∑ < ∑ CiC j
0≤ j i≤100
2n 200 2n P
202 Cn + 2 = m( Cn ) + 2
= S=
3 ∑
= ∑ CiC j
0 ≤i j≤100
n=100, m=202, P=200
(1 + x)100 ⇒ n = 100
Hence, n+m+p = 200+100+202=502
ab
S1 + S2 + S3 =

Sol 32: (1+x)(1+x+x2)……(1+x+x2+…. . +xn) S1 + S2 + S3 =


Max. power of x
n(n + 1)
∑< ∑ CiC j + ∑ < ∑ CiC j + ∑ ∑ CiC j
0 ≤i j≤100 0≤ j i≤100 0 ≤i j≤100
= 1+2+3+……. n =
2
1 + n(n + 1) n2 + n + 2
S1 = S2  ∑∑ CiC j < ∑ ∑ C jC
(a) Total terms = = 0 ≤i j≤100 0 ≤i j≤100
2 2
S3 = S1 + C0 C0 + C1C1 + C22 + ...... + C100
2
(b) 1+x=x+1

1+x+x2 = x2+x+7, So now product is S=


3 S1 + 2nCn

= (x+1) (x1+x+1) = ……. S1 + S2 + S3 = 2S1 + 2n Cn


M a them a ti cs | 4.65

= [ C0 (C1 + C2 + ...... + C100 ) +C1 (C2 + ........ + C100 ) + ....... ] 2×1


= 3 − 2×2 + = 3+1− 4 = 0
2!
2n
= C0 + 2nC1 + 2nC2 + ....... + 2nC2n
Or
n
When C1 + C2 + ...... + C100 =
2 = n − n−1C1 (n − 1) + n−1C2 (n − 2) + ......
22n 2=
== 200 100
4= 16
=50
ab +3 n−1Cn−3 ( −1)n−3 + 2 n−1Cn−2 ( −1)n−2
a + b = 16 + 50 = 66 = n − n−1C0 + 2 n−1C1 + 3 n−1C2 + .......

+( −1)n−1 n−1Cn−1

Exercise 2 = n − ( n−1C1 + n−1C0 ) + 0 = n − (n − 1 + 1) =0

8
Sol 1: (B) Given binomial is (21/3+3–1/3)n Sol 5: (C) t6 in  x −8/3 + x2 log10
x 
5600
=
 
∴ T7 = T6+1= nC6(21/3)n–6(3–1/3)6
( ) ( x log ) = 5600
3
8
C5 x −8/3 2 x
10
n −1/3 n−6 1/3 6
T7 ' from end = Cn−6 (3 ) (2 )

(log ) =
5
1T7n
C6 2n/32−23−2 (2.3)n/3 ⇒ x2 x
10 100
⇒ = = = = (6)(n/3)− 4
T7 ' 6 n
C6 3−n/33222 (6)2+ 2 ⇒x=
10
1 n
6(n/3)− 4 = ⇒ − 4 =−1 ⇒n=9
6 3
Sol 6: (B) (α + p)m−1 + (α + p)m−2 (α + q)

Sol 2: (C) We have 1523+2323 = (19–4)23+(19+4)23 +(α + p)3 (α + q)2 + ...... + (α + q)m−1

Coefficient of t
= 2  23C0 1923 + 23C2 1921 + ... + 23C22 19 
  m−1 
  α + q  α + q
= 2. 19K always divisible by 19 
 + ...... +  α + p 
m−1
= (α + p) 1 + 
 α +p   
So the remainder is zero 

  
m
1 −  α + q  
n n 2 n n−1 
Sol 3: (D) 4{ C1 + 4 C2 + 4 . C3 + ...... + 4 }

= (α + p)m−1  α +p 
= {4 nC1 + 42 nC2 + 43 nC3 + ..... + 4n nCn } 
 1 −  α + q  
  α + p  
= (1 + x)n = C0 + C1 x + C2 x2 + ..... + xn nCn 
At x = 4  (α + q) 
n 2 1 − α + p 
5 =
1 + 4C1 + 4 C2 + ...   (α + p)
= (α + p)m−1
4n
α +p−α −q
So C1 + 42 nC2 + 43 nC3 + ..... + 4n nCn = 5n − 1
  
m
1 −  α + q  

Sol 4: (A) n ≥ 3
 α + p   =  (α + p) − (α + q) 
m m

= (α + p)m    
(n − 1) (n − 1)(n − 2)  p−q   p−q 
n− (n − 1) + (n − 2)  
1! 2!  
m
(n − 1)(2 − n)(n − 3) Ct [pm−t − qm−t ]
− (n − 3) + ...... Coefficient of α t =
3! p−q
At n = 3

( )
2145
1.3 − (3 − 1) (3 − 1)(3 − 2) Sol 7: (B) 1 + x − 3x2 = a0 + a1 x + a2 x2 + ...
= (3 − 1) + (3 − 2) − 0
1 2!
Put x = - 1
4 . 6 6 | Binomial Theorem

= Number of ways of choosing n persons out of n men


⇒ (1 − 1 − 3 )
2145
= a0 − a1 + a2 − a3 + ....
and n women
⇒ a0 − a1 + a2 − a3 + .... ( −3)
2145
= Number of ways of choosing n person out of 2n
persons
Last digit of ( −3)
2145
is 3. = 2nCn

9
 4x2 3 
Sol 8: (B)  −  Sol 12: (D) aC0 + ( a + b ) C1 + .... + ( a + nb ) Cn
 3 2x 

9 −r
= C0 + C1 + .... + Cn 
r
 4x2   3  = C0 + C1 + .... + Cn 
=Tr +1 9 Cr   −  +b 0 × C0 + 1 × C1 + 2 × C2 + .... + nCn 
 3   2x  +b 0 × C0 + 1 × C1 + 2 × C2 + .... + nCn 
 
Power of x= 2(9 − r) + ( −1)r = a2n + bn2n−1
⇒ 18 − 2r − r = 18 − 3r =
6 (given)
= ( 2a + nb ) 2n−1
⇒ r 12
⇒ 3r = 18 − 6 = 12= = /3 4
9− 4 4 5 4
9 4  −3  9×8×7×6 4  3
Coefficient C4     =     Previous Years’ Questions
3  2  1.2.3.4  3   2 

210 × 34 Sol 1: We know, (1 + x)2n = C0 +C1x + C2x2 + ….. + C2nx2n


= 9×2×7× = 21 × 27 =
2688
5 4
3 ×2 On differentiating both sides w.r.t. x, we get
2n (1 + x )
2n−1
18 C1 2.C2 x
=+
 1 
Sol 9: (D)  9x −  ,x > 0 +3.C3 x2 + .... + 2nC2n x2n−1 … (i)
 3 x 
r And
18 18 −r  1 
Tr +1 = Cr (9x)   2n
 9x   1 1 1
1 −  =C0 − C1 . + C2 .
 x x x2
r 3r
Power of x = 18 − r − = 18 − = 0 (given)
2 2 1 1
−C3 . + .... + C2n . … (ii)
3
r r 3r
x x2n 
 1  18 −r − 18 −
0
α = 918 −r   = (9) 2 = (9) 2
= 9= 1 On multiplying Eqs. (i) and (ii), we get
 9
2n
 1
2n (1 + x )
2n−1
55 55 1 − 
x
Sol 10: (C)  x x++ x3x3−−11  +++ x x−− x3x3−−11  
       
= C1 + 2.C2 x + 3.C3 x2 + .... + 2n.C2n x2n−1 
 

( ) ( )
2
= 2  5 C0 x5 +5 C2 x3 x3 − 1 +5 C 4 x x3 − 1 
   1  1   1 
× C0 − C1   + C2   − .... + C2n  
 x  x2   x2n  
⇒ Highest power is 7.
1
The coefficient of   on the LHS
x
Sol 11: (C) We have
1  1 
 (1 + x ) ( x − 1)
n 2n−1 2n
= Coefficient of in 2n 
∑ (n Cr ) (n Cr )
C20 + C12 + C22 + ... + Cn2 = x 2n
x 
r =0

( )
n 2n−1

= (n Cr ) (n Cn − r ) [ nCr nCn − r ] = Coefficient of x2n−1 in 2n 1 − x2 (1 − x )
r =0
M a them a ti cs | 4.67

n−1 ( 2n−1) n
= 2n ( −1)
∑ ( −1)
r n
. Cn−1 Sol 3: Cr
r =0

= ( −1 ) ( 2n)
n ( 2n − 1) !
 1 3r 7r 15r 
(n − 1) !n!  r + 2r + 3r + 4r + ....upto mterms 
 2 2 2 2 
(2n)!
= − ( −1 ) n ⋅
n
⋅n … (iii) n r
(n!)2 1
∑ ( −1 )
r n
 Cr   +
r =0 2
= −( −1)n n ⋅ Cn r r
n n
3 7
( −1 ) Cr   + ∑ ( −1 ) n Cr   + ...
r r n
1 ∑ 4
Again, the coefficient of   on the RHS =r 0=   r 0 8
x
Upto m terms
(
= − C12 − 2.C22 + 3.C32 + .... + −2nC22n

… (iv)
 n
) n

using∑ ( −1 ) n Cr xr =( 1 − x) 
r
From Eqs. (iii) and (iv), we get 
 r =0 
( −1) n.Cn
n
C12 − 2.C22 + 3.C32 − .... − 2n.C22n =  1  3 
n
7
n n
=  1 −  +  1 −  +  1 −  + .....
 2  4  8
Sol 2: n+1 C1 + n+1
C2s1 + n+1
C3s2
Upto m terms
n+1 n n n
1 1  1
+.... + n+1
Cn+1sn =∑ n+1 Cr sr −1 =   +   +   + .....
r =1 2  4 8
1 − qn+1
Where sn = 1 + q + q2 + .... + qn = Upto m terms
1−q
  1  
m
n+1  1 − qr   1 −  n 
∴ ∑ n+1 Cr   1 
n
2   2mn − 1
 1−q  = =  
  
( )
r =1
 2   1 − 1  2mn 2n − 1
1  n+1 n+1 n+1   2n 
 ∑ Cr − ∑ Cr q 
n+1 r  
1 − q  r 1=r 1
= 
Sol 4: Let y = (x – a)m, where m is a positive integer,
1  r ≤m,
= (1 + 1 )n+1 − (1 + q)n+1 
1−q
dy
Now, = m ( x − a)
m−1

dx
1  n+1
2 − (1 + q ) 
n+1
= … (i)
1 − q   d2 y
= m (m − 1 )( x − a)
m− 2
 ⇒
2
dx
2 n
q+1 q+1 q+1 d4 y
Also,=
Sn 1  +  + ..... +   ⇒ = m (m − 1 )(m − 2 )(m − 3)( x − a)
m− 4
 2   2   2  dx 4

q+1
n+1 …………………………………..
1− 
 2  2n+1 − (q + 1)n+1 On differentiating r times, we get
= … (ii)
q+1 2n (1 − q)
1−  dr y
= m (m − 1 ) .... (m − r + 1 )( x − a)
m−r
 2   r
dx
From eqs. (i) and (ii), we get
n+1
Cr + n+1
C2s1 + n+1
C3s2 +.... + n+1
Cn+1sn =n
2 sn
=
m!
(m − r ) !
( x − a)
=
m−r
r! ( C ) ( x − a)
m
r
m−r

dr y
And for r > m, =0
dxr
4 . 6 8 | Binomial Theorem

(1 + x )
Now, n
2
+ x4
2n 2n =
∑ ar ( x − 2=
) ∑ br ( x − 3)
r r
(given) x2n
=r 0=r 0
Thus, a20 − a12 + a22 − a32 + .... + a2n
2
On differentiating both sides n times w.r.t. x, we get
= Coefficient of the term independent of x in
2n
∑ ar (n!) Cn ( x − 2 )
r r −n
1
(1 + x )
n
2
r =n + x4
2n
x

( )
2n n
∑ br (n!) Cn ( x − 3)
r r −n
= Coefficient of x2n in 1 + x2 + x 4
r =n

( )
n
On putting x = 3, we get = Coefficient of tn in 1 + t + t2 =an
2n
∑ ar (n!) Cn = (bn ) n!
r
Sol 6: To show that
r =n

1 2n 2k . n C0 . n Ck − 2k −1. n C1 . n−1 Ck −1 +2k −2. n C2 . n−2 Ck −2 − .... +


⇒ br =∑ ar (n!) Cn
r

n! r =n
( −1)
k n
= 2n + 1
Cn Ck n−k C0 =
n
Ck

= 2n + 1
Cn + 1 Taking LHS

2k . n C0 . n Ck − 2k −1. n C1 . n−1 Ck −1 +..... + ( −1 ) . Ck = Ck


n n k

( )
n
Sol 5: 1 + x + x2 =a0 +a1 x + .... + a2n x2n  … (i)
k
∑ ( −1)
r
Replacing x by –1/x, we get = .2k −r . n Cr . n−r Ck −r
r =0
n
 1 1 
1 − + 2 
k
n! (n − r ) !
∑ ( −1 )
r
x x  = .2k −r . .

r =0 r!(n − r)! (k − r ) ! (n − k ) !
a1 a2 a3 a2n
= a0 − + − + .... + … (ii) k
n! k!
∑ ( −1)
r
x x2 x3 x2n  = .2k −r . .
r =0 (n − k )!k! r! (k − r )!
Now, a20 − a12 + a22 − a32 + .... + a22n = coefficient of the
k
∑ ( −1)
r
term independent of x in = .2k −r . n Ck . k Cr
r =0
a + a x + a x2 + .... + a x2n 
 0 1 2 2n 
 k r 1 
 a a a  = 2k . n Ck ∑ ( −1 ) . . k Cr 
× a0 − 1 + 2 − .... + 2n  r =0 2r 
 x x 2
x2n 
k
 1
= Coefficient of the term independent of x in = 2k . n Ck  1 −  = n Ck = RHS
 2
n

( ) 1 2 
n
1 + x + x2 1 − + 2 
10
 x x  Sol 7: Let y = ∑ Ar (B10 Br − C10 Ar )
r =1
n

( ) 1 1 
n 10
2
Now, RHS = 1 + x + x 1 − + 2  ∑ ArBr = coefficient of x 20
in ((1 + x)10 (x + 1)20) – 1
 x x 
r =1

(1 + x + x ) ( x )
n n
2 2
− x +1 10 2
= = C20 – 1 = C10 – 1 and ∑ ( Ar ) = coefficient of x10 in ((1
x2n + x)10 (x + 1)10) – 1 = B10 – 1r =1
n
 2
( 2
)
2
 x +1 − x  ⇒ y = B10(C10 – 1) – C10(B10 – 1) = C10 – B10.
(1 + 2x2 + x 4 − x2 )n
=  =
2n 2n
x x
M a them a ti cs | 4.69

Sol 8: Let Tr – 1, Tr, Tr + 1 are three consecutive terms of


−1 + i 3
(1 + x)n + 5 Sol 11: Z = = ω
2
Tr−1 = n+5Cr−2 (x)r−2 , Tr = n+5Cr−1xr−1 , Tr+1 = n+5Crxr
( −ω)r ω2s 
Where, n+5Cr–2 : n+5Cr–1 : n+5Cr = 5 : 10 : 14. P= 
2s
 ω ωr 
n+5 n+5
Cr −2 Cr −1
So = ⇒ n − 3r =−3  ... (i) ( −ω)r
5 10 ω2s  ( −ω)r ω2s 
P2 =    
n+5 n+5
Cr −1 C  ω2s ωr   ω2s ωr 
−30 
= r ⇒ 5n − 12r = ... (ii)
10 14
 ( −ω)2r + (ω2s )2 ω2s ( −ω)r + ωr ω2s 
From equation (i) and (ii) n = 6 = 
ω2s ( −ω) + ωr ω2s ω4s + ω2r 
Sol 9: 2x1 + 3x2 + 4x3 = 11  ω4s + ω2r ω2s (ωr + ( −ω)r 
= 
Possibilities are (0, 1, 2); (1, 3, 0); (2, 1, 1); (4, 1, 0). ω2s (ωr + ( −ω)r ) ω4s + ω2r 

∴ Required coefficients = − I (Given)

= (4C0 × 7C1 × 12C2) + (4C1 × 7C3 × 12C0) + (4C2 × 7C1 × 12C1)


+ (4C4 × 7C1 × 1)

= (1 × 7 × 66) + (4 × 35 × 1) + (6 × 7 × 12) + (1 × 7) ω4s + ω2r = −1 and ω2s (ωr + ( −ω)r ) = 0


ωr + ( −ω)r = 0
= 462 + 140 + 504 + 7 = 1113.
r s r s
Sol 10: x9 can be formed in 8 ways 1 1 1 1
ii.e. x9, x1 + 8, x2 + 7, x3 + 6, x4 + 5, x1 + 2 + 6, x1 + 3 + 5, x2 + 3 + 4 2 2 3 3
and coefficient in each case is 1.
Total no. pairs = 1
⇒ Coefficient of x9 = 1 + 1 + 1 + ............... + 1 = 8
8 times
2017-18 100 &
op kers
Class 11 T
By E ran culty
-JE Fa r
IIT enior emie .
S fP r es
o titut
Ins

MATHEMATICS
FOR JEE MAIN & ADVANCED
SECOND
EDITION

Exhaustive Theory
(Now Revised)

Formula Sheet
9000+ Problems
based on latest JEE pattern

2500 + 1000 (New) Problems


of previous 35 years of
AIEEE (JEE Main) and IIT-JEE (JEE Adv)

5000+Illustrations and Solved Examples


Detailed Solutions
of all problems available

Topic Covered Plancess Concepts


Tips & Tricks, Facts, Notes, Misconceptions,
Permutation and Key Take Aways, Problem Solving Tactics
Combination
PlancEssential
Questions recommended for revision
5. P E R M U TAT I O N S A N D
C O M B I N AT I O N S

1. INTRODUCTION
The main subject of this chapter is counting. Given a set of objects the problem is to arrange some or all of them
according to some order or to select some or all of them according to some specification.

2. FUNDAMENTAL PRINCIPLE OF COUNTING


The rule of sum: If a first task can be performed in m ways, while a second task can be performed in n ways, and
the tasks cannot be performed simultaneously, then performing either one of these tasks can be accomplished in
any one of total m+n ways.
The rule of product: lf a procedure can be broken down into first and second stages, and if there are m possible
outcomes for the first stage and if, for each of these outcomes, there are n possible outcomes for the second stage,
then the total procedure can be carried out, in the designated order, in a total of mn ways.

Illustration 1: There are three stations, A. B and C. Five routes for going from station A to station B and four routes
for going from station B to station C. Find the number of different ways through which a person can go from A to
C via B. (JEE MAIN)

Sol: This problem is an application of the Fundamental Principle of Counting. The rule of product can be used to
solve this question easily.
Given there are five routes for going from A to B and four routes for going from B to C.

A C
B

Figure 5.1

Hence, by the fundamental principle of multiplication the total number of different ways
= 5 x 4 (i.e., A to B and then B to C) = 20 ways

Illustration 2: A hall has 12 gates. In how many ways, can a man enter the hall through one gate and come out
through a different gate.  (JEE MAIN)

Sol: The rule of product can be used to solve this problem.


5 . 2 | Permutations and Combinations

There are 12 ways of entering the hall. After entering the hall the man can come out through any of 11 different
gates.
Hence, by the fundamental principle of multiplication, the total number of ways are 12 x 11 = 132 ways.

Illustration 3: How many numbers between 10 and 10,000 can be formed by using the digits 1, 2, 3, 4, 5 if
(i) no digit is repeated in any number. (ii) digits can be repeated.  (JEE MAIN)

Sol: The numbers between 10 and 10,000 can be either two digit, three digit or four digit numbers. We consider
each of these cases and try to find the number of possibilities using 1, 2, 3, 4 and 5. Finally, we add them up to get
the desired result.
(i) Number of two digit numbers = 5 x 4 = 20
Number of three digit numbers = 5 x 4 x 3 = 60
Number of four digit numbers = 5 x 4 x 3 x 2 = 120
Total number of numbers = 20 + 60 +120 = 200
(ii) Number of two digit numbers = 5 x 5 = 25
Number of three digit numbers = 5 x 5 x 5 = 125
Number of four digit numbers = 5 × 5 × 5 × 5 = 625
Total number of numbers = 25 +125 +625 = 775

3. FACTORIAL NOTATION
An efficient way of writing a product of several consecutive integers is the factorial notation. The number n! (read
as “n-factorial”) is defined as follows :
For any positive integer n; n! = n(n – 1)(n – 2) …….. (3)(2)(1); For instance, 4! = 4.3.2.1 = 24

Note: (i) n! = n(n – 1)(n – 2) ………… 3.2.1 ; n! = n.(n – 1)! ; 0! = 1! = 1; (2n)! = 2n.n![1.3.5.7 ……… (2n – 1)]
(ii) n! = n(n – 1)! = n(n – 1)(n – 2)! = n(n – 1)(n – 2) (n – 3)!
n!
(iii) n(n – 1) …….. (n – r + 1) =
(n − r)!

Illustration 4: Find the sum of n terms of the series whose nth term is n!×n. (JEE ADVANCED)

Sol: Represent the general term in this question as a difference of two terms and then add them up to find the
answer.
The required sum = (1)! + 2(2)! + 3(3)! + ……. + n(n!) = (2 – 1)! + (3 – 1)2! + (4 – 1)3! + …….. + [(n + 1) – 1]n!
= (2! – 1!) + (3! – 2!) + (4! – 3!) + …….. + [(n + 1)! – n!] = (n + 1)! – 1

4. PERMUTATION
Each of the different arrangements, which can be made by taking some or all of a number of objects is called
permutation. The number of permutations of n different objects taken r at a time is represented as
n!
n
Pr = = n(n – 1)(n – 2) ……… (n – r + 1) (where, 0 ≤ r ≤ n)
(n − r)!

Note: (i) In permutation, the order of the items plays an important role.
(ii) The number of all permutations of n distinct objects taken all at a time is n!
M a them a tics | 5.3

Illustration 5: If 56Pr+6 : 54Pr+3 = 30800 : 1, find rP2  (JEE MAIN)

Sol: Use the formula for nPr.


56
Pr + 6 30800 56! (51 − r)! 30800
We have , = = × = ⇒ 56 x 55 (51 – r) = 30800 ⇒ r = 41
54
Pr +3 1 (50 − r)! 54! 1

∴ 41
P2 = 41 x 40 = 1640

Illustration 6: Three men have 4 coats, 5 waist coats and 6 caps. In how many ways can they wear them?
 (JEE ADVANCED)

Sol: Use the concept and understanding of Permutation, i.e. arrangement to find the answer.
The total number of ways in which three men can wear 4 coats is the number of arrangements of 4 different coats
taken 3 at a time. So, three men can wear 4 coats in 4P3 ways. Similarly, 5 waist coats and 6 caps can be worn by
three men in 5P3 and 6P3 ways respectively. Hence, the required no. of ways = 4P3 × 5P3 × 6P3 = (4!) × (5 × 4 × 3) ×
(6 × 5 × 4) = 172800.

Illustration 7: Suppose 8 people enter an event in a swim meet. In how many ways could the gold, silver, and
bronze prizes be awarded? (JEE ADVANCED)

Sol: Use the formula for nPr. The required number of ways is an arrangement of 3 people out of 8 i.e.
8! 8.7.6.5. 4 .3.2.1
8
P3 = = = 8 . 7 . 6 = 336.
5! 5. 4 .3.2.1

PLANCESS CONCEPTS

The following two steps are involved in the solution of a permutation problem:
Step 1: Recognizing the objects and the places involved in the problem.
Step 2: Checking whether the repetition of the objects is allowed or not.
Uday Kiran G (JEE 2012, AIR 102)

4.1 Permutation with Repetition


These kinds of problems occur with permutations of different objects in which some of the objects can be repeated.
The no. of permutations of n different objects taken r at a time when each object may be repeated any number of
times is nr.

Illustration 8: A student appears in an objective test which contains 10 multiple choice questions. Each question
has four choices in which one is the correct option. What maximum number of different answers can the student
give? How will the answer change if each question may have more than one correct answers? (JEE ADVANCED)

Sol: Use the concept of Permutation with Repetition.


For the first part each question has four possible answers. So, the total possible answers = 4 × 4 × .... 10 times = 410.
For the second part. Suppose the choices for each question are denoted by A, B, C and D. Now the choice A is
either correct or incorrect (two ways) similarly the other choices are either correct or incorrect. Thus, this particular
question can have 2 × 2 × 2 × 2 = 16 possible answers. But this includes the case when all the four choices are
incorrect. Thus the total number of answers = 15. Now, as there are 10 questions and each question has 15 possible
answers. Therefore the total number of answers = 1510.
5 . 4 | Permutations and Combinations

PLANCESS CONCEPTS

The mn or nm dilemma
Let us start with an example.
Q. There are 7 letters and 5 letter-boxes. In how many ways can you put the letters in the boxes?
Sol: This is the typically confusing question asked frequently from the P & C area. Let’s see how you can
solve this type of question.
The Exhaustive Approach: One way to solve this question is through (what we will call) The Exhaustive
Approach. While solving such problems, first decide which of the items (letters and letter-boxes here) is
exhaustive. Exhaustive here means the entity which is sure to be used up completely.
In this example, all the “letters” are sure to be placed in the boxes, whereas there is no such constraint as
regards the “letter-boxes”. Some boxes could go empty. Having decided this, just go by the options we
have for all the instances of the exhaustive item and you have your answer.
As you can see here, every letter has 5 boxes to choose from. Thus the total would
be (5 x 5 x........7 times) = (57)
A similar question could be: In how many ways can 10 rings be worn on 5 fingers? Try it yourself.
Chinmay S Purandare (JEE 2012, AIR 698)

4.2 Permutation of Alike Objects


This kind of problems involve permutations of different objects in which some of them are similar.
The number of permutations of n objects taken all at a time in which, p are alike objects of one kind, q are alike
objects of second kind & r are alike objects of a third kind and the rest (n – (p + q + r)) are all different is
n!
p!q!r!

Illustration 9: Determine the number of permutations of the letters of the word ‘SIMPLETION’ taken all at a time.
 (JEE MAIN)
Sol: In the given word the letter I occurs twice and the remaining letters occur only once. So, the concept of
Permutation of Alike Objects is used to find out the answer.
There are 10 letters in the word ‘SIMPLETION’ and out of these 10 letters two are identical. So, just selecting all 10
objects at a time will give twice the actual result. Hence, the number of permutations of taking all the letters at a
time = 10P10 / 2! = 10! / 2! = 181440.

4.3 Permutation under Restriction


(a) The number of permutations of n different objects, taken r at a time, when a particular object is to be always
included in each arrangement, is r. n–1Pr–1
The number of permutations of n different objects, taken r at a time, when a particular object is never taken
in each arrangement is n–1Pr.
(b) String method: The number of permutations of n different objects, taken all at a time, when m specified
objects always come together is m! × (n – m + 1)!.
(c) Gap Method: The number of permutations when no two given objects occur together.
In order to find the number of permutations when no two given objects occur together.
M a them a tics | 5.5

(a) First of all, put the m objects for which there is no restriction, in a line. These m objects can be arranged in
m! ways.
(b) Then count the number of gaps between every two m objects for which there is no restriction, including the
end positions. Number of such gaps will be (m + 1).
(c) If m is the number of objects for which there is no restriction and n is the number of objects, two of which are
not allowed to occur together, then the required number of ways = m! × m+1Cn × n!

The number of permutations when two types of objects are to be arranged alternately
(a) If their numbers differ by 1 put the object whose number is greater in the first, third, fifth.... places, etc. and
the other object in the second, fourth, sixth.... places.
(b) If the number of two types of objects is same, consider two cases separately keeping the first type of object
in the first, third, fifth places, etc. and the second type of object in the first, third, fifth places.... and then add.

4.4 Non-Consecutive Selection


The number of selections of r consecutive objects out of n objects in a row = n - r + 1
n, when r < n
The number of selections of r consecutive objects out of n objects along a circle = 
1, when r = n
Illustration 10: Find the numbers between 300 and 3000 that can be formed with the digits 0, 1, 2, 3, 4 and 5,
where no digit is repeated in any number. (JEE MAIN)

Sol: The numbers between 300 and 3000 can either be a three digit number or a four digit number. The solution is
divided into these two different cases and their sum will give us the desired result.
Any number between 300 and 3000 must be of three or four digits.
Case I: When number is of three digits: The hundreds place can be filled by any one of the three digits 3, 4 and 5
in 3 ways. The remaining two places can be filled by the remaining five digits in 5P2, ways.
5!
∴ The number of numbers formed in this case = 3 × 5P2 = 3 × =60
3!
Case II: When number is of four digits: The thousands place can be filled by any one of the two digits 1 and 2 in 2
ways and the remaining three places can be filled by the remaining five digits in 5P3 ways.
5!
∴ The number of numbers formed in this case = 2 × 5P3 = 2 × = 120
2!
∴ Total numbers = 60 + 120 = 180

Illustration 11: How many words can be formed from the letters of the word ARTICLE, so that vowels occupy the
even places? (JEE MAIN)

Sol: Clearly, this is an example of Permutation under Restriction. We identify the even places and the odd places
and try to find the number of ways in which the vowels and consonants can fill the spaces.
There are seven places: 3 even and 4 odd in which we have to fill 3 vowels and 4 consonants.
∴ The number of words = 3P3 . 4P4 = 3! × 4! = 6 × 24 = 144.

Illustration 12: How many different words can be formed with the letters of the word ORDINATE so that
(a) Four vowels occupy the odd places (b) Beginning with O (c) Beginning with O and ending with E. (JEE MAIN)

Sol: The concept of Permutation under Restriction can be used to solve this problem.
There are 4 vowels and 4 consonants. Total 8 letters.
(a) No. of words = 4! × 4! = 24 × 24 = 576. Because 4 vowels are to be adjusted in 4 odd place and the 4 consonants
in the remaining 4 even places.
5 . 6 | Permutations and Combinations

(b) 7! ways, O being fixed.


(c) 6! ways, O fixed in first and E fixed in last.

Illustration 13: Find the number of ways in which 5 boys and 5 girls can be seated in a row so that
(a) No two girls may sit together.
(b) All the girls sit together and all the boys sit together
(c) All the girls are never together. (JEE ADVANCED)

Sol: Since the number of girls and the number of boys are equal they have to sit alternately. This can be used to
solve (a). For (b), we keep the girls together and arrange the boys in five places. Also, the girls can be arranged
amongst themselves in 5! ways. This gives us the number of arrangements. Use the answer of the second part to
find (c).
(a) 5 boys can be seated in a row in 5P5 = 5! ways. Now, in the 6 gaps between 5 boys, the 5 girls can be arranged
in 6P5 ways. Hence, the number of ways in which no two girls sit together = 5! × 5P5 = 5! × 6!
(b) The two groups of girls and boys can be arranged in 2! ways. 5 girls can be arranged among themselves in
5! ways. Similarly, 5 boys can be arranged among themselves in 5! ways. Hence, by the fundamental principle of
counting, the total number of requisite seating arrangements = 2!(5! × 5!) = 2(5!)2.
(c) The total number of ways in which all the girls are never together = Total number of arrangements – Total
number of arrangements in which all the girls are always together = 10! – 5! × 6!

Illustration 14: The letters of the word OUGHT are written in all possible orders and these words are written out
as in a dictionary. Find the rank of the word TOUGH in this dictionary. (JEE MAIN)

Sol: The word TOUGH will appear after all the words that start with G, H and O. Then we look at the second letter
of the words starting with T and then third. Hence, the rank of the word TOUGH will be one more than the sum of
all the possibilities just mentioned.
Total number of letters in the word OUGHT is 5 and all the five letters are different, the alphabetical order of these
letters is G, H, O, T, U.
Number of words beginning with G = 4! = 24
Number of words beginning with H = 4! = 24
Number of words beginning with O = 4! = 24
Number of words beginning with TG = 3! = 6
Number of words beginning with TH = 3! = 6
Number of words beginning with TOG = 2! = 2
Number of words beginning with TOH = 2! = 2
Next come the words beginning with TOU and TOUH is the first word beginning with TOU.
∴ Rank of ‘TOUGH’ in the dictionary = 24 + 24 + 24 + 6 + 6 + 2 + 2 + 1 = 89

Illustration 15: There are 21 balls which are either white or black and the balls of same color are alike. Find the
number of white balls so that, the number of arrangements of these balls in a row is maximum.(JEE ADVANCED)

Sol: The property of a binomial coefficient can be used to solve this question.
Let there be r white balls so that the number of arrangements of these balls in a row be maximum. Number of
arrangements of these balls is
21! 21 + 1 21 – 1
A= A will be maximum when r = or i.e. 10 or 11
r!(21 − r)! 2 2
M a them a tics | 5.7

Illustration 16: The number plates of cars must contain 3 letters of the alphabet denoting the place and area to
which its owner belongs. This is to be followed by a three-digit number. How many different number plates can be
formed if:
(i) Repetition of letters and digits is not allowed. (ii) Repetition of letters and digits is allowed. (JEE ADVANCED)

Sol: This is a simple application of Permutation with and without repetition.


There are 26 letters of alphabet and 10 digits from 0 to 9.
(i) When repetition is not allowed
3 letters selected in 26 × 25 × 24 ways
3 digit numbers are = 9 x 9 x 8 (as zero can’t be in the hundreds place)
∴ The Number of plates = 26 x 25 x 24 x 9 x 9 x 8 = 10108800.
(ii) When repetition is allowed
3 letters are selected 26 × 26 × 26 ways
3 digit numbers are = 9 × 10 × 10 = 900
∴ The number of plates = 26 x 26 x 26 x 900 = 15818400.

PLANCESS CONCEPTS

Constraint based arrangement


Let us start with an example first:
Q. In how many ways can the word VARIETY be arranged so that exactly 2 vowels are together?
The problem could be easier if none or all vowels were to be kept together. Isn’t it? Well, we will do
exactly that!
Whenever question involving “constraints that has choices (2 vowels could be IE or AI or AE)” are asked,
“go for the backward approach.” Rather than finding the favorable cases, subtract the unfavorable ones
from the total possible cases. This method is more reliable.
So, the solution to the above question would be:
(Total arrangements of VARIETY) – (Arrangements with no vowels together + Arrangements with all the
vowels together)
Vaibhav Krishnan (JEE 2009, AIR 22)

5. COMBINATION
Each of the different groups or selection which can be made by some or all of a number of given objects without
reference to the order of the objects in each group is called a combination.
n!
The number of all combinations of n objects, taken r at a time is generally denoted by C(n, r) or nCr =
n
pr r!(n – r)!
(0 ≤ r ≤ n) =
r!
Note:
(a) The number of ways of selecting r objects out of n objects, is the same as the number of ways in which the
remaining (n - r) can be selected and rejected.
(b) The combination notation also represents the binomial coefficient. That is, the binomial coefficient nCr is the
combination of n elements chosen r at a time.
5 . 8 | Permutations and Combinations

(c) (a) nCr = nCn–r


n n n+1
(b) Cr + Cr −1 =Cr
(c) nCx = nCy ⇒ x = y or x + y = n
(d) If n is even, then the greatest value of nCr is nCn/2
n
(e) If n is odd, then the greatest value of nCr is C n+1
2
(f) nC0 + nCr +................+ nCn = 2n
(g) nCn + n+1Cn + n+2Cn + ……………. + 2n–1Cn = 2nCn+1

(d) Comparison of permutation and combination

Permutations Combinations
Different orderings or arrangement of the r objects are Each choice or subset of r object give one combination. Order
different permutations within the group of r objects does not matter.
n! n!
Pr =
n
Cr =
n

(n – r)! (n – r)!r!
Clue words: arrangement, schedule, order Clue words: group, committee, sample, selection, subset.

Illustration 17: A basketball coach must select two attackers and two defenders from among three attackers and
five defenders. How many different combinations of attackers and defenders can he select? (JEE MAIN)

Sol: The number of ways two attackers and two defenders can be selected is 3C2 and 5C2 respectively.
5. 4 .3.2.1
∴ He can select in 3C2 × 5C2 = = 30 different combinations.
2.2

Illustration 18: A soccer team of 11 players is to be chosen from 30 boys, of whom 4 can play only in goal, 12 can
play only as forwards and the remaining 14 in any of the other positions. If the team is to include five forwards and
of course, one goalkeeper, in how many ways can it be made up? (JEE MAIN)

Sol: Proceed according to the previous question.


There are: 4C1 ways of choosing the goalkeeper. l2C5 ways of choosing the forwards and 14C5 ways of choosing the
other 5 players. That is, 4C1 × 12C5 × 14C5 combinations altogether = 4 x 792 x 2002 = 6342336.

5.1 Combinations under Restrictions


(a) Number of ways of choosing r objects out of n different objects if p particular objects must be excluded.

Consider the objects A1, A2, A3,.....,Ap, Ap+1, …........, An. If the p objects A1, A2......Ap are to be excluded then we will

have to select r objects from the remaining n – p objects (Ap+1, Ap+2, …….. An).
Hence the required number of ways = (n–p)Cr

(b) Number of ways of choosing r objects out of n different objects if p particular objects must be included
(p ≤ r).
Consider the objects A1, A2, A3 ....... Ap, Ap+1, ……. An. If the p particular objects A1, A2, ...... Ap (say) must be
included in the selection then to complete the selection, we must select (r – p) more objects to complete the
selection. These objects are to be selected from the remaining n – p objects.
Hence, the required number of ways = n–pCr–p

(c) The total number of combinations of n different objects taken one or more at a time = 2n - 1.
M a them a tics | 5.9

5.2 Combinations of Alike Objects


(a) The number of combinations of n identical objects taking (r ≤ n) at a time is 1.
(b) The number of ways of selecting r objects out of n identical objects is n + 1.
(c) If out of (p + q + r + s) objects, p are alike of one kind, q are alike of a second kind, r are alike of the third kind
and s are different, then the total number of combinations is (p + 1)(q + 1)(r + 1)2s – 1
Note: The list of alike objects can be extended further
4. The number of ways in which r objects can be selected from a group of n objects of which p are identical, is
t t
∑ n−p C r , if r ≤ p and ∑ n−p C r if r > p
0 r =p

Illustration 19: There are 4 oranges, 5 apples and 6 mangoes in a fruit basket. In how many ways can a person
select fruits from among the fruits in the basket? (JEE MAIN)

Sol: Use the concept of Combination of Alike objects described above.


Here, we consider all fruits of the same type as identical.
Zero or more oranges can be selected out of 4 identical oranges in 4 + 1 = 5 ways.
Zero or more apples can be selected out of 5 identical apples in 5 + 1 = 6 ways.
Zero or more mangoes can be selected out of 6 identical mangoes in 6 + 1 = 7 ways
∴ The total number of selections when all the three types of fruits are selected (the number of any type of fruit
may be zero) = 5 × 6 × 7 = 210.
But in one of these selections number of each type of fruit is zero and hence there is no selection, this must be
excluded.
∴ The required number = 210 – 1 = 209.
Caution: When all fruits of same type are different, the number of selections
= (4C0 + 4C1 + …….. + 4C4)(5C0 + 5C1 + …… + 5C5) (6C0 + ……….. + 6C6) –1 = 24 x 25 x 26 – 1 = 215 – 1

Illustration 20: How many four digit numbers are there whose decimal notation contains not more than two
distinct digits? (JEE MAIN)

Sol: A four digit number can consist of either only one digit or two digits as per the question. Clearly, there are nine
four digit numbers with the same digit. Similarly, calculate the number of four digit numbers with two distinct digits
and hence the sum gives us the desired result.
Evidently any number so formed of four digits contains
(i) Only one digit (like 1111, 2222,...) and there are 9 numbers. (ii) Two digits
(a) if zero is one of the two, then the one more can be anyone of the nine, and these two digits can be arranged
in 9C1 [3C1 + 3C2 + 3C2 + 3C3] = 63.
(b) if zero is not one of them, then two of the digits have to be selected from 9, and these two can be arranged
in 9C2[4C1 + 4C2 + 4C3] = 504
Hence, the total number of required numbers = 567.

Illustration 21: In how many ways can a cricket team of eleven players be chosen out of a batch of 15 players, if
(a) there is no restriction on the selection
(b) a particular player is always chosen
(c) a particular player is never chosen (JEE MAIN)
5 . 1 0 | Permutations and Combinations

Sol: Using the concept of combination of alike objects we can get the answer.
15 × 14 × 13 × 12
(a) The total number of ways of selecting 11 players out of 15 is = 15C11 = 15C15–11 = 15C4 = = 1365
4 ×3× 2×1
(b) A particular player is always chosen. This means that 10 players are selected out of the remaining 14 players.
∴ The required number of ways = 14C10 = 14C14–10 = 14C4 = 1001

(c) The number of ways = 14C11 = 14C3 = 14 × 13 × 12 = 364


3× 2

Illustration 22: In a plane there are 37 straight lines, of which 13 pass through the point A and 11 pass through
the point B. Moreover, no three lines pass through one point, no line passes both points A and B. and no two are
parallel. Find the number of points of intersection of the straight lines. (JEE MAIN)

Sol: Two non parallel straight lines give us a point of intersection. Using this idea we find the total number of points
of intersection. Care should be taken that a point is not counted more than once.
The number of points of intersection of 37 straight lines is 37C2. But 13 straight lines out of the given 37 straight
lines pass through the same point A. Therefore, instead of getting 13C2 points, we get merely one point A. Similarly,
11 straight lines out of the given 37 straight lines intersect at point B. Therefore, instead of getting 11C2 points, we
get only point B. Hence, the number of intersection points of the lines in 37C2 – 13C2 – 11C2 + 2 = 535.

Illustration 23: How many five-digit numbers can be made having exactly two identical digits? (JEE MAIN)

Sol: Note that zero cannot occupy the first place. So we divide the solution into two cases when the common digit
is 0 and otherwise. Calculate the number of possibilities in these two cases and their sum gives us the desired result.
Case I: Two identical digits are 0, 0.
The number of ways to select three more digits is 9C3. The number of arrangements of these five digits is
(5! 2!) − 4! = 60 − 24 = 36 .
Hence, the number of such numbers is 9C3 × 36 = 3024  … (i)
Case II: Two identical digits are (1, 1) or (2, 2) or.... or (9, 9).
If 0 is included, then number of ways of selection of two more digits is 8C2.
The number of ways of arrangements of these five digits is 5! / 2! -4! / 2! =48.
Therefore, the number of such numbers is 8C2 × 48.
If 0 is not included, then selection of three more digits is 8C3.
Therefore, the number of such numbers is 8C3 × 5! / 2! = 8C3 × 60.
Hence, the total number of five-digit numbers with identical digits (1.1)......(9.9) is
9 × (8C2 × 48 + 8C3 × 60) = 42336  ... (ii)
From Eqs. (i) and (ii), the required number of numbers is 3024 + 42336 = 45360.

Illustration 24: How many words can be made with letters of the word “INTERMEDIATE” if
(i) The words neither begin with I nor end with E,
(ii) The vowels and consonants alternate in the words,
(iii) The vowels are always consecutive,
(iv) The relative order of vowels and consonants does not change,
(v) No vowel is in between two consonants,
(vi) The order of vowels does not change? (JEE MAIN)
M a them a ti cs | 5.11

Sol: This is an application of Permutation under restriction and Permutation of Alike objects. Proceed according to
the given conditions.
(i) The required number of words = (the number of words without restriction) – (the number of words beginning
with I) – (the number of word ending with E) + (the number of words beginning with I and ending with E)
(because words beginning with I as well as words ending with E contain some words beginning with I and
ending with E).
12!
The number of words without restriction =
2! 3! 2!
( There are 12 letters in which there are two I’s, three E’s and two T’s).
11!
The number of words beginning with I =
2!3!
( With E in the extreme left place we are left to arrange 11 letters INTERMEDIATE in which there are two T’s
and three E’s).
11!
The number of words ending with E =
2!2!2!
( With E in the extreme right place we are left to arrange 11 letters INTERMEDIATE in which there are two
I’s. two E’s and two T’s.)
10!
The number of words beginning with I and ending with E =
2!2!
( With I in the extreme left and E in the extreme right places we are left to arrange 10 letters INTERMEDIATE
in the which there are two T’s and Two E’s).
12! 11! 11! 10! 10!
∴ the required number or words = – – + = (12×11 – 11×2 – 11×3 +6) = 83 × 10!
2!3!2! 2!3! 2!2!2! 2!2! 2!3!2! 24
(ii) There are 6 vowels and 6 consonants. So, the number of words in which vowels and consonants alternate =
(the number of words in which vowels occupy odd places and consonants occupy even places) + (the number
of words in which consonants occupy odd places and vowels occupy even places)
6! 6! 6! 6! 6! 6!
= × + × = 2. . = 43200
2!3! 2! 2! 2!3! 2!3! 2!
7!
(iii) Considering the 6 vowels IEEIAE as one object, the number of arrangements of this with 6 consonants =
( there are two T’s in the consonants). 2!

For each of these arrangements, the 6 consecutive vowels can be arranged among themselves in 6! .
7! 6! 2!3!
∴ The required number of words = × (as above) = 151200
2! 2!3!
(iv) The relative order of vowels and consonants will not change if in the arrangements of letters, the vowels
occupy places of vowels, i.e.. 1st, 4th, 7th, 9th, 10th, 12th places and consonants occupy their places, i.e., 2nd. 3rd.
5th. 6th. 8th. 11th places, the required number of words
6! 6!
× = 21600
2!3! 2!

(v) No vowel will be between two consonants if all the consonants become consecutive
∴ the required number of words = the number of arrangements when all the consonants are consecutive
7! 6!
= × (as above) = 151200
2!3! 2!
(vi) The order of vowels will not change if no two vowels interchange places, i.e., in the arrangement all the vowels
are treated as identical.
(For example LATE. ATLE. TLAE. etc.. have the same order of vowels A. E. But LETA, ETLA.
TLEA. etc.. have changed order of vowels A. E. So. LATE is counted but LETA is not.
If A, E, are taken as identical say V then LVTV does not give a new arrangement by interchanging V, V.
5 . 1 2 | Permutations and Combinations

The required number of words,


= The number of arrangements of 12 letters in which 6 vowels are treated as identical
12!
= ( there are two T’s also).
6!2!

Illustration 25: India and South Africa play a one day international series until one team wins 4 matches. No match
ends in a draw. Find, in how many ways can the series can be won.  (JEE ADVANCED)

Sol: The team who wins the series is the team with more number of wins. The losing team wins either 0 or 1 or 2
or 3 matches. Using this we find the number of ways in which a team can win.
Let I for India and S for South Africa. We can arrange I and S to show the wins for India and South Africa respectively
Suppose India wins the series, then the last match is always won by India.
Wins of S Wins of I No. of ways
(i) 0 4 1
(ii) 1 4 4! / 3! = 4
5!
(iii) 2 4 =10
2!3!
6!
(iv) 3 4 = 20
3!3!
∴ Total no. of ways = 35
In the same number of ways South Africa can win the series
∴ Total no. of ways in which the series can be won = 35 × 2 = 70

Illustration 26: There are p intermediate railway stations on a railway line from one terminal to another. In how
many ways can a train stop at three of these intermediate stations, if no two of these stations (where it stops) are
to be consecutive? (JEE ADVANCED)
Sol: The train stops only at three intermediate stations implies that the train does not stop at (p – 3) stations. Using
this idea we proceed further to get the answer.
The problem then reduces to the following:
In how many ways can three objects be placed among (p – 3) objects in a row such that no two of them are next
to each other (at most 1 object is to be placed between any two of these (p – 3) objects). Since there are (p – 2)
positions to place the three objects, the required number of ways = p–2C3.

Illustration 27: Five balls are to be placed in three boxes. Each can hold all the five balls. In how many different
ways can we place the balls so that no box remains empty if
(i) balls and boxes are all different
(ii) balls are identical but boxes are different
(iii) balls are different but boxes are identical
(iv) balls as well as boxes are identical
(v) balls as well as boxes are identical but boxes are kept in a row?  (JEE MAIN)

Sol: Use the different cases of combination to solve the question according to the given conditions
As no box is to remain empty, boxes can have balls in the following numbers:
Possibilities 1, 1, 3 or 1, 2, 2
M a them a ti cs | 5.13

(i) The number of ways to distribute the balls in groups of 1, 1, 3 = 5C1 × 4C1 × 3C3.
But the boxes can interchange their content, no exchange gives a new way when boxes containing balls in equal
number interchange.
3!
∴ the total number of ways to distribute 1, 1, 3 balls to the boxes = 5C1 × 4C1 × 3C3 ×
2! 3!
Similarly, the total number of ways to distribute 1, 2, 2 balls to the boxes = 5C1 × 4C2 × 2C2 ×
3! 5 3! 2!
∴ the required number of ways = 5C1 × 4C1 × 3C3 × + C 1 × 4C 1 × 2C 2 × = 5 × 4 × 3 + 5 × 6 × 3 = 60 + 90 = 150
2! 2!
Note: Writing the whole answer in tabular form.
Possibilities Combinations Permutations
3!
1, 1, 3 5
C1 × 4C1 × 3C3 5
C 1 × 4C 1 × 3C 3 × = 5 × 4 × 3 = 60
2!
3!
1, 2, 2 5
C1 × 4C2 × 2C2 5
C 1 × 4C 2 × 2C 2 × = 5 × 6 × 3 = 90
2!
∴ the required number of ways = 60 + 90 = 150.
(ii) When balls are identical but boxes are different the number of combinations will be 1 in each case.
3! 3!
∴ the required number of ways = 1 × +1 × =3+3=6
2! 2!
(iii) When the balls are different and boxes are identical, the number of arrangements will be 1 in each case.
∴ the required number of ways = 5C1 × 4C1 × 3C3 + 5C1 × 4C2 × 2C2 = 5 × 4 + 5 × 6 = 20 + 30 = 50
(iv) When balls as well as boxes are identical, the number of combinations and arrangements will be 1 each in
both cases.
∴ the required number of ways = 1 × 1 + 1 × 1 = 2
(v) When boxes are kept in a row, they will be treated as different. So, in this case the number of ways will be the
same as in (ii).

Illustration 28: There are m points on one straight line AB and n points on another straight line AC, none of them
being A. How many triangles can be formed with these points as vertices? How many can be formed if point A is
also included?  (JEE MAIN)

Sol: A triangle has three vertices, so we select two points on one line and one on the other and vice versa. Also,
consider the case when one point of the triangle is the intersection of the two lines.
To get a triangle, we either take two points on AB and one point on AC. or one point on AB and two points on AC.
Therefore, the number of triangles, we obtain
m(m – 1) n(n – 1) 1 1
= (mC2)(nC1) + (mC1)(nC2) = n+m = mn(m – 1 + n – 1) = mn(m + n – 2)
2 2 2 2

If the point A is included, we get m n additional triangles. Thus, in this case we get
mn mn(m + n)
= (m + n – 2) + mn = triangles.
2 2

5.3 Division into Groups

(a) The number of ways in which (m + n) different objects can be divided into two unequal groups containing m
(m + n)!
and n objects respectively is .
m!n!
(2n)!
If m = n, the groups are equal and in this case the number of division is ; as it is possible to interchange the
n!n!2!
two groups without obtaining a new distribution.
5 . 1 4 | Permutations and Combinations

(b) However, if 2n objects are to be divided equally between two persons then the number of ways
(2n)! (2n)!
= 2! =
n!n!2! n!n!

(c) The number of ways in which (m + n + p) different objects can be divided into three unequal groups containing
(m + n + p)
m , n and p objects respectively is = ,m≠n≠p
m!n!p!
(3n)!
If m = n = P then the number of groups = . However, if 3n objects are to be divided equally among three
n!n!n!3!
(3n)! (3n)!
persons then the number of ways = 3! =
n!n!n!3! (n!)3
15!
For example, the number of ways in which 15 recruits can be divided into three equal groups is and the
5!5!5!3!
15!
number of ways in which they can be drafted into three different regiments, five in each, is
5!5!5!

(d) The number of ways in which mn different objects can be divided equally into m groups if order of groups is
mn!
not important is
(n!)m m!

(e) The number of ways in which mn different objects can be divided equally into m groups if the order of groups
mn! (mn)!
is important is × m! =
m
(n!) m! (n!)m

Illustration 29: In how many ways can 12 balls be divided between 2 boys, one receiving 5 and the other 7 balls?
 (JEE MAIN)

Sol: Simple application of division of objects into groups. Since the order is important, the number of ways in which
12 different balls can be divided between two boys who each get 5 and 7 balls respectively, is
12! 12.11.10.9.8.7!
= × 2! = × 2 1584
5!7! (5.4.3.2.1)7!

Alternative:
The first boy can be given 5 balls out of 12 balls in 12C5 ways. The second boy can be given the remaining 7 balls in
one way. But the order is important (the boys can interchange 2 ways).
12! 12.11.10.9.8.7!.2
Thus, the required number of ways = 12C5 × 1 × 2! = ×2= = 1584
5!7! 5.4.3.2.1.7!
Illustration 30: Find the number of ways in which 9 different toys can be distributed among 4 children belonging
to different age groups in such a way that the distribution among the 3 elder children is even and the youngest
one is to receive one toy more.  (JEE ADVANCED)

Sol: Using the concept of division of objects into groups we can solve this problem very easily.
The distribution should be 2, 2, 2 and 3 to the youngest. Now, 3 toys for the youngest can be selected in 9C3 ways,
the remaining 6 toys can be divided into three equal groups in
6!
ways and can be distributed in 3 ! ways.
(2!)3 .3!
6! 9!
Thus, the required number of ways = 9C3. 3! =
3
(2!) 3! 3!(2!)3
M a them a ti cs | 5.15

Illustration 31: Divide 50 objects in 5 groups of size 10, 10, 10, 15 and 5 objects. Also find the number of
distributions? (JEE MAIN)
50!
Sol: Same as the above question. Number of ways of dividing 50 objects into 5 groups as given = 3
(10!) (15)!(5)!(3)!
50!
Number of ways of distributing 50 objects into above formed groups = × 5!
(10)!3 .(15)!(5)!3!

PLANCESS CONCEPTS

Identical Objects and Distinct Choices


Questions involving identical objects tend to be tricky, especially when the choices that they have are
distinct.
Many choices image 1
An example would be :
Q. In how many ways can we place 10 identical oranges in 3 distinct baskets, such that every basket has
at least 2 oranges each?
One method is to place 2 oranges in every basket and make cases for the rest of them. However, in such
questions, the other approach results in fewer cases, and hence, simpler calculations and a more efficient
solution
For this question: Divide 10 into groups of 3 rather than placing 2 in each and dividing the remaining
four.
Vaibhav Gupta (JEE 2009, AIR 54)

6. CIRCULAR PERMUTATION
Let us consider that persons A, B, C and D are sitting around a round table. If all of them (A, B, C, D) are shifted one
place in an anticlockwise order, then we will get fig.(b) from fig.(a). Now, if we shift A, B, C, D in anticlockwise order
again, we will get fig. (c). We shift them once more and we will get fig.(d); and in the next time fig(a).

C B A D

D B C A B D A C

A D C B

A B C D D A B C C D A B B C D A

a b c d
Figure 5.2

Thus, we see that if 4 persons are sitting at a round table, they can be shifted four times and the four different
arrangements thus obtained will be same, because the anticlockwise order of A, B, C, D does not change. But if A,
B, C and D are sitting in a row and they are shifted in such an order that the last occupies the place of first, then the
four arrangements will be different.
Thus, if there are 4 objects, then for each circular arrangement number of linear arrangements is 4.
Similarly, if n different objects are arranged along a circle, for each circular arrangement the number of linear
arrangements is n.
Therefore, the number of circular arrangements of n different objects = the number of linear arrangements of n
different objects / n = n!/(n) = (n – 1)!
5 . 1 6 | Permutations and Combinations

Clockwise and Anticlockwise Arrangements


Let the four persons A, B, C and D sit at a round table in anticlockwise as well as clockwise directions. These two
arrangements are different. But if four flowers R (red), G (green), Y (yellow) and B (blue) are arranged to form
a garland in anticlockwise and in clockwise order, then the two arrangements are same because if we view the
garland from one side the four flowers R, G, Y, B will appear in anticlockwise direction and if seen from the other
side the four flowers will appear in the clockwise direction. Here the two arrangements will be considered as one
arrangement because the order of flowers does not change, rather only the side of observation changes. Here, two
permutations will be counted as one.
C C Y Y

D B B D B G G B

A A R R

Figure 5.3

Therefore, when clockwise and anticlockwise arrangements are not different, i.e. when observations can be made
from both sides, the number of circular arrangements of n different objects is (n – 1)!/2
Consider five persons A, B, C, D, E on the circumference of a circular table in an order which has no head. Now,
shifting A, B, C, D, E one position in anticlockwise direction we will get arrangements as follows:
We see, that arrangements in all figures are different.
∴ The number of circular permutation of n different objects taken all at a time is (n – 1)!, if clockwise and anticlockwise
orders are taken as different.

Note:
(a) The number of circular permutations of n different objects taken r at a time
n
Pr/r, when clockwise and anticlockwise orders are treated as different.
n
Pr/2r, when clockwise and anticlockwise orders are treated as same.
(b) The number of circular permutations of n different objects altogether
n
Pn/n = (n – 1)!, when clockwise and anticlockwise order are treated as different,
n
Pn/2n = 1/2(n – 1)!, when the above two orders are treated as same.

Illustration 32: In how many ways can 5 Indians and 4 Englishmen be seated at a round table if
(a) There is no restriction, (b) All the four Englishmen sit together,
(c) All four Englishmen don’t sit together, (d) No two Englishmen sit together. (JEE MAIN)

Sol: Clearly, this is a case of Circular Permutation. Using the formula (n – 1)!, we can find the answer according to
the given cases.
(a) Total number of persons = 5 + 4 = 9. These 9 persons can be seated at the round table in 8! Ways.
∴ Required number of ways = 8!
(b) Regarding 4 Englishmen as one person, we have only 5 + 1 i.e. 6 persons.
These 6 persons can be seated at the round table in 5! ways. Also, the 4 Englishmen can be arranged among
themselves in 4! ways.
∴ the required number of ways = 5! 4!
(c) The total number of arrangements when there is no restriction = 8!; the number of arrangements when all the
four English men sit together = 5! 4!
∴ The number of arrangements when all the four Englishmen don’t sit together = 8! – 5! 4!
M a them a ti cs | 5.17

(d) As there is no restriction on Indians, we first arrange the 5 Indians.


Now, 5 Indians can be seated around a table in 4! ways. If an Englishman sits between two Indians, then no
two Englishmen will sit together. Now, there are 5 places for 4 English men, therefore, 4 Englishmen can be
seated in 5P4 ways.
∴ The required number of ways = 4! × 5P4 = 4 × 5!

Illustration 33: Consider 21 different pearls on a necklace. How many ways can the pearls be placed in this
necklace such that 3 specific pearls always remains together? (JEE MAIN)

Sol: This is the case of circular permutation when there is no distinction between clockwise and anticlockwise
arrangements.
After fixing the places of three pearls. Treating 3 specific pearls = 1 unit. So we have now 18 pearls + 1 unit = 19
and the number of arrangements will be (19 – 1)! = 18!. Also the number of ways 3 pearls can be arranged between
themselves is 3! = 6. As there is no distinction between the clockwise and anticlockwise arrangements, the required
1
number of arrangements = 18!. 6 = 3 (18!).
2

Illustration 34: Six persons A, B, C, D, E and F are to be seated at a circular table. Find the number of ways this can
be done if A must have either B or C on his right and B must have either C or D on his right. (JEE MAIN)

Sol: Fix the position of some of the persons relative to each other as per the question and arrange the remaining
in the seats available.
When A has B or C to his right we have either AB or AC
When B has C or D to his right we have BC or BD.
Thus, we must have ABC or ABD or AC and BD.
For ABC, D, E, F in a circular number of ways = 3! = 6
For ABD, C, E, F in a circular number of ways = 3! = 6
For AC, BD E, F the number of ways = 3! = 6
Hence, the required number of ways = 18

7. LINEAR EQUATIONS WITH UNIT COEFFICIENTS


Consider the equation x1 + x2 + x3 + … + xk = m, in k variables whose sum must always be m,
The number of non-negative solutions to the above equation is given by the fictitious partition method which is
stated as:
Method of fictitious partition: Number of ways in which n identical objects may be distributed among p persons
if each person may receive none, one or more objects is = n+p–1Cn.

Coefficient Method
(a) The number of non-negative integral solutions of equation x1 + x2 + … + xr = n
= The number of ways of distributing n identical objects among r persons when each person can get zero, one or
more objects = coeff. of xn in [(1 + x + x2 + … + xn)(1 + x + x2 + … + xn) (1 + x + x2 + … + xn)…upto r factors]
= coeff. of xn in (1 + x + x2 + … + xn)r
r
 1 – xn+1 
= coeff. of x in 
n  = coeff. of xn in (1 – xn+1)r (1 – x)–r = coeff. of xn in (1 – x)–r

 1–x 
[leaving terms containing powers of x greater than n] = n+r–1Cr–1
5 . 1 8 | Permutations and Combinations

Note: If n is a positive integer, then


(–n) (–n)(–n – 1) (–n)(–n – 1)(–n – 2)
(1 – x)–n = 1 + (x) + (–x)2 + (–x)3 + … to ∞
1! 2! 3!
n n(n + 1) 2 n(n + 1)(n + 2) 3
=1+ x+ x + x + …. To ∞ = 1 + nC1x + n+1C2x2 + 2C3x3 + … to ∞
1! 2! 3!
Coeff. Xr in (1 – x)–n = n+r–1C ⇒ coeff. of xn in (1 –x)–r = n+r–1Cn = n+r–1Cr–1
(b) The number of positive integral solutions for the equation x1 + x2 + …. + xr = n
= The number of ways of distributing n identical objects among r persons when each person can get at least one
object
= coeff. of xn in [x + x2 + …+ xn) (x + x2 + … + xn) (x +x2 + … + xn) …. Upto r factors]
r
 1 – xn 
r
= coeff. of x in (x + x + … + x ) = coeff. of x in x 
n 2 n r n

 1–x 
 
= coeff. Of x in (1 – x ) (1 – x) = coeff. Of x in (1 – x)
n–r n r –r n–r –r

[Leaving terms containing powers of x greater than n – r]


= n–r+r–1Cr–1 = n–1Cr–1.

Illustration 35: How many integral solutions are there to x + y + z + w = 29, when x ≥ 1, y ≥ 2, z ≥ 3 and w ≥ 0?
 (JEE ADVANCED)
Sol: Application of multinomial theorem.
x + y + z + w = 29 .…(i)
x ≥ 1, y ≥ 2, z ≥ 3, w ≥ 0 ⇒ x – 1 ≥ 0, y – 2 ≥ 0, z – 3 ≥ 0, w ≥ 0
Let x1 = x – 1, x2 = y – 2, x3 = z – 3

⇒ x = x1 + 1, y = x2 + 2, z = x3 + 3 and then x1 ≥ 0, x2 ≥ 0, x3 ≥ 0, w ≥ 0

From (i), x1 + 1 + x2 + 2 + x3 + 3 + w = 29

⇒ x1 + x2 + x3 + w = 23

Hence, the total number of solutions = 23+4–1C4–1 = 26C3 = 2600

Illustration 36: Find the number of non-negative integral solution 3x + y + z = 24. (JEE MAIN)

Sol: Application of multinomial theorem.


3x + y + z = 24, x ≥ 0, y ≥ 0, z ≥ 0
Let x = k ∴ y + z = 24 – 3k .…(i)
Here, 0 ≤ 24 – 3k ≤ 24. Hence, 0 ≤ k ≤ 8
The total number of integral solutions of (1) is 24–3k+2–1C2–1 = 25–3kC1 = 25 – 3k
Hence, the total number of solutions of the original equation
8 8 8
8.9
= ∑ (25 – 3k) = 25 ∑ 1 – 3∑ k ⇒ 25.9 – 3.
2
= 225 – 108 = 117.
=k 0 =k 0=k 0

Illustration 37: Find the number of solutions of the equation x + y + z = 6, where x, y, z ∈ W. (JEE MAIN)

Sol: The number of solutions = 6+3–1C3–1 = 8C2 = 280.


M a them a ti cs | 5.19

Illustration 38: How many integers are there between 1 and 1000000 having the sum of the digits as 18?
 (JEE ADVANCED)
Sol: Let the digits be a1, …, a6 and use multinomial theorem we get the answer.
Any number between 1 and 1000000 must be of less than seven digits. Therefore, it must be of the form a1 a2 a3
a4 a5 a6
Where a1. a2.a3.a4.a5.a6 ∈ {0, 1, 2, … 9}

Thus a1 + a2 + a3 + a4 + a5 + a6 = 18, where 0 ≤ ai ≤ 9, i = 1, 2, … 9


The required number of ways = coeff. of x18 in (1 + x + x2 + … x9)6
6
 1 – x10 
= coeff. of x in 
18

 1–x 
 
= coeff. of x in [(1 – x ) (1 – x)–6] ; = coeff. of x18 in [(1 – 6C1 x10 …) (1 – x)–6]
18 10 6

[leaving terms containing powers of x greater than 18]


= coeff. of x18 in (1 – x)–6 – 6C1. coeff. of x8 in (1 – x)–6 = 6+18–1C5 – 6. 6+8–1C5 = 23C5 – 6.13C5

23.22.21.20.19 13.12.11.10.9
= – 6. = 33649 – 7722 = 25927
120 120

PLANCESS CONCEPTS

•• m different white balls and n different red balls are to be arranged in a line such that the balls of the
same colour are always together = (m! n! 2!)
•• m different white balls and n different red balls are to be arranged in a line such that all the red balls
are together = ((m + 1)! n!)
•• m different white balls and n different red balls are to be arranged in a line such that no two red balls
are together (m ≥ n – 1) = (m+1Cn m! n!)
•• m different white balls and m different red balls are to be arranged in a line such that colour of the
balls is alternating = (2 × (m!)2)
•• m identical white balls and n different red balls are to be arranged in a line such that no two red balls
are together (m ≥ n – 1) = (m+1Cn)
•• m identical white balls and n different red balls are to be arranged in a line such that no two red balls
are together (m ≥ n – 1) = (m+1Cn n!)
•• If n objects are arranged in a line the number of selections of r objects (n ≥ 2r – 1) such that no two
objects are adjacent is same number of ways of arranging n – r identical white balls and r identical red
ball in a line such that no two balls are together = (n–r+1Cr). e.g. suppose there are n stations on trains’s
route and a train has to stop at r stations such that no two stations are adjacent. The number of ways
must be n–r+1Cr.
•• suppose there are N seats in a particular row of a theatre. The number of ways of making n people
sit (N ≥ 2n – 1) such that no two people sit side by side is same as number of ways of arranging N – n
identical white balls (empty seats) and n different red balls (n people) such that no two red balls are
together. The required number of ways are N–n+1Cn × n!.
Nitish Jhawar (JEE 2009, AIR 7)
5 . 2 0 | Permutations and Combinations

8. DIVISIBILITY OF NUMBERS
The following table shows the conditions of divisibility of some numbers

Divisible by Condition
2 Whose last digit is even
3 sum of whose digits is divisible by 3
4 whose last two digits number is divisible by 4
5 whose last digit is either 0 or 5
6 which is divisible by both 2 and 3
7 If you double the last digit and subtract it from the rest of the number, answer is a multiple of 7
8 whose last three digits number is divisible by 8
9 sum of whose digits is divisible by 9
10 Whose last digit is 0
11 If you sum every second digit and then subtract sum of all other digits, answer is a multiple of 11
25 whose last two digits are divisible by 25

Illustration 39: How many four digit numbers can be made with the digits 0, 1, 2, 3, 4, 5 which are divisible by 3
(digits being unrepeated in the same number)? How many of these will be divisible by 6? (JEE ADVANCED)

Sol: A number is divisible by 3 if the sum of the digits is divisible by 3. This reduces the problem to the number of
non-negative integral solutions of equation x1 + x2 + … + xr = n.
Here, 0 + 1 + 2 + 3 + 4 + 5 = 15; so two digits are to be omitted whose sum is 3 or 6 or 9.
Hence, the number of four digits can be made by either
1, 2, 4, 5 or 0, 3, 4, 5 (omitting two digits whose sum is 3)
0, 1, 3, 5 or 0, 2, 3, 4 (omitting two digits whose sum 6)
0, 1, 2, 3 (omitting two digits whose sum is 9)
The number of 4-digit numbers that can be made with 1, 2, 4, 5 = 4P4 = 4!
The number of 4-digit numbers that can be made by the digits in any one of remaining four groups (each containing
0) = 4! – 3!
∴ The required number of 4-digit numbers divisible by 3 = 4! + 4(4! – 3!) = 24 + 4(24 – 6) = 96
Now, a number is divisible by 6 if it is even as well as divisible by 3.
So, the number of 4-digit numbers divisible by 6 that can be made with 1, 2, 4, 5 = 2 × 3! (∵ the number should
have an even digit in the units places).
The number of numbers of 4 digits, divisible by 6, that can be made with 0, 3, 4, 5 = (3! – 2!) + 3!
( The number should have 4 or 0 in units place and 0 should not come in thousands place).
Similarly, the number of numbers of 4 digits, divisible by 6, that can be made with 0, 1, 2, 3 =
(3! – 2!) + 3!
The number of 4-digit numbers divisible by 6 that can be made with the digits 0, 1, 3 = 3!
The number of numbers of 4 digits, divisible by 6, that can be made with 0, 2, 3, 4 = (3! – 2!) +
(3! – 2!) + 3!
( The number should have 4 or 2 or 0 in units place and 0 should not come in thousands place)
M a them a ti cs | 5.21

∴ the required 4-digit numbers divisible by 6


= 2 × 3! + (3! – 2!) + (3! – 2!) + 3! + 3! + (3! – 2!) + (3! – 2!) + (3! – 2!) + 3! = 12 + 4 + 6 + 4 + 6 + 6 + 4 + 4 + 6 = 52.

9. SUM OF NUMBERS
(a) For given n different digits a1, a2, a3 … an the sum of the digits in the units place of all numbers formed (if
numbers are not repeated) is
(a1 + a2 + a3 + … + an) (n – 1)! i.e. (sum of the digits) (n – 1)!
(b) Sum of the total numbers which can be formed with given different digits a1, a2, a3…… an is
(a + a2 + a3 + … + an) (n –1)! (111. ……….. n times)

Illustration 40: Find the sum of all 4 digit numbers formed using the digits 1, 2, 4 and 6.  (JEE MAIN)

Sol: Use formula, Sum = (a1 + a2 + a3 + … + an) (n− 1)! (111 …. N times)
Using formula, Sum = (1 + 2 + 4 + 6) 3! (1111) = 13 × 6 × 1111 = 86658

Alternate:
Here, the total 4-digit numbers will be 4! = 24. So, every digit will occur 6 times at every one of the four places.
Since the sum of the given digits = 1 + 2 + 4 + 6 = 13. So, the sum of all the digits at every place of all the 24
numbers = 13 × 6 = 78.
The sum of the values of all the digits
At first place = 78
At the tens place = 780
At the hundreds place = 7800
At the thousands place = 78000
∴ The required sum 78 + 780 + 7800 + 78000 = 86658

10. FACTORS OF NATURAL NUMBERS


Let N = pa . qb . rc … where p, q, r … are distinct primes & a, b, c … are natural numbers, then:
(a) The total number of divisors of N including 1 and N are = (a + 1) (b + 1) (c + 1) …
(b) The sum of these divisors is = (p0 + p1 + p2 +…+ pa) (q0 + q1 + q2 +…+ qb) (r0 + r1 + r2 +…+ rc)…
(c) The number of ways in which N can be resolved as a product of two factors is
 1
 (a + 1)( a2+ 1)( a3 + 1)... if N is not a perfect square
= 2 1
 1
(a + 1)(a 2 + 1)(a 3 + 1)... + 1 if N is a perfect square

 2 1

(d) The number of ways in which a composite number N can be resolved into two factors which are relatively
prime (or coprime) to each other is equal to 2n–1 where n is the number of different prime factors in N

Illustration 41: Find the number of factors of the number 38808 (excluding 1 and the number itself). Find also the
sum of these divisors. (JEE MAIN)

Sol: Factorise 38808 into its product of primes and then use the concept of combination to find the answer.
38808 = 23 . 32 . 72 . 11
5 . 2 2 | Permutations and Combinations

Hence, the total number of divisors (excluding 1 and itself) = (3 + 1) (2 + 1) (2 + 1) (1 + 1) – 2 = 70


The sum of these divisors = (20 + 21 + 22 + 23) (30 + 31 + 32) (70 + 71 + 72) (110 + 111)
= (15) (13) (57) (12) – 1 – 38808 = 94571

Illustration 42: In how many ways can the number 10800 be resolved as a product of two factors?  (JEE MAIN)

Sol: Check whether the number is a perfect square or not and accordingly use the formula to find the desired result.
10800 = 24 . 33 . 52
Here 10800 is not a perfect square ( power of 3 is odd).
1
Hence, the number of ways = (4 + 1) (3 + 1) (2 + 1) = 30.
2
Illustration 43: Find the number of positive integral solutions of x1.x2.x3 = 30.  (JEE ADVANCED)

Sol: Factorise 30 into primes and then use combination to get the desired result.
x1x2x3 = 2 × 3 × 5. If we treat 2, 3, 5 as objects and x1, x2, x3 as distinct boxes then finding the number of positive
integer solution is the same as finding the number of ways of distributing 3 distinct objects in 3 distinct boxes.
Thus, the required number of solutions is 35 = 27
(For example, if all the objects are held by x1 the corresponding solution is x1 = 30. x2 = 1, x3 = 1, if 2 and 3 are held
by x1 and 5 by x3 then x1 = 6, x2 = 1, x3 = 5 etc)

11. EXPONENT OF A PRIME P IN N!


Consider a prime p and we want to know its exponent in n!.

n
The number of multiples of p in n! is given by   . Even if a number k between 1 and n has two factors of p, this
p 
n
formula counts it as only one. Hence we need to evaluate   also. Similarly, for three and four to infinity. Hence,
2
 p 
∞  
n
the exponent of prime p in n! is given by ep(n) = ∑   ep is called Legendre’s function.
i
 p 
i=1 

Even though this is an infinite sum result, it is finite since for all pi greater than n, step function becomes zero.
Let’s understand this method by an example of finding exponent of 2 in 100!

 100   100   100   100   100   100   100 


Sol:=
  50;=
 2  25;=
 3  12;=
 4  6;=
 5  3;=
 6  1;=
 7  0
 2  2  2  2  2  2  2 
From this result we can infer that there are 50 numbers between 1 and 100 which have a factor of 2.
Out of these 50, there are 25 numbers that have a factor of 22.
Out of these 25, there are 12 numbers that have a factor of 23.
Out of these 12, there are 6 numbers that have a factor of 24.
Out of these 6, there are 3 numbers that have a factor of 25.
Out of these 3, there is 1 number that has a factor of 26.
But there is no number that has a factor of 27 or higher.
⇒ 100! can be written as r.2n where 2n = (26)1 (25)(3–1) (24)(6–3) (23)(12–6) (22)(25–12)(2)(50–25) and r being a natural number
which doesn’t have 2 as a factor.
M a them a ti cs | 5.23

⇒ 2n = (25.2)1 (24.2)(3–1)(23.2)(6–3) (22.2)(12–6) (2.2)(25–12) (2)(50–25)


= (25)1(24)(3–1) (23)(n–3)(22)(12–6) (2)(25 – 12) (2)50
= (24.2)1 (23.2)(3–1) (22.2)(6–3) (2.2)(12–6) (2)(25–12) (2)50
= (23.2)1 (22.2)(3–1) (2.2)(6–3) (2)(12–6) (2)25 (2)50
= (22.2)1 (2.2)(3–1) (2)(6–3) (2)12 (2)25 (2)50
= (2.2)1 (2)(3–1) (2)6 (2)12 (2)25 (2)50
= (2)1 (2)3 (2)6 (2)12 (2)25 (2)50
Hence, the exponent of 2 in 100! is = 50 + 25 + 12 + 6 + 3 + 1 = 97

PLANCESS CONCEPTS

Following method involves part of an advanced topic in mathematics called Modular Arithmetic.
Legendre’s function also has another result, which is
∞ n n – Sp (n)
ep(n) = ∑  pi  = p –1
i=1  

Where Sp(n) is the sum of digits of n when written in base p.


Converting n to base p is done by repeated division of n by p and by noting the remainders to form a
number starting with units place.
This procedure is similar to converting a decimal number to binary. In binary, the base is equal to 2. Let
us solve an example using this method.
Example: Determine the exponent of 3 in ((3!)!)!
Sol: ((3!)!)! = (6!)! = 720!
Let us convert 720 to base 3
3 720
3 240 R 0
3 80 R 0
3 26 R 2
38 R2
32 R2
30 R2
Hence 720 = (222200)3
S3(720 ) = 2 + 2 + 2 + 0 + 0 = 8
720 – 8 712
⇒ e3(720) = = = 356
3–1 2
You can verify this answer using previous method.
If you are confused by base conversion then do not use this method.
Akshat Kharaya (JEE 2009, AIR 235)
5 . 2 4 | Permutations and Combinations

Illustration 44: Find the exponent of 7 in 400!. (JEE MAIN)

Sol: Apply Legendre’s formula.


 400   400   400 
e7(400) =   +  2  +  3  = 57 + 8 + 1 = 66.
 7   7   7 

Illustration 45: Find all positive integers of n such that n! ends in exactly 1000 zeros. (JEE ADVANCED)
Sol: 10 is a multiple of 2 and 5. In order to get 1000 zeroes we must have 1000 as the exponent of 5 in n!. Now use
the definition of the GIF to find the range of numbers satisfying the given condition.
n   n 
There are clearly more 2’s than 5’s in the prime factorization of n!, hence it suffices to solve the equation   +  
+ … = 1000.  5   52 

n   n  n n n 1  n 1 n
But   +   + … < + + … =  1 + + ...  (as [x] < x) = . = .
5 2
  5  5 52 5  5  5 1 4
1–
5
Hence, n > 4000.
5
On the other hand, using the inequality [x] > x – 1, we have 1
1– 
n  n  n   n
1000 >  – 1  +  – 1  +  – 1  + 
 n  n  1 1
– 1  +  – 1  = 1 + +
1 1  n 5
+ +  –5= . – 5.
5  5 2
 53
 5 4
 5 5
 5 5 52
53 4
5  5 1
1–
5
1005 ⋅ 4 ⋅ 3125
So, N < < 4022.
3124
We narrowed n down to {4001, 4002, … , 4021}. Using Legendre’s formula we find that 4005 is the first positive
integer with the desired property and that 4009 is the last. Hence, n = 4005, 4006, 4007, 4008, 4009.
Second solution: It suffices to solve the equation e5(n) = 1000. Using the second form of Legendre’s formula, this
becomes n – s5(n) = 4000. Hence n > 4000. We work our way upward from 4000 looking for a solution. Since e5(n)
can change only at multiples of 5 (why?), we step up 5 each time:
4000 – 4
e5(4000) = = 999.
5–1
4005 – 5
e5(4005) = = 1000.
5–1
4010 – 6
e5(4010) = = 1001.
5–1
Any n > 4010 will clearly have e5(n) ≥ e5(4010) = 1001. Hence the only solutions are n = 4005, 4006, 4007, 4008, 4009.

12. INCLUSION-EXCLUSION PRINCIPLE


In its general form, the principle of inclusion-exclusion states that for finite sets A1,…An. One has the identity
n n
Ai ∑ Ai
= − ∑ | Ai ∩ Aj | + ∑ | Ai ∩ Aj ∩ Ak | – ... + (–1)n–1 | A1 ∩ ... ∩ An | .
i=1=i 1 1≤i< j≤n 1≤i< j<k ≤n

n n  
=
This can be compactly written as  Ai ∑ (–1)k +1  ∑ | Ai ∩ ... ∩ Ai | 
1 k 
i==
1 k 1  1≤i1 <........<ik ≤n 
In words, to count the number of elements in a finite union of finite sets, first sum the cardinalities of the individual
sets, then subtract the number of elements which appear in more than one set, then add back the number of
elements which appear in more than two sets, then subtract the number of elements which appear in more than
three sets, and so on. This process naturally ends since there can be no elements which appear in more than the
number of sets in the union.
M a them a ti cs | 5.25

In applications it is common to see the principle expressed in its complementary form. That is, taking S to be a
finite universal set containing all of the Ai and letting Ai denote the complement of Ai in S. By De Morgan’s laws.
n n n
We have,  Ai = S –  Ai = | S |= ∑ | Ai | + ∑ | Ai ∩ A j | –... + (–1)n | A1 ∩ ... ∩ An | .
=i 1 =i 1 =i 1 1≤i< j≤n

Illustration 46: 105 students take an examination of whom 80 students pass in English. 75 students pass in
Mathematics and 60 students pass in both subjects. How many students fail in both subjects? (JEE MAIN)

Sol: A simple application of Inclusion-Exclusion Principle.


Let X = the set of students who take the examination.
A = the set of students who pass in English
B = the set of students who pass in Mathematics
We are given that n(X) = 105, n(A) = 80, n(B) = 75, n(A ∩ B) = 60
Since, n(A ∪ B) = n(A) + n(B) – n(A ∪ B).
Therefore, n(A ∪ B) = 80 + 75 – 60 = 95.
The required number = n(X) – n (A ∪ B) = 105 – 95 = 10.
Thus, 10 students fail in both subjects.

Illustration 47: Find the number of permutations of the 8 letters AABBCCDD, taken all at a time, such that no two
adjacent letters are alike. (JEE ADVANCED)

Sol: Divide the question into cases when A’s are adjacent, B’s are adjacent and so on. Similarly proceed to find the
number of ways in which two alike objects are adjacent and so on. Then use Inclusion-Exclusion Principle to find
the result.
First disregard the restriction that no two adjacent letters be alike.
8!
The total number of permutation is then N = = 2250
2!2!2!2!
Now, apply the inclusion exclusion principle. Where a permutation has property α in case the A’s are adjacent,
property β in case the B’s are adjacent, etc. It can be calculated that
7! 6!
N(α) = = 630. N(α, β) = = 180
2!2!2! 2!2!
N(α, β, γ) = 60. N(α, β, γ, δ) = 24.
Hence, the answer is N – 4N (α) + 6N(α, β) – 4N(α, β, γ) + N(α, β, γ, δ) = 864.

13. DERANGEMENTS THEOREM


Derangements theorem is an important application of inclusion exclusion principle.
Suppose, there are n letters and n corresponding envelopes. The number of ways in which letters can be placed in the
 1 1 (–1)n 
envelopes (one letter in each envelope) so that no letter is placed in correct envelope is n! 1 – + + ... + 
 1! 2! n! 
Proof: n letters are denoted by 1, 2, 3, … , n. Let, Ai denote the set of distribution of letters in envelopes (one letter
in each envelope) so that the ith letter is placed in the corresponding envelope. Then, n(Ai) = (n – 1)! [since the
remaining n – 1 letters can be placed in n – 1 envelopes in (n – 1)! Ways] Then, n(Ai ∩ Aj) represents the number of
ways where letters i and j can be placed in their corresponding envelopes. Then N(Ai ∩ Aj) = (n – 2)!
Also, n (Ai ∩ Aj ∩ Ak) = (n – 3)!
5 . 2 6 | Permutations and Combinations

Hence, the required number is n(A’1 ∪ A’2 ∪ … ∪ A’n) = n! – n (Ai ∪ A2 ∪ … ∪ An)

= n! – Σ n(Ai ) – Σ n(Ai ∩ A j ) + Σ n(Ai ∩ A j ∩ Ak ) + ... + (–1) Σ n(A1 ∩ A2 ... ∩ An )


n

= n! – [(nC1)(n – 1)! –nC2 (n–2)! + nC3 (n – 3)! +…+(–1)n–1 × nCn1]

 n! n!   1 1 (–1)n 
= n! –  (n – 1)! (n – 2)!+ ... + (–1)n–1  = n! 1 – + + ... + 
 1!(n – 1) 2!(n – 2)!   1! 2! n! 

Remark: If r objects go to wrong place out of n object then (n – r) objects goes to original place.
1 1 1 1
A’r = r!(1 – + – + … + (–1)r )
1! 2! 3! r!

PLANCESS CONCEPTS
n
(–1)i 1
Number of derangements Dn = n! ∑ ; Interestingly, as n → ∞, Dn =
i=0 i! e
 n! 
This results in an interesting relating Dn =   .Where [x] is the nearest integer function.
e
Use this formula only if the given options are wide apart from one another.
ExampleYou have 6 ball in 6 different colors, and for every ball you have a box of the same color. How
many derangements do you have, if no ball is in a box of the same color?
Sol: We know that e = 2.71828. To make division simple let’s round it to 2.7. You have to keep in mind that
we have reduced the value of e, so the result which we get is greater than the actual result.
 6!   720   800 
∴ Dn =   =   =   = [266.66] = 267
e
   2.7   3 
Hence, the result will be close to 266.
This is a pretty good approximation as the actual answer is 265.
But, if the given options are all close to 266, then it is advised to calculate using the original formula or
by rounding the value of e to the number of significant digits equal to that of n! (numerator).
So if we use e = 2.72 we get 34
 6!   720   4500 
Dn ==
  =    = [264.70] = 265
 e   2.72   17 
 Vaibhav Krishnan (JEE 2009, AIR 22)

Illustration 48: A person writes letters to six friends and addresses the corresponding envelopes. In how many
ways can the letters be placed in the envelopes so that (i) atleast two of them are in the wrong envelopes. (ii) All
the letters are in the wrong envelopes. (JEE MAIN)

Sol: Application of Derangement theorem.


6
(i) The number of ways in which at least two of them in the wrong envelopes = ∑ 6 C6–rDr
r =2
= 6C6–2 D2 + 6C6–3D3 + 6C6–4 D4 + 6C6–5 D5 + 6C6–6 D6

 1 1  1 1 1  1 1 1 1
= 6C4.2!  1 – +  + 6 C 3 .3!  1 – + +  + 6 C2 .4!  1 – + – + 
 1! 2!   1! 2! 3!   1! 2! 3! 4! 
M a them a ti cs | 5.27

 1 1 1 1 1  1 1 1 1 1 1
+6 C1.5! 1– + – + +  + 6 C 0 .6! 1– + – + – + 
 1! 2! 3! 4! 5!   1! 2! 3! 4! 5! 6! 

(ii) The number of ways in which all letters be placed in wrong envelopes
 1 1 1 1 1 1 1 1 1 1 1 
= 6!  1 – + – + – +  ; = 720  – + – +  ; = 360 – 120 + 30 – 6 + 1 = 265
 1! 2! 3! 4! 5! 6!   2 6 24 120 720 

14. MULTINOMIAL THEOREM


(a) If there are l objects of one kind, m objects of a second kind, n objects of a third kind and so on; then the
number of ways of choosing r objects out these (i.e., l + m + n …) is the coefficient of xr in the expansion of
(1 + x + x2 + x3 + … + xl) (1 + x + x2 + x3 + … + xm) (1 + x + x2 + x3 + … + xn)
Further, if one object of each kind is to be included, then the number of ways of choosing r objects out of
these objects (i.e., l + m + n + …) is the coefficient of xr in the expansion of
(x + x2 + x3 + … + xl) (x + x2 + x3 + … + xm) (x + x2 + x3 + … + xn)…
(b) If there are objects of one kind, m objects of a second kind, n objects of a third kind and so on; then the
number of possible arrangements/permutations of r objects out of these object (i.e., l + m + m + …) is the
coefficient of xr in the expansion of

 x x2 x f  x x2 xm  x x2 xn 
r!  1 + + + .....  1 + + + ...  1 + + + ... 
 1! 2! 
l !  1! 2! 
m!  1! 2! n! 

Illustration 49: In an examination, the maximum marks for each of three papers is n and that for the fourth paper
is 2n. Prove that the number of ways in which candidate can get 3n marks is
1
(n + 1) (5n2 + 10n + 6). (JEE ADVANCED)
6

Sol: The maximum marks in the four papers are n, n, n and 2n. Consider a polynomial (1 + x + x2 + … + xn)3 (1 +
x + … + x2n). The number of ways of securing a total of 3n is equal to the co-efficient of the term containing x3n.
The number of ways of getting 3n marks
= coefficient of x3n in (1 + x + x2 + … + xn)3 (1 + x + … + x2n)
= coefficient of x3n in (1 – xn+1)3 (1 –x2n+1) (1 – x)–4
= coefficient of x3n in (1 – 3xn+1 + 3x2n+2 – x3n+3) (1 – x2n+1) × (1 + 4C1 x + 5C2x2 + 6C3 x3 + ….)
= coefficient of x3n in (1 – 3xn+1 – x2n+1 + 3x2n+2) ( 1 + 4C1 x + 5C2 x2 …)
= 3n+3C3n – 3.2n+2C2n–1 + 3 . n+1Cn–2 – n+2Cn–1

(3n + 3)! (2n + 2)! (n – 1)! (n + 2)!


= –3. +3 –
3!(3n)! 3!(2n – 1)! 3!(n – 2)! 3!(n – 1)!

= 1/6 (n + 1) (27n2 + 27n + 6 – 24n2 – 12n + 3n2 – 3n – n2 – 2n) = 1/6 (n + 1) (5n2 + 10n + 6)

PROBLEM-SOLVING TACTICS
In any given problem, first try to understand whether it is a problem of permutations or combinations. Now, think
if repetition is allowed and then try solving problem.
5 . 2 8 | Permutations and Combinations

A simple method to solve these problems where repetition is not allowed is as follows -
First draw series of dashes representing the number of places you want to fill or number of items you want to select.
Now start filling dashes by the number of objects available to choose from and multiply the numbers. This is the
final answer for a permutations problem.
If it is a combination problem then divide the answer with the factorial or number of items.
This calculation becomes complex if repetition is allowed.

FORMULAE SHEET
(a) Permutation (Arrangement of Objects): Each of the different arrangement, which can be made by taking
some or all of a number of objects is called permutation.
n!
(i) The number of permutations of n different objects taken r at a time is nPr = .
(n − r)!
(ii) The number of all permutations of n distinct objects taken all at a time is n!.

Permutation with Repetition: The number of permutations of n different objects taken r at a time when each
object may be repeated any number of times is nr.

Permutation of Alike Objects: The number of permutations of n objects taken all at a time in which, p are
alike objects of one kind, q are alike objects of second kind & r are alike objects of a third kind and the rest
n!
(n – (p + q + r)) are all different, is .
p!q!r!

Permutation under Restriction: The number of permutations of n different objects, taken all at a time, when m
specified objects always come together is m! × (n – m + 1)!.

(b) Combination (Selection of Objects): Each of the different groups or selection which can be made by some
or all of a number of given objects without reference to the order of the objects in each group is called a
combination.
n!
The number of all combinations of n objects, taken r at a time is generally denoted by C(n, r) or nCr =
n r!(n – r)!
Pr
(0 ≤ r ≤ n) =
r!
Note:
(a) The number of ways of selecting r objects out of n objects, is the same as the number of ways in which the
remaining (n - r) can be selected and rejected.
(b) The combination notation also represents the binomial coefficient. That is, the binomial coefficient nCr is the
combination of n elements chosen r at a time.
(c) (i) nCr = nCn–r
(ii) n Cr +n Cr −1 =
n+1
Cr
(iii) nCx = nCy ⇒ x = y or x + y = n
(iv) If n is even, then the greatest value of nCr is nCn/2
n
(v) If n is odd, then the greatest value of nCr is C(n+1)/2
(vi) C0 + Cr +................+ Cn = 2
n n n n

(vii) nCn + n+1Cn + n+2Cn + ……………. + 2n–1Cn = 2nCn+1


M a them a ti cs | 5.29

Combinations under Restrictions


(a) The number of ways of choosing r objects out of n different objects if p particular objects must be excluded
= (n–p)Cr
(b) The number of ways of choosing r objects out of n different objects if p particular objects must be included
(p ≤ r) = n–pCr–p
(c) The total number of combinations of n different objects taken one or more at a time = 2n - 1.

Combinations of Alike Objects


(a) The number of combinations of n identical objects taking (r ≤ n) at a time is 1.
(b) The number of ways of selecting r objects out of n identical objects is n + 1.
(c) If out of (p + q + r + s) objects, p are alike of one kind, q are alike of a second kind, r are alike of the third kind
and s are different, then total number of combinations is (p + 1)(q + 1)(r + 1)2s – 1
(d) The number of ways in which r objects can be selected from a group of n objects of which p are identical, is
t t
∑ n−p C r , if r ≤ p and ∑ n−p C r if r > p
0 r =p

Division into Groups


(a) The number of ways in which (m + n) different objects can be divided into two unequal groups containing m
(m + n)!
and n objects respectively is .
m!n! (2n)!
If m = n, the groups are equal and in this case the number of divisions is ; as it is possible to interchange
the two groups without obtaining a new distribution. n!n!2!

(b) However, if 2n objects are to be divided equally between two persons then the number of ways
(2n)! (2n)!
= 2! =
n!n!2! n!n!
(c) The number of ways in which (m + n + p) different objects can be divided into three unequal groups containing
(m + n + p)
m , n and p objects respectively is = ,m≠n≠p
m!n!p!
(3n)!
If m = n = P then the number of groups = . However, if 3n objects are to be divided equally among
n!n!n!3!
(3n)! (3n)!
three persons then the number of ways = 3! =
n!n!n!3! (n!)3
(d) The number of ways in which mn different objects can be divided equally into m groups if the order of groups
mn!
is not important is
(n!)m m!
(e) The number of ways in which mn different objects can be divided equally into m groups if the order of groups
mn! (mn)!
is important is × m! =
m
(n!) m! (n!)m

Circular Permutation

(a) The number of circular permutations of n different objects taken r at a time


n
Pr/r, when clockwise and anticlockwise orders are treated as different.
n
Pr/2r, when clockwise and anticlockwise orders are treated as same.
(b) The number of circular permutations of n different objects altogether
n
Pn/n = (n – 1)!, when clockwise and anticlockwise order are treated as different
5 . 3 0 | Permutations and Combinations

Pn/2n = 1/2(n – 1)!, when above two orders are treated as same
n

The number of non-negative integral solutions of equation x1 + x2 + … + xr = n


=T
 he number of ways of distributing n identical objects among r persons when each person can get zero or
one or more objects = n+r–1Cr–1
The number of positive integral solutions for the equation x1 + x2 + …. + xr = n
=T
 he number of ways of distributing n identical objects among r persons when each person can get at least
one object = n–r+r–1Cr–1 = n–1Cr–1.
(c) For given n different digits a1, a2, a3 … an the sum of the digits in the units place of all the numbers formed (if
numbers are not repeated) is
(a1 + a2 + a3 + … + an) (n – 1)! i.e. (sum of the digits) (n – 1)!
(d) The sum of the total numbers which can be formed with given different digits a1, a2, a3…… an is
(a + a2 + a3 + … + an) (n –1)! (111. ……….. n times)

Factors of Natural Numbers


Let N = pa . qb . rc … where p, q, r … are distinct primes & a, b, c … are natural numbers, then:
(a) The total number of divisors of N including 1 and N are = (a + 1) (b + 1) (c + 1) …
(b) The sum of these divisors is = (p0 + p1 + p2 +…+ pa) (q0 + q1 + q2 +…+ qb) (r0 + r1 + r2 +…+ rc)…
(c) The number of ways in which N can be resolved as a product of two factors is
 1
 (a + 1)(b + 1)(c + 1) if N is not a perfect square
= 2

 1 (a + 1)(b + 1)(c + 1)... + 1 if N is a perfect square
 2  

(d) The number of ways in which a composite number N can be resolved into two factors which are relatively
prime (or coprime) to each other is equal to 2n–1 where n is the number of different prime factors in N
∞ n
Exponent of a Prime P in N! = ∑  pi 
i=1  

Inclusion-Exclusion Principle: The principle of inclusion-exclusion states that for finite sets A1,…An. One has the
identity
n n
Ai ∑ Ai
= − ∑ | Ai ∩ Aj | + ∑ | Ai ∩ Aj ∩ Ak | – ... + (–1)n–1 | A1 ∩ ... ∩ An | .
i=1=i 1 1≤i< j≤n 1≤i< j<k ≤n

n n  
=
This can be compactly written as  Ai ∑ (–1)k +1  ∑ | Ai ∩ ... ∩ Ai | 
1 k 
i==
1 k 1  1≤i1 <........<ik ≤n 

Derangements Theorem: The number of ways in which letters n can be placed in n envelopes (one letter in each
 1 1 (–1)n 
envelope) so that no letter is placed in the correct envelope is n! 1 – + + ... + 
 1! 2! n! 

If n objects are arranged at n places then the number of ways to rearrange exactly r objects at right places is =

n!  1 1 1 1 n–r 1 
1 – + – + – ... + (–1) 
r  1! 2! 3! 4! (n – r)! 
M a them a ti cs | 5.31

Some Important results


(a) The number of totally different straight lines formed by joining n points on a plane of which m(<n) are
collinear is nC2 – mC2 + 1.
(b) The number of total triangles formed by joining n points on a plane of which m(<n) are collinear is nC3 – mC3.
(c) The number of diagonals in a polygon of n sides is nC2 – n.
(d) If m parallel lines in a plane are intersected by a family of other n parallel lines. Then total number of
parallelograms so formed are mC2 × nC2.
(e) Given n points on the circumference of a circle, then
the number of straight lines between these points are nC2
the number of triangles between these points are nC3
the number of quadrilaterals between these points are nC4
(f) If n straight lines are drawn in the plane such that no two lines are parallel and no three lines are concurrent.
Then, the number of parts into which these lines divide the plane is = 1 + Sn

Solved Examples

JEE Main/Boards Each of the above pairs can be associated with a third
station in (n – 2) ways. Thus, choosing a pair of stations
Example 1: Find the number of ways in which 5 identical and any third station can be done in (n – 1) (n – 2) ways.
balls can be distributed among 10 different boxes, if The above count also includes the case of three
exactly one ball goes into a box. consecutive stations. However, we can see that each
such case has been counted twice. For example, the pair
Sol: It is same as selecting 5 boxes from 10 boxes and S4S5 combined with S6 and the pair S5S6 combined with
distributing the balls in those 5 boxes. S4 are identical.
Number of boxes = 10 and Number of balls = 5. Hence, subtracting the excess counting, the number of
∴ Possible number of ways = C5 10 ways in which three stations can be chosen so that at
least two of them are consecutive
Example 2: There are n intermediate stations on a = (n – 1) (n – 2) – (n – 2) = (n – 2)2
railway line from one terminal to another. In how many
ways can the train stop at 3 of these intermediate Example 3: How many ways are there to invite 1 of 3
stations if friends for dinner on 6 successive nights such that no
friend is invited more than 3 times?
(i) all the three stations are consecutive.
(ii) at least two of the stations are consecutive. Sol: Divide the solution in different possible cases. 6 can
be partitioned in the following ways
Sol: The first part is very trivial. For the second part
1+2+3
consider a pair of consecutive stations and then select
a station such that it is not consecutive. Check for 0+3+3
multiple counting. 2+2+2
Let the intermediate stations be S1, S2,……, Sn Using this we can form different possibilities and
(i) The number of triplets of consecutive stations, as calculate the number of ways the friends can be invited.
S1S2S3, S2S3S4, S3S4S5, …. Sn–2Sn–1Sn, is (n – 2). Let x, y, z be the friends and let (a, b, c) denote the case
(ii) The total number of consecutive pairs of stations, as where x is invited a times, y, b times and z, c times. For
S1S2, S2S3,…………, Sn–1Sn is (n – 1). example, one possible arrangement corresponding to
5 . 3 2 | Permutations and Combinations

the triplet (3, 2, 1) is x, x, y, x, y, z 2 alike of one kind and 3 alike of different kind and so
on. Count the number of words in these cases and their
Then we have the following possibilities:
sum gives us the answer.
(i) (a, b, c) = (1, 2, 3); (1, 3, 2); (2, 3, 1); (2, 1, 3):
Proposition contains 11 letters PP, R, OOO, S, II, T, N.
(ii) (a, b, c) = (3, 3,0); (3,0,3) ; (0, 3, 3).
Following table given the number of words.
(iii) (a, b, c) = (2, 2, 2). So the total number of ways is 6 ×
Repeated letters: O(2), P(2), I(2)
6!/1! 2! 3! + 3 × 6!/3!3! + 6!/2!2!2!
Different letters: R, S, T, N
Note: We can also solve this problem using linear
equations. Letters No. of Words Total
A 5 Distinct 7
C5.5! 2520
Example 4: There are 2n guests at a dinner party.
Supposing that the master and mistress of the B 3 Alike 1.2C1.(51/3!2!) 20
house have fixed seats opposite one another. And
2 Alike
that there are two specified guests who must not be
placed next to one another. Show that the number C 3 Alike 1.6C2.(5!/3!) 300
of ways in which the company can be placed is 2 Different
(2n – 2)! (4n2 – 6n + 4)
D 2 Alike 3
C2.5C1.(5!/2!2!) 450
Sol: This is an application of division into groups. Find 2Other Alike
the total number of ways of arrangement of the guests
and then subtract the number of ways in which the two 1 Different
mentioned guests are together. E 2 Alike 3
C1.6C3.(5!/2!) 3600
Excluding the two specified guests, 2n – 2 persons can be
3 Different
divided into two groups, one containing n and the other
(2n – 2)! Total no. of words = 6890
(n – 2) in and can sit on either side of mister
n!(n – 2)!
and mistress in 2! ways and can arrange themselves in Example 6: There are 10 points in a plane where no
n!(n – 2)! three points are collinear except for 4 points which are
M collinear. Find the number of triangles formed by the
 n-2 points as vertices.
4
n G1 G2 Sol: A triangle is formed from three non-collinear
3 points. Select 3 points from 10 points in 10C3 ways and
m  2
 subtract the cases when the points are collinear, as they
1 would not form a triangle.
Now, the two specified guests where (n – 2) guests are Let us suppose that the 10 points are such that no three
seated will have (n – 1) gaps and can arrange themselves of them are collinear. Now, a triangle will be formed by
in 2! Ways. Number of ways when G1 G2 will always be any three of these ten points. Thus forming a triangle
together amount to selecting any three of the 10 points.
(2n – 2)! Now 3 points can be selected out of 10 point in 10
C3
= 2! n! (n – 2) ! (n – 1) 2! = (2n – 2)! 4(n – 1)
n!(n – 2)! ways.
Hence, the number of ways when G1 G2 are never ∴ Number of triangles formed by 10 points when no
together three of them are collinear = 10C3.
2!
= 2! n! n! – 4(n – 1) (2n – 2)! Similarly, the number of triangles formed by 4 points
n! n! 2!
then no 3 of them are collinear = 4C3
= (2n – 2)! [2n(2n – 1) – 4 (n – 1)] = (2n – 2)! [4n2 – 6n + 4]
∴ Required number of triangle formed = 10
C 3 – 4C 3 =
120 – 4 = 116.
Example 5: Find the number of words of 5 letters that
can be formed with the letters of the word Proposition. Example 7: From 6 gentlemen and 4 ladies, a committee
of 5 is to be formed. In how many ways can this be done
Sol: Divide the cases into words having 5 distinct letters, if the committee is to include at least one lady?
M a them a ti cs | 5.33

Sol: According to the question, the committee should (b)


include atleast one lady. Consider cases when the
committee consists of 1, 2, 3 or 4 ladies and find the
number of ways for all these cases. 5 5 5 5
Each place can be filled by any of the 5 numbers.
Different combinations are listed below: Therefore, the total number of arrangements is 54.

No. of Ladies No. of Gentlemen No. of Committees (c) Each card can be given to any of the 3 servants.

1 4 4
C 1 6C 4 ∴ No. of ways = 3 × 3 × 3 × 3 × 3 × 3 = 36 = 729.

2 3 4
C 2 6C 3 (d) There are ten letters in the word BENEVOLENT of
which three are E and two are N, and the rest five are
3 2 4
C 3 6C 2 different.
4 1 4
C 4 6C 1 10!
∴ Total number of arrangements =
3! 2!
Total number of committees
Example 9: n1 and n2 are five-digit numbers. Find the
= 4C1 6C4 + 4C2 6C3 + 4C3 6C2 + 4C4 6C1 = 246 total number of ways of forming n1 and n2, so that n2 can
be subtracted from n1 without borrowing at any stage.
Example 8: (a) In how many ways can the following
diagram be coloured, subject to two conditions: Each Sol: Two numbers can be subtracted without borrowing
of the smaller triangle is to be painted with one of three if all the digits in n1 is greater than all the corresponding
colours: red, blue, green and no two adjacent regions digits in the number n2. Using this information, find the
should have the same color? number of ways for different possible cases and add
them up to get the answer.
(b)How many numbers of four digits can be formed
with the digits 1, 2, 3, 4 and 5? Let n1 = x1 x2 x3 x4 x5 and n2 = y1 y2 y3 y4 y5 be two numbers.
n1 and n2 can be subtracted without borrowing at any
stage if xi ≥ yi.
Here, xi and yi denotes the digits at various places in the
number n1 and n2 respectively.

Value of x5 Value of ys
9 0,1,2,…9
8 0,1,2,…8
7 0,1,2,…7
(c) A gentleman has 6 friends to invite. In how many
ways can he send invitation cards to them if he has 3 6 0,1,2,3,4,5,6
servants to carry the cards? 5 0,1,2,3,4,5
(d) Find the number of arrangements of the letters of 4 0,1,2,3,4
the word ‘BENEVOLENT’.
3 0,1,2,3
Sol: For each of the parts (a), (b), (c) and (d), identify 2 0,1
the number of ways a particular cell can be coloured or
filled in and then use permutation / combination to get 1 0
the result. 0
(a) These conditions are satisfied if we proceed as Thus, x5 and y5 can be selected collectively by 10 + 9 +
follows: Just color the central triangle by one color, 8 + … 1 = 55 ways. Similarly, each pair (x4, y4), (x3, y3), (x2,
this can be done in three ways. Next paint other three y2) can be selected in 55 ways. But, pair (x1, y1) can be
triangles with remaining 2 colors. By the fundamental selected in 1 + 2 + 3 + … + 9 = 45 ways as in this pair
principle of counting. This can be done in 3 × 2 × 2 × 2 we cannot have 0.
= 24 ways.
Therefore total number of ways = 45(55)4.
5 . 3 4 | Permutations and Combinations

Example 10: Prove that the product of r consecutive Sol: List down different ways in which we get the sum of
positive integers is divisible by r!. 5 and 4 and get the answer.
Event E (the first throw resulting in 5) can happen in one
Sol: Simple application of the definition of nPr.
of four ways as:
Let P be the product of r consecutive positive integers
3 + 2; 4 + 1; 2 + 3; 1 + 4.
ending with n; then
Event F (the second throw resulting in 4) can happen in
P = n(n – 1) … (n – r + 1)
one of three ways as:
P n(n – 1)...(n – r + 1)
= 2 + 2; 1 + 3; 3 + 1.
r! r!
The two events can together happen in 4 × 3 = 12 ways.
[n(n – 1)(n – 2)...(n – r + 1)][(n – r)...3.2.1]
r!(n – r)...3.2.1
n! Example 3: An eight-oared boat is to be manned by
= = nCr = an integer a crew chosen from 11 men of whom 3 can steer but
r! n – r!
cannot row and the rest cannot steer. In how many ways
∴ P is divisible by r!. can the crew be arranged if two of the men can only row
in bow side?

JEE Advanced/Boards
Bow side

Example 1: How many numbers of n digits can be made


with the non-zero digits in which no two consecutive
digits are the same?
Stroke side
Sol: Using Permutation under Restriction we can easily
find the answer. Sol: Find the number of ways we can select for steering,
There are nine non-zero digits, namely 1, 2, 3, … and 9. rowing and arranging the remaining men. Their product
gives us the required result.
The total number of men = 11
  9   8 8 8 8
The number of men who can only steer = 3
In order the make an n-digit number we have to fill n
places by using the nine digits. As no two consecutive The number of other men = 8
digits are to be the same, a digit used in a place cannot The number of ways of selecting one man for steering
be used in the next place but it can be used again in the out of 3 = 3C1.
place coming after the next place.
The number of ways in which the two particular men
So, the first place can be filled in 9 ways; who only row on bow side
the second place can be filled in 8 ways (rejecting the Can be arranged on bow side = 4P2
digit used in the first place)
The number of ways in which remaining 6 men can be
the third place can be filled in 7 + 1, i.e., 8 ways (rejecting arranged in remaining 6 places = 6!
the digit used in the second place but including the
digit used in the first place) and so on. ∴ The required number = 3C1. 4P2 . 6!

∴ The total number of desired numbers = 3 × 4 × 3 ×6! = 25920

= 9 × 8 × 8 × 8 × … to n factors = 9 × 8n–1.
Example 4: The members of a chess club took part in
a round robin competition in which each plays every
Example 2: A dice is a six-faced cube, with the faces one else once. All members scored the same number of
reading 1, 2, 3, 4, 5 and 6. When two dice are thrown, points, except four juniors whose total score were 17.5.
we add the digits they show on top and take that sum as How many members were there in the club? (Assume
the result of the throw. In how many different ways the
that for each win a player scores 1 point, for draw 1
first throw of the 2 dice shows a total of 5, and second 2
point and zero for losing.)
throw of the 2 dice shows a total of 4?
M a them a ti cs | 5.35

Sol: Form an equation of the total number of points As the man wants to travel by one of the many possible
scored using the given information. Solve the equation shortest paths, he will never turn to the right or turn
to find the answer. downward. So a travel by one of the shortest paths is to
take 4 horizontal pieces and 4 vertical pieces of roads.
Let the number of members be n.
∴ A shortest path is an arrangement of eight objects
Total number of point = nC2.
L1, L2, L3, L4, U1, U2, U3, U4 so that the order of L’s and U’s
∴ nC2 – 17.5 = (n – 4) x (where x is the number of point do not change.
scored by each player)
( Clearly L2 cannot be taken without taking L1, U2
n (n – 1) – 35 = 2 (n – 4)x cannot be taken without taking U1, etc.)
n(n – 1) – 35 Hence, the number of shortest paths
2x = (where x takes the values 0.5, 1, 1.5 etc.)
n– 4
= The number of arrangements of L1, L2, L3, L4, U1, U2,
n2 – n – 35 U3, U4 where the order of Ls as well as the order of Us
= (must be an integer)
n– 4 do not change
n(n – 4) + 3(n – 4) – 23 23 = The number of arrangement treating Ls as identical
= = (n + 3) –
n– 4 n– 4 and Us as identical
23
⇒ must be an integer 8! 8.7.6.5
n– 4 = = = 2.7.5 = 70.
4! 4! 24
⇒ n = 27 is the only possibility.
Example 7: A condolence meeting being held in a hall
Example 5: If p, q, r, s, t are prime numbers. Find the which has 7 doors, by which mourners enter the hall.
number of ways in which the product, pq2r3st can be One can use any of the 7 doors to enter and can come
expressed as product of two factors, excluding 1 as a at any time during the meeting. At each door, a register
factor. is kept in which mourner has to affix his signature while
entering the hall. If 200 people attend the meeting,
Sol: Use the standard result to find the answer. how many different sequences of 7 lists of signatures
Total factors = 2 × 3 × 4 × 2 × 2 = 96 can arise?
96
Hence, the total ways = = 48. but this includes 1
2 Sol: Clearly, the total number of people is 200, hence the
and the number itself also. Hence, the required number sum of the entries is 200. Apply Multinomial theorem
of ways = 48 – 1 = 47 to find the total number of ways list can be made and
hence the answer.
Example 6: In the given figure you have the road plan There are 7 lists, say 1, 2,…. 7. Suppose, that list i has xi
of a city. A man standing at X wants to reach the cinema names; then,
hall at Y by the shortest path. What is the number of
different paths that he can take? x1 + ….. + x7 = 200 where xi ≥ 0 is an integer.

Y We need to first find the number of solutions of this


equation.
U4
(Note that this does not complete the solution to
U3 the questions as list 1 may contain 7 names which
would remain the same in 7!, arrangements of the
U2 names)
The number of solutions are = 200+7–1C7–1 = 206C6
U1
X But corresponding to any one solution (x1…x7) (i.e. list
L4 L4 L2 L1
f contains xf names) we can have 200! arrangements
Sol: If the man moves only in the upward and the consistent with distribution of xj names to jth list
leftward direction, then the path will be the shortest. ∴ The number of different sequences of 7 lists
Use this idea to calculate total number of shortest 206
paths. = 206C6 × 200! = .
6!
5 . 3 6 | Permutations and Combinations

JEE Main/Boards

Exercise 1 (ii) How many characters can be transmitted by using


(a) exactly five symbols? (b) at most five symbols?
Q.1 How many odd numbers less than 1000 can be
Q.11 In how many of the distinct permutation of the
formed using the digits 0, 1, 4 and 7 if repetition of
letter in MISSISSIPPI do the four I’s not come together?
digits is allowed?

Q.12 In how many ways 4 boys and 3 girls can be seated


Q.2 In how many ways can five people be seated in
in a row so that they are alternate?
a car with two people in the front seat and three in
the rear, if two particular persons out of the five cannot
drive? Q.13 A biologist studying the genetic code is interested
to know the number of possible arrangements of 12
molecules in a chain. The chain contains 4 different
Q.3 A team consisting of 7 boys and 3 girls play singles
molecules represented by the initials A (for adenine),
matches against another team consisting of 5 boys and
C (for Cytosine), G(for Guanine) and T (for Thymine)
5 girls. How many matches can be scheduled between
and 3 molecules of each kind. How many different such
the two teams if a boy plays against a girl and a girl
arrangements are possible in all?
plays against a boy?

(2n + 1) Q.14 Find the number of rearrangement of the letters of


Q.4 Prove that = 2n[1.3.5…(2n – 1)(2n + 1)]
n! the word ‘BENEVOLENT’. How many of them end in L?
Q.5 If nP4 =360, find n.
Q.15 How many words can be formed with the letters
of the word PATALIPUTRA’ without changing the relative
Q.6 Find the number of numbers between 300 and
order of the vowels and consonants?
3000 which can be formed with the digits 0, 1, 2, 3, 4
and 5, with no digit being repeated in any number.
Q.16 A person is to walk from A B
to B. However, he is restricted to
Q.7 How many even numbers are there with three digits
walk only to the right of A or
such that if 5 is one of the digits in a number then 7 is
upwards of A, but not necessarily
the next digit in that number?
in this order. One such path is
shown in the given figure
Q.8 Find the sum of 3 digit numbers formed by digits Determine the total number of
1, 2, 3 is paths available to the person A
from A to B.
Q.9 A telegraph has 5 arms and each arm is capable of
4 distinct positions, including the position of rest. What Q.17 In how many ways can three jobs I, II and III be
is the total number of signals that can be made? assigned to three persons A, B and C, if one person
is assigned only one job and all are capable of doing
Q.10 In telegraph communication, the Morse code is each job? Which assignment of jobs will take the least
used in which all the letters of the English alphabet, time to complete the jobs, if time taken (in hours) by an
digits 0 to 9 and even the punctuation marks, all usually individual on each job as follows?
referred as characters, are represented by ‘dots’ and
‘dashes’ Job persons I II III

For example, E is represented by a dot (.), T by a dash A 5 4 4


(–), O by three dashes (- - -), S by three dots (. . .) and so
1 1
on. Thus, SOS is represented by (. . . – – – . . .). B 4 3 4
4 2
(i) How many characters can be transmitted using one
symbol (dot or dash), two symbols, three symbols, C 5 3 5
four symbols? Also find the total number of characters
which can be transmitted using at most four symbols.
M a them a ti cs | 5.37

Q.18 If 15C3r = 15Cr+3’ find r. Exercise 2


n r n n− s
Q.19 Prove that Cr × Cs = Cs × Cr −s . Single Correct Choice Type

Q.20Find the value of the expression Q.1 If the letters of the word “VARUN” are written in all
5 possible ways and then are arranged as in a dictionary,
47
C4 + ∑ 52– j C3 . then rank of the word VARUN is:
j=1
(A) 98 (B) 99 (C) 100 (D) 101
Q.21 Prove that the product of r consecutive integers
is divisible by r!. Q.2 Number of natural numbers between 100 and 1000
such that at least one of their digits is 7, is
Q.22 From a class of 25 students, 10 are to be chosen
(A) 225 (B) 243 (C) 252 (D) none
for a field trip. There are 3 students who decide that
either all of them will join or none of them will join. In
Q.3 The 120 permutations of MAHES are arranged in
how many ways can the field trip members be chosen?
dictionary order, as if each were an ordinary five-letter
word. The last letter of the 86th word in the list is
Q.23 There are ten points in a plane. Of these ten
points, four points are in a straight line and with the (A) A (B) H (C) S (D) E
exception of these four points, no three points are in
the same straight line. Find- Q.4 A new flag is to be designed with six vertical strips
using some or all of the colors yellow green, blue and
(i) The number of triangles formed.
red. Then the number of ways this can be done such
(ii) The number of straight lines formed that no two adjacent strips have the same color is
(iii) The number of quadrilaterals formed, by joining (A) 12 × 81 (B) 16 × 192 (C) 20 × 125 (D) 24 × 216
these ten points.
Q.5 The number of 10-digit numbers such that the
Q.24 In an examination a minimum of is to be secured product of any two consecutive digits in the number is
in each of 5 subjects for a pass. In how many ways can a prime number, is
a student fail?
(A) 1024 (B) 2048 (C) 512 (D) 64
Q.25 In how many ways 50 different objects can be
Q.6 Consider the five points comprising of the vertices
divided in 5 sets three of them having 12 objects each
of a square and the intersection point of its diagonals.
and two of them having 7 objects each.
How many triangles can be formed using these points?
Q.26 Six “X”s (crosses) have to be placed in the squares (A) 4 (B) 6 (C) 8 (D) 10
of the figure given below, such that each row contains
at least one X. In how many different ways can this be Q.7 How many of the 900 three digit numbers have at
done? least one even digit?
(A) 775 (B) 875 (C) 100 (D) 101

Q.8 A 5 digit number divisible by 3 is to be formed


using the numbers 0, 1, 2, 3, 4 & 5 without repetition.
The total number of ways in which this can be done is:
Q.27 Five balls of different colors are to be placed in (A) 3125 (B) 600 (C) 240 (D) 216
three boxes of different sizes. Each box can hold all five
balls. In how many different ways can we place the balls Q.9 The number of different seven digit numbers that
so that no box remains empty? can be written using only three digits 1, 2 & 3 under the
condition that the digit 2 occurs exactly twice in each
Q.28 How many different words of 4 letters can be number is
formed with the letters of the word “EXAMINATION”?
(A) 672 (B) 640 (C) 512 (D) none
5 . 3 8 | Permutations and Combinations

Q.10 Out of seven consonants and four vowels, the Q.20 A students have to answer 10 out of 13 questions
number of words of six letters, formed by taking four in an examination. The number of ways in which he
consonants and two vowels is (Assume that each can answer if he must answer at least 3 of the first five
ordered group of letter is a word): questions is
(A) 210 (B) 462 (C) 151200 (D) 332640 (A) 276 (B) 267 (C) 80 (D) 1200

Q.11 All possible three digits even numbers which can Q.21 The number of three digit numbers having only
be formed with the condition that if 5 is one of the two consecutive digits identical is:
digit, then 7 is the next digit is:
(A) 153 (B) 162 (C) 180 (D) 161
(A) 5 (B) 325 (C) 345 (D) 365
Q.22 The interior angles of a regular polygon measure
Q.12 Number of 5 digit numbers which are divisible by 150º each. The number of diagonals of the polygon is
5 and each number containing the digit 5, digits being
(A) 35 (B) 44 (C) 54 (D) 78
all different is equal to k(4!), the value of k is
(A) 84 (B) 168 (C) 188 (D) 208 Q.23 The number of n digit numbers which consists of
the digits 1 & 2 only if each digit is to be used at least
Q.13 The number of six digit numbers that can be once, is equal to 510 then n is equal to:
formed from the digits 1, 2, 3, 4, 5, 6, & 7 so that digits
(A) 7 (B) 8 (C) 9 (D) 10
do not repeat and the terminal digit are even is:
(A) 144 (B) 72 (C) 288 (D) 720 Q.24 Number of four digit numbers with all digits
different and containing the digit 7 is
Q.14 The number of natural numbers from 1000 to
(A) 2016 (B) 1828 (C) 1848 (D) 1884
9999 (both inclusive) that do not have all 4 different
digits is
Q.25 An English school and a Vernacular school are
(A) 4048 (B) 4464 (C) 4518 (D) 4536 both under one superintendent. Suppose that the
superintendentship, the four teachership of English
Q.15 Number of positive integers which have no two and Vernacular school each, are vacant, if there be
digits having the same value with sum of their digits altogether 11 candidates for the appointments, 3 of
being 45, is whom apply exclusively for the superintendentship and
(A) 10! (B) 9! (C) 9.9! (D) 17.8! 2 exclusively for the appointment in the English school,
the number of ways in which the different appointment
Q.16 Number of 3 digit number in which the digit at can be disposed of is :
hundredth’s place is greater than the other two digit is (A) 4320 (B) 268 (C) 1080 (D) 25920
(A) 285 (B) 281 (C) 240 (D) 204
Q.26 A committee of 5 is to be chosen from a group of
Q.17 Number of permutation of 1, 2, 3, 4, 5, 6, 7, 8 and 9 people. Number of ways in which it can be formed if
9 taken all at a time, such that the digit 1 appearing two particular persons either serve together or not at
somewhere to the left of 2, 3 appearing to somewhere all and two other particular persons refuse to serve with
the left of 4 and 5 somewhere to the left of 6, is (e.g. each other, is
815723946 would be one such permutation) (A) 41 (B) 36 (C) 47 (D) 76
(A) 9.7! (B) 8! (C) 5!.4! (D) 8!.4!
Q.27 A question paper on mathematics consists of
Q.18 Number of odd integers between 1000 and 8000 twelve questions divided into three parts A, B and C,
which have none of their digit repeated, is each containing four questions, in how many ways can
an examinee answer five questions, selecting at least
(A) 1014 (B) 810 (C) 690 (D) 1736
one from each part?

Q.19 The number of ways in which 5 different books (A) 624 (B) 208 (C) 1248 (D) 2304
can be distributed among 10 people if each person can
get at most one book is:
(A) 252 (B) 105 (C) 510 (D) 10C5.5!
M a them a ti cs | 5.39

Q.28 Number of ways in which 7 green bottles and 8 Q.36 Number of different words that can be formed
blue bottles can be arranged in a row if exactly 1 pair of using all the letters of the word “DEEPMALA” if two
green bottles is side by side, is (Assume all bottles to be vowels are together and the other two are also together
alike except for the color). but separated from the first two is
(A) 84 (B) 360 (C) 504 (D) None (A) 960 (B) 1200 (C) 2160 (D) 1440

Q.29 The kindergarden teacher has 25 kids in her class. Q.37 In a unique hockey series between India &
She takes 5 of them at a time, to zoological garden as Pakistan, they decide to play on till a team wins 5
often as she can, without taking the same 5 kids more matches. The number of ways in which the series can
than once. Then the number of visits, the teacher makes be won by India, if no match ends in a draw is:
to the garden exceeds that of a kid by: (A) 126 (B) 252 (C) 225 (D) None
(A) C5 – C5 (B) C5
25 24 24
(C) C4
24
(D) None
Q.38 Sameer has to make a telephone call to his friend
Harish, Unfortunately he does not remember the 7 digit
Q.30 A rack has 5 different pairs of shoes. The number phone number. But he remembers that the first three
of ways in which 4 shoes can be chosen from it so that digits are 635 or 674, the number is odd and there is
there will be no complete pair is: exactly one 9 in the number. The maximum number of
(A) 1920 (B) 200 (C) 110 (D) 80 trials that Sameer has to make to be successful is
(A) 10,000 (B) 3402 (C) 3200 (D) 5000
Q.31 Number of ways in which 9 different toys can be
distributed among 4 children belonging to different Q.39 There are 12 guests at a dinner party. Supposing
age groups in such a way that distribution among the that the master and mistress of the house have fixed
3 elder children is even and the youngest one is to seats opposite one another, and that there are two
receive one toy more, is: specified guests who must always, be placed next to
(5!)
2
9! 9! one another; the number of ways in which the company
(A) (B) (C) (D) None
8 2 3!(2!)3 can be placed is :
(A) 20 . 10 ! (B) 22 . 10 ! (C) 44 . 10 ! (D) None
Q.32 There are 10 red balls of different shades & 9
green balls of identical shades. Then the number of Q.40 In a conference 10 speakers are present. If S1
such arrangements such that no two green balls are wants to speak before S2 and S2 wants to speak after
together in the row is: S3, then the number of ways all the 10 speakers can
(A) (10!).11P9 (B) (10!).11C9 (C) 10! (D) 10! 9! give their speeches with the above restriction if the
remaining seven speakers have no objection to speak
Q.33 A shelf contains 20 different books of which 4 are at any number is
10!
in single volume and the others form sets of 8, 5 and (A) 10C3 (B) 10P8 (C) 10P3 (D)
3
3 volumes respectively. Number of ways in which the
books may be arranged on the shelf, if the volumes of
Q.41 The number of all possible selection of one or
each set are together and in their due order is
more questions from 10 given questions, each question
20! having an alternative is:
(A) (B) 7! (C) 8! (D) 7.8!
8!5!3! (A) 310 (B) 210–1 (C) 310 – 1 (D) 210

Q.34 Number of ways in which 3 men and their wives


Q.42 Number of 7 digit numbers the sum of whose
can be arranged in a line such that none of the 3 men
digits is 61 is:
stand in a position that is ahead of his wife, is
(A) 12 (B) 24 (C) 28 (D) None
6!
(A) 3!.3! (B) 2.3!.3! (C) 3! (D)
2!2!2! Q.43 There are 2 identical white balls, 3 identical red
balls and 4 green balls of different shades. The number
Q.35 The number of different ways in which five ‘dashes’
of ways in which they can be arranged in a row so that
and eight ‘dots’ can be arranged, using only seven of
at least one ball is separated from the balls of the same
these 13 ‘dashes’ & ‘dots’ is
color, is:
(A) 1287 (B) 119 (C) 120 (D) 1235520 (A) 6 (7! – 4!) (B) 7 (6! – 4!) (C) 8 ! – 5! (D) None
5 . 4 0 | Permutations and Combinations

Q.44 Product of all the even divisors of N = 1000, is Q.52 The number 916238457 is an example of nine
digit number which contains each of the digit 1 to 9
(A) 32 . 102 (B) 64 . 214 (C) 64 . 1018 (D) 128 . 106
exactly once. It also has the property that the digits 1
to 5 occur in their natural order, while the digits 1 to 6
Q.45 A lift with 7 people stops at 10 floors. People
do not. Number of such numbers are
varying from zero to seven go out at each floor. The
number of ways in which the lift can get emptied, (A) 2268 (B) 2520 (C) 2975 (D) 1560
assuming each way only differs by the number of
people leaving at each floor, is Q.53 Number of functions defined from f : {1, 2, 3, 4, 5,
6} → {7, 8, 9, 10} such that the sum f(1) + f(2) + f(3) +
(A) 16C6 (B) 17C7 (C) 16C7 (D) None
f(4) +f(5) + f(6) is odd, is
Q.46 You are given an unlimited supply of each of the (A) 210 (B) 211 (C) 212 (D) 212 – 1
digits 1, 2, 3 or 4. Using only these four digits, you
construct n digit numbers. Such n digit numbers will Multiple Correct Choice Type
be called LEGITIMATE if it contains the digit 1 either
an even number times or not at all. Number of n digit Q.54 The continued product, 2.6.10.14… to n factors is
legitimate numbers are equal to:
(A) 2n + 1 (B) 2n+1 + 2 (C) 2n+2 + 4 (D) 2n–1(2n + 1) (A) 2nCn (B) 2nPn

Q.47 Distinct 3 digit numbers are formed using only the (C) 2n+1Cn (D) None
digits 1, 2, 3 and 4 with each digit used at most once
in each number thus formed. The sum of all possible Q.55 The maximum number of permutations of 2n
numbers so formed is letters in which there are only a’s & b’s, taken all at a
time is given by :
(A) 6660 (B) 3330 (C) 2220 (D) None
(A) 2nCn
Q.48 An ice cream parlor has ice creams in eight 2 6 10 4n – 6 4n – 2
(B) . . … .
different varieties. Number of ways of choosing 3 ice 1 2 3 n–1 n
creams taking at least two ice creams of the same
n + 1 n + 2 n + 3 n + 4 2n – 1 2n
variety, is (Assume that ice creams of the same variety (C) . . . … .
1 2 3 4 n–1 n
to be identical & available in unlimited supply)
2n 1.3.5...(2n – 3)(2n – 1)
(A) 56 (B) 64 (C) 100 (D) None (D)
n!
Q.49 There are 12 books on Algebra and Calculus (E) All of the above
in our library, the books of the same subject being
different. If the number of selection each of which
consists of 3 books on each topic is greatest then the Q.56 Number of ways in which 3 numbers in A.P. can be
number of books of Algebra and Calculus in the library selected from 1, 2, 3,… n is :
are respectively: 2
n–1 n(n – 2)
(A)   if n is even (B) if n is odd
(A) 3 and 9 (B) 4 and 8 (C) 5 and 7 (D) 6 and 6  2  4

Q.50 A person writes letters to his 5 friends and addresses (n – 1) n(n – 2)


the corresponding envelopes. Number of ways in which (C) if n is odd (D) if n is even
4 4
the letters can be placed in the envelope, so that at least
two of them are in the wrong envelopes, is,
(A) 1 (B) 2 (C) 118 (D) 119 Previous Years’ Questions
5
Q.51 For a game in which two partners oppose two Q.1 The value of the expression 47
C4 + ∑ 52– j C3 is
j=1
other partners, 8 men are available. If every possible equal to (1982)
pair must play with every other pair, the number of
games played is (A) 47C5 (B) 52C5

(A) 8C2 . 6C2 (B) 8C2 . 6C2 . 2 (C) 8C4 . 3 (D) None (C) 52C4 (D) None of these
M a them a ti cs | 5.41

Q.2 Eight chairs are numbered 1 to 8. Two women and Q.5 If r, s, t are prime numbers and p, q are the positive
three men wish to occupy one chair each. First the integers such that LCM of p, q is r2s4t2, then the number
women choose the chairs from amongst the chairs of ordered pairs (p, q) is  (2006)
marked 1 to 4, and then the men select the chairs
(A) 252 (B) 254 (C) 225 (D) 224
from amongst the remaining. The number of possible
arrangements is  (1982)
Q.6 The letters of the word COCHIN are permuted and
(A) 6C3 × 4C2 (B) 4P2 × 4P3 all the permutations are arranged in an alphabetical
(C) 4C2 + 4P3 (D) None order as in an English dictionary. The number of words
that appear before the word COCHIN is (2007)
Q.3 A five digits number divisible by 3 is to be formed (A) 360 (B) 192 (C) 96 (D) 48
using the numbers 0, 1, 2, 3, 4 and 5, without repetition.
The total number of ways this can be done, is  (1989) Q.7 The number of seven digit integers, with sum of the
(A) 216 (B) 240 (C) 600 (D) 3125 digits equal to 10 and formed by using the digits 1, 2
and 3 only, is  (2009)
Q.4 Number of divisors of the form (4n + 2), n ≥ 0 of (A) 55 (B) 66 (C) 77 (D) 88
integer 240 is  (1998)
(A) 4 (B) 8 (C) 10 (D) 3

JEE Advanced/Boards

Exercise 1
Q.6 In an election for the managing committee of a
Q.1 Consider all the six digit numbers that can be reputed club, the number of candidates contesting
formed using the digits 1, 2, 3, 4, 5 and 6, each digit elections exceeds the number of members to be
being used exactly once. Each of such six digit numbers elected by r(r > 0). If a voter can vote in 967 different
have the property that for each digit, not more than ways to elect managing committee by voting at least
two digits, smaller than that digit, appear to the right 1 of them & can vote in 55 different ways to elect (r –
of that digit. Find the number of such six digit numbers 1) candidates by voting in the same manner. Find the
having the desired property number of candidates contesting the election & the
number of candidates losing the elections.
Q.2 Find the number of five digit number that can be
formed using the digits 1, 2, 3, 4, 5, 5, 7, 9 in which Paragraph for question nos. 7 to 9:
one digit appears once and two digits appear twice (e.g 2 American men; 2 British men; 2 Chinese men and one
41174 is one such number but 75355 is not.) each of Dutch, Egyptian, French and German persons
are to be seated for a round table conference.
Q.3 Find the number of ways in which 3 distinct
numbers can be selected from the set {31, 32, 33, ….. 3100, Q.7 If the number of ways in which they can be seated
3101} so that they form a G.P. if exactly to pairs of persons of same nationality are
together is p(6!), then find p.
Q.4 Find the number of odd numbers between 3000 to
6300 that have all different digits. Q.8 If the number of ways in which only American pair
is adjacent is equal to q(6!), then find q.
Q.5 A man has 3 friend. In how many ways he can invite
one friend every day for dinner on 6 successive nights Q.9 If the number of ways in which no two people of
so that no friend is invited more than 3 times. the same nationality are together given by r (6!), find r.
5 . 4 2 | Permutations and Combinations

Q.10 For each positive integer k, let Sk denote the Q.18 (a) Find the number of non-empty subsets S of {1,
increasing arithmetic sequence of integers whose 2, 3, 4, 5, 6, 7, 8, 9, 10, 11, 12} such that if, S contains k
first term is 1 and whose common difference is k. For elements, then S contains no number less than k.
example, S3 is the sequence 1, 4, 7, 10 ….. Find the
(b) If the number of ordered pairs (S, T) of subsets of
number of values of k for which Sk contain the term 36!
{1, 2, 3, 4, 5, 6} are such that S ∪ T contains exactly three
elements 10λ, then find the value of λ.
Q.11 A shop sells 6 different flavors of ice-cream. In
how many ways can a customer choose 4 ice-cream
Q.19 Find the number of permutation of the digits 1,
cones if
2, 3, 4 and 5 taken all at a time so that the sum of the
(i) They are all of different flavors digits at the first two places is smaller than the sum of
the digit at the last two places.
(ii) They are not necessarily of different flavors
(iii) They contain only 3 different flavors Q.20 In a league of 8 teams, each team played every
(iv) They contain only 2 or 3 different flavors? other team 10 times. The number of wins of the 8 teams
formed an arithmetic sequence. Find the least possible
Q.12 (a) How many divisors are there of the number number of games won by the champion.
21600. Find also the sum of these divisors.
Q.21 Find the sum of all numbers greater than 10000
(b) In how many ways the number 7056 can be resolved formed by using the digits 0, 1, 2, 4, 5 no. digit being
as a product of 2 factors. repeated in any number.
(c) Find the number of ways in which the number
300300 can be split into 2 factors which are relatively Q.22 There are 3 cars of different make available to
prime. transport 3 girls and 5 boys on a field trip. Each car can
hold up to 3 children. Find
(d) Find the number of positive integers that are divisors
of at least one of the number 1010; 157; 1811. (a) the number of ways in which they can be
accommodated.
Q.13 How many 15 letter arrangement of 5A’s, 5 B’s and (b) the numbers of ways in which they can be
5 C’s have no A’s in the first 5 letters, no B’s in the next accommodated if 2 or 3 girls are assigned to one of
5 letters, and to C’s in the last 5 letters. the cars.

Q.14 Determine the number of paths from the origin to In both the cases internal arrangement of children
the point (9, 0) in the Cartesian plane which never pass inside the car is considered to be immaterial.
through (5, 5) in paths consisting only of steps going 1
unit North and 1 unit East. Q.23 Find the number of three elements sets of positive
integers {a, b, c} such that a × b × c = 2310.
Q.15 There are n triangles of positive area that have
one vertex A(0, 0) and the other two vertices whose Q.24 Find the number of integer between 1 and 10000
coordinates are drawn independently with replacement with a least one 8 nd at least one 9 as digits
from the set {0, 1, 2, 3, 4} e.g. (1. 2), (0, 1) (2, 2) etc. Find
the value of n. Q.25 Let N be the number of ordered pairs of non-
empty sets A and B that have the following properties:
Q.16 How many different ways can 15 Candy bars be (a) A ∪ B = {1, 2, 3, 4, 5, 6, 7, 8, 9, 10}
distributed between Ram, Shyam, Ghanshyam and
Balram, if Ram cannot have more than 5 candy bars (b) A ∩ B = φ
and Shyam must have at least two. Assume all Candy (c) The number of elements of A is not the element of B.
bars to be alike
(d) The number of elements of B is not an element of A.
Q.17 Find the number of three digits number from 100 Find N.
to 999 inclusive which have any one digit that is the
average of the other two. Q.26 In how many other ways can be letters of the word
MULTIPLE be arranged:
(i) Without changing the order of the vowels
M a them a ti cs | 5.43

(ii) Keeping the position of each vowel fixed and Q.5 Number of different natural numbers which are
without changing the relative order/position or vowels smaller than two hundred million and use only the
& consonants. digits 1 or 2 is
(A) (3) . 28 – 2 (B) (3) . 28 – 1
Q.27 Let N denotes the number of all 9 digits numbers if
(C) 2 (29 – 1) (D) None
(a) The digit of each number are all from the set {5, 6,
7, 8, 9} and
Q.6 There are counters available in x different colors.
(b) Any digit that appears in the number, repeats at The counters are all alike except for the color. The total
least three times. Find the value of N/5. number of arrangements consisting of y counters,
assuming sufficient number of counters of each color,
Q.28 How many integers between 1000 and 9999 have if no arrangement consists of all counters of the same
exactly one pair of equal digit such as 4049 or 9902 but color is:
not 4449 or 4040? (A) xy – x (B) xy – y (C) yx – x (D) yx – y

Q.29 How many 6 digits odd numbers greater than Q.7 If m denotes the number of 5 digit numbers of
60,000 can be formed from the digits 5, 6, 7, 8, 9, 0 if each successive digits are in their descending order
(i) Repetitions are not allowed magnitude and n is the corresponding figure, when the
digits are in their ascending order of magnitude then
(ii) Repetitions are allowed.
(m – n) has the value
(A) 10C4 (B) 9C5 (C) 10C3 (D) 9C3
Exercise 2
Q.8 There are m points on straight line AB & n points on
Single Correct Choice Type the line AC none of them being the point A. Triangles
are formed with these points as vertices, when
Q.1 An eight digit number divisible by 9 is to be formed
(i) A is excluded
by using 8 digits out of the digits 0, 1, 2, 3, 4, 5, 6, 7 8, 9
without replacement. The number of ways in which this (ii) A is included. The ration of number of triangles in
can be done is the two cases is:
(A) 9! (B) 2(7!) (C) 4(7!) (D) (36) (7!) m+n–2 m+n–2
(A) (B)
m+n m+n–1
Q.2 Number of 4 digit numbers of the form N = abcd m+n–2 n(n – 1)
(C) (D
which satisfy following three conditions m+n+2 (m + 1)(n + 1)
(i) 4000 ≤ N < 6000
Q.9 The number of 5 digit numbers such that the sum
(ii) N is a multiple of 5
of their digits is even is
(iii) 3 ≤ b < c ≤ 6 is equal to
(A) 50000 (B) 45000 (C) 60000 (D) None
(A) 12 (B) 18 (C) 24 (D) 48
Q.10 Number of ways in which 8 people can be
Q.3 5 Indian & 5 American couples meet at a party arranged in a line if A and B must be next each other
and shake hands. If no wife shakes hands with her own and C must be somewhere behind D, is equal to
husband and no Indian wife shakes hands with a male
(A) 10080 (B) 5040 (C) 5050 (D) 10100
then the number of handshakes that takes place in the
party is
Q.11 Seven different coins are to be divided amongst
(A) 95 (B) 110 (C) 135 (D) 150 three persons. If no two of the persons receive the same
number of coins but each receives at least one coin &
Q. 4 The 9 horizontal and 9 vertical lines on an 8 × 8 none is left over, then the number of ways in which the
chessboard form ‘r’ rectangles and ‘s’ squares, The ratio division may be made is
s/r in its lowest terms is
(A) 420 (B) 630 (C) 710 (D) None
1 17 4
(A) (B) (C) (D) None
6 108 27
5 . 4 4 | Permutations and Combinations

Q.12 Let there be 9 fixed point on the circumference of Q.19 Number of rectangles in the grid shown which are
a circle. Each of these points is joined to every one of not squares is
the remaining 8 points by a straight line and the points
are so positioned on the circumference that at most 2 
straight lines meet in any interior point of the circle. The
number of such interior intersection points is:
(A) 126 (B) 351 (C) 756 (D) None
(A) 160 (B) 162 (C) 170 (D) 185
Q.13 The number of ways in which 8 distinguishable
apples can be distributed among 3 boys such that every Q.20 All the five digit numbers in which each successive
boy should get at least 1 apple & at most 4 apples is K. digit exceeds its predecessor are arranged in the
7
P3 where K has the value equal to increasing order of their magnitude. The 97th number in
(A) 14 (B) 66 (C) 44 (D) 22 the list does not contains the digit
(A) 4 (B) 5 (C) 7 (D) 8
Q.14 There are five different peaches and three different
apples. Number of ways they can be divided into two Q.21 There are n identical red balls & m identical green
packs of four fruits if each pack must contain at least balls. The number of different linear arrangements
one apple, is consisting of n red ball but not necessarily all the green
(A) 95 (B) 65 (C) 60 (D) 30 balls’ is xCy then
(A) x = m + n, y = m
Q.15 Let Pn denote the number of ways in which three
people can be selected out of ‘n’ people sitting in a row, (B) x = m + n + 1, y = ,m
if no two of them are consecutive. If Pn+1 – Pn = 15 then (C) x = m + n + 1, y = m + 1
the value of ‘n’ is
(D) x = m + n, y = n
(A) 7 (B) 8 (C) 9 (D) 10
Q.22 A gentleman invites a party of m + n (m ≠ n) friends
Q.16 The number of positive integers not greater than to a dinner & places m at one table T1 and n at another
100, which are not divisible by 2, 3 or 5 is table T2, the table being round, if not all people shall have
(A) 26 (B) 18 (C) 31 (D) None the same neighbor in any two arrangement, then the
number of ways in which he can arrange the guests, is
Q.17 There are six periods in each working day of a (m + n)! 1 (m + n)!
(A) (B)
school. Number of ways in which 5 subjects can be 4mn 2 mn
arranged if each subject is allotted at least one period
(m + n)!
and no period remains vacant is (C) 2 (D) None
mn
(A) 210 (B) 1800 (C) 360 (D) 3600
Q.23 Consider a determinant of order 3 all whose
Q.18 An old man while dialing a 7 digit telephone entries are either 0 or 1. Five of these entries are 1 and
number remembers that the first four digits consists of four of them are ‘0’. Also aij = aji ∀ 1 ≤ i, j ≤ 3. The
one 1’s, one 2’s and two 3’s. He also remembers that number of such determinants, is equal to
the fifth digit is either a 4 or 5 while has no memorizing
(A) 6 (B) 8 (C) 9 (D) 12
of the sixth digit, he remembers that the seventh digit
is 9 minus the sixth digit. Maximum number of distinct
Q.24 A team of 8 students goes on an excursion, in
trials he has to try to make sure that he dials the correct
two cars, of which one can seat 5 and the other only 4.
telephone number, is
If internal arrangement inside the car does not matter
(A) 360 (B) 240 (C) 216 (D) None then the number of ways in which they can travel, is
(A) 91 (B) 182 (C) 126 (D) 3920
M a them a ti cs | 5.45

Q.25 One hundred management students who read at Q.33 The number of ways of choosing a committee
least one of the three business magazines are surveyed of 2 women & 6 men, if Mr. A refuses to serve on the
to study the readership pattern. It is found that 80 committee if Mr. B is a member & Mr. B can only serve,
read Business India, 50 read Business world, and 30 if Miss C is the member of the committee, is
read Business Today. Five students read all the three
(A) 60 (B) 84 (C) 124 (D) None
magazines. How many read exactly two magazines?
(A) 50 (B) 10 (C) 95 (D) 45 Q.34 Six person A, B, C, D, E and F are to be seated at a
circular table. The number of ways this can be done if
Q.26 Six people are going to sit in a row on a bench. A A must have either B or C on his right and B must have
and B are adjacent, C does not want to sit adjacent to either C or D on his right is:
D. E and F can sit anywhere. Number of ways in which
(A) 36 (B) 12 (C) 24 (D) 18
these six people can be seated, is
(A) 200 (B) 144 (C) 120 (D) 56 Q.35 There are 100 different books in a shelf. Number
of ways in which 3 books can be selected so that no two
Q.27 Number of cyphers at the end of 2002C1001 is of which are adjacent is
(A) 0 (B) 1 (C) 2 (D) 200 (A) 100C3 – 98 (B) 97C3 (C) 96C3 (D) 98C3

Q.28 Three vertices of a convex n sided polygon


are selected. If the number of triangles that can be Q.36 Number of ways in which four different toys
constructed such that none of the sides of the triangle and five indistinguishable marbles can be distributed
is also the side of the polygon is 30, then the polygon between Amar, Akbar and Anthony, if each child
is a receives at least one toy and one marble, is

(A) Heptagon (B) Octagon (A) 42 (B) 100 (C) 150 (D) 216

(C) Nonagon (D) Decagon Q.37 A 3 digit palindrome is a 3 digit number (not
starting with zero) which reads the same backwards as
Q.29 Given 11 points, of which 5 lie on one circle, other forwards. For example 171. The sum of all even 3 digit
than these 5, no 4 lie on one circle. Then the maximum palindromes, is
number of circles that can be drawn so that each
contains at least three of the given points is: (A) 22380 (B) 25700 (C) 22000 (D) 22400

(A) 216 (B) 156 (C) 172 (D) None Q.38 Two classrooms A and B having capacity of 25
and (n–25) seats respectively An denotes the number
Q.30 Number of 5 digit numbers divisible by 25 that of possible seating arrangements of room ‘A’, when
can be formed using only the digits 1, 2, 3, 4, 5, & 0 ‘n’ students are to be seated in these rooms, starting
taken five at a time is from room ‘A’ which is to be filled up full to its capacity.
(A) 2 (B) 32 (C) 42 (D) 52 If An – An–1 = 25! (49C25) then ‘n’ equals
(A) 50 (B) 48 (C) 49 (D) 51
Q.31 Let Pn denotes the number of ways of selecting
3 people out of ‘n’ sitting in a row, if no two of them Q.39 Number of positive integral solution satisfying
are consecutive and Qn is the corresponding figure the equation (x1 + x2 + x3) (y1 + y2) = 77, is
when they are in a circle. If Pn – Qn = 6, then ‘n’ is
equal to (A) 150 (B) 270 (C) 420 (D) 1024

(A) 8 (B) 9 (C) 10 (D) 12 Q.40 There are counters available in 3 different colors (at
least four of each color). Counters are all alike except for the
Q.32 Let m denote the number of ways in which 4 color. If ‘m’ denotes the number of arrangements of four
different books are distributed among 10 persons, counters if no arrangement consists of counters of same
each receiving none or one only and let n denote the color and ‘n’ denotes the corresponding figure when every
number of ways of distribution if the books are all alike. arrangement consists of counters of each color, then:
Then:
(A) m = 2 n (B) 6 m = 13 n
(A) m = 4n (B) n = 4m (C) m = 24 n (D) none
(C) 3 m = 5 n (D) 5 m = 3 n
5 . 4 6 | Permutations and Combinations

Q.41 Three digit numbers in which the middle one is a Multiple Correct Choice Type
perfect square are formed using the digits 1 to 9. Their
sum is: Q.46 The combinatorial coefficient C(n, r) is equal to
(A) 134055 (B) 270540 (A) number of possible subsets of r members from a set
of n distinct members.
(C) 170055 (D) none
(B) number of possible binary messages of length n
Q.42 A guardian with 6 wards wishes every one of with exactly r l’s.
them to study either Law of Medicine or Engineering. (C) number of non-decreasing 2-D paths from the
Number of ways in which he can make up his mind with lattice point (0, 0) to (r, n).
regard to the education of his wards if every one of
them be fit for any of the branches to study, and at least (D) number of ways of selecting r objects out of n
one child is to be sent in each discipline is: different objects when a particular object is always
included plus the number of ways of selecting ‘r’
(A) 120 (B) 216 (C) 729 (D) 540 objects out of n, when a particular object out of n, when
a particular object is always excluded.
Q.43 There are (p + q) different books on different
topics in Mathematics (p ≠ q) Q.47 There are 10 questions, each question is either
If L = The number of ways in which these books are True or False. Number of different sequences of
distributed between two students X and Y such that X incorrect answers is also equal to
get p books and Y gets q books. (A) Number of ways in which a normal coin tossed 10
M = The number of ways in which these books are times would fall in a definite order if both Heads and
distributed between two students X and Y such that Tails are present.
one of them gets p books and another gets q books. (B) Number of ways in which a multiple choice question
N = The number of ways in which these books are containing 10 alternatives with one or more than one
divided into two groups of p books and q books then, correct alternatives, can be answered.

(A) L = M = N (B) L = 2M = 2N (C) Number of ways in which it is possible to draw a


sum of money with 10 coins of different denominations
(C) 2L = M = 2N (D) L = M = 2N taken some or all at a time.

Q.44 Number of ways in which 5A’ and 6B’s can be (D) Number of different selection of 10 indistinguishable
arranged in a row which reads the same backwards and objects taken some or all at a time.
forwards, is
Q.48 The number of ways in which five different books
(A) 6 (B) 8 (C) 10 (D) 12 can be distributed among 3 persons so that each
person gets at least one book, is equal to the number
Q.45 Coefficient of x2 y3 z4 in the expansion of (x + y + of ways in which
z)9 is equal to
(A) 5 persons are allotted 3 different residential flats so
(A) The number of ways in which 9 objects of which 2 that and each person is allotted at most one flat and no
alike of one kind, 3 alike of 2nd kind, and 4 alike of 3rd two persons are allotted the same flat.
kind can be arranged.
(B) Number of parallelograms (some of which may
(B) The number of ways in which 9 identical objects can be overlapping) formed by one set of 6 parallel lines
be distributed in 3 persons each receiving at least two and other set of 5 parallel lines that goes in other
objects. direction.
(C) The number of ways in which 9 identical objects can (C) 5 different toys are to be distributed among 3
be distributed in 3 persons each receiving none one or children, so that each child gets at least one toy.
more.
(D) 3 mathematics professors are assigned five different
(D) The number of ways in which 9 different books can lecturers to be delivered, so that each professor gets at
be tied up in to three bundles one containing 2, other 3 least one lecturer.
and third containing 4 books.
M a them a ti cs | 5.47

Q.49 If k is odd then kCr is maximum for r equal to Q.54 If the number of arrangements of the letters of
the word W if all the S’s and P’s are separated is (k)
1 1
=
(A) r (k − 1) =
(B) r (k + 1)  10! 
2 2
  then k equals
 4!.4! 
(C) k ‒ 1 (D) k
6 4 3
(A) (B) 1 (C) (D)
Q.50 Which of the following statements are correct? 5 3 2
(A) Number of words that can be formed with 6 only
of the letters of the word “CENTRIFUGAL’ if each word Paragraph 2: 16 players P1, P2, P3, ….. P16 take part in
must contain all the vowels is 3 . 7! a tennis tournament. Lower suffix player is better than
any higher suffix player. These players are to be divided
(B) There are 15 balls of which some are white and the into 4 groups each comprising of 4 players and the best
rest black. If the number of ways in which the balls con from each group is selected for semifinals.
be arranged in a row, is maximum then the number of
white balls must be equal to 7 or 8. Assume balls of the Q.55 Number of ways in which 16 players can be
same color to be alike. divided into four equal groups, is
(C) There are 12 objects, 4 alike of one kind, 5 alike and 35 8 35 8
of another kind and the rest are all different. The total (A) ∏ (2r – 1)
27 r =1
(B) ∏ (2r – 1)
24 r =1
number of combinations in 240
(D) Number of selections that can be made of 6 letters 35 8 35 8
from the word “COMMITTEE” is 35.
(C) ∏ (2r – 1)
52 r =1
(D) ∏ (2r – 1)
6 r =1

Q.51 Number of ways in which the letters of the word Q.56 Number of ways in which they can be divided into
‘B U L B U L’ can be arranged in a line is a definite order 4 equal groups if the players P1, P2, P3 and P4 are in
is also equal to the different groups, is:
(A) Number of ways in which 2 alike Apples and 4 alike (11)! (11)! (11)! (11)!
(A) (B) (C) (D)
Mangoes can be distributed in 3 children so that each 36 72 108 216
child receives any number of fruits.
(B) Number of ways in which 6 different books can be Match the Columns
tied up into 3 bundles, if each bundle is to have equal Q.57
number of books.
Column-I Column-II
(C) Coefficient of x2y2z2 in the expansion of (x + y + z)6.
(A) Number of increasing permutations (p) nm
(D) Number of ways in which 6 different prizes can be
of m symbols are there from the n set
distributed equally in three children.
numbers {a1, a2, …, an} where the order
among the number is given by a1 < a2 < a3
Comprehension Type < … an–1 < an is

Paragraph 1: Consider the word W = MISSISSIPPI (B) There are m men and n monkeys. (q) mCn
Number of ways in which every monkey
Q.52 If N denotes the number of different selections has a master, if a man can have any number
of 5 letters from the word W = MISSISSIPPI then N of monkeys
belongs to the set (C) Number of ways in which n red balls are (r) nCm
(A) {15, 16, 17, 18, 19} (B) {20, 21, 22, 23, 24} (m – 1) green balls can be arranged in a
line, so that no two red balls are together, is
(C) {25, 26, 27, 28, 29} (D) {30, 31, 32, 33, 34} (balls of the same color are alike)

Q.53 Number of ways in which the letters of the word (D) Number of ways in which ‘m’ different (s) mn
W can be arranged if at least one vowel is separated toys can be distributed in ‘n’ children if every
from rest of the vowels child may receive any number of toys, is

8!.161 8!.161 8!.161 8! 165


(A) (B) (C) (D) .
4!.4!.2! 4.4!.2! 4!.2! 4!.2! 4!
5 . 4 8 | Permutations and Combinations

Q.58 Q.2 7 relatives of a man comprises 4 ladies and 3


gentlemen, his wife has also 7 relatives; 3 of them are
Column-I Column-II ladies and 4 gentlemen. In how many ways can they
(A) Four different movies are running in a (p) 11 invite a dinner party of 3 ladies and 3 gentlemen so
town. Ten students go to watch these four that there are 3 of man’s relative and 3 of the wife’s
movies. The number of ways in which every relatives?  (1985)
movie is watched by at least one student, is
(Assume each way differs only by number Q.3 A box contains two white balls, three black balls
of students watching a movies) and four red balls. In how many ways can three balls be
drawn from the box, if at least one black ball is to be
(B) Consider 8 vertices of a regular octagon (q) 36
included in the draw? (1986)
and its center. If T denotes the number
of triangles and S denotes the number of
straight lines that can be formed with these Q.4 Eighteen guests have to be seated half on each
9 points then the value of (T – S) equals side of a long table. Four particular guests desire to sit
one particular side and three other on the other side.
(C) In an examination, 5 children were (r) 52 Determine the number of ways in which the sitting
found to have their mobiles in their pocket. arrangements can be made.  (1991)
The Invigilator fired them and took their
mobiles in his possession. Towards the end Q.5 A committee of 12 is to be formed from 9 women
of the test, Invigilator randomly returned and 8 men. In how many ways this can be done if at
their mobiles. The number of ways in which least five women have to be included in a committee?
at most two children did not get their own In how many of these committees
mobiles is
(a) the women are in majority?
(D) The product of the digits of 3214 is 24. (s) 60
The number of 4 digit natural numbers (b) the men are in majority?  (1994)
such that the product of their digits is 12, is
Q.6 Match the conditions/expressions in column I with
(E) The number of ways in which a mixed (t) 84
statement in column II.
double tennis game can be arranged from
amongst 5 married couple if no husband & Consider all possible permutations of the letters of the
wife plays in the same game, is word ENDEANOEL. (2008)

Column I Column II
Previous Years’ Questions (A) The number of permutations containing (p) 5!
the word ENDEA. is
Q.1 Five balls of different colors are to be placed in
three boxes of different sizes. Each box can hold all five. (B) The number of permutations in which (q) 2 × 5!
In how many different ways can we place the balls so the letter E occurs in the first and the last
that no box remains empty?  (1981) positions, is
(C) The number of permutations in which (r) 7 × 5!
none of the letters D, L, N occurs in the last
five positions, is
(D) The number of permutations in which the (s) 21×5!
letters A, E, O occur only in odd positions, is
M a them a ti cs | 5.49

PlancEssential Questions
JEE Main/Boards JEE Advanced/Boards
Exercise 1 Exercise 1
Q.6 Q.10 Q.13 Q.16 Q.22 Q.5 Q.6 Q.12 Q.13 Q.16
Q.26 Q.27 Q.20 Q.22 Q.28 Q.30

Exercise 2 Exercise 2
Q.3 Q.13 Q.15 Q.18 Q.25 Q.1 Q.3 Q.8 Q.13 Q.20
Q.33 Q.43 Q.46 Q.47 Q.50 Q.26 Q.32 Q.39 Q.42 Q.43
Q.52 Q.49 Q.55 Q.58

Previous Years’ Questions Previous Years’ Questions


Q.4 Q.5 Q.7 Q.1 Q.3 Q.4 Q.6

Answer Key

JEE Main/Boards Q.13 369600

Q.14 302399, 30240


Exercise 1
Q.15 3600
Q.1 2 + 6 + 24 = 32
Q.16 126
Q.2 3 × 4 × 3 × 2 × 1 = 72 1 1
Q.17 3 + 4 + 4 = 11 Hours.
Q.3 35 + 15 = 50 4 2
Q.18 r = 3
Q.5 n = 6
Q.20 52C4 = 270725
Q.6 180
Q.22 817190
Q.7 365
Q.23 (i) 116 (ii) 40 (iii) 185
Q.8 1332
Q.24 31
Q.9 1023
50!
Q.25
Q.10 (i) 2 + 2 + 2 + 2 = 30 (ii) = 2 + 2 + 2 + 2 +
1 2 3 4 1 2 3 4
(12!) .(7!)2 3!
3

25 = 2 + 4 + 8 + 16 + 32 = 62
Q.26 26
Q.11 34650 – 840 = 33810
Q.27 150
Q.12 4! 3!
Q.28 2454
5 . 5 0 | Permutations and Combinations

Exercise 2

Single Correct Choice Type


Q.1 C Q.2 C Q.3 D Q.4 A Q.5 B Q.6 C
Q.7 A Q.8 D Q.9 A Q.10 C Q.11 D Q.12 B
Q.13 D Q.14 B Q.15 A Q.16 A Q.17 A Q.18 D
Q.19 D Q.20 A Q.21 B Q.22 C Q.23 C Q.24 C
Q.25 D Q.26 A Q.27 A Q.28 C Q.29 B Q.30 D
Q.31 C Q.32 B Q.33 C Q.34 D Q.35 C Q.36 D
Q.37 A Q.38 B Q.39 A Q.40 D Q.41 C Q.42 C
Q.43 A Q.44 C Q.45 C Q.46 D Q.47 A Q.48 B
Q.49 D Q.50 D Q.51 C Q.52 B Q.53 B Q.54 B
Q.55 E Q.56 D

Previous Years’ Questions


Q.1 C Q.2 D Q.3 A Q.4 A Q.5 C Q.6 C
Q.7 C

JEE Advanced/Boards
Exercise 1
Q.1 162 Q.2 7560 Q.3 2500 Q.4 826 Q.5 510 Q.6 10, 3
Q.7 60 Q.8 64 Q.9 244 Q.10 24 Q.11 (i)15, (ii) 126, (iii) 60 (iv) 105
Q.12 (a) 72; 78120; (b) 23 (c) 32; (d) 435 Q.13 2252 Q.14 30980 Q.15 276
Q.16 440 Q.17 121 Q.18 (a) 128; (b) 54 Q.19 48 Q.20 42
Q.21 3119976 Q.22 (a) 1680; (b) 1140 Q.23 40 Q.24 974 Q.25 186
Q.26 (i) 3359; (ii) 59; (iii) 359 Q.27 4201 Q.28 3888 Q.29 (i) 240, (ii) 15552

Exercise 2

Single Correct Choice Type


Q.1 D Q.2 C Q.3 C Q.4 B Q.5 A Q.6 A
Q.7 B Q.8 A Q.9 B Q.10 B Q.11 B Q.12 A
Q.13 D Q.14 D Q.15 D Q.16 A Q.17 B Q.18 B
Q.19 A Q.20 B Q.21 B Q.22 A Q.23 D Q.24 C
Q.25 A Q.26 B Q.27 B Q.28 C Q.29 B Q.30 C
Q.31 C Q.32 C Q.33 C Q.34 D Q.35 D Q.36 D
Q.37 C Q.38 A Q.39 C Q.40 B Q.41 A Q.42 D
Q.43 C Q.44 C Q.45 D
M a them a ti cs | 5.51

Multiple Correct Choice Type


Q.46 A, B, D Q.47 B, C Q.48 B, C, D Q.49 A, B Q.50 B, D Q.51 C, D

Comprehension Type
Q.52 C Q.53 B Q.54 B Q.55 A Q.56 C

Matric Match Type


Q.57 A → r; B → s; C → q; D → p Q.58 A → t; B → r; C → p; D → q; E → s

Previous Years’ Questions


Q.1 300 Q.2 485 Q.3 64 Q.4 9P4 × 9P3 × (11)!

Q.5 6062, (a) 2702 (b) 1008 Q.6 A → p; B → s; C → q; D → q.

Solutions

JEE Main/Boards =2n (1 × 2 × 3.....n)(1 × 3 × 5......(2n +1))

=2n.n! (1 × 3 × 5........ (2n + 1))


Exercise 1
(2n + 1)
= 2n. (1 × 3 × 5........ (2n + 1))
0, 1, 4, 7 n!

n!
Sol 5: nP4 = 360; = 360
Sol 1: (n – 4)!
n(n – 1)(n – 2)(n – 3) = 360 ⇒ 6 × 5 × 4 × 3 = 360
0, 1, 4, 7 1 or 7
n=6
Total numbers = 4 × 4 × 2 = 32
Sol 6: Case-I: 4 digits
10r2
Sol 2:

First arrange those 3 persons in rear seats 2 × 5C3 × 3! = 120


Then remaining in front.
Case-II: 3 digits
Total ways to seat = 5!
Two particular people cannot seat on the driver self
So for this case total ways ⇒ 4! + 4! 3 or 4
3 × 5C2 × 2! = 60
or 5
Therefore, required number of ways = 5! – 4! – 4! = 4!
Total number of numbers = 60 + 120 = 180
(5 – 2) = 3 × 4 × 3 × 2 × 1 = 72

Sol 7: 5 can be there only in the thousand’s digit


Sol 3: 7C1 × 5C1 + 3C1 × 5C1 = 35 + 15 = 50
Case-I: 5 is there 0, 2, 4, 6, 8
Sol 4: (2n + 1)! = 1 × 2 × 3........ (2n + 1) 5 7 5
= (2 × 4 × 6......2n)(1 × 3 × 5...... (2n + 1))
5 choices
5 . 5 2 | Permutations and Combinations

Therefore, total number for case I = 5 10!


Total rearrangements = – 1= 302400 – 1 = 302399
Case-II: 5 is not there 3! 2!
9!
Total rearrangements with L in the end = = 30240
3! 2!
1 1 2 2334 4 56 5
5 Sol 15: PATALI PUTRA
8 9
choices Vowels = 5
choices choices
Consonants = 6
Therefore, total number for case II = 8 × 9 × 5 = 360
Vowels: A(3) I(1) U(1)
Total ways = 5 + 360 = 365
Consonants: P(2) T(2)L(1)R(1)
Sol 8: 123+132+213+231+312+321 = 1332 5! 6!
Total no. of words = × = 3600
3! 2! 2!
Sol 9: 45 – 1 = 1024 – 1 = 1023
One case is excluded when all arms are at rest. 9!
Sol 16: = 126
4! 5!
Sol 10: (i) 21 + 22 + 23 + 24 = 30
Sol 17: 3! ways
(ii) (a) 25 = 32 (b) 21 + 22 + 23 ......... 25 = 62
A → III
B→I
Sol 11: M I S S I S S I P P I
C → II
M (1) 1 1
Time = 4 + 4 + 3 = 11 hours
S (4) 4 4
I (4)
Sol 18: 15C3r = 15Cr+3
P (2)
3
Ways = Total permutations – Permutations with 4 I’s (1) 3r = r + 3 ⇒ r =
2
together
(2) 3r + r + 3 = 12 ⇒ r =
15 ⇒ 4r = 3
11! 8!
= – = 33810 3
4! 4! 2! 4! 2! r= Not possible
2
B B B B r = 3 Possible
Sol 12: ↓ ↓ ↓ ↓
G G G
××× Sol 19: nCr × rCs = nCs × n–sCr–s

first arrange girls → 3! n! r!


LHS: nCr + rCs = ,
4 seats remains in which 4 boys will be r!(n – r)! s!(r – s)!

seated → 4! n! (n – s)! 1 n! (n – s)!


= × × = ×
s! (n – s)! (n – r)!.(r – s)! s!(n – s)! (n – r)!(r – s)!
Total ways = 3! × 4! = 144
= nCs × n–sCr–s = RHS
12!
Sol 13: = 369600
(3!)4 5
Sol 20: 47C4 + ∑ 52– j C3
j=1
Sol 14: BENEVOLENT
47
= C 4 + ( 47 C3 +48C3 ....... 51C3 = 48C4 + (48C3 + ...... + 51C3)
No. of letters = 10   
B (1), E (3), N (2), V (1), O (1), L (1), T (1)
= 51C4 + 51C3 = 52C4 = 270725
M a them a ti cs | 5.53

Sol 21: Product = (n + 1)(n + 2) ...... (n + r) 2A, 2A


4!
(n + r)! (n + r)!
3
C2 × = 18
= = r! × = r! × n+rCr 2! 2!
n! n! r!
Total = 2454
n+r
Cr will be integer
Hence product is divisible by r!. Exercise 2
Sol 22: 22C10 + 3C3 22C7= 817190 Single Correct Choice Type

Sol 23: Sol 1: (C)


ANRUV
4!
NARUV

(i) No. of triangles = 10C3 – 4C3 = 116 4!


RANUV
(ii) No. of straight lines = 6C2 + 6 × 4 + 1 4!
= 15 + 24 + 1 = 40 UANRV
4!
(iii) No. of quadrilaterals = 10C4 – 4C3 × 6 – 4C4 = 185 96
V A N R U 97
V A N U R 98
Sol 24: 25 –1 = 31 V A R N U 99
V A R U N 100
50!
Sol 25:
(12!)3 3! (7!)2 2! Sol 2: (C) Numbers = Total – Numbers with no digit 7
Total = 900
Sol 26: Cases not allowed

× ×
8 9 9
× × × × × × × ×
× × Number of numbers with at least
8
C6 – 2 = 26 One digit 7 = 900 – 8 × 9 × 9 = 252

Sol 27: Possible groups Sol 3: (D)


AEHMS
113
24
122 ASMHE 24
EAHMS
No. of ways 24
 5! 5!  ESMHA 48
= 3! ×  +  = 60 + 90 = 150 HAEMS
 1! 1! 3! (2!) 1! 2! 2! (2!)  24
HSMEA 72
Sol 28: EXAMINATION MAEHS 73
6
4 DIFF
MASHE 78
8
C4 × 4! = 1680 MEAHS
6
2 diff. 2 alike MESHA 84
4! MHAES
3
C 1 × 7C 2 × = 756
2! MHASE 86
5 . 5 4 | Permutations and Combinations

Case-II: 5 is not there

Sol 4: (A) Y, G, B, R
5
8 9
choices
4 3 3 3 3 3 choices choices

4 × 35 = 12 × 81 8 × 9 × 5 = 360
Total ways = 5 + 360 = 365
Sol 5: (B) Prime number in 0 – 9
2, 3, 5, 7 Sol 12: (B) 5

1 1 1 1 1 8 8 7 6

4 4 4 4 4 5 4 3 2 0
2 × 45 = 2048 4
C1 × 8 × 7 × 6

Total ways = 8×8×7×6+ 4C1×8×7×6 = 168 × 4!


Sol 6: (C)
Sol 13: (D) 4 odd, 3 even
5
C3 – 2 = 10 – 2 = 8
Arrangements: 4 + 2 3 + 3
Total numbers = 3C2 × 2 × 4! + 4C3 × 3C2 × 2 × 4! = 720
Sol 7: (A)
Sol 14: (B) 9000 – 9 × 9 × 8 × 7 = 4464
5 5 5
No. 5 with no even digit
Sol 15: (A) It can be a digit number with digits 1 to 9
900 – 53 = 775
or a 10 digit number with digits 0 to 9
9! + 9.9! = 10!
Sol 8: (D) The sum of the 5-digits used must be divisible
by 3.
Sol 16: (A) n
Only possible combinations are:
1, 2, 3, 4, 5&0, 1, 2, 4, 5 n n

↓↓ 9
9 × 10 × 19
5! 4.4! ∑ n2 = 6
= 285
n=1
Total = 9.4!
= 9 × 24 = 216 Sol 17: (A) Consider 1 & 2, 3 & 4, 5 & 6 to be identical
9!
Permutations = = 9.7!
Sol 9: (A) C2 × 2 = 672
7 5
(2!)3

Sol 10: (C) 7C4 × 4C2 × 6! = 151200 Sol 18: (D)

Last digit 9   Last digit not 9


Sol 11: (D) 5 can be there only in the thousand’s digit
9
Case-I: 5 is there
7 8 7 6 8 7 4
0, 2, 4, 6, 8
5 7 5 Total = 7 × 8 × 7 + 6 × 8 × 7 × 4 = 1736

5 choices
M a them a ti cs | 5.55

Sol 19: (D) Sol 28: (C) Combine those 2 green bottles
↓ ↓ ↓ ↓ ↓ ↓ ↓ ↓ ↓ ↓ ××××××××
× × × × × × × × × × 9 spaces
10 persons 6 × 9C6 = 84 × 6 = 504
10
C5 . 5!
Sol 29: (B) 25C5 – 24C4 = 24C5
Choose 5 persons from 10 getting books & distribute
books to then 5! ways .
Sol 30: (D) 5C4 × 24 = 80
Sol 20: (A) 5C3 × 8C7 + 5C4 × 8C6 + 5C5 × 8C5 Select 4 pairs out of 5 different pairs. Now in each pair
you can choose 2 different shoes.
= 80 + 5 × 28 + 56 = 276

Sol 31: (C) 2 2 2 3


Sol 21: (B)
3! 9! 9!
3 =
9 9 3! (2!) 3! 3! (2!)3
9 9

Total = 162
Sol 32: (B) × × × × × × × × × ×
(n – 2) 10 red balls
Sol 22: (C) × 180 = 150
n
11 spaces
6n – 12 = 5n ⇒ n = 12
choose 9 spaces to fill green balls. 10! ways to arrange
Diagonals = 12C2 – 12 = 54
red balls.
11
C9 × 10!
Sol 23: (C) 2n – 2 = 510
2n = 512
Sol 33: (C) 7! × 23
n=9
7! ways to arrange
2 order (ascending/descending)
Sol 24: (C) 7
7

9 8 7 8 8 7 6!
Sol 34: (D)
2! 2! 2!
9 × 8 × 7 + 8 × 8 × 7 × 3= 33 × 8 × 7= 1848
Consider man and wife to be identical and arrange them.
Sol 25: (D) Total appointment = 11 Similar concept as used in Q. 18

11 → 2 3 6 7! 7! 7! 7!
Sol 35: (C) + + .......
7! 0! 6! 1! 5! 2! 2! 5!
total ways to disposed = (2! 3! 6!) × 3 = 8640 × 3= 25920
7
C0 + 7C1 + 7C2 ....... 7C5 = 27 – 7C6 – 7C7 = 128 – 7 – 1 = 120
Sol 26: (A) Illegal ways = 2 × 7C4 + 7C3 – 2 × 5C2 = 85
No. of possible ways = 9C5 – 85= 126 – 85 = 41 Sol 36: (D)
× × × ×
Sol 27: (A) 1 1 3 EE AA

122 EAEA

3 × (4C1 × 4C1 × 4C3 + 4C1 × 4C2 × 4C2) AEAE

= 3 × (64 + 144) = 624 EAAE


4! × (5C2 × (2! + 2! + 1 + 1)) = 1440
5 . 5 6 | Permutations and Combinations

Sol 37: (A) The last match has to be won by India Total ways = 7 + 10–1C10–1= 16C9 = 16C7
1 + 5C1 + 6C2 + 7C3 + 8C4= 126
Sol 46: (D) Total ways = 3n+ nC2 .3n–2+nC4.3n–4+......
Sol 38: (B) (1 + 3)n 2n
= + = 22n–1 + 2n–1 = 2n–1(2n + 1)
2 2
635/674
Sol 47: (A) Each digit will be present at unit’s ten’s and
2(93 × 1 + 3 × 4 × 92) = 2(1701) = 3402 hundred’s place 6 times.
(1 + 2 + 3 + 4) × 6 = 60
Sol 39: (A)
60
Master
× × 60 ×
× × 60 × ×
× × Sum of digits =
× × 6660
× ×
× × Sol 48: (B) 8C1 + 8C1 × 7C1 = 64
Mistress

2 × 10 ×10!=Sol.20 10! Sol 49: (D) No. of books of algebra = No of books of


calculus
Sol 40: (D) S1 S3 S2
Sol 50: (D) No. of ways = Total ways – No letter is in
S 3S 1 S 2
wrong envelope = 5! – 1 = 119
Consider S1 S2 S3 identical & arrange
10! 10!
2× = Sol 51: (C) P1 P2 P3 P4
3! 3
4!
=3
Sol 41: (C) 310 – 1 (2!)2 2!
3 choices question its alternative & no question.
Total matches = 8C4 × 3

Sol 42: (C) 9 9 9 9 9 9 7 Sol 52: (B) 9C6 × 5 × 3! = 2520


9999988
Sol 53: (B) 6C1 ×21 × 25 + 6C3 × 23 × 23 + 6C5
7! 7!
+ f= 7 + 21 = 2s
6! 5! 2! × 25 × 21 = 26 (6C1 + 6C3 + 6C5)
26
9!
= 26 × = 211
Sol 43: (A) – 4! 3! = 6(7! – 4!) 2
2! 3!
Sol 54: (B) 2.6.10 ....... (4n – 2)
Sol 44: (C) 1000 = 23 × 53
n! (2n)! 2n
= 2n(1.3.5 ...... (2n –1)) × = = Pn
The product of even divisors of 1000 will be n! n!
( ) ( ) ( ) (
= 2 × 22 × 23 × ( 2 × 5 ) × 2 × 52 × 2 × 53 × 22 × 5 ) Sol 55: (E) No. of permutations
× (2 2
× 5 ) × (2 × 5 ) × (2 × 5) × (2 × 5 ) × (2
2 2 3 3 3 2 3
×5 )
3
(2n)! 2n . n![1.3.5......(2n – 3)(2n – 1)]
= 2nCn = =
n! n! n!
= (26)4 × (5 × 52 × 53) = (26)4 × (56)3 = 64 × 1018

Sol 45: (C) x1 + x2 + ........ x10 = 7 Sol 56: (D) n is odd


n+1
7 people are distributed to 10 floor 1, 2, 3 ......... ...... n
2
M a them a ti cs | 5.57

n–1 (n – 1)2
Total A.P.S. = 0 + 1.....+ +....+1+ 0 = Pq Number of ways
2 4
R0r2 1 way
n is even
R1r21 way
n n
1, 2, ....... , + 1, ....... n
2 2 R2r0, r1, r23 ways
Total A.P.S ∴ Total number of ways to select r = 5.
 n  n(n – 2) Selection of s as under
= 2 ×  0 + 1 + ....... – 1  =
 2  4
s0s41 way
s1s41 way
Previous Years’ Questions s2s41 way
5 s3s41 way
Sol 1: (C) Here, 47C4 + ∑ 52– j
C3
s4s41 way
j=1

= 47C4 + 51C3 + 50C3 + 49C3 + 47C3 ∴ Total number of ways to select s = 9.

= (47C4 + 47C3) + 48C3 + 49C3 + 50C3 + 51C3 Similarly, the number of ways to select t = 5.
∴ Total number of ways = 5 × 9 × 5 = 2Sol.25
(using nCr + nCr–1 = n+1Cr)

= (48C4 + 48C3) + 49C3 + 50C3 + 51C3 Sol 6: (C) Arrange the letters of the word COCHIN as in
the order of dictionary CCHINO.
= (49C4 + 49C3) + 50C43 + 51C3
Consider the words starting from C.
= (50C4 + 50C3) + 51C3 = 51C4 + 51C3 = 52C4
There are 5! Such words. Number of words with the two
C’ s occupying first and second place = 4!.
Sol 2: (D) Since, the first 2 women select the chairs
amongst 1 to 4 in 4P2 ways Sol 7: (C) There are two possible cases
Now, from the remaining 6 chairs, three men could be
arranged in 6P3. Case-I Five 1’s , one 2’s, one 3’s
7!
∴ Total number of arrangements=4P2×6P3. Number of numbers = = 42
5!
Sol 3: (A) Since, a five digits number is formed using Case-II Four 1’s three 2’s
the digits {0, 1, 2, 3, 4 and 5} divisible by 3 ie, only 7!
possible when sum of the digits is multiple of three. Number of numbers = = 35
4! 3!
Case I : Using digits 0, 1, 2, 4, 5 Total number of numbers = 42 + 35 = 77
Number of ways = 4 × 4 × 3 × 2 × 1 = 96
Case II : Using digits 1, 2, 3, 4, 5 JEE Advanced/Boards
Number of ways = 5×4×3×2×1 = 120
∴ Total number formed =120+96=216 Exercise 1

Sol 4: (A) Since, 240 = 24 × 3 × 5 Sol 1:


∴ Total number of divisors = (4 +1) (2) (2) = 20
1/2/3/4/5/6
1/2/3
Out of these 2, 6, 10, and 30 are of the fomr 4n + 2. 1/2/3/4/54
Therefore, (a) is the answer. 1/2/3/4

3
× 3
× 3 ×3 × 2 = 81 × 2 = 162
Sol 5: (C) Since, r, s, t are prime numbers. Filling first last two places
4 places
∴ Selection of p and q are as under
5 . 5 8 | Permutations and Combinations

5! 120 Sol 8: n(A) – 2n(A ∩ Β) + 2n(A ∩ B ∩ C)


Sol 2: 9C1 × 8C2 × = 9 × 28 × = 7560
2! 2! 84 = 8! – 2 × 7! + 6! = (56 + 14 + 1
6! = 43 6!
Sol 3: 31 32 33 ........ 351 ........ 398 399 3100 3101
Fix the middle element of G.P. the find the number of
Sol 9: 9! – n(A ∪ B ∪ C)
G.P.S possible
0 +1 + 2 +......+ 49 +50 + 49 +.....+ 1 + 0 = 2500 360
Sol 10: Should be an integer.
k
Sol 4: 3/5 3/5 ∴ k is a factor of 360
4
360 = 32 × 23 × 5

2 8 7 4 8 7 4 Total factors = 4 × 3 × 2= 24

2 × 8 × 7 × 4 =448 8 × 7 × 5 = 280
Sol 11: (i) 6C4 = 15
0/2
(ii) All, 4 diff. 6C4 = 15
6 1 6
2diff, 2Alike 6C2 × 4C1 = 60
7 4 2 7 5 2 alike, 2alike6C2 = 15
1 diff, 3alike 2 × 6C2 = 30
7 × 4 = 282 × 7 × 5 = 70
4 alike 6C1 = 6
Total = 826
126
Sol 5: Case-I: 2 2 2 (iii)3 different flavours = 60
6! (iv) 2 or 3 different flavours =60 +15 + 30 = 105
= 90
(2!)3
Sol 12: (a) x = 21600
Case-II: 1 2 3
6! x = 63 × 100 = 25 33 52
× 3! = 360
1! 2! 3! No. of divisors = 6 × 4 × 3 = 72

Case-III: 0 3 3 Sum = (20 + 21+....+25)

6! (30+31 +....33)(50+51+52)= 60 × 40 × 31 = 78120


× 3! = 60
(3!)2 2! (b) x = 7056

Total = 90 + 360 + 60 = 510 x = 24 × 32 × 72


Total factors = 5 × 3 × 3 = 45
45 + 1
A Answer = = 23
2
Sol 6:
B C (c) 300300 = 7 × 11 × 13 × 102 × 3
= 22 × 3 × 52 × 7 × 11 × 13
n(A) = 8! = n(B) = n(C) 26
= 32
2
n(A ∩ B) = n(B ∩ C) = n(A ∩ C) = 7!
(d) 1010 157 1811
n(A ∩ B ∩ C) = 6!
210 × 510, 37 × 57, 211 × 322
Sol 7: 3 × n(A ∩ B) – 3n(A ∩ B ∩ C) HCF of 1010, 157 & 1811 = 1
3 × 7! –3 × 6! HCF of 1010 & 157 = 57
= (63 – 3)6! = 60(6!) HCF of 1010 & 1811 = 210
M a them a ti cs | 5.59

HCF of 157 & 1811 = 37 Sol 20: Total wins = Total matches = 10 × 8C2
Total divisors = (11×11+8×8+12×23) – (8+11+8)+1=435 Wins of champion = A + 7d
5 8
[2A + 7d] = 280
Sol 13: ∑ (5 Cx )3 = 1 + 5 3
+ 103 + 103 + 53 + 1 = 2252 2
x =0
A = 70 – 7d
2
Sol 14: 18C9 – 10C5 × 8C4= 48620 – 252 × 70 = 30980 Now d ≠ 0 d = 2
A = 28
Sol 15: 52 – 1 = 24 ∴ 24C2 triangles
Wins of champion = 28 + 7 × 2 = 42
Sol 16: x + y + 2 + w = 13
S1 S2 S3 S4 S5
x ≤ 5y ≥ 2 Sol 21:
10
C2 + C2 +.... C2= 45 + 55 + 66 + 78 + 91 + 105 = 440
11 15
S1 = (1 + 2 + 4 + 5) × 4! = 288
S2 = S3 = S4 = S5 =(1 + 2 + 4 + 5) × 3 × 3! = 216
Sol 17: 2 digits should be odd or
Sum = S1 × 104 + S2(103 + 102 + 10 + 1)
2 digits should be even for average to be integer
= 288 × 104 + 216(1111) = 3119976
111 222 ......... 999 each repeats 3 times

C2× 5 C1 ×5 C1 +3C2× 4 C1 ×4 C1 + 2 C1 × 4 × 3 – 2
3
× 9 = 121 Sol 22: (a) 8C3 × 5C3 × 3! = 1680
    repetition 
odd even/0 zero 5! 5!
(b) 3C1 2 + 3C 2 × 3 × 2 × × 2!
3! 2! 1! 2! 2!
Sol 18: (a) 1 element 12C1 = 12 = 60 + 1080 = 1140
2 elements C2 = 45 10
a, b, c distinct
3 elements 8C3 = 56
Sol 23: {1, x, x} (5C1 + 5C2) = 15
4 elements 6C4 = 15
5! 5!
5 elements 0 {x, x, x} + = 25
1! 1! 3! 2! 1! 2! 2! 2!
Total = 128
Total = 15 + 25 = 40
(b) Select 3 elements
Each is this element is either present in S, T or both Sol 24: No. of integers
S and T
2 digit no. 89, 98 2
∴ Total = 6C3 × 33 = 540
3 digit no.
λ = 54
Zero included 890, 809 4

Sol 19: Sum at first places can be 6 at max. 980, 908

4 3 1 2 5 not allowed Zero excludes


First 2 digits Last 2 digits Permutations 2× 3C2 + 7C1 × 3!48
Sum 6: 15 43 4 4 digit no.
24 53 4 Tow zeroes 3C2 × 26
Sum5: 14 53/52 8 One zero 3C1 × (7C1 × 3! + 6)144
23 54/51 8 No zero 2 ×4C2×7×7+2×4C2×2×7 770+2×4C3+4C2= 974
Sum4: 13 54/52/42 12
Sum3: 12 54/53 12 Sol 25: A – 1B – 9 1
   Total 48 A – 2 B – 8 8C 1
5 . 6 0 | Permutations and Combinations

A – 3B – 78C2 Sol 2: (C)


A – 4B – 68C3
4
2 C2 2
A – 5B – 50
N = 2(1 + 8C1 + 8C2 + 8C3) 2 × 4C2 × 2 = 24
N = 186
Sol 3: (C) 20C2 – 10 – 5 × 9 = 190 – 10 – 45= 135
Sol 26: (i) MU LTI PLE
Sol 4: (B) r = 9C2 × 9C2 = 362
8!
– 1 = 3359 8 × 9 × 17
3! 2! s = 82 + 72 + 62 +........ 12 =
6
5!
(ii) – 1 = 59 s 17
2! =
r 108
5!
(iii)3! × – 1 = 359
2!
Sol 5: (A) 200,000,000
9!  28 – 1 
Sol 27: 3 digits 5C3 × = 16,800 28 + 28 + 27 + 26 +........ 21 = 28 + 2.   = 3. (2)8 –2
3! 3! 3!  2–1 
 
 9! 9! 
2 digits 2 × 5C2 ×  +  = 4,200
 3! 6! 4! 5!  y counters
1 digit 5C1 = 5 Sol 6: (A) ............
Total = 21005 x x x x
N
= 4201 xy – y
5

Sol 7: (B) M – n = 10C5 – 9C5 = 9C5


Sol 28: 3C2 × 9 × 8 + 9 × 3C2 × 8 × 8 + 9 × 3 × 9 × 8= 3888

Sol 8: (A) N1 = m+nC3 – mC3 – nC3


Sol 29: (i) 4 × 4! + 2 × 3 × 4! = 240
(ii) 4 × 64 × 3 = 15552 N2 = m+n+1C3 – m+1C3 – n+1C3

N1 m+n–2
=
N2 m+n
Exercise 2
Single Correct Choice Type Sol 9: (B) 10000 to 99999

90000
Number of numbers = = 45000
Sol 1: (D) We have 0 + 1 + 2 + 3 … + 8 + 9 = 45 2
To obtain an eight digit number exactly divisible by 9,
we must not use either (0, 9) or (2, 7) or (3, 6) or (4, 5). Sol 10: (B) AB D C E F G H
[Sum of the remaining eight digits is 36 which is exactly 7!
divisible by 9.] 2× = 5040
2!
When, we do not use (0, 9), then the number of required
8 digit number is 8!. 7!
Sol 11: (B) × 3! = 630
1! 2! 4!
When, one of (1, 8) or (2, 7) or (3, 6) or (4, 5) is not used,
the remaining digits can be arranged in 8! – 7! ways as
Sol 12: (A) 9C4 = 126
0 cannot be at extreme left.
Hence, there are 8! + 4(8! – 7!) = (36) (7!) numbers in
the desired category.
M a them a ti cs | 5.61

Sol 13: (D) Case-I: 1 3 4 = 24 + 15 + 8 + 3 = 50


8! Rectangles which are not squares = 210-50 = 160
× 3!
1! 3! 4!
Sol 20: (B)
Case-II: 2 3 3
 {15 + 10 + 6 + 3 + 1} + 
8!  
× 3!
2! 3! 3! 2! 1 _ _ _ _ _ _  {10 + 6 + 3 + 1} =70
 
 + {6 + 3 + 1} + {3 + 1} + 1 
Case-III: 2 2 4  
2 3 _ _ _ _ _ 10 + 6 + 3 + 1 = 20 + 70 = 90
8!
× 3! 2 4 5 _ _ _ _ 3 + 2 + 1 = 6 + 90 = 96
2! 2! 4! 2!
2 4 6 7 8 = 97th
Total = 4620 = 22 × 7P3

Sol 14: (D) Pack 1 Pack 2 Sol 21: (B) nCn + n+1Cn + n+2Cn + ........ n+mCn

Peaches Apples Peaches Apples =nC0+n+1C1+n+2C2+.....n+mCm–n+1C0+ n+1C0 = n+m+1Cm

3122
1 1 m+n
Sol 22: (A) × × Cm(m – 1)(n – 1)!
5
C3 × 3C1 = 30 2 2
1 (m + n)!
Sol 15: (D) = ×
4 mn
× × × ×
n–3
1 x x
(n – 2) choices. select 3 Sol 23: (D) x 1 x 3
C1 = 3
Pn = C3
n–2
x x 1
Pn+1 – Pn = n–1C3 – n–2C3 = n–2C2 = 15 1 x x
n–2=6⇒n=8 x 0 x 3C 1 × 3C 2 = 9
x x 0
Sol 16: (A) n(2) = 50n(2 ∩ 3) = 16
Total = 12
n(3) = 33 n(3 ∩ 5)=6 n(2 ∩ 3 ∩ 5) =5
n(5) = 20n(2 ∩ 5) = 10 Sol 24: (C) 5 3 or 4 4
n(2 ∪ 3 ∪ 5)=(50 + 33 + 20) – (16 + 6 + 10)+3 8
C5 + 8C4 = 126
n( 2 ∪ 3 ∪ 5 ) = 100 – 74 = 26
Sol 25: (A) 100 = (80 + 50 + 30) – n + 5
6! n = 65
Sol 17: (B) 5C1 × = 1800
2! People reading exactly 2 magazines
Choose subject with two periods and then arrange.
= 65 – 3 × 5 = 50

Sol 18: (B) • • • • Sol 26: (B) 2(5! – 4! × 2) = 144


4
C2 × 2 × 2 × 10
Sol 27: (B)
240 Degree of 5 in prime factorization of 2002C1001 = 1
2002
C1001 is clearly divisible by 2
Sol 19: (A) Rectangles = 7C2 × 5C2 = 21 × 10 = 210
Squares = 6 × 4 + 5 × 3 + 4 × 2 + 3 ×1
5 . 6 2 | Permutations and Combinations

Sol 28: (C) No. of triangles = nC3 – [(n(n –4)) + n] Sol 37: (C) S1 S2 S3
n(n – 1)(n – 2)
= – [(n2 – 3n)] = 30 ⇒ n = 9
3! S1 = S3 = (2 + 4 + 6 + 8) × 10 = 200
S2 = (0 + 1 + 2 + ....... 9) × 4 = 180
Sol 29: (B) 11C3 – 5C3 + 1 = 156
Sum = 200 + 180 × 10 + 200 × 100 = 22000
Sol 30: (C)
Sol 38: (A) An = nC25 × 25!
2 5 5 10
An – An–1 = 25! (nC25 – n–1C25) = 25!(n–1C24)

3! + 3 × 2 × 2! 4C3 × 3! = 18 = 24 n = 50

Total no.s = 18 + 24 = 42
Sol 39: (C) (x1 + x2 + x3) (y1 + y2) = 77 = 7 × 11

Sol 31: (C) Pn – Qn x1 + x2 + x3 = 4 & x1 + x2 + x3 = 8

The difference in Pn & Qn is the number of ways in which y1 + y2 = 9y1 + y2 = 5


first and last person of the row is selected No. of possible solutions
Pn – Qn = n – 4 = 6 = 6C2 × 10C1 + 10C2 × 6C1 = 15 × 10 + 45 × 6
n = 10 = 150 + 270 = 420

Sol 32: (C) m = 10C4 × 4! Sol 40: (B) m = 34 – 3 = 78


n = C4; m = 24 n
10
n = 34 – [3(24 - 2) + 3] = 81 – 45 = 36

Sol 33: (C) No. of ways of selecting a committee of 2W m 78 13


Hence, = =
and 3M from 5W and 6M n 36 6
= 4C1 × 4C2 + 5C2 × 5C2 = 124
Sol 41: (A) Sum of digits at units place
Sol 34: (D) ABC, ACD, ABD = (1 +2 +.....+ 9) × 3 × 9 = 45 × 27 =1215
3! + 3! + 3! = 18 Sum of digits at ten’s place
= (1+ 4 + 9) × 9 × 9 = 14 × 81 = 1134
Sol 35: (D) Ways in which 2 are neighbours
Sum of digits at hundred’s place
= 2 × 97 + 97 × 96 = 98 × 97
= (1 + 2 + ...... 9 ) × 3 × 9 = 45 × 27 = 1215
ways in which all 3 are neighbours = 98
Required ways = 100C3 – 982 = 152096 = 98C3 1215
1134 ×
1215 × ×
Sol 36: (D) Make 3 groups of boys 1, 1, 2 Sum =
134055
4!
× 31
! ways to distribute
2! (11)2 2! Sol 42: (D) 3 possible distribution of wards for each
subject.
Identical marbles distribution
1 1 4
1 2 2– 3 ways
1 2 3
1 1 3 – 3 ways
4! 2 2 2
Total ways = × 3! × 6 = 216  6! 6! 6! 
2! 2! Total ways = 3!  + +  = 540
 4! (1!)2 2! 1! 2! 3! (2!)3 3! 
 
M a them a ti cs | 5.63

Sol 43: (C) M = L × 2! 6!


(C) = 90
L=N ∴ 2L = M = 2N (2!)3

6!
(D) × 3! = 90
Sol 44: (C) There are 11 position (2!)3 3!
At the 6th position A should be present. In the 5 positions
left to 6th positions 2positions will have A. Comprehension Type
5
C2 ways Sol 52: (C) M I(4) S(4)P(2)
(i) 2C1 × 3C1 = 6
Sol 45: (D) 9C2 × 7C3 × 4C4
(ii) 2C1 × 2C1 = 4
9!
(A)
2! 3! 4! (iii) 2C1 × 3C1 = 6
9! (iv) 3C2 × 2 = 6
(D)
2! 3! 4!
(v) 3C1 = 3
Adding all these, we get = 25
Multiple Correct Choice Type
Sol 53: (B) Total ways in which all vowels are together
Sol 46: (A, B, D) n–1Cr–1 + n–1Cr = nCr
11! 8! 8!  165  8!.161
= – =  – 1 =
Sol 47: (B, C) 2 – 1 10 4! 4! 2! 4! 2! 4! 2!  4  4 4! 2!

Sol 48: (B, C, D) 2 2 1  10! 


Sol 54: (B) 6C2 × 8C4 × 5= 1 ×  4! 4! 
 
5! 120
× 3! = × 6 = 90
(2!) 2
1! 2! 8 16!
Sol 55: (A) No of ways =
(4!)5
Sol 49: (A, B) Let k = 2n + 1, then 2n+1Cr is maximum (1 × 3 × 5........ × 15) × 28 × 81
when r = n. Also 2n+1Cn = 2n+1Cn+1. Thus, kCr is maximum =
(4!)5
1 1
when
= r (k − 1) or =r (k + 1)
2 2 8
28 × 8 × 7 × 6 × 5 × 4!
= ∏ (2r – 1) ×
(4!)5
r =1
Sol 50: (B, D) (A) 4 vowels, 7 consonants
7
C2 × 6! = 3.7!
8
28 × 48 35 8
=35 ∏ (2r – 1) × = ∏ (2r – 1)
(24)4 27
15 r =1 r =1
(B) n = no of white balls
n!(15 – x)!
12! 12 × 11! 11!
12! Sol 56: (C) × 4!= =
(C) (3!) 4
4! 6 4 108
4! 5!

(D) 35
Match the Columns

Sol 51: (C, D) BULBUL Sol 57: A → r; B → s; C → q; D → p


6! (A) nCm (B) mn (C) mCn (D) nm
= 90
2! 2! 2!

(A) 4C2 × 6C2 = 6 × 15 = 90 Sol 58: A → t; B → r; C → p; D → q; E → s

6! (A) x1, x2, x3 & x4 are the students watching a particular


(B) = 15 movie
3
(2!) 3!
5 . 6 4 | Permutations and Combinations

x2 + x2 + x3 + x4 = 10 Number of ways = 3C3.3C3 = 1


xi ≥ 1 ∴ Total number of ways = 16 + 324 + 144 + 1 =485
9×8×7
9
C3 = 84
6 Sol 3: Case-I: When one black and two others ball S
are drawn
⇒ number of ways = 3C1 . 6C2 = 45
Case-II: When two black and one other balls are drawn
(B) T = 9C3 – 4 = 80
⇒ Number of ways = 3C2.6C1 = 18
S = 8C2 = 28
Case-III : When all three black balls are drawn
T – S = 52
⇒ Number of ways = 3C3 = 1
(C) 1 + 5C2 = 11
∴ Total number of ways = 45 + 18 + 1 = 64
(D) 12 = 1 × 2 × 2 × 3 = 1 × 1 × 4 × 3 = 1 × 1 × 2 × 6

4! 4! 4! Sol 4: Let the two sides be A and B. Assume that four


+ + = 36
2! 2! 2! particular guests wish to sit on side A. Four guests who
wish to sit on side A can be accommodated on nine
(E) 5C2 × 3C2 × 2 = 10 × 3 × 2 = 60
chairs in 9P4 ways and there guests who wish to sit on
M1M2 M1 M2 side B and be accommodated in 9P3 ways.
F 1 F 2 F 2F 1 Now, the remaining guests are left who can sit on 11
chairs on both the sides of the table in (11!) ways.
Hence, the total number of ways in which 18 persons
Previous Years’ Questions can be seated = 9P4 × 9P3 × (11)!.

Sol 1: Since, each box can hold five balls. Sol 5: There are 9 women and 8 men. A committee of
∴ Number of ways in which balls could be distributed 12, consisting of at least 5 women, can be formed by
so that none is empty are (2, 21) or (3, 1, 1). choosing:

ie, (5C2 3C2 1C1 + 5C3 2C1 1C1) × 3! (i) 5 women and 7 men
(ii) 6 women and 6 men
= (30 + 20) × 6 = 300
(iii) 7 women and 5 men
Sol 2: The possible cases are (iv) 8 women and 4 men

Case-I: A man invites 3 ladies and women invites 3 (v) 9 women and 3 men
gentlemen ∴ Total number of ways forming the committee
Number of ways = 4C3 . 4C3 = 16 = 9 C 5 × 8C 7 + 9C 6 × 8C 6 + 9C 7 × 8C 5 + 9C 8 × 8C 4 + 9C 9 × 8C 3
Case-II: A man invites (2 ladies, 1 gentlemen) and = 126 × 8 + 84 × 28 + 36 × 56 + 9 × 70 + 1 × 56 = 6062
women invites (2 gentlemen, 1 lady). (i) Clearly, women are in majority in (iii), (iv) and (v)
Number of ways cases as discussed above.

= (4C2.3C1) (3C1.4C1) = 324 (ii) So, total number of committees in which women are
in majority
Case-III: A man invites (1 lady, 2 gentlemen) and
= 9C 7 × 8C 5 + 9C 8 × 8C 4 + 9C 9 × 8C 3
women invites (2 ladies, 1 gentleman).
= 36 × 56 + 9 × 70 + 1 × 56 = 2702
Number of ways
Clearly, men are in majority in only (i) case as discussed
= (4C1.3C2).(3C2.4C1)= 144 above.
Case-IV: A man invites (3 gentlemen) and women So, total number of committees in which men are in
invites (3 ladies). majority
= 9C5 × 8C7 = 126 × 8 = 1008
M a them a ti cs | 5.65

Sol 6: A → p; B → s; C → q; D → q

(A) If ENDEA is fixed word, then assume this as a single


letter.
Total number of letters = 5
Total number of arrangements = 5!

(B) If E is at first and last places, then total number of


7!
permutation of = 21 × 5!
2!
(C) If D, L, N are not in last five positions
← D, I, N, N → ← E, E, E, A, O →
Total number of permutation
4! 5!
= × = 2 × 5!
2! 3!
(D) Total number of odd position = 5
5!
Permutation of AEEEO are
3!
Total number of even positions = 4
4!
Number of permutations of N, N, D, L =
2!
5! 4!
Hence, total number of permutation = × = 2 × 5!
3! 2!
2017-18 100 &
op kers
Class 11 T
By E ran culty
-JE Fa r
IIT enior emie .
S fP r es
o titut
Ins

MATHEMATICS
FOR JEE MAIN & ADVANCED
SECOND
EDITION

Exhaustive Theory
(Now Revised)

Formula Sheet
9000+ Problems
based on latest JEE pattern

2500 + 1000 (New) Problems


of previous 35 years of
AIEEE (JEE Main) and IIT-JEE (JEE Adv)

5000+Illustrations and Solved Examples


Detailed Solutions
of all problems available

Topic Covered Plancess Concepts


Tips & Tricks, Facts, Notes, Misconceptions,
Complex Number Key Take Aways, Problem Solving Tactics

PlancEssential
Questions recommended for revision
6. COMPLEX NUMBER

1. INTRODUCTION
The number system can be briefly summarized as N ⊂ W ⊂ I ⊂ Q ⊂ R ⊂ C , where N, W, I, Q, R and C are the
standard notations for the various subsets of the numbers belong to it.
N - Natural numbers = {1, 2, 3 …. n}
W - Whole numbers = {0, 1, 2, 3 …. n}
I - Integers = {….2, -1, 0, 1, 2 …..}
1 3 
Q – Rational numbers =  , ..... 
2 5 
IR – Irrational numbers = { 2, 3, π}
C – Complex numbers
A complex number is generally represented by the letter “z”. Every complex number z, can be written as, z= x + i y
where x, y ∈ R and i= −1 .
x is called the real part of complex number, and
y is the imaginary part of complex number.
Note that the sign + does not indicate addition as normally understood, nor does the symbol “i” denote a number.
These are parts of the scheme used to express numbers of a new class and they signify the pair of real numbers
(x, y) to form a single complex number. Y
Imaginary axis P(x,y)

X’ X
O Real axis

Y’
Figure 6.1: Representation of a complex number on a plane

Swiss-born mathematician Jean Robert Argand, after a systematic study on complex numbers, represented every
complex number as a set of ordered pair (x, y) on a plane called complex plane.
All complex numbers lying on the real axis were called purely real and those lying on imaginary axis as purely
imaginary.
Hence, the complex number 0 + 0i is purely real as well as purely imaginary but it is not imaginary.
6 . 2 | Complex Number

Note
Purely real if y = 0

z = x+iy is Purely imaginary if x = 0

x0
Complex number if
y0

Figure 6.2: Classification of a complex number

(a) The symbol i combines itself with real number as per the rule of algebra together with

i2 = −1 ; i3 = −i ; i4 = 1 ; i2014 = −1 ; i2015 = −i and so on.


In general, i4n = 1 , i4n+1 = i , i4n+ 2 = −1 , i4n+3 = −i , n ∈ I and i4n + i4n+1 + i4n+ 2 + i4n+3 =
0
Hence, 1 + i1 + i2 + ……. + i2014 + i2015 = 0
(b) The imaginary part of every real number can be treated as zero. Hence, there is one-one mapping between
the set of complex numbers and the set of points on the complex plane.

PLANCESS CONCEPTS
Complex number as an ordered pair: A complex number may also be defined as an ordered pair of real
numbers and may be denoted by the symbol (a, b) . For a complex number to be uniquely specified, we
need two real numbers in a particular order.
Vaibhav Gupta (JEE 2009, AIR 54)

2. ALGEBRA OF COMPLEX NUMBERS


(a) Addition: (a + ib) + (c + id) = (a + c) + i(b + d)
(b) Subtraction: (a + ib) − (c + id) = (a − c) + i(b − d)
(c) Multiplication: (a + ib) (c + id) = (ac − bd) + i (ad + bc)
(d) Reciprocal: If at least one of a, b is non-zero, then the reciprocal of a + ib is given by
1 a − ib a b
−i
a + ib (a + ib) (a − ib) 2 2
a + b2
2
= = a +b

(e) Quotient: If at least one of c, d is non-zero, then quotient of a + ib and c + id is given by

a + ib (a + ib) (c − id) (ac + bd) + i (bc − ad) ac + bd bc − ad


= = = +i
c + id (c + id) (c − id) 2
c +d 2
c2 + d2 c2 + d2

(f) Inequality in complex numbers is not discussed/defined. If a + ib > c + id is meaningful only if b = d = 0.


However, equalities in complex numbers are meaningful. Two complex numbers z1 and z2 are said to be equal
if Re (z1 ) = Re(z 2 ) and Im (z1 ) = Im(z 2 ). (Geometrically, the position of complex number z1 on complex plane)

(g) In real number system if p2 + q2 = 0 implies, p= 0= q . But if z1 and z 2 are complex numbers then z12 + z 22 =
0
does not imply z=1 z=2 0 . For e.g. z1 = i and z 2 = 1 .
However if the product of two complex numbers is zero then at least one of them must be zero, same as in
case of real numbers.
x, if x ≥ 0
(h) In case x is real, then | x | =  but in case of complex number z, | z | means the distance of the
point z from the origin. −x, if x < 0
M a them a tics | 6.3

PLANCESS CONCEPTS

• The additive inverse of a complex number z= a + ib is −z (i.e. − a − ib) .


1
• For every non-zero complex number z, the multiplicative inverse of z is .
z
• | z | ≥ Re(z) ≥ Re(z) and | z | ≥ Im(z) ≥ Im(z) .

z
• is always a uni-modular complex number if z ≠ 0 .
|z|
Vaibhav Krishnan (JEE 2009, AIR 22)

Illustration 1: Find the square root of 5 + 12i.  (JEE MAIN)

Sol: z= 5 + 12i
Let the square root of the given complex number be a + ib. Use algebra to simplify and get the value of a and b.
Let its square root = a + ib ⇒ 5 + 12i = a2 − b2 + 2abi
⇒ a2 − b2 =
5 … (i)
12 
⇒ 2ab = … (ii)
⇒ (a2 + b2 ) 2 = (a2 − b2 ) 2 + 4a2b2 ⇒ (a2 + b2 ) 2 = 25 + 144 =169 ⇒ a2 + b2 =
13  … (iii)
2 2
(i) + (iii) ⇒ 2a =
18 ⇒ a =
9 ⇒ a =± 3
If a = 3 ⇒ b =
2 If a = −3 ⇒ b =−2
∴ Square root = 3 + 2i, −3 − 2i ∴ Combined form ± (3 + 2i)

If z (x, y) ∈ C . Find z satisfying z 2 × (1 + i) = ( −7 + 17i) .


Illustration 2: = (JEE MAIN)

Sol: Algebra of Complex Numbers.


(x + iy) 2 (1 + i) =−7 + 17i
⇒ (x2 − y 2 + 2xyi) (1 + i) =−7 + 17i ; x2 − y 2 + i(x2 − y 2 ) + 2xyi − 2xy =−7 + 17i
⇒ (x2 − y 2 − 2xy) + i (x2 − y 2 + 2xy) =
−7 + 17i ⇒ x= 3, y= 2 ⇒ x =−3, y =−2
⇒ z =−3 + i ( −2 ) =−3 − 2i

Illustration 3: If x2 + 2(1 + 2i) x − (11 + 2i) =


0 . Solve the equation.  (JEE ADVANCED)

Sol: Use the quadratic formula to find the value of x.


−2 (1 + 2i) ± 4 − 16 + 16i + 44 + 8i
∴x =
2

⇒ 2x =( −2)(1 + 2i) ± 32 + 24i

⇒ x =( −1)(1 + 2i) ± 8 + 6i = −1 − 2i ± (3 + i) ; x = 2 – i, –4 – 3i
6 . 4 | Complex Number

Illustration 4: If f(x) = x 4 − 4x3 + 4x2 + 8x + 44 . Find f(3 + 2i) .  (JEE ADVANCED)

Sol: Let x = 3+2i, and square it to form a quadratic equation. Then try to represent f(x) in terms of this quadratic.
x= 3 + 2i
⇒ (x − 3) 2 =−4 ⇒ x2 − 6x + 13 =
0
x 4 − 4x3 + 4x2 + 8x + 44
= x2 (x2 − 6x + 13) + 2x3 − 9x2 + 8x + 44
= x2 (x2 − 6x + 13) + 2(x3 − 6x2 + 13x) + 3(x2 − 6x + 13) + 5
⇒ f(x) ⇒ f(x) =
5

3. IMPORTANT TERMS ASSOCIATED WITH COMPLEX NUMBER


Three important terms associated with complex number are conjugate, modulus and argument.
(a) Conjugate: If z = x + iy then its complex conjugate is obtained by changing the sign of its imaginary part and
denoted by z i.e. z = x – iy (see Fig 6.3).
The conjugate satisfies following basic properties
(i) z + z = 2Re (z) Y
(ii) z − z = 2i Im (z)
z = x+iy
z x2 + y 2
(iii) z =
(iv) If z lies in 1st quadrant then z lies in 4th quadrant and – z in
the 2nd quadrant. X’ X

(v) If x + iy = f (a + ib) then x – iy = f (a – ib)


For e.g. If (2 + 3i)3 = x + i y then (2 – 3i)3 = x – iy Conjugate of z , z = x+iy
Image of z in x-axis
and, sin(α + iβ) = x + iy ⇒ sin(α − iβ) = x – iy
Y’
(vi) z + z = 0 ⇒ z is purely imaginary
Figure 6.3: Conjugate of a complex number
(vii) z − z =0 ⇒ z is purely real

(b) Modulus: If P denotes a complex number z = x + iy then, OP = | z | = x2 + y 2 . Imaginary axis


Geometrically, it is the distance of a complex number from the origin.
Hence, note that | z | ≥ 0 , | i | = 1 i.e. −1 = 1. P(x, y)
r = lzl
All complex number satisfying |z| = r lie on the circle having centre at origin and

radius equal to ‘r’.
O Real axis
(c) Argument: If OP makes an angle θ (see Fig 6.4) with real axis in anticlockwise Figure 6.4: Modulus of a
sense, then θ is called the argument of z. General values of argument of z are complex number
given by 2nπ + θ, n ∈ I . Hence any two successive arguments differ by 2π .
Note: A complex number is completely defined by specifying both modulus and argument. However for the
complex number 0 + 0i the argument is not defined and this is the only complex number which is completely
defined by its modulus only.

(i) Amplitude (Principal value of argument): The unique value of θ such that −π < θ ≤ π is called
principal value of argument. Unless otherwise stated, amp z refers to the principal value of argument.
(ii) Least positive argument: The value of θ such that 0 < θ ≤ 2π is called the least positive argument.
y
If φ =tan−1 .
x
M a them a tics | 6.5

PLANCESS CONCEPTS

y y
• If x > 0, y > 0 (i.e. z is in first quadrant), then arg z = θ = tan−1 .
x -
y
• If x < 0, y > 0 (i.e. z is in 2nd quadrant, then arg z = θ = π − tan−1
x

y x
• If x < 0, y < 0 (i.e. z is in 3rd quadrant), then arg z = θ = − π + tan−1 O
x -+ -

y
• If x > 0, y < 0 (i.e. z is in 4th quadrant), then arg z = θ = − tan−1
x Figure 6.5

0, if x > 0
• If y = 0 (i.e. z is on the X-axis), then arg (x + i0) = 
 π, if x < 0
π
 , if y > 0
• If x = 0 (i.e. z is on the Y-axis), then arg (0 + iy) =  2
 3π , if y < 0
 2

Shrikant Nagori (JEE 2009, AIR 30)

Illustration 5: For what real values of x and y, are −3 + ix2 y and x2 + y + 4i complex conjugate to each other?
 (JEE MAIN)

Sol: As −3 + ix2 y and x2 + y + 4i are complex conjugate of each other. Therefore −3 + ix2 y = x2 + y + 4i .
−3 + ix2 y = x2 + y − 4i
Equating real and imaginary parts of the above question, we get
– 3 = x2 + y ⇒ y =−3 − x2  … (i)
and x2 y = – 4  … (ii)
2
Putting the value of y = −3 − x from (i) in (ii), we get
−3 ± 9 + 16 −3 ± 5 2 −8
x2 ( −3 − x2 ) = – 4 0 ⇒ x2 =
⇒ x 4 + 3x2 − 4 = = = , =1, − 4
2 2 2 2
∴ x2 =1 ⇒ x =±1
Putting value of x = ± 1 in (i), we get y = – 3 – (1)2 = – 3 – 1 = – 4
Hence, x = ± 1 and y = – 4.

1+i 1−i
Illustration 6: Find the modulus of − . (JEE MAIN)
1−i 1+i

Sol: As | z | = x2 + y 2 , using algebra of complex number we will get the result.

1+i 1−i (1 + i)(1 + i) (1 − i)(1 − i)


Here, we have − = −
1−i 1+i (1 − i) (1 + i) (1 + i) (1 − i)

1 + i2 + 2i 1 + i2 − 2i 1 − 1 + 2i 1 − 1 − 2i 2i ( −2i) 1+i 1−i


= − = − = − = i + i = 2i, ∴ ⇒ − =| 2i | =2 .
1+1 1+1 2 2 2 2 1−i 1+i
6 . 6 | Complex Number

Illustration 7: Find the locus of z if | z – 3 | = 3| z + 3|.  (JEE MAIN)

Sol: Simply substituting z = x + iy and by using formula | z | = x2 + y 2 we will get the result.
Let z = x + iy
| x + iy – 3 | = 3 | x + iy + 3 | | (x – 3) + iy | = 3 | (x + 3) + iy |

(x − 3) 2 + y 2 = 3 (x + 3) 2 + y 2 ; (x − 3) 2 + y 2 = 9(x + 3) 2 + 9y 2 .

β−α
Illustration 8: If α and β are different complex numbers with | β | = 1, then find .  (JEE ADVANCED)
1 − αβ

β−α
Sol: By using modulus and conjugate property, we can find out the value of .
1 − αβ

We have, | β | = 1 ⇒ | β | 2 = 1 ⇒ ββ = 1

Now,
β−α
1 − αβ
=
β−α
ββ − αβ
=
β−α
β (β − α )
=
|β−α|
|β||β−α |
=
1
|β|
= 1. {as | x + iy | =| x + iy |}

Illustration 9: Find the number of non-zero integral solution of the equation | 1 − i | x =.
2x (JEE ADVANCED)

Sol: As | z | = x2 + y 2 , therefore by using this formula we can solve it.

We have, | 1 − i | x =
2x
x x

( ) x
x
⇒  12 + 12  =2 x ⇒ = 2x 2 2x
⇒ 22 = ⇒ 0 ⇒ x =.
= 0
  2
∴ The number of non zero integral solution is zero.

a + ib a2 + b2
Illustration 10: If = p + iq. Prove that = p2 + q2 . (JEE MAIN)
c + id c + d2
2

a + ib
Sol: Simply by obtaining modulus of both side of = p + iq .
c + id
a + ib
We have, = p + iq
c + id

a + ib a2 + b2 a + ib a2 + b2
= ⇒ p + iq = p2 + q2 ; = | p + iq | ⇒ p2 + q2 .
=
c + id c2 + d2 c + id c2 + d2

x y
Illustration 11: If (x + iy) 1/3 = a + ib. Prove that + = 4 (a2 − b2 ) . (JEE ADVANCED)
a b
Sol: By using algebra of complex number. We have, (x + iy) 1/3 = a + ib
x + iy = (a + ib) 3 =a3 + i3b3 + 3a2ib + 3a(ib)2 = a3 − b3i + 3a2bi − 3ab2
x + iy = (a3 − 3ab2 ) + (3a2b − b3 )i ; x = a3 − 3ab2 = a (a2 − 3b2 ) ; y = 3a2b − b3
x y
+ = 4 (a2 − b2 ) .
a b
M a them a tics | 6.7

4. REPRESENTATION OF COMPLEX NUMBER

4.1 Graphical Representation


Every complex number x + iy can be represented in a plane as a point P (x, y). X-coordinate of point P represents
the real part of the complex number and y-coordinate represents the imaginary part of the complex number.
Complex number x + 0i (real number) is represented by a point (x, 0) lying on the x-axis. Therefore, x-axis is called
the real axis. Similarly, a complex number 0 + iy (imaginary number) is represented by a point on y-axis. Therefore,
y-axis is called the imaginary axis.
The plane on which a complex number is represented is called complex Y
Imaginary axis
number plane or simply complex plane or Argand plane (see Fig 6.6). The
figure represented by the complex numbers as points in a plane is known

lm(z) > 0
as Argand Diagram.

4.2 Algebraic Form Re(z) < 0 Re(z) > 0


X’ X
y O Real axis , lm(z) = 0
If z = x + iy; then x2 + y 2 ; z= x − iy , and θ =tan−1  
|z|=
x

lm(z) < 0
Generally this form is useful in solving equations and in problems
involving locus.

4.3 Polar Form Y’


Figure 6.6: Graphical representation
Figure 6.7 shows the components of a complex number along the x and
y-axes respectively. Then
z = x + iy = r (cos θ + isin θ) = r cis θ where | z | = r ; amp z = θ .
Aliter: z = x+iy Y
 
x y
⇒ z= x2 + y 2  +i 
 x2 + y 2 x + y2
2  z(x, y) = x+iy
 

z | z | ( cos θ + i sin=
θ ) r cis θ
r

= r sin

Note: (a) (cis α ) (cis β=
) cis(α + β) X
O r cos
(b) (cis α ) (cis( −β=
)) cis(α − β)
1 Figure 6.7: Polar form
−1
(c) = (cis α ) = cis( −α )
(cis α )

PLANCESS CONCEPTS

The unique value of θ such that −π < θ ≤ π for which=x r cos θ &=
y r sin θ is r (cos θ + i sin θ)
known as the principal value of the argument.
The general value of argument is (2nπ + θ) , where n is an integer and θ is the
principal value of arg (z). While reducing a complex number to polar form, we r c i s θ
always take the principal value. Figure 6.8
The complex number z = r (cos θ + isin θ) can also be written as r cis θ .
Nitish Jhawar (JEE 2009, AIR 7)
6 . 8 | Complex Number

4.4 Exponential Form


Euler’s formula, named after the famous mathematician Leonhard Euler, states that for any real number x,
eix cos x + isinx .
=
Hence, for any complex number
= z r (cos θ + isin θ) , z = reiθ is the exponential representation.
eix + e−ix eix − e−ix
Note: (a) cos x = and sin x = are known as Eulers identities.
2 2i
ex + e− x
(b) cos ix = = cos hx is always positive real ∀ x ∈ R and is > 1.
2
ex − e− x
and, sin ix = i = i sin hx is always purely imaginary.
2

4.5 Vector Representation


The knowledge of vectors can also be used to represent a complex number Y

z = x + iy. The vector OP , joining the origin O of the complex plane to the P (x, y), z = x+iy
point P (x, y), is the vector representation of the complex number z=x+iy,
 
(see Fig 6.9). The length of the vector OP , that is, | OP | is the modulus of z. OP

The angle between the positive real axis and the vector OP , more exactly, the
angle through which the positive real axis must be rotated to cause it to X’
O
X

have the same direction as OP (considered positive if the rotation is Y’
counter-clockwise and negative otherwise) is the argument of the complex Figure 6.9 Vector representation
number z.

 1  1
Illustration 12: Find locus represented by Re   < .  (JEE MAIN)
 x + iy  2
Sol: Multiplying numerator and denominator by x − iy.

 1  1  x − iy  1
We have, Re   < Re  <
 x + iy  2  x2 + y 2  2
 
x 1
⇒ < ⇒ x2 + y 2 − 2x > 0
2
x +y 2 2

Locus is the exterior of the circle with centre (1, 0) and radius = 1.

6π 6π
Illustration 13: If z = 1 + cos + isin . Find r and amp z. (JEE MAIN)
5 5

Sol: By using trigonometric formula we can reduce given equation in the form of
= z r ( cosθ + i sinθ ) .
3π 3π 3π 3π  3π 3π 
z = 2cos2 + 2isin cos = 2cos cos + isin 
5 5 5 5  5 5 

2π  2π 2π  2π  2π 2π  2π 2π
= −2cos  − cos + isin  = 2cos cos − isin  Hence, | z | = 2cos ; amp z = −
5  5 5  5  5 5  5 5
M a them a tics | 6.9

1
Illustration 14: Show that the locus of the point P(ω)denoting the complex number z + on the complex plane is
a standard ellipse where | z | = a, where a ≠ 0, 1. z (JEE ADVANCED)

Sol: Here consider w = x + iy and z = α + iβ and then solve this by using algebra of complex number.
1
Let w = z + where z = α + iβ , α2 + β2 = a2 (as | z | = a)
z
1 α − iβ  α  β   1   1 
x + iy = α + iβ + = α + iβ + =  α +  + i  β −  ∴ x =α  1 +  ; y =β 1 − 
α + iβ 2
α +β 2
 2
a   2
a   2
a   a2 

x2 y2 x2 y2
∴ + = α2 + β2 = a2 ; ∴ + = 1.
2 2 2 2
 1   1   1  1
1 + 2  1 − 2  a +  a − 
 a   a   a  a

5. IMPORTANT PROPERTIES OF CONJUGATE, MODULUS AND ARGUMENT


For z, z1and z 2 ∈ C ,

(a) Properties of Conjugate:

(i) z + z = 2Re (z)


(ii) z − z = 2i Im (z)
(iii) (z) = z

(iv) z1 + z 2 = z1 + z2

(v) z1 − z 2 = z1 − z2
z1 z 2 = z1 · z2
(vi)
 z1  z1
(vii)   = ; z2 ≠ 0
 z 2  z2

(b) Properties of Modulus:

(i) | z | ≥ 0 ; | z | ≥ Re(z) ; | z | ≥ Im(z) ;| z | =


| z |=
| −z |
1
(ii) zz = | z | 2 ; if | z | = 1, then z =
z
(iii) | z1 z 2 | = | z1 | · | z 2 |

z1 | z1 |
(iv) = , z2 ≠ 0
z1 | z2 |
n n
(v) | z | = | z |
2
(vi) | z1 + z 2 | + | z1 − z=
2 |
2
2 | z1 | 2 + | z 2 | 2 
 
(vii) | z1 | − | z 2 | ≤ | z1 + z 2 | ≤ | z1 | + | z 2 | [Triangle Inequality]
6 . 1 0 | Complex Number

(c) Properties of Amplitude:

(i) amp (z1 ·=


z 2 ) amp z1 + amp z 2 + 2kπ, k ∈ I

 z1 
(ii) amp =  amp z 1 − amp z 2 + 2kπ, k ∈ I
 z2 

(iii) amp
= (zn ) n amp(z) + 2kπ , where the value of k should be such that RHS lies in ( −π, π]

Based on the above information, we have the following

•• | Re(z) | + | Im(z) | ≤ 2 | z |

•• | z1 | − | z 2 | ≤ z1 − z 2 ≤ z1 + z 2 . Thus | z1 | + | z 2 | is the greatest possible value of | z1 + z 2 |

and | z1 | − | z 2 | is the least possible value of | z1 + z 2 | .

1 a + a2 + 4 −a + a2 + 4
•• If z + = a, the greatest and least values of |z| are respectively and .
z 2 2

•• | z1 + z12 − z 22 | + | z 2 − z12 − z 22 | = | z1 + z 2 | + | z1 − z 2 |

•• If z1 =z 2 ⇔ | z1 | =| z 2 | and arg z1 =argz 2

•• | z1 + z 2 | = | z1 | + | z 2 | ⇔ arg (z1 ) = arg(z 2 ) i.e. z1 and z 2 are parallel.

•• | z1 + z 2 | =| z1 | + | z 2 | ⇔ arg (z1 ) − arg(z 2 ) =2nπ, where n is some integer.

•• | z1 − z 2 | =| z1 | − | z 2 | ⇔ arg (z1 ) − arg(z 2 ) =2nπ, where n is some integer.


π
•• | z1 + z 2 | = | z1 − z 2 | ⇔ arg (z1 ) − arg(z 2 ) = (2n + 1) , where n is some integer.
2
If | z1 | ≤ 1, | z 2 | ≤ 1, then | z1 + z 2 | ≤ ( | z1 | − | z 2 | ) + ( arg (z1 ) − arg(z 2 ) ) , and
2 2 2
••
2.
| z1 + z 2 | 2 ≥ ( | z1 | + | z 2 | ) − ( arg (z1 ) − arg(z 2 ) )
2

z  z
Illustration 15: If z1= 3 + 5i and z 2= 2 − 3i , then verify that  1  = 1  (JEE MAIN)
z  z
 2 2

Sol: Simply by using properties of conjugate.

z1 3 + 5i (3 + 5i) (2 + 3i) 6 + 9i + 10i + 15i2 6 + 19i + 15( −1) 6 + 19i − 15 −9 + 19i −9 19


= = × = = = = = + i
z2 2 − 3i (2 − 3i) (2 + 3i) 4 − 9i2 4+9 13 13 13 13

 z   9 19  9 19
L.H.S. =  1=
 z   − 13 + 13 i  =
− − i
13 13
 2  

z1 3 + 5i 3 − 5i (3 − 5i) (2 − 3i)
R.H.S. = = = = ×
z2 2 − 3i 2 + 3i (2 + 3i) (2 − 3i)

6 − 9i − 10i + 15i2 6 − 19i + 15( −1) 6 − 19i − 15 9 19  z  z1


= = = = − − i ∴  1  =
4+9 13 13 13  z 2  z2
2
4 − 9i
M a them a ti cs | 6.11

Illustration 16: If z be a non-zero complex number, then show that (z −1 ) = (z)−1 . (JEE MAIN)

Sol: By considering z = a + ib and using properties of conjugate we can prove given equation.

Let z = a + ib Since, z ≠ 0 , we have x2 + y 2 > 0

z −1=
1
=
1
=
1
×
a − ib
z a + ib a + ib a − ib
=
2
a
a +b 2

2
ib
a +b 2
z −1
⇒=
2
a
a +b 2
+ ( )ib
a + b2
2
 … (i)

1 1 1 1 a + ib a b
and (z)−1 = = = = × = +i  … (ii)
z a + ib a − ib a − ib a + ib 2
a +b 2
a + b2
2

From (i) and (ii), we get (z −1 ) = (z)−1 .

(a + i) 2 (a2 + 1) 2
Illustration 17: If = p + iq, then show that p2 + q2 = . (JEE MAIN)
2a − i 4a2 + 1

Sol: Multiply given equation to its conjugate.


(a + i) 2
We have, p + iq =  … (i)
2a − i
 (a + i)2 
Taking conjugate of both sides, we get p + iq =
 
 (2a − i) 
 

(a + i)2   z1  z1  (a − i) 2
⇒ p − iq =    =  ⇒ p − iq = … (ii) using (z
= 2
) z= · z (z) 2 
· z z=
 
(2a − i) 
  z 2  z2  (2a + i)

 (a + i) 2   (a − i) 2 
Multiplying (i) and (ii), we get (p + iq) (p – iq) =   
 2a − i   2a + i 
  
(a2 − i2 ) 2 (a2 + 1) 2
⇒ p2 − i2q2 = ⇒ p2 + q2 = .
4a2 − i2 4a2 + 1

Illustration 18: Let z1 , z 2 , z3 , ……. zn are the complex numbers such that | z1 | = | z 2 | = ……. = | zn | = 1 . If z =
 n  n 1 
 ∑ zk   ∑  then prove that
=  k 1=   k 1 zk 
(i) z is a real number (ii) 0 < z ≤ n2  (JEE ADVANCED)

1  n  n 1 
Sol: Here | z1 | = | z 2 | = ……. = | zn | = 1, therefore zz = 1 ⇒ z = . Hence by substituting this to z =  ∑ zk   ∑ ,
z   z
we can solve above problem. =  k 1=  k 1 k 
1 1 1 
Now, z = (z1 + z 2 + z3 + ……. + zn )  + + ……. + 
 z1 z 2 zn 

= (z1 + z 2 + z3 + ……. + zn ) ( z1 + z2 + ……. + zn ) = (


(z1 + z 2 + z3 + ……. + zn ) z1 + z 2 + ……. + zn )
= | z1 + z 2 + z3 + ……. + zn | 2 which is real

≤ ( | z1 | + | z 2 | + | z3 | +…….+ | zn | ) = n2 ∴
2
0 < z ≤ n2 .
6 . 1 2 | Complex Number

Illustration 19: Let x1, x2 are the roots of the quadratic equation x2 + ax + b = 0 where a, b are complex numbers and
y1, y2 are the roots of the quadratic equation y 2 + | a | y + | b | =
0 . If | =
x1 | | =
x2 | 1 , then prove that | = y2 | 1 .
y1 | | =
 (JEE ADVANCED)
Sol: Solve by using modulus properties of complex number.

Let x2 + ax + b =0 where x1 and x2 are complex numbers


x1 + x2 = – a … (i)
and x1 x2 = b … (ii)
From (ii) | x1 | | x2 | = | b | ⇒ | b | =
1 Also | – a | = | x1 + x2 |

∴ | a | ≤ | x1 | + | x2 | or |a | ≤ 2

y1
Now consider y 2 + | a | y + | b | =
0 , where y1 and y 2 are complex numbers
y2

− | a | ±  4 − | a |2  i
− | a | ± | a |2 − 4 | b |   | a | 2 + 4− | a | 2
y1, 2 = = ∴ | y1, 2 | = =1
2 2 2

Hence, | = y2 | 1 .
y1 | | =

6. TRIANGLE ON COMPLEX PLANE


In a ΔABC, the vertices A, B and C are represented by the complex numbers z1, z2 and z3 respectively, then

z1 + z 2 + z3
(a) Centroid: The centroid ‘G’ is given by . Refer to Fig 6.10.
3
A(z1)

B(z2) D C(z3)

Figure 6.10: Centroid

a z1 + b z 2 + c z3
(b) Incentre: The incentre ‘I’ is given by . Refer to Fig 6.11.
a+b+c
A(z1)

B(z2) C(z3)

Figure 6.11: Incentre


M a them a ti cs | 6.13

z1 tanA + z 2 tanB + z3 tanC


(c) Orthocentre: The orhtocentre ‘H’ is given by .
∑ tanA
A(z1)

n
E
H

B(z2) p D q C(z3)

Figure 6.12: Orthocentre

p z3 + q z 2
Proof: From section formula, we have zD =
a
In ΔABD and ΔACD, p = c cos B and q = b cos C. Refer to Fig 6.12.
bcosC z 2 + c cosB z3
Therefore, zD =
a
Now, AE = c cos A;= = AEcosec C = c cos A cosec C
n AH
⇒n=2R cos A [Using Sine Rule]
and m = c cos B cot C or, m = 2R cos B cos C [Using Sine Rule]

mz1 + n ZD
Hence, zH = .
m+n
 bcosC z 2 + c cosB z3 
2R cosBcosC z1 + 2R cos A  
 a 
=
2R (cos A + cosBcosC)

acosB cosC z1 + b cos A cosC z 2 + c cos A cosB z3


=
a( − cos(B + C) + cosB cosC)

z1 (sinA cosB cosC) + z 2 (sinB cosC cos A) + z3 (sinC cos A cosB)


=
sinA (sinB sinC)

z1 tanA + z 2 tanB + z3 tanC z1 tanA + z 2 tanB + z3 tanC


∴ ZH = or
∑ tanA ∏ tanA
[If A + B + C = π , then tan A + tan B + tan C = tan A tan B tan C]

(d) Circumcentre:
Let R be the circumradius and the complex number z0 represent the circumcentre of the triangle as shown in Fig 6.11.

∴ | z1 − z 0 | = | z 2 − z 0 | = | z3 − z 0 |

Consider, | z1 − z 0 | 2 =| z 2 − z 0 | 2

(z1 − z 0 ) (z1 − z0 ) = (z 2 − z 0 ) (z2 − z0 )


6 . 1 4 | Complex Number

z1 (z1 − z 0 ) − z2 (z 2 − z=
0) z 0 (z1 − z 0 ) − (z 2 − z 0 )

) z 0 (z1 − z 2 ) 
z1 (z1 − z 0 ) − z2 (z 2 − z 0= … (i)

Similarly 1st and 3rd gives


) z 0 (z1 − z3 ) 
z1 (z1 − z 0 ) − z3 (z3 − z 0= … (ii)
On dividing (i) by (ii), z0 gets eliminated and we obtain z 0 .

Alternatively: From Fig 6.13, we have 


A(z1)
1 = ar PBC
BD m Ar. ∆ABD Ar. ∆PBD 2 = ar PCA
= = =
DC n Ar. ∆ADC Ar. ∆PDC 3 = ar PAB
m Ar. ∆ABD − Ar. ∆PBD ∆3 2
∴ = 3
n Ar. ∆ADC − Ar. ∆PDC ∆2 P(z0)
R2 1
sin2C
m 2 sin2C
∴ = = C(z3) D
B(z2)
n R 2 sin2B n m
sin2B
2
Figure 6.13: Circumcentre
sin2B (z 2 ) + sin2C(z3 )
Hence, ZD =
sin2B + sin2C

PA l ∆ABP ∆APC ∆ABP + ∆APC l ∆ + ∆2 sin2C + sin2B


Now, = = = = ∴ =3 =
PD k ∆PBD ∆CPD ∆PBD + ∆CPD k ∆1 sin2A

kz1 + l zD z1 sin2A + z 2 sin2B + z3 sin2C


Hence, z 0 = =
k+l
∑ sin2A

PLANCESS CONCEPTS
1
• The area of the triangle whose vertices are z, iz and z + iz is | z |2 .
2

3
• The area of the triangle with vertices z, ω z and z + ω z is | z |2 .
4
• If z1 , z 2 , z3 be the vertices of an equilateral triangle and z 0 be the circumcentre, then z12 + z 22 + z32 =
3z 02 .
• If z1 , z 2 , z3 ,…….zn be the vertices of a regular polygon of n sides and z 0 be its centroid, then
z12 + z 22 + ……. + zn2 = nz 02 .

• If z1, z2 , z3 be the vertices of a triangle, then the triangle is equilateral if (z1 ‒ z2)2 + (z2 ‒z3)2 + (z3 ‒ z1)2= 0
1 1 1
or z12 + z 22 + z32= z1 z 2 + z 2 z3 + z3 z1 or + + = 0.
z1 − z 2 z 2 − z3 z3 − z1
• If z1, z2 , z3 are the vertices of an isosceles triangle, right angled at z2 then z12 + 2z 22 + =
z32 2z 2 (z1 + z 3 ) .
2
• If z1, z2 , z3 are the vertices of a right-angled isosceles triangle, then (z1 − z 2 ) = 2 (z1 − z3 ) (z3 − z 2 ) .
• If z1, z2 , z3 be the affixes of the vertices A, B, C respectively of a triangle ABC, then its orthocentere
a (sec A) z1 + b(secB) z 2 + c(sec C) z3
  is .
asec A + bsecB + c sec C
Shivam Agarwal (JEE 2009, AIR 27)
M a them a ti cs | 6.15

z3
Illustration 20: If z1, z2 , z3 are the vertices of an isosceles triangle right angled
at z 2 then prove that z12 + 2z 22 + =
z32 2z 2 (z1 + z 3 ) (JEE MAIN)

Sol: Here (z1 − z 2 ) = (z3 − z 2 )e 2 . Hence by squaring both side we will get the result.
z1 z2
⇒ (z1 − z 2 )2 = i2 (z3 − z 2 )2
Figure 6.14
⇒ z32 + z 22 − 2z3 z 2 =−z12 − z 22 + 2z1 z 2 ⇒ z12 + 2z 22 + =
z32 2z 2 (z1 + z 3 ) .

Illustration 21: A, B, C are the points representing the complex numbers z1, z2 , z3 respectively and the circumcentre of
the triangle ABC lies at the origin. If the altitudes of the triangle through the opposite vertices meets the circumcircle at
D, E, F respectively. Find the complex numbers corresponding to the points D, E, F in terms of z1, z2 , z3. (JEE MAIN)
  A(z1)
Sol: Here the ∠BOD = π − 2B , hence OD = OB ei( π−2B) .
(/2)-B E()
 
i( π−2B)
From Fig 6.13, we have OD = OB e ; 2B
O
i ( π−2B) −i2B
α = z2 e = −z2 e  … (i) B(z2) C(z3)
-2B
also, z1 = z3 ei 2B  … (ii) D()
Figure 6.15
− z 2 z3
∴ αz1 =−z 2 z3 ⇒α=
z1
−z3 z1 −z z
Similarly, β = and γ = 1 2 .
z2 z3

Illustration 22: If zr (r = 1, 2, …,6) are the vertices of a regular hexagon then E(z5) D(z4)
6
prove that ∑ zr2 = 6z02 , where z 0 is the circumcentre of the regular hexagon.
r =1
 (JEE MAIN)
F(z6) z0 C(z3)

Sol: As we know If z1 , z 2 , z3 ,…….zn be the vertices of a regular polygon of n sides


and z 0 be its centroid, then z12 + z 22 + ……. + zn2 = nz 02 .
A(z1) B(z2)
Here by the Fig 6.14,
Figure 6.16
3z 20 = z12 + z32 + z52
6
and, 3z 20 = z 22 + z 24 + z 62 ⇒ 6z 20 = ∑ zr2 .
r =1

Illustration 23: If z1, z2 , z3 are the vertices of an equilateral triangle then prove that z12 + z 22 + z32= z1 z 2 + z 2 z3 + z3 z1
and if z0 is its circumcentre then 3z 20 = z12 + z 22 + z32 . (JEE ADVANCED)

Sol: By using triangle on complex plane we can prove


z1 + z 2 + z3
z12 + z 22 + z32= z1 z 2 + z 2 z3 + z3 z1 and by using z 0 = we can prove 3z 20 = z12 + z 22 + z32 .
3
Y
To Prove, z12 + z 22 + z32= z1 z 2 + z 2 z3 + z3 z1
A(z1)
As seen in the Fig 6.17,

C(z3)
z1 − z 2 (z3 − z 2 ) e 3 2
/3
 = ⇒ (z1 − z 2 ) (z1 − z3 ) =
−(z 2 − z3 )
z 2 − z3 iπ B(z2)
(z1 − z3 ) e3 O X

0 ∴ ∑ z12 = Figure 6.17


⇒ z12 − z1 z3 − z 2 z1 + z 2 z3 + z 22 + z32 − 2z 2 z3 = ∑ z1 z2
6 . 1 6 | Complex Number

Now if z0 is the circumcentre of the ∆, then we need to prove 3z 20 = z12 + z 22 + z32 .


z1 + z 2 + z3
Since in an equilateral triangle, the circumcentre coincides with the centroid, we have z 0 =
3
(3z0 )
2
⇒ (z1 + z 2 + z3 )2 =

⇒ ∑ z12 + 2∑ z1 z 2 =
9z 02 ∴ 3∑ z12 =
9z 02 .

Illustration 24: Prove that the triangle whose vertices are the points z1, z2 , z3 on the Argand plane is an equilateral
1 1 1
triangle if and only if + + = 0.  (JEE ADVANCED)
z 2 − z3 z3 − z1 z1 − z 2
Sol: Consider ABC is the equilateral triangle with vertices z1 , z 2 and z3 respectively.
A(z1)
Therefore | z 2 − z3 | = | z3 − z1 | = | z1 − z 2 | .
Let ABC be a triangle such that the vertices A, B and C are z1, z2 and z3 respectively. 60

Further, let α= z 2 − z3 , β= z3 − z1 and γ= z1 − z 2 . Then α + β + γ = 0  … (i)


As shown in Fig 6.16, let ∆ABC be an equilateral triangle. Then, BC = CA = AB
⇒ | z 2 − z3 | = | z3 − z1 | = | z1 − z 2 | ⇒ | α | = | β | = | γ | 60 60
B (z2) C (z3)
⇒ | α |2 =| β |2 =| γ |2 =λ(say)
Figure 6.18
⇒ αα = ββ = γγ = λ
λ λ λ
⇒α
= ,β
= ,=
γ  … (ii)
α β γ
Now, α + β + γ = 0 [from (i)]
λ λ λ
⇒ α + β + γ =0 ⇒ + + =0 [Using (ii)]
α β γ
1 1 1 1 1 1
⇒ + + =0⇒ + + 0 which is the required condition.
=
α β γ z 2 − z3 z3 − z1 z1 − z 2

Conversely, let ABC be a triangle such that


1 1 1 1 1 1
⇒ + + 0 i.e. ⇒ + + =
= 0
z 2 − z3 z3 − z1 z1 − z 2 α β γ
1 1 1
Thus, we have to prove that the triangle is equilateral. We have, + + =0
α β γ
1 1 1 1 β+ γ  1 α
⇒ =− +  ⇒ =−  ⇒ = ⇒ α2 = βγ ⇒ | α |2 = | βγ |
α β γ  α  βγ  α βγ

⇒ | α |2 = | β | | γ | ⇒ | α |3 = | α | | β | | γ |

Similarly, ⇒ | β |3 = | α | | β | | γ | and | γ |3 =| α | | β | | γ |
∴ |α|=|β|=| γ |
⇒ | z 2 − z3 | = | z3 − z1 | = | z1 − z 2 | ⇒ BC = CA = AB
Hence, the given triangle is an equilateral triangle.
M a them a ti cs | 6.17

1 1 1
Illustration 25: Prove that the roots of the equation + + 0 (where z , z , z are pair wise distinct
=
z − z1 z − z 2 z − z3 1 2 3

complex numbers) correspond to points on a complex plane, which lie inside a triangle with vertices z1, z2 , z3 excluding
its boundaries.  (JEE ADVANCED)

z − z1 z − z2 z − z3
Sol: By using modulus and conjugate properties we can reduce given expression as + +
| z − z1 | 2 | z − z2 | 2 | z − z3 | 2
1
= 0. Therefore by putting | z − zi | = , where i = 1, 2 and 3, we will get the result.
2
ti A(z1)
1
t1 (z − z1 ) + t2 (z − z2 ) + t3 (z − z3 ) = 0 where | z − z1 | 2 =etc and t1 , t2 , t3 ∈ R +
t1 t2

t1 (z − z1 ) + t2 (z − z 2 ) + t3 (z − z3 ) = 0 z’ t3
t z +t z +t z t1 z t1 + t2
(t1 + t2 + t3 ) z= t1 z1 + t2 z 2 + t3 z3 ⇒ z =1 1 2 2 3 3
t1 + t2 + t3
B (z2) C (z3)
t1 z1 + t2 z 2 t1 + t2 t3 z 3 t1 + t2 t3 z 3
⇒z · + = z' + Figure 6.19
t1 + t2 t1 + t2 + t3 t1 + t2 + t3 t1 + t2 + t3 t1 + t2 + t3
(t + t )z'+ t3 z3
⇒ z =1 2 ⇒ z lies inside the ∆z1 z 2 z3
t1 + t2 + t3

If t=
1 t=
2 t3 ⇒ z is the centroid of the triangle.
Also, it implies | z − z1 | =| z − z 2 | =| z − z3 | ⇒ z is the circumcentre .

Illustration 26: Let z1 and z 2 be roots of the equation z 2 + pz + q = 0, where the coefficients p and q may be
complex numbers. Let A and B represent z1 and z 2 in the complex plane. If ∠AOB = α ≠ 0 and OA = OB, where O
α
is the origin, prove that p2 = 4qcos2 .  (JEE ADVANCED)
2
 
Sol: Here OB = OAeiα . Therefore by using formula of sum and product of roots of quadratic equation we can prove
this problem.
Y
Since z1 and z 2 are roots of the equation z 2 + pz + q = 0
−p and z z = q B(z2)
z +z =
1 2 1 2 (1)
   A(z1)
Since OA = OB. So OB is obtained by rotating OA in anticlockwise direction through X’ X
angle α. O
  z z2
∴ ∴ OB = OAeiα ⇒ z2 = z1eiα ⇒ 2 = eiα ⇒ = cos α + isin α
Y’
z1 z1 Figure 6.20

z2 z 2 + z1 α α α α
⇒ + 1 = 1 + cos α + isin α ⇒ = 2cos  cos + isin
=  2cos e 2
z1 z1 2 2 2 2
iα 2
z 2 + z1 α  z + z1  α
⇒ 2cos e 2 ⇒  2
= 4 cos2 eiα
 =
z1 2  z1  2
2
 z + z1  α z2 α
⇒ ( z 2 + z1 ) =
2
⇒  2  = 4 cos2 4z1 z 2 cos2
 z1  2 z1 2
α α
⇒ ( − p) 2 = 4qcos2 ⇒ p 2 = 4qcos2 .
2 2
6 . 1 8 | Complex Number

Illustration 27: On the Argand plane z1, z2 and z3 are respectively the vertices of an isosceles triangle ABC with
AC = BC and equal angles are θ. If z4 is the incentre of the triangle then prove that (z2 ‒ z1) (z3 ‒ z1) = (1 + sec θ)
(z4 ‒ z1)2 (JEE ADVANCED)

Sol: Here by using angle rotation formula we can solve this problem. From Fig 6.21, we have
z 2 − z1 z 4 − z1
= eiθ /2 … (i) (clockwise) C(z3)
| z 2 − z1 | | z 4 − z1 |

z3 − z1 z 4 − z1
and = eiθ /2 … (ii) (anticlockwise)
| z3 − z1 | | z 4 − z1 |

I(z4)
Multiplying (i) and (ii) /2
/2 
(z 2 − z1 ) (z3 − z1 ) | (z 2 − z1 ) | | (z3 − z1 ) | AB| AC | 2(AD) (AC) 2(AD)2 AC A(z1) D B(z2)
= = = = ·
(z 4 − z1 ) 2 | z 4 − z1 | 2 (AI) 2
(AI) 2
(AI) 2 AD
Figure 6.21
θ
= 2cos2 sec θ = (1+ cos θ)sec θ .
2

7. REPRESENTATION OF DIFFERENT LOCI ON COMPLEX PLANE


(a) | z – (1 + 2i) | = 3 denotes a circle with centre (1, 2) and radius 3 (see Fig 6.22).
Y

(1,2)
Radius = 3
X’ X

Y’
Figure 6.22: Circle on a complex plane

(b) | z – 1 | = | z – i | denotes the equation of the perpendicular bisector of join of (1, 0) and (0, 1) on the Argand
plane (see Fig 6.24).
Y

B(0,1)

O Locus of z
X’ X
A(0,1)

Y’

Figure 6.23: Perpendicular bisector complex plane

(c) | z – 4i | + | z + 4i | = 10 denotes an ellipse with foci at (0, 4) and (0, – 4); major axis 10; minor axis 6 with
4
e= (see Fig 6.24). Y
5
(0,4)

X’ X

(0,-4)

Y’
Figure 6.24: Ellipse on a complex plane
M a them a ti cs | 6.19

 2 2 
2 36 64 4 x + y =1
e = 1− = ⇒ e= 9 25
100 100 5  

(d) | z – 1 | + | z + 1 | = 1 denotes no locus. (Triangle inequality).


(e) | z – 1 | < 1 denotes area inside a circle with centre (1, 0) and radius 1.
(f) 2 ≤ | z − 1 | < 5 denotes the region between the concentric circles of radii 5 and 2. Centred at (1, 0) including
the inner boundary (see Fig 6.25).
Y

(1,0)
X’ X

Y’

Figure 6.25: Circle disc on a complex plane

π
(g) 0 ≤ arg z ≤ (z ≠ 0) where z is defined by positive real axis and the part of the line x = y in the first quadrant.
4
It includes the boundary but not the origin. Refer to Fig 6.26.

y=x
X’ X
O

Y’

Figure 6.26

(h) Re (z 2 ) > 0 denotes the area between the lines x = y and x = – y which includes the x-axis.

0 ⇒ x2 − y 2 > 0
Hint: (x2 − y 2 ) + 2xyi = ⇒ (x – y) (x + y) > 0.

Illustration 28: Solve for z, if z 2 + | z |=


0 .  (JEE MAIN)

Sol: Consider z = x + iy and solve this using algebra of complex number.

Let z = x + iy ⇒ (x + iy) 2 + x2 + y 2 =
0 ⇒  x2 − y 2 + x2 + y 2  + (2ixy) =
0
 
⇒ Either x =0 or y =0 ; x = 0 ⇒ − y 2 + | y | =
0 ⇒ y= 0, 1, − 1 ∴ z= 0, i, − i
2
0
and, y = 0 ⇒ x + | x | = 0
⇒ x= ∴z =0
Therefore, z = 0, z = i, z = – i.

Illustration 29: If the complex number z is to satisfy |z| = 3, z − {a(1 + i) − i} ≤ 3 and | z + 2a – (a + 1) i | > 3
simultaneously for at least one z then find all a ∈ R . (JEE ADVANCED)

Sol: Consider z = x + iy and solve these inequalities to get the result.


6 . 2 0 | Complex Number

All z at a time lie on a circle | z | = 3 but inside and outside the circles | z – {a (1 + i) – i} | = 3 and | z + 2a – (a + 1)
i | = 3, respectively.
Let z = x + iy then equation of circles are x2 + y 2 =
9 … (i)
2 2
(x − a) + (y − a + 1) =9 … (ii)
and (x + 2a)2 + (y − a − 1)2 =9 … (iii)
Circles (i) and (ii) should cut or touch then distance between their centres ≤ sum of their radii.

+ +
⇒ (a − 0) 2 + (a − 1 − 0)2 ≤ 3 + 3 ⇒ a2 + (a − 1)2 ≤ 36
-
1-71 1+71
35
⇒ 2a2 − 2a − 35 ≤ 0 ⇒ a2 − a − ≤0 2 2
2
2 Figure 6.27
 1 71 1 − 71 1 + 71
⇒ a −  ≤ ∴ ≤a≤ … (iv)
 2 4 2 2

Again circles (i) and (iii) should not cut or touch then distance between their centres > sum of the radii

⇒ ( −2a − 0) 2 + (a + 1 − 0) 2 > 3 + 3 ⇒ 5a 2 + 2a + 1 > 6 ⇒ 5a 2 + 2a + 1 > 36

2a
⇒ 5a 2 + 2a − 35 > 0 ⇒ a2 + −7 > 0 +
5 +
-
-1-411 -1+411
 −1 − 4 11   −1 + 4 11 
Then  a −  a − >0 5 5
 5  5 
   Figure 6.28
 −1 − 4 11   −1 + 4 11 
∴ a ∈  −∞ , ∪ , ∞  … (v)
 5   5 
   
The common values of a satisfying (iv) and v are

 1 − 71 −1 − 4 11   −1 + 4 11 1 + 71 
a∈ , ∪ , 
 2 5   5 2 
   

8. DEMOIVRE’S THEOREM
Statement: (cos nθ + isin nθ) is the value or one of the values of (cos θ + isin θ) n , ∀ n ∈ Q . Value if n is an integer.
One of the values if n is rational which is not integer, the theorem is very useful in determining the roots of any
complex quantity.
Note: We use the theory of equations to find the continued product of the roots of a complex number.

PLANCESS CONCEPTS
The theorem is not directly applicable to (sin θ + icos θ) n , rather
n
n  π  π  π  π 
(sin θ + icos θ) = cos  − θ  + isin  − θ   = cos n  − θ  + isin n  − θ 
 2  2  2  2 
M a them a ti cs | 6.21

8.1 Application
Cube root of unity
−1 + i 3 −1 − i 3
(a) The cube roots of unity are 1, ,
2 2
[Note that 1 – i 3 = – 2 and 1 + i 3 = –2 ω2 ]

(b) If ω is one of the imaginary cube roots of unity then 1 + ω + ω2 = 0 .


In general 1 + ωr + ω2r = 0; where r = 1, and not a multiple of 3.
2π 2π 4π 4π
(c) In polar form the cube roots of unity are: cos 0 + isin 0 ; cos + isin ; cos + isin
3 3 3 3
(d) The three cube roots of unity when plotted on the argand plane constitute the vertices of an equilateral
triangle.
[Note that the 3 cube roots of i lies on the vertices of an isosceles triangle]

(e) The following factorization should be remembered.


For a, b, c ∈ R and ω being the cube root of unity,

(i) a3 − b3= (a − b) (a − ωb)(a − ω2b)

(ii) x2 + x + 1= (x − ω) (x − ω2 )

(iii) a3 + b3= (a + b) (a + ωb)(a + ω2b)

(iv) a3 + b3 + c3 − 3abc = (a + b + c) (a + ωb + ω2c) (a + ω2b + ωc)

nth roots of unity: If 1, α1 , α2 , α3 , ……., αn−1 are the n, nth roots of unity then
 2π 
i 
2π 2π
(i) They are in G.P. with common ratio e n 
= cos + isin
n n
(ii) 1p + α1p + αp2 + ……. + αnp−1 = 0 if p is not an integral multiple of n

1p + (α1 )p + (α2 ) p + ……. + (αn−1 )p =n if p is an integral multiple of n.



(iii) (1 − α1 ) (1 − α2 ) ……. (1 − αn−1 ) =n .

Steps to determine nth roots of a complex number


(i) Represent the complex number whose roots are to be determined in polar form.

(ii) Add 2mπ to the argument.

(iii) Apply De Moivre’s TheoremT

(iv) Put m = 0, 1, 2, 3, ……. (n – 1) to get all the nth roots.

1 1 1
 2mπ 2mπ 
Explanation: Let z = 1n = (cos 0 + isin 0)n = ( cos 2mπ + isin 2mπ ) n =  cos + isin 
 n n 
Put m = 0, 1, 2, 3, ……. (n – 1), we get

2π 2π 4π 4π 2(n − 1)π 2(n − 1)π


1, cos + isin , cos + isin , ……. , cos + isin (n, nth roots in G.P.)
n
 n n n n n
α
6 . 2 2 | Complex Number

1 − ( αp ) n 1 − ( αn ) p
Now, S = 1p + αp + α2p + α3p + ……. + α(n−1)p = =
1 − αp 1 − αp
 0
n p  = 0, if p is not an integral multiple of n
1 − (α ) non zero
= = 
1 − αp 0
 = indeterminant, if p is an integral multiple of n
0

Again, if x is one of the nth root of unity then xn – 1 = (x – 1) (x − α1 )(x − α2 ) ……. (x − αn−1 )
xn − 1
1 + x + x2 + …… + xn−1 = ≡ (x − α1 )(x − α2 ) ……. (x − αn−1 )
x −1
Put x = 1, to get (1 − α1 ) (1 − α2 ) ……. (1 − α n−1 ) =n

Similarly put x = – 1, is to get other result.

PLANCESS CONCEPTS

 | z | +a | z | −a 
Square roots of z = a + ib are ±  +i  for b > 0.
 2 2 

If 1, α1 , α2 , α3 , ……., αn−1 are the n, nth roots of unity then

(1 + α1 ) (1 + α2 ) ……. (1 + α n−1 ) = 0 if n is even and 1 if n is odd.

1 · α1 · α2 · α3 · ……. · αn−1 = 1 or – 1 according as n is odd or even.


0, if n = 3k

(ω − α1 ) (ω − α2 ) ……. (ω − αn−1 ) = 1, n 3k + 1 .
if =
1 + ω, if n= 3k + 2

Ravi Vooda (JEE 2009, AIR 71)

Illustration 30: If x= a + b , y = aω + bω2 and z = aω2 + bω , then prove that x3 + y 3 + z3= 3 (a3 + b3 )  (JEE MAIN)

Sol: Here x + y + z =.
0 Take cube on both side.
x + y + z =
0 ⇒ x3 + y 3 + z 3 =
3xyz ∴ LHS = 3xyz
= 3(a + b)(aω + bω2 )(aω2 + bω) = 3(a + b)(aω + bω2 )(aω2 + bω.ω3 ) = 3ω3 (a + b)(a + bω)(a + b=
ω2 ) 3 (a3 + b3 )

Illustration 31: The value of expression 1(2 − ω)(2 − ω2 ) + 2 (3 − ω) (3 − ω2 ) + ... + (n − 1)(n − ω)(n − ω2 ) .
 (JEE ADVANCED)

Sol: The given expression represent as x3 − 1 = (x – 1) (x − ω) (x − ω2 ) . Therefore by putting x = 2, 3, 4 … n, we will


get the result.
x3 − 1 = (x – 1) (x − ω) (x − ω2 )
Put x = 2 23 – 1 = 1 · (2 − ω) (2 − ω) 2 Put x = 3 33 – 1 = 2 · (3 − ω) (3 − ω2 ) :
Put x = n n3 – 1 = (n − 1) (n − ω) (n − ω2 )
2
 n (n + 1) 
= (23 + 33 + ……. + n3 ) − (n − 1) = (13 + 23 + 33 + ……=
∴ LHS . + n3 ) − n   −n
 2 
M a them a ti cs | 6.23

9. SUMMATION OF SERIES USING COMPLEX NUMBER

 nθ 
sin  
(a) cos θ + cos2θ + cos3θ + …… + cosnθ =  2  cos  n + 1  θ
 
θ
sin    2 
2

 nθ 
sin  
(b) sin θ + sin2θ + sin3θ + …… + sinnθ =  2  sin  n + 1  θ
 
θ
sin    2 
2

Note: If θ = , then the sum of the above series vanishes.
n

9.1 Complex Number and Binomial Coefficients


Try the following questions using the binomial expansion of (1 + x)n and substituting the value of x according to
the binomial coefficients in the respective question.
Find the value of the following
(i) C0 + C 4 + C8 + ……. (ii) C1 + C5 + C9 + …….
(iii) C2 + C6 + C10 + ……. (iv) C3 + C7 + C11 + …….
(v) C0 + C3 + C6 + C9 + …….
Hint (v): In the expansion of (1 + x)n , put x =1, ω,and ω2 and add the three equations.

Illustration 32: If 1, ω, ω2 , ……., ωn−1 are nth roots of unity, then the value of (5 − ω) (5 − ω2 ) ……. (5 − ωn−1 ) is equal
to (JEE MAIN)
1
Sol: Here consider x = (1)n , therefore xn − 1 =0 (has n roots i.e. 1, ω, ω2 , ……., ωn−1 ).
xn − 1
⇒ xn − 1= (x − 1)(x − ω) (x − ω2 ) …….(x − ωn−1 ) ⇒ = (x − ω) (x − ω2 ) …….(x − ωn−1 )
x −1
5n − 1
⇒ Putting x = 5 in both sides, we get ∴ (5 − ω) (5 − ω2 ) ……. (5 − ωn−1 ) = .
4

10. APPLICATION IN GEOMETRY

10.1 Distance Formula


Distance between A (z1 ) and B(z 2 ) is given by AB = | z 2 − z1 | . Refer Fig 6.29.

A(z1)

B(z2)
X’ X
O

Y’
Figure 6.29
6 . 2 4 | Complex Number

10.2 Section Formula n B(z2)

The point P(z) which divides the join of A(z1) and B(z2) in the ratio m: n is m P(z)
mz 2 + nz1
given by z = . Refer Fig 6.30. A(z1)
m+n
Figure 6.30

10.3 Midpoint Formula


1
Mid-point M(z) of the segment AB is given by z = (z + z 2 ) .
2 1

1 1
A(z1) M(z) B(z2)

Figure 6.31 Mid point formula

10.4 Condition For Four Non-Collinear Points


Condition(s) for four non-collinear A(z1 ), B(z 2 ), C(z3 ) and D(z 4 ) to represent vertices of a

(a) Parallelogram: The diagonals AC and BD must bisect each other


A(z1) D(z4)
1 1
⇔ (z1 + z 3 ) = (z + z 4 )
2 2 2
⇔ z1 + z 3 = z 2 + z 4

(b) Rhombus:
B(z2) C(z3)
(i) The diagonals AC and BD bisect each other Figure 6.32
⇔ z1 + z 3 = z 2 + z 4 , and A(z1)

D(z4)
(ii) A pair of two adjacent sides are equal, for instance AD = AB
⇔ | z 4 − z 1 | = | z2 − z 1 |

(c) Square:
B(z2) C(z3)
(i) The diagonals AC and BD bisect each other
Figure 6.33
⇔ z1 + z 3 = z 2 + z 4

(ii) A pair of adjacent sides are equal; for instance, AD = AB A(z1) D(z4)

⇔ | z 4 − z 1 | = | z2 − z 1 |

(iii) The two diagonals are equal, that is AC = BD
⇔ | z3 − z 1 | = | z 4 − z 2 |
B(z2) C(z3)
Figure 6.34
(d) Rectangle:
(i) The diagonals AC and BD bisect each other A(z1) D(z4)

⇔ z1 + z 3 = z 2 + z 4

(ii) The diagonals AC and BD are equal


⇔ | z3 − z 1 | = | z 4 − z 2 | B(z2) C(z3)

Figure 6.35
M a them a ti cs | 6.25

10.5 Triangle
In a triangle ABC, let the vertices A, B and C be represented by the complex numbers z1, z2, and z3 respectively. Then

(a) Centroid: The centroid (G), is the point of intersection of medians of ∆ABC . It is given by the formula
1
z= (z + z 2 + z3 )
3 1
A(z1)

B(z2) C(z3)

Figure 6.36 (a)

(b) Incentre: The incentre (I) of ∆ABC is the point of intersection of internal angular bisectors of angles of
∆ABC . It is given by the formula
az1 + bz 2 + cz3
z= ,
a+b+c A(z1)

B(z2) C(z3)

Figure 6.36 (b)

(c) Circumcentre: The circumcentre (S) of ∆ABC is the point of intersection of perpendicular bisectors of sides
of ∆ABC . It is given by the formula
| z1 | 2 z1 1 A(z1)
| z2 | 2 z2 1
2
2 2 2
| z1 | (z 2 − z3 )+ | z 2 | (z3 − z1 )+ | z3 | (z1 − z 2 ) | z3 | z3 1
z= =
z1 (z 2 − z3 ) + z2 (z3 − z1 ) + z3 (z1 − z 2 ) z1 z1 1 S
z2 z2 1
B(z2)
z3 z3 1 C(z3)

z1 (sin2A) + z 2 (sin2B) + z3 (sin2C) Figure 6.36 (c)


Also, z =
sin2A + sin2B + sin2C

H 2 G 1 S
(Orthocentre) (Centroid) (Circumcentre)

Figure 6.37

(d) Euler’s Line: The orthocenter H, the centroid G and the circumcentre S of a triangle which is not equilateral
lies on a straight line. In case of an equilateral triangle these points coincide.
G divides the join of H and S in the ratio 2 : 1 (see Fig 6.37).
1
Thus,=
zG (z + 2zS )
3 H
6 . 2 6 | Complex Number

10.6 Area of a Triangle A(z1)

Area of ∆ABC with vertices A(z1 ), B(z 2 ) and C(z3 ) is given by


z1 z1 1
1 1
∆ =| z 2 z2 1 | = Im(z1 z 2 + z2 z3 + z3 z1 )
4i 2
z3 z3 1
B(z2) C(z3)

Figure 6.38
10.7 Conditions for Triangle to be Equilateral
The triangle ABC with vertices A(z1 ), B(z 2 ) and C(z3 ) is equilateral
1 1 1
iff + + =0
z 2 − z3 z3 − z1 z1 − z 2
A(z1)
⇔ z12 + z 22 + z32= z 2 z3 + z3 z1 + z1 z 2 ⇔ z1 z2 = z 2 z3 = z3=
z1 ⇔ z12 z 2 z3 =
and z 22 z1 z3
60
1 z2 z3
z − z1 z3 − z 2
⇔ 1 z3 0 ⇔ 2
z1 = =
z3 − z 2 z1 − z 2
1 z1 z2
60 60
1 1 1 1
⇔ + + = 0 where z= (z1 + z 2 + z3 ) . B(z2) C(z3)
z − z1 z − z 2 z − z3 3 Figure 6.39

10.8 Equation of a Straight line

(a) Non-parametric form: An equation of a straight line joining the two points A(z1 ) and B(z 2 ) is

z z 1 C(z3)
 z − z1 
Arg   = 0 z1 z1 1 = 0
 z 2 − z1  z2 z2 1
B(z2)
z − z1 z − z1
or =
z 2 − z1 z2 − z1 A(z1)

or z (z1 − z2 ) − z(z1 − z 2 ) + z1 z2 − z 2 z1 =
0 Figure 6.40

(b) Parametric form: An equation of the line segment between the points A(z1 ) and B(z 2 ) is
z = tz1 + (1 − t)z 2 , t ( 0,1 ) where t is a real parameter.

(c) General equation of a straight line: The general equation of a straight line is az + az + b =0 where, a is
non-zero complex number and b is a real number.

10.9 Complex Slope of a Line


z1 − z 2
If A(z1 ) and B(z 2 ) are two points in the complex plane, then complex slope of AB is defined to be µ =
z1 − z2
Two lines with complex slopes µ1 and µ2 are
(i) Parallel, if µ1 =µ2 (ii) Perpendicular, if µ1 + µ2 = 0
 −a 
The complex slope of the line az + az + b =0 is given by   .
 a 
M a them a ti cs | 6.27

10.10 Length of Perpendicular from a Point to a Line A


p
Length of perpendicular of point A(ω) from the line az + az + b =0.
| aω + aω + b |
Where a ∈ C − {0}, and b ∈ R is given by p =
2|a|
Figure 6.41

10.11 Equation of Circle z


r or
(a) An equation of the circle with centre z 0 and radius r is | z − z 0 | = r
z= z 0 + reiθ ,0 ≤ θ < 2π (parametric form) or zz − z 0 z − z0 z + z 0 z0 − r 2 = 0 z0

(b) General equation of a circle is zz + az + az + b =0 … (i) Figure 6.42

Where a is a complex number and b is a real number such that aa − b ≥ 0 . Centre of (i) is – a and its radius is
aa − b

(c) Diameter form of a circle: An equation of the circle one of whose diameter is the segment joining A(z1) and
B(z2) is (z − z1 ) (z − z2 ) + (z − z1 )(z − z 2 ) =
0
P(z)
(d) An equation of the circle passing through two points A(z1) and B(z2)
B(z2)

Centre
z z 1
A(z1)
is (z − z1 ) (z − z2 ) + (z − z1 )(z − z 2 ) + i k z1 0 where k is a real parameter.
z1 1 =
z2 z2 1 Figure 6.43

(e) Equation of a circle passing through three non-collinear points.


Let three non-collinear points be A(z1 ), B(z 2 ) and C(z3 ) and P(z) be any point on the circle through A, B and C.
Then either ∠APB
∠ACB = [when angles are in the same segment]
or, ∠ACB + ∠APB = π [when angles are in the opposite segment] (see Fig 6.44).

 z − z2   z − z2   z3 − z 2   z − z1 
⇒ arg  3 − arg   = 0 or, arg   + arg   =
π
 z − z 
 3 1   z − z1   z3 − z1   z − z2 

 z − z 2   z − z1  
⇒ arg  3   0
  = P(z)

 z3 − z1   z − z 2   C(z3) 

 z − z 2   z − z1  
or, arg  3     = π
 z3 − z1   z − z 2  
B(z2)
A(z1) -
P(z)
(z − z1 ) (z3 − z 2 )
In any case, we get is purely real. Figure 6.44
(z − z 2 )(z3 − z1 )

(z − z1 ) (z3 − z 2 )(z − z1 ) (z3 − z2 )


⇔ =
(z − z 2 )(z3 − z1 ) (z − z2 )(z3 − z1 )

(f) Condition for four points to be concyclic.


(z 4 − z1 ) (z3 − z 2 )
Four points z1 , z 2 , z3 andz 4 will lie on the same circle if and only if is purely real.
(z 4 − z 2 )(z3 − z1 )
(z 4 − z1 ) (z3 − z 2 ) (z4 − z1 ) (z3 − z2 )
⇔ =
(z 4 − z 2 )(z3 − z1 ) (z4 − z2 )(z3 − z1 )
6 . 2 8 | Complex Number

PLANCESS CONCEPTS

z1 z1 1
Three points z 1 , z 2 and z3 are collinear if z 2 z2 1 = 0.
z3 z3 1

If three points A(z1 ), B(z 2 ) and C(z3 ) are collinear then slope of AB = slope of BC = slope of AC
z1 − z 2 z 2 − z3 z1 − z3
⇒ = =
z1 − z2 z2 − z3 z1 − z3

Akshat Kharaya (JEE 2009, AIR 235)

2z + 1
Illustration 33: If the imaginary part of is – 4, then the locus of the point representing z in the complex
plane is iz + 1

(a) A straight line (b) A parabola (c) A circle (d) An ellipse (JEE MAIN)

Sol: Put z = x + iy and then equate its imaginary part to – 4.

2z + 1 2 (x + iy) + 1 (2x + 1) + 2iy [(2x + 1) + 2iy] [(1 − y) − ix]


Let z = x + iy, then = = =
iz + 1 i (x + iy) + 1 (1 − y) + ix (1 − y)2 + x2

 2z + 1  2y (1 − y) − x(2x + 1)
As Im   = – 4, we get =–4
 iz + 1  x2 + (1 − y)2
⇒ 2x2 + 2y 2 + x − 2y= 4x2 + 4(y 2 − 2y + 1) ⇒ 2x2 + 2y 2 − x − 6y + 4 =0 . It represents a circle.

Illustration 34: The roots of z5= (z − 1) 5 are represented in the argand plane by the points that are
(a) Collinear (b) Concyclic
(c) Vertices of a parallelogram (d) None of these (JEE MAIN)

Sol: Apply modulus on both the side of given expression.


Let z be a complex number satisfying z5= (z − 1) 5 .
⇒ | z5 | =| (z − 1)5 | ⇒ | z | 5 =−
| z 1 |5 ⇒ | z | =| z − 1 |
1
Thus, z lies on the perpendicular bisector of the segment joining the origin and (1 + i0) i.e. z lies on Re(z) = .
2
z z
Illustration 35: Let z1 and z 2 be two non-zero complex numbers such that 1 + 2 = 1, then the origin and points
represented by z1 and z 2 z 2 z1

(a) Lie on straight line (b) Form a right triangle


(c) Form an equilateral triangle (d) None of these (JEE ADVANCED)

z1
Sol: Here consider z = and z1 and z 2 are represented by A and B respectively and O be the origin.
z2
z1 1
Let z = , then z + =1 ⇒ z2 − z + 1 =0
z2 z

1 ± 3i z 1 ± 3i
⇒ z= ⇒ 1 =
2 z2 2
M a them a ti cs | 6.29

If z1 and z2 are represented by A and B respectively and O be the origin, then

ΟA |z | 1 ± 3i 1 3
= 1 = = + = 1 ⇒ OA =
OB
OB | z2 | 2 4 4

AB | z − z1 | z 1 3  1 3 1 3
Also, = 2 = 1− 1 = 1− ± i =  i = + =1
| z2 | z2   2 2 4 4
OB 2 2 

OB
⇒ AB = Thus, OA = OB = AB ∴ ∆OAB is an equilateral triangle.

Illustration 36: If z1 , z 2 , z3 are the vertices of an isosceles triangle, right angled at the vertex z 2 , then the value
of (z1 − z 2 ) 2 + (z 2 − z3 ) 2 is
(a) -1 (b) 0 (c) (z1 − z3 ) 2 (d) None of these (JEE ADVANCED)

Sol: Here use distance and argument formula of complex number to solve this problem.
As ABC is an isosceles right angled triangle with right angle at B, C(z3)

z −z  π
BA = BC and ∠ABC = 90° ⇒ | z1 − z 2 | = | z3 − z 2 | and arg  3 2  =
 z1 − z 2  2
z3 − z 2 | z3 − z 2 |  π π 
⇒= cos   + isin    = i
z1 − z 2 | z1 − z 2 |  2
   2  B(z2) A(z1)

Figure 6.45
⇒ (z3 − z 2 )2 =
−(z1 − z 2 )2 ⇒ (z1 − z 2 )2 + (z 2 − z3 )2 =
0.

11. CONCEPTS OF ROTATION OF COMPLEX NUMBER


Let z be a non-zero complex number. We can write z in the polar form as follows:
z = r (cos θ + isin θ) = reiθ where r = | z | and arg (z) = θ (see Fig 6.46). Y
Q(zei)
Consider a complex number zeiα .
P(z)
zeiα = (r eiθ )eiα = rei( θ+α )

 X
O

Thus, ze represents the complex number whose modulus is r and argument is θ + α.
Figure 6.46
Geometrically, zeiα can be obtained by rotating the line segment joining
O and P(z) through an angle α in the anticlockwise direction.

Corollary: If A(z1 ) and B(z 2 ) are two complex number such that
| z2 |
θ , then z 2 =
∠AOB = z1eiθ (see Fig 6.47). Y
| z1 |
B(z2)
Let z1 = r1eiα and z 2 = r2eiβ where = , | z 2 | r2 .
| z1 | r1=
r2
iβ A(z1)
z 2 r2e r  r1
Then= = 2 ei(β−α ) 
z1 r eiα r1  X
1 O

z2 r2 | z2 |
Thus, = eiθ (∵ β − α = θ ) ⇒ z2 = z eiθ Figure 6.47
z1 r1 | z1 | 1
6 . 3 0 | Complex Number

PLANCESS CONCEPTS

Multiplication of a complex number,z with i.

 π π  π  π 
Let z = r (cos θ + isin θ) and i =  cos + isin  , then iz = r cos  + θ  + isin  + θ   .
 2 2  2  2 
Hence, iz can be obtained by rotating the vector z by right angle in the positive sense. And so on, to
multiply a vector by – 1 is to turn it through two right angles.
Thus, multiplying a vector by (cos θ + isin θ) is to turn it through the angle θ in the positive sense.
Anvit Tawar (JEE 2009, AIR 9)

Illustration 37: Suppose A( z1 ), B( z 2 ) and C( z3 ) are the vertices of an equilateral triangle inscribed in the circle
| z | = 2. If z1= 1 + 3 i , then z 2 and z3 are respectively.

(a) −2, 1 − 3 i (b) − 1 + 3 i, − 2


(c) −2, − 1 + 3 i (d) −2, 2 + 3 i  (JEE ADVANCED)

Sol: As we know x + iy = reiθ . Hence by using this formula we can obtain z 2 and z3 .

z1= 1 + 3 i = 2e 3
2 πi 2 πi Y
2π 2π
Since, ∠AOC =and ∠BOC = , z 2 = z1e 3 and z3 = z 2e 3 A(z1)
3 3
5 πi
B(z2) /3
⇒ z3 = 2eπi = 2(cos π + isin π) = −2 and z3 = 2e 3
O
X

  π  π 
= 2 cos  2π −  + isin  2π −   C(z3)
  3   3  Figure 6.48
 π π 1 3 
= 2 cos − isin  = 2  − i = 1 − 3 i .
 3 3  2 2 

PROBLEM-SOLVING TACTICS

(a) On a complex plane, a complex number represents a point.


(b) In case of division and modulus of a complex number, the conjugates are very useful.

(c) For questions related to locus and for equations, use the algebraic form of the complex number.

(d) Polar form of a complex number is particularly useful in multiplication and division of complex numbers. It
directly gives the modulus and the argument of the complex number.

(e) Translate unfamiliar statements by changing z into x+iy.

(f) Multiplying by cos θ corresponds to rotation by angle θ about O in the positive sense.
M a them a ti cs | 6.31

a + ib
(g) To put the complex number in the form A + iB we should multiply the numerator and the denominator
c + id
by the conjugate of the denominator.

(h) Care should be taken while calculating the argument of a complex number. If z = a + ib, then arg(z) is not
b
always equal to  tan−1   . To find the argument of a complex number, first determine the quadrant in which
a
it lies, and then proceed to find the angle it makes with the positive x-axis.
b π −3π
For example, if z = – 1 – i, the formula tan−1   gives the argument as  , while the actual argument is  .
a
  4 4

FORMULAE SHEET

(a) Complex number z= x + i y , where x, y ∈ R and i= −1 .

(b) If z = x + iy then its conjugate z = x – iy.

(c) Modulus of z, i.e. | z | = x2 + y 2

 −1 y
 tan x > 0, y > 0
 x
 y
 π − tan−1 x < 0, y > 0
 x
(d) Argument of z, i.e. θ =
−π + tan−1 y
x < 0, y < 0
 x

 −1 y
 − tan x
x > 0, y < 0

y -1 y
where,  =tan x
=- =

x
=-+ O =-

0, if x > 0
(e) If y=0, then argument of z, i.e. θ =
π, if x < 0
 π
 , if y > 0
(f) If x=0, then argument of z, i.e. θ = 2
 3 π , if y < 0
 2

(g) In polar form x = rsinθ , therefore


rcosθ and y = = z r ( cosθ + i sinθ )

(h) In exponential form complex number z = reiθ , where e=



cosθ + isinθ .
6 . 3 2 | Complex Number

eix + e−ix eix − e−ix


(i) cos x = and sin x =
2 2i
( j) Important properties of conjugate

(i) z + z = 2Re (z) and z − z = 2 Im(z)


(ii) z= z ⇔ z is purely real
(iii) z + z = 0 ⇔ z is purely imaginary
(iv)
= zz [Re(z)] 2 + [Im(z)] 2

(v) z1 + z 2 = z1 + z2

(vi) z1 − z 2 = z1 − z2

(vii) z1 z 2 = z1 z2

 z1  z1
(viii)
=  z  z if z 2 ≠ 0
 2 2

(k) Important properties of modulus


If z is a complex number, then

(i) | z | = 0 ⇔ z =
0
(ii) | z | = | z | =| −z | =| − z |
(iii) − | z | ≤ Re(z) ≤ | z |
(iv) − | z | ≤ Im(z) ≤ | z |
(v) zz = | z | 2
If z1 , z 2 are two complex numbers, then

(i) | z1 z 2 | = | z1 | | z 2 |

(ii) z1 | z1 | , if z ≠ 0
= 2
z2 | z2 |
2 2 2 2 2
(iii) | z1 + z 2 | = | z1 | + | z 2 | + z1 z 2 + z1 z2 = | z1 | + | z 2 | +2Re (z1 z2 )
2 2 2
(iv) | z1 − z 2 | = | z1 | + | z 2 | − z1 z 2 − z1 z2 = | z1 | 2 + | z 2 | 2 −2Re (z1 z2 )

(l) Important properties of argument


(i) arg(z) = − arg(z)
(ii) arg(z
= 1 z 2 ) arg(z1 ) + arg(z 2 )

In fact arg(z1=
z 2 ) arg(z1 ) + arg(z 2 ) + 2kπ
0, if − π < arg(z1 ) + arg(z 2 ) ≤ π

where,k= 1, if − 2π < arg(z1 ) + arg(z 2 ) ≤ −π
−1, if π < arg(z1 ) + arg(z 2 ) ≤ 2π

(iii) arg(z
= 1 z2 ) arg(z1 ) − arg(z 2 )

 z1 
(iv) arg=
  arg(z1 ) − arg(z 2 )
 z2 
M a them a ti cs | 6.33

π
(v) | z1 + z 2 | = | z1 − z 2 | ⇔ arg(z1 ) − arg(z 2 ) =
2
(vi) | z1 + z 2 | = | z1 | + | z 2 | ⇔ arg(z1 ) =
arg(z 2 )

If=
z1 r1 (cos θ1 + isin θ1 ) and=
z 2 r2 (cos θ2 + isin θ2 ) , then

z 2 | 2 | z1 | 2 + | z 2 | 2 +2 | z1 | | z 2 | cos(θ1 − θ2 ) = r12 + r22 + 2r1r2 cos(θ1 − θ2 )


(vii) | z1 + =

z 2 | 2 | z1 | 2 + | z 2 | 2 −2 | z1 | | z 2 | cos(θ1 − θ2 ) = r12 + r22 − 2r1r2 cos(θ1 − θ2 )


(viii) | z1 − =

(m) Triangle on complex plane

z1 + z 2 + z3
(i) Centroid (G), zG =
3
a z1 + b z 2 + c z3
(ii) Incentre (I), zI =
a+b+c
z1 tanA + z 2 tanB + z3 tanC
(iii) Orthocentre (H), zH
∑ tanA
z1 (sin2A) + z 2 (sin2B) + z3 (sin2C)
(iv) Circumcentre (S), zS
sin2A + sin2B + sin2C

( cos θ + i sin θ=
)
n
(n) cosnθ + i sin nθ

 z +x z −x
(o) z =x + i y =± +i   for  y > 0
 2 2 
 
(p) Distance between A (z1 ) and B(z 2 ) is given by | z 2 − z1 |
(q) Section formula: The point P (z) which divides the join of the segment AB in the ratio m : n
mz 2 + nz1
is given by z = .
m+n
1
(r) Midpoint formula: z = (z + z 2 ) .
2 1
(s) Equation of a straight line
(i) Non-parametric form: z (z1 − z2 ) − z(z1 − z 2 ) + z1 z2 − z 2 z1 =
0
(ii) Parametric form: z = tz1 + (1 − t)z 2
(iii) General equation of straight line: az + az + b =0
z1 − z 2
(t) Complex slope of a line, µ = . Two lines with complex slopes µ1 and µ2 are
z1 − z2
(i) Parallel, if µ1 =µ2
(ii) Perpendicular, if µ1 + µ2 = 0
(u) Equation of a circle: | z − z 0 | =
r
6 . 3 4 | Complex Number

Solved Examples

JEE Main/Boards ∴ L.H.S = 1 + ω3m+1 + ω6m+ 2 = 1 + ω + ω2 = 0

Example 1: f z1 and z 2 are 1 – i, – 2 + 4i respectively. (b) Let n = 3m + 2


z z 
Find Im  1 2  . 1 + ω3m+ 2 + ω6m+ 4 = 1 + ω2 + ω4 = 1 + ω2 + ω = 0.
 z1 
z1 z 2 (1 − i) ( −2 + 4i) −2 + 2i + 4i + 4 z −3
Sol: = = Example 4: Show that = 2 represents a circle.
z1 1+i 1+i z+3

2 + 6i 1 − i 2 + 6i − 2i + 6
= × = = 4 + 2i Sol: Consider z = x + iy and then by taking modulus we
1+i 1−i 2 will get the result.
z z 
2.
∴ Im  1 2  = Let z = x + iy
 z1 
z −3 x − 3 + iy
∴ 2 ⇒
= 2
=
Example 2: Find the square root of z = – 7 – 24i. z+3 x + 3 + iy

Sol: Consider z 0= x + iy be a square root then | 2 22 | x + 3 + iy | 2


∴ | x − 3 + iy=
z 02 =−7 − 24i .
2
– 7 – 24i = x − y + 2ixy2
or (
(x − 3) 2 + y 2= 4 (x + 3) 2 + y 2 )
⇒ 3x2 + 3y 2 + 30x + 27 =
0
Equating real and imaginary parts we get
which represents a circle.
x2 − y 2 =
−7  … (i)

and 2xy = – 24  … (ii) Example 5: If | z1 | = | z 2 | = ……. = | zn |= 1

(x2 + y 2 ) 2 = (x2 − y 2 )2 + 4x2 y 2 1 1 1


prove that | z1 + z 2 + ……. + z=
n | + + ……. +
z1 z 2 zn
= ( −7)2 + ( −24) 2 = 625

∴ x2 + y 2 = 
25 … (iii) Sol: | z j | = 1 ⇒ z j z j= 1 ∀ j= 1, …… , n

Solving (i) and (iii), we get, ( zz = | z 2 |)

(x, y) = (3, – 4);(– 3, 4) by (ii) L.H.S.

∴ z 0 =±(3 − 4i) . 1 1 1
| z1 + z 2 + ……. + z=
n | + + ……. + =
z1 z2 zn
Example 3: If n is a positive integer and ω be an
imaginary cube root of unity, prove that 1 1 1 1
+ + + ……. +
z1 z 2 z3 zn
3, when n is a mulitpleof 3
1 + ωn + ω2n 
0, when n is not a mulitpleof 3 1 1 1 1
= + + + ……. + = R.H.S.
z1 z 2 z3 zn
Sol: Case I: n = 3m; m ∈ I
∴ 1 + ωn + ω2n = 1 + ω3m + ω6m
Example 6: If | z1 + z 2 | = | z1 − z 2 | , prove that
3
= 1 + 1 + 1 [ ω =1] =3 π
argz1 − argz 2 = odd multiple of .
2
Case II: n = 3m + 1 or 3m + 2; m ∈ I
Sol: As we know | z | = z.z . Apply this formula and
(a) Let n = 3m + 1
M a them a ti cs | 6.35

consider
= z r ( cosθ + i sinθ ) . Equating real and imaginary parts.
n
| z1 + z 2 | 2 =| z1 − z 2 | 2 nπ
2 2 cos = a0 – a2 + a4 + …..
4
⇒ (z1 + z 2 ) (z1 + z2 ) = (z1 − z 2 ) (z1 − z2 ) or n

2 2 sin = a1 – a3 + a5 + …..
z1 z1 + z 2 z2 + z 2 z1 + z1 z2 = z1 z1 + z 2 z2 − z 2 z1 − z1 z2 4

or 0 ; Re (z1 z2 ) = 0
2(z 2 z1 + z1 z2 ) = Example 9: Solve the equation zn–1 = z :n∈N
Let=
z1 r1 (cos θ1 + isin θ1 ) and=
z 2 r2 (cos θ2 + isin θ2 );
Sol: Apply modulus on both side.
then
= z1 z2 r1r2 ( cos(θ1 − θ2 ) + isin(θ1 − θ2 ) )
Zn–1 = z ; |z|n–1 = | z | = |z|
∴ ∴= ( as Re(z1 z2 ) 0 )
cos (θ1 − θ2 ) 0=
∴ |z| = 0 or |z| = 1 If |z| = 0 then z = 0,
π
θ1 − θ2 = odd multiple of .
2 Let |z| = 1; then, zn = z z = 1
2mπ 2mπ
Example 7: If | z – 1 | < 3, prove that | iz + 3 − 5i |< 8 . ∴ z cos
= + isin : m = 0, 1, …., n – 1
n n
Sol: Here we have to reduce iz + 3 – 5i as the sum
1 − iz
of two complex numbers containing z – 1. because we Example 10: If z = x + iy and ω =
z −i
have to use
with |w| =1, show that, z lies on the real axis.
| z – 1 | < 3.
Sol: Substitute value of ω in |w| =1.
| iz + 3 – 5i | = | iz – i + 3 – 4i |
1 − iz
| ω |= = 1 ⇒ |1 – iz| = |z – i|
= | 3 – 4i + i (z – 1) | ≤ | 3 − 4i | + | i (z − 1) | z −i
(by triangle inequality) < 5 + 1 · 3 = 8 or, |1 – ix + y| = |x + i(y – 1)|
or, (1 + y)2 + x2 = x2 + (y – 1)2 or, 4y = 0
Example 8: If (1 + x) = a0 + a1x + a2x +……….…+ anx ,
n 2 n
Hence z lies on the real axis.
then show that
n
nπ Example 11: If a complex number z lies in the interior
(a) a0 – a2 + a4 + ….. = 2 2 cos
4 or on the boundary of a circle of radius as 3 and centre
n
nπ at (0, – 4) then greatest and least value of |z + 1| are-
(b) a1 – a3 + a5 + … = 2 2 sin
4
(A) 3 + 17 , 17 − 3 (B) 6, 1
Sol: Simply put x = i in the given expansion and then
by using formula (C) 17 , 1 (D) 3, 1

z r ( cosθ + i sinθ ) and ( cos θ + i sin θ )


n
= Sol: Greatest and least value of |z + 1| means maximum
= cosnθ + i sin nθ , we can solve this problem. and minimum distance of circle from the point (– 1, 0).
In circle greatest and least distance of it from any point
Put x = i in the given expansion is along the normal.
Y
(1 + i)n = a0 + a1i + a2i2 + …+ anin.
n
  π π 
 2  cos + isin   (-1,0) (0,0)
  4 4  X

= (a0 – a2 + a4 - …) + i (a1 – a3 + a5 - ….)


(0,-4)
n/2  nπ nπ 
2  cos + isin 
 4 4 

= (a0 – a2 + a4 + …….) + i(a1 – a3 + a5 + …..)


6 . 3 6 | Complex Number

∴ Greatest distance = 3 + 12 + 42 = 3 + 17 equilateral triangle. Also show that |z1| = |z2| = |z3|.

Least distance = 12 + 42 − 3 = 17 − 3 Sol: Use algebra to solve this problem.

Given z1 + z2 + z3 = 0, and from 2nd relation z2z3 + z3z1


Example 12: Find the equation of the circle for which + z1z2 = 0
 z − 6 − 2i 
arg   = π/ 4. ∴ z2z3 = – z1(z2 + z3) = – z1 (– z1) = z12
 z − 2 − 2i 
∴ z13 = z1z2z3 = z23 = z33
 z − 6 − 2i 
Sol: arg   = π / 4 represent a major arc of
 z − 2 − 2i  ∴ |z1|3 = |z2|3 = |z3|3
circle of which Line joining (6, 2)) and (2, 2) is a chord
Above shows that distance of origin from A, B, C is
π same.
that subtends an angle at circumference.
4
Origin is circumcentre, but z1 + z2 + z3 = 0

Z implies that centroid is also at the origin so that the


A O triangle must be equilateral.
A
A
A(2,2) M B(6,2) JEE Advanced/Boards
Example 1: For constant c ≥ 1, find all complex numbers
Clearly AB is parallel to real (x) axis, M is mid-point, z satisfying the equation z + c | z + 1| + i = 0
M ≡ (4, 2), OM = AM = 2
Sol: Solve this by putting z = x + iy.
∴ O = (4, 4) and OA = OM + AM = 2 2 Equation of
2 2 2
Let z = x + iy.
required circle is
The equation z + c |z + 1| + i = 0 becomes
| z − 4 − 4i | =
2 2
x + iy + c (x + 1)2 + y 2 + i =0
Example 13: If |z| ≥ 3, prove that the least value of
1 8 or x + c (x + 1)2 + y 2 + i(y + 1) =
0
z+ is .
z 3
Equating real and imaginary parts, we get
1 1
Sol: Here z + ≥ | z | − . y+1=0⇒y=–1 … (i)
z |z|
Now |z| ≥ 3
and x + c (x + 1)2 + y 2 =
0: x < 0  ...(ii)
1 1 1 1
∴ ≤ or – ≥−  … (i) Solving (i) and (ii), we get
|z| 3 |z| 3
Adding the two like inequalities x+c (x + 1)2 + 1 =0 or x2 = c2[(x + 1)2 + 1]
1 1 8
|z|− ≥3 − =  … (ii) or (c2 – 1) x2 + 2c2x + 2c2 = 0
|z| 3 3
1 8 If c =1, then x = – 1. Let c > 1 ; then,
Hence from (i) and (ii), we get z + ≥
z 3
8 −2c2 ± 4c 4 − 8c2 (c2 − 1) −c2 ± c 2 − c2
∴ Least value is x= =
3 2(c2 − 1) c2 − 1

As x is real and c > 1, we have: 1 < c ≤ 2


Example 14: If z1, z2, z3 are non-zero complex numbers
such that z1 + z2 + z3 = 0 and z1−1 + z 2−1 + z3−1 =
0 then (Thus, for c > 2 , there is no solution). Since both
prove that the given points are the vertices of an values of x satisfy (ii), both values are admissible.
M a them a ti cs | 6.37

Example 2: Find the sixth roots of z = 64i. Re(z) > 3 (see the Figure above)
π π (b) Let z = x + iy, then, x > 0 and y > 0
Sol: Here i = cos + isin and sixth root of z
2 2
i.e. zr = z1/6. y π y π
arg z = tan−1 tan ≤ ≤ tan
x 6 x 4
 π π
=z 64  cos + isin  ∴ zr = z1/6 Y
 2 2
y=x
 π π
 2rπ + 2rπ + 
= 2 cos 2 + isin 2 
 6 6  arg(z)=6
 
X
Where r = 0, 1, 2, 3, 4, 5 O

The roots z0, z1, z2, z3, z4, z5 are given by 1 y


≤ ≤ 1 ; x ≤ 3y and y ≤ x
 π π  3 x
=z 0 2  cos + isin 
 12 12  Hence the given inequality represents the region
1
 5π 5π  bounded by the rays y = x and y = x except the
=z1 2  cos + isin  3
 12 12  origin.

=
 9π 9π 
z 2 2  cos + isin  Example 4: If z12 + z22 – 2z1z2 cos θ = 0, show that the
 12 12  points z1, z2 and the origin, in the argand plane, are the
vertices of an isosceles triangle.
 13π 13π   π π 
=z3 2  cos + isin −2  cos + isin 
 =
 12 12   12 12  Sol: By using formula of roots of quadratic equation we
can solve it.
 17π 17π   5π 5π 
=z 4 2  cos + isin =−2  cos + isin  z12 + z22 - 2z1z2 cos θ = 0
 12 12   12 12 
2
z  z 
 21π 21π   9π 9π  ⇒  1  − 2  1  cos θ + 1 = 0
=z5 2  cos + isin −2  cos + isin 
 =
 12 12   12 12   z2   z2 

 z  2cos θ ± 4 cos2 θ − 4
⇒  1  =
Example 3: Locate the region in the Argand plane for
 z2  2
the complex number z satisfying
π π
(a) |z – 4| < |z – 2| (b) ≤ arg z ≤ B(z)
6 4
Sol: Consider z = x + iy and solve by using properties B(z1)
of modulus and argument.
(a) Let z = x + iy O

|x + iy – 4| < |x + iy – 2| = cos θ ± i sin θ


(x – 4) + y < (x – 2) + y or – 4x + 12 < 0
2 2 2 2
z1
⇒ = | cos θ ± i sin q| – 1
y z2
z1
⇒ =1 ⇒ |z1| = |z2| or OA = OB
z2
x’ x
O Hence points A(z), B(z) and the origin are the vertices of
an isosceles triangle.

y’ x=3
6 . 3 8 | Complex Number

Example 5: Let three vertices A, B, C (taken in clock


eiA −eiB −eiC
wise order) of an isosceles right angled triangle with
right angle at C, be affixes of complex numbers z1, z2, =−1 −eiA eiB −eiC , by (2)
z3 respectively. Show that (z1 – z2)2 = 2(z1 – z3) (z3 – z2). −eiA −eiB eiC
z 2 − z3
Sol: Here = e−iπ /2 . Therefore solve it using
z1 − z3 Take eiA, eiB and eiC common from C1, C2 and C3 and
algebra method. again put ei(A+B+C) = eiπ = –1.
π 1 −1 −1
Given CB = CA and angle ∠C =
2 ∴ ∆ = (– 1) (– 1) −1 1 −1
z 2 − z3 −1 −1 1
= e−iπ /2 or (z3 – z2)2 = i2(z1 – z3)2
z1 − z3
Now make two zeros and expand
(z3 – z2)2 = – (z1 – z3)2
∆ = – 4 which is purely real.
A(z1)

Example 7: Prove that |a + b|2 + |a – b|2


= 2(|a|2 + |b|2). Interpret the result geometrically and

deduce that c + c2 − d2 + c − c2 − d2 = |c + d| + |c
C(z3) B(z2)
– d| ; all numbers involved being complex
or

z32 + z22 – 2z2z3 + z12 + z32 – 2z1z3 = 0 Sol: By using algebra of complex number and modulus
property we can prove given expresson. And then by
Add and subtract 2z1z2, we get using Appolonius theorem we can interpret the result
geometrically.
z12 + z22 – 2z1z2 + 2z32 – 2z2z3 – 2z1z3 + 2z1z2 = 0, or
S = |a + b|2 + |a – b|2
(z1 – z2) + 2[z3(z3 – z2) – z1(z3 – z2)] = 0 or
2

= |a|2+ |b|2 + 2Re(ab) +|a|2+ |b|2 - 2Re(ab) = 2(|a|2 +


(z1 – z2)2 + 2(z3 – z1) (z3 – z2) = 0, or
|b|2) (proved)
(z1 – z2) = 2(z1 – z3) (z3 – z2).
2
Now |a + b|2 + |a – b|2 = 2(|a|2 + |b|2)

Example 6: If A, B, C be the angles of triangle then This is nothing but Appolonius theorem. In DOAB, M
is midpoint of AB on applying Appolonius theorem we
e2iA e−iC e−iB get
prove that e−iC e2iB e−iA is purely real. (b)
−iB −iA 2iC B(a+b)
e e e

Sol: Here A+B+C = π, therefore epi = cos π+i sin π = – 1. M(a)


And by using properties of matrices we can solve this O
problem.
e–pi = – 1  … (i)
A(a - b)

OA2 + OB2 = 2(AM2 + OM2)


ei(B+C) = ei(π–A) = epi e–iA = – e–iA
c+d c−d
e–i(B+C) = – eiA  … (ii) Now take a = and b =
2 2
Take eiA, eiB and eiC common from R1, R2 and R3 Then using result
respectively. ∆ = ei(A+B+C)
|a + b|2 + |a – b|2 = 2(|a|2 + |b|2)
iA −i(A + C) −i(A +B)
e e e  2 2
c+d c−d 
e −i(B + C)
e iB
e −i(B + A)
=RHS 2  + = |c + d| + |c – d|
 2 2 
−i(B + C) −i(C + A)
e e eiC  
M a them a ti cs | 6.39

2
2
 z1 a 1
 c+d c−d  c+d c−d 
L.H.S. 
= +  +  −  we get z 2 b 1 =0
 2 2   2 2 
    z3 c 1
On simplifying we get
Example 9: If b1 + b2 + b3 + b4 = 0 where b1 etc. are
L.H.S. = c + c2 − d2 + c − c2 − d2 non-zero real numbers, sum of no two being zero,
and b1z1 + b2z2 + b3z3 + b4z4 = 0 where no three of the
points z1, z2, z3, z4 are collinear then prove that the four
Example 8: Show that the triangles whose vertices are
point concyclic if b1b2 |z1 – z2|2 = b3b4 |z3 – z4|2.
z1, z2, z3 and a, b, c are

z1 a 1 Sol: Here the four points A, B, C, D will be concyclic if


PA.PB = PC.CD. therefore obtain PA, PB, PC and CD and
similar if z 2 b 1 = 0.
simplify.
z3 c 1
Here b1 + b2 = – (b3 + b4)
Sol: Consider triangle ABC and DEF are similar, therefore Also b1z1 + b2z2 = – (b3z3 + b4z4)
AB BC
= and ∠ABC = ∠DEF.
DE EF b1 z1 + b2 z 2 b3 z3 + b 4 z 4
Dividing these, =
Suppose z1, z2, z3 are given by A, B, C respectively and a, b1 + b2 b3 + b 4
b, c are given by D, E, F respectively. Since the triangle
AB BC z3
ABC and DEF are similar and
z1
= A C
DE EF b2
b4
∠ABC = ∠DEF = α(say)
P b1
 z − z2  a−b
b3
We have ∠B = arg  1
 z − z  = arg  c − b 
z4 z2
 3 2  D B
z1 − z 2 AB
⇒ = (cos α + i sin α)  … (i)
z3 − z 2 BC The left side gives the point that divides the line
DE segment joining A(z1), B(z2) in the ratio b2: b1 and the
a−b
and = (cos α + i sin α)  … (ii) right side gives the point that divides the line segment
c −b EF
joining the points C(z3), D(z4) in the ratio b4: b3. So the
A(Z1) D(a) line segments intersect at P which is
b1 z1 + b2 z 2 b3 z3 + b 4 z 4
Represented by as well as
b1 + b2 b3 + b 4
Now, AB = |z1 – z2|
B(X2) C(z3) E(b) F(c)
b2 b1
∴ PA
= z 2 ) & PB
(z1 − = (z1 − z 2 )
b1 + b2 b1 + b2
AB DE
Since AB = BC we have =
DE EF BC EF Also, CD = |z3 – z4|

Thus, from (i) and (ii) we get b4 b3


∴ PC
= z 4 ) & PD
(z3 − = (z3 − z 4 )
z1 − z 2 z − z2 a − b b3 + b 4 b3 + b 4
a−b
= ⇒ 1 =0
z3 − z 2 c − b z3 − z 2 c − b The four points A, B, C, D will be concyclic if PA.PB =
PC.CD
z1 − z 2 a−b 0 b1b2 2 b3b 4 2
⇒ z2 b 1 =0 i.e. = z1 − z 2 z3 − z 4
(b1 + b2 )2 (b3 + b 4 )2
z3 − z 2 c −b 0
i.e. b1b2 |z1 – z2|2 = b3b4 |z3 – z4|2
Applying R1 → R1 + R2 and R3 → R3 + R2
(∵ b1 + b2 = – (b3 + b4)}
6 . 4 0 | Complex Number

Example 10: Show that all the roots of the equation 2i sinnθ
= = itannθ . Taking negative sign,
z cos q0 + z cos q1 + z cos q2+......... + z cos qn–1 +
n n–1 n–2
2cosnθ
1 z 2n − 1 −2isin nθ
cos qn = 2 lie outside the circle | z |= where q0, q1 etc. similarly we get = = – i tan nθ
2 z 2n + 1 2cosnθ
are real.
z 2n − 1
∴ = | ± i tan nq| = |tan nq|,
Sol: By using triangle inequality. z 2n + 1
Here |zn cos q0 + zn–1 cos q1 + zn–2 cos q2 + …… + z For |± i| = 1.
cos qn–1 + cos qn| = 2  …. (i)

By triangle inequality. Example 12: Find the complex number z which satisfies
the condition |z – 2 + 2i| = 1 and has the least absolute
2 = |zn cos θ n + 2n–1 cos q1 + 2n–1 cos q2 + ….. + z
value.
cos qn–1 + cos qn | ≤ | zn cos qn| + |zn–1 cos q1|+
Sol: Here z – 2 + 2i = cos θ + i sin θ, therefore by obtaing
|zn–2 cos q2| + …..++ |z cos qn–1| + |cos qn| modulus of z we can solve above problem.

= |zn| |cos qn| + |zn–1| |cos qn| + ……..+ |z| |cos qn–1| + |z – 2 + 2i| = 1

|cos qn| ≤ |z|n + |z|n–1 + …. + |z| + 1 ⇒ z – 2 + 2i = cos θ + i sin θ

(∵ | cos q1| ≤ 1 and |zn+1| = |z|n+1) Where θ is some real number.

1− | z |n+1 1 1 ⇒ z = (2 + cos θ) + (sin θ – 2)i


= < ∴ 2<
1− | z | 1− | z | 1− | z |
⇒ |z| = [(2 + cos θ)2 + (sin θ – 2)2]1/2
1
So 1 – |z| is positive and 1-|z| <
2 = [8 + cos2 θ + sin2 θ + 4(cos θ – sin θ)]1/2
1 1
∴ |z| > 1 – =
1/2
  π 
2 2 = 9 + 4 2 cos  θ +  
  4 
1
∴ All z satisfying (i) lie outside the circle | z |=
2 |z| will be least if cos (θ + π/4) is least, that is, if cos

1 z 2n − 1 (θ + π/4)= – 1 or θ = . Thus, least value of |z| is
Example 11: If z + = 2cos θ , prove that 4
z
= |tan nq|. z 2n + 1
( )  1   1 
1/2
9−4 2 for z =2 −  + i − 2
Sol: By using formula of roots of quadratic equation,  2  2 
we can solve this problem.
Example 13: For every real number c ≥ 0, find all the
1
Here z + = 2 cos θ; complex numbers z which satisfy the equation. 2|z| – 4
z cz + 1 + ic = 0.
∴ z2 – 2 cos θ. z + 1 = 0
Sol: Substitute z = x + iy and equate real and imaginary
2cos θ ± 4 cos2 θ − 4 part to zero.
∴ z= = cos θ ± i sin θ
2
2 x2 + y 2 - 4c (x + iy) + 1 + ic = 0
Taking positive sign, z = cos θ + i sin θ
1 1
∴ = (cos θ + i sin θ)–1 = cos θ – i sin θ ∴ – 4 cy + c = 0 ⇒ y =  … (i)
z 4
1 1
2n zn − n 2 x2 + − 4cx + 1 =0 or
z −1 (cos θ + isin θ)n − (cos θ − isin θ)n
∴ = z
=
16
z + 1 zn + 1
2n
(cos θ + isin θ)n + (cos θ − isin θ)n  1 
n 4  x2 +  = (4 cx – 1)
2
z  16 
3
=
cosnθ + isin θ − (cosnθ − isinnθ) 4x2 (4c2 – 1) – 8 cx + =0
cosnθ + isinnθ + (cosnθ − isinnθ 4
M a them a ti cs | 6.41

π
8c ± 64c2 − 12(4c2 − 1) z3 − z1 z3 − z1 i 4  π π
∴ x= = = e 2  cos + isin 
8(4c2 − 1) z 2 − z1 z 2 − z1  4 4

4c ± 4c2 + 3 ⇒ z3 = z1 + (z2 – z1) (1 + i)


or x =  … (ii)
4(4c2 − 1) Similarly z4 = z1 + i(z3 – z1)
x is real as c ≥ 0, z = (x, y) as given by (i) and (ii), c ≥ 0. A(z1) D(z4)

Example 14: Consider a square ABCD such that z1, z2, z3, /4
and z4 represent its vertices A, B, C and D respectively.
Express ‘z3’ and ‘z4’ in terms of z1 & z2.

Sol: Consider the rotation of AB about A through an


π B(z2) C(z3)
angle .
4
π
z3 − z1 z3 − z1 i 4
Therefore = e .
z 2 − z1 z 2 − z1

JEE Main/Boards

Exercise 1

Q.10 If z = 1 + i tan α, where π < α < . find the value
Q.1 Find all non-zero complex numbers z satisfying of |z| cos α. 2
z = iz 2 .
Q.11 If 1, ω, w2 be the cube roots of unity, find the roots
1 + 2i + 3i2
Q.2 Express in the form A + iB. of the equation (x – 1)3 + 8 = 0.
1 − 2i + 3i2

Q.3 Find x and y if (x + iy) (2 – 3i) =(4 + i) Q.12 If |z| < 4, prove that |i z + 3 – 4i| < 9.

(1 + i) x − 2i (2 − 3i)y + i
Q.4 Find x and y if + i
= Q.13 2 + i 3 is a vertex of square inscribed in circle |z –
3+i 3−i
1| =2. Find other vertices.
Q.5 If x = a + b, y = aα + bβ and z = aβ +bα, where α and
β are complex cube roots of unity, show that xyz = a3 + b3. Q.14 Find the centre and radius of the circle formed
by the points represented by z = x +iy satisfying the
1 + 7i |z−α|
Q.6 in the polar form. relation = k(k ≠ 1) where α & β are constant
(2 − i)2 | z −β|
Q.7 Find the square root of – 8 – 6i. complex number’s given by α =α1 + iα2 & β =β1 + iβ2
Q.15 Prove that there exists no complex number z such
Q.8 Find the value of smallest positive integer n, for 1 a

n that | z |< and ∑ ar zr =1 where |ar| < 2.


1 + i 3 r =1
which   =1.
1 −i Q.16 Let a complex number α, α ≠ 11, be a root of
Q.9 Show that the complex number z = x + iy which the equation zp + q – zp – zq + 1 = 0, where p, q are
z − 5i distinct primes. Show that either 1 + α + α2 + ……. +
satisfies the equation = 1 lies on the x-axis. αp–1 = 0 or 1 + α + α2 + ……. + αq–1 = 0, but not both
z + 5i
together.
6 . 4 2 | Complex Number

Q.17 Show that the area of the triangle on the Argand Q.28 If |z| ≤ 1, |w| ≤ 1, show that
diagram formed by the complex numbers: z, iz and
|z – w|2 ≤ (|z| – |w|)2 + (arg z – arg w)2.
1
z + iz is: | z |2 .
2
Q.29 Let A and B be two complex numbers such
Q.18 If iz3 + z2 – z + i = 0 then show that |z| =1. A B
that + = 1, prove that the origin and the points
B A
Q.19 Find the value of the expression represented by A and B form the vertices of an
equilateral triangle.
1(2 – ω) (2 – w2) + 2(3 – ω) (3 – w2) + ….

+ (n – 1) (n – ω) (n – w2) where ω is an imaginary cube Q.30 Let z1, z2, z3 be three complex numbers and a, b, c
root of unity. be real number not all zero, such that a + b + c = 0 and
az1 + bz2 + cz3 = 0.
Q.20 If x =
1
2
( )
5 − 3i , then find the value of Show that z1, z2, z3 are collinear.
x4 – x3 – 12x2 + 23x + 12.
Q.31 If |z – 4 + 3i | ≤ 2, find the least and the greatest
Q.21 Let the complex numbers z1, z2 and z3 be the vertices values of |z| and hence find the limits between which
of an equilateral triangle. Let z0 be the circumcentre of |z| lies.
the triangle. Then prove that: z12 + z22 + z32 = 3z02. z1 − z 2
Q.32 If |z1| < 1 and < 1, then show that |z2| < 1.
1 − z1 z 2
Q.22 If z1, z2, z3 are the vertices of an isosceles triangle,
right angled at z2, prove that z12 + 2z22 + z32 = 2z2 (z1 + z3). | z |2 − | z | + 1
Q.33 Find the locus of points z if log < 2.
3 2+ | z |
Q.23 Show that the equation
2 2 2
Q.34 For complex numbers z and ω, prove that |z|2ω – |
2
A B C H ω |2 z = z – ω if and only if z = w or zω =1 .
+ + + ...... = x+,
x −a x −b x −c x −h
Where A, B, C, …., a, b, c, …. and  are real, cannot have
imaginary roots. Exercise 2
Single Correct Choice Type
Q.24 Find the common roots of the equation
z3 + 2z2 + 2z + 1 = 0 and z1985 + z100 + 1 = 0. Q.1 |z + 4| ≤ 3, Z∈ C: then the greatest and least value
of |z + 1| are:
Q.25 If n is an odd integer greater than 3 but not a (A) (7, 1) (B) (6, 1) (C) (6, 0) (D) None
multiple of 3, prove that [(x + y)n – xn – yn] is divisible by
xy (x + y) (x2 + xy + y2). Q.2 The maximum & minimum values of |z +1| when |z
+ 3| ≤ 3 are
Q.26 If α and β are any two complex numbers, (A) (5, 0) (B) (6, 0) (C) (7, 1) (D) (5, 1)

show that α + α2 − β2 + α − α2 − β2
Q.3 The points z1 = 3 + 3 i and z2 = 2 3 + 6i are given
= |α + β| + |α – β| on a complex plane. The complex number lying on the
bisector of the angle formed by the vectors z1 and z2 is:

Q.27 Let z1 = 10 + 6i and z2 = 4 + 6i. If z is any complex = (3 + 2 3) 3+2


(A) z + i
z − z1 π 2 2
number such that the argument of is , then
z − z2 4 (B) z = 5 + 5i
prove that |z – 7 – 9i| =3 2 . (C) z = – 1 – I
(D) None of these
M a them a ti cs | 6.43

Q.4 If z1, z2, z3, z4 are the vertices of a square in that Q.9 If q1, q2, q3 are the roots of the equation, x3 + 64 = 0,
order, then which of the following do(es) not hold q1 q2 q3
good?
then the value of the determinant q2 q3 q1 is:
z1 − z 2
(A) is purely imaginary q3 q1 q2
z3 − z 2 (A) 1 (B) 4
z1 − z3 (C) 10 (D) none of these
(B) is purely imaginary
z2 − z 4

z1 − z 2 Q.10 z = (3 + 7i) (p + iq) where p, q ∈ I – {0} purely


(C) is purely imaginary imaginary then minimum value of |z|2 is
z3 − z 4

(D) None of these 3364


(A) 0 (B) 58 (C) (D) 3364
3

Q.5 Let z1, z2 and z3 be the complex numbers Q.11 On the complex plane triangles OAP & OQR are
representing the vertices of a triangle ABC respectively similar and  (OA) = 1. If the points P and Q denotes the
and a, b, c are lengths of BC, CA, AB. If P is a point complex numbers z1 & z2 then the complex number ‘z’
representing the complex number z0 satisfying: denoted by the point R is given by:
a(z1 – z0) + b(z2 – z0) + c(z3 – z0) = 0, then w.r.t. the z1 z2 z1 + z 2
triangle ABC, the point P is its: (A) z1z2 (B) (C) (D)
z2 z1 z2
(A) Centroid (B) Orthocentre
Q.12 If A and B be two complex numbers satisfying
(C) Circumcentre (D) Incentre
A B
+ = 1 . Then the two points represented by A and B
B A
Q.6 Three complex numbers α, β & γ are represented
in the Argand diagram by the three points A, B, C and the origin form the vertices of
respectively. The complex number represented by D (A) An equilateral triangle
where A, B, C, D form a parallelogram with BD on a
(B) An isosceles triangle which is not equilateral
diagonal is:
(C) An isosceles triangle which is not right angled
(A) α – β + g (B) – α + β + g
(D) A right angled triangle
(C) α + β – g (D) α – β – g

Q.13 The solutions of the equation in z, |z|2 – (z + z ) +


Q.7 If the complex number z satisfies the condition |z| ≥ i (z – z ) + 2 = 0 are:
1
3, then the least value of z + is (A) 2 + i, 1 – i (B) 1 + i, 1 – i
z
(C) 1 + 2i, – 1 – I (D) 1 + i, 1 + i
5 8 11
(A) (B) (C) (D) None of these
3 3 3
Q.14 If z1 = – 3 + 5i: z2 = – 5 – 3i and z is a complex
number lying on the line segment joining z1 & z2 then
Q.8 Point z1 & z2 are adjacent vertices of a regular
arg z can be:
octagon. The vertex z3 adjacent to z2(z3 ≠ z1) can be
represented by: 3π π π 5π
(A) – (B) − (C) (D)
1 4 4 6 6
(A) z 2 + (1 ± i)(z1 + z 2 )
2 Q.15 The points of intersection of the two curves
1 |z – 3| = 2 and |z| = 2 in an argand plane are:
(B) z 2 + (1 ± i)(z1 − z 2 )
2
1 1
1 (A) (7 ± i 3) (B) (3 ± i 7)
(C) z 2 + (1 ± i)(z 2 − z1 ) 2 2
2
3 7 7 3
(C) ±i (D) ±i
(D) None of these 2 2 2 2
6 . 4 4 | Complex Number

Q.16 Let z to be complex number having the argument Previous Years’ Questions
π
θ, 0 < θ < and satisfying the equality |z – 3i| = 3.
2 Q.1 The smallest positive integer n for which
6 n
Then cot θ – is equal to: 1 + i
z   = 1 , is (1980)
1 −i
(A) 1 (B) – 1 (C) i (D) – i
(A) 8 (B) 16
Q.17 The locus represented by the equation, |z – 1| + (C) 12 (D) None of these
|z + 1| = 2 is:
(A) An ellipse with foci (1, 0): (– 1, 0) Q.2 The complex numbers z = x + iy which satisfy the
(B) One of the family of circles passing through the z − 5i
equation = 1 lie on  (1981)
points of intersection of the circles |z + 1| = 1 z + 5i
(C) The radical axis of the circles |z – 1| = 1 and |z + 1| = 1 (A) The x-axis

(D) The portion of the real axis between the points (B) The straight line y = 5
(1, 0) and (– 1, 0) (C) A circle passing through the origin
(D) None of these
Q.18 Let P denotes a complex number z on the
Argand’s plane, and Q denotes a complex number
Q.3 If z = x + iy and w = (1 – iz) / (z – i), then |w| = 1
π 
2 | z |2 cos  + θ  where θ = amp z if ‘O’ is the origin, implies that, in the complex plane  (1983)
 4 
(A) z lies on the imaginary axis
then the DOPQ is:
(B) z lies on the real axis
(A) Isosceles but not right angled
(C) z lies on the unit circle
(B) Right angled but not isosceles
(D) None of these
(C) Right isoscles
(D) Equilateral Q.4 The points z1, z2, z3, z4 in the complex plane are the
vertices of a parallelogram taken in order, if and only if
Q.19 Let z1, z2, z3 be three distinct complex numbers  (1983)
satisfying |z1 – 1| = |z2 –1| = |z3 – 1|. (A) z1 + z4 = z2 + z3 (B) z1 + z3 = z2 + z4
If z1 + z2 + z3 = 3 then z1, z2, z3 must represent the (C) z1 + z2 = z3 + z4 (D) None of these
vertices of:
(A) An equilateral triangle Q.5 If z1 and z2 are two non-zero complex numbers
such that |z1 + z2| = |z1| + |z2|, then arg (z1) – arg (z2) is
(B) An isoseles triangles which is not equilateral
equal to (1987)
(C) A right triangle π π
(A) – π (B) – (C) 0 (D)
(D) Nothing definite can be said 2 2
Q.6 The complex numbers sin x + i cos 2x and cos x – i
Q.20 If p = a + bω + cw ; q = b + cω + aw ; and
2 2 sin 2x are conjugate to each other, for (1988)
r = c + aω + bw2 where a, b, c ≠ 0 and ω is the complex (A) x = nπ (B) x = 0
cube root of unity, then:
 1
(C) x =  n +  π (D) No value of x
(A) p + q + r = a + b + c  2
(B) p2 + q2 + r2 = a2 + b2 + c2
Q.7 If ω (≠ 1) is a cube root of unity and (1 +ω)7 = A +
(C) p2 + q2 + r2 = 2(pq + qr + rp)
Bω, then A and B are respectively  (1995)
(D) None of these
(A) 0, 1 (B) 1, 1 (C) 1, 0 (D) – 1, 1
M a them a ti cs | 6.45

Q.8 Let z and w be two non-zero complex numbers z2


such that |z| = |w| and arg (z) + arg(w) = π, then z equals Q.16 If z ≠ 1 and is real, then the point represented
z −1
 (1995)
by the complex number z lies (2012)
(A) w (B) – w (C) ω (D) −ω
(A) Either on the real axis or on a circle passing through
the origin.
Q.9 If ω is an imaginary cube root of unity, then
(B) On a circle with centre at the origin.
(1 + ω – ω2)7 is equal to (1998)
(C) Either on the real axis or on a circle not passing
(A) 128 ω (B) – 128 ω (C) 128 w2 (D) – 128 w2
through the origin.
13
Q.10 The value of sum ∑ (in + i−n+1 ) where i= −1 (D) On the imaginary axis.
n=1 n
equals  (1998)
Q.17 If z is a complex number of unit modulus and
(A) i (B) i – 1 (C) – i (D) 0 1+z 
argument θ , then arg   equals (2013)
1 + z 
6i −3i 1
Q.11 If 4 3i −1 = x + iy , then  (1998) π
(A) −θ (B) θ (C) π − θ (D) −θ
2
20 3 i

(A) x = 3, y = 1 (B) x = 1, y = 1 Q.18 If z is a complex number such that | z | ≥ 2 , then


(C) x = 0, y = 3 (D) x = 0, y = 0 1
the minimum value of z +  (2014)
2

Q.12 If z1, z2 and z3 are complex numbers such that 5


(A) Is equal to
2
1 1 1
| z= 2 | | z3 | =
1 | | z= + + 1,
= (B) Lies in the interval (1, 2)
z1 z 2 z3
then |z1 + z2 + z3| is  (2000) 5
(C) Is strictly greater than
2
(A) Equal to 1 (B) Less than 1 3 5
(D) Is strictly greater than but less than
2 2
(C) Greater than 3 (D) Equal to 3
Q.19 A complex number z is said to be unimodular if
1 3 |z| = 1. Suppose z1 and z2 are complex numbers such
Q.13 Let ω = − + i , then value of the determinant
2 2 z − 2z 2
that 1 is unimodular and z 2 is not unimodular.
1 1 1 2 − z1 z2
1 −1 − ω 2
ω2 is  (2002) Then the point z1 lies on a:  (2015)
1 ω2 ω (A) Straight line parallel to y-axis

(A) 3w (B) 3 ω (ω – 1) (C) 3ω 2 (D) 3 ω (1 – ω) (B) Circle of radius 2


(C) Circle of radius 2
Q.14 If ω (≠1) be a cube root of unity and (1 + w2)n =
(D) Straight line parallel to x-axis
(1 + ω4)n, then the least positive value of n is (2004)
(A) 2 (B) 3 (C) 5 (D) 6 2 + 3i sin θ
Q.20 A value of θ for which is purely
1 − 2i sin θ
Q.15 A man walks a distance of 3 units from the origin imaginary, is: (2016)
towards the North-West (N 45° E) direction. From there,  3
π
he walks a distance of 4 units towards the North-West (A) (B) sin−1  
6  4 
(N 45° W) direction to reach a point P. Then, the position  
of P in the Argand plane is (2007)  1  π
(C) sin−1   (D)
 3 3
(A) 3eiπ / 4 + 4i (B) (3 − 4i)eiπ / 4
(C) (4 + 3i)eiπ / 4 (D) (3 + 4i)eiπ / 4
6 . 4 6 | Complex Number

JEE Advanced/Boards

Exercise 1 Q.8 If the expression z5 – 32 can be factorised into linear


and quadratic factors over real coefficients as (z5 – 32)
Q.1 Prove that with regard to the quadratic equation = (z – 2) (z2 – pz + 4) (z2 – zq + 4) then find the value
z2 + (p + ip′) z + q + iq′ = 0 where p, p′, q, q′ are all real. of (p2 + 2p).

(i) If the equation has one real root then q′2 – pp′ q′ +
Q.9 Let z1 & z2 be any two arbitrary complex numbers
qp′2 = 0
then prove that:
1 z z
(ii) If the equation has two equal roots then p2 – q′2 = | z1 | + | z 2 | ≥ (| z1 | + | z 2 |) 1 + 2 .
4q and pp′ = 2q ′. 2 | z1 | | z 2 |

state whether these equal roots are real or complex. Q.10 Let zi (i = 1, 2, 3, 4) represent the vertices of a
square all of which lie on the sides of the triangle with
Q.2 Let z = 18 + 26i where z0 = x0 + iy0 (x0, y0∈ R) is the vertices (0, 0), (2, 1) and (3, 0). If z1 and z2 are purely
cube roots of z having least positive argument. Find the real, then area of triangle formed by z3, z4 and origin
value of x0 y 0 (x0 + y 0 ) . is m (where m and n are in their lowest form). Find the
value of (m + n).
Q.3 Show that the locus formed by z in the equation
z3 + iz =1 never crosses the coordinate axes in the
Q.11 (i) Let Cr ’s denotes the combinatorial coefficients
Argand’s plane.
in the expansion of (1 + x)n, n ∈ N. If the integers
− Im(z)
Further show that | z |= an = C0 + C3 + C6 + C9 + ……
2Re(z)Im(z) + 1
bn = C1 + C4 + C7 + C10 + ……
Q.4 Consider the diagonal matrix An = dia (d1, d2, d3, …. and cn = C2 + C3 + C8 + C11+…..
dn) of order where
i2 π
then prove that
Di =ai–1, 1 ≤ i ≤ n and α =e n ; i= −1 , is the nth root
of unity. (a) an3 + bn3 + cn3 – 3anbncn = 2n.

Let L: represent the value of Tr. (A7)7. (b) (an – bn)2 + (bn – cn)2 +(cn – an)2=2

M: denotes the value of det (A2n+1) + det (A2n). (ii) Prove the identity:
(C0 – C2 + C4 – C6 + ……… )2
Find the value of (L + M).
+ (C1 – C3 + C5 – C7 + ….. )2 = 2n.
[Note: Tr(A) denotes trace of square matrix A]

Q.12 Let z1, z2, z3, z4 be the vertices A, B, C, D


Q.5 Let z1, z2 ∈ C such that z12 + z22 ∈ R. If z1 (z12 – 3z22) respectively of a square on the Argand diagram taken
in anticlockwise direction then prove that:
= 10 and z2(3z12 – z22)=30. Find the value of (z12 + z22).
(i) 2z2 = (1 + i) z1 + (1 – i) z3 & (ii) 2z4 = (1 – i) z1 + (1 + i) z3
Q.6 If the equation (z + 1) + z = 0 has roots z1, z2, ……
7 7

z7, find the value of Q.13 A function f is defined on the complex number by
7 7 f(z) = (a + bi)z, where ‘a’ and ‘b’ are positive numbers.
(a) ∑ Re(Zr ) and ∑ Im(Zr ) This function has the property that the image of each
r =1 r =1
point in the complex plane is equidistant from that
point and the origin. Given that |a + bi| = 8 and that
Q.7 If z is one of the imaginary 7th roots of unity, then
find the equation whose roots are (z + z4 + z2) and u
b2 = where u and v are co-primes. Find the value
(z6 + z3 + z5). v
of (u + v).
M a them a ti cs | 6.47

Q.14 Prove that Q.21 C is the complex number. f: C → R is defined


(a) cos x+ nC1 cos 2x + nC2 cos 3x + ….. + nCn cos (n + 1) by f(z)= |z3 – z + 2|. Find the maximum value of f(z)
if |z| = 1.
x n+2
x = 2n. cosn .cos  x
2  2  Q.22 Let a, b, c are distinct integers and w, w2 are the
(b) sin x + nC1 sin 2x + nC2 sin 3x + …….. + nCn sin(n + 1) imaginary cube roots of unity. If minimum value of |a +
1
x n+2 bw + cw2| + |a + bw2 + cw| is n 4 where n ∈ N, then find
x = 2 . cos
n n
.sin   x.
2  2  the value of n.

Q.15 Let f(x) = ax3 + bx2 + cx + d be a cubic polynomial Q.23 If the area of the polygon whose vertices are the
with real coefficients satisfying f(i)= 0 and f(1 + i) = 5. solutions (in the complex plane) of the equation
Find the value of a2 + b2 + c2 + d2.
x7 + x6 + x5 + x4 + x3 + x2 + x + 1 = 0
a b +c
Q.16 Let w1, w2, w3, …. wn be the complex numbers. A can be expressed in the simplest form as , find
the value of (a + b + c + d). d
line L on the complex plane is called a mean line for the
points w1, w2, w3, …. wn if L contains the points (complex
n Q.24 If a and b are positive integer such that
numbers) z1, z2, z3, …. zn such that ∑ (zk − ωk ) = 0 . N = (a + ib)3 – 107 i is a positive integer.
k =1

Now for the complex number w1 = 32 + 170i, w2 = Find N.


– 7 + 64i, w3 = –9 + 200i, ω4 = 1 + 27i and ω5 = – 14 + 43i,
Q.25 If the biquadratic x4 + ax3 + bx2 + cx + d = 0 (a, b, c, d
there is a unique mean line with y-intercept 3. Find the
∈ R) has 4 non real roots, two with sum 3 + 4i and the
slope of the line. other two with product 13 + i. Find the value of ‘b’.

Q.17 A particle start to travel from a point P on the Q.26 Resolve z5 + 1 into linear and quadratic
curve C1: |z – 3 – 4i| = 5, where |z| is maximum. From
factors with real coefficients. Deduce that:
3
P, the particle moves through an angle tan−1 in π π
4 4 sin cos = 1 .
anticlockwise direction on |z – 3 – 4i| = 5 and reaches 10 5
at point Q. From Q, it comes down parallel to imaginary
axis by 2 units and reaches at point R. Find the complex Q.27 If x = 1 + i 3 : y = 1 – i 3 & z = 2,
number corresponding to point R in the Argand plane.
prove that xp + yp = zp for every prime p > 3.
p
32  10  2qπ 2qπ  
Q.18 Evaluate: ∑ (3p − 2)  ∑  sin − icos Q.28 Dividing f(z) by z – i, we get the remainder i and
q 1 
= p 1=  11 11   dividing it by z + i we get the remainder 1 + i. Find the
remainder upon the division of f(z) by z2 + 1.
Q.19 Let a, b, c be distinct complex numbers
Q.29 (a) Let z = a + b be a complex number, where x
a b c
such that = = = k. and y are real numbers. Let A and B be the sets defined
1−b 1−c 1−a by
Find the value of k.
A = {z| |z| ≤ 2} and

Q.20 Let α, β be fixed complex numbers and z is a B = {z| (1 – i)z + (1 + i) z ≥ 4} .


variable complex number such that
Find the area of the region A ∩ B .
2 2
| z − α | + | z − β | =k
Find out the limits for ‘k’ such that the locus of z is a
circle. Find also the centre and radius of the circle.
6 . 4 8 | Complex Number

1 Q.2 The complex numbers whose real and imaginary


(b) For all real numbers x, let the mapping f(x) = , parts are integers and satisfy the relation zz 3 + z3 z =
350
x −i
form a rectangle on the Argand plane, the length of
where i= −1 . If there exist real number a, b, c and d
whose diagonal is-
for which f(a), f(b), f(c) and f(d) form a square on the
complex plane. Find the area of the square. (A) 5 (B) 10 (C) 15 (D) 25

Q.3 Let z1 and z2 be non-zero complex numbers


Q.30
satisfying the equation, z12 – 2z1z2 + 2z22 = 0.
Column I Column II The geometrical nature of the triangle whose vertices
(A) L et ω be a non-real cube root of unity then (p) 4 are the origin and the points representing z1 & z2.
the number of distinct elements in the set
(A) An isosceles right angled triangle
{(1 + ω + ω 2
}
+ ... + ωn )m |m,n∈N
(B) A right angled triangle which is not isosceles

(C) An equilateral triangle


(B) Let 1, ω, w2 be the cube root of unity. The (q) 5
least possible degree of a polynomial with (D) An isosceles triangle which is not right angled
real coefficients having roots

2ω,(2+3ω),(2+3w2),(2–ω-w2), is Q.4 The set of points on the Argand diagram which


π
satisfy both | z | ≤ 4 & Arg z = is:
3
(C) α=6+4i and β=(2 +4i) are two complex (r) 6 (A) A circle and line (B) A radius of a circle
numbers on the complex plane. A complex
number z. A complex number z satisfying (C) A sector of a circle (D) An infinite part line
z−α π
amp   = moves on the major Q.5 If z1 & z2 are two complex numbers & if
 z −β  6
z1 + z 2 π
segment of a circle whose radius is arg = but |z1 + z2| ≠ |z1 – z2| then the figure
z1 − z 2 2
formed by the points represented by 0, z1, z2 & z1 + z2 is:
(s) 7
(A) A parallelogram but not a square rectangle or a
rhombus
(B) A rectangle but not a square
Exercise 2
(C) A rhombus but not a square
Single Correct Choice Type (D) A square

Q.1 The set of points on the complex plane such that z2


Q.6 If z1, z2, z3 are the vertices of the ∆ABC on the
+ z + 1 is real and positive. (where z = x + iy, x, y ∈ R) is:
complex plane & are also the roots of the equation z3 –
(A) Complete real axis only 3az2 + 3bz + x = 0, then the condition for the ∆ABC to
be equilateral triangle is:
(B) Complete real axis or all points on the line 2x + 1 = 0
(A) a2 = β (B) α = b2
(C) Complete real axis or a line segment joining points
(C) a2 = 2β (D) α = 3b2
 1 3  1 3
− , &  − , −  excluding both.
 2 2   2 2  Q.7 Let A, B, C represent the complex numbers z1, z2, z3
   
respectively on the complex plane. If the circumcentre of
(D) Complete real axis or set of points lying inside the the triangle. ABC lies at the origin then the orthocenter
rectangle formed by the lines. is represented by the complex number:

2x + 1 = 0; 2x – 1 = 0; 2y – 3 = 0 & 2y + 3 =0 (A) z1 + z2 – z3 (B) z2 + z3 – z1


(C) z3 + z1 – z2 (D) z1 + z2 + z3
M a them a ti cs | 6.49

Q.8 Which of the following represents a point in an Q.15 If Ar (r = 1, 2, 3, ….., n) are the vertices of a regular
argand’s plane, equidistant from the roots of equation polygon inscribed in a circle of radius R, then
(z + 1)4 = 16z4?
(A1A2)2 + (A1A3)2 + (A1A4)2 + …+ (A1An)2 =
 1  1   2 
(A) (0, 0) (B)  − ,0  (C)  ,0  (D)  0,  nR 2
 3  3   5 (A) (B) 2nR2
2
π
Q.9 The equation of the radical axis of the two circles (C) 4R2 cot (D) (2n – 1) R2
presented by the equations. 2n

|z – 2| =3 and |z – 2 – 3i| = 4 on the complex plane is: Q.16 If the equation z4 + a1z3 + a2z2 + a3z + a4 = 0,
(A) 3y + 1 = 0 (B) 3y – 1 = 0 where a1, a2, a3, a4 are real coefficients different from
zero has a pure imaginary root then the expression
(C) 2y – 1 = 0 (D) None of these a3 aa
+ 1 4 has the value equal to
a1a2 a2a3
Q.10 Number of real solution of the equation, z3 + iz – 1
= 0 is (A) 0 (B) 1 (C) – 2 (D) 2
(A) Zero (B) One (C) Two (D) Three
Q.17 All roots of the equation (1 + z)6 + z6 = 0
Q.11 A point ‘z’ moves on the curve |z – 4 – 3i| = 2 in (A) Lie on a unit circle with centre at the origin
an argand plane. The maximum and minimum values
(B) Lie on a unit circle with centre at (– 1, 0)
of |z| are:
(C) Lie on the vertices of a regular polygon with centre
(A) 2, 1 (B) 6, 5 (C) 4, 3 (D) 7, 3
at the origin
 π (D) Are collinear
Q.12 Let z = 1 – sin α + i cos α where α ∈ 0,  , then
 2
the modulus and the principle value of the argument of Q.18 Number of roots of the equation z10 – z5 – 992 = 0
z are respectively: with real part negative is:
π α π α (A) 3 (B) 4 (C) 5 (D) 6
(A) 2(1 − sin α ),  +  (B) 2(1 − sin α ),  − 
 4 2  4 2
Q.19 z1 and z2 are two distinct points in argand plane.
π α π α
(C) 2(1 + sin α ),  +  (D) 2(1 + sin α ),  −  az1 bz 2
4 2 4 2 If a |z1| = b|z2|, then the point + is a point on
bz 2 az1
the (a, b ∈ R)
Q.13 z1 and z2 are complex numbers. Then
(A) Line segment [– 2, 2] of the real axis

z1 + z 2 z1 + z 2 (B) Line segment [– 2, 2] of the imaginary axis


Equ. + z1 z 2 + − z1 z 2 =
2 2 (C) Unit circle |z| = 1
(D) The line with arg z = tan–1 2
(A) 2 z1 + z 2 (B) 2 z1 − z 2

 2 2   2 2  Q.20 If ω is an imaginary cube root of unity, then the


(C) 2  z1 + z2  (D)  z1 + z2  value of (p + q)3 + (pω + qω2)3 + (pω2 + qω)3 is
   
(A) p3 + q3
Q.14 If α, β be the roots of the equation u – 2u + 2 = 0 2
(B) 3(p3 + q3)
n n
(x + α ) − (x + β)
and if cot θ = x + 1, then is equal to: (C) 3(p3 + q3) – pq (p + q)
α −β
(D) 3(p3 + q3) + pq (p + q)
sinnθ cosnθ sinnθ cosnθ
(A) (B) (C) (D)
n n n
sin θ cos θ cos θ sinn θ
6 . 5 0 | Complex Number

( ) Q.26 For two complex numbers z1 and z2


x
Q.21 The solution set of the equation 1 + i 3 − 2x =
0

(A) Form an A.P. (B) Form a G.P.


( az1 + bz1 )( cz2 + dz2 ) = ( cz1 + dz1 )( az2 + bz2 )
If (a, b, c, d ∈ R):
(C) Form an H.P. (D) Is a empty set
a c a b
(A) = (B) =
b d d c
Multiple Correct Choice Type
(C) |z1| = |z2| (D) arg z1 = arg z2
Q.22 In the quadratic equation x2 + (p + iq) x + 3i = 0,
p and q are real. If the sum of the squares of the roots
Q.27 Let z1, z2 be two complex numbers represented by
is 8 then
point on the circle |z1| = 1 and |z2| = 2 respectively, then:
(A) p = 3, q = – 1 (B) p = 3, q = 1
(A) Max |2z1 + z2| = 4 (B) Min |z1 – z2| = 1
(C) p = –3, q = – 1 (D) p = – 3, q = 1
1
(C) z 2 + ≤3 (D) None of these
Q.23 Let z1 and z2 be complex numbers such that z1 ≠ z2 z1
and |z1| – |z2|. If z1 has positive real part and z2 has negative
z + z2 Q.28 If α, β any two complex numbers such that
imaginary part, then 1 may be
z1 − z 2 α −β
= 1 , then
1 − αβ
(A) Zero (B) Real & positive
(C) Real and negative (D) Purely imaginary (A) |α| = 1 (B) |β| = 1
(C) α = e iθ , θ ∈ R (D) β = eiθ , θ ∈ R
Q.24 Given a, b, x, y ∈ R then which of the following
statement(s) hold good? Q.29 On the argand plane, let α = – 2 + 3z, β = – 2 – 3z
(A) (a + ib) (x + iy) = a – ib ⇒ x + y = 1
–1 2 2 and |z| = 1. Then the correct statement is:

(B) (1 – ix) (1 + ix)–1 = a – ib ⇒ a2 + b2 = 1 (A) α moves on the circle, centre at (– 2, 0) and radius 3

(C) (a + ib) (a – ib)–1 = x – iy ⇒ |x + iy| = 1 (B) α and β describe the same locus

(D) (y – ix) (a + ib)–1 = y + ix ⇒ |a – ib| = 1 (C) α and β move on different circles


(D) α – β moves on a circle concentric with |z| = 1
Q.25 If z = x = iy = r (cos θ + i sin θ) then the values of
Q.30 The value of in + i–n, for i= −1 and n ∈ I
z is equal to:
is:
(A) ±
1
2
( )
r + x + i r − x for y ≥ 0
(A)
2n
+
(1 + i)2n
(B)
(1 + i)2n
+
(1 − i)2n
(1 − i)2n 2n 2n 2n
(B) ±
1
2
( )
r + x − i r − x for y ≥ 0
(1 + i)2n 2n 2n 2n
(C) + (D) +
2n (1 − i)2n (1 + i)2n (1 − i)2n
(C) ±
1
2
( )
r + x + i r − x for y ≤ 0

Q.31 A complex number z satisfying the equation,


(D) ±
1
2
( )
r + x − i r − x for y ≤ 0
log14 (13 | z 2 − 4i |) + log196
1
0
=
(13 + | z 2 + 4i |)2
(A) Can be purely real
(B) Can be purely imaginary
(C) Must be imaginary
(D) Must be real or purely imaginary
M a them a ti cs | 6.51

Q.32 Let S be the set of real values of x satisfying the z1 z1′ 1


inequality
(A) z 2 z′2 1 = 0
| x + 1 + 2i | −2
1 − log2 ≥ 0 , then S contains: z3 z3′ 1
2 −1
(B) The two triangles are congruent
(A) [– 3, – 1) (B) (– 1, 1]
(C) The two triangles are similar
(C) [– 2, 2) (D) [1, 2]
(D) The two triangles have the same area.
Q.33 If x = cos α; y = cos β ; z = cos γ ; Where α, β, γ ∈
Q.38 If ‘z’ be any complex number in a plane (|z| ≠ 0)
R, then
then the complex number z for which the multiplication
inverse is equal to the additive inverse is:
(A) ∑× = Π × ⇒ cos (α – β) = 1
(A) 0 + i (B) 0 – i (C) 1 – i (D) 1 + i
x−y α −β
(B) Π 8Π cos
=
z 2 1 − ix
Q.39 Given z =a + bi = ; a, b, x ∈ R, then which of
1 + ix
x+y
(C) Π is real the following holds good?
z
π
(A) – < arg z ≤ 0 (B)– π < arg z ≤ 0
(D) ∑= Im(x) sin ( ∑ α )
(Re)x cos ( ∑ α ) , ∑ = 2
(C) |z| = 1 (D) arg z = π ; |z| = 1
π
Q.34 If xr = cos   for 1 ≤ r ≤ n; r, n ∈ N then:
 2r 
 n   n  Previous Years’ Questions
(A) Lim Re  ∏ xr  = −1 (B) Lim Re  ∏ xr  = 0
n→ ∞ r = 1  n→ ∞ r = 1 
    Q.1 If z is any complex number satisfying |z – 3 – 2i| ≤ 2,
then the minimum value of |2z – 6 + 5i| is ……  (2011)
 n   n 
(C) Lim Im  ∏ xr  = 1 (D) nLim Im  ∏ xr  = 0 2π
n→ ∞ r = 1  →∞ r = 1  i
    Q.2 Let ω =e and a, b, c, x, y, z be non-zero complex
3

number such that a + b + c = x, a + bω + cω2 = y,


Q.35 If 1, z1, z2, z3 …… zn–1 be the nth roots of unity and a + bω2 + cω = z. Then, the value of
ω be a non real complex cube root of unity then the
n −1 | x |2 + | y |2 + | z |2
is …… (2011)
product ∏ ( ω − zr ) can be equal to: | a |2 + | b |2 + | c |2
r =1

(A) 0 (B) 1 (C) – 1 (D) 2


Paragraph (for Q.3, 4, 5)

Q.36 Identify the correct statements(s). Read the following passage and answer the following
questions.
(A) No non zero complex number z satisfies the
equation, z = −4z Let A, B, C be three sets of complex number as defined
below
(B) z = z implies that z is purely real
(C) z = −z implies that z is purely imaginary A = {z: |Im z ≥ 1|}

(D) If z1, z2 are the roots of the quadratic equation B = {z: |z – 2 – i| = 3}


az2 + bz + c = 0 such that Im (z1z2) ≠ 0 then a, b, c must
C = {z: Re ((1 – i)z) + 2 ) (2008)
be real numbers

Q.37 If the complex numbers z1, z2, z3 & z1′, z2′ and z3′ Q.3 The number of elements in the set A ∩ B ∩ C is
are representing the vertices of two triangles such that (A) 0 (B) 1 (C) 2 (D) ∞
z3 = (1 – z0) z1 + z0z2 and z3 ‘ = (1 – z0) z1’ + z0z2’ where
z0 is also a complex number then:
6 . 5 2 | Complex Number

Q.4 Let z be any point in A ∩ B ∩ C . The |z + 1– i|2 + | Q.12 If i= −1 , then


z – 5 – i|2 lies between
334 365
 1 i 3  1 i 3
(A) 25 and 29 (B) 30 and 34 4 + 5 − +  + 3 − + 
 2 2   2 2 
(C) 35 and 39 (D) 40 and 44  
is equal to (1999)
Q.5 Let z be any point in A ∩ B ∩ C and let w any point
(A) l – i 3 (B) – 1 + i 3 (C) i 3 (D) − i 3
satisfying |w – 2 – i|< 3. Then |z| – |w| + 3 lies between
(A) – 6 and 3 (B) – 3 and 6 Q.13 If arg (z) < 0, then arg (– z) – arg (z) equal(2000)
(C) – 6 and 6 (D) – 3 and 9 π π
(A) π (B) – π (C) − (D)
2 2
5 5
 3 i  3 i
Q.6 If z =  +  + −  , then (1982) Q.14 If z1 = a + ib and z2 = c + id are complex numbers
 2 2  2 2
    such that |z1| = |z2| =1 and Re ( z1 z2 ) = 0 , then the pair
(A) Re (z) = 0 of complex numbers w1 = a + ic and w2 = b + id satisfied
(B) Im (z) = 0 by (1985)
(A) |w1| = 1 (B) |w2| = 1
(C) Re (z) > 0, Im (z) > 0
(D) Re (z) > 0, Im (z) < 0 (C) Re ( w1 w2 ) = 0 (D) None of these

Q.7 The inequality |z – 4| < |z – 2| represents the region Q.15 Let z1 and z2 be two distinct complex numbers
given by  (1982) and let z = (1 – t) z1 + tz2 for some real number t with 0
< t < 1. If arg (w) denotes the principal argument of a
(A) Re (z) ≥ 0 (B) Re (z) < 0 non-zero complex number w, then (2010)
(C) Re (z) > 0 (D) None of these (A) |z – z1| + |z – z2| = |z1 – z2|

Q.8 If a, b, c and u, v, w are the complex numbers (B) arg (z – z1) = arg (z – z2)
representing the vertices of two triangles such that z − z1 z − z1
c= (1–r) a+rb and w = (1 – r) u + rv, where r is a complex (C) =0
z 2 − z1 z2 − z1
number, then the two triangles  (1985)
(A) Have the same area (B) Are similar (D) arg (z – z1) = arg (z2 – z1)

(C) Are congruent (D) None of these


Q.16 Match the statements in column I with those in
6 column II. (2010)
 2πk 2πk 
Q.9 The value of ∑  sin 7
− icos
7 
 is (1987)
[Note: Here z takes values in the complex plane and Im
k =1 
z and Re z denote, respectively, the imaginary part and
(A) – 1 (B) 0 (C) – i (D) i the real part of z.]

Q.10 Let z and w be two complex numbers such that Column I Column II
|z| ≤ 1, |w| ≤ 1 and |z + iw| = |z – iw | = 2, then z equals (A) The set of points z (p) An ellipse with
 (1995) satisfying |z-i|z|| = |z + i|z|| 4
(A) 1 or i (B) i or – i (C) 1 or – 1 (D) i or –1 eccentricity
Is contained in or equal to 5

Q.11 For positive integers n1, n2 the value of expression (B) The set of points z (q) The set of points z
n n n2 n
satisfying |z+4|+|z-4|=10 is satisfying Im = z = 0
(1 + i) 1 + (1 + i3 ) 1 + (1 + i5 ) + (1 + i7 ) 2 ,Here i= −1 contained in or equal to
is a real number, if and only if (1996) (C) If |w|=2, then the set (r) The set of points z
(A) n1 = n2 + 1 (B) n1 = n2 – 1 1 satisfying | Im z | ≤ 1
of points z= w − is
w
(C) n1 = n2 (D) n1 > 0, n2 > 0 contained in or equal to
M a them a ti cs | 6.53

Then the number of distinct matrices in the set S is


(D) If |w| = 1, then the set (s) The set of points z
 (2011)
1 satisfying | Re z | ≤ 2
of points z= w + is (A) 2 (B) 6 (C) 4 (D) 8
w
contained in or equal to
The set of points z Q.22 Let z be a complex number such that the imaginary
satisfying | z | ≤ 3 part of z is nonzero and a = z 2 + z + 1 is real. Then a
cannot take the value  (2012)
1 1 3
2π 2π (A) -1 (B) (C) (D)
Q.17 Let ω be the complex number cos + i sin . 3 2 4
3 3
Then the number of distinct complex number z
1
Q.23 Let complex numbers α and lies on circle
satisfying  (2010) α
( x − x0 ) + ( y − y 0 ) r 2 and ( x − x0 ) + ( y − y0 ) =
2 2 2 2
= 4r 2 ,
2
z +1 ω ω
2
respectively. If z=
0 x0 + iy 0 satisfies the equation
ω z+ω 1 = 0 is equal to
ω2 1 z+ω |2 r 2 + 2 , then | α |=
2| z 0 = (2013)

1 1 1 1
(A) (B) (C) (D)
Q.18 Let z1 and z 2 be two distinct complex numbers 2 2 7 3
and let z = (1 − t ) z1 + t z2 for some real number t with
3 +i
0 < t < 1. If Arg (w) denotes the principal argument of a
nonzero complex number w, then  (2010)
Q.24 Let ω =
2
and P =ωn : n = {
1,2,3,..... . Further }
 1  −1 
z ∈ C : Rez >  and H2 =
H1 = z ∈ C : Rez <  ,
(A) | z − z1 | + | z − z 2 |= | z1 − z 2 |  2  2 
where C is the set of all complex numbers. If
(B) Arg ( z − z1=
) Arg ( z − z2 )
z1 ∈ P ∩ H1 , z 2 ∈ P ∩ H2 and O represents the origin,
z − z1 z − z1
(C) =0
z 2 − z1 z2 − z1 then ∠ z1 Oz 2 = (2013)

π π 2π 5π
(D) Arg ( z − z=
1) Arg ( z 2 − z1 ) (A) (B) (C) (D)
2 6 3 6

Q.19 if z is any complex number satisfying z − 3 − 2i ≤ 2


Q.25 Let ω be a complex cube root of unity with ω ≠ 1
, then the minimum value of 2z − 6 − 5i is  (2011)
and P = pij  be a n × n matrix with pij = ωi+ j . Then
 
Q.20 Let ω =ei π /3 , and a, b, c, x, y, z be non-zero P2 ≠ 0 , when n = (2013)
complex numbers such that (A) 57 (B) 55 (C) 58 (D) 56
a+b+c=x
Paragraph (for Q.26 and Q.27)  (2013)
a + b ω + c ω2 = y
Let S =S1 ∩ S2 ∩ S3 , where
a + b ω2 + c ω = z
  z − 1 + 3 i 
| x |2 + | y |2 + | z |2 S1 ={z ∈ C : | z | < 4} , S2 =
z ∈ C : Im   > 0
Then the value of is  (2011)   1 − 3 i  
| a |2 + | b |2 + | c |2
and S3 ={z ∈C : Re Z > 0}
Q.21 Let ω ≠ 1 be a cube root of unity and S be the set
Q.26 Area of S =
of all non-singular matrices of the form
 1 a b 10 π 20 π 16 π 32 π
  2
(A) (B) (C) (D)
 ω 1 c  where each if a, b and c is either ω or ω . 3 3 3 3
ω2 ω 1 
 
6 . 5 4 | Complex Number

Q.27 Min 1 − 3i − z = kπ kπ


z∈S Q.29 For any integer k, =
let αk cos   + i sin  ,
 7   7 
2− 3 2+ 3 3− 3 3+ 3 where i= −1 . The value of the expression.
(A) (B) (C) (D) 12
2 2 2 2
∑ | αk +1 − αk |
k =1
is  (2015)
Q.28 Match the following: (2014) 3
∑ | α 4 k −1 − α 4k −2 |
k =1
Column I Column II
(i) The number of polynomials f(x) with (p) 8 −1 + 3 i
non-negative integer coefficients of
Q.30 Let z = , where i= −1 , and r, s ∈ {1,2,3} .
2  
degree ≤ 2 , satisfying f (0) = 0 and  −z r z 2s 
1
Let P = 
( )  and I be the identity matrix of order
∫ f ( x )dx = 1 , is  z 2s
 z r 

0
2. Then the total number of ordered pairs (r, s) for which
(ii) The number of points in the (q) 2
P2 = −I is  (2016)
interval  − 13, 13  at which
 
( )
f ( x ) sin x2 + cos x2 attains its
= ( ) Q.31 Let a, b ∈  and a2 + b2 ≠ 0.
maximum value, is  1 
Suppose S =z ∈  : z = t ∈  , t ≠ 0  , where
(r) 4  a + ibt' 
2
3x2
(iii) ∫ dx equals i= −1 . If z = x + iy and z ∈ S, then (x, y) lies on
−2 (1 + e )x
 (2016)

(s) 0 1  1 
 1/2 (A) The circle with radius and centre  , 0  for
1 + x   2a  2a 
 ∫ cos 2x.log   dx  a > 0, b ≠ 0
  1 − x  
(iv)  −1/2
equals 1  1 
 1/2 1 + x   (B) The circle with radius − and centre  − , 0  for
 ∫ cos 2x.log   dx  a < 0, b ≠ 0 2a  2a 
  1 − x  
 0
(C) The x-axis for a ≠ 0, b = 0
Codes:
(D) The y-axis for a = 0, b ≠ 0
(i) (ii) (iii) (iv)
(A) r q s p
(B) q r s p
(C) r q p s
(D) q r p s
M a them a ti cs | 6.55

PlancEssential Questions
JEE Main/Boards JEE Advanced/Boards

Exercise 1 Exercise 1
Q.6 Q.9 Q.15 Q.7 Q.11 Q.13

Q.18 Q.22 Q.24 Q.16 Q.18 Q.25

Q.28 Q.31 Q.34 Q.29 Q.30

Exercise 2 Exercise 2
Q.2 Q.8 Q.10 Q.2 Q.6 Q.9

Q.13 Q.16 Q.18 Q.15 Q.19 Q.22

Q. 25 Q.27 Q.31
Previous Years’ Questions
Q. 33 Q. 36 Q. 39
Q.2 Q.4 Q.7

Q.10 Q. 13 Q.15 Previous Years’ Questions


Q.2 Q.4 Q. 8

Q.11 Q.14 Q.15

Answer Key

JEE Main/Boards
Exercise 1

3 i 5 14
Q.1 z = 0, i, ± – Q.2 – i Q.3
= x = ,y
2 2 13 13
  3π   3π  
Q.4 x = 3, y – 1 Q.6 2 cos   + isin    Q.7 ± (1 – 3i)
  4   4 
Q.8 n = 4 Q.10 – 1 Q.11 – 1, 1 – 2ω, 1 – 2ω2

Q.13 z1 =(1 − 3) + i ; z 2 = −i 3 ; z3 =(1 + 3) − i

 α − K 2β  α + β −K (n − 1)n 2
Q.14 Centre   , radius Q.19 [n + 3n + 4]
2
 1 − K  1 − K2 4
6 . 5 6 | Complex Number

Q.20 5 Q.24 ω, w2

Q.31 3 ≤ | z | ≤ 7 Q.33 Interior of circle x2 + y2 = 25

Exercise 2

Single Correct Choice Type

Q.1 C Q.2 D Q.3 B Q.4 C Q.5 D Q.6 A Q.7 B

Q.8 C Q.9 D Q.10 D Q.11 A Q.12 A Q.13 D Q.14 D

Q.15 B Q.16 C Q.17 D Q.18 C Q.19 A Q.20 C

Previous Years’ Questions

Q.1 D Q.2 A Q.3 B Q.4 B Q.5 C Q.6 D Q.7 B

Q.8 D Q.9 D Q.10 B Q.11 D Q.12 A Q.13 B Q.14 B

Q.15 D Q.16 A Q.17 B Q.18 B Q.19 B Q.20 C

JEE Advanced/Boards
Exercise 1
7
Q.2 12 Q.4 7 Q.5 10 Q.6 (a) − , (b) zero Q.7 x2 + x + 2 = 0 Q.8 4
2
Q.10 41 Q.13 259 Q.15 26 Q.16 163 Q.17 (3 + 7i) Q.18 48 (1 – i)
2
Q.19 – ω or –w2 Q.20 k > α − β

Q.21 If (z) is maximum when z = ω, when ω is the cube root unity v and If (z) = 13

Q.22 144 Q.23 8 Q.24 198 Q.25 51

iz 1
Q.26 (z + 1) (z2 – 2z cos 36° + 1) (z2 – 2z cos 108° + 1) Q.28 + + i
2 2
1
Q.29 (a) π – 2; (b) Q.30 A → s; B → q; C → p
2

Exercise 2

Single Correct Choice Type

Q.1 C Q.2 B Q.3 A Q.4 B Q.5 C Q.6 A Q.7 D

Q.8 C Q.9 B Q.10 A Q.11 D Q.12 A Q.13 D Q.14 A

Q.15 B Q.16 B Q.17 D Q.18 A Q.19 A Q.20 B Q.21 A


M a them a ti cs | 6.57

Multiple Correct Choice Type

Q.22 B, C Q.23 A, D Q.24 A, B, C, D Q.25 A, D Q.26 A, D Q.27 A, B, C Q.28 A, B, C, D

Q.29 A, B, D Q.30 B, D Q.31 A, B, D Q.32 A, B Q.33 D Q.34 A, D Q.35 A, B, D

Q.36 A, B, C Q.37 A, C Q.38 A, B Q.39 A, B, C, D

Previous Years’ Questions

Q.1 5 Q.2 3 Q.3 B Q.4 C Q.5 D Q.6 B Q.7 D

Q.8 B Q.9 D Q.10 C Q.11 D Q.12 C Q.13 A Q.14 A, B, C

Q.15 A, C, D Q.16 A → q, r; B → p; C → p, s, t; D → q, r, s, t Q.17 3 Q.18 A, C, D Q.19 5

Q.20 3 Q.21 A Q.22 D Q.23 C Q.24 C, D Q.25 B, C, D Q.26 B

Q.27 C Q.28 C Q.29 4 Q.30 1 Q.31 A, C, D

Solutions

JEE Main/Boards Sol 3: (x + iy)(2–3i) = 4 + i


⇒(2x + 3y) + i(2y– 3x) = 4 + i
Exercise 1
⇒2x + 3y = 4 and 2y – 3x = 1

Sol 1: z = i(z2) ⇒ Let x = a + ib 5 14


⇒x = and y =
13 13
⇒ a – ib = i(a2 – b2 + 2abi)
⇒ a = –2ab and –b = a2 – b2 (1 + i)x – 2i (2 – 3i)y + i
Sol 4: + i
=
a(1 + 2b) = 0 and a = b – b
2 2 3+i 3–i
1 [x + i(x – 2)][3 – i] + [3 + i][2y + i(1 – 3y)]
a = 0 or b = – =
2 10
if a = 0 ⇒ b = 0, 1 [3x + x – 2 + i(3x – 6 – x)] + [6y + 3y – 1 + i(2y + 3 – 9y)]
=
1 3 10
if b = – ,a=±
2 2 4 x – 2 + i(2x – 6) + (9y – 1) + i(–7y + 3)
=
3 i 10
Complex numbers are z = 0, i, ± −
2 2 4x + 9y – 3 + i(2x – 7y – 3)
= =i
10
1 + 3i2 + 2i −2 + 2i 1 − i
Sol 2: = =
1 + 3i2 – 2i −2 − 2i 1 + i 2x – 7y – 3 = 10 and 4x + 9y – 3 = 0
⇒x = 3 and y = –1
1 + i2 − 2i
=  1 − i  1 − i  =
  
= −i
 1 + i  1 − i  1 − i2
6 . 5 8 | Complex Number

Sol 5: x = a + b Solving (i) and (ii), we get


y = αa + bβ a = –1 & b = +3
z = aβ + bα a = 1 & b = –3
α and β are complex cube roots of unity So square root
⇒ αβ = 1, α2 = β, β2 = α = (–1 + 3i) and (1 – 3i)
(as α = ω, β = ω2, α2 = β)  ….. (i) n
1 + i
xyz = (a + b)(αa + bβ) (aβ + bα) Sol 8:   =1
1 −i
= (αa2 + αab + βab + b2β) (aβ + bα) (1 + i)2n
=1
= αa3β + αa2bβ + α2bβ2 + ab2β2 [(1 + i)(1 − i)]n
n
+ α2a2b + α2ab2 + ab2aβ + b3αβ (1 + i)2 
(1 + i)2n n
 = 1 ⇒ [2i] = 1
=1⇒ 
= aβ(a3 + b3 + a2b + ab2) + α2(a2b + b2a) 2n 2n 2n

+ β2 (a2b + b2a) ⇒in = 1 ⇒ n = 4, 8, 12

from eqn. (i) Minimum value of n is 4

= a3 + b3 +a2b+ab2+ (a2b + b2a) (α2 + b2)


z − 5i
Sol 9: =1
= a3 + b3 + a2b + ab2 + (a2b + b2a) (–1) z + 5i

= a3 + b3 + a2b + ab2 – a2b – ab2 z = x + iy


⇒|x + i(y – 5)| = |x + i(y+5)|
= a3 + b3 hence proved.
⇒x2 + (y – 5)2 = x2 + (y + 5)2
2 2
1 + 7i (2 + i) (1 + 7i)(2 + i) ⇒y = 0
Sol 6: × =
(2 − i) 2 2
(2 + i) 25
i.e. complex part of z is zero.
(1 + 7i)(3 + 4i) –25 + 25i z is pure real i.e. it lies on x axis.
= =
25 25
z = –1 + i Sol 10: z = 1 + itana
z(θ) = |z|e iq
|z| = 1 + tan2 =
α sec2 =
α sec α
|z| = 1 + 1 =2  π 3π 
For α ∈  , 
1 2 2 
tanθ = = –1
–1 secα < 0 ⇒ |seca| = –seca
3π |z| = –seca
⇒ θ = tan–1(–1) =
4
3π |z|cosα = –1
i   3π   3π  
⇒ z(θ) = 2e 4 = 2 cos   + isin   
  4   4  Sol 11: (x – 1)3 = –8

Sol 7: –8 – 6i= a + ib x – 1 = (–8)1/3

–8 – 6i = a2 – b2 + i(2ab) x – 1 = (81/3)(–1)1/3 ⇒ 1 – x = (8)1/3(1)1/3

⇒a2 – b2 = –8  …… (i) (–x + 1) = 2, 2ω, 2ω2

2ab = – 6 x = –1, 1 – 2ω, 1 – 2ω2

⇒ab = – 3  …… (ii)
M a them a ti cs | 6.59

Sol 12: |z| < 4  2α − 2β k 2   2α − 2β k 2 


⇒ x2 + y2 + x  1 2 1  +y  2 2

|3 + i(z – 4)| < 9(To prove)  k 2 − 1 
 k − 1 
We know that |z1 + z2| ≤ |z1| + |z2|  k 2β2 + k 2β2 − α2 − α2 
+  1 2 1 2
 =0
|iz + (3 – 4i)| ≤ |z| + |3 – i4|  2
k −1 

|z| < 4 Eqn. of circle

⇒|iz + (3 – 4i)| < 4 + 5  β k2 − α β k2 − α 


with centre as  1 1
, 2 2

 k 2 − 1 k 2 − 1 
⇒|iz + (3 – 4i)| < 9 Hence proved
 α − k 2β 
or  2

Sol 13: 2 + i 3 is vertex of square inscribed in  1 − k 
|z – 1| = 2
(α1 + β1 − k)2 + (α2 + β2 )2 α +β −k
and radius = =
1−k 2
1 − k2

n
1
Sol 15: |z| <
3
and ∑ ar zr = 1  … (i)
r =1

⇒ |a1z1 + a2z2 + a3z3 …… anzn| = 1

|z1 + z2 + z3+ …..zn| ≤ |z1| + |z2| + …..|zn| … (ii)

Here one vertex is A (2, 3) and equation of circle is ⇒ 1 ≤ |a1z| + |a2z2| + |a3z3| …..|anzn|
(x − 1)2 + y 2 =
4
⇒ |a1||z| + |a2||z|2 + |a3||z|3 …..|an||z|n ≥ 1
Radius of the circle is 2. Hence side of the square will
1
be 2 2 . ⇒ |z| + |z|2 …….|z|n ≥
2
Points that lie on the circle and are at a distance 2 2 ⇒ (Limiting case n → ∞)
from A are B (1 − 3,1) and D (1 + 3, −1) . z 1 1
≥ ⇒ z ≥ − (2)
The point C will be the other end of diameter of A. 1− z 2 3
Hence C (0, − 3) . From (i) and (ii), we can say that there is no ‘z’ satisfying
both conditions.
Hence the four vertices are

2 + i 3 , 1 − 3 + i , − 3i and 1 + 3 − i Sol 16: zp+q ‒ zp ‒ zq + 1 = 0


zp(zq ‒ 1).(zq ‒ 1) = 0
z−α (zp ‒ 1).(zq ‒ 1) = 0
Sol 14: =k
z −β zp = 1 or zq = 1
⇒|z – α|2 = k2|z – β| If α is roots, then αo, α, α2,………….. αn-1 also roots of
zp = 1 or zq = 1
α = α1 + iα2, β = b1 + iβ2 Sum of roots = 1 + α + α2 +…………………..+ αn-1 = 0
⇒ (x – α1)2 + (y – α2)2 = k2[(x – B1)2 + (y – β2)2]
Sol 17: z, iz, z + iz
⇒ x2 + α12 + y2 + α22 – 2xα1 – 2yα2
(x, y) (–y, x) (x – y, y + x)
= k2x2 + k2y2 + k2b12 + k2b22 – 2xb1k2 – 2yb2k2
x y 1
1 1
⇒ x (k –1) + y (k –1)+x(2α1 – 2b1k )
2 2 2 2 2
∆= −y x 1 = |–x2 – y2|
2 2
x−y x+y 1
+y(2α2–2b2k2)+k2b12+k2b22–α12–α22=0
6 . 6 0 | Complex Number

1 2 1 2 = (x – 3)[x3 + 2x2 – 6x + 5] + 27
⇒ |D| = (x + y2) = z
2 2
= (x – 3)[(x – 3)(x2 + 5x + 9) + 32] + 27

Sol 18: iz3 + i + z2 – z = 0 = (x – 3)2[x2 + 5x + 9] + 32(x – 3) + 27

iz(z2 + i) + i(z2 + i) = 0 = (x – 3)2[x2 – 3x + 8x – 24 + 33]+32(x – 3)+27

(iz + 1) (z2 + i) = 0 = (x – 3)3 (x + 8) + 33(x – 3)2 + 32(x – 3) + 27

Either z = i or z2 = i = w3(x + 8) + 33(w2) + 32w + 27

If z = I ⇒ |z| = |i| = 1 = w + 1 + 32w + 33w2 + 27

If z 2 =⇒
i | z |2 =1 ⇒ | z |=1 = w + w2 + 32w + 32w2 + 38

Hence, |z| = 1 = –1 – 32 + 38 = 5

Sol 19: 1(2 – w) (2 – w2) + 2(3 – w) (3 – w2) + …… (n – 1) Sol 21: z2 – z1 = (z3 – z1)e–iπ/3
(n – w) (n – w2)
z3 – z2 = (z1 – z2) e–iπ/3
Tn = n(n + 1 – w) (n + 1 – w ) 2
z3 − z1
z 2 − z1
=
= (n + n – nw) (n + 1 – w )
2 2 z3 − z 2 z1 − z 2

= n3 + n2 – n2w2 + n3 + n – nw2 – n2w – nw + nw3 2 2 2


⇒ z1 + z 2 + z3 = z1z2 + z2z3 + z3z1
= n3 + n2(2 – w2 – w) + n(1 – w – w2 + 1)  z1 + z 2 + z3 
2
2
  = z 0
= n3+ 3n2 + 3n  3 

Sn = STn = Σ(n3 + 3n2 + 3n) z12 + z 22 + z32 + 2(z12 + z 22 + z32 )


⇒ = z 20
9
= Sn3 + 3Sn2 + 3Sn
⇒ z12 + z 22 + z32 2
= 3 z0
2
 n(n + 1)  3n(n + 1)(2n + 1) 3n(n + 1)
=   + +
 2  6 2 Sol 22: z1 z2 z3 of an isosceles ∆L at z2
z1
n(n + 1)  n(n + 1) 
=  + (2n + 1) + 3
2  2 
n(n + 1) 2
= [n + n + 6 + 4n + 2]
4
n(n + 1) 2
Sn = STn = [n + 5n + 8]
4
z2
(n − 1)n 2 z3
Sn–1 = [n + 1 − 2n + 5n − 5 + 8]
4 Considering z2 at origin
(n − 1)n 2
= [n + 3n + 4] z1-z2
4

1 1
Sol 20: x = (5 − i 3) =(6 − 1 − i 3) =
3+w
2 2
x4 – x3 – 12x2 + 23x + 12
O z3-z2
= x3(x – 3) + 2x2(x – 3) – 6x(x – 3) (z1 – z2) = (z3 – z2)e iπ/2

+ 5(x – 3) + 27 z1 – z2 = (z3 – z2)i


M a them a ti cs | 6.61

(z1 – z2)2 = –(z3 – z1)2


2 α + α2 − β2 α − α2 − β2
⇒ z1 + z3 + 2z2 = 2z1z3 + 2z1z2
2 2 2

   α + α2 − β2 
⇒ z12 + 2z22 + z32 = 2z2(z1 + z3) =  α + α2 + β2   α + α2 − β2  +  
    

A2 B2 H2  2 2 
 α − α − β  + 2|α – a + b |
2 2 2
Sol 23: + .......... x +  Let x = p + iq
=
x −a x −b x −H  
2  2 2
2
A2 A (p− a) − iq = 2  α + α2 − β2  + 2 β
(p +  ) + iq = ∑ =∑  
(p − a) + iq (p − a)2 + q2
2 2
= 2 α + 2 β + 2 α2 − β2
A2 (p− a) A2q
= ∑ (p − a)2 + q2 − i∑ (p − a)2 + q2 2
= 2 α +2 β +2 α +β α −β
2

Equating the imaginary terms we get = αα + ββ + αβ + αβ + αα + ββ –


2
A q αβ − βα + 2 α + β α − β
q+∑ 0
=
(p − a)2 + q2 2 2
= α +β + α −β +2 α +β α −β
 A2 
⇒ q1 + ∑  =0 2
 (p − a)2 + q2  =  α + β + α − β 

A2 T = α + β + α − β Hence proved
⇒ q = 0 or ∑ (p − a)2 + q2 = −1

Sol 27: z1 = 10 + 6i; z2 = 4 + 6i


Here A, p, a and q are all real hence q = 0 is the only
solution. z − z1 x − 10 + (y − 6)i  (x − 4) − i(y − 6) 
=  
⇒ x cannot have imaginary roots. z − z2 x − 4 + (y − 6)i  (x − 4) − i(y − 6) 

(x − 10)(x − 4) + (y − 6)2 + i (y − 6)(x − 4) + (x − 10)(6 − y)


Sol 24: z3 + 2z2 + 2z + 1 = 0 =
(x − 4)2 + (y − 6)2
⇒ z = –1, ω , ω2
z − z1 π
z1985 + z100 + 1 = 0 arg =
z − z2 4
⇒ z = ω , ω2
⇒ (y – 6)(x – 4)+(x – 10)(6 – y)
Common roots are ω , ω2
= (x – 10) (x – 4) + (y – 6)2
Sol 25: Let f(x,y) = (x+y) – x –y n n n
⇒ (y–6)(x – 4 – x + 10)=x2 – 14x + 40 + (y–6)2
xy(x+y) (x +xy+y ) = (x-0)(y-0)(x+y)(x-wy)(x-w y)
2 2 2
⇒ 6(y – 6) = x2 – 14x + 40 + y2 + 36 – 12y
f(x=0)=0 and f(y=0)=0
⇒ x2 + y2 – 18y – 14x + 112 = 0
f(y=–x)=(x–x)n–(x)n–(–x)n=0
f(x=wy)=(wy+y)n–wnyn–yn ; = yn[w2n + wn +1] = 0 ⇒ (x – 7)2 + (y – 9)2 = 18

Similarly f(x=w2y)=0 ⇒ |z – 7 – 9i| = 3 2


∴ f(x) is divisible by xy(x+y)(x2+xy+y2)
( z − w )( z − w )
2
Sol 28: z − w =

Sol 26: T = α + α2 − β2 + α − α2 − β2 2 2
= z + w − z w − zw + 2 z w − 2 z w

( )
2 2 2
T2 = α + α2 − β2 + α − α2 − β2 + = z −w − z w − zw + 2 z w

Let z = r1cosθ1; w = r2cosθ2


6 . 6 2 | Complex Number

z1 – z2 = k1(z3 – z2)
(z − w)
2
= − 2r1r1 cos ( θ1 − θ2 ) + 2r1r2
⇒ z1, z2 and z3 are collinear (by vector)
  θ − θ 
( )
2
=z − w + 2r1r1  2sin2  1 2  
 
  2  Sol 31: |z – 4 + 3i| ≤ 2
2
 θ − θ2 
( )
2
=z − w + 4r1r1  sin 1 
 2 

We know, sin θ ≤ θ and r1, r2, ≤ 1


2
 θ − θ2 
( )
2
≤ z −w + 4 ×  1 
 2 

( ) + ( θ1 − θ2 )2
2
≤ z −w

( ) + (arg z − arg w )
2 2
≤ z − w
The shaded area show z
Min and max value = Distance of centre from origin ±
A B radius
Sol 29: + = 1
B A
= 5 ± 2 = 3, 7
A2 + B2 = AB
⇒ 3 ≤ |z| ≤ 7
A B
= 1−
B A
R z1 − z 2
A A −B Sol 32: <1
= 1 − z1 z 2
B A
2 2
lA-Bl z1 − z 2 < 1 − z1 z 2
lRl
A2
= A −B (z1 – z2) (z1 − z2 ) < (1 − z1 z 2 )(1 − z1 z2 )
B
lAl ⇒ |z1|2 + |z2|2 – z1 z2 – z1 Z2
2 O A
B
= B−A < – z1 z2 – z2 z1 + |z1|2|z2|2 + 1
A
⇒ |z1|2 – 1 < (|z1|2 – 1) |z2|2
2 2
A B
A −B = B − A ⇒ = 2
z1 − 1
B A
|z2| <
2
2
⇒|A | = |B |3 3 z1 − 1

⇒|A| = |B| = |A – B| |z2|2 < 1 ⇒ |z2| < 1

i.e. all sides are equal it forms an equilateral D


 z 2 − z + 1
Sol 30: a + b + c = 0 ⇒ b = –a – c Sol 33: log  <2
3  z +2 
az1 + bz2 + cz3 = 0  
2
z − z +1
az1 + cz3 – (a + c)z2 = 0 <3
z +2
az1 + cz3
z2 = ⇒ |z|2 – |z| + 1 < 3|z| + 6
a+c
⇒a(z1 – z2) + c(z3 – z2) = 0 ⇒ |z2| – 4|z| – 5 < 0
−c
⇒(z1 – z2) = (z3 – z2) ⇒ (|z| + 1) (|z| – 5) < 0
a
M a them a ti cs | 6.63

⇒ |z| + 1 ≥ 0 z1

So (|z| – 5) < 0 z3

Interior of circle x2 + y2 = 25

z2
Sol 34: |z| ω – | ω 2|z = z – ω
2
zˆ 3 = zˆ 2e ; zˆ 1 = zˆ 3eiθ

⇒ zzω − ωωz = z − ω
24i
zˆ 32 = zˆ 1 zˆ 2 =
⇒ z ω (z − ω) = z – ω 24

⇒ z(zω − 1) = ω(z ω − 1)  ….. (i) 1+i


ẑ3 = i=
2
⇒ z(zω − 1) = ω(z ω − 1) 1+i
Unit vector along bisector is , complex number
⇒ Either z ω = zω = 1 or z = ω 2
lying along this vector is 5(1+i)
x ω
= ⇒ zz = ωω ⇒ |z| = | ω |
ω z Sol 4: (C) z1, z2, z3, z4 vertices of square
|z| ( ω – z) = (z – ω )
2
z1 – z2 = i2Im(z1)
( ω – z) (|z| + 1) = 0 ⇒ ω = z
2
z3 – z2 = –2Re(z1)

Exercise 2
z1(x+iy)
z4(-x+iy)

Single Correct Choice Type


z3(-x-iy) z2(x-iy)
Sol 1: (C) z + 4 ≤ 3 Least and greatest value of |z + 1|
z1 − z 2
is imaginary
z3 − z 2
z2 – z4 = [–x + iy – (x – iy)]
-4 = 2Re(z) – i2lm(z)
z1 – z3 = 2x + 2iy = 2Re(z) + i2Im(z)
i.e. distance of z from (–1, 0) z1 − z3 2x + 2iy x + iy x2 − y 2 + 2ixy
= = =
Least is 0; maximum is 2r = 6 z 2 − z 4 2x − 2iy x − iy x2 + y 2

(x = y as it is aqueous)
Sol 2: (D) |z + 3| ≤ 3 Least & greatest value of |z + 1|
ie its minimum and maximum distance from (–1,0) is 1 2ixy
= (Purely Imaginary)
and 5. x + y2
2

az1 + bz 2 + cz3
Sol 5: (D) = z0
(-3,0)(-1,0) a+b+c
A(z1)

Sol 3: (B) z1 = 3 + i 3
c b
z2 = 2 3 + 6i

B(z2) a C(z3)
z0 is incentre.
6 . 6 4 | Complex Number

Sol 6: (A) δ – γ = α – b[parallel vector] |z3| = 1 ⇒ | z | =


| z1 z 2 |
δ=α+γ–β ⇒ z = z1z2
D() A()

A B
Sol 12: (A) + =1
B A
A
C() B() Let y =
B
1
Sol 7: (B) z ≥ 3 y+ 1
=
y
1 1 1 8
z+ ≥ z − =3– = ⇒ y2 − y + 1 =0
z z 3 3
1+ i 3
⇒y=
2
Sol 8: (C)
iπ iπ A 1+ i 3
⇒ =

± z3

± z2
z 2 = z1e 4
z3 = z 2e 4 B 2
/4 z1
(1 ± i) A
z3 – z2 = (z2 – z1) ⇒ 1
= A
2 B

(1 ± i) From Rotation Theorem
z3 = z2 + (z2 – z1)
2 A A iθ B
= e 
B B O
Sol 9: (D) x3 = (4)3 (–1)1/3
1+ i 3
x = –4, –4w, –4w2 ⇒ eiθ =
2
q1 q2 q3 −4 −4ω −4ω2 ⇒ θ =60o
2
q2 q3 q1 = −4ω −4ω −4
and A = B ⇒ ∠OAB = ∠OBA = 60o
q3 q1 q2 −4ω2 −4 −4ω
⇒ AOB is equilateral triangle
2
1 ω ω 0 0 0
2 2
= –64 ω ω 1 = –64 ω ω 1 =0 Sol 13: (D) |z|2 – (z + z ) + i(z – z ) + 2 = 0
2 2
ω 1 ω ω 1 ω
z z – (z + z ) + i(z – z ) + 2 = 0
Let z = a + ib
Sol 10: (D) z = (3 + 7i) (p + iq) is purely imaginary.
a2 + b2 – 2a + i(i2b) + 2 = 0
⇒ 3p – 7q = 0
a2 – 2a + 1 + b2 – 2b + 1 = 0
7q
⇒p= (a – 1)2 + (b – 1)2 = 0
3
2
|z|2 = |3 + 7i|2 |p + iq|2 = 58  p2 + q2  ⇒a=b=1
 
for minimum |z|, q = 3, p = 7 ⇒z=1+i
|z|2 = 58(49+9) = 3364
Sol 14: (D) z1 = –3 + 5i

Sol 11: (A) R(z) Q(z2)


P(z1) z2 = –5 – 3i

A(z3) Eqn of line y = 4x + 17


z1 z z1 z 2
= ⇒z=  4x + 17   17 
z3 arg z = tan–1 
z3 z 2  = tan 4 +
–1

 x   x 
M a them a ti cs | 6.65

 −5 −3  = |z|[(cosθ – sinθ) + i(sinθ + cosθ)]


x ∈ [–3, –5] ⇒ arg z ∈ tan−1 , tan−1 
 3 5  |z|i
= |z|[(cosθ+isinθ)+ (–sinθ + icosθ)]
 5π  i
Only option left is   |z|
 6  = |z| [cosθ + isinθ] + [–cosθ – isinq]
i
Sol 15: (B) | z − 3=| 2 : ( x − 3) + y=
22
4 = P + iP
2 2
| z=| 2 : x + y = 4  Q −P  π
⇒ arg  =
 P  2
Points of intersection lie on the radical axis S1 − S2 =
0
This is right angle triangle with |P| = |Q–P| and right
angle at P.

3 2
Sol 19: (A) |z1 – 1| = |z2 – 1| = |z3 – 1|
2
z 1 + z 2 + z3 = 3
Centroid is at z = 1
3
Radical axis is x =
2 
z1
z2  
x +y =4
2 2 

9 9
⇒ + y2 =
4 ⇒ y2 = 4 – 
4 4
7 ± 7
⇒ y2 = ⇒y=
4 2 Distance of vertical z1 , z 2 and z3 from centroid is same,
3 i 7 which mean centroid coincides with circumcenter.
Complex no. is ±
2 2 Therefore, ∆ is equilateral.

Sol 16: (C) |z – 3i| = 3 ⇒ (x)2 + (y – 3)2 = 9 Sol 20: (C) p = a + bw + cw2
y q = b + cw + aw2
⇒ tanθ =
x
6 x 6(x − iy) r = c + aw + bw2
x 6(x − iy)
cotθ = – = −
z y (x + iy)(x − iy) y x2 + y 2 p + q + r = a(1 + w + w2) + b(1 + w + w2)
x2 + y2 = 6y + c(1 + w + w2)
x 6(x − iy) = (a + b + c) (1 + w + w2) = 0
⇒ − =i
y 6y
p2+q2+r2=(p + q + r)2 – 2pq – 2qr – 2rp

Sol 17: (D) |z – 1| + |z + 1| = 2 = –2(pq + qr + rp)

pq + qr + rp = ab + acw + a2w2 + b2w2 + b2w + bcw2 +


abw3 + cbw2 + c2w3 + caw + …….
(-1,0) (1,0)
= ab+2acw+ab+c2+(a2+2bc)w2+b2w

The portion of real axis between (–1,0) & (1,0) as the = 2ab + c2 + 2acw + (b2 + 2bc)w2 + a2w + 2bc + a2 +
distance between both the point is 2 2baw + (b2 + 2ac)w2 + c2w + 2ac + b2 + 2bcw + (c2 +
a2 + 2ab)w2
2 π 
Sol 18: (C) Q = 2 z cis  + θ  = (2ab + 2bc + 2ac) (1 + w + w2)
4 
+ (c2 + b2 + a2) (1 + w2 + w)
 π  π 
= |z| 2 cos  + θ  + isin  + θ  
 4  4  p2 + q2 + r2 = 0 = 2(pq + qr + rp)
6 . 6 6 | Complex Number

Previous Years’ Questions ⇒


z1 + z3
=
z2 + z 4
⇒ z1 + z3 = z2 + z4
2 2
n
1 + i
Sol 1: (D) Since,   =1 Sol 5: (C) Given, |z1 + z2| = |z1| + |z2|
1 −i
On squaring both sides, we get
n
1 + i 1 + i
⇒  × 1
 = |z1|2 + |z2|2 + 2|z1| |z2|cos (arg z1 – z2)
1 −i 1 +i
n = |z1|2 + |z2|2 + 2|z1| |z2|
 2i 
⇒   = 1 ⇒ in=1
2 ⇒ 2|z1| |z2| cos (arg z1 – arg z2) = 2 |z1| z2|
The smallest positive integer n for which in = 1 is 4
⇒ cos (arg z1 – arg z2) = 1
∴n=4
⇒ arg (z1) – arg (z2) = 0
z − 5i
Sol 2:(A) Given, =1
z + 5i Sol 6: (D) Since, sin x + i cos 2x
⇒ |z – 5i| = |z + 5i| = cos x – i sin 2x
(if |z – z1| = |z – z2|,
⇒ sin x –i cos 2x = cos x – i sin 2x
Then it is a perpendicular bisector of z1
⇒ sin x = cos x and cos 2 x = sin 2 x < x
y
⇒ tan x = 1 and tan 2x = 1
π π
(0,5) ⇒x= and x = which is not possible at same time.
x’ 4 8
x
Hence, no such value exists.
(0,-5)

Sol 7: (B) (1 + ω)7 = (1 + ω) (1 + ω)6


y’
= (1 + ω) (–ω 2)6
∴ Perpendicular bisector of (0, 5) and (0, – 5) is x-axis.
=1+ω
⇒ A + Bω = 1 + 0 ⇒ A = 1, B = 1
Sol 3: (B) Since, |w| = 1
1 − iz
⇒ = 1 ⇒ |z – i| = |1 – iz| Sol 8: (D) Since, |z| =|w| and arg (z) =π – arg(w)
z −i
⇒ |z – i| = |z + i| Let w = reiθ, then ω = re–iθ

∴ z re
= =( ) rei π .e−i θ
π−θ
(∵ |1 – iz| = |– i| |z + i| = |z + i|)
∴ It is a perpendicular bisector of (0, 1) and (0, –1)     = – re–iθ = – ω
i.e., x-axis ∴ z lies on the perpendicular bisector of the line joining
Thus, z lies on real axis. –iw and – iω is the mirror image of – iw in the x-axis,
the locus of z is the x-axis
Sol 4: (B) Since, z1, z2, z3, z4 are the vertices of parallelogram.
Letz = x + iy and y = 0
D(z4) C(z3)
Now,|z| ≤ 1 ⇒ x2 + 02 ≤ 1
⇒–1≤x≤1 ….. (i)
∴ z may take values given in (i).

A(z1) B(z2)
Sol 9: (D)
∴ Mid-point of AC = mid-point of BD (1 + ω − ω2 )7 = ( −ω2 − ω2 )7 ( 1 + ω + ω2 = 0)
=( −2ω2 )7 =( −2)7 ω14 =−128ω2
M a them a ti cs | 6.67

Sol 10: (B) Applying R 2 → R1 : R3 → R3 − R1


13 13 13
1 1 1
∑ (in + in+1 ) =∑ in (1 + i) =(1 + i) ∑ in
2 2
=n 1=n 1 =n 1 = 0 −2 − ω ω −1
2 3 13  i(1 − i)  2
= (1 + i)(i + i + i + ..... + i ) = (1 + i)   0 ω −1 ω−1
 1−i 
=(1 + i)i =−1 + i = ( −2 − ω )(ω − 1) − (ω2 − 1)2
2

=−2ω + 2 − ω3 + ω2 − (ω4 − 2ω2 + 1)


Alternate solution: = 3ω2 − 3ω = 3ω(ω − 1) ( ω4 = ω)
Since, sum of any four consecutive powers of iota is
zero.
Sol 14: (B)
13
∴ ∑ (i + i n n+1 2 13 2 3
) =(i + i + ..... + i ) + (i + i + ..... + i ) 14
Given, (1 + ω2 )n= (1 + ω4 )n
n=1
⇒ ( −ω)n = ( −ω2 )n (ω3 = 1 and 1 + ω + ω2 = 0)
=i + i2 =i − 1
⇒ ωn =1
⇒ 3 is the least positive value of n.
n=
Sol 11: (D)
6i −3i 1 Sol 15: (D) Let OA=3, so that the complex number
Given, 4 3i −1 = x + iy associated with A is 3eiπ / 4 . If z is the complex number
20 3 i associated with P, then

6i 1 1 z − 3eiπ / 4 4 −iπ /2 4i
= e = −
⇒ − 3i 4 −1 −1 = x + iy 0 − 3e iπ / 4 3 3
20 i i ⇒ 3z − 9eiπ / 4 =
12ieiπ / 4
⇒ x + iy =
0 ( C2 and C3 are identical) ⇒ z =(3 + 4i)eiπ / 4
⇒ x=
0, y =
0 y
p

Sol 12: (A) 4


i/4
Given, | z=
1 | | z=
2 | | z=
3 | 1
3e
A
Now, | z1 |= 1
3
⇒ | z1 |2 =
1
/4
⇒ z1 z 2 =
1, z3 z3 =
1 x’
x
O
1 1 1
Again now, + + 1
= y’
z1 z 2 z3
⇒ | z1 + z 2 + z3 |=
1 z2
Sol 16: (A) Given: is real
⇒ | z1 + z 2 + z3 | =
1 z −1
⇒ | z1 + z 2 + z3 |=
1 Let z = a + ib, then

( a + ib ) a2 − b2 + 2aib × ( a − 1) − ib


2
z2
Sol 13: (B) =
=
( a − 1 ) + ib  ( a − 1 ) − ib 
z − 1 a + ib − 1    
1 1 1
Let =
∆ 1 −1 − ω 2
ω2 ⇒
( )
−b a2 − b2 + 2ab ( a − 1 )
0
=
( a − 1 ) + b2
2
1 ω2 ω

⇒ − a2b + b3 + 2a2b − 2ab =0


⇒ a2b + b3 − 2ab =
0
(
⇒ b a2 + b2 − 2a =
0 )
6 . 6 8 | Complex Number

b 0 or a2 + b2 −=
⇒= 2a 0 2 + 4i sin θ + 3 i sin θ − 6 sin2 θ
=
1 + 4 sin2 θ
⇒ Either real axis or circle passing through origin.
2 − 6 sin2 θ
Given =0
1+z  1 + 4 sin2 θ
Sol 17: (B) Let θ =arg  
1 + z  1
  ⇒ sin2 θ =
1+z  3
⇒ θ = arg   {| z |= 1 ⇒ z z = 1} 1
1 + 1  ⇒ sin θ = ±
  3
 z
 1  −1  1 
⇒ θ =arg ( z ) =⇒ θ sin−1   , − sin  
 3  3
1
Sol 18: (B) Given: The expression z + and z ≥ 2
2
Using triangle in equality JEE Advanced/Boards
z1 + z 2 ≥ | z1 | − | z 2 |
Exercise 1
1 1
⇒ z+ ≥ | z |− Sol 1: z2 + (p + ip’)z + (q + iq’) = 0
2 2
One real root
1 1
⇒ z+ ≥ z−
2 2 (z2 + pz + q) + i(p’z + q’) = 0
If z is real
1 3
⇒ z+ ≥ lies in (1, 2) −q'
2 2 P’z = –q’ ⇒ z =
p'
z2 + pz + q = 0
Sol 19: (B) z1 − 2z 2 = 1
2 − z1 z2 q'2 pq'
− +q=0
⇒ z1 − 2z 2 =2 − z1 z2 p' 2 p'
q'2 − pp'q'+ qp'2 =
0
⇒ ( z1 − 2z 2 )( z1 − 2 z2 ) =( 2 − z1 z2 )( 2 − z1 z 2 )
If eqn. has 2 equal roots
2 2
⇒ z1 − 2 z1 z2 − 2 z1 z 2 + 4 z 2
(p + ip’)2 = 4(q + iq’)
2 2
4 − 2 z1 z 2 − 2z1 z2 + z1
= z2 2
p= − p'2 4q and
= p'2 4q2
2 2 2 2
⇒ z1 + 4 z 2 4 z2
=+ z1 −(p + ip')
The roots are i.e. roots (equal) are imaginary.
2
⇒ z1 +  1 − z 2  − 4  1 − z 2  =
2 2 2
0
   
Sol 2: z = 18 + 26i
⇒  1 − z 2   z1 − 4  =
2 2
0 z0 = x0 + iy0 is cube root of z
  
z0 = (r{cosθ + isinq})1/3 = r1/3 eiθ/3
⇒ z1 = 4
2
{ z2 ≠1 }
 θ θ
⇒ z1 =
2 = r1/3  cos + isin 
 3 3 
2 + 3i sin θ r = (1000)1/2
Sol 20: (C)
1 − 2i sin θ r1/3 = (103/2)1/3 = 101/2

=
( 2 + 3i sin θ )(1 + 2i sin θ ) cosθ =
18
sinθ =
26

1 + 4 sin2 θ 1000 1000


M a them a ti cs | 6.69

cosθ = 0.57, sinθ = 0.82 2 π7(0)


i
(A7) =
7 e 7 + ei2π(1) + .......ei2π(6)
θ θ
cosθ = 3cos – 4cos3  
3 3 =cos0+isin0 + cos2π + isin2π…cos6π + isin6p
0.57 = 3t – 4t3
= [1+1 + …….. 7 times + i (0 + 0……….)] = 7
θ
cos   = 0.2, 0.74, 0.95 i

(K −1)
π  (2n+1)
i  2n

−2n 
3 det A2n = ∏ e 2n = e n 2 

θ
cos = 0.95
3 πi  4n2 −2n 
π
n 2  i(2n2 −n)
θ = e   = en = eiπ(2n – 1)
sin = 0.3162
3 det A2n+1 = ei2pn
z= 10 (0.95 + i 0.3162) = 3 + i det A2n + det A2n+1 = eiπ(2n – 1) + ei2pn
x0 = 3, y0 = 1 =cos(2n–1)π + isin(2n–1)π + cos2nπ + isin2np
x0y0(x0 + y0) = 3.4 = 12 M=0⇒L+M=7

Sol 3: z3 + iz = 1 Sol 5: z1(z12 – 3z22) = 10


z(z2 + i) = 1 z2(3z12 –z22) = 30
z can never be Purely real as (z2 + i) will be imaginary z1 = a + ib,Z2 = c + id
which multiplied with z will not be a real number,
0 as z12 + z22 is real
ab + cd =
hence its locus will never cut x-coo axis Z can never be
imaginary (pure) as (z2 + i) will be of form (a + ib) which ⇒ z13 – 3z1z22 + 3z12z2 – z23 = 40
multiplied by z, cannot form 1 (real) therefore its locus
⇒ z12(z1 + 3z2) – z22(z2 + 3z1) = 40
can never cut y axis.
⇒ (z1 + iz2)3 = z13 + iz23 – 3z12iz2 – 3z1z22
z(z2 + i) = 1
= 10 – 30i
(x + iy) [x2 – y2 + i(2xy + 1)] = 1
⇒ (z1 + iz2)3 = 10 + 30i
x3–xy2+i(2x2y+x+x2y–y3) – 2xy2 – y = 1
⇒ ((z1 + iz2)2)3 = 1000 ⇒ z12 + z22 = 10
x3–xy2–2xy2–y+i(3x2y+x–y3) = 1
x3 – 3xy2 – y = 1 & 3x2y + x – y3 = 0 Sol 6: (z + 1)7 + z7 = 0 has roots z1 …….. z7
x(x – 3y ) = 1 + y & y(y – 3x ) = x
2 2 2 2
Re(z1) + Re(z2) ……… Re(z7) = ?
1 1 + y x  1  y + x2 + y 2 
|z|2= −  +  ⇒ |z|2 = −   (z + 1)7 = (–z)7
2 x y 2  xy 
( ( a + 1) + ib ) = ( −a − ib )7
7

1   2xy + 1  − y
|z|2 = − y + | z |2  ⇒ |z|2  =
2xy    2xy  2xy One of the solution is
−1
−Im(z) ⇒ a + 1 = –a ⇒ a = and b = 0
⇒ |z| = 2
2Re(z)Im(z) + 1 −1
⇒z=
2
−7
Sol 4: An = dia(d1, d2, ……dn) z1 + z2 + z3 ……. z7 = + i(0)
2
di = ai-1; α = ei2π/n −7
Re(z1 +……….+ z7) =
2

⇒ dn = i (n−1) Im (z1 + z2 …+ z7) = 0
e n
L = Tr(A7)7
Sol 7: z = (1)1/7
M = det A(2n+1) + det(A2n)
α + β = (z + z2 + z3 + z4 + z5 + z6) = (-1) = -1
6 . 7 0 | Complex Number

αβ = (z + z4 + z2) (z3 + z5 + z6) a = 2c; b = 3c; b + c = 3


3 3 9
= z4 + z6 + z7 + + z7 + z9 + z10 + z5 + z7 +z8 4c = 3 ⇒ c = ,a = ,b =
4 2 4
= z 4 + z6 + 3 + z5 + z + z 3 + z2
0 0 1
= 3 + z + z 4 + z6 + z5 + z3 + z2 = 3 – 1 = 2
1 9 3
∆(0, z3, z4) = 1
Eqn → x2 + x + 2 = 0 2 4 4
3 3
1
Sol 8: z5 – 32 = z5 – 25 2 4
1 3 3 9 m
= (z – 2) (z2 – pz + 4) (z2 – qz + 4) = × × = =
2 4 4 32 n
= z5 + z4(–q – p – 2) + z3(+pq + 8 + 2p + 2q) + z2(–4p ⇒ m + n = 32 + 9 = 41
– 4q + (pq + 8) –2)

+ z(16 + 8p + 8q) – 32 = 0 Sol 11: (1 + x)n


an = C0 + C3 + C6 + …….C9
⇒p+q+2=0
bn = C1 + C4 + ………
2p + 2q + 8 + pq = 0
cn = C2 + C5 + C8 + ………..
⇒ pq + 4 = 0 ⇒ pq = –4
a3 + b3 + c3 – 3abc
4
p + q = –2 ⇒ p – = –2 (1+x)n = C0 + C1x+ C2x2+ C3x3……..Cnxn
p
⇒ p2 + 2p – 4 = 0 2n = C0 + C1 + C2 + ……..Cn

⇒ p2 + 2p = 4 (1+w)n = C0+w(C1 + C4 + C7 + ……..)


+ w2(C2 + C5 + C8 + ……..) + C3 + C6 ……

Sol 9: z + z ≥ 1 z + z z1 z2 (1 + w2)n = C0 + w2(C1 + C4 + C7 ……)


1 2
2 1 2 ( ) z1
+
z2
RHS + w(C2 + C5 + C8 + …..) + C3 + C6 …….

1 z1 z2 2n+ (1 + w)n + (1 + w2)n = 3(C0 + C3+C6 …)


(
z + z2
2 1
) z1
+
z2 2n+ (–w2)n + (–w)n = 3(C0 + C3+C6 …)

z1 z2 z1 z2 2nw2 + w(–w2)n + (–w)n = 3w2(C1 + C2+C7 …)


+ ≤ 2
=
z1 z2 z1 z2
2n + ( −ω2 )n + ( −ω)n
an =
1 1 z1 z2
2
(
× 2 z1 + z 2 ≥ )
z + z2
2 1
( ) z1
+
z2
3
2n ω2 + ω( −ω2 )n + ( −ω)n
bn =
1 z1 z2 3ω2
z1 + z 2 ≥
2 1
(
z + z2 ) z1
+
z2
ω2 2n + ( −ω2 )n + ω( −ω)n
cn =
Sol 10: 3ω2

z1 − z 2 iπ /2
Sol 12: = e= i
z4
(2,1)
z3 z3 − z 2

z1 − iz3 z1 (1 + i) + z3 (1 − i)
(0,0) z1 z2 (3,0) = z 2 ⇒ Z2 =
1−i 2
z1 − z 4
z1 = (a, 0), z2 = (b, 0), z3 = (b, c), z4 = (a, c) e−iπ /2 = −i ⇒ z1 – z4 = –i(z3 – z4)
z3 − z 4
b – a = c(square)
M a them a ti cs | 6.71

z1 (1 − i) + z3 (1 + i) a (1 + i ) + b (1 + i ) + c (1 + i ) + d =
3 2
⇒ z4 = 5
2
( ) ( )
⇒ a 1 + i3 + 3i2 + 3i + b 1 + i2 + 2i + c (1 + i ) + d =
5
Sol 13: f(z) = (a + ib)(z) = c + id
⇒ a (1 − i − 3 + 3i) + b (1 − 1 + 2i) + c (1 + i) + d =5
z = (x + iy)
⇒ a ( −2 + 2i) + b ( 2i) + c (1 + i) + d =5
Image = c – id
(c – x)2 + (y + d)2 = c2 + d2 ⇒ − 2a + c + d = 5 and 2a + 2b + c =0

x2 + y 2 − 2cx + 2dy =
0 From (i), we have

a2 + b2 = 64 d – a = 5 and 3a + 2d = 0

c = ax – by ⇒ a =c =−2 and b= d= 3
( −2) + 32 + ( −2) + 32
2 2
d = bx – ay ∴ a2 + b2 + c2 + d2 =

⇒ x2 + y2 – 2x(ax – by) + 2y(bx – ay) = 0 =4+9+4+9


= 26
⇒ x2(1 – 2a) + y2(1 – 2a) –2bxy + 2bxy = 0
⇒ (x2 + y2) (1 – 2a) = 0 n

1 1 255 u
Sol 16: ∑ ( zk − wk ) =
0
k =1
⇒a= ⇒ b2 = 64 – = =
2 4 4 v ⇒ z1 + z2 + z3 + z4 + z5
⇒ u + v = 259 ⇒ Z1 + Z 2 + Z3 + Z 4 + Z5
= 32 + 170i − 7 + 64i − 9 + 200i + 1 + 27i − 14 + 43i
Sol 14: = 3 + 504i

cosx + nC1cos2x + nC2cos3x + .......... + nCncos(n + 1)x If y-intercept is 3, then eq. of line

= cosx + nC1cos2x + nC2cos33x + .......... + nCncosn + 1x y = mx + 3

1 ++nnCCcisx n n 2 2 n n yi = mxi + 3 for i=1,2…….5


==cos
Cisx [1 11cosx++C2Cciscos + ..........++n CnC
x +x.............. cos
nCis x x]
 2 n 
Now, z1 + ……………..z5 = 3 + 504i
= cosx[1 + cosx]n
(x1 + x2 + ……… + x5) + i(y1 + y2 + ……..y5) = 3 + 504i +
n
 x x i{m(x1 + x2 ………… + x5) + 15} = 3 + 504i
=
= cosx
Cisx 2cos cosx
Cisx 
 2 2 x1 + x2 + ………….. + x5 = 3
xx nn
==22nnCos
Cosnn cos
Cis cos
Cis Cisx
Cisx m(x1 + x2 + ……….. + x5) + 15 = 504
22 22
⇒ 3m + 15 = 504
xx nn++22 ⇒m= 163
cosnn cos
==22nncos cis
cis xx
22  22 
R
x (n+ 2) x n + 2   Sol 17: P(3,9)
n n
2 cos cos + isin   x Q 37
2 2  2   53
(3,4)
By comparing, we get the desired results. Hence, proved
53

Sol 15: f ( x ) = ax3 + bx2 + cx + d f ( i ) = 0


⇒ − ai − b + ci + d = 0 + io ⇒ − a + c = 0 and d – b = 0
3
⇒a =c and b = d …(i) z − 3 − 4i =5 tan–1 (37°) in clockwise direction
4
And f (1 + i) = 5 It reaches Q (3, 9) above centre
Then it moves 2 unit downwards i.e. R (3, 7)
6 . 7 2 | Complex Number

p
32  10  2qπ 2qπ   2k + 2α1β1 + 2α2β2 − α12 − α22 − β12 − β22
Sol 18: ∑ ( 3p + 2 )  ∑  sin − icos  Radius =
= p 1=  q 1 11 11  4

p
  10 2qπi  Sol 21: f(z) = |z3 – z + 2 |z| = 1
= ∑ ( 3p + 2 )  −i  ∑ e 11 − 1 
  q=0 
 (f(z))2 = |z3 – z + 2|2=(z3 – z + 2)( z 3 – z + 2)

= 1–z2+2z3– z 2 +z z –2z+2 z 3 –2 z +4
32 32  32 p 
= ∑ (3p + 2 ) ip = ∑ (3p ) ip 2 ∑ i = 0  = 6 – (z2 – z 2 ) –2(z + z ) + 2(Z3 + z 3 )
p =1 p =1  1 
= 6–2(a2–b2)–4(a)+2(z+ z )(z2+ z 2 – z z )
= 3i(1 − 3 +  − 31) − 3(2 − 4 +  − 32) =48(1 − i)
= 6–2(a2–b2)–4(a)+4a(2a2–2b2–1)

a b c = 6–8a–2(2a2–1) + 8a(2a2–1)
Sol 19: = = = k
1−b 1−c 1−a f(z) = 16a3 – 4a2 – 16a + 8
a = k – kb f’(z) = 48a2 – 8a – 16 = 8(6a2 – a – 2)
b = k – kc = 8(6a2–4a + 3a – 2)
c = k – ka = 8(2a(3a – 2) + 1(3a – 1))
a = k – k2 + k2(k – ka) = 8(2a + 1) (3a – 2)
a = k – k + k –k a)
2 3 3
−1 2
a= a=
k −k +k 2 3 2 3
a= = b= c
1 + k3 2
For a =
3
but a ≠ b ≠ c 8 4 2
f(z) = 16 × –4× – 16 × +8
i.e. k = –1 ⇒ k = –w, –w
3 2 27 9 3
128 – 48 − 288 + 202
= <0
Sol 20: |z – a|2 + |z – b|2 = k 27
Locus of z is a circle −1
For a = ; f (z) = –2 – 1 + 8 + 8 = 13
(x – a1)2+(x – b1)2+(y – a2)2+(y – b2)2 = k 2
Maximum value of f(z) = 13
⇒ 2x2 + 2y2 – 2x(a1 + b1) – 2y2(a2 + b2)

+ a12 + a22 + b12 + b22 – k = 0 Sol 22: |a + bw + cw2| + |a + bw2 + cw| ≥


 α + β1   α2 + β2  |a + bw + cw2 + a + bw2 + cw|
⇒ x2 + y2 – 2x  1  − 2y  
 2   2  = |2a – b – c| = |a – b + a – c|
 α2 + α22 −k 
+ β12 + β22 Minimum value will occur when
+  1 =0
 2 
  a – b = k, a – c = –k
r > 0 i.e. g + f – c > 0
2 2
i.e. k = 1
2 2
 α1 + β1   α2 + β2   α2 + α2 + β2 + β2 − k 
+ − 2  1 2 1 2 >0 a – b = 1 and a – c = –1
 2   2   4 
      with a & b being least possible value integer values
⇒ – α12 + α22 − β12 − β22 + 2α1β1 + 2α2β2 + k > 0 ⇒ a = 2, b = 1, c = 3
⇒ k > α12 + α22 + β12 + β22 − 2α1β1 – 2α2β2 |a + bw + cw2| + |a + bw2 + cw|

⇒ k > (α1 − β1 )2 + (α2 − β2 )2 ⇒ k > |α – b|2 = 2|2 + w + 3w2| = 2|1 + 2w2|

 α + β1 α2 + β2  = 2|w2 – w| = 2|(– 3 )| = 2 3
Centre =  1 , 
 2 2  = 12 = n1/4 ⇒ n = 144
M a them a ti cs | 6.73

Sol 23: x7 + x6 + ……….. 1 = 0 ⇒ b = 26 + 25 = 51

 x8 − 1 
 =0 Sol 26: z5 + 1
 x −1  2
  3 1 = (z3 + 1) z2 + (1 – z2)

4 = (z + 1) (z2 – z + 1) z2 + (1 – z2)
= (1 + z)[(z2 – z + 1) z2 + 1 – z]
5 7 = (1 + z)[z4 – z3 + z2 – z + 1]
6
= (1 + z)[z2 + az + 1] [z2 + bz + 1]
Total area = unshaded area + shaded area
⇒ b + a = –1
ba + 2 = 1
0 0 1
Unshaded area = 6 × 1 0 1 1 ab = –1
1 1 1
1 ⇒a– = – 1 ⇒ a2 + a – 1 = 0
2 2 a
6 1 6 −1 ± 5 −1 + 5
= × = ⇒a= ⇒a= = –2cos108°
2 2 2 2 2 2
−1 − 5
1 1 1 1 And b = = –2cos36°
Shaded area = 2 × × × = 2
2 2 2 2
∴ Factors are (1 + z) (z2 – 2zcos36° + 1) (z2 – 2cos108° + 1)
6 1 6+ 2 6 2 +2 Since ab = –1
Total area = + = =
2 2 2 2 2 4 π π
⇒ 4cos36°cos108° = -1 ⇒ 4 cos cos = 1
5 10
3 2 +1 a b +c
= =
2 d Sol 27: x = 1 + i 3
a+b+c+d=3+2+1+2=8 y=1–i 3
z=2
Sol 24: N = (a + ib)3 – 107i x = –2wy = –2w2z = 2
= a3 – ib3 + 3a2ib – 3ab2 – 107i xp + ypp > 3 prime (P is odd)
N = (–b + 3a b – 107)i + a – 3ab
3 2 3 2
= –2pwp – 2p w2p = –2p(wp + w2p) = 2p = zp
⇒ 3a b – b = 107 ⇒ b(3a – b ) = 107
2 3 2 2

⇒ b =1 and 3a2 – b2 = 107 ⇒ a2 = 36 Sol 28: f(z) = a(z – i) + i ⇒ f(i) = i  … (i)

⇒ N = a3 – 3ab2 = 216 – 18 = 198 f(z) = b(z + i) +(1+ i) ⇒ f(–i) = 1+i  ... (ii)
f(z) = c(z2 + 1) + k1z + k2 … (iii)
Sol 25: x + ax + bx + cx + d has 4 non-real roots.
4 3 2
Substituting values from (i) and (ii) in (iii)
α + β = 3 + 4i ⇒ i = k1i + k2 and i + 1 = –k1i + k2
gδ = 13 + i(γ = α , δ = β ) 1 i
⇒ k2 = i + and k1 =
= 13 + i aβ = 13 – i
⇒ αβ 2 2
,
α + β = 3 + 4i iz 1
∴ Remainder = + i +
2 2
⇒ aβ + bγ + gδ + dα + aγ + bδ = b
⇒ aβ + β α + αβ + α α + β β + α β = b
⇒ 13 – i + 13 + i +( α + β )(α + β) = b
= 26+ (3 + 4i) (3 – 4i) = b
6 . 7 4 | Complex Number

Sol 29: (a) (c) α = 6 + 4iβ = 2 + 4i


z−α π
amp  =
 z −β  6
 (x − 6) + (y − 4)i (x − 2) − i(y − 4)  π
tan–1  (x − 2) + i(y − 4) (x − 2) − i(y − 4)  = 6
 
Re(z) = (x – 6) (x – 2) + (y – 4)2
Im(z) = (x – 2) (y – 4) + (x – 6) (y – 4)
A = {z| |z| ≤ 2}
(x − 2)(y − 4) + (x − 6)(4 − y) 1
B = {z| (1 – i)z + (1 + i) z ≥ 4} ⇒ =
2
(x − 6)(x − 2) + (y − 4) 3
Let z = a + ib
−4x − 2y + 8 + 4x + 6y − 24 1
⇒ (1 – i)(a + ib)+(1 + i)(a – ib) ≥ 4 ⇒
2 2
=
x + y − 8x − 8y + 12 + 16 3
⇒a+b+a+b ≥ 4⇒a+b ≥ 2
πr 2 1 2  π 1  2 ⇒ x2 + y2 – 8x – 8y + 28 = 4 3 y – 16 3
area = − r = −  r = (π – 2)
4 2  4 2 ⇒ x2+y2–8x+y(–8–4 3 )+28+16 3 =0
1 1  x +i
(b) f(x) = =  
x −i x −i x +i ⇒r= 16 + (4 + 2 3)2 − 28 − 16 3
x  1 
= + i 
2
x + 1  x2 + 1  = 16 + 16 + 12 − 28 + 16 3 − 16 3
k
x – coordinate = = 16 = 4(p)
2
k +1
1
y – coordinate =
k2 + 1
∴ Locus of the function will be
Exercise 2
x2 + y2 – y = 0 Single Correct Choice Type
This is circle with diameter 1
1 Sol 1: (C) z2 + z + 1 is real and +ve
Hence the area of the square =
2 x2 – y2 + x + 1 + i(2xy + y)
Sol 30: (a) (1 + w + ……..wn)m, m, n ∈ N x2 – y2 + x + 1 > 0 and 2xy + y = 0
−1
n ∈ 3k⇒ 1 ⇒ y(2x + 1) = 0 ⇒ y = 0 or x =
2
n ∈ 3k + 1⇒ (–w2)m If y = 0 x2 + x + 1 > 0 ⇒ z represents real axis
n ∈ 3k + 2⇒ 0 −1 3
If x = − y 2 > 0 , Line segment joining
m = 1 ⇒ (–w2), m = 2 ⇒ (w), m = 3 ⇒ (–1) 2 4
 1 − 3  1 3
m = 4 ⇒ (w2), m = 5 ⇒ (–w), m = 6 ⇒ (1) − ,  to  − , 
 2 2   2 2 
No. of distinct elements are 7(S)
(b) Real coefficient,
Sol 2: (B) zz 3 + z3 z =
350
Root → 2w, 2+3w, 2+3w , 2–w–w2 2

|z|2 (z2 + z 2 ) = 350


Roots are 2w, 2 + 3w, 2 + 3w2, 3

Other root will be 2ω -z- z


lzl
Total no. of roots are 5 (q)

-z -
z
M a them a ti cs | 6.75

Length of diagonal 2|z| ⇒ x ∈  −2,2 & y ∈  −2 3,2 3 


 
|z|2 (x2 - y2 + 2ixy + x2 – y2 – 2ixy)=350
Set of points lie on radius of circle
350
(x + y ) (x – y ) =
2 2 2
= 175 2
2
Sol 5: (C) z2
As x and y are integers z1+z2
x = ±4; y = ±3
z1-z2
⇒ |z| = 5
z1
Length of diagonal is 10

Sol 3: (A) z12 − 2z1z 2 + 2z 22 =


0
z1 + z 2 π
arg =
( z1 − z2 )
2
+ z 22 =
0 z1 − z 2 2

⇒ ( z1 − z 2 ) = z 2 and ( z1 − z 2 ) =
2
−z 22 =z1 − z 2 = iz 2  z + z2  π
tan–1  1  =
and  z1 − z 2  2

2 |z1 + z2| ≠ |z1 – z2|


 z1  z1
  − 2 + 2 =0 Diagonals are perpendicular but not equal ie figure
 z2  z2 z1
represented is rhombus but not a square.
z
⇒ 1 =+ 1 i z1
z2 z1- z2
Sol 6: (A) Condition for equilateral
z1
⇒ =2 z2 ∆ : z12 + z 22 + z32= z1 z 2 + z 2 z3 + z3 z1
z2
O z2 z3 – 3az2 + 3bz + x = 0
Now,
z1, z2, z3
z1 − 0 z1 i∠z1oz2
= e ⇒ z1 + z2 + z3 = 3a
z2 − 0 z2
i∠z1oz2 1+ i ⇒ z1z2 + z2z3 + z3z1 = 3b
e = = eiπ / 4
2
⇒ z1, z2, z3 = –x
⇒ ∠z1oz 2 = 45o = ∠oz1z 2 2 2 2
(z1 + z2 + z3)2 = z1 + z 2 + z3 + 2(3β)
Also, (3α)2 = 9b
z 2 − z1 z 2 − z1 i∠0z2 z1
= e a2 = b
z2 z2

⇒e
i∠oz2 z1
= i ⇒ ∠oz 2 z1 = 90o Sol 7: (D) z1

Sol 4: (B) 2 1
23 O G C
-2 z2 z3
4

23 z1 + z 2 + z3
= Centroid,
= zc 0
3
π
|Z| ≤ 4 and Arg ( z ) =
3 z1 + z 2 + z3 2z C + z O
= ⇒ zO = z1 + z2 + z3
3 3
x2 + y2 ≤ 16 and y = 3 x
6 . 7 6 | Complex Number

Sol 8: (C) Sol 11: (D) |z-4-3i|=2


z lies on a circle shown in the figure |z|=|z-0| is nothing
but distance from origin
-1 1 y
3
Q
C
(z + 1)4 = 16z4
(4i3)
⇒ z + 1 = 2z, 2iz, –2z, –2iz P
1 1 1 1 −1 − 2i −1 + 2i
⇒ z = 1, , − , = − , 1, ,
2i − 1 3 −1 − 2i 3 5 5 O x
z1 + z 2
Point equidistant is z =
2 Minimum = OP = OC-CP
 −1  =42 + 32 − 2 =5-2=3
Where z1 = (1, 0), z2 =  ,0 
 3 
Maximum = OC + CQ =5+9=7
1
1−
z= 3 =1
2 3 Sol 12: (A) z = 1 – sinα + icosa

|z| = 1 + sin2 α − 2sin α + cos2 α


Sol 9: (B) |z – 2| = 3
= 2 − 2sin α = 2(1 − sin α )

cos α  1 
arg z = tan–1 = tan−1  
1 − sin α  sec α − tan α 
2
 α α
 sin + cos 
1 + sin α  2 2
= tan–1 = tan–1
cos α 
2
2 α 2 α
 cos − sin 
 2 2
|z – 2 – 3i| = 4  α α  α
 sin + cos   1 + tan 
⇒S1 = (x – 2) + y = 9
2 2
= tan−1  2 2  = tan–1  2
⇒S2 = (x – 2)2 + (y – 3)2 = 16  cos α − sin α   1 − tan α 
   
 2 2  2
Both circles are intersecting. So, radical axis will be
  π α
S1 – S2 = 0 ⇒ 9 – 6y = 7 ⇒ 3y – 1 = 0 = tan–1 tan  π + α   = +
  4 2  4 2

Sol 10: (A) z3 + iz − 1 =0


z1 + z 2 z1 + z 2
Let z=k (real) Sol 13: (D) + z1 z 2 + − z1 z 2
2 2
⇒ k + ik ‒ 1 = 0
3

k3 + ik ‒ 1 = 0 and k = 0 z1 + z 2 + 2 z1 z 2 + z1 + z 2 − 2 z1 z 2
=
If k = 0, then 0 ‒ 1 = 0 2

( ) +( )
2 2
Not possible z1 + z 2 z1 − z 2
=
Therefore z ≠ k (real) 2
Hence, no real solution
2 ( z1 ) + 2 ( z 2 )
2 2 2 2
= = z1 + z2
2
M a them a ti cs | 6.77

Sol 14: (A) u2 – 2u + 2 = 0 a3 a1a4 b2 a4 b 4 a2 + a4 a2 b2a22


+ = a + = = = 1
⇒(u – 1)2 = –1 a1a2 a2a3 2 a2b2 a22b2 b2a22
⇒u – 1 = ±i
Sol 17: (D) (1+z)6+z6=0
⇒u = 1 + i, 1 – i = (α, β)
⇒ (1 + z ) =
6
cotθ = x + 1 −z6
Now,
(x + α)
n
( cot θ + i) − ( cot θ − i)
n n
− (x + β)n
= 6
α −β 2i  1+ z 
⇒ −1
 =
 z 
( cos θ + isin θ ) − ( cos θ − isin θ )
n n
(4, 3)
= 1
⇒ 1 + = ( −1)
n 1/6
2isin θ
z
eiθn − e−iθn sinnθ
= = 1 2Kπ + π
n
2isin θ sinn θ =
⇒ cos −1
z 6
Sol 15: (B) A1…….An vertices of regular polygon in a
circle of radius R 1  2Kπ + π  2Kπ + π π  
⇒ =− 2sin cos  − 
z  12  12 2  
A2
A1
A3 1   π 2Kπ + π  
⇒ z =− cos  − 
R 2Kπ + π  2 12  
2sin
12
A4
1 2Kπ + π 
z=
− 1 + icot 
(A1A2)2 + (A1A3)2 …….(A1An)2 2 12 
π 1
A1A2 = 2Rsin ⇒ z lies on line x =

n 2
2π ⇒ All roots are collinear.
A1A3 = 2Rsin
n

A1A2 = 2R Sol 18: (A)
n
 2π 2 2π 2 3π 2 (n − 1)π 
4R2 sin n + sin n + sin n ...sin n 


 2π 4π π
=−2R 2 1 − n + cos + cos + ...cos(2n − 1) 
 n n n
 
  z10 – z5 = 992
sin πk 2nπ 
=–2R2 1 − n + − cos = 2nR2
 π n  z5(z5 – 1) = 32(31)
 sinn   
 n  z5 = t
⇒ t2 – t = 992 ⇒ t2 – 32t + t – 992 = 0
Sol 16: (B) z4 + a1z3 + a2z2 + a3z + a4 = 0
⇒ t = 32, –31 ⇒ z5 = 32, –31
z = ib
⇒ z5 = 32 has 2 roots with –ve real Part
b4 – ib3a1 – b2a2 + iba3 + a4 = 0
and z5 = –31 has 3 roots with –ve real part
b – b a2 + a4 = 0 ⇒ b a2 + a4a2 = b a2
4 2 4 2 2

⇒ ba3 – b3a1 = 0 ⇒ a3 = b2a1


6 . 7 8 | Complex Number

Sol 19: (A) a|z1| = b|z2| Sol 24: (A, B, C, D) (a) a, b, x y is ∈ R

a z2 az bz az1 a + ib
= a − ib
= T = 1 + 2 let =z x + iy
b z1 bz 2 az1 bz 2
a2 − b2 + 2abi
⇒ x + iy =
1z z a2 + b2
T=z+ =z+
zz | z |2 2
a −b 2
2ab
⇒x= ,y=
|z| = 1 ⇒ T = z + z = 2Re(z) 2
a +b 2
a + b2
2

Re(z) ∈ (–1, 1) ⇒ x2 + y2 = 1 A is correct

⇒ T ∈ [–2, 2] 1 − ix
(b) = a − ib
1 + ix
Sol 20: (B) (p+q)3 + (pw+qw2)3 + (pw2+qw)3 1 − x2 − 2ix
⇒ = a − ib
= p3+q2+3p2q+3pq2+p3+q3+3pq2w5 1 + x2
1 − x2 2x
⇒a= ,b=
+3p2qw4+p3+q3+3p2qw5+3pq2w4 1+x 2
1 + x2
= 3(p3+q3) ⇒a2 + b2 = 1 B is correct
x
1 + i 3 
( )
x a + ib
Sol 21: (A) 1 + i 3 x
2 ⇒ 
=  =1 (c) = x − iy

 2  a − ib
⇒ (–w)x = 1 (a + ib)2
⇒ = x − iy
⇒ x = 6, 12, 18, ……... a2 + b2
a2 − b2 + 2iab
It forms an AP ⇒ x + iy =
a2 + b2
a2 − b2 − 2iab
⇒ x + iy = ⇒ |x + iy| = 1
Multiple Correct Choice Type a2 + b2
y − ix
Sol 22: (B, C) x2+(p+iq)x + 3i = 0 (d) = y + ix
a + ib
a2 + b2 = 8 y 2 − x2 − 2ixy
⇒ = a + ib
(α + β) = –(p + iq) y 2 + x2
aβ = 3i y 2 − x2 + 2ixy
⇒a – ib = ⇒ |a – ib| =1
x2 + y 2
⇒ (α + β)2 = 8 + 6i = p2 – q2 + 2ipq
p2 – q2 = 8 Sol 25: (A, D) z = x + iy = r(cosθ + isinθ)
2pq = 6
z
= x + iy
= r(cos θ + isin θ)
⇒p = 3, q = +1 or p = –3, q = –1
 θ θ
r eiθ /2
== r cos + isin  for y > 0
 2 2
Sol 23: (A, D) |z1| = |z2| ⇒ a2 + b2 = c2 + d2
z1 = a + ib, a > 0  1 + cos θ 1 − cos θ 
= r +i 
z2 = c + id, d < 0  2 2 
z1 + z 2 (a + c) + i(b + d) r  r +x +i r −x r +x +i r −x
= =   = [A]
z1 − z 2 (a − c) + i(b − d)
2  r  2
(a + c) & (b + d) can be zero, so value can be zero Similarly
(a – c) & (b – d) can be simultaneously zero ⇒ purely 1 
imaginary. = r + x − i r − x  for y < 0
2 
M a them a ti cs | 6.79

Sol 26: (A, D) α = eiθ = β [C and D are correct]


( az1 + bz1 )( cz2 + dz2 ) = ( cz1 + dz1 )( az2 + bz2 )
Sol 29: (A, B, D) α = 3z – 2
adz1 z2 + bc z1 z2 = bcz1 z2 + ad z1 z2
β = –3z – 2
⇒ ad = bc
|z| = 1
a c
= (A is correct)
b d |α + 2| = 3 → A is correct
b d
⇒ arg z1 = tan–1 a , arg z2 = tan–1 c |β + 2| = 3 → B is correct

α – β = 6Z D is correct
⇒ arg z1 = arg z2
z1 z2 ⇒ α – β and z, both move on same circle
or z1 z2 = z1 z 2 ⇒ =
z1 z2
a + ib c + id Sol 30: (B, D) (in + i–n) is
⇒ = ⇒ –ad + bc = 0
a − ib c − id
22n + 22n 2.2n 2n+1
(A) = = (1 + i)2n =
2n (1 − i)2n (1 − i)2n
2 2n
Sol 27: (A, B, C)
(2i)n
= 2in= in + in
n−1
2

(1 + i)2n + (1 − i)2n (2i)n + ( −2i)n


(B) = =in + (–i)n
2n 2n
= in + i–n
−(2i)n 2n
(C) + = –in + in = 0
|z1| = 1 and |z2| = 2 2n ( −2i)n
2n 2n
Max |2z1 + z2| (D) + i−n + in
=
n n
(2i) ( −2i)
|2z1 + z2| ≤ 2|z1| + |z2| = 4 (A is correct)

Min |z1 – z2| ≥ |z1| – |z2| Sol 31: (A, B, D)


 
|z1 – z2| ≥ 1(B is correct)  
( 2
log14 13 + z − 4i + log196  ) 1
=0
( )
2
1 1  13 + z 2 + 4i 
z2 + ≤ z2 + ≤ 2 (C is correct)  
z1 z1  
13 + z 2 − 4i
log14 =0
13 + z 2 + 4i
α −β
Sol 28: (A, B, C, D) =1 ⇒ 13 + |z2 – 4i| = 13 + |z2 + 4i|
1 − αβ
2 2 ⇒ |z2 – 4i| = |z2 + 4i|
α − β = 1 − αβ

⇒ (α – β) ( α – β ) = (1 – α β) (1 − αβ ) ⇒ |x2 – y2 + 2ixy – 4i| = |x2 – y2 + 2ixy + 4i|

|a|2 + |b|2 – α β – β α =1 – α β – β α + |a|2 |b|2 ⇒ (x2 – y2)2 + (2xy – 4)2 = (x2 – y2)2 + (2xy +4)2

|a|2 (α – |b|2) = (1 – |b|2)⇒(|a|2 – 1) (1 – |b|2) =0 ⇒ –16xy = 16xy ⇒ either x = 0 or y = 0

⇒|a| = 1 A is correct If y = 0 can be purely real

⇒|b| = 1 B is correct x = 0 can be purely imaginary


Must be real or purely imaginary.
6 . 8 0 | Complex Number

 x + 1 + 2i − 2  z = z implies z is purely real (B is correct)


Sol 32: (A, B) 1 − log2  ≥0
 2 − 1  z = −z implies z is purely imaginary (C is correct)

x + 1 + 2i − 2
⇒ log2 ≤1 Sol 37: (A, C)
2 −1 z2
⇒ |x + 1 + 2i| – 2 ≤ 2( 2 − 1 )

⇒ |x + 1 + 2i| ≤ 2 2 ⇒ (x + 1)2 + 4 ≤ 8

⇒ (x + 1)2 ≤ 4 ⇒ –2 ≤ x + 1 ≤ 2 z1 z3

⇒ –3 ≤ x ≤ 1 ⇒ x ∈ [–3, 1] z3 = (1 – z0)z1 + z0z2

z'3 = (1 – z0)z1' + z0z2'


Sol 33: (D) x = cos a
y = cos b
z1 z1 1
z = cos g z3 − z1 z'3 − z1'
z0 = = ⇒ z2 z2 1 =
0
z 2 − z1 z' − z'
Sx = cosα + cosβ + cosg 2 1
z3 z3 1
= cosα+cosβ+cosγ+i(sinα+sinβ+sinγ) angle is same
Px=cosa cosb cosγ = eiα eiβ eiγ = ei(α + β + γ) The two triangles are similar

π Sol 38: (A, B) Z multiplicative inverse is same as


Sol 34: (A, D) xr = cos  
 2r  additive inverse
 π π 
n i + ......  π π 
i + +  1
 r r +1  =−a − ib
Lim ∏xr = e2 2  =e2 4 
a + ib
n→∞
r =1
a − ib
π =−a − ib
i
 1 a2 + b2
2 1 − 
= e  2 = eiπ ⇒ a = –a3 – ab2 ⇒ a(1 + a2 + b2) = 0
= cosπ + isinπ = –1 + 0i ⇒ a = 0 and
Real part is –1 b = a2b + b3 ⇒ b(1 – a2 – b2) = 0
Imaginary part is 0 b ≠ 0 so a2 + b2 = 1 ⇒ b = ±1

n−1 z=0±i
Sol 35: (A, B, D) ∏ ( ω − zr )
r =1
Sol 39: (A, B, C, D) Z = a + bi =
(1 − ix ) (1 − ix )
ωn − 1 (1 + ix ) (1 − ix )
(w – z1) (w – z2) (w – z3)….. (w – zn-1) =
ω−1 1 − x2 − 2ix
a + bi =
If n is a multiple of 3, value can be zero as z1 can be w 1 + x2
or w2 ⇒ |z| = 1
If n = 3k + 1 value is 1  2x 
Arg z = tan–1  2 
If n = 3k + 2 value is 1 + w  x −1
Arg (z) ≡ ( −π, π 
Sol 36: (A, B, C) z = −4z
x – iy= –4x – 4iy
5x + i(3y) = 0
x = y = 0 (A is true)
M a them a ti cs | 6.81

Previous Years’ Questions ⇒ x + y =2  ….(ii)

Sol 1: Given, |z – 3 – 2i| ≤ 2  …. (i) And


To find minimum of |2z – 6 + 5i| Im ( z ) ≥ 1
5 ⇒ Im ( x + iy ) ≥ 1
Or 2 z − 3 + i ,
2
⇒ y ≥ 1  ….(iii)
By using triangle inequality

i.e., | z1 | − | z 2 | ≤ z1 + z2

5
∴ z −3+ i
2
5 9
= z − 3 − 2i + 2i + i = (z − 3 − 2i) + i
2 2

9
≥ | z − 3 − 2i | − From (i), (ii) and (iii) A ∩ B ∩ C has only one point P
2
shown in the figure.
9 5
≥ 2− ≥
2 2 Sol 4: (C) z + 1 − i + z − 5 − i
2 2

5 5
⇒ z − 3 + i ≥ or |2z – 6 + 5i| ≥ 5 2 2
2 2 = z − ( −1 + i ) + z − (5 + i )

The point A (-1, 1) and dB (5, 1) are end points of one


i
2π of the diameter
Sol 2: ω =e 3
In right angle ∆ APB
| x |2 + | y |2 + | z |2
Then, =3
| a |2 + | b2 | + | c |2
B(5,1)

Note: Here, ω = ei2π /3 , then only integer solution exists.

Sol 3: (B)
=A {z : Im ( z ) ≥ 1} A (-1,1)

( AB )
2
AP2 + BP2 =
B
= {z : z − 2 −=i 3}
{z : Re (1 − i) z + 2 }
2 2
=C ⇒ z +1 −i + z −5 −i

(=
6)
2
Taking z − 2 − i =3 = 36

Let z= z + iy
x + iy − 2 − i =3 Sol 5: (D) w − 2 − i < 3

2 From triangle in equality


⇒ ( x − 2) + i ( y − 1) =
(3)
2

z−w > |z|−|w|


⇒ ( x − 2) + ( y − 1) =
2 2
9  …(i)
⇒ − z − w < | z | − | w | < | z− w |  …(i)
And
Given w − 2 − i < 3 , which means that w lies inside the
Re (1 − i) z  =2 given circle. Since z is presented by point P and W has
Re (1 − i)( x + iy )  =2 to be the other end of diameter

| z − w | = length of diameter
6 . 8 2 | Complex Number

From (i), we get a u 1


−6 < | z | − | w | < 6 Consider b v 1
c w 1
⇒ − 6 + 3 < | z | − | w | + 3 < 6 +3
Applying R3 → R3 – {(1 – r) R1 + rR2}
⇒ −3 < | z | − | w | + 3 < 9
a u 1
5 5 = b v 1
 3 i  3 i
Sol 6: (B) Given, z =  +  + −  c − (1 − r)a − rb w − (1 − r)u − rv 1 − (1 − r) − r
 2 2  2 2
   
a u 1
 −1 + i 3 −1 − i 3 
=ω = and ω2  = b=v 1 0 (from eq. (i))
 2 2 
  0 0 0
3 +i  −1 + i 3 
Now, =–i   = −iω Hence, two triangles are similar.
2  2 
 

3 −i  −1 − i 3  6
 2kπ 2kπ 
And = i  = iω2 Sol 9: (D) ∑  sin − icos 
2  2  k =1  7 7 
 
 6 i2kπ 
∴ z = (– iω)5 + (iw2)5 = – iw2 + iω 6
 2kπ 2kπ  = −i  e 7 
∑ i  cos 7 + isin 7  ∑ 
( ) k =1   k =1 
= i (ω – w2) = i i 3 = – 3

⇒ Re (z) < 0 and Im (z) = 0  6 i2k π   6 i2k π 


∑ e = 0 = −i  ∑ e − 1 = −i 0 − 1  = i
k =0 7  k =0 7 
Alternate solution:
We known z + z =
2Re(z)
Sol 10: (C) Given, | z + iw |=| z − iw | 2 =2
5 5
 3 i  3 i ⇒ | z − ( −iw) |= | z − (iw) |= 2
If z =  +  + −  , then z is purely real,
 2 2  2 2 ⇒ | z − ( −iw) |= | z − ( −iw) | .
   
i.e. Im (z) = 0
iw iw
w
Sol 7: (D) Given, |z – 4| < |z – 2|
Since, |z – z1| > |z – z2| represents the region on right -1 1
side of perpendicular bisector of z1 and z2
w
∴ |z – 2| > |z – 4| -i w
⇒ Re(z) > 3 and Im (z) ∈ R
∴ z may take values given in (c).
y
Alternate solution
x’ y
x
O (2,0) (3,0) (4,0)

(-z)
y’ r -

x
O -

Sol 8: (B) Since a, b, c and u, v, w are the vertices of two r

triangles. (z)

Also, c = (1 – r) a + rb and w = (1 – r) u + rv … (i)


M a them a ti cs | 6.83

| z + iw |≤| z | + | iw | = | z | + | w |≤ 1 + 1= 2 Sol 12: (C) If in a complex number a + ib, the ratio a: b


∴ | z + iw |≤ 2 is 1 : 3 , then always convert the complex number in
⇒ | z + iw |=
2 holds when the form of ω .
Arg z – arg iw = 0 1 3
Since, ω= − + i
2 2
Similarly, when | z − iw |=2 334 365
 1 i 3  1 i 3
z ∴ 4 + 5 − +  + 3 − + 
Then is purely imaginary  2 2   2 2 
w  
Now, given relation
= 4 + 5ω334 + 3ω365
| z + iw |=| z − iw |=2 = 4 + 5 ⋅ (ω3 )111 ⋅ ω + 3 ⋅ (ω3 )121 ⋅ ω2
Put w = i, we get = 4 + 5ω + 3ω2 ( ω3 = 1)

| z + i2 |=z + i2 |=2 = 1 + 3 + 2ω + 3ω + 3ω2


⇒ | z − 1 |= 2 = 1 + 2ω + 3(1 + ω + ω2 ) = 1 + 2ω + 3 × 0
⇒ z= −1 (| z |≤ 1) = 1 + ( −1 + 3i) = 3i.
Put w = - i, we get
| z − i2 |=| z − i2 |= 2 Sol 13: (A) Since, arg (z) < 0
⇒ | z + 1 |= 2
⇒ arg(z) = −θ
⇒ =z 1 (| z |≤ 1) ⇒=z r cos( −θ) + isin(=
−θ) r(cos θ − isin θ)
and − z = −r[cos θ − isin θ]
∴ z = 1 or – 1 is the one correct option given.
= r[cos( π − θ) + isin( π − θ)]
∴ arg ( −z) = π − θ
Sol 11: (D)
Thus, arg (-z) –arg (z)
n n n2 n2
(1 + i) 1 + (1 − i) 1 + (1 + i) + (1 − i)
n n1 n n = π − θ − ( −θ) = π
= [ 1 C0 + C1i + 1 C2i2 + 1 C3i3 + ......]
n n n n Alternate solution:
+ [ 1 C0 − 1 C1i + i C2i2 − 1 C3i3 + ......]
 −z 
n n n n
+ [ 2 C0 + 2 C1i + 2 C2i2 + 2 C3i + .....] Reason: arg (-z) – arg z =arg   = arg( −1) =π
 z 
n n n n
+ [ 2 C0 − 2 C1i + 2 C2i2 − 2 C3i3 + ...]  z 
And also arg z – arg (-z) =arg   = arg( −1) =π
n
= 2[ 1 C0 +
n1
C2i2
n1
+ C 4 i4 + ....]  −z 
n n n
+2[ 2 C0 + 2 C2i2 + 2 C 4 i4 + ....]
= 2[
n1
C0 −
n1 n
C2 + 1 C 4= ....] Sol 14: (A, B,C) Since, z1 =
a + ib and z 2 =
c + id
n n n ⇒ | z1 |2 = a2 + b2 = 1 and | z 2 |2 = c2 + d2 = 1 ...(i)
+2[ 2 C0 − 2 C2 + 2 C 4 − ....]
(| z=
1 | | z=
2 | 1)
This is a real number irrespective of the values of n1 and n2 Also, Re (z1 z 2 ) =0 ⇒ ac + bd =0
Alternate solution a d
⇒ =− =λ (say)...(ii)
n n n n b c
{(1 + i) 1 + (1 − i) 1 } + {(1 + i) 2 + (1 − i) 2 }
From Eqs.(i) and (ii), b2 λ2 + b2= c2 + λ2c2
⇒ a real number for all n1 and n2 ∈ R.
2
n n ⇒ b= c2 and a=
2
d2
[ z=+ z 2Re(z) ⇒ (1 + i) 1 + (1 − i) 1
is real number for all n ∈ R] Also, given w1 =
a + ic and w2 =
b + id
Now, | w1 |= a2 + c2 = a2 + b2 = 1
| w2 |= b2 + d2 = a2 + b2 = 1
6 . 8 4 | Complex Number

and Re(w1 w2 )= ab + cd= (bλ )b + c( −λc) aq = 4 and 2a = 10


4
λ (b2 − c2 ) =
= 0 ⇒e=
5
B→p
(1 − t)z1 + tz 2
Sol 15: (A, C, D) Given, z = 1
(1 − t) + t (C) z= w −
w
A P B 1
⇒ z = w− ×w
z1 z z2 ww
t : (1 - t) w
= w−
| w |2
Clearly, z divides z1 and z 2 in the ratio of t:
w
= w−
(1-t), 0 < t < 1 4
⇒ AP + BP = AB Let w = p + I q, then
ie, | z − z1 | + | z − z 2 |= | z1 − z 2 | 3P 5q
z
= +i
⇒ Option (a) is true. 4 4
Now, let z = x + iy
And arg (z − =
z1 ) arg (z 2 −=
z) arg(z 2 − z1 )
3P 4x
⇒ x= ⇒ P=
⇒ (b) is false and (d) is true. 4 3
5q 4y
Also, arg (z − z1=
) arg(z 2 − z1 ) ⇒ y= ⇒ q=
4 5
 z − z1 
⇒ arg  0
 = | w |2 = P2 + q2 = 4
 z 2 − z1 
z − z1
∴ is purely real.
z 2 − z1

z − z1 z − z1 z − z1 z − z1
=
⇒ or= 0
z 2 − z1 z 2 − z1 z 2 − z1 z 2 − z1

Sol 16: (A) | z − i | z ||=


| z + i | z || 16 x2 16 y 2
⇒ + 4
=
9 25
⇒ | z − i | z ||2 =| z + i | z ||2
x2 y2
⇒ ( z − i | z |) ( z + i| z |) = ( z + i | z |) ( z − i | z |) ⇒ + 1 , its an ellipse
=
9 / 4 25 / 4
⇒ z z + i z| z | − i | z | z + | z |2 b2
e2= 1 −
a2
=z z − i z | z | + i| z | z + | z |2
9 16
⇒ 2i z | z |=
2i| z | z =1− =
25 25

⇒ 2i | z | ( z − z ) =
0 4
⇒e=
5
| z | 0 or z=
⇒= −z 0
From figure, we can conclude that
⇒ Im ( z ) =
0
Re ( z ) ≤ 2
Also Im ( z ) ≤ 1

A → q, r And z ≤ 3

(B) |z + 4| + |z - 4| = 0
C → p, s, t
Its an equation of ellipse having
M a them a ti cs | 6.85

1 w w z − z1 z 2 − z1
(D ) z= w + w
=+ w
=+ w w
=+ =
w ww | w |2 z − z1 z2 − z1
Let w = a + i b z − z1 z − z1
⇒ 0
=
z = a + i b + a – i b = 2a z 2 − z1 z2 − z1

Now, |z| = 2 |a| and Im (z) = 0, Im ( z ) ≤ 1 Sol 19: 2z − 6 − 5i


⇒ Re ( z ) ≤ 2 and | z | ≤ 3 5
= 2 z −3+ i
2
D → q, r , s and t
 5 
Here z −  3 − i  is nothing but the distance between
Sol 17: (B)  2 
 −5 
z +1 ω ω2 any point on the circle z − 3 − 2i ≤ 2 and point  3, 
 2 
ω z + ω2 1 =0
2
ω 1 z+ω
C1 → C1 + C2 + C3 (3,2)

z ω ω2
z z + ω2 1 =0
z 1 z+ω

 2π 2π )3, -5 )
=ω cos + i sin 2

 3 3 
1 ω ω2 5
∴ Minimum value =
2
1 z + ω2 1 =0
1 1 z+ω 5
2z − 6 + 5i = 2× =5
min 2
Expanding the determinant, we get
Sol 20: Given: a + b + c = x
 (
z  z + ω2 ) ( z + ω) − 1 − ω ( z + ω − 1) + ω (1 − z − ω ) =0
2 2
a + b ω + c ω2 = y
⇒ z  z 2 + z ω + zω2 + ω2 − 1 − zω − ω2 + ω + ω2 − ω2 z − ω4  =0
  a + b ω2 + c ω = z
⇒ z  z2  =
0 | x |2 + | y |2 + | z |2 xx + yy+zz
  =
2 2 2
|a| + |b| + |c| | a |2 + | b |2 + | c |2
⇒z=0

Sol 18: (A, C, D) Given z =(1 − t ) z1 + t z2


( ) (
= ( a + b + c ) a + b + c + a + b ω + c ω2 + a + b ω+ c ω2 ) ( )
This equation represents line segment between z1 and ( 2
+ a + bω + cω a + bω + c ω)( 2
)
z2 | a | + | b | + | c |2
2 2

From fig.  i π /3
 ω =e
| z − z1 | + | z − z 2 |= | z1 − z 2 |  −i π /3
 ω =e
 π
ω2 = e2i 3 = − ω

( )
= ( a + b + c ) a + b + c + ( a + b ω− ω c ) a + b ω + c ω2 ( )
Arg ( z − z=
1) Arg ( z 2 − z1 ) (
+ ( a − b ω + c ω) a + b ω + c ω 2
)
| a | + | b | + | c |2
2 2
6 . 8 6 | Complex Number

(
3 | a |2 + | b |2 + | c |2
= 3
) 1 1 
4r 2
⇒  − z 0   − z0  =
 α  α 
| a |2 + | b |2 + | c |2
 1 z z 
⇒  − 0 − 0 + | z 0 |2  =
4r 2
 α α α α 
1 a b
   | α |2 
Sol 21: (A) Given  ω 1 c 2
α α = | α | ⇒ α = 
ω2  α 
 ω 1 
1 z0 α z0 α
1 a b ⇒ − − + | z 0 |2 =
4r 2
2 2 2
  |α| |α| |α|
Determinant (D )=  ω 1 c
ω2 ω 1 
 ⇒ 1 − z 0 α − z0 α+ | α |2 | z 0 |2 = 4r 2 | z |2  …(ii)
|= (
D | 1 (1 − c ω ) − a ω − c ω 2
) + b (ω2
−ω 2
) Subtracting (ii) from (i), we get
⇒ | D | = 1 − ( a + c ) ω + acω 2
1− | α |2 + | α |2 | z 0 |2 − | z 0=
|2 r 2 4 | α |2 −1 ( )
| D | ≠ 0 , only when a = ω and c = ω
r2 + 2
(
∴ ( a,b,c ) ≡ ( ω, ω, ω) or ω, ω2 , ω ) 1− | α |2 +
2
(
| α |2 −= )
1 r 2 4 | α |2 −1 ( )
∴ Two non-singular matrices are possible.  r2 + 2  2
(
⇒ | α |2 −1 
 2
)
−1
=

2
 r 4 | α | −1 ( )
Sol 22: (D) a = z 2 + z + 1 = 0  r2  2
⇒ z2 − z + 1 − a =0 (
⇒ | α |2 −1  =
2
 
) 2
 r 4 | α | −1 ( )
Discriminant < 0 {Since imaginary part of z us not zero}
⇒ | α |2 −1 = 8 | α |2 −2
1 – 4 (1 - a) < 0
⇒ 7| α |2 =1
⇒ 1 − 4 + 4a < 0
1
⇒ 4a < 3 ⇒ | α |=
7
3
⇒a <
4 Sol 24: (C,=
D) P {=
w :n n
1,2,3,..... }
3 1
Sol 23: (C) ( x − x0 ) + ( y − y 0 ) =
2 2
r w= + ⇒ | w |= 1
2 2
⇒ | z − z 0 |=
r ⇒ All the complex number belong to set P lie on circle
of unit radius, centre at origin.
α lies on it, then

| α − z 0 |=r

⇒ ( α − z 0 ) ( α − z0 ) =r 2

| |2 − z0 α − z 0 α + | z 0 |2 =
=α r2  ….(i)

Similarly,

( x − x0 ) + ( y − y 0 )
2 2
2r
=

⇒ | z − z 0 |=
2r
1
lies on it, then
α
1
− z0 =
2r
α
M a them a ti cs | 6.87

All the complex number belong to PnH1 lie right of the  


 z − 1 + 3i 
1 S2 =
line x = on circle. Possible positions are shown in the z ∈ C : Im   > 0
2   1 − 3 i  
figure as A1 , A2 , A3 .

Similarly, all the complex number belong to PnH2 lie left (


 z −1 + 3i 1 + 3i 
Im   >0 )( )
−1 4 
of the line x = on circle. Possible positions are shown
2  
in the figure as B1 ,B2 ,B2 < z1 0 z 2 = angle between A2 Let z = x + i y
2π 5π
and B3 =
3
or angle between A1 and B3 =
6 ( ) (
 ( x + iy ) 1 + 3 i − 1 − 3 i 1 + 3 i 
⇒ Im   >0 )( )
4 
 

( ) (
 x + 3 y + i 3x + y − 4 
⇒ Im   >0 )
4 
 

3x + y
⇒ > 0 ⇒ 3x+ y > 0
4
{
S3 z ∈ C : Re ( z ) > 0 }
⇒ z lies in either first or fourth quadrant.
Now, points of intersection of circle x2 + y 2 =
16 and

Sol 25: (B, C, D) P = pij  pij = ωi+ j 3x + y =0 is x2 + 3x2 =


16
n×n
⇒ 4x2 =
16 ⇒ x =± 2
 ω2 ω3 ω4  ω 
n+1
  ⇒ y =± 2 3
ω3 ω4 ω5  ωn+ 2 
P=  1 2 5π
      Area =
2
( 4) × 6
ω2n+1 ωn+ 2 ωn+3  ωn+n 
  1 5 π 20 π
= × 16 × = .sq. unit2
2 6 3
ω4 + ω6 + ω8 + .... ω5 + ω7 + ω9  
 
    Sol 27: (C) | 1 − 3i− z |
⇒ P2 =
    
  = z − 1 + 3i = z − (1 − 3i )
n 3 n 5
 ω + ω + .....
+ +  

Lets take element P11 =ω4 + ω6 + ω8 + ....n terms ( )


Min z − (1 − 3i) = distance of point (1, -3) from line

=
(
ω4 ω2n − 1 ) 3 x+y =0

ω−1 3×1 − 3 3−3


= =
If n is multiple of 3, then this element will vanish. Which 3+1 2
is the case for every element.
3− 3
=
∴ n can not be multiple of 3, for P2 ≠ 0 2
∴ Possible values of n are 55, 58, 56
2πx 2k π
Sol 28: (C) zk =
cos + i sin ,k =
1,2,....9
10 10
{z C :| z | < 4}
Sol 26: (B) S1 =∈
i
2 πk
(p) zk = e 10
z lies inside of a circle given by |z| = 4
6 . 8 8 | Complex Number

2π π π π
i (k + j) 12 ik i 12 i
⇒ zk .z=
j e 10 = 1 ∑ e 7 e 7 −1 ∑ e 7 −1
k =1 k =1
2π 2π = =
⇒ cos
10
( k + j) + i sin
10
(k + j ) =
1 3 i
( 4k −2) i
π 3 i
π

∑e 7 e 7 −1 ∑e 7 −1
If k + j = 10 m (multiple of 10), then above equation is k =1 k =1

True.

(q) z1 .z = zk 12
π π
2 πk
∑ 2 sin 14 12 × 2 sin
i (k −1) = k =1
= 4 =4
zk e 10 i 2π 3 π
⇒ z= = = e 10 π
z1 i
2π ∑ 2 sin 14 3 × 2 sin
4
e 10 k =1

Clearly, this equation has many solutions


 −z r z 2s 
⇒ Q is False Sol 30: P = 
( )
 z 2s zr 
(r) Consider, z10 = 1 

z10 − 1 = ( z − 1) ( z − z1 )( z − z2 )( z − z3 ) .... ( z − z9 )
10
 −z 2r + z 4s
P = 
2
( ) ( ) ( −z ) z 2s + zr z 2s 
r

Where zk represents the roots of equation z = 10 
( −z ) z + z z
r 2s r 2s 2r 4s
z +z 
=1 + z + z 2 + .... + z 9
 
z 2s ( −z ) + zr  
2r 4s r


Z10 − 1
( z − z1 ) ( z − z2 ) ...... ( z − z9 )
=  z +z  
z −1 = 
 z 2s  ( −z )r + zr  z 2r + z 4s 
= 1 + z + z 2 + z3 + .... + z 9    

z10 − 1
( z − z1 ) ( z − z2 ) .... ( z − z9 )
= 1 0  2r 4s
z −1 Given that P2 − I =  ⇒ z +z = −1
 0 1 
= 1 + 1 + 1 …… 10
= 10 2s  r
And z ( −z ) + z  =
r
0
10  
(s) We know that sum of roots of z − 1 =0 is zero
−1 + i 3
2k π9 9
2k π Now, we have z = =ω
⇒ 1 + ∑ cos 0 ⇒ ∑ cos
= −1
= 2
k =1 10 k =1 10
⇒ ω2r + ω4s = −1 and ω2s ( −ω) + ωr  = 0
r
9
2k π  
⇒ 1 − ∑ cos =2
k =1 10 Only (r, s ) ≡ (1, 1 ) satisfies both the equation.
kπ Only one pair exists.
kπ kπ i
Sol 29: Given
= αk cos + i sin =e 7
7 7
 1 
Sol 31: (A, C, D) S =z ∈ C : z = , t ∈R, t ≠ 0 
12 i
(k +1) π i
kπ  a + ib t 
12
∑ | αk +1 − αk | ∑e 7 −e 7
1 a−i b t
k =1
k =1 =z ×
= a + ib t a − ib t
3
3 i ( 4k −1 )
π ( 4k −2)
∑ | α 4 k −1 − α 4k −2 | ∑e 7
i
−e 7
a−i b t
k =1 ⇒z= = x + i y (Let)
k =1
a2 + b2 t2
a bt
=⇒x = and y
22 2
a +b t a + b2 t 2
2
M a them a ti cs | 6.89

x a ay
⇒ = ⇒t=
y bt bx
a
Substituting ‘t’ in x =
a + b2 t 2
2
a
x=
a2 y 2
a2 + b2
b2 x 2

ax2
⇒ x= ⇒ a2 x2 + a2 y 2 − ax =
0
a2 x2 + a2 y 2
x
⇒ x2 + y 2 − 0
=
a
2 2
 1  2 1 
x −  + y = 
 2a   2a 

 1 
Centre ≡  , 0 
 2a 
1
Radius = , when a > 0 , b ≠ 0
2a

If=
b 0, a ≠ 0
y = 0 ⇒ x − axis
If=a 0, b ≠ 0
x = 0 ⇒ y − axis
2017-18 100 &
op kers
Class 11 T
By E ran culty
-JE Fa r
IIT enior emie .
S fP r es
o titut
Ins

MATHEMATICS
FOR JEE MAIN & ADVANCED
SECOND
EDITION

Exhaustive Theory
(Now Revised)

Formula Sheet
9000+ Problems
based on latest JEE pattern

2500 + 1000 (New) Problems


of previous 35 years of
AIEEE (JEE Main) and IIT-JEE (JEE Adv)

5000+Illustrations and Solved Examples


Detailed Solutions
of all problems available

Topic Covered Plancess Concepts


Tips & Tricks, Facts, Notes, Misconceptions,
Trigonometric Ratios, Key Take Aways, Problem Solving Tactics
Identities and Equations
PlancEssential
Questions recommended for revision
7. TRIGONOMETRIC
R AT I O S , I D E N T I T I E S
A N D E Q U AT I O N S
1. INTRODUCTION
The equations involving trigonometric functions of unknown angles are known as Trigonometric equations e.g.
cos
= θ 0,cos2 θ − 4 cos
= θ 1,sin2 θ + sin
= θ 2cos2 θ − 4 sin
= θ 1.

2. TRIGONOMETRIC FUNCTIONS (CIRCULAR FUNCTIONS)

Function Domain Range


sin A R [–1, 1]
cos A R [-1, 1]
tan A R − ( 2n + 1 ) π / 2,n ∈ I  R= ( − ∞, ∞ )
cosec A R − nπ,n ∈ I  ( −∞, −1 ∪ 1, ∞ )
sec A
{
R − ( 2n + 1 ) π / 2,n ∈ I } ( −∞, −1 ∪ 1, ∞ )
cot A R − nπ,n ∈ I  ( −∞, ∞ )
We find, sinA ≤ 1, cos A ≤ 1,sec A ≥ 1 or sec A ≤ −1 and cosec A ≥ 1or cosec A ≤ −1

2.1 Some Basic Formulae of Trigonometric Functions Y

(a) sin2 A + cos2 A =


1. II quadrant I quadrant

(b) sec2 A − tan2 A =


1 cosec }
only sin 
are + ve All t-ratios are + ve

2 2
(c) cosec A − co t A =
1 X’ X
(d) sinA
= cosec A tanA
= cot A cos
= A sec A 1 III quadrant IV quadrant

A system of rectangular coordinate axes divide a plane into four }


only tan 
cot 
are + ve only cos 
sec } are + ve

quadrants. An angle θ lies in one and only one of these quadrants.


The signs of the trigonometric ratios in the four quadrants are shown in Y’
Fig 7.1. Figure 7.1
7 . 2 | Trigonometric Ratios, Identities and Equations

PLANCESS CONCEPTS

A crude way to remember the sign is “Add Sugar to Coffee”. This implies the 1st letter of each word gives
you the trigonometric functions with a +ve sign.
Eg. Add-1st word ⇒ 1st quadrant 1st letter=A ⇒ All are positive to-3rd word ⇒ 3rd quadrant 1st letter-t
⇒ tan θ (cot θ ) are positive.
Ravi Vooda (JEE 2009, AIR 71)

Sine, cosine and tangent of some angles less than 90º:

Trigonometric 0º 15º 18º 30º 36º


ratios
sin 0
6− 2 5 −1 1 10 − 2 5
4 4 2 4

cos 1
6+ 2 10 + 2 5 3 5 +1
4 4 2 4

tan 0
2− 3 25 − 10 5 1
5−2 5
5 3
37º 45º 53º 60º 90º
sin ≈ 3/5 1 ≈ 4/5 1
3
2
2

cos ≈ 4/5 1 ≈ 3/5 0


1
2 2

tan ≈ 3/4 1 ≈ 4/3 Not defined


3

Illustration 1: Prove the following identities:

(i) (1 + tan A ) +  1 + tan1 A  =


2
2 2
1
sin A − sin 4
A

2 (JEE MAIN)
1 + tan2 θ
 1 − tan θ 
(ii) =  
1 + cot θ  1 − cot θ 
2

Sol: (i) Simply by using Pythagorean and product identities, we can solve these problems.
 1 
(i) L.H.S. = (1 + tan2 A) +  1 +  = sec A + (1 + cot A)
2 2

 tan2 A 
1 1 sin2 A + cos2 A 
=sec2 A + cosec2=
A= + =  sin2 θ + cos2 θ 1
cos2 A sin2 A sin2 A.cos2 A  
M a them a tics | 7.3

1 1
= = = R.H.S.  cos2 θ = 1 − sin2 θ 
2
(
sin A 1 − sin A 2
) sin A − sin4 A
2  

Hence proved.
1 + tan2 θ sec2 θ sin2 θ
(ii) L.H.S. = = = = tan2 θ  … (i)
1 + cot θ cosec θ cos2 θ
2 2
2 2
    2
2
 1 − tan θ   1 − tan θ   1 − tan θ   1 − tan θ 
Now, R.H.S. = =  =    =  . tan θ  = tan2 θ  … (ii)
 ( )
 1 − cot θ  1 − 1   tan θ − 1   − 1 − tan θ 
    
 tan θ   tan θ 
From (i) and (ii), clearly, L.H.S. = R.H.S. Proved.

Illustration 2: Prove the following identities: (JEE MAIN)


sin2 A cos2 A 1
(i) =
+ = − 2 sec2 A cosec2 A − 2
2 2 2 2
cos A sin A sin A cos A

(ii) sec4 A (1 – sin4 A) – 2 tan2 A = 1

Sol: Use algebra and appropriate identities to solve these problems.

(sin A ) + (cos A )
2 2
2 2
sin2 A
cos2 A sin4 A + cos4 A + 2sin2 A cos2 A − 2sin2 A.cos2 A
(i) += =
cos2 A sin2 A sin2 A cos2 A sin2 A cos2 A

(sin )
2
2
A + cos2 A − 2sin2 A cos2 A 1 − 2sin2 A cos2 A
= =
sin2 A cos2 A sin2 A cos2 A

1 2sin2 A cos2 A
= − = sec2 A cosec2 A – 2 = R.H.S.  Proved.
2 2 2 2
sin A cos A sin A cos A
sin4 A
(ii) L.H.S. = sec4 A (1 – sin4 A) – 2 tan2 A = sec 4 A − − 2 tan2 A = sec 4 A − tan4 A − 2 tan2 A
4
cos A

( )
2
= 1 + tan2 A − tan4 A − 2 tan2 A = 1 + 2 tan2 A + tan4 A – tan4 A – 2 tan2 A = 1 = R.H.S.  Proved.

Illustration 3: Prove the following identities:

1 + cos α 1 + sin α
(i) = cosecα + cot α (ii) = secα + tan α  (JEE MAIN)
1 − cos α 1 − sin α

Sol: By rationalizing L.H.S. we will get required result.

(1 + cos α )
2
1 + cos α 1 + cos α 1 + cos α
(i) L.H.S. = = × =
1 − cos α 1 − cos α 1 + cos α 1 − cos2 α

(1 + cos α=
)
2
1 + cos α 1 cos α
= = + = cosecα + cot
= α R.H.S.  Proved.
sin α 2 sin α sin α sin α

(1 + sin α=)(1 + sin α ) (1 + sin α ) (1 + sin α )


2 2
1 + sin α
(ii) L.H.S. =
= =
1 − sin α (1 − sin α )(1 + sin α ) 1 − sin2 α cos2 α
7 . 4 | Trigonometric Ratios, Identities and Equations

1 + sin α 1 sin α
= = + = sec α + tan=
α R.H.S.  Proved.
cos α cos α cos α

Illustration 4: In each of the following identities, show that:


cot Α + tanB sin2 A − sin2 B
(i) = cot A. tanB (ii) tan2 A − tan2 B =  (JEE ADVANCED)
cotB + tanA cos2 A cos2 B

Sol: Apply tangent and cotangent identity.

cos A sinB cos A cosB + sinA sinB


+
cot A + tanB sinA cosB sinA cosB
(i) L.H.S. = = =
cotB + tanA cosB sinA cos A cosB + sinA sinB
+
sinB cos A sinBcos A

sinBcos A  cos A  sinB 


= =  =  cot
= A tanB R.H.S.  Proved.
sinA cosB  sinA  cosB 

sin2 A sin2 B sin2 A cos2 B − cos2 A sin2 B


(ii) L.H.S. = tan2 A − tan2 B = − =
cos2 A cos2 B cos2 A cos2 B

=
( ) ( )
sin2 A 1 − sin2 B − 1 − sin2 A sin2 B
=
sin2 A − sin2 A sin2 B − sin2 B + sin2 A sin2 B
=
sin2 A − sin2 B
= R.H.S.  Proved.
cos2 A cos2 B cos2 A cos2 B cos2 A cos2 B

1 1 1 1
Illustration 5: Prove the following identities: − = −  (JEE ADVANCED)
cosecθ − cot θ sin θ sin θ cosecθ + cot θ

1 1 2
Sol: By rearranging terms we will get + = , and then using Pythagorean identity
cosecθ − cot θ cosecθ + cot θ sin θ
we can solve this problem.

1 1 1 1
We have, − = −
cosecθ − cot θ sin θ sin θ cosecθ + cot θ
1 1 1 1 1 1 2
⇒ + = + ⇒ + =
cosecθ − cot θ cosecθ + cot θ sin θ sin θ cosecθ − cot θ cosecθ + cot θ sin θ

1 1 cosecθ + cot θ + cosecθ − cot θ


Now, L.H.S. = + =
cosecθ − cot θ cosecθ + cot θ ( cosecθ − cot θ )( cosecθ + cot θ )

2cosecθ  cosec2 θ − cot2 θ =1


=
(cosec θ − cot θ)
2 2  

2cosecθ 2  1 
= = = R.H.S.  cosecθ =   Proved.
1 sin θ  sin θ 

Alternative Method

R.H.S
=
1

1
= cosec θ −
( cosec θ − cot θ )
sin θ cosec θ + cot θ cosec2 θ − cot2 θ
= cosec θ − cosec θ + cot θ
= cot θ  Proved.
M a them a tics | 7.5

Illustration 6: Prove that:

(i)
(1 + cot A + tanA )( sinA − cos A ) = sin2 A.cos2 A
sec3 A − cosec3 A
sinA cos A
(ii) + 1
= (JEE ADVANCED)
sec A + tanA − 1 cosecA + cot A − 1

Sol: Using algebra and appropriate identities, we can prove this.

(i) L.H.S. =
(1 + cot A + tanA )( sinA − cos A )
sec3 A − cosec3 A

 cos A sinA 
1 + +  ( sinA − cos A )
sinA cos A 
= 
( sec A − cosecA ) ( sec
( )
 a3 − b3 = ( a − b ) a2 + ab + b2 
2
A + sec A cosec A + cosec A 2
)  

(sinA cos A + cos 2


)
A + sin2 A ( sinA − cos A )  sinA
( sinA cos A + 1)  sinA −
cos A


sinA cos A   cos A sinA cos A
= =
( sec A − cosecA )  2 + cos A1sinA + 12  ( sec A − cosecA )  sin A + sinA
 1   2
cos A + cos A 
2

 cos A sin A   sin2 A cos2 A 

=
( sinA cos A + 1)( sec A − cosecA ) × sin2 A cos2 A  sin2 θ + cos2 θ =1 = sin2 A cos2 A = R.H.S.  Proved.
( sec A − cosecA )(1 + sinA cos A )  

sinA cos A
(ii) L.H.S. = +
sec A + tanA − 1 cosec A + cot A − 1

sinA cosecA + sinA cot A − sinA + cos A sec A + cos A tanA − cos A
=
( sec A + tanA − 1)( cosecA + cot A − 1)
1 + cos A − sinA + 1 + sinA − cos A 2
= =
 1 sinA  1 cos A   1 + sinA − cos A  (1 + cos A − sinA )
 + − 1 + − 1  
 cos A cos A   sinA sinA   cos A  sinA

2sinA cos A 2sinA cos A


= =
1 + ( sinA − cos A )  1 − ( sinA − cos A )  1 − ( sinA − cos A )2
  

2sinA cos A  ( a + b )( a − b ) = a2 − b2 
=
( 2 2
1 − sin A + cos A − 2sinA cos A )  

2sinA cos A 2sinA cos A 2sinA cos A


= =  sin2 θ + cos2 θ =1 = = 1= R.H.S.  Proved.
1 − (1 − 2sinA cos A ) 1 − 1 + 2sinA cos A   2sinA cos A

Illustration 7: Prove that:


 1 1  2 1 − sin2 θ.cos2 θ
 +  sin θ.cos2 θ =  (JEE ADVANCED)
 sec2 θ − cos2 θ cosec2 θ − sin2 θ  2 + sin2 θ.cos2 θ

Sol: Write L.H.S. in terms of cosine and sine functions.


7 . 6 | Trigonometric Ratios, Identities and Equations

 1 1  2
L.H.S. =  +  sin θ.cos2 θ
 sec2 θ − cos2 θ cosec2 − θ sec2 θ 
 
 
 1 1   cos2 θ sin2 θ  2
=  +  sin2 θ cos
= 2
θ  + 2
 sin θ cos θ
 1 1  4 4
 1 − cos θ 1 − sin θ 
 − cos2 θ − sin2 θ 
 cos2 θ sin2 θ 

 
cos2 θ sin2 θ
=  + ( )( )
 sin2 θ cos2 θ  a2 − b2 = a − b a + b 
( )(
 1 + cos2 θ 1 − cos2 θ
 ) (
1 − sin2 θ 1 + sin2 θ )( ) 


 
cos2 θ sin2 θ
=  +  sin2 θ cos2 θ
( )
 1 + cos2 θ sin2 θ cos2 θ 1 + sin2 θ
 ( ) 


=
cos4 θ
+ 
=
sin4 θ (
cos4 θ 1 + sin2 θ + sin4 θ 1 + cos2 θ 
 ) ( )
2 2
1 + cos θ 1 + sin θ 2
1 + cos θ 1 + sin θ2
( )( )
=
cos4 θ + sin2 θ cos4 θ + sin4 θ + sin4 θ cos2 θ
=
(
sin4 θ + cos4 θ + sin2 θ cos2 θ cos2 θ + sin2 θ )
(1 + cos θ)(1 + sin θ)
2 2
(1 + cos θ)(1 + sin θ)
2 2

(sin θ) + (cos θ) + 2sin θ cos θ − sin θ cos θ


2 2
2 2 2 2 2 2

=
(1 + cos θ)(1 + sin θ)
2 2

=
(sin θ + cos θ) =
2 2
− sin θ cos θ 2 2
1 − sin θ cos θ
=
2 2
1 − sin2 θ cos2 θ
= R.H.S.  Proved.
1 + sin2 θ + cos2 θ + sin2 θ cos2 θ 1 + 1 + sin2 θ cos2 θ 2 + sin2 cos2 θ

3. TRANSFORMATIONS

3.1 Compound, Multiple and Sub-Multiple Angles

Circular functions of the algebraic sum of two angles can be expressed as circular functions of separate angles.
sin (A ± B) = sin A cos B ± cos A sin B; cos (A ± B) = cos A cos B  sin A sin B

tanA ± tanB cot A cotB  1


tan ( A ± B ) = ; cot ( A ± B ) =
1  tanA. tanB cotB ± cot A
Circular functions of multiples of an angle can be expressed as circular functions of the angle.
2 tanA
sin2A 2sinA
= = cos A
1 + tan2 A
1 − tan2 A
cos2A = cos2 A − sin2 A = = 2 cos2 A – 1 = 1 – 2 sin2 A
1 + tan2 A
2 tanA
tan2A = ; = 3sinA − 4 sin3 A
sin3A
2
1 − tan A
3tanA − tan3 A
cos3A 4 cos3 A − 3cos A ;
= tan3A =
1 − 3tan2 A
M a them a tics | 7.7

Circular functions of half of an angle can be expressed as circular functions of the complete angle.

A 1 − cos A A 1 + cos A A 1 − cos A 1 − cos A sinA


sin = ; cos = ; tan
= = =
2 2 2 2 2 1 + cos A sinA 1 + cos A

3.2 Complementary and Supplementary Angles

sin ( −θ ) = − sin θ π 
cos  + θ  = − sin θ
 2 
cos ( −θ
= ) cos θ π 
tan  + θ  = − cot θ
2 
tan ( −θ ) = − tan θ sin ( π − θ=
) sin θ
π 
sin  − θ=
 cos θ cos ( π − θ ) = − cos θ
2 
π 
cos  − θ= sin θ tan ( π − θ ) = − tan θ
2 
π 
tan  − θ= cot θ sin ( π + θ ) = − sin θ
2 
π 
sin  + θ=
 cos θ cos ( π + θ ) = − cos θ
2 
tan ( π + θ=
) tan θ

3.3 Product to Sum and Sum to Product


C +D C −D C +D C −D
sinC + sinD =
2sin .cos ; sinC − sinD =
2cos .sin
2 2 2 2
C +D C −D C +D D−C
cosC + cosD =
2cos .cos ; cosC − cosD =
2sin .sin
2 2 2 2

π π
C + −D C − +D
π  2 2
Note: sinC + cosD = sinC + sin  − D  = 2sin .cos
2  2 2

sinC sinD sin ( C + D ) 1


tanC + tanD = + =
cosC cosD cosC.cosD
; sinA.cosB
=
2
{ }
sin ( A + B ) + sin ( A − B )

1 1
sinA.sinB
=
2
{ }
cos ( A − B ) − cos ( A + B ) ; cos A.cosB
=
2
{
cos ( A − B ) + cos ( A + B ) }
sin ( A + B ) .sin ( A − B=
) sin2 A − sin2 B ; cos ( A + B ) .cos ( A − =
B ) cos2 A − sin2 B

3.4 Power Reduction

1 1
sin2=
A
2
(1 − cos2A ) cos2=
A
2
(1 + cos2A )

1 − cos2A 3sinA − sin3A 3cos A + cos3A
tan2 A = ; sin3 A = ; cos3 A =
1 + cos2A 4 4
7 . 8 | Trigonometric Ratios, Identities and Equations

PLANCESS CONCEPTS

2 3
 sin2n A
• cos A.cos2A.cos2 .A cos2= A......cos2n−1 A  if A ≠ nπ
 2n sinA

 1 if A= 2nπ
 −1 if A = (2n+ 1)π

• sin
= ( A1 + A2 + .... + An ) cos A1 cos A2 ....cos An (S1 − S3 + S5 − S7 + .... )
• cos
= ( A1 + A2 + .... + An ) cos A1 cos A2 ....cos An (1 − S2 + S4 − S6 .... )
S − S3 + S5 − S7 + ....
• tan ( A1 + A2 + .... + An ) =1
1 − S2 + S 4 − S6 + ....
Where,
S1 = tan A1 + tan A2 + …. + tan An = The sum of the tangents of the separate angles.
S2 = tan A1 tan A2 + tan A1 tan A3 + …. = The sum of the tangents taken two at a time.
S3 = tan A1 tan A2 tan A3 + tan A2 tan A3 tan A4 + ….. = Sum of tangents three at a time, and so on.
If A1 = A2 = ….. = An = A, then S1 = n tan A, S2 = nC2 tan2 A. S3 = nC3 tan3 A, …..

Vaibhav Gupta (JEE 2009, AIR 54)

4. TRIGONOMETRIC IDENTITY
A trigonometric equation is said to be an identity if it is true for all values of the angle or angles involved. A given
identity may be established by (i) Reducing either side to the other one, or (ii) Reducing each side to the same
expression, or (iii) Any convenient, modification of the methods given in (i) and (ii).

4.1 Conditional Identity


When the angles, A, B and C satisfy a given relation, we can establish many interesting identities connecting the
trigonometric functions of these angles. To prove these identities, we require the properties of complementary and
supplementary angles. For example, if A + B + C = π , then

1. sin (B + C ) = − cos ( C + A ) 2. cos ( A + B ) =


sinA,cosB = sin ( A + B )
− cosC,sinC =

A +B C C A +B
3. tan ( C + A ) = − cot (B + C ) 4.=
− tanB,cot A = cos sin
= ,cos sin
2 2 2 2

C+A B A B+C B+C A B C+A


5.=
sin cos
= ,sin cos 6.
= tan cot
= ,tan cot
2 2 2 2 2 2 2 2

Some Important Identities: If A + B + C =π , then

tanA tanB tanC 2. cotB cot C + cot C cot A + cot A cotB =


1. tanA + tanB + tanC = 1
B C C A A B A B C A B C
3. tan tan + tan tan + tan tan =1 4. cot + cot + cot =cot cot cot
2 2 2 2 2 2 2 2 2 2 2 2
M a them a tics | 7.9

5. sin2A + sin2B + sin2C =


4 sinA sinB sinC 6. cos2A + cos2B + cos2C =−1 − 4 cos A cosB cosC
A B C
7. cos2 A + cos2 B + cos2 C =
1 − 2cos A cosB cosC 8. sinA + sinB + sinC =
4 cos cos cos
2 2 2
A B C
9. cos A + cosB + cosC =
1 + 4 sin sin sin
2 2 2

Illustration 8: Show that:

1
(i) sin ( 40º +θ ) cos (10º +θ ) − cos ( 40º +θ ) sin (10º +θ ) =
2
π  π  π  π 
 sin ( θ + φ ) 
(ii) cos  − θ  cos  − φ  − sin  − θ  sin  − φ= (JEE MAIN)
4  4  4  4 

Sol: Use sum and difference formulae of sine and cosine functions.
(i) L.H.S. = sin ( 40º +θ ) cos (10º +θ ) − cos ( 40º +θ ) sin (10º +θ )
1
{ }
= sin ( 40º +θ ) − (10º +θ )  sin ( A
= − B ) sinA cosB − cos A sinB  = sin30º
= = R.H.S. 
2
Proved.

π  π  π  π 
(ii) L.H.S. = cos  − θ  cos  − φ  − sin  − θ  sin  − φ 
4  4  4  4 
 π  π   π 
= cos  − θ  +  − φ    cos ( A= + B ) cos A cosB − sinA =
sinB  cos  − ( θ + φ ) 
 4  4   2 
 π  
= sin ( θ + φ ) =R.H.S.  cos  − θ=  sin θ   Proved
  2  

1  3π  5 π 
Illustration 9: Find the value of tan ( α + β ) , given that cot α= , α ∈  π,  and sec β = − , β ∈  , π  . 
2  2  3 2 
 (JEE MAIN)
tan α + tan β
Sol: As we know, tan ( α + β ) = , therefore by using product and Pythagorean identities we can obtain
1 − tan α. tan β
the values of tanα and tanβ .
1
Given, cot α = ⇒ tan α =2
2
5 25 4
Also, sec β = − . Then tan β = sec2 β − 1 =± − 1 =±
3 9 3
π  4
But β ∈  , π  ⇒ tan β = −  tan β is − ve in II quadrant 
2  3
 4 2
2 + −  +
4  3  = 3 = + 2
Substituting tan α =2 and tan β = − in (1), we get tan ( α + β ) =
3  4  11 11
1 − (2)  − 
 3  3
Illustration 10: Prove that: tan3A tan2A tanA = tan3A − tan2A − tanA  (JEE MAIN)

tan α + tan β
Sol: Here we can write tan3A as tan ( 2A + A ) , and then by using tan ( α + β ) = we can solve this
problem. 1 − tan α tan β

tan2A + tanA
We have: 3A = 2A + A ⇒ tan3A = tan ( 2A + A ) ⇒ tan3A =
1 − tan2A tanA
7 . 1 0 | Trigonometric Ratios, Identities and Equations

⇒ tan3A (1 − tan2A tanA ) =tan2A + tanA ⇒ tan3A − tan3A tan2A tanA = tan2A + tanA

tan3A tan2A tanA  Proved.


⇒ tan3A − tan2A − tanA =

Illustration 11: Prove that: 2 + 2 + 2 + 2cos8θ = 2cos θ  (JEE MAIN)

Sol: Use 1 + cos2


= θ 2cos2 θ , to solve this problem.

L.H.S. = 2 + 2 + 2 + 2cos8θ = 2 + 2 + 2 (1 + cos8θ )

= 2 + 2 + 2(2cos2 4θ)  1 + cos2


= θ 2cos2 θ 
 

= 2 + 2 + 2cos 4θ= 2 + 2 (1 + cos 4θ ) = (


2 + 2 2cos2 2θ = ) 2 + 2cos2θ

= 2(1 + cos2
= θ) 2.2cos
= 2
θ 4 cos2 θ = 2cos θ = R.H.S.  Proved.

m 1 π
Illustration 12: If tan A = and tan B = , prove that A – B =  (JEE ADVANCED)
m−1 2m − 1 4
tanA − tanB
Sol: Simply using tan (A – B) = , we can prove above equation.
1 + tanA. tanB
m 1
We have, tan A = and tan B =
m−1 2m − 1

tanA − tanB
Now, tan (A – B) =  ... (i)
1 + tanA. tanB

Substituting the values of tan A and tan B in (i), we get


m 1
2m2 − m − m + 1 (m − 1 )( 2m − 1 )

tan (A – B) = m − 1 2m −1 = × 1
=
 m  1 
1+  
( m − 1 )( 2m − 1 ) 2m2 − 3m + 1 + m
 m − 1   2m − 1 

π  π  π
⇒ tan ( A − B ) =
tan  tan = 1 ⇒ A − B =  Proved.
4  4  4

nsin α cos α
Illustration 13: If tan β = ; prove that tan ( α − β =
) (1 − n) tan α  (JEE ADVANCED)
1 − nsin2 α
tan α − tan β
Sol: Same as above problem tan ( α − β ) = , therefore by substituting
1 + tan α tan β
nsin α cos α
tan β = , we can prove given equation.
1 − nsin2 α
tan α − tan β
L.H.S. = tan ( α − β ) =  ... (i)
1 + tan α tan β
sin α nsin α cos α

nsin α cos α cos α 1 − nsin2 α  sin α 
Substituting tan β = in (i), we get L.H.S. =  tan α = 
2
1 − nsin α sin α nsin α cos α  cos α
1+ .
cos α 1 − nsin2 α
M a them a ti cs | 7.11

=
( )
sin α 1 − nsin2 α − nsin α cos2 α
=
3
sin α − nsin α − nsin α cos α 2

cos α (1 − nsin α ) + nsin α cos α


2 2 2 2
cos α − nsin α cos α + nsin α cos α

sin α − nsin α ( sin α + cos α ) sin α − nsin α


2 2

=
cos α cos α

 sin2 α + cos2 α =1 = (


1 − n) sin α
  cos α
= (1 − n) tan α= R.H.S.  Proved.

k sin α sin α
Illustration 14: If θ + φ = α and sin=
θ k sin φ , prove
= that tan θ = ,tan φ (JEE ADVANCED)
1 + k cos α k + cos α 

Sol: Here φ = α − θ , substitute this in sin=


θ k sin φ and then use compound angle formula to obtain required
result.
We have, θ + φ = α ⇒ φ = α − θ  ... (i)
θ k sin φ 
and sin= ... (ii)
⇒ sin
= θ k sin ( α − θ ) [Using = (i)] k sin α cos θ − cos α sin θ 
⇒ sin
= θ k sin α cos θ − k cos α sin θ  ... (iii)
Dividing both sides of (iii) by cos θ, we get tan
= θ k sin α − k cos α. tan θ
k sin α
⇒ tan θ + k cos α. tan
= θ k sin α ⇒ tan θ (1 + k cos α=
) k sin α ⇒ tan θ =  Proved.
1 + k cos α
Again, θ k sin φ
sin= ⇒ sin ( α −=
φ ) k sin φ θ + φ = α ⇒ θ = α − φ 
⇒ sin α cos φ − cos α sin
= φ k sin φ  ... (iv)
Dividing both side of (iv) by cos φ , we get
sin α
⇒ sin α − cos α tan
= φ k tan φ ⇒ (k + cos α ) tan φ= sin α ⇒ tan φ = Proved.
k + cos α

α+β β+γ γ+α


Illustration 15: Prove that: cos α + cos β + cos γ + cos ( α + β + γ ) = 4 cos cos cos (JEE ADVANCED)
2 2 2 
α+β α −β
Sol: Use cos α + cos β = 2cos   cos   , to solve this problem.
 2   2 
L.H.S = cos α + cos β + cos γ + cos ( α + β + γ ) = ( cos α + cos β ) + cos γ + cos ( α + β + γ ) 

α+β α −β α+β+ γ + γ α+β+ γ −γ


= 2cos   cos   + 2cos   .cos  
 2   2   2   2 

α+β α −β  α + β + 2γ  α+β


= 2cos   cos   + 2cos   cos  
 2   2   2   2 

 α + β   α −β  α + β + 2γ  
= 2cos   cos   + cos  
 2    2   2  

  α − β α + β + 2γ   α + β + 2γ α − β  
+ −
 α + β   
2 2
 
2 2 

= 2cos   2cos   cos  
 2   2   2 
   
     
7 . 1 2 | Trigonometric Ratios, Identities and Equations

 α + β   α+γ  β + γ   α+β β+ γ  γ+α


= 2cos   2cos   cos    = 4 cos  2  cos  2  cos  2  = R.H.S.  Proved.
 2    2   2        

 2π   4π 
Illustration 16: If x cos
= θ y cos  θ +=  z cos  θ +  , then show that xy + yz + zx = 0. (JEE ADVANCED)
 3   3 

 2π   4π 
Sol: Consider x cos
= θ y cos  θ +=  z cos  θ + =  k , obtain the value of x , y and z in terms of k, and solve
L.H.S. of given equation.  3   3 

 2π   4π 
Let x cos
= θ y cos  θ +=  z cos  θ + =  k ... (i)
 3   3 

 2π   4π 
cos  θ +  cos  θ + 
1 cos θ 1  3  1  3 
=
⇒ = , = ,
x k y k z k
xyz xyz xyz 1 1 1 
Now, L.H.S. = xy + yz + zx = + + = xyz  + + 
z x y z x y
  4π   2π  
 cos  θ +  cos  θ + 
 3 + cos θ  3  xyz   4π   2π  
= xyz  + [Using
= (i)]  cos  θ +  + cos  θ +  + cos θ 
 k k k  k   3   3  
 
 

xyz  2θ + 2π π  xyz  π  xyz  1 


= 2cos cos + cos θ  = 2cos ( π + θ ) cos + cos θ  =  −2cos θ.   + cos θ 
k  2 3  k  3  k  2 

xyz xyz
=  − cos θ + cos θ  = 0  = 0 ⇒ xy + zy + zx =
0 Proved.
k  k  

Illustration 17: Prove that: cos θ cos2θ cos 4θ......cos2n−1 θ =


( )
sin 2n θ
(JEE ADVANCED)
2n ( sin θ )

Sol: Multiply and divide L.H.S. by 2sinθ and apply sin ( 2θ=
) 2sin θ cos θ .
Here, we observe that each angle in L.H.S. is double of the preceding angle.
L.H.S. = cos θ cos2θ cos 4θ......cos2n−1 θ

=
1
2sin θ
(
( 2sin θ.cos θ ) cos2θ.cos 4θ.....cos2n−1 θ = 2 1 ( 2sin2θ.cos2θ ) cos 4θ......cos2n−1 θ
2 sin θ
)
1
= ( 2sin 4θ.cos 4θ ) cos8θ cos16θ......cos2n−1 θ  sin=
2n θ 2 sin n θ cos n θ 
23 sin θ

=
1
( 2sin8 θ.cos8 θ)  cos16 θ......cos2n−1 
θ =
1 
2sin2n−1
θ cos2
=n−1 
θ
( )
sin 2n θ
= R.H.S.  Proved.
24 sin θ   2n sin θ   2n sin θ

acos φ + b θ a−b φ
Illustration 18: If cos θ = , prove that tan = tan  (JEE ADVANCED)
a + bcos φ 2 a+b 2
θ φ
1 − tan2 1 − tan2
2 2 acos φ + b
Sol: Substitute cos θ = and cos φ = in given equation i.e. cos θ = .
2 θ 2 φ a + bcos φ
1 + tan 1 + tan
2 2
M a them a ti cs | 7.13

 2 φ
 1 − tan 
a.  2 +b
1 − tan2
θ  1 + tan2 φ 
acos φ + b  
Now, cos θ = ⇒ 2 =  2 [Using (i)]
a + bcos φ θ  2 φ
1 + tan2  1 − tan 
2 a + b 2
 1 + tan φ 
2
 
 2

θ a 1 − tan2 φ  + b 1 + tan2 φ  φ φ
1 − tan2  2   2  a − atan2 + b + b tan2
⇒ 2 =     = 2 2
2 θ  2 φ  2 φ 2 φ 2 φ
1 + tan a 1 + tan  + b 1 − tan  a + atan + b − b tan
2  2  2 2 2

Applying componendo and dividendo, we get

θ φ φ
2 tan2
2
2atan2   − 2b tan2  
2 2
( a − b ) tan2 2φ θ a−b φ
= ⇒ tan = tan  Proved
2 2a + 2b a+b 2 a+b 2

5. SOLUTION OF TRIGONOMETRIC EQUATION


A solution of a trigonometric equation is the value of the unknown angle that satisfies the equation.
1 π 3π 9π 11π
Eg.: if sinθ
= ⇒=
θ , , , ,......
2 4 4 4 4
Thus, the trigonometric equation may have infinite number of solutions (because of their periodic nature). These
solutions can be classified as:
(i) Principal solution (ii) General solution

5.1 Principal Solution


 , π ) are called principal solutions.
The solutions of a trigonometric equation which lie in the interval −π

Methods for Finding Principal Value


1
Suppose we have to find the principal value of θ satisfying the equation sin θ = − . Since sin θ is negative, θ
2
will be in 3rd or 4th quadrant. We can approach the 3rd and the 4th quadrant from two directions. Following the
anticlockwise direction, the numerical value of the angle will be greater than π . The clockwise approach would
result in the angles being numerically less than π. To find the principal value, we have to take the angle which is
numerically smallest.
Y

X’ /6 /6 X

B
Y’

Figure 7.2
7 . 1 4 | Trigonometric Ratios, Identities and Equations

For Principal Value


(a) If the angle is in the 1st or 2nd quadrant, we must select the anticlockwise direction and if the angles are in the
3rd or 4th quadrant, we must select the clockwise direction.
(b) Principal value is never numerically greater than π .
(c) Principal value always lies in the first circle (i.e. in first rotation)
π 5π π π
On the above criteria, θ will be − or − . Among these two − has the least numerical value. Hence − is the
6 6 6 6
1
principal value of θ satisfying the equation sin θ = −
2
From the above discussion, the method for finding principal value can be summed up as follows:
(a) First identify the quadrants in which the angle lies.
(b) Select the anticlockwise direction for the 1st and 2nd quadrants and select clockwise direction for the 3rd and
4th quadrants.
(c) Find the angle in the first rotation.
(d) Select the numerically least value among these two values. The angle thus found will be the principal value.
(e) In case, two angles, one with a positive sign and the other with a negative sign have the same numerical value,
then it is the convention to select the angle with the positive sign as the principal value.

5.2 General Solution


The expression which gives all solutions of a trigonometric equation is called a General Solution.

General Solution of Trigonometric Equations


In this section we shall obtain the general solutions of +1
trigonometric equations
sin
= θ 0,cos
= θ 0,tan
= θ 0 and cot θ =0 .
General Solution of sin θ =0 -2 -3/2 - -2
O
2

3/2
2

By Graphical approach:
The graph clearly shows that sin θ =0 at
-1

θ= 0, π, 2 π,......, − π, −2 π ..... Figure 7.3

So the general solution of sin θ =0 is θ = nπ : n ∈ I where n = 0, ± 1, ± 2 …………………..


Note: Trigonometric functions are periodic functions. Therefore, solutions of trigonometric equations can be
generalized with the help of periodicity of trigonometric functions.

PLANCESS CONCEPTS

A trigonometric identity is satisfied by any value of an unknown angle while a trigonometric equation is
satisfied by certain values of the unknown.
Vaibhav Krishnan (JEE 2009, AIR 22)

Method for Finding Principal Value


(a) First note the quadrants in which the angle lies.
(b) For the 1st and 2nd quadrants, consider the anticlockwise direction. For the 3rd and 4th quadrants, consider the
clockwise direction.
M a them a ti cs | 7.15

(c) Find the angles in the 1st rotation.


(d) Select the numerically least value among these two values. The angle found will be the principal value.

Illustration 19: Principal value of tan θ = −1 is  (JEE MAIN)

Sol: Solve it by using above mentioned method.


Y
 tanθ is negative
B +
∴ θ will lie in 2nd or 4th quadrant 3
For the 2 quadrant, we will choose the anticlockwise direction and for the 4 quadrant,
nd th 4
we will select the clockwise direction. X’  X
-
π 3π + - 4
In the first circle, two values − and are obtained.
4 4
π A
Among these two, − is numerically least angle.
4 Y’
π
Hence, the principal value is − Figure 7.4
4

1
Illustration 20: Principal value of cos θ = is:  (JEE MAIN)
2
Sol: Here cos θ is ( + ) ve hence θ will lie in 1st or 4th quadrant.
 cos θ is ( + ) ve ∴ θ will lie in the 1st or the 4th quadrant.
Y
B
For the 1st quadrant, we will select the anticlockwise direction and for the 4th quadrant,

we will select the clockwise direction.
3
π −π
As a result, in the first circle, two values and are found. X’
O 
X
3 3 -
π π 3
Both and − have the same numerical value.
3 3
A
π
In such a case, will be selected as the principal value, as it has a positive sign. Y’
3
Figure 7.5
Illustration 21: Find the general solutions of the following equations:
3 
(i) sin2θ =0 (ii) cos  θ  =0 (iii) tan2 2θ =0  (JEE MAIN)
2 
Sol: By using above mentioned method of finding general solution we can solve these equation.

(i) We have, sin2θ = 0 ⇒ 2θ = nπ ⇒ θ = where, n = 0, ± 1, ± 2, ± 3......
2

Hence, the general solution of sin2θ =0 =
is θ ,n ∈ Z
2

(ii) We know that, the general solution of the equation cos θ =0 is =


θ ( 2n + 1) 2π ,n ∈ Z
 3θ 
Therefore, cos   = 0 ⇒
3θ π
= ( 2n + 1 ) ⇒ =
θ ( 2n + 1) 3π , where n = 0, ± 1, ± 2 …….
 2  2 2
 3θ 
Which is the general solution of cos   = 0
 2 
(iii) We know that the general solution of the equation tan θ =0 is θ = nπ , n ∈ Z

Therefore, tan2 2θ =0 ⇒ tan2θ =0 ⇒ 2θ = nπ ⇒ θ = , where n = 0, ± 1, ± 2 ………
2
Which is the required solution.
7 . 1 6 | Trigonometric Ratios, Identities and Equations

6. PERIODIC FUNCTION
A function f (x) is said to be periodic if there exists T > 0 such that f (x + T) = f (x) for all x in the domain of definition
of f (x). If T is the smallest positive real number such that f (x + T) = f (x), then it is called the period of f (x).
( 2nπ + x ) sinx,cos=
We know that, sin= ( 2nπ + x ) cos x , =
tan (nπ + x ) tanx for all n ∈ Z
Therefore, sinx, cosx and tanx are periodic functions. The period of sinx and cosx is 2π and the period of tanx is π .
Function Period

sin ( ax + b ) ,cos ( ax + b ) ,sec ( ax + b ) ,cosec ( ax + b ) 2π / a

tan ( ax + b ) ,cot ( ax + b ) π/a

sin ( ax + b ) , cos ( ax + b ) , sec ( ax + b ) , cosec ( ax + b ) π/a

tan ( ax + b ) , cot ( ax + b ) π / 2a

(a) Trigonometric equations can be solved by different methods. The form of solutions obtained in different
methods may be different. From these different forms of solutions, it is wrong to assume that the answer
obtained by one method is wrong and those obtained by another method are correct. The solutions obtained
by different methods may be shown to be equivalent by some supplementary transformations.
To test the equivalence of two solutions obtained from two different methods, the simplest way is to put
values of n = ……….. – 2, – 1, 0, 1, 2, 3, ……. etc. and then to find the angles in [0, 2π]. If all the angles in both
the solutions are same, the solutions are equivalent.
(b) While manipulating the trigonometric equation, avoid the danger of losing roots. Generally, some roots are
lost by cancelling a common factor from the two sides of an equation. For example, suppose we have the
equation tan x = 2 sin x. Here by dividing both sides by sin x, we get cos x = 1/2.
(c) While equating one of the factors to zero, we must take care to see that the other factor does not become
infinite. For example, if we have the equation sin x = 0, which can be written as cos x tan x = 0. Here we cannot
put cos x = 0, since for cos x = 0, tan x = sin x / cos x is infinite.
(d) Avoid squaring: When we square both sides of an equation, some extraneous roots appear. Hence it is
necessary to check all the solutions found by substituting them in the given equation and omit the solutions
that do not satisfy the given equation.
For example: Consider the equation, sin θ + cos θ =1  …. (i)
Squaring, we get 1 + sin2θ =1 or sin2θ =0  …. (ii)
This gives θ= 0, π / 2, π,3π / 2 ………
Verification shows that π and 3π / 2 do not satisfy the equation as sin π + cos π = −1, ≠ 1 and
sin3π / 2 + cos3π / 2 =−1, ≠ 1 .
The reason for this is simple.
The equation (ii) is not equivalent to (i) and (ii) contains two equations: sin θ + cos θ =1 and sin θ + cos θ = −1 .
Therefore, we get extra solutions.
Thus if squaring is a must, verify each of the solutions.

Some Necessary Restriction: If the equation involves tan x, sec x, take cos x ≠ 0. If cot x or cosec x appear, take
sin x ≠ 0. If log appears in the equation, then number > 0 and base of log > 0, ≠ 1.
Also note that f ( θ )  is always positive. For example, sin2 θ = sinθ , not ± sin θ .
 

Verification: Students are advised to check whether all the roots obtained by them satisfy the equation and lie in
the domain of the variable of the given equation.
M a them a ti cs | 7.17

7. SOME TRIGONOMETRIC EQUATIONS WITH THEIR GENERAL SOLUTIONS

Trigonometric equation General solution

If sin θ =0 θ = nπ

If cos θ =0 θ= (nπ + π / 2)= ( 2n + 1) π / 2

If tan θ =0 θ = nπ

If sin θ =1 θ= 2nπ + π / 2= ( 4n + 1) π / 2
If cos θ =1 θ= 2nπ

θ = nπ + ( −1 ) α where α ∈  −π / 2, π / 2 
n
If sin θ
= sin α

If cos=
θ cos α θ= 2nπ ± α where α ∈ 0, π 

If tan=
θ tan α θ = nπ + α where α ∈  −π / 2, π / 2 

If sin2=
θ sin2 α θ = nπ ± α

2
If cos= θ cos2 α θ = nπ ± α

2
If tan= θ tan2 α θ = nπ ± α

If sin θ= sin α θ= 2nπ + α


cos= θ cos α

If sin θ= sin α θ= 2nπ + α


tan= θ tan α

If tan=
θ tan α θ= 2nπ + α
cos=
θ cos α

Note: Everywhere in this chapter, “n” is taken as an integer.

Illustration 22: Solve: sinmθ + sinnθ = 0  (JEE MAIN)


α+β α −β
Sol: By using sin α + sin β = 2sin   cos   , we can solve this problem.
 2   2 
We have, sinmθ + sinnθ = 0

m+n m−n m+n m−n


⇒ sin   θ.cos   θ =0 ⇒ sin   θ =0 or cos   θ =0
 2   2   2   2 
m+n m+n
Now, sin   θ =0 ⇒   θ = rπ,r ∈ Z  ... (i)
 2   2 
7 . 1 8 | Trigonometric Ratios, Identities and Equations

m−n m−n π
And cos   θ =0 ⇒ cos  2  θ =cos 2
 2   

m−n  2p + 1 
⇒ =θ ( 2p + 1) 2π , p ∈ Z=
⇒θ   π,  … (ii)
 2   m−n 
2rπ  2p + 1 
From (i) and (ii), we have θ = or
= θ   π where, m,n ∈ Z
m+n  m−1 

Illustration 23: Solve: 4 sinx cos x + 2sinx + 2cos x + 1 =0 (JEE ADVANCED)

Sol: Simply using algebra and method of finding general equation, we can solve above equation.
We have, 4 sinx cos x + 2sinx + 2cos x + 1 =0
⇒ 2sinx ( 2cos x + 1 ) + 1 ( 2cos x + 1 ) =
0 ⇒ ( 2sinx + 1 )( 2cos x + 1 ) =
0
1 1
⇒ 2sinx + 1 =0 or 2cos x + 1 =0 ⇒ sinx = − or cos x = −
2 2
1  π π
sinx = − = sin  − 
⇒ sinx ⇒ x= − The general solution of this is
2  6 6
 −1 ) 
(
n+1
n π n+1  π 
x = nπ + ( −1 )  −  = nπ + ( −1 )   ⇒ x = 
π n+  … (i)
 6  6
   6 
 
1  π 2π 2π
and cos x = − ⇒ cos
= x cos  π − = cos ⇒ x=
2  3 3 3

2π  1
The general solution of this is x= 2nπ ± i.e. x = 2π  n ±   ... (ii)
3  3
 −1 ) 
(
n+1

From (1) and (2), we have π n +  and 2π  n ± 1  are the required solutions
  
6   3
 

8. METHODS OF SOLVING TRIGONOMETRIC EQUATIONS

8.1 Factorization
Trigonometric equations can be solved by use of factorization.

sin2 x 
Illustration 24: Solve: ( 2sinx − cos x )(1 + cos x ) = (JEE MAIN)

Sol: Use factorization method to solve this illustration.


( 2sinx − cos x )(1 + cos x ) = sin2 x ⇒ ( 2sinx − cos x )(1 + cos x ) − sin2 x = 0
( 2sinx − cos x )(1 + cos x ) − (1 − cos x )(1 + cos x ) =
0 ; (1 + cos x )( 2sinx − 1 ) =
0
1 + cosx = 0 or 2 sin x – 1 = 0
1
cos x = – 1 or sin x =
2
cos x = cos π or sin x = sin π / 6
( 2n + 1) π,n ∈ I or x = nπ + ( −1 ) π / 6,n ∈ I
n
⇒ x=
∴ The solution of given equation is ( 2n + 1 ) π,n ∈ I or nπ + ( −1 ) π / 6, n ∈ I
n
M a them a ti cs | 7.19

8.2 Sum to Product


Trigonometric equations can be solved by transforming a sum or difference of trigonometric ratios into their
product.

Illustration 25: If sin 5x + sin 3x + sin x = 0 and 0 ≤ x ≤ π / 2 , then x is equal to.  (JEE MAIN)

α+β α −β
Sol: By using sum to product formula i.e. sin α + sin β = 2sin   cos  .
 2   2 
0 ⇒ sin3x ( 2cos2x + 1 ) =
sin 5x + sinx = – sin 3x ⇒ 2sin3x cos2x + sin3x = 0
⇒ sin3x = −1 / 2 ⇒ x = nπ, x = nπ ± ( π / 3)
0,cos2x =

0
Illustration 26: Solve cos3x + sin2x − sin 4x = (JEE MAIN)

Sol: Same as above illustration, by using formula


α +β α −β
sin α − sin β = 2cos   sin   We can solve this illustration.
 2   2 
cos 3x + sin 2x – sin 4x = 0 ⇒ cos3x + 2cos3x.sin − x =
0( )
0 ⇒ cos3x (1 − 2sinx ) =
⇒ cos3x − 2cos3x.sinx = 0
π 1
0 or 1 – 2 sin x = 0
⇒ cos3x = ⇒ 3x = ( 2n + 1 ) ,n ∈ I or sinx =
2 2

( 2n + 1) 6π ,n ∈ I or x = nπ + ( −1 )
π
n
⇒ x= ,n ∈ I
6
π n π
∴ Solution of given equation is ( 2n + 1 ) ,n ∈ I or nπ + ( −1 ) ,n ∈ I
6 6

8.3 Product to Sum


Trigonometric equations can also be solved by transforming product into a sum or difference of trigonometric
ratios.

Illustration 27: The number of solutions of the equation sin5x cos3x = sin6x cos2x, in the interval 0, π  , is:
 (JEE MAIN)
Sol: Simply by using product to sum method.
1
The given equation can be written as
2
( sin8x + sin2x )= 12 ( sin8x + sin 4x )
⇒ sin 2x – sin 4x = 0 ⇒ – 2 sin x cos 3x = 0
π
Hence sin x = 0 or cos 3x = 0. That is, x = nπ (n ∈ I ) , or 3x = kπ +
2
(k ∈ I ) .
π π 5π
Therefore, since x ∈ 0, π  , the given equation is satisfied if x = 0, π, , or .
6 2 6
Hence, no. of solutions is 5.
7 . 2 0 | Trigonometric Ratios, Identities and Equations

8.4 Parametric Methods


General solution of trigonometric equation acos θ + b sin θ = c
b
To solve the equation acos θ + b sin θ = c , put a= r cos φ,b= r sin φ such that=
r a2 + b2=
, φ tan−1
a
Substituting these values in the equation, we have, r cos φ cos θ + r sin φ sin θ = c
c c
cos ( θ − φ ) =
r
(
⇒ cos θ − φ = )
a + b2
2

If c > a2 + b2 , then the equation acos θ + b sin θ = c has no solution.

c
If c ≤ a2 + b2 , then put = cos α , so that cos ( θ − φ=
) cos α
a2 + b2
⇒ ( θ − φ )= 2nπ ± α ⇒ θ= 2nπ ± α + φ

2
Illustration 28: Solve: sinx + 3 cos x = (JEE MAIN)

Sol: Solve by using above mentioned parametric method.

Given, 3 cos x + sinx =2 , dividing both sides by a2 + b2

3 1 2 1  π π
⇒ cos x + sinx = = ⇒ cos  x −  =
cos  
2 2 2 2  6 4
π π π π 5π π
⇒ x − = 2nπ ± ⇒ x= 2nπ ± + ⇒ x= 2nπ + , 2nπ − where n ∈ I
6 4 4 6 12 12
Note: Trigonometric equations of the form a sin x + b cos x = c can also be solved by changing sin x and cos x into
their corresponding tangent of half the angle. i.e t=tan x/2. The following example gives you insight.

Illustration 29: Solve: 3 cos x + 4 sin x = 5 (JEE MAIN)

x x
1 − tan2 2 tan
Sol: As we know, cos x = 2 and sinx = 2 . Therefore by substituting these values and solving we
2 x 2 x
1 + tan 1 + tan
will be get the result. 2 2

3 cos x + 4 sin x = 5  … (i)


x x
1 − tan2 2 tan
 cos x = 2 & sinx = 2 ∴ Equation (i) becomes
2 x 2 x
1 + tan 1 + tan
2 2
 2 x   x 
 1 − tan   2 tan 
⇒ 3 2  + 4 2 = 5 … (ii)
 1 + tan2 x   1 + tan2 x 
   
 2  2
x
Let tan = t ∴ Equation (ii) becomes
2
 1 − t2   2t 
5 ⇒ 4t2 − 4t + 1 = 0 ⇒ ( 2t − 1 ) = 0 ⇒ t = 1 / 2  t = tanx / 2
2
3  + 4 =
 1 + t2  2 
  1 + t 
M a them a ti cs | 7.21

x 1 x 1 x −1  1 
⇒ tan =⇒ tan = where, α tan   ,n ∈ I
tan α , where tan α = ⇒ = nπ + α ⇒ x= 2nπ + 2α =
2 2 2 2 2 2

8.5 Functions of sin x and cos x


Trigonometric equations of the form P (sin x ± cos x, sin x cos x) = 0, where P (y, z) is a polynomial, can be solved
by using the substitution sin x ± cos x = C.

Illustration 30: Solve: sin x + cos x = 1 + sin x. cos x (JEE MAIN)

Sol: Consider sin x + cos x = t, and solve it by using parametric method.


 sin x + cos x = 1 + sin x. cos x ... (i)
Let sin x + cos x = t
t2 − 1
⇒ sin2 x + cos2 x + 2sinx.cosx =
t2 ⇒ sinx.cos x =
2
t2 − 1 t2 − 1
Now, put sin x + cos x = t and sin x. cos x = in (i), we get t = 1 +
2 2
⇒ t2 − 2t + 1 =0 ⇒t=1  t = sin x + cos x ⇒ sinx + cos x =
1 … (ii)

Dividing both sides of equation (ii) by 2 , we get:

1 1 1  π π π π
⇒ sinx + cos x. = ⇒ cos  x −  = cos ⇒ x − = 2nπ ±
2 2 2  4  4 4 4
π
If we take the positive sign, we get x = 2nπ + , n ∈ I
2
If we take the negative sign, we get x = 2nπ, n ∈ I

8.6 Using Boundaries of sin x and cos x


Trigonometric equations can be solved by the use of boundness of the trigonometric ratios sinx and cos x.

PLANCESS CONCEPTS

(i) The answer should not contain such values of angles which make any of the terms undefined or
infinite.
(ii) Never cancel terms containing unknown terms on the two sides, which are in product. It may cause
loss of the general solution.
Suppose the equation is sin x = (tan x)/2. Now, cancelling sinx on both the sides, we get only
1
cos x = , sin x = 0 is not counted.
2
(iii) Check that the denominator is not zero at any stage while solving equations.
(iv) While solving a trigonometric equation, squaring the equation at any step must be avoided if
possible. If squaring is necessary, check the solution for extraneous values.
Suppose the equation is sin x = – sin x. We know that the only solution of this is sin x = 0 but on squaring,
we get (sin x)2=(sin x)2 which is always true.
(v) Domain should not change, it if changes, necessary corrections must be made.
Shivam Agarwal (JEE 2009, AIR 27)
7 . 2 2 | Trigonometric Ratios, Identities and Equations

Illustration 31: Solve: sin 3x + cos 2x = – 2 (JEE ADVANCED)

Sol: By using boundary condition of sin x and cos x.


Since sin3x ≥ −1 and cos2x ≥ −1 , we have, sin3x + cos2x ≥ −2
Thus, the equality holds true if and only if sin3x =
−1 and cos2x =
−1
n π nπ n π π
⇒ 3x = nπ + ( −1 )  −  and 2x = 2nπ ± π i.e. x = + ( −1 )  −  and x = nπ ± , n ∈ I
 2  3  6  2

   
∴ Solution set is, x | x = nπ + ( −1)n  − π   ∩ x | x = nπ ±  π  
 3  6     2  
Note: Here, unlike all other problems, the solution set consists of the intersection of two solution sets and not the
union of the solution sets.

 x   x 
Illustration 32: sinx  cos − 2sinx  +  1 + sin − 2cos x  ( cos x ) =
0 . Find the general solution. (JEE ADVANCED)
 4   4 
Sol: Open all brackets of given equation and then by using sum to product formula and method of finding general
solution we will get the result.
x x
sinx cos − 2sin2 x + cos x + sin cos x − 2cos2 x =
0
4 4

 x 5x 5x
sin  x +  + cos x =
2 ⇒ sin + cos x =2 ⇒ sin = 1 and cos x = 1
 4 4 4
5x 5x π π
sin = 1 ⇒ = 2nπ + =
⇒ x 2 ( 4n + 1 ) ; cos x = 1 ⇒ x = 2mπ
4 4 2 5

⇒ x = 2π,10π,18π ……..AP ⇒ x = 2π + m − 1 8π ( )
( )
⇒ x = 2π 4m − 3 m ∈ I

 π
Illustration 33: Find the general solution of 2sin  3x +  = 1 + 8 sin2x cos2 2x (JEE ADVANCED)
4
 
Sol: First square on both side and then using sum and difference formula we can solve this illustration.
2 2
 π  sin3x cos3x 
 = 1 + 8 sin2x cos 2x
2
4 sin2  3x +  = 1 + 8 sin2x cos2 2x ⇒ 4  +
 4   2 2 
4 sin2 3x 4 cos2 3x
⇒ + 1 + 8 sin2x cos2 2x
+ 4 sin3x cos3x =
2 2
⇒ 2sin2 3x + 2cos2 3x + 2sin6x =
1 + 8 sin2x cos2 2x
⇒ 1 + 2 sin 6x = 8 sin 2x cos2 2x ⇒ 1 + 2 sin 6x = 4 sin 4x cos 2x
⇒ 1 + 2 sin 6x = 2 (sin 6x + sin 2x) ⇒ 1 = 2 sin 2x ⇒ sin 2x = ½
π sin x cos4 x 1
4
⇒ x= + 2nπ x= + = n ∈ I
12 2 3 5

9. SIMULTANEOUS EQUATIONS
Two equations are given and we have to find the value of variable θ which may satisfy both the given equations,
M a them a ti cs | 7.23

like cos=
θ cos α and sin θ= sin α
So, the common solution is θ= 2nπ + α , n ∈ I
Similarly, sin θ= sin α and tan=
θ tan α
So, the common solution is θ= 2nπ + α , n ∈ I

1
Illustration 34: The most general value of θ satisfying the equations cos θ = and tan θ = −1 is: (JEE MAIN)
2
Sol: As above mentioned method we can find out the general value of θ .
1 π
cos=
θ = cos  
2 4
π 9π 7π
⇒ θ= 2nπ ± ; n ∈ I Put n = 1 θ = ,
4 4 4

 −π  3π 7π
tan θ = −1 = tan   ⇒ θ = nπ − π / 4 , n ∈ I Put n = 1, θ = ; Put n= 2, θ=
 4  4 4
 7π 
The common value which satisfies both these equation is   .
 4 

Hence, the general value is 2nπ + .
4

1
Illustration 35: The most general value of θ satisfying equations sin θ = − and tan θ =1 / 3 are:  (JEE MAIN)
2
Sol: Similar to above illustration.
We shall first consider values of θ between 0 and 2π
1 π  π
sin θ = − = − = sin  π +  or sin ( 2π − π / 6 )
2 6  6
∴ θ = 7π / 6,11π / 6 ; tan
= θ 1/ =3 tan ( π /= 6 ) tan ( π + π / 6 )
∴ θ = π / 6,7π / 6
Thus, the value of θ between 0 and 2π which satisfies both the equations is 7π / 6 .
Hence, the general value of θ is 2nπ + 7π / 6 where n ∈ I

PROBLEM SOLVING TACTICS


nπ + ( −1 ) A
n
A
(a) Any formula that gives the value of sin in terms of sin A shall also give the value of sin .
2 2
A 2nπ ± A
(b) Any formula that gives the value of cos in terms of cos A shall also give the value of cos .
2 2
A nπ ± A
(c) Any formula that gives the value of tan in terms of tan A shall also give the value of tan .
2 2
7 . 2 4 | Trigonometric Ratios, Identities and Equations

(d) If α is the least positive value of θ which satisfies two given trigonometric equations, then the general value
of θ will be 2nπ + α . For example, sin θ= sin α and cos= θ cos α , then, θ= 2nπ + α ,n ∈ I
sin (nπ + θ ) = ( −1) sin θ, n ∈ I
n
(i)
cos (nπ + θ ) = ( −1 ) cos θ, n ∈ I
n
(ii)
sin (nπ − θ ) = ( −1 ) sin θ, n ∈ I
n−1
(iii)

FORMULAE SHEET

Tangent and cotangent


sin θ cos θ
Identities tan θ = and cot θ =
cos θ sin θ

Product Identities sin θ× cosec θ = 1 , cos θ× sec θ = 1 , tan θ× cot θ = 1

Pythagorean Identities 2
+ 1 sec2 θ , 1 + cot=
sin2 θ + cos2 θ =1 , tan θ = 2
θ csc2 θ

sin ( −θ ) = − sin θ , cos ( −θ


= ) cos θ , tan ( −θ ) = − tan θ ,
Even/Odd Formulas
cot ( −θ ) = − cot θ , sec ( −θ
= ) sec θ , cosec ( −θ ) = − cosec θ

Periodic Formulas
sin ( 2n π + θ=
) sin θ , cos ( 2n π + θ=
) cos θ , tan(n π + =
θ) tan θ ,
(If n is an integer)
θ) cot θ , sec ( 2n π + θ=
cot(n π + = ) sec θ , cosec ( 2n π =
+ θ ) cosec θ

sin ( 2θ=
) 2sin θ cos θ , sin3
=θ 3sin θ − 4 sin3 θ
Double and Triple Angle
Formulas
cos ( 2=
θ ) cos2 θ − sin2 θ cos3
= θ 4 cos3 θ − 3cos θ

2 tan θ 3tan θ − tan3 θ


tan ( 2θ ) = , tan3θ =
1 − tan2 θ 1 − 3tan2 θ

π  π  π 
Complementary angles sin  ± θ=
 cos θ , cos  ±=θ   sin θ , tan  ±=θ   cot θ ,
 2   2   2 
π  π  π 
cot  − θ=
 tan θ , sec  = − θ  cosec θ , cosec  − θ=
 sec θ
 2   2   2 

1 1 1 − cos ( 2θ )
sin2=
θ 1 − cos ( 2θ )  , cos2=
θ 1 + cos ( 2θ )  , tan2 θ =
Half Angle 2   2   1 + cos ( 2θ )

sin ( α ± β=
) sin α cos β ± cos α sinβ ,
Sum and Difference
cos ( α ± β=
) cos α cos β  sin α sinβ ,
tan α ± tan β
tan ( α ± β ) = ,
1  tan α tan β
M a them a ti cs | 7.25

1
sin α=
sin β cos ( α − β ) − cos ( α + β )  ,
Product to Sum 2 
1
sin α =
cos β sin ( α + β ) + sin ( α − β )  ,
2 
1
cos α=
cos β cos ( α − β ) + cos ( α + β )  ,
2 
1
cos α=
sin β sin ( α + β ) − sin ( α − β )  ,
2 

α+β α −β
Sum to Product sin α + sin β = 2sin   cos  ,
 2   2 

α +β α −β
sin α − sin β = 2cos   sin  
 2   2 

α+β α −β
cos α + cos β = 2cos   cos  
 2   2 

α +β α −β
cos α − cos β = −2sin   sin  
 2   2 

Solved Examples

JEE Main/Boards
Example 1: Solve: 2cos2θ + 2sin θ = 2  1 − cos x  cos x + 1
⇒ 8=
  : cos x ≠ 0, −1
1  1 + cos x  cos x
Sol: Solve this example by using sin2=
θ 1 − cos ( 2θ )  .
2 
or (8 – 8 cosx)cosx = (cosx + 1)2
θ 2 (1 − cos2θ=
2sin= ) 4 sin2 θ or 8 cosx – 8 cos2x = cos2x + 2cosx + 1
4
∴ 2=
sin θ 16 sin θ : sin θ ≥ 4
or 9cos2x – 6 cosx + 1 = 0
1
sin θ =0 or sin3 θ = ∴ sin θ =0 or or (3cosx –1)2 = 0
8
1 n π
, θ= mπ : m ∈ I or θ = nπ + ( −1 ) : n∈ I or cosx =
1 1
= cos β, ( say ) , β =cos−1  
2 6 3 3
x ∴ x= 2nπ ± β : n ∈ I
Example 2: Solve: 8 tan2 = 1 + sec x
2
Sol: As we know that Example 3: Solve: sinx + cos x − 2 2 sinx cos x =
0
x 1 − cos x , substitute this to solve above
tan2 = Sol: We can write given equation a sinx + cosx =
2 1 + cos x
example. sinx + cos x = 2 sin2x , multiplying and dividing L.H.S. by 2 , we will
x get the result.
8 tan2 = 1 + sec x  ... (i)
2
7 . 2 6 | Trigonometric Ratios, Identities and Equations

or
 1
2 sinx +
1 
cos x  =
2 sin2x (
a2 − 2a + 1 + tan2 π ( a + x ) =0 )
 2 2  2
or ( a − 1 ) + tan π ( a + x )  =
2
0
 π
or sin  x +  =sin2x
 4 ⇒ a−1 =0 and tan π ( a + x ) =
0
n π
⇒ 2x = nπ + ( −1 )  x +  : n ∈ I ⇒ tan (1 + x ) π =0 ⇒ (1 + x ) π = nπ : n ∈ I
 4
∴x = n−1 : n∈ I : a = 1
Example 4: Find the general value of θ which satisfies
3 1 Example 7: Solve the equation cos7 x + sin4 x =
1
both the equations cos θ = − and sin θ = .
2 2
Sol: Use the method for simultaneous equations. Sol: Here c os7 x ≤ cos2 x and sin4 x ≤ sin2 x , hence by
solving this we will get the result.
3 5π 7π
cos θ = − ⇒θ= , ….
2 6 6 cos7 x ≤ cos2 x and sin4 x ≤ sin2 x

1 π 5π ∴
= 1 cos7 x + sin4 x ≤ cos2 x + sin2 x =
1
sin θ = ⇒ θ = , .....
2 6 6 ∴ c os7 x =
cos2 x and sin4 x = sin2 x
Hence, the general solution is given by

(
cos7 x = cos2 x ⇒ cos2 x cos5 x − 1 =
0 )
θ 2nπ+
= , n∈ I ∴ cos
= x 0 or cos
= x 1
6

Example 5: Show that the equation


∴ x= ( 2n + 1) 2π ; n∈I

x π or x = 2mπ; m ∈ I  …(i)


2cos2   sin2 =
x x2 + x −2 for 0 < x ≤
2
  2
sin2 x = sin4 x ⇒ sin2 x cos2 x =
0
has no real solution.
⇒ cos= = 0 
x 0 or sinx ... (ii)
x 1
Sol: Here 2cos2 sin2 =x x2 + holds only If x2 =1,
2 x2 Since (i) satisfies the system (ii),
hence by substituting x =1 in above equation we can
conclude that given equation has real solution or not. ∴ Solution set is given by (i)

1
x2 + ≥ 2∀x with equality for Example 8: Solve for x and y:
x2
x 12sinx + 5cos x = 2y 2 − 8y + 21
x2 = 1 alone. Since 2cos2 sin2 x ≤ 2 ,
2
Sol: Multiply and divide L.H.S. by 13 and solve to get
x 1
∴ 2cos2 sin2 x =
x2 + holds only the result.
2 x2
x 12sinx + 5cos x = 2y 2 − 8y + 21
If x2 =1 ∴ x = 1 and cos sinx = ±1
2
 1C 
 12 5 
(
∴ 122 + 52  sinx + cos x = 2 y 2 − 4y + 4 + 13
 13 13 
)
i.e. cos   sin1C = ±1 , which is not true.
 2 
  or 13cos ( x − α ) 2 ( y − 2) + 13
2
=
Hence, the given equation has no solution.
5 12
: cos α
= and
= sin α
13 13
Example 6: Determine ‘a’ for which the equation
Thus, cos ( x − α ) =1 and y = 2 or
( )
a2 − 2a + sec2 π ( a + x ) =
0 has solutions and find the
solutions. x − α= 2nπ : n ∈ I and y = 2

Sol: By using algebra and tangent of angle we can get ∴ x= 2nπ + α : n ∈ I and y = 2
the result.
M a them a ti cs | 7.27

JEE Advanced/Boards ⇒ 2x = nπ + ( −1 )
n π
: n ∈ I General solution is
6
Example 1: Solve for x, y: x cos3 y + 3x cos y sin2 y =
14 nπ n π
3 2 x= + ( −1 ) ,n ∈ I
x sin y + 3x cos y sin y =
13 2 12

Sol: Divide equation 1 by equation 2 and then by Example 3: Solve: 3tan ( θ − 15º
= ) tan ( θ + 15º )
applying componendo and dividendo we can solve this
problem. tan ( θ + 15º ) 3
Sol: We can write given equation as = ,
We note that, “x = 0; sin y = 0 or cos y = 0” do not yield tan ( θ − 15º ) 1
a solution to given system. hence by applying componendo and dividendo we will
3 2 get the result.
cos y + 3cos y sin y 14
=
3 2
sin y + 3cos y sin y 13 Given, 3tan ( θ − 15º
= ) tan ( θ + 15º )
By componendo and dividendo, tan ( θ + 15º ) 3
or =
3 2 2
cos y + 3cos y sin y + 3cos y sin y + sin y 3
tan ( θ − 15º ) 1
3 2 2 3
cos y + 3cos y sin y − 3cos y sin y − sin y
tan ( θ + 15º ) + tan ( θ − 15º ) 4
3 or =
=
14 + 13  cos y + sin y 
or  (3)
3 tan ( θ + 15º ) − tan ( θ − 15º ) 2
= 27
=
14 − 13  cos y − sin y 
sin ( θ + 15º +θ − 15º ) π
cos y + sin y 1 + tan y 3 or = 2 or sin2θ = 1 = sin
or = 3 or = sin ( θ + 15º −θ + 15º ) 2
cos y − sin y 1 − tan y 1
1 π nπ n π
⇒ 2θ = nπ + ( −1 ) + ( −1 )
n
tan y = = tan α ; y = nπ + α : n ∈ I : n ∈ I ∴=
θ : n∈ I
2 2 2 4
Since sin y and cos y have signs, we have the following
cases: Example 4: Find value of θ for sin2
= θ cos3θ , where
0 ≤ θ ≤ 2π ; Use the above equation to find the value
1 2 of sin 18º.
(i) sin y = and cos y = ; then
5 5
π 
 8 2 1 Sol: Here as we know= sin θ cos  − θ  , hence we can
x +3 .  = 14 ⇒ x = 5 5 2 
5 5
5 5 write given equation as cos3
=θ cos ( π / 2 ) − 2θ . { }
1 2
(ii) sin y = − and cos y = − ; then Therefore by comparing their angle we will get the
5 5 result.
 −8  −2  1 
x + 3   = 14 ⇒ x =−5 5 The given equation is sin2
=θ cos3θ or, cos3
=θ sin2θ
 5 5  5  5 
or, cos3
= {
θ cos ( π / 2 ) − 2θ }
7
Example 2: Solve: sin4 x + cos4 x =
2
sinx cos x or, 3θ= 2nπ ± {( π / 2) − 2θ} where n ∈ I
Taking + sign, 3θ= 2nπ + {π / 2 − 2θ}
Sol: By substituting 2sinx cos x = t and solving we will
be get the result. or 5=
θ ( 4n + 1)( π / 2)
7
sin4 x + cos4 x = sin2x ; ∴ sin2x > 0 θ ( 4n + 1 )( π / 10 ) , where n ∈ I 
or, = … (i)
4

( ) 7 Again taking – sign, 3θ − 2nπ − {( π / 2 ) − 2θ}


2
sin2 x + cos2 x − 2sin2 x cos2 x = sin2x
4
or =θ ( 4n + 1 )( π / 2 )  … (ii)
t2 7t 2
∴1 − = or 2t + 7t − 4 =
0
2 4 Putting n = 0, 1, 2, 3, …….in (i) the values of θ in the
1 π interval 0 ≤ θ ≤ 2π are given by
or ( 2t − 1 )( t + 4 ) =
0 ∴ sin2x =
t= sin
=
2 6
θ = π / 10,5π / 10,9π / 10,13π / 10,17π / 10 or 18º, 90º,
162º, 234º, 346º.
7 . 2 8 | Trigonometric Ratios, Identities and Equations

Again putting n = 0, ±1, ±2,......, in (ii) the value of θ in only for those (integral) values of k, n, m for which the
corresponding right members of the relations (i) are
the interval 0 ≤ θ ≤ 2 π is 3π / 2 i.e. 270º only.
positive.
Hence the required values of θ in 0 ≤ θ ≤ 2π are 18º, It is easy to see that of the first equation of (i) that
90º, 162º, 234º, 270º, 306º. is positive for integer k > 0, the right side of second
equation of (i) is positive for integral n > 0; and the
Example 5: Solve the equation: right side of the third equation of (i) is positive for
m ≥ 0 . Thus, we have to solve (i) only for the indicated
( ) ( ) ( )
cos π3x − 2cos2 π3x + 2cos 4 π3x − cos 7π3x ( ) values of k, m, n. The resulting values of x are then the
roots of the original equation:
= sin ( π3 ) + 2sin ( π3 ) − 2sin ( 4 π3 )
x 2 x x
 1 2k 
x log3  − +  =
= ;k 1,2,......
+2sin ( π3 ) − sin ( 7π3 )
x +1 x
 6 6 

n 1 1 
= x log
= 3   ,n 1,2,...... = log3  +  m= 0,1,2
Sol: Substitute π 3x =
y , and then by using sum to 2 8 m
product formula we can solve this equation.
Example 6: Solve the equation:
Denote π 3x by y to get

cos y − 2cos2 y + 2cos 4y − cos7y 1 


x − 1  2 tanx (1 + 4 sinx )
17 sec2 x + 16  tanx sec=
2 
=sin y + 2sin2 y − 2sin 4y +2sin3y − sin7y  …(i)

Transposing all terms to the left side, Sol: Solve it like algebra by using product and
Pythagorean identity.
we have, ( cos y − cos7y ) + ( sin7y − sin y )
The given equation is
+2 ( cos 4y + sin 4y ) − 2 ( sin3y + 1 ) =
0
1 
17 sec2 x + 16  tanx sec x − 1 
or, 2sin 4y sin3y + 2cos 4y sin3y 2 
+2 ( cos 4y + sin 4y ) − 2 ( sin3y + 1 ) =
0 = 2 tanx (1 + 4 sinx )  … (i)
[Use C & D formulae]
⇒ 17 sec2 x + 8 tanx sec x − 16 × 1
or, 2sin3y ( sin 4y + cos 4y ) + 2 = 2 tanx (1 + 4 sinx )
+2 ( cos 4y + sin 4y ) − 2 ( sin3y + 1 ) =
0
⇒ 17
⇒ 17 sec
2
sec2 xx +
+88 tanx
tanx sec
sec xx −− 16 (
16 ×sec
⇒ 17 sec2 x + 8 tanx sec x − 16 × 1
2 2
1 x − tan x )
or, ( sin3y + 1 )( sin 4y + cos 4y − 1 ) =
0 tanx(1 sinx))
= 2
=
=
tanx
22tanx (1+++444sinx
1 ( sinx )
This enables us to write down three groups of solutions:
π 2kπ nπ π mπ
⇒ 16 sec222 xx ++ 88tanx
17tan tanxsec ((
16 2sec
secxx +− sec x 2
2
x − tan2 x
⇒ 17 sec x + 8 tanx sec x − 16 sec x − tan2 x ))
y1 =− + , y 2= ,y= + = 2 tanx (1 + 4 sinx )
6 3 2 3 8 2 = 2 tanx (1 + 4 sinx )
where k, n and m are arbitrary integers. Recalling ⇒ 16 tan22 x + 8 tanx sec x + sec22 x
⇒ 16 tan x + 8 tanx sec x + sec x
that y = π3x , we obtain an infinity of equations for
determining the roots of the original equations:
= 2 tanx (1 + 4 sinx )
1 2k n
3x =− + , k = 0, ± 1, ± 2 ,…… 3x= ,n= 0, ± 2,
( 4 tanx ) + 2 × 4 tanx.sec x + sec2 x
2
6 3 2
1 m
… = + ,m = 0, ± 1, ± 2, .... = 2 tanx (1 + 4 sinx )
8 2
4 tanx + sec= x 2 tanx (1 + 4 sinx )
The equation 3x = a has a (unique) root only
for positive a and it is given by the formula
x = log3 a . Therefore, the equation (i) has a solution
M a them a ti cs | 7.29

⇒ 8 sinx. tanx − 2 tanx − sec x =


0 Example 8: Consider the system of linear equations in
2
sin x sinx 1 x, y and z:
⇒8 −2 − 0
=
cos x cos x cos x ( sin3θ ) x − y + z =0  … (i)
2
⇒ 8 sin x − 2sinx − 1 =0
( cos2θ ) x + 4y + 3z = 0  ... (ii)
1 1 π
∴ sinx =and sinx =
− 2x + 7y + 7z = 0 ... (iii)
2 4 4

 1 Find the value of θ for which the system has a non-


= sin π / 6 ⇒ x = sin–1  − 
sinx trivial solution.
 4
∴ Solution of (i) is given by Sol: Here we can write given linear equation in matrix
π 1 form, and as we know for the system having non-trivial
and x = nπ + ( −1 )
n+1
x = nπ ( −1 )
n
sin−1   ,
6 4 solution A must be 0.
where n ∈ I We can write the given linear equation in the form of
AX=O
x  0 
Example 7: Solve the equation:    
 sin3θ −1 1     
29  
sin10 x + cos10 x = cos4 2x A= cos2θ 4 3  , X =  y  and O = 0  .
16    
 2 7 7     

z  0 
Sol: we can represent given equation as    
5 5 As the system has a non-trivial solution,
 2sin2 x   2cos2 x  29
  +  = cos4 2x , then use half
 2   2  16 A must be 0
   
angle formula to solve this problem. sin3θ −1 1
29 ∴ cos2θ 4 3 =0
Given, sin10 x + cos10 x = cos4 2x
16 2 7 7
5 5
 2sin2 x   2cos2 x  29 ⇒ 7 sin3θ + 14 cos2θ − 14 = 0
  +  = cos4 2x
 2   2  16
    ⇒ sin3θ + 2cos2θ − 2 = 0 ,

(3sin θ − 4 sin θ) + 2 (1 − 2sin θ) − 2 =0 or


5 5
 1 − cos2x   1 + cos2x  29 3 2
  +  = cos4 2x
 2   2  16
4 sin3 θ + 4 sin2 θ − 3sin θ = 0
(1 − cos2 x)2 .(1 − cos2 x)3 + (1 + cos2 x)2 .(1 + cos2 x)3
32
( )
sin θ 4 sin2 θ + 4 sin θ − 3 = 0

29 4 sin θ ( 2sin θ − 1 )( 2sin θ + 3) = 0


= cos 2x
16
∴=sin θ 0 or= sin θ 1 / 2 or sin θ = −3 / 2
⇒ 10 cos 4 2x + 20 cos2 2x + 2 =
58 cos4 2x
1
But sin θ =0 sin θ = is possible
⇒ 48 cos 4 2x − 20 cos2 2x − 2 =0 2
−3
And sin θ = is not possible.
2
⇒ 24 cos 4 2x – 10 cos22x – 1 = 0
Now, sin θ =0 ∴ θ = nπ ; n ∈ I
⇒ (2 cos22x –1) (12 cos22x + 1) = 0 and sin= 2 sin ( π / 6 ) ,
θ 1 /=
∴ θ = nπ + ( −1 ) π / 6 ; n ∈ I
n
∴ 2 cos2 2x − 1 =0 12 cos2 2x + 1 ≠ 0 
 
⇒ cos 4x =
0 Therefore the required values of θ are
θ = n π and nπ + ( −1 ) ( π / 6 ) , where n ∈ I
n
π
⇒ 4x = nπ +
2
nπ π
⇒ x= + ,n ∈ I
4 8
7 . 3 0 | Trigonometric Ratios, Identities and Equations

Example 9: Find the value of x, 1

Minimum value of 2 cos2 x =2


1
cos2 x
1
2 y2 − y + ≤1 2 2
2  1 1 1
Minimum value of y −  +  =
 2 2
   2
Sol: Re-write the expression inside the square root and
then by using algebra we can find out the value of x. 1
⇒ Minimum value of (i) is 2 × = 1.
Given 2
1 2 2
1
1 cos2 x
 1 1 1
2 cos2 x 2
y −y+ ≤1 ...(i) ∴ 2 1 ⇒ cos2 x =
y −  +  = 1 and y =
2  2 2
   2

1 ⇒ cos2 2nπ =1
cos2 x
1 1
2 y2 − y + + ≤ 1 ∴ x= 2nπ
4 4
1 2 2
cos2 x
 1 1
2 y −  +  ≤ 1
 2 2

JEE Main/Boards

Exercise 1 Q.11 Solve the equation: tanx + sec x =


2cos x

Q.1 Solve the following trigonometric equations:


Q.12 Solve: 2sin2 x − 5sinx cos x − 8 cos2 x =
−2
1 1
(i) sin2θ = (ii) cos5θ = −
2 2 Q.13 Solve: 4 sinx sin2x sin 4x = sin3x

Q.2 Solve 7 cos2 θ + 3sin2 θ = 4


Q.14 Solve the equation

Q.3 Solve: tanx + tan2x + 3 tanx tan2x =


3 (1 − tan θ )(1 + sin2θ ) = 1 + tan θ

Q.4 Solve: 3tan ( θ − 15º


= ) tan ( θ + 15º ) Q.15 Solve sinx + 3 cos x =
2

Q.5 Solve the equations


Q.16 Find the general solution of the following
3 1 trigonometric equations:
sin ( x − y ) = and cos ( x + y ) =
2 2 1
(i) tan3θ = −1 (ii) cos5x = −
Q.6 Solve the equation sin x = tanx 2

Q.17 Solve the following trigonometric equations:


Q.7 Solve the equation 2 tan θ − cot θ + 1 = 0
(i) 3cos2 θ + 7 sin2 θ = 4
Q.8 Solve the equations sinmx + sinnx =
0 (ii) tanx + tan2x + tan3x =
tan2x tan3x
π  π 
Q.9 Solve the equation sec2 2x= 1 − tan2x (iii) tan  + θ  + tan  − θ  = 4
4  4 

Q.10 Solve the equation: Q.18 Solve the equation tanx + cot x =
2
4 sinx cos x + 2sinx + 2cos x + 1 =0
M a them a ti cs | 7.31

Q.19 Find the general solution of the trigonometric Exercise 2


equation: tan3 x − 3tanx =
0
Single Correct Choice Type
Q.20 Solve the following trigonometric equations:
A B 3
1 (ii) sec x − tanx =
(i) cos x + sinx = 3 Q.1 If in a triangle ABC, b cos2 + acos2 = c , then
a, c, b are: 2 2 2
1
(iii) sinx + cos x = (iv) cos x + 3 sinx =
1 (A) In A.P. (B) In G.P. (C) In H.P. (D) None
2
π 
Q.2 Given a2 + 2a + cosec2  ( a + x )  =
0
Q.21 Find the degree measures corresponding to the 2 
following radian measures.
then, which of the following holds good?
C C x
π  4π  (A) a = 1; ∈ I
 (iii) (1.2 )
C
(i)   (ii)  2
6  5  x
(B) a = –1; ∈ I
2
Q.22 The angles in a triangle are in A.P. and the ratio
of the smallest angle in degree to the greatest angle (C) a ∈ R; x ∈ φ
in radians is 60 : π . Find the angles of the triangle in (D) a, x are finite but not possible to find
degrees and radians.
C C
Q.3 In any triangle ABC, ( a + b ) sin2 + ( a − b ) cos2 =
2 2

Q.23 Assuming the distance of the earth from the 2 2


moon to be 38400 km and the angle subtended by the (A) c (a + b) (B) b (c + a) (C) a (b + c) (D) c2
moon at the eye of a person on the earth to be 31’, find
the diameter of the moon.
Q.4 If in a ∆ABC,sin3 A + sin3 B + sin3 C

Q.24 Assuming that a person of normal sight can read = 3sinA ⋅ sinB ⋅ sinC then
print at such a distance that the letters subtend an
(A) ∆ABC may be a scalene triangle
angle of 5’ at his eye, find the height of the letters that
he can read at a distance of 12 meters. (B) ∆ABC is a right triangle
(C) ∆ABC is an obtuse angled triangle
Q.25 Solve the equation 4 cos2 x sinx − 2sin2 x =
3sinx (D) ∆ABC is an equilateral triangle

Q.26 Solve the equation: Q.5 sin3


=θ 4 sin θ sin2θ sin 4θ in 0 ≤ θ ≤ π has:
θ (A) 2 real solutions (B) 4 real solutions
5cos2θ + 2cos2 + 1= 0, −π < θ < π
2
(C) 6 real solutions (D) 8 real solutions
Q.27 Solve the equation: 4 sin4 x + cos4 x =
1

Q.6 With usual notations, in a triangle ABC,


Q.28 Solve the equation: tan2θ tan θ =1 acos (B − C ) + b cos ( C − A ) + c cos ( A − B ) is equal to

abc abc 4abc abc


Q.29 Show that the equation: esin x − e− sin x − 4 =
0 has (A) (B) (C) (D)
2 2 2
no real solution. R 4R R 2R 2

2cos β − 1 α β
Q.30 Does the equation sin4 θ − 2sin2 θ − 1 = 0 has a Q.7 If cos α = then tan cot has the value
2 − cos β 2 2
solution?
equal to, where ( 0 < α < π and 0 < β < π )

(A) 2 (B) 2 (C) 3 (D) 3


7 . 3 2 | Trigonometric Ratios, Identities and Equations

 2π   4π  Q.17 The maximum value of


Q.8 If x sin
= θ y sin  θ + =  z sin  θ +  then
 3   3  ( 7 cos θ + 24 sin θ ) × ( 7 sin θ − 24 cos θ )
(A) x + y + z =0 (B) xy + yz + zx =
0 for every θ ∈ R .
625 625
(C) xyz + x + y + z =1 (D) None of these (A) 25 (B) 625 (C) (D)
2 4

Q.9 If acos2 α + 3acos α sin2 α =m Q.18 If x = , satisfies the equation
3 2
2
and asin α + 3acos α sin α =n . Then x x
sin − cos = 1 − sinx & the inequality
(m + n) + (m − n)
2/3 2/3
is equal to: 2 2

(A) 2 a2 (B) 2a1/3 (C) 2 a2/3 (D) 2 a3 x π 3π


− ≤ , then :
2 2 4
Q.10 The number of solutions of (A) n = –1, 0, 3, 5 (B) n = 1, 2, 4, 5
tan (5π cos=
θ ) cot (5π sin θ ) for θ in ( 0,2π ) is (C) n = 0, 2, 4 (D) n = – 1, 1, 3, 5
(A) 28 (B) 14 (C) 4 (D) 2
Q.19 The number of all possible triplets ( a1 ,a2 ,a3 )
B C 2
such that a1 + a2 cos2x + a3 sin x =
0 for all x is
Q.11 In a ∆ABC if B + C = 3A then cot .cot has the
value equal to 2 2
(A) 0 (B) 1 (C) 3 (D) Infinite
(A) 4 (B) 3 (C) 2 (D) 1
Q.20 If A and B are complimentary angles, then:
Q.12 The set of value of ‘a’ for which the equation,
2a 7 possess a solution is-
cos2x + asinx =−
 A  B
(A)  1 + tan  1 + tan  =
2
 2  2
(A) ( −∞ ,2 ) (B) [2, 6] (C) ( 6,∞ ) (D) ( −∞ , ∞ )
 A  B
(B)  1 + cot  1 + cot  =
2
A B  2  2
∑ cot2 2 .cot2 2
Q.13 In ∆ABC , the minimum value of
A  A  B
is ∏ cot2 2 (C)  1 + sec  1 + cosec  =
2
 2  2
(A) 1 (B) 2
(D) A set containing two values
(C) 3 (D) Non-existent

Q.14 The general solution of Previous Years’ Questions


sinx + sin5x = sin2x + sin 4x is:
x
(A) 2nπ (B) nπ Q.1 The equation 2cos2   sin2 =
x x2 + x −2 ,
2
 
(C) nπ / 3 (D) 2nπ / 3 Where n ∈ I π
0 < x ≤ has  (1980)
2
Q.15 Number of values of θ ∈ 0,2π  satisfying the (A) No real solution
equation cotx – cosx = 1 – cotx. cosx
(B) One real solution
(A) 1 (B) 2 (C) 3 (D) 4
(C) More than one real solution
(D) None of above
Q.16 The exact value of
cos2 73º + cos2 47º + ( cos73º.cos 47º ) is
Q.2 The smallest positive root of the equation, tan
1 1 3 x – x = 0 lies in  (1987)
(A) (B) (C) (D) 1
4 2 4  π π   3π   3π 
(A)  0,  (B)  , π  (C)  π,  (D)  ,2π 
 2 2   2   2 
M a them a ti cs | 7.33

Q.3 The number of solution of the equation Q.9 Given A = sin2 θ + cos4θ, then for all real values

( )
sin ex= 5x + 5− x is  (1991)
of θ 
3
(1980)
(A) 1 ≤ A ≤ 2 (B) ≤ A ≤ 1
(A) 0 (B) 1 (C) 2 (D) Infinitely many 4
13 3 13
(C) ≤ A ≤1 (D) ≤A≤
Q.4 The number of integral values of k for which the 16 4 16
equation 7 cos x + 5sinx = 2k + 1 has a solution, is
 (2002) Q.10 The expression  (1986)
(A) 4 (B) 8 (C) 10 (D) 12
  3π  
3 sin4  − α  + sin4 ( 3π + α ) 
  2  
Q.5 The set of values of θ satisfying the in equation
2sin2 θ − 5sin θ + 2 > 0 , where 0 < θ < 2π , is (2006)  π  
−2 sin6  + α  + sin6 (5π − α )  is equal to
 π   5π   π   5π   2  
(A)  0,  ∪  ,2π  (B) 0,  ∪  ,2π 
 6   6   6   6  (A) 0 (B) 1

 π   2π  (C) 3 (D) sin 4α + cos6α


(C) 0,  ∪  ,2π  (D) None of these
 3  3  π
Q.11 If α + β = and β + γ = α , then tanα , equals
 2 (2001)
Q.6 The number of solutions of the pair of equations
2sin2 θ − cos2θ = 0 and 2cos2 θ − 3cos θ = 0 in the (A) 2 ( tan β + tan γ ) (B) tan β + tan γ

interval 0,2π  is (2007)
(C) tan β + 2 tan γ (D) 2 tan β + tan γ
(A) 0 (B) 1 (C) 2 (D) 4
 π
( tan θ ) ,t2 =( tan θ )
tan θ cot θ
Q.7 Let P = {θ : sin θ − cos θ = }
2 cos θ and Q.12 Let θ ∈  0,  and t1 =
 4 
Q= {θ : sin θ + cos θ = }
2 sin θ be two sets. Then t3
= ( cos θ )
tan θ
and=
t4 ( cot θ )
cot θ
, then (2006)
 (2011)
(A) P ⊂ Q and Q − P ≠ ∅ (A) t1 > t2 > t3 > t 4 (B) t 4 > t3 > t1 > t2

(B) Q ⊄ P (C) t3 > t1 > t2 > t 4 (D) t2 > t3 > t1 > t 4
(C) P ⊄ Q
(D) P = Q tan A cot A
Q.13 The expression + can be
1 − cot A 1 − tan A
Q.8 If α + β + γ = 2π , then  (1979) written as  (2013)
(A) secA cosecA + 1 (B) tanA + cotA
α β γ α β γ
(A) tan + tan + tan =tan tan tan
2 2 2 2 2 2 (C) secA + cosecA (D) sinA cosA + 1

α β β γ γ α
(B) tan tan + tan tan + tan tan =1 Q.14 If 0 ≤ x < 2 π , then the number of real values of
2 2 2 2 2 2
x, which satisfy the equation cosx + cos2x + cos3x +
(C) tan
α β γ α β γ cos4x = 0 is:  (2016)
+ tan + tan =− tan tan tan
2 2 2 2 2 2
(A) 5 (B) 7 (C) 9 (D) 3
(D) None of these
7 . 3 4 | Trigonometric Ratios, Identities and Equations

JEE Advanced/Boards

Exercise 1  π
Q.13 2sin  3x +  = 1 + 8sin2x.cos2 2x
 4
Q.1 Solve the equation: sin5x = 16 sin5 x
Q.14 Find the number of principal solutions of the
equation.
Q.2 Find all the solutions, of 4cos2xsinx ﹘ 2sin2x = 3sinx
sinx − sin3x + sin5x = cos x − cos3x + cos5x

Q.3 Find the number of solutions of the equation


1 + cos x + cos2x + sinx + sin2x + sin3x =0. Q.15 Find the general solution of the trigonometric
π π equation
Which satisfy the condition < 3x − ≤ π .
2 2 1 
 +log3 ( cos x + sin x ) 
2 log2 ( cos x −sin x )
3  −2 2
=
Q.4 Solve for x, ( −π ≤ x ≤ π ) the equation;
2 ( cos x + cos2x ) + sin2x (1 + 2cos x ) =
2sinx Q.16 Find all values of θ between 0º & 180º satisfying
the equation; cos6θ + cos 4θ + cos2θ + 1 = 0
Q.5 Find the general solution of the following equation:
Q.17 Find the solution set of the equation,
2 ( sinx − cos2x ) − sin2x (1 + 2sinx ) + 2cos x =
0
log
− x2 −6x
( sin3x + sinx ) =
log 2
− x −6x
( sin2x )
Q.6 Find the values of x, between 0 & 2π . Satisfying the 10 10

3x x
equation cos3x + cos2x = sin + sin .
2 2
Q.18 Find the value of θ , which satisfy
2 2
Q.7 Solve: tan 2x + cot 2x + 2 tan2x + 2cot 2x =
6 3 ﹘ 2cos θ ﹘ 4sinθ ﹘ cos2θ + sin2θ = 0.

x Q.19 Find the sum of the roots of the equation


sec2
Q.8 Solve the equation: 1 + 2 cosecx = – 2 and cos 4x + 6 =7 cos2x on the interval 0,314  .
2
2cos7x
the inequality > 2cos 2x Q.20 Find the least positive angle measured in degrees
cos3 + sin3
satisfying the equation

sin3 x + sin3 2x + sin3 3x = ( sinx + sin2x + sin3x )


3
 x  x
Q.9 Solve sin   + cos  =2 sin x
 2   2 
   
Q.21 Find the number of solution of the equation
π 
Q.10 Find all values of ‘a’ for which every root of the sin ( π − 6x ) + 3 sin  + 6x  = 3 in 0,2π 
equation, a cos 2x + a cos 4x + cos 6x = 1 is also a 2 
1
root of the equation,= sinxcos2x
sinx cos2x = sin2x
sin2xcos3x
cos3x− sin5x
sin5x, Q.22 Find the general values of θ for which the
2
quadratic function
and conversely, every root of the second equation is
also a root of the first equation. ( sin θ ) x2 + ( 2cos θ ) x + cos θ 2+ sin θ
is the square of a linear function.
Q.11 Solve for x, the equation 13 − 18 tanx =6 tan x – 3,
where −2π < x < 2π .
Q.23 Prove that the equations
(a) sinx ⋅ sin2x ⋅ sin3x =
1
Q.12 Determine the smallest positive value of x which
satisfy the equation 1 + sin2x − 2 cos3x =
0 (b) sinx ⋅ cos 4x ⋅ sin5x =
−1 / 2
M a them a ti cs | 7.35

(c) sinx cos x cos2x + 1 / 2 =


0 Q.27
(d) 4 sin2x + cos x =
5
Column I Column II
(e) sin3x − cos x =
2
(A) The general solution of the (p) nπ(where n ∈ I)
Have no solution equation sin2 x + cos2 3x = 1 is
equal to
4
(
Q.24 Let f ( x ) = sin x + cos x + k sin x + cos x
6 6 4
) for (B) The general solution of the

some real number k. Determine equation (q)
4
(a) All real number k for which f(x) is constant for all 2θ
ecot + sin2 θ − 2cos2 2θ +=
4 4 sin θ, is
values of x.
cot θ 2
sin2 θ − 2cos2 2θ +=
(b) All real numbers k for which ethere +exists a real 4 4 sin θ, is
number ‘c’ such that f(c) = 0 (C) For all real values of a, the
π
(c) If k = – 0.7, determine all solutions to the equation general solution of the equation (r) nπ +
4
f(x) = 0 a2 sinx − asin2x + sinx = 0 , is
equal to
Q.25 If α and β are the roots of the equation, (D) The general solution of the
π
acos θ + b sin θ = c then match the entries of column I equation (s) (4n + 1)
2
with the entries of column II. 3
2 tan θ − 1 + 3 tan θ − 1 =1 , is

Column I Column II
(A) sinα + sinβ
(p) 2b Exercise 2
a+c

(B) sinα.sinβ Single Correct Choice Type


(q) c − a
c+a
Q.1 If in a ∆ABC, cos A. cos B + sin A sinB sin 2C =1
α β 2bc then, the statement which is correct?
(C) tan + tan (r)
2 2 a2 + b2 (A) ∆ABC is isosceles and right angled
α β c2 − a2 (B) ∆ABC is acute angled
(D) tan . tan (s)
2 2 2 2
(C) ∆ABC is not right angled
a +b
π
(D) Least angle of the triangle is
3
Q.26 Solve the equations for ‘x’ given in column I and
match the entries of column II. Q.2 The set of values of x satisfying the equation,
 π
 π sin2  x − 
Column I Column II tan x −   4
2  4 − 2 ( 0.25 ) cos 2x +1 =0 is
(A) cos3x.cos3 x + sin3x.sin3 x =
0 π
(p) nπ ±
3 (A) An empty set
(B) sin3
= (
α 4 sin α sin x + α ) (q) nπ +
π (B) A singleton
,n ∈ I
sin ( x − α ) 4
(C) A set containing two elements
Where α is a constant ≠ nπ
(D) An infinite set

nπ π
(C) 2 tanx − 1 + 2cot x − 1 =2 (r) + ,n ∈ I Q.3 The number of solutions of the equation,
4 8 5

29 nπ π ∑ cos (rx ) = 0 lying in (0, π) is:


(D) sin10 x + cos10 x =cos4 2x (s) ± r =1
16 2 4
(A) 2 (B) 3 (C) 5 (D) More than 5
7 . 3 6 | Trigonometric Ratios, Identities and Equations

1º 1º 1º 1º Q.11 The value of x that satisfies the relation


Q.4 The value of cot 7 + tan67 − cot 67 − tan7
is 2 2 2 2 x = 1 − x + x2 − x3 + x 4 − x5 + .............∞

(A) A rational number (B) Irrational number (A) 2cos36° (B) 2cos144°

(
(C) 2 3 + 2 3 ) (
(D) 2 3 − 3 ) (C) 2sin18° (D) None of these

Q.5 If A = 580º then which one of the following is true? Multiple Correct Choice Type

A Q.12 An extreme value of 1+4sin θ +3cos θ is:


(A) 2sin   = 1 + sinA − 1 − sinA
2 (A) -3 (B) -4 (C) 5 (D) 6
A
(B) 2sin   =− 1 + sinA + 1 − sinA 4
2 Q.13 It is known that =
sin β & 0 < β < π then the
value of 5
A
(C) 2sin   =− 1 + sinA − 1 − sinA 2
2 3 sin ( α + β ) − cos ( α + β )
π
A cos
(D) 2sin   = 1 + sinA + 1 − sinA 6 is:
2 sin α
(A) Independent of α for all β in (0, π /2)
x2 − x
Q.6 If tan α = and 5
x2 − x + 1 (B) for tan β >0
3
1
=tan β
2
( x ≠ 0,1) , where
2x − 2x + 1 3(7 + 24 cot α )
(C) for tan β <0
π 15
0 < α , β < , then tan ( α + β ) has the value equal to:
2
3 (D) None of these
(A) 1 (B) –1 (C) 2 (D)
4
1 1
Q.7 Minimum value of 8 cos2 x + 18 sec2 x ∀ x ∈ R Q.14 If sint +cost= then tan is equal to:
5 5
wherever it is defined, is:
(A) -1 (B) -1/3 (C) 2 (D) -1/6
(A) 24 (B) 25 (C) 26 (D) 18

Q.8 If 𐐭 is eliminated from the equations x=a cos(𐐭-∝) Previous Years’ Questions
x2 y 2 2xy
and y=b cos(𐐭- β ) then + − is
cos(α − β)isequalto
a2 b2 ab Q.1 Show that the equation esin x − e− sin x − 4 =
0 has no
equal to real solution. (1982)
(A) cos2 (α − β) (B) sin2 (α − β)
Q.2 Find the values of x ( −π, π ) which satisfy the
(C) sec2 (α − β) (D) cosec2 (α − β)
1 + cos x + cos2 x +.....
equation 2 =4  (1984)
Q.9 The general solution of the trigonometric equation
tanx + tan2x + tan3x =tanx.tan2x.tan3x is Q.3 If exp {(sin x + sin
2 4
) }
x + sin6 x + ..... + ∞ loge 2 ,
π 2
(A) x = nπ (B) nπ ± satisfies the equation x − 9x + 8 =0 , find the value of
3
cox π
nπ ,0 < x < . (1991)
(C) x= 2nπ (D) x = Where n ϵ I cos x + sinx 2
3

Q.10 Number of principal solutions of the equation Q.4 Determine the smallest positive value of x (in
tan3x-tan2x-tanx=0 , is degree) for which tan (x + 100º) = tan (x + 50º) tan (x)
tan (x – 50º)  (1993)
(A) 3 (B) 5 (C) 7 (D) More than 7
M a them a ti cs | 7.37

Q.5 Find the smallest positive number p for which the  π π


equation cos (p sin x) –sin (pcos x)=0 has a solution Q.12 The number of values of θ in the interval  − , 
nπ  2 2
x ∈ 0,2π   (1995) such that θ ≠ for n = 0, ± 1, ± 2 and tan
= θ cot5θ
5
 π π as well as sin2
=θ cos 4θ is .... (2010)
Q.6 Find all value of θ in the interval  − ,  satisfying
the equation  2 2
2θ Q.13 The positive value of n > 3 satisfying the equation
(1 − tan θ )(1 + tan θ ) sec2 θ + 2tan =0  (1996)
1 1 1
= + is ...... (2011)
π  2π   3π 
Q.7 The values of θ lying between θ =0 and θ = π / 2 sin   sin   sin  
and satisfying the equation n  n   n 

1 + sin2 θ cos2 θ 4 sin 4θ


2 2 Q.14 The number of all possible values of θ , where
sin θ 1 + cos θ 4 sin 4θ 0 
= (1988)
0 < θ < π , for which the system of equations
sin2 θ cos2 θ 1 + 4 sin 4θ
2cos3θ 2sin3θ
( y + z=
) cos3θ ( xyz ) sin3θ x=
sin3θ
y
+
z
7π 5π 11π π
(A) (B) (C) (D)
24 24 24 24 And ( xyz ) sin3θ= ( y + 2z ) cos3θ + y sin3θ
∞ have a solution ( x0 , y 0 , z 0 ) with y 0 z 0 ≠ 0 , is....(2010)
Q.8 For 0 < φ < π / 2 , if x = ∑ cos2n φ , y =
n= 0
 π π
∞ ∞ Q.15 The number of values of θ in the interval  − , 
∑ sin 2n
φ, z = ∑ cos 2n
φ sin φ , then
2n
(1993)

 2 2
n= 0 n= 0
such that θ ≠ for n = 0, ± 1, ± 2 and tan
= θ cot 5 θ
5
= xy + y (B) xyz
(A) xyz = xy + z
as well as sin=
2 θ cos 4 θ is  (2010)
(C) xyz = x + y + z (D) xyz
= yz + x
Q.16 The maximum value of the expression
4xy 1
Q.9 sec2 θ = is true if and only if (1996) is  (2010)
(x + y)
2
sin2 θ + 3sin θ cos θ + 5cos2 θ
(A) x= y ≠ 0 (B)=
x y, x ≠ 0

(C) x = y (D) x ≠ 0, y ≠ 0
Q.17 Let P = {θ : sin θ − cos θ = 2 cos θ }
and Q = {θ : sin θ + cos θ = }
2 sin θ be two sets. Then
sin4 x cos4 x 1  (2011)
Q.10 If + = , then  (2009)
2 3 5 (A) P ⊂ Q and Q − P ≠ ∅ (B) Q ⊄ P
2 2 sin8 x cos8 x 1 (C) P ⊄ Q (D) P = Q
(A) tan x = (B) + =
3 8 27 125
Q.18 The positive integer value of n > 3 satisfying the
1 2 sin8 x cos8 x 2 equation
(C) tan x = (D) + =
3 8 27 125
1 1 1
= + is  (2011)
π π  2π   3π 
Q.11 For 0 < θ < , the solution(s) of sin   sin   sin  
2 n  n   n 
6  (m − 1) π  cosec  θ + mπ  = 4
∑ cosec  θ + 4   
4 
2 is/are
m=1   
 (2009)
π π π 5π
(A) (B) (C) (D)
4 6 12 12
7 . 3 8 | Trigonometric Ratios, Identities and Equations

Q.19 Let θ, φ ∈ 0, 2 π  be such that Q.21 The number of distinct solution of the equation

 θ 5
2 cos θ (1 − sin=
θ
φ ) sin2 θ  tan + cot  cos φ − 1 cos2 2x + cos4 x + sin4 x + cos6 x + sin6 x =
2
2 2 4
 ,
In the interval 0,2 π  is  (2015)
3
tan ( 2 π − θ ) > 0 and −1 < sin θ < −
2
 π
Then φ cannot satisfy (2012) S x ∈ ( −π, π ) : x ≠ 0 , ±  .
Q.22 Let =
 2 
π π 4π
(A) 0 < φ < (B) < φ < The sum of all distinct solutions of the equation
2 2 3
3 sec x + cosec x + 2 ( tan x − cot x ) =
0 in the set S is
4π 3π 3π equal to (2016)
(C) <φ< (D) < φ < 2π
3 2 2
7π 2π 5π
(A) − (B) − (C) 0 (D)
9 9 9
Q.20 For x ∈ ( 0, π ) , the equation sin x + 2 sin 2x – sin
3x = 3 has (2014)
(A) Infinitely many solutions
(B) Three solutions
(C) One solution
(D) No solution

PlancEssential Questions
JEE Main/Boards JEE Advanced/Boards

Exercise 1 Exercise 1
Q.11 Q.17 Q.24 Q.7 Q.12 Q.17
Q.29 Q.21 Q.24 Q.26

Exercise 2 Exercise 2
Q.6 Q.9 Q.14 Q.4 Q.7 Q.8
Q.17 Q.19 Q.11

Previous Years’ Questions Previous Years’ Questions


Q.1 Q.4 Q.5 Q.4 Q.5 Q.6
Q.8 Q.10 Q.13 Q.8 Q.10 Q.12
M a them a ti cs | 7.39

Q.13 Q.14
Answer Key

JEE Main/Boards
5π π
Q.15 2nπ + ,2nπ − ,n ∈ I
Exercise 1 12 12
nπ π 2nπ 3π
nπ n π 2nπ 2π Q.16 (i) − ,n ∈ I (ii) ± ,n ∈ I
Q.1 (i) + ( −1 ) ,,nn∈∈II (ii) ± ,,nn∈∈II 3 12 5 20
2 12 5 15
n π nπ
Q.17 (i) n π + ( −1 )  ±  (ii)
= x ,n ∈ I
π 2π  6 3
Q.2 2nπ ± ,2nπ ± , n∈I
3 3 nπ
(iii) nπ,nπ ± ,n ∈ I
nπ π 3
Q.3 + ,n ∈ I
3 9 π
Q.18 nπ + ,n ∈ I
4
nπ n π
Q.4 + ( −1 ) ,n ∈ I π
2 4 Q.19 nπ,nπ ± ,n ∈ I
3
1 n π π π π
Q.5=
x nπ + ( −1 ) + 2mπ ±  ,n,m ∈ I, Q.20(i) 2nπ ± + ,n ∈ I i.e.
2 3 3 4 4

1 π n π
π π π
2nπ or 2nπ + ,n ∈ I (ii) 2nπ ± + ,n ∈ I
=y 2mπ ± − nπ − ( −1 )  ,n,m ∈ I 2 3 6
2 3 3
7π 2π
Q.6 x = nπ,n ∈ I (iii) 2nπ + ,n ∈ I (iv) 2nπ,2nπ + ,n ∈ I
12 3
1 Q.21 (i) 30º (ii) 144º (iii) 68º 43’ 37.8”
Q.7 tan α =
2 π π π
Q.22 30º, 60º, 90º and , ,
Q.8 x =
=
2aπ
or x
( 2b + 1) π ,a,b ∈ I
6 3 2
m+n m−n Q.23 3466.36 km

nπ mπ π Q.24 17.4 cm
Q.9 =
x , − ,n ∈ I
2 2 8 3π n π
Q.25 x = nπ,nπ + ( −1 ) ,nπ + ( −1 )
n+1

π 2π 10 10
Q.10 x = nπ − ( −1 )
n
,2nπ ± ,n ∈ I
6 3 where n = 0, ±1, ±2,.......

π π π −π 3
Q.11 x = nπ + ( −1 )
n
or (2n+ 1) ,n ∈ I Q.26
= θ , , π − cos−1  
6 2 3 3 5
2
3 Q.27 x = nπ, x = nπ ± α where sin α =
Q.12 x = nπ + tan ( − ) or x = n π + tan−1 (2)
−1
5
4
π
nπ π Q.28 θ = nπ ±
Q.13 nπ or ± , where n ∈ I 6
3 9
π Q.30 NO real Solution
Q.14 nπ,nπ − , where n ∈ I
4
7 . 4 0 | Trigonometric Ratios, Identities and Equations

Exercise 2

Single Correct Choice Type

Q.1 A Q.2 B Q.3 D Q.4 D Q.5 D Q.6 A

Q.7 D Q.8 B Q.9 C Q.10 A Q.11 C Q.12 B

Q.13 A Q.14 C Q.15 B Q.16 C Q.17 C Q.18 B

Q.19 D Q.20 A

Previous Years’ Questions

Q.1 A Q.2 C Q.3 A Q.4 B Q.5 A Q.6 C

Q.7 D Q.8 A Q.9 B Q.10 B Q.11 C Q.12 B

Q.13 A Q.14 B

JEE Advanced/Boards Q.12 x = π / 16

Exercise 1 Q.13 x = n π +
π
;n ∈ I
12
π
Q.1 x = nπ or x = nπ ± Q.14 10 solutions
6
π n  3π  π
Q.2 nπ;nπ + ( −1 ) or nπ + ( −1 )  
n
Q.15 x= 2nπ +
10  10  12
Q.3 2 Q.16 30º, 45º, 90º, 135º, 150º
±π −π
Q.4 , , ±π Q.17 No Solution
3 2
π
π n n π Q.18 θ= 2nπ or 2nπ + ;n ∈ I
Q.5 x = 2nπ or x = nπ + ( −1 )  −  or x = nπ + ( −1 ) 2
 2 6
Q.19 4950 π
π 5π 9π 13π
Q.6 , ,π ,
7 7 7 7 Q.20 72º
Q.21 13
nπ π nπ π nπ 5π
Q.7 x = + , − , − π
2 8 2 24 2 24 Q.23 2nπ + or ( 2n + 1 ) π − tan−1 2;n ∈ I
4
π
Q.8 x= 2nπ −
2 3  1 nπ π
2 2
Q.24 (a) − ; (b) k ∈  −1, −  ; (c) =
x ±
 π  4mπ π  2  2 2 6
x  4nπ −  or=
Q.9 = x  + 
 2  3 2
Q.25 A → r; B → s; C → p; D → q
where m,n ∈ W
Q.26 A → s; B → p; C → q; D → r
Q.10
= x 0 or a < −1
Q.27 A → q; B → s; C → p; D → r
2
Q.11 α − 2π; α − π, α + π, where tan α =
3
M a them a ti cs | 7.41

Exercise 2

Single Correct Choice Type

Q.1 A Q.2 A Q.3 A Q.4 B Q.5 C Q.6 A

Q.7 C Q.8 B Q.9 D Q.10 C Q.11 C

Multiple Correct Choice Type

Q.12 B, D Q.13 A, B, C Q.14 B, C

Previous Years’ Questions

 π 2π  3 −1 π π
Q.2 ± , ±  Q.3 Q.4 30º Q.5 Q.6 θ = ± Q.7 A C
 3 3 2 2 2 3

Q.8 B, C Q.9 A, B Q.10 A, B Q.11 D Q.12 3 Q.13 7

Q.14 3 Q.15 3 Q.16 2 Q.17 D Q.18 7 Q.19 A, C, D

Q.20 D Q.21 8 Q.22 C

Solutions

JEE Main/Boards 2π
x = 2nπ ± ,n∈I
3
Exercise 1 2π
∴ 5θ = 2nπ ± ,n∈I
3
1
Sol 1: (i) sin 2 θ = 2n π 2 π
2 ⇒θ= ± ,n∈I
5 15
1
General solution of sinx = is
2 Sol 2: 7cos2θ + 3sin2θ = 4
π
x = nπ + (–1)n 6 , n ∈ I Since sin2θ + cos2θ = 1
π ∴ 4cos2θ + 3(cos2θ + sin2θ) = 4
∴ 2θ = nπ + (–1)n 6
⇒ 4cos2θ + 3 = 4
nπ π 1 1 1
⇒θ= + ( −1)n ,n∈I ∴ cos2θ = or cosθ = , −
2 12 4 2 2
1
(ii) cos 5 θ = – π 2π
2 ∴ θ = 2nπ ± , 2nπ ± ,n∈I
3 3
1
General solution of cosx = − is
2
7 . 4 2 | Trigonometric Ratios, Identities and Equations

Sol 3: tanx + tan2x + 3 tanx tan2x = 3 nπ n π


∴θ= + ( −1 ) ,n∈I
2 4
⇒ tanx + tan2x = 3 (1 – tanx tan2x)
tanx + tan2x 3 1
∴ 3
= Sol 5: sin(x – y) = , cos(x + y) =
1 − tan x tan 2 x 2 2
tanA + tanB π
∵ = tan(A + B) ∴ x – y = nπ + (–1)n n ∈ I. … (i)
1 − tanA tanB 3

Applying the above formula π


and x + y = 2mπ ± n ∈ I … (ii)
3
tan (x + 2x) = 3 ⇒ tan3x = 3 π π
∴ 2x = nπ + (–1)n + 2mπ ±
General solution of tanθ = 3 is 3 3
π [Adding (i) and (ii)]
θ = nπ + ,n∈I
3
1 n π π
π nπ π and x = nπ + ( −1) + 2mπ ± 
∴ 3x = nπ + ⇒x= + 2 3 3
3 3 9
Similarly,
Sol 4: 3tan(θ – 15°) = tan(θ + 15°)
1  π  3 π 
We can write it as =y 2n π ± −  n π + ( −1 )   , m,n ∈I
2  3  3 
 π 
tan  θ + 
tan ( θ + 15° )  12  [Subtracting (i) from (ii)]
= = 3
tan ( θ − 15° )  π 
tan  θ −  Sol 6: sinx = tanx
 12 
sinx sinx
sin θ ∵ tanx = ⇒sinx =
Applying tanθ = cos x cos x
cos θ
⇒ sin x cos x=sin x ⇒ sin x(cos x–1) = 0
 π   π 
sin  θ +  cos  θ −  ∴ sin x = 0 or cos x = 1 for sin x = 0
 12   12 
⇒ 3
= x = nπ, n ∈ I
 π   π 
sin  θ −  cos  θ + 
 12   12  and for cos x = 1, x = 2mπ, m ∈ I
1 As the equation is valid for sin x = 0 or cos x = 1, is the
Using sin A cos B = [sin(A + B) + sin(A – B)]
2 solution will be union of both.
Above expression can be written as ∴ x = nπ, n ∈ I
1  π π   π π 
sin  θ + + θ −  + sin  θ + − θ +  Sol 7: 2tanθ – cotθ + 1 = 0
2  12 12   12 12 
=3
1  π π   π π  1
cotθ =
sin  θ + + θ −  + sin  θ − − θ −  tanθ
2  12 12   12 12  
1
π ∴ 2tanθ – +1=0
sin2 θ + sin tanθ
⇒ 6 = 3
π 2 tan2 θ + tan θ − 1
sin2 θ − sin ⇒ 0
=
6 tan θ
π π (2 tan θ − 1)(tan θ + 1)
⇒ 2sin2θ = 4sin ⇒ sin2θ = 2 × sin =1 ⇒ =0
6 6 tan θ
∴ sin2θ = 1 1
⇒ tanθ ≠ 0 and tanθ = or tanθ = –1
General solution of sin x = 1 2
π ∴ From tanθ ≠ 0, θ ≠ nπ and
is x = nπ + (–1)n , n ∈ I
2 π
tanθ = –1, θ = nπ – n∈I
4
M a them a ti cs | 7.43

1 ∴ 1 + sinx = 2(1 – sin2x) = 2(1 – sinx) (1 + sinx)


∴ Solution of equation is tanθ = ,–1
2 ⇒ [2(1 – sinx) – 1] (1 + sinx) = 0
1 π
i.e. θ = nπ + tan 2 , nπ – n ∈ I
–1
4 ⇒ [(1 – 2sinx) (1 + sinx)] = 0
1
∴ sinx = or –1
Sol 8: sin mx + sin nx = 0 2
1 π
C +D  C −D  For sin x = x = nπ + (–1)n 6 n ∈ I
Qsin C + sin D = 2sin   cos  
 2   2  2
π
 x  x For sin x = –1 x = (2n + 1) n∈I
∴ 2sin  (m + n)  cos  (m − n)  =
0 2
 2  2 π π
∴ x = nπ + (–1)n or (2n + 1) n ∈ I
x  x 6 2
∴ sin(m + n) = 0 or cos  (m − n)  =0
2  2 
x x π Sol 12: 2 sin2x – 5sinx cosx – 8cos2x + 2 = 0
⇒(m + n) = aπ or (m – n) = (2b + 1) a,b ∈ I
2 2 2 We can write 2 as 2(cos2x + sin2x)

∴x=
2a
π or
(2b+ 1)
π a,b ∈ I ∵ cos2x + sin2x = 1
m+n m−n
∴ 2sin2x–5sinxcosx–8cos2x+2cos2x+2sin2x = 0

Sol 9: sec22x = 1 – tan2x ⇒ 4sin2x – 5sinx cosx – 6cos2x = 0

[1 + tan2θ = sec2q] 4sin2x – 8sinx cosx + 3sinx cosx – 6cos2x = 0

∴ 1 + tan22x = 1 – tan2x 4sinx(sinx – 2cosx) + 3cosx(sinx – 2cosx) = 0

⇒ tan22x + tan2x = 0 ∴ (4sinx + 3cosx) (sinx – 2cosx) = 0

⇒ tan2x(1 + tan2x) = 0 3
⇒ sinx = − cosx or sinx = 2cosx
4
tan2x = 0 or tan2x = –1
3
π or tanx = − or tanx = 2
2x = nπ or 2x = mπ – n, m ∈ I 4
4  3
nπ mπ π x = nπ + tan–1  −  or x = nπ + tan–1(2)
∴x= or − n, m ∈ I  4
2 2 8
Sol 13: 4sinx sin2x sin4x = sin3x
Sol 10: 4sinx cosx + 2sinx + 2cosx + 1 = 0
2sinA sinB = cos(A – B) – cos(A + B)
⇒ (2sinx + 1) × (2cosx + 1) = 0
2sinA cosB = sin(A + B) + sin(A – B)
1 1
⇒ sin x = − or cosx = − ∴ 2sinx[2sin2x sin4x] = 2sinx[cos2x – cos6x]
2 2
π 1 ⇒ 2sinx cos2x – 2sinx cos6x = sin3x
∴ x = nπ– (–1)n (when sin x = − )
6 2 ⇒ sin3x + sin(–x) – [sin7x + sin(–5x)] = sin3x
2π 1 ⇒–sinx = sin7x – sin5x
or x = 2nπ ± (when cos x = − )
3 2
–sinx = 2cos6x sinx
Sol 11: tanx + secx = 2cosx C +D   C −D 
sinC + sinD = 2sin   cos  
sinx 1  2   2 
tanx = , sec x =
cos x cos x ⇒ sinx(2cos6x + 1) = 0

1 + sin x 1
sinx = 0 or cos6x = −
∴ = 2 cos x 2
cos x
2π nπ π
⇒ 1 + sin x = 2cos2x x = nπ or 6x = 2nπ ± ⇒ x = nπ, ±
3 3 9
[∵ sin2x + cos2x = 1]
7 . 4 4 | Trigonometric Ratios, Identities and Equations

Sol 14: (1 – tanθ) (1 + sin2θ) = (tanθ + 1) 3π


θ = 2nπ ± n∈I
sin θ 4
tanθ =
cos θ 3π 2n π 3 π
∴ 5x = 2nπ ± ⇒x= ± ,n∈I
4 5 20
 cos θ − sin θ   cos θ + sin θ 
∴   (1 + sin2θ) =  
 cos θ   cos θ  Sol 17: (i) 3cos2θ + 7sin2θ = 4
cos θ + sin θ 3(cos2θ + sin2θ) + 4sin2θ = 4
⇒ (1 + sin2θ) =
cos θ − sin θ
⇒ 4sin2θ = 4 – 3 = 1
( cos θ + sin θ )
2
1 1 n π
= ∴ sinθ = , − ⇒ θ = n π + ( −1 )  ± 
( cos θ − sin θ )( cos θ + sin θ ) 2 2  6

cos2 θ + sin2 θ + 2sin θ cos θ (ii) tanx + tan2x + tan3x = tanx tan2x tan3x
∴ (1 + sin2θ) =
cos2 θ − sin2 θ ⇒ tanx + tan2x + tan3x (1 – tanx tan2x) = 0
1 + sin2θ tanx + tan2x
(1 + sin2θ) = ⇒ = –tan3x
cos2θ 1 − tanx tan2x
⇒ cos2θ (1 + sin2θ) – (1 + sin2θ) = 0 tanA + tanB
∵ = tan(A + B)
(cos2θ – 1) (sin2θ + 1) = 0 1 − tanA tanB
sin2θ = –1 or cos2θ = 1 ∴ tan(x + 2x) = –tan3x
π 2tan3x = 0
∴ 2θ = 2nπ – or 2θ = 2nπ, n ∈ I
2
i.e. tan3x = 0
π
∴ θ = nπ – or nπ n ∈ I nπ
4 ∵ 3x = nπ, n ∈ I or x = ,n∈I
3

Sol 15: sin x + 3 cos x = 2 π  π 


(iii) tan  + θ  + tan  − θ  = 4
4  4 
1 3 2
⇒ sinx + cosx = tanA ± tanB
2 2 2 tan (A ± B) =
1  tanA tanB
π π π
⇒ sin sin x + cos cos x =
cos
6 6 4 π  1 + tan θ
∴ tan  + θ  =
 π π 4  1 − tan θ
⇒ cos  x −  =cos
 6  4 π  1 − tan θ
and tan  − θ  =
π π 4  1 + tan θ
x − = 2n π ±
6 4
1 + tan θ 1 − tan θ
5π π ∴ + 4
=
∴ x = 2nπ + , 2nπ – n∈I 1 − tan θ 1 + tan θ
12 12
(1 + tan θ ) + (1 − tan θ )
2 2

Sol 16: (i) tan3θ = –1 ⇒ =4


General solution of tan x = –1 is
(1 − tan θ) 2

π ∴ (1 + tan2θ) × 2 = 4(1 – tan2θ) ⇒ 3tan2θ = 1


x = nπ – , n ∈ I
4 1 π
π nπ π tanθ = ± ⇒ θ = nπ ± ,n∈I
∴ 3θ = nπ – ⇒θ= − ,n∈I 3 6
4 3 12
1 Sol 18: tanx + cotx = 2
(ii) cos5x = −
2 1 1
1 cotx = ∴ tanx + =2
General solution for cos θ = − is tanx tanx
2
M a them a ti cs | 7.45

tan2x – 2tanx + 1 = 0 π π
⇒ x = 2nπ ± +
3 4
⇒ (tanx – 1)2 = 0 ⇒ tanx = 1
7x π
π 2nπ + or 2nπ –
∴ x = nπ + ,n∈I 12 12
4
(iv) cosx + 3 sinx = 1
Sol 19: tan3x – 3tanx = 0,
tanx (tan2x – 3) = 0 Divide the equation by 2

tanx = 0 or tanx = ± 3 1 3 1
cosx + sinx =
π 2 2 2
x = nπ or x = nπ ± ,n∈I
3  π π
⇒ cos  x −  = cos  
 3 3
Sol 20: (i) cosx + sinx = 1 π π
∴ x = 2nπ ± +
1 3 3
Multiply whole equation by
2 2π
i.e. x = 2nπ or 2nπ + ,n∈I
1 1 1 3
⇒ cosx + sinx =
2 2 2 π
C
Sol 21: (i)  
π π  π 1 6
cos cosx + sin sinx = cos  x −  =
4 4  4  2 180° = π radian
π π 0
∴ x – = 2nπ ± 180
4 4 ∴ 1 radian →
π
π π π π 180 π
∴ x = 2nπ ± + = 2nπ or 2nπ + ,n∈I → × = 30°
4 4 2 6 π 6
(ii) secx – tanx = 3  4π
C
(ii)  
1 tanx 1 − sinx  5 
⇒ sec x − 1 ⇒
= =1
3 3 3 cos x 4π 180 4 π
radian = × = 144°
5 π 5
⇒ 3 cosx + sinx = 1
(iii) (1.2)c
Divide the equation by 2
180
1.2 radian = × 1.2 = 68°43’37.8”
3 1 1 π
⇒ cos x + sinx =
2 2 2 Note: 1° = 60’, 1’ = 60”
 π π
cos  x −  = cos  
 6 3 Sol 22: a + b + c = 180°
π π B
∴ x = 2nπ ± + ,n∈I
3 6
b
π π
x = 2nπ – or 2nπ + , n ∈ I
6 2
1
(iii) sinx + cosx = a c
2 A C
1
Multiply the equation by Given angles are in A. P.
2
Let common difference = d
1 1 1
⇒ sinx + cosx = ∴ b = a + d, c = a + 2d
2 2 2
 π π ⇒ a + (a + d) + (a + 2d) = 3(a + d) = 180°
⇒ cos  x −  = cos  
 4  3
7 . 4 6 | Trigonometric Ratios, Identities and Equations

a + d = 60° … (i) i.e. x = nπ


π
Now if x ≠ nπ i.e. sinx ≠ 0 ⇒ 4cos2x = 3 + 2sinx
a a 60° 3 1
Also given = = = =
c a + 2d π π 3 ∴ 4 – 4sin2x = 3 + 2sinx ⇒ 4sin2x + 2sinx – 1 = 0

⇒ 3a = a + 2d ⇒ a = d … (ii) −2 ± (2)
2
− 4 ( 4 )( −1 ) −2 ± 20
sinx= =
From (i) and (ii) 2× 4 8

a = d = 30° −1 ± 5
⇒ sinx =
∴ a = 30°, b = 60°, c = 90° Ans. 4
∴ Possible solution is
Sol 23: 5 −1  π  π
sinx = = sin   ⇒ x = nπ + (–1)n 10
4  10 
A
−1 − 5  3π   3π 
O and sinx = = sin  −  = sin  π + 
d 31 man 4  10   10 
C
 3π  3π
B ∴ x = nπ + (–1)n  −  = nπ + (–1)n+1 10
 10 
moon 384400 km
Earth
π 3π
Line OC divides AB into two equal parts ∴ x = nπ, nπ + (–1)n 10 , nπ + (–1)n+1 10

In DOBC
θ
 31'  OB Sol 26: 5cos2θ + 2cos2 2 + 1 = 0 − π < θ < n
tan  = ∴ OB = 384400 × tan(15.5°)
 2  OC cos2θ = 2cos2θ – 1
0
15.5
= 384400 × tan = 173318 Km θ
60 ∴ 2cos2 = cos θ + 1
2
AB = 2(OB) = 346636 Km and cos2θ = 2cos2θ + 1
∴ Diameter of moon = 346636 Km. Putting both these in given equation
∴ 5(2cos2θ – 1) + cosθ + 1 + 1 = 0
Sol 24:
A 10cos2θ + cosθ – 3 = 0
10cos2θ – 5cosθ + 6cosθ – 3 = 0
O
Letter B 5’ man
 (5cosθ + 3) (2cosθ – 1) = 0
3 1
C ⇒ cosθ = − or cosθ =
5 2
12 m
 3 π 3
Assuming letter to be symmetrically placed ∴ θ = cos–1  − 5  or θ = ± = π – cos–1 5
  3
BC
tanθ = 3
OB π π
∴θ= , – , π – cos–1 5 (As – π < θ < π)
BC 3 3
tan(2.5’) =
12
Sol 27: 4sin4x + cos4x = 1
∴ BC = 12tan (2.5’) = 0.0873 m
4sin4x = 1 – cos4x = (1 – cos2x) (1 + cos2x)
∴ Total length of letter = 2BC = 0. 174 m = 17.4 m.
4sin4x = sin2x(1 + cos2x)
Sol 25: 4cos2x sinx – 2sin2x = 3sinx One of obvious solution is sinx = 0 i.e. x = np
One of the obvious solution is sinx = 0 If sinx ≠ 0
M a them a ti cs | 7.47

⇒ 4sin2x = 1 + cos2x = 2 – sin2x Exercise 2


2
⇒ sin2x =
5 Single Correct Choice Type

2
sinx = ± A B 3
5 Sol 1: (A) b cos2 + a cos2 = c
2 2 2
2
∴ x = nπ ± sin–1 5
b
( cos A + 1) + a ( cosB+ 1) =
3
c
2 2 2
2
∴ x = nπ, nπ ± α,sinα = ,n∈I bcosA + b + acosB + a = 3c
5
⇒ a + b + (acosB + bcosA) = 3c
Sol 28: tan2θ tanθ = 1 ⇒ acosB + bcosA = c
2 tan θ
tan2θ = , ∴ a + b = 2c ⇒ a,b,c are in A.P.
1 − tan2 θ
Substitute this in given equation, we get π 
Sol 2: (B) a2 + 2a + cosec2  2 ( a + x )  =
0
 
2 tan2 θ [cosec2θ = 1 + cot2q]
∴ = 1 ⇒ 3 tan2 θ = 1
1 − tan2 θ π 
⇒ a2 + 2a +1 + cot2  2 ( a + x )  =
0
1 π  
tanθ = ± ∴ θ = nπ ±
3 6 π 
⇒ (a + 1)2 + cot2  2 ( a + x )  =
0
 
Sol 29: esinx – e–sinx – 4 = 0
1 For the above equation to be valid
esinx – =4
esin x π 
a + 1 = 0 and cot  ( a + x )  =
0
Max. Value of L. H. S. can be attained only when esinx is 2 
π π
max and
1
is min. ⇒ a = –1 and ( a + x ) = ( 2n + 1 )
sin x 2 2
e
⇒ a + x = 2n + 1 ⇒ x = 2n + 2
As max. Value of sinx is 1
x
1 1 ∴ a = –1 and ∈I
∴ esinx ≤ e1 and ≥ 2
esin x e
1 C C
Sol 3: (D) ( a + b ) sin2+ ( a − b ) cos2
2 2
∴Max. Value of LHS = e – ≈ 2 35
e 2 2
So there is no real value of x for which LHS = 4 C C
= (a2 + b2 + 2ab) sin2 + (a2 + b2 – 2ab) cos2
2 2
Sol 30: sin4θ – 2sin2θ – 1 = 0 π  c c
= (a2 + b2) + 2ab  sin2 − cos2 
Let sin2θ = t ⇒ t2 – 2t – 1 = 0 3  2 2
= a2 + b2 + 2ab(–cos C)
2
2 ± ( −2) + 4 × 1 2± 8 a2 + b2 − c2
∴t= = =1± 2 2 = a2 + b2 – 2ab = c2
2 2 2ab
∴ sin2θ = 1 + 2 or 1 – 2
Sol 4: (D) sin3A + sin3B + sin3C = 3sinA sinB sinC
Since –1 < sinθ < 1 and 0 < sin2θ < 1
a b c
∴ No real solution. ∵ In triangle = = = k
sin A sinB sinC

i.e. sinA ∝ a, sinB ∝ b, sinC ∝ c


∴ a3 + b3 + c3 = 3abc
7 . 4 8 | Trigonometric Ratios, Identities and Equations

a3 + b3 + c3 – 3abc = (a+b+c)(a2+b2+c2–ab–bc–ca)=0  β  β
2 1 − tan2  − 1 − tan2 
∴ a2 + b2 + c2 – ab – bc – ca = 0 2  2
⇒ = 
⇒ (a – b)2 + (b – c)2 + (c – a)2 = 0 ⇒ a = b = c β  β 
2(1 + tan2 ) − 1 + tan2 
2  2
∴ Triangle should be equilateral
β β
2 − 2 tan2− 1 − tan2
Sol 5: (D) sin3θ = 4sinθ sin2θ sin4θ θ ∈ [0, p] ⇒ = 2 2
2 β β
sin3θ = 2sinθ [cos2θ – cos6q] 2 + 2 tan − 1 + 2 tan2
2 2
sin3θ = sin3θ + sin(–θ) – [sin7θ + sin(–5θ)] α
1 − tan2
⇒ sin7θ – sin5θ = –sinq 1 − 3tan2 β / 2 2
⇒ =
2
1 + 3 tan β / 2 1 + tan2 α
∴ –sinθ = 2cos6θ sinq
2
−1
sinθ [2cos6θ + 1] = 0 ⇒ sinθ = 0 or cos6θ =
2  β  β
2π nπ π ⇒ 1 − 3 tan2  1 + tan2 
θ = nπ or 6θ = 2nπ ± ⇒θ= ±  2  2
3 3 9
 2 β  2 β
π 4π 7π 2π 5π 8π = 1 − tan  1 + 3 tan 
∴ θ=0, π, , , , , ,  2  2
9 9 9 9 9 9
β α α β
Sol 6: (A) acos(B–C)+bcos(C – A) + ccos(A – B) ⇒ 1 − 3tan2 + tan2 − 3tan2 tan2
2 2 2 2
= a(cosB cosC + sinB sinC) + b(cosC cosA + sinC sinA) β α α β
1 + 3 tan2
= − tan2 =− 3tan2 tan2
+ c(cosA cosB + sinA sinB) 2 2 2 2
α β
= cosC[acosB + bcosA] + c cosA cosB ⇒ 2 tan2 6 tan2
=
2 2
+ a sinB sinC + b sinC sinA + c sinA sinB
α β
⇒ tan2 3 tan2
=
a = 2RsinA, b = 2RsinB, c = 2RsinC 2 2
and acosB + bcosA = c α β
⇒ tan2 3 tan2
=
abc abc abc 2 2
= c cosC + c cosA cosB + + +
( 2R ) ( 2R ) ( 2R )
2 2 2
α β
⇒ tan2 ⋅ cot =3
3abc 2 2
= c[cos(π – (A + B)) + cosAcosB] +
4R 2
3abc  2π   4π
= c[–cosAcosB + sinAsinB + cosAcosB] + Sol 8: (B) x sinθ = y sin  θ +  =z sin θ + 
4R 2  3   3 
3abc abc 3abc abc 
= c sinAsinB + = + =  1 3
4R 2
4R 2
4R 2
R2 sin θ  −  + cos θ  + 
x  2  2 
⇒ =   = − 1 + 3 cot θ
2 cos β − 1 y sin θ 2 2
Sol 7: (D) cos α =
2 − cos β  1  3
sin θ  −  + cos θ  − 
 2 β x  2  2  1 3
1 − tan  =  = − − cot θ
2
2 −1 z sin θ 2 2
β
1 + tan2 x x
⇒ = 2 ∴ + =−1 ⇒ xz + xy + yz = 0
 2 β y z
1 − tan 
2
2− 
β Sol 9: (C) m = acos3α + 3acosα sin2a
1 + tan2
2
n = asin3α + 3acos2α sina
M a them a ti cs | 7.49

(m+n)=a(sin3α+cos3α+3cosα sinα(cosα+sinα)) B C s
∴ cot cot =
2 2 s − a
= a(sinα + cosα)3

(m–n)=a(cos3α–sin3α+3cosα sinα (sinα – cosα)) (a + b + c) ( a + 3a)


2 2 2a
= or =
= a(cosα – sinα) 3
(a + b + c) −a
 a + 3a 
 −a
2a − a

∴ (m + n)2/3 + (m – n)2/3 2  2 
B C
= a2/3(sinα + cosα)2 + a2/3(cosα – sinα)2 = 2a2/3 ∴ cot cot = 2
2 2

Sol 10: (A) tan(5pcosθ) = cot(5psinθ)


Sol 12: (B) cos2x + asinx = 2a – 7
sin(5π cos θ) cos(5π sin θ)
= 1 – 2sin2x + asinx = 2a – 7
cos(5π cos θ) sin(5π sin θ)
2sin2x – asinx + 2a – 8 = 0
⇒ cos(5pcosθ) cos(5psinθ) – sin(5pcosθ) sin(5psinθ) = 0
⇒ cos(5pcosθ + 5psinθ) = 0
a ± a2 − 4(2a − 8) × 2
sinx =
2×2
−2 ± 4 + 4 × 4 1± 5
⇒ 5π(cosθ + sinθ) = 2nπ ± = −
(a − 8)
2
2× 4 4 a± a ± | a− 8 |
 2n 1  ∴ sinx = =
⇒ cosθ + sinθ =  ±  4 4
 5 10  For a > 8
1 1 1  2n 1  a ± | a− 8 | 2a − 8
⇒ cos θ + sin θ =  ±  sinx = ⇒ sinx = or 2
2 2 2  5 10  4 4
Since sinx ≤ 1 no value of a > 8 satisfies the equation
 π 1  2n 1 
sin  θ +  =  ± n ∈ I For a < 8
 4 2  5 10 
a ± ( −a + 8 )
For this relation to satisfy sinx =
4
1  2n 1  2a − 8
 ±  ∈  −1,1 sinx = 2,
2  5 10  4
2n 1 2a − 8
∴− 2 ≤ + ≤ 2 ∴ –1 ≤ ≤1
5 10 4
2n 1 2≤a≤6
and − 2 ≤ − ≤ 2
5 10 ∴ The solution is a ∈ [2, 6]
From following both condition
A B
 1 5  1 5
− 2 −  ≤ n ≤  2 − 
∑ cot2 2 cot2 2 1 1 1
 10  2  10  2 Sol 13: (A) = + +
A C B C
 1 5  1 5
∏ cot2 2 cot2
2
cot2
2
cot2
2
and  − 2 ≤ n ≤  2 + 
 10 2  10  2 A B C
= tan2 + tan2 + tan2
Considering values of n and θ ∈ [0, 2p] total of 28 values 2 2 2
are possible. 2
 A B C A B
=  tan + tan + tan  – 2 ∑ tan + tan
B C s−a  2 2 2 2 2
Sol 11: (C) tan tan =
2 2 s A B
For triangle ∑ tan + tan = 1
2s = a + b + c 2 2
Given b + c = 3a  A B C
2
=  tan + tan + tan  − 2
 2 2 2
7 . 5 0 | Trigonometric Ratios, Identities and Equations

2 Sol 17: (C) (7cosθ + 24sinθ) × (7sinθ – 24cosθ)


∴ For LHS to be minimum  ∑ tan A  should take its
 2  7 cos θ + 24 sin θ   7 sin θ − 24 cos θ 
minimum value which is only possible for A = B = C =
π = 25   × 25  
3  25   25 
2 2
 A B C  1  = –625 sin(α + θ) cos(α + θ)
∴  tan + tan + tan  =  × 3 = 3
 2 2 2  3  24
7
Where sinα = , cosα =
∴ Min. Value of given function = 3 – 2 = 1 25 24
625
= − sin 2 ( α + θ )
2
Sol 14: (C) sinx + sin5x = sin2x + sin4x
∴ Maximum value occurs when
2sin3x cos2x = 2sin3x cosx
625
2sin3x(cos2x – cosx) = 0 sin2(α + θ) = –1 ∴ Maximum value =
2
∴ 3x = nπ or cos2x = cosx
x x
Sol 18: (B) sin – cos = 1 – sinx
nπ 2n π 2 2
x= or 2x = 2nπ ± x ⇒ x = , 2np 2
3 3  x x
We know that 1 – sinx =  sin − cos 
π 2π 4π  2 2
∴x= , , π, .........
3 3 3 2
x x  x x
nπ ⇒ sin – cos =  sin − cos 
∴General solution is 2 2  2 2
3
x x 1
= cos or sin  x − π  =
 
Sol 15: (B) cotx – cosx = 1 – cotx cosx ⇒ sin
2 2 2 4 2
 1 − sinx  sinx − cos2 x x π x π π
cosx   = ⇒ = mπ + or − = mπ + ( −1)m
 sinx  sinx 2 4 2 4 4
For sinx ≠ 0 i.e. x ≠ np π π π
⇒ x = (4m+ 1) or x = (4m + 1) + ( −1)m
cosx – cosx sinx = sinx – cos x2 2 2 2

(cosx – sinx) + cosx(cosx – sinx) = 0 x π 3π


Also − ≤
⇒(1 + cosx) (cosx – sinx) = 0 2 2 4

cosx = –1 or cosx = sinx 3π x π 3π 3π π x 3π π


⇒− ≤ − ≤ ⇒ − + ≤ ≤ +
x 3π 4 2 2 4 4 2 2 4 2
∴ x = π, ,
4 4 π x 5π π 5π
⇒ − ≤ ≤ ⇒ − ≤x≤
Qn ≠ π as sinx ≠ 0 4 2 4 2 2
x 3π π 3π 4 π 5π
∴x= , ∴x= , π, , ,
4 4 2 2 2 2
∴ Two solutions
i.e. n = 1, 2, 4, 5
Sol 16: (C) cos 73 + cos 47 + (cos73 cos47)
2 2

Sol 19: (D) a1 + a2cos2x + a3sin2x = 0


= (cos73 + cos47)2 – cos47 cos73
2 a1 + a2(1 – 2sin2x) + a3sin2x = 0
 120 26   cos120 + cos26 
⇒  2cos cos  −   ⇒ a1 + a2 + (a3 –a2)sin2x = 0
 2 2   2 
For all x of this function has to be zero
1 1 
= cos213–  − + 2cos2 13 − 1 
2 2  then a1 + a2 = 0, a3 – 2a2 = 0

1 1 1 1 3 As there are three Variables and two equations so


= cos213 + – cos213 + = + = infinite solution are possible.
4 2 4 2 4
M a them a ti cs | 7.51

 π A  B Sol 4: (B) We know


Sol 20: (A) Given A + B =1 + tan   1 + tan 
 2 2 2
 π  – a2 + b2 ≤ asinx + bcos x ≤ α2 + b2
  − A 
 A  B = 
1 + tan
A 
1 + tan 2 
 1 + tan   1 + tan     ∴ − 74 ≤ 7cos x + 5sinx ≤ 74
2 2 2  2
  
 π    i.e. ∴ − 74 ≤ 2k + 1 ≤ 74
  − A 
 A  2 
=  1 + tan   1 + 1tan A  Since, k is integer, −9 < 2k + 1 < 9
 2
A  
− tan 2
2=
 
 1 + tan  A  ( 2 )
=  1 + tan   1 + = 2 ⇒ −10 < 2k < 8 ⇒ −5 < k < 4
 2   A   2 A
 1 + tan  1 + tan
 2 2 ⇒ Number of possible integer values of k is equal to 8.

Sol 5: (A) Since, 2sin2 θ − 5sin θ + 2 > 0


Previous Years’ Questions
⇒ ( 2sin θ − 1 )( sin θ − 2 ) > 0
Sol 1: (A) Given equation is
 Where, ( sin θ − 2 ) < 0 for all θ ∈ R 
 
x π
2cos2   sin2 =
x x2 + x −2 , x ≤
2
  9 ⇒ ( 2sin θ − 1 ) < 0
x 1
LHS = 2cos2   sin2 x < 2 and RHS = x2 + ≥2 y
2
  x2
∴ The equation has no real solution.
y= 1
2
( x ) tanx − x
Sol 2: (C) Let f= x’  5  2
x
π 6 6
We know, for 0 < x < y’
2
1
⇒ tanx > x ⇒ sin θ <
2
∴ f(x) = tan x – x has no root in ( 0, π / 2 )  π   5π 
∴ From the graph, θ ∈  0,  ∪  ,2π 
For π / 2 < x < π , tan x is negative  6  6 
∴ f (x) = tan x – x < 0
2 2 1
π  Sol 6: (C) 2sin θ − cos2θ = 0 ⇒ sin θ =
So, f(x) = 0 has no root in  , π  4
2 
1
3π Also, 2cos2=
θ 3sin θ ⇒ sin θ =
For < x < 2π,tanx is negative 2
2
⇒ Two solutions exist in the interval 0,2π  .
∴ f ( x=
) tanx − x < 0
 3π
So, f ( x ) = 0 has no root in  ,2π 
 Sol 7: (D) P ={θ : sin θ − cos θ = }
2 cos θ

 2  ⇒ cos θ ( 2 + 1 ) = sin θ
 3π  3π 3π
We have, f ( π ) = 0 −π < 0 and f  =  tan − >0
 2 2 2 ⇒ tan=
θ 2 +1 …… (i)

∴ f (x) =
0 has at least one root between π and
2
Q= {θ : sin θ + cos θ = } ⇒ sin θ (
2 sin θ )
2 − 1 = cos θ

( )
Sol 3: (A) Given equation is sin ex= 5x + 5− x is tan=
θ
1
2 −1
×
2 +1
=
2 +1
( 2 + 1) ….. (ii)

LHS = sin e ( )
x x
≤ 1 , for all x ∈ R and RHS = 5 + 5 −x
≥2 ∴P =Q

( )
∴ sin ex= 5x + 5− x has no solution.
7 . 5 2 | Trigonometric Ratios, Identities and Equations

Sol 8: (A) α + β + γ = 2 π = 1 − 2sin2 α cos2 α + 2 cos2 α sin2 α

tan ( α + β + γ ) = 0 =1

π tan α + tan β
tan ( α + β ) + tan γ Sol 11: (C) α + β= ⇒ tan ( α + β )= = 0
⇒ 0
= 2 1 − tan α tan β
1 − tan ( α + β ) tan γ
γ=α−β

tan α + tan β ⇒ tan α tan β =1


⇒ + tan γ =0
1 − tan α tan β γ=α−β
⇒ tan α + tan β + tan=
γ tan α tan β tan γ tan α − tan β
tan γ =
1 + tan α tan β
Sol 9: (B) A sin2 θ + cos4 θ
tan α − tan β
⇒ tan γ =
( ) 2
2
= sin2 θ + 1 − sin2 θ
⇒ 2 tan=
γ tan α − tan β
2 4 2
= sin θ + sin θ + 1 − 2 sin θ ⇒ tana = 2tang + tanb

= 1 + sin4 θ − sin2 θ
 π
2 Sol 12: (B) θ ←  0, 
 1 1  4
=  sin2 θ −  + 1 −
 2  4
⇒ tan θ∈ (1, 0 )
2
 1 3
=  sin2 θ −  +  1 
 2  4 ⇒ cos θ∈  , 1
 2 
1
For minimum, sin θ =
2 ⇒ cot θ∈ (1, 0 )
3
A=  π
4 As θ∈  0, 
 4
For maximum, sin θ =0
cot θ > tan θ
A=1
⇒ ( cot θ ) > ( tan θ )
cot θ cot θ
3
⇒ ≤ A ≤1
4
⇒ t 4 > t2
  3π  
Sol 10: (B) 3 sin4  − α  + sin4 ( 3 π + α ) 
  2   tan A cot A
Sol 13: (A) +
 6π   1 − cot A 1 − tan A
−2 sin  + 2  + sin6 (5 π − 2 ) 
 2   tan A 1 / tan A
= +
1 1 − tan A
= 3 cos2 α + sin4 α  − 2 cos6 α + sin6 α  1−
    tan A

= 3cos4 α + 3sin4 α − 2cos4 tan2 A 1


= +
( )
α 1 − sin2 α − 2sin4 α 1 − cos2 α ( ) 1 − tan A tan A (1 − tan A )

− tan3 A + 1
= sin4 α + cos4 α + 2cos4 α sin2 α + 2sin4 α cos2 α =
(1 − tan A ) tan A
(sin α + cos α ) − 2sin α cos α
2
2 2 2 2
=
(1 − tan A ) (1 + tan2 A + tan A )
+2cos α sin α ( cos α + sin α )
2 2 2 2 =
tanA (1 − tan A )
M a them a ti cs | 7.53

sec2 A + tanA 2π π
= ⇒ sin4x = ⇒ sinx = ±
tan A 3 3
π
sec2 A ∴ x = nπ ± , np
= +1 6
tan A
= sec A cosec A + 1 Sol 2: 4cos2x sinx – 2sin2x = 3sinx
One above solution is sinx = 0 i.e. x = np
Sol 14: (B) cos x + cos 2x + cos 3x + cos 4x = 0
If sinx ≠ 0
⇒ cos x + cos3x + cos2x + cos 4x =
0
4cos2x = 3 + 2sinx
⇒ 2cos 2x cos x + 2cos 3x cos x =
0 4(1 – sin2x) = 3 + 2sinx
⇒ 2cos x ( cos 2x + cos 3x ) =
0 4sin2x + 2sinx – 1 = 0

 5x x −2 ± 4 + 4 × 4 1 ± 5
⇒ 2cos x 2 cos cos  =
0 =sinx = =
 2 2 2×4 4
x 5x 3π π
⇒ cos= 0 or cos
= x 0 or cos= 0 x = nπ + (–1)n+1 or nπ + (–1)n
2 2 10 10
x π π 5x π

2
( 2p + 1 ) or x =
2
( 2q + 1 ) or
2 2
=( 2r + 1 )
2 Sol 3: 1 + cosx+cos2x+sinx+sin2x+sin3x = 0
π π
Given condition < 3x − ≤ π
⇒ x= ( 2p + 1) π or x= ( 2q + 1) 2π or x= ( 2r + 1) 5π 2 2
1+cosx + 2cos2x – 1 +(sinx + sin3x)+ sin2x = 0
π 3π 7 π 9 π π 3π
=⇒x , , , , , π,
5 5 5 5 2 2 cosx(2cosx + 1) + 2sin2x cosx + sin2x = 0

Total 7 solutions (cosx + sin2x) (2cosx + 1) = 0


(2sinx + 1) (2cosx + 1) cosx = 0
1 1
JEE Advanced/Boards sinx = − or cosx = − or cosx = 0
2 2
Exercise 1 According to given condition
π π
< 3x − ≤ π
Sol 1: sin5x = 16sin x 5
2 2
sin(3x + 2x) = sin3x cos2x + cos3x sin2x π π π π
⇒ < 3x − ≤ π and −π ≤ 3x − < −
= (3sinx – 4sin3x) (1 – 2sin2x) 2 2 2 2

+ (–3cosx + 4cos3x)2sinx cosx 3π π


⇒ π < 3x ≤ and − ≤ 3x < 0
2 2
= sinx[(3 – 4sin2x) (1 – 2sin2x)
 π  π π 
+ 2(3 – 4(1 – sin2x)) (1 – sin2x)] = 16sin5x ∴ common condition is x ∈  − , 0  ∪  , 
 6  3 2 
One of the obvious solution is Only two solutions are possible
sinx = 0 ⇒ x = np 1
sinx = − and cosx = 0
If sinx ≠ 0 2

⇒ 3 – 10sin2x + 8sin4x Sol 4: 2(cosx + cos2x) + sin2x (1 + 2cosx) = 2sinx


+ 2(– 1 + 4sin x)(1 – sin x) = 16sin x
2 2 4
2(cosx + 2cos2x – 1) + sin2x (1 + 2cosx) = 2sinx
⇒ 3 – 10sin2x + 8sin4x + 10sin2x – 8sin4x – 2 = 16sin4x 2cosx(2cosx + 1) – 2 + sin2x(1 + 2cosx) = 2sinx
⇒ 1 = 16sin4x 2(cosx + sin2x) (2cosx + 1) – 2(1 + sinx) = 0
7 . 5 4 | Trigonometric Ratios, Identities and Equations

2cosx(1 + sinx) (2cosx + 1) – 2(1 + sinx) = 0 Sol 7: tan22x+cot22x+2tan2x+2cot2x = 6


(1 + sinx) [cosx(2cosx + 1) – 1] = 0 Let tan2x = t
⇒ sinx + 1 = 0 or 2cos2x + cosx – 1 = 0 1 2
⇒ t2 + + 2t + =6
i.e. (2cosx – 1) (cosx + 1) = 0 t 2 t
⇒ t4 + 1 + 2(t2 + 1)t = 6t2
1
∴ sinx = –1 or cosx = or cosx = –1
2 ⇒ t4 + 2t3 – 6t2 + 2t + 1 = 0
π
x = (4n – 1) ⇒ (t – 1) (t – 1) (t2 + 4t + 1) = 0
2
π − 4 ± 16 − 4
x = 2nπ ± or x = (2n + 1) p ⇒ tan2x = 1 or = −2 ± 3
3 2
∵ x ∈ [–π, p] ∴ 2x = nπ +
π
or 2x = nπ –
π
or nπ –

4 12 12
π π π
∴x= − , − , ; –π, +p nπ π nπ π nπ 5π
2 3 3 ⇒x= + , − , −
2 8 2 12 2 12
Sol 5: 2(sinx – cos2x) – sin2x(1 + 2sinx) + 2cosx = 0 x
− sec2
2(sinx+2sin2x) – 2 – sin2x(1 + 2sinx)+ 2cosx = 0 Sol 8: 1 + 2cosecx = 2
2
(2sinx – sin2x) (1 + 2sinx) – 2(1 – cosx) = 0
2 1 1
2sinx(1 – cosx) (1 + 2sinx) – 2(1 – cosx) = 0 1+ =− ×
sin x 2 x
cos2
2(1 – cosx) [sinx(1 + 2sinx) – 1] = 0 2
⇒ cosx = 1or 2sin2x + sinx – 1 = 0 2 + sinx 1
⇒ = −
1 x x x
2sin cos 2cos2
(2sinx – 1) (sinx + 1) = 0 ⇒ sinx = or –1 2 2 2
2
π π x x x
x = 2nπ or x = nπ + (–1)n 6 or x = (4n – 1) cos ≠0⇒ ≠ 2nπ ± ⇒ x ≠ 4nπ ± p
2 2 2 2
π π x
⇒ x = 2nπ, nπ + (–1)n+1 2 , nπ + (–1)n 6 ⇒ 2 + sinx = – tan
2
x
2 tan
3x x ⇒2+ 2 = − tan x (Let tan x = t)
Sol 6: cos3x + cos2x = sin + sin
2 2 x 2 2
1 + tan2
5x x x 2
⇒ 2cos cos = 2sinx cos 2t
2 2 2 ⇒2+ = − t ⇒ 2 + 2t2 + 2t = –t – t3
1 + t2
x  5x 
⇒ cos  cos − sinx  = 0
2 2  ⇒ t3 + 2t2 + 3t + 2 = 0
x 5x π  ⇒ (t + 1) (t2 + t + 2) = 0
cos = 0 or cos = sinx = cos  − x 
2 2 2  ⇒ t = – 1 or t2 + t + 2 = 0
x π 5x π  x
⇒ = 2nπ ± or = 2nπ ±  − x  ∴ tan = −1
2 2 2 2  2
x π π
π π π π = nπ – ⇒ x = 2nπ –
⇒ x =4nπ ± π or x = 4n + or 4n − n ∈ I 2 4 2
7 7 3 3
∵ x ∈ (0, 2π)  x  x
π 5π 9π 13π Sol 9: sin   + cos  =2 sin x
∴ x = π, , , ,  2   2 
   
7 7 7 7
1  x 1  x π 
sin  + cos   = cos  − x 
2  2  2  2  2 
   
M a them a ti cs | 7.55

 x π π  π 
⇒ cos  − =  cos  − x  ⇒ cos  − 2x  = cos6x
 2 4   2  2 
 
π
⇒ − 2x= 2nπ ± 6x
2
x π π 
∴ − = 2n π ±  − x  nπ π nπ π
2 4 2  ⇒x= + , −
4 16 2 8
4n π π π π
⇒ x= + , - 4nπ + ∴ Smallest positive value =
3 2 2 16
2 2
 4nπ π   π  π
∴x=  +  ,  4nπ −  n ∈ I Sol 13: 2sin  3x +  =
 3 2  2 1 + 8 sin2x cos2 2x
 4

 1 1 
Sol 10: As the roots are same both equations should ⇒ 2 sin3x + cos3x 
be same.  2 2 
Let us solve the second equation
= 1 + 4(2sin2x cos2x)cos2x
1
sinx cos2x = sin2x cos3x – sin5x
2 ⇒ [ 2 (sin3x + cos3x)]2 = 1 + 4sin4x cos2x
⇒ 2sinx cos2x=2sin2x cos3x – sin5x ⇒ 2(sin23x + cos23x + 2sin3x cos3x)
⇒ sin3x – sinx = sin5x – sinx – sin5x = 1 + 4sin4x cos2x
⇒ sin3x = 0 ∴ x = np ⇒ 2(1 + 2sin3x cos3x) = 1 + 4sin4x cos2x
Put x = nπ is given equation ⇒ 1 = 2(2sin4x cos2x) – 2(2sin3x cos3x)
a cos2x + |a| cos4x + cos6x = 1 = 2(sin6x + sin2x) – 2sin6x
a cos2nπ + |a| cos4nπ + cos6nπ = 1 1
⇒ sin2x =
⇒ a + |a| + 1 = 1 2
 π π
∴a≤0 ∴  2x −  = 2nπ ±
 2 3
5π π
Sol 11: 13 − 18 tan x = 6tanx – 3, –2π , x < 2p x = nπ + , nπ + ,n∈I
12 12
⇒ 13 – 18tanx = (6 tanx – 3)2 5π  π
If x = ⇒ sin  3x +  = –1
13 – 18tanx = 36tan2x + 9 – 36tanx 12  4
18 tan2x – 9tanx – 2 = 0 Which is not possible

⇒ (6tanx + 1) (3tanx – 2) = 0 π
∴ x = nπ +
12
1
Also 6tanx – 3 > 0 ⇒ tan x >
2
2 Sol 14: sinx – sin3x + sin5x = cosx – cos3x + cos5x
∴ tanx =
3 1 1
2 (sinx – cosx) – (sin3x – cos3x)
x = a – 2π, a – π, a, a + π, where a = tan–1 3 2 2
1
+ (sin5x – cos5x) = 0
Sol 12: ( 1 + sin 2x − 2 cos 3x =
0 ) 2

1 + sin2x = 2cos 3x = 2 (1 – sin 3x)


2 2  π  π  π
sin  x −  – sin  3x −  + sin  5x −  = 0
 4   4   4
⇒ 2sin23x + sin2x = 1
 π
⇒ 1 – cos6x + sin2x = 1  6x − 
2  x − 5x   π
⇒ 2sin   cos   − sin  3x −  =
0
⇒ sin2x = cos6x  2   2   4
 
 
7 . 5 6 | Trigonometric Ratios, Identities and Equations

 π Sol 17: log (sin3x + sinx) =


log 2 (sin2x)
⇒ sin  3x −  [2cos2x – 1] = 0 − x2 −6x − x −6x
 4 10 10

 π 1 ⇒ –x2 – 6x > 0 and –x2 – 6x ≠ 10


∴ sin  3x −  = 0 or cos2x =
 4  2 ⇒x(x + 6) < 0 and x2 + 6x + 10 ≠ 0
π π ∴ x ∈ (–6, 0)
⇒ 3x = nπ + or 2x = 2nπ ±
4 3
But if x is negative then sin2x will be negative.
nπ π π
∴x= + , nπ ± Hence no solution is possible.
3 12 6
π π 9 π 13 π
∴ Principle solution are , , , , Sol 18: 3 – 2cosθ – 4sinθ – cos2θ + sin2θ=0
12 12 12 12
3 – 2cosθ – 4sinθ – (1 – 2sin2θ) + 2sinθ cosθ = 0
17 π 21 π π 5 π 7π 11 π
, , , , , ⇒ 2cosθ(sinθ–1)+2sinθ (sinθ–1)–2(sinθ – 1) = 0
12 12 6 6 6 6
⇒ (sinθ – 1) (2cosθ + 2sinθ – 2) = 0
i.e. 10 solutions.
∴ sinθ = 1 or sinθ + cosθ = 1
1 
 +log3( cos x + sin x ) 
2 log2 ( cos x −sin x ) π  π π
Sol 15: 3  −2 2
= ⇒ θ = nπ + (–1)n 2 or cos  θ −  = cos  
 4 4
1
π π
∵ θ = 2nπ ±
log (cosx + sinx) log2 (cosx −sinx)
32 3 3 −2 2
= +
4 4
3 (cosx + sinx) – (cosx – sinx) = 2 π
∴ θ = 2nπ, 2nπ +
2
3 −1 3 +1 1
⇒ cos x + sinx =
2 2 2 2 2 Sol 19: cos4x + 6 = 7cos2x
cos4x – cos2x = 6(cos2x – 1)
 5π  π
cos  x −  = 2nπ ±
 12  3 ⇒ 2sin3x sin(–x) = 6(–2sin2x)
cosx – sinx and cosx + sinx > 0 ⇒ sin3x sinx = 6sin2x
π 3π sin3x sinx = 6sin2x
∴ x = 2nπ + , 2nπ +
12 4 sinx(sin3x – 6sinx) = 0

∵ For x = 2nπ + , cosx + sinx < 0, which is not a sinx(3sinx – 4sin3x – 6sinx) = 0
solution. 4
⇒ sin2x(–3 – 4 sin2x) = 0
π
∴ x = 2nπ + −3
12 ⇒ sinx = 0 or sin2x =
4
Sol 16: cos6θ + cos4θ + cos2θ + 1 = 0 Which is not possible
2cos4θ cos2θ + cos4θ + 1 = 0 ∴ x = np
2cos4θ cos2θ + 2cos 2θ = 0 2
∴ Possible solutions are
2cos2θ (cos4θ + cos2θ) 0, π, 2π, 3π, . . . . . . . 99p
2cos2θ × 2cos3θ cosθ = 0 ∵ 100π = 314.159 > 314
⇒ cosθ = 0 or cos2θ = 0 or cos3θ = 0 99 × 100 π
∴ Sum = = 4950p
π π 2n π π 2
θ = 2nπ ± or θ = nπ + or θ = ±
2 4 3 6
π π π 5π 3π Sol 20: sin3x + sin32x + sin33x
∴θ= , , , ,
6 4 2 6 4 = (sinx + sin2x + sin3x)3
= 30°, 45°, 90°, 150°, 135° a3 + b3 + c3 = (a + b + c)3 – 3(a + b)
M a them a ti cs | 7.57

(b + c) (c + a) 2cos2x sin2x – 2cos4x sin2x = 4


⇒ (sinx + sin2x) (sin2x + sin3x) (sin3x + sinx) =0 ⇒ sin4x + sin2x – sin6x = 4
 3x x  5x x As maximum value of sinθ = 1 so if sin4x, sin2x takes
 2sin cos  2sin cos  (2sin2xcosx)=0 maximum value 1 and sin6x takes the value –1 even
 2 2  2 2
then LHS will be less than RHS so no solution possible.
3x 5x
⇒ sin = 0 or sin = 0 or sin2x = 0 or 1
2 2 (b) sinx cos4x sin5x = −
2
x
cos = 0 or cosx = 0 (cos4x – cos6x) cos4x = –1
2
2n π 2n π π 2cos24x – 2cos6x cos4x = – 2
∴x= , , nπ, 4nπ ± π, 2nπ ±
3 5 2 ⇒ 1+cos8x–cos10x–cos2x=– 2
2π ⇒ cos8x – cos10x – cos2x = –3
∴ Least positive angle would be = 72°.
5
For LHS = RHS
π 
Sol 21: sin(π – 6x) + 3 sin  + 6x  = 3 in [0, 2p] cos10x = cos2x = 1 and cos8x = 1
 2 
∴ 8x = 2nπ and 10x = (2m + 1) π and
1 3 3
sin6x + cos6x =
2 2 2 2x = (2m + 1)p
 π π nπ π π
cos  6x −  = cos   ⇒x= , x = (2m + 1) , x = (2m + 1)
 6 6 4 10 2
nπ π nπ π
π π nπ nπ π = (2m + 1) and = (2m + 1)
6x – = 2nπ ± ⇒x= , + 4 10 4 2
6 6 3 3 18
There is no integer value of n and m for which above
π 2π 4π 5π π
∴ x = 0, , , π, , , 2π, , results hold. So no solution.
3 3 3 3 18
(c) 2sinx cosx cos2x = –1
7 π 13 π 19 π 25 π 31 π
, , , , 2sinx[cos3x + cosx] = –2
18 18 18 18 18
⇒ sin4x + sin(–2x) + sin2x=– 2
∴ Total no. of solutions = 13
⇒ sin4x = –2
cos θ + sin θ
Sol 22: (sinθ)x2 + (2cosθ)x + ∴ No solution
2
(d)4sin2x + cosx = 5
For perfect square of linear equation D = 0
For this result to hold
⇒ b2 – 4ac = 0
sin2x = 1 and cosx = 1
 cos θ + sin θ 
4cos2θ – 4   sinθ = 0 π
 2  ∴ 2x = nπ + (–1)n 2 and x = 2mp
2cos2θ = (cosθ + sinθ)sinq
nπ π
⇒ 2cotθ = 1 + tanq ∴x= + (–1)n 2 and x = 2mp
2
⇒ tan2θ + tanθ – 2 = 0 There exist no m, n for which the above relation as
(tanθ + 2) (tanθ – 1) = 0 valid. No solution
tanθ = – 2, 1 (e) sin3x – cosx = 2
π For this
n π + , n π − tan−1 2 n ∈ I
4 sin3x = 1 and cosx = –1
π
Sol 23: (a) sinx sin2x sin3x = 1 3x = nπ + (–1)n 2 and x = (2m + 1)p
1 π
(cos(–2x) – cos4x)sin2x = 1 nπ
tanx x= + (–1)n 6 and x = (2m + 1)p
3
7 . 5 8 | Trigonometric Ratios, Identities and Equations

Both the values cannot be same for any integral value θ


2 tan
of m and n. So no. solution 2
sinθ = ,
θ
1 + tan2
Sol 24: (a) f(x) = sin6x + cos6x + k(sin4x + cos4x) 2

= (sin2x + cos2x)3 – 3sin4x cos2x θ


1 − tan2
cosθ = 2
– 3sin2x cos4x + k(1 – 2sin2x cos2x)
θ
1 + tan2
= 1 + k – 2ksin2x cos2x – 3sin2x cos2x 2

= (1 + k) –
( 2k + 3) sin22x Let tan
θ
=t
4 2
For f(x) = constant ∴ acosθ + bsinθ = c
2k + 3 3 a(1 – t2) + b(2t) = c(1 + t2)
=0 ⇒k= −
4 2 = (a + c)t2 – 2bt + c – a = 0
(b) (1 + k) –
( 2k + 3 ) sin22x = 0
α β 2b
4 ∴ tan + tan = (P) (sum of roots)
2 2 a+c
(1 + k)4
⇒ sin22x = α β c−a
(2k + 3) tan tan =
2 2 c+a
4 (1 + k )  −1  (Q) (Product of roots)
∴0≤ ≤ 1 ⇒ k ∈  −1, 
2k + 3  2

(c) If k = –0. 7 Sol 26:

2( −0.7) + 3 (A) cos3x cos3x + sin3x sin3x = 0


⇒ (1–0. 7)– sin22x= 0
4 ⇒ (4cos3x – 3cosx) cos3x+(3sinx – 4sin3x) sin3x = 0
0.3 × 4 3 ⇒ 4cos6x–4sin6x+3sin4x–3cos4x=0
⇒ sin22x = =
1.6 4
⇒ 4[(cos2x)3–(sin2x)3]
3
∴ sin2x = ± +3(sin2x–cos2x)(sin2x+ cos2x)=0
2
π nπ π ⇒ 4(cos2x – sin2x)(1 + cos2x sin2x) – 3cos2x = 0
⇒ 2x = nπ ± ⇒x= +
3 2 6 ⇒ 4cos2x(1 + cos2x sin2x) = 3cos2x
⇒ cos2x + 4cos2x(sin2x)2 = 0
Sol 25: a & b
⇒ cos2x(1 + sin22x) = 0
a cos θ + b sin θ =c
sin22x = – 1
⇒ a cos θ = c − bsin θ
Not possible (No real solution)
2 2 2 2 2
⇒ a cos θ= c − b sin θ − 2bc sin θ π nπ π
cos2x = 0 ⇒ 2x
= 2n π ± ⇒ =
x ±
⇒a 2
(1 − sin θ=) c + b sin θ − 2bc sin θ
2 2 2 2 2 2 4
(B) sin3α=4sinasin(x + α)sin(x – α)
⇒ ( a + b ) sin θ − 2bc sin θ + c − a =0
2 2 2 2 2

sin3α = 2sinα [cos2α – cos2x]


If α & β are roots, then
sin3α = sin3α + sin(–α) – 2sinacos2x
2bc
sin α + sin β = ⇒ sinα (1 + 2cos2x) = 0
a2 + b2
1
c2 − a2 cos2x = −
sin α .sin β = 2
a2 + b2 2π
2x = 2nπ ±
α β 3
(c) tan + tan
2 2 π
x = nπ ±
3
M a them a ti cs | 7.59

(C) |2tanx – 1| + |2cotx – 1| = 2 ⇒ 2cos22x – 1 = 0


For tanx > 2 ⇒ cos4x = 0
2tanx – 1 + 1 – 2cotx = 2 π nπ π
4x = nπ + or x = + n∈I
1 2 4 8
tanx – =1
tanx
Sol 27: (A) sin2x + cos23x = 1
tan2x – tanx – 1 = 0
cos23x = cos2x
1± 5
tanx = <2
2 ⇒ cos3x = ±cosx
∴ No. solution. ∴ 3x = 2nπ ± x or 3x = 2nπ ±(π – x)
1  nπ nπ π π
for tanx ∈  , 2  x = nπ, or x = + , nπ –
2  2 2 4 2
2tanx – 1 + 2cotx – 1 = 2 From options one can say that all the options which
2 satisfying the equation are p, q, r, s
2tanx + =4 2θ
tanx (B) ecot + sin2θ – 2cos22θ + 4 = 4sinq
tan2x – 2tanx + 1 = 0 ecot

is not possible at θ = nπ, n ∈ I
1  π
tanx = 1 ∈  , 2  So at θ =
2  4
π 1 4
∴ x = nπ + is one solution ⇒ e1+ – 2(0) + 4, =2 2
4 2 2
1
For x < L. H. S. ≠ R. H. S.
2
2 π
1 – 2tanx + –1=2 at θ =
tanx 2
⇒ e0 + 12 – 2(1)2 + 4 = 4(1)
tan2x + tanx – 1 = 0
1+1–x+4=4
−1 ± 5
tanx =
2 L. H. S. = R. H. S.
−1 − 5 π
tanx = is acceptable So Ans (s) (4n + 1)
2 2
(C) a2sinx – asin2x + sinx = 0
∴ From option (Q)
(a2 + 1)sinx = 2asinx cosx
29
(D) sin10x + cos10x = cos42x sinx[(a2 + 1) – 2acosx] = 0
16
5 5 a2 + 1
 1 − cos2x   1 + cos 2x  29 sinx = 0,cosx = ≥1
  +  = cos4 2x 2a
 2   5  16
From options Ans. (p)
Let cos2x = t
(D) 3 2 tan θ − 1 + 3 tan θ − 1 =1
5 5
1 − t  1 + t  29 4
  +  = t From all the given options we can directly reject P, Q, S
 2   2  16
π
as they are not satisfying the given equation and is
not in domain of tanθ. 2
24t4 – 10t2 – 1 = 0 or (2t2 – 1) (12t2 + 1) = 0
1
⇒t=±
2
1
∴ cos2x =
2
7 . 6 0 | Trigonometric Ratios, Identities and Equations

Exercise 2 ⇒ cos 3x =cos ( 2n + 1 )


0=
π
2
Single Correct Choice Type π π
⇒ x =,
6 2
Sol 1: (A) If B = C then ∠A = 90° ∠B = 45° ∠C= 45° ⇒ Two solutions

 π
 π sin2  x −  1 1 1 1
tan x −   4 Sol 4: (B) cot 7 ° + tan67 ° − cot 67 ° − tan7 °
Sol 2: (A) 2  4 − 2 ( 0.25 ) cos 2x +1 =0 2 2 2 2

   π  A B B A
π = cot + tan – cot – tan
sin2  x −   1 − cos2  x −   2 2 2 2
 4 1   4 
=
cos 2x 2 cos 2x A = 15°, B = 135°

1 (1 − sin 2x ) 1 π  1 − tan2
A B
tan2 − 1
= = tan  − x  2 + 2
2 cos 2x 2  4  =
A B
tan tan
 π 2 2
∴ Take tan  x −  = t
 4
= 2cotA – 2cotB = 2(cot15° – cot135°)
Then expression would be
= 2(2 + 3 + 1) = 2(3 + 3)
t/2
1
2t – 2   +1=0 Which is an irrational number.
4
1
t 2π
⇒ 2t – 2   + 1 = 0 Sol 5: (C) A = 580° = 3π +
9
2
A A A A
(2t)2 + (2)t – 2 = 0 1 + sinA= sin2 + cos2 + 2sin cos
2 2 2 2
(2t + 2) (2t – 1) = 0
2
⇒ 2t = – 2 (Not possible) or 2t = 1  A A A A
=  sin + cos  = sin + cos
⇒t=0  2 2 2 2

 π π 2π
∴ tan  x −  = 0 ; x = nπ + ∵ for A = 3π +
 4  4 9
1  A A
But in equation does not exist at 1 + sinA =
−  sin + cos 
cos2x  2 2
π
x = nπ + , therefore no value of x exists. A A A A
4 1 − sinA= sin2 + cos2 − 2sin cos
2 2 2 2
5
Sol 3: (A) ∑ cos r x = 0 A A
r =1 = sin − cos
2 2
cos x + cos 2x + cos 3x + cos 4x + cos 5x = 0

For A = 3π +
⇒ cos x + cos 5x + cos 2 x + cos 4x + cos 3x =
0 9

⇒ 2 cos 3x cos 2x + 2 cos 3x cos x + cos 3x =


0  A A
1 − sinA =
−  sin − cos 
 2 2
⇒ cos 3 x 2 cos 2x + 2 cos x + 1 =
0
A
∴ 2sin = – 1 + sinA – 1 − sinA
 3x x  2
⇒ cos 3 x 2 × 2 cos cos + 1 =
0
 2 2 
 3x x 
⇒ cos 3x  4 cos cos + 1 =0
 2 2 
M a them a ti cs | 7.61

x2 − x 1
Sol 6: (A) tanα = and tanβ = xy x2 y2
2
x − x +1
2
2x − 2x + 1 ⇒ cos ( α − β=
) + 1− . 1−
ab a2 b2
1
⇒2(x2 – x) + 1 =
tanβ xy x2 y 2 x2 y 2
⇒ cos ( α − β=
) + 1− − +
ab a2 b2 a2 b2
 1 − tan β  1
x2 – x =  
 tan β  2 2  2 2 2 2 
2
 xy   1− x − y + x y
⇒  cos ( α − β ) − =  
1  1 − tan β  1  1 − tan β   ab   a2 b2 a2 b2 
     
2  tan β  2  tan β 
∴ tanα = =
1  1 − tan β  1  1 + tan β  x2 y 2 xy
  +1   ⇒ cos2 ( α − β ) + − 2 cos ( α − β ) ×
2  tan β  2  tan β  ab2 2 ab

1 − tan β  x2 y 2  x2 y 2
⇒ tanα = 1  +
=− +
1 + tan β  a2 b2  a2 b2
 
⇒ tanα + tanα tanβ = 1 – tanb
x2 y2 2xy
⇒ + − cos ( α − β ) = 1 − cos2 ( α − β )
tan α + tan β a2
b 2 ab
⇒ = tan(α +=
β) 1
1 – tan α tan β
= sin2 ( α − β )

Sol 7: (C) 8cos2x + 18sec2x


Sol 9: (D) tanx + tan2x + tan3x = tanx. tan2x. tan3x
18
f (X) = 8X + 2
let cos x = X
2
tanx + tan2x
X2 = − tan3x
1 − tanx tan2x
36
f’(X) = 16 X – =0 ⇒ tan3x = – tan3x ⇒ tan3x = 0
X3
1/ 4 ⇒ 3x = np
 36 
X=   = (2 25)1/4 > 1
 16  nπ
⇒ x=
3
∴ Min. Value of this function will be
When cosx = 1 Sol 10: (C) tan3x – tan2x – tanx = 0
∴ Min. Value = 8 + 18 = 26 tan(3x–2x)[1+tan3x tan2x] – tanx = 0
⇒ tanx tan2x tan3x = 0
=
Sol 8: (B) x a cos ( φ − α )
nπ nπ
∴ x = nπ, ,
=y b cos ( φ − β ) 2 3
π 2π 4 π 5π
x ∴ x = 0, , , π. , , 2p
⇒ cos−1   = φ − α  …..(i) 3 3 3 3
a i.e. 7 solutions
y
⇒ cos−1   = φ − β  …..(ii) π
b ∵x≠
2
(ii) – (i)
π
x y As at x = tan x, is not defined

= α − β cos−1   − cos−1   2
a b
 xy x2 y2  Sol 11: (C) x = 1 – x + x2 – x3 + x4 – x5 + . . . . . . . ∞
⇒ α − β cos−1 
= + 1− 1− 
 ab a2 b2  x = 1 – x(1 – x + x2 – x3 + x4 . . . . . . . ∞)

x = 1 – x. x
7 . 6 2 | Trigonometric Ratios, Identities and Equations

⇒ x2 + x – 1 = 0 2  12 7
=  cot α + 
−1 ± 5 35 10 
x= = 2sin18°
2
[24 cot α + 7]
= 3
15
Multiple Correct Choice Type
1
4 3  Sol 14: (B, C) sin t + cost =
Sol 12: (B, D) 1 + 4sinθ + 3cosθ = 1 + 5  sin θ + cos θ  5
 5 5  t t
4 2 tan1 − tan2
= 1 + 5sin(θ + α)cosα = 2 + 1
2 =
5 ⇒
t t 5
∴ Maximum value is 1 + 5 = 6 1 + tan2 1 + tan2
2 2
When sin(θ + α) = 1 t
Let tan =a
Minimum value is 1 – 5 = – 4 2

When sin(θ + α) = –1 5(2a + 1 – a2) = 1 + a2


⇒ 6a2 – 10a – 4 = 0 ⇒ 3a2 – 5a – 2 = 0
2 ⇒ (3a + 1) (a – 2) = 0
Sol 13: (A,B, C) 3 sin(α + β) − cos(α + β)
π
cos 1
6 ∴a= − ,2
sin α 3
t −1
tan = ,2
 3  2 3
 3× sin(α + β) − 2cos(α + β) 
=  2  2
 sin α  3
  Previous Years’ Questions
 
3  1
2 2(
sin α cos β + cos α sin β ) − 2(cos α cos β − sin α sin β)  Sol 1: Given, esin x − 4
=
sin x
=   e
3 sin α 
( ) ( )
2
  ⇒ esin x − 4 esin x − 1 =0
2 3 3 
=  cos β + cot αsinβ − 2cot α cos β + 2sin β  4 ± 16 + 4
3 2 2  ⇒ esin x = 2± 5
=
2
4 3 But since, e ~ 2.72 and we know, 0 < esin x < e
It sinβ = and cosβ =
5 5 ∴ esin x =
2 ± 5 is not possible.
 
i.e. tanβ > 0 i.e. β ∈  0, π  Hence, no solution.
 2
R. H. S. 1 + cos x + cos2 x + cos3 x +.....
Sol 2: Given, 2 = 22
2 3 3 3 4 3 4
=  × + cot α × − 2cot α × + 2 × 
1
3 2 5 2 5 5 5 1 − cos x 1
⇒2 = 22 ⇒ =2
1 − cos x
2  9 8 5 1
=  + = ⇒ cos x = ±
3 10 5  3 2
3 ( x ∈ ( −π, π))
If cosβ = − i.e. tanβ < 0
5
⇒ R. H. S. Thus, the solution set is

2 3  3  3 4 3 4  π 2π 
=   −  + cot α × + 2cot α × + 2 ×  + , ± 
3 2  5  2 5 5 5  3 3
M a them a ti cs | 7.63

Sol 3: Exp {(sin x + sin


2 4
) }
x + sin6 x + ..... + ∞ loge 2 ⇒ sin ( 4x + 100º ) + sin100º + sin200º =
0

sin2 x sin2 x
⇒ sin ( 4x + 100º ) + 2sin150º cos50º =
0
loge 2 loge 2
= e 1 −sin2 x = e cos2 x 1
⇒ sin ( 4x + 100º ) + 2. sin ( 90º −50º ) =
0
2
2x
⇒ 2tan satisfys x2 − 9x + 8 =0 ⇒ sin ( 4x + 100º ) + sin 40º =
0

⇒ x = 1, 8 ⇒ sin ( 4x + 100º ) + sin ( −40º )


tan2 x tan2 x
⇒ 4x n (180º ) + ( −1 ) ( −40º ) − 100º
n
∴ 2 = 1 and 2 =8 =

⇒ tan2 x = 0 and tan2 x = 3 1


n (180º ) + ( −1 ) ( −40º ) − 100º 
n
⇒x
=
2 
4 
 π
⇒ x = nπ and tan2 x =  tan  The smallest positive value of x is obtained when n = 1.
 3
1
⇒ x = nπ and x = nπ ±
π Therefore,=
x
4
(180º + 40º −100º )
3
π 1
Neglecting x = nπ as 0 < x < ⇒x
=
4
(120º
= ) 30º
2
π  π
⇒ x = ∈  0,  Sol 5: Given,
3  2
1
cos x 2 1 3 −1 cos (p sinx ) = sin (p cos x ) , ∀x ∈ 0,2π 
∴ = = ×
cos x + sinx 1 3 1+ 3 3 −1
+ π 
2 2 ⇒ cos (psinx ) =
cos  − pcos x 
 2 
cos x 3 −1 π 
⇒ = ⇒ psinx= 2nπ ±  − pcos x  , n ∈ I
cos x + sinx 2 2 
( cos θ= cos α ⇒ θ= 2nπ ± α ,n ∈ I )
Sol 4: tan ( x + 100º ) = tan ( x + 50º ) tanx tan ( x − 50º )
⇒ psinx + pcos x= 2nπ + π / 2
tan ( x + 100º ) or psinx − pcos x= 2nπ − π / 2 , n ∈ I
⇒ tan ( x + 50º ) tan ( x − 50º )
=
tanx
⇒ p ( sinx + cos x ) 2nπ + π / 2
sin ( x + 100º ) cos x sin ( x + 50º ) sin ( x − 50º )
⇒ ⋅ =
cos ( x + 50º ) cos ( x − 50º ) or p ( sinx − cos x )= 2nπ − π / 2 , n ∈ I
cos ( x + 100º ) sinx
 π π  π
⇒ p 2  cos sinx + sin cos x = 2nπ −
⇒ sin ( 2x + 100º ) + sin100º  cos100º + cos2x   4 4  2

= cos100º − cos2x  × sin ( 2x + 100º ) − sin100º   π π  π


or p 2  cos sinx − sin cos x = 2nπ − ,n ∈ I
 4 4  2
⇒ sin ( 2x + 100º ) ⋅ cos100º
⇒ p 2 sin ( x + π / 4 )  =
( 4n + 1) π
+ sin ( 2x + 100º ) ⋅ cos2x + sin100º cos100º 2

+ sin100º cos2x or p 2 sin ( x − π / 4=


) ( 4n − 1) 2π , n ∈ I
= cos100º sin ( 2x + 100º ) Now, −1 ≤ sin ( x ± π / 4 ) ≤ 1
− cos100º sin100º − cos2x sin
⇒ −p 2 ≤ p 2 sin ( x ± π / 4 ) ≤ p 2
( 2x + 100º ) + cos2x sin100º
⇒ −p 2≤
( 4n + 1) π ≤ p 2 , n∈ I
⇒ 2sin ( 2x + 100º ) cos2x +2sin100º cos100º =
0 2
7 . 6 4 | Trigonometric Ratios, Identities and Equations

Second inequality is always a subset of first, therefore, Sol 7: (A, C) Given,


we have to consider only first.
1 + sin2 θ cos2 θ 4 sin 4θ
It is sufficient to consider n ≤ 0 , because for n > 0, the 2 2
sin θ 1 + cos θ 4 sin 4θ 0
=
solution will be same for n ≥ 0 .
2 2
sin θ cos θ 1 + 4 sin 4θ
If n ≥ 0, − 2p ≤ ( 4n + 1 ) π / 2

⇒ ( 4n + 1 ) π / 2 ≤ 2p Applying
For p to be least, n should be least. R3 → R3 − R1 and R 2 → R 2 − R1 we get
⇒n=0
1 + sin2 θ cos2 θ 4 sin 4θ
⇒ 2p ≥ π / 2 −1 1 0 0
=
π −1 0 1
⇒ p≥
2 2
π Applying C1 → C1 + C2
Therefore, least value of p =
2 2
2 cos2 θ 4 sin 4θ
Sol 6: Given, ⇒ 0 1 0 0
=
2θ −1 0 1
(1 − tan θ )(1 + tan θ ) sec2 θ + 2tan =0

( )(
⇒ 1 − tan2 θ ⋅ 1 + tan2 θ + 2 ) tan2 θ
=0 ⇒ 2 + 4 sin 4θ =0

⇒ 1 − tan4 θ + 2tan =0 −1
⇒ sin 4θ =
2
Substitute tan2 θ =x
n π
∴ 1 − x 2 + 2x =
0 ⇒ 4θ = nπ + ( −1 )  − 
 6
⇒ x2 − 1 =2x nπ n+1  π 
⇒ 4=
θ + ( −1 )  
x 2
Note: 2 and x − 1 are incompatible functions, 4  24 
therefore, we have to consider range of both functions. 7π 11π
Clearly, θ = , are two values of θ lying between
Curvesy = x – 1 2
24 24
π
0º and
Andy = 2x 2
It is clear from the graph that two curves interest at one
point at x = 3, y = 8. Sol 8: (B, C) For 0 < φ < / π / 2 we have

y x= ∑ cos2n φ = 1 + cos2 φ
n= 0

+ cos4 φ + cos6 φ + .....

x’ x It is clearly a GP with common ratio of cos2 φ which is


-1 O 1
≤1.
-1
1 1
y’ Hence, x = =
2
1 − cos φ sin2 φ
Therefore, tan2 θ =3
 a 
⇒ tan θ = ± 3  S=
∞ , −1 < r < 1
 1−r 
π 1
⇒θ=± Similarly, y =
3 cos2 φ
1
And z =
1 − sin φ cos2 φ
2
M a them a ti cs | 7.65

1 1 4
Now, x +=
y + 4 ± 16 − 4 × 5 ×
2 2
sin φ cos φ ⇒ sin2 x = 5
10
cos2 φ + sin2 φ 1
=
2 2
cos φ sin φ cos φ sin2 φ
2
4 ± 16 − 16
=
1 1 10
Again, = 1 − sin2 φ cos2 φ = 1 −
x xy 2
⇒ sin2 x =
5
1 xy − 1
⇒ = ⇒ xy = xyz − z 2
x xy ⇒ sin x =
±
5
⇒ xy + z =xyz
2
sin2 x =
Therefore, (b) is the answer from eq. (i) (putting the 5
value of xy)
3
⇒ cos2 x =
⇒ xyz = x + y + z 5
5
⇒ sec2 x =
Sol 9: (A, B) We know that, sec2 θ ≥ 1 3
4xy
⇒ ≥1 ⇒ tan2 x = sec2 x − 1
(x + y)
5 2
⇒ 4xy ≥ ( x + y )
2
= −1 =
3 3
⇒ ( x + y ) − 4xy ≤ 0
2
sin8 x cos8 x
Now, +
⇒ (x − y) ≤ 0 8 27
2

4 4
⇒ x−y =0 2 3
   
5 5 2 3 1
⇒ x=y =  +  = + =
8 27 5 4
5 4 125
Therefore , x + y = 2x (add x both sides)
But x + y ≠ 0 since it lies in the denominator, Sol 11: (D)
⇒ 2x ≠ 0
6
⇒ x≠0  mπ π  mπ 
∑ cosec θ + 4
−  cosec  θ +
4
 =4 2
4 
Hence, x = y, x ≠ 0 is the answer. m=1  
 m  m π π 
sin4 x cos4 x 1 sin  θ +  −  θ + − 
Sol 10: (A, B) + =
6
  π   4 4 
2 3 5 ∑ 2×
 mπ   mπ π 
4 2
=
m=1
sin  θ +  sin  θ + − 
3 sin4 x + 2 cos4 x 1  4   4 4
⇒ =
6 5
 m π  mπ π   m π  m π π
( 6
)
4 4 4
sin  θ +  cos  θ + −  − cos  θ +  sin  θ + − 
⇒ sin x + 2 sin x + cos x = 6 4 4 4 4 4 4
5 ∑   
 m π 



m π

π 

m=1
sin  θ +  sin  θ + − 
( 6
⇒ sin4 x + 2 1 − 2 sin2 x cos2 x  =
  5 ) 6
 4   4 4

  mπ π   m π 
∑ 2 x cot  θ + −  − cot  θ +  = 4 2
4 2
(
⇒ sin x + 2 − 4 sin x 1 − sin x =
6
5
2
) m=1   4 4  4 

6   3 π 
⇒ sin4 x + 2 − 4 sin2 x + 4 sin4 x = 2 cot θ − cot  θ +  = 4 2
5   2 
4  3π 
⇒ 5 sin4 x − 4 sin2 x + 0
= ⇒ cot θ − cot  + θ = 4
5
 2 
7 . 6 6 | Trigonometric Ratios, Identities and Equations

⇒ cot θ − tan θ = 4
Sol 13: Given, n > 3 ∈ Integer
1 1 1
⇒ tan2 θ − 4 tan θ + 1 = 0 and = +
π  2π   3π 
4 ± 16 − 4 sin   sin   sin  
⇒ tan θ = n
  n
   n 
2
1 1 1
4 ± 12 ⇒ − =
= π 3π 2π
2 sin sin sin
n n n
4± 2 3
=
2 3π π
sin − sin
⇒ n n = 1
⇒ tan θ = ± 2 3
π 3π 2π
sin .sin sin
5π n n n
⇒θ=
12 π 3π
 2π  sin .sin
π n n
⇒ 2cos   .sin =
Sol 12: Given, tan
= θ cot5θ n
  n 2 π
sin
n
π 
⇒=tan θ tan  − 5θ 
2 4π 3π
 
⇒ = π− ⇒ 7π = π ; ⇒ n = 7
π n n n
⇒ − 5θ = nπ + θ
2
π
⇒ 6θ= − nπ Sol 14: ( y + z ) =
cos 3 θ xyz sin 3 θ  …..(i)
2
π nπ sin3 θ 2z cos 3 θ + 2y sin 3 θ 
xyz= …..(ii)
⇒=θ −
12 6
π 
xyz sin3 θ= ( y + 2z ) cos 3 θ + y sin 3 θ  ….(iii)
Also, cos 4=
θ sin2=
θ cos  − 2θ 
 2  From (i), (ii) and (iii), we get
π 
⇒ 4θ= 2nπ ±  − 2θ  ( y + z ) cos
= 3 θ 2z cos 3 θ + 2y sin 3 θ
2 
= ( y + 2z ) cos 3 θ + y sin 3 θ
Taking positive ∆ sign
⇒ y ( cos 3 θ − 2sin
= 3 θ ) z cos 3 θ and
π
⇒ 6θ= 2nπ + y ( cos 3 θ − sin 3 θ ) =0
2
nπ π
⇒=
θ + ⇒ cos =
3 θ sin 3 θ
3 12
π
Taking negative sign ⇒ tan 3 θ = 1 = tan
4
2π 2π 3π
⇒ 2sin .cos = sin π
n n n ⇒ 3 θ= n π +
4
4π 3π
⇒ sin = sin nπ π
n n ⇒=
θ +
3 4
π
⇒ 2θ= 2nπ − π 7 π 11 π
2 ⇒ , ,
4 12 12
π
⇒ θ = nπ − Total three solution in ( 0, π )
4
Above values of θ suggest that there are only 3
common solutions. Sol 15: tan
= θ cot5 θ
π 
tan θ tan  − 5 θ 
=
2 
M a them a ti cs | 7.67

π 1
⇒ θ= n π + − 5θ 1
2 is =2
2
sin θ + 3sin θ cos θ + 5cos θ 2 1/2
π
⇒6 θ = nπ +
2

nπ π π ±π ± 5 π
Sol 17: (D) P = {θ : sin θ − cos θ = 2 cos θ }
⇒θ = + ⇒θ= ± , , .....  …. (i)
6 12 12 4 12 ⇒ sin=
θ ( 2 + 1) cos θ
sin=
2 θ cos 4 θ
⇒ tan =
θ 2 +1
π 
⇒ cos
= 4 θ cos  − 2 θ 
2  Q= {θ : sin θ + cos θ = 2 sin θ}
π  ⇒ cos=θ ( 2 − 1 ) sin θ
⇒ 4=θ 2 m π ±  − 2θ
2  1
mπ π π ⇒ tan =
θ = 2 +1
⇒=θ + , m π− 2 −1
3 12 4
∴P =Q
π π 5π
⇒=
θ ,− ,  ….(ii)
12 4 12 1 1 1
Sol 18: = +
π 2π 3π
In (i) and (ii) only (iii) solutions are common sin sin sin
n n n
π π 5π
− , ,
4 12 12 3π π
sin− sin
n n 1
⇒ =
π 3π π
Sol 16: Given sin sin sin 2
n n n
1
sin θ + 3sin θ cos θ + 5cos2 θ
2  3π π   3π π 
 +   − 
n n n n
Let 2 cos  sin 
2 2 1
⇒ =
A sin2 θ + 3sin θ cos θ + 5cos2 θ
= π 3π 2π
sin sin sin
n n n
= 1 + 4 cos2 θ + 3sin θ cos θ

3 2π π
= 1 + 2 (1 + cos 2 θ ) + sin2 θ 2 cos
sin
1
2 ⇒ n n =
π 3π π
3 sin sin sin 2
= 3 + 2cos 2 θ + sin 2 θ n n n
2
a cos θ + b sin θ ≥ − a2 + b2  2π 2π 3π
  = 2=
sin cos sin
n n n
3
2 4π 3π
3 − 22 +  
Amin = ⇒ sin sin
=
n n
2
4π 3π
9 ⇒ sin − sin 0
=
=3 − 4 + n n
4
7π π
25 5 1 ⇒ 2 cos sin 0
=
=3 − =3 − = 2n 2n
4 2 2
7π π
The maximum value of given expression ⇒ cos = 0 or sin = 0
2n 2n
7 . 6 8 | Trigonometric Ratios, Identities and Equations

7  1 
⇒ n= ⇒ − 4 sin x cos2 x − − cos x  =3
2m + 1  2 
For integral values of n > 3  3
2
1
⇒ − 4 sin x  cos x −  −  =3
 2 4
Sol 19: (A, C, D) θ ∈ 0, 2 π   
2
 1 3 3
 θ θ ⇒  cos x −  − =−
2 cos θ (1 − sin =
φ ) sin2 θ  tan + cot  cos φ − 1  2 4 4 sin x
 2 2 
2
 θ   1 3 3
sin θ  tan2 + 1 
2
⇒  cos x −  = −
 2  cos φ − 1 2 4 4 sin x
⇒ 2cos θ (1 − sin φ )
= 
θ
tan 2
2  1 3 3 
⇒  cos x −  =  1 − 
θ  2 4 sin x 
sin2 θ.sec2
⇒ 2cos θ (1 − sin φ ) 2 cos φ − 1
= L.H.S. > 0 and R.H.S. < 0
θ
tan
2 ∴ No solution

⇒ 2cos θ (1 − sin=
φ ) 2sin θ ( cos φ ) − 1
5
Sol 21: cos2 2x + cos4 x + sin4 x + cos6 x + sin6 x =
2
⇒ 1 + 2 cos
= θ 2 sin ( θ + φ )  ….(i) 4
3 5
Now, given tan ( 2 π − θ ) > 0, − 1 < sin θ < − ⇒ cos2 2x + cos4 x + sin4 x + cos4 x
2 4
3π 5π 1 − sin2 x  + sin4 x 1 − cos2 x  =
2
⇒ tan θ < 0 and <θ<    
2 3



2
<θ<

3

5
4
( )
cos2 2x + 2 cos4 x + sin4 x − sin2 x cos2 x

 3 π 5π 
From (i), as θ ∈  ,
(sin x + cos x ) =
2 2
2

 2 3 
⇒ cos 2x + 2 ( sin x + cos x )
5 2  2 2
2 
− 2sin2 x cos2 x 
⇒ 1 < 2 cos θ + 1 < 2 4  
⇒ 1 < 2 sin ( θ + φ ) < 2 − sin2 x cos2 x =
2
1  π 5π  π
⇒ < sin ( θ + φ ) < 1 ⇒ θ + φ ∈  ,  ,θ + φ ≠ ⇒
5
cos2 2x − 5sin2 x cos2 x =
0
2 6 6  2 4
Or
5 5
⇒ cos2 2x = sin2 2x
 13 π 17 π  5π π 5π 4 4
θ+φ∈ ,  ,θ + φ ≠ ⇒ −θ < φ < −θ
 6 6  2 6 6
⇒ tan2 2x =
1
13 π 17 π
or −θ < φ < −θ  π
6 6 ⇒ tan 2x = ± = tan  ± 
 3π 2π   2π 7π   4
As, θ ∈  − ,−  or  ,  π
 2 3   3 6  ⇒ 2x= n π ±
4
Sol 20: (D) sin x + 2 sin 2x – sin 3x = 3 nπ π
⇒ x= ±
2 8
⇒ sin x − sin 3x + 2sin 2x =
3
π 3 π 5 π 7 π 11 π 13 π 15 π
=⇒x , , , , , , →
⇒ 2 cos 2x sin ( −x ) + 2sin 2x =
3 8 8 8 8 8 8 8
Total 8 solution.
⇒ − 2 sin x 2 cos2 x − 1 + 2 × 2 sin x cos x =3
 
M a them a ti cs | 7.69

Sol 22: (C) 3 sec x + cosec x + 2 ( tanx − cot x ) =


0

3 1  sin x cos x 
⇒ + + 2 − =0
cos x sin x  cos x sinx 
⇒ 3 sin x + cos x + 2 sin2 − cos2 x  =
0
 

⇒ 3 sin x + cos x =
2 cos 2x

3 1
⇒ sin x + cos x =
cos 2x
2 2
π π
⇒ sin sinx + cos cos x =
cos2x
3 3

 π
⇒ cos  x −  = cos 2x
 3
π
⇒ x − = 2n π ± 2x
3
 π 2n π π
⇒ x = −  2n π +  or x = +
 3  3 9

π π 5π 7 π
⇒x=− , ,− ,
3 9 9 9
Sum = 0
2017-18 100 &
op kers
Class 11 T
By E ran culty
-JE Fa r
IIT enior emie .
S fP r es
o titut
Ins

MATHEMATICS
FOR JEE MAIN & ADVANCED
SECOND
EDITION

Exhaustive Theory
(Now Revised)

Formula Sheet
9000+ Problems
based on latest JEE pattern

2500 + 1000 (New) Problems


of previous 35 years of
AIEEE (JEE Main) and IIT-JEE (JEE Adv)

5000+Illustrations and Solved Examples


Detailed Solutions
of all problems available

Topic Covered Plancess Concepts


Tips & Tricks, Facts, Notes, Misconceptions,
Straight Line Key Take Aways, Problem Solving Tactics

PlancEssential
Questions recommended for revision
8. STRAIGHT LINE

1. INTRODUCTION
Co-ordinate geometry is the branch of mathematics which includes the study of different curves and figures by
ordered pairs of real numbers called Cartesian co-ordinates, representing lines & curves by algebraic equation. This
mathematical model is used in solving real world problems.

2. CO-ORDINATE SYSTEM
Co-ordinate system is nothing but a reference system designed to locate position of any point or geometric
element in a plane of space.

2.1 Cartesian Co-ordinates


Let us consider two perpendicular straight lines XOX’ and YOY’ passing through the origin
y
O in the plane. Then, =90
x
Axis of x: The horizontal line xox’ is called axis of x. x’ O

Axis of y: The vertical line yoy’ is called axis of y. y’

Co-ordinate axis: x-axis and y-axis together are called axis of co-ordinates or axis of Figure 8.1
reference.
Origin: The point ‘O’ is called the origin of co-ordinates or just the origin.
Oblique axis: When xox’ and yoy’ are not at right angle, i.e. if the both axes are not perpendicular, to each other,
then axis of co-ordinates are called oblique axis.

2.2 Co-ordinate of a Point


The ordered pair of perpendicular distances of a point from X- and Y-axes are called co-ordinates of that point.
If the perpendicular algebraic distance of a point p from y-axis is x and from x-axis is y, then co-ordinates of the
point P is (x, y). Here,
(a) x is called x-co-ordinate or abscissa.
(b) y is called y-co-ordinate or ordinate.
(c) x-co-ordinate of every point lying upon y-axis is zero.
(d) y-co-ordinate of every point lying upon x-axis is zero.
(e) Co-ordinates of origin are (0, 0).
Note: A point whose abscissa and ordinate are both integers is known as lattice point.
8 . 2 | Straight Line

2.3 Polar Co-ordinates y


P (r ,)
Let OX be any fixed line, known as initial line, and O be the origin. If the distance of any
point P from the origin O is ‘r’ and ∠XOP = θ, then (r, θ) are known as polar co-ordinates r
of point P. If (x, y) are the Cartesian co-ordinates of a point P, then x = rcos θ; y = rsin θ and

x
y
| r | = x2 + y 2 , θ = tan–1   θ ∈ ( −π , π )
O
x Figure 8.2

Illustration 1: If the Cartesian co-ordinates of any point are ( 3, 1) , find the polar co-ordinates.  (JEE MAIN)
y
Sol: Polar co-ordinates of any point are (r, θ), where r = x2 + y 2 and θ = tan–1   .
x
x= 3;y=1
Let their polar co-ordinates be (r, θ) ⇒ x = r cos θ; y = r sinθ

So r ⇒ x2 + y 2 r = 3+1

y  1  π
θ ⇒ tan−1   = 2 θ ⇒ tan−1  =
x  3 6
 π
∴(r, θ) =  2 ,  .
 6
y
3. DISTANCE FORMULA
Q(x2,y2)
B(r2,2)
d
The distance between two points P(x1, y1) and Q(x2, y2) is ,y 1)
P(x 1
PQ = (x1 – x2 )2 + (y1 – y 2 )2 = (x2 − x1 )2 + (y 2 − y1 )2 (r1,1) A
x2 - x1
Distance of a point P(x1, y1) from the origin O(0, 0) is
O x2
OP = 2
X +y 2
1 1
Figure 8.3
Distance between two polar co-ordinates A(r1, q1) and B(r2, q2) is
given by
AB = r12 + r22 − 2r1r2 cos(θ1 − θ2 )

Proof: AB = (x2 − x1 )2 + (y 2 − y1 )2 x1 = r1 cos q1, x2 = r2 cos q2, y1 = r1sin q1, y2 = r2sinq2

AB = (r2 cos θ2 − r1 cos θ1 )2 + (r2 sin θ2 − r1 sin θ1 )2

AB = (r2 cos θ2 )2 − 2r1r2 cos θ1 cos θ2 + (r1 cos θ1 )2 + (r2 sin θ2 )2 − 2r1r2 sin θ1 sin θ2 + (r1 sin θ1 )2

AB = r12 + r22 − 2r1r2 cos(θ1 − θ2 )

PLANCESS CONCEPTS
Distance between two polar co-ordinates A(r1, q1) and B(r2, q2) is given by
AB = r12 + r22 − 2r1r2 cos(θ1 − θ2 )
Vaibhav Krishnan (JEE 2009, AIR 22)
M a them a tics | 8.3

 π  π
Illustration 2: Find the distance between P  2, −  and Q  3,  .  (JEE MAIN)
 6  6

Sol: The distance between two points = r12 + r22 − 2r1r2 cos(θ1 − θ2 ) Therefore,
.
 π π π 1
PQ = r12 + r22 − 2r1r2 cos(θ1 − θ2 ) = 4 + 9 − 2.2.3cos  − −  = = 4 + 9 − 12cos   = 13 − 12. = 7
 6 6 3 2

Illustration 3: The point whose abscissa is equal to its ordinate and which is equidistant from the points A(1, 0),
B(0, 3) is  (JEE MAIN)

Sol: Given, abscissa = ordinate. Therefore distance can be found by considering the co-ordinates of required point
be P(k, k).
Now given PA = PB ⇒ (k − 1)2 + k 2 = k 2 + (k − 3)2
2k2 – 2k + 1 = 2k2 – 6k + 9 ⇒ 4k = 8 ⇒ k = 2

4. SECTION FORMULA
Let R divide the two points P(x1, y1) and Q(x2, y2) internally in the ratio m:n.
Let (x, y) be the co-ordinates of R.
y
Draw PM, QN, RK perpendicular to the x-axis. Q
R
F
Also, draw PE and RF perpendicular to RK and QN. P E

PR m
Here, = .
RQ n
x’ O M K N x
Triangles PRE and RFQ are similar.
PR PE PE m y’
∴ = ⇒ =
RQ RF RF n
Figure 8.4
But PE = x – x1 and RF = x2 – x.

x − x1 m mx2 + nx1
∴ = ⇒ x=
x2 − x n m+n
ER m
In the same way, =
FQ n
y − y1 m my 2 + ny1  mx2 + nx1 my 2 + ny1 
i.e., = ⇒y= The co-ordinates of R are  , 
y2 − y n m+n  m+n m + n 
PR ' m
If R’ divides PQ externally, so that = , triangles PER’ and QR’F are similar. y
QR ' n
PR ' PE F R’
∴ =
R 'Q R 'F Q
P E
But PE = x – x1 and R’F = x – x2.
x
x − x1
m mx2 − nx1 x’ O M N K

= i.e., x =
x − x2 n m−n
my 2 − ny1 y’
Similarly, y = .
m−n
Figure 8.5
 mx − nx1 my 2 − ny1 
The co-ordinates of R’ are  2 , 
 m−n m − n 
8 . 4 | Straight Line

PR ' m m
Alternate Method: =− = By changing n into –n in the co-ordinates of R, we can obtain the co-ordinates
R 'Q n –n
of R’:

mx2 − nx1 my 2 − ny1


,
m−n m−n
 x + x 2 y1 + y 2 
Cor. The mid-point joining the two points (x1, y1) and (x2, y2) is  1 , 
 2 2  .

 x + λx2 y1 + λy 2 
Cor. From the above cor., the co-ordinates of a point dividing PQ in the ratio λ:1 are  1 ,  . Considering
 1+ λ 1+ λ 
λ as a variable parameter, i.e. of all values positive or negative, the co-ordinates of any point on the line joining the

points (x1, y1) and (x2, y2) can be expressed in the above forms.

5. SPECIAL POINTS OF A TRIANGLE

5.1 Centroid C(x3, y3)

Let the vertices of the triangle ABC be (x1, y1), (x2, y2) and (x3, y3), respectively. F 1 D
 x + x3 y 2 + y 3  2 G
The mid-point D of BC is  2 ,  G, the centroid, divides AD internally
2 2 
in the ratio 2:1.  (x1, y1)A E B(x2, y2)

Let G be (x, y), Figure 8.6

2. ( (x2 + x3 ) / 2 ) + 1.x1 x1 + x2 + x3
then x = = and
2+1 3
2. ( (y 2 + y 3 ) / 2 ) + 1.y1 y1 + y 2 + y 3  x + x 2 + x3 y1 + y 2 + y 3 
y= = ∴ G is  1 ,  .
2+1 3  3 3 

5.2 Incentre
Let A (x1, y1), B (x2, y2), C (x3, y3) be the vertices of the triangle.
Let AD bisect angle BAC and cut BC at D.
A
BD AB c
We know that = =
DC AC b
cx3 + bx2 cy 3 + by 2 I
Hence the co-ordinates of D are , C
c+b c+b
D
Let (x, y) be the incentre of the triangle
B
CD b BC b + c ca AI AB c b+c
= ∴ = ∴ BD = = = = Figure 8.7
BD c DB c b + c ID BD ( ca / (b + c) ) a
(b + c) ( (cx3 + bx2 ) / (c + b) ) + ax1 ax1 + bx2 + cx3
= x = ,
b+c+a a+b+c
(b + c) ( (cx3 + bx2 ) / (c + b) ) + ax1 ax1 + bx2 + cx3 (b + c) ( (cy 3 + by 2 ) / (c+ b) ) + ay1 ay1 + by 2 + cy 3
∴ x = = , y =
b+c+a a+b+c b+c+a a+b+c
(b + c) ( (cy 3 + by 2 ) / (c+ b) ) + ay1 ay1 + by 2 + cy 3
y =
b+c+a a+b+c
M a them a tics | 8.5

5.3 Ex-centres
A
The centre of the circle which touches the side BC and the extended portions of sides I3 I2
AB and AC is called the ex-centre of ∆ABC with respect to the vertex A. It is denoted by
I1 and its co-ordinates are as follows: B C

 −ax1 + bx2 + cx3 −ay1 + by 2 + cy 3 


I1 =  , 
 −a + b + c −a + b + c 
I1

Similarly ex-centres of ∆ABC with respect to vertices B and C are denoted by I2 and I3,
respectively, and Figure 8.8

 ax – bx2 + cx3 ay1 − by 2 + cy 3 


I2 =  1 ,  ,
 a−b + c a−b + c 

 ax + bx2 − cx3 ay1 + by 2 − cy 3 


I3 =  1 ,  .
 a+b−c a+b−c
 

5.4 Circumcentre A(x1, y1)

It is the point of intersection of perpendicular bisectors of the sides of the triangle.


It is also the centre of a circle passing through the vertices of the triangle. If O is the
O
circumcentre of any ∆ABC, then, OA = OB = OC.
B E C(x3, y3)
 x sin2A + x2 sin2B + x3 sin2C y1 sin2A + y 2 sin2B + y 3 sin2C 
Circumcentre:  1 ,  (x2, y2)
 Σ sin2A Σ sin2A 
Figure 8.9
Note: For a right-angled triangle, its circumcentre is the mid-point of hypotenuse.

A C

Figure 8.10
A(x1, y1)
5.5 Orthocentre
The point of intersection of altitudes of a triangle that can be obtained by solving the D F
equation of any two altitudes is called Orthocentre. It is denoted by H
H

 x tanA + x2 tanB + x3 tanC y1 tanA + y 2 tanB + y 3 tanC  B E C


Orthocentre:  1 , 
(x2, y2) (x3, y3)
 Σ tanA Σ tanA  Figure 8.11
Note: In a right angle triangle, orthocentre is the point where right angle is formed.
A
Remarks:
(a) In an equilateral triangle, centroid, incentre, orthocentre, circumcentre coincide.
G H
(b) Orthocentre, centroid, and circumcentre are always collinear. Centroid divides the O
Orthocentre and circumcentre joining line in a 2: 1 ratio.
B C
Proof: H, G and O are collinear and ∆’s OGD & AGH are similar. D N

But OD (distance of c.c. from BC) = R cos A Figure 8.12


8 . 6 | Straight Line

HA = distance of orthocentre from vertex A = 2R cos A

AH AG HG
∴ = 2= = ⇒ G divides line joining H and O in 2:1.
OD GD GO
(c) In an isosceles triangle centroid, orthocentre, incentre, circumcentre lie on the same line.

5.6 Nine-Point Circle


Nine-point circle can be constructed for any given triangle, and is so named because it touches nine significant
concyclic points throughout the triangle.
These nine points are as follows:
•• Mid-point of each side of the triangle
•• Foot of each altitude
•• Mid-point of the line segment from each vertex of the triangle to the orthocentre.

Figure 8.13

PLANCESS CONCEPTS

•• The centroid, incentre, orthocentre and circumcentre coincide in an equilateral triangle.


•• In an isosceles triangle, centroid, orthocentre, incentre and circumcentre lie on the same line.
•• Orthocentre, centroid and circumcentre are always collinear, and centroid divides the line joining
orthocentre and circumcentre in the ratio 2:1.

Saurabh Gupta (JEE 2010, AIR 443)

Illustration 4: If G be the centroid of the triangle ABC, prove that AB2 + BC2 + CA2 = 3(GA2 + GB2 + GC2).
 (JEE MAIN)

Sol: Distance formula of two points can be used to prove AB2 + BC2 + CA2 = 3(GA2 + GB2 + GC2).
In triangle ABC, let B be the origin and BC the x-axis. Let A be (h, k) and

a+h k  A(h, k)
C be (a, 0). Then centroid G is  , .
 3 3
LHS
= AB2 + BC2 + CA2 = (h – 0)2 + (k – 0)2 + a2 + (h – a)2 + (k – 0)2
C (a, 0)
= 2h2 + 2k2 + 2a2 – 2ah B

Figure 8.14
M a them a tics | 8.7

∴ RHS
 a + h 2
 k
2
 a+h
2
 k
2
 a+h
2
 k 
= 3  − h +  − k  +  − 0 +  − 0 +  − a  +  − 0 
 3  3   3  3   3  3  

= 1/3 [(a – 2h)2 + 4k2 + (a + h)2 + k2 + (h – 2a)2 + k2] = 2h2 + 2k2 + 2a2 – 2ah
Hence, it is equal on both sides.
C
y
5.7 Area of a Triangle
Let A, B, C be the vertices of the triangle having (x1, y1), (x2, y2) and (x3, y3) as their B

respective co-ordinates. Draw AL, BM, CN perpendicular to the x-axis. A

Then ∆ABC = trapezium ALNC + trapezium CNMB – trapezium ALMB

1 1 1
= (LA + NC) LN + (NC + MB) NM – (LA + MB) LM
2 2 2 x
1 1 1 O L N M
= (y1 + y3) (x3 – x1) + (y3 + y2) (x2 – x3) – (y1 + y2) (x2 – x1)
2 2 2
Figure 8.15
1
= {x1(y2 – y3) + x2(y3 – y1) + x3(y1 – y2)}
2
x1 y1 1
1
= x2 y2 1
2
x3 y3 1

If the area of the triangle formed by the points (x1, y1), (x2, y2) and (x3, y3) is zero, the points lie on a straight line.
Using this, we can determine whether three point are in a straight line. i.e. the condition for (x1, y1), (x2, y2) and (x3,

x1 y1 1
y3) to be collinear is that x2 y 2 1 = 0.
x3 y3 1
1
For example, the area of the triangle formed by the points (1, 4), (3, –2) and (–3, 16) is {1(–2 –16) + 3} {(16 – 4) – 3
(4 + 2)} = 0. The three points lie on a straight line. 2

Illustration 5: The vertices of a triangle ABC are A(p2, –p), B(q2, q) and C(r2, –r). Find the area of the triangle.
 (JEE MAIN)
x1 y1 1
1
Sol: Area of the triangle formed by the points (x1, y1), (x2, y2) and (x3, y3) is x2 y 2 1 . Substituting the given
co-ordinates, we can obtain area of given triangle. 2
x y 1
3 3

p2 −p 1 p2 − q2 −(p + q) 0 p − q −1 0
1 2 1 2 2 1
D= q q 1 = q −r q+r 0 = (p + q) (q + r) q − r 1 0
2 2 2 2
r −r 1 r2 −r 1 r2 −r 1

1 1
= (p + q) (q + r) [(p – q) + (q – r)] = (p + q) (q + r) (p – r)
2 2
8 . 8 | Straight Line

Note:
(a) If area of the triangle is zero, then the three points are collinear.
(b) The area of a polygon with vertices A1(xi, yi), i = 1, …, n (vertices taken in anti-clockwise order)
1
[(x y – x2y1) + (x2y3 – x3y2) +…+(xny1 – x1yn)]
2 1 2

6. LOCUS
Locus is a set of points which satisfies a given geometrical data. Thus, for example, locus of a point moving at a
constant distance from a given point is a circle. Locus of a point which is equidistance from two fixed points is a
perpendicular bisector of the line joining the two points.
All the points in a locus can be represented by an equation. For example,
(a) If the distance of the point (x, y) from (2, 3) is 4, then
(x – 2)2 + (y – 3)2 = 42.
i.e. x2 + y2 – 4x – 6y – 3 = 0.
This equation will represent a circle with its centre at (2, 3) and radius 4.

(b) If (x, y) be the point equidistant from the points (3, 4) and (2, 1), then
(x – 3)2 + (y – 4)2 = (x – 2)2 + (y – 1)2
i.e. x + 3y = 10.
From the geometrical constraint, which governs the motion, we can find a relation (locus) between the co-
ordinates of the moving point in any of its positions. Equation of locus is therefore merely on equation
relating the x and y co-ordinates of every point on the locus.

Steps to find locus


(i) Assume the co-ordinates of point for which locus is to be determined as (h, k).
(ii) Apply the given geometrical conditions.
(iii) Transform the geometrical conditions into algebraic equation and simplify.
(iv) Eliminate variables (if any).
(v) Replace h → x and k → y to get the equation of locus.

Note:
•• Locus should not contain any other variables except x and y.
•• The algebraic relation between x and y satisfied by the co-ordinates at every point
on the curve and not off the curve is called the equation of curve.

y
Illustration 6: Find the equation of locus of a point which moves so that its distance P(x, y)
(0, 1)
from the point (0, 1) is twice the distance from x-axis. (JEE MAIN)
N
Sol: Here we can obtain the equation of locus of given point by using given condition
and distance formula of two points.
x
Let the co-ordinates of such a point be (x, y). Draw PM ⊥ to x-axis. x’ 0 M
y’
Hence, PM = y
Figure 8.16
PN = 2PM (given)
M a them a tics | 8.9

i.e. (x – 0)2 + (y – 1)2 = 4y2


i.e. x2 – 3y2 – 2y + 1 = 0.

Illustration 7: Locus of the centroid of the triangle whose vertices are (a cos t, a sin t), (b sin t, – b cos t) and (1, 0),
where t is a parameter is  (JEE MAIN)
(A) (3x – 1)2 + (3y)2 = a2 + b2 (B) (3x + 1)2 + (3y)2 = a2 + b2

(C) (3x + 1)2 + (3y)2 = a2 – b2 (D) (3x – 1)2 + (3y)2 = a2 – b2

Sol: The centroid (h, k) of a triangle formed by points (x1, y1), (x2, y2) and (x3, y3) will be
x1 + x2 + x3 y1 + y 2 + y 3
h = and k .
3 3
(A) If (h, k) is the centroid, then

acos t + bsint + 1 asint − bcos t + 0


h= ,k= ⇒ (3h – 1)2 + (3k)2 = (a cos t + b sin t)2 + (a sint t – b cos t)2 = a2 + b2
3 3
∴ Locus of (h, k) is (3x – 1)2 + (3y)2 = a2 + b2

7. STRAIGHT LINE
Definition: It is defined as the locus of a point such that any two points of this locus have a Y
constant inclination (gradient).
Inclination: If a straight line intersects the x-axis, the inclination of the line is defined

as the measure of the smallest non-negative angle which the line makes with the X’
positive direction of the x-axis 180- 
 π
Slope (or gradient): If the inclination of a line (i.e. non-vertical line) is θ and  θ ≠  ,
then the slope of a line is defined to be tan θ and is denoted by m.  2
Y’
∴ m = tanq Figure 8.17
(a) Slope of x-axis is zero.
(b) Slope of y-axis is not defined.

7.1 Slope
(x2, y2)Q l
Let P(x1, y1) and Q(x2, y2) be two points on a line, then slope will be
y2 - y1
(x1, y1)P
MQ M
m
= tan=
θ x2 - x 1
MP
y 2 − y1 
m
= tan=
θ
x2 − x1 O
x y 1
Figure 8.18
(a) Line passing through two points (x1, y1) and (x2, y2) is x1 y1 1 = 0
x2 y2 1

Note: Above-mentioned matrix form is a condition for three points to be collinear. (x1, y1) (x2, y2)
(b) Equation of the median through A(x1, y1) is
Figure 8.19
8 . 1 0 | Straight Line

A(x1, y1)

P(x, y)

B C
M (x3, y3)
(x2, y2)
( x2 +x3 y2 +y3
2
,
2 )
Figure 8.20

x y 1 x y 1 x y 1
x1 y1 1 = 0 or x1 y1 1 + x1 y1 1 = 0
x 2 + x3 y 2 + y3 x2 y2 1 x3 y3 1
1
2 2

(c) Equation of internal and external angle bisectors of A are

A(x1, y1)
x y 1 x y 1
b x1 y1 1 ± c x1 y1 1 =
0

P(x, y)
x2 y2 1 x3 y3 1
P(x, y)

(x2, y2)B b E
D C(x3, y3)

( bx2 + cx3 by2 + cy3


b+c
,
b+c ) ( bx2 - cx3 by2 - cy3
b-c
,
b-c )
Figure 8.21

7.2 Angle between Two Lines


Two lines intersecting each other make two angles between them, one acute and the other obtuse. Figure 8.22
shows lines L1 and L2 intersecting each other, acute angle θ and obtuse angle φ.
Let line L1 makes angle q1 with x-axis and L2 makes q2.

y
L2

L1




1 -2
A x
O B

Figure 8.22
M a them a ti cs | 8.11

Therefore slope of L1 is m1= tan q1


Slope of L2 is m2 = tanq2
Now in ∆ABC,
q1 + π – q2 + θ = π
θ = θ2 – θ1
tan θ2 − tan θ1 m2 − m1
tanθ = tan (q2 – q1) ⇒ tanθ = ⇒ tan θ = , this gives the acute angle between lines.
1 + tan θ2 . tan θ1 1 + m1m2
Note:
(i) If m1 = m2, then θ = 0º, i.e. lines are parallel or coincident.
(ii) If m1m2 = −1, then θ = 90 º , i.e. lines are perpendicular to each other.

π 1
Illustration 8: If the angle between two lines is and slope of one of the lines is . Find the slope of the other.
 4 2 (JEE MAIN)
m2 – m1
Sol: We know that, tan θ = , where m1 and m2 are the slope of lines and θ is the angle between them.
1+m1m2

1 π π m – (1 / 2) m – (1 / 2)
Let m1 = , m2 = m and θ = So, tan = ⇒ 1 =± ⇒ m = 3 or –(1/3)
2 4 4 1+(1 / 2)m 1+(1 / 2)m

Illustration 9: Line through the point (–2, 6) and (4, 8) is perpendicular to the line through the point (8, 12) and
(x, 24). Find the value of x.  (JEE MAIN)

Sol: Given two lines are perpendicular to each other. Therefore, product of their slope will be -1.
8–6 2 1
Slope of the line through the points (–2, 6) and (4, 8) is m1= = =
4 – (–2) 6 3
24 – 12 12
Slope of the line through the points (8, 12) and (x, 24) is m2 = =
x–8 x–8
Since two lines are perpendicular m1m2 = – 1
1 12
⇒ × = –1 ⇒x=4
3 x–8

7.3 Collinearity

C
B

Figure 8.23

If three points A, B, C are collinear, then


Slope of AB = Slope of BC = slope of AC
8 . 1 2 | Straight Line

PLANCESS CONCEPTS

Collinearity of three given points:


Three given points A, B, C are collinear if any one of the following conditions is satisfied.
•• Area of triangle ABC is zero.
•• Slope of AB = Slope of BC = Slope of AC.
•• AC = AB + BC.
•• Find the equation of the line passing through two given points, if the third point satisfies the equation of
the line, then three points are collinear.
1
If any one line is parallel to y-axis, then the angle between two straight lines is given by tan θ = ± ,
where m is the slope of other straight line. m

A line of gradient m is equally inclined with the two lines of gradient m1 and m2.
m1 – m m2 – m
Then =- .
1+m1m 1+m2m
Aman Gour (JEE 2012, AIR 230)

7.4 Equation of a line


(a) Point slope form: Suppose P0(x0, y0) is a fixed point on a non-vertical line L whose P(x, y)
slope is m. Let P(x, y) be an arbitrary point on L. Then by definition, the slope of L
y – y0 P0(x0, y0)
is given by m = ⇒ y – y 0 = m(x – x0 )
x – x0
This is called point slope form of a line.
Figure 8.24
(b) Two point form: Let line L passes through two given points P1(x1, y1) and P2(x2, y2).
Let P(x, y) be a point on the line. So slope P1P = slope P1P2
y – y1 y 2 – y1
⇒ = P2(x2, y2)
x – x1 x2 – x1
P(x, y)
y 2 – y1
⇒ y – y1 = (x – x1 ) P1(x1, y1)
x2 – x1

This is called two-point form of the line.


Figure 8.25

(c) Slope intercept from: Case-I: If slope of line is m and makes y-intercept c, then equation is
(y – c) = m (x – 0) ⇒ y = mx + c
Case-II: If slope of line is m and makes x-intercept d, then equation is
y = m(x – d)
These equations are called slope intercept form. (0, c)

(d, 0)

Figure 8.26
M a them a ti cs | 8.13

(d) Intercept form: Suppose a line L makes intercept a on x-axis and intercept b on L
y-axis, i.e. the line meets x-axis at (a, 0) and y-axis (0, b). (0, b)

b–0
So, y – 0 = (x – a) b
0–a
x y
i.e. + = 1 . This is called intercept form of the line.
a b
a (a, 0)

Figure 8.27

(e) Normal form: If P is perpendicular distance from origin to the line AB and makes angle α with x-axis, then
equation of the line is x cos α + y sin α = P
OM y
Proof: cos α = OM = OL cos α = x cos α
OL
B
In DPNL,
PN
sin α =
PL
Q
PN = PL sin α = y sin α M P(x,y)
90-
MQ = PN = y sin α 
N 
x
O x A
Now P = OQ = OM + MQ = x cos α + y sin α L
Figure 8.28
So x cos α + y sin α = P

This is called normal form of the line.

(f) Parametric form or distance form: The equation of the line passing through (x1, y1) and making an angle θ
with the positive x-axis is
x – x1 y – y1
= = r , where ‘r’ is the signed value.
cos θ sin θ

Hence, the co-ordinate of any point at a distance r on this line is

x = x1 + r cos θ y = y1 + r sin θ

PLANCESS CONCEPTS

Point of intersection of two lines a1x + b1y + c1 = 0 and a2x + b2y + c2 = 0 is given by
 b1 b2 c1 c2 
 
 b1c2 – b2c1 c1a2 – c2a1   c1 c2 a1 a2 
( x', y')=  a b – a b , a b – a b  =  a1 a2
,
a1 a2 
 1 2 2 1 1 1 2 1  
 b1 b2 b1 b2 

Saurabh Gupta (JEE 2010, AIR 443)


8 . 1 4 | Straight Line

Illustration 10: A straight line is drawn through the point P(2, 3) and is inclined at an angle of 30° with positive
x-axis. Find the co-ordinate of two points on it at a distance 4 from P on either side of P.  (JEE MAIN)
x – x1 y – y1
Sol: By using formula = = r , we can obtain co-ordinates of point.
cos θ sinθ
The equation of line
x – x1 y – y1 x–2 y –3
= = ±r ⇒ = = ± 4 ⇒ x = 2 ± 2 3, y = 3 ± 2
cos θ sinθ cos 30° sin 30°

So, co-ordinate of two points are 2± 2 3 ,3± 2 ( )


Illustration 11: If two vertices of a triangle are (–2, 3) and (5, –1). Orthocentre of the triangle lies at the origin and
centroid on the line x + y = 7, then the third vertex lies at  (JEE MAIN)
(A) (7, 4) (B) (8, 14)
(C) (12, 21) (D) None of these

Sol: (D) The line passing through the third vertex and orthocentre must be perpendicular to line through (–2, 3)
and (5, –1). Therefore, product of their slope will be -1.
Given the two vertices B(–2, 3) and C(5, – 1); let H(0, 0) be the orthocentre; A(h, k) the third vertex.
Then, the slope of the line through A and H is k/h, while the line through B and C has the slope
(–1 –3)/(5 + 2) = – 4/7. By the property of the orthocentre, these two lines must be perpendicular,
 k  4  k 7
So we have    −  = – 1 ⇒ =  … (i)
h
  7  h 4

5 − 2 + h −1 + 3 + k
Also + = 7 ⇒ h + k = 16 … (ii)
3 3
Which is not satisfied by the points given in (A), (B) or (C).

Illustration 12: In what direction should a line be drawn passing through point (1, 2) so that its intersection point
6
with line x + y = 4 is at a distance of units.  (JEE ADVANCED)
3
Sol: By using x = x1 + r cos θ and y = y1 + r sin θ, we can obtain the required angle.
For co-ordinates of B

6 6 6
Substitute r= ∴ x = 1+ cos θ & =
y 2+ sin θ B
3 3 3
A(1,2)
Substituting in x + y =4 6
 3
6 6 3
⇒ 1+ cos θ + 2+ sin θ = 4 ∴ (cos θ + sin θ) =
3 3 6
1 1 3 1
∴ cos θ + sin θ = (Multiple by )
2 2 2 2 Figure 8.29

sin (45° + θ) = sin 60°


∴ θ = 15°
or, sin (45° + θ)= sin 120° ∴ θ = 75°
M a them a ti cs | 8.15

Illustration 13: If sum of the distances of the points from two perpendicular lines in a plane is 1, then find its locus.
(JEE ADVANCED)
Sol: If (h, k) be any point on the locus, then |h| + |k| = 1
Y
Let the two perpendicular lines be taken as the co-ordinate axes.
⇒ locus of (h, k) is |x| + |y| = 1 (0, 1)

This consists of four line segments which enclose a square as -1

x
y=

+
y
x-
shown in figure.

=1
X
 (-1, 0) O (1, 0)

x
=1

+
y

y
x-

=1
(0, -1)

Figure 8.30

Illustration 14: If the circumcentre of a triangle lies at the origin and the centroid is the mid-point of the line
joining the points (a2 + 1, a2 + 1) and (2a, –2a), then the orthocentre lies on the line.  (JEE ADVANCED)
(A) y = (a2 + 1)x (B) y = 2ax (C) x + y = 0 (D) (a – 1)2 x – (a + 1)2 y = 0

Sol: (D) We know from geometry that the circumcentre, centroid and orthocentre of a triangle lie on a line. So the
 (a + 1)2 (a − 1)2 
orthocentre of the triangle lies on the line joining the circumcentre (0, 0) and the centroid  , 
 2 2 
2 2 
(a + 1) (a − 1)
y= x or (a – 1)2 x – (a + 1)2 y = 0.
2 2

PLANCESS CONCEPTS

Equation of parallel and perpendicular lines:


• Equation of a line which is parallel to ax + by + c = 0 is ax + by + k = 0.
• Equation of a line which is perpendicular to ax + by + c = 0 is bx – ay + k = 0.
• If y = m1x + c1, y = m1x + c2, y = m2x + d1 and y = m2x + d2 are sides of a parallelogram then its
(c1 – c2 )(d1 – d2 )
area is .
m1 – m2
x y
• The equation of a line whose mid-point is (x1, y1) in between the axes is + =2.
x1 y1
c2
• Area of the triangle made by the line ax + by +c = 0 with the co-ordinate axes is .
2 | ab |

• A line passing through (x1, y1) and if the intercept between the axes is divided in the ratio m:n at this
nx my
point then the equation is + =m + n .
x1 y1
• The equation of a straight line which makes a triangle with the co-ordinates axes whose centroid is
x y
(x1, y1) is + =1.
3x1 3y1
B Rajiv Reddy (JEE 2012, AIR 11)
8 . 1 6 | Straight Line

7.5 Foot of the Perpendicular


A(x1, y1) ax+by+c=0
The foot of the perpendicular (h, k) from (x1, y1) to the line ax + by + c = 0 is given by
h – x1 k – y1 –(ax1 +by1 + c)
= = .
a b a2 +b2
Hence, the co-ordinates of the foot of perpendicular is

 b2 x – aby – ac a2 y – abx – bc 
 1 1
, 1 1 .
 a2
+b 2
a2
+b 2  B(h, k)
 
Figure 8.31

The image of a point with respect to the line mirror: The image of A(x1, y1) with respect to the line mirror ax +
h – x1 k – y1 –2(ax1 +bx1 + c)
by + c = 0, B (h, k) is given by = =
a b a2 +b2 .
Special Cases
(a) Image of the point P(x1, y1) with respect to x-axis is (x1, – y1).
(b) Image of the point P(x1, y1) with respect to y-axis is (– x1, y1).
(c) Image of the point P(x1, y1) with respect to the line mirror y = x is Q(y1, x1).
(d) Image of the point P(x, y) with respect to the origin is the point (-x, -y).

Illustration 15: Find the equation of the line which is at a distance 3 from the origin and the perpendicular from
the line makes an angle of 30° with the positive direction of the x-axis.  (JEE MAIN)

Sol: By using xcosα + ysinα = P, we can solve this problem. Here α = 30° and P = 3.
3 y
So equation is x cos 30° + y sin 30° = 3 x + =3 ⇒ 3x + y = 6
2 2
Position of a point w.r.t. a line
Let the equation of the given line be ax + by + c = 0 and let the co-ordinates of the two given points be P(x1, y1)
and Q(x2, y2). Let R1 be a point on the line.
m –ax1 – by1 – c
The co-ordinates of R1 which divides the line joining P and Q in the ratio m:n are =
n ax2 +by 2 + c
.

Thus, the two points (x1, y1) and (x2, y2) are on the same (or opposite) sides of the straight line ax + by + c = 0
m
whether Point R1 divides internally or externally or sign of .
n
Note:
⇒ A point (x1, y1) will lie on the side of the origin relative to a line ax + by + c = 0, if ax1 +by1 + c and c have the
same sign.
⇒ A point (x1, y1) will lie on the opposite side of the origin relative to the line ax + by + c = 0, if
ax1 + by1 + c and c have the opposite sign.

Illustration 16: For what values of the parameter α does the point M (α, α + 1) lies within the triangle ABC of
vertices A(0, 3), B(– 2, 0) and C(6, 1).  (JEE ADVANCED)
Sol:Here, the point M will be inside the triangle if and only if |Area ∆MBC| + |Area ∆MCA| + |Area ∆MAB| = |Area
∆ABC|. And each individual area must be non-zero.
M a them a ti cs | 8.17

α α +1 1
1 1
Area MBC = –2 0 1 = 7α + 6
2 2
6 1 1

α α +1 1
1 1
Area MCA = 6 1 1 = –8α +12
2 2
0 3 1

α α +1 1
1 1
Area MAB = 0 3 1 = α+4
2 2
–2 0 1
Figure 8.32
0 3 1
1 1
Area ABC = –2 0 1 = .22
2 2
6 1 1
6 3
The above equation has critical points – 4, – and .
7 2
For α ≤ –4, the equation is
–7α – 6 – 8α + 12 – α – 4 =22
5 5
⇒ α=– which is not a solution, since – > –4
4 4
 6 6
For a∈  –4, –  , then equation is – 7α – 6 – 8α + 12 + α + 4 = 22 ⇒ α = –
 7 7
6
which is solution of equation but area MBC = 0 ⇒ M lies on BC ⇒ α = – is not the desired value.
7
 6 3
For a∈  – ,  , the equation is 7α + 68α + 12 + α + 4 = 22.
 7 2
 6 3
⇒ All α in the interval  – ,  satisfy the equation.
 7 2
3  3
Finally over  , ∞  , we get α = implies area MCA become zero.
 2  2
 6 3
⇒ The desired values of α lie in the interval  – ,  .
 7 2

7.6 Length of the Perpendicular P(x1,y1)


The perpendicular distance ‘p’ of a point P(x1, y1) from the line p
| ax1 +by1 + c |
ax + by + c = 0 is p = ax+by+c
a2 +b2 M
Figure 8.33
(a) Distance between parallel lines: The distance between the parallel lines a

x+ by + c1 = 0 and ax + by + c2 = 0 is c1 – c2
a2 +b2 y=mx+c
( 
(b) Lines making angle α with given line: The equations of the two straight lines (

passing through P(x’, y’) and making an angle α with the line y = mx + c P(x’,y’)

(where m = tan θ) are Figure 8.34


8 . 1 8 | Straight Line

y – y’ = tan (θ + α) (x – x’)
π
Note: If θ + α or θ – α is an odd multiple of , the corresponding line has equation x = x’.
2

(c) Concurrency of lines: Lines aix + biy + c1 = 0, where i = 1, 2, 3 are concurrent if they meet at a point. The

a1 b1 c1
condition for concurrency is
a2 b2 c2 = 0
a3 b2 c3

Illustration 17: The equation of the two tangents to the circle are 3x – 4y + 10 = 0 and 6x – 8y + 30 = 0. Find
diameter of the circle.  (JEE MAIN)

c1 – c2
Sol: By using formula of distance between two parallel line, i.e. , we can find the
a2 +b2
diameter of given circle. d

These are two parallel lines


3x – 4y + 10 = 0 .....(i) Figure 8.35

6x – 8y + 30 = 0 .....(ii)
15 − 10
Dividing second equation by 2 gives 3x – 4y + 15 = 0; ∴ d = =1
32 + 42

PLANCESS CONCEPTS

(a) A triangle is isosceles if any two of its median are equal.


(b) Triangle having integral co-ordinates can never be equilateral.
(c) If arx + bry + cr = 0 (r = 1, 2, 3) are the sides of a triangle then the area of the triangle is given
2
a1 b1 c1
1
by a2 b2 c2 where C1, C2 and C3 are the cofactors of c1, c2 and c3 in the determinant.
2C1C2C3
a3 b3 c3

(d) Area of parallelogram:


(i) Whose sides are a and b and angle between them is θ is given by ab sin q. Area of
ABCD = ab.sinθ a
D C

b b

A a B

Figure 8.36

(ii) Whose length of perpendicular from one vertices to the opposite sides are p1 and p2 and angle
PP
between sides is θ is given by Area = 1 2
Sin θ

p2
p1
θ

Figure 8.37

Krishan Mittal (JEE 2012, AIR 199)


M a them a ti cs | 8.19

8. FAMILY OF LINES
Consider two intersecting lines L1: a1x + b1y + c1 = 0 and L2: a2x + b2y + c2 = 0, then
Type-1: The equation of the family of lines passing through the intersection of the lines
L1 + λL2 = 0
⇒ (a1x +b1y + c1) + λ(a2x +b2y + c2) = 0 where λ is a parameter.
Type-2: Converse, L1 + λL2 = 0 is a line which passes through a fixed point, where L1 = 0 and L2=0 are fixed lines
and the fixed point is the intersection of L1 and L2.

Type-3: Equation of AC ≡ u2u3 – u1u4 = 0 and BD ≡ u3u4 – u1u2= 0

u1a1x+b1y+c1=0
B C

u2a2x+b2y+d2=0 u4a2x+b2y+d1=0

A D
u3a1x+b1y+c2=0
Figure 8.38

Note that second degree terms cancel and the equation u2u3 – u1u4 = 0 is satisfied by the co-ordinate points B
and D.

Illustration 18: If a, b, c are in A.P., then prove that the variable line ax + by + c = 0 passes through a fixed point.
 (JEE MAIN)
Sol: By using given condition we can reduce ax + by + c = 0 to as L1 + λL2 = 0. Hence we can obtain co-ordinate
of fixed point by taking L1 = 0 and L2=0.
2b =a + c ⇒ c = 2b – a ⇒ ax + by + 2b – a = 0
∴ a (x – 1) + b(y + 2) = 0 This is of the form L1 + λL2 = 0, where b/a = l
∴ Co-ordinates of fixed point is (1, – 2).

9. ANGULAR BISECTOR

9.1 Bisectors of the Angle Between Two Lines


(a) Equations of the bisectors of angle between the lines ax + by + c = 0 and a1x + b1y + c1 = 0 are
ax +by + c a1 x +b1 y + c1
=± (ab1 ≠ a1b)
a2 +b2 a12 +b12
(b) To discriminate between the bisectors of the angle containing the origin and that of angle not containing
the origin, rewrite the equations, ax + by + c = 0 and a’x+ b’y + c’ = 0 such that the terms c, c’ are positive,
ax +by + c a'x +b'y + c'
then =+ gives the equation of the bisector of the angle containing origin and
2 2
a +b a'2 +b'2
ax +by + c a'x +b'y + c'
=– gives the equation of the bisector of the angle not containing origin.
2 2
a +b a'2 +b'2

(c) Acute angle bisector and obtuse angle bisector can be differentiated from the following methods:
Let two lines a1x + b1y + c1 = 0 and a2x + b2y + c2 = 0 intersect such that constant terms are positive.
8 . 2 0 | Straight Line

If a1a2 + b1b2 < 0, then the angle between the lines that contain the origin is acute and the equation for
a x +b1 y + c a x +b2 y + c2 ax1 +by1 + c a x +b2 y + c2
the acute angle bisector is 1 =+ 2 . Therefore =– 2 is the
2 2 2 2 2 2
a +b a2 +b2 a +b a22 +b22
equation of other bisector. If, however, a1a2 + b1b2 > 0, then the angle between the lines containing the origin
a1 x +b1 y + c1 a2 x +b2 y + c2
is obtuse and the equation of the bisector of the obtuse angle is =+ ; therefore
a1 x +b1 y + c1 a2 x +b2 y + c2 a12 +b12 a22 +b22
=– is acute angle bisector.
a12 +b12 a22 +b22

(d) Few more methods of identifying an acute and obtuse angle bisectors are as follows:
Let L1 = 0 and L2 = 0 are the given lines and u1 = 0 and u2 = 0 are the bisectors L1=0
between L1 = 0 and L2 = 0. Take a point P on any one of the lines L1 = 0 or L2 = 0 and
draw a perpendicular on u1 = 0 and u2 = 0 as shown. If P p
q
|p| < |q| ⇒ u1 is the acute angle bisector. L2=0

|p| > |q| ⇒ u1 is the obtuse angle bisector.


|p| = |q| ⇒ the lines L1 and L2 are perpendicular.

Note: The straight lines passing through P(x1, y1) and equally inclined with the lines u2=0
u1=0
a1x + b1y + c1 = 0 and a2x + b2y + c2 = 0 are those which are parallel to the bisectors
between lines and passing through the point P. Figure 8.39

PLANCESS CONCEPTS

(a) Algorithm to find the bisector of the angle containing the origin: Let the equations of the
two lines be a1x + b1y + c1 = 0 and a2x + b2y + c2 = 0. The following methods are used to find the
bisector of the angle containing the origin:
Step I: In the equations of two lines, check if the constant terms c1 and c2 are positive. If the terms are
negative, then make them positive by multiply both the sides of the equation by –1.

Step II: Obtain the bisector corresponding to the positive sign, i.e. a1 x +b1 y + c1 a2 x +b2 y + c2
=
a12 +b12 a22 +b22
L1

Acute bisector

L2

Obtuse bisector
Figure: 8.40

This is the required bisector of the angle containing the origin, i.e. the bisectors of the angle between the
lines which contain the origin within it.
(b) Method to find acute angle bisector and obtuse angle bisector
(i) Make the constant term positive by multiplying the equation by –1.
(ii) Now determine the sign of the expression a1a2 + b1b2.
M a them a ti cs | 8.21

PLANCESS CONCEPTS

(iii) If a1a2 + b1b2 > 0, then the bisector corresponding to ‘+ve’ and ‘–ve’ signs give the obtuse and acute
angle bisectors, respectively, between the lines.
(iv) If a1a2 +b1b2 < 0, then the bisector corresponding to ‘+ve’ and ‘–ve’ signs give the acute and obtuse
angle bisectors, respectively.
Both the bisectors are perpendicular to each other. If a1a2 + b1b2 > 0, then the origin lies in the
obtuse angle and if a1a2 + b1b2 < 0, then the origin lies in the acute angle.

T P Varun (JEE 2012, AIR 64)

PLANCESS CONCEPTS

Incentre divides the angle bisectors in the ratios (b + c):a, (c + a):b and (a + b):c . Angle bisector divides
the opposite sides in the ratio of remaining sides.

Figure: 8.41
BD AB c
= =
DC AC b
Aishwarya Karnawat (JEE 2012, AIR 839)

9.2 Bisectors in Case of Triangle


Two possible models are as follows:
Case-I: When vertices of a triangle are known, compute the sides of the
A(x1,y1)
triangle and the incentre. All the internal bisectors can be known, using the
co-ordinates of incentre and vertices of triangle.
Note: If the triangle is isosceles/equilateral, then one can easily get the c b
incentre.
Case-II: When the equations of the sides are given,compute tan A, tan B,
(x2,y2)B C(x3,y3)
tan C by arranging the lines in descending order of their slope. Compute the a
acute/obtuse angle bisectors as the case may be. Plot the lines approximately
Figure 8.42
and bisectors containing or not containing the origin.
8 . 2 2 | Straight Line

Illustration 19: The line x + y =a meets the x- and y-axes at A and B, respectively. A triangle AMN is inscribed in
the triangle OAB, O being the origin, with right angle at N. M and N lie respectively on OB and AB. If the area of
the triangle AMN is 3/8 of the area of the triangle OAB, then AN/BN is equal to. (JEE ADVANCED)
(A) 3 (B) 1/3 (C) 2 (D) 1/2

Sol: (A) Here simply by using the formula of area of triangle, Y


1 B(0, a)
i.e. {x1(y2 – y3) + x2(y3 – y1) + x3(y1 – y2)} , we can solve the problem.
2
N
AN  a λa 
Let = λ . Then the co-ordinates of N are  , ,
BN  1 + λ 1 +λ M
where (a, 0) and (0, a) are the co-ordinates of A and B, respectively.
Now equation of MN perpendicular to AB is X
O A(a, 0)
λa a 1–λ  λ –1 
y– =x– or x – y = a So the co-ordinates of M are  0, a
1+ λ 1+ λ 1+λ  λ +1  Figure 8.43

Therefore, area of the triangle AMN is

1   –a  1 – λ 2 λa2
= a
   + a  =
2   λ + 1  (1 + λ )
2
 (1 + λ )2

Also area of the triangle OAB = a2/2.


λa2 3 1 2
So that according to the given condition: = . a ⇒ 3l2 – 10λ + 3 = 0 ; ⇒ λ = 3 or λ = 1/3.
(1 + λ ) 2 8 2
For λ = 1/3, M lies outside the segment OB and hence the required value of λ is 3.

10. PAIR OF STRAIGHT LINES

10.1 Pair of Straight Lines Through Origin


(a) A homogeneous equation of degree two of the type ax2 + 2hxy + by2 = 0 always represents a pair of straight
lines passing through the origin and if
(i) h2 > ab ⇒ lines are real and distinct.
(ii) h2 = ab ⇒ lines are coincident.
(iii) h2 < ab ⇒ lines are imaginary with real point of intersection, i.e. (0, 0)
(b) If y = m1x and y = m2x be the two equations represented by ax2 + 2hxy + by2 = 0, then

2h a
m1 + m2 =
– and m1m2 =
b b

Angle between two straight lines:


(c) If θ is the acute angle between the pair of straight lines represents by ax2 + 2hxy + b, then

2 h2 – ab
tan θ =
a+b

The condition that these lines are:


(i) At right angles to each other if a + b = 0, i.e. sum of coefficients of x2 and y2 is zero.
M a them a ti cs | 8.23

(ii) Coincident if h2 = ab and (ax2 + 2hxy + by2) is a perfect square of ( ax + by)2 .


(iii) Equally inclined to the axis of x if h = 0, i.e. coefficient of xy =0.

Combined equation of angle bisectors passing through origin: The combined equation of the bisectors of
the angles between the lines ax2 + 2hxy + by 2 =
0 (a pair of straight lines passing through origin) is given by
x2 – y 2 xy
= .
a–b h

10.2 General Equation for Pair of Straight Lines


ax2 + 2hxy + by 2 + 2gx + 2fy + c =
(a) 0 represents a pair of straight lines if:
a h g
2 2 2
abc + 2fgh – af – bg – ch = 0 , if h b f = 0
g f c

(b) The slope of the two lines represented by a general equation is the same as that between the two lines
represented by only its homogeneous part.

10.3 Homogenisation
The equation of the two lines joining the origin to the points of intersection of the line lx + my +n = 0 and the curve
ax2 + 2hxy + by 2 + 2gx + 2fy + c =0 is obtained by homogenising the equation of the curve using the equation of
the line.
The combined equation of pair of straight lines joining origin to the points y
of intersection of the line given by lx + my + n = 0  …. (i)
The second degree curve:
lx+my+n=0
ax2 + 2hxy + by 2 + 2gx + 2fy + c = 
0 …. (ii)
Using equation (i) and (ii) x
O
2
 lx + my   lx + my   lx + my 
ax2 + 2hxy + by 2 + 2gx   + 2fy   + c 0 
 = …. (iii) Figure 8.44
 –n   –n   –n 

 lx + my 
Obtained by homogenizing (ii) with the help of (i), by writing (i) in the form:   = 1.
 –n 

PLANCESS CONCEPTS

Through a point A on the x-axis, a straight line is drawn parallel to y-axis so as to meet the pair of straight
lines.
ax2 + 2hxy + by 2 =
0 in B and C. If AB = BC, then 8h2 = 9ab.
Krishan Mittal (Jee 2012, Air 199)
8 . 2 4 | Straight Line

Illustration 20: The orthocentre of the triangle formed by the lines xy = 0 and x + y = 1 is  (JEE MAIN)
(A) (1/2, 1/2) (B) (1/3, 1/3) (C) (0, 0) (D) (1/4, 1/4)

Sol: (C) Here the three lines are x = 0, y = 0 and x + y = 1.


Since the triangle formed by the line x = 0, y = 0 and x + y = 1 is right angled, the orthocentre lies at the vertex
(0, 0), the point of intersection of the perpendicular lines x = 0 and y = 0.

Illustration 21: If θ is an angle between the lines given by the equation 6x2 + 5xy – 4y 2 + 7x + 13y – 3 = 0 then
equation of the line passing through the point of intersection of these lines and making an angle θ with the positive
x-axis is  (JEE ADVANCED)
(A) 2x + 11y + 13 = 0 (B) 11x – 2y + 13 = 0
(C) 2x – 11y + 2 = 0 (D) 11x + 2y – 11 = 0

Sol: (B) By taking the term y constant and using the formula of roots of quadratic equation, we can get the

2 h2 – ab
equation of two lines represented by the given equation and then by using tan θ = , we will get the
required result. a+b

Writing the given equation as a quadratic in x, we have

–(5y + 7) ± (5y + 7)2 + 24(4y 2 – 13y + 3)


6x2 + (5y + 7) x – (4y 2 – 13y + 3) =
0⇒ x=
12
–(5y + 7) ± 121y 2 – 242y + 121 –(5y + 7) ± 11(y – 1) 6y – 18 –16y + 4
= = = or
12 12 12 12

⇒ 2x – y + 3 = 0 and 3x + 4y – 1 = 0 are the two lines represented by the given equation and the point of
intersection is (– 1, 1), obtained by solving these equations.

2 h2 – ab 2 (5 / 2)2 – 6(–4) 121 11


Also tan θ = , where a = 6, b = – 4, h = 5/2 = = =
a+b 6–4 4 2
11
So the equation of the required line is y=
–1 (x + 1) ⇒ 11x – 2y + 13 = 0
2

Illustration 22: If the equation of the pair of straight lines passing through the point (1, 1), and making an angle
θ with the positive direction of x-axis and the other making the same angle with the positive direction of y-axis is
x2 – (a + 2)xy + y 2 + a(x + y – 1) =
0 , a ≠ – 2, then the value of sin 2θ is  (JEE ADVANCED)
2 2
(A) a – 2 (B) a + 2 (C) (D)
(a + 2) a

Sol: (C) As both line passes through (1, 1) and one line makes angle θ with x-axis and other line with y–axis, slopes
of line are tan θ and cot θ
Equations of the given lines are y – 1 = tan θ (x – 1) and y – 1 = cot θ (x – 1)
So, their combined equation is [(y – 1) – tan θ (x – 1)] [(y – 1) – cot θ (x – 1) ] = 0
⇒ (y – 1)2 – (tan θ + cot θ) (x – 1) (y – 1) + (x – 1)2 = 0
⇒ x2 – (tan θ + cot θ) xy + y2 + (tan θ + cot θ – 2) (x + y – 1) = 0
Comparing with the given equation we get tan θ + cot θ = a + 2

1 2
⇒ = a + 2 ⇒ sin2θ =
sin θ cos θ a+2
M a them a ti cs | 8.25

Illustration 23: If two of the lines represented by x 4 + x3 y + cx2 y 2 – xy 3 + y 4 =


0 bisect the angle between the
other two, then the value c is (JEE ADVANCED)
(A) 0 (B) –1 (C) 1 (D) – 6

Sol: (D) As the product of the slopes of the four lines represented by the given equation is 1 and a pair of line
represents the bisectors of the angles between the other two, the product of the slopes of each pair is –1.
So let the equation of one pair be ax2 + 2hxy – ay2 = 0.

x2 – y 2 xy
The equation of its bisectors is = .
2a h
By hypothesis x 4 + x3 y + cx2 y 2 – xy 3 + y 4
≡ (ax2 + 2hxy – ay2) (hx2 – 2axy – hy2) = ah(x4 + y4) + 2(h2 – a2) (x3y – xy3) – 6ahx2y2
Comparing the respective coefficients, we get ah =1 and c = –6ah = –6

11. TRANSLATION AND ROTATION OF AXES Y Y’


P

11.1 Translation of Axes


O’ X’
Let OX and OY be the original axes, and let the new axes, parallel to original axes, be O’X’
and O’Y’. Let the co-ordinates of the new origin O’ referred to the original axes be (h, k). If X
the point P has co-ordinates (x, y) and (x’, y’) with respect to original and new axes,
O

respectively, then x = x’ + h; y = y’ + k Figure 8.45

11.2 Rotation of Axes


Let OX and OY be the original system of axes and let OX’ and OY’ be the new system Y
of axes and angle XOX’ = θ (the angle through which the axes are turned). If the point
P
Y’ X’
P has co-ordinates (x, y) and (x’, y’) with respect to original and new axes, respectively,
then
x = x’ cos θ – y’ sin θ and y = x’ sin θ + y’ cos θ

X
O
in matrix form it is as follows:
Figure 8.46
 x  cos θ − sin θ   x' 
 =  
 y   sin θ cos θ   y '

PLANCESS CONCEPTS

If origin is shifted to point (α, β) , then new equation of curve can be obtained by putting x + α in place
of x and y + β in placed of y.
Vaibhav Krishnan (JEE 2009, AIR 22)

Illustration 24: The line L has intercepts a and b on the co-ordinate axes. The co-ordinate axes are rotated through
a fixed angle, keeping the origin fixed. If p and q are the intercepts of the line L on the new axes, then
1 1 1 1
– + – is equal to  (JEE MAIN)
2 2 2
a p b q2
(A) –1 (B) 0 (C) 1 (D) None of these
8 . 2 6 | Straight Line

Sol: (B) By using intercept form of equation of line, we will get equation of line before and after rotation. As their
perpendicular length from the origin does not change, by using distance formula the result can be obtained.
x y X Y
Equation of the line L in the two co-ordinate system is + = 1, + = 1 Where (x, y) are the new co-ordinates
a b p q
of a point (x, y) when the axes are rotated through a fixed angle, keeping the origin fixed. As the length of the
perpendicular from the origin has not changed:
1 1 1 1 1 1 1 1 1 1
= ⇒ + = + ⇒ – + – =0
2 2 2 2 2 2 2
(1 / a2 ) + (1 / b2 ) (1 / p2 ) + (1 / q2 ) a b p q a p b q2

Illustration 25: Let 0 < α < π/2 be a fixed angle. If P = (cos θ, sin θ) and Q = (cos (α – θ), (sin (α – θ)). Then Q is
obtained from P by  (JEE ADVANCED)
(A) Clockwise rotation around origin through an angle α.
(B) Anti-clockwise rotation around origin through an angle α.
(C) Reflection in the line through the origin with slope tan α.
(D) Reflection in the line through the origin with slope tan α/2.

Sol: As we know angle decreases during clockwise rotation and increases during anticlockwise rotation.
D Clockwise rotation of P through an angle α takes it to the point (cos (θ – α), sin(θ – α)) and anticlockwise takes it
to (cos (α + θ), sin (α + θ))
sin θ – sin(α – θ) 2cos(α / 2)sin(θ – α / 2)
Now slope of PQ = = − cot ( α / 2 )
=
cos θ – cos(α – θ) –2sin(α / 2) – sin(θ – α / 2)
⇒ PQ is perpendicular to the line with slope tan(α/2). Hence, Q is the reflection of P in the line through the origin
α
with slope tan   .
2

PLANCESS CONCEPTS

RELATION BETWEEN THE COEFFICIENT


Conditions for two lines to be coincident, parallel, perpendicular and intersecting: Two lines
a1x + b1y + c1 = 0 and a2x + b2y + c2 = 0 are
a1 b1 c1
• Coincident, if = =
a2 b2 c2

a1 b1 c1
• Parallel, if = ≠
a2 b2 c2

a1 b1
• Intersecting, if ≠
a2 b2
• Perpendicular, if a1a2 + b1b2 = 0
The three lines a1x + b1y + c1 = 0, a2x + b2y + c2 = 0 and a3x + b3y + c3 = 0 are concurrent, if
a1 b1 c1
a2 b2 c2 = 0
a3 b3 c3
Aman Gour (JEE 2012, AIR 230)
M a them a ti cs | 8.27

PROBLEM-SOLVING TACTICS

(a) In most of the questions involving figures like triangle or any parallelogram, taking origin as (0,0) helps a lot
in arriving at desired solution. One must ensure that conditions given are not violated.
(b) One must remember that in an isosceles triangle, centroid, orthocentre, incentre and circumcentre lie on the
same line.
(c) The centroid, incentre, orthocentre and circumcentre coincide in an equilateral triangle.
(d) If area of the triangle is zero, then the three points are collinear.
(e) Find the equation of the line passing through two given points, if the third point satisfies the equation of the
line, then three points are collinear
(f) Whenever origin is shifted to a new point (α, β), then new equation can be obtained by putting x + α in place
of x and y + β in placed of y.

FORMULAE SHEET

(a) Distance Formula: The distance between two points P(x1, y1) and Q(x2, y2) is

PQ = (x1 – x2 )2 + (y1 – y 2 )2 = (x1 – x2 )2 + (y1 – y 2 )2

And between two polar co-ordinate A(r1, q1) and B(r2, q2) is AB = r12 + r22 − 2r1r2 cos(θ1 − θ2 )

(b) Section Formula: If P(x1, y1), Q(x2, y2) and the point R(x, y) divide the line PQ internally in the ratio m:n then
the co-ordinates of R will be
mx2 + nx1 my 2 − ny1  mx + nx1 my 2 + ny1 
x= and y = i.e. R  2 , 
m+n m−n ,  m+n m + n 
 x + x 2 y1 + y 2 
And if R is a mid-point of line PQ, then the co-ordinates of R will be  1 , 
 2 2 

(c) Centroid of Triangle: If A(x1, y1), B(x2, y2) and C(x3, y3) be the vertices of the triangle ABC and G is
 x + x 2 + x3 y1 + y 2 + y 3 
Centroid, then co-ordinate of G will be  1 ,  .
 3 3 A
  I3 I2
ax1 + bx2 + cx3 ay1 + by 2 + cy 3
(d) Co-ordinates
= of Incentre: x = , y B C
a+b+c a+b+c

(e) Co-ordinates of Ex-centre: As shown in figure, ex-centres of ∆ABC I1


with respect to vertices A, B and C are denoted by
I1, I2 and I3, respectively,
Figure 8.47

 −ax1 + bx2 + cx3 −ay1 + by 2 + cy 3   ax1 – bx2 + cx3 ay1 − by 2 + cy 3 


I1 =  ,  ; I2 =  ,  ,
−a + b + c −a + b + c  a−b + c a−b + c 
 
 ax + bx2 − cx3 ay1 + by 2 − cy 3 
I3 =  1 , 
 a+b−c a+b−c 
8 . 2 8 | Straight Line

(f) Co-ordinates of Circumcentre: If O is the circumcentre of any ∆ABC, then its co-ordinates will be

 x sin2A + x2 sin2B + x3 sin2C y1 sin2A + y 2 sin2B + y 3 sin2C 


O =  1 , 
 Σ sin2A Σ sin2A 

(g) Co-ordinates of Orthocentre: If H is the orthocentre of any ∆ABC, then its co-ordinates will be

 x tanA + x2 tanB + x3 tanC y1 tanA + y 2 tanB + y 3 tanC 


H=  1 ,
 Σ tanA Σ tanA 
 
y 2 − y1
(h) Slope of Line: Slope of line made by joining of points P(x1, y1) and Q(x2, y2) is given by m
= tan=
θ
x2 − x1
m2 − m1
(i) Angle between two Lines: tan θ =
1 + m1m2
( j) Equation of a Line:
y 2 – y1
(i) Slope point form: y – y1 = m(x – x1) ; (ii) Two point form: y – y1 = (x – x1 )
x2 – x1
x y
(iii) Slope intercept form: y = mx + c; (iv) Intercept form: + =1
a b
(v) Normal form: x cos α + y sin α = P
x – x1 y – y1
(vi) Parametric form or distance form: = = r ; and x = x1 + r cos θ and y = y1 + r sin θ
cos θ sin θ

(k) Length of Perpendicular: The perpendicular distance ‘p’ of a point P(x1, y1) from the line ax + by + c = 0 is
| ax1 +by1 + c |
p =
a2 +b2
c1 – c2
(i) Distance between parallel lines:
a2 +b2
(ii) Lines making angle α with given line: y – y’ = tan (θ + α) (x – x’) and y – y’ = tan (θ – α) (x – x’)
a1 b1 c1
(iii) Concurrency of lines: The lines are concurrent if a2 b2 c2 = 0
a3 b2 c3

ax +by + c a1 x +b1 y + c1
(l) Equation of bisector of the angle between two lines: =± (ab1 ≠ a1b)
a2 +b2 a12 +b12
(m) Pair of straight line:
(i) ax2 + 2hxy + by2 = 0 always represents a pair of straight lines passing through the origin and if
•• h2 > ab ⇒ lines are real and distinct.
•• h2 = ab ⇒ lines are coincident.
•• h2 < ab ⇒ lines are imaginary with real point of intersection, i.e. (0, 0)
2h a
(ii) m1 + m2 =
− and m1m2 =
b b

2 h2 – ab
(iii) tan θ =
a+b

(n) General equation for pair of straight lines: ax2 + 2hxy + by 2 + 2gx + 2fy + c =0 represents a pair of straight
a h g
lines, if h b f = 0
g f c
M a them a ti cs | 8.29

Solved Examples

JEE Main/Boards 20 2 – 10 2 + 30 2 5
= =
5 2 +5 2 + 6 2 2
Example 1: Find the ratio in which y – x + 2 = 0 divides
the line joining A (3, – 1) and B (8, 9). ay + by 2 + cy 3
y= 1
a+b+c
Sol: By considering the required ratio be λ:1, and using
section formula, we can solve above problem. –10 2 + 20 2 + 30 2 5
= =
The point of division P is internal as A and B lie on 5 2 +5 2 + 6 2 2
opposite sides of given line.
Let required ratio be λ:1. Example 3: A rectangle PQRS has its side PQ parallel to
the line y = mx and vertices P, Q, S lie on lines y = a, x = b
 8λ + 3 9λ – 1  and x = –b, respectively. Find the locus of the vertex R.
Since, P  ,  lies on y – x + 2 = 0,
 λ +1 λ +1 
Sol: Here sides PQ and QR must be perpendicular to
9λ – 1 8λ + 3 2
∴ – +2 = 0 or λ = each other. Therefore product of their slopes will be –1.
λ +1 λ +1 3
Let R(h, k) be any point on the locus and let S and Q
y-x+2=0
have co-ordinates (–b, β) and (b, α), respectively, as T is
mid-point of SQ and PR.
 1 α–a
A B Thus P has co-ordinates (–h, a) =m
P b+h
⇒ α = a + m (b + h)
1 α –k
– = slope of QR =
Hence, required ratio is 2/3:1 or 2:3 m b–h
1
⇒α=k– (b – h)
Example 2: Find the incentre I of ∆ABC, if A is (4, – 2) B m
is (– 2, 4) and C is (5, 5). 1
a + m(b + h) = k – (b – h)
ax1 + bx2 + cx3 ay1 + by 2 + cy 3 m
Sol: Using x
= = , y , ∴ locus of R is
a+b+c a+b+c
we can obtain the incentre. x(m2 – 1) 2 my + b + am + bm2 = 0.

a = BC = (5 + 2)2 + (5 – 4)2 =
5 2 Example 4 Two equal sides AB and AC of an isosceles
b = CA = 2 2
(5 – 4) + (5 + 2) = 5 2 triangle ABC have equation 7x – y + 3 = 0 and x + y – 3
= 0, respectively. The third side BC of the triangle passes
c = AB = (–2 – 4)2 + (4 + 2)2 = 6 2 through point P(1, – 10). Find the equation of BC.

A Sol: For isosceles triangle ABC, AD is perpendicular


bisector of side BC and it also bisects angle BAC. Hence
by using equation of bisector formula, i.e.
ax +by + c a1 x +b1 y + c1
=± ,
a2 +b2 a12 +b12
B C
we can obtain slope of AD.
If incentre I is (x, y) , then, Equations of AB and AC are 7x –y + 3 = 0 and
– x – y + 3 = 0, respectively.
ax1 + bx2 + cx3
x= a1 = 7; b1 = – 1, c1 = 3;
a+b+c
8 . 3 0 | Straight Line

a2 = b2 = – 1, c2 = 3. Comparing the coefficients on both sides, we have


As c1 > 0, c2 > 0 and a1a2 + b1b2 = –6< 0 2h a
m 1 + m 2 = – ; m1m 2 =  …..(i)
b b
Equation of the bisector of the acute angle BAD is
Now, bx2 – 2hxy + ay2 = 0
7x – y + 3 –x – y + 3
= , i.e. 3x + y = 3 2h a
49 + 1 2 ⇒ x2 – xy + y 2 =
0
b b
A
⇒ x2 + (m1 + m2) xy + m1m2y2 = 0 (by (i))

⇒ (x + m1y) (x + m2y) = 0
–1 –1
⇒ y
= = x;y x
m1 m2
B D C P
1 Example 7: Find the angle φ between the straight lines
As slope of AD is –3, slope of BC is
3 π
Equation of BC through P(1, –10) is (x2 + y2) sin2α = (x cos θ – y sin θ)2, where 0 < 2α < .
1 2
y + 10 = (x – 1) or x – 3y = 31.
3 2 h2 – ab
Sol: We know tan φ = . Solving it, angle φ
can be obtained. (a + b)
Example 5: Find the equation of the line passing
through the intersection of lines x – 3y + 1 = 0, 2x +
x2 (cos2 θ – sin2 α ) – 2xy cos θ sin θ + y 2 (sin2 θ – sin2 α ) =
0
5y – 9= 0 and whose distance from the origin is 5.
a = cos2θ – sin2α, 2h = – 2 cos θ sin θ,
Sol: Equation of any line passing through the
b = sin2θ – sin2α
intersection of two other lines will be L1 + λL2 = 0.
Therefore, by using perpendicular distance formula of 2 h2 – ab
point to line, i.e = tan φ =
(a + b)
| ax1 +by1 + c |
p=
a2 +b2 2 cos2 θ sin2 θ – (cos2 θ – sin2 α )(sin2 θ – sin2 α )
=
we can obtain required equation of line. | (cos2 θ – sin2 α ) + sin2 θ – sin2 α |
Any line through the point of intersection of given lines is
2sin α cos α sin2α
= = = tan2α ∴ φ = 2a
x – 3y + 1 + λ (2x + 5y – 9) = 0 cos2α cos2α
or (1 + 2λ) x + (– 3 + 5λ) y + 1 – 9λ = 0
Example 8: The point A divides the line joining P ≡ (–5, 1)
| 0 + 0 + 1 – 9λ | and Q = (3, 5) in the ratio k:1. Find the two values of k for
5=
(1 + 2λ )2 + (–3 + 5λ )2 which the area of ∆ABC where B ≡ (1, 5), C ≡ (7, 2) is equal
to two square units.
7
Squaring and simplifying, we get λ = .
8 Sol: By using section formula, we can obtain the co-
ordinates of point A and then values of k by using the
Hence, required line has the equation 2x + y – 5 = 0.
triangle formula.
Co-ordinates of A, dividing the line joining points
Example 6: Show that bx2 – 2hxy +ay2 = 0 represent a
P ≡ (– 5, 1) and Q ≡ (3, 5) in the ratio k:1, are given by
pair of straight lines which are at right angles to the pair
of lines given by ax2 + 2hxy + by2 = 0. (3k – 5 / k + 1, 5k + 1 / k + 1) . Also, area of the ∆ABC
1
Sol: Here if the product of slopes of a pair of straight is given by ∆ = ∑ x1 (y 2 – y 3 )
2
lines represented by the given equations is –1, then
1
they are right angle to each other. = | [x (y – y 3 ) + x2 (y 3 – y1 ) + x3 (y1 – y 2 )] |
2 1 2
Let ax + 2hxy + by
2 2

1  3k – 5  5k + 1   5k + 1  
= (y – m1x) ( y – m2x)  (7) +  –2 –  + 7 – 5  = 2
2  k + 1  k + 1   k + 1  
M a them a ti cs | 8.31

3k – 5  5k + 1   5k + 1  JEE Advanced/Boards
(7) +  –2 –  + 7 – 5 =
±4
k +1  k +1   k +1 
Example 1: If A(–1, 5), B(3,1) and C(5, 7) are vertices of
⇒ 14k – 66 =
4k + 4,10k =
70,k =
7 a ∆ABC and D, E, F are the mid-points of BC, CA and AB,
respectively, then show that area ∆ABC = 4 times area
or 14k – 66 = – 4k – 4, 18k = 62,
(∆DEF).
k = (31/9).
Sol: Co-ordinates of D, E and F are first obtained by
Therefore, value of the k = 7, 31/9
using mid-point formula, and prove the given equation
Example 9: Prove that the sum of the reciprocals of the by using formula of area of triangle.
intercepts made on the co-ordinate axes by any line Co-ordinates of D, E, F are (4, 4), (2, 6) and (1, 3),
not passing through the origin and through the point respectively.
of intersection of the lines 2x + 3y = 6 and 3x + 2y = 6 –1 5 1
is constant. 1
∴ Area of ∆ABC = 3 1 1 = 16
2
5 7 1
Sol: Equation of any line through the points of
intersection of the given lines is L1 + λL2=0.
4 4 1
2x + 3y – 6 + k (3x + 2y – 6) =
0 1
Area of ∆DEF = 2 6 1 =4
2
(2 + 3k) x + (3 + 2k)y – 6(k + 1) =
0 1 3 1
x y A
⇒ + 1
=
((6(k + 1)) / (2 + 3k)) ((6(k + 1)) / (3 + 2k))
F E
Where k ≠ – 1
and in this case, sum of the reciprocals of the intercepts
made by this line on the co-ordinate axis is equal to
B C
D

2 + 3k + 3 + 2k 5(k + 1) 5 Hence, area of ∆ABC = 4 area (∆DEF)
= = .
6(k + 1) 6(k + 1) 6
Example 2: Point P(a2, a + 1) is a point of the angle (which
However, for k = – 1, the line become
contains the origin) between the lines 3x – y + 1 = 0,
x = y, which passes through the origin. x + 2y – 5 = 0. Find interval for values of ‘a’.

Sol: Given origin and P lie on same side of each line.


Example 10: Find the straight lines represented by
Substituting P in the given equation of line, we can
6x2+ 13xy + 6y2 +8x + 7y + 2 = 0 and also find their
obtain the required interval.
point of intersection.
a2 + 2a + 2 – 5< 0 and 3a2 – (a + 1) + 1 > 0
Sol: Taking term y as a constant and using quadratic
i.e. (a + 3) (a – 1) < 0 and a (3a – 1) > 0
2
−b ± b − 4ac 1 
roots formula, i.e. x = , we can obtain ∴ a ∈ (–3, 1) and a ∈ (– ∞, 0) ∪  , ∞ 
2a 3 
equations of required straight lines and after that by 1 
∴ a ∈ (– 3, 0) ∪  , 1 
3 
solving them we will get their point of intersection.
Rewrite the given equation as Example 3: Find the equations of the lines passing
through P(2, 3) and making an intercept AB of length
–(13y + 8) ± (13y + 8)2 – 24(6y 2 + 7y + 2)
x= 2 units between the lines y + 2x = 3 and y + 2x = 5.
12
Sol: Using equation of line in parametric form, i.e.
–(13y + 8) ± (5y + 4) –(2y + 1) –(3y + 2)
= = , x – x1= r cos θ and y – y1 = r sin θ, we can obtain the
12 3 2 required equation of line.
Hence, 3x + 2y + 1 = 0 and 2x + 3y + 2 = 0
1 4  Let equation of the line, in parametric form, be
are the required lines and they intersect at  , –  . x – 2 = r cos θ; y – 3 = r sin θ.
5 5 
8 . 3 2 | Straight Line

Then, A(2 + r1cos θ, 3 + r1 sin θ) and B(2 + r2 cos θ, ab2 a2b h k 


3 + r2 sin θ) lie on y+2x=3 and y+2x=5, respectively. By (i) and (ii) h = ,k= F is  , 
2 2
2 2 2 2
a +b a +b
∴ (3 + r1 sin θ) + 2 (2 + r1 cos θ) = 3 …. (i) ab
Slope of BE = .
and (3 + r2 sin θ) + 2 (2 + r2 cos θ) = 5 …. (ii) a2 + 2b2
∴ (r2 – r1) (sin θ + 2 cos θ) = 2 a2 + 2b2
Slope of AF =
–ab
Product of slopes of BE and AF is equal to (–1). Hence
AF ⊥ BE.
y+2x=3 A

Example 5: A square lying above the x-axis and has


y+2x=5 one vertex at the origin. A side passing through the
B  π
origin makes an angle α  0 < α <  with the positive
P  4
direction of x-axis. Prove that the equations of its
∴ sin θ + 2 cos θ = ± 1 (as |r2 – r1| = 2) diagonals are y (cos α – sin α) = x (sin α + cos α) and
y(sin α + cos α) + x (cos α – sin α) = 0, where a is the
3 cos2θ + 4 sin θ cos θ = 0
length of a side of the square.
π 3
θ= or tan θ = – ∴ Required lines are x = 2 and
2 4 Sol: Using slope point form of equation of line, i.e.
4y + 3x = 18 y – y1 = m(x – x1), we can obtain the result. Here m = tan
θ and x1 , y1 is 0.
Example 4: In a triangle ABC, AB = AC. If D is mid-point  π
Equation of diagonal OB is= y tan  α +  x
of BC, E is the foot of perpendicular from D on AC, and  4
F is the mid-point of DE, show that AF is perpendicular  π  π
or y cos  α += x sin  α + 
to BE.  4  4

Sol: As the geometrical fact to be established does or y (cos α – sin α) = x (cos α + sin α) ….(i)
not depend on position of ABC, we may assume that y
‘’D is the origin; BC and AD are along x and y axes B
respectively (as shown)”. Therefore by using intercept
form of equation of line, we can obtain required result.
C
Let BD = DC = a, and A and E have co-ordinates (0, b)
and (h, k), respectively. A
/4
x y
Line AC has the equation + = 1 
a b O
x
y
A(0,b) From the figure, point A is (a cos α, a sin α).
As diagonal AC is perpendicular to diagonal OB,
equation of AC is
cos α – sin α
E Y – a sin α = (x – acos α )
F cos α + sin α
or x (cos α – sin α) + y (cos α + sin α) = a
x
B(-a,0) D C(a,0)
Example 6: Two sides of a rhombus lying in the first
h k quadrant are given by 3x – 4y = 0 and 12x – 5y = 0. If
∴ + =1  ….(i)
a b the length of the longer diagonal is 12 units, find the
equations of the other two sides of the rhombus.
Also, (k/h) (–b/a) = – 1
( AC ⊥ DE)  ….(ii) Sol: Using formula of equation of bisector of the angle,
we can obtain equation of AC. Given the length AC, we
M a them a ti cs | 8.33

can obtain co-ordinates of C. ⇒ y – 9x = 0 or y + x = 0


Let ABCD be the rhombus with AC as the longer A(, )
diagonal, where A has co-ordinates (0, 0). AC is bisector y=9x
y=-x
of angle ∆BAD. The equations of the two angle bisectors
F E
(of angles formed by given lines) are:
3x − 4y 12x − 5y
= ± or 21x + 27y = 0 and 99x – 77y = 0.
5 13
B(x1,y1) C(x2,y2)
Since diagonal AC has positive slope, its equation is
x y The mid-point E of AC lies on
=
7 9
y y – 9x = 0

C β + y2 – 9(α + x2) = 0  ….(i)
D
12x=5y

Since AC is perpendicular to y – 9x = 0,
y –β
B We have 2 (9) = –1 Rewrite (i) and (ii) as 9x2 – y2
x2 – α
3x - 4y = β – 9a
x
A x2 + 9y2 = α + 9β

In parametric form, we get 9β – 40α 9α + 40β


∴ x2 = y2 =
x y 41 41
= ; Since AC = 12 units,
7 / 130 9 / 130  α + x1 β + y1 
Similarly, F  ,  , the mid-point of AB lies on
 7.12 9.12   2 2 
C has co-ordinates  , .
 130 130  y + x = 0.

Let sides DC and BC have equations 3x – 4y = a and 12x ∴ α + x1 + β + y1 = 0


– 5y = b, respectively. Substituting co-ordinates of C in or x1 +y1 = – (α + β)  …..(iii)
these equations yields
Since, AB is perpendicular to y + x = 0,
3.84 4.108 –180

= a = – ; We have,
130 130 130
β – y1
(–1) = –1 or β – y1 = α – x1
12.84 5.108 468 α – x1
= b = –
130 130 130 or x1 – y1 = α – β  ….(iv)
Solving (iii) and (iv), we have
Hence, equation of sides DC and BC are 3x – 4y +
180 468 x1 = – β and y1 = – a
= 0 and 12x – 5y = , respectively.
130 130 As B(x1, y1), C(x2, y2) and D(f, g) are collinear
we have,
Example 7: The base of a triangle ABC passes through
a fixed point (f, g) and its other two sides are bisected f g 1
at right angles by the lines y2 – 8xy – 9x2 = 0. Find the x1 y1 1 = 0
locus of the vertex x2 y2 1
Sol: Let vertices be A(α, β), B(x1, y1) and C(x2, y2) and let or f(y1 – y2) – g(x1 – x2) + (x1y2 – x2y1) = 0
(f, g) lie on BC. The mid-point of side AB and AC must
be lies on two perpendicular lines represented by y2 –  9α + 40β   9β – 40α 
or f  – α –  – g  –β −
8xy – 9x2 = 0, respectively. Hence by solving them we 41  41 

 
will get locus of the vertex.
Let y2 – 8xy – 9x2 = 0 9α + 40β 9β – 40α
–β +α =0
41 41
⇒ (y – 9x) (y + x) = 0
8 . 3 4 | Straight Line

or f(50α + 40β) + g(40α – 50β) + 40(α2 + β2 ) =0 Locus lx2 + my 2 + n BP


⇒ – =λ=  …. (i)
of (α, β) is 4(x2 + y 2 ) + (4g + 5f)x + (4f – 5g)y =
0 lx3 + my 3 + n PC

Similarly, we obtain
Example 8: If the vertices of a triangle have integral
co-ordinates, prove that the triangle cannot be equilateral. CQ lx + my 3 + n
=– 3  …. (ii)
QA lx1 + my1 + n
Sol: Obtaining the area of triangle using D = (1/2) bc
sin A and using the co-ordinate form, we can conclude AR lx + my1 + n
and =– 1  …. (iii)
that the triangle cannot be equilateral if vertices have RB lx2 + my 2 + n
integral co-ordinates.
Multiplying (i), (ii) and (iii), we get the required
Consider a triangle ABC with vertices A ≡ (x1, y1),
result.
B ≡ (x2, y2), C ≡ (x3, y3). Let x1, x2, x3, y1, y2, y3 be the integers.
BC2 = (x2 – x3)2 + (y2 – y3)2 a positive integer. Example 10: The circumcentre of a triangle having
vertices A = (a, a tan α), B = (b, b tan β), C = (c, c tan
If the triangle is equilateral, then AB = BC = CA = a
γ) is the origin, where α + β + γ = π. Show that the
and ∠A =∠B =∠C =60° .
orthocentre lies on the line.
Area of the triangle = (1/2) bc sin A = (1/2) a2 sin 60°
 α β γ  α β γ
= (a2 / 2).( 3 / 2) = ( 3 / 4)a2 which is irrational.  4 cos cos cos  x – 4  sin sin sin  y – y = 0
 2 2 2  2 2 2
 a2 is a positive integer.
Sol: Consider the circumcentre ‘O’ to be the origin and
Now, the area of the triangle in terms of the co-ordinates
the equation of the circumcircle be x2 + y2 = r2. As
= (1/2) [(x1(y2 – y3) + x2 (y3 – y1) + x3 (y1 – y2 )] vertices of triangle lies on this circle, we can obtain the
which is rational number. co-ordinates of centroid by using the respective
formula.
This contradicts that the area is an irrational number, if
the triangle is equilateral. Since vertex A(a, a tan α) is r distance from the
circumcenter.
Example 9: A line L intersects the three sides BC, CA Therefore a2(1 + tan2α) = r2 ⇒ a = r cos α
and AB of a triangle ABC at P, Q and R, respectively.
A = (r cos α, r sin α)
 BP   CQ   AR  Similarly B = (r cos β, r sin β)
Show that      = –1 .
 PC   QA   RB 
C = (r cos γ, r sin γ)
Sol: Using equation of line lx + my + n = 0 and section A
formula, we can prove the given equation.
O
Consider a triangle ABC with vertices A(x1, y1), B(x2, y2) G
and C(x3, y3), and let lx + my + n = 0 be equation of H

the line L. If P divides BC in the ratio λ:1, then the co- B C

 λx + x 2 λy 3 + y 2 
ordinates of P are  3 , 
 λ +1 λ + 1  Centroid G
Also, as P lies on L, we have
 r(cos α + cos β + cos γ ) r(sin α + sin β + sin γ ) 
 λx + x 2   λy 3 + y 2   , 
l  3  + m   3 3 
  + n =0
 λ +1   λ +1 
Circumcentre O’ (0, 0) and let orthocentre H (h, k).
A We know that O, G, H are collinear. Therefore slope of
R OG = slope of OH
Q sin α + sin β + sin γ k
i.e. = '
cos α + cos β + cos γ h
1B P
C Point (h, k) will be on
M a them a ti cs | 8.35

x(sin α + sin β + sin γ) – y (cos α + cos β + cos γ) = 0  x + 3 y2 – 1 


Also D  2 , the mid-point of AC lies on the
 2 , 2 
 α β γ  α β γ  
⇒ x  4 cos cos cos  – y  1 + 4 sin sin sin  = 0 median through B,
 2 2 2  2 2 2
i.e. 6x + 10y – 59 = 0
And hence the result is α + β + γ = π.
 x +3  y2 – 1 
⇒ 6  2  + 10 
 0
 – 59 =
Example 11: ABC is a variable triangle with the fixed  2   2 
vertex C(1, 2) and vertices A and B with co-ordinates
(cos t, sin t) and (sin t, – cos t), ⇒ 3x2 + 5y2 – 55 = 0  ….(ii)

Respectively, where t is a parameter. Find the Solving (i) and (ii), we get x2 =10, y2 = 5,

locus of the centroid of the ∆ABC. i.e. the co-ordinates of C are (10, 5) and thus
the equation of AC is 6x – 7y = 25
Sol: We can obtain co-ordinates of centroid G(α, β)
 x + x 2 + x3 y1 + y 2 + y 3  A(3, -1)
using the formula  1 ,  and
 3 3  Angle bisector
we will get required equation of locus of centroid by Median
solving them simultaneously.
Let G(α, β) be the centroid in any position .Then G(α, β) 6x + 80y - 59 = 0
x - 4y + 10= 0
 1 + cos t + sint 2 + sint – cos t  B(x1, y1) C(x2, y2)
= –  or
 3 3 
Let the slope of BC be m1. Since BC and AC are equally
1 + cos t + sint 2 – sint – cos t
=∴ α = ,β inclined to the angle bisector
3 3
x – 4y + 10 = 0,
∴ or 3α – 1 = cos t + sin t  …. (i)
(1 / 4) – m (6 / 7) – (1 / 4) 1 – 4m 17
3β =2 – sin t – cos t  …. (ii) = ⇒ =
1 + (1 / 4)m 1 + (6 / 7) × (1 / 4) 4 + m 34
Squaring and adding equations (i) and (ii), we get
2
(3α – 1)2 + (3β – 2)2 ⇒m= – Equation of BC is
9
= (cos t + sin t)2 + (sin t – cos t)2 2
y – 5 = – (x – 10) and 6x1 + 10y1 = 59
= 2 (cos t + sin t) = 2
2 2
9
∴ the equation of the locus of the centroid is (3x – 1)2+ Solving these equations, we get X1 = – 7/2, y1 = 8
(3y – 2)2 = 2
8 +1
Hence, equation of AB is y + 1 = (x – 3)
9(x2 + y2) –6 x – 12y + 3 = 0 –7 / 2 – 3
∴ 3(x2 + y2) – 2x – 4y + 1 = 0
Example 13: A triangle has the lines y = m1x and y
Example 12: Find equations of the sides of the = m2x for two of its sides, with m1 and m2 being roots
triangle having (3, –1) as a vertex, x – 4y + 10 = 0 and of the equation bx2 + 2hx + a = 0. If H(a, b) is the
6x + 10y – 59 = 0 being the equations of an angle orthocentre of the triangle, show that the equation of
bisector and a median, respectively, drawn from the third side is (a + b) (ax + by) =ab(a + b – 2h).
different vertices.
Sol: Line OD passes from orthocentre. Therefore it
Sol: Consider the vertices of the triangle to be A(3, – 1), must be perpendicular to the side AB. By considering
B(x1, y1) and C(x2, y2). Here the mid-point of AC lies on equation of AB as y = mx +c, we will get co-ordinates
the median through B. of A and B. Using slope point form of equation of line,
we can solve the problem.
Equation of the median through B be 6x + 10y – 59 = 0
and the equation of the angle bisector from C be The given lines y = m1x and y = m2x intersect at the
origin O (0, 0). Thus one vertex of the triangle is at the
x – 4y + 10 = 0; x2 – 4y2 + 10 = 0 ….(i)
origin O. Therefore, let OAB be the triangle and OA and
8 . 3 6 | Straight Line

OB be the lines since m1 and m2 are the roots of the equation


y = m1x  ....(i) bx2 + 2hx + a = 0

and y = m2x  .…(ii) m1+ m2 = – 2h/b and m1m2 = a/b


From (vii), we have
Y
–[a / b + 2hm / b + m2 ]a –[a + 2hm + bm2 ]a
B c= =
m(a / b + 1) m(a + b)
D
y=m2x y=mx+c From (iii), the equation of third side AB is
(a + 2hm + bm2 )a
H(a,b) A y = mx –
m1 (a + b)
y=m1x
X a (a – 2ha / b + ba2 / b2 )a
O or y = – x –
b (–a / b)(a + b)

Let the equation of the third side AB be or (ax + by)(a + b)= ab(a + b – 2h)
y = mx +c  …..(iii)
Given that H(a, b) is the orthocentre of the OAB, Example 14: Find the co-ordinates of the centroid,
circumcentre and orthocentres of the triangle
∴ OH ⊥ AB formed by the lines 3x – 2y = 6, 3x + 4y + 12 = 0 and
⇒ (b/a) × m = – 1 ⇒ m = –a/b  ….(iv) 3x – 8y + 12 = 0.

Solving (iii) with (i) and (ii), the co-ordinates of Sol: Solving the given equations, we can obtain the
 c cm1  co-ordinates of vertices of triangle. Using appropriate
A =  ,  and formula for finding the co-ordinates of centroid,
m
 1 – m m1 –m
circumcentre and orthocentre, the problem can be
 c cm2  solved.
B =  , 
 m2 – m m2 – m  Let sides AB, BC and CA have the

Now equation of line through A equations 3x – 2y – 6 = 0  ….(i)

perpendicular to OB is 3x – 8y + 12 = 0  ….(ii)
3x + 4y + 12 = 0  ….(iii)
cm1 1  c 
y– =–  x –  or Solving (ii), (iii) we get y = 0, x = – 4,
m1 – m m2  m1 – m 
A
x c(m1m2 + 1)
y= – +  ….(v)
m2 m2 (m1 – m)

Similarly, equation of line through B 3x - 2y- 6 =0 3x - 4y+ 12=0

perpendicular to OA is

x c (m1 m2 + 1)
y= − +  ….(vi)
m1 m1 (m2 − m) B C
3x - 8y+12=0
C = (– 4, 0)
The point of intersection of (v) and (vi) is the orthocentre
H (a, b). Solving (i), (ii) we get y = 3, x = 4
∴ Subtracting (vi) from (v), we get
B = (4, 3)
–cm(m1m2 + 1)
x=a=
(m1 – m)(m2 – m) Solving (i,) (iii) we get y = – 3, x = 0;

–[m1m2 – m(m1 + m2 ) + m2 ]a A = (0 – 3)
or c =  …..(vii)
m(m1m2 + 1)
M a them a ti cs | 8.37

 x + x 2 + x3 y1 + y 2 + y 3  Example 15: One diagonal of a square is the position


Centroid G =  1 ,  b
 3 3  m+
of the line tan( ±45°) = a =±1 which is intercepted
where vertices are (x, y), etc. b
1–m
a
 0 + 4 – 4 –3 + 3 + 0  between the axes. Find the co-ordinates of other two
G = ,  (0,0)
 3 3  vertices of the square. Also prove that if two opposite
vertices of a square move on two perpendicular lines,
To find the circumcentre: the other two vertices also move on two perpendicular
lines.
Let M(α, β) be the circumcentre.
m2 − m1
MA = MB = MC Sol: Using tan θ = , we can obtain the slopes
1 + m1m2
(α – 0)2 + (β+ 3)2 = (α – 4)2+ (β – 3)2 = (α + 4)2 + (β – 0)2
of AB and AD. As the slope of the given lines is –b/a,
α2 + β2 + 6β + 9 the two vertices are clearly on the diagonal BD of the
2 2 2 2 square ABCD.
= α + β – 8α – 6β + 25 = α + β – 8α + 16
If m be the slope of the line inclined at an angle of 45°
= 6β + 9 = –8α – 6β + 25 = 8α + 16
to BD,
6β + 9= –8α – 6β + 25 and 6β + 9 = 8α + 16 m + (b / a)
tan( ±45°) = =±1
8 α + 12 β – 16 = 0 1 – m(b / a)

2 α + 3 β – 4 =0  …. (i) C

8 α – 6 β + 7 =0  …. (ii)
1 23
Solving (i) and (ii), we get =
α ,=
β
12 18 (0,b)D
E
45 B
 1 23  45 (a, 0)
Circumcentre =  , 
 12 18 
O


A(0, -3)
A
E
a–b (a + b) a–b
m= or – AB is y – 0 = (x – a)
M a+b a–b a+b
a+b
AD is y– b = – (x – 0)
B(4, 3) D C(-4, 0) a–b
By solving these equation we get
MG 1 a–b b – a
Use = = The point A is 
GH 2 ,  . C is obtained by using the
 2 2 
fact that mid-point of AC and BD is same.
Let H(α, β) be the orthocentre

1 a+b a+b
α+2 C=  , 
12 1  2 2 
=0 = ⇒α –  …. (i)
3 6
The opposite vertices B, D move on two perpendicular
23 lines x-axis and y-axis. Now the point
β+2  –1 –23 
18 23
=0 = ⇒β – Then H  , . a–b b – a a+b a+b
3 9  6 9  A ,  lies on y = –x and point C  , 
 2 2   2 2 
lies on y = x.
8 . 3 8 | Straight Line

Example 16: If the image of the point (x1, y1) with x1 + x2 y1 + y 2


and a. + b. +c =0  ….(ii)
respect to the mirror ax + by + c = 0 be (x2, y2), show 2 2
that x – x y 2 – y1 –2(ax1 + by1 + c) From (ii), a(x1 + x2) + b(y1 + y2) + 2c = 0
2 1
= =
a b a2 + b2
Sol: As the line PQ joining the points P(x1, y1) and Q(x2, or (ax1 +by1 + c) + (ax1 +by1 + c) = 0  ….(iii)
y2) is perpendicular to the line ax + by +c = 0. Also the
x1 – x2 y1 – y 2 a(x1 – x2 ) + b(y1 – y 2 )
mid-point M of PQ is on the lines ax + by +c= 0. Hence From (i), = =
product of their slopes will be –1 and co-ordinates of M a b a2 + b2
lies on ax + by +c= 0.
By ratio and proportion
y1 – y 2  a 
 –  = –1  …..(i)
=
(ax1 + by1 + c) – (ax2 + by 2 + c)
x1 – x2  b 
a2 + b2
P(x1,y1)
2(ax1 + by1 + c)
= , using (iii)
a2 + b2
x2 – x1 y 2 – y1 – 2(ax1 + by1 + c)
ax+by+c=0 = =
a b a2 + b2
M

Q(x2,y2)

JEE Main/Boards

Exercise 1 Q.7 Find the coordinates of the vertices of a square


inscribed in the triangle with vertices A(0, 0), B(2, 1) and
Q.1 Find the slope of the line joining (4, – 6) and (– 2, – 5). C(3, 0); Given the two vertices are on the side AC.

Q.2 Show that the line joining (2, – 3) and (– 5,  1) Q.8 Find the equation of the straight line which passes
is (i) Parallel to the line joining (7, – 1) and (0, 3), through the origin and trisects the intercept of line
(ii) Perpendicular to the line joining (4, 5) and (0, – 2). 3x + 4y = 12 between the axes.

Q.3 A quadrilateral has the vertices at the points Q.9 A straight line passes through the point (3, – 2).
(– 4,  2), (2, 6), (8, 5) and (9, – 7). Show that the mid- Find the locus of the middle point of the portion of the
points of the sides of this quadrilateral are vertices of a line intercepted between the axes.
parallelogram.
Q.10 Find the equation of the straight line which passes
Q.4 Find the values of x and y for which A(2, 0), through the point (3, 2) and whose gradient is 3/4. Find
B(0, 2), C(0, 7) and D(x, y) are the vertices of an isosceles the coordinates of the point on the line that are 5 units
trapezium in which AB ||DC. away from the point (3, 2).

Q.5 Find the equations of the diagonals of the rectangle, Q.11 Find the distance of the point (2, 5) the lines 3x
whose sides are x = a, x = a’, y = b and y= b’. +y + 4 = 0 measured parallel to line having slope 3/4.

Q.6 In what ratio is the line joining the points (2, 3) and Q.12 The extremities of a diagonal of a square are
(4, –5) divided by the line joining the points (6, 8) and (1, 1), (– 2, – 1). Obtain the other two vertices and the
(– 3, – 2) ? equation of the other diagonal.
M a them a ti cs | 8.39

Q.13 In the given figure, PQR is an equilateral Q.23 A ray of light is sent along the line x – 2y – 3 = 0.
triangle and OSPT is a square. If OT =2 2 units, find Upon reaching the line 3x – 2y – 5 = 0, the ray is reflected
the equation of the lines OT, OS, SP, OR, PR and PQ. from it. Find the equation of the line containing the
y reflected ray.

Q.24 Find the image of the point (– 8, 12) with respect


K to the line mirror 4x + 7y + 13 = 0.
15 L
Q.25 The equations of two sides of a triangle are
45 3x – 2y + 6 = 0 and 4x + 5y = 20 and the orthocentre is
S
(1, 1). Find the equation of the third side.
45 45
60 45 45 45
x
M O Q Q.26 Find the equations of the straight lines passing
through the point of intersection of the lines x + 3y +
4 = 0 and 3x + y + 4 = 0 and equally inclined to the
axis.
Q.14 Find the equation of the medians of a triangle
formed by the lines x + y – 6= 0, x – 3y – 2 =0 and Q.27 Show that the straight lines x (a + 2b) + y(a + 3b)
5x – 3y + 2 =0. = a + b, for different values of a and b pass through a
fixed point.
Q.15 Find the coordinates of the orthocentre of the
triangle whose vertices are (0, 0), 2, – 1) and (–1, 3). Q.28 The equation of the base of an equilateral triangle
is x + y = 2 and the vertex is (2, – 1). Find the length of the
Q.16 Two vertices of a triangle are (3, – 1) and (–2, 3) side of the triangle.
and its orthocentre is origin. Find the coordinates of the
third vertix.

Q.17 If the lines 3x + y – 2 =0, px + 2y – 3 = 0 and


Exercise 2
2x – y – 3 =0 are concurrent, find the value of p.
Single Correct Choice Type

Q.18 Find the angle between the lines y – 3x – 5 = 0 and


Q.1 The pair of points which lie on the same side of the
3y – x + 6 =.
0 straight line, 3x – 8y – 7 = 0 is
(A) (0, – 1), (0, 0) (B) (0, 1), (3, 0)
Q.19 Prove that the points (2, – 1), (0, 2), (3, 3) and (5, 0)
are the vertices of a parallelogram. Also find the angle (C) (– 1, – 1), (3, 7) (D) (– 4, –3), (1, 1)
between its diagonals.
Q.2 Equation of the bisector of the acute angle between
Q.20 A and B are the points (–2, 0) and (0, 5). Find the the lines, 3x – 4y + 7 = 0 and 12x + 5y – 2= 0 is
Coordinates of two points C and D such that ABCD is (A) 11x – 3y + 9 = 0 (B) 11x + 3y – 9 = 0
a square.
(C) 3x – 11y + 9 = 0 (D) None

Q.21 Find the equations of the lines through the point


(3, 2), which make an angle of 45° with the line x – 2y = 3. Q.3 A ray of light passing through the point A (1, 2)
is reflected at a point B on the x-axis and then passes
through (5, 3). Then the equation of AB is
Q.22 A line 4x + y = 1 through the point A(2, – 7)
meets the line BC whose equation is 3x – 4y + 1 = 0 at (A) 5x + 4y = 13 (B) 5x – 4y = – 3
the point B. Find the equation of the line AC, so that (C) 4x + 5y = 14 (D) 4x – 5y = – 6
AB = AC.
8 . 4 0 | Straight Line

Q.4 The line x + 3y – 2 = 0 bisects the angle between a Q.11 The straight line, ax + by = 1 , makes with the
pair of straight lines of which one has equation x – 7y + curve px2 + 2axy + qy2 = r a chord which subtends a
5 = 0. The equation of the other line is right angle at the origin. Then:
(A) 3x + 3y – 1 = 0 (B) x – 3y + 2 = 0 (A) r(a2 + b2) = p + q (B) r(a2 + p2) = q + b
(C) 5x + 5y – 3 = 0 (D) None (C) r(b2 + q2) = p + a (D) None

Q.5 A is point (3, – 5) with respect to a given system of


axes. If the origin is moved to (4, – 3) by a translation Q.12 The lines y – y1 = m(x – x1) ± a 1 + m2 are tangents
of axes, then the new co-ordinates of the point A are to the same circle. The radius of the circle is:
given by (A) a/2 (B) a (C) 2a (D) None
(A) (1, – 2) (B) (– 1, 2)
Q.13 The equation of the pair of bisectors of the angles
(C) (– 1, – 2) (D) None of these
between two straight lines is, 12x2 – 7xy – 12y2 = 0. If
the equation of one line is 2y – x = 0, then the equation
x y 3 4 of the other line is
Q.6 The set of lines given by + = 1 , where + = 5
a b a b
(A) 41x – 38y = 0 (B) 38x – 41y = 0
are concurrent at a fixed point, then point is -
(C) 38x + 41y = 0 (D) 41x + 38y = 0
3 4
(A)  ,  (B) (0, b)
5 5 
Q.14 If the point B is symmetric to the point A(4, – 1)
(C) (a, 0) (D) None
with respect to the bisector of the first quadrant, then
the length AB is
Q.7 If P = (1, 0 ); Q = (– 1, 0) & R= (2, 0) are three
given points, then the locus of the points S satisfying (A) 3 2 (B) 4 2 (C) 5 2 (D) None
the relation, SQ2 +SR2 = 2 SP2 is
Q.15 The co-ordinates of the points A, B, C are (– 4, 0),
(A) A straight line parallel to x-axis.
(0, 2) & (– 3, 2) respectively. The point of intersection of
(B) A circle passing through the origin. the line which bisects the angle CAB internally and the
(C) A circle with the centre at the origin. line joining C to the middle point of AB is

(D) A straight line parallel to y-axis.  7 4  5 13 


(A)  – ,  (B) – , 
 3 3  2 2 
Q.8 Area of the rhombus bonded by the four lines,
 7 10   5 3
ax ± by ± c =0 is: (C)  – ,  (D) – , 
 3 3   2 2
c2 2c2
(A) (B)
2ab ab
Q.16 The sides of ∆ABC are 2x – y + 5 = 0, x + y – 5 =
2
4c ab 0 and x – 2y – 5 = 0. Sum of the tangents of its interior
(C) (D) angle is
ab 4c2
(A) 6 (B) 27/4 (C) 9 (D) None
Q.9 If the sum of the distances of a point from two
perpendicular lines in a plane is 1, then its locus is
Q.17 Equation of a straight line passing through the
(A) A square (B) Circle origin and making with x-axis an angle twice the size
(C) A straight line (D) Two intersecting lines of the angle made by the line y = 0.2x with the x-axis,
is:
Q.10 If the straight line x + 2y = 9, 3x – 5y = 5 & ax + (A) y = 0.4x (B) y = (5/12)x
by = 1 are concurrent, then the straight line 5x + 2y = 1
(C) 6y – 5x = 0 (D) None of these
passes through the point
(A) (a, – b) (B) (– a, b)
(C) (a. b) (D) (–a, –b)
M a them a ti cs | 8.41

Q.18 The shortest distance from the point M(– 7, 2) to π


(III) Rotation through an angle about the origin in
the circle x2 + y2 – 10x – 14y – 151 = 0 is 4
(A) 1 (B) 2 (C) 3 (D) None the counter clockwise direction. Then, the final position
of the point is given by the coordinates  (1980)

Q.19 The image of the pair of lines represented by


ax2+2hxy + by2 = 0 by the line mirror y=0 is
 1 7 
(A)  ,
 2 2
 (B) – 2, 7 2 ( )
(A) ax2 – 2hxy – by2 = 0 (B) bx2 – 2hxy + ay2 = 0
(C) bx2 + 2hxy + ay2 = 0 (D) ax2 – 2hxy +by2 = 0
 1 7 
(C)  –

,  (D)
2 2
( 2,7 2 )
Q.20 The pair of straight lines x2 – 4xy + y2 = 0 together
Q.4 The straight lines x + y = 0, 3x + y – 4 = 0,
with the line x + y + 4 6 = 0 form a triangle which is
x + 3y – 4 = 0 form a triangle which is  (1983)
(A) Right angle but not isosceles
(A) Isosceles (B) Equilateral
(B) Right isosceles
(C) Right angled (D) None of these
(C) Scalene
(D) Equilateral Q.5 If the sum of the distance of a point from two
perpendicular lines in a plane is 1, then its locus is
Q.21 Points, A & B are in the first quadrant; point ‘O’ is  (1992)
the origin. If the slope of OA is 1, slope of OB is 7 and (A) Square (B) Circle
OA = OB, then the slope of AB is
(C) Straight line (D) Two intersecting lines
(A) – 1/5 (B) – 1/4 (C) –1/3 (D) – 1/2
Q.6 The orthocentre of the triangle formed by the lines
xy = 0 and x + y = 1, is  (1995)
Previous Years’ Questions 1 1 1 1
(A)  ,  (B)  , 
Q.1 The points (–a, – b), (0, 0), (a, b) and are  (1979) 2 2 3 3

(A) Collinear (C) (0, 0)


1 1
(D)  , 
(B) Vertices of a rectangle 4 4

(C) Vertices of a parallelogram


Q.7 The graph of the function cos x cos (x + 2) – cos2
(D) None of the above (x + 1) is  (1997)
(A) A straight line passing through (0, – sin21) with slope 2.
Q.2 Given the four lines with the equations, x + 2y – 3 = 0,
3x + 4y – 7 = 0, 2x +3y – 4 = 0, 4x + 5y – 6 = 0, then (B) A straight line passing through (0, 0).
 (1980) (C) A parabola with vertex (1, – sin21).
(A) They are all concurrent π 
(D) A straight line passing through the point  , – sin2 1 
(B) They are the sides of a quadrilateral and parallel to the x-axis.  2 

(C) Only three lines are concurrent


(D) None of these Q.8 The diagonals of a parallelogram PQRS are along
the lines x + 3y = 4 and 6x –2y = 7. Then PQRS must
be a  (1998)
Q.3 The point (4, 1) undergoes the following three
transformations successively (A) Rectangle (B) Square
(I) Reflection about the line y = x. (C) Cyclic quadrilateral (D) Rhombus
(II) Translation through a distance 2 unit along the
positive direction of x-axis.
8 . 4 2 | Straight Line

Q.9 Let PS be the median of the triangle with vertices Q.16 Let PS be the median of the triangle with vertices
P (2, 2), Q(6, – 1) and R(7, 3). The equation of the line P(2, 2), Q(6, -1) and R(7, 3). The equation of the line
passing through (1, – 1) and parallel to PS is  (2000) passing through (1, -1) and parallel to PS is (2014)
(A) 2x – 9y – 7 =0 (B) 2x – 9y – 11 = 0 (A) 4x - 7y - 11 = 0 (B) 2x+ 9y+ 7 = 0
(C) 2x + 9y – 11 = 0 (D) 2x + 9y + 7 = 0 (C) 4x+ 7y+ 3 = 0 (D) 2x - 9y - 11 = 0

Q.10 The incentre of the triangle with vertices Q.17 Let a, b, c and d be non-zero numbers. If the point
(1, 3),(0, 0) and (2, 0) is  (2000) of intersection of the lines 4ax + 2ay + c = 0 and 5bx +
2by + d= 0 lies in the fourth quadrant and is equidistant
 3 2 1  from the two axes then  (2014)
(A)  1,  (B)  , 
 2  3 3 (A) 2bc - 3ad= 0 (B) 2bc+ 3ad= 0
 

2 3  1  (C) 3bc - 2ad= 0 (D) 3bc+ 2ad= 0


(C)  ,  (D)  1, 
3 2   3
 
Q.18 Locus of the image of the point (2, 3) in the line
( 2x − 3y + 4 ) + k ( x − 2y + 3) = 0, k ∈ R , is a:  (2015)
Q.11 The number of integer values of m, for which the
x-coordinate of the point of intersection of the lines (A) Straight line parallel to y-axis
3x +4y = 9 and y = mx + 1 is also an integer, is (2001) (B) Circle of radius 2
(A) 2 (B) 0 (C) 4 (D) 1 (C) Circle of radius 3
(D) Straight line parallel to x-axis.
Q.12 Area of the parallelogram formed by the lines
y = mx, y = mx + 1, y = nx and y = nx + 1 equals (2001)
Q.19 The number of points, having both co-ordinates
|m+n| 2 as integers, that lie in the interior of the triangle with
(A) (B)
(m – n)2 |m+n| vertices (0, 0), (0, 41) and (41, 0), is:  (2015)
1 1 (A) 861 (B) 820 (C) 780 (D) 901
(C) (D)
|m+n| |m–n|
Q.20 Two sides of a rhombus are along the lines,
x – y + 1 = 0 and 7x – y - 5 = 0. If its diagonals intersect
Q.13 Let P = (– 1, 0), Q = (0, 0) and R = (3, 3 3) be
at (–1, –2), then which one of the following is a vertex
three points. Then, the equations of the bisector of the of this rhombus?  (2016)
angle PQR is  (2002)
1 8
(A) (-3, -8) (B)  , − 
3  3 3
(A) x + y = (B)
0 x + 3y =
0
2  10 7
(C)  − , −  (D) (-3, -9)
3  3 3
(C) 3x + y = (D)
0 x+ y=
0
2

Q.14 If the line 2x + y = k passes through the point


which divides the line segment joining the points (1, 1)
and (2, 4) in the ratio 3 : 2, then k equals  (2012)

29 11
(A) (B) 5 (C) 6 (D)
5 5

Q.15 The x-coordinate of the incentre of the triangle


that has the coordinates of mid points of its sides as
(0, 1) (1, 1) and (1, 0) is  (2013)

(A) 2 − 2 (B) 1 + 2

(C) 1 − 2 (D) 2 + 2
M a them a ti cs | 8.43

JEE Advanced/Boards

Exercise 1 Q.5 Find the equation of the straight lines passing


through (– 2, – 7) & having an intercept of length 3
Q.1 Points O, A, B, C………are shown in figure where between the straight lines 4x + 3y = 12, 4x + 3y = 3.
OA = 2AB = 4BC =………so on. Let A is the centroid
of a triangle whose orthocentre and circumcentre are Q.6 Two sides of a rhombus ABCD are parallel to the
7 5 lines y = x + 2 & y = 7x + 3. If the diagonals of the
(2, 4) and  ,  respectively. If an insect starts moving rhombus intersect at the point (1, 2) & the vertex A is
2 2
on the y-axis, find the possible coordinates of A.
from the point O(0, 0) along the straight line is zig-zag
fashions and terminates ultimately at point P(α, β), then
find the value of (α + β) Q.7 Let O(0, 0), A(6, 0) and B(3, 3 ) be the vertices
of ∆OAB. Let R be the region consisting of all those
y points P inside OAB which satisfy d(P, OA) = minimum
{d(P, OB)), d(P, AB)}, where d(P, OA), d(P, OB) and
d(P, AB) represent the distance of P from the sides
C OA, OB and AB respectively. If the area of region R is
A 45 9(a – b), where a and b are coprime. Then, find the value of
45
(a + b).
45
45 B
x
O (0, 0) Q.8 Find the equations of the sides of a triangle having
(4, – 1) as a vertex. If the lines x – 1 = 0 and x – y – 1 = 0
are the equations of two internal bisectors of its angles.

Q.2 Let ABC be a triangle such that the coordinates of


Q.9 P is the point (– 1, 2), a variable line through P cuts
A are (– 3, 1). Equation of the median through B is 2x
the x & y axes at A & B respectively. Q is the point on
+ y – 3 = 0 and equation of the angular bisector of C
AB such that PA, PQ, PB are in HP. Find the locus of Q.
is 7x – 4y – 1 = 0. Then match the entries of column-1
with their corresponding correct entries of column-II.
Q.10 The equations of the altitudes AD, BE, CF of a
triangle ABC are x + y = 0, x + 4y = 0 and 2x– y =
Column I Column II
0 respectively. The coordinates of A are (t, –t). Find
(A) Equation of the line AB is (p) 2x + y – 3 = 0 coordinates of B & C. Prove that if t varies the locus of
(B) Equation of the line BC is (q) 2x – 3y + 9 = 0
the centroid of the triangle ABC is x + 5y = 0.

(C) Equation of CA is (r) 4x + 7y + 5 = 0


Q.11 The distance of a point (x1, y1) from each of two
(s) 18x – y – 49 = 0 straight lines which passes through the origin of co-
ordinates is δ; find the combined equation of these
straight lines.
Q.3 The equations of the perpendicular of sides AB and
AC of triangle ABC are x – y – 4 = 0 and 2x – y – 5 = 0
Q.12 Consider a, ∆ABC whose sides AB, BC and CA are
respectively. If the vertex A is (–2, 3) and point of intersection
represented by the straight lines 2x + y = 0. x + py = q
3 5 and x – y = 3 respectively. The point P is (2, 3)
of perpendicular bisector is  ,  , find the equation of
2 2
(i) If P is the centroid, then find the value of (p + q)
medians to the sides AB and AC respectively.
(ii) If P is the orthocentre, then find the value of (p + q)
Q.4 The interior angle bisector of angle A for the (iii) If P is the circumcentre, then find the values of
triangle ABC whose coordinates of the vertices are (p + q)
A(– 8, 5); B(– 15, – 19) and C(1, – 7) has the equation ax
+ 2y + c = 0. Find ‘a’ and ‘c’.
8 . 4 4 | Straight Line

Q.13 The sides of a triangle have the combined equation Q.20 The triangle ABC, right angled at C, has median
x2 – 3y2 – 2xy + 8y – 4 = 0. The third side, which is variable AD, BE and CF. AD lies along the line y = x + 3, BE lies
always passes through the point (– 5, – 1). If the range of along the line y = 2x + 4. If the length of the hypotenuse
values of the slope of the third line so that the origin is is 60, find the area of the triangle ABC.
an interior point of the triangle, lies in the interval (a, b),
 1  Q.21 A triangle has side lengths 18, 24 and 30. Find
then find  a + .
 b2  the area of the triangle whose vertices are the incentre,
circumcentre and centroid of the triangle.
Q.14 Consider a line pair 2x2 + 3xy – 2y2 – 10x + 15y
– 28 = 0 and another line L passing through origin
with gradient 3. The line pair and line L form a triangle Q.22 The points (1, 3) & (5, 1) are two opposite vertices
whose vertices are A, B and C. of a rectangle. The other two vertices lie on the lines
y = 2x + c. Find c & the remaining vertices.
(i) Find the sum of the cotangents of the inter ior angles
of the triangle ABC.
Q.23 A straight line L is perpendicular to the line
(ii) Find the area of triangle ABC. 5x – y = 1. The area of the triangle formed by the line L
(iii) Find the radius of the circle touching all the 3 sides & the coordinate axes is 5. Find the equation of the line.
of the triangle.
Q.24 Two equal sides of an isosceles triangle are given
Q.15 Show that all the chords of the curve by the equations 7x – y + 3 = 0 and x + y –3 = 0 & its
3x2 – y2 – 2x + 4y = 0 which subtend a right angle at the third side passes through the point (1, – 10). Determine
origin are concurrent. Does this result also hold for the the equation of the third side.
curve, 3x2 + 3y2 + 2x + 4y = 0 ? If yes, what is the point
of concurrency & if not, give reasons. Q.25 The equations of the perpendicular bisectors of
the sides AB & AC of a triangle ABC are x – y +5 = 0 &
Q.16 A straight line is drawn from the point (1, 0) to the x + 2y = 0, respectively. If the point A is (1, –2). Find the
curve x2 + y2 + 6x – 10y + 1 = 0, such that the intercept equation of the line BC.
made on it by the curve subtends a right angle at the
origin. Find the equations of the line. Q.26 Let P be the point (3, 2). Let Q be the reflection
of P about the x-axis. Let R be the reflection of Q about
Q.17 The two line pairs y2 – 4y + 3 = 0 andx2 + 4xy + the lines y = – x and Let S be the reflection of R through
4y2 – 5x –10y + 4 = 0 enclose a 4 sided convex polygon the origin. PQRS in a convex quadrilateral. Find the area
find. of PQRS.
(i) Area of the polygon.
Q.27 Two parallel lines 1 and  2 having non-zero
(ii) Length of the diagonals.
slope, are passing through the points (0, 1) and (– 1,0)
respectively. Two other lines 1 and  2 are drawn
Q.18 Find the equations of the two straight lines which through (0, 0) and (1, 0) which are perpendicular to 1
together with those given by the equation 6x2 – xy – y2 and  2 respectively. The two sets of lines intersect in
+ x + 12y – 35 = 0 will make a parallelogram whose four points which are vertices of a square. If the area of
diagonals intersect in the origin. p
this square can be expressed is the form q where p ∈
N, then the least value of (p + q)?
Q.19 A straight line passing through O(0, 0) cuts the
lines x = α, y = β and x + y = 8 at A, B and C respectively
such that OA . OB . OC = 482 and f(α, β) = 0 where Q.28 In an acute triangle ABC, the base BC has the
y 3 equation 4x – 3y + 3 = 0. If the coordinates of the
f(x,=
y) – + (3π – 2y)6 + ex + 2y – 2e – 6 orthocentre (H) and circumcentre (P) of the triangle are
x 2
(1, 2) and (2, 3) respectively, then the radius of the circle
(i) Find the point of intersection of lines x = α and y = β. m
circumscribing the triangle is , where m and n are
(ii) Find the value of (OA + OB + OC) a
relatively prime. Find the value of (m+ n).
(iii) Find the equation of line OA.
M a them a ti cs | 8.45

(You may use the fact that the distance between Q.4 The lines 3x + 4y = 9 & 4x – 3y + 12 = 0 intersect
orthocentre and circumcentre of the triangle is given at P. The first line intersects x-axis at A and the second
R 1 – 8 cos A cosB cosC ) line intersects y-axis at B. Then the circum radius of the
triangle PAB is

Q.29 The points (– 6, 1), (6, 10), (9, 6) and (– 3, – 3) are (A) 3/2 (B) 5/2 (C) 10 (D) None
the vertices of a rectangle. If the area of the portion of
this rectangle that lies above the x axis is a/b, find the Q.5 If the lines ax + y + 1 = 0, x + by + 1 = 0 & x + y +
value of (a +b), given a and b are coprime. c = 0, where a, b & c are distinct real numbers different
from 1 are concurrent, then the value of
Q.30 Consider the triangle ABC with sides AB and AC 1 1 1
+ + =
having the equation L1 = 0 and L2 = 0. Let the centroid. 1–a 1–b 1–c
Orthocentre and circumcentre of the ∆ABC and G, H and
S respectively. L = 0 denotes the equation of sides BC. (A) 4 (B) 3 (C) 2 (D) 1

(i) If L1: 2x – y = 0 and L2: x + y = 3 and G(2, 3) then find Q.6 The points A(a, 0), B(0, b), C(c, 0) & D(0, d) are such
the slope of the line L= 0. that ac = bd & a, b, c, d are all non zero. The points
thus:
(ii) If L1: 2x + y = 0 and L2: x – y + 2 = 0 and H(2, 3) then
find the y-intercept of L = 0. (A) Form a parallelogram (B) Do not lie on a circle

(iii) If L1: x + y – 1 = 0 and L2: 2x – y + 4 = 0 and S(2, 1) (C) Form a trapezium (D) Are concyclic
then find the x-intercept of the line L= 0.
Q.7 The angles between the straight lines joining the
origin to the points common to 7x2 + 8y2 – 4xy + 2x –
Exercise 2 4y – 8 = 0 and 3x – y = 2 is
π π π
(A) tan–1 2 (B) (C) (D)
Single Correct Choice Type 3 4 2

Q.1 Given the family of lines, a (3x + 4y + 6) + b (x + y Q.8 Distance between two lines represented by the line
+ 2) = 0. The line of the family situated at the greatest pair, x2 – 4xy + 4y2 + x – 2y – 6 = 0 is
distance from the point P(2, 3) has equation: 1
(A) (B) 5 (C) 2 5 (D) None
(A) 4x + 3y + 8 = 0 (B) 5x + 3y + 10 = 0 5
(C) 15x + 8y + 30 = 0 (D) None
Q.9 If the straight lines joining the origin and the points
of intersection of the curve
Q.2 On the portion of the straight line, x + 2y = 4 5x2 + 12xy – 6y2 + 4x – 2y + 3 = 0.
intercepted between the axes, a square is constructed
on the side of the line away from the origin. Then the And x + ky – 1= 0 are equally inclined to the co-ordinate
point of intersection of its diagonals has co-ordinates: axes then the value of k:

(A) (2, 3) (B) (3, 2) (C) (3, 3) (D) None (A) Is equal to 1
(B) Is equal to – 1
Q.3 The base BC of a triangle ABC is bisected at the (C) Is equal to 2
point (p, q) and the equation to the side AB & AC are
(D) Does not exist in the set of real numbers
px + qy = 1 and qx + py = 1. The equation of the
median through A is:
Q.10 If the vertices P and Q of a triangle PQR are given
(A) (p – 2q) x + (q – 2p)y + 1 = 0 by (2,5) and (4,–11) respectively, and the point R moves
(B) (p + q) (x + y) – 2 = 0 along the line N: 9x + 7y + 4 = 0, then the locus of the
centroid of the triangle PQR is a straight line parallel to
(C) (2pq – 1)(px+qy –1) =(p2 + q2 – 1)(qx+py – 1)
(A) PQ (B) QR (C) RP (D) N
(D) None
8 . 4 6 | Straight Line

Q.11 Let the co-ordinates of the two points A & B


Q.17 A is a point on either of two lines y + 3 | x |=2 at
be (1, 2) and (7, 5) respectively. The line AB is rotated
through 45° in anticlockwise direction about the point 4
a distance of units from their point of intersection.
of trisection of AB which is nearer to B. The equation of 3
the line in new position is The co-ordinates of the foot of perpendicular from A
(A) 2x – y – 6 = 0 (B) x – y – 1= 0 on the bisector of the angle between them are
(C) 3x – y – 11 = 0 (D) None of these
 2   2 
(A)  – ,2  (B) (0, 0) (C)  ,2  (D) (0, 4)
 3   3 
Q.12 If the line y =mx bisects the angle between the
lines ax2 + 2hxy + by2 = 0, then m is a root of the
quadratic equation: Q.18 All the points lying inside the triangle formed by
the points (1, 3), (5, 6) & (– 1, 2) satisfy
(A) hx2 + (a – b) x – h = 0
(A) 3x + 2y ≥ 0 (B) 2x + y + 1 ≥ 0
(B) x2 + h(a – b) x – 1 = 0
(C) (a – b)x2 + hx – (a – b) = 0 (C) 2x + 3y – 12 ≥ 0 (D) – 2x + 11 ≥ 0

(D) (a – b)x2 – hx – (a – b) = 0 x y
Q.19 Line + = 1 cuts the co-ordinate axes at
a b
x y
Q.13 A Triangle is formed by the lines 2x – 3y – 6 = 0; A(a, 0) & B(0, b) & the line + = –1 at A’ (–a’, 0) &
3x – y + 3 = 0 and 3x + 4y – 12 = 0. If the points P(α, 0) a' b'
and Q(0, β) always lie on or inside the ∆ABC, then B’(0, – b’ ). If the points A, B, A’, B’ are concyclic then the
orthocentre of the triangle ABA’ is
(A) α ∈ [– 1, 2] & β ∈ [– 2, 3]
(B) α ∈ [– 1, 3] & β ∈ [– 2, 4]  aa'   bb' 
(A) (0, 0) (B) (0, b’) (C)  0,  (D)  0, 
(C) α ∈ [– 2, 4] & β ∈ [– 3, 4]  b   a 

(D) α ∈ [– 1, 3] & β ∈ [– 2, 3]
Q.20 If one vertex of an equilateral triangle of side
‘a’ lies at the origin and the other lies on the line
Q.14 In a triangle ABC, side AB has the equation 2x + 3y
= 29 and the side AC has the equation, x + 2y = 16. If the x – 3y = 0 then the co-ordinates of the third vertex
mid-point of BC is (5, 6), then the equation of BC is are:
 3a a   3a a 
(A) x – y = – 1 (B) 5x – 2y = 13 (A) (0,a) (B)  , –  (C) (0, – a) (D)  – , 
 2 2   2 2
  
(C) x + y = 11 (D) 3x – 4y = – 9
Q.21 Three vertices of a triangle are A(4, 3); B(1, –1) and
Q.15 The vertex of a right angle o triangle lies on the C(7, k). Value(s) of k for which centroid, orthocentre,
straight line 2x + y – 10 = 0 and the two other vertices, incentre and circumcentre of the ABC lie on the same
at point (2, – 3) and (4, 1) then the area of triangle in straight line is/are:
sq. units is - 19
33 (A) 7 (B) – 1 (C) – (D) None
(A) 10 (B) 3 (C) (D) 11 8
5
Q.22 Equation of a line through (7, 4) and touching the
Multiple Correct Choice Type circle x2 + y2 – 6x + 4y – 3 = 0 is
(A) 5x – 12y + 13 = 0 (B) 12x – 5y – 64 = 0
Q.16 The area of triangle ABC is 20 cm2. The co-
ordinates of vertex A are (– 5, 0) and B are (3, 0). The (C) x – 7 = 0 (D) y = 4
vertex C lies upon the line, x – y = 2. The co-ordinates
of C are Q.23 The circumcentre of the triangle formed by the
lines, xy + 2x + 2y + 4 = 0 and x + y +2 = 0 is
(A) (5, 3) (B) (– 3, – 5) (C) (– 5, – 7) (D) (7, 5)
(A) (– 2, – 2) (B) (–1 , – 1)
(C) (0, 0) (D) (– 1, – 2)
M a them a ti cs | 8.47

Q.24 The sides of a triangle are x + y = 1, 7y = x and Match the conditions expressions in column I with
– 3y + x = 0. Then the following is an interior point of statement in column II
the triangle.
(A) Circumcentre (B) Centroid Q.2 Consider the lines given by  (2008)

(C) Incentre (D) Orthocentre L1: x + 3y – 5 = 0 ; L2: 3x – ky – 1 = 0


L3: 5x + 2y – 12 = 0
Q.25 Equation of a straight line passing through the
1 Column I Column II
point (2, 3) and inclined at an angle of tan−1   with
the line y + 2x = 5 is 2
(A) L1, L2, L3 are concurrent, if (p) k = – 9
(A) y =3 (B) x = 2
(B) One of L1, L2, L3 is parallel to at least 6
(q) k = –
(C) 3x + 4y – 18 = 0 (D) 4x + 3y – 17 = 0 one of the other two, if 5

(C) L1, L2, L3 form a triangle, if 5


Q.26 A ray of light travelling along the line x + y = 1 (r) k =
6
is incident on the x-axis and after refraction it enters
(D) L1, L2, L3 do not form a triangle, if (s) k = 5
the other side of the x-axis by turning π/6 away from
the x-axis. The equation of the line along which the
refracted ray travels is
Q.3 Three lines px + qy + r = 0, qx + ry + p = 0 and
(A) x + (2 – 3)y =
1 (B) (2 – 3)x + y =
1 rx + py + q = 0 are concurrent, if  (1985)
(C) y + (2 + 3)x =2 + 3 (D) None of these (A) p + q + r = 0 (B) p2 + q2 + r2 = pr + rq
(C) p3 +q3 + r3 = 3 pqr (D) None of these
Q.27 Consider the equation y – y1 = m(x – x1). If m &
x1 are fixed and different lines are drawn for different
Q.4 All points lying inside the triangle formed by the
values of y1, then
points (1, 3), (5, 0) and (– 1, 2) satisfy  (1986)
(A) The lines will pass through a fixed point
(A) 3x + 2y ≥ 0 (B) 2x +y – 13 ≥ 0
(B) There will be a set of parallel lines
(C) 2x – 3y – 12 ≤ 0 (D) – 2x + y ≥ 0
(C) All the lines interest the line x = x1
(D) All the lines will be parallel to the line Q.5 Let L1 be a straight line passing through the origin and
L2 be the straight line x+y=1. If the intercepts made by the
circle x2 + y2 – x + 3y = 0 on L1 and L2 are equal, then which
Previous Years’ Questions of the following equation can represent L3? (1999)
(A) x + y = 0 (B) x – y = 0
The codes (A), (B), (C) and (D) deformed as follows.
(A) Statement-I is true, statement-II is also true; (C) x + 7y = 0 (D) x – 7y = 0
statement-II is the correct explanation of statement-I.
(B) Statement-I is true, statement-II is also true; Q.6 If the distance between the plane Ax – 2y + z = d and
statement-II is not the correct explanation of x –1 y –2 z –3
statement-I. the plane containing the lines = = and
2 3 4
(C) Statement-I is true; statement-II is false. x–2 y –3 z– 4
= = is 6 , then |d| is……….. (2010)
(D) Statement-I is false; statement-II is true. 3 4 5

Q.1 Lines L1: y – x = 0 and L2: 2x + y =0 intersect the line L3: Q.7 The straight line 2x – 3y = 1 divides the circular
y + 2 = 0 at P and Q, respectively. The bisector of the acute region x2 + y2 ≤ 6 into two parts. If
angle between L1 and L2 intersects L3 at R.  (2007)
 3   5 3   1 1   1 1  
Statement-I: The ratio PR: RQ equals 2 2 : 5 . S =  2,  ,  ,  ,  , –  ,  ,   ,  (2011)
 4   2 4   4 4   8 4  
Because
Then the number of point(s) lying inside the smaller
Statement-II: In any triangle, bisector of an angle
part is……………
divides the triangle into two similar triangles.
8 . 4 8 | Straight Line

Q.8 For points P = (x1, y1) and Q = (x2, y2) of the Q.11 Using coordinate geometry, prove that the three
coordinate plane, a new distance d(P,Q) is defined by altitudes of any triangle are concurrent. (1998)
d(P, Q) = | x1 – x2 | + | y1 – y2 | . Let Q = (0, 0) and A =
(3, 2). Prove that the set of points in the first quadrant Q.12 A Straight line L through the point (3, -2) is
which are equidistant (with respect to the new distance) inclined at an angle 60° to the line 3x + y =. 1 If L
from O and A consists of the union of a line segment also intersects the x-axis, then the equation of L is
of finite length and an infinite ray. Sketch this set in a  (2011)
labelled diagram.  (2000)

Q.13 For a > b > c > 0, the distance between (1, 1) and
Q.9 A straight line L through the origin meets the line x the point of intersection of the lines ax + by + c = 0 and
+ y = 1 and x + y = 3 at P and Q respectively. Through bx + ay + c = 0 is less than 2 2 . Then (2013)
P and Q two straight lines L1 and L2 are drawn, parallel
to 2x – y = 5 and 3x + y = 5 respectively. Lines L1 and L2 (A) a + b – c > 0 (B) a – b + c < 0
intersect at R, show that the locus of R as L varies, is a (C) a – b + c > 0 (D) a + b – c < 0
straight line.  (2002)

Q.14 For a point P in the plane, let d1 (P ) and d2 (P ) be


Q.10 Let S be a square of unit area. Consider any the distance of the point P from the lines x – y = 0 and x
quadrilateral which has one vertex on each side of S. If a, b, + y = 0 respectively. The area of the region R consisting
c and d denote the length of the sides of the quadrilateral, of all points P lying in the first quadrant of the plane
prove that 2 ≤ a2 + b2 + c2 + d2 ≤ 4.  (1997) and satisfying 2 ≤ d1 (P ) + d2 (P ) ≤ 4  (2014)

PlancEssential Questions
JEE Main/Boards JEE Advanced/Boards

Exercise 1 Exercise 1
Q.10 Q.13 Q.23 Q.2 Q.6 Q.10
Q.24 Q.26 Q.27 Q.15 Q.19 Q.24
Q.27
Exercise 2
Q.11 Q.12 Q.16 Exercise 2
Q.18 Q.2 Q.6 Q.8
Q.12 Q.18 Q.21
Previous Years’ Questions Q.23 Q.26
Q.2 Q.7 Q.10
Q.11 Q.13 Previous Years’ Questions
Q.2 Q.5 Q.7
Q.9 Q.10
M a them a ti cs | 8.49

Answer Key

JEE Main/Boards
Q.14 x = 2, x + 9y – 14 = 0, 7x – 9y – 2 = 0
Exercise 1
Q.15 (– 4, – 3)
1
Q.1 – 36 45
6 Q.16 – ,–
7 7
Q.4 (7, 0) or (2, 5) Q.17 5

Q.5 (b – b')x + (a'– a)y + ab'– a'b Q.18 30° or 150°


= 0,(b – b')x + (a – a')y + a'b'– ab= 0 22
Q.19
= θ tan–1 ( − )
3
Q.6 5 : 97 externally
Q.20 D = (– 7, 2), C = (– 5, 7)
3   9  9 3 3 3  Q.21 3x – y – 7 = 0 and x + 3y – 9 = 0
Q.7 E  ,0  , F  ,0  , G  ,  , H  , 
 2   4   4 4  2 4  Q.22 52x + 89y + 519 = 0 or 4x + y = 1
Q.8 3x – 8y = 0, 3x – 2y = 0 Q.23 29x – 2y – 31 = 0
Q.9 3y – 2x = 2xy Q.24 (– 16, – 2)
Q.10 (7, 5) and (– 1, – 1) Q.25 26x – 122y – 1675 = 0
Q.11 5 Q.26 x + y + 2 = 0
 3 3  3 13  3 1 3  Q.27 P (2, – 1)
Q.12 C =  – ,  – ,, A
, ,=–,   , – 
 2 2  2 22  2 2 2 
2
Q.28
Q.13 OT → y = x, OS → y = –x, SP → y = x + 4, 3
PQ → y = –x + 4, PR → y = (2 – 3 )x + 4

Exercise 2

Single Correct Choice Type

Q.1 C Q.2 A Q.3 A Q.4 C Q.5 C Q.6 A


Q.7 D Q.8 B Q.9 A Q.10 C Q.11 A Q.12 B
Q.13 A Q.14 C Q.15 D Q.16B Q.17 B Q.18 B
Q.19 D Q.20 D Q.21 D

Previous Years Questions


Q.1 A Q.2 C Q.3 C Q.4 A Q.5 A Q.6 C
Q.7 D Q.8 D Q.9 D Q.10 D Q.11 A Q.12 D
Q.13C Q.14 C Q.15 A Q.16 B Q.17 C Q.18 B
Q.19 C Q.20 B
8 . 5 0 | Straight Line

JEE Advanced/Boards Q.15 (1, – 2), yes ( −1, −2 )


Q.16 x +y = 1; x + 9y = 1
Exercise 1
Q.17 (i) area – 6 sq. units, (ii) diagonals ar 5 & 53
Q.1 8
Q.18 6x2 – xy – y 2 – x – 12y – 35 = 0
Q.2 (A) R; (B) S; (C) Q
Q.19 (i) α =2 and β =3; (ii) 9 2 ; (iii) x – y = 0
Q.3 x + 4y = 4 ; 5x + 2y = 8
Q.20 400 sq. units
Q.4 a = 11, c = 78
Q.21 3 units
Q.5 7x + 24y - 182 = 0 or x = – 2
Q.22 c = – 4; B (2, 0) ; D (4, 4)
 5
Q.6 (0, 0) or  0,  Q.23 x + 5y + 5 2 = 0 or x + 5y – 5 2 = 0
 2
Q.7 5 Q.24 x – 3y – 31 = 0 or 3x+ y + 7 = 0
Q.8 2x – y +3 =0, 2x + y – 7 =0, x – 2y – 6 =0 Q.25 14x + 23y = 40
Q.9 y = 2x Q.26 15
 2t –t   t 
Q.10 B  – .  .C  .t  Q.27 6
 3 6  2 
Q.28 63
Q.11 (y12 – δ2 )x2 – 2x1 y1 xy + (x12 – δ2 )y 2 =
0
Q.29 533
Q.12 (a) 74 ; (b) 50 ; (c) 47
3
Q.30 (a) 5 ; (b) 2 ; (c)
Q.13 24 2
50 63 3
Q.14 (a) ; (b) ; (c) (8 5 – 5 10 )
7 10 10

Exercise 2
Single Correct Choice Type

Q.1 A Q.2 C Q.3 C Q.4 B Q.5 D Q.6 D


Q.7 D Q.8 B Q.9 B Q.10 D Q.11 C Q.12 A
Q.13 D Q.14 C Q.15 B

Multiple Correct Choice Type

Q.16 B, D Q.17 B, C Q.18 A, B, D Q.19 B, C Q.20 A, B, C, D Q.21 B, C

Q.22 A, C Q.23 B, C Q.24 B, C Q.25 B, C Q.26 A, C Q.27 B, C

Previous Years Questions


Q.1 C Q.2 A → s; B → p, q; C → r; D → p, q, s Q.3 A , C Q.4 A, C
Q.5 B , C Q.6 6 Q.7 2 Q.13 A Q.14 6
M a them a ti cs | 8.51

Solutions

JEE Main/Boards 5
−1 +
2 3 1
Slope of GH = = =
Exercise 1 17 5
− 12 4
2 2
Sol 1: Slope of line joining [4, –6] & [–2, –5] 5
4+
−5 + 6 1 Slope of EH = 2 = −13
= = 5
−2 − 4 −6 7
−1 −
2
Sol 2: Line 1 joining (2, –3) and (–5, 1) EF || GH and FG || EH, hence midpoints E, F, G, H form a
parallelogram.
4
⇒ Slope = −
7
Sol 4: AB || DC
 2 → line joining (7, –1) and (0, 3)
2−0
−4 Slope of AB = = –1
⇒ Slope = 0−2
7
7−y y −7
 3 → line joining (4, 5) and (0, –2) Slope of DC = =
0−x x
7 y – 7 = –x ⇒ x + y = 7 – x …(i)
⇒ Slope =
4
1 ||  2 and  m2 ⋅ m3 =
−1 A B

i.e. 1 and  2 are perpendicular to  3

Sol 3: A(–4, 2) ; B(2, 6) ; C(8, 5); D(9, –7) mid points of D C


Trapezium is isosceles i. e. AD = BC
 −4 + 2 2 + 6 
AB = E  ,  = (–1, 4) (x – 2)2 + y2 = 25
 2 2 
x2 + y2 – 4x = 21 …(ii)
2 + 8 6 + 5  11 
BC = F  ,  = 5,  x2 + y2 + 2xy = 49
 2 2   2
4x + 2xy = 28 ⇒ 2x(2 + y) = 28 from equation (i)
 8 + 9 5 − 7   17 
CD = G  ,  =  , − 1
 2 2  2  y= 7-x

 9 − 4 −7 + 2   5 −5  ⇒ x(9 – x) = 14 ⇒ x2 – 9x + 14 = 0
AD = H  , =  , 
 2 2  2 2  ⇒x = 7, 2

 11  If x = 7, y = 0 and if x = 2, y = 5
 2 − 4 3 1
Slope of EF =   = = Ans is (7, 0) and (2, 5)
 5 + 1  12 4
 
Sol 5:
11 D C y = b’
+1 −13
13
Slope of FG = 2 = = y=b
17 −7 7 A B
5−
2

x=a x = a’
8 . 5 2 | Straight Line

Coordinates are A(a, b), B(a′, b), C(a′, b′) and D(a, b′) It is a square hence b – a = c
b′ − b H lies of x = 2y ⇒ a = 2c ⇒ b = 3c
Slope of line AC =
a′ − a G lies on x + y = 3 ⇒ b + c = 3
b′ − b 4c = 3
Slope of line BD =
a − a′
3 9 6
Equation of line AC ⇒c= ; b = ; a=
4 4 4
 b′ − b  Coordinates of square are
⇒y–b=   (x – a)
 a′ − a 
3  9  9 3 3 3 
⇒ (b′ – b)x + (a – a′)y – ab′ + a⇒b = 0 E  ,0  , F  ,0  , G  ,  , H  , 
 2   4   4 4  2 4 
Equation of line BD
b′ − b
⇒y–b= (x – a′) Sol 8:
a − a′
⇒ (b′ – b)x – (a – a′)y – a'b' + ab = 0 (0,3) A
B

Sol 6: 1 (line joining (2, 3) and (4, –5)) (4,0) 3x+4y=12

 2 (line joining (6, 8) and (–3, –2))

(6, 8)  1(4,0) + 2(0,3)  4 6


A=  ⇒  , 
 3  3 3
m 1
 2(4,0) + 1(0,3)  8 3
(2, 3) F (4, -5) B=  ⇒  , 
 3  3 3
(-3, -2) 3x
Line OA → y = ⇒ 3x – 2y = 0
2
Lets assume it divides 1 in ratio m : 1
3x
Line OB → y = ⇒ 3x – 8y = 0
 4m + 2 3 − 5m  8
Coordinates of F are  , 
 m+1 1+m 
Sol 9:
y −8 10
Equation of  2 is =
x−6 9
3 − 5m
−8
10 −5 − 13m 10 B
⇒ 1+m = ⇒ =
4m + 2 9 −2m − 4 9
−6 A
m+1
−5
⇒ 45 + 117m = 20m + 40 ⇒ m = (3,-2)
97
i. e. in ratio 5 : 97 (externally).
y+2
=m
x −3
Sol 7:
y + 2 = m(x – 3)
B  2 
H G A ⇒  3 + ,0  [x intercept]
 m 
A E F C B ⇒ (0, –2 – 3m) [y intercept]

 2 
 3 + m −2 − 3m 
E = (a, 0), F = (b, 0), G = (b, c) Mid point of AB  , 
 2 2 
H = (a, c)  
M a them a ti cs | 8.53

−3m − 2 3m + 2 −3
y= ,x= Slope of AC =
2 2m 2
 2y + 2  BD = 13
m = – 
 3  Length of side =
13
 −2y − 2   −2y − 2  2
⇒ 2x   = 3  +2
 3   3  Let slope of AB be m

⇒ –4xy – 4x = –6y – 6 + 6 2
m−
tan 45º = 3 =1
⇒ 2xy + 2x – 3y = 0 2m
1+
3
y −2 3
Sol 10: = and y = 3x – 1 m 5 5m −1 −1
x −3 4 = ⇒ m = 5 or = ⇒m=
3 3 3 3 5
Points which are 5 units away from (3, 2) are
 13 13 
A = 1 + cos θ, 1 + sin θ 
(3 ± 5 cos θ, 2 ± 5 sin θ )
 2 2 
3 −1 −5
tan θ = ⇒(3 ± 4, 2 ± 3)⇒ [7, 5] [–1, –1] Now cos θ = , sin θ =
4
26 26

Sol 11: Coordinates of A at r units from B  1 −3 


⇒A  , 
A(2 + r cosθ, 5 + r sin θ) 2 2 
−1 1 −5
m= , sin θ = , cosθ =
B 5 26 26
2,5
3/4  13 13 
r C = 1 + cos θ,1 + sin θ 
 2 2 
3x + y + 4 = 0
A
 3 3
3 ⇒ C = − , 
tan θ =  2 2
4
 4r 3r  Sol 13: OT → y = x [O(0, 0), T (2, 2)]
A 2 + ,5 + 
 5 5
OS → y = –x [O(0, 0), S (–2, 2)]
A lies on given line SP → y = x + 4 [as OP = 4]
 4r  3r PQ → y = –x + 4 [as OQ = 4]
⇒ 3 2 +  + 5 + +4=0
 5  5
PR → y = mx + 4
15 + 3r = 0 ⇒ r = –5
3 −1
A[2 – 4, 5 – 3] = [–2, 2] m = tan 150 → =2– 3
3 +1
r = 5 units ⇒ y = (2 – 3 )x + 4
OR → y = mx
Sol 12:
R[4 + 4 2 cos75, 4 2 sin 75]
(-2,-1)
D C  ( 3 − 1) ( 3 + 1) 
R 4 + 4 2 ,4 2 
m  2 2 2 2 
A B (1,1)
R[4 + 2( 3 –1), 2( 3 +1)]
2
Slope of BD = R[2( 3 +1), 2( 3 +1)]
3
⇒y=x
8 . 5 4 | Straight Line

Sol 14 : A Sol 16: A (3, -1)

x-3y=2 x+y-6=0 O
G E
F (0, 0)
C B B C
5x-3y+2=0
(-2, 3) (a, b)
Solving
Orthocentre is (0, 0)
x + y – 6 = 0, x – 3y – 2 = 0 gives A[5,1]
1
x + y – 6 = 0, 5x – 3y + 2 = 0 gives B[2,4] (slope)OA = −
3
x – 3y – 2 = 0, 5x – 3y + 2 = 0 gives C[–1,–1] b−3
(slope)BC = 3 =  …. (i)
 7 5  1 3  a+2
Midpoints are E  ,  , F  ,  , G[2, 0]
2 2 2 2 −3
(slope)OB =
2
Equation of median
2 b +1
1 (slope)AC = =  …. (ii)
y −1 3 a−3
AF ⇒ = 2 ⇒ x + 9y = 14
x −5 9 Solving (i) and (ii), we get

2 b = 3a + 9 and 3b = 2a – 9
y−4 1
BG ⇒ = ⇒x=2 9a + 27 = 2a – 9
x−2 0
36 −108 + 63 45
y +1 7 a= − ; b= = −
CE ⇒ = ⇒ 7x – 9y = 2 7 7 7
x +1 9
 36 45 
⇒ C − , − 
Sol 15:  7 7 
C(-1,3)
Sol 17: Lines are concurrent intersection of 1 and M  3
gives x = 1 y = –1
It lies on  2 ⇒ p – 2 – 3 = 0 ⇒ p = 5
A(0,0)
B(2,-1)
x−6
−4 Sol 18: 1 ⇒ y = 3x + 5 2 ⇒ y =
Slope of line BC = 3
3 1
m1 = 3 m2 =
3 3
Slope of line perpendicular to BC =
4 1
3−
⊥ bisector through A 3 1
tan θ = =
3x 3 3
y= …. (i) 1+
4  3
Slope of line AC = –3
⇒ θ = 30º, 150º
1
Slope of line perpendicular to AC =
3 Sol 19:
⊥ bisector through B
3
A(2, –1) mAB = −
3y = x – 5 …. (ii) 2
Solving (i) & (ii), we get 1
B(0, 2) mBC =
x = –4; y = –3 3
−3
C(3, 3) mCD =
Ans is (–4, –3) 2
M a them a ti cs | 8.55

1
D(5, 0) mAD = A (2, -7)
3
BC || AD and AB || CD B
4x+y=1
⇒ ABCD is a parallelogram
C
⇒ Diagonals are AC & BD 3x-4y+1=0
−1 − 3
Slope of AC → =4 Solving equations we get B
2−3
3x + 1
2 − 0 −2 = 1 – 4x ⇒ 19x = 3
Slope of BD → = 4
0 −5 5 3 7
−2 ⇒x= ,y=
m1 = 4; m2 = 19 19
5
2 3 7
4+ ∴ B , 
5 22 −22  19 19 
tan θ = = =
8 −3 3
1− 2 2
5  3   7 
AB = 2 −  +  + 7
 19   19 
 −22 
φ = tan–1  
 3  (35)2 + (140)2 35
= = 17
19 19
Sol 20: A(–2, 0) ; B(0, 5)
C = (2 + r cosθ, –7 + r sinθ)
5
mAB = rAB = 29 3(2 + r cosθ) – 4(–7 + r sinθ) + 1 = 0
2
35 + 3r cosθ – 4r sinθ = 0
C = 0 + 29 cos θ,5 + 29 sin θ 
  3 17 4 17
1+ cosθ – sinθ = 0
D =  −2 + 29 cos θ,0 + 29 sin θ  19 19
  19
–3 cosθ + 4 sinθ =
−2 2 −5 17
tan θ = ; sin θ = ; cos θ =
5 29 29 −52
tan θ = –4 or
C = [–5, 7] D = [–7, 2] 89
y+7 −52
Equation of AC ⇒ = –4 or =
x−2 89
y −2
Sol 21: =m 52x + 89y + 519 = 0 or 4x + y = 1
x −3

1 Sol 23:
m−
2 m=m
tan 45 = m=1/2 m=–2/3
m
1+
2 x–2y–3=0
2m − 1
= ±1
m+2
1 m=3/2 3x–2y–5=0
m = +3, m = −
3
Points of intersection is (1, –1) and both the lines
⇒y – 2 = 3x – 9 ⇒ y = 3x – 7 x – 2y – 3 = 0 and reflected Line are equally inclined to
normal on 3x – 2y 5 = 0
1
& y – 2 = − (x – 3) ⇒ x + 3y = 9 2 −2 1
3 m+ −
3 = 3 2
2m 1
Sol 22: 1− 1−
3 3
8 . 5 6 | Straight Line

3m + 2 −7 13
⇒ = ⇒ Slope of BC =
3 − 2m 4 61
33
⇒ 2(6m + 4) = –21 + 14m y −b 13 y+
⇒ = ⇒ 2 = 13
⇒ 2m = +29 x−a 61 x + 13 61
29 61 × 33
⇒ m=+ ⇒ 61y = 13x + 13×13 –
2 2
29 ⇒ 26x – 122y – 1675 = 0
⇒ (y + 1) = (x – 1)
2
⇒ 2y + 31 = 29x Sol 26: The equation of line passes through point of
intersection of x+3y+y=0 and 3x+y+4=0 is
Sol 24:
3x+y+4 + λ (x+3y+4) = 0
(-8, 12)
(λ+3)x + (1+3 λ)y + 4 + 4 λ =0
The obtained line is equally inclined to axes, then
4x+7y+13
Slope of line = ± 1
λ+3 λ+3
(a, b) − 0
= and − −1
=
1 + 3λ 1 + 3λ

a+8 b − 12 65 ⇒ λ + 3 = - 1 -3 λ or λ + 3 = 1 +3 λ
= = –2 .
4 7 65 ⇒ λ = -1 or ⇒ λ = 1
a+8 b − 12 Eqn of line is (-1 + 3)x + (1 - 3)y + 4 - 4=0
⇒ = = –2
4 7
⇒ 2x − 2y= 0 ⇒ x= y and
⇒ a = –16 & b = –2
(1+3)x + (1 + 3)y + 4 + 4=0
Image ≡ (–16, –2)
⇒ 4x + 4y + 8 =0
⇒ x+y+2 = 0
Sol 25: A

Sol 27: x(a + 2b) + y(a + 3b) – a – b = 0


3x-2y+6 4x+5y=20 a(x + y – 1) + b(2x + 3y – 1) = 0
(1,1)
Fixed point is the intersection of given 2 lines
B C x + y = 1 & 2x + 3y = 1
 10 84 
A=  ,  ⇒ y = –1; x = 2; P (2, – 1)
 23 23 
B = (a, b) ⇒ 3a – 2b + 6 = 0 Sol 28:
b −1 5 (2, -1)
= ⇒ 4b = 5a – 1
a−1 4
Solving these, we get
60
6a + 12 = 5a – 1
x+y=2
−33
a = –13 b= −1 1
2 Distance of vertex from line is =
84 2 2
−1 Length of base = 2 a
−61
AB slope = 23 =
10 13
−1 2
23 =2
3
M a them a ti cs | 8.57

2 1 1 1
1 2 m+ − −
sin 60º = ⇒a= = 3
2a 3 3 = 3 7
m 1
1− 1−
3 21
Exercise 2
3m + 1 −10 3m + 1 −1
⇒ = ⇒ =
Single Correct Choice Type 3−m 20 3−m 2

⇒ 6m + 2 = m – 3 ⇒ m = –1
Sol 1: (C) 3x – 8y – 7 = 0
 −1 7 
3(0) − 8( −1) − 7 =
1 It passes through  , 
(A)  different sides  10 10 
3(0) − 8(0) − 7 =−7  7
y−
10 = –1⇒ x + y = 7 – 1 ⇒ 5x + 5y = 3
3(0) − 8(1) − 7 =−15
(B)  different sides 1 10 10
3(3) − 8(0) − 7 =2  x+
10
3( −1) − 8(1) − 7 =−2 
(C)  same sides Sol 5:(C) X = x – h = 3 – 4 = –1
3(3) − 8(7) − 7 =−54 
Y = y – k = –5 + 3 = –2
Sol 2: (A) 3x – 4y + 7 = 0; 12x + 5y – 2 = 0
x y
p1p2 + q1q2 = 36 – 20 = 16 > 0 Sol 6:(A) + =1
a b
3x − 4y + 7 −(12x + 5y − 2) 3 4
= + = 1 are concurrent at a fixed point.
5 13 5a 5b
39x – 52y + 91 = –60x – 25y + 10 3 4
Point is x = and y =
5 5
99x – 27y + 81 = 0 ⇒ 11x – 3y + 9 = 0
3 4 
⇒  , 
Sol 3: (A) 5 5 
(5,3)
(1,2) C
A Sol 7: (D) P (1, 0) ; Q (–1, 0) ; R (2, 0)
 
2SP2 = SR2 + SQ2
B
(x+1)2+y2+(x–2)2+y2 = 2(x–1)2+2y2
⇒ x2+1+2x+x2+4–4x=2(x2+1–2x)
C’
(5,-3)

⇒ 5 – 2x = 2 – 4x
C’ is the reflection of C w.r.t. x-axis. 3
2x = –3 ⇒ x = −
∴ Eq. of AB = Eq. of AC’ 2

y +3 2+3 5 Which is a straight line parallel to y-axis.


AC’ = = −
x −5 1 −5 4
Sol 8: (B) ax ± by ± c = 0
⇒ 5x + 4y = 13
x y
⇒ ± ± 1
=
Sol 4: (C) ca cb
x-7y+5=0 (m = 1/7)
1 c c  2c2
∴ Area = 4  ⋅ ⋅  =
2 a b ab

x+3y=2 (m=-1/3)
 Sol 9: (A) Lets assume 2 lines are x and y axis so the
distances from 2 lines are …. .
8 . 5 8 | Straight Line

y
2a 1 + m2
2r = =2a ⇒r=a
1 + m2
x
Sol 13: (A) 12x2 – 16xy + 9xy – 12y2 = 0
4x(3x – 4y) + 3y(3x – 4y)
(4x + 3y)(3x – 4y) = 0
|x|+|y|=1
2y=x
It forms a square.
3x-4y=0

Sol 10: (C) Given 3 lines are concurrent m

1 2 9 3 3 1
3 −5 5 = 0 m− −
4
= 4 2
a b 1 3m 3 1
1+ 1+ ×
4 4 2
–5 – 5b – 2(3 – 5a) + 9(3b + 5a) = 0
–5 – 5b – 6 + 10a + 27b + 45a = 0 4m − 3 2
= ⇒ 44m – 33 = 8 + 6m
55a + 22b – 11 = 0 4 + 3m 11
41x
5a + 2b = 1 38m = 41 ⇒ y =
38
The straight line 5x + 2y = 1 passes through (a, b)
Sol 14: (C) A = (4, –1)
Sol 11: (A)
B is symmetric to A w. r. t. y = x
B is [–1, 4]

AB = 25 + 25 = 5 2

Sol 15: (D) A(–4, 0); B(0, 2); C(–3, 2)


C(-3,2)

px2 + 2axy + qy2 = r (homogenising) m

px2 + 2axy + qy2 = r[ax + by]2 (-4,0)A B(0,2)


D
x2(p – ra2) + y2(q – rb2) + 2axy – 2abxy =0 (-2,1)

Lines are perpendicular i. e. a + b = 0 y −1 −1


Eq. of CD ⇒ =
p + q = r(a2 + b2) x+2 1
y – 1 = –2 – x
Sol 12: (B) y – y1 = m(x – x1) ± a 1 + m2 x + y = –1
1
C1 m1 mAB = mAC = +2
2
1
2r m−
2−m 2 = 2m − 1
C2 ⇒ =
1 + 2m m 2+m
m2 1+
C1 − C2 2
2r =
1 + m2 ⇒ 4 – m2 = 4m2 – 1 ⇒ m = ± 1
y
=1
x+4
M a them a ti cs | 8.59

Eq. AD y=x+4 Sol 20: (D) x2 + y2 – 4xy = 0


Intersection point –1 – x = x + 4 ⇒ 2x = –5; 2y = 3 m1 . m2 = 1
+4
m 1 + m2 = = +4
Sol 16: (B) 2x – y + 5 = 0 m=2 1
x + y –5 = 0 m = –1 ⇒ m1 = 2 – 3
1 m2 = 2 + 3 & m3 = −1
x – 2y – 5 = 0 m=
2 It forms an equilateral triangle.
1 1
−1 − −2
3 2
∑ tan θ = + + 2
1−2 1 1+1
1−
2
3 3 27
=3+3+ = 6+ =
4 4 4

Sol 17: (B) y = 0.2x


m = 0.2
tan θ = 0.2
2 tan θ 0.4 0.4 5
tan 2θ = = = =
2
1 − tan θ 1 − 0.04 0.96 12 Sol 21: (D) y = x (A lies on line 1)
5x A(x, x)
⇒y=
12 y = 7x (B lies on this line  2 )
(y, 7y)
Sol 18: (B) x2 + y2 – 10x – 14y – 151 = 0
M(–7, 2) OA = OB ⇒ x 2 = y 50

49 + 4 + 70 – 28 – 151 < 0 x = 5y

Point is inside 7y − x 2y −1
Slope of line AB = = =
y−x −4y 2
A(–7, +2) and Centre (5, 7)

r= 25 + 49 + 151 = 15

AC = 144 + 25 = 13 Previous Years’ Questions


(r – AC) = 2 (minimum distance)
Sol 1: (A) The point O(0,0) is the mid point of A(-a,-b)
and B(a,b).
Sol 19: (D) ax2 + 2hxy + by2 = 0
Therefore, A,O,B are collinear and equation of line AOB
Image by line mirror y = 0
b
is y = x
a
Since, the fourth point D(a2, ab) satisfies the above
equation. Hence, the four points are collinear.

Sol 2: (C) Given lines, x+2y–3=0 and 3x+4y–7=0 intersect


at (1,1), which does not satisfy 2x+3y–a=0 and 4x+5y–
Replace y by (–y) 6=0. Also, 3x+4y–7=0 and 2x+3y–4=0 intersect at
ax2 – 2hxy + by2 = 0 (5,-2) which does not satisfy x+2y–3=0
4x+5y–6=0. Lastly, intersection point of x+2y-3=0 and
2x+3y-4=0 is (-1,2) which satisfy 4x+5y-6=0. Hence,
only three lines are concurrent.
8 . 6 0 | Straight Line

Sol 3: (C) Let B, C, D be the position of the point A(4, 1) Sol 9: (D) Since, S is the mid point of Q and R.
after the three operations I, II and III respectively. Then,
7 + 6 3 − 1 13
B(1, 4), C(1+2, 4) ie, (3, 4). The point D is obtained from ∴ S≡ , = ,1
2 2 2
C by rotating the coordinate axes through an angle π/4
in anticlockwise direction. 2 −1 2
Now, slope of PS = m = = −
Therefore, the coordinates of D are given by 2 − 13 / 2 9

π 1 Now, equation of the line passing through (1, -1) and


x = 3cos= − parallel to PS is
4 2
π π 7 2
and Y =3sin + 4 cos = y + 1 =− (x − 1)
4 4 9
2
⇒ 2x + 9y + 7 =0

 1 7 
∴ Coordinates of D are  − ,  Sol 10: (D) Let the vertices of triangle be A (1, 3) , B(0,
 2 2 0 ) and C (2, 0).
Here AB = BC = CA = 2.
Sol 4: (A) The point of intersection of three lines are
A(1, 1), B(2, -2), C(-2, 2). Therefore, it is an equilateral triangle. So the incentre
coincides with centroid.
Now, | AB |= 1+9 = 10 ,| BC |= 16 + 16 = 4 2,
and | CA |= 9 +1 = 10 0 +1+ 2 0 + 0 + 3   1 
∴ I= ,  =  1, 
 3 3   3
 
∴ Triangle is an isosceles
Sol 11: (A) On solving equations 3x + 4y = 9 and y =
Sol 5: (A) By the given condition, we can take two mx + 1, we get
perpendicular lines as x and y axes. If (h, k) is any point 5
on the locus, then |h|+|k|=Sol 1 Therefore, the locus is x=
3 + 4m
|x|+|y|=1 This consist of a square of side 1.
Hence, the required locus is a square. Now, for x to be an integer
3 + 4m = ± 5 or ± 1
Sol 6: (C) Orthocentre of right angled triangle is at
the vertex of right angle. Therefore, orthocentre of the The integral values of m satisfying these conditions are – 2
triangle is at (0, 0). and – 1

Let y cos x cos(x + 2) − cos2 (x + 1)


Sol 7: (D)= Sol 12: (D) Let lines OB : y = mx
= cos(x + 1 − 1)cos(x + 1 + 1) − cos2 (x + 1) CA : y = mx + 1
2 2 2
= cos (x + 1) − sin 1 − cos (x + 1) BA : y = nx + 1
y = − sin2 1. and OC : y = nx
This is a straight line which is parallel to x-axis It passes
The point of intersection B of OB and AB has x-coordinate
through ( π / 2, − sin2 1).
1
.
Sol 8: (D) Slope of line x+3y=4 is -1/3 m–n y

And slope of line 6x-2y=7 is 3. A


 −1 
Here, 3 ×   = −1 C D B
 3  x’ x
Therefore, these two lines are perpendicular which O
show that both diagonals are perpendicular.
Hence, PQRS must be a rhombus.
y’
M a them a ti cs | 8.61

Now, area of parallelogram 8 14 30


2× + k ⇒
= k
=
OBAC = 2 × area of DOBA 5 5 5

1 1 1 ⇒k=
6
= 2 × × OA × DB = 2 × ×
2 2 m–n
1 1 Sol 15: (A) Sides OA = OB = 2 and AB = 2 2
= =
m–n |m–n| X-coordinates of incentre of ∆ OAB

Sol 13: (C) The equation of the line passing through


points P(-1, 0) and Q(0, 0) is;
Y = 0 … (i)

R(3, 3 3)

Q(0, 0)
P(-1, 0) 2 2 ×0 + 2 × 0 + 2 × 2

2 2 +2+2
Equation of the line passing through points Q(0, 0) and
R(3, 3 ) is; 4 2
= = = 2− 2
4 + 2 2 2+ 2 2
y −0 3 3 y 3 3
= ⇒=
x−0 3−0 x 3
 6 + 7 −1 + 3   13 
⇒ y =3x  … (ii) Sol 16: (B) S ≡  , ; ≡  , 1
 2 2   2 

Therefore, the equations of the bisector of the angle 2 −1 2


Slope of PS = = −
PQR is 13 9
2−
2
y 3x − y 3x − y
=± ⇒ y=±
12 3+1 2

3x − y 3x + y
⇒ y = or y=

2 2
⇒ 3x
= 3x + y 0 .
− 3y 0 or =

2
Sol 14: (C) Point T is given by Equation of line passes through having slope −
9
 3 × 2 + 2 × 1 3 × 4 + 2 × 1   8 14  2
T ≡
 3+2
,
3+2
 ≡ ,
 5 5 
 y + 1 =−
9
( x − 1) ⇒ 9y + 9 =−2x + 2
⇒ 2x + 9y − 7 =0

Sol 17: (C) The given equation of lines are


4ax + 2ay + c = 0 and 5bx + 2by + d = 0
Since, these lines intersect in fourth quadrant and point
is equidistant from axes, then

Which lies on the line 2x + y = k


8 . 6 2 | Straight Line

The point can be of form (k, -k), k > 0 On solving, we get


⇒ 4ak − 2ak + c =0 and 5bk − 2bk + d =0 B ≡ (1, 2 )
⇒ 2ak + c =0 and 3bk + d =
0 We know that diagonals in rhombus bisect each other, so
c d D ≡ ( −3, − 6 )
⇒− =−
2a 3b
Since BC || AD
⇒ 3bc − 2ad =
0
⇒ Equation of AD 7x − y + λ1 = 0 , Passes through
Sol 18: (B) (2x - 3y + 4) + k (x – 2y + 3) = 0 (-3, -6)

Is a family of equation passes through T (1, 2) ⇒ 7x − y + 15 =0

From figure

Similarly, AB || DC
PT = P’T
⇒ Equation of DC x – y + λ2 = 0, passes through
(P' T )
2
⇒ PT2 = (-3, -6)
⇒ (h − 1 ) + (k − 2 ) =( 2 − 1 ) + ( 3 − 1 )
2 2 2 2
⇒ x − y −3 =0
⇒ (h − 1 ) + (k − 2 ) = 1 + 1 = 2
2 2

1 8  7 4
⇒ ( x − 1) + ( y − 2) =
2
2
2 ⇒ C  , −  and A  − , − 
3 3  3 3
Locus is a circle.

Sol 19: (C) Equation of AB, x + y = 41


JEE Advanced/Boards
On line x = 1, there are 39 points inside ∆ OAB
Similarly Exercise 1
Sol 1: OA = 2AB = 4BC

A 1 2
S G H
(7/2, 5/2) (2, 4)

2+7 4 +5
G= , = (3, 3)
3 3
A = (3, 3) = (a, a)
On line x = 2, there are 38 points inside ∆ DAB  3a a
B =  ,a − 
Total points  2 2 
= 39 + 38 + ….. + 2 + 1
 a a a a
39 ( 40 ) C =  a + + ,a − + 
= = 780  2 4 2 4
2  a a a a a a 
⇒  a + + + + ...,a − + − ....x 
 2 4 8 2 4 8 
Sol 20: (B) Let two sides AB and BC be x – y + = 0 and
7x – y – 5 = 0 respectively.
M a them a ti cs | 8.63

  A = (–2, 3)
 a a   2a 
N=  ,  = 2a,  ⇒ Eq. of AB is x + y = 1  … (i)
1 − 1 1 + 1   3
  Eq. of AC is x + 2y = 4  … (ii)
 2 2
8a 3 5
α+β= =8 S=  , 
3 2 2
⇒ Eq. of perpendicular bisector of AB and AC are
Sol 2: y = x + 1 … (iii)
A(-3,1)
1
y = 2x –  … (iv) respectively.
2
(,) Eq. (i) and (iii) ⇒ E = (0, 1) [Midpoint of A, B]

7x-4y=1 2x+y=3 Eq. (ii) and (iv) ⇒ F = (1, 3/2) [Midpoint of A, C]



(2+3,2b-1) ⇒ B = (2, –1) and C = (4, 0)
B m
y −0 1−0
Eq. of median to AB is = ⇒ x + 4y = 4
7(2α + 3) – 4(2β – 1) = 1 x−4 0−4
14α – 8β + 24 = 0 ⇒7α – 4β + 12 = 0 Eq. of median to AC is
3
2α + β – 3 = 0 y +1
+1
= 2 ⇒ 5x + 2y = 8
α = 0, β = 3 ⇒ C = (3, 5) x−2 1−2
2
Slope of AC =
3 Sol 4: A(-8,5)
7 7 2
m− −
4 = 4 3 ⇒ 4m − 7 = 13 = 1  
7m 7 2 7m + 4 26 2
1+ 1+ . m
4 4 3
8m – 14 = 7m + 4 ⇒ m = 18
B C
y −5 2 (1, -7)
Equation of AC = = (-15, -19)
x −3 3
24
⇒ 3y – 9 = 2x mAB =
7
y −5
Equation of BC = = 18 12 −4
x −3 mAC = =
−9 3
⇒ 18x – y = 49
24 4
−m m+
2x + y = 3; 18x – y = 49 7 3
=
26 −11 24m 4m
x= ,y= 1+ 1−
10 5 7 3
24 − 7m 3m + 4
 26 −11  ⇒ =
B=  ,  24m + 7 3 − 4m
 10 5 
−11 ⇒ 72 – 117m + 28m2 = 72m2 + 117m + 28
−1
y −1 −16 −4
Equation of AB = = 5 = = ⇒ 44m2 + 234m – 44 = 0
x+3 26 28 7
+3
10 ⇒ 22m2 + 117m – 22 = 0
⇒ 7y – 7 = –4x – 12 ⇒ 4x + 7y + 5 = 0 ⇒ (11m – 2) (2m + 11) = 0
⇒ m = 2/11, –11/2
Sol 3: x – y = 4 [Line perpendicular to AB]
y −5 2 11
2x – y = 5 [Line perpendicular to AC] = or –
x+8 11 2
8 . 6 4 | Straight Line

⇒ 11y – 55 = 2x + 16 or 2y – 10 = –11x – 88 ⇒ y – 7x – C2 = ±(5y – 5x – 5C1)


⇒ 2x – 11y + 71 = 0 or 2y + 11x + 78 = 0 ⇒ 4y+2x–5C1+C2=0 and 6y – 12x – 5C1 – C2=0
⇒a = 11; c = 78 go through point (1, 2) (given)
⇒ 10 – 5C1 + C2 = 0 and 5C1 + C2 = 0
Sol 5: 4x+3y=12 –2C2 = 10 ⇒ C2 = –5

9/5 3 C1 - C2
=
9 –5C1 = 5 ⇒ C1 = 1
2
a +b
2
5
m ⇒ Diagonals are
4x+3y=3
4y + 2x – 10 = 0 ⇒ 2y + x – 5 = 0
(-2, -7)
6y – 12x + 0 = 0 ⇒ y = 2x
(–2+r cosθ, –7+r sinθ) lies on 4x + 3y = 3 If A vertex is at y-axis ⇒ x = 0
–2+(r+3)cosθ, –7+(r+3)sinθ lies on 4x+3y = 12 y = 2(0) = 0 ⇒ (0, 0)
4(rcosθ – 2) + 3(rsinθ – 7) = 3  …. (i) 2y + 0 – 5 = 0 ⇒y = 5/2 ⇒(0, 5/2)
4((r+3)cosθ – 2) + 3((r+3)sinθ–7)=12  ….(ii)
On solving (i) and (ii), we get Sol 7: O(0, 0) ; A(6, 0) ; B(3, 3)

–12cosθ – 9sinθ = –9 B(3,3)


⇒ 4cosθ + 3sinθ = 3 23 I 23
⇒ 9+16cos2θ – 24 cosθ = 9 – 9 cos2θ A
O (6,0)
⇒ 25 cos2θ = 24 cos θ
⇒ cos θ = 0 or cos θ = 24/25 d(P, OA) = d(P, OB) if P lies along the angular bisector of
−7 AOB. Similarly if d(P, OA) = d(P, AB) then P lies along the
⇒ sin θ = 1 or sin θ = angular bisector of OAB
24
−7 Hence the region where
⇒ m = ∞,
24
d(P, OA) ≤ minimum {d(P, OB), d(P, AB)} is OAI.
y+7 −7
⇒ = or ∞ 1  OA AOB 
x+2 24 Area of OAI = OA  tan 
2  2 2 
⇒ x = –2 or 24y + 168 = –7x – 14
⇒ 7x + 24y = 182 or x = –2
=
1
2
(
6 3tan150 ) = 9 (2 − 3 )
⇒a+b=2+3=5
Sol 6:
1 Sol 8:
(4,-1)
A
(0,a) x=1 A
(1,2) x-y=1
7 (2,4-a) (1,) 1
2
 O
 1 (1, )
(, -1) B C
Sides of the rhombus are
Reflection of (4, –1) about both of bisectors lie on the
y = x + C1; y = 7x + C2 line BC
Eq. of diagonals So reflection about x – 1 = 0 is A’(2, 1)
y − 7x − C2  y − x − C1  Reflection about x – y = 1 is A’’(0, 3)
⇒ = ±  
50  2  Hence Eq. of BC is y – 3 = 2x
M a them a ti cs | 8.65

B and C are intersections of BC with Sol 10:


x – y – 1 = 0 and x = 1 respectively A(t, -t)

⇒ B = (4, 5); C = (1, 5) (-1/2)


(-1)
⇒ Eq. of AB is x – 2y = 6 F
O
E
(4)
Eq. of AC is 2x + y = 7
B C
Sol 9: The equation of line D 1

(y – 2) = m(x + 1) …. (i) mBC = 1


 −2  mAC = 4 ⇒ y = 4x – 5t  … (i)
⇒A=  − 1,0  ; B = (0, 2 + m)
 m  −1
mAB= ⇒ 2y+ x + t = 0  …(ii)
P 2
(-1,2) B Solving AB and BE we get x = –4y
Q
t  −4t t 
A t = 2y ⇒ y = ⇒B=  , 
2  2 2
CF and AC ⇒ y = 2x
Assume Q = (h, k)
 5t 
Q is on the line 2x = 5t ⇒ C =  ,5t 
2 
⇒ k = 2 + m(h + 1)
G = (h, k)
PQ = (h+ 1)2 + (k − 2)2 5t 4t t
3h = t + − ; 3k = t + 5t +
2 2 2
= (h+ 1)2 + m2 (h+ 1)2 = | h+ 1 | m2 + 1 t 3t
h= ;k= ⇒ k = 3h
2 2
2
 −2  2
PA = 
2
− 1 + 1 + 2 = 1 + m2
m  m Sol 11: y = m1x; y = m 2x
y2 – (m1 + m2)xy + m1m2x2 = 0
PB = 1 + m2
m1 x1 − y1 m2 x1 − y1
It’s given that PA, PQ, PB are in H. P. = =δ
1 + m12 1 + m22
2 1 1
⇒ = +
PQ PA PB (mx1 – y1)2 = d2(1 + m2)
2 m 1 1 m2x12 + y12 – 2mx1y1 = d2 + d2m2
⇒ = +
| h+ 1 | m2 + 1 2 1 + m2 1 + m2 m2(x12 – d2) + m(–2x1y1) + y2 – d2 = 0
2 1 k −2 2x1 y1 y 2 − δ2
⇒= +1 m 1 + m2 = ; m 1m 2 =
| h+ 1 | 2 h + 1 x12 − δ2 x12 − δ2

⇒ 2|x + 1| + |y – 2| = 4 2x1 y1 y12 − δ2


y2 – xy + x2 = 0
If m < 0 then y =2x x12 −δ 2
x12 −δ 2

If m > 0 then 2x + y = 4 (If x > 1 and y > 2) (x12 – d2)y2 + (y12 – d2)x2 – 2x1y1xy = 0

= 4 (If x < 1 and y < 2)


8 . 6 6 | Straight Line

Sol 12: A(1,-2) β −5


3− 
 2  11 − β
⇒ –1 = ⇒ –1 =
β +1 3−β
2x+y=0 2− 
P(2,3)  2 
x-y=3
14
⇒ β – 3 = 11 – β ⇒ β = =7
(,-2)B x+py=q C(,-3) 2
⇒ 7 + 4p = q
If P is centroid
⇒ –13 + 26p = 5q
⇒α+β+1=6 …(i)
⇒ 35 + 20p = 5q
⇒ –2 – 2α + β – 3 = 9
⇒ –48 + 6p = 0
⇒ 2α – β = –14 …(ii)
⇒ p = 8 ; q = 39
⇒α+β=5
⇒ p + q = 47
⇒ α = –3; β = 8
B and C lie on line BC Sol 13: x2 – 3y2 – 2xy + 8y – 4 = 0
α−
⇒ α 2pα
− 2p α= q
=q Assume eq. are
+ p(β − 3) =q
⇒ β + p(β − 3) =
β q (x + ay + c)(x + by + d) = 0
⇒ –3 + 6p = q ⇒ db = – 3; dc = – 4
⇒ 8 + 5p = q ⇒ a+b = –2; bc+ad = 8
⇒ p = 11; q = 63 ⇒ p + q = 74 ⇒ (a–b)2= 4 + 12 =42d + c = 0
If P is orthocentre a – b = 4⇒ d = –c
m(AP) = 5 ⇒ a = 1⇒ –c2 = –4
−1 −1 ⇒ d = –3 c = ±2
mBC = = ⇒p=5
5 p bc – ac = 8
m(AC) = 1 8
(b – a) =
3 + 2α c
m(BP) = –1 =
2−α ⇒ c = –2, ⇒ d = 2
⇒ α = –5 Eq. are (x + y – 2)(x – 3y + 2) = 0
⇒ –5 – 10(–5) = q ⇒ q = 45 x-3y+2=0
⇒ p + q = 50 A
-2 (1,1)
If P is circumcentre
(0,0) x y
+ =1
 α + 1 −2 − 2α   α + 1 
Mid point of AB =  , =  , − (α + 1)  (-5,-1) 2 2
 2 2   2 
1 3 − ( −α − 1)
m(PM) =
= 
2  α +1
2− 
 2  For A ⇒x + y = 2 ⇒ x = 2 – y
1 2(4 + α )
⇒ = ⇒2 – y – 3y + 2 = 0
2 (3 − α )
−13 4y = 4 ⇒ y = 1
⇒ 3 – α = 16 + 4α⇒α =
5 ⇒x = 2 – 1 = 1

1 + β β −5 1
Mid point of AC =  , Slope of L1 [(–5, –1) to (0, 0)] =
 5
 2 2 
M a them a ti cs | 8.67

x y −2
Slope of + =1⇒ = –1
2 2 2 1 3 1
1
Range (–1, 1/5) Area = 4 12 1
2
1 18
(∴ Third line is go through (0, 0) and for triangle parallel − 1
5 5
x y
to + = 1 meet at infinity)
2 2  
 1 = 1 1  12 − 18  − 3  4 + 1  + 1  18.4 + 1.12  
 −1,  = (a, b) 2  5   5   5 5 
 5
1  42 3.21 72 + 12 
 1  =  − + 
⇒  a + 2  = –1 + 52 = –1 + 15 = 24 2 5 5 5 
 b 
1  84 + 42 − 63  63
=   = 10
Sol 14: 2x + 3xy – 2y – 10x + 15y – 28 = 0
2 2 2 5 

y = 3x ⇒ a1 = 9 + 81 = 3 10
(-1/5,18/5)
36 9 3
C B ⇒ a2 = + = 45 = 45
(4,12) 25 25 5
A
(1,3) 2 2
 21   18  441 + (42)2 21
⇒ a3 =   +  12 −  = = 5
 5   5  25 5
2x2+9x2–2(9x2)–10x+15(3x)–28 = 0
Incentre will be
–7x2 + 35x – 28 = 0
x2 – 5x + 4 = 0 ⇒ x = 1, 4 1 × 21 5 4 × 3 5 1
+ − × 3 10
5 5 5 ,
∴ y = 3, 12
15 10 3 5 21 5
+ +
∂p 5 5 5
= 4x + 3y – 10 = 0
dx
∂p 3 × 21 5 12 × 3 5 18
= –4y + 3x + 15 = 0 + + 10
dy 5 5 8
18 24 5 + 15 10
25y = 90 ⇒ y = 5
5
54  11 5 − 10 33 5 + 18 10 
10 − =  , 
x= 5 = −1
 8 5 + 5 10 8 5 + 5 10 
4 5
3 Radius = distance of incentre from any of the sides.
−1
⇒ m1 = 3, m2 = 5 = , 4 − 3x 18 10 + 3 10 21
−6 2 = =
5 10 10(8 5 + 5 10 ) 8 5 + 5 10
18
12 −
5 42 21(8 5 − 5 10 ) 3
⇒ m3 = = =2 = = (8 5 − 5 10 )
1 21 70 10
4+
5
3−2 1 Sol 15: y = mx + c
⇒ tanq1 = =
1+6 7  y − mx   y − mx 
3x2–y2– 2x   +4y   =0
1  c   c 
3+
⇒ tanq2 = 2 = –7
2xy 2mx2 4y 2 4mxy
3 3x2–y2 – + + − =0
1− c c c c
2
1 50
⇒ cotq1 + cotq2 + cotq3 = 17 + +0=
7 7
8 . 6 8 | Straight Line

 4m 2  Sol 17: x2 + 4xy + 4y2 – 5x – 10y + 4 = 0


x2  3 + 2m  + y2  4 − 1  + xy  − −  =0
 c  c   c c ⇒ y2 – 4y + 3 = 0
a
Now = –1 ⇒ y2 – 3y – y + 3 = 0 ⇒ y = 1, 3
b
For y = 1
3c + 2m
⇒ = –1 ⇒ x2 + 4x + 4 – 5x – 10 + 4 = 0
4−c
x2 – x – 2 = 0 ⇒ x = –1, 2
⇒ 3c + 2m = c – 4
For y = 3
⇒ 2c + 2m = –4
⇒ x2 + 12x + 36 – 5x – 30 + 4 = 0
⇒ c + m = –2
⇒ x2 + 7x + 10
Point is (1, –2)
⇒ x = –5, –2
for 3x2 + 3y2 + 2x + 4y = 0
equation will be

 3m  24   −4m 2  2
x2  3 −  + y  − 1  + xy  +  =0
-1
 c   c   c c

3c − 2m The points are (–1, 1) (2, 1)


= – 1 ⇒3c – 2m = c – 4
4−c
(–5, 3) (–2, 2)
c – m = –2 ⇒ point is (–1, –2)
−1 1
y
Sol 16: y = m(x – 1) – +x=1 2 1
m 1 1
−2 3 = [–1+6–5–6–(2–2–15–3)]
2 2 2
 y  y  y −5 3
⇒ x2+y2+6x  x −  –10  x −  + 1 x −  = 0
 m  m  m −1 1

6xy 10y 2 1
⇒ x2 + y2 + 6x2 – – 10xy + + x2 = [–6 – (–18)] = 6 units
m m 2
Diagonals are = 49 + 4 , −1 + 4 = 53 , 5
y2 2xy
+ – =0
m2 m
Sol 18: 6x2 – xy – y2 + x + 12y – 35 = 0
 10 1   6 2
⇒ 8x2 + y2 1 + + + xy  − − 10 −  = 0 (y – m1x – c1)(y – m2x – c2) = 0
m 2 m m
 m   
y2 – m2xy – c2y – m1xy + m1m2x2
a 8
⇒ = = –1 + m1xc2 – c1y + c1m2x + c1c2
b 10 1
1+ +
m m2 m1m2x2 + y2 + xy(–m2 – m1) + x(m2c1 + m1c2)

1 10 + y(–c1 – c2) + c1c2 = 0


⇒ –8 = + +1
m2 m m1m2 = –6; m1 + m2 = –1
⇒ –9m2 = 1 + 10m This gives m1 = 2 and m2 = –3, –3, 2
⇒ 9m2 + 10m + 1 = 0 ⇒ c1c2 = 35
⇒ (9m2 + 9m + m + 1) = 0 ⇒ c1 + c2 = 12
⇒ 9m(m + 1) + 1(m + 1) = 0 ⇒ m2c1 + m1c2 = –1
−1 ⇒ –3c2 + 2c1 = –1
⇒ m = –1,
9 ⇒ c2 = 5, c1 = 7
⇒ y = 1 – x, 9y = 1 – x
M a them a ti cs | 8.69

y = –3x + c1 …. (i) i. e. α = 2, β = 3


y = 2x + c2 …. (ii) OA + OB + OC
y = –3x + 7 …. (iii) 8 1 + m2 m2 + 1
= + β + α 1 + m2
y = 2x + 5 …. (iv) 1+m m2
(, )
8 1 + m2 3
(1) = + 1 + m2 + 2 1 + m2
1+m m
(0,0)
(3) = 4 2 +3 2 +2 2 = 9 2
(-, - )
Equation is y = mx ⇒ y = x
(4) (2)

⇒ 3α + β + 7 = 0 Sol 20: Area(ABC) = 3 Arc (AMB)


3α + β = c1 In ∆ACB A
⇒ c1 = –7 AF=BF=CF=
AB
= 30
2
y = 2a + 5
CM:MF=2:1 E F
–y = –2a + c2 M
⇒ MF =
10
c2 = –5
By Apollonius theorem C D B
Combined equation will be
6x2–xy – y2 + x(–1) + y(–12) – 35 = 0
2 2
AM + BM = 2 CF + AF ( 2 2
)
AM2 + BM2 =
2000

Sol 19: Let ∠AMB =


θ

, MBE − MAD 2 −1 1


tan
= θ = =
B (/m,) 1 + MBEMAD 1 + 2 3
(,m)
A 33 11
⇒ cosθ
⇒Cos
Cosθθ and
and sinθ
Sin
Sin
θθ==
=
10
10 10
10
(0,0) (y,my) In ∆AMB
y = mx AM2 + BM2 − AB2
cosθθ =
Cos
8 2AM.BM
y=
1+m 3 2000 − 3600
=
OA. OB. OC = 48 10 2AM.BM

β2 1600
y 2 + m2 y 2 β2 + α2 + α2m2 = 48 ⇒ AM.BM =
− 10
6
m2
1
8
αβ
(1 + m2 )3/2
= 48
=
Area of ∆AMB
2
( AM)(BM) Sin θ
(1 + m) (m)
1 1600 1 400
= × 10 × =
⇒ (1 + m2)3/2 = 2 m(1 + m) 2 6 10 3
⇒ (1 + m2)3 = 2m2(1+m2+2m) Area of ABC 3 × Area of ∆AMB
⇒ m + 1 + 3m + 3m = (m + m + 2m )
6 2 4 4 2 3 2
400

= 400 Sq. units
=
m + m + m + 1 = 4m
6 4 2 3
3
⇒m=1
β 3
f(α, β) = − + (3x − 2y)2 + e(x − 2) + 2(y − 3) = 0
α 2
8 . 7 0 | Straight Line

Sol 21: c2 = 2; c=± 2


(0,24)
30 5y + x = ± 5 2
24
A(0,3)
(0,0) 18 (18,0)
x+y=3 7x-y+3=0
18 24
Centroid = , = (6, 8)
3 3 B (,3-) C(,7+3)
D(1,-10)
 18 24 
Circumcentre =  ,  = (9, 12) mBD = mBC
 2 3 
Incentre 13 − β 7α + 13
=
 24.18 18.24   24.18  β −1 α −1
=  , =  ,6  = (6, 6)
 30 + 24 + 18 30 + 24 + 18   72  13α–13–aβ+β = 7aβ–7α+13β–13
6 6 1 8aβ = 20α – 12β
1
Area = 6 8 1
2 β2 + β2 = α2 + (7α )2
9 12 1
1 1 2b2 = 50a2
= [6(–4)–6(6–9)+1(72–72)] = [–24 + 18] = 3 sq. units
2 2 β = ±5α
Case I: 40a2 = 20α – 60α
Sol 22:
40a2 = –40α
(1,3)
(,2+3) α = –1; β = –5
y + 10 13 + 5
(5,1) = = –3
(,2+c) x −1 −6
y + 3x = –7
Case II: β = –5α
 2β + c − 1   2β + c + 3 
    = –1 –40a2 = 80α
 β −5   β −1 
α = –2; β = +10
b2 – 6β + 5= –(2β + c – 1)(2β + c – 3)
y + 10 3 1
= =
α+β 2α + 2β + 2c x −1 9 3
= 3; =2
2 2 3y + 30 = x – 1
α + β = 6; α+β+c=2 x – 3y = 31
c = –4
b2 – 6β + 5 = –(2β – 5)(2β – 7) = –(4b2 – 24β + 35) Sol 24: Let slope of BC is m, then equations is
y + 10 = m(x -1)
5b2 – 30β + 40 = 0
∠ACB =
∠ABC
β = 2, 4 A(0,3)
tan ∠ACB =tan ∠ABC
Coordinates are (2, 0) and (4, 4) 7x-y+3=0 x+y-3=0
7 −m m − (1)
=
Sol 23: 5x – y = 1 1 + 7m 1 + m ( −1) C
 
B(1,-10)
−1
y= x+c 7 −m m +1
5 ⇒ =
1 + 7m 1 − m
A(0, 0) ; B(0, c) ; C(5c, 0)
1 ⇒ 7 − 7m − m + m2 =1 + m + 7m2 + 7m
× 5c2 = 5
2
M a them a ti cs | 8.71

Sol 26:
⇒ 6m2 + 16m − 6 =0
⇒ 3m2 + 8m − 3 =0
P
⇒ 3m2 + 8m − 3 =0
⇒ 3m2 + 9m − m − 3 =0
⇒ (3m− 1)(m+ 3) =
0 Q

⇒m= −3, 1/ 3 R
y=-x

Equations
P(3, 2), Q(3, –2), R(2, –3), S = (–2, 3)
3x + y=
+ 7 0 or x − 3y −
= 31 0
3 2
Sol 25:
3 −2
1
Area of PQRS will be, A = 2 −3
A(1,-2) 2
−2 3
3 2
m=-1 1 1
D E = [–6–9+6–4–(6–4+6+9)] = [–13 – 17] = 15 units
m=2 2 2

(,-1-)B C ( -10 5
,
3 3 ) Sol 27:
(,2-4)
x+2y=0 x-y+5=0
l3
l4
y+2 (0,1)
Eq. of AB = = –1
x −1 A
(1,0)
y+2=1–x (-1,0)
(0,0)
B
C
x+y+1=0 D l1
y+2 l2
Eq. of AC = =2
x −1
y – 1 = mx → 1
y + 2 = 2x – 2 ⇒ y + 4 = 2x
y = m(x + 1) →  2
 α + 1 −α − 3   β + 1 2β − 6 
D→  ,  ; E →  ,  −1x
 α α   2 2  y= → 3
m
α +1  α +3 −1(x − 1)
+  +5 = 0 y= → 4
2  2  m
−x
α + 7 = 0 ⇒ α = –7 1 intersection  3 → mx + 1=
m
−m 1
β +1  2β − 6  x= ,y=
= −2   1+m 2
1 + m2
2  2 
11 −(x − 1)
β + 1 = –4β + 12; β= 1 intersection  4 → mx +1 =
5 m
 11 2  −x 1
(–7, 6)  ,  mx + 1 = +
m m
 5 5
y −6 28 −14  m2 + 1  1−m
= =  x =
x+7 −46 23  m  m
23y + 14x = 138 – 98 1−m
x=
14x + 23y = 40 1 + m2
8 . 7 2 | Straight Line

m − m2 m+1 5
⇒ x1 + x2 + x3 = … (iii)
y= +1 =
1 + m2 m2 + 1 8
⇒ y1 + y 2 + y 3 = … (iv)
 1−m m+1 
B 2 ,  From (i), (ii), (iii) and (iv), we get
 m + 1 m2 + 1 
−x ⇒ 4 ( x 2 + x3 ) − 3 ( y 2 + y 3 ) + 6 =0
2 intersect 3 → m(x + 1) =
m ⇒ 4 (5 − x1 ) − 3 ( 8 − y1 ) + 6 =0
x
mx + = –m
m ⇒ 20 − 4x1 − 24 + 3y1 + 6 =0
2
−m
x= ⇒ −4x1 + 3y1 + 2 =0 … (v)
1 + m2
 −m2 Now AH ⊥ BC
m m 
y= C  , 
1 + m2 2 2 y1 − 2 4
 1 + m 1 + m  × =−1
x1 − 1 3
−1
2 intercept  4 ⇒ m(x + 1) = (x − 1)
m ⇒ 4y1 − 8y =
−3x1 + 3
x 1
⇒ mx + = –m ⇒ 3x1 − 4y1 =
11
m m … (vi)
(1 − m2 ) From (v) and (vi), we get
x=
2
1+m  41 38 
(x
( x1,1, yy11)) =  , 
 2   25 25 
y = m
2 
1 +m  2 2
 41   38 
 (1 − m2 ) 2m  Radius = AO= 2 −  + 3 − 
 25   25 
⇒ D , 
2 2
 1 + m m + 1 
2 58 m
 m−1  = =
Area of square =   =
(m − 1)2 ( 25) n
 1 + m2  m2 + 1
AB = BD
⇒ m 58, =
= n 25 ⇒ m + n =83
m2 + m4 = 2m(1 + m2) Sol 29: (–6, 1) (6, 10), (9, 6), (–3, –3)
m2 = 2m
(6, 10)
m=2 B

1 p
Area = =
5 q A
C
(9, 6)
(-6, 1)
p+q=6 E
(1,0)
2 1
Sol 28: P(2,3)
D
H(1,2) C (-3,-3)

5 3 Area of rectangle – Area of ∆ =a / b


A
Centroid (c) =  , 
3 3 Area of rect = 81 + 144 16 + 9 = 15. 5 = 75
Let coordinates of vertices of y +3 3
∆ABC be A (x, y), B(x2, y2), C (x3, y3) Eq. of CD =
B C x+3 4
4x-3y+3=0
4x2 - 3y2 + 3=0 … (i) Point E = (1, 0)
4x3 - 3y3 + 3=0 … (ii) y +3 −4
Eq. of AD = =
x1 + x2 + x3 5 x+3 3
and =
3 3 −9 −21
− 3 = x; x=
4 4
M a them a ti cs | 8.73

α – 2 = 2α + 3 ⇒ α = –5
1 1  25 
[x1 − x2 ] + 3 = Area of ∆ =   × 3 = 75
2 2 4  8 and –8β + 8α = 5β + 10α + 10

75 × 7 525 a −13β − 10
Area = = = α= ,β=0
8 8 b 2
β(–5, +10)C, (0, 2)
a + b = 533
−8
m(BC) =
5
Sol 30:
y − 2 −8
A(1,2) =
x 5
y intercept = 2
L1=0 L2=0
(iii) L1 = x + y – 1 = 0

(,2)B C(,3-) L2 = 2x – y + 4 = 0
L=0
A
(i) L1 = 2x – y = 0
L2 = x + y = 3 E F
S
G = (2, 3), A(1, 2) (2,1)
⇒α+β+1=6 B C
(,1 - ) D
(,2+4)
⇒ 2 + 2α + 3 – β = 9
⇒ 2α – β = 4 ⇒ 3α = 9 S(2, 1)
α = 3, β = 2 A(–1, 2)
B(3, 6)C(2, 1)  α + β 2β − α + 5 
D , 
y −1  2 2 
Eq. ⇒ =5  α −1 3 − α  β −1 
x−2 E=  ,  ; F =  ,β + 3
 2 2   2 
y + 5 = 5x
m=5 Now, m(SE) = 1
(ii) If H = (2, 3) α −1
−2
L1 = 2x + y = 0 / L2 = x – y + 2 = 0 ⇒ 2 =1
3−α
−1
−2 4  −2 4  2
x= y= A  , 
3 3  3 3
⇒ α – 5 = 1 – α; α = 3
B(α, –2α), C (β, β + 2) −1
m(SF) =
4 2
3−
3 5
Slope of AH = = β −1
2 8 −2
β −5
2+
3 ⇒ 2 = = –2
β + 3 −1 2(β + 2)
−8 B + 2α + 2
Slope of BC = = –4(β + 2) = β – 5
5 β−α
3 + 2α −3
Slope of BH = 5β = –3β =
2−α 5
2  −3 14 
β+ β = (3, –2), C  , 
α−2 3 =1
Slope of AC = =  5 5 
3 + 2α 2
β+ y+2 −24 y+2 −4
3 = ⇒ =
x −3 18 x −3 3
8 . 7 4 | Straight Line

3 3 2 1
⇒ x intercept 3 – = sin θ = , cos θ =
2 2 5 5
C = (6, 4)
Exercise 2 D = 0 + 2 5 cosθ, 2 + 2 5 sinθ = (2, 6)
Eq. of AC
Single Correct Choice Type
y −2 1
= ⇒ x = 3y – 6 ….(i)
Sol 1: (A) a(3x + 4y + 6) + b(x + y + 2) = 0 x−0 3
(2, 3) is situated at greater distance y −0
Eq. of BD = = –3
x−4
a(6 + 12 + 6) + b(2 + 3 + 2) y
D= x=4– ….(ii)
2 2
(3a + b) + (4a + b) 3
Solving eq. (i) & (ii) we get the required Point is (3, 3)
24a + 7b 24T + 7
D= =
(3a + b)2 + (4a + b)2 (3T + 1)2 + (4T + 1)2 Sol 3: (C)
A
 a
Where  T = 
 b 
px+qy=1 qx+py=1
dD
=0
dT
B C
⇒ (3T + 1)2 + (4T + 1)2 × 24 (p,q)

(24T + 7)2[(3T + 1)3 + 4(4T + 1)] px + qy = 1 × q


=
2 (3T + 1)2 + (4T + 1)2 qx + py = 1 × p

⇒ 24[(3T+1)2 + (4T+1)2 pqx + q2y = q

= (24T + 7)(9T + 16T + 7) pqr + p2y = p

⇒ 24[25T2 + 2 + 14T] = [24T + 7][25T + 7] ----------------

⇒ 48 + 24×14T = 24T×7+25×7T+7×7 y(q2 – p2) = (q – p)


1
⇒ 24×7T = 49 – 48 + 25×7T y=
p+q
−1 a
⇒ –7T = 1 ⇒ T = = 1
7 b x=
p+q
⇒ –(3x + 4y + 6) + 7(x + y + 2) = 0
 1 1 
A=  , 
4x + 3y + 8 = 0 p + q p + q
Median through AB
Sol 2: (C)
1
q−
y −q p+q
2
5 ⇒ =
x −p 1
p−
(0,2) A C p+q
2 5
5 2 y −q  pq + q2 − 1 
⇒ =  
B x −p 2
 p + pq − 1 
(4,0)
(p2 + pq – 1)y – q(p2 + pq – 1)
C = 4 + 2 5 cosθ, 2 5 sinθ = x(q2 + qp – 1) – p(q2 + pq – 1)
tan θ = 2 x(pq+q2–1) + p = q + y(p2 + pq – 1)
M a them a ti cs | 8.75

= (2pq – 1)(px + qy – 1) 9 7
⇒ 4β – 8 = 3α – ⇒ 3α – 4β + =0
= (p2 + q2 – 1) (qx + py – 1) 2 2

 27 36  25β − 36 4
Sol 4: (B) 3x + 4y = 9 × 4 Mid point of AC=  ,  = =
 25 25  25α − 27 3
⇒ 12x + 16y = 36 (Slope of ⊥ to AC)
4x – 3y + 12 = 0 × 3 4α
⇒ 100α – 108 = 75β – 108 ⇒β =
⇒12x – 9y = –36 3
3
25y = 72 α= β=2
2
72
y= 9 5
25 Circumradius = +4 =
4 2
4 × 72
9−
25 4 × 24 21
⇒x = = 3− = − a 1 1
3 25 25
Sol 5: (D) 1 b 1 = 0
A(3,0) 1 1 c

a(bc – 1) – 1(c – 1) + 1(1 – b) = 0


(,) abc – a – b – c + 2 = 0
(0,4)B [ 21 72
C - 25 , 25 ] a + b + c = abc + 2
1 1 1
+ +
 21 72  1−a 1−b 1−c
A(3, 0) B(0, 4]C  − 25 , 25 
  1 + bc − b − c + 1 + ab − a − b + 1 + ac − a − c
=
(1 − a)(1 − b)(1 − c)
3 0 1
1 3 + ab + bc + ca − 2a − 2b − 2c
DPAB = 0 4 1 =
2 1 − a − b − c + ab + bc + ca − abc
21 72
− 0
25 25 3 − 2(abc + 2) + ab + bc + ca
=
1 − abc − (abc + 2) + ab + bc + ca
1  72   84  
= 3 4 −  + 10 +  −1 − 2abc + ab + bc + ca
2  25   25   = =1
−1 − 2abc + ab + bc + ca
1  3 × 28 84  84
=  +  = 25
2  25 25 
Sol 6: (D) A(a, 0) ; B(0, b) ; C (c, 0) ; D(0, d)
72
−72 −3 d c
25 = lines are not parallel i. e. not a trapezium, not
mAC = = = a b
21 96 4
− −3
25 a parallelogram

72 OA OC = OB OD (For concyclic points)


4−
mBC = 25 = 24 = 8
Assuming origin as centre
21 21 7
25 ac = bd i. e. ABCD are concyclic.
4
mAB = −
3 Sol 7: (D) 7x2 + 8y2 – 4xy + 2x – 4y – 8 = 0
3  β−2 3 Homogenising
Mid point of AB  ,2 ⇒ = (Slope of ⊥ AB)
2  3 4  3x − y 
α−
2 7x2 + 8y2 – 4xy + 2x  
 2 
8 . 7 6 | Straight Line

 3x − y   3x − y 
2 12x2 + y2(3k2 – 2k – 6) + xy(6k + 4k + 10) = 0
– 4  y – 8  =0
 2   2  −2h −(10k + 10)
m 1 + m2 = = =0 ⇒ k = –1
7x2 + 8y2 – 4xy + 3x2 – xy – 6xy b 3k 2 − 2k − 6

+ 2y2 – 2(9x2 + y2 – 6xy) = 0 Sol 10: (D) R = (h, k)


⇒ –8x2 + 8y2 – 11xy + 12xy = 0
h+6 k −6
⇒ 8x2 – 8y2 – xy = 0 Centroid is  , 
 3 3 
2 h2 − ab
⇒ tan θ = =∞ h+6 k −6
a+b 9  + 7  +4=0
 3   3 
1
⇒ h = − a = 8b = –8
2 9h + 7k + 54 – 42 + 12 = 0
π 9h + 7k + 24 = 0
⇒θ=
2
parallel to N.
Sol 8: (B) x2 – 4xy + 4y2 + x – 2y – 6 = 0
Sol 11: (C) A(1, 2) & B(7, 5)
y = mx + C1y = mx + C2
3 1
(y – mx – a)(y – mx – C2) = 0 AB line ⇒m = =
6 2
y2 – mxy – C2y – mxy + m2x2
2 1
+ C2mx – C2y + C1mx + C1C2 = 0
A 2 C 1B
m x + y – 2mxy + x (C1m + C2m)
2 2 2
A C (7,5)B
(1,2)
+ y(–C1 – C2) + C1C2 = 0 (1,2) (7,5)

1 2 1  14 + 1 10 + 2 
=4⇒ − = –4 ⇒ m = + C=  ,  ⇒ (5,4)
m 2  3 3 
m2
2 2m − 1
(C1 + C2) + =1 = ±1
a m+2
1 2m – 1 = –2 – m
(C1 – C2) =
2 1 1
m= − or m = 3(m = − is not possible)
6 3 3 3
C 1C 2 = − = −
4 2 ⇒ y = 3x – 11 ⇒ 3x – y – 11 = 0
C1 − C2
Distance between lines =
1 + m2 Sol 12: (A) ax2 + 2hxy + by2 = 0
1
C 1 + C2 =
2 m2
−6
C 1C 2 =
4 m
1
+6
C1 − C2 4 5
= = = 5
1 + m2 1 5 m1
1+ 2
4 4
m − m1 m2 − m
=
1 + mm1 1 + m2m
Sol 9: (B) 5x2 + 12xy – 6y2 + 4x(x + ky) [Homogenising]
⇒ m + m 2m 2 – m 1 – m 1m 2m
– ay(x + ky) + 3(x + ky)2 = 0
= m2 – m + mm1m2 – m2m1
9x2 – 6y2 – 2ky2 + 10xy + 4kxy + 3(x2 + k2y2 + 2kxy) = 0
⇒ 2m – (m1 + m2) + (m1 + m2)m2 = 2m1m2m
M a them a ti cs | 8.77

2h 2h 2 2a mAB. mBC = –1
⇒ 2m + – m = m
b b b 9 − 2x
mAB =
⇒ 2mb – 2am = –2h + 2hm 2 x−4

hm2 + m(a – b) – h = 0 13 − 2x
mBC =
x−2
Sol 13: (D) (9 – 2x)(13 – x) = –1(x2 – 6x + 8)
3x-y=-3 117 – 18x – 26x + 4x2 = –x2 + 6x – 8
5x2 – 50x + 125 = 0
(0,3)
2x-3y = 6 x2 – 10x + 25 = 0 ⇒ x = 5; y = 0
1 1
Area = × BC × AB = 9+9 1+1 = 3
(-1,0)
2 2
(3,0)

(0,-2)
3x+4y = 12 Multiple Correct Choice Type

Sol 16: (B, D) Let the co-ordinate of the vertex C is (h,


From figure h – 2) then area of triangle will be
α ∈ [–1, 3] −5 0 1
1
β ∈ [–2, 3] ⇒ 3 0 1 =
20
2
h h− 2 1
Sol 14: (C)
⇒ (h – 2)8 = ± 40
A
⇒ h – 2 = ±5

2x+3y=29 x+2y=16 ⇒ h = 7 or –3
∴ Co-ordinate are (7, 5) or (–3, –5)

(,)B C(10-,12-)
(5, 6)
Sol 17: (B, C) y + 3x = 2
2α + 3β = 29
y– 3x = 2
10 – α + 24 – 2β = 16
x = 0, y = 2
α + 2β = 18  …(i)
2α + 4β = 36  …(ii) -1/3 A
m
4/3
On solving, we get 1/3 60
β = 7, α = 4 2/3
So we have, B(4, 7) A
y −7
Eq. of BC = = –1 ⇒ x + y = 11 2/3
x−4

Sol 15: (B) 1 1


−m m+
3 3 1 − 3m 3m + 1
A(4,1) = ⇒ =
m m m+ 3 3 −m
1+ 1−
3 3

B
C(2,-3) ⇒ 3 – m –3m+m2 3 =m2/ 3 +m+3m+ 3
(x,10-2x)
2x+y=0
8 . 7 8 | Straight Line

⇒m=0
B(0, b)
Bisector line is y = 2
(, )
P 2
⇒ cos 60º = ; P= A’(-a’, 0) A(0, a)
4/ 3 3

 2  B’(0, b’)
⇒ Given point is  ,2 
 3  −b
m(AB) =
a
Or the other possibility is m = ∞
b
⇒ Foot of perpendicular = (0, 0) m(BA′) =
a′
Let orthocentre be at (α, β)
Sol 18: (A, B, D)
A(1,3) b−β
=∞⇒α=0
0−α

β−0 a
P(x,y) =
α + a′ b
(5,6)B C(-1,2)
aa′
β= = b′ [From (i)]
y −6 3 b
AB - =
x −5 4
Sol 20: (A, B, C, D)
4y – 24 = 3x – 15 ⇒ 3x – 4y + 9 = 0
y −6 2 C(0,a)
BC - =
x −5 3 B(x,x/3)
60
3y – 18 = 2x – 10 ⇒ 2x – 3y + 8 = 0 30
y −3 (0,0) -30
1
AC - = x2 + x2/3 = a2
x −1 2
2y – 6 = x – 1 ⇒ x – 2y + 5 = 0 (0,-a) x=3a/2

C and P on same side C either lies on y axis so C(0,a) or C(0,–a)


3(–1) – 4(2) + 5 < 0 3x – 4y + 9 < 0 1
or on y = −
B and P on same side of AC 3x
5 – 2(6) + 5 < 0 x – 2y + 5 < 0  −1 
C  x, 
 3x 
A and P on same side of BC
2(1) – 3(3) + 8 > 0 2x – 3y + 8 > 0 x2
⇒x2 + = a2
3
So we can see [A,B,D] are correct.
±a 3
x=
x y 2
Sol 19: (B, C) + = 1A (a, 0) & B(0, b)
a b a
y= 
x y 2
line + = 1 C(–a′, 0) D(0, –b′)
a′ b′  3a −a   − 3a a 
⇒ (0, a), (0, –a),  ,   , 
AB A′B′ are concyclic  2 2   2 2
   
i. e. OAOA′ = OBOB′
aa′ = bb′
aa′
b′ =
b
M a them a ti cs | 8.79

Sol 21: (B, C) Vertices are (4, 3)(1, –1) & (7, k) Sol 23: (B, C) (x + 2) (y + 2) = 0; x + y + 2 = 0

A(4,3) Circumcentre lies on mid-point of hypotenuse i. e. AB


( −2,0) + (0, −2)
= (–1, –1)
2

x=-2 y
(1,-1)B C(7,k)

 2+k  A
C =  4,  x
 3 
That occurs only in isosceles ∆ B C y=-2
(1 – 4)2 + (3 + 1)2 = (k – 3)2 + 9
y=-x-2
AC = AB
Sol 24: (B, C) x + y = 1; x = 7y; x = 3y
16 = (k – 3)2
Centroid and In centre always Lie inside of the triangle.
⇒ k – 3 = ±4
⇒ k = 7, –1 or (7/8, 1/8)
25 = (k + 1) + 36
2

≠ not possible (AB = BC) (3/4, 1/4)


or (k – 3) + 9 = (k + 1) + 36
2 2

(0,0)
(BC = AC)
Sol 25: (B, C)
18 – 6k = 37 + 2k
y+2x=5
−19
⇒ 8k = –19 ⇒ k = m
8
For k = 7AB = 5
AC = 5, BC = 10
[∆ is not possible] m − ( −2)
tan θ =
1 + m( −2)
−19
So k = –1 or m+2 1
8 =±
1 − 2m 2
Sol 22: (A, C) (7, 4) 2m + 4 = ±(1 – 2m)
y – 4 = m(x – 7) ⇒ 2m + 4 = 2m – 1 ⇒(m = ∞)
Centre ≡ (3, –2) ⇒ x = 2 (B)
(3 − 7)m + 4 + 2 2m + 4 = 1 – 2m
⇒ =4
1 + m2 3
4m = –3 ⇒ m = −
⇒ (–4m + 6) = 16(1 + m )
2 2
4
⇒ 9 + 4m2 – 12m = 4 + 4m2 y −3 −3
=
x−2 4
5
⇒ m= or m = ∞
12 3x + 4y = 18 (C)
5
⇒ y–4= (x – 7)
12
⇒5x – 12y + 13 = 0 and x = 7
8 . 8 0 | Straight Line

Sol 26: (A, C)


2 2
=
   5
y
y=-2x
(1,0) y=x
45
/4
/6 x’ x
O
L1 /2 L2

/2
y – a = m(x – 1) L3 P R Q
y=-2
(-2,-2) (1,-2)
y = m(x – 1)
y’
− sin75º
m = – tan 75º =
cos75º
Sol 2: (A) Solving equations L1 and L2.
−( 3 + 1)
= = – (2 + 3) x y 1
( 3 − 1) ⇒ = =
–36 + 10 –25 + 12 2 – 15
y = – (2 + 3) (x – 1) ∴ x = 2, y = 1
(2 + 3) x + y = (2 + 3) (C) L1, L2, L3 are concurrent if point (2, 1) lies on L2

x + (2 − 3) y = 1 ∴ 6 – k – 1= 0 ⇒ k = 5
(A) → (S)
Sol 27: (B, C) y – y1 = m(x – x1) (B) Either L1 is parallel to L2 or L3 is parallel to L2, then
y = y1 + m(x – x1) 1 3 3 –k
= or =
(B) Set of parallel lines 3 –k 5 2
–6
(C) All these lines pass through x = x1 ⇒ k = – 9 or k =
5
(B) → (p, q)

(C) L1, L2, L3 form a triangle, if they are not concurrent,
or not parallel.
6 5
∴ k ≠ 5, – 9, – ⇒k=
5 6
(c) → (r)
(D) L1, L2, L3 do not form a triangle. If
6
Previous Years’ Questions k = 5, – 9, –
5
(D) → (p, q, s)
Sol 1: (C) It is not necessary that the bisector of an
angle will divide the triangle into two similar triangle, Sol 3: (A, C) Given lines px + qy + r = 0, qx + ry + p = 0
therefore, statement-II is false.
Now we verify statement-I and rx + py + q = 0 are concurrent.

∆ OPQ, OR is the internal bisector of ∠POQ p q r


∴ q r p = 0
PR OP
∴ = r p q
RQ OQ

PR 22 + 22 Applying R1 → R1 + R2 + R3 and taking common from R1


⇒ =
RQ 12 + 22 1 1 1
⇒ (p + q + r) q r p
r p q
M a them a ti cs | 8.81

⇒ (p + q + r) (p2 + q2 + r2 – pq – qr – pr) = 0 Given plane is


⇒ p3 + q3 + r3 – 3pqr = 0. x – 2y +z = d  …….(ii)
Equations (i) and (ii) are parallel.
Sol 4: (A, C) Since, 3x + 2y ≥ 0 ….(i)
Clearly, A = 1
Where (1, 3) (5, 0) and (– 1, 2) satisfy Equation (i)
Now, distance between plane
∴ Option (a) is true.

Again 2x + y – 13 ≥ 0 d
= = 6
is not satisfied by (1, 3), 1+ 4 +1
⇒ |d| = 6
∴ Option (b) is false.
2x – 3y – 12 ≤ 0, Sol 7: x2 + y2 ≤ 6 and 2x – 3y = 1 is shown as
is satisfied for all points,
∴ Option (c) is true.
L
And – 2x + y ≥ 0,
is not satisfied by (5, 0), 1/2

∴ Option (d) is false, 1/3

Sol 5: (B, C) Let equation of line L1 be y =mx. Intercepts


For the point to lie in the shaded part, origin and the
made by L1 and L2 on the circle will be equal ie, L1 and L2
point lie on opposite side of straight line L.
are at the same distance from the centre of the circle.
∴ For any point in shaded part L > 0 and for any point
Centre of the given circle is (1/2, – 3/2). Therefore, inside the circle S < 0.
m 3  3
+ Now, for  2,  , L : 2x – 3y – 1
|1 / 2–3/ 2–1|
= 2 2  4
1+1 m2 + 1 9 3
L : 4 – – 1= >0
4 4
2 |m+3|
⇒ = 9
2 2 and S : x2 + y 2 – 6, S : 4 + –6<0
2 m +1 16
⇒ 8m + 8 = m2 + 6m + 9
2  3
⇒  2,  lies in shaded part.
⇒ 7m2 – 6m – 1 = 0  4
5 3
⇒ (7m + 1) (m – 1) = 0 For  ,  L : 5 – 9 – 1 < 0 (neglect)
2 4
–1
m
⇒= = , m 1 1 1 1 3
7 For  , –  L : + – 1 > 0
Thus, two chords are x + 7y = 0 and y – x = 0 Therefore, 4 4 2 4
(b) and (c) are correct answers. 1 1
∴  –  lies in the shaded part.
4 4
Sol 6: Equation of plane containing the given lines is 1 1  1 3
For  ,  L : – – 1 < 0 (neglect)
x –1 y –2 z –3 8 4 4 4
2 3 4 =0
⇒ Only 2 points lie in the shaded part.
3 4 5

⇒ (x – 1)(– 1) – (y – 2)(– 2) + (z – 3)(– 1) = 0 Sol 8: Note: d : (P, Q) = | x1 – x2 | + | y1 – y2 |. It is new


method representing distance between two points
⇒ – x +1 + 2y – 4 – z + 3 = 0
P and Q and in future very important in coordinate
⇒ x + 2y – z = 0  ……(i) geometry.
8 . 8 2 | Straight Line

Now, let P(x, y) be any point in the first quadrant. We  1 m 


have P≡ , 
m+1 m+1
d(P, 0) = | x – 0 | + | y – 0 | = | x | + | y | = x + y ( x,
y > given)  3 3m 
Q≡ , 
 m + 1 m +1
d (P, A) = |X – 3| + |Y – 2| (given)
y
d(P, 0) = D(P, A)
⇒ x + y = |x – 3| + |y – 2| L1
Case I : When 0 < x < 3, 0 < y < 2 R Q
In this case I Eq.(i) becomes L2 L

x + y= 3 – x + 2 – y P
x
O
5
⇒ 2x + 2y = 5 ⇒x+y=
2 x+y=1 x+y=3

Case II : When 0 < x < 3, y ≥ 2


m–2
Now, Eq. (i) becomes Now, equation of L1 : y – 2x =  …. (i)
m+1
x + y = 3 – x + y – 2 3m + 9
And equation of L2 : y + 3x =  ….(ii)
1 m+1
⇒ 2x = 1 ⇒x=
2 By eliminating ‘m’ from Equ. (i) and (ii), we get locus of
R as x – 3y + 5 = 0, which represents a straight line.
Case III : When x ≥ 3, 0 < y < 2

Infinite segment
Sol 10: Let the square S is to be bounded by the lines x
y 1 1
x=1/2 =± and y = ±
2 2
(1/2,2)

Finite segment

y=2
2 2 2
x+y=5/2,

 1 1 
We have,
= a2  x1 –  +  – y1 
 2 2 
1
y

x A(x1,1/2) 1/2
O 1/2 (5/2, 0) 3
D(-1/2,y2) d
a
1/2
Now, Eq.(i) becomes x + y = x – 3 + 2 – y x’ O
x
-1/2 B(1/2, y1)
c
1
⇒ 2y = – 1 or y= – b
2 C(x2-1/2) -1/2
Hence, no solution. y’
Case IV : When x ≥ 3, y ≥ 2
1
= x12 – y12 – x1 – y1 +
In this case (i) changes to x + y = x – 3 + y – 2 2
⇒0=–5 1
Similarly,
= b2 x22 – y12 – x2 + y1 +
2
Which is not possible.
1
Hence, this solution set is {(x, y)} | x = 12, y ≥ 2} ∪ {(x, y)} | c2 = x22 – y 22 + x2 + y 2 +
2
x + y = 5/2, 0 < x < 3, 0 < y > 2 } 1
d2 = x12 – y 22 + x1 + y 2 +
The graph is given in adjoining figure. 2

∴ a2 + b2 + c2 + d2= 2(x12 + y12 + x22 + y 22 ) + 2


Sol 9: Let the equation of straight line L be
1
y = mx Therefore 0 ≤ x12 , x22 , y12 , y 22 ≤
4
M a them a ti cs | 8.83

0 ≤ x12 + x22 + y12 + y 22 ≤ 1 Sol 11: Let the vertices of a triangle be, O(0, 0) A(a, 0)
and B(b, c) equation of altitude BD is x =b.
⇒ 0 ≤ 2(x12 + x22 + y12 + y 22 ) ≤ 2
c
Slope of OB is .
But 2 ≤ 2(x12 + x22 + y12 + y 22 ) + 2 ≤4 b
b
Alternate Solution Slope of AF is – .
c
2
c= x22 + y 22  ….(i) Now, the equation of altitude AF is
y
y
B(b,c)
A(x1,1)
a
d B(1,y1) F 90
90 E
P
(0,y2)D
b x’ x
c O D(b,0) A(a,0)
x’ x y’
O C(x2,0)
b
y – 0 = – (x – a)
y’ c

=b2 (1 – x2 )2 + y12  …. (ii) Suppose, BD and OE intersect at P.

=a2 (1 – y1 )2 + (1 – x1 )2  …..(iii)   a – b 
Coordinates of P are b,b  
 c 
2
d= x12 + (1 – y 2 ) 2
.....(iv) 
On adding Eqs. (i), (ii), (iii) and (iv), we get a–b
Let m1 be the slope of OP =
2 2
a +b +c +d = 2 2
{x12 + (1 – x1 ) } 2
+ {y12 2
+ (1 – y1 ) } c

+ {x22 + (1 – x2 )2 } + {y 22 + (1 – y 2 )2 } c
and m2 be the slope of AB =
b–a
Where x1, y1, x2, y2 all vary in the interval [0, 1].
 a – b  c 
Now, consider the function y = x2 + (1 – x)2, 0 ≤ x ≤ 1 Now, m1m2 =
=   –1
 c  b – a 
dy
Differentiating ⇒ ⇒ 2x – 2(1 – x) . For maximum
= We get that the line through O and P is perpendicular
dx
dy to AB.
or minimum =0.
dx
⇒ 2x – 2(1 – x ) = 0 ⇒ 2x – 2 + 2x = 0 Sol 12: Since, line L make 60° with line 3 x + y =
1, then
1
⇒ 4x = 2 ⇒x=
2
d2 y
Again, =2+2 = 4
dx2
⇒ Which is positive.
1
Hence, y is minimum at x = and its minimum value
1 2
is . − 3 + tan 60°
= m1 = 0
4
Clearly, value is maximum when x = 1.
1 − − 3 ( tan 60° ) ( )
− 3 − tan 60° −2 3 −2 3
1 1 1 1 m2 = = = = 3
∴ Minimum value of a2 + b2 + c2 + d2 = + + + =2
2 2 2 2 ( )
1 + − 3 tan 60° 1−3 −2

And maximum value is 1 + 1 + 1 + 1 = 4


Equation of line having slope 3 and passes through
(3, -2)
8 . 8 4 | Straight Line

y +=
2 3 ( x − 3) α −β α+β
⇒2≤ + ≤4
⇒ y + 2= 3x − 3 3 2 2
⇒ y − 3x + 2 + 3 3 =0

Sol 13: (A) The point of intersection of lines ax + by + c


= 0 and bx + ay + c = 0 is
 −c −c 
P , 
a+b a+b
Given that distance of point P from (1, 1) is less than
2 2
2 2
 c   c 
1 +  + 1 +  <2 2
 a+b  a+b
Since, Point P lies in the first quadrant α ; β > 0
a+b+c
⇒ 2 <2 2 Case 1: α≥β
a+b
α −β α+β
a+b+c a+b+c 2≤ + ≤4
⇒ < 2 ⇒ −2 < <2 2 2
a+b a+b

⇒ a + b + c < 2a + 2b ⇒ a + b − c > 0 ⇒ 2≤α≤2 2

Case 2: α<β
Sol 14: Let P be ( α , β ) , then
−α + β α+β
α −β 2≤ + ≤4⇒ 2 ≤β≤2 2
d1 (P ) = 2 2
2
( ) − ( 2) = 8 – 2
2 2
α+β Area = 2 2
d2 (P ) =
2 = 6 sq. unit

⇒ 2 ≤ d1 (P ) + d2 (P ) ≤ 4
2017-18 100 &
op kers
Class 11 T
By E ran culty
-JE Fa r
IIT enior emie .
S fP r es
o titut
Ins

MATHEMATICS
FOR JEE MAIN & ADVANCED
SECOND
EDITION

Exhaustive Theory
(Now Revised)

Formula Sheet
9000+ Problems
based on latest JEE pattern

2500 + 1000 (New) Problems


of previous 35 years of
AIEEE (JEE Main) and IIT-JEE (JEE Adv)

5000+Illustrations and Solved Examples


Detailed Solutions
of all problems available

Topic Covered Plancess Concepts


Tips & Tricks, Facts, Notes, Misconceptions,
Circle Key Take Aways, Problem Solving Tactics

PlancEssential
Questions recommended for revision
9. CIRCLE

1. INTRODUCTION
Definition: The locus of a point which moves in a plane such that its distance
from a fixed point in that plane always remains the same (i.e., constant) is P moving point
known as a circle.
The fixed point is called the centre of the circle and the distance between the .
fixed point
fixed point and moving point is called the radius of the circle.

2. DIFFERENT FORM OF EQUATION OF CIRCLE


Figure 9.1

2.1 General Form


The general equation of a circle is of the form x2 + y2 + 2gx + 2fy + c = 0 where g, f, and c are constants.
1 1
(a) Centre of the circle ≡ (–g, –f). i.e., (– coefficient of x, – coefficient of y).
2 2
(b) Radius of the circle = g2 + f 2 − c .

Nature of the circle:


(a) If g2 + f2 –c > 0, then the radius of the circle will be real. Hence, it is possible to draw a circle on a plane.
(b) If g 2+ f2 – c = 0, then the radius of the circle will be zero. Such a circle is known as point circle.
(c) If g2 + f2 – c < 0, then the radius g2 + f 2 − c of the circle will be an imaginary. Hence, it is not possible to
draw a circle.
The condition for the general second degree equation to represent a circle:
The general equation ax2 + 2hxy + by2 + 2gx +2fy + c=0 represents a circle iff
(a) a = b ≠ 0 i.e. the coefficient of x2 = the coefficient of y2 ≠ 0.
(b) h = 0 i.e. the coefficient of xy is 0.
a h g
(c) ∆ = h b f = abc + 2hgf – af2 – bg2 – ch2 ≠ 0, it implies that the general equation is non degenerate (i.e.
g f c
equation cannot be written into two linear factors)
(d) g2 + f2 – c ≥ 0
9 . 2 | Circle

PLANCESS CONCEPTS

• The general equation ax2 + 2hxy + by 2 + 2gx + 2fy + c =0 can be written in matrix form as

 a h g  x 
a h   x    
 x y      + 2gx + 2fy + c =0 and  x y 1 h b f   y  = 0
h b   y  g f c  1 
  
• Degeneracy condition depends on determinant of 3x3 matrix and the type of conic depends on
determinant of 2x2 matrix.
• Also the equation can be taken as intersection of z =ax2 + 2hxy + by 2 and the plane z =
− 2gx + 2fy + c ( )
Vaibhav Gupta (JEE 2009, AIR 54)

2.2 Standard Form


The equation of circle with center (0,0) and radius ‘a’ is x2 + y 2 =
a2 .

P(x,y)

X’ X
O (0,0)

2 2 2
X +Y =a

Y’

Figure 9.2: Standard Form

2.3 Central Form


The equation of the circle with centre (h, k) and radius ‘r’ is (x – y)2 + (y – k)2 = r2.
Y
P(x,y)
r

C(h,k)

2 2 2
(x-h) +(y-k) =r

X’ X
O

Y’
Figure 9.3: Central form
M a them a tics | 9.3

2.4 Diametric Form


Circle on a given diameter: The equation of the circle with (x1, y1) and (x2, y2) as (x,y)
the end points of the diameter is given by
(x – x1)(x – x2) + (y – y1) (y – y2) = 0
(x1,y1) (x2,y2)
 x + x 2 y1 + y 2 
Centre ≡  1 , 
 2 2 
2 2
 x2 − x1   y 2 − y1 
and, Radius =   +   . Figure 9.4: Diametric Form
 2   2 

2.5 Parametric Form


For x2 + y2 = r2, parametric co-ordinates of any point on the circle is given by (r cosθ, r sinθ), (0 ≤ θ < 2π).
(a) The parametric co-ordinates of a point on the circle (x – h)2 + (y – k)2 = r2 is given by (h + r cosθ, k + r sin θ),
(0 ≤ θ < 2π).
(b) The parametric co-ordinates of any point on the circle x2 + y2 + 2gx + 2fy + c = 0 are x = –g + r cos θ and
y = –f + r sinθ, (where r = g2 + f 2 − c , and 0 ≤ θ < 2π)

2.6 Equation of Circle under Special Conditions


(a) The equation of the circle through three points A(x1, y1), B(x2, y2), C(x3, y3) is

x2 + y 2 x y 1
x12 + y12 x1 y1 1
=0
x22 + y 22 x2 y2 1
x32 + y 32 x3 y3 1

The concept of family of circles can be used to derive this form.


Taking any two of the given three points as extremities of diameter, we get the equation of a circle S = 0 and the
equation of straight line passing through these two points L = 0. Then the equation of circle passing through the
intersection of circle and line is S + λL = 0, where λ is a parameter. The value of λ can be found by substituting the
third point in the equation as it also lies on the circle.
(b) Equation of circle circumscribing the triangle formed by the lines ar x + br y + cr =
0 where r = 1, 2, 3 is

a12 + b12
a1 b1
a1 x + b1 y + c1
a22 + b22
a2 b2 = 0
a2 x + b2 y + c2
a32 + b32
a3 b3
a3 x + b3 y + c3

PLANCESS CONCEPTS

Whenever the problem seems to be very complicated using formulas and geometrical approach, then
try to apply trigonometric approach as well. Like the given circle may be in circle or ex-circle of some
triangle. May be using properties of triangle we can solve it.
Vaibhav Gupta (JEE 2009, AIR 54)
9 . 4 | Circle

Illustration 1: Find the equation of the circle which passes through the point of intersection of the lines x – 4y – 1 = 0
and 4x + y – 21 = 0 and whose centre is (2, –3). (JEE MAIN)

Sol: By solving given equation of lines simultaneously we will get point of intersection of lines i.e. P. Therefore
using distance formula we will get radius of circle and using centre and radius we will get required equation.
Let P be the point of intersection of the lines
x – 4y – 1 = 0  …. (i)
and 4x + y – 21 = 0  …. (ii)
( ) ( )
From (i) and (ii), we get x = 5 , y = 1 . So, coordinates of P are 5,1 .Let C 2, −3 be the centre of the circle.
Since the circle passes though P, therefore

(5 − 2 ) + (1 + 3 )
2 2
CP= radius ⇒ = radius ⇒ radius = 5

Hence the equation of the required circle is (x – 2)2 + (y + 3)2 = 25.

Illustration 2: Find the equation of a circle of radius 10 whose centre lies on x-axis and passes through the point
(4, 6).  (JEE MAIN)

Sol: Consider coordinates of centre of circle as (a, 0). Now Y


by using distance formula, we can calculate the value
of ‘a’ and then by using central form, we get required
equation.
A(4,6)
Here centre C lies on x-axis, and the circle passes through
A (4,6).
10
Let C be (a,0)
X’ X
∴ CA = radius C2 O C1

(a − 4 ) + (0 − 6)
2 2
⇒ CA =10 ⇒ = 10
⇒ (a – 4)2 + 36 = 100 ⇒ (a – 4)2 = 64
⇒ a – 4 = ±8 ⇒ a = 12 or a = – 4.
Thus, the coordinates of the centre are (12, 0) or − 4,0 . ( )
Hence, the equations of the required circles are Y’
Figure 9.5
( x − 12) + ( y − 0 ) 102 and ( x + 4 ) + ( y − 0 ) =
2 2 2 2
= 102

Illustration 3: Find the equation of the circle concentric with the circle x2 + y 2 + 4x + 6y + 11 =
0 and passing
( )
through the point 5, 4 . (JEE MAIN)

Sol: Since both circle are concentric therefore their centre should be same. (5,4)
Hence equation of a required circle can be written as x2 + y 2 + 4x + 6y +
(constant term) = 0.
Let the equation of the concentric circle be
x2 + y2 + 4x + 6y + k = 0
(-2,-3)
Since the point (5,4) lies on this circle,
∴ (5)2 + (4)2 + 4(5) + 6(4) + k = 0
⇒ 25 + 16 + 20 + 24 + k = 0 ⇒ k = –85
Therefore, the equation of the required circle is
x2 + y2 + 4x + 6y – 85 = 0 Figure 9.6
M a them a tics | 9.5

Illustration 4: Find the equation of a circle passing through the origin and making intercepts 4 and 3 on the y and
x axis respectively. (JEE MAIN)

Sol: By observing the problem we conclude that given intercepts are end Y
(0,4)
points of diameter of this circle. Therefore by using diametric form we can
P
obtain the equation of circle.

Let the intercepts be OP = 4, OQ = 5


∴ The co-ordinates of P and Q are (0, 4) and (3, 0) respectively.
Since POQ = 90°, hence PQ is a diameter. X’ X
O Q(3,0)
∴ The required equation of the circle is
(x − 3)(x − 0) + (y − 0)(y − 4) =
0 ⇒ 0
x2 + y 2 − 3x − 4y = Y’
Figure 9.7

(
Illustration 5: Find the equation of the circle which is circumscribed about the triangle whose vertices −2,3 , 5,2 )( )
(
and 6, −1 . ) (JEE ADVANCED)

Sol: Consider (a, b) as the centre of circle and r as the radius. As circle passes from given vertices, therefore their
distance from the centre are same. Therefore by using distance formula, we will get the value of a, b and r.
( )( )
Since the circle passes through the points −2,3 , 5,2 and 6, −1 . ( )
∴ (–2 – a) + (3 – b) = r ⇒ a + 4a + 4 + b – 6b + 9 = r 
2 2 2 2 2 2
…. (i)
(5 – a)2 + (2 – b)2 = r2 ⇒ a2 – 10a + 25 + b2 – 4b + 4 = r2  …. (ii)
(6 – a)2 + (–1 – b)2 = r2 ⇒ a2 – 12a + 36 + b2 + 2b + 1 = r2 …. (iii)
Subtracting (ii) from (i), we have
14a – 21 – 2b + 5 = 0 i.e., 14a – 2b = 16 …. (iv)
Subtracting (iii) from (ii), we get
2a – 11 – 6b + 3 = 0 ⇒ 2a – 6b = 8  …. (v)
Solving (iv) and (v), we get
a = 1 and b = –1
Putting the values of a = 1 and b = −1 in (i), we get
1 + 4 + 4 + 1 + 6 + 9 = r2 ⇒ 25 = r2 ⇒r=5
Thus, the required equation of the circle is
(x – 1)2 + (y + 1)2 = 25 ⇒ x2 + 2x + 1 + y2 + 1 + 2y = 25 ⇒ x2 + y2 – 2x + 2y – 23 = 0
9 . 6 | Circle

3. EQUATION OF CIRCLE IN SOME SPECIAL CASES


Equation Centre/Radius Properties Figures
(a) x2 + y2 = a2 ( 0, 0 ) ; a When the centre of the circle Y
coincides with the origin
center = (0,0)

X’ a X
O

Y’

Figure 9.8
(b) ( x – h )2 + ( y ± a )2 = a2 ( h , ±a ) ; a Touches x axis only, y Y
coordinate of centre = ±a

C1 (h,a)

X’ O X
G

C2 (h,-a)

Y’

Figure 9.9
(c) ( x ± a )2 + ( y – k )2 = a2 ( ±a, k ) ; a Touches y–axis only, x Y
coordinate of centre = ±a

(-a,k) (a,k)
C1 C2

X’ X
O

Y’

Figure 9.10
M a them a tics | 9.7

Equation Centre/Radius Properties Figures


(d) (x ± a )2 + (y ± a)2 = a2 ( ±a, ±a ) ; a Touches both the axes Y
depending on the quadrant
center = ( ±a , ±a)

C2 C1
(-a,a) (a,a)

X’ X
O
(-a,-a) (a,-a)

C3 C4

Y’

Figure 9.11
(e) x2 + y2 – 2ax = 0 C (a, 0) ; a When the circle passes Y
through the origin and
centre lies on x axis

X’ X
O (a,0)

Figure 9.12
(f) x2 + y2 – 2ay = 0 C (0, a); a When the circle passes Y
through the origin and
centre lies on y axis.

(0,a)

X’ X
O

Y’

Figure 9.13
9 . 8 | Circle

Equation Centre/Radius Properties Figures


(g) x2 + y2 –αx –βy =0 α β Passes through (0, 0) and has Y’
 , ; intercepts α and β on x &
 2 2 y axes respectively.

1
α2 + β2 , 
2 2 2


X’ X
O

Y’

Figure 9.14
(h, h); h When circle touches both
( ) + ( y − h)
2 2
(h) x − h h2 or
=
the axes
x2 + y 2 − 2hx − 2hy + h2 =
0
(h,h)

Figure 9.15

Y
Illustration 6: Find the equation of the circle which passes through two points on
the x–axis which are at distances 12 from the origin and whose radius is 13.
C
Sol: There are two circles which passes through two points A and A’ on x–axis which
are at a distance 12 from the origin and whose radius is 13. The centre of these circles X’
A’ O A
X
lie on y–axis (perpendicular bisector of chord AA’)
In ∆AOC, AC2 = OA2 + OC2
⇒ 132 = 122 + OC2 ⇒ OC = 5
So the coordinates of the centre of the required circles are (0, 5) and C’ (0, –5). Hence Y’
the equations of the required circles are Figure 9.16
(x – 0 ) + (y ± 5 ) = 13 ⇒ x + y ± 10y – 144 = 0.
2 2 2 2 2

4. INTERCEPTS ON THE AXES Y

The lengths of intercepts made by the circle x2 + y2 +2gx + 2fy + c = 0 on X and Y axes
are 2 g2 − c and 2 f 2 − c respectively.
Therefore,
(a) The circle x2 + y2 +2gx + 2fy + c = 0 cuts the x–axis in real and distinct points, X’
O
X
touches or does not meet in real points according as g2 >c- Distinct points
Y’
g2 = c – Touches
Figure 9.17: Intercept made
g2 < c – Does not meet by circle on x-axis

(b) Similarly, the circle x2 + y2 +2gx + 2fy + c = 0 cuts the y–axis in real and distinct points, touches or does not
meet in real points according as f2 >, = or < c .
M a them a tics | 9.9

(-g,-f)

X’ X
O

Y’

Figure 9.18: Intercept made by circle on y-axis

Illustration 7: Find the equation to the circle which touches the positive axis of y at a distance 4 from the origin
and cuts off an intercept of 6 from the axis of x. (JEE MAIN)
Sol: As circle touches Y axis therefore Y coordinate of centre of circle is 4 so by Y
using formula of intercept we will get the value of X coordinate of centre of circle
and c.
Consider a circle x2 + y2 + 2gx + 2fy + c = 0. (5,4)
This meets the axis of y in points given by y2 + 2fy + c = 0 5
4
3
The roots of this equation must be each equal to 4, so that it must be equivalent X’
O
X
to (y – 4)2 = 0 ⇒ 2f = –8 & c = 16
∵ Intercept made on the x-axis = 6
Y’
Figure 9.19
⇒ 6 = 2 g2 − 16 ⇒ g = ±5.
Hence, the required equation is x2 + y2 ± 10x – 8y + 16 = 0.

5. POSITION OF A POINT W.R.T A CIRCLE


(a) If CP < radius, then the point P lies inside the circle. (Refer Fig. 9.20 (i))

r
C

(i)
Figure 9.20 (i)

(b) If CP = radius, then the point P lies on the circumference.

P
r
C

(ii)
Figure 9.20 (ii)
9 .10 | Circle

(c) CP > radius, then the point P lies outside the circle.
P

r
C

Figure 9.20 (iii)


(iii)

Hence, any point (x, y) lies outside, on or inside if

(x1 + g)2 + (y1 + f)2 > = < g2 + f 2 − c ⇒ (x1 + g2) + (y1 + f )2 >

= < (g2 + f2 – c )
⇒ x12 + y12 + 2gx1 + 2fy1 + c > = < 0
Or, S1 > = < 0 where S1 = x12 + y12 + 2gx1 + 2fy1 + c
Therefore, a point (x1, y1) lies outside, on or inside a circle
S ≡ x2 + y2 + 2gx + 2fy + c = 0 according as S1 ≡ x12 + y12 + 2gx1 + 2fy1+ c is positive, zero or negative.

5.1 Power of a Point w.r.t. a circle


Let P ( x1 , y1 ) be a point and a secant (a line which cuts the curve in two point) PAB is drawn.
T
B
A

P (x1,y1)

C
D
Figure 9.21

The power of P(x1, y1) w.r.t. S = x2 + y2 + 2gx + 2fy + c = 0 is equal to PA.PB which is S1, where S1 = x12 + y12 + 2gx1 + 2fy1 + c .
Power remains constant for the circle i.e. independent of A and B
PA×PB = PC×PD = PT2 = square of the length of a tangent

5.2 The Least and Greatest distance of a Point from a Circle


Let S = 0 be a circle and P ( x1 , y1 ) be a point. If the diameter of the circle through P
P
intersect the circle at Q and R, Q
then QP = PC – r= least distance; and C
PR = PC + r = greatest distance R

where ‘r’ is the radius and C is the centre of the circle.


(Refer Fig. 9.23)
Figure 9.22
M a them a ti cs | 9.11

Illustration 8: The coordinates of the point on the circle x2 + y2 – 2x – 4y – 11 = 0 farthest from the origin are
 (JEE MAIN)
 8 4   4 8   8 4 
(A)  2 + ,1+  (B)  1 + , 2+  (C)  1 + , 2+  (D) None of these
 5 5  5 5  5 5

Sol: (B) The required point lies on the normal to circle through the origin, i.e. on the line 2x = y. Therefore by
substituting y = 2x in above equation of circle we will get coordinates of required point.
4  4 
x2 + 4x2 – 2x – 8x – 11 = 0 ⇒ 5x2 – 10x – 11 = 0 ⇒ x = 1 ± and y = 2  1 ± 
5  5
 4 8 
and the required point farthest from the origin is  1 + , 2+ .
 5 5

Illustration 9: The point (1, 3) is inside the circle S whose equation is of the form x2 + y2 – 6x – 10y + k = 0, k
being an arbitrary constant. Find the possible values of k if the circle S neither touches the axes nor cuts them.
 (JEE ADVANCED)

Sol: As (1, 3) lies inside the circle S therefore S1 < 0 and it does not touches x and y axes. On the basis of this we
can solve the problem and will get range of k.
12 + 32 – 6×1 –10×3 + k < 0; ∴ k < 26  …(i)
Solving y = 0 and x + y – 6x – 10y + k = 0, we get x – 6x + k = 0
2 2 2

Since the circle S does not intersect with the x-axis,


⇒ discriminant < 0 i.e., 36 – 4k < 0 ⇒ k > 9  …(ii)
Solving x = 0 and x2 + y2 – 6x – 10y + k = 0, we get y2 – 10y + k = 0
Since the circle S does not intersect with the y-axis,
⇒ discriminant < 0 i.e., 100 – 4k < 0 ⇒ k > 25  …(iii)
From (i), (ii) and (iii), we get 25 < k < 26, i.e., k ∈ (25, 26).

6. LINE AND A CIRCLE


a2 (1 + m2 ) − c2
The length of the intercept cut off from the line y = mx + c by the circle x2 + y2 = a2 is 2 .
1 + m2
(a) If a2 (1 + m2 ) > c2, or, | c | < a 1 + m2

i.e., the line will intersects the circle at two real and different points.

(b) If a2 (1 + m2 ) = c2, | c | a 1 + m2
or, =

i.e., the line will touch the circle at only one point i.e. the line will be a tangent. (2,4)

(c) If a2 (1 + m2 ) < c2, or, | c | > a 1 + m2

i.e., the line will meet the circle at two imaginary points.

Figure 9.23

Illustration 10: Show that the line 3x – 4y – c = 0 will meet the circle having centre at (2, 4) and the radius 5 in real
and distinct points if – 40 < c < 20.  (JEE MAIN)

Sol: Since the line cuts the circle so length of perpendicular from centre of circle upon line is less than the radius
of circle.
9 .12 | Circle

3× 2 − 4 × 4 − c
<6 ⇒ 10 + c < 30
9 + 16
⇒ –30 < 10 +c < 30 ⇒ –40 < c < 20

Illustration 11: If 4l2 – 5m2 + 6l +1 = 0 then show that the line lx + my +1 = 0 touches a fixed circle. Find the radius
and centre of the circle. (JEE ADVANCED)

Sol: If line touches the circle then perpendicular distance from centre of circle to the line is equal to the radius of
circle so by using distance formula of point to line we will get one equation and other is given 4l2 – 5m2 + 6l +1 =
0 . hence by solving these two equation we will get required answer.
Let the circle be (x – α)2 + (y – β)2 = a2
The line lx + my + 1 = 0 touches the circle if
l α + mβ + 1
a= or a2(l 2 + m2) = (lα + mβ +1)2
2 2
l +m
or (a2 – α2) l2 + (a2 – β2) m2 –2 lβα m – 2 l α – 2mβ – 1 = 0 ... (i)
But 4l – 5 m + 6l + 1 = 0
2 2
... (ii)
It is possible to find α,β, a if (i) and (ii) are identical.
The condition is
a2 − α2 a2 − β2 −2α 2β −1
= = = =  ... (iii)
4 −5 6 0 1
a2 − α2
∴ β = 0, α =3 and = –1 which implies a2 – 32 = –4, i.e., a = 5 .Also α = 3, β = 0, a = 5 satisfies equation
4
( )
2
(iii) and hence the line touches the fixed circle (x – 3 )2 +(y – 0)2 = 5

or x2 + y2 – 6x + 4 = 0, whose centre = (α, β) = (3, 0) and radius = a = 5.

Illustration 12: Find equation of a line with slope gradient 1 and such that x2 + y2 = 4 and x2 + y2 – 10x – 14y + 65 = 0
intercept equal length on it ? (JEE ADVANCED)

Sol: As given slope of line is 1, therefore its equation will be y = x + c Hence by using perpendicular distance
formula we will get distance of line from centre of respective circle and then
by using Pythagoras we can obtain length of intercepts made by line to these
A’
circles and which are equal. Therefore we can obtain value of c and required
equation of circle.
C(5,7)
Let 2 be the length of the intercept made by the two circle.
For x2 + y2 = 4, centre ≡ (0,0) and radius = 2, and A
For x2 + y2 – 10x – 14y + 65 = 0, centre ≡ (5,7) and
O(0,0)
radius = 3.
c 5−7+c c−2
∴ OA = and CA’ = ⇒CA’ = Figure: 9.24
2 2 2

Now, from the diagram we get

4 – OA2 = 2  …(i)
M a them a ti cs | 9.13

and 9 – CA’2 = 2  …(ii)


2 2
c c−2
⇒4– =9–   [from (i) and (ii)]
2  2 
3
⇒c=–
2
The equation of line is 2x - 2y = 3

Illustration 13: Find the values of α for which the point (2α, α +1) is an interior point of the larger segment of the
circle x2 + y2 – 2x –2y – 8 = 0 made by the chord whose equation is x – y + 1 = 0. (JEE ADVANCED)

Sol: As point (2α, α +1) lies inside the circle S, therefore S1 < 0. Hence by substituting
the point in the equation, we will get the range of α and as it lies in larger segment =0
+1
made by line x – y + 1 = 0. The centre of circle i.e. (1, 1) and (2α, α +1) will have the x-y
same sign.
∴ (2α)2 + (α +1)2 – 2×2α – 2(α + 1) – 8 < 0
Centre
⇒ 5a2 – 4α – 9 < 0 or (5α – 9)(α + 1) < 0
(2,+1)
⇒ –1 < α <9/5  ... (i)
Also, as the point lies in the larger segment, the centre (1, 1) and the point (2α, α + 1) Figure 9.25
must be on the same side of the line x – y + 1 = 0.
Clearly, 1 – 1 + 1 > 0; So, 2α – (α + 1) + 1 > 0;
∴ α > 0 … (ii)
 9
∴ The set of values of α satisfying (i) and (ii) is  0,  .
 5

7. TANGENTS

7.1. Point Form


(a) The equation of tangent at (x1 ,y1) to circle x2 + y2 = a2 is xx1 + yy1 - a2 = 0.

2 2 2
x +y=a
P(x1,y1)

Figure 9.26

(b) The equation of tangent at (x1, y1) to circle x2 + y2 + 2gx + 2fy + c = 0 is xx1 + yy1 + g(x + x1) + f(y + y1) + c = 0.

7.2. Parametric Form


Since parametric co-ordinates of a point on the circle x2 + y2 = a2 is (a cos θ, a sin θ), then equation of tangent at (a
cos θ, a sin θ) is x a cos θ + y a sin θ =a2 or, x cos θ + y sin θ = a.
9 .14 | Circle

7.3. Condition for Tangency


A line L = 0 touches the circle S = 0. If length of perpendicular (p) drawn from the centre of the circle to the line
is equal to radius of the circle i.e. p = r. This is the condition of tangency for the line L = 0. Circle x2 + y2 = a2 will
touches the line y = mx + c if c = ±a 1 + m2

Illustration 14: For what value of c will the line y = 2x + c be a tangent to the circle x2 + y2 = 5.  (JEE MAIN)

Sol: The equation of the tangent to the circle x2 + y2 = a2 in slope form is y = mx + a 1 + m2 .


On comparison, we get a = 5 and m = 2.

∴c=± 5 × 1 + 22 ⇒ c = ± 5

The required equation is =


y 2x ± 5

7.4. Slope Form


The straight line y = mx + c touches the circle x2 + y2 = a2 if c2 = a2 (1 + m2). Therefore, the equation of the tangent
 ma ±a 
in the slope form is y = mx ± a 1 + m2 and the point of contact is  ,  .
 2 
 1+m 1 + m2 
7.5 Length of Tangent

The length of the tangent drawn from a point P(x1, y1 ) to the circle S =x2 + y2 + 2gx +2fy + c = 0 is PT1 = PT2 =

x12 + y12 + 2gx1 + 2fy1 + c


∴ Length of Tangent = S1

Note:
(i) P
 T2 is called the power of the point with respect to a given circle, where PT is the tangent from a point P to a
given circle.
(ii) Area of quadrilateral PT1C T2 = 2 × (Area of triangle PT1C), and

 r 
 
(iii) The angle between tangents PT1 and PT2 is equal to 2tan–1 
P(x1,y1)
S .
 1

Illustration 15: If OA and OB are tangents from the origin O, to the circle x2 + y2 +2gx +2fy
 
+ c= 0, c > 0 and C is the centre of the circle, then area of the quadrilateral OACB is 
 (JEE MAIN)

(A)
1
2
( )
c g2 + f 2 − c (B) ( )
c g2 + f 2 − c

g2 + f 2 − c
(C) c g2 + f 2 − c (D)
c
T1
Sol: (B) As we know quadrilateral OACB is formed by two right angle triangle OAC and r
C
T3
triangle OBC. Line OA and OB are tangent to the circle from common point O. Therefore
OA = OB and (AC = CB) radius of circle, hence both triangle are equal. Therefore Area of
the quadrilateral OACB = 2 Area of the triangle OAC. 2 2
x +y -2gx+2fc c=0
OA = OB = S1 = c (Length of the tangent from the origin) Figure 9.27
M a them a ti cs | 9.15

and, CA = CB = g2 + f 2 − c (Radius of the circle)

∴ Area of the quadrilateral OACB = 2 Area of the triangle OAC C


1 B
= 2 ×   OA × CA = c g2 + f 2 − c A
2

Illustration 16: The locus of a point which moves such that the tangents from it to the
two circles x2 + y2 – 5x – 3 = 0 and 3x2 + 3y2 + 2x + 4y – 6 =0 are equal, is (JEE MAIN)
(A) 7x + 4y – 3 = 0 (B) 17x + 4y + 3 = 0 (C) 3x – 4y + 9 = 0 (D) 13x – 4y + 15 =0

Sol: (B) Use the formula for length of tangent. Let P(h, k) be any point on the locus.

The length of the tangent from P to the first circle is h2 + k 2 − 5h − 3 ,  …(i)

2 4 6
Similarly, the length of the tangent to the other circle is h2 + k 2 + h + k − . …(ii) O (0,0)
3 3 3
On equating (i) and (ii), we get 17h + 4k + 3 = 0, Figure 9.28

Therefore, the required locus is 17x + 4y + 3 = 0.

7.6 Pair of Tangents


From a given point P(x1, y1) two tangents PA and PB can be drawn to the circle S = x2 + y2 +2gx +2fy + c = 0.
The combined equation of the pair of tangents is A
SS1 = T2, where
S = 0 is the equation of circle,
T = 0 is the equation of tangent at (x1, y1), and
S1 = x12 + y12 +2gx1 +2fy1 + c
(S1 is obtained by replacing x by x1 and y by y1 in S) P(x1,y1) B

Pair of tangents from point (0, 0) to the circle are at right angles if g2 + Figure 9.29
f2 = 2c.

Illustration 17: Find the equation of the pair of the tangents drawn to the circle x2 + y2 – 2x + 4y = 0 from the
point (0, 1). (JEE MAIN)

Sol: Here (x1, y1) = (0, 1). So by using formula SS1 = T2 we can get required equation, where S = x2 + y 2 − 2x + 4y =
0,
S1 ≡ x12 + y12 +2gx1 +2fy1 + c and T = xx1 + yy1 – (x+x1) +2(y+x1).
0
Given circle is S ≡ x2 + y 2 − 2x + 4y = …(i)
Let P be the point (0, 1).
∴ S1 ≡ x12 + y12 +2gx1 +2fy1 + c ⇒ S1 ≡ 02 + 12 – 2.0 + 4.1 = 5
And, T ≡ xx1 + yy1 – (x+x1) +2(y+x1) ⇒ T ≡ x(0) + y(1) – (x + 0) + 2(y + 1)
i.e., T ≡ – x + 3 y + 2.
Hence, the equation of pair of tangents from P (0, 1) to the given circle is SS1 = T2

( )
i.e. 5 x2 + y 2 − 2x + 4y = ( −x + 3y + 2 )
2

⇒ 5x2 + 5y2 – 10x + 20y = x2 + 9y2 + 4 – 6xy – 4x + 12y


9 .16 | Circle

⇒ 4x2 – 4y2 – 6x + 8y + 6xy – 4 = 0


⇒ 2x2 – 2y2 + 3xy – 3x + 4y – 2 = 0  …(ii)
Note: From (ii), we have 2x2 + 3(y – 1) x – (2y2 – 4y + 2) = 0.
This is a quadratic equation in x, hence by using quadratic formula we get

3(y − 1) ± 9(y − 1)2 + 8(2y 2 − 4y + 2)


x= or, 4x – 3y + 3 = ± 25y 2 − 50y + 25
4
or, 4x – 3y + 3 = ±5(y – 1).
∴ Separate equations of tangents are x – 2y + 2 = 0 and 2x + y - 1 = 0.

Illustration 18: From a point on the line 4x – 3y = 6 tangents are drawn to the circle x2 + y2 – 6x –4y + 4 = 0 which
24
make an angle of tan–1 between them. Find the coordinates of all such points and the equations of tangents.
 7 (JEE ADVANCED)

Sol: Consider a point P on the line 4x – 3y = 6 and use the formula.


A
Let P (x1 , y1) be a point on the line 4x – 3y = 6.
24
If θ is the angle between the tangents, then tan θ = . P(x1,y1) 
7
C
For the given circle, Centre C = (3, 2) and
Radius = CA = 32 + 22 − 4 = 3 for tan θ
B
∴ The length of tangent, PA = S1 = x12 + y12 − 6x1 − 4y1 + 4
Figure 9.30
θ AC 3
∴ tan = =
2 PA S1
θ 9
1 − tan2 2 2 1−
S1 − 9 7
⇒ tan2
θ
=
9
or, 2 = x1 + y1 − 6x1 − 4y1 + 4 − 9 or −S1
= = ⇒ S1 = 16
2 S1 2 θ x 2
+ y 2
− 6x − 4y + 4 + 9 9 S1 + 9 25
1 + tan 1 1 1 1 1+
2 S1

7 x12 + y12 − 6x1 − 4y1 − 5  24 


∴ = 2  tan θ = 
25 x1 + y12 − 6x1 − 4y1 + 13  7 

or, x12 + y12 – 6x1 – 4y1 – 12 = 0  … (i)


As (x1, y1) is on the line 4x – 3y = 6, we get 4x1 – 3y1 = 6 … (ii)
Solving (i) and (ii), we get
2
 4x − 6   4x − 6 
x12
+  1  – 6x1 – 4  1  –12 = 0
 3   3 
⇒ 9x12 + (4x1 – 6)2 – 54x1 – 12(4x1– 6) – 108 = 0
⇒ 25x12 – 150x1 = 0 ⇒ x1 (x1 – 6) = 0
4x1 − 6 6 18
⇒ x1 = 0, 6 and, y1 = =– , = –2, 6.
3 3 3
∴(x1, y1) ≡ (0, –2) and (6, 6).
The equation of the pair of tangents is given by SS1 =T2
where S ≡ x2 + y2 – 6x – 4y + 4,
S1 = x12 + y12 – 6x1 – 4y1 + 4, and
M a them a ti cs | 9.17

T = xx1 + yy1 – 3(x + x1) – 2(y + y1) + 4


∴ The pair of tangents from (0, –2) is
(x2 + y2 – 6x – 4y + 4)⋅(0 + 4 – 0 + 8 + 4) = (0 + y(–2)– 3(x) –2(y – 2) + 4)2
⇒ 16(x2 + y2 – 6x –4y + 4) = (–3x –4y + 8)2
⇒ 16(x2 + y2 – 6x –4y + 4) = 9x2 + 16y2 + 64 +24xy – 48x – 64y
⇒ 7x2 – 24xy –48x = 0 ⇒ x(7x – 24y – 48) = 0
∴ The tangents from (0, –2) are x = 0, and 7x –24y –48 =0.
Similarly, the equation of the pair of tangents from (6, 6) is
(x2 + y2 – 6x – 4y + 4) ⋅ (36 + 36 – 6 ⋅ 6 – 4 ⋅ 6 + 4) = {x ⋅ 6 + y ⋅ 6 – 3(x + 6) – 2(y + 6) + 4}2
⇒ 16(x2 + y2 – 6x – 4y + 4) = (3x + 4y – 26)2 = 9x2 + 16y2 + 676 + 24xy – 156x – 208y
⇒ 7x2 – 24xy + 60x + 144y – 612 = 0
⇒ (7x – 24y + 102) (x – 6) = 0
∴ The tangents from (6, 6)are x – 6 = 0, and 7x – 24y + 102 =0.

Illustration 19: Obtain the locus of the point of intersection of the tangents to the circle x2 + y2 = a2 which include
an angle α. (JEE ADVANCED)

Sol: Consider (x1, y1) as the point of intersection of tangents to the given circle (x1,y1)

α a
and then use tan = to get the desired result.
2 S 1
O
Let (x1, y1) be the point of intersection of a pair of tangents to the given circle.
If the pair of straight lines includes an angle α, then
a
2 2
x +y =a
2 2

α a S1
⇒ tan = ⇒ tan α = Figure 9.31
2 S1 a2
1−
S1
2a x12 + y12 − a2
⇒ tan α =
y12 + x12 − 2a2

(
⇒ x12 + y12 − 2a2 )
2
( 2 2
tan2 α = 4a2 x1 + y1 − a
2
)
Hence, the required locus is (x2 + y2 – 2a2)2 tan2 α =4a2 (x2 + y2 – a2).

7.7 Director Circle


The locus of the point of intersection of two perpendicular tangents to a circle is called
the Director circle.
For the circle x2 + y2 = a2, the equation of the director circle is x2 + y2 = 2a2. P
Hence, the centre of the director circle is same as the centre of the given circle, and the
radius is 2 times the radius of the given circle.
General Form: For the general form of the circle x2 + y2 + 2gx + 2fy + c = 0, the
equation of the director circle is given by x2 + y2 + 2gx + 2fy + 2c – g2 – f2 = 0. Figure 9.32
9 .18 | Circle

Illustration 20: Find the equation of the director circle of the circle (x – 2)2 + (y + 1)2 = 2.  (JEE MAIN)

Sol: As we know, for the circle x2 + y2 = a2, the equation of the director circle is x2 + y2 = 2a2 .
For the given circle, Centre ≡(2, –1) & Radius = 2.
∴ The centre of the director circle ≡ (2, – 1), and the radius of the director circle = 2 × 2 = 2.
∴ The required equation is (x – 2)2 + (y + 1)2 = 4.

8. NORMALS
The normal of a circle at any point is a straight line, perpendicular to
the tangent and passing through the centre of the circle.
(a) Equation of normal: The equation of normal to the general l
form of the circle x2 + y2 + 2gx + 2fy + c = 0 at any point rma C
No
(x1, y1) on the circle is P
y1 + f x − x1 y − y1
(y – y1) = (x – x1) or, =
x1 + g x1 + g y1 + f
Tangent
The equation of normal to the circle x2 + y2 = a2 at any point (x1 ,y1) is
x y Figure 9.33
xy1 – xy1 = 0 or, = .
x1 y1
x y
(b) Parametric Form: Equation of normal at (a cos θ, a sin θ) to the circle x2 + y2 = a2 is =
acos θ asinθ
x y
or, = or, y = x tan θ or, y = mx (where m = tan θ).
cos θ sin θ

Illustration 21: Find the equation of the circle having the pair of lines x2 + 2xy + 3x + 6y = 0 as its normal and
having the size just sufficient to contain the circle x(x – 4) + y(y – 3) = 0. (JEE ADVANCED)

Sol: By solving equation x2 + 2xy + 3x + 6y = 0 we will get point of intersection of normals i.e. centre of required
circle. As given circle x(x – 4) + y(y – 3) = 0 lies inside the required circle hence distance between centres will be
equal to the difference between their radius, therefore we can find out radius of required circle by using distance
formula.
Given the equation of pair of normal is x(x + 3) + 2y(x + 3) = 0
⇒ (x + 3)(x + 2y) = 0
∴ Either (x+3)=0 …(i) or (x+2y)=0  …(ii)
3
On solving (i) and (ii), we get x = −3 and y =
2 (-3, 3/2) (2, 3/2)
 3
∴ The centre ≡  −3,  (The point of intersection of the normals).
 2
For the circle x2 + y2 – 4x – 3y = 0 …(iii)
2
 3  −3  5
( −2)
2 Figure 9.34
centre =  2,  and radius, r = +   − 0 = .
 2  2  2
If the circle x2 + y2 – 4x – 3y = 0 lies inside another circle of radius ‘a’, then
 3  3
a – r = distance between the centres  −3,  and  2, 
 2   2
2
5 5 15
( −3 − 2) +  32 − 32 
2  
⇒a– = ⇒a=5+ ∴a= .
2   2 2
M a them a ti cs | 9.19

2 2
 3  15 
Hence, the equation of the circle is (x + 3) +  y −  =   or, x2 + y2 + 6x – 3y = 45.
2
 2  2 

9. CHORD OF CONTACT (x’,y’)Q

Consider a point P(x1, y1) lying outside the circle. Tangents are drawn to touch the
given circle at Q and R respectively (as shown in the diagram). The chord joining P
Chord of
the points of contact of the two tangents to a circle (or any conic) from the point (x1, y1)
contact
P, outside it, is known as the chord of contact.
(x”, y”)R
9.1 Equation of Chord of contact Figure 9.35

The equation of the chord of contact of tangents drawn from a point (x1 , y1) to
the circle x2 + y2 = a2 is xx1 + yy1 = a2. Equation of chord of contact at (x1, y1) to the circle x2 + y2 + 2gx + 2fy + c = 0
is xx1 + yy1 + g(x + x1) + f(y + y1) + c =0 .
Clearly, the equation of the chord of contact coincides with the equation of the tangent.

Length of chord of contact


Consider a circle of radius ‘r’ and the length of perpendicular from the centre to the chord of contact be ‘p’, then
the length of the chord, QR = 2 r 2 − p2 .

PLANCESS CONCEPTS

( )
3/2
1 a x12 + y12 − a2 RL3
• Area of ∆PQR = × PM × QR = =
2 x12 + y12 R 2 + L2

where, the length of the tangent, L = x12 + y12 − a2 and,
radius of circle, R = a.

 quation of circle circumscribing the triangle PQR is


• E
(x – x1) (x + g) + (y – y1) (y + f) = 0.
Note: Circumscribing Circle also passes through centre Figure 9.36
of original Circle
Vaibhav Krishnan (JEE 2009, AIR 22)

Illustration 22: Find the equation of the chord of contact of the tangents drawn from (1, 2) to the circle x2 + y2 – 2x
+ 4y + 7 = 0. (JEE MAIN)

Sol: Equation of chord of contact is T = 0


Given circle is S ≡ x2 + y2 – 2x + 4y + 7 = 0 …(i)
For point P ≡ (1, 2),
S1 > 0, ⇒ the point P lies outside the circle. and, T ≡ x(1) + y(2) – (x + 1) + 2(y + 2) + 7 i.e. T ≡ 4y + 10
∴ The equation of the chord of contact is T = 0 i.e. 2y + 5 = 0.
9 .20 | Circle

Illustration 23: The locus of the point of intersection of the tangents at the extremities of a chord of the circle x2
+ y2 = a2 which touches the circle x2 + y2 – 2ax = 0 passes through the point (JEE ADVANCED)
a   a
(A)  , 0  (B)  0,  (C) (0, a) (D) (a, 0)
2   2

Sol: (A) and (C) Apply the condition of tangency to the equation of chord of contact.
Let P (h, k) be the point of intersection of the tangents at the extremities of the chord AB of the circle x2 + y2 = a2.
∴ The equation of the chord of contact AB w.r.t. the point P is hx + ky = a2.
h(a) + k(0) − a2
The line hx + ky = a2 touches the circle x2 + y2 – 2ax = 0 if =a
h2 + k 2
⇒ (h – a)2 = h2 + k2
Therefore, the locus of (h, k) is (x – a)2 = x2 + y2 or, y2 = a(a – 2x).
Clearly, points (A) and (C) satisfy the above equation.

9.2 Chord Bisected at a given Point B


P(x1,y1)
The equation of the chord of the circle A

S ≡ x + y + 2gx + 2fy + c = 0 bisected at the


2 2
C
point (x1, y1) is given by T = S1.
i.e., xx1 + yy1 + g ( x + x1 ) + f ( y + y1 ) + c =x12 + y12 + 2gx1 + 2fy1 + c .

Figure 9.37: Chord bisected by point P

PLANCESS CONCEPTS

The smallest chord of a circle passing through a point 'M' at a maximum distance from the centre is the
one whose middle point is M.
Shrikant Nagori (JEE 2009, AIR 30)

Illustration 24: Find the equation of the chord of the circle x2 + y2 + 6x + 8y – 11 = 0, whose middle point is (1, –1).
 (JEE MAIN)
Sol: Use T = S1
Given, S ≡ x2 + y2 + 6x + 8y – 11 = 0
For point L (1, 1), S1 = 12 + (–1)2 + 6.1 + 8(–1) – 11 = –11 and
T = x.1 + y.(–1) + 3(x + 1) + 4(y – 1) – 11 i.e. T = 4x + 3y – 12
Now equation of the chord having middle point, L(1, –1) is
∴ 4x + 3y – 12 = –11 ⇒ 4x + 3y – 1 = 0
Second method:
Let C be the centre of the given circle, C ≡ (–3, –4)
−4 + 1 3
∴ Slope of CL = =
−3 − 1 4
∴ Equation of chord whose middle point is L, is
M a them a ti cs | 9.21

4
∴ y + 1 = – (x – 1) [ chord is perpendicular to CL]
3
Or, 4x + 3y – 1 = 0

Illustration 25: Find the locus of the middle points of chords of given circle x2 + y2 = a2 which subtends a right
angle at the fixed point (p, q). (JEE ADVANCED)

Sol: As M(h, k)be the midpoint of the chord AB which subtends an angle of 900 at the point N (p , q) therefore a
circle can be drawn with AB as the diameter and passing through the point N, hence AM = MN.
∴ AM = MN ⇒ AM2 = MN2 ⇒a2 – (h2 + k2) = (h - p)2 + (k - q)2 A
⇒ a2 – h2 – k 2 =
h2 + p2 – 2hp + k 2 + q2 − 2kq
⇒ 2h2 + 2k 2 – 2ph − 2qk + p2 + q2 − a2 =
0 N(p,q) M(h,k)
1 2
⇒ h2 + k 2 – ph − qk +
2
(
p + q2 − a2 = )0

1
( )
B
Hence, the required locus is x2 + y 2 – px − qy + p2 + q2 − a2 =
0. Figure 9.38
2

10. COMMON CHORD OF TWO CIRCLES


Definition: The chord joining the points of intersection of two given circles is called their common chord.

Equation of common chord: The equation of the common chord of two circles
S1 ≡ x2 + y2 + 2g1x + 2f1y + c1 = 0 …. (i)
and S2 ≡ x2 + y2 + 2g2x + 2f2y + c2 = 0 …. (ii)
is given by S1 – S2 = 0 i.e., 2x(g1 – g2) + 2y (f1 – f2) + c1 – c2 = 0.

C1 M C2(2,-4)
=

Figure 9.39: Common Chord

Length of the common chord: AB = 2(AM) = 2 C1 A2 − C1M2

Where, C1A = radius of the circle S1 = 0, and


C1M = length of the perpendicular from the centre C1 to the common chord AB.

Note: If the two circles touch each other, then the length of common chord is zero and the common chord is the
common tangent to the two circles at the point of contact.
9 .22 | Circle

PLANCESS CONCEPTS

The length of the common chord AB is maximum when it is the diameter of the smallest circle.
Nitish Jhawar (JEE 2009, AIR 7)

Illustration 26: Find the equation and the length of the common chord of two circles.
2x2 + 2y2 + 7x – 5y + 2 = 0 and x2 + y2 – 4x + 8y – 18 = 0 (JEE MAIN)

Sol: Use the formula for equation of common chord and length of common chord. Equation of common chord of
circle is S1 – S2 = 0 i.e., 2x(g1 – g2) + 2y (f1 – f2) + c1 – c2 = 0 and length of common chord is 2 C2 A2 − C2M2
.
7 5
Given S1 = x + y + x – y + 1 =0 
2 2
…. (i)
2 2
S2 = x2 + y2 – 4x + 8y – 18 = 0 …. (ii)
Therefore, the equation of the common chord AB is S1 – S2 = 0
15 21
i.e. x– y + 19 = 0 ⇒ 15x – 21y + 38 = 0 .… (iii)
2 2
30 + 84 + 38 152
The length of the perpendicular from the centre C2(2, –4) to the common chord AB is C2M = =
Radius of the circle S2 = 0 is, C2A = 38 152 + 212 666

∴ The length of the common chord = AB = 2AM


2
 152  1102
= 2 C2 A2 − C2M2 ; = 2 38 −   =2
 666  333

Illustration 27: Tangents are drawn to the circle x2 + y2 = 12 at the points where it is met by the circle x2 + y2 – 5x
+ 3y – 2 = 0; find the point of intersection of these tangents.  (JEE ADVANCED)

Sol: As we know that, if (x1, y1) is a point of intersection of tangents of circle x2 + y2 = a2 then equation of chord of
contact is xx1 + yy1=a2 and the equation of common chord of two circles are S1 – S2 = 0 i.e., 2x(g1 – g2) + 2y (f1 – f2)
+ c1 – c2 = 0. By using these two formulae we can solve the problem.
Given circles are S1 ≡ x2 + y2 – 12 = 0 … (i)
and S2 ≡ x2 + y2 – 5x + 3y – 2 = 0 … (ii)
The equation of common chord is S1 – S2 = 0 i.e. 5x – 3y – 10 = 0  … (iii)
Let this line meet circle (i) at A and B, and P(α, β) be the point of intersection of the tangents at A and B. Therefore,
the equation of the chord of contact AB is xα + yβ – 12 = 0 … (iv)
As (iii) and (iv) represent the same line, therefore on comparison, we get
α β 6 18  18 
= = ∴ α = 6 and β = – . Hence, P ≡  6, − .
5 −3 5 5  5 

O
11. DIAMETER OF CIRCLE P(h,k)

The locus of the middle points of a system of parallel chords of a circle Diameter
x+my=0
is known as the diameter of the circle.
Let the equation of parallel chords be y=mx+c
Figure 9.40
M a them a ti cs | 9.23

y = mx + c (where, c is a parameter).
∴The equation of the diameter bisecting parallel chords
of the circle x2 + y2 = a2 is given by x + my = 0.

12. POLE AND POLAR


Let P (x1,y1) be any point inside or outside the circle. Passing through the point P chords AB and A’ B’ are drawn. If
the tangents at point A and point B intersect at Q (h, k) , then the locus of Q is a straight line and is called the polar
of point P with respect to circle and P is called the pole. Similarly, if the tangents to the circle at A’ and B’ meet at
Q’, then the locus of Q’ is the polar with P as its pole.
Q’ Q(h,k1) A
B’
A’

Polar
A’ Polar P(x1,y1)
B
Pole B
P(x1,y1) A
Q’ B’

Q(h,k1)

Figure 9.41(a): Polar of a point P outside the circle Figure 9.41(b): Polar of a point P inside the circle

Equation of polar of the circle x2 + y2 + 2gx + 2fy + c = 0 w.r.t. point P (x1, y1) is xx1 + yy1 + g(x + x1) + f(y + y1) + c = 0
i.e. T = 0.
If the circle is x2 + y2 = a2, then its polar w.r.t. (x1, y1) is xx1 + yy1 – a2 = 0 i.e. T = 0.
Pole of a line w.r.t. the circle x2 + y2 = a2
Consider a line lx + my + n = 0 and let (x1, y1) be the pole of the line w.r.t. the circle x2 + y2 = a2.
For the point (x1, y1),
The equation of polar w.r.t. the circle x2 + y2 = a2 is xx1 + yy1 – a2 = 0.
Since lx + my + n = 0 and xx1 + yy1 – a2 = 0 represent the same line.
x1 y1 −a2 a2l a2m
∴ = = ⇒ x1 = – and y1 = – .
l m n n n
 a2l a2m 
Hence, the pole of the line lx + my +n = 0 is  − ,− 
 n n 

Pole of a line w.r.t. the circle x2 + y2 + 2gx + 2fy + c = 0
Consider a line lx + my + n = 0.
If (x1, y1) is the pole, then the equation of polar is xx1 + yy1 + g(x + x1) + f(y + y1) + c = 0.
Now, since lx + my + n = 0 and xx1 + yy1 + g(x + x1) + f(y + y1) + c = 0 represent the same line,
x1 + g y1 + f gx1 + fy1 + c
∴ = =
l m n
x1 + g y1 + f g2 + f 2 − c
On simplification, we get = =
l m l g+ fm − n

x1 + g y1 + f r2
⇒ = = , where r is radius of the circle.
l m l g+ mf − n
9 .24 | Circle

12.1 Conjugate Points and Conjugate Lines


(a) If the polar of point P (x1 , y1) w.r.t. a circle x2 + y2 = a2, passes through Q(x2 , y2), then the polar of Q will pass
through P. Such points are called conjugate points and they satisfy the relation x1x2 + y1y2 = a2
(b) If the pole of the line l1x + m1y + n1 = 0 w.r.t. a circle lies on another line l2x + m2y + n2 = 0, then the pole of
the second line will lie on the first and such lines are said to be conjugate lines.
Consider the circle x2 + y2 = a2,
 a2l a2l 
The pole P of the line l1x + m1y + n1 = 0 w.r.t. the circle is given by  − 1 , − 1  .
 n1 n1 
 
 a2l   a2m 
⇒ l2  − 1  + m2  − 1 
+n = 0 ∴ a (l1l2 + m1m2 ) =
2
n1n2
 n1   n1  2
   

PLANCESS CONCEPTS

Points P (x1, y1) and Q (x2, y2) are conjugate points w.r.t. the circle x2 + y2 + 2gx + 2fy + c = 0 if x1x2 + y1y2
+ g (x1 + x2) + f(y1 + y2) + c = 0.
If P and Q are conjugate points w.r.t. a circle with centre at O and radius ‘a’ then PQ2 = OP2 + OQ2 – 2a2.
Shivam Agarwal (JEE 2009, AIR 27)

Illustration 28: Find the pole of the line 3x + 5y + 17 = 0 with respect to the circle x2 + y2 + 4x + 6y + 9 = 0. 
 (JEE MAIN)

Sol: If P(α, β) be the pole of line with respect to the given circle. Then the equation of polar of point P(α, β) w.r.t.
the circle is xα + yβ + 2(x + α) + 3(y + β) + 9 = 0. And this equation represent same line which is represented by
equation 3x + 5y + 17 = 0. By solving these two equation simultaneously we will get required pole.
Given circle is x2 + y2 + 4x + 6y + 9 = 0 ... (i)
and, given line is 3x + 5y + 17 = 0  … (ii
⇒ (α + 2) x + (β + 3) y + 2α + 3β + 9 = 0  ... (iii)
Since equation (ii) and (iii) represent the same line,
α+2 β + 3 2α + 3β + 9
∴ = = ⇒ 5α + 10 = 3β + 9
3 5 17
⇒ 5α – 3β = –1 ... (iv)
and, 17α + 34 = 6α + 9β + 27 ⇒ 11α – 9β = –7 ... (v)
From (iv) and (v), we get α = 1, β = 2
Hence, the pole of the line 3x + 5y + 17 = 0 w.r.t. the circle x2 + y2 + 4x + 6y + 9 = 0 is (1, 2).

Illustration 29: A variable circle is drawn to touch the axis of x at origin. Find locus of pole of straight line lx + my
+ n = 0 w.r.t. circle. (JEE ADVANCED)

Sol: As circle touches x-axis at origin therefore let (0, λ) be its centre then equation of circle will be x2 + (y – λ)2 =
λ2. Hence by considering P(x1, y1) be the pole and using polar equation we will get required result.
Let the centre of the circle be (0, λ).
Then the equation of the circle is x2 + (y – λ)2 = λ2
M a them a ti cs | 9.25

⇒ x2 + y2 – 2λy = 0.
Let P(x1, y1) be the pole of the line lx + my + n = 0 w.r.t. the circle,
then, the equation of the polar is xx1 + yy1 – λ(y + y1) = 0 (0,)
xx1 + y (–λ + y1) – λy1 = 0
x −λ + y1 −λy1
∴ On comparison, we =
get 1 = .
l m n
Figure 9.42
Hence, the locus of the pole is ly2 = mxy – xn.

Illustration 30: Prove that if two lines at right angles are conjugate w.r.t. circle then one of them passes through
centre.  (JEE ADVANCED)

Sol: Let two perpendicular lines which are conjugate to each other be
ax + by + c = 0 ... (i)
bx – ay + λ = 0  ... (ii)
∴ The equation of the polar of a point (x1, y1) is xx1 + yy1 – r2=0 … (iii)
x1 y1 −r 2
On comparing (i) and (iii), we get = = .
a b c
From the definition of conjugate lines, we know that the point (x1, y1) should satisfy the equation bx – ay + λ = 0,
−br 2a ar 2b
hence + + λ = 0 ⇒ λ = 0.
c c
Therefore, bx - ay + λ = 0 passes through (0, 0).

13. COMMON TANGENTS TO TWO CIRCLES


Different cases of intersection of two circles:
Let the two circles be (x – x1)2 + (y – y1)2 = r12 … (i) and, (x – x1)2 + (y – y2)2 = r22 … (ii)

Then following cases may arise:
Case I: When the distance between the centres is greater than the sum of radii. C1 C2 > r1 + r2

Direct common
tangent

r1
r2
P C1
C2 T

Transverse common
tangents
Figure 9.43: Common tangents for non-intersecting and non-overlapping circles

In this case four common tangents can be drawn, in which two are direct common tangents and the other two are
transverse common tangents.
9 .26 | Circle

The points P and T , the point of intersection of direct common tangents and transverse common tangents
respectively, always lie on the line joining the centres of the two circles. The point P and T divide the join of C1 and
C2 externally and internally respectively in the ratio r1 : r2.
C1P r1 C1 T r1
i.e. = (externally) and = (internally)
C2P r2 C2 T r2
r x −r x r y −r y   r1 x2 + r2 x1 r1 y 2 + r2 y1 
∴ P ≡  1 2 2 1 , 1 2 2 1  and T ≡  , .
 r1 − r2 r1 − r2   r1 + r2 r1 + r2 

Steps to find Equations of Common Tangents


Let the equation of tangent of any circle in the slope form be (y+f) = m(x+g) + a 1 + m2 where, a is radius of circle
and m is the slope of tangent.
The value of ‘m’ can be obtained by substituting the co-ordinates of the point P and T in the above equation.

Note: Length of an external (or direct) common tangent, Lext = d2 − (r1 − r2 )2 , and Length of an internal (or

transverse) common tangent, Lint = d2 − (r1 + r2 )2 . where, d is the distance between the centres of the two circles,

and r1, r2 are the radii of the two circles. Therefore, the length of internal common tangent is always less than the
length of the external common tangent.
Case-II: When the distance between the centres is equal to the sum of radii (Circles touching externally)

C1 C2= r1 + r2
Direct common
tangent

r1
r2
P C1
C2 T

Transverse common
tangents

Figure 9.44: Common tangents of circles touching externally

In this case three common tangents can be drawn, two direct common tangents and one transverse common
tangent.
Case III: When the distance between the centres is less than the sum of radii. (Intersecting circles)

r1 − r2 < C1 C2 < r1 + r2
Direct common
Tangent

P C1
C2

Figure 9.45: Common tangents for intersecting circles


M a them a ti cs | 9.27

In this case two direct common tangents can be drawn as shown in the diagram.
Case IV: When the distance between the centres is equal to the difference of the radii. (Circles touching each other
internally), i.e.C1C2 = r1 –r2.

Tangent at
the point of
contact
r2
P C1
C2
r1

Figure 9.46: Common tangents for circles touching each other internally

In this case the total number of common tangents is one.


Case V: When the distance between the centres is less than the difference of the radii. (Circles neither touch each
other nor intersect), i.e. C1C2<r1 – r2.

r2
C1
C2
r1

Figure 9.47

In this case, the number of common tangents is zero.

Illustration 31: Examine if the two circles x2 + y2 – 2x – 4y = 0 and x2 + y2 – 8y – 4 = 0 touch each other externally
or internally. (JEE MAIN)

Sol: When distance between centre of circle is equal to the sum of their radius then they touches eachother
externally and when it is equal to the difference of their radius then circle touches eachother internally.
Let C1 and C2 be the centres and r1 and r2 the radii of S1 ≡x2 + y2 – 2x – 4y = 0 and S1 ≡x2 + y2 – 8y – 4 = 0 respectively.
∴ C1 ≡ (1, 2), C2 ≡ (0, 4), r1 = 5 , r2 = 2 5

(1 − 0 ) + ( 2 − 4 )
2 2
Now, C 1C 2 = = 5 and

r1 +r2 = 3 5 ,r1 –r2 = 5


Thus, C1C2 = r1 –r2, hence the two circles touch each other internally,

Illustration 32: Prove that the two circles x2 + y2 + 2ax + c = 0 and x2 + y2 +2by + c = 0 touch each other,
1 1 1
if + =  (JEE ADVANCED)
2
a b 2 c

Sol: Two circles touch each other if distance between centres of these two circles are equal to the sum or difference
of their radius.
Let centres of given circles be C1 and C2 and their radii be r1 and r2 respectively.
9 .28 | Circle

∴ C1 ≡ (–a, 0); r1 = a2 − c and C2 ≡ (0, – b); r2 = b2 − c

Two circles touch each other, if C1C2 = r1 ± r2


⇒ a2 + b2= a2 − c ± b2 − c ⇒ a2 + b2 = a2 – c + b2 – c ±2 (a − c )(b
2 2
−c )
⇒ c =± a b − a c − b c + c
2 2 2 2 2
⇒ c = a b – a c – b c + c
2 2 2 2 2 2

1 1 1
⇒ a2c+ b2c = a2b2 ∴ + =.
a 2 2
b c

Illustration 33: An equation of a common tangent to the circles x2 + y2 + 14x – 4y + 28 = 0


and x2 + y2 – 14x + 4y – 28 = 0 is  (JEE ADVANCED)
(A) x – 7 = 0 (B) y + 7 = 0 (C) 28x + 45y + 371 = 0 (D) None of these

Sol: (C) Calculate the distance between the centres and use the different cases of two circles.
Let S1 ≡x2 + y2 + 14x – 4y + 28 = 0 …(i) A2
A1
⇒ C1 = (-7 , 2) and r1 = 5
9
5
and, S2 ≡x + y - 14x + 4y - 28 = 0
2 2
…(ii) P
⇒ C2 = (7 , -2) and r2 = 9

( 7 + 7 ) + ( −2 − 2)
2 2
∴ C1C=
2 > r1 + r2.
Figure 9.48
Hence, four common tangents are possible.
For x – 7 = 0, … (iii)
Clearly, C2 lies on the (iii).
For y + 7 = 0 … (iv)
Length of perpendicular from C1 = 9 > r1.
For 28x + 45y + 371 = 0 … (v)
o
180 -
28( −7) + 45(2) + 371 265
Length of perpendicular from=
C1 = = r1 . r1 r2
282 + 452 53 C1
d C2

28(7) + 45( −2) + 371 477


Length of perpendicular from C2 = = = r2 .
282 + 452 53
Figure 9.49

14. ANGLE OF INTERSECTION OF TWO CIRCLES


The angle of intersection between two circles S = 0 and S’ = 0 is defined as the angle between their tangents at
their point of intersection.
If S ≡ x2 + y2 + 2g1x + 2f1y + c1 = 0;
P
S’ ≡ x2 + y2 + 2g2x + 2f2y + c2 = 0
90°
are two circles with radii r1, r2 and d be the distance between their centres then
r 2 + r 2 − d2
C1 C2
the angle of intersection θ between them is given by cos (180 − θ ) = 1 2 (-g1, -f1) (-g2, -f2)
2r1r2
2 ( g1g2 + f1 f2 ) − ( c1 + c2 )
or, cos (180 − θ ) = .
2 g12 + f12 − c1 g22 + f22 − c1 Figure 9.50: Angle of intersection
M a them a ti cs | 9.29

Condition of Orthogonality: Two circles are said to be orthogonal to each other if the angle of intersection of the
two circles is 90°.
⇒ 2(g1g2 + f1f2) = c1 + c2 .

PLANCESS CONCEPTS

If two circles are orthogonal, then the polar of a point ‘P’ on first circle w.r.t. the second circle passes
though the point Q which is the other end of the diameter through P. Hence locus of a point which
moves such that its polar w.r.t. the circles S1 = 0, S2 =0 & S3 = 0 are concurrent in a circle which is
orthogonal all the three circles.
Ravi Vooda (JEE 2009, AIR 71)

Illustration 34: If a circle passes through the point (3, 4) and cuts the circle x2 + y2 = a2 orthogonally, the equation
of the locus of its centre is (JEE MAIN)
(A) 3x + 4y – a2 = 0 (B) 6x + 8y = a2 + 25
(C) 6x + 8y + a2 + 25 = 0 (D) 3x + 4y =a2 + 25

Sol : (B)
As we know Two circle are said to be orthogonal if 2(g1g2 + f1f2) = c1+c2. So by considering required equation of
circle as x2 + y2+ 2gx + 2fy + c = 0 and As point (3, 4) satisfies this equation so by solving these two equation we
will get required equation of the locus of its centre.
Let the equation of the circle be x2 + y2+ 2gx + 2fy + c = 0 ... (i)
As the point (3, 4) lies on (i), we have 9 + 16 + 6g +8f + c = 0
⇒ 6g + 8f + c = – 25 ... (ii)
⇒ 2g×0 + 2f×0 = c – a2 ⇒ c = a2 .
∴ From equation (ii), we have 6g + 8f + a2 + 25 = 0.
Hence locus of the centre (–g, –f) is 6x + 8y – (a2 + 25) = 0.

Illustration 35: Obtain the equation of the circle orthogonal to both the circles x2 + y2 +3x – 5y + 56 = 0 and
4x2 + 4y2 – 28x + 29 = 0 and whose centre lies on the line 3x + 4x + 1 = 0. (JEE ADVANCED)

Sol: By considering the required circle to be S ≡ x2 + y2 + 2gx + 2fy + c = 0 and using orthogonality formula
2(g1g2 + f1f2) = c1+c2 we will get a relation between g and f. Also as the centre lies on the line 3x + 4x + 1 = 0 , by
solving these equation we will get required result.
Let the required circle be S ≡ x2 + y2 + 2gx + 2fy + c = 0 … (i)
Given S1 ≡ x + y + 3x – 5y + 56 = 0
2 2
… (ii)
29
and, S2 ≡ x2 + y2 – 7x + = 0. ... (iii)
4
Since (i) is orthogonal to (ii) and (iii)
3  5
∴ 2g   + 2f  −  =c + 6 + 2f ⇒ 3g – 5f = c + 6 ... (iv)
2
   2
9 .30 | Circle

 7 29 29
and 2g  −  + 2f.0 ⇒ c+ ⇒ –7g = c +  ...(v)
 2 4 4

From (iv) and (v), we get 40g –20f = –5 ... (vi)
Given line is 3x + 4y = –1  ... (vii)
(–g, –f) also lies on the line (vii). ⇒ –3g – 4f = –1 ... (viii)
1 29
∴ g =0, f = and c =– [From (vi) and (viii)]
4 4
1 29
∴ The equation of the circle is x2 + y2 + y– = 0 or, 4(x2 + y2) + 2y –29 = 0
2 4

15. FAMILY OF CIRCLES


(a) The equation of the family of circles passing through the point of intersection of two given circle S = 0 and
S’ = 0 is given by S + λS’ = 0, (where λ is a parameter, λ≠–1)

S’=0

S=0

Figure 9.51

(b) The equation of the family of circles passing through the point of intersection of circle S = 0 and a line L = 0
is given by S + λL = 0, (where λ is a parameter)

S+L=0 S=0
L=0

Figure 9.52

(c) The equation of the family of circles touching the circle S =0 and the line L = 0 at their point of contact P is
S + λL = 0, (where λ is a parameter)

S+L=0 S=0

L=0
Figure 9.53
M a them a ti cs | 9.31

(d) The equation of a family of circles passing through two given points P(x1, x1) and Q(x2, x2) can be written in the
form
x y 1
(x –x1) (x – x2) + (y –y1)(y – y2) + λ x1 y1 1 = 0, (where λ is a parameter)
x2 y 2 1

Figure 9.54

In this equation, (x –x1) (x – x2) + (y –y1)(y – y2) = 0 is the equation of the circle with P and Q as the end points
x y 1
of the diameter and x1 y1 1 =0 is the equation of the line through P and Q.
x2 y2 1

(e) The equation of the family of circles touching the circle S ≡ x2 + y2 + 2gx +2fy +c = 0 at point P(x1, y1) is x2 +
y2 + 2gx + 2fy + c + λ {xx1 + yy1 + g(x +x1) + f(y + y1) +c} = 0 or, S + λL = 0, where, L = 0 is the equation of
the tangent to the circle at P(x1, y1) and λ ∈ R.

p(x1,y1)

Figure 9.55

(f) The equation of family of circle, which touch y – y1 = m(x –x1) at (x1, y1) for any finite m is
(x – x1)2 + (y – y1)2 + λ {(y – y1) –m(x – x1)} = 0. And if m is infinite, the family of circle is
(x – x1)2 + (y – y1)2 + λ(x – x1) = 0, (where λ is a parameter)

p(x1,y1)

Figure 9.56
9 .32 | Circle

Note that (x – x1)2 + (y – y1)2 = 0 represents the equation of a point circle with centre at (x1, y1)
(g) Equation of the circles given in diagram is
(x – x1)(x – x2) + (y – y1) (y – y2) ±cot θ {(x – x1)(y – y2) – (x – x2) (y – y1)} = 0

(h) Family of circles circumscribing a triangle whose side are given by L1 = 0; L2 = 0 and L3 = 0 is given by L1 L2 + λL2
L3 + µL3 L1 = 0 provided coefficient of xy = 0 and co-coefficient of x2 = co-efficient of y2.

(x1,y1)

 

(x2,y2)

Figure 9.57

(i) Equation of circle circumscribing a quadrilateral whose sides in order are represented by the lines L1 = 0, L2 =
0, L3 = 0 & L4 = 0 are L1 L3 + λL2 L4 = 0 where value of λ can be found out by using condition that co-efficient
of x2 = y2 and co-efficient of xy = 0.

Illustration 36: Find the equation of circle through the points A(1, 1) & B(2, 2) and whose radius is 1.(JEE MAIN)

Sol: As we know that, equation of family of circle passing through (x1, y1) and
(x2, y2) is given by (x − x1 )(x − x2 ) + (y − y1 )(y − y 2 ) + λ(x − y) = 0
Equation of AB is x – y = 0
∴ Equation of the family of circle passing through A and B is
(x – 1)(x – 2) + (y – 1) (y – 2) + λ (x – y) = 0 or x2 + y2 + (λ – 3)x – (λ + 3) y + 4 = 0

( λ − 3) ( λ + 3)
2 2

∴ Radius = + −4 .
4 4

( λ − 3) ( λ + 3)
2 2

According to the question, + −4 =1


4 4
or (λ – 3)2 + (λ +3)2 – 16 = 4 or 2λ2 = 2 or λ = ±1
∴ Equation of circle is x2 + y2 – 2x – 4y + 4 = 0 and x2 + y2 – 4x –2y + 4 = 0

Illustration 37: Find the equations of circles which touches 2x – y + 3 = 0 and pass through the points of intersection
of the line x + 2y – 1 = 0 and the circle x2 + y2 –2x + 1 = 0. (JEE MAIN)

Sol: Here in this problem the equation of family of circle will be S + λL = 0 by solving this equation we will get centre
and radius of required circle in the form of λ and as this circle touches the line 2x – y + 3 = 0 hence perpendicular
distance from centre of circle to the line is equal to the radius of circle.
Let the equation of the family of circles be S + λL = 0.
∴ x2+ y2 – 2x + 1 + λ (x + 2y – 1) = 0 or, x2 + y2 – x(2 – λ) + 2λy + (1 – λ) = 0  ... (i)
⇒ Centre (–g, –f) is ({2 – λ}/2, – λ)

(2 − λ )
2
1 λ
⇒r= g2 + f 2 −=
c + λ 2 − (1 − =
λ) λ2
5= 5.
4 2 2
M a them a ti cs | 9.33

Since the circle touches the line 2x – y + 3 = 0,

2. ( 2 − λ ) / 2 − ( −λ ) + 3 λ λ
∴ = 5 or, 5 = ± 5 ⇒ λ = ±2  … (ii)
± 5 2 2

Hence, the required circles are x2 + y2+ 4y – 1 = 0 and x2 + y2 – 4x – 4y +3 = 0.

Illustration 38: If P and Q are the points of intersection of the circles x2 + y2 + 3x +7y +2p – 5 = 0
and x2 + y2 +2x + 2y + p2 = 0, then there is a circle passing through P, Q and (1, 1) for (JEE MAIN)
(A) All except two values of p (B) Exactly one value of p
(C) All values of p (D) All except one value of p.

Sol: (D) Here in this problem the equation of family of circle will be S + λL = 0 . and as the circle passes through
(1, 1), we can find the values of P such that λ is any real no. except – 1.
Equation of a circle passing through P and Q is
x2 + y2 + 3x +7y +2p – 5 + λ (x2 + y2 + 2x + 2y – p2) = 0 … (i)
Since (i) also passes through (1 , 1), we get (7 + 2p) – λ (p2 – 6) = 0
7 + 2p
⇒λ= ≠ –1 ⇒ p ≠ –1.
p2 − 6

Illustration 39: C1 and C2 are circles of unit radius with centres at (0, 0) and (1, 0) respectively. C3 is a circle of unit
radius, passes through the centres of the circles C1 and C2 and have its centre above x-axis. Equation of the common
tangent to C1 and C3 which does not pass through C2 is  (JEE ADVANCED)
(A) x – 3 y + 2 = 0 (B) 3x–y+2=0

(C) 3 x – y – 2= 0 (D) x + 3 y + 2 = 0

Sol: (B) Equation of any circle passing through any two point (x1, y1)
Y
and (x2, y2) is given by
x y 1
( x – x1 ) ( x – x2 ) + ( y – y1 ) ( y – y 2 ) + λ 0 0 1 = 0
1 0 1

Equation of any circle passing through the centre of C1 and C2 is


x y 1
(2 2 )
C3 1 , 3

(x – 0) (x – 1) + (y – 0) (y – 0) + λ 0 0 1 = 0
1 0 1
⇒ x + y – x + λy = 0.
2 2
… (i) X’
C1 C2
X
(0,0) (1,0)
If (i) represents C3, its radius = 1
⇒ 1 = (1/4) + (λ2/4) ⇒ λ = - 3 (as λ cannot be +ve)
Y’
Hence, the equation of C3 is x2 + y2 – x – 3y = 0.
Figure 9.58
Since the radius of C1 and C3 are equal, their common tangents will
be parallel to the line joining their centres (0, 0)
1 3 
and  , 
2 2 
 
9 .34 | Circle

So, let the equation of a common tangent be y = 3 x + k. … (ii)


k
From the condition of tangency on C1, we get = 1 ⇒ k = ±2
3+1

Since the tangent does not pass through C2, the equation of the required common tangent is 3 x – y + 2 = 0.

Illustration 40: Find the equation of circle circumscribing the triangle whose sides are 3x – y – 9 = 0, 5x – 3y– 23 = 0
& x + y – 3 = 0. (JEE ADVANCED)
Sol: Given L1 ≡ 3x – y – 9 = 0 L2 ≡ 5x – 3y – 23 = 0 L3 ≡ x + y – 3 = 0
Family of circles circumscribing a triangle whose side are L1 = 0; L2 = 0 and L3 = 0 is
L1L2 + λL2 L3 + µL3 L1 = 0 provided coefficient of xy = 0 & co-coefficient of x2 = co-efficient of y2 .
∴ L1L2 + λL2L3 + µL1L3 = 0
⇒ (3x – y – 9)(5x – 3y – 23) + λ(5x – 3y – 23)(x + y – 3) + µ(3x – y – 9) (x + y – 3) = 0
⇒ (15x2 + 3y2 – 14xy – 114x + 50y + 207) + λ(5x2 – 3y2 + 2xy – 38x – 14y + 69)
+ µ(3x2 – y2 + 2xy –18x – 6y + 27) = 0
⇒ (5λ + 3µ + 15) x2 + (3 – 3λ – µ) y2 + xy(2λ + 2µ – 14) – x(114 + 38λ + 18µ)
+ y(50 – 14λ – 6µ) + (207 + 69λ +27µ) = 0 ... (i)
The equation (i) represents a circle if
coefficient of x2 = coefficient of y2
⇒ 5λ + 3µ + 15 = 3 – 3λ – µ ⇒ 8λ + 4µ + 12 = 0 ; 2λ + µ + 3 = 0 ... (ii)
and, coefficient of xy = 0
⇒ 2λ + 2µ – 14 = 0 ⇒λ + µ – 7 = 0  ... (iii)
From equation (ii) and (iii), we have λ = –10, µ = 17
Putting these values of λ & µ in equation (i), we get 2x2 + 2y2 – 5x + 11y – 3 = 0

16. RADICAL AXIS AND RADICAL CENTRE


P(x1,y1)

16.1 Radical Axis


The radical axis of two circles is defined as the locus of a point which c2
moves such that the lengths of the tangents drawn from it to the two c1
s2 =0
circles are equal. The radical axis of two circles is a straight line.
S1 = S2 s1 =0

⇒ S1 = S2 ⇒ S1 − S2 = 0
Figure 9.59
Consider two circles given by S1 = 0 and S2 = 0. Then the equation of
the radical axis of the two circle is S1 – S2 = 0
i.e. 2x (g1 – g2) + 2y(f1 – f2) + c1 – c2 = 0,
which is a straight line.
S1
Properties of the radical axis S1-S2=o
S2
(a) For two intersecting circles the radical axis and common chord are identical. Also, the
radical axis and the common tangent are same for two circles touching each other. Figure 9.60
M a them a ti cs | 9.35

(b) The radical axis is perpendicular to the line joining the centres of the two circles.
(c) If two circles cut a third circle orthogonally, the radical axis of the two circles will pass through the centre of
the third circle.
(d) Radical axis does not exist if circles are concentric.
(e) Radical axis does not always pass through the mid-point of the line joining the centre of the two circles.
(f) The radical axis of two circles bisects all common tangents of the two circles.

L
16.2 Radical Centre
s1=0
The point of intersection of the radical axis of three circles, taken in pairs,
is known as their radical centre. s3=0
Let the three circles be
S1 = 0 …..(i), S2 = 0 ...(ii), S3 = 0 ...(iii)
O
Refer to the diagrams shown alongside.
N
Let the straight line OL be the radical axis of the circles S1 = 0 & S3 = 0
and the straight line OM be the radical axis of the circles S1 = 0 & S2 = 0.
The equation of any straight line passing through O is given by (S1 – S2) M
s2=0
+ λ(S3 – S1) = 0 , where λ is any constant.
Figure 9.61
For λ= 1 , this equation become S2 – S3 = 0 , which is, equation of ON.
Clearly, the third radical axis also passes through the point where the
straight lines OL and OM meet. Hence, the point of intersection of the three radical axis, O is the radical centre.

Properties of radical center


(a) Co-ordinates of radical centre can be found by solving the equation S1 = S2 = S3.
(b) The radical centre does not exist if the centre of three circles are collinear.
(c) The circles with centre at radical centre and radius is equal to the length of tangents from radical centre to any
of the circle will cut the three circle orthogonally.
(d) If circles are drawn on three sides of a triangle as diameter then radical centre of the three circles is the
orthocenter of the triangle. Hence, in case of a right angled triangle, the radical centre of the three circles with
the sides as diameter is the vertex with the right angle.

PLANCESS CONCEPTS

Alternate approach to find the equation of the tangent of a circle passing through a point lying on a
given circle.
Consider a point (x1 , y1) on the given circle S1 = 0. Then the equation of a point circle with (x1 , y1) as the
centre is S2 ≡ (x – x1)2 + (y – y1)2= 0. Now we have two circles - one given circle and another point circle.
We now have to find the radical axis of those two circles, which is S1 – S2 = 0.
E.g.: Given a circle x2 + y2 = 8 and the point on circle is (2, 2), we need to find equation of a tangent to
the circle at point (2, 2).
Point circle: (x – 2)2 + (y – 2)2= 0 ⇒ x2 + y2 – 4x – 4y +8= 0
Hence, the radical axis is S1 – S2 = 0. ⇒ x + y = 4, which is also the tangent to the given circle at the
point (2, 2).
Akshat Kharaya (JEE 2009, AIR 235)
9 .36 | Circle

Illustration 41: Find the co-ordinates of the point from which the lengths of the tangents to the following three
circles be equal. 3x2 + 3y2 + 4x – 6y – 1= 0, 2x2 + 2y2 – 3x – 2y – 4 = 0, 2x2 + 2y2 – x + y – 1 = 0. (JEE MAIN)

Sol: Here by using formula S1 – S2 = 0, S2 – S3 = 0, and S3 – S1 = 0 we will get equations of radical axis and solving
these equations we will get required co-ordinate.
Reducing the equation of the circles to the standard form,
4 1
S1 ≡x2 + y2 + x – 2y – = 0
3 3
3
S2 ≡x2 + y2 - x–y–2=0
2
1 1 1
S3 ≡x2 + y2 - x + y – = 0
2 2 2
Hence, the equations of the three radical axis is given by
17 5
L1 ≡ x – y + = 0 … (i)
6 3
3 3
L2 ≡–x – y – = 0,  … (ii)
2 2
11 5 1
and, L3 ≡– x+ y+ = 0. … (iii)
6 2 6
 16 31 
Solving (i) and (ii), we get the point  − ,  , which also satisfies the equation (iii).
 21 63 
This point is called the radical centre and by definition the length of the tangents from it to the three circles are
equal.

Illustration 42: Find the equation of the circle orthogonal to the three circles x2 + y2 – 2x + 3y – 7 = 0, x2 + y2 +
5x – 5y + 9 = 0 and x2 + y2 + 7x – 9y + 29 = 0 (JEE ADVANCED)

Sol: By using formula of radical axis we will get co- ordinate of radical centre which is also equal to the centre of
required circle.
The given circles are
S1 ≡x2 + y2 – 2x + 3y – 7 = 0 … (i)
S2 ≡x2 + y2 + 5x – 5y + 9 = 0 … (ii)
and S3 ≡x + y + 7x – 9y + 29 = 0
2 2
... (iii)
The radical axis of S1 = 0 and S2 = 0 is 7x – 8y + 16 = 0 ... (iv)
The radical axis of S2 = 0 and S3 = 0 is x – 2y + 10 = 0  ...(v)
∴ The radical centre is (8, 9).
Therefore, the length of the tangent from (8, 9) to each of the given circles is 149 .
∴ The required equation is (x – 8)2 + (y – 9)2 = 149 or x2 + y2 – 16x – 18y – 4 = 0.

Illustration 43: If two circles intersect a third circle orthogonally. Prove that their radical axis passes through the
centre of the third circle. (JEE ADVANCED)

Sol: By considering equation of these circles as Sr = x2 + y2 + 2grx + 2fry + cr = 0


(r = 1, 2, 3) and using radical axis formula we will prove given problem.
Let the given circles be Sr = x2 + y2 + 2grx + 2fry + cr = 0 (r = 1, 2, 3)
M a them a ti cs | 9.37

Let S1 and S2 cut each other orthogonally, then we have


2g1g2 + 2f1f2 = c1 + c2  ... (i)
Similarly, let S2 and S3 cut each other orthogonally, then we have
2g2g3+ 2f2f3 = c2 + c3 ... (ii)
Subtracting (ii) from (i), we get 2(g1 – g3)g2 + 2(f1 – f3)f2 = c1 – c3  ... (iii)
Now the radical axis of S1 and S3 is 2(g1 – g3)x +2(f1 – f3)y + c1 –c3 = 0 ... (iv)
From (iii) and (iv), the point (–g2, –f2) lies on the line (iv). Hence, proved.

Illustration 44: Prove that the square of the length of tangent that can be drawn from any point on one circle to
another circle is equal to twice the product of the perpendicular distance of the point from the radical axis of the
two circles, and the distance between their centres. (JEE ADVANCED)

Sol: Consider two circle as S1 ≡ x2 + y2 = a2 and S2 ≡ (x – h)2 + y2 = b2 and then by using radical axis formula and
perpendicular distance formula we will prove given problem.
We have to prove that PQ2 = 2 ×PN×C1 C2
Let the equation of the two circles be
S1 ≡ x2 + y2 = a2, and … (i)
S2 ≡ (x – h)2 + y2 = b2 … (ii)
Let P ≡ (a cos θ, a sin θ) be a point on the circle S1 =0 ∴ PQ = 2 2 2
a − b + h − 2ahcos θ
and, Radical axis is {x2 + y2 - a2} – {(x – h)2 + y2 - b2 } = 0
h2 + a2 − b2
⇒ –2 hx + h2 + a2 - b2 = 0 or x=
2h
h2 + a2 − b2 h2 + a2 − b2 − 2ahcos θ
⇒ PN = − acos θ =
2h 2h

h2 + a2 − b2 − 2ahcos θ PQ2
⇒ PN×C1C2 = ×h ⇒ PN × C1C2 =
2h 2
∴ PQ2 = 2 PN×C1C2

17. CO-AXIAL SYSTEM OF CIRCLES


A system (or a family) of circles, every pair of which have the same radical axis, are called co-axial circles.
(1) The equation of a system of co-axial circles, when the equation of the radical axis is P ≡ lx + my + n = 0 and,
one circle of the system is S ≡ x2 + y2 + 2gx + 2fy + c = 0 respectively, is S + λP = 0 (λ is an arbitrary constant).

S+P=0 S+P=0

S+P=0
S=0
P=0
Figure 9.62

(2) The equation of a co-axial system of circles, when the equation of any two circles of the system are S1 ≡ x2 + y2
+ 2g1x + 2f1y + c1 = 0 and S2 ≡ x2 + y2 + 2g2x + 2f2y + c2 = 0 respectively, is S1 + λ(S1 – S2) = 0
9 .38 | Circle

S2 + (S1 - S2) = 0
S2 + (S1 - S2) = 0
S1 + (S1 - S2) = 0
S1+S1= 0 S2 = 0

S1 = 0
S1 = 0 S2 = 0 S1 - S2 = 0
Figure 9.63

or S2 + λ1(S1 – S2) = 0
Other form
S1 + λS2 = 0, (λ ≠ –1)

Properties of co-axial System of Circles


(a) Centres of all circles of a coaxial system lie on a straight line which is perpendicular to the common radical axis
as the line joining the centres of two circles is perpendicular to their radical axis.
(b) Circles passing through two fixed points P and Q form a coaxial system, because every pair of circles has the
same common chord PQ and therefore, the same radical axis which is perpendicular bisector of PQ.

PLANCESS CONCEPTS

The equation of a system of co-axial circles in the simplest form is x2 + y2 + 2gx + c = 0 , where g is a
variable and c is a constant. This is the system with center on x-axis and y-axis as common radical axis.
Anvit Tawar (JEE 2009, AIR 9)

Illustration 45: Find the equation of the system of coaxial circles that are tangent at ( )
2, 4 to the locus of point
of intersection of mutually perpendicular tangents to the circle x2 + y2 = 9. (JEE ADVANCED)

Sol: The locus of point of intersection of mutually perpendicular tangents is known as the Director circle. Hence by
using formula of director circle and co-axial system of circle we will get required result.
∴ The equation of the locus of point of intersection of perpendicular tangents is
x2 + y2 = 18 ... (i)
Since, ( )
2, 4 satisfies the equation x2 + y2= 18,

∴ The tangent at ( )
2, 4 to the circle x2 + y2 = 18 is x⋅ 2 + y⋅4 = 18 ... (ii)

The equation of the family of circles touching (i) at ( )


2, 4 is

x2 + y2 –18 +λ ( 2x + 4y – 18) = 0 or x2 + y2 + 2 λx + 4λy – 18(λ + 1) = 0 ... (iii)

Also any two circles of (iii) have the same radical axis 2x + 4y – 18 = 0

∴ The required equation of coaxial circles is (iii).


M a them a ti cs | 9.39

Illustration 46: Find equation of circle co-axial with S1=x2+y2+4x+2y+1=0 and S2 = 2x2 + 2y2 – 2x – 4y – 3 = 0 and
centre of circle lies on radical axis of these 2 circles.  (JEE MAIN)

Sol: By using S1 – S2 = 0 and S1 + λL = 0 we will get equation of radical axis and equation of co-axial system of
circle respectively.
5
S1 – S2 = 0 ⇒ 5x + + 4y = 0 ⇒ 10x +8y + 5 = 0
2
∴ The equation of the radical axis is 10x + 8y + 5 = 0
The equation of the coaxial system of circles is x2 + y2 + 4x + 2y + 1 +λ(10x + 5 + 8y) = 0
23
⇒ Centre ≡ [ −(2 + 5λ ), − (1 + 4λ )] which lies on radical axis, after substituting we get ⇒ λ = −
82

Illustration 47: For what values of l and m the circles 5(x2 + y2) + ly – m = 0 belongs to the coaxial system
determined by the circles x2 + y2 + 2x + 4y – 6 = 0 and 2(x2 + y2) – x = 0 ? (JEE ADVANCED)

Sol: By using radical axis formula i.e. S1 – S2 = 0 we will get equations of radical axis and by solving them
simultaneously we will get required value of l and m.
Let the circles be S1 ≡ x2 + y2 + 2x + 4y – 6 = 0;
1
S2 ≡ x2 + y2 – x = 0;
2
l m
S3 ≡ x2 + y2 + y– = 0.
5 5
The equation of the radical axis of circles S1 = 0 and S2 = 0 is S1 – S2 = 0,
 1 
i.e., x2 + y2 + 2x + 4y – 6 –  x2 + y 2 − x  = 0 or, 5x + 8y – 12 = 0 ... (i)
 2 
The equation of the radical axis of circles S2 = 0 and S3 = 0 is S2 – S3 = 0,
1  l m
i.e., x2 + y2 – x –  x2 + y 2 + y −  = 0 or, 5x + 2ly – 2m = 0 ... (ii)
2  5 5
5 8 −12 4 6
On comparing (i) and (ii), = = ⇒ 1= = ∴ l = 4, m = 6.
5 2l −2m l m

18. LIMITING POINTS


Limiting point of system of co-axial circles are the centres of the point circles belonging to the family
Let the circle be x2 + y2 + 2gx + c = 0 where g is a variable and c is a constant.
∴ Centre ≡ (–g, 0) and Radius = g2 − c .
A circle is said to be a point circle, if the radius is equal to 0, i.e. g2 − c = 0 ⇒ g = ± c
Thus, we get the two limiting points of the given co-axial system as ( c , 0) and (– c , 0).
Depending on the sign of c, either the limiting points are real and distinct, real and coincident or imaginary.

18.1 System of Co-axial Circles when Limiting Points are given


Let (a, b) and (α , β) be two limiting points of a coaxial system of circles. Then, the equation of the corresponding
point circles are S1 ≡ (x – a)2 + (y – b)2 = 0 and S2 ≡ (x – α)2 + (y – β)2 = 0.
∴ The coaxial system of circles is given by S1 + λS2 = 0, λ ≠ –1.
or, (x – a)2 + (y – b)2 + λ{(x – α)2 + (y – β)2} = 0, λ ≠ –1.
9 .40 | Circle

PLANCESS CONCEPTS

If origin is a limiting point of the coaxial system containing the circles x2 + y2 + 2gx + 2fy + c = 0
 −gc −fc 
then the other limiting point is  , .
 g2 + f 2 g2 + f 2 
 
π
A common tangent drawn to any two circles of a coaxial system subtends an angle of at the limiting
points. 2

Chinmay S Purandare (JEE 2012, AIR 698)

Illustration 48: Equation of a circles through the origin and belonging to the co-axial system, of which the limiting
points are (1, 2), (4, 3) is (JEE ADVANCED)
(A) x2 + y2 – 2x + 4y = 0 (B) x2 + y2 – 8x – 6y = 0
(C) 2x2 + 2y2 – x – 7y = 0 (D) x2 + y2 – 6x – 10y = 0

Sol: (C) As we know, if (a, b) and (α , β) be two limiting points of a coaxial system of circles. Then, the equation of
the corresponding point circles are S1 ≡ (x – a)2 + (y – b)2 = 0 and S2 ≡ (x – α)2 + (y – β)2 = 0 so by using the formula
of co-axial system i.e. S1 + λS2 = 0 we will get required result.
Equations of the point circles having (1, 2) and (4,3) as centres is
S1 ≡ (x – 1)2 + (y – 2)2 = 0 ⇒ x2 + y2 – 2x – 4y + 5 = 0
and, S2 ≡ (x – 4)2 + (y – 3)2 = 0 ⇒ x2 + y2 – 8x – 6y + 25 = 0
∴ The co-axial system of circles is S1 + λ S2 = 0.
i.e. x2 + y2 – 2x –4y + 5 + λ(x2 + y2 – 8x – 6y + 25) = 0 … (i)
If (0, 0) lies on the circle given by equation (i), then
02 + 02 – 2(0) – 4(0) + 5 + λ(02 + 02 – 8(0) – 6(0) + 25) = 0
1
⇒ 5 + 25λ = 0 or, λ = –   .
5
∴ The equation of the required circle is 5(x2 + y2 – 2x – 4y + 5) – (x2 + y2 – 8x – 6y + 25) = 0
⇒ 4x2 + 4y2 – 2x – 14y = 0 ⇒ 2x2 + 2y2 – x – 7y = 0.

19. IMAGE OF THE CIRCLE BY LINE MIRROR


Here, let us consider a general equation of a circle x2 + y2 + 2gx + 2fy + c = 0 and a line lx + my + n = 0. If we take
the image of the circle in the given line, then the radius of image circle remains unchanged and the centre lies on
the opposite side of the line at an equal distance.
Let the centre of image circle be (x1, y1).
∴ Slope of C1C2 × Slope of (lx + my +n = 0) = –1 ..... (i)
And the mid-point of C1 and C2 lies on the line lx + my + n = 0
 x1 − g   y1 − f 
l   + m 
  + n =0  ….. (ii)
 2   2 
M a them a ti cs | 9.41

From (i) and (ii), we get the centre of the image circle and the radius is
hence the equation of the image.
(g
2
)
+ f 2 − c (same as the given circle), and

C1 Given
circle
r

Given line

C2 Image of the circle

Figure 9.64: Image of a circle

PROBLEM-SOLVING TACTICS

(a) Let S = 0, S’ = 0 be two circles with centers C1, C2 and radii R1, R2 respectively.
(i) If C1C2 > r1 + r2 then each circle lies completely outside the other circle.
(ii) If C1C2 = r1 + r2 then the two circles touch each other externally. (Trick) the point of contact divides C1C2
in the ratio r1 : r2 internally.
(iii) If |r1 – r2| < C1C2 < r1 + r2 then the two circles intersect at two points P and Q.
(iv) If C1C2 = |r1 – r2| then the two circles touch each other internally. (Trick) The point of contact divides C1C2
in the ratio r1 : r2 externally.
(v) If C1C2 < |r1 – r2| then one circle lies completely inside the other circle.

(b) Two intersecting circles are said to cut each other orthogonally if the angle between the circles is a right angle.
Let the circles be S = x2 + y2 + 2gx + 2fy + c = 0, S’ = x2 + y2 + 2g’x + 2f’y + c’ = 0.
And let d be the distance between the centers of two intersecting circles with radii r1, r2. The two cir-
cles will intersect orthogonally if and only if

(i) D2 = and
(ii) 2g g’ + 2f f’ = c + c’.
9 .42 | Circle

FORMULAE SHEET

 1. General equation of a circle: x2 + y2 + 2gx +2fy + c = 0


(i) Centre of the circle = (–g, –f).
1 1
g= coefficient of x, and f = coefficient of y.
2 2

(ii) r = g2 + f 2 − c

2 ax2 + 2hxy + by2 + 2gx + 2fy + c = 0 represents a circle if

(i) a = b ≠ 0 (ii) h = 0 (iii) ∆ = abc + 2hgf – af2 – bg2 – ch2 ≠ 0 (iv) g2 + f2– c ≥ 0

 3. if centre of circle is (h, k) and radius ‘r’ then equation of circle is: (x – h)2 + (y – k)2 = r2

 4. The equation of the circle drawn on the straight line joining two given points (x1 , y1) and (x2 , y2) as diameter
is : (x – x1) (x – x2) + (y – y1) (y – y2) = 0

2 2
 x + x 2 y1 + y 2   x2 − x1   y 2 − y1 
Centre :  1 .r=  + 
 2 , 2   
   2   2 

 5. (i) In parametric form:

x=–g+ (g 2
)
+ f 2 − c cos θ and y = –f + (g 2
)
+ f 2 − c sin θ , (0 ≤ θ < 2π)

 6. (i) Circle passing through three non-collinear points


x2 + y 2 x y 1
x12 + y12 x1 y1 1
A(x1 , y1), B(x2, y2), C(x3 ,y3) is represented by =0
x22 + y 22 x2 y2 1
x32 + y 32 x3 y3 1

 7. Circle circumscribing the triangle formed by the lines


a12 + b12
a1 b1
a1 x + b1 y + c1
a22 + b22
ai x + bi y + ci = 0 ( i= 1,2,3) : a2 b2 = 0
a2 x + b2 y + c2
a32 + b32
a3 b3
a3 x + b3 y + c3

 8. Intercepts length made by the circle On X and Y axes are 2 g2 − c and 2 f 2 − c respectively.

 9. Position of point (x1, y1) lies outside, on or inside a circle S ≡ x2 + y2 + 2gx + 2fy + c = 0.
When S1 ≡ x12 + y12 + 2gx + 2fy + c > = < 0 respectively.

 10. The power of P(x1, y1) w.r.t. S = x2 + y2 + 2gx + 2fy + c = 0 is equal to PA. PB which is S1 = x12 + y12 + 2gx1 + 2fy2 + c.
PA. PB = PC.PD = PT2 = square of the length of a tangent
M a them a ti cs | 9.43

 11. Intercept length cut off from the line y =mx + c by the circle x + y = a is 2 2 2 2
( )
a2 1 + m2 − c2
2
1+m
 12. The equation of tangent at (x1, y1) to circle x2 + y2 + 2gx + 2fy + c = 0 is xx1 + yy1 + g(x +x1) + f(y + y1) + c = 0.

 13. The equation of tangent at (a cos θ , a sin θ) is x cos θ + y sin θ = a

 14. Condition for tangency:


line y = mx + c is tangent of the circle x2 + y2 = a2 if c2 = a2(1 + m2)

  ma ±a 
and the point of contact of tangent y = mx ± a 1 + m2 is  , 
 2 
 1+m 1 + m2 

 15. The length of the tangent from a point P(x1,y1) to the circle S = x2 + y2 +2gx + 2fy + c = 0 is equal to

x12 + y12 + 2gx1 + 2fy1 + c

 16. Pair of tangent from point (0, 0) to the circle are at right angles if g2 + f2 = 2c.

 17. Equation of director circle of the circle x2 + y2 = a2 is equal to x2 + y2 = 2a2 .

 18. Equation of Director circle of circle x2 + y2 + 2gx + 2fy + c = 0 is


x2 + y2 + 2gx + 2fy + 2c – g2 – f2 = 0.
x y
 19. The equation of normal at any point (x1, y1) to the circle x2 + y2 = a2 is xy1 – x1y = 0 or = .
x1 y1
 20. Equation of normal at (a cosθ , a sinθ) is y = x tan θ or y = mx .

 21. The equation of the chord of contact of tangents drawn from a point (x1, y1)
to the circle x2 + y2 = a2 is xx1 + yy2 = a2. And
to the circle x2+ y2 + 2gx + 2fy + c = 0 is xx1 + yy1 + g(x + x1) + f(y + y1) + c = 0.
3

 22. Area of ∆ APQ is given by


(
a x12 + y12 − a2 ) 2
.=
RL3
. Where L & R are length of tangent and radius of circle.
x12 + y12 R 2 + L2
P
L

A R

 23. The equation of the chord of the circle x2 + y2 + 2gx + 2fy + c = 0. Bisected at the point (x1, y1) is given T = S1.
i.e., xx1 + yy1 + g(x + x1) + f(y + y1) + c = x12 + y12 + 2gx1 + 2fy1 + c.

 24. The equation of the common chord of two circles x2 + y2 + 2g1x + 2f1y + c1 = 0 and x2 + y2 + 2g2x + 2f2y + c2 = 0
is equal to 2x(g1 – g2) + 2y(f1 – f2) + c1 – c2 = 0 i.e., S1 – S2 = 0.

2 2
 25. Length of the common chord : PQ = 2(PM) = 2 C1P − C1M . Where,
C1P = radius of the circle S1 = 0
C1M =perpendicular length from the centre C1 to the common chord PQ.
9 .44 | Circle

 26. Equation of polar of the circle x2 + y2 + 2gx + 2fy + c = 0 and x2 + y2 = a2


w.r.t. (x1, y1) is xx1 + yy1+ g(x + x1) + f(y + y1) + c = 0 and xx1 + yy1 – a2 = 0. Respectively.

 a2l a2m 
 27. The pole of the line lx + my + n = 0 with respect to the circle x2 + y2 = a2:  − ,− 
 n n 

 28. P (x1, y1) and Q(x2, y2) are conjugate points of the circle x2 + y 2 + 2gx + 2fy + c =0
When x1 x2 + y1 y 2 + g(x1 + x2 ) + f(y1 + y 2 ) + c =.
0
If P and Q are conjugate points w.r.t. a circle with centre at O and radius r then PQ2 = OP2 + OQ – 2r2.

 29. The points P and T are a intersection point of direct common tangents and transverse. Common tangents
respectively, and it divide line joining the centres of the circles externally and internally respectively in the
ratio of their radii.
C1P r1
= (externally)
C2P r2
C1 T r1
= (internally)
C2 T r2
Hence, the ordinates of P and T are.

r x −r x r y −r y   r1 x2 + r2 x1 r1 y 2 + r2 y1 
P ≡  1 2 2 1 , 1 2 2 1  and T ≡  , 
 r1 − r2 r1 − r2   r1 + r2 r1 + r2 

 30. If two circles S ≡ x2+ y2 + 2g1x + 2f1y + c1 = 0 and S’ ≡ x2+ y2 + 2g2x + 2f2y + c2 = 0 of r1 , r2 and d be the distance
between their centres then the angle of intersection θ between them is given by

r12 + r22 − d2 2 ( g1g2 + f1 f2 ) − ( c1 + c2 )


(
cos 180 − θ = ) or cos = (180 − θ ) = .
2r1r2 2 g12 + f12 − c1 g12 + f12 − c2

 31. Condition for orthogonality: 2g1g2 + 2f1f2 = c1 + c2

 32. S1 – S2 = 0 the equation of the radical axis of the two circle. i.e. 2x (g1 – g2) + 2y (f1 – f2) + c1 – c2 = 0 which is
a straight line.

 33. The two limiting points of the given co-axial system are ( c , 0) and ( − c , 0).

 34. If two limiting points of a coaxial system of circles is (a, b) and (α, β).
then S1 + λS2 = 0, λ ≠ –1. or, {(x – a)2 + (y – b)2} + λ{(x – α)2 + (y – β)2} = 0, λ ≠ –1 is the
Coaxial system of circle.

 35. If origin is a limiting point of the coaxial system containing the circle x2 + y2 +2gx + 2fy + c = 0 then the other
 −gc −fc 
limiting point is  , .
 g2 + f 2 g2 + f 2 
 
M a them a ti cs | 9.45

Solved Examples

JEE Main/Boards Required circle is x2 + y2 – 6x – 6y + 9 = 0

Example 3: Lines 5x + 12y –10 = 0 and 5x – 12y – 40 =


Example 1: The given curves ax2 + 2hxy + by2 + 2gx + 0 touch a circle C1 (of diameter 6). If centre of C1 lies in
2fy + c = 0 and Ax2 + 2Hxy + By2 + 2Gx + 2Fy + c = 0 the first quadrant, find concentric circle C2 which cuts
intersect each other at four concyclic points then prove intercepts of length 8 units on each given line.
a−b A −B
that = .
h H Sol: Consider centre of required circle is (h, k) and by
using perpendicular distance formula from centre to
Sol: Equation of second degree curve passing through
given tangent we will get value of h and k.
the intersections of the given curves is S1 + λS2 = 0
Let centre of circle C1 be O (h, k), where h > 0 and k > 0
ax2 + 2hxy + by2 + 2gx + 2fy + c + λ(Ax2 + 2Hxy + By2
+ 2Gx + Fy + C) = 0  ….. (i) WP = 4 and OP = 3 … (given)
intersection points of of the two curves are concyclic, In OWP,
(i) must be a circle for some λ .
∴ Coefficient of x2 = coefficient of y2 and coefficient of OW = OP2 + WP2 = 42 + 32 = 5
xy = 0.
OW = 5 = radius of C2.
∴ a + λA = b + λB 5h + 12k − 10 5h − 12k − 40
⇒ = =3
and 2h + λ ⋅ 2H = 0 or a – b = λ(B – A) 13 13
and h = –λH
a − b λ (B − A ) a−b A −B Z
∴ = ; ∴ = . Q
h −λH h H
Y

Example 2: Find the equation of a circle which cuts the C1 C2


W O(h,k)
circle x2 + y2 – 6x +4y – 3 = 0 orthogonally and which
passes though (3, 0) and touches the y-axis. P

Sol: When two circle intersects each other orthogonally


X
then 2(g1g2 + f1f2) = c1 + c2. Hence by considering
centre as (h, k) and using given condition we can solve
problem. 5h + 12k − 10  5h − 12k − 40 
= ± 
13  13 
Let C(h, k) be the centre of required circle
⇒ Either 5h + 12 k – 10 = 5h – 12 k – 40
(h − 3 )
2
radius of circle = + k 2 = |h| ⇒ 24 k = - 30
∴ (h – 3) + k = h
2 2 2
−30
⇒k= (Not possible)
or k – 6h + 9 = 0 
2
…..(i) 40
Required circle is (x – h)2 + (y – k)2 = h2 Or 5h + 12 k – 10 = - 5h + 12 k + 40

or x2 + y2 – 2hx – 2ky + k2 = 0 ⇒ 10 h = 50

It is intersected by x2 + y2 – 6x + 4y – 3 = 0 , orthogonally; ⇒ H=5

∴ 2(–3)(–h) + 2(2)(–k) = k2 – 3 Substituting h = 5 in

or 6h – 4k + 3 = k2  …..(ii) 5h + 12k − 10
=3
13
Solve (i) and (ii) : h = 3, k = 3
9 .46 | Circle

⇒ k = 2 (as k > 0) Example 5: From a point P tangents are drawn to


circles x2 + y2 + x – 3 = 0,
Hence, equation of required circle is
(x – 5)2 + (y – 2)2 = 25 5
x2 + y2 –   x + y = 0 and 4x2 + 4y2 + 8x + 7y + 9 = 0,
3
Example 4: Find the locus of the middle points of the and they are of equal lengths. Find equation of a circle
chords of the circle x2 + y2 = a2 which pass through a passing through P and touching the line x + y = 5 at
given point (x1, y1). A(6, –1).

Sol: As line joining centre of given circle to the mid


Sol: By reading the problem we get that P is a radical
point of chord is perpendicular to the chord and
centre of these circles. Hence by radical axis formula
hence product of their slope will be – 1. Therefore by
we can obtain co-ordinate of point P, as required circle
considering mid point of chord as (α, β) and by finding
is passing from these points so we can obtain required
their slope we will get required equation.
equation.
Let M (α, β) be the middle point of any chord PQ
Write third circle as
through the given point (x1, y1). The centre of the circle
is O (0, 0). Clearly MO is perpendicular to PQ. 7 9
x2 + y2 + 2x +   y +   = 0
4
  4
By definition, P is radical centre of three circles. Equation
of two of the radical axis are
O(0,0) 8 7  21 
  x – y – 3 = 0 and x +   y +   = 0
3 4
   4 
which intersect at P(0, –3). Let required circle be
M(,) Q
P(x1,y1) x2 + y2 + 2gx + 2fy + c = 0

β − y1 with centre Q(–g, –f )


Now, slope of PQ =
α − x1 P(0, –3) lies on it
β−0 β ⇒ –6f + c + 9 = 0  … (i)
slope of OM = =
α−0 α A(6, –1) lies on it
β − y1 β
∴ · = –1 ⇒ 12g – 2f + c + 37 = 0 … (ii)
α − x1 α
Since, PA is perpendicular to x + y = 5
or α(α – x1) + β(β – y1) = 0
 −f + 1 
∴ the equation of the locus of M (α, β) is ∴   ( −1 ) =−1  … (iii)
 −g − 6 
x(x – x1) + y(y – y1) = 0
⇒ f–g=7
Alternative Solving (i), (ii) and (iii) for f, g and c, we have
The equation of chord when mid-point is known is 7 7
f = , g = – and c = 12.
T = S1 2 2
Hence equation of required circle is
Let the mid-point be ( α, β) x2 + y2 – 7x + 7y + 12 = 0.
∴ x α + y β − a2 = α2 + β2 − a2
∵ It passes through (x1, y1) we get Example 6: Find the equation of a circle which touches
the line x + y = 5 at the point P(–2, 7) and cut the circle
x, α + y , β = α2 + β2
x2 + y2 + 4x –6y + 9 = 0 orthogonally.
⇒ α ( α − x1 ) + β ( β − y1 ) = 0

∴ Required locus is Sol: Using the concept of family of circle and the
condition for two circles to be orthogonal, we can find
x ( x − x1 ) + y ( y − y1 ) =
0 the equation of the required circle.
As the circle is touching the line x + y = 5. It (-2, 7).
M a them a ti cs | 9.47

Consider the equation of circle as Sol: As polar of point (x1, y1) with respect to the circle x2
x y
( x + 2) + ( y + 7 ) + λ ( x + y − 5) =0
2 2
+ y2 = c2 is same as line + = 1.
a b
⇒ x2 + y 2 + x ( 4 + λ ) + y ( λ − 14 ) + 53 − 5λ = 0 ….(i) On comparing the two equations, we can prove the
 given statement.
∴ As the circle given equation (i) is orthogonal to
Let the pole be (x1, y1). Then the polar of (x1,y1) with
x2 + y 2 + 4x − 6y + 9 =0, respect to the circle x2 + y2 = c2 is
We have xx1 + yy1 = c2 … (i)
( 4 + λ ) .2 + ( λ − 14 )( −3=) 53 − 5λ + 9 x y
Now, the line (i) and + = 1 must be the same line.
⇒ 8 + 2λ − 3λ + 42= 62 − 5λ a b
⇒ 4λ =12 x1 y1 c2
∴ comparing coefficients, = =
1/a 1/b 1
⇒λ =3
or ax
= 1 by
= 1 c2 ,
∴ Equation of the circle is x + y + 7x − 11y + 38 =
2 2
0.
∴ ax1 = by1

Example 7: Find the equation of the circle described on ∴ (x1 , y1) always lies on the line ax = by which is a fixed
the common chord of the circles x2 + y2 - 4x – 5 = 0 and line.

x2 + y2 + 8y + 7 = 0 as diameter.
Example 9: Inside the circle x2 + y2 = a2 is inscribed
Sol: Use Geometry to find the centre and the radius of an equilateral triangle with the vertex at (a , 0). The
the required circle. equation of the side opposite to this vertex is

For x2 + y 2 − 4x − 5 = (A) 2x – a = 0 (B) x + a = 0


0
(C) 2x + a = 0 (D) 3x – 2a = 0
Centre ≡ ( +2, 0 )
Radius = 3 Sol: (C) As P (a , 0) be the vertex of the equilateral
triangles PQR inscribed in the circle x2 + y2 = a2 Let
For x2 + y 2 + 8y + 7 =0 M be the middle point of the side QR, then MOP is
Centre ≡ ( 0, − 4 ) C perpendicular to QR and O being the centroid of the
triangle OP = 2 (OM).
Radius = 3
(Circumcentre and Centroid of an equilateral triangle
A M B are same)
(2,0) (0,-4)
So if (h, k) be the coordinates of M, then
Y

The mid point of AB is the centre of the required circle


i.e. M ≡ (1, − 2 )
( 2a , 23 a) Q

and Radius
= AC2 − AM2
= 9 −5

=2
1
X
M O P(a,o)

Equation of circle is ( x − 1 ) + ( y − 2 ) =
2 2
4.

Example 8: Prove that, for all c ∈ R, the pole of the line


R
x y ( -a2 , - 23 a)
+ = 1 with respect to the circle x2 + y2 = c2 lies on
a b
a fixed line. 1
Y
9 .48 | Circle

2h + a 2k + 0 Sol: As we know Equation of any tangent to x2 + y2 = 1,


= 0 and =0
3 3 is y = mx ± 1 + m2 and perpendicular distance from
a centre to tangent is equal to its radius. By using this
⇒ h = −   and k = 0
2 condition we can solve above problem.

and hence the equation of BC is As they are drawn from A(–2, 0), conditions are
−2m ± 1 + m2
0=
a
x = – or 2x + a = 0. 1
2 ⇒m=±
3
Example 10 : Find the radical centre of the three circles Equations of tangents become
x2 + y2 = a2, (x – c)2 + y2 = a2 and x2 + (y – b)2 = a2.
T1 : 3y= x + 2
Sol: Here by using the formula
T2 : 3y =−x − 2
S 1 – S2 = 0 , S 2 − S 3 =
0 and S3 − S1 =
0
Circles touching C and having T1 and T2 as tangents
we will get equation of radical axis and by solving them must have their center on x-axis (the angle bisector of
we can obtain requird radical centre. T1 and T2).
Radical axis of first & second circle is given by Let C1 and C2 be the 2 circles and M (h1, 0) & L(h2, 0) be
(x + y ) – (x + y – 2cx + c ) = 0
2 2 2 2 2 their respective centers where

c h1 > 0 and h2 < 0


or x=
2 By tangency of T1, perpendicular distance from centre
Also the radical axis of first and third circle is given M is equal to radius r1 of the circle C1
by h1 + 2
∴ r1 =
(x + y ) – (x + y – 2by + b ) = 0
2 2 2 2 2 2
b As C1 and C touch each other r1 = h1 – 1
or y =
2
c b Y
⇒ The radical centre =  ,  .
2 2
T1
C2
JEE Advanced/Boards
X’ C1 (0,0)
X
(-2,0)
Example 1: Two distinct chords drawn from the point
P(a, b) to the circle x2 + y2 – ax – by = 0, (ab ≠ 0), are
bisected by the x-axis. Show that a2 > 8b2. T2

Sol: As Circle passes through (0, 0) and P(a, b) .Consider


the chord PQ intersect x-axis at A; then, Q is (α, –b). Y’
Hence by substituting this point to given equation of h1 + 2
or + 1 = h1
circle we can solve above problem. 2
∴ α2 + b2 – aα + b2 = 0 or α2 – aα + 2b2 = 0 or h1 = 4

Hence, Discriminant > 0 ∴ For circle C1 : centre is M(4, 0) and radius = 3.

⇒ a2 > 8b2 h2 + 2
Similarly for circle C2, –h2 – 1 =
2
Example 2: Let T1, T2 be two tangents drawn from (–2 , 0) ⇒ –2h2 – 2 = h2 + 2
to the circle C : x2 + y2 = 1. Determine circles touching C
and having T1, T2 as their pair of tangents. Further find (∴ h2 > –2; see figure)
the equation of all possible common tangents to these ⇒ –3h2 = 4
circles, when taken two at time.
4 1
or h2 = – and radius = .
3 3
M a them a ti cs | 9.49

Equations of two circles are (x – 4)2 + y2 = 9 and 2


 2r  r2
 4
2
1 or x12 +  y1 +  =
2  3  9
x +  + y = 
 3 9
C1 & C have x = 1 as transverse common tangent and ∴ Locus of (x1, y1) is a circle.
C2 & C have x = –1 as transverse common tangent.
Example 4: Derive the equation of the circle passing
Example 3 : Let AB be a chord of the circle x2 + y2 = r2 through the point P(2, 8) and touches the lines 4x – 3y
subtending a right angle at the centre O. Show that the – 24 = 0 and 4x + 3y – 42 = 0 and coordinates of the
centroid of the triangle PAB as P moves on the circle is centre less than or equal to 8.
a circle.
Sol: Here using Equations of bisectors of angle between
Sol: By considering point P(r cos θ , r sinθ) and centroid the lines we will get co-ordinate of centre of circle i.e
as point (x1, y1) we can obtain required result. O. and as OA = OP we can obtain required equation of
circle. consider O is the center of circle.
∆OAB is isosceles with
Let L1 ≡ 4x – 3y – 24 = 0
OA = OB = x(say)
L2 ≡ 4x + 3y – 42 = 0
We may assume AB is parallel to and below x-axis
and Let A and B denote the respective points of contact
r
∴ x2 + x2 = r2 ⇒ x =
2 Equations of bisectors of angle between the lines are;
 r r   r r 
∴ B is  ,−  and A is  − ,− 
 2 2  2 2 -24=
0
A 4x-3y
Let P be (r cos θ , r sinθ) and centroid of ∆PAB be G (x1, y1) P(2,8)
Y
P
C O

A
r
B
X’ X 4x+3y
O - 42=0
4x − 3y − 24 4x + 3y − 42

5 5
33
B i.e., y = 3 & x=
4
Y’ Since O lies on one of these bisectors and x-coordinate
of O is less then or equal to 8,
r r
r cos θ + − ∴ O lies on y = 3.
2 2,
∴ x1 = Let O be (a,3). Then, OA = CP
3
2
r r  4a − 33 
 =( a − 2 ) + 25
2
r sin θ − − or 
y1 = 2 2  5 
3 or 16a2 – 264a + (33)2 = 25{a2 – 4a + 29}
3x1 = r cos θ ; 3y1 = r(sin θ – 2) or 9a2 + 164a – 364 = 0
Eliminating θ, we get or (a – 2)(9a + 182) = 0
2
 3x1   3y1 
2 182
∴ a =2 or a = −
∴  r  +  r + 2  =
1 9
   
and radius = OP.
9 .50 | Circle

Example 5: Coordinates of a diagonal of a rectangle Sol: The condition for one circle to be within the other is
are (0, 0) and (4, 3). Find the equations of the tangents
C1C2 < r1 − r2
to the circumcircle of the rectangle which are parallel to
this diagonal. Without the loss of generality,

Sol: Here centre of circle is the mid-point of line OP Let λ > µ


hence by using slope point form we can get required
λ−µ λ2 µ2
equation of tangents. ∴ C1 C2 < r1 − r2 ⇒  < −c − −c
 2  4 4
Two extremities are O (0, 0) and P (4, 3). Middle point of λ 2 µ2 λ µ λ2 µ2
⇒ + −2× × < −c+ −c
 3 4 4 2 2 4 4
the diagonal OP is M  2,  which is the centre of the
 2  λ2   µ2 
9 5 −2  − c − c
circumscribed circle and radius is OM = 4 + =  4  4 
4 2   

 λ2   µ2  λµ
⇒2  − c − c  < 2. − 2c
 4  4  4
  

λ 2 µ2  λ 2 µ2  2 λ 2 µ2
− c + +c < + c2 −
16  4 4  16
P(4,3)  
λµ
2×c×
4
 λ 2 µ2 λ µ
c + − 2× ×  > 0
 4 4 2 2 
O(0,0) 
2
λ µ
c −  > 0
2 2
⇒C>0
Also ∴ λ > µ
3  −µ 
A line parallel to OP is y = x+c  , 0  will be inside
4  2 
It is a tangent to the circumscribed circle.
x 2 + y 2 + λx + c = 0
Therefore length of perpendicular from
µ2 µλ
3 ⇒ +0− +c < 0
 3 5 4
( 2) − 23 + C 5 4 2
M  2,  to it = ⇒ = µ2
 +c>0
 2 2 9 2 4
1+
16 λ µ µ2
5 5 25 ∴ > +c
or C =± ⋅ =± 2 4
2 4 8
λµ
3 25 ∴ >0
Hence tangents are y = x± 2
4 8
⇒ λµ>0
25
or 3x – 4y ± = 0. Hence, proved.
2

Example 6: The equations two circles are Example 7: A circle touches the line y = x at a point P
x + y + λx + c = 0 and x + y + µx + c = 0. Prove that
2 2 2 2 such that OP = 4 2 where O is the origin. The circle
one of the circles will be within the other if λµ > 0 and contains the point (–10, 2) in its interior and the length
c > 0. of its chord on the line x + y = 0 is 6 2 . Find the
equation of the circle.
M a them a ti cs | 9.51

Sol: In this question, the concept of rotation of axes Example 8: Derive the equation of the circle passing
would be useful. through the centres of the three given circles x2 + y2 –
4y – 5 = 0,
Let the new co-ordinate axis be rotated by an angle of
45° in the clockwise direction. Then x2 + y2 + 12x + 4y +31 = 0 and
=X x cos ( θ) + y sin ( θ ) x2 + y2 + 8x + 10y + 32 = 0.
Y =−x sin ( θ ) + y cos ( θ )
Sol: Find the relation between the centres of the circle
Where θ= 45° and there use the appropriate form of circle.
x−y Let P, Q and R denote the centres of the given circle
∴ X =
2 P ≡ ( 0, 2 ) , Q =( −6, − 2 ) and
x+y
Y =
2
R≡ ( −4, − 5)
The image after rotation would be −2 − 2 −4 2
∴ mPQ = = =
−6 − 0 −6 3
−5 + 2 −3
mQR
= =
−4 + 6 2
2 −3
∴ mPQ . mQR =× −1
=
C1 (0, 4 2) 3 2
⇒ PQ is perpendicular to QR
A
B
∴ Using diameter form, we get
3 2 (x - 0) (x + 4) + (y - 2) (y + 5) =0
C2 (0, -4 2)

Example 9: Area of Quadrilateral PQRS is 18, side PQ


|| RS and PQ = 2RS and PS ⊥ PQ and RS. then radius a
circle drawn inside the quadrilateral PQRS touching all
the sides is,
In ∆ ABC, AC, =
4 2 3
(A) 3 (B) 2 (C) (D) 1
AB = 3 2 2
Sol: (B) Let r be the radius of the circle, then PS = 2r.
( ) ( )
2 2
∴ Radius = 4 2 + 3 2 b 2
= Let P be the origin and PQ and PS as x-axis and y-axis
respectively.
∴ Equation of the circle is
∴ The coordinates of P, Q, R, S are (0, 0), (2a, 0), (a, 2r)
(X + 5 2 ) + (Y  4 2 ) =
(5 2 )
2 2 2
and (0, 2r) respectively.
1
2 2 ∴ Area (PQRS) =   (a + 2a)(2r) = 18
x−y
Or, 
 2
 x+y
+5 2 + 
  2

 4 2 =

5 2 ( ) 2
⇒ ar = 6.
or, ( x − y + 10 ) + ( x + y ± 8 ) =
2 2
100 S(0,2r) R(a,2r)

But, since (-10, 2) lies inside the circle.


r
The equation of the circle is
(r,r)
( x − y + 10 ) + ( x + y + 8 )
2 2
=100
Or, x2 + y 2 + 100 − 2xy − 20y + 20x
P(0,0) Q(2a,0)
+ x2 + y 2 + 64 + 2xy + 16y + 16x =
100
∴ Equation of QR is
Or, 2x2 + 2y 2 + 36x − 4y + 64 =
0
 y 2 − y1 
Or, x2 + y 2 + 18x − 2y + 32 =
0 (=
y − y1 )   ( x − x1 )
 x2 − x1 
9 .52 | Circle

 0 − 2r  Similarly, equation of BQ is
⇒ (=
y − 2r )   ( x − a)
 2a − a  asin β − 0
(y − v)
=
a cos β − a
( x − a)
−2r
⇒ ( y − 2r
= ) ( x − a)
a β β
2 a.sin
.cos
⇒ ay − 2ar =
−2rx + 2ar 2 2 x−a
=⇒y
β
( )
⇒ 2rx + ay − 4ar =
0 −a × 2 sin
2
∴ QR is a tangent to the circle β
⇒y=− cot   ( x − a)  …(ii)
2r 2 + ar − 4ar 2
∴ r
=
Now, we eliminate α , β using (i) and (ii)
4r 2 + a2
 α − β =2r
r ( 2r − 3a)
⇒ r
= α β
2 2 ⇒ − = r
4r + a 2 2
α β
⇒ ( 2r − 3a) = 4r 2 + a2
2
⇒ tan  −  =tan r
 2 2
⇒ 4r 2 + 9 a2 − 12 ar = 4r 2 + a2
α β
2
tan − tan
⇒8a =
12 ar ⇒ 2 2 = tan r
α β
⇒ 2a2 =
3 ar 1 + tan . tan
2 2
y a−x
⇒ 2a2 =×
3 6 −
x+a y
⇒a =3 ⇒ tan γ
=
y a−x
1+ ×
∴ r 2 (=
=  ar 6 ) a+ x y

y 2 − a2 + x2
Example 10: A circle having centre at (0, 0) and radius ⇒ tan γ
=
ay + xy + ay − xy
equal to 'a' meets the x - axis at P and Q. A(α) and B(β)
are points on this circle such that α – β = 2γ, where γ ⇒ x2 + y 2 − 2ay tan γ − a2 =0
is a constant. Then locus of the point of intersection of
PA and QB is a a-r
(A) x – y – 2ay tan γ = a
2 2 2 

(B) x2 + y2 – 2ay tan γ = a2 r

(C) x2 + y2 + 2ay tan γ = a2


(D) x2 – y2 + 2ay tan γ = a2 r

Sol: (B) Let the equation of the circle be x2 + y 2 =
a2 

∴ P ≡ ( −a, 0 ) and Q = ( a, 0 )
∴ Equation of PA is
a sin α − 0
( y − 0)
=
acos α + a
( x + a)
a sin α
=⇒y
a ( cos α + 1 )
( x + a)
α α
.cos a.2 sin
2 2 x+a
=⇒y ( )
2 α
a.2 cos
2
α
⇒ y tan
=
2
x+a  ( ) …(i)
M a them a ti cs | 9.53

JEE Main/Boards

Exercise 1 Q.15 Show that the line x + y = 2 touches the circles


x2 + y2 = 2 and x2 + y2 + 3x + 3y – 8 = 0 at the point
Q.1 Find the equation of the circle whose centre lies on where the two circles touch each other.
the line 2x – y – 3 = 0 and which passes through the
Q.16 One of the diameters of the circle circumscribing
points (3, –2) and (–2, 0).
the rectangle ABCD is 4y = x + 7. If A and B are the
Q.2 Show that four points (0, 0), (1, 1), (5, –5) and (6, –4) points (–3, 4) and (5, 4) respectively, find the area of the
are concylic. rectangle.

Q.3 Find the centre, the radius and the equation of the Q.17 A circle of radius 2 lies in the first quadrant
circle drawn on the line joining A(–1, 2) and B(3, –4) as and touches both the axes of co-ordinates, Find the
diameter. equation of the circle with centre at (6, 5) and touching
the above circle externally.
Q.4 Find the equation of the tangent and the normal to
the circle x2 + y2 = 25 at the point P(–3, –4).  1 
Q.18 If  mi ,  ; i = 1, 2, 3, 4 are four distinct point on
 mi 
Q.5 Show that the tangent to x2 + y2 = 5 at (1, –2) also
a circle, show that m1m2m3m4 = 1.
touches the circle x2 + y2 – 8x + 6y + 20 = 0

Q.6 Find the equation of the tangents to the circle x2 + Q.19 Show that the circle on the chord
y2 – 2x + 8y = 23 drawn from an external point (8, –3). xcosα + ysinα – p = 0 of the circle x2 + y2 = a2 as
diameter is x2 + y2 – a2 – 2p (xcosα + y sin α – p) = 0.
Q.7 Find the equation of the circle whose centre is
(–4, 2) and having the line x – y = 3 as a tangent Q.20 Find the length of the chord of the circle x2 + y2 = 16
which bisects the line joining the points (2, 3) and (1, 2)
Q.8 Find the equation of the circle through the points perpendicularly.
of intersections of two given circles
Q.21 Find the angle that the chord of circle x2 + y2 – 4y=0
x2 + y2 – 8x – 2y + 7 = 0 and along the line x + y = 1 subtends at the circumference
x2 + y2 – 4x + 10y + 8 = 0 and passing through (3, –3). of the larger segment.

Q.9 Find the equation of chord of the circle x2 + y2 – 4x Q.22 Prove that the equation x2 + y2 – 2x – 2λ y – 8=0,
= 0 which is bisected at the point (1, 1). where λ is a parameter, represents a family of circles
passing through two fixed points A and B on the x-axis.
Q.10 Find the equation of chord of contact of the circle Also find the equation of that circle of the family,
x2 + y2 – 4x = 0 with respect to the point (6, 0). the tangents to which at A and B meet on the line
x + 2y + 5 = 0.
Q.11 Find the length of the tangent drawn from the
point (3, 2) to the circle 4x2 + 4y2 + 4x + 16y + 13 = 0. Q.23 Find the area of the quadrilateral formed by
a pair of tangents from the point (4, 5) to the circle
Q.12 Obtain the equations of common tangents of the x2 + y2 – 4x – 2y – 11 = 0 and a pair of its radii.
circles x2 + y2 = 9 and x2 + y2 – 12x + 27 = 0.
Q.24 If the lines a1x + b1y + c1=0 and a2x + b2y + c2=0
Q.13 The centres of the circle passing through the cut the co-ordinate axes in concyclic points, prove that
points (0, 0), (1, 0) and touching the circle x2 + y2 = 9 a1a2 = b1b2.
1 
are  , ± 2  . Q.25 Show that the length of the tangent from any
2 
point on the circle
Q.14 The abscissae of two points A and B are the roots
of the equation x2+ 2ax – b2 = 0 and their ordinates x2 + y2 + 2gx + 2fy + c = 0 to the circle
are the roots of the equation x2 + 2px – q2 = 0. Find x2 + y2 + 2gx + 2fy + c1 = 0 is c1 − c .
the equation and the radius of the circle with AB as
diameter.
9 .54 | Circle

Q.26 Find the point from which the tangents to the (A) Are the vertices of a right triangle
three circles x2 + y2 – 4x + 7 = 0,
(B) The vertices of an isosceles triangle which is not
2x + 2y – 3x + 5y + 9 = 0
2 2
regular
and x2 + y2 + y = 0 are equal in length. Find also this (C) Vertices of a regular triangle
length.
(D) Are collinear
Q.27 The chord of contact of tangents from a point on
the circle x2 + y2 = a2 to the circle x2 + y2 = b2 touches Q.2 2x2 + 2y2 + 2λ x + λ2 = 0 represents a circle for :
the circle x2 + y2 = c2. Show that a, b, c are in G.P. (A) Each real value of λ
Q.28 Obtain the equation of the circle orthogonal to (B) No real value of λ
both the circles (C) Positive λ
x2 + y2 + 3x – 5y + 6 = 0 and (D) Negative λ
4x + 4y – 28x + 29 = 0 and whose centre lies on the
2 2

line 3x + 4y + 1 = 0. Q.3 The area of an equilateral triangle inscribed in the


circle x2 + y2 – 2x = 0 is
Q.29 From the point A (0, 3) on the circle x2 + 4x +
(y – 3)2 = 0, a chord AB is drawn and extended to a 3 3 3 3
(A) (B)
point M such that AM = 2AB. Find the equation of the 4 2
locus of M.
3 3
(C) (D) None of these
Q.30 From the origin, chords are drawn to the circle 8
(x – 1)2 + y2 = 1. Find the equation to the locus of the
middle points of these chords. Q.4 A circle of radius 5 has its centre on the negative
x-axis and passes through the point (2, 3). The intercept
Q.31 Tangent at any point on the circle x2 + y2=a2 meets made by the circle on the y-axis is
the circle x2 + y2 = b2 at P and Q. Find the condition on a
(A)10 (B) 2 21
and b such that tangents at P and Q meet at right angles.
(C) 2 11 (D) imaginary y-intercept
Q.32 The tangent from a point to the circle x2 + y2 = 1
is perpendicular to the tangent from the same point to
Q.5 The radii of the circle x2 + y2 = 1, x2 + y2 – 2x – 6y = 6
the circle x2 + y2 = 3. Show that the locus of the point
and x2 + y2 – 4x – 12y = 9 are in
is a circle .
(A) A.P. (B) G.P.
Q.33 A variable circle passes through the point A (a, b)
(C) H.P. (D) None of these
and touches the x-axis. Show that the locus of the other
end of the diameter through A is (x – a)2 = 4 by.
Q.6 If the equation x2 + y2 + 2λx + 4 = 0 and x2 + y2 –
Q.34 AB is a diameter of a circle. CD is a chord parallel 4λy + 8 = 0 represent real circles then the value of λ
to AB and 2CD = AB. The tangent at B meets the line AC can be
(produced) at E. Prove that AE = 2AB. (A) 5 (B) 2
(C) 3 (D) All of these

Exercise 2
Q.7 The equation of the image of the circle x2 + y2 +16x
Single Correct Choice Type – 24y + 183 = 0 by the line mirror 4x + 7y + 13 = 0 is;
(A) x2 + y2 + 32x – 4y + 235 = 0
Q.1 Centres of the three circles
(B) x2 + y2 + 32x + 4y – 235 = 0
x2 + y2 – 4x – 6y – 14 = 0
(C) x2 + y2 + 32x – 4y – 235 = 0
x2 + y2 + 2x + 4y – 5 = 0
(D) x2 + y2 + 32x + 4y + 235 = 0
and x2 + y2 – 10x – 16y + 7 = 0
M a them a ti cs | 9.55

Q.8 The circle described on the line joining the points y= 4 and the x-axis is
(0, 1), (a, b) as diameter cuts the x-axis in points whose y
abscissae are roots of the equation : y=4
(A) x + ax + b = 0
2
(B) x – ax + b = 0
2
x/2
y=
(C) x2 + ax – b = 0 (C) x2 – ax – b = 0 x
O

Q.9 A straight line l1 with equation x – 2y + 10 = 0 8 5


meets the circle with equation x2 + y2 = 100 at B in the (A) 4 + 2 5 (B) 4 +
5
first quadrant. A line through B, perpendicular to l1 cuts
6 5
the y-axis at P (0, t). The value of ‘t’ is (C) 2 + (D) 8 + 2 5
5
(A) 12 (B) 15 (C) 20 (D) 25

Q.15 From the point A (0, 3) on the circle x2 + 4x +
 1  1  1  1 (y – 3)2 = 0 a chord AB is drawn and extended to a point
Q.10 If  a,  ,  b,  ,  c,  and  d,  are four distinct M such that AM = 2AB. The equation of the locus of M is,
 a   b   c   d
point on a circle of radius 4 units then, abcd is equal to (A) x2 + 8x + y2 = 0
1 (B) x2 + 8x + (y – 3)2 = 0
(A) 4 (B) (C) 1 (D) 16
4 (C) (x –3)2 + 8x + y2 = 0
Q.11 The radius of the circle passing through the (D) x2 + 8x + 8y2 = 0
vertices of the triangle ABC, is
A Q.16 If L1 and L2 are the length of the tangent from (0, 5)
to the circles x2 + y2 + 2x – 4 = 0 and x2 + y2 – y + 1 = 0
12 12 then
(A) L1 = 2L2 (B) L2 = 2L1 (C) L1 = L2 (D) L21 = L2
B 6 C
Q.17 The line 2x – y + 1 = 0 is tangent to the circle
at the point (2, 5) and the centre of the circles lies on
8 15 3 15 x–2y = 4. The radius of the circle is
(A) (B) (C) 3 15 (D) 3 2
5 5
(A) 3 5 (B) 5 3 (C) 2 5 (D) 5 2
Q.12 The points A(a, 0), B(0, b), C(c, 0) and D(0, d) are
such that ac = bd and a, b, c, d are all non-zero. Then Q.18 Coordinates of the centre of the circle which
the points bisects the circumferences of the circles x2 + y2 = 1 ; x2
(A) Form a parallelogram (B) Do not lie on a circle + y2 + 2x – 3 = 0 and x2 + y2 + 2y – 3 = 0 is

(C) Form a trapezium (D) Are concyclic (A) (–1, –1) (B) (3, 3) (C) (2, 2) (D) (–2, –2)

Q.19 The anglebetween the two tangents from the


Q.13 Four unit circles pass through the origin and origin to the circle (x –7)2 + (y + 1)2 = 25 equals
have their centres on the coordinate axes. The area π π π π
of the quadrilateral whose vertices are the points of (A) (B) (C) (D)
6 3 2 4
intersection (in pairs) of the circle, is
(A) 1 sq. unit Q.20 In a right triangle ABC, right angled at A, on
the leg AC as diameter, a semicircle is described. The
(B) 2 2 sq. units chord joining A with the point of intersection of the
(C) 4sq. units hypotenuse and the semicircle, then the length AC
equals to
(D) Cannot be uniquely determined, insufficient data
AB ⋅ AD AB ⋅ AD
(A) (B)
2
AB + AD 2 AB +AD
Q.14 The x-coordinate of the center of the circle in the
1 AB ⋅ AD
(C) AB ⋅ AD (D)
first quadrant (see figure) tangent to the lines y = x,
2 AB2 − AD2
9 .56 | Circle

Q.21 Locus of all point P (x, y) satisfying x3 + y3 + 3xy = Q.7 The triangle PQR is inscribed in the circle x2 + y2
1 consists of union of = 25. If Q and R have coordinates (3, 4) and (–4, 3)
respectively, then ∠ QPR is equal to  (2002)
(A) A line and an isolated point
π π π π
(B) A line pair and an isolated point (A) (B) (C) (D)
2 3 4 6
(C) A line and a circle
Q.8 The number of common tangents to the circles
(D) A circle and an isolated point.
x2 + y2 = 4 and x2 + y2 – 6x – 8y = 24 is  (1998)

(A) 0 (B) 1 (C) 3 (D) 4


Previous Years’ Questions
Q.9 Tangents are drawn from the point (17, 7) to the
Q.1 The circle passing through the point(–1, 0) and circle x2 + y2 = 169.  (2007)
touching the y-axis at (0, 2) also passes through the
point  (2011) (A) Statement-I is true, statement-II is true and
statement-II is correct explanation for statement-I.
 3   5   3 5
(A)  − ,0  (B)  − ,2  (C)  − ,  (D) (–1, –4) (B) Statement-I is true, statement-II is true and
 2   2   2 2
statement-II is NOT the correct explanation for statement-I.
Q.2 Consider the two curves C1 : y2 = 4x (C) Statement-I is true, statement-II is false.

C2 : x2 + y2 – 6x + 1 = 0, then  (2008) (D) Statement-I is false, statement-II is true.

(A) C1 and C2 touch each other only at one point Statement-I: The tangents are mutually perpendicular.
(B) C1 and C2 touch each other exactly at two points
Statement-II: The locus of the points from which a
(C) C1 and C2 intersect (but do not touch) at exactly two mutually perpendicular tangents can be drawn to the
points given circle is x2 + y2 = 338.
(D) C1 and C2 neither intersect nor touch each other
Q.10 Find the equation of circle touching the line
Q.3 If one of the diameters of the circle x + y – 2x – 6y
2 2 2x + 3y + 1 = 0 at the point (1, –1) and is orthogonal to
+ 6 = 0 is a chord to the circle with centre (2, 1), then the circle which has the line segment having end points
the radius of the circle is  (2004) (0, –1) and (–2, 3) as the diameter.  (2004)

(A) 3 (B) 2 (C) 3 (D) 2


Q.11 Let C1 and C2 be two circles with C2 lying inside
C1. A circle C lying inside C1 touches C1 internally and C2
Q.4 The centre of circle inscribed in square formed by externally. Identify the locus of the centre of C (2001)
the lines x2 – 8x + 12 = 0 and y2 – 14y + 45 = 0, is
 (2003)
Q.12 Consider the family of circles x2 + y2 = r2, 2< r < 5.
(A) (4, 7) (B) (7, 4) (C) (9, 4) (D) (4, 9) If in the first quadrant, the common tangent to a circle
of this family and the ellipse 4x2 + 25y2 = 100 meets the
Q.5 If the tangent at the point P on the circle x2 + y2 + coordinate axis at A and B, then find the equation of the
6x + 6y = 2 meets the straight line 5x – 2y + 6 = 0 at a locus of the mid points of AB. (1999)
point Q on the y-axis, then the length of PQ is  (2002)
(A) 4 (B) 2 5 (C) 5 (D) 3 5 Q.13 C1 and C2 are two concentric circle the radius of C2
being twice that of C1. From a point P on C2, tangents
PA and PB are drawn to C1. Prove that the centroid of
Q.6 If the circle x2 + y2 + 2x + 2ky + 6 = 0 and x2 + y2 + the triangle PAB lies on C1. (1998)
2ky + k = 0 intersect orthogonally, then k is (2000)
3 3 Q.14 The length of the diameter of the circle which
(A) 2 or – (B) –2 or –
2 2 louches the x-axis at the point (1,0 ) and passes
3 3 through the point ( 2,3) is  (2012)
(C) 2 or (D) –2 or
2 2
10 3 6 5
(A) (B) (C) (D)
3 5 5 3
M a them a ti cs | 9.57

Q.15 The circle through (1, ﹘2) and touching the axis of
x at (3,0) also passes through the point  (2013)
3 3 1 1
(A) (B) (C) (D)
(A) ( 2, −5 ) (B) (5, −2 ) (C) ( −2,5 ) (D) ( −5,2 ) 2 2 2 4

Q.18 The number of common tangents to circles


Q.16 The equation of the circle passing through the x2 + y 2 − 4x − 6y − 12 =
0 and
x2 y 2 x2 + y 2 + 6x − 18y + 26 =
0 , is  (2015)
foci of the ellipse + =1 , and having centre at
16 9
( ) is 
0,3
(2013) (A) 2 (B) 3 (C) 4 (D) 1

2 2 2 2
(A) x + y − 6y + 7 =0 (B) x + y − 6y − 5 =0 Q.19 The centres of those circles which touch the
2 2
(C) x + y − 6y + 5 =0 2 2
(D) x + y − 6y − 7 =0 circle, x2 + y 2 − 8x − 8y − 4 =
0 , externally and also
touch the x-axis, lie on: (2016)
Q.17 let C be the circle with centre at (1,1) and radius (A) An ellipse which is not a circle
=1. If T is the circle centred at ( 0, y ) , passing through
origin and touching the circle C externally, then radius (B) A hyperbola
of T is equal to  (2014)
(C) A parabola

(D) A circle

JEE Advanced/Boards

Exercise 1 2x2 + 2y2 + 3x + 8y + 2c = 0 touches the circle x2 + y2 +


3
Q.1 Let S : x2 + y2 – 8x – 6y + 24 = 0 be a circle and O is 2x – 2y + 1 = 0. Show that either g = or f = 2
4
the origin. Let OAB is the line intersecting the circle at
Q.6 Consider a family of circles passing through two
A and B. On the chord AB a point P is taken. The locus
fixed points A (3, 7) & B (6, 5). The chords in which the
of the point P in each of the following cases.
circle x2 + y2 – 4x – 6y + 3 = 0 cuts the members of the
(i) OP is the arithmetic mean of OA and OB family are concurrent at a point. Find the coordinates
(ii) OP is the geometric mean of OA and OB of this point.
(iii) OP is the harmonic mean between OA and OB Q.7 Find the equation of circle passing through (1, 1)
Q.2 A circle x2 + y2 + 4x – 2 2y + c = 0 is the director belonging to the system of co-axial circles that are
circle of circle S1 and S1 is the director circle of circle S2 tangent at (2, 2) to the locus of the point of intersection
and so on. If the sum of radii of all these circles is 2, of mutually perpendicular tangent to the circle x2 + y2 = 4.
then the value of c is equal to n where n ∈ N. Find
Q.8 The circle C : x2 + y2 + kx + (1 + k)y – (k + 1) = 0
the value of n.
passes through two fixed points for every real number
Q.3 If the circle x2 + y2 + 4x + 22y + a = 0 bisects the k. Find
circumference of the circle x2 + y2 – 2x + 8y – b = 0 (i) the coordinates of these points.
(where a, b > 0), then find the maximum value of (ab).
(ii) the minimum value of the radius of a circle C.
Q.4 Real number x, y satisfies x2 + y2 =1. If the maximum
4−y Q.9 Find the equation of a circle which is co-axial with
and minimum value of the expression z = are M
7−x circles 2x2 + 2y2 – 2x + 6y – 3 = 0 and
and m respectively, then find the value (2M + 6m). x2 + y2 + 4x + 2y + 1 = 0. It is given that the centre of
Q.5 The radical axis of the circle the circle to be determined lies on the radical axis of
these two circles.
x2 + y2 + 2gx + 2fy + c = 0 and
9 .58 | Circle

Q.10 Find the equation of the circle passing through the column-I, the ratio of b/a is
points of intersection of circles x2 + y2 – 4x – 6y – 12 = 0 and
x2 + y2 + 6x + 4y – 12 = 0 and cutting the circle x2 + y2 Column I Column II
– 2x – 4 = 0 orthogonally.
(A) C1 and C2 touch each other
(p) 2 + 2
Q.11 The centre of the circles S = 0 lie on line 2x – 2y
(B) C1 and C2 are orthogonal (q) 3
+ 9 = 0 & S = 0 cuts orthogonally the circle x2 + y2 = 4.
Show that circle S = 0 passes through two fixed points (C) C1 and C2 intersect so that
(r) 2 + 3
& find their coordinates. the common chord is longest
(D) C2 passes through the
Q.12 Find the equation of a circle passing through the (s) 3 + 2 2
centre of C1
origin if the line pair, xy – 3x + 2y – 6 = 0 is orthogonal
to it. If this circle is orthogonal to the circle x2 + y2 – kx (t) 3 – 2 2
+ 2ky – 8 = 0 then find the value of k.
Q.20 A circle with centre in the first quadrant is tangent
Q.13 Find the equation of the circle which cuts the
to y = x + 10, y = x – 6, and the y-axis. Let (h, k) be the
circle x2 + y2 – 14x – 8y + 64 = 0 and the coordinate
axes orthogonally. centre of the circle. If the value of (h + k) = a + b a
where a is a surd, find the value of a + b.
Q.14 Show that the locus of the centres of a circle
which cuts two given circles orthogonally is a straight Q.21 Circles C1 and C2 are externally tangent and they
line & hence deduce the locus of the centres of the are both internally tangent to the circle C3. The radii
circles which cut the circles x2 + y2 + 4x – 6y + 9 = 0 & of C1 and C2 are 4 and 10, respectively and the centres
x2 + y2 – 5x + 4y + 2 = 0 orthogonally. Intercept the locus. of the three circles are collinear. A chord of C3 is also
a common internal tangent of C1 and C2. Given that
Q.15 Find the equation of a circle which touches the m n
line x + y = 5 at the point (–2, 7) and cuts the circle the length of the chord is where m, n and p are
p
x2 + y2 + 4x – 6y + 9 = 0 orthogonally. positive integers, m and p are relatively prime and n is
not divisible by the square of any prime, find the value
Q.16 Find the equation of the circle passing through
of (m + n + p).
the point (–6, 0) if the power of the point (1, 1) w.r.t. the
circle is 5 and it cuts the circle x2 + y2 – 4x – 6y – 3 = 0 Q.22 Find the equation of the circle passing through
orthogonally. the three points (4, 7), (5, 6) and (1, 8). Also find the
coordinates of the point of intersection of the tangents
Q.17 As shown in the figure, the five circles are tangent
to the circle at the points where it is cut by the straight
to one another consecutively and to the lines L1 and
line 5x + y + 17 = 0.
L2. If the radius of the largest circle is 18 and that of
the smaller one is 8, then find the radius of the middle Q.23 The line 2x – 3y + 1 = 0 is tangent to a circle
circle. S = 0 at (1, 1). If the radius of the circle is 13 . Find the
equation of the circle S.
L1
Q.24 Find the equation of the circle which passes
through the point (1, 1) & which touches the circle
x2 + y2 + 4x – 6y – 3 = 0 at the point (2, 3) on it.
L2

Q.18 Find the equation of a circle which touches the line


7x2 – 18xy + 7y2 = 0 and the circle x2 + y2 – 8x – 8y = 0
and is contained in the given circle.

Q.19 Consider two circle C1 of radius ‘a’ and C2 of radius


‘b’ (b > a) both lying in the first quadrant and touching
the coordinate axes. In each of the conditions listed in
M a them a ti cs | 9.59

Exercise 2 Q.8 A rhombus is inscribed in the region common to


the two circles x2 + y2 – 4x – 12 = 0 and x2 + y2 + 4x –
Single Correct Choice Type 12 = 0 with two of its vertices on the line joining the
centres of the circles. The area of the rhombus is
Q.1 B and C are fixed points having co-ordinates
(3, 0) and (–3, 0) respectively. If the vertical angle BAC (A) 8 3 sq. units (B) 4 3 sq. units
is 90°, then the locus of the centroid of the ∆ABC has (C) 16 3 sq. units (D) None of these
the equation :
(A) x2 + y2 = 1 (B) x2 + y2 = 2 Q.9 From (3, 4) chords are drawn to the circle x2 + y2
– 4x = 0. The locus of the mid points of the chords is:
(C) 9(x2 + y2) = 1 (D) 9(x2 + y2) = 4
(A) x2 + y2 – 5x – 4y + 6 = 0
Q.2 Number of points in which the graphs of |y| = x + 1 (B) x2 + y2 + 5x – 4y + 6 = 0
and (x – 1)2 + y2 = 4 intersect, is
(C) x2 + y2 – 5x + 4y + 6 = 0
(A) 1 (B) 2 (C) 3 (D) 4
(D) x2 + y2 – 5x – 4y – 6 = 0

Q.3 y - 1 = m1(x – 3) and y – 3 = m2(x – 1) are two family


of straight lines, at right angles to each other. The locus Q.10 The line joining (5, 5) to (10 cos θ, 10 sin θ) is
of their point of intersection is divided internally in the ratio 2 : 3 at P. If θ varies then
the locus of P is :
(A) x2 + y2 – 2x – 6y + 10 = 0
(A) A pair of straight lines
(B) x2 + y2 – 4x – 4y + 6 = 0
(B) A circle
(C) x2 + y2 – 2x – 6y + 6 = 0
(C) A straight line
(D) x2 + y2 – 4x – 4y – 6 = 0
(D) A second degree curve which is not a circle
Q.4 The points (x1, y1), (x2, y2), (x1, y2) and (x2, y1) are always:
Q.11 The normal at the point (3, 4) on a circle cuts the circle
(A) Collinear (B) Concyclic
at the point (–1, –2). Then the equation of the circle is:
(C) Vertices of a square (D) Vertices of a rhombus
(A) x2 + y2 + 2x – 2y – 13 = 0

Q.5 Consider 3 non-collinear points A, B, C with (B) x2 + y2 – 2x – 2y – 11 = 0


coordinates (0, 6), (5, 5) and (–1, 1) respectively. (C) x2 + y2 – 2x + 2y + 12 = 0
Equation of a line tangent to the circle circumscribing (D) x2 + y2 – 2x – 2y + 14 = 0
the triangle ABC and passing through the origin is
(A) 2x – 3y = 0 (B) 3x + 2y = 0 Q.12 The shortest distance from the line 3x + 4y = 25
(C) 3x – 2y = 0 (D) 2x + 3y = 0 to the circle x2 + y2 = 6x – 8y is equal to
7 9 11 32
(A) (B) (C) (D)
Q.6 A (1, 0) and B(0, 1) and two fixed points on the circle 5 5 5 5
x2 + y2 = 1. C is a variable point on this circle. As C moves, π
the locus of the orthocenter of the triangle ABC is Q.13 The equation of a line inclined at an angle to
4
(A) x2 + y2 – 2x – 2y + 1 = 0 the axis X, such that the two circles x2 + y2 = 4 and x2 +
(B) x2 + y2 – x – y = 0 y2 – 10x – 14y + 65 = 0 intercept equal lengths on it, is
(C) x2 + y2 = 4 (A) 2x – 2y – 3 = 0 (B) 2x – 2y + 3 = 0
(D) x2 + y2 + 2x – 2y + 1 = 0 (C) x – y + 6 = 0 (D) x – y – 6 = 0

Q.7 A straight line with slope 2 and y - intercept 5 Q.14 The locus of the midpoint of a line segment that
touches the circle, x2 + y2 + 16x +12y + c =0 at a point is drawn from a given external point P to a given circle
Q. Then the coordinates of Q are with centre O (where O is origin) and radius r, is
(A) (–6, 11) (B) (–9, –13) (A) A straight line perpendicular to PO
(C) (–10, –15) (D) (–6, –7) (B) A circle with centre P and radius r
9 .60 | Circle

(C) A circle with centre P and radius 2r Q.19 The circles x2 + y2 + 2x + 4y – 20 = 0 and x2 + y2
r + 6x – 8y + 10 = 0
(D) A circle with centre at the midpoint PO and radius
2 (A) Are such that the number of common tangents on
Multiple Correct Choice Type them is 2
(B) Are not orthogonal
Q.15 Locus of the intersection of the two straight lines
passing through (1, 0) and (–1, 0) respectively and (C) Are such that the length of their common tangent
1
including an angle of 45° can be a circle with  12  4
is 5  
(A) Centre (1, 0) and radius 2 .  5 
(B) Centre (1, 0) and radius 2. (D) Are such that the length of their common chord is
(C) Centre (0, 1) and radius 2. 3
5 .
2
(D) Centre (0, –1) and radius 2.
Q.20 Three distinct lines are drawn in a plane. Suppose
Q.16 Consider the circles there exist exactly n circles in the plane tangent to all
the three lines, then the possible values of n is/are
S1 : x2 + y2 + 2x + 4y + 1 = 0
(A) 0 (B) 1 (C) 2 (D) 4
S2 : x2 + y2 – 4x + 3 = 0
S3 : x2 + y2 + 6y + 5 = 0 Q.21 The equation of a circle C1 is x2+y2+14x–4y + 28=0.
Which of this following statement are correct? The locus of the point of intersection of orthogonal
tangents to C1 is the curve C2 and the locus of the point
(A) Radical centre of S1, S2 and S3 lies in 1st quadrant.
of intersection of perpendicular tangents to C2 is the
(B) Radical centre of S1, S2 and S3 lies in 4st quadrant. curve C3 then the statement (s) which hold good?
(C) Radical centre of S1, S2 and S3 orthogonally is 1. (A) C3 is a circle
(D) Circle orthogonal to S1, S2 and S3 has its x and y (B) Area enclosed by C3 is 100π sq. unit
intercept equal to zero.
(C) Area of C2 is 2 times the area of C1.
(D) C2 and C3 are concentric circles.
Q.17 Consider the circles
C1 : x2 + y2 – 4x + 6y + 8 = 0
Q.22 The circles x2 + y2 – 2x – 4y + 1 = 0 and x2 + y2 +
C2 : x + y – 10x – 6y + 14 = 0
2 2 4x + 4y – 1 = 0
Which of the following statement (s) hold good in (A) Touch internally
respect of C1 and C2?
(B) Touch externally
(A) C1 and C2 are orthogonal.
(C) Have 3x + 4y – 1 = 0 as the common tangent at the
(B) C1 and C2 touch each other. point of contact.
(C) Radical axis between C1 and C2 is also one of their (D) have 3x + 4y + 1 = 0 as the common tangent at the
common tangent. point of contact.
(D) Middle point of the line joining the centres of C1
and C2 lies on their radical axis. Q.23 Which of the following is/are True? The circles x2
+ y2 – 6x – 6y + 9 = 0 and x2 + y2 + 6x + 6y + 9 = 0 are
such that
Q.18 A circle passes through the points (–1 , 1), (0, 6)
and (5, 5). The point (s) on this circle, the tangent (s) (A) They do not intersect.
at which is/are parallel to the straight line joining the
(B) They touch each other.
origin to its centre is/are:
(C) Their direct common tangents are parallel.
(A) (1, –5) (B) (5, 1) (C) (–5, –1) (D) (–1, 5)
(D) Their trannsverse common tangents are
perpendicular.
M a them a ti cs | 9.61

Q.24 Two circles x2 + y2 + px + py – 7 = 0 and x2 + y2 Q.30 Let A(x1, y1), B(x2, y2) and C(x3, x3) are the vertices
– 10x + 2py + 1 = 0 intersect each other orthogonally of a triangle ABC.
then the value of p is
Statement-I : If angel C is obtuse then the quantity
(A) 1 (B) 2 (C) 3 (D) 5 (x3 – x1)(x3 – x2) + (y3 – y1) (y3 –y2) is negative.
Statement-II: Diameter of a circle subtends obtuse
Q.25 Which of the following statements is/are incorrect? angle at any point lying inside the semicircle.
(A) Two circles always have a unique common normal.
(B) Radical axis is always perpendicular bisector to the Q.31 Let C be a circle with centre ‘O’ and HK is the
line joining the centres of two circles. chord of contact of pair of the tangents from point
A. OA intersects the circle C at P and Q and B is the
(C) Radical axis is nearer to the centre of circle of smaller midpoint of HK, then
radius.
Statement-I: AB is the harmonic mean of AP and AQ.
(D) Two circles always have a radical axis.
Statement-II: AK is the Geometric mean of AB and AO
and OA is the arithmetic mean of AP and AQ.
Assertion Reasoning Type
(A) Statement-I is true, statement-II is true and Comprehension Type
statement-II is correct explanation for statement-I.
(B) Statement-I is true, statement-II is true and Paragraph for questions 32 to 34
statement-II is NOT the correct explanation for statement-I. Let A, B, C be three sets of real numbers (x, y) defined as
(C) Statement-I is true, statement-II is false. A : {(x, y): y ≥ 1}
(D) Statement-I is false, statement-II is true. B : {(x, y): x2 + y2 – 4x – 2y – 4 = 0}

Q.26 Consider the lines L : (k + 7)x – (k – 1)y – 4(k – 5)=0 C : {(x, y): x + y = 2}
where k is a parameter and the circle
C : x2 + y2 + 4x + 12y – 60 = 0 Q.32 Number of elements in the A ∩ B∩ C is

Statement-I: Every member of L intersects the circle ‘C’ (A) 0 (B) 1 (C) 2 (D) infinite
at an angle of 90°
Q.33 (x + 1)2 + (y – 1)2 + (x – 5)2 + (y – 1)2 has the value
Statement-II: Every member of L tangent to the circle C.
equal to

Q.27 Statement-I: Angle between the tangents drawn (A) 16 (B) 25 (C) 36 (D) 49
from the point P(13, 6) to the circle S : x2 + y2 – 6x +
8y – 75 = 0 is 90° . Q.34 If the locus of the point of intersection of the pair
of perpendicular tangents to the cirlc B is the curve S
Statement-II: Point P lies on the director circle of S.
then the area enclosed between B and S is

Q.28 Statement-I: From the point (1, 5) as its centre, (A) 6π (B) 8π (C) 9π (D) 18π
only one circle can be drawn touching the circle
x2 + y2 – 2x = 7. Paragraph for questions 35 to 36
Statement-II: Point (1, 5) lies outside the circle Consider a circle x2 + y2 = 4 and a point P(4, 2). θ denotes
x2 + y2 – 2x = 7. the angle enclosed by the tangents from P on the circle
and A, B are the points of contact of the tangents from
Q.29 Statement-I: Let C1 (0, 0) and C2(2, 2) be centres P on the circle.
of two circle and L : x + y –2 = 0 is their common chord.
Q.35 The value of θ lies in the interval
If length of common chord is equal to 2 , then both
circles intersect orthogonally. (A) (0, 15º) (B) (15º, 30º)
Statement-II: Two circles will be orthogonal if their (C) (30º, 45º) (D) (45º, 60º)
centres are mirror images of each other in their common
chord and distance between centres is equal to length
of common chord.
9 .62 | Circle

Q.36 The intercept made by a tangent on the x-axis is Q.4 Let PQ and RS be tangents at the extremities of the
diameter PR of a circle of radius r If PS and RQ intersect
9 10 11 12
(A) (B) (C) (D) at a point X on the circumference of the circle, then 2r
4 4 4 4 equals  (2001)
PQ + RS
(A) PQ ⋅ RS (B)
Paragraph for questions 37 to 39 2

Consider the circle S : x2 + y2 – 4x – 1 = 0 and the line L 2PQ ⋅ RS PQ2 + RS2


(C) (D)
: y = 3x – 1. If the line L cuts the circle at A and B then PQ + RS 2

Q.37 Length of the chord AB equal Q.5 Let AB be a chord of the circle x2 + y2 = r2 subtending
a right angle at the centre. Then the locus of centroid of
(A) 2 5 (B) 5 (C) 5 2 (C) 10 the triangle PAB as P moves on the circle is (2001)
(A) A parabola (B) A circle
Q.38 The angle subtended by the chord AB in the
minor arc of S is (C) An ellipse (D) A pair of straight lines

3π 5π 2π π
(A) (B) (C) (D) Q.6 If two distinct chords, drawn from the point
4 6 3 4
(p, q) on the circle x2 + y2 = px + qy (where pq ≠ 0) are
bisected by the x-axis, then (1999)
Q.39 Acute angel between the line L and the circle S is
(A) p2 = q2 (B) p2 = 8q2
π π π π
(A) (B) (C) (D)
2 3 4 6 (C) p2 <8 q2 (D) p2 > 8q2

Q.7 Consider
Previous Years’ Questions L1 : 2x + 3y + p – 3 = 0
L2 : 2x + 3y + p + 3 = 0
Q.1 Tangents drawn from the point P (1, 8) to the circle
where p is a real number and
x2 + y2 – 6x – 4y – 11 = 0 touch the circle at the point A
and B. The equation of the circumcircle of the triangle C : x2 + y2 – 6x + 10y + 30 = 0 (2008)
PAB is (2009)
Statement-I: If line L1 is a chord of circle C, then line L2
(A) x2 + y2 + 4x – 6y + 19 = 0 is not always a diameter of circle C. and
(B) x2 + y2 – 4x – 10y + 19 = 0
Statement-II: If line L1 is a diameter of circle C, then line
(C) x2 + y2 – 2x + 6y – 29 = 0 L2 is not a chord of circle C.
(D) x2 + y2 – 6x – 4y + 19 = 0
Paragraph 1: Let ABCD be a square of side length 2
unit. C2 is the circle through vertices A, B, C, D and C1 is
Q.2 Let ABCD be a quadrilateral with area 18, with side
the circle touching all the sides of square ABCD. L is the
AB parallel to the side CD and AB = 2CD. Let AD be
line through A. (2006)
perpendicular to AB and CD. If a circle is drawn inside
the quadrilateral ABCD touching all the sides, its radius
is  (2007) Q.8 If P is a point of C1 and Q is a point on C2, then
3 PA2 + PB2 + PC2 + PD2
(A) 3 (B) 2 (C) (D) 1 is equal to
2 QA2 + QB2 + QC2 + QD2
Q.3 The locus of the centre of circle which touches (A) 0.75 (B) 1.25 (C) 1 (D) 0.5
(y – 1)2 + x2 = 1 externally and also touches x axis, is
 (2005)
Q.9 A circle touches the line L and the circle C1 externally
(A) {x2 = 4y , y ≥ 0}∪{(0, y), y < 0}
such that both the circle are on the same side of the
(B) x2 = y line, then the locus of centre of the circle is
(C) y = 4x2 (A) Ellipse (B) Hyperbola
(D) y2 = 4x ∪ (0, y), y∈R (C) Parabola (D) Parts of straight line
M a them a ti cs | 9.63

Q.10 A line M through A is drawn parallel to BD. Point S Q.15 Let T1, T2 and be two tangents drawn from
moves such that its distances from the line BD and the (–2, 0) onto the circle C : x2 + y2 = 1. Determine the
vertex A are equal. If locus of S cuts M at T2 and T3 and circles touching C and having T1, T2 as their pair of
AC at T1. then area of ∆T1T2T3 is tangents. Further, find the equations of all possible
1 common tangents to these circles when taken two at a
2
(A) sq unit (B) sq unit time. (1999)
2 3
(C) 1 sq unit (D) 2 sq unit
Q.16 Two parallel chords of a circle of radius 2 are at
a distance 3 + 1 apart. If the chords subtend at the
Paragraph 2: A circle C of radius 1 is inscribed in an
π 2π
equilateral triangle PQR. The points of contact of C with center, angles of and , where k > 0 , then the
the sides PQ, QR, RP are D, E, F respectively. The line k k
value of k  is
PQ is given by the equation 3 x + y – 6 = 0 and the [Note : k  denotes the largest integer less than or
3 3 3 equal to k ]  (2010)
point D is  ,  . Further, it is given that the origin
 2 2
 
and the centre of C are on the same side of the line PQ. Q.17 The circle passing through the point ( −1,0 ) and
 (2008) touching the y − axis at ( 0,2 ) also passes through the
point (2011)
Q.11 The equation of circle C is
 3   5 
(A) (x – 2 3 )2 + (y – 1)2 = 1 (A)  − ,0  (B)  − ,2 
2  2   2 
 1
(B) (x – 2 3 ) +  y +  = 1
2
2  3 5
 (C)  − ,  (D) ( −4,0 )
(C) (x – 3 ) + (y + 1)2 = 1
2  2 2

(D) (x – 3 )2 + (y – 1)2 = 1
Paragraph 3: A tangent PT is drawn to the circle
Q.12 Point E and F are given by x2 + y 2 =
4 at the point P ( )
3,1 . A straight line

 3 3  3 1 L, perpendicular to PT is a tangent to the circle


(A)  , ,
 2 2
 
( 3,0 ) (B)  , ,
 2 2
 
( 3,0 ) ( x − 3)
2
1.
+ y2 =

 3 3  3 1 3 3  3 1 Q. 18 A common tangent of the two circles is  (2012)


(C)  ,  , ,  (D)  ,  , , 
 2 2  2 2 2 2   2 2
        (A) x = 4 (B) y = 2

Q.13 Equations of the sides QR, RP are (C) x + 3y =


3 (D) x + 2 2 y =
6

2 2
(A) y = x + 1, y = – x –1 Q.19 A possible equation of L is  (2012)
3 3
1 (A) x − 3y =
1 (B) x + 3y =
1
(B) y = x, y = 0
3 (C) x − 3y =
−1 (D) x + 3y =
5
3 3
(C) y = x + 1, y = – x–1 2
2 2 Q.20 Let S be the focus of the parabola y = 8x
and let PQ be the common chord of the circle
(D) y = 3 x, y = 0
x2 + y 2 − 2x − 4y =
0 and the given parabola. The area
of the triangle PQS is  (2012)
Q.14 Let 2x2 + y2 – 3xy = 0 be the equation of a pair of
tangents drawn from the origin O to a circle of radius 3 Q.21 The locus of the mid-point of the chord of contact
with centre in the first quadrant. If A is one of the points of tangents drawn from points lying on the straight line
of contact, find the length of OA.  (2001) 2 2
4x – 5y = 20 to the circle x + y =9 is (2012)
9 .64 | Circle

(A) 20(x2 + y2) ﹘ 36x + 45y = 0 1 1 2


(A) f  2  ≥ f (1 ) (B) f  3  ≤ f 
(B) 20(x2 + y2) + 36x ñ 45y = 0     3
(C) 36(x2 + y2) ﹘ 20x + 45y = 0
f ' (3) f ' (2)
(D) 36(x2 + y2) + 20x ñ 45y = 0 (C) f ( 2 ) ≤ 0 (D) ≥
f (3) f (2)

Q.22 Circle(s) touching x-axis at a distance 3 from the


Q.26 The circle C1 : x2 + y 2 =
3 , with centre O, intersects
origin and having an intercept of length 2 7 on y-axis 2
the parabola x = 2y at the point P in the first quadrant.
is (are) (2013)
Let the tangent to the circle C1 at P touches other two
2 2
(A) x + y − 6x + 8y + 9 =0 circles C2 and C3 at R 2 and R 3 , respectively. Suppose
2 2
(B) x + y − 6x + 7y + 9 =0 C2 and C3 have equal radii 2 3 and centres Q2 and
2 2 Q3 respectively. If Q2 and Q3 lie on the y-axis, then
(C) x + y − 6x − 8y + 9 =0
 (2016)
2 2
(D) x + y − 6x − 7y + 9 =0
(A) Q2Q3 = 12

Q.23 The common tangents to the circle x + y = 2 2 2 (B) R 2R3 = 4 6


2
and the parabola y = 8x touch the circle at the points
(C) Area of the triangle OR 2R3 is 6 2
P, Q and the parabola at the points R, S. Then the area
of the quadrilateral PQRS is  (2014) (D) Area of the triangle PQ2Q3 is 4 2
(A) 3 (B) 6 (C) 9 (D) 15
2 2
Q.27 Let RS be the diameter of the circle x + y = 1,
Q.24 A circle S passes through the point (0, 1) and where S is the point (1, 0). Let P be a variable point
is orthogonal to the circles ( x − 1 ) + y 2 = (other than R and S) on the circle and tangents to the
2
16 and
x2 + y 2 =
1 . Then (2014) circle at S and P meet at the point Q. The normal to the
circle at P intersects a line drawn through Q parallel to
(A) Radius of S is 8 (B) Radius of S is 7 RS at point E. Then the locus of E passes through the
(C) Centre of S is (-7, 1) (D) Centre of S is (-8, 1) point(s)  (2016)
1 1 
(A)  , 
Q.25 Let 3 3 
x
 n 1 1
 nn ( x + n)  x + n  .....  x + n   (B)  , 
  2  n   4 2
f ( x ) = lim   ,
 n! x2 + n2  x2 + n  ....  x2 + n  
2 2
( )
n→ ∞
    1 1 
 4   n2   (C)  , − 
   3 3
for all x > 0. Then (2016)
1 1
(D)  , − 
 4 2
M a them a ti cs | 9.65

PlancEssential Questions
JEE Main/Boards JEE Advanced/Boards

Exercise 1 Exercise 1
Q.12 Q.18 Q.21 Q.5 Q.9 Q.14
Q.23 Q.29 Q.17 Q.19 Q.21 Q.24

Exercise 2 Exercise 2
Q.3 Q.7 Q.14 Q.2 Q.4 Q.9
Q.15 Q.20 Q.13 Q.16 Q.21
Q.22 Q.25 Q.27
Previous Years’ Questions
Q.22 Q.25 Q.26
Q.1 Q.3 Q.5 Q.29 Q.32
Q.8 Q.11 Q.13
Previous Years’ Questions
Q.1 Q.3 Q.6
Q.7 Q.9 Q.13

Answer Key

JEE Main/Boards Q.12 x = 3 and y = ±3.

Exercise 1 Q.14 x2 + y2 + 2ax + 2py – (b2 + q2) =0; a2 + b2 + p2 + q2

Q.15 x2 + y2 + 18x – 2y + 32 = 0
Q.1 x2 + y2 + 3x + 12y + 2 = 0
Q.16 32 sq. units
Q.3 (1, –1), 13 , x2 + y2 – 2x + 2y – 11 = 0
Q.17 x2 + y2 – 12x – 10y + 52 = 0
Q.4 3x - 4y = 7, 4x + 3y = 0
Q.20 4 2
Q.6 13x + 9y = 77, 3x – y – 27 = 0
1
Q.21 cos–1
Q.7 2x2 + 2y2 + 16x – 8y – 41 = 0 2 2
Q.8 23x2 + 23y2 – 156x + 38y + 168 = 0 Q.22 x2 + y2 – 2x – 6y – 8 =0

Q.9 y = x Q.23 8 sq. units.

Q.10 x = 3 Q.26 (2, –1) ; 2


109 Q.28 4(x2 + y 2 ) + 2y − 29 =
0
Q.11
2
9 .66 | Circle

Q.29 x2 + y 2 + 8x − 6y + 9 =0 Q.30 x2 + y2 – x = 0 Q.31 2 a2 = b2

Exercise 2
Single Correct Choice Type

Q.1 D Q.2 B Q.3 A Q.4 B Q.5 A Q.6 D

Q.7 D Q.8 B Q.9 C Q.10 C Q.11 A Q.12 D

Q.13 C Q.14 A Q.15 B Q. 16 C Q.17 A Q.18 D

Q.19 C Q.20 D Q.21 A

Previous Years’ Questions

Q.1 D Q.2 B Q.3 C Q.4 A Q.5 C Q.6 A

Q.7 C Q.8 B Q.10 2x2 + 2y2 – 10x – 5y + 1 = 0 Q.11 (a,b) and (0,0)

Q.12 4x2+25y2=4x2y2 Q.14 A Q.15 B Q.16 D Q.17 D

Q.18 B Q.19 C

JEE Advanced/Boards

Exercise 1
Q.1 (i) x2 + y2 – 4x – 3y = 0, (ii) x2 + y2 = 24, (iii) 4x + 3y = 24 Q.2 32

 23 
Q.3 625 Q.4 4 Q.6  2, 
 3 
1 1 1
Q.7 x2 + y2 – 3x – 3y + 4 = 0 Q.8 (1, 0)&  ,  ; r = Q.9 4x2 + 4y2 + 6x + 10y – 1 = 0
2 2 2 2
 1 1
Q.10 x2 + y2 + 16x + 14y – 12 = 0 Q.11 (–4, 4);  − ,  Q.12 x2 + y2 + 4x – 6y = 0; k= 1;
 2 2

Q.13 x2 + y2 = 64 Q.14 9x – 10y + 7 =0; radical axis Q.15 x2 + y2 + 7x – 11y + 38 = 0

Q.16 x2 + y2 + 6x – 3y = 0 Q.17 12 Q.18 x2 + y2 – 12x –12y + 64 = 0

Q.19 (A) S; (B) R; (C) Q ; (D) P Q.20 10 Q.21 19

Q.22 (–4, 2), x2 + y2 – 2x – 6y – 15 = 0

Q.23 x2+ y2 – 6x + 4y = 0 OR x2 + y2 + 2x – 8y + 4 = 0 Q.24 x2 + y 2 +x – 6y + 3 = 0


M a them a ti cs | 9.67

Exercise 2
Single Correct Choice Type

Q.1 A Q.2 C Q.3 B Q.4 B Q.5 D Q.6 A


Q.7 D Q.8 A Q.9 A Q.10 B Q.11 B Q.12 A
Q.13 A Q.14 D

Multiple Correct Choice Type

Q.15 C, D Q.16 B, C, D Q.17 B, C Q.18 B, D Q.19 A, C, D Q.20 A, C, D


Q.21 A, B, D Q.22 B, C Q.23 A, C, D Q.24 B, C Q.25 A, B, D

Assertion Reaosing Type

Q.26 C Q.27 A Q.28 D Q.29 A Q.30 A Q.31 A

Comprehension Type

Paragraph 1: Q.32 B Q.33 C Q.34 C


Paragraph 2: Q.35 D Q.36 B
Paragraph 3: Q.37 D Q.38 A Q.39 C

Previous Years’ Questions

Q.1 B Q.2 B Q.3 A Q.4 A Q.5 B Q.6 D


Q.7 C Q.8 A Q.9 C Q.10 C Q.11 D Q.12 A
2 2
 4 1 5  4
Q.13 D Q.14 3(3 + 10 ) Q.15  X +  + y =   ; y = ± x +  Q.16 3
 3 3 39  5

Q.17 D Q.18 D Q.19 A Q.20 4 Q.21 A Q.22 C
Q.23 D Q.24 B C Q.25 A C D Q.26 C Q.27 A C
9 .68 | Circle

⇒ 3y + 12 = 4x + 12
Solutions
⇒ 4x − 3y =
0

JEE Main/Boards ∴ Slope of tangent at P is


−1
=
−3
4/3 4
Exercise 1 ∴ Equation of tangent is ( y + 4=
−3
) 4
( x + 3)
Sol 1: ∴ Centre lies on 2x – y – 3 = 0 ⇒ 4y + 16 =−3x − 9
∴ Let the centre be C ≡ (h, 2h –3) ⇒ 3x + 4y + 25 =
0
It also passes through A ≡ (3, –2) and B ≡ (–2, 0)
∴ AC = BC Sol 5: Equation of tangent at (1, –2) is

⇒ (h – 3)2 + (2h – 1)2 = (h + 2)2 + (2h – 3)2 x x1 + y y1 = a2

⇒ – 6h + 9 – 4h + 1 = 4h + 4 – 12h + 9 – 2h =3 x – 2y = 5

3  −3  1 5
∴h=– ∴
= C  , −6  ∴y = x–
2 2 2
 2 
∴ Equation of the circle is Equation of C2 is (x – 4)2 + (x + 3)2 = ( 5 )2

(x – h)2 + (y – k)2 = R2 Now the tangent will touch C2 If c2 = r2(1 + m2)


2
2 2 5
 3  −3  c2 =  
⇒  x +  + (y + 6)2 =  , − 3  + ( −6 + 2 )
2
2
 2   2  2
 1 5
r (1 + m ) = 5 ×  1 +  =  
2 2

⇒ x2 + 3x +
9
+ y2 + 12y + 36 =
81
+16  4 2
4 4 ∴The given line is tangent to C2
x2 + y2 + 3x + 12y + 2 = 0
Sol 6: Equation of circle is
Sol 2: We can see that (0, 0), (1, 1) & (6, –4) form a right C ≡ (x – 1)2 + (y + 4)2 = (2 10 )2
angled triangle with (0, 0) & (6, –4) as diameter
Shifting origin to (1, –4)
Equation of circle is (x – 0) (x – 6) + y(y + 4) = 0
∴ C’ ≡ X2 + Y2 = (2 10 )2 & P = (7, 1)
⇒ C = x(x – 6) + y (y + 4) = 0
∴ Y – 1 = m(X – 7)
We can see that (5, –5) satisfies this equation
∴Y = mX + (1 – 7m)
∴ 4 points are concyclic
∴c2 = a2(1 + m2)
Sol3: A ≡ (–1, 2), and B ≡ (3, –4) (1 − 7m)
2
(
= 40 1 + m2 )
Equation of the circle is (x + 1) (x – 3) + (y – 2) (y + 4) ⇒ 9m – 14m – 39 = 0
2

⇒ x + y – 2x + 2y – 11 = 0
2 2
⇒ 9m2 – 27m + 13m – 39 = 0
∴ C =( −g, − f ) =(1, − 1 ) −13
m = 3 or m =
9
Radius = g2 + f 2 − c = 1 + 1 + 11 = 13
Since slope remains same in both system
∴Equation of lines in old co-ordinates are
Sol 4: Given equation of circle is x2 + y2 = 25 P ≡ ( −3 , − 4 )
−13
0+4 4 (y + 3) = 3(x – 8) &(y + 3) = (x – 8)
∴ Slope of normal=
OP = 9
0+3 3
Or 3x − y − 27 =0 and 13x + 9y =
77
4
∴ Equation of normal is ( y + 4 )= ( x + 3)
3
M a them a ti cs | 9.69

Sol 7: Centre = (– 4, 2) Sol 12: C1 ≡ x2 + y2 = 9


Tangent is x – y = 3 Centre = (0, 0 & R1 = 3)
−4 − 2 − 3  9  C2 ≡ x2 + y2 – 12x + 27 = 0
∴Radius = = 
2  2 Centre = (6, 0 & R2 = 3)
81 ∴ The circles touch each other externally
∴C ≡ (x + 4)2 + (y – 2)2 =
2
T2 T1
3
Sol 8: Using the concept of family of circles, let the
equation of circle be

(x 2
) ( )
+ y 2 − 8x − 2y + 7 + λ x2 + y 2 − 4x + 10 y + 8 =
0
-3
3
6
T3
As (3, -3) lies on it
∴ ( 9 + 9 − 24 + 6 + 7 ) + λ ( 9 + 9 − 12 − 30 + 8 ) =0 ∴ The equation of tangents are
⇒ 7 − 16 λ =0 y = 3, x = 3 & y = –3 (from figure itself)
7
⇒λ=
16 Sol 13: Family of circles passing through two points is
∴ Equation of the circle is (x – x1) (x – x2) + (y – y1) (y –y2) + λL = 0

(x2 + y 2 − 8x − 2y + 7 + )
7 2
16
(
x + y 2 − 4x + 10y + 8 = 0 ) ∴x(x – 1) + y2 + ly = 0
∴x2 + y2 – x + ly = 0
= 023 x2 + 23y 2 − 156 x + 38y + 168 =
or 0
 1 −λ 
Centre =  , 
2 2 
Sol 9: C ≡ x2 + y2 – 4x = 0 Now since the circle touches internally [∵ (0, 0), & (1, 0)
Centre = (2, 0) lie inside the circle]
1−0 ∴r1 – r2 = distance between their centres
Slope of line perpendicular to chord = = −1
1−2
1 λ2 1 λ2
∴ Slope of chord = 1 ∴3 – + = +
4 4 4 4
⇒ y – 1 = 1 (x – 1)
 1 + λ2 
∴ y = x is the equation of chord ∴9 = 4  
 4 
 
Alternative 1 
∴λ = ±2 2 ∴Centre =  , ± 2 
Equation of a chord bisected at a given point is T = S1 2 

∴x x1 + y y1 − 2 ( x + x1 ) = x12 + y12 − 4x1 Sol 14: Let the coordinates of diameter be (h1, k1) &
Or, x + y − 2x − 2 = 1 + 1 − 4 (h2, k2)

Or, x – y = 0 ∴Equation of circle is


(x – h1) (x – h2)+(y – k1) (y – k2) = 0
Sol 10: Equation of chord of contact ⇒ x2 – y2 – (h1 + h2)x – (k1 – k2)y + (h1h2+k1k2)=0
xx1 + yy1 + g(x + x1) + f(g+g1) + c = 0
⇒ x2 + y2 – (–2a)x – (–2p)y + (– b2 – q2) = 0
⇒ 6x – 2(x + 6) = 0 ; x = 3
⇒ x2 + y2 + 2ax + 2py – (b2 + q2) = 0

Sol 11: Length of tangent from a point ∴R = a2 + p2 + b2 + q2

= x12 + y12 + 2gx1 + 2fy1 + C


Sol 15: Given equation of line is x + y = 2  …(i)
2 2
4(3) + 4(2) + 4 × 3 + 16 × 2 + 13 109 109 2 2
2 , we get
On solving (i), with x + y =
= = =
4 4 2
9 .70 | Circle

x2 + ( 2 − x ) =
2
2 As circle with center (6,5) touches it externally

⇒ x2 + 4 − 4x + x2 =2 Y

⇒ 2x2 − 4x + 2 =0 (6,5)

⇒ ( x − 1 ) = 0 ⇒ x = 1 This means the line represented


2 C2

by (i) and the circle intersects only at (1, 1) (2,2)

Similarly, on solving x + y = 2 and C1


X
x2 + y 2 + 3x + 3y − 8 =,
0 we get
∴ C1 C2 =+
r1 r2
2x2 − 4x + 4 + 3 ( 2 ) − 8 =0 , we get
⇒ 2x2 − 4x + 2 =0 ⇒ (6 − 2)2 + (5 − 2)2 = 2 + r

⇒ 2 ( x − 1) =
0
2 ⇒ r2 + 4r + 4 = 16 + 9

⇒x =1 ⇒ r2 + 4r – 21 = 0

Hence, the line intersects only at one point (1, 1) ⇒ r2 + 7r – 3r – 21 = 0

Hence, proved. ∴r = 3 ( r cannot be negative)


∴Equation of C2 is (x – 6)2 + (y – 5)2 = 9
Sol 16: Let C ≡ (h, k) be the center of the circle
∴4k = h + 7 Sol 18: Let the equation of circle be

∴ AC = BC x2 + y2 + 2gx + 2fy + c = 0

⇒ (4k – 7 + 3)2 + (k – 4)2 = (4k – 12)2 + (k – 4)2  1


Let  m,  be point on the circle.
 m
(
⇒ ( 4k − 4 ) = 4k − 122
2
) On substitution we get
⇒K=
2
If m4 + 2gm3 + 2fm + cm2 + 1 = 0
∴ C ≡ (1, 2 )
m1, m2, m3, m4 are roots of this equation
Now
A(-3,4) B(5,4) 4y=x+7 then, m1m2m3m4 = 1

Sol 19: Equation of line


C(1,2)
⇒ x cos x + y sin x – p = 0
C D
Now family of circle passing
Equation of chord AB is y – 4 = 0 through the intersection of
the circle & line is
∴Perpendicular distance of center from chord AB is
2−4 x2 + y2 – a2 + λ (x cos x + y sin
=2 x – p) = 0
1
∴ AB = 8 and BC = 2PQ = 4 ∴ Radius of circle = AM
= a2 − p2
∴Area of rectangle = 8 × 4 = 32
2 2
 λ cos x   λ sinx  2
⇒   +  + a + λp = a2 − p2
Sol 17: Radius of C1 = 2  2   2 
∴Centre = (2, 2) λ2
⇒ + lp + p2 = 0
4
(λ + 2p)2 = 0 ⇒ λ = – 2p
∴S ≡ x2 + y2 – 2px cos x
– 2py sin x + 2P2 – a2 = 0
M a them a ti cs | 9.71

Sol 20: Slope of AB = 1 Let θ be the angle subtended at the circumference


∴ Slope of L1 × slope of AB =
−1 ∴ Angle subtended at circumference
⇒ Slope of L1 =
−1 1
= (Angle subtended at centre)
2
1 1
∴ cosθ = ⇒ θ = cos–1
2 2 2 2

Sol 22: Given x2 + y 2 − 2x − 2λy − 8 =0

( )
⇒ x2 + y 2 − 2x − 8 − 2λ ( y ) =0  ….(i)
Let S ≡ x2 + y 2 − 2x − 8 =0 and  …(iii)
0 
L≡ y = …(iii)
∴ The equation is represents a family of circles passing
3 5
And mid-point of AB, M ≡  ,  through the intersection of S = 0 & L = 0.
2 2
∴ On solving (ii) and (iii), we get
∴ Equation of line L1 is
x2 − 2x − 8 =0
 5  3
y −  =−1  x −  2 ± 4 + 32
 2   2  x
⇒= = 4 or − 2
2
Or, (2y - 5) = - (2x - 3) ∴ The fixed point are A (4, 0) and B (-2, 0) from the
Or 2x + 2y – 8 = 0 diagram, the perpendicular bisector of AB is con.
Current with the tangents at P
Or, x + y – 4 = 0
∴ Length of perpendicular from (0, 0) on L1 is

0+0−4
=2 2
2

( )
2
( a)
2
∴ Length of the chord
= 2 − 2 2

( )
2
= 2 16 − 2 2

=4 2

Sol 21: Equation of circle is x2 + y2 – 4y = 0


∴ M ≡ (1, 0 )
∴ Centre = (0, 2) & radius = 2
And Equation of line MP is x = 1  …..(iv)
Perpendicular distance of center from the line x + y = 1 is
∴ On solving (iv) with x + 2y + 5 = 0
0 + 2 −1 1
= We get 1 + 2y + 5 = 0
2 2
−6
⇒ 2y + 6 = 0 ⇒ y = =−3
2
∴ P ≡ (1, − 3 )
Centre of circle (i) is C ≡ (1, λ )
If P is the point of intersection of tangents then CB is
perpendicular to BP
λ −0 0+3
∴ × =−1
1 − 4   4 −1
9 .72 | Circle

λ 3 The equation of radical axes are S1 – S2 = 0


∴ × =−1 ⇒ λ = 3
−3 3 3  5 7−9
∴ S 1 – S2 =  − 4  x – y + =0
∴ Equation of the required circle is 2  2 2

x2 + y 2 − 2x − 6y − 8 =0 ⇒5x + 5y – 5 = 0 ⇒ x + y – 1 = 0
and S1 – S3 = 0 ⇒ – 4x – y + 7 = 0
Sol 23: Length of tangent = S11 4x + y – 7 = 0

∴ QP = 42 + 52 − 42 − 10 − 11 = 2 ∴The radical centre is (2, –1)


Length of tangent = S1 = 22 + 12 − 8 + 7 =2
P
Sol 27: Let (h, k) be the point on circle x2 + y2 = a2
Q S ⇒ ∴ h2 + k 2 =
a2  …(i)
Equation of chord of contact for x2 + y 2 =is
b2
R b2 …(ii)
hx + ky =

1 As (ii) touches the circle x2 + y 2 =


c2
Area of PQRS = 2DPQS = 2 × × PS x QPPS
2 −b2
∴ c
=
= Radius of circle = 22 + 12 + 11 = 4 h2 + k 2

∴ Area of PQRS = 4 × 2 = 8 ⇒ b2 =
ac
∴ a, b and c are in G.P.
Sol 24: The equation of any curve passing through the
intersection of Sol 28: Let the required circle be
L1 ≡ a1x + b1y + c1 = 0 x2 + y2 + 2gx + 2fy + c = 0  … (i)
L2 ≡ a2x + b2y + c2 = 0 The given circles are
L3 ≡ y = 0 & L4 ≡ x = 0 is L1 L2 + λ L3 L 4 x2 + y2 + 3x – 5y + 6 = 0  … (ii)
⇒ (a1x + b1y + c1) (a2x + b2y + c2) + lxy = 0 29
and x2 + y2 – 7x + =0 … (iii)
where λ is a parameter 4
Now 1, 2 & 1, 3 are orthogonal
This curve represents a circle if coeff. of x2 = coeff. of y2
3 −5
∴ a1a2 = b1b2 ∴2g + 2f =c+6
2 2
−7 29
Sol 25: Let any point on c2 be (h, k) 3g – 5f = c + 6 & 2g × + 2f × 0 = c +
2 4
Length of tangent from any point to circle 29
⇒–7g = c +
= S1 4
−5
∴l = h2 + k 2 + 2gh + 2fk + c1 ∴10g – 5f =
4
Now since (h, k) satisfies circle 1 ∴8g – 4f = – 1
∴h + k + 2gh + 2fk = – c
2 2
Equation of circle is
∴l = c1 − c (8g + 1)
x2 + y2 + 2gx + y+c=0
2
Sol 26: The tangents to the these circle are equal in The centre lies on the line
length 3x + 4y + 1 = 0
∴The point is radical centre (8g + 1)
⇒ 3(– g) – 4 +1=0
4
M a them a ti cs | 9.73

⇒ –11g = 0 ∠ OPR
= 90° − ∠ QPR= 45°
1 29
⇒ g = 0, f = and c = – OM
4 4 ∴ In ∆OMP, sin 45° =
∴Equation of circle is OP
4x2 + 4y2 + 2y – 29 = 0 1 a
⇒ =
2 b
Sol 29: Given equation of circle is x 2 + 4x + ( y − 3) =
2
0 ⇒ b =2 a .
0+h 3+k 
Let M ≡ (h, k ) ∴B ≡  , 
 2 2  Sol 32: According to condition
h 3+k 
∴B ≡  ,  S1 S2
2 2 
Y 1
(h, k)
1
2
R 3

(0,3) θ1 + θ2 = 90º
∴tanθ1tanθ2 = 1
B r1 1
X tanθ1 = =
M Length of tangent h2 + k 2 − 1
3
tanθ2 =
As point B lies on the circle
h2 + k 2 − 3
h2 h 3+k 
2
According to condition –
∴ + 4× + − 3 =0
4 2  2  ∴ 3 = (h2 + k2 –1) (h2 + k2 – 3)

h2 k2 9 k 3 3 = (h2 + k2)2 –4 (h2 + k2) + 3


⇒ + 2h + + −2 × =0
4 4 4 2 2 ∴ h2 + k2 = 0
⇒ h2 + k 2 + 8h − 6k + 9 =
0 or h2 + k2 = 4

∴ The value of point B is Now h2 + k2 ≠ 0 as no tangent will be possible.

x 2 + y 2 + 8x − 6y + 9 =0 ∴ The locus of point is a circle

Sol 30: Let (h, k) be middle points Sol 33: Let the other end of diameter be (h, k)

Equation of chord through (h, k) is ∴ Equation of circle is

xh – (x + h) + yk = h2 – 2h + k2 … (i) (x – a) (x – h) + (y – b) (y – k ) = 0

As the chord given by equation (i) passes through (0, 0) a+h b+k 
∴ Center ≡  , 
 2 2 
∴ On substituting, x = 0 and y = 0, we get
Since the circle touches the x-axis
– h = h2 – 2h + k2
∴ | y-coordinate| = radius
∴Locus of midpoint is x2 – x + y2 = 0
2 2
b+k a+h b +k 
⇒ =   +  − (ah + bk)
Sol 31: Given, OM = a and OP = b 2  2   2 
From the diagram, 2
a+h
∠ PRQ =
90°
∴  = (ah + bk)
 2 
And PR = QR
∠ QPR =
∠ PQR =
45°
9 .74 | Circle

∴Locus of point is λ2
Sol 2: (B) S ≡ x2 + y2 + lx + =0
x2 + 2ax + a2 = 4ax + 4by 2
(x – a)2 = 4by λ
2
λ2 λ2
Radius = g2 + f 2 − c =   − = −
2 2 4
Sol 34: Let G be perpendicular from C on AB
∴Radius is not defined for any real value of l
And M be midpoint of CD
Let radius = R Sol 3: (A) For an equilateral B
E triangle inscribed in circle of
radius r, in ∆ OAB using cosine a r
rule, we get
o
120
r O
r 2 + r 2 − a2
cos120º = A
C M D F 2r 2 a/2
⇒ – r2 = 2r2 – a2
A
G O
B ⇒a= 3r
Area of equilateral triangle
3 2 3 3 3 2
∴MO2 + MD2 = OD2 (O is centre) = a = × ( 3 r)2 = r
4 4 4
R2
MO2 = R2 –
4 Radius of given circle = g2 + f 2 − c = 1
3R
MO = 3 3
4 ∴A =
4
3R
⇒ CG =
4 Sol 4: (B) Let the centre of circle be (– h, 0)
R R
AG = AO – GO = AO – CM = R – = where h > 0
2 2
3R 2 R2 Radius = 5
AC2= AG2 + GC2 = +
4 4 ∴ Equation of circle is (x + h)2 + y2 = 25
∴ AC = R It passes through the point (2, 3)
AE AB ∴(h + 2)2 = (4)2 ⇒ h = 2 or h = –6
∴ = (As ∆ AEB  ∆ ACG )
AC AG
But h > 0 ⇒ h = 2 ⇒ ( 2 + h) + 9 =
2
h 2 or − 6
25 ⇒=
AE AB ∴ Equation of the circle is x2 + y2 + 2x – 21 = 0
⇒ = ⇒ AE = 2AB
R R
∴ Intercept made on y-axis = 2 f 2 − c = 2 21
2

Sol 5: (A) S1 : x2 + y2 = 1
Exercise 2
S2 : x2 + y2 – 2x – 6y = 6
Single Correct Choice Type S3 : x2 + y2 – 4x – 12y = 9

Sol 1: (D) The centers are A = (2, 3) ; B = (–1, –2) ; C = r1 = 1; r2 = 12 + 32 + 6 =4; r3= 22 + 62 + 9 =7
(5, 8)
∴r1, r2, r3 are in A.P.
3 − ( −2) 5
∴ Slope of AB = =
2 − ( −1) 3 Sol 6: (D) S1: x2 + y2 + 2lx + 4 = 0
8−3 5 S2: x2 + y2 – 4lx + 8 = 0
and slope AC = =
5−2 3 Since both represent real circles
∴ The three points are collinear
M a them a ti cs | 9.75

∴r1 ≥ 0 & r2 ≥ 0
∴l2 – 4 ≥ 0 ∴ λ ≤ –2 or λ ≥ 2 ... (i)  1
Let  x,  be a point on the circle.
∴4l2 – 8 ≥ 0 ∴λ ≤ – 2 or λ ≥ 2  ... (ii)  x

From 1, 2 λ (– ∞, –2] U [2, ∞) ∴ x4 + 2gx3 + cx2 + 2fx + 1 = 0


1
All of these lie within the range ⇒ abcd = =1
1
abc
Sol 7: (D) s = x2 + y2 + 16x – 24y + 183 = 0 Sol 11: (A) Circumradius R = , where ∆ is the area
4∆
Centre ≡ (–8, 12) Radius = 5 of a triangle
12 × 12 × 6
Let (x1, y1) be the image of (–8, 12) w.r.t. to the line ⇒ R=
1 
4x + 7y + 13 = 0 4 ×  × 6 × height 
2 
x1 − ( −8) y1 − 12
∴ = Height = 122 − 32 = 3 15
4 7

=
{
−2 4 × ( −8 ) + 7 × 12 + 13 } ∴R=
12 × 6
=
8 15
2
4 +7 2 3 15 5

x1 + 8 y1 − 12 Sol 12: (D) Given, ac = bd


= =–2
4 7
⇒ AO × OC = OB × OD
x1 = – 16, y1 = – 2
Y
Equation of required circle is
(x + 16)2 + (y + 2)2 = 52
D
x2 + y2 + 32x + 4y + 235 = 0

Sol 8: (B) Equation of circle is


(x – 0) (x – a) + (y – 1) (y – b) = 0  …(i) B X
O
Let the circle given by eq. (i) cut the x-axis at (h, 0) A C

h(h – a) + b = 0 This is true in case of circle and two secants


h – ah + b = 0
2
∴A, B, C and D lie on a circle.
The abscissa are roots of equation x – ax + b = 0
2
Sol 13: (C)

Sol 9: (C) x = 2y – 10 & x2 + y2 = 100


⇒ 4y2 – 40y + y2 = 0
⇒ 5y(y – 8) 0
∴ y = 8 (as point lies in 1st quadrant & x = + 6)
The line perpendicular to x – 2y + 10 =0 passing
through (6, 8) is (y – 8) = –2(x – 6)
2x + y = 20
It cuts the y-axis at (0, 20)
Since the centres lie on co-ordinate axes
Sol 10: (C) Let equation of circle be The centre are (1, 0), (–1, 0), (0, 1) and (0, –1)
x + y + 2gx + 2fy + c = 0
2 2
Consider two circles with centre (1, 0) & (0, 1)
Their point of intersection will lie on the line y = x
9 .76 | Circle

Putting y = x in (x – 1)2 + y2 = 1 Alternate:


⇒ 2x2 – 2x = 0 AM AB
Since, =2 ⇒ 1
=
AB BM
⇒ x = 1 & y = 1 (ignoring x = y = 0)
Let M be (h, k)
(1, 1) is the point
h k +3
By symmetry the other 3 points are Then, B ≡  , 
2 2 
(1, –1) (–1, 1) (–1, –1). Which lies on Circle.
It is a square of side 2 units Substitute to get the required Locus.
Area = 4 sq. units
Sol 16: (C) P = (0, 5)
Sol 14: (A) The y co-ordinate = 2, centre = (h, 2) & S1 = x2 + y2 + 2x – 4 = 0
radius = 2
x S2 = x2 + y2 – y + 1 = 0
On using the condition of tangency on y = ,
2 L1 = 25 − 4 = 21
2×2 −h
we get =±2 L2 = 21
5
⇒ h=4±2 5 ∴L1 = L2
But h > 0
Sol 17: (A) Let centre of circle be (h, k)
x coordinate is 4 + 2 5 .
∴h – 2k = 4 Y

Sol 15: (B) Let the midpoint of chord be (h, k) ⇒ h = 2k + 4 P(2,5)

∴Equation of chord is T = S1 ∴Centre is (2k + 4, k) C(h,k)


⇒ xh + 2( x + h) + yk – 3(y + k) + 9
= h2 + 4h + k2 – 6k + 9
X’ X
Since (0, 3) lies on this chord
2h + 3k – 3(3+k) = h2 + 4h + k2 – 6k
Locus of midpoint is
h2 + 2h + k2 – 6k + 9 = 0
Y’
1 3 Now CP ⊥ tangent
A (h, k) B M 5−k
∴ × 2 =−1
2 − 2k − 4
∴Let M be (x , y)
(2k + 2)
 3× 0 + x 9 + y  ∴5 – k =
(h, k) =  ,  2
 4 4 
∴5 – k = k + 1
Substituting in 1 we get locus of M.
∴k = 2
2 2
x 2(x)  y + 9  6 × (y + 9)
∴  + +  − +9=0 Center is (8, 2)
4
  4  4  4
(8 − 2) + (2 − 5)
2 2
⇒ x2 + y2 + 8x – 6y + 81 – 216 + 144 = 0 Radius = 3 5
=

⇒ x2 + 8x – (y – 3)2 = 0
Sol 18: (D) Let circle be S ≡ x2+y2+2gx+2fy + c = 0
S1 ≡ x2 + y2 = 1
S2 ≡ x2 + y2 + 2x – 3 = 0
M a them a ti cs | 9.77

S3 ≡ x2 + y2 + 2y – 3 = 0 [From (i) and (ii)]


⇒ S – S1 = 0 is the equation of chord of contact & it AB2 AD2
AC2 =
passes through centre of S1 AB2 − AD2
⇒ 2gx + 2fy + c + 1 = 0 AB.AD
AC =
Satisfying (0, 0) ⇒ c = – 1, AB2 − AD2
Similarly S – S2 = 0
Sol 21: (A) x3 + y3 + 3xy – 1 = 0
⇒ (2g –2)x + 2fy + 2 = 0
⇒ (x +y)3 – 3xy(x + y) + 3xy – 1 = 0
Satisfying (-1, 0), we get 2 – 2g + 2 = 0
⇒ (x +y)3 – 3xy(x + y – 1) – 13 = 0
⇒g=2
⇒ (x +y)3 – 13 = 3xy(x + y – 1)
Similarly, S - S3 = 0
⇒ (x +y –1){(x + y)2 + (x + y) + 1} – 3xy(x + y – 1) = 0
⇒ (2gx + (2f – 2)y + 2 = 0
We get,
(Satisfying (0, –1), we get ⇒ f = 2
∴ (x +y –1){(x + y)2 + (x + y) + 1 – 3xy} = 0
∴ Centre is (–2, –2)
(x +y –1) (x2 + y2 – xy + x + y + 1) = 0
Sol 19: (C) Let tangent from origin be y = mx For the curve x2 + y2 – xy + x + y + 1 = 0
Using the condition of tangency, we get 1
ab – h2 = 1 – >0
7m + 1 4
⇒ =5
m2 + 1 and
(7m + 1)2 = 25(m2 + 1) a h g 1 −1 / 2 1 / 2
⇒ 24m + 14m – 24 = 0
2 ∆ = h b f = −1 / 2 1 1/2
g f c 1/2 1/2 1
⇒ 12m2 + 7m – 12 = 0
⇒ 12m2 + 16m – 9m – 12 = 0  1 1  1 1 1  1 1
= 1 × 1 −  +  − −  +  − − 
 4 2  2 4 2  4 2
(4m – 3) (3m + 4)
3 4 3 3
∴m = and m = − = – =0
4 3 4 4
π
The angle between tangents = ∴ It is a point
2
Sol 20: (D) Since A, D, C lies on the circle with AC as
the diameter
AD ⊥ DC B
Previous Years’ Questions
Sol 1: (D) Equation of circle passing through a point
D (x1 , y1 ) and touching the straight line L, is given by
(x − x1 )2 + (y − y1 )2 + λL = 0

A C
∴ Equation of circle passing through (0, 2) and touching
x=0
∴ ∆ ADC  ∆ ABC ⇒ (x − 0)2 + (y − 2)2 + λx = 0  …..(i)
AC AD
⇒ =  …(i) Also, it passes through (-1, 0)
BC AB
⇒ 1 + 4 − λ= 0 λ ⇒ 5
Also, BC2 = AB2 + AC2  …(ii)
∴ Eq. (i) becomes,
AD2
AC2 = (AB2 + AC2) x2 + y 2 − 4y + 4 + 5x =0
AB2
⇒ x2 + y 2 + 5x − 4y + 4 =
0,
9 .78 | Circle

For x-intercept put y = 0 Sol 4: (A) Given, circle is inscribed in square formed by
2
⇒ x + 5x + 4 = 0 the lines
(x + 1)(x + 4) =
0 x2 − 8x=
+ 12 0 and y 2 − 14y =
+ 45 0
∴ x =−1, −4 ⇒ x 6 and
= = x =2, y 5 and= y 9

Sol 2: (B) For the point of intersection of the two given Which could be plotted as
curves y

C1 : y 2 = 4x
D (2,9) C (6,9)
y=9
and C 2 : x2 + y 2 − 6x + 1 =0
y

(1, 2)
y=5
A (2,5) B (6,5)

x’ x x’ x
O
(0, 0) (3, 0)
y’

(1, -2) Where ABCD clearly forms a square

y’ ∴ Centre of inscribed circle


= Point of intersection of diagonals
We have, x2 + 4x − 6x + 1 =0
= Mid point of AC or BD
⇒ x2 − 2x + 1 =0
 2 + 6  5 + 9 
⇒ (x − 1)2 =
0 = =  ,  (4,7)
 2   2 
⇒ x= 1 (equal real roots)
⇒ y =2, −2 ⇒ Centre of inscribed circle is (4, 7)

Thus, the given curves touch each other at exactly two Sol 5: (C) The line 5x-2y+6=0 meets
point (1, 2) and (1, -2).
The y-axis at the point (0, 3) and therefore the tangent
has to pass through the point (0, 3) and required length
Sol 3: (C) Here radius of smaller circle,
= x12 + y12 + 6x1 + 6y1 − 2
AC= 12 + 32 − 6= 2 Clearly, from the figure the radius
AC– = 02 + 32 + 6(0) + 6(3) − 2
of bigger circle
= 02 + 32 + 6(0) + 6(3) −=
2 25 5
=
2 2 2 2
r = 2 + [(2 − 1) + (1 − 3) ] = 25 5
=
2
r =9 Sol 6: (A) Since, the given circles intersect orthogonally.
⇒r= 3
2 ( g1 g2 + f1 f2 ) =
G + C2

∴ 2( −1)(0) + 2( −k)( −k) = 6 + k


A
3
⇒ 2k 2 − k − 6 =0 ⇒ k =− ,2
2 2

C C1(2,1)
(1,3) Sol 7: (C) Let O is the point at centre and P is the point
2 at circumference. Therefore, angle QOR is double the
angle QPR. So it is sufficient to find the angle QOR.
B
M a them a ti cs | 9.79

y Sol 11: Let the given circles C1 and C2 have centres O1


and O2 and radii r1 and r2 respectively.
Q (3,4)
Let the variable circle C touching C1 internally, C2

4,3)
externally have a radius r and centre at O

x’ R (- O (0,0)
x
C2 C1
O
r 2
2

P r
y’ O1 O
c
Now, slope of OQ, m1 = 4/3, slope of OR, m2 = ‒3/4 r1
Here, m1m2 = −1
Threfore, ∠QOR =
π/2
Which implies that ∠QPR =
π/4
Now, OO2 =
r + r2 and OO1 =
r1 − r
⇒ OO1 + OO2 =+
r1 r2
Sol 8: (B) Given, x2 + y 2 =
4
Centre ≡ C1 ≡ (0,0) and R1 =
2 Which is greater than

Again, x2 + y 2 − 6x − 8y
= − 24 0, then C2 ≡ (3, 4) O1O2 as O1O2 < r1 + r2 (C2 lies inside C1 )

and R 2 = 7 again, C1C2= 5= R 2 − R1 ⇒ Locus of O is an ellipse with foci O1 and O2


Since, the given circles touch internally therefore, they Alternate Solution
can have just one common tangent at the point of
contact. Let equations of C1 be x2 + y2 = r12 and of C2 be
(x − a)2 + (y − b)2 =
r22
Sol 9: Since, the tangents are perpendicular. Let centre C be (h, k) and radius r, then by the given
So, locus of perpendicular tangents to circle condition

x2 + y 2 =
169 is a director circle having equation (h − a)2 + (k − b)2 =r + r2 and h2 + k 2 =r1 − r
x2 + y 2 =
338
⇒ (h − a)2 + (k − b)2 + h2 + k 2 = r1 + r2

Sol 10: The equation of circle having tangent Required locus is


2x+3y+1=0 at (1, -1)
(x − a)2 + (y − b)2 + x2 + y 2 = r1 + r2
⇒ (x − 1)2 + (y + 1)2 + λ(2x + 3y + 1) = 0
Which represents an ellipse whose foci are at (a, b) and
x2 + y 2 + 2x(λ − 1) + y(3λ + 2) + (λ + 2) =0  ….(i) (0, 0).
Which is orthogonal to the circle having end point of
diameter (0, -1) and (-2, 3) Sol 12: Equation of any tangent to circle x2 + y 2 =
r 2 is
⇒ x(x + 2) + (y + 1)(y − 3) =
0 x cos θ + y sin θ = r  ….(i)
or 2 2
x + y + 2x − 2y − 3 =0  …(ii) Suppose Eq. (i) is tangent to 4x + 25y = 2
100 2

2(2λ − 2) 2(3λ + 2) x2 y 2
∴ ⋅1 + ( −1) = λ + 2 − 3 Or + =1at(x1 , y1 )
2 2 25 4
⇒ 2λ − 2 − 3λ − 2 = λ − 1 xx yy
Then, Eq. (i) and 1 + 1 = 1 are identical
⇒ 2λ = −3 ⇒ λ = −3 / 2 25 4
y1
∴ From Eq. (i) equation of circle, x1 / 25 4 = 1
∴ =
2 2 cos θ sin θ r
2x + 2y − 10x − 5y + 1 =0 25cos θ 4 sin θ
⇒ x1
= ,=
y1
r r
9 .80 | Circle

The line (i) meet the coordinates axes in A(r sec θ,0)   π π π π 
and β(0,r cosecθ) . Let (h, k) be mid point of AB.   θ− +θ+ θ− −θ−  
1 3 3 ⋅ cos 3 3  + 2r sin θ 
= r  2sin
3  2 2  
r sec θ r cosec θ    
Then,
= h = and k   
2 2
1
r r = [r{2cos θ cos π / 3} + 2r cos θ]
Therefore,
= 2h = and 2k 3
cos θ sin θ
1
25 4 = [r ⋅ cos θ + 2r cos θ=] r cos θ
∴ x=
1 and y=1 3
2h 2k 1  π  π
x2 y 2 and q
= [r sin  θ −  + r sin  θ +  + 2r sin θ]
As (x1 , y1 ) lies on the ellipse + 1
= 3  3  3
25 4
1  π  π
1  625  1  4  = [r{sin  θ −  + sin  θ + } + 2r sin θ]
We get  +   = 1 3  3   3 
25  4h2  4  k 2 
1
= [r(2sin θ cos π / 3) + 2r sin θ]
25 1 3
⇒ + =1
4h2 k 2 =
1
[r(sin θ) + 2r sin θ]
or 2
25k + 4h = 2 2 2
4h k 3
= r sin θ
Therefore, required locus is 4x2 + 25y 2 = 4x2 y 2
Now, (p,q) = (r cos θ,r sin θ) lies on x2 + y 2
Sol 13: Let the coordinate of point P be (2r cos θ, 2r sin θ) = r 2 , which is C1
We have, OA = r, OP = 2r
Sol 14: (A) Eq. of circle touching x − a × y at (1,0 ) u
Since, ∆OAP is a right angled triangle. 2 2
( x − 1) + ( y − k ) k2
=

C2 Circle passes through ( 2,3) , then


A
C1 2 2
- ( x − 1) + (3 − k ) k2
=

P  O 1 + 9 − 6k + k 2 =k2

B ⇒ 6k =
10
10
⇒ 2k =
3
cos φ =1 / 2
Sol 15: (B) The eq. of circle touching the
⇒ φ = π/3
∴ Coordinates of A and B are
( )
a − a × u at 3,0 is
2 2
{r cos(θ − π / 3),r sin(θ − π / 3)] and (1 − 3) + ( −2, −k ) =k 2

⇒ 4 + 4 + 4k + k 2 =
k2
  π    π
r cos  θ +   , r sin  θ +  ⇒ 4k =
−8
  3    3
⇒ k =−2
If p,q is the centroid of ∆PAB , then 2 2
Circle: ( x − 3) ( −2 − k ) =k 2
1
=p [r cos(θ − π / 3) + r cos(θ + π / 3) + 2r cos θ] Point (5, −2 )
3
2 2
=
1
[r{cos(θ − π / 3) + cos(θ + π / 3)} + 2r cos θ] (5, −3) + ( −2 + 2 ) = 2+2 = 4
3
Only (5, −2 ) lies on circle.
  π π π π 
  θ− +θ+ θ− −θ−  
1 3 3 ⋅ cos 3 3  + 2r cos θ 
r  2cos
3  2 2  
  
 
M a them a ti cs | 9.81

x2 y 2
Sol 16: (D) + 1
=
10 9

to is = (± 7 ,0 )
Circle having cente as ( 0,3)
2
x2 + ( y − 3) =γ 2 passes through focus , then
2 2
Sol 19: (C) x + y − 8x − 8y − 4 =
0
(± 7 )
2 2
+ ( 0 − 3) =γ2
2 2
2
(x − 4) + (y − 4) 36
=
7 + 9 =γ
Circles touch each other exotically
⇒ γ 2 =16
2 2
⇒ x2 + ( y − 3) =
16
2 k + 6= (n − 4 ) + (k − 4 )
⇒ x2 + y 2 − 6y − 7 =0 k 2 + 36 + 12k = h2 + 16 − 3h + k 2 − 3k + 16
⇒ h2 − 3h − 9 =20k
Sol 17: (D) If circles C and T touch each other externally
⇒ x2 − 3x − 20y − 4 =
0
then
2 2 If y < 0
1 + y= (1 − 0 ) + (1 − y )
2
⇒ (1 + y ) =1 + 1 − y 2 ( )
⇒ 12 + y 2 + 2y =1 + 1 + y 2 − 2y (4,4) (h,k)

1 K
⇒y=
4
(h,-k)

2 2 2
(1,1) ( −k + 6 ) = (n − 4 ) + (k − 4 )
⇒ h2 − 8h + 4k − 4 =
0
T ⇒ x2 − 8x + 4y − 4 =
0
Locus is Parabola

Sol 18: (B) ( x − 2 ) + ( y − 3 ) =


25
2 2
JEE Advanced/Boards

Exercise 1
2 2
( x + 3) + ( y + 9 ) 64
=

2 2
C1C2 = ( 2 + 3) + (3 + 9 ) Sol 1: The equation of line through origin is y = mx
Let point on circle be (h1, mh1) and (h2, mh2)
= 25 + 144
S = x2 + y2 – 8x – 6y + 24 = 0
= 169 13
=
O = origin
r1 + r2 = 3 + 5 = 13
(i) The equation of chord of S whose mid-point is
⇒ r1 + r2 =
c1 c2 (h, k) is
hx+ ky – 4(x + h) – 3(y + k) + 24
Circles touch each other externally therefrom, three
tangents are possible = h2 + k2 – 8h – 6k + 24
9 .82 | Circle

Since it passes through origin Sol 3: Equation of common chord


∴–4h – 3k = h2 + k2 – 8h – 6k = x2 + y2 + 4x + 22y + a–(x2 + y2 – 2x + 8y - b)
∴ Locus of point is = 6x + 14 y + (a + b) = 0
x2 + y2 – 4x – 3y = 0 Now centre of second circle lies on this
(ii) OP = OA × OB ∴ 6 × 1 + 14 × (– 4) + (a + b) = 0
It is a known property that ∴ (a + b) = 50
T B Now a , b > 0
∴ AM > GM
A a+b
⇒ > ab
2
O ∴ 25 > ab
ab < 625
OA × OB = OT2=OP2
∴OP = OT = constant k Sol 4: x2 + y2 = 1

OT = S(0,0) = 24 y−4
Z=
x−7
∴The locus of P is the circle of radius 24 and centre 4
= origin In this the slope from the point (7, 4); tanq2 =
7
⇒ x2 + y2 = 24 is the locus of P
(7, 4)
2OA.OB OA. OB m
(iii) OP = =
OA + OB OM 

∴OP × OM = OA × OB
∴ A and M are harmonic conjugates of P & B
2 1

A P M B
AM AB AM
∴ = ⇒ =2
PM MB PM
∴ P is mid-point of A & M
m − tan θ2
tanθ = tan (θ1 – θ2) =
∴Locus of P: 1 + mtan θ2
x + y – 8x – 6y + 24 – (x + y – 4x – 3y) = 0
2 2 2 2
r m− 4 /7
∴4x + 3y = 24 is locus of P =
 tangent 4
1 + m×
7
Sol 2: Radius of given circle= 4 + 2 − C = 6 − C
1 m− 4 /7 7m − 4
⇒ ± = =
r = 2 r1 and r1 = 2 r2, r2 = 2 r3 8 4m 7 + 4m
1+
Sum of radii of all circles 7
3 5
r r r r ∴M = and m =
=r+ + +...............= ⇒ =2 4 12
2 2 1 1
1− 1− 2×3 6 ×5
2 2 ∴2M + 6m = + =4
∴r = 2 – 2 4 12

⇒ 6−C = 2 – 2 ⇒6–C=4+2–4 2

∴C = 4 2 = 32 ⇒ n = 32
M a them a ti cs | 9.83

Sol 5: The radical axis of 2 circles is – 6 + λ(–4) = 0


 3 −3
 2g −  x + (2f – 4)y = 0 λ=
 2 2
Centre of the given circle = (–1, 1) Equation of circle is

and radius = 1 x2 + y2 – 8 – 3x – 3y + 12 = 0

Since it is a tangent to the circle ⇒ x2 + y2 – 3x – 3y + 4 = 0


 3
(2f − 4) −  2g − 
2 Sol 8: C: x2 + y 2 + y − 1 + k ( x + y − 1 ) =0
⇒  =1
2 It is the family of circle passing through points of
 3
 2g −  + (2f − 4)
2
intersection of a circle & L.
 2 
2
Putting x = 1 – y in C1
 3  3
⇒ (2f – 4) +  2g −  + 2(2f – 4)
2
 2g −  We get y2 – 2y + 1+fy2 + y – 1 = 0
 2  2
1 1
2 ⇒ 2y2 – y = 0 ⇒ y = 0, & x = 1 or
 3 2 2
=  2g −  + (2t – 4)2
 2 1 1
∴ The point of intersection are A(1, 0) and B  , 
 3 2 2
∴ (2f – 4)  2g −  = 0
2 The minimum value of radius is when point act as

diameter
3
∴ Either f = 2 or g = 2 2
1
4 1 1 1 1 1
∴rmin =   +  = × 2 =
2 2 2 2 2 2 2
Sol 6: The line passing through points A(3, 7) and
B(6, 5) is 2x + 3y – 27 = 0 Sol 9: The equation of circle co-axial with 2 circle is S1
+ lS2=0
The family of circles passing through these points is
2x2 + 2y2 – 2x + 6y – 3 +
(x – 3) (x – 6) + (y – 7) (y – 5) + λ (2x + 3y – 27) = 0
λ(x2 + y2 + 4x + 2y + 1) = 0
⇒ x2 – 9x + 18 + y2 – 12y + 35 + λ (2x + 3y – 27) = 0
= (2 + λ)x2 + (λ + 2)y2 + (4λ – 2)x
∴Chord of contact = s1 – s2
+ (2λ +6)y + λ – 3 = 0
⇒ –5x – 6y + 50 + λ(2x + 3y – 27) = 0
2 − 4λ 1 − 2λ
⇒ L1 + λL2 = 0 xcentre = =
2(2 + λ ) λ+2
The point which passes through intersection of L1 and −(λ + 3)
L2 is the point of intersection of all λ ycentre =
λ+2
5x + 6y = 50 5
Radical axis of the two circle is s1 – s2 ≡ 5x – y + =0
2
2x + 3y = 27 Centre lies on radical axis

∴x = 2 & y =
23
∴ P=
 23 
 2,  ∴5 ×
(1 − 2λ ) + λ+3 5
+ =0
3  3  λ+2 λ+2 2
⇒ 10 – 20λ + 2λ + 6 + 5λ + 10 = 0
Sol 7: The locus of point of intersection of mutually
perpendicular tangent is the director circle ⇒ 13λ = 26∴λ = 2

∴Locus of point = x2 + y2 = 8 ∴Equation of circle is 4x2 + 4y2 + 6x + 10y – 1 = 0

The equation of family of circle touch a given circles &


at (x1, y1) is S + λ(L) where L = tangent Sol 10: s1 ≡ x2 + y2 – 4x – 6y – 12 = 0

x2 + y2 – 8 + λ (x × 2 + y × 2 – 8) =0 s2 ≡ x2 + y2 + 6x + 4y – 12 = 0

Now this passes through (1, 1) s3 ≡ x2 + y2 – 2x – 4 = 0


9 .84 | Circle

The circle passing through point of intersection of s1 (x + 2) (y – 3) = 0


and s2 is s ≡ s1 + ls2 = 0
The centre is point of intersection of these two lines
⇒ x + y – 4x – 6y – 12 + λ
2 2
c ≡ (–2, 3)
(x2 + y2 + 6x + 4y – 12) = 0
g = 2 and f = – 3
⇒(λ + 1)x2 + (λ + 1)y2 + (6λ – 4)x
s ≡ x2 + y2 + 4x – 6y = 0
+ (4λ – 6)y – 12 (λ +1) = 0
s1 = x2 + y2 – kx + 2ky – 8 = 0
Since it is orthogonal to s3
Since s & s1 are orthogonal
∴2gg1 + 2ff1 = c+c1
∴2gg1 + 2ff1 = 0 - 8
(6λ − 4) −12(λ + 1) ⇒ 2( – k) + (–3) × 2k = 0 – 8
⇒ x – 1 + 0= –4
λ +1 (λ + 1)
∴k = 1
∴4 – 6λ = – 16 (λ + 1)
Sol 13: Since the circle cuts co-ordinate axis orthogonally
10λ = – 20
∴C ≡ (0, 0)
λ=–2
∴s ≡ x2 + y2 – a2 = 0
s ≡ – x2 – y2 – 16x – 4y + 12 = 0
s ≡ x2 + y2 – 14x – 8y + 64 = 0
∴x2 + y2 + 16x + 14y – 12 = 0
Since s & s1 are orthogonal
Sol 11: Let s = x + y + 2gx + 2fy + c = 0
2 2
∴ 2 × 0 × –7 + 2 × 0 × –4 = – a2 + 64
Now (–g, –f) lies on 2x – 2y + 9 = 0 ∴ a2 = 64
⇒–2g + 2f + 9 = 0 ∴ s ≡ x2 + y2 – 64 = 0
and it is orthogonal to x2 + y2 – 4 = 0
Sol 14: Let the given circles
2g × 0 + 2f × 0 = c – 4
S1 ≡ x2 + y2 + 2g1x + 2f1y + c1 = 0
C=4
S2 ≡ x2 + y2 + 2g2x + 2f2y + c2 = 0
9
and f = g –
2 Let the circle orthogonal to the two circles be
 9
s = x2 + y2 + 2gx + 2  g −  y + 4 = 0 x2 + y2 + 2gx + 2fy + c = 0
 2
∴2gg1 + 2ff1 = c1 + c
s ≡ x + y – 9y + 4 + 2g (x + y)
2 2

and 2gg2 + 2ff2 = c + c2


∴ It passes through point of intersection of S and L
⇒ 2g(g1 – g2) + 2f(f1 – f2) = c1 – c2
Putting x = – y in s
Now the centre is (–g, –f)
2y2 – 9y + 4 = 0 ⇒ 2y2 – 8y – y + 4 = 0
1 ∴x =–g & y = –f substituting instead of g & f
1
∴y = or y = 4 & x = – , – 4
2 2 We get 2x(g1 – g2) + 2y(f1 + f2) = (c1 – c2)
 −1 1  Which is the radical axis & (straight line)
∴The points are  ,  & (–4, 4)
 2 2 The locus of centres of given s1, s2 is s1 – s2=0
Sol 12: Let the equation of circle be 4x + 5x – 6y – 4y + 7 = 0
x + y + 2gx + 2fy = 0
2 2
9x – 10y + 7 = 0
(it passes through origin)
The line pair is Sol 15: Consider a point circle at (–2, 7)

xy – 3x + 2y – 6 = 0 (x + 2)2 + (y –7)2 = 0

x(y – 3) + 2(y – 3) = 0 Now the equation a circle touching a circle at point is


s + λL
M a them a ti cs | 9.85

Where L is tangent to L 2nd smallest circle be r1 & 2nd largest circle be r2


∴s ≡ (x + 2)2 + (y – 7)2 + λ(x + y –5) = 0
⇒ x2 + y2 + (λ + 4)x + (λ − 14)y + (53 – 5λ) = 0
r1 r r2
∴s2 ≡ x2 + y2 + 4x – 6y + 9 = 0 8 15

Since s & s2 are orthogonal


∴(λ + 4) + (λ – 14) x – 3 = 53 – 5λ + 9
∴ r12 = 8r & r22 = 18 r
⇒4λ = 12 ⇒ λ = 3
r2 = r1 r2
∴ Equation of circle
∴r4 = 8r × 18 r
s ≡ x2 + y2 + 7x – 11y + 38 = 0
r= 8 × 18 = 12

Sol 16: Let the circle be x2 + y2 + 2gx + 2fy + c = 0


Sol 18: The pair of lines is
(–6, 0) lies on the circle
7x2 – 18 xy + 7y2 = 0
∴ 36 – 12g + c = 0  … (i)
Since co-eff of x = coeff of y.
The power of (i, i) is 5
angle bisectors are
⇒ 1 + 1 + 2g + 2f + c = 5
(x – y) = 0 & x + y = 0
⇒ 2g + 2 f + c = 3  … (ii)
Since the given circle lies in the 1st quadrant
S is orthogonal to
∴Our circle should also lie in the 1st quadrant
x + y – 4x – 6y – 3 = 0
2 2
∴ Its centre should lie on y = x
⇒ 2g(– 2) + 2f(– 3) = c – 3
Centre ≡ (h , h)
∴ 4g + 6f + c – 3 = 0  … (iii)
Now (x – h)2 + (y – h)2 = k2
From (ii) and (iii)
Let y = mx be equation of tangent
2g + 2c = 6 ⇒ g + c = 3  … (iv)
h − mh
=R
From i and iv g = 3
1 + m2
−3
∴c = 0 & f = ∴R2(m2 + 1) = h2(m2 – 2m + 1)
2
s ≡ x2 + y2 + 6x – 3y = 0 ∴(R2 – h2)m2 + 2h2m + R2 – h2 = 0
Comparing to pair of lines
Sol 17: Radius of largest circle = 18 2h2 −18
We get =
Radius of smallest circle = 8 R −h2 2 7

14h2 = – 18R2 + 18h2


∴4h2 = 18R2
r1 r2 r3
3
∴h = R h is in 1st quadrant
2
When 3 circle touching each other have direct common Since C touches C1
tangent
= (R1 – R) = distance between centres
The radius of the middle circle is GM of radius of other 2 2
2 circles  3R   3R 
∴(4 2 – R)2 =  4 −  +4 − 
∴ r22 = r1r3  2  2

In the given problem  3R 


⇒4 2 – R = ± 2 4 − 
Let radius of middle circle be r,  2
9 .86 | Circle

 3R  Equation of common chord is


∴4 2 – R = – 2 4 − 
 2 2(b – a)x + 2(b – a)y = b2 – a2
3 2R ∴2x + 2y = a + b
8 2 = +R
2 It passes through (a, a)
8 2 =4R 4a = a + b
b = 3a
R=2 2
b
⇒ =3
3× 2 2 a
h= =6
2 (D) C2 passes through centre of C1
Equation is (x – 6)2+(y – 6)2 = (2 2 )2 ∴a2 + a2 – 2ab – 2ab + b2 = 0
⇒ b2 – 4ab + 2a2 = 0
Sol 19: (r, p, q) (A) Centre of C1 ≡ (a, a) & radius = a for
2
C2 centre ≡ (b, b) & radius = b b b
⇒   − 4  + 2 =0
C1 & C2 cannot touch other internally a a

b 4 ± 16 − 8 4±2 2
∴ = = =2± 2
a 2 2
C1 But b > a
C2
∴b = 2 + 2
a

Sol 20: y = x + 10 & y = x – 6 are tangents


(b − a)2 + (b − a)2 = (b + a)
The centre of circle passes through
∴ 2 (b – a) = (b + a)
(10 − 6)
y=x+ =y=x+2
∴( 2 – 1)b = ( 2 + 1)a 2
1
Also radius, = ⊥ distance between lines
b 2 +1 2
∴ = =3+2 2
a 2 −1 1 c − c2 1 16
= × 1 = × =4 2
2 2
a +b 2 2 2
(B) Equation of
C1 ≡ x2 + y2 – 2ax – 2ay + a2 = 0 ∴Circle is (x – h)2 + (y – (h + 2))2 = (4 2 )2

C2 ≡ x2 + y2 – 2bx – 2by + b2 = 0 h + k = 2h + 2 = a + b a

C1 & C2 are orthogonal Since y – axis is tangent

2(–ax – b) + 2(–ax – b) = a2 + b2 ∴h = Radius

4ab = a2 + b2 ∴h = 4 2

a2 – 4ab + b2=0 and h + k = 2h + 2 = 8 2 + 2


∴a + b = 10
b 4 ± 16 − 4 4 ± 12
= =
a 2 2
Sol 21:
C2 C3
=2± 3 C1
b
But b > a ∴ =2+ 3 C
a r1 r2
(C) C1 and C2 intersect such that common chord is longest
∴ C2 bisects C1
M a them a ti cs | 9.87

Since, centres of the Circle are collinear.  (2 + 3λ ) 


2
∴(λ – 1) + 
2
 – λ – 2 = 13
2r1 + 2r2  2 
∴Radius of bigger circle = = 14
2 ∴l2 = 4 ∴ λ = ± 2
Now distance of point of intersection from centre = R –
∴ Equation of circles are
(2r1) = 14 – 2 × 4 = 6 = d
x2 + y2 + 2x – 8y + 4 = 0 or x2 + y2 – 6x + 4y = 0
Length of chord

= 2 R 2 − d2 = 2(14)2 – (6)2= 4 40 = 8 10 Sol 24: Equation of circle touching other. circle is at


point is s + λ (L) = 0
m + n + p = 1 + 8 + 10 = 19
Where L is equation of tangent at the point
Sol 22: Equation of a circle passing through two points x2 + y2 + 4x – 6y – 3 + λ (2x + 3y
(x – 1) (x – 4) + (y – 7) (y – 8) + λ(L) =0 + 2(x + 2) – 3 (y + 3) – 3) = 0
L passing through (4, 7) & (1, 8) It passes through (1, 1)
−1
is y – 8 = (x – 1) −(1 + 1 + 4 − 6 − 3) 3 −3
3 ∴λ = = =
(2 + 3 + 6 − 12 − 3) −4 4
3y + x – 25 = 0
∴ Equation of circle is
∴ (x – 1) (x – 4) + (y – 7) (y – 8) + λ(3y + x – 25) =0
4x2 + 4y2 + 16x – 24y – 12 – 3 (4x – 8) = 0
(5, 6) satisfies this equation
4x2 + 4y2 + 4x – 24y + 12 = 0
(4 + 2)
λ=+ =3
+2 x2 + y2 + x – 6y + 3 = 0
Equation of circle is x2 + y2 – 2x – 6y – 15 = 0
Let the points of intersection of tangent be (h , k) Exercise 2
chord of contact is
Single Correct Choice Type
hx + ky – (x + h) – 3(y + k) – 15 = h2 + k2 – 2h – 6h – 15
(h – 1)x + (k – 3)y + h + 3k – h2 – k2 = 0 Sol 1: (A) Since BAC = 90º

(h − 1) 5 locus of A is the circle with (3, 0), (–3, 0) as diameter


Now, =  … (i)
h + 3k − h2 − k 2 17 Let A = (h, – k)
k −3 1 (h –3) ( h + 3) + k2 = 0
and =
h + 3k − h2 − k 2 17 Now, centroid
h−1 h+3−3 k +0 +0
= 5 ⇒ h –1 = 5( k –3) C(x, y) =  , 
k −3  3 3 
h = 5(K – 3) + 1 Substituting h, k in terms of (x, y)
Substituting in 1 we get k = 2 (3x – 3) (3x + 3) + (3y)2 = 0
∴h = – 4 x2 + y2 = 1 is the equation of centroid
∴Point is (–4, 2)
Sol 2: (C) | y | = x + 1 & (x –1)2 + y2 = 4
Sol 23: The equation of circle which touches a given Substituting value of | y |
line at a point is
(x –1)2 + (x + 1)2 = 4
(x–1)2 + (y – 1)2 + λ(2x – 3y +1) = 0
x2 = 1
∴x2 – y2 + 2(λ – 1)x – (2 + 3λ)y + λ + 2 = 0
x=±1
R= 13 For x = –1; y = 0
9 .88 | Circle

For x = +1; | y | = 2 ∴ y ± 2 3 − 2m
Now y = mx is tangent to the circle ∴ = 13
∴ Three possible solutions are possible 1 + m2
4m2 – 12m + 9 = 13m2 + 13 ⇒ 9m2 + 12m + 4 = 0
Alternate method
9m2 + 6m + 6m + 4 = 0
Plotting the graph of |y| = x + 1 and (x – 1)2 + y2 = 4
(3m + 2)2 = 0
2
m=–
3
∴Equation of line is 3y + 2x = 0
-1 1

Sol 6: (A) The circumcenter of triangle A,B,C is (0, 0)


Let c ≡ (h , k)
We can directly see that three possible intersection are 1 +h 1 +k 
And centroid (c1) is  , 
possible  3 3 
Let the orthocentre be (x, y)
Sol 3: (B) Line 1 passes through (3, 1) and The centroid divides O and C in ratio 2 : 1
Line 2 passes through (1, 3)
1 +h 1 +k  x y
Lines L1 and L2 are ⊥ ∴ locus of point of intersection is
∴ ,  =  , 
 3 3  3 3
a circle with (3, 1) & (1,3) as ends of diameter
∴h = (x – 1) and k = (y – 1)
Locus of points is (x – 3) (x – 1) + (y – 1) (y – 3) = 0
(x –1)2 + (y – 1)2 = 1
∴x2 + y2 – 4x – 4y + 6 = 0
∴x2 + y2 – 2x – 2y + 1 = 0

Sol 4: (B) Plotting the point on a graph


Sol 7: (D) Centre of circle is (– 8, –6)
y2 A B Equation of line is y = 2x + 5
∴ Q is the foot of perpendicular of (– 8, –6) on 2x – y
+5=0
y1 D C
x − ( −8) y − ( −6) −( −5)
∴ = =
x1 x2 2 −1 5

It is not necessary that ∴x = – 6 & y = – 7

|x2 – x1| = |y2 – y1| ∴Q ≡ (–6, –7)

With (x2, y1) & (x1, y2) as ends of diameter ∠ABC = 90º
Sol 8: (A) Centre of C1 = (2, 0) R1 = 4 & R2 = 4
and ∠ADC = 90º
Centre of C2 = (– 2, 0)
∴ ABCD are concyclic

Sol 5: (D) Let A = (0, 6), B = (5, 5) & C = (–1, 1)


−1
Slope of AB = & mAC = 5 (-2, 0) (2, 0)
5
∴ AB ⊥ AC
Circumcentre is midpoint of BC
∴The other 2 points of rhombus lie on y axis put in
O = (2, 3) x = 0 we get
1
And radius = 62 + 4 2 = 13 Y=±2 3
2
M a them a ti cs | 9.89

∴ Length of 1st diagonal is (2 – (–2) = 4 and length of


Length intercepted = 2 R 2 − ( ⊥ dis tance)2
2nd diagonal = 4 3
2
1 1 O−O+C 2 C2
Area of rhombus = ab = ×16 3 = 8 3 sq. units ∴ l1 = 2 22 −   =2 2 −
2 2  2  2

(5 − 7 + C)2
Sol 9: (A) From (3, 4) chords are drawn to l2 = 2 (3)2 −
2
x2 + y2 – 4x = 0 l1 = l2
Let mid points of chord be (b, h) C2 (C − 2)2
∴4 – =9–
∴h2 + k2 – 4h = xh + yk –2(h + x) 2 2
Now (3, 4) pass through these chords ∴C2 – 4C + 4 – C2 = 10

∴h2 + k2 – 4h = 3h + 4k –2(h + 3) 3
C=–
2
∴Locus of mid-point is x2 + y2 – 5x – 4y + 6 = 0
3
Line is y = x –
2
Sol 10: (B) Let p = (x, y)
2x – 2y – 3 = 0
 20 cos θ + 15 20 sin θ + 15 
(x, y) =  , 
 5 5  Sol 14: (D) Equation of circle is x2 + y2 = r2
x −1 y −1 Let P ≡ (a, b)
cos θ = & sinθ =
4 4
Let the midpoint of a point (h, k) on circle &
(x – 1)2 + (y –1)2 = 16 P(a, b) be M(x, y)
This is a circle. a+h b+k 
(x, y) =  , 
 2 2 
Sol 11: (B) (3, 4) & (–1, –2) are ends of diameter h = 2x – a ; k = 2y – b
(x – 3)(x + 1) + (y – 4)(y + 2) = 0
(2x –a)2 + (2y –b)2 = r2 is locus of M
x + y – 2x – 2y – 11 = 0
2 2
2 2 2
 a  b r
x −  +y −  =  
Sol 12: (A) Shortest distance from line to circle  2  2 2
= ⊥ distance – radius
Multiple Correct Choice Type
Centre of circle ≡ (3, –4) & radius = 5
Sol 15: (C, D) Let h, k be the point of intersection
9 − 16 − 25 32
∴ ⊥ distance = =
25 5  k   k 
∴ Slope of lines is   and  
h−1 h+1
32 7
∴shortest distance = –5= For point (1, 0) and (–1, 0)
5 5
tan θ − tan θ1
And tan (θ – θ1) =
Sol 13: (A) Slope of the line is 1 1 + tan θ tan θ1
∴ The angle between lines is either 45º or 135º
∴y = x + c
θ – θ1 = 45º or 135º
The two circle are
s1 ≡ x2 + y2 = 4 k k

c1 = (0, 0) & R = 2 ± 1 = h−1 h+1
k2
1+ 2
s2 ≡ x2 + y2 – 10x – 14y + 65 = 0 h −1
c2 = (5, 7) & R = 3 2k
±1=
h + k2 − 1
2
9 .90 | Circle

∴h2 + k2 – 2k – 1 = 0 C ≡ (0, 1) R = 2 S : (x + 1)(x –5) + (y –1) (y – 5) = 0

∴h2 + k2 + 2k – 1 = 0 C ≡ (0, –1) R = 2 ∴x2 + y2 – 4x – 6y = 0


Center c = (2, 3) ; r = 13
Sol 16: (B, C, D) s1 ≡ x + y + 2x + 4y + 1 = 0
2 2 3
The line joining origin to center is y = x
2
s2 ≡ x2 + y2 – 4x + 3 = 0 ∴3x – 2y = 0
s3 ≡ x + y + 6y + 5 = 0
2 2
The points will lie on the line ⊥ to 3x – 2y = 0 & passing
Radical axes of s1 and s2 is through (2, 3) at a distance of r from (2, 3)

6x + 4y – 2 = 0 −2 −2
L:y–3= (x –2) tanθ =
3 3
3x + 2y - 1 = 0
2x + 3y – 13 = 0
Radical axes of s3 and s2 is
Let points be (h, k)
6y + 4x + 2 = 0
When θ is in 2nd quadrant
3y + 2x + 1 = 0
sinθ > 0 & cosθ < 0
5x + 3y = 0
h = a + r cosθ ; k = a + r sinq
x = 1 y = –1
−3
(1, -1) is the radical centre ∴h = 2 + 13 ×
13
It is a known property that circle which is orthogonal to 2
3 circle has its center equal to radical center & radius k=3+ 13 ×
13
= length of tangent from radical center to any circles.
∴P1 = (–1, 5)
Radices = 1+1+ 2− 4 +1 = 1
When q lies in 4th quadrant
Equation of orthogonal circle is (x –1) + (y +1) = 1
2 2
sinθ < 0 & cosθ > 0
This circle touches both x & y axis.
3
h=2+ × 13
Its x & y-intercept are 1 13
 −2 
k=3+  × 13 
Sol 17: (B, C) c1 ≡ x2 + y2 – 4x + 6y + 8 = 0
 13 
c2 ≡ x2 + y2 – 10x – 6y + 14 = 0 ∴P2 = (5, 1)
Centre of c1 ≡ (2, –3)
Sol 19: (A, C, D) s1 : x2 + y2 + 2x + 4y – 20 = 0
Centre of c2 ≡ (5, 3)
s2 ≡ x2 + y2 + 6x – 8y + 10 = 0
r1 = 4+9−8 = 5
c1 = (–1, –2) & c2 = (–3, 4)
r2 = 25 + 9 − 14 = 2 5
r1 = 12 + 22 + 20 = 5
c1c2 = r1 + r2
r2 = 32 + 42 − 10 = 15
c1c2 = (5 − 3)2 + (6)2 = 3 5
c1c2 = 22 + 62 = 40 = 2 10
∴ c1 & c2 touch each other
c1c2 = r1+r2
∴ Radical axis is the common tangent and the mid-
point of c1c2 doesn’t lie on radical axis as their radius and c1c2 > | r1 – r2 |
are not the same. ∴ The two circles intersect each other at 2 points
2gg1 + 2ff1 = 2 × 3 + 4x – 4 = – 10 = c + c1
Sol 18: (B, D) A = (–1, 1); B = (0, 6); C = (5, 5)
AB ⊥ BC
∴The circle passing through ABC will have AC as a
diameter
M a them a ti cs | 9.91

The 2 circle are orthogonal 2 circle are possible


When all 3 lines are parallel no circles are possible
r1 r2
C1 C2
Sol 21: (A, B, D) c1 = (x + 7)2 + (y – 2)2 = 25
∴r1 = 5
c2 is director circle of c1
∴ r2 = 5 2

Length of common tangents And c3 director circle of c2


∴ r3 = 5 2 × 2 = 10
= (c1c2 )2 − (r2 − r1 )2 = 40 − (5 − 15)2
Area enclosed by c3 = pr2 = 100 p
= 10 15 = 5(12/5)4 Area enclosed of c2 = π ×( 2 r)2 = 2pr2
The equation of common chord is s1 – s2 = 2 times area enclosed by c1
⇒ 4x – 12y + 30 = 0
⇒ 2x – 6y + 15 = 0 Sol 22: (B, C) S1 ≡ x2 + y2 – 2x – 4y + 1 = 0 r1 = 2
−2 + 12 + 15 25 G ≡ (1, 2 ) , r1 =
2
Perpendicular from c1 on this ⇒ =
40 40
S2 ≡ x2 + y2 + 4x + 4y – 1 = 0
Length of common chord = 2 r 2 − a2 C2 ≡ ( −2, − 2 ) , r2 =3
2
 25  625 C 1C 2 = 32 + 42 = 5
=2 25 −   =2 25 −
 40  40 The two circle touch each other externally and common
tangent is S2 – S1 = 0
75 10 3 3
=2 = =5 6x + 8y – 2 = 0
8 2 2 2
3x + 4y – 1 = 0
Sol 20: (A, C, D) Consider 2 lines not parallel to one
another and when the third line passes through Sol 23: (A, C, D) S1 ≡ x2 + y2 – 6x – 6y + 9 = 0
intersection of both lines, no circle is possible. S2 ≡ x2 + y2 + 6x + 6y + 9 = 0
When the third line doesnot pass through point of C1 = (–g, –t) = (3, 3)
intersection of the lines & is not parallel to either of
them 4 circle are possible. r1 = 32 + 32 − 9 = 3

and C2 = (–3, –3)

C1 r2 = 32 + 32 − 9 = 3

C 1C 2 = 62 + 62 = 6 2
C3 C4
r1 + r2 = 6
C2
They do not intersect with each other
Since their radius are same
When the 3 line is parallel to one of the line then
rd
∴ External direct common tangents are parallel

r

C1
M

Also, the point of intersection of transverse common


tangents is midpoint of C1 and C2 (same radii)
9 .92 | Circle

M = (0, 0) Every line L is normal to circle


r 3 1 Statement-I is true & statement-2 is false
sinθ = = =
MC1 32 + 32 2
Sol 27: (A) Length of tangent from (13, 6)
θ = 45º
Angle between tangents = 2θ = 90º = 132 + 62 − 13 × 6 + 8 × 6 − 75 = 10

Sol 24: (B, C) S1 ≡ x2 + y2 + px + py – 7 = 0


S2 ≡ x2 + y2 – 10x + 2py + 1 = 0
C  P
S1 & S2 are orthogonal
∴2gg1 + 2ff1 = c + c1
⇒ p(–5) + p . p = – 6
32 + ( −4 ) + 75
2
∴ Radius of circle
= = 10
⇒ p2 – 5p + 6 = 0
∴tanθ = 1
⇒ P =2 or p = 3
∴θ = 45º
Sol 25: (A, B, D) (A) Two circles having the same center. Angle between tangents = 2θ = 2 × 45 = 90º
Have infinitely many common normal.
Director circle of a circle S1 is such that the angle
(B) Radical axis is always perpendicular to the line between the tangents drawn from any point on director
joining center but it does not necessarily bisect the line circle to S1 is 90º
joining the centres. It bisects only when r1 = r2
(C) Let the centres of the two circles be C1 & C2. Sol 28: (D) (1, 5) lies outside the circle
Consider a point O, on radical axis centres which lies on as 1 + 25 – 2 – 7 = 17 > 0
the line C1C2
Now OC12 = r12 + OT12
C1

OC22 = r22 + OT22


Since length of tangent is same
∴ OC12 < OC22 if r1 < r2 C2
⇒ OC1 < OC2 ∴Two circles shown C1, C2 are possible
∴ Statement-I is false
O
C1 r1 C1 Sol 29: (A) Since x + y – 2 = 0 is ⊥ bisector of C1C2
r2
T1 Radius of both the circles is same
T2
Since length of common chord = 2 2

(D) Consider two circles having same centre these ABCD is a square since diagonals are equal & ⊥ to
circles donot have a radical axis each other B
r r
Assertion Reasoning Type

L L
A C
 1  2  (0, 0) 2 2 (2, 2)
Sol 26: (C) L : k(x − y − 4) + 7x + y + 20 = 0 r r
L are the lines passing through intersection of L1 & L2
D
Point of intersection is (– 2, – 6)
Which is center of circle c
M a them a ti cs | 9.93

When their centres are mirror image of each other AH2 – HB2 = AP × AQ – [AB × AP
1
then the common chord bisects C1C2 and × length + AB × AQ – AB2 – APAQ]
2
1 AB2 = AP × AQ – [AB(AP + AQ) – AB2 – APAQ]
of common chord = c1c2
2 2AP × AQ
∴ AB =
AP + AQ

∴ Statement-I is true
C 1 C2
2 Statement-II: AK2 = AB × AO & AK2 = AP × AQ
C1 C2
C1 C2 (AP + AQ)
2 ∴AB × = AP × AQ
2
tanθ = 1 2AP × AQ
∴AB =
(AP + AQ)
θ = 45º
The circles are orthogonal
Comprehension Type
When the centres are mirror image & length of chord
= distance between centres then the two circles are
Paragraph 1: (32-34)
orthogonal. The inverse is not true
∴ Statement-II is wrong Sol 32: (B) A : {(x, y) : y ≥ 1 }
B : {(x, y) : x2 + y2 – 4x – 2y – 4 = 0}
Sol 30: (A) Let AB = diameter
C : {(x, y) : x + y = 2

A
B
C
P
A B 2

The circle with AB as diameter is


There is only one point P of intersection of region A,
(x – x1) (x – x2) + (y – y1).(y – y2) = 0 B, C
when C is obtuse, then C lies inside the circle
D(x3, y3) < 0 Sol 33: (C) B : x2 + y2 – 4x – 2y – 4 = 0

(Power of a point inside a circle < 0) ⇒ 2x2 + 2y2 – 8x – 4y – 8 = 0


⇒ (x – 5)2 + (x + 1)2 + (y – 1)2 + (y – 1)2 – 36= 0
Sol 31: ∴f(x) = 36
H
Sol 34: (C) S is director circle of B
∴B : (x – 2)2 + (y – 1)2 = 9
Q A
O B P s : (x – 2)2 + (y – 1)2 = 18
Arc of B = 9p
K
Arc of s = 18p
Since KPHQ are concyclic Area of S-Area of B = 9π
∴ PB × BQ = HB × BK = (HB)2
∴ (AB – AP) (AQ – AB) = (HB)2
Also AH2 = AP × AQ (from property of tangents)
9 .94 | Circle

Paragraph 2: (35-36) 1 10
= × =1
2 5/2
Sol 35: (D) Let m be slope of tangents
∴ (y – 2) = m( x – 4) are equation of tangent θ = 45º
π 3π
s = x2 + y2 = 4 Angle at minor arc = π – =
4 4
For tangents c2 = a2 (1 + m2)
∴(2 – 4m)2 = 4(1 + m2) Sol 39: (C)
A
12 m2 – 16m = 0 1
/4 
4 m(3m – 4) = 0 90
O
4 4
m = 0 or m = ⇒ tanθ =
3 3
θ ∈ (45º, 60º) Ans.(D) B

π π π
Sol 36: (B) the tangents are q1 = – =
2 4 4
y = 2 & 4x – 3y – 10 = 0 π
10 5 & θ + q1 =
∴ Intercepts made on x axis by 2nd tangent = = 2
4 2 π
θ=
4
Paragraph 3: (37-39)

Sol 37: (D) s : x2 + y2 – 4x – 1 = 0 Previous Years’ Questions


L : y = 3x – 1
Sol 1: (B) For required circle, P(1, 8) and O(3,2) will be
Centre of circle = (2, 0)
the end point of its diameter.
Radius = 5
Length of chord AB A

= 2 r 2 − (perpendicular distance from centre )


2
(1,8) P O (3,2)

Perpendicular distance from centre B

6 −1 5
= = ∴ (x − 1)(x − 3) + (y − 8)(y − 2) =
0
10 2
⇒ x2 + y 2 − 4x − 10y + 19 =
0
5 5
AB = 2 5− = 2 = 10
2 2
1
Sol 2: (B) 18
= (3α )(2r) ⇒ α=
r 6
2
Sol 38: (A)
y

 2
 D C (, 2r)
(0, 2r)
B

(r, r)

Angle subtends at minor arc = 180 – angle at major arc x’ x


1  AB (0, 0) A B (2, 0)
tanθ =
2 ± distance y’
M a them a ti cs | 9.95

2r Sol 3: (A) Let the locus of centre of circle be (h, k)


Line, y =− (x − 2α ) is tangent to circle
α touching
(x − r)2 + (y − r)2 = r2 (y − 1)2 + x2 =
1 and x-axis shown as
2α 3r and
= = αr 6
Clearly, from figure,
r=2 y
x
r x r
B A

 (0, 1) (h, k)
r lkl
2r O A
O 1
90 l kl
- x’ x
r O B

y’
C 2 x-r D
2x
Distance between O and A is always 1+|k|,
x
r x r ie, (h − 0)2 + (k − 1)2 =1+ | k |,
B A
⇒ h2 + k 2 − 2k + 1
=1 + k 2 + 2 | k |
 r
2r ⇒ h2 =+
2 | k | 2k
O
90 ⇒ x2 =
2 | y | +2 y
-
r  y, y ≥ 0
where | y |=  y, y ≥ 0
where | y |= − y, y < 0
− y, y < 0
C 2 x-r D 
2
∴ x2 =2y + 2y, y ≥ 0
2x ∴ x =2y + 2y, y ≥ 0
and x22 = 2y + 2y, y < 0
and x = 2y + 2y, y < 0
Alternate solution 2
⇒ = x2 4y when y ≥ 0
⇒ = x 4y when y ≥ 0
1 and x22 0 when y < 0
=
(x + 2x) × 2r =
18 and x
= 0 when y < 0
2 ∴ {(x,
= y) : x22 4y, when y ≥ 0} ∪ {(0, y) : y < 0}
xr = 6 ...(i) ∴ {(x,
= y) : x 4y, when y ≥ 0} ∪ {(0, y) : y < 0}

x −r
In ∆AOB, tan θ =
r Sol 4: (A) From figure it is clear that ∆PRQ and ∆RSP
and in ∆DOC are similar.
2x − r
tan(90o − θ) =
r
x −r r P
r r
R
∴ = 90- 
r 2x − r 
⇒ x(2x − 3r) =
0
3r
⇒ x= ....(ii) X
2
-


90

From Eqs. (i) and (ii) we get


Q S
r=2
9 .96 | Circle

PR PQ There are given two distinct chords which are bisected


∴ =
RS RP at x-axis then, there will be two distinct values of h
⇒ PR 2 =
PQ ⋅ RS satisfying Eq. (i).
So, discriminant of this quadratic equation must be > 0.
⇒ PR =
PQ.RS
⇒ 2r = PQ ⋅ RS ⇒ D >0
⇒ ( −3p) − 4 ⋅ 2(p2 + q2 ) > 0
2

Sol 5: (B) Choosing OA as x-axis, A=(r, 0), B=(0, r) and ⇒ 9p2 − 8p2 − 8q 2 > 0
any point P on the circle is (r cos θ,r sin θ) . If (x, y) is the ⇒ p2 − 8q2 > 0
centroid of ∆PAB , then
y
⇒ p2 > 8q2

Sol 7: Equation of given circle C is

(x − 3)2 + (y + 5)2 =9 + 25 − 30
A (p,q)
ie, (x − 3)2 + (y + 5)2 =
22
C (h,0)
x’ x
O Centre = (3, -5)
B
y’
If L1 is diameter, then 2(3) + 3( −5) + p − 3 = 0 ⇒ p = 12

3x r cos θ + r + 0
= ∴ L1 is 2x + 3y + 9 =0
and 3y
= r sin θ + 0 + r L2 is 2x + 3y + 15 =
0
∴ (3x − r)2 + (3y − r)2 =
r2 Distance of centre of circle from L2 equals

Hence, locus of P is a circle. 2(3) + 3( −5) + 15 6


= < 2 (radius of circle)
22 + 33 13
Sol 6: (D) From equation of circle it is clear that circle
passes through origin. Let AB is chord of the circle. ∴ L2 is a chord of circle C.
y Statement-II, false.

Sol 8: (A) Let the, equation of circles are


C1 : (x − 1)2 + (y − 1)2 =
(1)2

A (p,q)
and C2 : (x − 1)2 + (y − 1)2 =
( 2)2

C (h,0)
x’ x C2
O Q
B
D (0,2) C (2,2)
y’ C1 P

A ≡ (p,q) ⋅ C is mid point and coordinate of C is (h, 0)


2 2
(1,1) x + y - 2x - 2y =0

Then coordinates of B are (-p + 2h, -q) and B lies on the


circle x2 + y 2 = px + qy. we have
A (0,0) B (2,0)
2 2
( −p + 2h) + ( −q) = p( −p + 2h) + q( −q)
x2 + y2 - 2x - 2y =0
⇒ p2 + 4h2 − 4ph + q2 =−p2 + 2ph − q2
⇒ 2p2 + 2q2 − 6ph + 4h2 =
0
∴ Coordinates of P(1 + cos θ,1 + sin θ)
⇒ 2h2 − 3ph + p2 + q2 =
0 ....(i)
M a them a ti cs | 9.97

and Q(1 + 2 cos θ,1 + 2 sin θ) (Rejecting ‘2’ because origin and centre of C are on the
same side of PQ).
∴ PA2 + PB2 + PC2 + PD2
The point ( 3,1) satisfies Eq. (i).
= {(1 + cos θ)2 + 1 + sin θ)2 } + {(cos θ − 1)2 + (1 + sin θ)2 }
+ {(cos θ − 1)2 + (sin θ − 1)2 } ∴ Equation of circle C is (x − 3)2 + (y − 1)2 =
1.
+ {(1 + cos θ)2 + (sin θ − 1)2 }
Sol 12: (A) Slope of line joining centre of circle to point
= 12 D is
Similarly, QA2 + QB2 + QC2 + QD2 =
16 3
−1
2 1
∑ PA2
12 tan θ
= =
∴ = 0.75
= 3 2 3
∑ QA 16 2 − 3
2
It makes an angle 30o with x-axis.
Sol 9: (C) Let C be the centre of the required circle.
Now, draw a line parallel to L at a distance of r1
(radius of C1 ) from it.
Now, CC1 = AC
⇒ C lies on a parabola.

∴ Point E and F will make angle 150o and -90o with


Sol 10: (C)
x-axis.
Since, AG = 2
∴ E and F are given by
1
∴ AT1 =T1G = x− 3 y −1
2 = = 1
o
cos150 sin150o
As A is the focus, T1 is the vertex and BD is the directrix x− 3 y −1
of parabola. and = = 1
cos( −90 ) sin( −90o )
o

Also, T2 T3 is latus reetum.  3 3


∴ = E  ,  and
= F ( 3,0)
 2 2
 
D C

Sol 13: (D) Clearly, points E and F satisfy the equations


G given in option (d).
T1
T2
B
Sol 14:
A
2x2 + y 2 − 3xy =
0 (given)
T3 2 2
⇒ 2x − 2xy − xy + y = 0
1
∴ T2 T3 =
4⋅ ⇒ 2x(x − y) − y(x − y) =
0
2
⇒ (2x − y)(x − y) =
0
1 1 4
∴ Area of ∆T1 T2 T3 = × × =1 sq unit
2 2 2 ⇒ y = 2x, y = x are the equations of straight lines
passing through origin.
Sol 11: (D) Let centre of circle C be (h, k) Now, let the angle between the lines be 2θ and the
line y = x
3h + k − 6
Then, =1 Makes angle of 45o with x-axis.
3+1
Therefore, tan(45o + 2θ) =2 (slope of the line y =2x)
⇒ 3h + k − 6 =+2
⇒ 3h + k =4 ....(i)
9 .98 | Circle

E D Again, in ∆OCA
3 3
30 tan θ= ,OA
=
OA tan θ
(3,1) 3
=
( −3 + 10 )
3(3 + 10 )
F
∴ =
( −3 + 10 )(3 + 10 )
3(3 + 10 )
tan 45o + tan2θ = = 3(3 + 10 )
⇒ 2
= (10 − 9)
1 − tan 45o × tan2θ
Sol 15:
y y
y = 2x
y=x T1
3 B1
C
3
A 30 A1 L
x’ x
S c1 M O N C2

A2

45 B2
x’
O T2

y’ y’

1 + tan2θ
⇒ = 2 l
1 − tan2θ
C1 C2
(1 + tan2θ) − (1 − tan2θ) 2 −1 1
⇒ = =
(1 + tan2θ) + (1 − tan2θ) (2 + 1) 3
2 tan2θ 1
⇒ =
2 3
1 From figure it is clear that, triangle OLS is a right triangle
⇒ tan2θ = with right angle at L.
3
2 tan θ 1 Also, OL = 1 and OS = 2
⇒ =
1 − tan θ 3
2
1
∴ sin(∠LSO)= ⇒ ∠LSO= 30o
2
⇒ (2 tan θ) ⋅ 3 = 1 − tan2 θ
Since, SA
= SA2 , ∆SA1 A2 is an equilateral triangle.
⇒ tan2 θ + 6 tan θ − 1 = 0 1

The circle with centre at C1 is a circle inscribed in the


−6 ± 36 + 4 × 1 × 1
⇒ tan θ = ∆SA1A2. Therefore, centre C1 is centroid of ∆SA1A2. This,
2
C1 divides SM in the ratio 2:1. Therefore, coordinates of
C1 are (-4/3,0) and its radius C1 M=1/3
−6 ± 40
⇒ tan θ = ∴ Its equation is (x + 4 / 3)2 + y 2 = (1 / 3)2  …(i)
2
⇒ tan θ = −3 ± 10 The other circle touches the equilateral triangle SB1B2
 π ∆
⇒ tan θ = −3 + 10  0 < θ <  Externally. Its radius r is given by r = ,
 4 s−a
1 3
where B=
1B2 a. But
= ∆ (a)(SN)
= a
2 2
3 a
and s − a= a − a=
2 2
Thus, r =3
M a them a ti cs | 9.99

⇒ Coordinates of C2 are (4,0) 2π π


∠POQ = =
∴ Equation of circle with centre at C2 is 3 k
π π
(x − 4)2 + y 2 =
32  …(ii) ∠ROS = =
3 k
Equations of common tangents to circle (i) and circle ⇒k =3
C are
1 Sol 17: (D)
x= −1 and y = ± (x + 2) [T1 and T2 ]
3
Equation of common tangents to circle (ii) and circle C
are
1
x= −1 and y = ± (x + 2) [T1 and T2 ]
3
Two tangents common to (i) and (ii) are T1 and T2 at O.
To find the remaining two transverse tangents to (i) and
(ii), we find a point I which divides the joint of C1C2 in
the ratio= r1 : r2 1=
/3:3 1:9 2 2
( x − h) + ( y − 2 ) h2
=
Therefore, coordinates of I are (-4/5,0)
Equation of any line through I is y = m (x+4/5). It will
Passes through ( −1,0 ) , then
2 2
touch (i) if ( −1, −h) + ( 0 − 2 ) =h2
2
 −4 4 
m + −0 (1 + h) − h2 = −4
 3 5 1
= ⇒ (1 + h − h)(1 + h + h) =−4
1+m 2 3

⇒ −
8m 1
= 1 + m2 ⇒ (1 )( 2h + 1 ) =−4
15 3
h = −5 / 2
⇒ 64m2 =
25(1 + m2 )
Circle is
⇒ 39m2 =
25
2 2
5  5 2 5
⇒ m= ±  x +  + ( y − 2) = 
39  2 2

Therefore, these tangents are Only ( −4,0 ) satisfies the eq. of circle.

5  4 D is the Answer.
y=
± x + 
39  5 
2 2
Sol 18: (D) Any tangent to circle x + y =4 and
2 2
Sol 16: Let equation of Circle be x + y =
4 and parallel 2

chords =are x 1 and − 13


( x − 3) 1 , then
+ y2 =

3x1 + 0 × y1 − 4
=1
x12 + y12

3x1 − 4
=1
y
⇒ 3x1 − 4 =
2

⇒ x1 =
2,2 / 3
2 4 2 
P ≡ (1,13) , Q ≡ (1, −13) ⇒ ( x1 , y1 ) ≡ ( 2,0 ) &  , 
3 3 
 
( ) (
R ≡ − 3,1 ,S ≡ − 3, −1 )
9 .100 | Circle

Tangents If (h,k) mid-point, the eq. of chord of contact T=S1


2x 4 2
2 . x + 0 = 4 ⇒ x = 2 and + 4
=
3 3
⇒ x+2 2 = 6

Sol 19: (A) The tangent to circle x2 + y 2 =


4 at ( 3,1) 4
PT ≡ 3x + y =4
Eq. of L is x − 3y =
λ
Circle ( x − 3) + y 2 =
2
1 is touching L, then
xh + yk = h2 + k 2  … (ii)
(i) & (ii) are identical, then
3− 3×0 − λ
=1 h k h2 + k 2
1+ 3 = =
5f 4f − 20 45
3 − λ =2 9h
t=
λ =1,5 h + k2
2

45k
Tangents x − 3y =
1 4t − 20 =
h + k2
2

Sol 20: Let P be (2t2, 4t) lies on circle 9h × 4 45k


⇒ − 20 =
2 2
4t 4 + 16t2 − 4t2 − 16t =
0 h +k h + k2
2

⇒ t 4 + 4t2 − t2 − 4t =
0 ⇒ 36h − 20 n2 + k 2 = (
45k )
(
⇒ t ( t − 1) t + t + 4 =
2
0 ) ( )
⇒ 20 h2 + k 2 − 36h + 45k =
0
⇒t=0,1
( )
⇒ 20 x2 + y 2 − 36x + 45y =
0

Sol 22: (C) Let circle touching x-axis be


(x − α) + (y − k )
2 2
=k 2  … (i)
Also for y-axis intercepts

(0 − α ) = ( y − k ) = k2
2 2

⇒ ( y − k ) = k 2 − α2
2

P ≡ ( 2, 4 ) Q ≡ ( 0,0 ) S ≡ ( 2,0 )
⇒ y = k ± k 2 − α2
1
∆= × 2 × 4 = 4 sq units
2 = 2 k 2 −=
Intercept α2 2 7

 4t − 20  ⇒ k 2 = 7 + α2
Sol 21: (A) Let point P be  t, 
 5  From (i) α =3
⇒ k 2 = 7 + 9 = 16
 4t − 20 
Eq. of chord of contact xt + y  =9 ⇒ k =±4
 5 
(5t ) x + y ( 4t − 20 ) =
45  … (i) Circle: ( x − 3) + ( y − 4 ) =
2
16
2

( x − 3) + ( y + 4 )
2 2
16
=
M a them a ti cs | 9.101

Sol 23: (D) Let tangent to parabola y2=8x Passes through (2, 8), then
Be ty=x+2t2 8=2m-2m-m3 ⇒ m=-2

It is also tangent to circle, then


Normal
2t2
= 2
y=-2x=12
1 + t2

( 4 − 2) + ( 4 − 8 )
2 2
(
⇒ 4t 4 =2 1 + t2 ) SP = = 4 + 16 = 2 5 units

⇒ 2t 4 − t2 − 1 =0 Let SQ : QP = 1 : λ

( )(
⇒ 2t2 + 1 t2 − 1 =
0 ) 1
.
λ
S ( 2,8 ) Q (h,k ) P ( 4, 4 )
⇒ t =±1

⇒ S ≡ ( 2, 4 ) & R ≡ ( 2, −4 )  4 + 2λ 8λ + y 
Q (h,k ) ≡  ,  lies on
 1+ λ 1+ λ 
⇒ P ≡ ( −1,1) & Q ( −1,1)
Circle, then
1
Area=
2
( 2 + B ) × 3= 15 sq units  4 + 2λ
2
  8λ + 4 
2

 − 2 +  − 8 =
4
 1+ λ   1+ λ 
2 2
Sol 24: (B, C) Let circle be x + y + 2yx + 2y + C =
0 2 2
 2   −4 
Applying condition for orthogenality ⇒  + 4
 =
 1+ λ   1+ λ 
2gx − 1 + 2f × 0 = C + ( −15)
20
⇒ 4
=
15 and 2g × 0 + 2f × 0 = C − 1
⇒ 2g + c = (1 + λ )
2

⇒C=
1
⇒g=
7 ⇒ 1+ λ = 5
⇒=
λ 5 −1
Also,
1 + 2f + C =
0
⇒ f =−1
SQ
=
QP
( 5 −1 )
Centre ≡ ( −g, −f ) ≡ ( −7,1)
x – intercept of normal at P is 6 slope of tangent at Q
Radius
= 2
g +f −C
= 2
49 + 1 −=
1 7 1
is
2
Hence, B and C are the correct options

Sol 26: (C) For point of intersection


Sol 25: (A, C, D) (x-2)2+(y−8)2=4
2y+y2=3
Shortest distance is measured along common normal
⇒ y 2 + 2y − 3 =0
The equation of normal to parabola
y=mx-2am-am3 ⇒ y=mx-2m-m3 ⇒ (y + 3)(y − 1) =
0
9 .102 | Circle

⇒ y = 1, -3 Sol 27: (A, C) Let point P be ( cos θ,sin θ ), The tangent


and normal are
⇒= ( 2,1)
The eq. of tangent at 1, 2 ( ) x cos θ + y sin θ =1 x sin θ − y cos θ = 0

3
2x + y = (i)  1 − cos θ 
⇒ θ ≡ 1, 
 sin θ 
Eqs. of circle C2 and C3
 y cos θ 
C2 ≡ x2+(y-y2)2 =12 ⇒ E ≡ 1, , y  ≡ (h,k ) (let )
 sin θ 
C2 ≡ x2+(y-y3)2 =12
 1 − Cosθ  cos θ
⇒ h=
 .
If line (i) touches circle, then  Sinθ  sin θ
2 ×0 + y −3  1 − Cosθ 
=2 3 k= 
2 +1  Sinθ 

⇒ y −3 =
6

⇒ y −3 =±6 h
1−
⇒y=−3,9
K= h + K2
2

K
⇒ y2 =
−3 and y 3 =
9 1−
h2 + K 2
⇒ Centres Q 2 ≡ ( 0, −3)
Q 3 ≡ ( 0,9 )
⇒ Q 2Q 3 =
12

For point of contact R2 and R3 ⇒ K2 + h = h2 + K 2 ⇒ y 2 + x = x2 + y 2

( )
R 2 ≡ 2 2, −1 and R 3 ≡ −2 2,7 ( )
(4 2)
2
+ (8) =
2
R 2R3= 32 + 64= 96= 16 × 6= 4 6

0(0,0), R2( 2 ,-1), R3(-2 2 ,7)

0 0 1
1
Area of ∆ OR2R3= 2 2 −1 1
2
−2 2 7 0

=
1
2
( 1
7×2 2 −2 2 = ×6×2 2
2
)
= 6 2 sq. units
Now
Area of ∆ PQ2Q3

2 1 1
1
= 0 −3 1
2
0 9 1

1
= 2 ( −3 −=
9 )  6 2 sq units
2 
2017-18 100 &
op kers
Class 11 T
By E ran culty
-JE Fa r
IIT enior emie .
S fP r es
o titut
Ins

MATHEMATICS
FOR JEE MAIN & ADVANCED
SECOND
EDITION

Exhaustive Theory
(Now Revised)

Formula Sheet
9000+ Problems
based on latest JEE pattern

2500 + 1000 (New) Problems


of previous 35 years of
AIEEE (JEE Main) and IIT-JEE (JEE Adv)

5000+Illustrations and Solved Examples


Detailed Solutions
of all problems available

Topic Covered Plancess Concepts


Tips & Tricks, Facts, Notes, Misconceptions,
Parabola Key Take Aways, Problem Solving Tactics

PlancEssential
Questions recommended for revision
10. PA R A B O L A

1. INTRODUCTION TO CONIC SECTIONS


1.1 Geometrical Interpretation
Conic section, or conic is the locus of a point which moves in a plane such that its distance from a fixed point is in
a constant ratio to its perpendicular distance from a fixed straight line.
(a) The fixed point is called the Focus.
(b) The fixed straight line is called the Directrix.
(c) The constant ratio is called the Eccentricity denoted by e.
(d) The line passing through the focus and perpendicular to the directrix is called the Axis.
(e) The point of intersection of a conic with its axis is called the Vertex.

Sections on right circular cone by different planes


(a) Section of a right circular cone by a plane passing through its vertex is a pair of straight lines passing through
the vertex as shown in the figure.
(b) Section of a right circular cone by a plane parallel to its base is a circle.
(c) Section of a right circular cone by a plane parallel to a generator of the cone is a parabola.
(d) Section of a right circular cone by a plane neither parallel to a side of the cone nor perpendicular to the axis
of the cone is an ellipse.
(e) Section of a right circular cone by a plane parallel to the axis of the cone is an ellipse or a hyperbola.

3D View:

Circle

Ellipse Parabola Hyperbola

Figure 10.1

1.2 Conic Section as a Locus of a Point


If a point moves in a plane such that its distances from a fixed point and from a fixed line always bear a constant
ratio 'e' then the locus of the point is a conic section of the eccentricity e (focus-directrix property). The fixed point
is the focus and the fixed line is the directrix.
1 0 . 2 | Parabola

(a) If e > 1, it is a hyperbola. L P

(b) If e = 1, it is a parabola.
(c) If e < 1, it is an ellipse. S
(d) If e = 0, it is a circle. M
(e) If the focus is (α, β) and the directrix is ax + by + c = 0 then the equation of the conic Figure 10.2
(ax + by + c)2
section whose eccentricity =e, is (x – α)2 + (y – β)2 = e2.
a2 + b2
1.3 General Equation of Conic
The general equation of a conic with focus (p, q) and directrix lx + my + n = 0 is:
(l2 + m2) [(x – p)2 + (y – q)2] = e2(lx + my + n)2
This equation, when simplified, can be written in the form ax2 + 2gx + 2hxy + by2 + 2fy + c = 0.
This general equation represents a pair of straight lines if it is degenerate, i.e. ∆ = 0,
a h g
where ∆ = h b f . Further, this equation represents
g f c
(a) A pair of parallel straight lines, if ∆ = 0 and h2 = ab
(b) A pair of perpendicular straight lines, if ∆ = 0 and a + b = 0
(c) A point, if ∆ = 0 and h2 < ab
The general equation given above, represents a conic section if it is non-degenerate, i.e. ∆ ≠ 0 , also this equation
represents
(a) A circle, if ∆ ≠ 0, a = b and h = 0
(b) A parabola, if ∆ ≠ 0 and h2 = ab
(c) An ellipse, if ∆ ≠ 0 and h2 < ab
(d) A hyperbola, if ∆ ≠ 0 and h2 > ab

PLANCESS CONCEPTS

Always use a geometrical approach in the coordinate geometry problem.


Vaibhav Gupta (JEE 2009, AIR 54)

2. GENERAL EQUATION OF PARABOLA


Definition: A parabola is the locus of a point which moves such that its distance P
M
from a fixed point is equal to its distance from a fixed straight line. (x,y)
F
Standard equation of the parabola: Z X
A (a,0)
y+a=0
Directrix

The standard form of a parabola is taken with the origin as its vertex and the focus Focus
lying on the x-axis.
Let F be the focus and ZM be the directrix of the parabola.
Figure 10.3
Let P be a point on the parabola.
Join FP and from P drawn PM ⊥ to ZM. ∴ FP = PM
M a them a ti cs | 10.3

To find the co-ordinates of focus and equation of directrix: From F draw FZ ⊥ on ZM. Bisect FZ in A, i.e. FA =
AZ. Then A lies on the parabola. Let FZ = 2a then AF = AZ = a.
(a,l)
Take A as the origin, AF as the x-axis and AY as the y-axis. L

x+a=0
Then the co-ordinates of F are (a, 0) and the equation of the directrix is x = –a, of x + a = 0. A
(0,0) F(a,0)
Let the co-ordinates of P be (x, y). We know that for a parabola FP = PM.
L’
(x − a)2 + (y − 0)2 = x + a

⇒ x2 – 2ax + a2 + y2 = x2 + 2ax + a2 ⇒ y 2 = 4ax Figure 10.4

Which is the standard equation of a parabola.

Note: Focus F is (a, 0); Directrix is x + a = 0; Vertex A is (0, 0); Axis AF of the parabola is y = 0.

Latus rectum LL′ is 4a as calculated below:


Let the co-ordinates of L be (a, l). The point L (a, l) lies on the parabola y2 = 4ax.
∴ l2 = 4a.a or l = 2a.
Latus rectum LSL′ = 2l = 2(2a) = 4a = 4.FA

Illustration 1: Find the equation of the parabola whose focus is (1, 1) and the tangent at the vertex is x + y = 1.
Also find its latus rectum.  (JEE MAIN)

Sol: In order to get the equation of a parabola, we need to find the equation of a directrix. Using the equation of
the tangent at the vertex and the focus we can find the directrix and hence the equation of the parabola.
The directrix is parallel to the tangent at the vertex V.
∴ The directrix will be of the form x + y = λ …(i)
Now, V is the foot of the perpendicular from S(1, 1) to the line x + y = 1.
Let V = (α, β). Then α + β = 1  …(ii)
β −1
and .(–1) = –1, i.e., α = β …(iii)
α −1
1 1 1
Solving (ii), (iii) we get α = β = . So V = , 
2 2 2
Let M = (x1, y1). As MV = VS, V is the middle point of MS. M V S(1,1)
x1 + 1 1 y1 + 1 1
∴ = , =
2 2 2 2 directrix
x+y=1
∴ x1 = 0, y1 = 0. So M = (0, 0)
Figure 10.5
As M is on the directrix, (0, 0) satisfies (i). Hence, λ = 0
∴ the equation of the directrix is x + y = 0. 2
 x+y 
Using focus-directrix property, the equation of the parabola is (x – 1) + (y – 1) = 
2 2

 2 2 
 1 +1 
⇒ 2[(x – 1) + (y – 1) ] = (x + y)
2 2 2

⇒ 2(x2 + y2 – 2x – 2y + 2) = x2 + y2 + 2xy
⇒ x2 + y2 – 2xy – 4x – 4y + 4 = 0
⇒ (x – y)2 = 4(x + y – 1)
2 2
1  1  1
Length of latus rectum = 4 × VS = 4  − 1 +  − 1 =4. = 2 2
2  2  2
1 0 . 4 | Parabola

Illustration 2: Find the equation of the parabola whose focus is (–1, –2) and equation of the directrix is
x – 2y + 3 = 0. (JEE MAIN)

Sol: Use the standard definition of a parabola.


Let P(x, y) be any point on the parabola whose focus is S(–1, –2) and the directrix x – 2y + 3 = 0
By definition, SP = PM
2
 x − 2y + 3 
⇒ (x + 1)2 + (y + 2)2 =  
 1+4 
⇒ 5[(x + 1)2 + (y + 2)2] = (x – 2y + 3)2
⇒ 5(x2 + y2 + 2x + 4y + 5) = (x2 + 4y2 + 9 – 4xy + 6x – 12y)
⇒ 4x2 + y2 + 4xy + 4x + 32y + 16 = 0
This is the equation of the required parabola.

Illustration 3: Find the equation of the parabola whose focus is the point (4, 0) and whose directrix is x = –4. Also,
find the length of the latus rectum. (JEE MAIN)

Sol: Same as the previous question. Refer Fig. 10.6


In this illustration, the focus is (4, 0) and directrix is x + 4 = 0
Let P(x, y) be any moving point then draw ZZ′ ⊥ PM from P to the directrix.

x = -4
M P
FP (x,y)
= 1; FP = PM ⇒ FP = (PM)
2 2
PM Z X
2 (0,0)
x+4 F(4,0)
(x – 4)2 + (y – 0)2 =  
 1  Z’
⇒ x2 – 8x + 16 + y2 = x2 + 8x + 16
Figure 10.6
⇒ y2 = 16x
Length of latus rectum = coefficient of x = 16.

3. STANDARD FORMS OF PARABOLA

Right handed parabola: The equation of this type of parabola is of the M Y


form y2 = 4ax, a > 0. See Fig. 10.7
A
X’ X
Z F

Y’
2
y = 4ax

Figure 10.7
Left handed parabola: The equation of this type of parabola is of the Y M
form y2 = –4ax, a > 0. See Fig. 10.8
A
X’ X
F Z

Y’
2
y = -4ax

Figure 10.8
M a them a ti cs | 10.5

Upward parabola: The equation of this type of parabola is of the form x2 Y


= 4ay, a > 0. See Fig. 10.9
F
A
X’ X
Z M

Y’
2
x = 4ay

Figure 10.9

Downward parabola: The equation of this type of parabola is of the form Y


x2 =– 4ay, a > 0. See Fig. 10.10
Z M

A
X’ X

Y’
2
x = -4ay

Figure 10.10

Equation of the parabola


y2 = 4ax y2 = –4ax x2 = 4ay x2 = –4ay
Properties
Vertex (Co-ordinates) (0, 0) (0, 0) (0, 0) (0, 0)
Focus (Co-ordinates) (a, 0) (–a, 0) (0, a) (0, –a)
Latus rectum (length) 4a 4a 4a 4a
Axis (Equation) y=0 y=0 x=0 x=0
Directrix (Equation) x = –a x=a y = –a y=a
Symmetry (about) x-axis x-axis y-axis y-axis

PLANCESS CONCEPTS

Don’t get confused between x2 = 4ay and y2 = 4ax.


Almost every condition is different for both parabolas.
Nivvedan (JEE 2009, AIR 113)

Illustration 4: Find the vertex, the axis, the focus, the directrix, and latus rectum of the parabola, 4y2 + 12x – 20y + 67 = 0.
 (JEE MAIN)

Sol: Represent the given equation in the standard form and then compare it with the standard forms to get vertex,
axis, focus, directrix and latus rectum to get the answers.
1 0 . 6 | Parabola

The given equation is


4y2 + 12x – 20y + 67 = 0 ⇒ y2 + 3x – 5y + 67/4 = 0
2 2
5 67 5
⇒ y2 – 5y = –3x – 67/4 ⇒ y2 – 5y +   = – 3x – +  
2
  4 2
2 2
 5 42  5  7
⇒  y −  = –3x – ⇒  y −  = –3  x +   ... (i)
 2 4  2  2
Let x = X – 7/2, y = Y + 5/2 … (ii)
Using these relations, equation (i) reduces to Y2 = –3X … (iii)
This is of the form Y2 = –4aX. On comparing, we get a = 3/4.

Vertex: The coordinates of the vertex of Y2 = –3X are (X = 0, Y = 0) So, the coordinates of the vertex before
 7 5
transformation are  − ,  .
 2 2
Axis: The equation of the axis of the parabola Y2 = –3X is Y = 0. So, the equation of the axis before transformation
is y = 5/2.
 17 5 
Focus: The coordinates of the focus are (X = –3/4, Y = 0). So, the coordinates of the focus are  − , .
 4 2
Directrix: The equation of the directrix is X = +a = +3/4. So, the equation of the directrix before transformation is
x = –11/4
Latus rectum: The length of the latus rectum of the given parabola is 4a = 3.

Illustration 5: Prove that 9x2 – 24xy + 16y2 – 20x – 15y – 60 = 0 represents a parabola. Also find its focus and
directrix. (JEE MAIN)

Sol: A general equation of a conic represents a parabola if ∆ ≠ 0 and h2 = ab. In order to get the focus and the
directrix, convert the given equation into the standard form and compare with the standard form.

9 −12 −10
−15
Here h2 – ab = (–12)2 – 9(16) = 144 – 144 = 0. Also, ∆ = −12 16 ≠0
2
−15
−10 −60
2
∴ The given equation represents a parabola. Now, the equation is (3x – 4y)2 = 5(4x + 3y + 12).
Clearly, the lines 3x – 4y = 0 and 4x + 3y + 12 = 0 are perpendicular to each other. So, let

3x − 4y 4x + 3y + 12
= Y, = X … (i)
32 + ( −4)2 42 + 32
1
The equation of the parabola becomes Y2 = X = 4. X
4
1
∴ Here a = in the standard equation.
4
1 
∴ The focus = (a, 0)X, Y =  , 0 
 4 X, Y
1
If, X = , Y = 0, then from the equations of transformation in (i), we get
4
3x − 4y 4x + 3y + 12 1 −43
= 0, = ⇒ 3x – 4y = 0, 4x + 3y =
5 5 4 4
M a them a ti cs | 10.7

3x 1  43  3x 1  43 
⇒ y= , y = − − 4x  ∴ = − − 4x 
4 3 4  4 3 4 
−43 3 3 −43 −129
⇒ 9x = –43 – 16x ; ∴ x = and y = x = . =
25 4 4 25 100
 −43 −129 
∴ Focus =  , 
 25 100 
1
The equation of the directrix is X + a = 0, i.e., X + =0
4
4x + 3y + 12 1 5 53
or + = 0 or 4x + 3y = – – 12 = –
5 4 4 4
53
∴ The equation of the directrix is 4x + 3y + = 0.
4

Illustration 6: Find the equation of the parabola whose latus rectum is 4 units, the axis is the line
3x + 4y – 4 = 0 and the tangent at the vertex is the line 4x – 3y + 7 = 0. (JEE ADVANCED)

Sol: The square of the distance of a point from the directrix is equal to the product of latus rectum and the distance
of the point from the axis.
Let P(x, y) be any point on the parabola and let PM and PN be perpendiculars from P to the axis and to the tangent
at the vertex respectively. Then
2
 3x + 4y − 4   4x − 3y + 7 
PM = (Latus rectum) (PN)
2
⇒   = 4  ⇒ (3x + 4y – 4)2 = 20(4x – 3y + 7)
 2 2   2 2 
 3 +4   4 + ( −3) 
This is the equation of the required parabola.
Note: In the above examples, we have learnt how to find the vertex, the focus, the axis, the directrix etc. of
parabolas reducible to one of the various forms given. If the equation of a parabola is quadratic in both x and y,
then to find its vertex, focus, axis, etc., we follow the following algorithm.
Step I: Obtain the equation of the parabola and express it in the form (ax + by + c)2 = (Constant) (bx – ay + c′)
It should be noted here that ax + by + c = 0 and bx – ay + c′ = 0 represent perpendicular lines.
2
2 2
 ax + by + c   bx − ay + c′ 
Step II: Divide both sides by a + b to obtain   = (Constant)  
 2 2   2 2 
 a +b   a +b 
ax + by + c bx − ay + c′
Step III: Now substitute = Y and = X in step II to obtain Y2 = (Constant) X.
2 2
a +b a2 + b2

Step IV: Compare the equation obtained in step III with Y2 = 4ax to obtain various elements like vertex, focus, axis,
etc., and use the transformation in step III to obtain the corresponding elements of the given parabola.

Illustration 7: Find the equation of the parabola whose axis is parallel to the y-axis and which passes through the
points (0, 4), (1, 9) and (–2, 6). Also find its latus rectum. (JEE ADVANCED)

Sol: Consider a standard equation of a parabola with the vertex at (α , β) such that the axis is parallel to the Y-axis.
Substitute the points given and find the. (see Fig. 10.11) equation
As the axis is parallel to the y-axis, it will be of the form x – α = 0 and the tangent to the vertex (which is
perpendicular to the axis) will be of the form y – β = 0 for some β.
Hence, the equation of the parabola will be of the form
(x – α)2 = 4a(y – β) … (i)
where α, β, a are the unknown constants and 4a being the latus rectum.
1 0 . 8 | Parabola

(i) passes through (0, 4), (1, 9) and (–2, 6). So


(0 – α)2 = 4a(4 – β),
i.e., a2 = 4a(4 – β)  … (ii)
and (1 – α)2 = 4a(9 – β)
i.e, 1 – 2α + a2 = 4a(9 – β) … (iii)
and (–2 – α)2 = 4a(6 – β),
i.e, 4 + 4α + a2 = 4a(6 – β)  … (iv)
(iii) – (ii) ⇒ 1 – 2α = 20a  … (v)
(iv) – (iii) ⇒ 3 + 6α = –12a
1 − 2α 20a
∴ = ⇒ –3(1 – 2α) = 5(3 + 6α) Z
3 + 6α −12a x- = 0
3
⇒ 24α = –18; ∴ α = − Y=0
4
3 5 1 X=0
∴ (v) ⇒ 1 + = 20a; ∴ a= =
2 40 8 y- = 0
2 Directrix
 3 1
∴ (ii) ⇒  −  = 4. (4 – β) Figure 10.11
 4  8
9 1 9 23
⇒ = (4 – β), i.e., 4 – β = ⇒ β =
16 2 8 8
2
 3 1  23 
∴ From (i), the equation of the parabola is  x +  = 4. .  y − 
 4 8  8 
3 9 1 23 3 1
⇒ x2 + x+ = y – ⇒ x2 + x– y +2=0
2 16 2 16 2 2
1 1
∴ 2x2 + 3x – y + 4 = 0 and its latus rectum = 4a = 4. =
8 2

4. PARAMETRIC EQUATIONS OF A PARABOLA


For the standard equation of the parabola y 2 = 4ax we write the parametric equations as x = at2 and y = 2at . Thus,
the parametric coordinates of a point on the parabola are ( at2 , 2at). Unlike the rest of conics, there is no physical
significance for the parameter t.

5. A POINT AND A PARABOLA


The point (x1, y1) lies inside, on, or outside y2 = 4ax according to y12 – 4ax1 <, =, or > 0.

Illustration 8: Find the set of values of α in the interval [π/2, 3π/2] for which the point (sin α, cosα) does not lie
outside the parabola 2y2 + x – 2 = 0. (JEE MAIN)

Sol. Use the concept of the Position of a point w.r.t. a parabola. If the point (sin α, cos α) lies inside or on the
parabola 2y2 + x – 2 = 0.
2cos2α + sin α – 2 ≤ 0 ⇒ 2 – 2 sin2α + sin α – 2 ≤ 0
⇒ sin α(2 sinα – 1) ≥ 0 ⇒ sin α ≤ 0 or, 2 sin α – 1 ≥ 0
⇒ α ∈ [π, 3π/2] or, α ∈ [π/2, 5π/6] ⇒ α ∈ [π/2, 5π/6] ∪ [π, 3π/2]
M a them a ti cs | 10.9

6. CHORD

6.1 Equation af a Chord


Let P (at12, 2at1) and Q(at22, 2at2) be two points on the parabola y2 = 4ax. Then, the equation of the chord PQ is
y(t1 + t2) = 2x + 2at1t2.

Note: (a) If the chord joining points t1 and t2 on the parabola y2 = 4ax passes through the focus then t1 t2 = –1.
(b) If one end of a focal chord of the parabola y2 = 4ax is P(at2, 2at), then the coordinates of the other end is
 a −2a 
Q , .
 t2 t 

Length of a focal chord

(a) Let P(at2, 2at) be one end of a focal chord PQ of the parabola y2 = 4ax. Then, the length of the focal chord
with ends as P and Q is a(t + 1/t)2
2
1  1
(b) We know that, t + ≥ 2 for all t ≠ 0. ∴ a  t +  ≥ 4a ⇒ PQ ≥ 4a.
t  t

Thus, the length of the smallest focal chord of the parabola is 4a which is the length of its latus rectum.For
this reason, the latus rectum of a parabola is the smallest focal chord.
(c)  he semi-latus rectum of the parabola y2 = 4ax is the harmonic mean between the segments of any focal
T
chord of the parabola.
(d) The circle described on any focal chord of a parabola as a diameter of that circle, also touches the directrix.
(e)  he line y = mx + c meets the parabola y2 = 4ax at two points that can be real, coincident or imaginary
T
according to a >,=,< cm ⇒ condition of tangency is, c = a/m.
 4 
(f) Length of the chord intercepted by the parabola on the line y = mx + c is:  a(1 + m2 )(a − mc)
2 
m 

PLANCESS CONCEPTS

The length of the chord joining two points t1 and t2 on the parabola y2 = 4ax is

a(t1 – t2) (t1 + t2 )2 + 4 


Nitin Chandrol (JEE 2012, AIR 134)

Illustration 9: A quadrilateral ABCD is inscribed in y2 = 4ax and 3 of its sides AB, BC, CD pass through fixed points
(α, 0), (β, 0) and (γ, 0), then show that the 4th side also passes through a fixed point. Also find this fixed point.

Sol: Consider four parametric coordinates and form equations according to the given conditions.
Let t1, t2, t3 and t4 be the parametric coordinates of A, B, C and D respectively. C (t3)
Equation of AB is y(t1 + t2) = 2x + 2at1t2. Given that this line passes through (α, 0). (t4)
α D
⇒ t1t2 = −
a α γ
t1 t2 t3 t 4 − . −αγ
β γ
Similarly, t2t3 = − , t3t4 = − Accordingly, t1t4 = = a a = A
a a t t
2 3 β βa (t1)
a B (t2)
 αγ 
From the above result we can say that AD always passes through a point  , 0 Figure 10.12
 β 
1 0 . 1 0 | Parabola

Illustration 10: Find the relation between the line y = x + 1 and the parabola y2 = 4x. (JEE MAIN)

Sol: Solve the two given equations and based on the intersection we can find the relation between the two.
Solving the line and parabola, we get (x + 1)2 = 4x ⇒ (x – 1)2 = 0. Therefore y = x + 1 is a tangent to the parabola.

Illustration 11: Through the vertex O of a parabola y2 = 4x, the chords OP and OQ are drawn at right angles to one
another. Show that for all positions of P, PQ intersects the axis of the parabola at a fixed point. Also find the locus
of the middle point of PQ. (JEE MAIN)

Sol: Use parametric coordinates for P (t1) and Q (t2) to find the relation between them and obtain the equation of
chord PQ. Let the coordinates of P and Q be (t12, 2t1) and (t22, 2t2) respectively. Then,
2 2
m1 = Slope of OP = ; m2 = Slope of OQ =
t1 t2
Since OP is perpendicular to OQ. Therefore, m1m2 = –1
2 2
⇒ × = –1 ⇒ t1t2 = –4
t1 t2
The equation of chord PQ is y(t1 + t2) = 2x + 2t1t2
⇒ y(t1 + t2) = 2x – 8

Clearly, it passes through (4, 0) for all values of t1 and t2. Thus, PQ cuts x-axis at a fixed point (4, 0) for all position
of point P.
Let R(h, k) be the mid-point of PQ. Then, 2h = t12 + t22 and k = t1 + t2 ... (ii)
∴ (t1 + t2)2 = t12 + t22 + 2t1t2 ⇒ k2 = 2h – 8 [Using (i) and (ii)]
Hence, the locus is y = 2x – 8.
2
(This is also a parabola)

Illustration 12: Find the locus of the centre of the circle described on any focal chord of a parabola y2 = 4ax as
diameter.  (JEE ADVANCED)

Sol: Use the parametric form.


Let P(at12, 2at1) and Q(at22, 2at2) be the extremities of a focal chord PQ of the parabola y2 = 4ax. Then, t1t2 = –1.
Let (h, k) be the coordinates of the centre of the circle described on PQ as the diameter. Then
h = a/2 (t12 + t22) and k = a(t1 + t2)
⇒ 2h/a = t1 + t2
2 2
and (k/a)2 = (t1 + t2)2
⇒ 2h/a = t12 + t22 and k2/a2 = t12 + t22 + 2t1t2
⇒ k2/a2 = 2h/a – 2 [ t1t2 = –1]
⇒ k2 = 2a(h – a)
Hence, the locus of (h, k) is y2 = 2a(x – a)

Illustration 13: A triangle ABC of area ∆ is inscribed in the parabola y2 = 4ax such that the vertex A lies at the vertex
of the parabola and side BC is a focal chord. Prove that the difference of the distances of B and C from the axis of
the parabola is 2∆/a.  (JEE ADVANCED)

Sol: Use the parametric form for the points B and C and proceed according to the conditions given in the question.
Let the coordinates of B and C be (at12, 2at1) and (at22, 2at2) respectively.
Since BC is a focal chord of the parabola y2 = 4ax. Therefore,
Since, ∆ = Area of ∆ABC
M a them a ti cs | 10.11

0 0 1
1 2
⇒ ∆ = at1 2at1 1 ⇒ ∆ = |a2t1t2(t1 – t2)| ⇒ ∆ =|–a2(t1 – t2)| [ t1t2 = –1]
2 2
at2 2at2 1

⇒ ∆ = a2|t2 – t1|
We have, BL = 2at1 and CM = 2at2 ∴ |BL – CM| = |2at1 – 2at2|
2a|t1 – t2| = 2a × ∆/a2 = 2∆/a

Illustration 14: Let PQ be a variable focal chord of the parabola y2 = 4ax whose vertex is A. Prove that the locus of
the centroid of ∆APQ is a parabola whose latus rectum is 4a/3. (JEE ADVANCED)

Sol: Take two general points on the focal chord of the parabola and use the formula for a centroid to obtain a
relation between the ordinate and the abscissae of the centroid.
Let the coordinates of P and Q be (at12, 2at1) and (at22, 2at2) respectively. Since PQ is a focal chord.
Therefore t1t2 = –1 … (i)
Let (h, k) be the coordinates of the centroid of ∆APQ. Then,
at12 + at22 2at1 + 2at2 3h
h= and k = ⇒ = t12 + t22 and 3k/2a = t1 + t2 … (ii)
3 3 a
2
 3k  3h 9k 2 3h − 2a
Now (t1 + t2)2 = t12 + t22 + 2t1t2 ⇒   = – 2 ⇒ =
2a
  a 4a 2 a
4a 4a  2a 
⇒ k2 = (3h – 2a) ⇒ k2 = h − 
9 3  3 
4a  2a 
Hence, the locus of (h, k) is y2 = x − 
3  3 
4a
Clearly, it represents a parabola whose latus rectum is
3

Illustration 15: A variable chord through the focus of the parabola y2 = 4ax intersects the curve at P and Q. The
straight line joining P to the vertex cuts the line joining Q to the point (–a, 0) at R. Show that the locus of R is y2 +
8x2 + 4ax = 0. (JEE ADVANCED)

Sol: Consider two points on the parabola and obtain the equation of the straight line passing through P and Q.
Then obtain the locus of the intersection point.
Let the coordinates of P and Q be (at12, 2at1) and (at22, 2at2) respectively. It is given that the chord PQ passes through
the focus S(–a, 0). Therefore t1t2 = –1.
2
Equation of OP is y = x  … (i)
t1
2at2 − 0 2t2
Equation of OQ is y – 0 = (x + a) ⇒ y = (x + a) … (ii)
2 2
at2 + a t2 + 1
2 2t2
Let R(h, k) be the point of intersection of OP and OQ. Then, k = h and k = (h + a)
t1 2
t2 + 1
2t2
⇒ k = –2ht2 and k = (h + a) [ t1t2 = –1]
2
t2 + 1
−k / h
⇒ k= (h + a) ⇒ k2 + 4h2 = –4h2 – 4ah ⇒ k2 + 8h2 + 4ah = 0
2
k
1+ 2
4h
Hence, the locus of (h, k) is obtained by replacing (h, k) with (x, y) in the above equation.
1 0 . 1 2 | Parabola

Illustration 16: AP is any chord of the parabola y2 = 4ax passing through the vertex A. PQ is a chord perpendicular
to AP. Find the locus of the mid-point of PQ. (JEE ADVANCED)

Sol: Use the parametric form to find the equation of the line PQ and then the locus of the mid-point of PQ.
Let the coordinates of P and Q be (at12, 2at1) and (at22, 2at2) respectively. Then
2 2
m1 = Slope of AP = m2 = Slope of PQ =
t1 t1 + t2
Since AT ⊥ PQ. Therefore, m1m2 = –1
4
⇒ = –1 ⇒ t1(t1 + t2) = –4 … (i)
t1 (t1 + t2 )

Let R(h, k) be the mid-point of PQ. Then, 2h = a(t12 + t22)  … (ii)


k = a(t1 + t2) … (iii)
kt1 −4a
From (i) and (iii), we have = –4 ⇒ t1 =
a k

From (i), we have t12 + t1t2 = –4 ⇒ t1t2 = –t12 – 4

16a2  4a 
⇒ t1t2 = – – 4  t1 = − 
k2  k 
4
⇒ t1t2 = − (4a2 + k 2 )  … (iv)
k2
k2 2h 8
Now (t1 + t2)2 = (t12 + t22) + 2t1t2 ⇒ = − (4a2 + k2) [Using (2), (3) and (4)]
a 2 a k2
⇒ k4 = 2k2ha – 8a2(4a2 + k2) Hence, the locus of (h, k) is
y4 = 2y2xa – 8a2(4a2 + y2) or, y4 + 8a2y2 – 2axy2 + 32a4 = 0

Illustration 17: Find a point K on axis of y2 = 4ax which has the property that if chord PQ of the parabola is drawn
1 1
through it then 2
+ is same for all positions of the chord.
PK KQ2
Sol: Consider a point of the axis of the parabola and use the distance form of a straight line to find the relation
1 1
between the parameters. The next step is to prove that 2
+ remains unchanged.
PK KQ2
1 1 x −b y −0
+ = constant = = r (r sinθ)2 = 4a( r cosθ + b)
PK 2
KQ 2 cos θ sin θ
P
⇒ r2sin2θ – 4a r cosθ – 4ab = 0 
K
r12 + r22 (r1 + r2 )2 − 2r1r2
⇒ =
r12 r22 r12 r22
Q

( ) 
( )
2
2 2 Figure 10.13
 (4acos θ) / (sin θ) − 2 ( −4ab) / (sin θ)  16a2 cos2 θ + 8ab sin2 θ
⇒  =
2 2 4
16a b / sin θ 16a2b2

8a(2acos2 θ + b sin2 θ)
f(θ) = ; f ′(θ) = sin2θ(b – 2a) = 0 b = 2a
16a2b2

Equation of the chord bisected at a given point: The equation of the chord of the parabola y2 = 4ax which is
bisected at (x1, y1) is yy1 – 2a(x + x1) = y12 – 4ax1 or T = S1
where S1 = y12 – 4ax1 and T = yy1 – 2a(x + x1)
M a them a ti cs | 10.13

6.2 Diameter of a Parabola


The locus of the middle points of a system of parallel chords is called a Y P(x1,y1)
diameter and in the case of a parabola this diameter is shown to be a straight
y=mx+c
line which is parallel to the axis of the parabola.
R(h,k)
The equation of the diameter bisecting the chords of the parabola y2 = 4ax, X’ X
A
with slope m, is y = 2a/m. Diameter

Y’ Q(x2,y2)

Figure 10.14
PLANCESS CONCEPTS
1
1. The area of a triangle formed inside the parabola y2 = 4ax is (y1 – y2)(y2 – y3)(y3 – y1) where y1, y2, y3
8a
are the ordinates of vertices of the triangle.
2. If the vertex and the focus of a parabola are on the x-axis and at distance a and a′ from the origin
respectively, then the equation of the parabola is y2 = 4(a′ – a)(x – a)

Vijay Senapathi (JEE 2009, AIR 71)

Illustration 18: A ray of light is coming along the line y = b, (b > 0) from the positive direction of the x-axis and
strikes a concave mirror whose intersection with the x-y plane is the parabola y2 = 4ax, (a > 0). Find the equation
of the reflected ray and show that it passes through the focus of the parabola. (JEE ADVANCED)

Sol: In this question, we need to use the concept of angle between two lines. Use this concept to find the equation
of the reflected ray and to show that the focus lies on the reflected ray.
 b2 
Let P be the point of incidence. Then P is the intersection of the line y = b and the parabola y2 = 4ax. ∴P =  , b 
 4a 
 
 b 
2
b 2
∴ The equation of the tangent PT at P is y.b = 2a  x +  or by = 2ax +  … (i)
 2  2

2a 2a
‘m’ of (i) is . So, tan θ =
b b
m − (2a / b) Y T
Let the slope of the reflected ray PQ be m. ∴ tan θ =
1 + m.(2a/ b) P 

2a m − (2a/ b) 2a y=b
m − (2a / b) 
or = ∴ =±
b 1 + m.(2a/ b) 1 + m.(2a/ b) b
X
2a  2a 4a2  2a 2a 4a2 2
or m – = ±  + m.  ∴ m – = + m. Q y =4ax
b b b2  b b b2

Figure 10.15
2a 2a 4a2  4a2  4a
and m – =– – m. ∴ m1 −  =
b b b2  2 
b  b

 4a2   4a2  4ab
and m  1 +  = 0. But m ≠ 0 ∴ m1 −  = 4a ⇒ m =
 b2   2 
b  b b − 4a2
2
 
4ab  b2 
∴ The equation of the reflected ray PQ is y – b =  x− 
b2 − 4a2  a2 
or (b2 – 4a2)y – b(b2 – 4a2) = 4abx – b3 or (b2 – 4a2)y – 4abx + 4a2b = 0

This will pass through the focus (a, 0) if (b2 – 4a2)0 – 4ab . a + 4a2b = 0, which is true.
∴ The reflected ray passes through the focus.
1 0 . 1 4 | Parabola

7. TANGENT
The equation of tangent at (x1, y1) to any conic section can be obtained by replacing x2 by xx1, y2 by yy1, x
x + x1 y + y1 xy1 + x1 y
by , y by and xy by and without changing the constant (if any) in the equation of the curve.
2 2 2

7.1 Equation of Tangent to Parabola (Standard Form)


The equation of the tangent to the parabola y2 = 4ax at a point (x1, y1) is given by yy1 = 2a(x + x1). The equation of
tangents to all standard forms of parabola at point (x1, y1) are given below for ready reference.

Equation of parabola Equation of tangent


y2 = 4ax yy1 = 2a(x + x1)
y = –4ax
2
yy1 = –2a(x + x1)
x2 = 4ay xx1 = 2a(y + y1)
x = –4ay
2
xx1 = –2a(y + y1)

7.2 Parametric Form


The equation of the tangent to the parabola y2 = 4ax at a point (at2, 2at) is given by ty = x + at2. The parametric
equations of tangents to all standard forms of parabola are as given below:

Equation of parabola Point of contact Equation of the tangent


y2 = 4ax (at2, 2at) ty = x + at2
y2 = –4ax (–at2, 2at) ty = –x + at2
x2 = 4ay (2at, at2) tx = y + at2
x2 = –4ay (2at, –at2) tx = –y + at2

7.3 Slope Form of a Tangent


The equation of the tangent with slope m to the parabola y2 = 4ax is y = mx + a/m
The equation of tangents to various standard forms of the parabola in terms of the slope of the tangent are as
follows:

Equation of parabola Equation of the tangent Condition of tangency Point of contact

a a  a 2a 
y2 = 4ax y = mx + c=  2, 
m m m m 

a a  a −2a 
y2 = –4ax y = mx – c=– − 2 , 
m m  m m 

a a  2a a 
x2 = 4ay x = my + c=  , 2
m m m m 

a a  −2a −a 
x2 = –4ay x = my – c=–  ,
m m 2 
 m m 
M a them a ti cs | 10.15

7.4 Point of Intersection of Tangents


The x-coordinate of the point of intersection of tangents at P(at12, 2at1) and Q(at22, 2at2) is the G.M. of the
x-coordinates of P and Q and the y-coordinate is the A.M. of the y-coordinates of P and Q that is, the tangents at
the points (at12, 2at1) and (at2, 2at2) intersect at (at1t2, a(t1 + t2))

7.5 Director Circle


The locus of the point of intersection of perpendicular tangents to a conic is known as its director circle. The
director circle of a parabola is its directrix.

7.6 Pair of Tangents


We see that two tangents can be drawn from a external point to a parabola. The two tangents are real and distinct
or coincident or imaginary according to whether the given point lies outside, on or inside the parabola and the
combined equation of the pair of tangents drawn from an external point (x1, y1) to the parabola y2 = 4ax is (y2 – 4ax)
(y12 – 4ax1) = {yy1 – 2a(x + x1)}2 or, SS1 = T2
where S ≡ y2 – 4ax, S1 = y12 – 4ax1 and T ≡ yy1 – 2a(x + x1).

7.7 Chord of Contact


The chord joining the points of contact of two tangents drawn from an external point P to a parabola is known as
the chord of contact of tangents drawn from P.
The chord of contact of tangents drawn from a point P(x1, y1) to the parabola y2 = 4ax is yy1 = 2a(x + x1)

7.8 Common tangents to two conics


In this section, we shall discuss some problems on finding the common tangents to two conics. The following
algorithm may be used to find the common tangents to two given conics.

Algorithm
Step I: Observe the equations of the two conics.
Step II: Identify the conic whose equation is either in standard form or it is reducible to standard form.
Step III: Write the equation of the tangent in slope form to the conic obtained in step II.
Step IV: Apply the condition that the tangent obtained in step III also touches the second conic and find the value
of m (m is the slope).
Step V: Substitute the value(s) of m obtained in step IV in the equation written in step III. The equation obtained
is the required tangent(s).

7.9 Important Results


(a) The tangent at the extremities of a focal chord of a parabola intersect at right angles on the directrix.
(b) The tangent at any point on a parabola bisects the angle between the focal distance of the point and the
perpendicular on the directrix from the point.
(c) The portion of the tangent to a parabola cut off between the directrix and the curve subtends a right angle
at the focus.
(d) The perpendicular drawn from the focus on any tangent to parabola, intersects on the tangent at the
vertex.
(e) The orthocentre of any triangle formed by three tangents to a parabola lies on the directrix.
1 0 . 1 6 | Parabola

(f) The circumcircle of the triangle formed by any three points on a parabola passes through the focus of the
parabola.
(g) The tangent at any point on a parabola is equally inclined to the focal distance of the point and axis of the
parabola.
(h) If SZ is perpendicular to the tangent at a point P of a parabola, then Z lies on the tangent at the vertex and
SZ2 = AS.SP, where A is the vertex of the parabola.
(i) The image of focus w.r.t. any tangent to a parabola lies on its directrix.
( j) The length of the subtangent at any point on a parabola is equal to twice the abscissa of that point.
(k) If the tangents to the parabola y2 = 4ax at the points P and Q intersect at T, then TP and TQ subtend equal
angles at the focus.

PLANCESS CONCEPTS

• The locus of the point of the intersection of the tangent at P and the perpendicular from the focus to
this tangent is the tangent at the vertex of the parabola.
• y = mx – am2 is a tangent to the parabola x2 = 4ay for all values of m and its point of contact is (2am, am2).
The formula given in the previous table for the parabola x2 = 4ay is different as in that case, the slope
of the tangent is 1/m. Don’t confuse between the two formulae
• Angle between the tangents at two points P(at12, 2at1) and Q(at22, 2at1) on the parabola
t2 − t1
y2 = 4ax is θ = tan–1
1 + t1 t2
B Rajiv Reddy (JEE 2012, AIR 11)

Illustration 19: Prove that the straight line y = mx + c touches the parabola y2 = 4a(x + a) if c = ma + a/m.
 (JEE MAIN)

Sol: Apply the tangency condition for standard form.


Equation of the tangent of slope m to the parabola y2 = 4a(x + a) is y = m(x + a) + a/m
 1
⇒ y = mx + a  m +  but the given tangent is y = mx + c ∴ c = am + a/m
 m

Illustration 20: A tangent to the parabola y2 = 8x makes an angle of 45º with the straight line y = 3x + 5. Find its
equation and its point of contact. (JEE MAIN)

Sol: Find the slopes of the lines making an angle of 45o. Then use the standard equation of the tangent to get the
answer.
3±1
Slope values of the required tangents are m = ⇒ m1 = – 2, m2 = 1/2
1  3
 Equation of the tangent of slope m to the parabola y2 = 4ax is y = mx + a/m
1 
∴ Tangents are y = –2x – 1 at  , – 2  and 2y = x + 8 at ( 8,8 ) .
2 

Illustration 21: Find the equation of the common tangents of the parabola y2 = 4ax & x2 = 4by.  (JEE MAIN)

Sol: Use the standard slope form of the equation of the tangent to find the slope and hence, the equation of the
common tangent.
M a them a ti cs | 10.17

Equation of tangent to y2 = 4ax is y = mx + a/m … (i)


1 b
Equation of tangent to x2 = 4by is x = m1y + b/m1 ⇒ y = x−  … (ii)
m1 (m )2
1
For common tangent, (i) & (ii) must represent the same line,
1/3
1 a b a  a
∴ = m and =– ⇒ = – m 2b ⇒ m = − 
m1 m m12 m  b
1/3 1/3
 a  b
∴ Equation of the common tangent is y = −  x + a − 
 b  a

Illustration 22: A chord of the parabola y2 = 4ax subtends a right angle at the vertex. Find the locus of the point
of intersection of tangents at its extremities. (JEE MAIN)

Sol: Find the relation between the parametric coordinates of the point subtending right angle at the vertex. Then,
use this relation to find the locus of the intersection of the tangents.
Let P(at12, 2at1) and Q(at22, 2at2) be two points on the parabola y2 = 4ax such that the chord PQ subtends a right
angle at the vertex O(0, 0). Then, Slope of OP × Slope of OQ = –1
2 2
⇒ × = –1 ⇒ t1t2 = –4 ….(i)
t1 t2
Let R(h, k) be the point of intersection of tangents at P and Q. Then,
h = at1t2 and k = a(t1 + t2) ⇒ h = –4a
Hence, the locus of R(h, k) is x = –4a.

Illustration 23: The inclinations θ and φ of two tangents to the parabola y2 = 4ax with the axis are given by tan θ
= 1/m and tan φ = m/2. Show that, as m varies, the point of intersection of the tangents traces a line parallel to the
directrix of the parabola. (JEE ADVANCED)

Sol: Use the parametric form to find the relation between the two points and hence, find the locus of the intersection
point.
Let P(at12, 2at1) and Q(at22, 2at2) be two points on the parabola y2 = 4ax such that the tangents at P and Q are
inclined at angles θ and φ with the axis of the parabola. Then
tan θ = Slope of the tangent at P
and, tan φ = Slope of the tangent at Q
⇒ tan θ = 1/t1 and tan φ = 1/t2
⇒ 1/m = 1/t1 and m/2 = 1/t2
1
⇒ 1/m × m/2 = ⇒ t1t2 = 2 … (i)
t1 t2
Let R(h, k) be the point of intersection of tangents at P and Q. Then
h = at1t2 and k = a(t1 + t2) ⇒ h = 2a [Using (1)]
Hence, the locus of R(h, k) is x = 2a, which is a line parallel to the directrix of the parabola.

Illustration 24: Tangents PQ and PR are drawn to a parabola y2 = 4ax. If p1, p2, p3 be the perpendiculars from P, Q
and R to any tangent of the parabola, prove that p1 is the geometric mean of p2 and p3. (JEE ADVANCED)

Sol: Consider two points on the parabola and find the point of intersection of the tangents at these points. Then,
consider a tangent in the general form and find p1, p2 and p3.
Let the coordinates of Q and R be (at12, 2at1) and (at22, 2at2) respectively. Then, the equations of the tangents at Q
and R are t1y = x + at12 , t2y = x + at22 respectively.
1 0 . 1 8 | Parabola

The coordinates of P are (at1t2, a(t1 + t2)). Let ty = x + at2 … (i)


be any tangent to the parabola y2 = 4ax
Then, p1 = Length of the perpendicular from P (at1t2, a(t1 + t2)) on (1)

at1 t2 − a(t1 + t2 )t + at2 (t − t1 )(t − t2 )


= =a
2
t +1 t2 + 1

p2 = Length of the perpendicular from Q(at12, 2at1) on (1)

2att1 − at12 − at2 a(t − t1 )2


⇒ p2 = = and, p3 = Length of the perpendicular from R(at22, 2at2) on (i)
2 2
t +1 t +1

2att2 − at22 − at2 a(t − t2 )2


p3 = = Clearly, p12 = p2p3.
2 2
t +1 t +1

Hence, p1 is the geometric mean of p2 and p3.

Illustration 25: Two straight lines are perpendicular to each other. One of them touches the parabola y2 = 4a( x + a)
and the other touches the parabola y2 = 4b(x + b). Prove that the point of intersection of the lines is x + a + b = 0.
 (JEE ADVANCED)

Sol: Consider the two equations of the tangent in the slope form. Find the intersection of the two tangents and
prove the above result.
The equation of the tangent of slope m1 to y2 = 4a(x + a) is
y = m1(x + a) + a/m1  … (i)
The equation of the tangent to the slope m2 to the parabola y2 = 4b(x + b) is
y = m2(x + b) + b/m2  … (ii)
It is given that (1) and (2) are perpendicular to each other. Therefore m2 = –1/m1
Putting m2 = –1/m1 in (1), we get y = –1/m1(x + b) – bm1  … (iii)
The x-coordinate of the point of intersection of (1) and (3) is obtained by subtracting (3) from (1),
 1   1   1 
we get 0 =  m1 +  x + a  m1 +  + b  m1 + 
 m1   m1   m1 
 1 
⇒ 0 = (x + a + b)m1  m1 +  ⇒ x + a + b = 0 ⇒ x = –(a + b).
 m1 

Clearly, the point (–(a + b), y) lies on the line x + a + b = 0 for all values of y. Thus, the point of intersection of (i)
and (ii) lies on the line x + a + b = 0.

Illustration 26: Prove that the circle circumscribing the triangle formed by any three tangents to a parabola passes
through the focus. (JEE ADVANCED)

Sol: Consider three points on a parabola and find the intersection of the tangents at these
points. Then, find the equation of the circle passing through these three points and prove P(t1)
B
that it passes through the focus. C
R(t3)
Let P(at12, 2at1), Q(at22, 2at2) and R(at32, 2at3) be the three points on the parabola y2 = 4ax.
A
Q(t2)
The equations of tangents at P and Q are
yt1 = x + at12 ; yt2 = x + at22 Figure 10.17
M a them a ti cs | 10.19

The point of intersection C of the tangents at P and Q is {at1t2, a(t1 + t2)}.


Similarly, other points of intersection of tangents are
B = {(at3t1, a(t3 + t1)}, A = {(at2t3, a(t2 + t3)}
Let the equation of the circumcircle of the ∆ABC be
x2 + y2 + 2gx + 2fy + c = 0  … (i)
(i) will pass through the focus (a, 0) if
a2 + 2ga + c = 0  … (ii)
A, B, C are points on (i). So
a2t12t22 + a2(t1 + t2)2 + 2g.at1t2 + 2f.a(t1 + t2) + c = 0  … (iii)
a2t22t32 + a2(t2 + t3)2 + 2g.at2t3 + 2f.a(t2 + t3) + c = 0  … (iv)
a2t32t12 + a2(t3 + t1)2 + 2g.at3t1 + 2f.a(t3 + t1) + c = 0  … (v)
(iii) – (iv) ⇒ a t2 (t1 – t3 ) + a (t1 – t3)(t1 + 2t2 + t3) + 2g.at2(t1 – t3) + 2f.a(t1 – t3) = 0
2 2 2 2 2

or a2t22(t1 + t3) + a2(t1 + 2t2 + t3) + 2gat2 + 2fa = 0  … (vi)


Similarly, (iv) – (v)
a2t32(t2 + t1) + a2(t1 + t2 + 2t3) + 2gat3 + 2fa = 0  … (vii)
Also, (vi) – (vii) ⇒ a2[t22(t1 + t3) – t32(t2 + t1) + t2 – t3] + 2ga(t2 – t3) = 0
or a[t2t3(t2 – t3) + t1(t22 – t32) + (t2 – t3)] + 2g(t2 – t3) = 0
or a[t2t3 + t1t2 + t1t3 + 1] + 2g = 0
∴ 2g = –a(1 + t1t2 + t2t3 + t3t1).
From t3 × (vi) – t2 × (vii), we get 2f = –a(t1 + t2 + t3 – t1t2t3).
Substituting the values of 2g, 2f in (iii), we get c = a2(t1t2 + t2t3 + t3t1)
∴ the equation of the circumcircle is x2 + y2 – a(1 + St1t2)x – a(St1 – t1t2t3)y + a2St1t2 = 0.
It passes through (a, 0) because a2 – a2(1 + St1t2) + a2 St1t2 = 0. Hence proved.

8. POLE AND POLAR


Let P be a point lying within or outside a given parabola. Suppose any straight line drawn through P intersects the
parabola at Q and R. Then, the locus of the point of intersection of the tangents to the parabola at Q and R is called
the polar of the given point P with respect to the parabola, and the point P is called the pole of the polar.
The polar of a point (x1, y1) with respect to the parabola y2 = 4ax is yy1 = 2a(x + x1)

PLANCESS CONCEPTS

• The chord of contact and the polar of any point on the directrix always pass through the focus.
• The pole of a focal chord lies on the directrix and the locus of the poles of the focal is a directrix.
• The polars of all points on the directrix always pass through a fixed point and this fixed point is the
focus.
• The polar of the focus is the directrix and the pole of the directrix is the focus.

Anurag Saraf (JEE 2009, AIR 226)


1 0 . 2 0 | Parabola

25
Illustration 27: The general equation to a system of parallel chords of the parabola y2 = x is 4x – y + k = 0. What
7
is the equation of the corresponding diameter? (JEE MAIN)

Sol: Solve the equation of the line and the parabola. Then use the definition of the diameter to find the answer.
Let PQ be a chord of the system whose equation is 4x – y + k = 0 … (i)
Where k is a parameter.
P
Let M(α, β) be the middle point of PQ. The locus of M is the required diameter.
M
25
The equation of the parabola is y2 = x
… (ii)
7 Q y =
2 25
x
2 7
7y
Solving (i) and (ii), 4 × – y + k = 0 or 28y2 – 25y + 25k = 0. 4x-y+k = 0
25
y + y2 25 25 Figure 10.17
Let its roots be y1, y2. Then β = 1 = = ∴ The equation of the locus of
2 2 × 28 56
25
M(α, β) is y =
56

Illustration 28: Find the locus of the middle points of the normal chords of the parabola y2 = 4ax. (JEE ADVANCED)

Sol: The locus of the middle points of the normal chords is nothing but the diameter
corresponding to the normal chords. Using this, we can easily find the answer. 2
P(at1 , 2at1)
Let PQ be a normal chord to the parabola y2 = 4ax, which is normal at P(at12, 2at1) M(,)
Let the chord extend to intersect the parabola again at Q(at2 , 2at2)
2
2
Q(at2 , 2at2)
The equation of the normal at P(at12, 2at1) is
Figure 10.18
y + t1x = 2at1 + at13 … (i)
Q(at22, 2at2) satisfies it. So
2at2 + t1 . at22 = 2at1 + at13
or 2a(t2 – t1) + at1(t22 – t12) = 0. As t1 ≠ t2, we get
2 + t1(t2 + t1) = 0  … (ii)
Let M(α, β) be the middle point of the normal chord. Then
at12 + at22 a 2 2
α= = (t + t )  … (iii)
2 2 1 2
2at1 + 2at2
β= = a(t1 + t2)  … (iv)
2
β β −2a
(4) ⇒ t1 + t2 = (2) ⇒ 2 + t1. = 0 ∴ t1 =
a a β
−2a  2a  β  2a  β 2a
∴ (2) ⇒ 2 +  t2 −  = 0 or −  t2 −  = 0 ∴ t2 = +
β  β  a  β  a β

Substituting t1, t2 in (3) we get

a  −2a   β 2a 
2 2
a  4a2 β2 4a2 
α =   + +   =  2 + 2 + 2 + 4
2  β   a β   2  β a β 
 

a  8a2 y 2  4a3 y2
∴ The equation of the required locus is x =  2 + 2 + 4 or x= + + 2a
2  y a  y2 2a
M a them a ti cs | 10.21

Illustration 29: Show that the locus of the poles of the tangents to the parabola y2 = 4ax with respect to the
parabola y2 = 4bx is the parabola ay2 = 4b2x.  (JEE ADVANCED)

Sol: Consider the general equation of the tangent to parabola. Taking a point as the pole find the polar w.r.t.
y2 = 4bx . Compare the two equations and prove the above result.
Any tangent to the parabola y2 = 4ax is ty = x + at2 ….(i)
Let (α, β) be the pole of (1) with respect to the parabola y = 4bx.
2

Then (1) is the polar of (α, β) with respect to y2 = 4bx ∴ (1) and yβ = 2b(x + α) are identical.
So, comparing these,
t 1 at2 β 2 α
= = ; ∴ t= , t =
β 2b 2bα 2b a
2
 β  α
∴   = or ab = 4b α
2 2
2b
  a

∴ The equation of the required locus of the poles is ay2 = 4b2x.

9. NORMAL

9.1 Equation of Normal


y1
Point Form: The equation of the normal to the parabola y2 = 4ax at a point (x1, y1) is given by y – y1 = – (x – x1).
2a
The equation of the normals to all standard forms of parabola at (x1, y1) are given below for ready reference.

Equation of the parabola Equation of the normal


y1
y2 =4ax y – y1 = – (x – x1)
2a

y2 = –4ax y1
y – y1 = (x – x1)
2a

x2 =4ay x1
x – x1 = – (y – y1)
2a
x2 = –4ay x1
x–x = (y – y1)
1
2a

SLOPE FORM: The equations of normals to various standard form of the parabola in terms of the slope of the
normal are as given below.

Equation of the parabola Equation of the normal (Feet of the normal)


y2 =4ax y = mx – 2am – am3 (am2, –2am)
y2 = –4ax y = mx + 2am + am3 (–am2, 2am)
x2 =4ay x = my – 2am – am3 (–2am, am2)
x2 = –4ay x = my + 2am + am3 (2am, –am2)

Also y = mx + c is normal to the parabola y2 = 4ax if c = –2am – am3.

Parametric Form: The equation of the normal to the parabola y2=4ax at point (at2, 2at) is given by y+tx=2at+at3.
The equation of normals to all standard forms of parabola in terms of parameter ‘t’ are listed below for ready
reference.
1 0 . 2 2 | Parabola

Equation of the parabola Parametric coordinates Equation of the normal


y2 =4ax (at2, 2at) y + tx = 2at + at3
y2 = –4ax (–at2, 2at) y – tx = 2at + at3
x2 =4ay (2at, at2) x + ty = 2at + at3
x2 = –4ay (2at, –at2) x – ty = 2at + at3

The point of intersection of normals at any two points P(at12, 2at1) and Q(at22, 2at2) on the parabola y2 = 4ax is R{2a
+ a(t12 + t22 + t1t2), – at1t2(t1 + t2)]
(at12, 2at1)
P
Y
R
X’ A X

Y’ Q
2
(at2 , 2at2)

Figure 10.19

Illustration 30: If the two parabolas y2 = 4ax and y2 = 4c(x – b) have a common normal other
b
than x-axis then prove that > 2 . (JEE MAIN)
a−c

Sol: Consider the slope of the normal to be m.


Write the equation of two normals w.r.t. the two parabolas
and solve them to prove the above inequality.
Normal to y2 = 4ax is y = mx – 2am – am3
Normal to y2 = 4c(x – b) is y = m(x – b) – 2cm – cm3
⇒ 2am + am3 = bm + 2cm + cm3 ⇒ m2(a – c) + 2(a – c) = b
Figure 10.20
b b
⇒ m2 = – 2 ⇒ > 2.
a−c a−c

9.2 Co-normal Points


We see that in general, three normals can be drawn from a point to a parabola. We shall also study the relations
between their slopes and conditions so that the three normals are distinct.
Co-normal Points: The points on the parabola at which the normals pass through a common point are called
co-normal points. The co-normal points are also called the feet of the normals.
Let P(h, k) be a point and y2 = 4ax be a parabola. The equation of any normal to the parabola y2 = 4ax is ;
y = mx – 2am – am3
If it passes through the point P(h, k) then
k = mh – 2am – am3 ⇒ am3 + m(2a – h) + k = 0  ….(i)
This is a cubic equation in m. So, it gives three values of m, say (m1, m2 and m3). Corresponding to each value of m
there is a normal passing through the point P(h, k).
Let, A, B, C be the feet of the normals. Then, AP, BP and CP are three normals passing through point P. Let m1, m2
and m3 respectively, be their slopes. Then, their equations are
y = m1x – 2am1 – am13, y = m2x – 2am2 – am23, y = m3x – 2am3 – am33
M a them a ti cs | 10.23

The coordinates of A, B and C are


A(am12, 2ma1), B(am22, –2am2) and C(am32, – am3).
Since, m1, m2, m3 are the roots of the equation (i). Therefore, we have
m1 + m2 + m3 = 0  ….(ii)
2a − h
m1m2 + m2m3 + m3m1 =  ….(iii)
a
−k
and, m1m2m3 =  ….(iv)
a
We have the following results related to co-normal points and the slopes of the normals at co-normal points.
Note:
(a) The algebraic sum of the slopes of the normals at co-normal point is zero.
(b) The sum of the ordinates of the co-normal points is zero.
(c) The centroid of the triangle formed by the co-normal points on a parabola lies on its axis.

9.3 Useful Results


(a) If the normal at the point P(at12, 2at1) meets the parabola y2 = 4ax again at (at22, 2at2), then
2
t2 = –t1 – .
t1
(b) The tangent at one extremity of the focal chord of a parabola is parallel to the normal at the other extremity.
(c) If the normals at points P(at12, 2at1) and Q(at22, 2at2) to the parabola y2 = 4ax meet on the parabola,
then t1.t2 = 2.
(d) If the normals at two points P and Q of a parabola y2 = 4ax intersect at a third point R on the curve, then the
product of the ordinates of P and Q is 8a2.
(e) If the normal chord at a point P(at2, 2at) to the parabola y2 = 4ax subtends a right angle at the vertex of the
parabola, then t2 = 2.
(f) The normal chord of a parabola at a point whose ordinate is equal to the abscissa subtends a right angle at
the focus.
(g) The normal at any point of a parabola is equally inclined to the focal distance of the point and to the axis of
the parabola.
(h) The sub-normal of a point on a parabola is always constant and equal to semi-latus rectum of the parabola.
(i) The normal at any point P of a parabola bisects the external angle between the focal distance of the point and
the perpendicular on the directrix from the point P.
Remark: It follows from this property, that is if there is a concave parabolic mirror whose intersection with xy-plane
is the parabola y2 = 4ax, then all rays of light coming from the positive direction of x-axis and parallel to the axis of
the parabola, after reflection, will pass through the focus of the parabola.

PLANCESS CONCEPTS

If a circle intersects a parabola at four points, then the sum of their ordinates is zero.
Anand K (JEE 2009, AIR 47)

Illustration 31: If a chord which is normal to y2 = 4ax at one end subtends a right angle at the vertex then find the
angle at which it is inclined to the axis. (JEE MAIN)
1 0 . 2 4 | Parabola

Sol: Let m1 and m2 be the slopes of the line joining the vertex and the two ends of the chord. Using the relation
between m1 and m2 find the slope of the chord and hence the angle it makes with the positive direction of X-axis.
2at1 2
m1 = = The point at which the normal intersects the parabola is (at12, 2at1)
at12 t1
2 2 2t1
t2 = –t1 – ⇒ m2 = = m1
t1 2 −t12 − 2 m3
t1 −
t1
4
m1m2 = -1 ⇒ = 1 ⇒ t12 + 2 = 4 m2
t12 + 2
2
t1 = ± 2 ⇒ m3 = = –t1
2
(at2 , 2at2)
t1 + t2
Figure 10.21
⇒ tan θ = ± 2 ⇒ θ = tan−1 ( ± 2)

Illustration 32: Find the equation of the normal to the parabola y2 = 4x, which is (i) parallel to the line y = 2x – 5,
(ii) perpendicular to the line 2x + 6y + 5 = 0. (JEE MAIN)

Sol: Use the slope form of the normal to get the two equations accordingly.
The equation of the normal to the parabola,
y2 = 4ax at (am2, –2am) is y = mx – 2am – am3
Where m is the slope of the normal. Here, a = 1. So, the equation of the normal at (m2, –2m) is
y = mx – 2m – m3 …(i)
(a) If the normal is parallel to the line y = 2x – 5. Then, m = Slope of the line y = 2x – 5 is 2
Putting the value of m in (i), we obtain y = 2x – 12
as the equation of the required normal at (4, –4).
(b) If the normal in (i) is perpendicular to the line 2x + 6y + 5 = 0. Then, m = 3
hence equation of the normal is y = 3x – 33

Illustration 33: Show that the distance between a tangent to the parabola y2 = 4ax and the parallel normal is a
sec2θ cosec θ, where θ is the inclination of the either of them with the axis of the parabola.  (JEE MAIN)

Sol: In the equation of the normal, use the concept of distance between two parallel lines to prove that the distance
between the tangent and the normal is a sec2θ cosec θ.
Let m be the slope of the tangent or parallel normal to the parabola y2 = 4ax. Then, m = tanθ.
The equations of the tangent and normal of slope m to the parabola y2 = 4ax are
y = mx + a/m and y = mx – 2am – am3
(a / m) + 2am + am3
The distance between these two parallel lines is given by d=
1 + m2

| c1 − c2 | a + 2am2 + am4 a(1 + m2 )2 a(1 + m2 )3/2


Using: d = ⇒ d= ⇒ ⇒ d=
a2 + b2 m 1 + m2 m 1 + m2 m

⇒ d = a(1 + tan2θ)3/2 cotθ [ m = tan q] ⇒ d = asec2θ cosecθ.

Illustration 34: Find the values of θ for which the line y = xcosθ + 4cos3θ – 14cosθ – 1 is a normal to the parabola
y2 = 16x. (JEE ADVANCED)
M a them a ti cs | 10.25

Sol: Compare the given equation with the standard equation of the normal and obtain the value of θ.
The slope of the given line is m = cosθ.
We know that the line y = mx + c is a normal to the parabola y2 = 4ax, if c = –2am – am3. Therefore, the given line
will be a normal to the parabola y2 = 16x, if
4cos3θ – 14cosθ – 1 = –18cosθ – 4cos3θ ; ⇒ 8cos3θ – 6cosθ = 1
⇒ 2(4cos3θ – 3 cosθ) = 1 ⇒ 2 cos 3θ = 1
1
⇒ cos3θ = ⇒ cos 3θ = cos π/3 = 1
2
2nπ π
⇒ 3θ = 2nπ ± π/3, n ∈ Z ⇒ θ = ± ,n∈Z
3 9
1
Illustration 35: Prove that three normals can be drawn from the point (c, 0) to the parabola y2 = x if c > and
2
then one of the normals is always the axis of the parabola. Also find c for which the other two normals will be
perpendicular to each other. (JEE ADVANCED)

Sol: The standard equation of the normal of a parabola is a cubic equation in ‘m’ (slope). Find the condition for
the cubic equation to have three real roots. Once we have the slope of the three normals, we can easily find the
condition for the two normals, other than the axis of the parabola, to be perpendicular to each other.
Let (t2, 1) be a foot of one of the normals to the parabola y2 = x from the point (c, 0).
Now, the equation of the normal to y2 = x at (t2, t) is
−1 −1  2 dy dy 1 
y–t= .(x − t2 ) ⇒ y–t= .(x − t2 )  y =x ⇒ 2y =1 ; ∴ = 
 dy  (1 / 2t)  dx dx 2y 
 
 dx t2 , t
⇒ y – t = –2t(x – t2) ⇒ y + 2tx = t + 2t3  ….(i)
It passes through (c, 0) if 0 + 2tc = t + 2t3
2c − 1
⇒ 2t3 + t(1 – 2c) = 0 ⇒ t[2t2 – (2c – 1)] = 0 ∴ t = 0, ±
2
Three normals can be drawn if t has three real distinct values.
1
So, 2c – 1 > 0, i.e., c > .
2
The foot of one of the normals is (t2, t) where t = 0, i.e., the foot is (0, 0).
From (i), the corresponding normal is y = 0, i.e., the x-axis which is the axis of the parabola.

2c − 1
For the other two normals t = ± .
2
2c − 1 2c − 1
From (i), ‘m’ of a normal = –2t ∴ ‘m’ of the other two normals are –2 . ,2. .
2 2
2c − 1 2c − 1
They are perpendicular if –2 . ×2. = –1
2 2

⇒ –2(2c – 1) = –1 ⇒ c = 3/4

10. SUBTANGENT AND SUBNORMAL


Let the parabola be y2 = 4ax. Let the tangent and normal at P(x1, y1) meet the axis of parabola at T and G respectively,
and the tangent at P(x1, y1) makes an angle Ψ with the positive direction of x-axis.
A(0, 0) is the vertex of the parabola and PN = y. Then,
1 0 . 2 6 | Parabola

(a) Length of tangent = PT = PN cosec Ψ = y1 cosec Ψ


(b) Length of normal = PG = PN cosec(90 – Ψ) = y1 sec Ψ
(c) Length of subtangent = TN = PN cot Ψ = y1 cot Ψ
(d) Length of subnormal = NG = PN cot(90º – Ψ) = y1 tan Ψ
2a
Where, tan Ψ = = m, {Slope of tangent at P(x, y)]
y1
Note: (a) Length of the tangent at (at2, 2at) = 2at cosec Ψ = 2at (1 + cot2 Ψ ) = 2at 1 + t2

(b) Length of the normal at (at2, 2at) = 2at sec Ψ = 2at (1 + tan2 Ψ ) = 2a t2 + t2 tan2 Ψ = 2a (t2 + 1)
(c) Length of subtangent at (at2, 2at) = 2at cotΨ = 2at2
(d) Length of subnormal at (at2, 2at) = 2at tanΨ = 2a.

PLANCESS CONCEPTS

Two parabolas are said to be equal when their latus rectums are equal.
The sub tangent at any point on the parabola is twice the abscissa or proportional to square of the
ordinate of the point.
GV Abhinav (JEE 2012, AIR 329)

PROBLEM-SOLVING TACTICS

A Working Rule to find the equation of a parabola when focus & directrix are given:
Step 1: Find the distance between focus and general point P(x, y) by the distance formula.
Step 2: Find the perpendicular distance from the point P(x, y) to the given directrix.
ax1 + by1 + c
(The perpendicular distance from a point P(x1, y1) to the line ax + by + c = 0 is )
a2 + b2
Step 3: Equate the distances calculated in step 1 and step 2. On simplification we get the required equation of the
parabola.

A Working Rule to find the equation of a parabola when the vertex and the focus are given:
y 2 − y1
Step 1: Find the slope of the axis formed by joining the focus and the vertex by the formula
x2 − x1
Step 2: Find the slope of the directrix by the formula m1.m2 = –1; where m1 is the slope of the axis of the parabola
and m2 is the slope of the directrix.
Step 3: Find a point on the directrix as the vertex, which is the middle point between the focus and the point on
the directrix, by means of the mid-point formula.
Step 4: Write the equation of the directrix, using the slope point formula.
Step 5: The focus and the directrix are now known so we can find the equation of the parabola by the method
given above.
M a them a ti cs | 10.27

FORMULAE SHEET

 1. Definition: A parabola is the locus of a point which moves so that its distance from a fixed point is equal to
its distance from a fixed straight line.
For e.g. if the focus is (α, β) and the directrix is ax + by + c = 0 then the equation of the parabola is
(ax + by + c)2
(x – α)2 + (y – β)2 =
a2 + b2
 2. The general equation of the second degree ax2 + 2hxy + by2 + 2gx + 2fy + c = 0 represents a parabola if
∆ ≠ 0 and h2 = ab.
 3.
Equation of the parabola
y2 = 4ax y2 = –4ax x2 = 4ay x2 = –4ay
Properties
Vertex (Co-ordinates) (0, 0) (0, 0) (0, 0) (0, 0)
Focus (Co-ordinates) (a, 0) (–a, 0) (0, a) (0, –a)
Latus rectum (length) 4a 4a 4a 4a
Axis (Equation) y =0 y =0 x=0 x=0
Directrix (Equation) x = –a x=a y = –a y=a
Symmetry (about) x-axis x-axis y-axis y-axis

 4. The equation of the chord joining points P(at12, 2at1) and Q(at22, 2at2) on the parabola y2 = 4ax is y(t1 + t2) =
2x + 2at1t2.

 5. If the equation of the chord joining points t1 and t2 on the parabola y2 = 4ax passes through the focus then
t1t2 = –1.
In other words, if one end of a focal chord of the parabola y2 = 4ax is P(at2, 2at) then the co-ordinates of the
 a −2a 
other end is Q  , .
 t2 t 
 a −2a 
 6. The length of the focal chord passing through P(at2, 2at) and Q  ,  is a(t + 1/t) .
2

 t 2 t 
 4  2
 7. The length of the chord intercepted by the parabola on the line y = mx + c is   a(1 + m )(a − mc) .
 m2 

 8. The length of the chord joining two points ‘t1’ and ‘t2’ on the parabola y2 = 4ax is a(t1 – t2) (t1 + t2 )2 + 4 .

 9. The equation of the tangent to the parabola y2 = 4ax at a point (x1, y1) is given by yy1 =2a(x +x1)
 10. Parametric Form

Equation of the parabola Point of contact Equation of the tangent


y = 4ax
2
(at , 2at)
2
ty = x + at2
y2 = –4ax (–at2, 2at) ty = –x + at2
x2 = 4ay (2at, at2) tx = y + at2
x2 = –4ay (2at, – at2) tx = –y + at2
1 0 . 2 8 | Parabola

 11. Slope form

Equation of the parabola Equation of the tangent Condition of tangency Point of contact

a a  a 2a 
y2 = 4ax y = mx + c=  2, 
m m m m 

a a  −a −2a 
y2 = –4ax y = mx – c=–  2, 
m m m m 

a a  2a a 
x2 = 4ay x = mx + c=  , 2
m m m m 

a a  −2a −a 
x2 = –4ay x = mx – c=–  ,
2 
m m  m m 

 12. The point of intersection of tangents at the points (at12, 2at1) and (at22, 2at2) is given by (at1t2, a(t1 + t2))

 13. If SZ be perpendicular to the tangent at a point P of a parabola, then Z lies on the tangent at the vertex and
SZ2 = AS × SP, where A is the vertex of the parabola.
 14. Angle between tangents at two points P(at12, 2at1) and Q(at22, 2at2) on the parabola y2 = 4ax is θ = tan–1
t2 − t1
1 + t1 t2

 15. Equation of normal in different forms

Equation of the parabola Equation of the normal

y1
y2 = 4ax y – y1 = – (x – x1)
2a

y1
y2 = –4ax y – y1 = (x – x1)
2a
x1
x2 = 4ay x – x1 = – (y – y1)
2a
x1
x2 = –4ay x – x1 = – (y – y1)
2a

Equation of the parabola Parametric coordinates Equation of the normal


y2 = 4ax (at2, 2at) y + tx = 2at + at3
y2 = –4ax (–at2, 2at) y – tx = 2at + at3
x2 = 4ay (2at, at2) x + ty = 2at + at3
x2 = –4ay (2at, – at2) x – ty = 2at + at3

Equation of the parabola Equation of the normal (Feet of the normal)


y2 = 4ax y = mx – 2am – am3 (am2, –2am)
y2 = –4ax y = mx + 2am + am3 (–am2, 2am)
x2 = 4ay x = my – 2am – am3 (–2am, am2)
x2 = –4ay x = my + 2am + am3 (2am, –am2)
M a them a ti cs | 10.29

 16. The point of intersection of normals at any two points P(at12, 2at1) and Q(at22, 2at2) on the parabola y2 = 4ax is
given by R[2a + a(t12 + t22 + t1t2), –at1t2(t1 + t2)]

 17. (i) The algebraic sum of the slopes of the normals at the co-normal point is zero.
(ii) The centroid of a triangle formed by the co-normal points on a parabola lies on its axis.
2
 18. If the normal at the point P (at12, 2at1) meets the parabola y2 = 4ax again at (at22, 2at2). Then t2 = –t1 – .
t1
 19. If the normal drawn at the point P(at12, 2at1) and Q(at22, 2at2) to the parabola y2 = 4ax intersect at a third point
on the parabola then t1.t2 = 2.

 20. If the normal chord at a point P(at2, 2at) to the parabola y2 = 4ax subtends a right angle at the vertex of the
parabola, then t2 = 2.

 21. The chord of contact of tangents drawn from a point P(x1, y1) to the parabola y2 = 4ax is yy1 = 2a(x + x1).

 22. The combined equation of the pair of tangents drawn from an external point (x1, y1) to the parabola y2 = 4ax is
SS1 = T2 where, S = y2 – 4ax, S1 = y12 – 4ax1 and T = yy1 – 2a(x + x1).

 23. The equation of the chord of the parabola y2 = 4ax which is bisected at (x1, y1) is
yy1 – 2a(x + x1) = y12 – 4ax1 or, T = S1.

 24. The polar of a point (x1, y1) with respect to the parabola y2 = 4ax is yy1 = 2a(x + x1).

 25. The equation of the diameter of the parabola y2 = 4ax bisecting chords of slope m is y = 2a/m.

 26. A circle on any focal radii of a point P(at2, 2at) as diameter touches the tangent at the vertex and intercepts a

chord of length a 1 + t2 on a normal at the point P.

 27. If the tangents at P and Q meet at T, then


(i) TP and TQ subtend equal angles at the focus S.
(ii) ST2 = SP×SQ, and
(iii) Δ SPT and Δ STQ are similar.

 28. Tangents and normals at the extremities of the latus rectum of a parabola y2 = 4ax constitute a square, their
points of intersection being (–a, 0) and (3a, 0).

 29. The semi latus rectum of the parabola y2 = 4ax, is the harmonic mean between segments of any focal chord
2bc 1 1 1
of the parabola, i.e. 2a = or, + = .
b+c b c a
 30. The orthocentre of any triangle formed by tangents at any three points P(t1), Q(t2) and R(t3) on a parabola
y2 = 4ax lies on the directrix and has the coordinates (–a, a(t1 + t2 + t3 + t1t2t3)).

 31. If a normal drawn to a parabola passes through a point P(h, k), then k = mh – 2am – am3, i.e,
am3 + m(2a – h) + k = 0,
2a − h k
⇒ m1 + m2 + m3 = 0; m 1m 2 + m 2m 3 + m 3 m 1 = ; and m1m2m3 = − .
a a
 32. The equation of a circle circumscribing the triangle formed by three co-normal points and which passes
through the vertex of the parabola is 2(x2 + y2) – 2(h + 2a)x – ky = 0.

 33. The area of a triangle formed inside the parabola y2 = 4ax is 1 (y1 – y2)(y2 – y3)(y3 – y1) where y1, y2, y3 are the
ordinates of the vertices of the triangle. 8a

 34. If the vertex and the focus of a parabola are on the x-axis and at a distance a and b from the origin respectively
then the equation of the parabola is y2 = 4(b – a)(x – a).
1 0 . 3 0 | Parabola

Solved Examples

JEE Main/Boards Y2 = 4ax is X = –a


⇒ x – 1/2 = –(–1) ⇒ x = 3/2
Example 1: If a parabolic reflector is 20 cm in diameter
is the equation of the directrix of the given parabola.
and 5 cm deep, find the focus.

Sol: Refer to Fig. 10.22. If we consider the origin to be Example 3: If the focus of a parabola divides a focal
the vertex of the parabola. Then we know that the point chord of the parabola in segments of length 3 and 2,
(5 , 10) will lie on the parabola. Using this we can solve the length of the latus rectum of the parabola is-
the question easily. (A) 3/2 (B) 6/5 (C) 12/5 (D) 24/5
M
Sol: Let y2 = 4ax be the equation of the parabola, then
10 cm the focus is S(a, 0). Let P(at12, 2at1) and Q(at22, 2at2) be
5 cm
A
B
x vertices of a focal chord of the parabola, then t1t2 = –1.
Let SP = 3, SQ = 2
10 cm

N SP = a2 (1 − t12 ) + 4a2 t12 = a(1 + t12) = 3  ..(i)

Let ‘MAN’ be the parabolic reflector such that MN is its  1


and SQ = a  1 +  = 2  ..(ii)
diameter and AB is its depth. It is given that AB = 5 cm  t12 

and MN = 20 cm
From (i) and (ii), we get t12 = 3/2 and a = 6/5. Hence, the
∴ MB = BN = 10 cm length of the latus rectum = 24/5.
Taking the equation of the reflector as
Example 4: The tangent at the point P(x1, y1) to the
y2 = 4ax  … (i)
parabola y2 = 4ax meets the parabola y2 = 4a(x + b) at
Co-ordinates of point M are (5, 10) and lies on (i). Q and R, the coordinates of the mid-point of QR are-
Therefore,
(A) (x1 – a, y1 + b) (B) (x1, y1)
(10)2 = 4(a)(5) ⇒ a=5
(C) (x1 + b, y1 + a) (D) (x1 – b, y1 – b)
Thus, the equation of the reflector is
Sol: Consider a mid point of the chord and find the
y2 = 20x
equation w.r.t. y2 = 4a(x + b). Compare this equation
Its focus is at (5, 0), i.e., at point B. with the equation of the tangent to y2 = 4ax and get the
Hence, the focus is at the mid-point of the given coordinates of the mid point.
diameter. Equation of the tangent at P(x1, y1) to the parabola y2 =
4ax is yy1 = 2a(x + x1)
Example 2: The equation of the directrix of the parabola
or 2ax – y1y + 2ax1 = 0 …(i)
y2 + 4y + 4x + 2 = 0 is-
If M(h, k) is the mid-point of QR, then the equation of
(A) x = –1 (B) x = 1 (C) x = –3/2 (D) x = 3/2
QR, a chord of the parabola y2 = 4a(x + b) in terms of
its mid-point is
Sol: Rewrite the given equation in the standard form
and compare with the equation of directrix. ky – 2a(x + h) – 4ab = k2 – 4a(h + b) (Using T = S′)
The given equation can be written as or 2ax – ky + k2 – 2ah = 0 …(ii)
(y + 2) = –4x + 2 = –4(x – 1/2)
2
Since (i) and (ii) represent the same line, we have
Which is of the form Y2 = 4aX 2a y 2ax1
= 1 =
Where Y = y + 2, X = x – 1/2, a = –1 2a k 2
k − 2ah
⇒ k = y1 and k2 – 2ah = 2ax1
The directrix of the parabola
M a them a ti cs | 10.31

⇒ y12 – 2ah = 2ax1 ⇒ 4ax1 – 2ax1 = 2ah ⇒ m2 = 1 ⇒ m = ±1


(As P(x1, y1) lies on the parabola y2 = 4ax) Hence, the common tangents are y = x + a and y = – x –
a, which intersect at the point (–a, 0) Which is the focus
⇒ h = x1 so that h = x1, k = y1 is the mid point of QR.
of the parabola y2 = –4ax.
Example 5: P is a point on the parabola whose ordinate
Example 7: The locus of the vertices of the family of
equals its abscissa. A normal is drawn to the parabola at
parabolas
P to meet it again at Q. If S is the focus of the parabola
then the product of the slopes of SP and SQ is- a3 x2 a2 x
y= + − 2a is-
(A) – 1 (B) 1/2 (C) 1 (D) 2 3 2

(A) xy = 64/105 (B) xy = 105/64


Sol: Proceed according to the given condition. Clearly,
the point with the same abscissa and the ordinate is the 3
(C) xy = (D) xy = 35/16
point (4a , 4a). 4

Let P(at2, 2at) be a point on the parabola y2 = 4ax, then Sol: Convert the given equation to the standard form.
at2 = 2at ⇒ t = 2 and thus the coordinates of P are The equation of the parabola can be written as
2
(4a, 4a).  ax
y 3 3
=  +  − –2
Equation of the normal at P is y = –tx + 2at + at3 a  
4  16
 3
⇒ y = –2x + 4a + 8a 2
 3  a2  35 
⇒ 2x + y = 12a  …(i)
or x +  = y + a
 4a  3a  16 
Which meets the parabola y = 4ax at points given by
2
Vertex is x = –3/4a, y = –35a/16
y2 = 2a(12a – y) ⇒ y2 + 2ay – 24a2 = 0
Locus of the vertex is xy = 105/64.
⇒ y = 4a or y = –6a
y = 4a corresponds to the point P Example 8: Find the locus of the foot of the
perpendicular drawn from a fixed point to any tangent
and y = – 6a ⇒ x = 9a from (i) to a parabola.
So that the coordinates of Q are (9a, –6a). Since the
coordinates of the focus S are (a,0), slope of SP = 4/3 Sol: Take a fixed point and use it to find the foot of the
and slope of SQ = –6/8. Product of the slopes = –1. perpendicular on a general equation of a tangent.
Let the parabola be y2 = 4ax and the fixed point be
Example 6: The common tangents to the circle (h, k)
x2 + y2 = a2/2 and the parabola y2 = 4ax intersect at the
(h,k)
focus of the parabola.
P(at2, 2at)
(A) x2 = 4ay (B) x2 = –4ay
M
(C) y2 = –4ax (D) y2 = 4a(x + a) (,)

Sol: In this case, first we need to find the two common


tangents and then find the point of intersection. Start The tangent at any point P(at2, 2at) is
with the standard equation of the tangent to a parabola ty = x + at2  ... (i)
and apply the condition of tangency on the circle to get
Let M(α, β) be the foot of the perpendicular to the
the slope of the tangents and proceed to find the point
tangent (i) from the point (h, k)
of intersection.
β −k 1
The equation of a tangent to the parabola y2 = 4ax is y Using perpendicularly, . = –1  … (ii)
= mx + a/m. α −h t
As M(α, β) is on (i), tβ = α + at2  … (iii)
If it touches the circle x2 + y2 = a2/2
We have to eliminate t from (ii) and (iii)
a a
=   1 + m2 ⇒ 2 = m2(1 + m2)
m 2 β −k
From (ii), t = − . Putting in (iii),
⇒ m4 + m2 – 2 = 0 ⇒ (m2 – 1)(m2 + 2) =0 α −h
1 0 . 3 2 | Parabola

 β −k 
2 The equation of the normal of a parabola
 β −k 
β −  = α + a.  
 α − h   α −h y2 = 4ax is y = –tx + 2at + at3  …(i)
or –β(β – k)(α – h) = α(α – h)2 + a(β – k)2 The joint equation of the lines joining the vertex (origin)
to the points of intersection of the parabola and the
∴ The equation of the locus of the foot M is
line (i) is
x(x – h)2 + y(x – h)(y – k) + a(y – k)2 = 0.  y + tx 
y2 = 4ax 
3
 2at + at 
Example 9: Tangents to the parabola at the extremities
of a common chord AB of the circle x2 + y2 = 5 and the ⇒ (2t + t3)y2 = 4x(y + tx)
parabola y2 = 4x intersect at the point T. A square ABCD
⇒ 4t x2 – (2t + t3)y2 + 4xy = 0
is constructed on this chord lying inside the parabola,
then [(TC)2 + (TD)2]2 is equal to ? Since these lines are at right angles co-efficient of x2 +
coefficient of y2 = 0
Sol: Find the point of intersection of the circle and the
⇒ 4t – 2t – t3 = 0 ⇒ t2 = 2
parabola. Then get the equation of the chord and the
point of intersection of the tangents at the end of the For t = 0, the normal line is y = 0, i.e. the axis of the
chord. In the last step use simple geometry to find parabola which passes through the vertex (0, 0).
[(TC)2 + (TD)2]2.
The points of intersection of the circle and the parabola Example 2: A parabola is drawn touching the x-axis at
are A(1, 2), B(1, –2) the origin and having its vertex at a given distance k
from the x-axis. Prove that the axis of the parabola is a
The equation of the common chord is x = 1, which is
tangent to the parabola x2 + 8k(y – 2k) = 0.
the latus rectum of the parabola.
x=-1 Sol: Use the relation between the tangent at the vertex
A(1,2)
D(5,2) and the axis of the parabola to prove it.

T Let the equation of the parabola be Y2 = 4ax.


t O
x=5
x=0 y=0
C(5,-2) x=0
B(1,-2)

∴ Tangents at the extremities of AB intersect on the V P T y=0


directrix x = –1. 2
(at , 2at) 2
tY - X - at =0
Coordinates of T are (–1, 0)
Any tangent to it at the point (at2, 2at) is
Since the length of AB = 4, the sides of the square ABCD
are of length 4, and the coordinates of C are Yt = X + at2 … (i)
(–5, 2) and of D are (5, 2). The normal at the point (at2, 2at) is
(TC)2 = (TD)2 = (5 + 1)2 + 4 = 40. Y + tX = 2at + at3 … (ii)
⇒ [(TC)2 + (TD)2]2 = 802 = 6400. Take the equations of transformation
tY − X − at2
= y … (iii)
JEE Advanced/Boards 1 + t2
Y + tX − 2at − at3
and = x … (iv)
Example 1: If the normal chord at a point ‘t’ on the 1 + t2
parabola y2 = 4ax subtends a right angle at the vertex, ∴ in x, y coordinates P = (0, 0) and PT is the x-axis
then a value of t is- which is the tangent to the parabola at the origin.
(A) 4 (B) 3 (C) 2 (D) 1 Now, (3) ⇒ tY – X – at2 = y 1 + t2  … (v)
Sol: Use the concept of homogenization of a conic and (4) ⇒ Y + tX – 2at – at3 = x 1 + t2  … (vi)
a straight line and then apply the condition of the sum
of the co-efficients of x2 and y2 equal to zero. ∴ (5) × t + (6) ⇒ (t + 1)Y – 2at – 2at
2 3
M a them a ti cs | 10.33

= yt 1 + t2 + x 1 + t2 Where Y = y, X = x – 8/k and A = k/4

∴ The axis of the parabola (Y = 0) becomes Equation of the directrix is


X = – A ⇒ x – 8/k = –k/4
–2at3 – 2at = (yt + x) 1 + t2
Which represents the given line x – 1 = 0
−2at(1 + t2 ) 2
or yt + x = = –2at 1 + t  … (vii) 8 k
1 + t2 If − =1
k 4
The distance of the vertex V(0, 0) in the X, Y coordinates
from PT ⇒ k2 + 4k – 32 = 0 ⇒ k = –8 or 4

−at2 For k = 4, the parabola is y2 = 4(x – 2) whose vertex


= = k (from the equation) is (2, 0) and touches the circle x2 + y2=4 at the vertex.
1 + t2 Therefore k ≠ 4.
∴ From (vii), the equation of the axis of the parabola in For k = –8, the parabola is y2 = –8(x + 1) which intersects
x, y coordinates becomes the circle x2 + y2 = 4 at two real distinct points.
 −at2 
yt + x = −2at  
 k  Example 4: A variable chord PQ of the parabola y = x2
 
subtends a right angle at the vertex. Find the locus of
2a2 t3 points of intersection of the normals at P and Q.
or yt + x – = 0 … (viii)
k
Sol: Take two points on the parabola and find the
The given parabola is x2 = –8k(y – 2k) … (ix) relation between the parametric coordinates. Use this
Solving (viii) and (ix), we get relation to find the locus.

1 2a2 t3  The vertex V of the parabola is (0, 0) and any point on y


x2 = –8k.  −x +  + 16k2 = x2 has the coordinates (t, t2).
t  k 
k So let us take P = (t1, t12), Q(t1, t22) and ÐPVQ = 90º
or x = 8 x – 16a2t2 + 16k2
2
t
2
or tx2 – 8kx + 16t(a2t2 – k2) = 0 P(t1, t1 )

Here, D = 64k2 – 64t2(a2t2 – k2) M

= 64[k2 – a2t4 + t2k2] V(0,0)


 a2 t 4 
= 64 (1 + t2 ). − a2 t 4  = 0 Q(t2, t22)
2
 1+t 
∴ The axis, given by (8) touches the given parabola. t12 − 0
As ‘m’ of VP = = t1
2 t1 − 0
at
Note: If we take k = , the points of intersection
t22 − 0
1 + t2 and ‘m’ of VQ = = t2,
of the axis and the given parabola will be imaginary. t2 − 0
VP ⊥ VQ ⇒ t1 . t2 = –1 … (i)
Example 3: If the line x – 1 = 0 is the directrix of the The equation of the normal to a curve at (x1,y1) is
parabola y2 – kx + 8 = 0, k ≠ 0 and the parabola intersects
the circle x2 + y2 = 4 in two real distinct points, then the −1
y – y1 = .(x − x1 )
value of k is-  dy 
 
(A) – 4 (B) – 8 (C) 4 (D) None  dx x
1 ,y1

Sol: Represent the parabola in the standard form. The normals at P & Q intersect at
Compare the equation of the directrix with the given
equation and form a quadratic in k. Solve the quadratic ( ( )
M(x,y)= 2a t12 + t22 + t1 t2 − at1 t2 ( t1 + t2 ) )
for two real roots to get the desired value of k.
From (1) t1 t2 =-1
The equation of the parabola can be written as
∴ y=a ( t1 + t2 ) x=2a( t12 + t22 -1)
y = k(x – 8/k) which is of the form Y = 4AX
2 2 ;
1 0 . 3 4 | Parabola

2
y= (
a2 t12 + t22 − 2 ) (ii) – (i) ⇒ –4aα =
4abm 1 + m2
1 + m2
y2 x
⇒ + 1 = ⇒ 2y 2 + 2a2 =
xa
a2 2a b2m2 m2 α2
or a2 = ∴ =  ... (iv)
Here, a=1/4 1 + m2 1 + m2 b2
2
∴ Locus is 16y= 2x − 1 Putting in (iii) from (iv)

a2 α2
Example 5: A parabola is drawn to pass through A and a2 + a2 + b2 = + b2
B, the ends of a diameter of a given circle of radius b2
a, and to have as directrix a tangent to a concentric  a2 
∴  1 − 2  a2 + b2 = b2 – a2
circle of radius b; the axes of reference being AB and  b 

a perpendicular diameter, prove that the locus of the
b2 − a2 α2 β2
x2 y2 or a2 + b2 = b2 – a2 ∴ + =1
focus of the parabola is + = 1. b2 b2 b2 − a2
b2 b2 − a2
Sol: Consider a circle with its centre at the origin. Let ∴ The equation of the locus of the focus (α, β) is
the two points A and B lie on the X-axis. Write the x2 y2
equation of the tangent in standard form and apply the + = 1.
focus-directrix property to prove the given statement. b2 b2 − a2

Let A = (–a, 0) and B = (a, 0) Example 6: Let (xr, yr) ; r = 1,2,3,4 be the points of the
The centre of the circle = (0, 0) intersection of the parabola y2 = 4ax and the circle

The equation of the concentric circle will be x2 + y2 + 2gx + 2fy + c = 0

x2 + y2 = b2 Prove that y1 + y2 + y3 + y4 = 0.

Any tangent to x2 + y2 = b2 is Sol: Solve the equation of the circle and the parabola.
y = mx + b 1 + m 2 Then use the theory of equations to prove y1 + y2 + y3
+ y4 = 0.
Which is the directrix of the parabola.
Let x2 + y2 + 2gx + 2fy + c = 0  … (i)
Let (α, β) be the focus.
y2 = 4ax  … (ii)
Then by focus-directrix property, the equation of the
parabola will be Solving (i) and (ii), we get the coordinates of points of
2
 y − mx − b 1 + m2  intersection
(x – α)2 + (y – β)2 =  
y2
 1+m 2  From (ii), x = putting in (i),
  4a
2
It passes through A (–a, 0), B (a, 0); so  y2  y2
  + y 2 + 2g. + 2fy + c = 0
2  4a  4a
 ma − b 1 + m2   
(a + α) + b = 
2 2  1  g
 1 + m2  or y 4 +  1 +  y 2 + 2fy + c = 0
  (4a)2
 2a 
m2a2 + b2 (1 + m2 ) − 2abm 1 + m2 It has four roots.
=  ... (i)
1 + m2
2
Its roots are y1, y2, y3 and y4.
 −ma − b 1 + m2
(a – α)2 + b2 =   coefficientof y 3
  Now, sum of roots = −
 1 + m2  coefficientof y 4
m a + b (1 + m ) + 2abm 1 + m2
2 2 2 2 0
=  ... (ii) ∴ y1 + y2 + y3 + y4 = − = 0.
1 + m2 1 / (4a)2

m2
or a2 + b2 = a2 + b2  ... (iii) Example 7: From the point, where any normal to the
1 + m2 parabola y2 = 4ax meets the axis, a line perpendicular to
M a them a ti cs | 10.35

the normal is drawn. Prove that this line always touches the line PQ whose equation is
the parabola y2 + 4a(x – 2a) = 0.  1
y  t −  = 2(x – a) is given by
Sol: Get the equation of the line perpendicular to the  t
normal, passing through the intersection of the normal 2a 2a
p= =
and the axis. Use the theory of equation 2
(t − 1 / t) + 22 (t + 1 / t)

The normal at any point (at2, 2at) of the parabola y2 = 4a2


So that .p2 = × a(t + 1/t)2 = 4a3,
4ax is y + tx = 2at + at3. (t + 1 / t)2
It cuts the axis y = 0 of the parabola at which is constant.
(2a + at2, 0).
2
(2a + at , 0) Paragraph for Example No. 9 to 11
2
(at , 2at) C: y = x2 – 3, D: y = kx2, L1: x = a, L2: x = 1. (a ≠ 0)

Example 9: If the parabolas C & D intersect at a point


A on the line L1, then the equation of the tangent line L
at A to the parabola D is-

∴ The equation of the line through this point drawn (A) 2(a3 – 3)x – ay + a3 – 3a = 0
perpendicular to the normals is (B) 2(a3 – 3)x – ay + a3 – 3a = 0
1 (C) (a3 – 3)x – 2ay – 2a3 + 6a = 0
y–0= (x – 2a + at2 )
t
(D) None of these
{ ‘m’ of normal = –t}
or ty = x – 2a – at2 … (i) Sol: In this case we need to calculate the point of
intersection of C and D and then find the equation of
We have to prove that (1) touches the parabola the tangent to the parabola y = kx2.
y2 + 4a(x – 2a) = 0 … (ii) C and D intersect at the points for which x2 – 3 = kx2.
Solving (i) and (ii), y2 + 4a(ty + at2) = 0 But x = a(given)
or y2 + 4aty + (2at)2 = 0 or (y + 2at)2 = 0 a2 − 3
⇒ k= .
∴ y = –2at, –2at a2
∴ (i) cuts (ii) at coincident points, i.e., (i) touches (ii). So the coordinates of A are (a, a2 – 3)
The equation of the tangent L at A to D: y = kx2 is
Example 8: Consider a parabola y2 = 4ax, the length 1 a2 − 3
of focal chord is  and the length of the perpendicular (y + a2 – 3) = xa
2 a2
from the vertex to the chord is p then-
⇒ 2(a2 – 3)x – ay – a3 + 3a = 0 (L)
(A)  .p is constant (B)  .p2 is constant
(C)  2.p is constant (D) None of these Example 10: If the line L meets the parabola C at a
point B on the line L2, other than A then a is equal to-
Sol: A quantity is constant if it does not depend on the
(A) –3 (B) –2 (C) 2 (D) 3
parameter. Represent  and p in terms of the parameter
and look for the quantity in which the parameter gets
eliminated. Sol: Proceed further from the previous solution.

Let P(at2, 2at) and Q(a/t2, –2a/t) be a focal chord of the The line L meets the parabola
parabola (as t1t2 = –1) C: y = x2 – 3 at the points for which
2 2 2 2
The length of PQ = = (at − a / t ) + (2at + 2a / t) 2(a2 − 3)
x2 – 3 = x − a2 + 3 ⇒ (x – a)(ax + 6 – a2) = 0.
= a (t2 − 1 / t2 )2 + 4(t + 1 / t)2 a
But x = 1 & x ≠ a.
= a(t + 1/t) (t − 1 / t)2 + 4 = a(t + 1/t)2
a2 − 6
The length of the perpendicular from the vertex (0,0) on ⇒x= = 1 ⇒ a2 – a – 6 = 0
a
1 0 . 3 6 | Parabola

Example 11: If a > 0, the angle subtended by the chord p q r s


AB at the vertex of the parabola C is- a p q r
(A) tan–1 (5/7) (B) tan–1 (1/2)
b q r s
(C) tan–1 (2) (D) tan–1 (1/8)
c p r s
Sol: Calculate the point of intersection of C and D
depending the value of ‘a’ and hence find the angle. d p q s

If a > 0, then a = 3. The coordinates of A and B are (3, 6)


and (1, –2) respectively, and the equation of C: is Sol: Use the standard results and simple transformations
to match the given option.
y = x2 – 3 or x2 = y + 3
For the parabola y2 = 4ax,
The coordinates of the vertex O of the parabola C are
(0, –3). The equation of the tangent at (x1, y1) is

Slope of OA = 3, slope of OB = 1 yy1 = 2a(x + x1)

∴ The angle between OA and OB is The equation of the normal at (x1, y1) is

3 −1 y1
tan–1 = tan–1(1/2). y – y1 = – (x – x1)
1+3 2a
⇒ xy1 = 2a(y1 – y) + x1y1
Example 12: Let y2 = 4ax be the equation of a parabola,
Next, the equation of the focal chord through (x1, y1)
then
and (a, 0) is
(A) yy1 = 2a(x + x1) (p) Equation of thenormal at (x1,y1) y1 − 0
y= (x – a)
(B) xy1=2a(y1–y)+x1y1 (q) Equation of the focal chord x1 − a
through (x1,y1) ⇒ xy1 = y(x1 – a) + ay1
(C) xy1=y(x1–a)+ay1 (r) Equation of the through (x1,y1) Lastly, the equation of the line joining (–a, 0), the point
and the point of intersection of axis of intersection of the axis y = 0 and the directrix x + a
with the direct = 0 with (x1, y1) is
(D) (x+a)y1=(x1+a)y (s) Equation of the tangent at (x1,y1) (x + a)y1 = (x1 + a)y

JEE Main/Boards

Exercise 1 Q.5 Show that lx + my + n = 0 will touch the parabola


y2 = 4ax, if ln = am2.
Q.1 Find the equation of the parabola whose focus is
(1, –1) and vertex is (2, 1). Q.6 If x + y + 1 = 0 touches the parabola y2 = lx, then
show that λ = 4.
Q.2 Find focus, vertex, directrix and axis of the parabola
4x2 + y – 3x = 0. Q.7 Find the equation of the tangent, to the parabola
y2 = 8x, which makes an angle of 45º with the line
y = 3x+5.
Q.3 The focal distance of a point on the parabola y2 =
12x is four units. Find the abscissa of this point.
Q.8 Find equation of the tangent and the normal to the
parabola y2 = 4x at the point (4, –4).
Q.4 A double ordinate of parabola y2 = 4ax is of length 8a.
Prove that the lines joining vertex to the end points of
this chord are at right angles. Q.9 Find the equation and the point of contact of the
tangents to y2 = 6x drawn from the point (10, –8).
M a them a ti cs | 10.37

Q.10 Find the equation of the common tangent to the Q.22 Show that the portion of the tangent to a parabola
parabola y2 = 32x and x2 = 108y. cut off between the directrix and the curve subtends a
right angle at the focus.
Q.11 Find the point where normal to the parabola
1 1 Q.23 If the tangent to the parabola y2 = 4ax meets
y2 = x at  ,  cuts it again. the axis in T and the tangent at the vertex A in Y and
 4 2
rectangle TAYG is completed, show that the locus of G
Q.12 Find shortest distance between y2 = 4x and is y2 + ax = 0.
x2 + y2 – 24y + 128 = 0.
Q.24 Two equal parabolas have the same vertex and
Q.13 AB is a chord of the parabola y = 4ax with the
2 their axes are at right angles. Prove that they cut again
end A at the vertex of the given parabola. BC is drawn 3
at an angle tan–1 .
perpendiculars to AB meeting the axis of the parabola 4
at C. Find the projection of BC on this axis.
Q.25 Find the locus of the point of intersection of the
Q.14 M is the foot of the perpendicular from a point P tangents to the parabola y2 = 4ax which include an
on the parabola y2 = (x – 3) to its directrix and S is the angle α.
focus of the parabola, if SPM is an equilateral triangle,
find the length of each side of the triangle. Q.26 Find the set of points on the axis of the parabola
y2 – 4x – 2y + 5 = 0 from which all the three normals
Q.15 PQ is a double ordinate of a parabola y2 = 4ax. If drawn to the parabola are real and distinct.
the locus of its points of trisection is another parabola
length of whose latus rectum is k times the length of Q.27 Show that the locus of points such that two of
the latus rectum of the given parabola, then find the the three normals to the parabola y2 = 4ax from them
value of k. coincide is 27ay2 = 4(x – 2a)3.

Q.16 Find the equation of the parabola, the extremities Q.28 If a circle passes through the feet of normals
of whose latus rectum are (1, 2) and (1, –4). drawn from a point to the parabola y2 = 4ax, Prove that
the circle also passes through origin.
Q.17 Prove that the normal chord to a parabola at the
point whose ordinate is equal to the abscissa subtends Q.29 The middle point of a variable chord of the parabola
a right angle at the focus.
y2 = 4ax lies on the line y = mx + c. Show that it always
2
Q.18 If from the vertex of the parabola y2 = 4ax a pair  2a   c
touches the parabola  y +  = 8a  x +  .
of chords be drawn at right angles to one another and  m  m
with these chords as adjacent sides a rectangle be
drawn, prove that the locus of the vertex of the farther
angle of the rectangle is the parabola y2 = 4a(x – 8a).
Exercise 2
Q.19 Prove that the locus of the middle points of
Single Correct Choice Type
all chords of the parabola y2 = 4ax which are drawn
through the vertex is the parabola y2 = 2ax. Q.1 The length of the chord intercepted by the parabola
y2 = 4x on the straight line x + y = 1 is-
Q.20 Show that the locus of the middle point of all
chords of the parabola y2 = 4ax passing through a fixed (A) 4 (B) 4 2 (C) 8 (D) 8 2
point (h, k) is y2 – ky = 2a(x – h).
Q.2 A parabola is drawn with its focus at (3, 4) and
Q.21 Prove that the area of the triangle formed by the vertex at the focus of the parabola y2 – 12x – 4y + 4 =
tangents at points t1 and t2 on the parabola y2 = 4ax 0. The equation of the parabola is-
a2 (A) x2 – 6x – 8y + 25 = 0 (B) x2 – 8x – 6y + 25 = 0
with the chord joining these two points is |t – t |3.
2 1 2
(C) x2 – 6x + 8y – 25 = 0 (D) x2 + 6x – 8y – 25 = 0
1 0 . 3 8 | Parabola

Q.3 The curve describes parametrically by x = t2 – 2t + (A) x = 3 cos t ; y = 4 sin t


2, y = t2 + 2t + 2 represents-
t
(B) x2 – 2 = –cos t; y = 4cos2
(A) Straight line (B) Pair of straight line 2
(C) Circle (D) Parabola (C) x = tan t; y = sec t
1
Q.4 If y = 2x – 3 is a tangent to the parabola (D) x = 1 − sint ; y = sin2 + cos
2
 1
y2 = 4a  x −  , then ‘a’ is equal to-
 3 Q.10 From an external point P, pair of tangent lines
14 −14 are drawn to the parabola, y2 = 4x. If q1 and q2 are the
(A) 1 (B) –1 (C) (D) inclinations of these tangents with the axis of x such
3 3
π
that, q1 + q2 = , then the locus of P is-
Q.5 Two tangents to the parabola y2=4a make angles 4
a1 and a2 with the x-axis. The locus of their point of (A) x – y + 1 = 0 (B) x + y – 1 = 0
cot α1 (C) x – y – 1 = 0 (D) x + y + 1 = 0
intersection if = 2 is-
cot α2
(A) 2y2 = 9ax (B) 4y2 = 9ax Q.11 From the point (4, 6) a pair of tangent lines are
drawn to the parabola, y2 = 8x. The area of the triangle
(C) y2 = 9ax (D) None
formed by these pair of tangent lines and the chord of
contact of the point (4, 6) is-
Q.6 Through the vertex ‘O’ of the parabola y2 = 4ax,
variable chords OP and OQ are drawn at right angles. If (A) 8 (B) 4 (C) 2 (D) None
the variable chord PQ intersects the axis of x at R, then
distance OR- Q.12 Let PSQ be the focal chord of the parabola,
(A) Varies with different positions of P and Q y2 = 8x. If the length of SP = 6 then, (SQ) is equal to-

(B) Equals the semi latus rectum of the parabola (A) 3 (B) 4 (C) 6 (D) None
(C) Equals latus rectum of the parabola
Q.13 The line 4x – 7y + 10 = 0 intersects the parabola, y2
(D) Equals double the latus rectum of the parabola = 4x at the points A and B. The coordinates of the point
of intersection of the tangents drawn at the points A
Q.7 A point P moves such that the difference between and B are-
its distances from the origin and from the axis of ‘x’ is
always a constant c. The locus- 7 5  5 7 5 7  7 5
(A)  ,  (B)  − ,  (C)  ,  (D)  − , 
(A) A straight line having equal intercepts C on the axis 2 2  2 2 2 2  2 2
 c
(B) A circle having its centre at  0, −  & passing Q.14 A line passing through the point (21,30) and
 2
 c normal to the curve y = 2 x can have the slope-
through  c 2, 
 2
(A) 2 (B) 3 (C) –2 (D) –5
 c
(C) A parabola with its vertex at  0, −  & passing
 2
 c Q.15 If the chord of contact of tangents from a point
through  c 2, 
 2 P to the parabola y2=4ax touches the parabola x2=4by,
(D) None of these the locus of P is-
(A) Circle (B) Parabola
Q.8 Tangents are drawn from the point (–1, 2) on
(C) Ellipse (D) Hyperbola
the parabola y2 = 4x. The length, these tangents will
intercept on the line x = 2, is-
Q.16 If M is the foot of the perpendicular from a point
(A) 6 (B) 6 2 (C) 2 6 (D) None P of a parabola y2=4ax its directrix and SPM is an
equilateral triangle, where S is the focus, the SP is equal
Q.9 Which one of the following equations represented to-
parametric-cally represents equation to a parabolic
(A) a (B) 2a (C) 3a (D) 4a
profile ?
M a them a ti cs | 10.39

Q.17 The latus rectum of a parabola whose focal chord Q.3 The equation of the directrix of the parabola y2 +
is PSQ such that SP = 3 and SQ = 2 is given by- 4y + 4x + 2 = 0 is- (2000)
(A) 24/5 (B) 12/5 (C) 6/5 (D) None (A) x = –1 (B) x = 1 (C) x = –3/2 (D) x = 3/2

Q.18 The normal chord of to a parabola y2 = 4ax at Q.4 The locus of the midpoint of the line segment
the point whose ordinate is equal to the abscissa, then joining the focus to a moving point on the parabola
angle subtended by normal chord at the focus is- y2=4ax is another parabola with directrix- (2000)
π π a a
(A) (B) tan–1 2 (C) tan-1 2 (D) (A) x = –a (B) x = − (C) x = 0 (D) x =
4 2 2 2
Q.19 P is any point on the parabola, y2=4ax whose Q.5 The equation of the common tangent to the curves
vertex is A. PA is produced to meet the directrix in D y2 = 8x and xy = –1 is- (2000)
and M is the foot of the perpendicular from P on the
directrix. The angle subtended by MD at the focus is- (A) 3y = 9x + 2 (B) y = 2x + 1

(A) π/4 (B) π/3 (C) 5π/12 (D) π/2 (C) 2y = x + 8 (D) y = x + 2

Q.20 A parabola y = ax2 + bx + c crosses the x-axis at Q.6 Let (x, y) be any point on the parabola y2 = 4x. Let
(α, 0), (β, 0) both to right of the origin. A circle also P be the point that divides the line segment from (0, 0)
passes through these two points. The length of a to (x, y) in the ratio 1: 3. Then, the locus of P is- (2011)
tangent from the origin to the circle is- (A) x2 = y (B) y2 = 2x (C) y2 = x (D) x2 = 2y
bc b c
(A) (B) ac2 (C) (D) Q.7 Let P(x1, y1) and Q(x2, y2), y1 < 0, y2 < 0, be the end
a a a
points of the latus rectum of the ellipse x2 + 4y2 = 4.
Q.21 TP and TQ are tangents to the parabola, y2 = 4ax The equations of parabolas with latus rectum PQ are-
at P and Q. If the chord PQ passes through the fixed  (2008)
point (–a, b) then the locus of T is-
(A) x2 + 2 3y=3+ 3 (B) x2 – 2 3y=3+ 3
(A) ay = 2b(x – b) (B) bx = 2a(y – a)
(C) x2 + 2 3y=3– 3 (D) x2 – 2 3y=3– 3
(C) by = 2a(x – a) (D) ax = 2b(y – b)
Q.8 Let L be a normal to the parabola y2 = 4x. If L passes
Q.22 The triangle PQR of area ‘A’ is inscribed in the through the point (9, 6), then L is given by- (2011)
parabola y2 = 4ax such that the vertex P lies at the
vertex of the parabola and the base QR is a focal chord. (A) y – x + 3 = 0 (B) y + 3x – 33 = 0
The modulus of the difference of the ordinates of the (C) y + x – 15 = 0 (D) y – 2x + 12 = 0
point Q and R is-
A A 2A 4A Q.9 The point of intersection of the tangents at the ends
(A) (B) (C) (D) of the latus rectum of parabola y2 = 4x is…….. (1994)
2a a a a

Q.10 Find the shortest distance of the point (0, c) from


the parabola y = x2 where 0 ≤ c ≤ 5.  (1982)
Previous Years’ Questions
Q.11 Find the equation of the normal to the curve
x2 = 4y which passes through the point (1, 2).  (1984)
Q.1 If x + y = k is normal to y2 = 12x, then k is- (2000)
(A) 3 (B) 9 (C) –9 (D) –3 Q.12 Through the vertex O of parabola y2 = 4x, chords
OP and OQ are drawn at right angles to one another.
Q.2 If the line x – 1 = 0 is the directrix of the parabola Show that for all positions of P, PQ cuts the axis of the
y2 – kx + 8 = 0, then one of the values of k is- (2000) parabola at a fixed point. Also find the locus of the
1 middle point of PQ. (1994)
1
(A) (B) 8 (C) 4 (D)
8 4
1 0 . 4 0 | Parabola

Q.13 From a point ‘A’ common tangents are drawn Q.15 At any point P on the parabola y2 – 2y – 4x + 5 = 0
a2 a tangent is drawn which meets the directrix at Q. Find
to the circle x2 + y2 = and parabola y2 = 4ax. Find the locus of point R, which divides QP externally in the
2
1
the area of the quadrilateral formed by the common ratio :1. (2004)
tangents, the chord of contact of the circle and the 2
chord of contact of the parabola.  (1996)
Q.16 The slope of the line touching both the parabolas
y 2 = 4x and x2 = −32 y is  (2014)
Q.14 The angle between a pair of tangents drawn from
1 3 1 2
a point P to the parabola y2 = 4ax is 45º. Show that the (A) (B) (C) (D)
locus of the point P is a hyperbola. (1998) 2 2 8 3

JEE Advanced/Boards

Exercise 1 Q.9 If the normal at P(18, 12) to the parabola y2 = 8x


cuts its again at Q, show that 9PQ = 80 10 .
Q.1 Find the equations of the tangents to the parabola
y2 = 16x, which are parallel & perpendicular respectively Q.10 O is the vertex of the parabola y2=4ax and L is
to the line 2x – y + 5 = 0. Also find the coordinates of the upper end of the latus rectum. If LH is drawn
their points of contact. perpendicular to OL meeting OX in H, prove that the
length of the double ordinate through H is 4a 5.
Q.2 Find the equations of the tangents of the parabola
y2 = 12x, which pass through the point (2, 5). Q.11 The normal at a point P to the parabola y2 = 4ax meets
its axis at G. Q is another point on the parabola such that
Q.3 Show that the locus of points of intersection of two QG is perpendicular to the axis of the parabola. Prove that
tangents to y2=4ax and which are inclined at an angle QG2 – PG2 = constant.
α is (y2 – 4ax)cos2α = (x + a)2sin2α.
Q.12 Find the condition on ‘a’ and ‘b’ so that the two
Q.4 Two straight lines one being a tangent to y2 = ax tangents drawn to the parabola y2 = 4ax from a point
and the other to x2 = 4by are right angles. Find the are normals to the parabola x2 = 4by.
locus of their point of intersection.
Q.13 Prove that the locus of the middle points of all
Q.5 Prove that the locus of the middle point of portion tangents drawn from points on the directrix to the
of a normal to y2=4ax intercepted between the curve parabola y2 = 4ax is y2(2x + a) = a(3x + a)2.
and the axis is another parabola. Find the vertex and
the latus rectum of the second parabola. Q.14 Prove that, the normal to y2 = 12x at (3, 6) meets
the parabola again in (27, –18) and circle on this normal
Q.6 Show that the locus of a point, such that two of the chord as diameter is x2 + y2 – 30x + 12y – 27 = 0.
three normals drawn from it to the parabola y2 = 4ax
are perpendicular is y2 = a(x – 3a). Q.15 Show that, the normals at the points (4a, 4a) and
at the upper end of the latus rectum of the parabola
Q.7 A variable chord t1t2 of the parabola y2 = 4ax. Show y2 = 4ax intersect on the same parabola.
that its passes through a fixed point. Also find the
co-ordinates of the fixed point. Q.16 If from the vertex of a parabola a pair of chords
be drawn at right angles to one another, & with these
Q.8 Through the vertex O of the parabola y2 = 4ax, a chords as adjacent sides a rectangle be constructed,
perpendicular is drawn to any tangent meeting it at P then find the locus of the outer corner of the rectangle.
and the parabola at Q. Show that OP.OQ = constant.
M a them a ti cs | 10.41

Q.17 Three normals to y2 = 4x pass through the point Q.28 A variable tangent to the parabola y2 = 4ax meets
(15, 12). Show that one of the normals is given by the circle x2 + y2 = r2 at P and Q. Prove that the locus of
y = x – 3 and find the equations of the others. the mid point of PQ is x(x2 + y2) + ay2 = 0.

Q.18 A circle is described whose centre is the vertex and Q.29 A variable chord PQ of a parabola y2=4ax, subtends
whose diameter is three-quarters of the latus rectum of a right angle at the vertex. Show that it always passes
a parabola y2 = 4ax. Prove that the common chord of through a fixed point. Also show that the locus of the
the circle and parabola bisects the distance between point of intersection of the tangents at P and Q is a
the vertex and the focus. straight line. Find the locus of the mid point of PQ.

Q.19 TP and TQ are tangents to the parabola and the


normals at P and Q meet at a point R on the curve. Exercise 2
Prove that the centre of the circle circumscribing the
triangle TPQ lies on the parabola 2y2 = a(x – a). Single Correct Choice Type

Q.20 Find the equation of the circle which passes Q.1 The tangent at P to a parabola y2=4ax meets the
through the focus of the parabola x2 = 4y and touches directrix at U and the latus rectum at V then SUV (where
it at the point (6, 9). S is the focus).
(A) Must be a right triangle
Q.21 P and Q are the point of contact of the tangents
(B) Must be an equilateral triangle
drawn from the point T to the parabola y2 = 4ax. If PQ
be the normal to the parabola at P, prove that TP is (C) Must be an isosceles triangle
bisected by the directrix.
(D) Must be a right isosceles triangle
Q.22 Prove that the locus of the middle points of the
Q.2 If the distances of two points P and Q from the focus
normal chords of the parabola y2 = 4ax is
of a parabola y2=4ax are 4 and 9, then the distance of
y 2 4a3 the point of intersection of tangents at P and Q from
+ = x – 2a.
2a y 2 the focus is-
(A) 8 (B) 6 (C) 5 (D) 13
Q.23 Two tangents to the parabola y2=8x meet the
tangent at its vertex in the point P and Q. If PQ = 4 units, Q.3 The chord of contact of the pair of tangents drawn
prove that the locus of the point of the intersection of from each point on the line 2x + y = 4 to the parabola
the two tangents is y2=8(x + 2). y2=–4x passes through a fixed point.

Q.24 Two perpendicular straight lines through the (A) (–2, 1) (B) (–2, –1)
focus of the parabola y2=4x meet its tangents to the (C) (1/2, 1/4) (D) (–1/2, –1/4)
parabola parallel to the perpendicular lines intersect in
the mid point of TT′.
Q.4 The locus of the foot of the perpendiculars drawn
from the vertex on a variable tangent to the parabola
Q.25 A variable chord PQ of the parabola y2 = 4x is
y2 = ax is-
drawn parallel to the line y=x. If the parameters of the
points P and Q on the parabola are p and q respectively, (A) x(x2 + y2) + ay2 = 0 (B) y(x2 + y2) + ax2 = 0
show, that p + q = 2. Also show that the locus of the (C) x(x2 – y2) + ay2 = 0 (D) None of these
point of intersection of the normals at P and Q is
2x – y = 12.
Q.5 Locus of the point of intersection of the perpendicular
tangents of the curve y2 + 4y – 6x – 2 = 0 is-
Q.26 Show that the circle through three points the
normals at which to the parabola y2 = 4ax are concurrent (A) 2x – 1 = 0 (B) 2x + 3 = 0
at the point (h, k) is 2(x2 + y2) – 2(h + 2a)x – ky = 0. (C) 2y + 3 = 0 (D) 2x + 5 = 0

Q.27 Find the condition such that the chord t1t2 of the
Q.6 If the tangent at the point P(x1, y1) to the parabola
parabola y2 = 4ax passes through the point (a, 3a). Find
y2 = 4ax meets the parabola y2 = 4a(x + b) at Q and R,
the locus of intersection of t tangents at t1 and t2 under
then the mid point of QR is-
this condition.
1 0 . 4 2 | Parabola

(A) (x1 + b, y1 + b) (B) (x1 – b, y1 – b) Q.15 A variable circle is drawn to touch the line
3x – 4y = 10 and also the circle x2 + y2 = 4 externally
(C) (x1, y1) (D) (x1 + b, y1)
then the locus of its centre is-

Q.7 The point (s) on the parabola y2 = 4x which are (A) Straight line
closest to the circle, x2 + y2 – 24y + 128 = 0 is/are- (B) Circle
(A) (0, 0) (B) (2, 2 2) (C) (4, 4) (D) None (C) Pair of real, distinct straight lines
(D) Parabola
Q.8 Length of the focal chord of the parabola y2 = 4ax
at a distance p from the vertex is- Q.16 The tangent and normal at P(t), for all real positive
2a 2
a 3
4a 3
p 2 t, to the parabola y2=4ax meet the axis of the parabola
(A) (B) (C) (D) in T and G respectively, then the angle at which the
p p 2
p 2
a2
tangent at P to the parabola is inclined to the tangent
Q.9 If two normals to a parabola y2=4ax intersect at at P to the circle through the points P, T and G is-
right angles then the chord joining their feet passes (A) cot–1t (B) cot–1t2 (C) tan–1t (D) tan–1t2
through a fixed point whose co-ordinates are-
(A) (–2a, 0) (B) (a, 0) (C) (2a, 0) (D) None Q.17 P is a point on the parabola y2=4x where abscissa
and ordinate are equal. Equation of a circle passing
Q.10 Locus of a point p if the three normals drawn through the focus and touching the parabola at P is-
from it to the parabola y2 = 4ax are such that two of (A) x2 + y2 – 13x + 2y + 12 = 0
them make complementary angles with the axis of the
(B) x2 + y2 – 13x – 18y + 12 = 0
parabola is-
(C) x2 + y2 + 13x – 2y – 14 = 0
(A) y2 = a(x + a) (B) y2 = 2a(x – a)
(D) None of these
(C) y2 = a(x – 2a) (D) y2 = a(x – a)

Q.18 A circle is described whose centre is the vertex


Q.11 A tangent to the parabola x2 + 4ay = 0 cuts the
and whose diameter is three – quarters of the latus
parabola x2 = 4by at A and B the locus of the mid point
rectum of the parabola y2 = 4ax. If PQ is the common
of AB is-
chord of the circle and the parabola and L1L2 is the latus
(A) (a + 2b)x2 = 4b2y (B) (b + 2a)x2 = 4b2y rectum, then the area of the trapezium PL1L2Q is-
(C) (a + 2b)y2 = 4b2x (D) (b + 2x)x2 = 4a2y 2+ 2 
(A) 3 2 a2 (B) 2 2 a2 (C) 4a2 (D)   a2
 2 
Q.12 The circle drawn on the latus rectum of the  
parabola 4y2 + 25 = 4(y + 4x) as diameter cuts the axis
of the parabola at the points- Multiple Correct Choice Type

1 1 9 1 1 1 1 9 Q.19 Let P, Q and R are three co-normal points on the


(A)  ,  ,  ,  (B)  ,  ,  , 
2 2 2 2 2 2 2 2 parabola y2 = 4ax. Then the correct statement(s) is/are-

1 1 1 7 1 9 (A) Algebraic sum of the slopes of the normals at P, Q


(C)  ,  , (0, 0) (D)  ,  ,  ,  and R vanishes
2 2 2 2 2 2
(B) Algebraic sum of the ordinates of the points P, Q
Q.13 The distance between a tangent to the parabola and R vanishes
y2 = 4Ax(A > 0) and the parallel normal with gradient (C) Centroid of the triangle PQR lies on the axis of the
1 is- parabola
(A) 4A (B) 2 2 A (C) 2A (D) 2A (D) Circle circumscribing the triangle PQR passes
through the vertex of the parabola
Q.14 The equation to the directrix of a parabola if the
two extremities of its latus rectum are (2, 4) and (6, 4) Q.20 Let A be the vertex and L the length of the latus
and the parabola passes through the point (8, 1) is- rectum of the parabola, y2 – 2y – 4x – 7 = 0. The equation
(A) y – 5 = 0 (B) y – 6 = 0 (C) y – 1 = 0 (D) y – 2 = 0 of the parabola with A as vertex, 2L, the length of the
M a them a ti cs | 10.43

latus rectum and the axis at right angles to the of the Q.2 The equation of the common tangent touching the
given curve is- circle (x – 3)2 + y2 = 9 and the parabola y2 = 4x above
the x-axis is- (2001)
(A) x2 + 4x + 8y – 4 = 0 (B) x2 + 4x – 8y + 12 = 0
(A) 3 y = 3x + 1 (B) 3 y = –(x + 3)
(C) x2 + 4x + 8y + 12 = 0 (D) x2 + 8x – 4y + 8 = 0
(C) 3 y = x + 3 (D) 3 y = –(3x + 1)
Q.21 Two parabolas have the same focus. If their
directrices are the x-axis and the y-axis respectively,
then the slope of their common chord is- Q.3 The focal chord to y2 = 16x is tangent to
(x – 6)2 + y2 = 2, then the possible values of the slope of
(A) 1 (B) –1 (C) 4/3 (D) 3/4
this chord, are- (2003)

Q.22 Equation of common tangent to the circle, (A) {–1, 1} (B) {–2, 2} (C) {–2, 1/2} (D) {2, –1/2}
x2 + y2 = 50 and the parabola, y2 = 40 x can be-
(A) x + y – 10 = 0 (B) x – y + 10 = 0 Q.4 Axis of a parabola is y = x and vertex and focus are
at a distance 2 and 2 2 respectively from the origin.
(C) 1 + y + 10 = 0 (D) x – y – 10 = 0
Then equation of the parabola is- (2006)
(A) (x – y)2 = 8(x + y – 2) (B) (x + y)2 = 2(x + y – 2)
Q.23 The equation y2 + 3 = 2(2x + y) represents a
parabola with the vertex at- (C) (x – y)2 = 4(x + y – 2) (D) (x + y)2 = 2(x – y + 2)
1 
(A)  ,1  and axis parallel to x-axis Q.5 Normals at P, Q, R are drawn to y2=4x which
2 
intersect at (3, 0). Then
 1
(B)  1,  and axis parallel to x-axis Column I Column II
 2
(A) Area of DPQR (p) 2
1  3 
(C)  ,1  and focus at  ,1 
 2  2  (B) Radius of circumcircle of DPQR 5
(q)
1  2
(D)  ,1  and axis parallel to y-axis
2  (C) Centroid of DPQR 5 
(r)  ,0 
2 
Q.24 Let y2 = 4ax be a parabola and x2 + y2 + 2bx =
0 be a circle. If parabola and circle touch each other (D) Circumcentre of DPQR 2 
externally then- (s)  ,0 
3 
(A) a > 0, b > 0 (B) a > 0, b < 0
(C) a > 0, b > 0 (D) a < 0, b < 0 Q.6 Equation of common tangent of y=x2 and y=–x2 +
4x – 4 is- (2006)
Q.25 P is a point on the parabola y2 = 4ax (a > 0) whose (A) y = 4(x – 1) (B) y = 0
vertex is A. PA is produced to meet the directrix in D and (C) y = –4(x – 1) (D) y = –30x–50
M is the foot of the perpendicular P on the directrix. If a
circle is described on MD as a diameter then it intersects
the x-axis at a point whose co-ordinates are- Q.7 The tangent PT and the normal PN to the parabola
y2 = 4ax at a point P on it meet its axis at points T and
(A) (–3a, 0) (B) (–a, 0) (C) (–2a, 0) (D) (a, 0)
N, respectively. The locus of the centroid of the triangle
PTN is a parabola whose- (2009)
Previous Years’ Questions (A) Vertex is  2a ,0  (B) Directrix is x = 0
 3 
Q.1 The curve described parametrically by x = t2 + t + 1,
y = t2 – t + 1 represent (1999) (C) Latus rectum is 2a (D) Focus is (a, 0)
3
(A) A pair of straight lines (B) An ellipse
(C) A parabola (D) A hyperbola
1 0 . 4 4 | Parabola

Q.8 Let A and B be two distinct points on the parabola Q.16 Let L be a normal to the parabola y2 = 4x. If L
y2 = 4x. If the axis of the parabola touches a circle of passes through the point (9, 6), then L is given by
radius r having AB as its diameter, then the slope of the  (2011)
line joining A and B can be- (2010) (A) y – x + 3 = 0 (B) y + 3x – 33 = 0
1 1 2 2 (C) y + x – 15 = 0 (D) y – 2x + 12 = 0
(A) − (B) (C) (D) −
r r r r
Q.17 Let (x, y) be any point on the parabola y 2 = 4x .
Q.9 Consider the parabola y2 = 8x. Let D1 be the area Let P be the point that divides the line segment from
of the triangle formed by the end points of its latus (0, 0) to (x, y) in the ratio 1 : 3. Then the locus of P is
1   (2011)
rectum and the point P  ,2  on the parabola and D2
2  (A) y 2 = y (B) y 2 = 2x (C) y 2 = x (D) x2 = 2y
be the area of the triangle formed by drawing tangents
∆ Q.18 Let the straight line x = b divide the area
at P and at the end points of the latus rectum. Then 1
∆2
(1 − x ) , y =
2
enclosed by y = 0 and x = 0 into two parts
is …… (2011)
1
R1 ( 0 ≤ x ≤ b ) and R 2 (b ≤ x ≤ 1 ) such that R1 − R 2 = .
4
Q.10 Suppose that the normals drawn at three different
Then b equals  (2011)
points on the parabola y2 = 4x pass through the point
(h, 0). Show that h > 2. (1981) 3 1 1 1
(A) (B) (C) (D)
4 2 3 4
Q.11 Three normals are drawn from the point (c, 0) to
1 Q.19 Consider the parabola y 2 = 8x . Let ∆1 be the area
the curve y2 = x. Show that c must be greater than
2 of the triangle formed by the end points of its latus and
. One normal is always the x-axis. Find c for which the 1 
other two normals are perpendicular to each other. the point P  ,2  on the parabola, and ∆2 be the area
2 
 (1991) of the triangle formed by drawing tangents at P and at

Q.12 Show that the locus of a point that divides a chord the end points of the latus rectum. Then 1 is (2011)
∆2
of slope 2 of the parabola y2 = 4ax internally in the
ratio 1: 2 is a parabola. Find the vertex of this parabola. Paragraph (Questions 20 and 21)
 (1995)
Let PQ be a focal chord of the parabola y 2 = 4ax . The
Q.13 Points A, B and C lie on the parabola y = 4ax. The
2
tangents to the parabola at P and Q meet at a point
tangents to the parabola at A, B and C, taken in pairs, lying on the line y = 2x + a, a > 0.
intersect at points P, Q and R. Determine the ratio of the
areas of the triangle ABC and PQR. (1996) Q.20 Length of chord PQ is

Q.14 Let C1 and C2 be, respectively, the parabolas (A) 7a (B) 5a (C) 2a (D) 3a
x2 = y – 1 and y2 = x – 1. Let P be any point on C1 and
Q be any point on C2. Let P1 and Q1 be the reflections Q.21 If chord PQ subtends an angle θ at the vertex of
of P and Q, respectively, with respect to the line y = x. y 2 = 4ax , then tan θ =
Prove that P1 lies on C2, Q1 lies on C1 and PQ ≥ min
{PP1, QQ1}. Hence or otherwise, determine points 2 −2 2 −2
(A) 7 (B) 7 (C) 5 (D) 5
P0 and Q0 on the parabolas C1 and C2 respectively such 3 3 3 3
that P0Q0 ≤ PQ for all pairs of points (P, Q) with P on C1
and Q on C2. (2000) Q.22 A line L : y = mx + 3 meets y-axis at E(0, 3) and
the arc of the parabola = y 2 16 x, 0 ≤ y ≤ 6 at the point
Q.15 Normals are drawn from the point P with slopes F ( x0 , y 0 ) . The tangent to the parabola at F ( x0 , y 0 )
m1, m2, m3 to the parabola y2 = 4x. If locus of P with intersects the y -axis at G(0, y1). The slope m of the line
m1m2 = α is a part of the parabola itself, then find α. L is chosen such that the area of the triangle EFG has a
 (2003) local maximum.
M a them a ti cs | 10.45

Match List I with List II and select the correct answer Q.23 The value of r is  (2014)
using the code given below the lists: (2013) 1 2
t +1 1 2
t −1
(A) − (B) (C) (D)
List I List II t t t t

1 Q.24 If st = 1, then the tangent at P and the normal


(i) m= (p)
2 at S to the parabola meet at a point whose ordinate is
 (2014)
(t ) ( )
(ii) Maximum area of ∆ EFG is (q) 4 2
2 2
−1 a t2 − 1
(A) (B)
(iii) y0 = (r) 2 2t3 2t3

( ) ( )
2 2
(iv) y1 = (s) 1 a t2 − 1 a t2 + 2
(C) (D)
t3 t3
(A) i → s, ii → p, iii → q, iv → iii
x2 y 2
(B) i → r, ii → s, iii → p, iv → ii Q.25 Suppose that the foci of the ellipse + =1
9 5
(C) i → p, ii → r, iii → q, iv → iv are (f1, 0) and (f2, 0) where f1 > 0 and f2 > 0. Let P1 and
(D) i → p, ii → r, iii → s, iv → ii P2 be two parabolas with a common vertex at (0, 0) and
with foci at (f1, 0) and (2f2, 0), respectively. Let T1 be a
tangent to P1 which passes through (2f2, 0) and T2 be a
Paragraph (Questions 23 and 24)
tangent P2 which passes through (f1, 0), The m1 is the
Let a, r, s, t be non-zero real numbers. Let P(at2, 2at), slope of T1 and m2 is the slope of T2, then the value of
Q (ar2, 2ar) and S(as2, 2as) be distinct points on the  1 2
parabola y2 = 4ax. Suppose that PQ is the focal chord  2 + m2  is  (2015)
m 
and lines QR and PK are parallel, where K is the point
(2a, 0).

PlancEssential Questions
JEE Main/Boards JEE Advanced/Boards
Exercise 1 Exercise 1

Q.12 Q.15 Q.18 Q.19 Q.7 Q.13 Q.18 Q.21


Q.23 Q.27 Q.29 Q.25 Q.29

Exercise 2 Exercise 2
Q.1 Q.10 Q.16 Q.21 Q.1 Q.7 Q.9 Q.15
Q.19 Q.21 Q.22 Q.25
Previous Years’ Questions
Q.4 Q.7 Q.12 Q.15 Previous Years’ Questions
Q.3 Q.6 Q.7 Q.9
Q.11 Q.14
1 0 . 4 6 | Parabola

Answer Key

JEE Main/Boards
Exercise 1
3 1 3 9 
Q.1 4x2 + y2 – 4xy + 8x + 46y – 71 = 0 Q.2  ,  ,  ,  , 8y–5 = 0, 8x–3=0
 8 2   8 16 
Q.3 1 Q.7 y + 2x + 1 = 0, 2y = x + 8
 50 
Q.8 x + 2y + 4 = 0, y – 2x + 12 = 0
Q.9 x + 2y + 6 = 0 at (6, –6) 3x + 10y + 50 = 0 at  , −10 
 3 
9 3
Q.10 2x + 3y + 36 = 0 Q.11  , − 
 4 2
Q.12 4( 5 − 1) Q.13 y (y/x) = y2/x = 4ax/x = 4a
Q.14 2 × 4 = 8 Q.15 k = 1/9
Q.16 (y + 1)2 = –3(2x – 5) ; (y + 1)2 = 3(2x + 1) Q.25 (x + a)2 tan2α = y2 – 4ax
Q.26 {(x, 1) ; x > 3}

Exercise 2
Single Correct Choice Type

Q.1 C Q.2 A Q.3 D Q.4 D Q.5 A Q.6 C


Q.7 C Q.8 B Q.9 B Q.10 C Q.11 C Q.12 A
Q.13 C Q.14 D Q.15 D Q.16 D Q.17 A Q.18 D
Q.19 D Q.20 D Q.21 C Q.22 C

Previous Years’ Questions

Q.1 B Q.2 C Q.3 D Q.4 C Q.5 D Q.6 C


1 1
Q.7 B,C Q.8 A,B,D Q.9 (–1, 0) Q.10 c− , ≤ c ≤ 5 Q.11 x + y = 3
4 2
15a2
Q.12 y2 = 2(x – 4) Q.13 Q.15 (x + 1)(y – 1)2 + 4 = 0 Q.16 A
4

JEE Advanced/Boards

Exercise 1
Q.1 2x – y + 2 = 0, (1, 4) ; x + 2y + 16 = 0, (16, –16) Q.2 3x – 2y + 4 = 0 ; x – y + 3 = 0
Q.4 (ax + by)(x2 + y2) + (ay – bx)2 = 0 Q.5 (a, 0) ; a
Q.7 [a(t02 + 4), –2at0] Q.12 a2 > 8b2
Q.16 y2 = 4a(x – 8a) Q.17 y = –4x + 72, y = 3x – 33
M a them a ti cs | 10.47

Q.20 x2 + y2 + 18x – 28y + 27 = 0 Q.27 2 – 3(t1 + t2) + 2t1t2 = 0 ; 2x – 3y + 2a = 0

Q.28 x (x2 + y2) + ay2 = 0 Q.29 (4a, 0) ; x + 4a = 0 ; y2 =2a(x – 4a)

Exercise 2
Single Correct Choice Type

Q.1 C Q.2 B Q.3 A Q.4 D Q.5 D Q.6 C

Q.7 C Q.8 C Q.9 B Q.10 D Q.11 A Q.12 A

Q.13 B Q.14 B Q.15 D Q.16 C Q.17 A Q.18 D

Multiple Correct Choice Type

Q.19 A, B, C, D Q.20 A, B Q.21 A, B Q.22 B, C Q.23 A, C Q.24 A, D

Q.25 A, D

Previous Years’ Questions


Q.1 C Q.2 C Q.3 A Q.4 A Q.5 A → p ; B → q ; C → s ; D → r

Q.6 A,B Q.7 A,D Q.8 C,D Q.9 2 Q.10 h > 2 Q.11 c = 3/4

2 8 1 5  5 1
Q.12  ,  Q.13 2 Q.14 P0  ,  , Q0  ,  Q.15 α = 2 Q.16 A,B,D
9 9 2 4  4 2
Q.17 C Q.18 B Q.19 2 Q.20 B Q.21 D Q.22 A

Q.23 B Q.24 D Q.25 4

Solutions

JEE Main/Boards ∴ x + 2y – 9 = 0
Let (x, y) be the point on parabola
Exercise 1 x + 2y − 9
∴ = (x − 1)2 + (y + 1)2
5
Sol 1: Vertex is the midpoint of point of intersection
∴ (x + 2y – 9)2 = 5 [(x – 1)2 + (y +1)2]
of directrix (let say M(a, b)) & the focus and axis of
parabola. ∴ x2+ 4y2 + 81 + 4xy – 18x – 36y
 a+1 b −1 = 5x2 + 5y2 – 10x +10 y + 10
∴ (2, 1) =  , 
 2 2  ∴ 4x2 + y2 – 4xy + 8x + 46y – 71 = 0
∴ M(a, b) = (3, 3)
9 9
Also directrix ⊥ to axis. Sol 2: 4x2 – 3x + =–y+
16 16
1
∴ Slope of directrix = –  9 
2 2  −y + 
 3 16
The equation of directrix is (y – 3) = –
1
(x – 3) ∴  2x −  =  
2  4 1
1 0 . 4 8 | Parabola

2 Sol 7: Let slope of line be m


 3 1 9 
∴ x −  =  −y +  m−3
 8 4 16  ∴ tan ±450 =
1 + 3m
9 3 (m − 3)2
Let Y = -y + &X=x 1=
16 8
(3m + 1)2
Y
∴ X2 = – ∴ 8m2 + 12m – 8 = 0
4
1 2m2 + 3m – 2 = 0 ⇒ (2m – 1) (m + 2) = 0
Comparing to X2 = – 4aY ⇒ a =
16 1
∴ m = – 2 or m =
Vertex is (X = 0, Y = 0) 2
a
∴ Coordinates of vertex in original Cartesian system ∴ Equation of tangent is y = mx + (a = 2)
m
3 9  2 1 2
is  ,  ∴ y = – 2x + or =y x+
 8 16  −2 2 1
 1  3 1 2
Focus =  0, −  ⇒ actual focus =  ,  ⇒ 2x + y + 1 = 0 or x – 2y + 8 = 0
 16  8 2
 1 
and foot of directrix = (0, a) =  0, 
 16  Sol 8: y2 = 4x (x1, y1) = (4, – 4)
3 5 Equation of tangent is yy1 = 2(x+ x1)
∴ Actual foot =  , 
8 8
∴ –4y = 2x + 8
5 3
equation of directrix is y = & axis is x = . y1
8 8 Equation of normal = (y – y1) = – (x – x1)
2a
Sol 3: y2 = 12x ∴ a =3 ⇒ y + 4 = 2(x – 4)
Focal distance = distance from directrix ∴ 2x – y – 12 = 0
∴ x+a=4 ⇒x=1 6
Sol 9: Let tangents be y = mx +
4m
Sol 4: Length of double ordinate = 8a 3
Now (10 – 8) lies on it – 8 = 10m +
∴ The ends of ordinate are (x, 4a) & (x – 4a). Substituting 2m
20m2 + 16m + 3 = 0
in y2 = 4ax we get x = 4a
⇒ 20m2 + 10m + 6m + 3 = 0
∴ A(4a, 4a) & B(4a, – 4a) are the ends of ordinates
⇒ (10m + 3) (2m + 1) = 0
mOA = 1 & mOB = – 1
3 1
∴ These points subtend 90º at origin. ∴m=– or m = –
10 2
3
∴ Equation is y = – x – 5 or 3x + 10y + 50 = 0
l n 10
Sol 5: Equation of line is y =
− x− .
m m 1
& y = – x – 3 or x + 2y + 6 = 0
but equation of tangent to a parabola with slope s 2
a  a 2a 
is y = sx + ∴ Points of contact are  , 
m m 
2
s
l a n ∴ For line 3x + 10y + 50 point of contact
∴ s=– & = − ⇒ am2 = ln
m s m
 3 3   50 
= × 100,  =  , −10 
Sol 6: x + y + 1 = 0 & y2 = lx  2 × 9 –3 / 10   3 
From above result ln = am2 and for line 2y + x + 6 = 0 point of contact
λ
∴1×1= × (–1)2 ⇒λ=4
4
M a them a ti cs | 10.49

  & let C be (h, 0)


 3 3 
× 4, we have to find |h – at12|
=   = (6, – 6)
 2×1 1
 −  Since BC ⊥ AB
 2
2at1 2at1
∴ × −1
=
at12 −h at12
Sol 10: Tangent to parabola y = 32x is
2

8 ∴ 4a = h – at12
y = mx + & tangent to parabola
m ∴ Projection of BC on x-axis = 4a
y
x2 = 108y is x = + 27m
m Sol 14: Changing the coordinate system to y = 4 &
8 x = x + 3 won’t change the dimensions of parabola
∴ = – 27m2
m ∴ Let parabola be y2 = 8x
8 2
∴ m3 = – ⇒m=– ∴ P = (at2, 2at)
27 3
2
∴ Common tangent is y = – x – 12
2
P(at , 2at)
3 M = (-a, 2at)

∴ 2x + 3 y + 36 = 0 S = (a, 0)

1
Sol 11: For the parabola a =
4
1 1
∴ Point =  , 
 4 2 M = (–a, 2at) S = (a, 0)
1 Since PM = SP always
∴ 2at = ⇒t=1
2
∴ (2a)2 + (2at)2 = (a + at2)2
2
∴ t2 = – t1 – ⇒ t2 = – 1 – 2 = – 3
t1 ⇒ 4 + 4t2 = t4 + 2t2 + 1
∴ Point, when the normal again cuts the parabola ∴ t4 – 2t2 – 3 = 0 ⇒ t2 = 3
 1 1   9 −3  ∴ Side of triangle = a + at2 = 2 + 2 × 3 = 8
is  t22 , t2  =  , 
4 2  4 2 
Sol 15: y2 = 4ax
Sol 12: The circle lies outside the parabola the shortest
P = (at2, 2at) & Q = (at2, –2at)
distance is when normal to parabola is normal to circle,
i. e. , it passes through center of circle Let M = point of trisection = (x, y)
The equation of normal in parametric form is  2at2 + at2 2 × (2at) − 2at 
∴ (x, y) =  , 
 3 3 
y = – tx + 2at + at3 ⇒ y = – tx + 2t + t3  
 2 2at 
⇒ It passes through (0, 12) ∴ (x, y) =  at , 
 3 
∴ t3 + 2t – 12 = 0 2at
∴ x = at2 & y =
(t – 2) (t2 + 2t + 6) = 0 3
2
t = 2 is only positive point  3y 
∴ x=a×  
∴ Point is (4, 4) & shortest distance  2a 
= 42 + (4 − 12)2 − r = 80 – 4 = 4( 5 − 1) 4ax
∴ y2 =
9
Sol 13: Let B = (at12 , 2at1 ) Length of latus rectum of y2 = 4ax is 4a. Length of latus
A = (0, 0) 4ax 4a 1
rectum of y2 = is ;∴ k =
9 9 9
1 0 . 5 0 | Parabola

Sol 16: Focus = mid point of ends of latus rectum = Sol 19: Let P(at2, 2at) be a point on parabola
(1, – 1)
∴ The middle point of segment OP is
6 3
a= =  at2 
4 2 (h, k) =  ,at 
 2 
Since latus rectum is ⊥ to x-axis  
2
∴ Only co-ordinates are shifted. ak 
∴ h=  
2a
 3   1  5 
vertex =  1 ± , −1  ⇒  − , −1  or  , − 1  ∴ Locus of M is y2 = 2ax
 2   2  2 
When vertex lies to left of latus rectum equation
parabola is
( )
Sol 20: Let A at12 , 2at1 & B(at22 ,2at2 ) be two points,
then mid point

 a(t2 + t2 ) 2a(t + t ) 
(x, y) =  1 2 , 1 2 
 2 2 
 
2 2 2x y
∴ (t1 + t2 ) = & t1 + t2 =
a a
The chord passing through A, B is
2
y – 2at1 = (x – at12 )
t1 + t2
∴ Equation of parabola is
Since (h, k) satisfies this
 1
(y + 1)2 = 6  x +  ⇒ (y + 1)2 = 3(2x + 1) ∴ (t1 + t2)k – 2at1t2 = 2h
 2
 5 yk
or (y +1)2 = – 6  x −  ⇒ (y + 1)2 = – 3(x – 5) ∴ – a((t1 + t2)2 – (t12 + t22 ) ) = 2h
2 a

yk  y 2 2x 
⇒ – a −  = 2h
Sol 18: Let AP be the chord with A = (0, 0) & a  a2 a 

(
P (at2, 2at). Let Q = at22 , 2at2 ) ∴
y(y − k)
= 2(x – h)
Since AQ ⊥ AP a
∴ y2 – ky = 2a (x – h)
2at2 2at 4
∴ × −1 ⇒ t2 = –
=
at22 at 2 t
Sol 21: Since point of intersection of
 16a 8a 
∴ Q=  ,−  P= (at12 ,2at1 ) & Q= (at22 ,2at22 ) is [at1t2a(t1 + t2)]
 t2 t 
Let R be the required point = (h, k) Area of PQR
∴ Since OPQR is a rectangle at12 2at1 1
1
∴ Midpoint of OR = Mid-point of P & Q APQR = at22 2at2 1
2
 2 16a  at1 t2 a(t1 + t2 ) 1
at + 2 2at − 8a 
 h k   t , t 
∴  , = at12 2at1 1
2 2  2 2 
  1
  ∴A = at22 2at2 1
4
 2 16   4 2at1 t2 2a(t1 + t2 ) 2
∴ h = at + ; k = 2a  t − 
2  t
 t   R3 → R3 – R1 – R2
2
k  h
∴  +8 = ⇒ k2 = (h – 8a) × 4a
2a a 1 at12 2at1 1
  A= at22 2at2 1
∴ Locus of R is y2 = 4a(x – 8a) 4
2
a(t1 − t2 ) 0 0
M a them a ti cs | 10.51

1 Sol 24: Let the parabolas be y2 = 4ax & x2 = 4ay


∴ A = 2a|(t1 – t2)| × a(t1 – t2)2
4 Let P be their point of intersection

a2 ⇒ x4 = (4a)2 × (4ax)
∴A= |t – t |3
2 1 2 (4a)2
∴ x = 4a & y = = 4a
4a
Sol 22: Consider the parabola with focus S & L as the ∴ P = (4a, 4a)
directrix & P is a point
The equation of tangents for y2 = 4ax at (4a, 4a) is
L
4ay = 2a (x + 4a)
M P
 1
 ∴ Slope =
O 2
R and equation of tangent to x2 = 4ay at (4a, 4a) is
S
4ax = 2a (y + 4a)
∴ Slope = 2  1 
 2− 
∴ Angle between parabolas = tanθ =  2  = 3
SP = PM & tangent bisects PM & SP 1 + 2× 1  4
 
 2
∴ PO ⊥ SM
a
Sol 25: Let tangents be y = mx +
∴ ∠PSM = 90 – θ. ∠PMR = 90º m
It passes through h, k
∴ ∠SMR = 90 – ∠PMS = θ
a
Now in ∆MOR & DSOR ∴ k = mh +
m
MO = SO & OR is common &
⇒ m2h – km + a = 0
∠MOR =∠SOR = 90º
∴ By RHS ∆MOR = DSOR a k
m1m2 = and m1 + m2 =
h h
∴∠ROS = ∠RNO = θ
Angle between tangents = a
∴ Angle subtended by the position of tangent Between
P and R = θ + (90 – θ) = 90º m1 − m2
∴tanα =
1 + m1m2
Sol 23: Equation of tangent to parabola is 2
k  a
a   − 4×
y = mx + h h k 2 − 4ha
m ∴ tan2α =   ⇒ tan2α =
 a
2
(h + a)2
Tangent at A is x = 0  1 + 
 h
 a 
T = − , 0 ∴ Locus is y2 = tan2α(x + a)2 + 4ax
2
 m 
 a
A = (0, 0) & Y =  0,  Sol 26: Let (h, 0) be point on axis of parabola
 m
 −a a  Equation of normal is y = mx – 2am – am3
Coordinations of G are  , 
 m2 m  (h, 0) passes through it
a a
∴ x=– ;y= ∴ am3 + 2am – mh = 0
m 2 m
a m(am2 + 2a – h) = 0
∴ x=– (2a − h)
a
2 For m to be real and distinct – >0
  a
y (h − 2a)
∴ >0
∴ y2 + ax = 0 is the locus of point G a
1 0 . 5 2 | Parabola

The parabola is (y – 1)2 = 4(x – 1) If the points (am2, –2am) lies on circle then
∴ a=1y=Y+1&x=X+1 (am2)2 + (–2am)2 + 2g(am2) + 2t (–2am) + c = 0
h – 2a > 0 & y – 1 = 0 This is a biquadratic equation in m. Hence these are
four values of m, say m1, m2, m3 & m4 such that circle
x > (2a + 1) & y = 1
passes through these points
∴ The points which satisfy are (x, 1) where x > 3
m 1 + m2 + m3 + m4 = 0

Sol 27: The equation of normal to parabola is 0 + m4 = 0; m4 = 0

y = mx – 2am – am3 ∴ (0, 0) lies on the circle

∴ (h, k) satisfies it
Sol 29: Let (h, k) be the middle point
∴ Let f(m) = am3 + (2a – h) m + k
∴ Equation of chord is
Two of the 3 tangents coincide
yk – 2a(x + h) = k2 – 4ah
∴ f(m) has two equal root
2a c
Now let y = Y – &x=X–
∴ f’(m) at m = p is 0 & f(p) = 0 m m
f’(m) = 3am2 + (2a – h)  2a   c 
∴  Y −  k – 2a  X − + h  = k – 4ah
2

(h − 2a)  m  m 
∴ f’(m) is O at m2 =
3a 2a  2ak 2ac 
Y= X+  + 2ah + k 2 − 4ah − /k
Let m = p k  m m 

f(m) = m(am2+ (2a – h)) + k & parabola is Y2 = 8aX


 h − 2a  Line touches the parabola if mc = a
∴ f(p) = p  + 2a − h  +k = 0
 3  2a  2a 2 2ac 
∴ mc =  + 2ah + k − 4ah − /k
(h − 2a) k m m 
∴ k= 2 p
3
2a  2ak 2ac 
h − 2a =  − 2ah + k 2 − /k
9k = 4(h – 2a) × p = 4(h – 2a) ×
2 2 2 2 k  m m 
3a
∴ 4(h – 2a)3 = 27ak2 2a  2ak 2a 
=  − 2ah + k 2 − (k − mh)  / k
k  m m 
∴ For two tangents to be coincide locus of P is 27ay2
= 4(h – 2a)3 2a 2ak 2ak 
=  − 2ah + k 2 − + 2ah  / k = 2a
k  m m 
Sol 28: Let A(am12, – 2am1), B(am22, 2am2) and C(am32, ∴ It is tangent to parabola
–2am3) be 3 points on parabola y2 = 4ax

C Exercise 2
Single Correct Choice Type
(, )
A Sol 1: (C) x = 1 – y
∴ y2 = 4 – 4y
B
∴ y2 + 4y – 4 = 0

Since point of intersection of normals is (α, β) then (y2 – y1)2 = (4)2 + 4 × 4 = 32 ⇒ y 2 − y1 =


4 2

am3 × (2a – α)m + β = 0 |x2– x1| = (1 – y2) – (1 – y1) = |y2 – y1|

m 1 + m2 + m3 = 0 ∴ Length of chord = 2 × |y2 – y1| = 8

Let the equation of circle through A, B, C be


Sol 2: (A) y2 – 12x – 4y + 4 = 0
x + y + 2gx + 2fy + c = 0
2 2
M a them a ti cs | 10.53

(y – 2)2 = 12x 2
Sol 6: (C) Let P be (at1 , 2at1 ) and Q be (at22 ,2at2 )
Let x = X and y = Y + 2 OP ⊥ OQ
∴ Y2 = 12X 2t1 2t2
∴ × −1
=
Focus of parabola is (3, 0) t12 t22
∴ Focus in original coordinate system is (3, 2) 4
∴ t2 = −
∴ Vertex of new parabola is (3, 2)&focus =(3, 4) t1
∴ a=2 The equation of PQ is

Let x = X + 3 & y = Y + 2 2
(y – at1) = (x − at12 )
4
∴ Equation is X2 = 8Y ⇒ (x – 3)2 = 8(y – 2) t1 −
t1
∴ x2 – 6x – 8y + 25 = 0 is the equation of parabola  4 −2at
xR =  t1 −  × + at12 = 4a
 t1  2
Sol 3: (D) x = t2 – 2t + 2, y = t2 + 2t + 2
∴ R = (0, 4a) ∴ OR = 4a
x + y – 4 = 2t2
y – x = 4t
Sol 7: (C) x2 + y 2 – y = c
∴ (x + y – 4, y – x) lies on a parabola
∴ x2 + y2 = y2 + 2cy + c2
⇒ (x + y, y – x) lies on a parabola by rotating axis (x, y) lies
on parabola  c  c
x2 = 2c  y +  ∴ Vertex is  0, − 
 2   2 
 1
Sol 4: (D) y2 = 4a  x −   c
 3 It passes through  c 2, 
1  2
Let y = Y and x = X +
3 1
Sol 8: (B) Equation of tangent is y = mx +
∴ Equation of parabola is Y = 4aX 2
m
7 (–1, 2) lies on it, then
∴ equation of line is Y = 2X –
3 1
a 7 a 2=–m+ ⇒ m2 + 2m – 1 = 0
c= ⇒– = m
m 3 2
m − m1
14 x = 2, so y1 – y2 = 2(m1 – m2) + 2
∴ a=– m1m2
3
2
 1 
tan α2
Sol 5: (A) cota1 = 2cota2∴ tana1= ∴ (y – y2) = (m1 – m2)  2 − m m 
2 2

2  1 2 

let P = (at12 , 2at1 ) & Q = (at22 , 2at2 ) 2


 1 
T = point of intersection = (at1t2, a(t1 + t2)) = (x, y) = [(m1 + m2)2 – 4m1m2]  2 − 
 m1m2 
1 1
Slope of P = and slope of Q = 2
t1 t2  1 
= [(m1 + m2) – 4m1m2]  2 −
2

1 1  m1m2 
⇒ =
t1 2t2
(y1 – y2)2 = (4 + 4) (2 + 1)2 = 8 × 9
∴ t1 = 2t2
∴ |y1 – y2| = 6 2
∴ x = 2at22 y = 3at2
2
 y 
∴   × 2a = x
 3a 
∴ 2y2 = 9ax is locus of T
1 0 . 5 4 | Parabola

Sol 9: (B) ∴4[(t2 – 1)2 + 4t2] = 36


(A) is an ellipse ∴ (t2 + 1)2 = 9 ⇒ t2 = 2
2
(B) x2 = 2 – cost  1
The length of focal chord of a parabola is a  t + 
 2 t  y   t
= 2 –  2cos − 1  = 2 –  − 1 
 2  2 
2
(t + 1) 2
9
= 2 = 2. = 9
1 t 2 2
∴ x2 = (6 – y)
2 ∴ (SQ) = (PQ) – (SP) = 3
∴ It is a parabola
(C) is a line in 1st quadrant Sol 13: (C) Let (h, k) be the point of intersection

t t ∴ Chord of contact is yk = 2(x + h)


(D) =
x , y sin + cos
1 − sint=
2 2 ⇒ 2x – ky + 2h = 0 ⇒ 4x – 2ky + 4h = 0
t t t t 7 5
⇒ y 2 =sin2 + cos2 + 2sin cos =1 + sint ∴k= and h =
2 2 2 2 2 2
=1+1 – x2 (A circle) 5 7
∴ Point of contact is  , 
2 2
Sol 10: (C) Let A (at12 , 2at2 ) & B(at22 , 2at2 ) be points on
parabola. P = (at1 t2, a(t1+ t2)) = (x, y) Sol 14: (D) The curve is y2 = 4x & y > 0
1 1 Normal to parabola is
slope of AP is & slope of BP is
t1 t2 y = mx – 2m – m3
tan(q1 + q2) = 1
It passes through (21, 30)
1 1 m3 + 2m – 21m + 30 = 0
+
t1 t2
∴ =1 m3 – 19m + 30 = 0
1
1−
t1 t2 ⇒ m = 2, m = 3 & m = – 5

t1 + t2 For m = 2, 3; y – coordinate < 0


∴ =1
t1 t2 − 1 ∴ m = – 5 is only possible solution

y
∴ =1 Sol 15: (D) Let P = (h, k)
x −1
∴ x – y – 1 = 0 is locus of P The chord of contact is given by
ky = 2a(x + h)
Sol 11: (C) Parabola is y = 8x. a = 2
2
k
∴x= y −h
The point P is (2 × (2 × 1), 2(2 + 1)) 2a

∴ The points of contact Q, R are It is a tangent to x2 = 4by

(2 × (2)2, 2 × 2 × 2) & (2 × (1)2, 2 × 2 × 1) b


∴–h= ⇒ – kh = 2ab
k
∴ Q & R are (8, 8) & (2, 4) 2a
4 6 1 ∴ xy = – 2ab is locus of P. It is a hyperbola.
1 1
∆= 8 8 1 = |4| = 2
2 2
2 4 1 Sol 16: (D) Let P = (at2, 2at)
∴M = (– a, 2at) ; S = (a, 0)
Sol 12: (A) Let P = (at2, 2at)
For SPM to be equilateral triangle
 a = 2 P = (2t2, 4t); S = (2, 0)
SM = PM (as PM = SP always)
SP = (2t2 − 2)2 + (4t)2 = 6
∴ (2a)2 + (2at)2 = (at2 + a)2
M a them a ti cs | 10.55

(2a)2 = (at2 – a)2 ⇒ at2 = 3a ⇒ t = 3 ∴C = aβ

SP = MP = at2 + a = 4a ∴ Length of tangent = 800 = C = αβ


c c
aβ = ∴ LT =
Sol 17: (A) Let P = (at2, 2at) a a
∴ (at2 – a)2 + (2at)2 = (3)2  … (i)
2 Sol 21: (C) Let P = (at12 ,2at1 ) & Q = (at22 , 2at2 )
 1
The length of focal chord = a  t +  =
5 … (ii) and PQ passes through F(–a, b)
 t

(1) ⇒ (at2 + a)2 = 9  … (iii) 2at1 − b 2at2 − b
∴ =
a(t2 + 1) = 3 ⇒ a(t2 + 1) = 3 2at12 + a at22 + 9

9 9 ∴ (2at1 – b) (t2 + 1) = (2at2 – b) (t12 + 1)


∴ a× = 5 ⇒ t2 =
a2 × t2 5a
∴ 2at1 t22 ++ 2at1 – bt22 = 2at2 t12 + 2at2 – bt12
2
9  ∴ 2at(t2 – t1) = 2a(t2 – t1) + b(t2 – t1) (t1+ t2)
From (iii)  + a  = 9
5 
9 6 ∴ 2at1t2 = 2a + b(t1 + t2)
⇒a=3– = (Since a cannot be negative)
5 5 Let T = (x, y) = (at1t2, a(t1 + t2))
24 by
∴ Length of latus rectum = 4a = ∴ 2x = 2a + ⇒ 2a(x – a) = by
5 a

Sol 18: (D) Ordinate = abscissa Sol 22: (C) A


∴ at = 2at ⇒ t = 2
2

S
2 B
t2 (other end of normal) = – t – =–3 (a, 0)
t
P = (4a, 4a); Q = (9a, – 6a) and S = (a, 0) C
4 3
∴ Slope of SP = and slope of SQ = – Area of ∆ABC = area of ∆ABS + area ∆BCS
3 4
1 1
∴ Angle subtend at focus = 90º = × BS × h1 + BS × h2
2 2
1
Sol 19: (D) Let P be (at2, 2at) = ax (h1 + h2)
2
2x
∴ Equation of PA is y = h1 + h2 = diff. in y-coordinates of A & C
t
1
 2a  ∴ a|y – y1| = A
M = (–a, 2at); D =  −a, −  2 2
 t 
2A
2at ∴ |y2 – y1| =
Slope of SM is =–t a
−2a
−2a 1
and slope of SD is =
t × ( −2a) t Previous Years’ Questions
∴ SD ⊥ SM ∴ angle between = 90º Sol 1: (B) If y = mx + c is normal to the parabola
y2 = 4ax,
Sol 20: (D) Let equation of circle be Then c = –2am – am3.
x + y + 2gx + 2fy + c = 0
2 2
From given condition, y2 = 12x
It passes through (α, 0) and (β, 0) ⇒ y2 = 4 . 3 . x ⇒ a = 3 and x + y = k
∴ a + 2yα + c = 0 and b + 2gβ + c = 0
2 2
⇒ y = (–1)x + k ⇒ m = –1
∴ C(β – α) + aβ(α – β) = 0
1 0 . 5 6 | Parabola

And c = k 2
Sol 5: (D) Tangent to the curve y2 = 8x is y = mx + .
∴ c = k = –2(3) (–1) – 3(–1)3=9 m
Substituting this in x y = - 1
 2
Sol 2: (C) Given,y2 = kx – 8 ⇒ x.  mx +  = –1
 m 
 8 2
⇒ y2 = k  x −  ⇒ mx2 + x+1=0
 k  m

Shifting the origin Since, it has equal roots.

Y2 = kX, where Y = y, X = x – 8/k. ∴D=0


k 4
Directrix of standard parabola is X = − ⇒ – 4m = 0 ⇒ m3 = 1 ⇒ m = 1
4 m2
8 k Hence, equation of common tangent is y = x + 2.
Directrix of original parabola is x = −
k 4
⇒ (x – y) = 8(x + y – 2)
8 k
Now, x = 1 also coincides with x = −
k 4
On solving, we get k = 4 Sol 6: (C) By section formula,
y
(x, y)Q
Sol 3: (D) Given, y2 + 4y + 4x + 2 = 0 3
⇒ (y + 2)2 + 4x – 2 = 0 P(h, k)
1 x
 1 O
⇒ (y + 2)2 = –4  x −  (0,0)
 2
1
2
y =4x
Replace y + 2 = Y, x – =X
2
x+0 y+0
We have,Y2 = –4X h= ,k=
4 4
This is a parabola with directrix at X = 1 ∴x = 4h, y = 4k
1 3 Substituting in y2 = 4x
⇒x– =1⇒x=
2 2
(4k)2 = 4(4h) ⇒ k2 = h

Sol 4: (C) Let P(h, k) be the midpoint of the line segment Or y2 = x is required locus.
joining the focus (a, 0) and a general point Q(x, y) on 2 
the parabola. Then (C) Centroid of DPQR =  ,0 
3 
x+a y Equation of circle passing through P,Q,R is
h= ,k=
2 2
(x–1)(x–1) + (y–2)(y+2) + λ(x–1) = 0
⇒ x = 2h – a, y = 2k
⇒ 1 – 4 – λ = 0 ⇒ λ = –3
Substitute these values of x & y in y2 = 4ax, we get
∴ required equation of circle is
4k2 = 4a(2h – a)
x2 + y2 – 5x = 0
⇒ 4k2 = 8ah – 4a2
5  5
⇒ k2 = 2ah – a2 ∴ Centre  ,0  and radius .
2  2
So, locus of P(h, k) is y2 = 2ax – a2
Sol 7: (B, C) The equation x2 + 4y2 = 4 represents an
 a
⇒ y2 = 2a  x −  ellipse with 2 and 1 as semi-major and semi-minor axes
 2 
3
a a and eccentricity .
Its directrix is x – =– 2
2 2  1
Thus, the ends of latus rectum are  3, 
⇒ x = 0 ⇒ y-axis  2
M a them a ti cs | 10.57

 1  1  1 and y(-2) = 4(x+1)/2


and  3, −  ,  − 3,  and  − 3, −  .
 2  2  2 which gievs -2y = 2(x + 1)  … (ii)
According to the question, we consider only The points of intersection of these tangents can be
 1  1
obtained by solving these two equatiosn simultaneously.
P  − 3, −  and Q  3, −  , y, y 2 < 0
 2  2 Therefore, -2(x +1) = 2(x + 1)
which gives 0 = 4(x +1)
y
this yields x = -1 and y = 0.

(0,1) Hence, the required point is (-1, 0).

Sol 10: Let the point be Q(x, x2) on x2 = y whose


x’ distance from (0, c) is minimum.
(-2,0) A (2,0)
Now, PQ2 = x2 + (x2 – c)2
P(-3, -1/2) P(3, -1/2)
Let f(x) = x2 + (x2 – c)2 …(i)
A’ (0,-1) f ′(x) = 2x + 2(x2 – c).2x
y’
= 2x(1 + 2x2 – 2c)
Now,PQ = 2 3  1
= 4x  x2 − c + 
Thus, the coordinates of the vertex of the parabolas are  2
y
 −1 + 3   −1 − 3 
A  0,  and A′  0,  and corresponding
 2   2 
   
x2=y
equations are (0,c)P
3 1− 3
(x – 0)2 = –4. y +  2

2  2  Q(x,x )
x’ x
3 1− 3 O
and (x – 0)2 = 4. y − 
2  2 
y’
i.e.,x + 2 3 y = 3 –
2
3 and x – 2 3 y = 3 +
2
3
 1  1
= 4x  x − c −  x + c − 
 2   2 
 
Sol 8: (A,B,D) Normal to y2 = 4x, is
1
When c >
y = mx – 2m – m3 which passes through (9, 6) 2
⇒ 6 = 9m – 2m – m3 For maxima, put f ′(x) = 0
⇒ m3 – 7m + 6 = 0  1
⇒ 4x  x2 − c + 
 2
⇒ m = 1, 2, –3
∴ Equation of normals are, 1
⇒ x = 0, x = ± c−
2
y – x + 3 = 0, y + 3x – 33 = 0 and y – 2x + 12 = 0
 1
Now, f ′′(x) = 4  x2 − c +  + 4x[2x]
Sol 9: The coordinates of the extremities of the latus  2
rectum of y2 = 4ax are (1, 2) and (1, -2).
1
At x = ± c −
The equations of tangents at these points are gievn by 2
y.2 = 4(x+1)/2
f ′′ (x) > 0.
This gives 2y = 2(x + 1) ... (i)
∴f(x) is minimum
1 0 . 5 8 | Parabola

 1  1 
Hence, minimum value of f(x) = |PQ| ⇒ h = 2  m2 + & k= 2  − m 
2 
 m  m 
2

2  2   2 
1 1 1 1 
=  c −  +  c −  − c ⇒ h = 2 m −  + 2  & k = 2 − m

 2    2    m   m 
   
2 Eliminating m, we get 2h = k2 + 8
1  1  1 1
= c − + c − − c = c− , ≤ c ≤ 5.
2  2  4 2 Or y2 = 2(x – 4) is required equation of locus.

Sol 13: Equation of any tangent to the parabola,


Sol 11: Equation of normal to x2 = 4y is x = my – 2m – m3
a
and passing through (1, 2). y2 = 4ax is y = mx + .
m
∴1 = 2m – 2m – m3 a2
This line will touch the circle x2 + y2 =
⇒ m3 = –1 or m = –1 2
y
Thus, the required equation of normal is, x = –y + 2 + E
1 or x + y = 3.
x=-a/2 B
x=a
Sol 12: Let the equation of chord OP be y = mx and
1 x’
then, equation of chord will be y = – x and P is A(-a,0) K O L
m
 4 4
point of intersection of y = mx and y2 = 4x is  ,  C
m m
2

1 y’ D
and Q is point intersection of y = − and y2 = 4x
mn
is (4m2, –4m) 2
a a2
If   = (m2 + 1) [Tangency condition]
Now, equation of PQ is m 2
4
+ 4m 1 1
y + 4m = m (x − 4m2 ) ⇒
2
= (m2 + 1) ⇒ 2 = m4 + m2
2
4 2 m
− 4m
m2 ⇒ m4 + m 2 – 2 = 0
y ⇒ (m2 – 1)(m2 + 2) = 0
P
⇒ m2 – 1 = 0, m2 = –2
x ⇒ m = ±1 (m2 = –2 is not possible)
O L
Q Therefore, two common tangents are
2
y = 4x y = x + a and y = –x – a
y’
m These two intersect at A(–a, 0)
⇒ y + 4m = (x – 4m2)
2
1−m The chord of contact of A(–a, 0) for the circle
⇒ (1– m )y + 4m – 4m3 = mx – 4m3
2
x2 + y2 = a2/2 is
⇒ mx – (1 – m2)y – 4m = 0
(–a)x + 0.y = a2/2 ⇒ x = –a/2
This line meets x-axis, where y = 0
i.e., x = 4 ⇒ OL = 4 which is constant as independent Sol 14: Let P(α, β) be any point on the locus. Equation of
of m. pair of tangents from P(α, β) to the parabola y2 = 4ax is

Again, let (h, k) be the midpoint of PQ, then [by – 2a(x + α)]2 = (b2 – 4aα)(y2 – 4ax)
4 4 [ T2 = S.S1]
4m2 + − 4m
h= m2 and k = m ⇒ b2y2+4a2(x2+a2+2x.a)–4aby(x + α)
2 2
M a them a ti cs | 10.59

= b2y2 – 4b2ax – 4aay2 + 16a2ax


⇒ b2y2 + 4a2x2 + 4a2a2 4
+ (h + 1) = 0
= b y – 4b ax – 4aay + 16a ax
2 2 2 2 2 (1 − k)2

– 4abxy – 4abay …(i) or(k – 1)2(h + 1) + 4 = 0


Now, coefficient of x2 = 4a2 ∴ Locus of a point is (x + 1)(y – 1)2 + 4 = 0
Coefficient of xy = –4aβ
Sol 16: (A) Let any point P on the Parabola y2 = 4x be
Coefficient of y2 = 4aα
(t2, 2t)
Again, angle between the two of Eq.(i) is given as 45º
Eq. of tangent is ty= x + t2
2 h2 − ab Now, this equation is tangent to x2 = −32 y , then
∴tan45º =
a+b  x + t2 
x2 = −32  
2
2 h − ab  t 
⇒ 1= ⇒ a + b = 2 h2 − ab  
a+b
⇒ x2 + 32 x + 32 t2 =
0
⇒ (a + b)2 = 4(h2 – ab) ⇒D=
0
⇒ (4a +4aα) = 4[4a b –(4a )(4aα)]
2 2 2 2 2
⇒ ( 32 ) − 4 × 32 t2 × t =
2
0 ⇒ t3 =
8
⇒ 16a2(a+α)2 = 4.4a2[b2–4aa] ⇒ t=
2
⇒ a2 + 6aα + a2 – b2 = 0 1 1
Slope of tangent= =
⇒ (α + 3a) – b = 8a
2 2 2 t 2
Thus, the required equation of the locus (x + 3a)2 – y2
=8a2 which is hyperbola. JEE Advanced/Boards
Sol 15: Given equation can be written as
Exercise 1
(y – 1)2 = 4(x – 1)
Whose parametric coordinate are Sol 1: Slope of tangent || to L =2
x – 1 = t2 and y – 1 = 2t 1
Slope of tangent ⊥ to L = –
2
i.e., P(1 + t2, 1 + 2t)
P : y2 = 16x ⇒ a = 4
∴ Equation of tangent at P is,
∴ Equation of tangent || to L is
t(y – 1) = x – 1 + t2, which meets the directrix x = 0 at Q.
4
1 y = 2x + ∴ y = 2(x + 1)
⇒y=1+t– 2
t 1 4
Equation of tangent ⊥ to L is y = – x+
 1 2 1
or Q  0, 1 + t −  ∴ 2y + x + 16 = 0 −
 t 2
1  a 2a 
Let R(h, k) which divides QP externally in the ratio : 1 Point of contact =  , 
m m 
2
or Q is mid point of RP, 2

 4 2× 4 
h + t2 + 1 For || line P1 =  ,  = (1, 4)
⇒ 0= or t2 = –(h+1)  ...(i)  22 2 
2
1 k + 2t + 1  
and 1 + t – =
t 2  
 4 2× 4 
2 For ⊥ line P1 =  ,  = (16, – 16)
or t =  …(ii) 1 −1 
2
1−k
2 2 
∴ From Eqs. (i) and (ii),   
1 0 . 6 0 | Parabola

3 ⇒ m2y + mx + b = 0 & m2x – my + a = 0


Sol 2 Let equation be y = mx +
m both have a common root
It passes through (2, 5)
∴ (c1a2 – a2c1)2 = (b1c2 – b2c1) (a1 b2 – a2b1)
3
∴ 5 = 2m + ⇒ (ay – bx)2 = (ax + by) (–y2 – x2)
m
∴ 2m2 – 5m + 3 = 0 ∴ (x2 + y2) (ax + by) + (ay – bx)2 = 0
∴ 2m2 – 2m – 3m + 3 = 0 is the locus of (h, k)
3
∴ m = 1 or m =
2 Sol 5: For a point (at12 , 2at1 ) the equation of normal is
∴ Equation of tangents are
y = – tx + 2at + at3
3x
y = x + 3 and y = + 2 ⇒ 3x – 2y + 4 = 0 ∴ Interception axis = (2a + at2, 0)
2
∴ M =(a + at, at) = (x, y)
Sol 3: Let two points be (at12 , 2at1 ) & (at22 ,
2at1 ) point a + y2
∴ x=
1 a
of intersection is (a(t1t2), a(t1 + t2)) slope of T1 = &
t1 ∴ y2 = a(x – a)
1
that of T2 = a
t2 Vertex is (a, 0) and latus rectum = ×4=a
4
1 1

t1 t2 Sol 6: The equation of normals is y = mx – 2am – am3
∴ tanα =
1 P(h, k) satisfies it
1+
t1 t2
∴ am3 + m(2a – h) + k = 0
t2 − t1
∴ tanα = m1 + m 2 + m 3 = 0
1 + t1 t2
m 1m 2 = – 1
2
(t1 + t2 ) − 4t1 t2 k k
∴ tan2α= m 1m 2m 3 = –⇒ m3 =
(1 + t1 t2 )2 a a
2a − h
x y m1m2 + m3(m1 + m2) =
= t1t2 & = (t1 + t2) a
a a
2 2a − h 2a − h
y 4x ⇒ m1m2 – m32 = ⇒ −(1 + m32 ) =
  − a a
a a
∴ tan2α = k2 h − 2a
2 ⇒1+ = ⇒ k2 = a(h – 3a) or y2 = a(x – 3a)
 x a2 a
1 + 
 a

y 2 − 4ax Sol 7: Let P = (at12 , 2at1 )


tan α =
2

(x + a)2 R = (at20 ,2at0 )


∴ (x + a)2 sin2α = cos2α (y2 – 4ax) Q = (at20 , 2at0 )

a 2a(t1 − t0 ) 2a(t2 − t0 )
Sol 4: Let y = mx + be tangent to ∴ × =–1
m a(t12 − t02 ) a(t22 − t20 )
y2 = 4ax the tangent to x2 = 4by is
4
(t + t0) = – 1
y = m1x – bm12 where m1 is slope of tangent m1m = – 1 (t1 + t0 ) 2
1 y − 2at1
⇒ m1 = – 2
m The equation of chord PQ is =
1 a x − at12 t1 + t2
∴y = – x–
m m2
M a them a ti cs | 10.61

4 2 2
t2 = – t0 – ∴ PQ =  2× | 2 |   44 
t1 − t0  18 −  +  12 + 
 9   3 
2(x − at12 )
∴ y – 2at1 = 1 80
4 = (80)2 + 9(80)2 = 10
t1 − t0 − 9 9
t1 + t0
∴ 9PQ = 80 10
∴ (y – 2at1) (t12 − t02 − 4) = 2(t1 + t0) (x − at12 )
∴ t12 y – t20 y – 4y – 2at13 +2at1 t2 + 8at1 Sol 10: O = (0, 0), L = (2a, a)
0
= 2(t1x – at13
+ t0x – at2 t0) Let H = (h, 0)
1
∴ t12 y − t02 y − 4y + 2at1 t20 + 8at1 a 1
∴ × = – 1 ⇒ a = – 4a + h
= 2t1x + 2t0x – 2at12 t0 2a − h 2
∴ h = 5a
t12 y + 2at1t0(t1 + t0) + 8at1 – 2t1x
∴ H = (5a, 0)
= t20 y + 4y + 2 t0x
∴Length of double ordinate= 2 4a × 5a = 4a 5
∴ t1(t1y + 2at0t1 + 2at20 + 8a – 2a)
= t20 y + 4y + 2t0x Sol 11: y2 = 4ax
∴ It passes through intersection if 2t0x + 4y + t20 y =0 Equation of normal at (at2, 2at) is
t1y – 2x + 2at0t1 + 2at20 + 8a = 0 y = – tx + 2at + at3
∴ The point of intersection which is the required fixed It meets y = 0 at G
point (a(t20 + 4) , – 2at0)
∴ G = (2a + at2, 0)

Sol 8: Equation of tangent is y = mx +


a QG = 4ax = 4a(2a + at2 )
m
Equation of ⊥ line through origin is PG = (at2 − (2a + at2 ))2 + (2at)2
1 ∴ QG2 – PG2 = 4a(2a + at)2 – (4a2 + 4a2t2)
y=– x
m = 8a2 – 4a2 = 4a2
∴y = 4a (–my)
2

∴y = – 4am is the y-coordinate of Q. & x = 4am2 Sol 12: The equation of tangent to y2 = 4ax of slope

( ) a
2
∴ OQ = (4am)2 + 4am2 = 4am 1 + m2 m is y = mx +
m
a ∴ xm2 – my + a = 0 …(i)
OP = (⊥ distance of (0, 0) from line
m 1 + m2 The equation of normal to x2 = 4by of slope m
a b
y = mx + ) is y = mx + 2b +
m m2
∴ OP × OQ = 4a2 = constant ∴ m3x + (2b – y)m2 + b = 0 …(ii)
Let the point (x, y) satisfy both the equation.
Sol 9: P = (2(3)2, 4(3))
a y
∴ Parameter (t1) = 3 ∴ From (i) m1m2 = & m1+m2 =
x x
2 2 11 These two tangents are normal to x2 = 4by
t2 = – t1 – =–3– =–
t1 3 3
∴ m1,m2 satisfy (ii)
 2 × 121 −44  b b
∴ Q = ,  ∴m1,m2,m3 = – ⇒ m3 = –
 9 3  x a
y − 2b y y 2b
m 1 + m 2+ m 3 = ⇒ + m3 = −
x x x x
1 0 . 6 2 | Parabola

2b And let t4 be the other end of normal for Q


⇒ m3 = – & m1m2 + m3(m1 + m2) = 0
x 2
∴ t4 = – 2 – =–3
a b 2b a 2
∴ m3 = – ⇒– =− =

y a x y ∴ t3 = t4
a2
⇒ x = 2ay = ∴ The both the normals intersect on parabola it self
b
Now m1, m2 are distinct & real
Sol 16: Let O be vertex & P be (at2, 2at)
∴ D of equation (i) > 0
2
∴ Slope of OP is
a4 t
∴ y2 – 4ax > 0 ⇒ > 8a2 ⇒ a2 > 8b+2
b2 Let Q be (at22 , 2at2 )
2 2 4
Sol 13: Let point be P(at2, 2at) OQ ⊥ OP ⇒ × = −1 ⇒ t2 = –
t t2 t
The directrix is x = – a
 16a 8a 
1 Q = ,− 
The equation of tangent is y = x + at  t2 t 
x
∴ The point where is meets the directrix is Now let R be to other end of rectangle (x, y) since it is a
  1  rectangle & OP ⊥ OQ
Q =  −a,a  t −  
  t  ∴ Midpoint of R & O = midpoint of P & Q
 2 16a  4
(a(t2 − 1)) a 1  at + 2 2a  t −  
M = (x, y) = , (3t − )] midpoint x y  t ,  t 
2 2 t ∴  ,  =
2x + 1 2 2  2 2 
 
t2 = 2yt = a (3t2 – 1)  
a
  2x 
2  4
2   4
 2x + a  
∴ 4y × 
2
 = a 3× 
2
+ 1 − 1 ∴ x = a   t −  + 8  ; y = 2a  t − 
 a    a    t   t
 
( 2x + a)
∴ 4y2 = 22(3x + a)2  y2  (x − 8a) × 4a2
a ∴x = a  + 8  ; ∴ y2 =
 4a2  a
∴ y2(2x + a) = a(3x + a)2  
∴ y2 = 4a(x – 8a) is the locus of other end
Sol 14: Parabola y2 = 12x
a=3 Sol 17: The equation of normals is y = mx – 2am – am3

∴ P = (3, 6) ∴ t = 1 ⇒ a = 1 & (15, 12) lies on it


2 ∴ 12 = 15m – 2m – m3
∴ t2 to the other point = – t – =–3
t
∴ Q = (a(–3)2, 2a(–3)) = (27, – 18) ∴ m3 – 13m + 12 = 0

∴ Equation of circle with PQ as diameter is (m – 1) (m2 + m – 12) = 0

(x – 3) (x – 27) + (y – 6) (y + 18) = 0 (m – 1) (m + 4) (m – 3) = 0

∴ x2 + y2 – 30x + 12y – 27 = 0 ∴ 1, – 4, 3 are three possible normals


∴ Equation is (y – 12) = (x – 15) ⇒ y = x – 3
Sol 15: Let upper end of latus rectum be P & and (y – 12) = 3(x – 15) ⇒ y = 3x – 33
Q = (4a, 4a) and (y – 12) = – 4 (x – 15) ⇒ 4x + y – 72 = 0
∴ P = (a, 2a) ∴ t = 1
& for Q (4a, 4a) ∴ t2 = 2 Sol 18:
3
∴ let t3 be the other end of normal for P Centre of circle = (0, 0) diameter = 2r = × 4a
4
2 3a
∴ t3 = – 1 – =–3 ∴ r=
1 2
M a them a ti cs | 10.63

 3a 
2 Sol 20: The focus of x2 = 4y is (0, 1)
∴ Equation of circle is x + y =   2 2

2 ∴ Tangent to parabola at (6, 9) is 6x = 2(y + 9)


⇒ 4x + 16ax – 9a = 0
2 2
∴ 3x – y – 9 = 0
⇒ 4x + 18ax – 2ax – 9a = 0
2 2
∴ Equation of normal is
⇒ (2x – a) (2x + 9a) = 0 1
(y – 9) = – (x – 6) ⇒ x + 3y – 33 = 0
9a 3
But x = – is not possible as y becomes imaginary
2 Centre lies on it ∴ g + 3f + 33 = 0
a (–g – 6)2 + (–t – 9)2 = (–g – 0)2 + (– f – 1)2
∴ x= is the abscissa of the two points of intersection
2
∴ 12g + 36 + 18 + 81 = 2f + 1
∴ The common chord bisects the line joining Vertex.
∴ 12g + 16f + 116 = 0

Sol 19: Let P(at12 , 2at1 ) & Q (at22 , 2at2 ) ⇒ 3g + 4f + 29 = 0

Let R be (aT2, 2aT) on the parabola y2 = 4ax Solving we get g = 9 & f = – 14

2 2 ∴ r= ( −g)2 + ( −f − 1)2 = g2 + 132 = 250


T = – t1 – = – t2 – ⇒ t1t2 = 2
t1 t2
Tangents at P and Q intersect at T (at1t2, a(t1 + t2)) ∴ C = g2 + f2 – r2 = 27

T(2a, a(t1 + t2)) ∴ Equation of circle is x2 + y2 + 18x – 28y + 27 = 0

Coordinates of R the point of intersection, are


Sol 21: Let P(at12 , 2at1 )
(2a + a ( + + t t ) , −at t ( t
t12 t22 1 2 1 2 1 + t2 ) ) since Q is the other end of the normal from P
≡ ( 4a + a ( t + t ) ,2a ( t + t ) )
2
1
2
2 1 2 2
t2 =– t1 –
⇒ ∠TPR = ⇒ ∠TQR = 90º ⇒ ∠TPR + ∠TQR = 180º t1
  2 
⇒ Quadrilateral TPQR is a cyclic quadrilateral & the  2  2
Q = a  t1 +  , −2a  t1 +  
centre of circle lies on the midpoint of TR.   t1  
 t1  
 
R
  2   −2  
T =  at1 ×  −t1 −  ,a   
 t1   t1  
 
∴ The x-coordinate of midpoint of T & P is
P
T Q at12 − 2a + at12
x=– =–a
2
Let midpoint be M (h, k)
∴ TP is bisected by directrix
∴ 2h = 2a + 4a + a(t12 + t22)
2h − 6a Sol 22: Let P = (at12 , 2at1 )
= (t1 + t22) – 2t1t2
a
The other end of normal chord Q = (at22 , 2at2 )
2h − 2a
= (t1 + t2)2( t1t2 = 2) 2
a ∴ t2 = – t1 –
t1
2k = a(t1 + t2) – 2a(t1 + t2)
  2 
2  2
2k
= (t1 – t2) ∴ Q =  a  t1 +  , −2a  t1 +  
  t1  
 t1  
a  
2
 2h − 2a   −2k  Let M(x, y) be midpoint of P & Q
∴   =  
 9   a  −2a 2a
∴ 2R2 = a(h – a) ∴ y= ∴ t1 = –
t1 y

∴ Locus of M(h, k) is 2y2 = a(x – a)
1 0 . 6 4 | Parabola

2
 −2a y 
2 ∴ Midpoint of T and T’ is
 −2a 
⇒ 2x = a ×   + a  y − a  a 
 y    A =  −a, − am 
 m 
4a3 4a3 y2
∴ 2x = + + + 4a ∴ Point of intersection of tangents is the midpoint of
y2 y2 a T and T’
4a3 y2
∴ x – 2a = +
y2 2a Sol 25: Let P = (at12 , 2at1 )
Q = (at22 , 2at2 )
Sol 23: Let A = (at12 , 2at1 ) a = 2
2a(t2 − t1 ) 2
Slope of PQ = = = 1
B= (at22 , 2at2 ) a(t2 − t1 ) t1 + t2
2 2

1 ∴ t1 + t2 =2
Tangent at A is y = x + at1
t1
Point of intersection of points with parameter t1 & t2 is
1
Tangent at B is y = x + at2 (2a + a(t12 + t22 + t12 ) , –at1t2(t1 + t2))
t2
∴ x = 2a + a((t1 + t2)2 – t1t2)
The y-coordinate of point of intersection of A & tangent
at vertex (x = 0) is y = – a(t1 + t2) × t1t2
y1 = (2t1); y2 = 2t2 ∴ x × (t1 + t2) – y = 2a(t1 + t2) + a(t1 + t2)2 = 0
∴PQ = 2(t1 – t2) = 4 2x – y = 2a × 2 + a × 8
∴ |t1 – t2| = 2 Putting a = 1
& point of intersect is (x, y) = (2t1t2, 2(t1 + t2) ∴ 2x – y =12 is the locus of point of intersection of
normals.
(t1 + t2) – 4t1t2 = (t1 – t2)
2 2

y2
∴ – 2x = 4 Sol 26: Let A(am12 ,2am1 ), B(am22 , −2am2 ) and
4
∴ y2 – 8x = 16
C(am32 , −2am3 ) be points on parabola y2 = 4ax
∴ y = 8(x +2) is the locus of point of intersection
2
Let point of intersection of normals be (h, k) then

Sol 24: Let m be the slope of 1st line am3 + (2a – h)m + k = 0

1 m 1 + m2 + m3 = 0  … (i)
∴– is the slope of the other line equation of tangents (2a − h)
m m 1 m 2 + m 2m 3 + m 3 m 1 =  … (ii)
a a
is y = mx +
m k
m 1m 2m 3 = – … (iii)
1 a
and y = – x – am
m Let equation of circle through ABC be
 −a 
⇒ T ( −a, 2am) & T'  −a,  x2 + y2 + 2gx + 2fy + c = 0
 m
The point (am2, –2am) lies on it
∴ Point of intersection of tangents
a2m4 + (4a2 + 2ag)m2 – 4afm + c = 0  … (iv)
 a 
M =  −a, − am 
 m  ∴ m1 + m2 + m3 + m4 = 0
One line passing through (a, 0) with slope m is ∴ From (i)
(y) = m(x – a) m4 = 0 – 0 = 0
1 ∴ (0, 0) two lies on circle
And ⊥ line is (y) = – (x – a)
m
∴ c=0
∴ T = (–a, –2am)
From (4) a2m4 + (4a2 + 2ag)m2 – 4afm = 0
 2a 
T’ =  −a,  ⇒ am3 + (4a + 2g)m – 4f = 0  … (v)
 m
M a them a ti cs | 10.65

Now, equation A & 5 are identicals ⇒ x (x2 + y2) + ay2 = 0


4a + 2g 4t
∴ 1= = −
2a − b k Sol 29: Let P be (at12 , 2at1 ) & Q be (at22 , 2at2 )
∴ 2g = – (2a + h) ∴ OP ⊥ OQ
k 2 2 4
2t = – ⇒ × = −1 ⇒ t2 = –
2 t1 t2 t1
k
∴ The equation of circle is x2 + y2 – (2a + h)x – y=0 ∴ Equation of PQ is
2
or 2(x2 + y2) – 2(h + 2a)x – ky = 0 y − 2at1 2(t2 − t1 )
=
2 x − at12 (t22 − t12 )
Sol 27: Let P(at12 ,2at1 ) and Q be (at2 , 2at2 )
y − 2at1 2
Now chord PQ passes through A(a, 3a) ∴ =
x − at12  4
2at2 − 3a 2at1 − 3a  t1 − 
∴ =  t1 
at22 − a at12 − a
∴ (y – 2at1) (t12 − 4) = 2t1 (x – at12 )
2t2 − 3 2t1 − 3
∴ = ∴ t12 y – 4y – 2at13 + 8at1 = 2t1x – 2at13
t22 −1 t12 −1
∴ t1(t1y + 8a – 2x) – 4y = 0
2t12 t2 + 3t12 – 2t + 3 = 2t1 t22 – 3t22 – 2t + 3 ∴ The line passes through point of intersection of
1

∴ 2t1t2(t2 – t1) – 3(t2 – t1) (t2 + t1) + 2(t2 – t1) = 0 t1y + 8a – 2x = 0 & y = 0
∴ 2t1t2 – 3(t1 + t2) + 2 = 0 The point is (4a, 0)
Point of intersection of tangent at t1t2 is Let M = (x, y) be midpoint
(at1t2, a(t1 + t2)) a(t12 + t22 ) 2a(t1 + t2 )
∴ (x, y) = ,
∴ x = at1t2 and y = a(t1+ t2) 2 2
x 3y 2x = a(t12 + t22 ) = a(t1 + t2) – 2t1t2)
2
∴ 2 − +2 =0
a a t1 t2 = – 4
∴ 2x – 3y + 2a = 0 is the locus of point of intersection
∴ 2x = a(t1+ t2)2 + 8
of tangent
y = a(t1 + t2)
Sol 28: Equation of tangent is  y2 
∴ 2x = a  + 8
a a a 2 
y = mx + ⇒ mx – y +  
m m y = 2ax – 8a = 2a(x – 4a) is the locus of M.
2 2

∴ Midpoint of P & Q is foot of ⊥ of (0, 0) on the tangent


let M be (x, y)
a
  Exercise 2
x−0 y −0 m
∴ = = −   Single Correct Choice Type
m −1 2
m +1
a a Sol 1: (C) Let equation of tangent be
∴ x=– y=
2 2
m +1 m(m + 1) a
y = mx + directrix is x = – a & latus rectum is
a m
∴ m2 = – 1 –
x x = a; S = (a, 0)
a2 a2
∴ y =2
2
y =
2
 1    1 
 a  a (x + a) a2 U =  −a,a  − m   ; V =  a,a  m +  
−  1 +  1 − 1 −  −
x  x x x2  m    m 

∴ y2(x + a) + x3 = 0  1  1
SU = 4a2 + a2  m2 − 2 + = m+
2  m
 m 
1 0 . 6 6 | Parabola

1 ∴ Y2 = 6X
SV = m +
m 3
The locus of ⊥ tangents is X = – a = –
∴ It is always an is isosceles triangle 2
3
Angle between SU & SV is not always 90º as slope of SV ∴ x+1=– ⇒ 2x + 5 = 0
2
= ∞ and slope of SU depends on m
∴ It is just an isosceles triangle Sol 6: (C) The tangent at (x1, y1) to y2 = 4ax

2 2 ⇒ yy1 = 2a(x + x1)


Sol 2: (B) Let points be (at1 , 2at1 ) & (at2 , 2at2 )
⇒ 2ax – yy1 + 2ax1 = 0  …(i)
∴ (a(t12 − 1))2 + 4a2 t12 =
16
Let (h, k) be midpoint
& (a(t22 − 1))2 + 4at22 = 81
∴ Locus of chord to y2 = 4a(x + b) with M(h, k) is
∴ (at12 2
+ a) = 16 yk – 2a(x + h) – 4ab = k2 – 4ah – 4ab
∴ at12 + a=±4 … (i) ⇒ ky – 2ax = k2 – 2ah
And at22 + a = ± 9 … (ii) or 2ax – ky + k2 – 2ah = 0
The point of intersection is (at1t2, a(t1+ t2)) ∴ 2ax – yy1 + 2ax1 = 0
∴ PS = a (t1t2 – 1) + a (t1 + t2)
2 2 2 2 2
And 2ax – ky + k2 – 2ah = 0 represent the same line
2
=a (t12 t22 + t12 t22 + 1) = 1 × 2 = 36 ∴ k = y1 & k2 – 2ah = 2ax1
2ah = y12 – 2ax1
Sol 3: (A) Let (h, k) be a point on line (2x + y = 4)
y12 – 4ax = 0 ⇒ y12 – 2ax = 2ax
∴ Chord of contact is ky = – 2(x + h) 1 1 1

⇒ 2ah = 2ax1 ⇒ h =x1


∴ 2x + ky = – 2h & (h,k) also satisfy
2h + k = 4 & 2h + ky = – 2x Sol 7: (C) For closest points normal to parabola should
∴ It passes through y = 1 & x = – 2 be normal to circle equation of normal at (at2, 2at) is

∴ (–2, 1) y = – tx + 2at + at3


a=1
Sol 4: (D) Parabola is y2 = ax
∴ y = – tx + 2t + t3 is equation of normal it should pass
a through (0, 2)
∴ a’ =
4 ∴12 = 2t + t3 ⇒ t3 + 2t – 12 = 0
a
Tangent is y = mx + (t – 2) (t2 + 2t + 6) = 0
4m
a
or mx – y + =0 t = 2 is only possible solution
4m
∴ The point on parabola closest to the circle is (4, 4)
∴ Foot of ⊥ from (0, 0) is (x, y)

x−0 y −0  a  Sol 8: (C) P = (at2, 2at)


∴ = =–  
m −1 2
 4m(m + 1)  2at
∴ Equation of focal chord is y = (x – a)
−a a a(t2 − 1)
∴ x= and y = 2tx – (t2 – 1)y – 2at = 0
2
4(m + 1) 4m(m2 + 1)
The distance from (0, 0) is
x −ay 2
⇒m=– ⇒x= 2t
y 4(x2 + y 2 ) =P
∴ 4x(x2 + y2) + ay2 = 0 (t + 1)2
2

| 2at |
∴ =P
Sol 5: (D) (y + 2)2 = 6(x + 1) t2 + 1
let y = Y – 2 & x = X – 1
M a them a ti cs | 10.67

 1
2 Let midpoint of A, B be (h, k)
Now length of focal chord is a  t + 
 t ∴ Equation of chord through (h, k) is
 1 2a xh – 2b(y + k) = h2 – 4bk
∴ t +  =
 t  P
hx – 2by + 2bk – h2 = 0  … (ii)
2 3
4a 4a
∴ Lf = a × = Equation (i) and (ii) are the same lines
P2 P2
m m2 a
∴= =
Sol 9: (B) Let the points be A(am12 , −2am1 ) h 2b 2bk − h2

& B = (am22 − 2am2 ) 2ab (ab)2


∴ =
1 2bk − h2 (2bk − h2 )2
m1m2 = – 1 ⇒ m2 = –
m
∴ 2b(2by – x2) = ax2
The line joining A & B is
2a(m2 − m1 ) ∴ x2(a + 2b) = 4b2y is locus of M
(y + 2am1) = (x − am12 )
a(m1 − m2 )(m1 + m2 )
Sol 12: (A) 4y2 – 4y + 1 = 16x – 24
−2am1
y + 2am1 = (x − am12 )  3  1 
2
3
m12 −1 (2y – 1)2 = 16  x −  ⇒  y −  = 4  x − 
 2   2   2
∴ ym12 – y + 2am13 – 2am1 = – 2m1x + 2am13 3 1
let x = X + and y = Y +
∴ y + m1 (–m1y + 2a – 2x) = 0 2 2
∴ It passes through intersection of y = 0 & ∴ Y2 = 4aX ⇒ a=1
– m1y + 2a – 2x = 0 The circle cuts the axis at (–a, 0) and (3a, 0)
∴ It always passes through (a, 0) 1 1 9 1
∴ The points in original system are  ,  &  , 
2 2 2 2
Sol 10: (D) Let (h, k) be the point Sol 13: (B) m = 1
The equation of normals through (h, k) is ∴ Equation of tangent is y = x + A and equation of
∴ k = mh – 2am – am 3 normal is

∴ am3 + (2a – h)m + k = 0 y = mx – 2Am – Am3


m = 1 ∴ y = x – 3A
q1 and q2 are complimentary
∴ tan(q1 + q2) = tan(90º) A − ( −3A)
∴ ⊥ distance = = 2 2A
2
∴ m 1m 2 = 1
m1 + m 2 + m3 = 0 Sol 14: (B) 4a = 4 ∴ a = 1
2a − h ∴ Equation of latus rectum is y = 4
m1 m2 + m3(m1 + m2) =
a ∴ Equation of directrix is y = 4 ± 2
k k
m 1m 2m 3 = – ⇒ m3 = – Now focus = (4, 4)
a a
2 2a − h When directrix is y = 6
∴ 1 – m3 =
a ∴ Tangent at vertex is y = 5
∴ y + 2a – ax = a
2 2 2
And parabola lies below y = 5
∴ y = ax – a ⇒ y = a(x – a) is locus of P
2 2 2
When directrix is y = 2
∴ Tangent at vertex x is y = 3
Sol 11: (A) Let the tangent to parabola be
∴ Parabola lies above y = 3
a
x = my – or mx – m2y + a = 0  … (i) But y-coordinate of point is (1)
m
1 0 . 6 8 | Parabola

∴ y = 6 is the directrix Centre passes through this


∴ k = 12 – 2h
Sol 15: (D) Consider a line L2, 2 units to left of L1 &
∴ Centre is (h, 12 – 2h)
parallel to L1
and distance from focus = distance from (4, 4)
L2 L1
∴ (h – 4)2 + (8 – 2h)2 = (h – 1)2 + (12 – 2h)2
∴ – 8h – 32h + 64 + 16
r Cr 2 2
x +y =4 = – 2h – 48h + 144 + 10h = 145 – 80
13
∴ h=
2
2
 13   11  5 5
∴ Distance of centre of circle c from L2 = 2 + r ∴Centre =  , − 1  and radius =   =
 2   2  2
& distance of C from origin = 2 + r
∴Equation of circle is x2+y2–13x + 2y + 12 = 0
∴Locus of C is a parabola

Sol 18: (D) Three quarters of the latus rectum = 3a


Sol 16: (C) P = (at2, 2at) 2
 3a 
x ∴ Equation of circle is x2 + y2 =  
Equation of tangent is y = + at 2
t
and equation of parabola is y2 = 4x
Equation of normal is y = – tx + 2at + at3
∴ Point of intersection of parabola & circle is
T = (–at2, 0) & G = (2a + at2, 0) 2
 3a 
Since PT ⊥ PG x2 + 4ax –   = 0
2
∴ The circle passing through PTG will have its centre at ∴ 4x + 16ax – 9a2 = 0
2

midpoint of T and G
4x2 + 16ax – 2ax – 9a2 = 0
∴ C = (a, 0)
But y2 > 0 & x >0.
2at 2t a
Slope PC = = ∴ x= is only possible solution
2 2
a(t − 1) t −1 2
1 − t2 ⇒y=± 2a
∴ Slope of tangent at P =
2t a  a 
1 ∴ P =  , 2a  ; Q =  , − 2a 
Slope of tangent to parabola = 2  2 
t
L1 L2 = 4a
1 (1 − t2 )
− (1 + t2 )t 1
tan(q1 – q2) = t 2t = = |t| h (PQ + L1L2)
Area of trapezium =
1 (1 − t2 ) (1 + t2 ) 2
1+ ×
t 2t 1 a 2+ 2 
= × (2 2a + 4a) =   a2 Ans (D)
∴ |q1 – q2| = tan–1t 2 2  2 
 

Sol 17: (A) Let P be (h, h) Multiple Correct Choice Type


∴ h2 – 4h = 0 ⇒ h = (4, 4) or h = (0, 0)
Sol 19: (A, B, C, D)
Let centre be (h, k)
Equation of normal is y = mx – 2am – am3
Centre lies on normal at (4, 4)
or am2 + (2a – x)m + y = 0
2t = 4 ⇒ t = 2
The points are (ami2 , −2ami )
∴ Equation of normal is y = – 2x + 4 + 8
m1 + m2 + m3 = 0
⇒ y + 2x = 12
Algebraic sum of ordinates is –2a(m1+ m2 + m3) = 0
M a them a ti cs | 10.69

The y-coordinate of centroid of triangle is 1 


∴ Vertex is at  , 1 
−2a 2 
(m1 + m2 + m3) = 0
3 And axis is parallel to x-axis a = 1
∴ It lies on x-axis 3 
∴ Focus is at  , 1 
2 
Sol 20: (A, B) y2 – 2y – 4x – 7 = 0
∴(y – 1)2 = 4(x + 2) a = 1 Sol 24: (A, D) The Parabola & circle both pass through
origin
Its axis is x-axis
∴ The circle touches parabola at (0, 0)
a’ of the 2nd parabola = 2 × a = 2
∴ Centre of circle = (–b, 0)
∴ Equation of 2nd parabola is (x + 2)2 = ± 8(y – 1)
If a > 0 ∴ –b < 0 ∴ b > 0 & if a < 0 b < 0
∴ x2+ 4x – 8y +12 = 0 & x2 + 4x + 8y – 4 = 0
can be the equation of 2nd parabola Sol 25: (A, D) Let P = (at2, 2at)
2x
Sol 21: (A, B) Equation of parabola 1 is ∴ PA is y =
t
(x – a)2 + (y – b)2 = y2 and of parabola 2 is  2a 
M = (–a, 2at) and D =  −a, −  , end points of diameter
(x – a) + (y – b) = x
2 2 2
 t 
Their common chord is such that x2 = y2 ∴ Equation of circle is
2
y  2a 
∴   = 1 y→= y±=
x ±x (x + a) (x + a) + (y – 2at)  y +  = 0,
 t 
x
∴ Slope = ±1 It intersects x-axis therefore satisfying y = 0

(x + a)2 = 4a2
Sol 22: (B, C) For parabola
= y 2 40
= x, a 10
∴ x = 2a – a or x = – 2a – a
∴ Equation of tangent to parabola is
∴ It intersects x-axis at (a, 0) & (–3a, 0)
10
∴ y = mx +
m
10
∴ ⊥ distance from origin is
m 1 + m2
Previous Years’ Questions
Since it is tangent to circle
Sol 1: (C) Given curves are x = t2 + t + 1  ....(i) ... (i)
∴ ⊥ from centre = radius
and y2=t2 -2 t + 1  ... (ii)
102 and y = t − t + 1 ......(ii)
⇒ =5 2 On subtracting Eq. (ii) from Eq.(i),
m 1 + m2
x−y = 2t
∴ 2 = m2(1 + m2) Now, substituting the value of 't' in (i)
2
∴ m 4 + m2 – 2 = 0 x−y x−y
⇒x 
=  +  +1
m4 + 2m2 – m2 – 2 = 0  2   2 
∴ m2 = 1 ∴ m = ± 1 ⇒ 4x = (x − y)2 + 2x − 2y + 4

∴ Possible equation of tangents are y = x + 10 ⇒ (x − y)2 = 2(x + y − 2)


⇒ x2 + y 2 − 2xy − 2x − 2y + 4 = 0
and x + y + 10 = 0
Now, ∆ = 1 ⋅ 1 ⋅ 4 + 2 ⋅ ( −1)( −1)( −1)

Sol 23: (A, C) y2 – 2y = 4x – 3 −1 × ( −1)2 − 1 × ( −1)2 − 4( −1)2


=4 − 2 − 1 − 1 − 4 =−4
 1
∴ (y – 1)2 = 4  x −  ∴∆ ≠ 0
 2
and ab − h2 = 1 ⋅ 1 − ( −1)2 = 1 − 1 = 0
⇒ x2 + y 2 − 2xy − 2x − 2y + 4 = 0
Now, ∆ = 1 ⋅ 1 ⋅ 4 + 2 ⋅ ( −1)( −1)( −1)
1−01. ×
70 | 2 Parabola
( −1) − 1 × ( −1)2 − 4( −1)2
=4 − 2 − 1 − 1 − 4 =−4
∴∆ ≠ 0 y
y2 = 16x
and ab − h2 = 1 ⋅ 1 − ( −1)2 = 1 − 1 = 0 Tangant
A as focal chord
Hence, it represents a equation of a parabola.
2
2
P  C (6,0)
Sol 2: (C) Any tangent to y 2 = 4x is of the form x’ x

11 (4,0)
yy=+mx
mx
=+ ,(,(
aa=1) and this touches the circle
1)
= 2
m m
B
(x − 3)2 + y 2 =
9,

1 y’
m(3) + −0
If m =3 ∴ Slope of focal chords as tangent to circle= ±1
m2 + 1
Sol 4: (A)
[∴ Centre of the circle is (3,0) and radius is 3] y

3m2 + 1 y=x
⇒ ± 3 m2 + 1
=
m P F
2 2
⇒ 3m + 1 =±3m m + 1 (2, 2)
N
⇒ 9m4 + 1 +=
6m2 9m2 (m2 + 1) M
4 2 4 2
(1, 1) V
⇒ 9m + 1 + 6m= 9m + 9m
⇒ 3m2 = 1 x+y-2=0
1 x’ x
⇒m= ± O
3
If the tangent touches the parabola and circle above
the x-axis, then slope m should be positive. y’

1 1 Since, distance of vertex from origin is 2 and focus


∴ m= and the equation is y= x+ 3
3 3 is 2 2
or 3y= x + 3. ∴ V (1,1) and F(2,2)(ie, lying on y =
x)
2
Sol 3: (A) Here, the focal chord of y = 16x is tangent Where, length of latusrectum
to circle (x − 6)2 + y 2 =
2 = 4a
= 4 2 (=a 2)
⇒ Focus of parabola as (a,0) ie. (4,0) ∴ By definition of parabola
2 2
Now, tangents are drawn from (4,0) to (x − 6) + y =
2
PM2 = (4a)(PN)
Since, PA is tangent to circle.
Where, PN is length of perpendicular upon x+y-2=0 (ie,
∴ tan θ = slope of tangent tangent at vertex).

AC 2 (x − y)2 x + y −2
tan=
θ = = 1, ⇒ = 4 2 
AP 2 2  2 
Slope of other tangent = – tan θ = −1 ⇒ (x − y)2 = 8(x + y − 2)

Sol 5: Since, equation of normal to the parabola


y2 = 4ax is y + xt = 2at + at3 passes through (3, 0)
⇒ 3t = 2t + t3 ( a = 1)
⇒ t = 0, 1, –1
M a them a ti cs | 10.71

∴Coordinates of the normals are 2at


k=
P(1, 2), Q(0, 0), R(1, –2). Thus, 3
2
 3k  9k 2
1 ⇒ 3h = 2a + a.   ⇒ 3h = 2a +
(A) Area of DPQR = ×1×4=2  2a  4a
2
2  ⇒ 9k2 = 4a(3h – 2a)
(C) Centroid of DPQR =  ,0 
3  4a  2a 
∴Locus of centroid is y2 = x − 
Equation of circle passing through P, Q, R is 3  3 
(x – 1)(x – 1) + (y – 2)(y + 2) + λ(x – 1) = 0  2a 
∴Vertex  ,0  ;
⇒ 1 – 4 – λ = 0 ⇒ λ = –3  3 
2a a a
∴ Required equation of circle is x2 + y2 – 5x = 0 Directrix x – = − ⇒x=
3 3 3
5  5 4a
∴ Centre  ,0  and radius Latus rectum =
2  2 3
 a 2a 
Sol 6: (A, B) The equation of tangent to y = x2, be ∴ Focus  + ,0  i.e., (a, 0)
3 3 
m2
y = mx – .
4 Sol 8: (C, D) Here, coordinates of
Putting in y = –x2 + 4x – 4, we should only get one value
of x i.e.,  t2 + t2 
M =  1 2 ,t1 + t2  i.e., mid point of chord AB.
Discriminant must be zero.  2 
 
m2
∴mx – = – x2 + 4x – 4 y B
4
M
m2 (t12,2t1)
⇒ x2 + x(m – 4) + 4 – =0 r
4 A
x
D=0 A’ P r
Now,(m – 4)2 – (16 – m2) = 0 M’
C
⇒ 2m(m – 4) = 0
B’
⇒ m = 0, 4
MP = t1 + t2 = r …(i)
∴y = 0 and y = 4(x – 1) are the required tangents.
Hence, (A) and (B) are correct answers. 2t2 − 2t1 2
mAB = =
t22 − t12 t2 + t1

Sol 7: (A, D) Equation of tangent and normal at point
(When AB is chord)
P(at2, 2at) is ty = x = at2 and y = –tx + 2at + at3
2
Let centroid of DPTN is R(h, k) ⇒ mAB = [from Eq.(i)]
r
at2 + ( −at2 ) + 2a + at2 2a + at2 2
∴h = = Also, mA′B′ = –
3 3 r
(When A′B′ is chord)
2
(at ,2at)
P
2
2
(-at ,0) N(2a+at ,0)
3
T y=-tx+2at+at
ty=x+at2
1 0 . 7 2 | Parabola

Sol 9: (2) y= 8x = 4.2.x 1


At,c = ⇒m=0
2
Now, for other normals to be perpendicular to each
A (2, 4)
other, we must have m1 . m2 = –1
m2  1 
or +  − c  = 0, has m1m2=–1
B(0, 2) P(1/2, 2) 4 2 
(0, 0) (2, 0) 1 
C  − c
2
⇒   = –1
1/4

M 1 1 3
⇒ –c=– ⇒c=
x = -2 (2, -4) 2 4 4

Sol 12: Let A(t12, 2t1) and B(t22, 2t2) be coordinates of the
∆ LPM
=2 end points of a chord of the parabola y2 = 4x having
∆ ABC slope 2.
∆1 Now, slope of AB is
=2
∆2 2t2 − 2t1 2(t2 − t1 )
m= =
t22 − t12 (t2 − t1 )(t2 + t1 )
Sol 10: If three different normals are drawn from (h, 0)
to y2 = 4x. Then, equation of normals are 2
=
y = mx – 2m – m which passes through (h, 0)
3 t2 + t1
y
⇒ mh – 2m – m3 = 0
A(t12,2t1)
⇒h=2+m 2

1
2 + m2 ≥ 2 P(h,k)
x’ x
∴ h > 2 (Neglect equality as if 2 + m2 = 2 ⇒ m = 0) O 2
B(t22,2t2)
Therefore, three normals are coincident.
∴h>2
y’

Sol 11: We know that, normal for y2 = 4ax is given by, But m = 2(given)
y = mx – 2am – am3.
2
∴Equation of normal for y2 = x is ⇒2=
t2 + t1
m m3  1
y = mx – –  a =  ⇒ t1 + t2 = 1 …(i)
2 4  4
Let P(h, k) be a point on AB such that, it divides AB
Since, normal passes through (c, 0)
internally in the ratio 1 : 2.
m m3 2t12 + t22 2(2t1 ) + 2t2
∴mc – – =0
2 4 Then, h = and k =
2+1 2+1
 1 m2 
⇒ m  c − −  =0 ⇒ 3h = 2t1 + t222
…(ii)
 2 4 
 1 and 3k = 4t1 + 2t2 …(iii)
⇒ m = 0 or m2 = 4  c − 
 2 On substituting value of t1 from Eq. (i) in Eq. (iii)
⇒ m = 0, the equation of normal is y = 0 3k = 4(1 – t2) + 2t2
Also, m ≥ 0
2
⇒ 3k = 4 – 2t2
1 1 3k
⇒c– ≥0⇒c≥ ⇒ t2 = 2 – …(iv)
2 2 2 
M a them a ti cs | 10.73

On substituting t1 = 1 – t2 in Eq.(ii), we get Applying R3 → R3 – R2 and


3h = 2(1–t2)2 + t22 R2 → R2 – R1, we get
= 2(1 – 2t2 + t22) + t22 = 3t22 – 4t2 + 2 at2 2at1 1
 2  2 1 2 2
 4 2  2 2 D1 = a(t2 − t1 ) 2a(t2 − t1 ) 0
= 3  t22 − t2 +  = 3  t2 − 2  + 2 − 4  = 3  t2 −  +

2
 3 3  3 3 9  3 3 a(t32 − t22 ) 2a(t3 − t2 ) 0
 
2 2
2  2  2  3k 2  2 2
1 a(t2 − t1 ) 2a(t2 − t1 )
⇒ 3h – =3  t2 −  ⇒ 3 h −  = 3  2 − −  =
3  3  9  2 3 2 a(t32 − t22 ) 2a(t3 − t2 )
[From Eq. (iv)]
2 2 1 (t − t )(t + t ) (t2 − t1 )
 2  4 3k   2 9 8 = a.2a 2 1 2 1
⇒ 3h −  = 3 −  ⇒ h −  = k −  2 (t3 − t2 )(t3 + t2 ) (t3 − t2 )
 9   3 2   9  4  9 
2
 8 4 2 t2 + t1 1
⇒ k −  = h −  = a2(t2 – t1)(t3 – t2)
 9  9  9  t3 + t 2 1

On generating, we get the required locus = a2|(t2 – t1)(t3 – t2)(t1 – t3)|


2
 8 4 2 Again, let D2 = area of the triangle PQR
y −  = x − 
 9  9  9  at1 t2 a(t1 + t2 ) 1 t1 t2 (t1 + t2 ) 1
2 8 1 1
This represents a parabola with vertex at  ,  = at t a(t2 + t3 ) 1 = a.a t2 t3 (t2 + t3 ) 1
9 9 2 23 2
at3 t1 a(t3 + t1 ) 1 t3 t1 (t3 + t1 ) 1
Sol 13: Let the three points on the parabola be
A(at12, 2at1), B(at22, 2at2) and C(at32, 2at3). Applying R3 → R3 – R2, R2 → R2 – R1, we get

Equation of the tangent to the parabola at (at2, 2at) is t1 t2 t1 + t2 1


a2
ty = x + at 2
= t (t − t ) t3 − t1 0
2 2 3 1
Therefore, equations of tangents at A and B are t3 (t1 − t2 ) t1 − t2 0

t1y = x + at12 …(i) t1 t2 t1 + t2 1


a2
And t2y = x + at2 2
…(ii) = (t − t )(t − t ) × t2 1 0
2 3 1 1 2
t3 1 0
From Eqs. (i) and (ii)
t1y = t2y – at22 + at12 a2 t 1
= (t3 – t1)(t1 – t2) 2
⇒ t1y – t2y = at12 – at22 2 t3 1

⇒ y = a(t1 + t2)( t1 ≠ t2) a2


= |(t – t )(t – t )(t – t )|
And t1a(t1 + t2) = x + at12 2 3 1 1 2 2 3
Therefore,
[from Eq.(i)] ⇒ x = at1t2
Therefore, coordinates of P are (at1t2, a(t1 + t2)) ∆1 a2 | (t2 − t1 )(t3 − t2 )(t1 − t3 ) |
= =2
∆2 1 2
Similarly, the coordinates of Q and R are respectively a | (t3 − t1 )(t1 − t2 )(t2 − t3 ) |
2
[at2t3, a(t2 + t3)] and [at1t3, a(t1 + t3)]
Sol 14: Let coordinates of P be (t, t2 + 1)
Let D1 = Area of the triangle ABC Reflection of P in y = x is P1(t2+1, t)
at12 2at1 1 Which clearly lies on y2 = x – 1
1 2
= at 2at2 1 Similarly, let coordinates of Q be (s2 + 1, s)
2 22
at3 2at3 1 Its reflection in y = x is
1 0 . 7 4 | Parabola

Q1(s, s2 + 1) which lies on x2 = y – 1 k3


k
⇒ − − (2 – h) + k = 0
We have, α3 α
PQ12 = (t – s)2 + (t2 – s2)2 = P1Q2 ⇒ k2 = a2h – 2a2 + a3

⇒ PQ1 = P1Q ⇒ y2 = a2x – 2a2 + a3

Also,PP1 || QQ1 On comparing with y2 = 4x

( both perpendicular to y = x) ⇒ a2 = 4 and –2α2 + α3 = 0 ⇒α=2

Thus, PP1QQ1 is an isosceles trapezium.


Sol 16: (A,B,D) Equation of normal to parabola y 2 = 4x
Also, P lies on PQ1 and Q lies on P1Q, we have PQ ≥ is given by
min{PP1, QQ1}
y = mx − 2m − m3
Let us take min {PP1, QQ1} = PP1
It passes through Point (9, 6)
∴ PQ2⇒ PP12=(t2+1–t)2+(t2+1–t)2
6 = 9m − 2m − m3
= 2(t +1–t) =f(t) (say)
2 2
⇒ m3 − 7m + 6 =0
⇒ m3 − 1 − 7m + 7 =0
2
C1 y x =y-1

y=x ( )
⇒ (m − 1 ) m2 + 1 + m − 7 (m − 1 ) =0

⇒ (m − 1 ) (m + m − 6) =
P1
2
Q1 0
P
(0,1) 2
y =-1 ⇒ (m − 1 ) (m + 3)(m − 2 ) =
0
Q
x’
O(1,0)
x ⇒m=−3, 1, 2
∴ The equations of normal are
C2
y’ y − x + 3= 0, y + 3x − 33= 0 and y − 2x + 12 =
0
We have,
Sol 17: (C) let Co-ordinates of point Q are (h, k)
f ′(t) = 4(t2+1–t)(2t–1) = 4[(t–1/2)2+3/4][2t–1]
According to the given condition
Now, f ′(t) = 0 ⇒ t = 1/2
Also, f ′(t) < 0 for t < 1/2 and f ′(t) > 0 for t > 1/2
Corresponding to t = 1/2, point P0 on C1 is (1/2, 5/4)
and P1 (which we take as Q0) on C2 are (5/4, 1/2). Note
that P0Q0 ≤ PQ for all pairs of (P, Q) with P on C1 and Q
on C2.

Sol 15: We know, equation of normal to y2=4ax is


y = mx – 2am – am3
Thus, equation of normal to y2 = 4x is,
y = mx – 2m – m3, let it passes through (h, k).
0 × 3 + x ×1 x
⇒ k = mh – 2m – m3 h= = ⇒ x = 4h
1+3 4
Or m3 + m(2 – h) + k = 0  …(i)
0 × 3 + y ×1 y
Here,m1 + m2 + m3 = 0, k= = ⇒ y = 4k
1+3 4
m1m2+m2m3+m3m1 = 2–h P (x, y) lies on the Parabola y 2 = 4x
m1m2m3 = –k, where m1m2 = α
( 4k ) = 4 ( 4h) ⇒ k =
2 2
k h
α
⇒ m = – it must satisfy Eq.(i)
3
⇒ Locus is y 2 = x
M a them a ti cs | 10.75

Sol 18: (B) Tangents at end points of latus rectum are y = x + 2 and
–y = x + 2, intersection point (-2, 0)

1 
Equation of tangent at P  , 2  is given by y = 2x +
2 
1 
1. Points of intersection of tangent at P  , 2  and
2 
tangents at latus rectum are (-1, -1) and (1, 3)

Area of ∆ formed by Points (-2, 0), (-1, -1) and (1, 3)

∫ (1 − x )
2
R=
1 dx
o
1

∫ (1 − x )
2
R=
2 dx
b
1
Given, R1 − R 2 =
4
b 1
1
∫ (1 − x ) dx − ∫ (1 − x ) dx =
2 2

o b
4 −2 0 1
1
b 1 ∆2 = −1 −1 1
 1−x 3  1−x 3

⇒ −
( )  +
( )  =1 2
1 3 1
 3   3  4
  o  b 1 1
=  −2 ( −1 − 3) + 0 + 1 ( −3 + 1 )  = 8 − 2 = 3
2   2 
 1 − b)  1
(
3
−1   ∆ 6
(1 − b ) − 1 + 0 − 3  =
3 
⇒ ⇒ 1 == 2
3  4 ∆2 3
 


−2
3
(
3 1 1
1 − b) = − =
4 3

1
12
( )
Sol 20: (B) Let Point P be at2 , 2at then other end of
 a 2a 
1 focal Chord Q is  , − 
1
⇒ 2 (1 − b ) = ⇒ (1 − b ) =
3
3
 t2 t 
4 3
We know that point of intersection of tangent at
1
⇒ (1 − b ) =⇒ b =
2
1
2
(at , 2at ) and (at , 2at ) is given by at t , a ( t
2
1 1
2
2 2 1 2 1 + t2 ) 

Sol 19: (2) The end points of latus rectum are A (2, 4),
B (2, -4)
1 
Area of ∆ formed by A (2, 4), B (2, -4) and P  , 2 
2 
2 4 1
1
=
∆1 2 −4 1
2
1
2 1
2
1   1  1 
= 2 ( −4 − 2 ) − 4  2 −  + 1  4 + 4 ×  
2  2  2 
  1 
1 ⇒ T  −a, a  t −   , which lies on y = 2x + a
= 6  6 sq. units
12 − 6 + =  t 
2  
1 0 . 7 6 | Parabola

 1 Tangent at F yt = x + 4t2
⇒ a  t −  =−2a + a =−a
 t a : x = 0 y = 4t (0, 4t)
1 1 (4t2, 8t) satisfies the line 8t = 4mt2 + 3
⇒ t− =−1 =t2 + =1 + 2
t t2 4mt2 - 8t + 3 = 0
2
 1
⇒ t +  = 5 0 3 1
 t
 1
2 Area =
1
2 2
0 4t 1 =
1
2
( )
4t2 ( 3 − 4 t ) = 2t2 ( 3 − 4t )
= a  t +  = 5a
Length of Chord 4t 8t 1
 t

2 =A 2 3t2 − 4t3 
Sol 21: (D) Slope OP =  
t dA
=2 6t − 12t2  =24t (1 − 2t )
Slope OQ = −2t dt  

2 2 1 
− ( −2t ) + 2t 2  t + t 
- + -

tan θ = t = t=   0 1/2
2 1−4 −3
1 + ( −2t ) t = 1/2 maxima
t
G(0, 4t) ⇒ G (0, 2)
2× 5 2 5 y1 = 2
= = −
−3 3
(x0, y0) = (4t2, 8t) = (t, 4)
y0 = 4
3 1 3−2 1
Area = 2  −  = 2  =
 4 2  4  2

Sol 23: (B)

2
 1
t +  =
5
 t

From previous question


1 PK || QR
Since, t − =− 1
t
2a
1 2ar +
⇒ t + =5 2at − 0 t
t =
at2 − 2a a
ar 2 −
t2
Sol 22: (A)
 1
(0,6) 2r + 
2t  t t 1
⇒ = ⇒ =
F(x0y0) 2
t −2  1  1 t −2 r −1
2
r +  r −  t
(0,3)E
 t t
G
t2 − 1
⇒ tr − 1 = t2 − 2 ⇒ tr = t2 − 1 ⇒ r =
  9
4
,0 t
M a them a ti cs | 10.77

Sol 24: (B) x−0 y −0 0 + 0 + 4 


= = −2 
Equation of tangent at O 1 1 12 + 12
t y= x + at2  … (i) x−0 y −0
⇒ == −4
Equation of normal at S 1 1
y=−sx + 2as + as3  … (ii) ⇒ V' ( x, y ) ≡ ( −4, − 4 )
From (i) and (ii)
Image of focus (1, 0) about the line
−s t y − at2  + 2as + as2
y= x+y+4=0
 
2 3
⇒y=−s t y + as t + 2as + as x −1 y − 0 1 + 0 + 4 
= −2 
= −5
=
2a a 1 1 12 + 12
⇒ y =− y + at + + st =1 
t t3 
⇒ F' ( x, y ) ≡ ( −4, − 5 )
( )
2
4 2 a 1 + t2
at + 2at + a ⇒y=
⇒ 2y = The line y = 5 Passes through the focus of the parabola
t3 2 t3
C, so y = -5 is latus rectum of Parabola (C), and. A and
B are end points of latus rectum.
Sol 25: The given Parabola y 2 = 4x has vertex (0, 0)
and focus (1, 0) The length of latus of C is same as of y 2 = 4x

Image of Vertex (0, 0) about the given line ⇒ AB = 4a = 4 × 1 =4

x + y + 4 = 0 is given by
2017-18 100 &
op kers
Class 11 T
By E ran culty
-JE Fa r
IIT enior emie .
S fP r es
o titut
Ins

MATHEMATICS
FOR JEE MAIN & ADVANCED
SECOND
EDITION

Exhaustive Theory
(Now Revised)

Formula Sheet
9000+ Problems
based on latest JEE pattern

2500 + 1000 (New) Problems


of previous 35 years of
AIEEE (JEE Main) and IIT-JEE (JEE Adv)

5000+Illustrations and Solved Examples


Detailed Solutions
of all problems available

Topic Covered Plancess Concepts


Tips & Tricks, Facts, Notes, Misconceptions,
Ellipse Key Take Aways, Problem Solving Tactics

PlancEssential
Questions recommended for revision
11. ELLIPSE

1. INTRODUCTION
An ellipse is defined as the locus of a point which moves such that the ratio of its distance from a fixed point (called
focus) to its distance from a fixed straight line (called directrix, not passing through fixed point) is always constant
and less than unity. The constant ratio is denoted by e and is known as the eccentricity of the ellipse.
Ellipse can also be defined as the locus of a point such that the sum of distances from two fixed points (foci)
is constant. i.e. SP + S’P = constant where S1S' are foci (two fixed points), P being a point on it. It has a lot of
applications in various fields. One of the most commonly known applications is Kepler’s first law of planetary
motion, which says that the path of each planet is an ellipse with the sun at one focus.

Illustration 1: Find the equation of the ellipse whose focus is (1, 0) and the directrix x + y + 1 = 0 and eccentricity
1
is equal to .  (JEE MAIN)
2
Z
Sol: Using the definition of ellipse we can easily get the equation of ellipse.
Let S (1, 0) be the focus and ZZ’ be the directrix. Let P(x, y) be any point on the ellipse
and PM be the perpendicular drawn from P on the directrix. Then by definition
P(x,y)
M
1
SP = e. PM, where e = .
2
x + y + 1 = 10

2
2 2 2 2 2 1 x + y + 1
⇒ SP = e PM ⇒ (x − 1) + (y − 0) = 
2 1+1 
S (focus)
⇒ 4{(x − 1)2 + y 2 } = (x + y + 1)2

⇒ 4x2 + 4y 2 − 8x + 4 = x2 + y 2 + 1 + 2xy + 2x + 2y

⇒ 3x2 + 3y 2 − 2xy − 10x − 2y + 3 =0 Z’


Figure 11.1
2 2
Note: The general equation of a conic can be taken as ax + 2hxy + by + 2gx + 2fy + c =0

This equation represents ellipse if it is non degenerate (i.e. eq. cannot be written into two linear factors)
 a h g
 
Condition: ∆ ≠ 0, h < ab. Where  ∆ = h b f 
2

 g f c 

1 1 . 2 | Ellipse

PLANCESS CONCEPTS

• The general equation ax2 + 2hxy + by 2 + 2gx + 2fy + c =0 can be written in matrix form as

 a h g  x 
a h   x    
 x y      + 2gx + 2fy + c =0 and  x y 1 h b f   y  = 0
h b   y  g f c  1 
  
Degeneracy condition depends on determinant of 3x3 matrix and the type of conic depends on
determinant of 2x2 matrix.
•  Also the equation can be taken as the intersection of z =ax2 + 2hxy + by 2 and the plane
z= − ( 2gx + 2fy + c )
Vaibhav Gupta (JEE 2009, AIR 54)

2. STANDARD EQUATION OF ELLIPSE


Let the origin be the centre of the ellipse and the major and minor axis be on the x-axis and y-axis respectively. It
means foci lies on x-axis and the coordinates of F1 are (-c, o) and F2 be (c, o). Let P be any point (x, y) on the ellipse.
By the definition of the ellipse, the sum of the distances from any point P(x, y) to foci F1 and F2 = constant.
Let us consider this constant to be 2a for the sake of simplicity.
PF1 + PF2 = 2a …(i)

PF1 2 = (x + c)2 + (y –0) 2

⇒ PF1 = (x + c)2 + y 2  …(ii)

Similarly, PF2 = (x − c)2 + y 2  …(iii)

Putting the value of PF1 and PF2 in (i) from (ii) and (iii), we get

(x + c)2 + y 2 + (x − c)2 + y 2 =
2a ⇒ (x + c)2 + y 2 = 2a − (x − c)2 + y 2

On squaring, we get

(x + c)2 + y 2 = 4a2 − 4a (x − c)2 + y 2 + (x − c)2 + y 2



Y
x2 + 2cx + c2 + y 2 = 4a2 − 4a (x − c)2 + y 2 + x2 − 2cx + c2 + y 2

C P(x,y)
⇒ 4cx = 4a2 − 4a (x − c)2 + y 2
A O B
⇒ 4a (x − c)2 + y 2 = 4a2 − 4cx X’ X
F₁(-c,0) F₂(c,0)
c
⇒ (x − c)2 + y 2 =a − x D
a
Squaring both sides, we get
2 Y’
c 
(x − c)2 + y 2 = a2 − 2cx +  x  Figure 11.2
a 
c2
⇒ x2 − 2cx + c2 + y 2 =a2 − 2cx + x2
2
a
M a them a ti cs | 11.3

 c2 
⇒  1 − 2  x2 + y 2 = a2 − c2
 a 

 a2 − c2  2
⇒   x + y 2 = a2 − c2
 a2 
 

x2 y2 x2 y 2
+ 1
= + 1
= …(iv)
2
a a2 − c2 a2 b2
2
[taking b= a2 − c2 ]
This is the standard form of the equation of an ellipse

x2 y2
+ = b2 a2 (1 − e2 ) i.e. b > a
1 , Where =
2 2
a b

PLANCESS CONCEPTS

x2 y2
Domain and range of an ellipse + 1 are [–a, a] and [–b, b] respectively.
=
a2 b2
Vaibhav Krishnan (JEE 2009, AIR 22)

3. TERMS RELATED TO AN ELLIPSE


Vertices: The points A and A’, in the figure where the curve meets the line joining the foci S and S’, are called the
vertices of the ellipse. The coordinates of A and A’ are (a, 0) and (–a, 0) respectively.

Major and Minor Axes: In the figure, the distance AA’= 2a and BB’= 2b are called the major and minor axes of the
b2 a2 (1 − e2 ) . Therefore a > b ⇒ AA’ > BB’.
ellipse. Since e<1 and =

Foci: In figure, the points S (ae, 0) and S’ (–ae, 0) are the foci of the ellipse.

Directrix: ZK and Z’K’ are two directrix of the ellipse and their equations are x = a/e and x=– a/e respectively.

Centre: Since the centre of a conic section is a point which bisects every chord passing through it. In case of the
x2 y2
ellipse + 1 every chord is bisected at C (0, 0). Therefore, C is the centre of the ellipse in the figure and C is
=
a2 b2
the mid-point of AA’.
2 2
x2 y2  Minor axis  b
Eccentricity of the Ellipse: The eccentricity of ellipse + 1 , a > b is e =
= 1− 1− 
 =
 Major axis  a
2 2
a b

Ordinate and Double Ordinate: Let P be a point on the ellipse and let PN be
perpendicular to the major axis AA’ such that PN produced meets the ellipse at M L
P’. Then PN is called the ordinate of P and PNP’ the double ordinate of P.

Latus Rectum: It is a double ordinate passing through the focus. In Fig. 3, LL’ S S’

is the latus rectum and LS is called the semi-latus rectum. MSM’ is also a latus
M’ L’
2b2 x2 y2
rectum. The length of latus rectum of the ellipse + 1 , is = 2a(1 − e2 ) .
=
a2 b2 a
Figure 11.3
1 1 . 4 | Ellipse

x2 y 2
Focal Distances of a Point on the Ellipse: Let P(x, y) be any point on the ellipse + 1 as shown in Fig. 11.4.
=
Then, by definition, we have a2 b2

SP= ePN and S’P = e PN’


Y
⇒ SP = eP’Q and S’P = e (P’Q‘)
⇒ SP = e (CQ – CP) and S’P = e (CQ’ + CP’) P
N’ N
a  a 
⇒ SP = e  − x  S’P = e  + x 
e  e  P S X
Q’ S’ C Q
⇒ SP = a – ex and S’P = a + ex
Thus, the focal distances of a point P(x, y) on the ellipse
x2 y2
+ =1 are a –ex and a + ex.
a2 b2
Also, SP + S’P= a – ex + a + ex = 2a = Major axis (constant) Figure 11.4
Hence, the sum of the focal distances of a point on the ellipse is constant and is equal to the length of the major
axis of the ellipse.

PLANCESS CONCEPTS

The above property of an ellipse gives us a mechanical method of tracing an ellipse as explained below:
Take an inextensible string of a certain length and fasten its ends to two fixed knobs. Now put a pencil
on the string and turn it round in such a way that the two portions of the string between it and the fixed
knobs are always tight. The curve so traced will be an ellipse having its foci at the fixed knobs.

Shrikant Nagori (JEE 2009, AIR 30)

4. PROPERTIES OF ELLIPSE

 x2 y 2 
Ellipse  2 + 2 = 1
Important Terms  a b 

For a > b For b > a


Centre (0, 0) (0, 0)
Vertices (±a, 0) (0, ±b)
Length of major axis 2a 2b
Length of minor axis 2b 2a
Foci (±ae, 0) (0, ± be)
Equation of directrices x = ±a/e y = ±b/e
Relation in a, b and e b2 = a2 (1 –e2) a2=b2 (1–e2)
Length of latus rectum
2b2 2a2
a b
M a them a ti cs | 11.5

Ends of latus rectum   a2 


b2 
 ±ae, ±   ± , ± be 
 a   b 
  

Parametric equations (a cos φ, b sinφ) (a cos φ, b sin φ)

(Discussed later) (0 ≤ φ < 2π) (0 ≤ φ < 2π)


Focal radii SP = a – ex1 SP = b –ey1

S’P = a + ex1 S’P = b + ey1


Sum of focal radii 2a 2b

SP + S’P =
Distance between foci 2ae 2be
Distance between directrices 2a/e 2b/e
Tangents at the vertices x = –a, x = a y = b, y = –b

PLANCESS CONCEPTS

The vertex divides the join of the focus and the point of intersection of directrix with the axis internally
and externally in the ratio e: 1
Misconceptions: If a>b it is a horizontal ellipse, if b > a it is a vertical ellipse unlike hyperbola.

Nitish Jhawar (JEE 2009, AIR 7)

Illustration 2: Find the equation of the ellipse whose foci are (4, 0) and (–4, 0) and whose eccentricity is 1/3.
 (JEE MAIN)
Sol: Use the property of the centre of an ellipse and the foci to find the equation.
Clearly, the foci are on the x-axis and the centre is (0, 0), being midway between the foci. So the equation will be
in the standard form.
x2 y2
Let it be + 1.
=
a2 b2
Foci are (acos θ, bsin θ) . Here they are ( ±4,0) .
∴ ae = 4
1
Given e=
3
1
∴ a. = 4 , i.e., a = 12
3
b2 a2 (1 − e2 )
Again, =

 1  8
⇒ b2= 122.  1 − = 122. = 128
2
 3  32
x2 y2
P(acos θ, bsin θ) . The equation of the ellipse is + 1
=
144 128
1 1 . 6 | Ellipse

Illustration 3: From a point Q on the circle x2 + y2 = a2 perpendicular QM is drawn to x-axis, find the locus of point
‘P’ dividing QM in ratio 2 : 1. (JEE MAIN)

Sol: Starting from a point on the circle find the foot of the perpendicular on the X-axis and hence find the locus.
Let by secφ + ax cosec φ + (a2 + b2 ) = 0 , M ≡ (acos θ,0) and P ≡ (h,k)
2 2
asin θ  3k   h 

= h acos θ , k = ⇒   +  = 1
3  a  a
x2 y2
⇒ Locus of P is + 1.
=
a2 (a/ 3)2

Illustration 4: Draw the shape of the given ellipse and find their major axis, minor axis, value of c, vertices, directrix,
foci, eccentricity and the length of the latus rectum.  (JEE MAIN)
(i) 36x2 + 4y 2 = (ii)
144 4x2 + 9y 2 =
36

Sol: Using the standard form and basic concepts of curve tracing, sketch the two ellipses.

1. Ellipse 36x2 + 4y2 = 144 4x2 + 9y2 = 36


x2 y 2 x2 y 2
or + 1
= or + 1
=
4 36 9 4

2. Shape y2 x2
Since the denominator of is larger then Since the denominator of is greater than
36 9
x2 y2
the denominator of , so the major axis lies the denominator of , so the major axis lies
4 4
along y-axis along x-axis

Directrix
Y
Y

Directrix
Directrix

X’ O X

X’ X
O
x2 y2
+ =1
b2 a2 x2 + y2 =1
Y’ b2 a2
Y’
Directrix

Figure 11.5 Figure 11.6

3. Major axis 2a = 2 × 6 = 12 2a = 2 × 3 = 6
4. Minor axis 2b = 2 × 2 = 4 2b = 2 ×2 =4
5. Value of c a2 = 36, b2 = 4
a2 = 9, b2 = 4 c = a2 − b2 = 9−4 = 5
c= a2 − b2 = 36 − 4= 4 2

6. Vertices (0, –a) and (0, a) (–a, 0) and (a, 0)

(0, –6) and (0, 6) (–3, 0) and (3, 0)


7. Directrices
a2 36 9 a2 9
y=
± =
± =
± x=
± =
±
c 4 2 2 c 5
M a them a ti cs | 11.7

8. Foci (0, –c), (0, c) (–c, 0) and (c, 0)

(0, –4 2 ), (0, 4 2) (– 5 , 0) and ( 5 , 0)


9. Eccentricity
c 4 2 2 2 c 5
e= = = e= =
a 6 3 a 3

10. Length of latus


2b2 2× 4 4 2b2 2× 4 8
rectum 2l = = = 2l = = =
a 6 3 a 3 3

PLANCESS CONCEPTS

The semi-latus rectum of an ellipse is the harmonic mean of the segments of its focal chord.

Shivam Agarwal (JEE 2009, AIR 27)

Illustration 5: Show that x2 + 4y 2 + 2x + 16y + 13 = 0 is the equation of an ellipse. Find its eccentricity, vertices,
foci, directrices, length of the latus rectum and the equation of the latus rectum. (JEE ADVANCED)

Sol: Represent the equation given in the standard form and compare it with the standard form to get the eccentricity,
vertices etc.
We have,
x2 + 4y 2 + 2x + 16y + 13 =
0 ⇒ (x2 + 2x + 1) + 4(y 2 + 4y + 4) =
4

(x + 1)2 (y + 2)2
⇒ (x + 1)2 + 4(y + 2)2 =
4 ⇒ + 1
= … (i)
22 12
Shifting the origin at (–1, –2) without rotating the coordinate axes and denoting the new coordinates with respect
to the new axes by X and Y,
we have x= X − 1 and y= Y − 2  … (ii)
Using these relations, equation (i) reduces to

X2 Y2
+ 1 , where 
= … (iii)
2
2 12
X2 Y2
This is of the form + 1 , where a = 2 and b = 1.
=
a2 b2
Thus, the given equation represents an ellipse.
Clearly a > b, so, the given equation represents an ellipse whose major and minor axes are along the X and Y axes respectively.
b2 1 3
Eccentricity: The eccentricity e is given by e = 1− = 1− =
a 2 4 2
Vertices: The vertices of the ellipse with respect to the new axes are (X = 0) i.e. (X =
±a, Y = 0) .
±2, Y =
So, the vertices with respect to the old axes are given by
( ± 2–1, –2) i.e., (–3, –2) and (1, –2) [Using (ii)]
Foci: The coordinates of the foci with respect to the axes are given by
(X = 0) i.e. (X =
±ae, Y = 0) .
± 3, Y =
1 1 . 8 | Ellipse

So, the coordinates of the foci with respect to the old axes are given by
( ± 3 − 1, − 2) [Putting X = 0 in (ii)]
± 3, Y =
Directrices: The equations of the directrices with respect to the new axes are

a  b2 
X= ± i.e. ea2  1 −=  4a2e2 cos2 θ
e  2 
d 

So, the equations of the directrices with respect to the old axes are

4 4 4  4 
x=
+ − 1 i.e.=
x − 1 and x =
− − 1 Putting X = ± in (ii)
3 3 3  3 

2b2 2
Length of the latus rectum: The length of the latus rectum = = = 1.
a 2
Equation of latus rectum: The equations of the latus rectum with respect to the new axes are

X = ±ae i.e. X = ± 3
So, the equations of the latus rectum with respect to the old axes are

± 3 − 1
x= [Putting X = ± 3 in (ii)]
i.e., =
x 3 − 1 and x =
− 3 −1 .

Illustration 6: A straight rod of given length slides between two fixed bars which include an angle of 90º. Show
that the locus of a point on the rod which divides it in a given ratio is an ellipse. If this ratio is 1/2, show that the
eccentricity of the ellipse is 3 / 2 . (JEE ADVANCED)
Sol: Consider a rod of particular length and write the coordinates of the point in terms of the parameter. Elliminate
the parameters to get eccentricity equal to 3 / 2 .
Let the two lines be along the coordinate axes. Let PQ be the rod of length a such that ∠OPQ = θ . Then, the
coordinates of P and Q are (acos θ, 0) and (0, asinθ) respectively. Let R(h,k) be the point dividing PQ in the ratio

acos θ λ asin θ
λ : 1 . Then, h = and k = .
λ +1 λ +1
h k
⇒ cos θ= (λ + 1) and sin=
θ (λ + 1)  Q
a aλ
h2 k2
⇒ cos2 θ + sin2=
θ (λ + 1)2 + (λ + 1)2 a
2 2 2
a aλ
h2 k2
⇒ + 1.
=
((a / (λ + 1)) ( aλ / (λ + 1))
2 2 
O P
x2 y2
Hence, the locus of R (h, k) is + 1
=
( a / (λ + 1)) ( aλ / (λ + 1))
2 2
Figure 11.7

 1 − λ2 if λ < 1

which is an ellipse of eccentricity given by e = 
2
 1 − (1 / λ ) if λ > 1

1 1 3
When λ = , we have e = 1− = .
2 4 2
M a them a ti cs | 11.9

Illustration 7: A man running a race course notes that the sum of the distances from the two flag posts from him
is always 10 metres and the distances between the flag posts is 8 metres. Find the equation of the path traced by
the man. (JEE ADVANCED)

Sol: Use the basic definition of an ellipse. Clearly, the path traced by the man is an ellipse having its foci at two flag
posts. Let the equation of the ellipse be

x2 y2
+ = b2 a2 (1 − e2 )
1 , where =
a2 b2
Y
It is given that the sum of the distances of the man from the two flag
posts is 10 metres. This means that the sum of the focal distances of
P
a point on the ellipse is 10 m.
⇒ PS + PS’ = 2a = 10 ⇒ a = 5 …(i)
ae ae
It is also given that the distance between the flag posts is 8 metres. X’ X
S O S’
∴ 2ae = 8 ⇒ ae = 4 …(ii)
2 2 2 2 2 2
Now, b =a (1 − e ) =a − a e =25 − 16
⇒ b2 = 9 ⇒ b = 3 [Using (i) and (ii) ] Y’

x2 y 2 Figure 11.8
Hence, the equation of the path is + 1.
=
25 9

5. AUXILIARY CIRCLE
A circle with its centre on the major axis, passing through the vertices of the ellipse is called an auxiliary circle.
x2 y2
If + 1 is an ellipse, then its auxiliary circle is x2 + y 2 =
= a2 .
a2 b2

Figure 11.9

x2 y2
Eccentric angle of a point: Let P be any point on the ellipse = 1 . Draw PM perpendicular from P on the
+
a2 b2
major axis of the ellipse and produce MP to meet the auxiliary circle in Q. Join OQ. The angle ∠QOM = φ is called
the eccentric angle of the point P on the ellipse.
Note that the angle ∠XOP is not the eccentric angle of point P.

PLANCESS CONCEPTS

A circle defined on the minor axis of an ellipse as diameter x2 + y 2 =


b2 is called a minor auxiliary circle.
Ravi Vooda (JEE 2009, AIR 71)
1 1 . 1 0 | Ellipse

6. PARAMETRIC FORM

6.1 Parametric Co-Ordinates of a Point on an Ellipse

x2 y2
Let P(x, y) be a point on an ellipse. + 1 and Q be the corresponding
= Y
a2 b2
Q
point on the auxiliary circle x2 + y 2 =
a2 .
P

φ.
Let the eccentric angle of P be φ. Then ∠XCQ = 
X
C M
Now, x = CM
⇒ x = CQ cos f [ CQ = radius of x2 + y 2 =
a2 ]

x2 y2 a2 cos2 φ y2
Since P(x, y) lies on + 1 ⇒
= + 1.
=
a2 b2 a2 b2 Figure 11.10
2
⇒ y= b2 (1 − cos2 =
φ) b2 sin2 φ ⇒ =y bsin φ .

Thus, the coordinates of point P having eccentric angle φ can be written as (a cosφ, b sin φ) and are known as the
parametric coordinates of an ellipse.

6.2 Parametric Equation of an Ellipse


x2 y 2
The equations x = acosφ, y = bsinφ taken together are called the parametric equations of the ellipse + 1,
=
where φ is the parameter. a2 b2

PLANCESS CONCEPTS

Always remember that θ is not the angle of P with x-axis. It is the angle of corresponding point Q.
Rohit Kumar (JEE 2012, AIR 79)

x2 y2
Illustration 8: Find the distance from the centre to the point P on the ellipse + 1 which makes an angle α
=
a2 b2
with x-axis. (JEE MAIN)

Sol: Establish a relation between the angle α and the eccentric angle. Use parametric coordinates of an ellipse and
the distance formula to find the distance.
Let P ≡ (acos θ,bsin θ) ∴ (b / a) tan=
θ tan α ⇒ =
tan θ ( a / b ) tan α

a2 cos2 θ + b2 sin2 θ = a2 cos2 θ + b2 sin2 θ


OP =  sin2 θ + cos2 θ =1
2
sin θ + cos θ2

a2 + b2 tan2 θ a2 + b2 × (a2 / b2 )tan2 α


= =
1 + tan2 θ 1 + (a2 / b2 )tan2 α

M a them a ti cs | 11.11

7. SPECIAL FORMS OF AN ELLIPSE


(a) If the centre of the ellipse is at point (h, k) and the directions of the axes are parallel to the coordinate axes,
(x − h)2 (y − k)2
then its equation is + 1.
=
a2 b2
(lx + my + n)2 (mx − ly + p)2
(b) If the equation of the curve is + 1 , where x2 + 2y 2 − 6x − 12y + 23 =
= 0 and
a2 b2
lx + my + n mx − ly + p
mx – ly + p =are
0 perpendicular lines, then we substitute =X, = Y , to put the equation
l2 + m2 l2 + m2
in the standard form.

Illustration 9: Find the equation to the ellipse whose axes are of lengths 6 and e2 cos2 φ + cos φ − 1 = 0 and their
equation are x − 3y + 3 = 0 and 3x + y − 1 = 0 respectively. (JEE MAIN)

Sol: Given the equation of the axis, we can find the centre. Use the length of the axes of the ellipse to find
the required equation of the ellipse. Let P (x, y) be any point on the ellipse and let p1 and p2 be the lengths of
perpendiculars drawn from P on the major and minor axes of the ellipse.
x − 3y + 3 3x + y − 1
Then, p1 = and p2= .
1+9 9 +1
Let 2a and 2b be the lengths of major and minor axes of the ellipse respectively. We have, 2a = 6 and 2b= 2 6 .
p12 p22
⇒ a = 3 and b = 6 . The equation of the ellipse is + 1
=
b2 a2
(x − 3y + 3)2 (3x + y − 1)2
⇒ + 1 ⇒ (x − 3y + 3)2 + 2(3x + y − 1)2
=
60 90
⇒ 21x2 − 6xy + 29y 2 + 6x − 58y − 151 =
0

8. EQUATION OF A CHORD
x2 y2
Let P (acos α , bsin α ), Q (acos β , bsin β ) be any two points of the ellipse + 1.
=
a2 b2
x α+β y α+β α −β
Then, the equation of the chord joining these two points is cos   + sin  =cos  .
a  2  b  2   2 

x2 y 2
Illustration 10: Find the angle between two diameters of the ellipse + 1 , whose extremities have eccentric
=
a2 b2
π
angles α and β = α + .  (JEE MAIN)
2

Sol: Find the slope of the two diameters and then use the relation between the given angles.

x2 y2
Let the ellipse be + 1
=
a2 b2
bsin α b bsin β −b π
Slope of OP ==
m1 = tan α ; Slope of OQ =
= m2 = cot α given β = α +
acos α a acos β a 2

m1 − m2 (b / a)(tan α + cot α ) 2ab


∴ tan θ = = =
1 + m1m 2 2
1 − (b / a ) 2
(a − b2 )sin2α
2
1 1 . 1 2 | Ellipse

Illustration 11: lf the chord joining the two points whose eccentric angles are α and β, cut the major axis of an
α β c−a
ellipse at a distance c from the centre, show that tan tan = . (JEE ADVANCED)
2 2 c+a
Sol: Use the fact that the point (c, 0) lies on the chord joining points whose eccentric angles are α and β . The
equation of the chord joining points whose eccentric angles are α and β on the ellipse

x2 y2 x α+β y α+β α −β


+ 1 , is cos 
=  + sin  =cos  
a2
b2 a  2  b  2   2 
This will cut the major axis at the point (c, 0) if

c α+β α −β cos ( (α + β) / 2 ) a cos ( (α + β) / 2 ) + cos ( (α − β) / 2 ) a + c


cos   = cos  ⇒ = ⇒ =
a  2   2  cos ( (α − β) / 2 ) c cos ( (α + β) / 2 ) − cos ( (α − β) / 2 ) a − c

2cos(α / 2) cos(β / 2) a + c α β c−a


⇒ = ⇒ tan tan = .
−2sin(α / 2) sin(β / 2) a − c 2 2 c+a

Illustration 12: The eccentric angle of any point P on the ellipse is φ . If S is the focus nearest to the end A of the
θ 1+e φ
major axis A’A such that ∠ASP =
θ . Prove that tan = tan . (JEE ADVANCED)
2 1−e 2

Sol: Find the distance of the point P from the X-axis and the horizontal distance of the point from nearest focus.
Use trigonometry to get the desired result.
In ∆PSL, we have
PL = bsinφ and SL = a cos φ –ae P

bsin φ 2 tan(θ / 2) 2 1 − e2 tan(φ / 2)
∴ tan θ = ⇒ = A’ S’ C S L A
acos φ − ae 1 − tan2 (θ / 2) (1 − e) − (1 + e)tan2 (φ / 2)

2t an(θ / 2) 2 (1 + e) / (1 − e) tan(φ / 2) θ 1+e φ


⇒ = ⇒ t an = tan
2
1 − tan (θ / 2) (
1 − (1 + e) / (1 − e) ) 2 1−e 2
Figure 11.11

9. POSITION OF A POINT W.R.T. AN ELLIPSE


x2 y2 x12 y 12
The point P(x1 , y1 ) lies outside, on or inside the ellipse + 1 according to
= + −1 > 0,
a2 b2 a2 b2
x12 y12
= 0 or < 0 respectively. S1 = + −1
a2 b2

x2 y 2
Illustration 13: Find the set of value(s) of ‘ α ’ for which the point P(α , − α ) lies inside the ellipse + =1 .
16 9
 (JEE MAIN)
Sol: Apply the concept of position of a point w.r.t. the ellipse.
If P(α , − α ) lies inside the ellipse 2a2 S1 < 0

α2 α2 25 144  12 12 
⇒ + −1 < 0 ⇒ , α2 < 1 ⇒ α2 < ∴α ∈− , .
16 9 144 25  5 5 
M a them a ti cs | 11.13

10. LINE AND AN ELLIPSE


x2 y2
Consider a straight line of the form =
y mx + c and ellipse + 1.
=
a2 b2
(mx + c )
2
x2
By solving these two equations we get, + 1
=
a2 b2

⇒ (b 2
)
+ a2m2 x2 + 2a2mcx + a2 c2 − b2 =
0 ( )
For this equation

(
D 4 a4m2c2 − b2 + a2m2 a2 c2 − b2
⇒= ( ) ( ))
D 4a2b2 b2 − c2 + a2m2
⇒= ( )
x2 y2
∴ The line =
y mx + c intersects the ellipse + 1 in two distinct points if a2m2 + b2 > c2 , in one point if
=
2 2
a b
2
c
= a m + b and does not intersect if a m + b < c2 .
2 2 2 2 2 2

x2 y2
Illustration 14: Find the condition for the line lx + my + n =0 to touch the ellipse + 1 .
= (JEE MAIN)
a2 b2

Sol: Use the theory of equations or the standard form of the tangent. The equation of the line is lx + my + n =0
 l   n x2 y 2
⇒ y =  −  x +  −  . We know that the line=
y mx + c touches the ellipse + if c2 a2m2 + b2 .
1=
=
 m  m
2 2
a b
2 2
 −n   l 
⇒   = a2  −  + b2 ⇒ n2 = a2l2 + b2m2 .
m
   m

x2 y2
Illustration 15: Find the condition for the line xcos α + ysin α = p to be a tangent to the ellipse + =1 .
a2 b2
 (JEE MAIN)
Sol: Use the theory of equations or the standard form of the tangent.
The equation of the given line  is x cos α + y sin α = p ⇒ y = (–cot α ) x – p cosec α

x2 y2 2 2
This will touch + 1 , If ( −pcosec=
= α) a2 cot2 α + b2 [Using:
= c2 a2m2 + b2 ]
2 2
a b
a2 cos2 α + b2 sin2 α
⇒ p2cosec
= 2
α ⇒ p2 a2 cos2 α + b2 sin2 α
=
2
sin α

x2 y 2
Illustration 16: Find the set of value(s) of ' λ ' for which the line 3x − 4y + λ = 0 intersect the ellipse + 1 at
=
16 16
two distinct points. (JEE ADVANCED)

Sol: Same as previous illustration.


(4y − λ )2 y 2
Solving the given line with ellipse, we get
9 × 16
+
9
= 1 ⇒ 32y 2 − 8λ + λ2 − 144 = 0 ( )
Since the line intersects the parabola at two distinct points:
∴ Roots of above equation are real & distinct ∴ D > 0

⇒ ( 8λ )
2
(
− 4.32 λ2 − 144 > 0 ⇒ −12 2 < λ < 12 2 )
1 1 . 1 4 | Ellipse

11. TANGENT TO AN ELLIPSE

11.1 Equation of Tangent

x2 y2 xx1 yy1
(a) Point form: The equation of the tangent to the ellipse + 1 at the point (x1 , y1 ) is
= + 1.
=
2 2 2
a b a b2
x2 y2
(b) Slope form: If the line =
y mx + c touches the ellipse + 1 , then
= = c2 a2m2 + b2 .
2 2
a b
mx ± a2m2 + b2 always represents the tangents to the ellipse.
Hence, the straight line y =

 0 − b  0 − b  x2 y 2
(i) Point of contact: Line    = −1 touches the ellipse + 1 at
=
 ae − 0   −ae − 0  a2 b2
 ±a2m  b2 
 , .
 2 2 2 
 a m +b a2m2 + b2 

x y
(c) Parametric form: The equation of tangent at any point (acos φ, bsin φ ) is cos φ + sin φ =1 .
a b
Remark: The equation of the tangents to the ellipse at points p(acos θ1 ,bsin θ1 ) and Q(acos θ2 ,bsin θ2 ) are
x y x y
cos θ1 + sin θ1 =1 and cos θ2 + sin θ2 =1
a b a b
 acos ( (θ1 + θ2 ) / 2 ) b sin ( (θ1 + θ2 ) / 2 ) 
And these two intersect at the point  , 
 cos ( (θ − θ ) / 2 ) cos ( (θ − θ ) / 2 ) 
 1 2 1 2 

11.2 Equation of Pair of Tangents

Pair of tangents: The equation of a pair of tangents PA and PB is SS1 = T2.


x2 y2
Where S ≡ + −1 A
a2 b2
2 2 (x,y) P
x1 y1
S1 ≡ + −1
a2 b2 B
xx1 yy1
T≡ + −1 Figure 11.12
2
a b2

PLANCESS CONCEPTS

The portion of the tangent to an ellipse intercepted between the curve and the directrix subtends a right
angle at the corresponding focus.
B Rajiv Reddy (JEE 2012, AIR 11)
M a them a ti cs | 11.15

11.3 Director Circle

Definition The locus of the point of intersection of the perpendicular tangents to an ellipse is known as its director
circle.

Equation of the director circle the equation of the director circle, is (x ± ae)2 =y 2 − 4a2 . Clearly, it is a circle
x2 y2
concentric to the ellipse and radius equal to + 1.
=
a2 b2
It follows from the definition of the director circle that the tangents drawn from any point on the director circle of
a given ellipse to the ellipse are always at right angles.

PLANCESS CONCEPTS

Director circle is the circumcircle of ellipse’s circumrectangle whose sides are parallel to the major and
minor axis.

Figure 11.13 Anvit Tanwar (JEE 2009, AIR 9)

Illustration 17: A tangent to the ellipse x2 + 4y2 = 4 meets the ellipse x2 + 2y2 = 6 at P and Q. Prove that the
tangents at P and Q of the ellipse x2 + 2y2 = 6 are at right angles.  (JEE ADVANCED)

Sol: Use the condition of tangency and the standard equation of tangent. The equations of the two ellipses are
x2 y2
+ 1
=
42 12  …(i)

x2 y 2
and + 1 
= …(ii) respectively.
6 3
Suppose the tangents P and Q to ellipse (ii) intersect at R(h, k). PQ is the chord of contact of tangents drawn from
R(h, k) to ellipse (ii). So, the equation of PQ is

hx ky
+ 1
= …(ii)
6 3
ky −hx hx 3
⇒ = + 1 ⇒ y =− + . This touches the ellipse given in (i). Therefore,
3 6 2k k
2
9  −h 
= 4  +1
: c2 a2m2 + b2 ]
[Using =
 2k 
2
k

⇒ h2 + k 2 =9 ⇒ (h,k) lies on the circle x2 + y 2 =


9.

Clearly, x2 + y 2 = 9 is the director circle of the ellipse (ii). Hence, the angle between the tangents at P and Q to the
ellipse is a right angle.
1 1 . 1 6 | Ellipse

11.4 Chord of Contact


x2 y 2
If PQ and PR are the tangents through point P (x1 , y1 ) to the ellipse + 1 , then the equation of the chord of
=
2 2
xx yy a b
contact QR is 1 + 1 = 1 or T = 0 at (x1 , y1 ) .
2
a b2
Y

Q
(x1, y1) P
X’ X
C
R

Y’
Figure 11.14

x2 y2
Illustration 18: Prove that the chord of contact of tangents drawn from the point (h, k) to the ellipse 1
+
=
a2 b2
h2 k2 1 1
will subtend a right angle at the centre, if + = + . (JEE ADVANCED)
4 4 2
a b a b2

Sol: Make the equation of the ellipse homogeneous using the chord and then apply the condition for the pair of
straight lines to be perpendicular.
x2 y 2
The equation of the chord of contact of tangents drawn from (h, k) to the ellipse + 1 is
=
a2 b2
hx ky
+ = 1 …(i)
a2 b2
The equation of the straight lines joining the centre of the ellipse i.e. the origin, to the points of intersection of the
ellipse and (i) is obtained by making a homogeneous equation with the help of (i) and the ellipse and is given by
2
x2 y2
 hx ky   1 h2   1 k 2  2hk
+ − +  0 or x2  −  + y 2 
= − − 0
xy = …(ii)
a2 b2  a2 b2   a2 a4   b2 b 4  a2b2
   
If the chord of contact of tangents subtends a right angle at the centre, then the lines represented by (ii) should
be at right angles.

 1 h2   1 k 2  h2 k 2 1 1
⇒ − + −  0 ⇒
= + = + .
 a2 a4   b2 b 4  a 4
b 4
a2
b2
   

Illustration 19: Find the equations of the tangents to the ellipse 3x2 + 4y 2 =
12 which are perpendicular to the line
y + 2x = 4 . (JEE MAIN)

Sol: Use the slope form of the tangent. Let m be the slope of the tangent. Since the tangent is perpendicular to
the line y + 2x =4
1 x2 y 2
∴ m ( −2 ) =−1 ⇒ m = ; Now, 3x2 + 4y 2 =
12 ⇒ + 1
=
2 4 3
x2 y2
Comparing this with + 1 , we get a2 = 4 and b2 = 3 .
=
2 2
a b
mx ± a2m2 + b2
So, the equations of the tangents are y =

1 x
i.e. y = x ± 4(1 / 4) + 3 ⇒ y = ± 2 ⇒ 2y = x ± 4 .
2 2
M a them a ti cs | 11.17

Illustration 20: Find the equations of the tangents to the ellipse 9x2 + 16y 2 =
144 which pass through the point
(2, 3). (JEE MAIN)

Sol: Put the given point in the standard equation of the tangent and find the value of m.
x2 y2
The equation of the ellipse is 9x2 + 16y 2 =
144 ⇒ + 1
=
42 32
x2 y2
This if of the form + 1 , where a2 = 42 and b2 = 32 .
=
a2 b2
mx ± a2m2 + b2 i.e. y =
The equation of any tangents to this ellipse is y = mx ± 16m2 + 9  …(i)

2m + 16m2 + 9
If it passes through (2, 3) then 3 =
2
⇒ (3 − 2m)= 16m2 + 9 x2 + y 2 =
4 ⇒ m = 0, − 1

Substituting these values of m in (i), we obtain y = 3 and y = –x + 5 as the equations of the required tangents.

Note: If the question was asked to find combined eq. of a pair of tangents then use SS1 = T2.

Illustration 21: The locus of the points of intersection of the tangents at the extremities of the chords of the ellipse
x2 + 2y 2 =
6 which touch the ellipse x2 + 4y 2 =
4 is. (JEE MAIN)

(A) x2 + y 2 =
4 (B) x2 + y 2 =
6 (C) x2 + y 2 =
9 (D) None of these.

Sol: Find the equation of the tangents for the two ellipses and compare the two equations.
x2 y 2
We can write x2 + 4y 2 =
4 as + = 1 …(i)
4 1
x
Equation of a tangent to the ellipse (i) is cos θ + y sin θ =1  …(ii)
2
x2 y 2
Equation of the ellipse x2 + 2y 2 =
6 can be written as + 1
= …(iii)
6 3
Suppose (ii) meets the ellipse (iii) at P and Q and the tangents at P and Q to the ellipse (iii) intersect at (h, k), then
hx ky
(ii) is the chord of contact of (h, k) with respect to the ellipse (iii) and thus its equation is + 1
= …(iv)
6 3
Since (ii) and (iv) represent the same line

a2
DA = CA − CD = − x1 ⇒ h = 3 cos θ, k = 3 sin θ and the locus of (h, k) is x2 + y 2 =
9.
x1

x2 y2
Illustration 22: Show that the locus point of intersection of the tangents at two points on the ellipse + =1,
2 2 a2 b2
x y
whose eccentric angles differ by a right angle is the ellipse + 2 .
= (JEE MAIN)
2
a b2
π
Sol: Solve the equation of the tangents at the two points whose eccentric angles differ by .
2
π
Let P (acos θ, bsin θ) and Q (acos φ, bsin φ) be two points on the ellipse such that θ − φ = . The equations of
tangents at P and Q are 2

x y
cos θ + sin θ =1  …(i)
a b
x y
and, cos φ + sin φ =1  …(ii) respectively.
a b
1 1 . 1 8 | Ellipse

2 x y
Since , so (i) can be written as - sin φ + cos φ =1  …(iii)
3 a b
Let (h, k) be the point of intersection of (i) and (ii). Then,

h k h k
cos θ + sin θ =1 and − sin θ + cos θ =1
a b a b
2 2
h k   h k  h2 k2
⇒  cos θ + sin θ  +  − sin θ + cos θ  = 1 + 1 ⇒ + 2
=
 a b   a b  a2 b2
x2 y2
Hence, the locus of (h, k) is + 2.
=
a2 b2

x2 y2
Illustration 23: Prove that the locus of the mid-points of the portion of the tangents to the ellipse + 1
=
a2 b2
intercepted between the axes a2 y 2 + b2 x2 =
4x2 y 2 . (JEE ADVANCED)

Sol: Starting from the equation of the tangent, find the mid point of the tangent intercepted between the axes.
Eliminate the parameter to get the locus.
x2 y2
The equation of the tangent at any point (acos θ, bsin θ) on the ellipse + 1 is
=
a2 b2
x y
cos θ + sin θ =1
a b
 a   b 
This cuts the coordinates axes at A  , 0  and B  0, 
 cos θ   sin θ

a b
Let P(h, k) be the mid-point of AB. Then, = h and =k
2cos θ 2sin θ

a b a2 b2 a2 b2
⇒ cos θ = and sin θ = ⇒ cos2 θ + sin2 θ = + ⇒ + =1
2h 2k 4h2 4k 2 4h2 4k 2
1
Hence, the locus of P (h, k) is , a2 y 2 + b2 x2 =
4x2 y 2 .
52

x2 y2
Illustration 24: Let d be the perpendicular distance from the centre of the ellipse + 1 to the tangent
=
a2 b2
 b2 
drawn at a point P on the ellipse. If F1 and F2 are two foci of the ellipse, then show that (PF1 − PF2 )2 = 4a2  1 −  .
 d2 

 (JEE ADVANCED)
Sol: Use the fact that focal distances of a point (x, y) on the ellipse are a+ex and a–ex.
Let the coordinates of P be (acos θ, bsin θ) , where θ is a parameter. The coordinates of F1 and F2 are (ae, 0) and
(–ae, 0) respectively. We know that.
Therefore, PF1 = a + ae cos θ and PF2 = a – ae cos θ
i.e., PF1 = a (1 + ecos θ) and PF2 = a (1 − ecos θ)

∴ (PF1 –PF2)2 = {a( 1 + ecos θ ) –a( 1 − ecos θ )}2 = 4a2e2 cos2 θ  …(i)
x y
The equation of the tangent at P (acos θ, bsin θ) is cos θ + sin θ =1  ...(ii)
a b
∴ d = Length of the perpendicular from (0, 0) on (ii)
M a them a ti cs | 11.19

(0 / a)cos θ + (0 / b)sin θ − 1
⇒ d=
cos2 θ / a2 + sin2 θ / b2

1 cos2 θ sin2 θ b2 b2 b2 b2
⇒ = + ⇒
= cos2 θ + sin2 θ ⇒ 1 − = 1 − cos2 θ − sin2 θ
2 2 2 2 2 2 2
d a b d a d a

b2 b2  b2 
⇒ 1 − = cos2 θ − cos=
2
θ cos2 θ  1 − =  e2 cos2 θ
d2 a2  a2 
 
 b2 
⇒ 4a2  1 −=  4a2e2 cos2 θ  …(iii)
 2 
d 

 b2 
Hence, from (i) and (iii), we have (PF1 − PF2 )2 = 4a2  1 −  .
 d2 

x2 y2
Illustration 25: The tangent at point P(cosθ, bsinθ) of an ellipse = 1 , meets its auxiliary circle on two
+
a2 b2
points, the chord joining which subtends a right angle at the centre. Show that the eccentricity of the ellipse is
(1 + sin2 θ)−1/2 .  (JEE ADVANCED)

Sol: Homogenize the equation of the ellipse using the equation of the tangent and then use the condition for the
pair of straight lines to be perpendicular.
x2 y2
The equation of the tangent at P (acos θ,bsin θ) to the ellipse + 1 is
=
a2 b2
x y
cos θ + sin θ =1  …(i)
a b

The equation of the auxiliary circle is x2 + y 2 =


a2  …(ii)
The combined equation of the lines joining the origin with the points of intersection of (i) and (ii) is obtained by
making (ii) homogeneous w.r.to (i)
2
2 x
2 y
2 
∴ x +y
= a  cos θ + cos θ 
a b 
 a2  a
⇒ x2 (1 − cos2 θ) + y 2  1 − sin2 θ  − 2xy sin θ cos θ =0
 b 2  b
 
These two lines are mutually perpendicular. Therefore, coefficient of x2 + Coefficient of y 2 = 0

a2  a2  a2 − b2 2
⇒ sin2 θ + 1 − sin2 θ ⇒ sin2 θ  1 −  + 1= 0 ⇒ sin θ= 1
b2  b2  b2

a2e2 sin2 θ
⇒ = 1 ⇒ e2 sin2 θ = 1 − e2 ⇒ e = (1 + sin2 θ)−1/2 .
a2 (1 − e2 )

x2 y2
Illustration 26: If the tangent at (h, k) to the ellipse + 1 cuts the auxiliary circle x2 + y 2 =
= r 2 at points
2 2
a b
1 1 2
whose ordinates are y1 and y2, show that + = . (JEE ADVANCED)
y1 y 2 k
Sol: Form a quadratic in y using the equation of the tangent and the ellipse and then use the sum and product of
the roots to prove the above result.
1 1 . 2 0 | Ellipse

x2 y2 hx ky
The equation of the tangents to the ellipse + 1 and point (h, k) is
= + 1 . The ordinates of the points
=
2 2 2
a b a b2
of intersection of (i) and the auxiliary circle are the roots of the equation
2
a4  b2 − ky 

2 
 + y2 =
a2
h  b 2 

( )
⇒ y 2 a4k 2 + b 4h2 − 2a4b2ky + a4b 4 − a2b 4h2 =
0

Since y1 and y2 are the roots of this equation.

2a4b2k a4b 4 − a2b 4h2


Therefore, y1 + y 2 = and y1 y 2 =
a4k 2 + b 4h2 a4k 2 + b 4h2

1 1 2a4b2k 1 1 2ka2 1 1 2a2k


⇒ = + ⇒= + ⇒ + = ⇒ 1 + 1 =2
y1 y 2 a4b 4 − a2b 4h2 y1 y 2 (a2 − h2 )b2 y1 y 2 a2k 2 y1 y 2 k

Illustration 27: Find the locus of the foot of the perpendicular drawn from the centre on any tangent to the ellipse.
 (JEE ADVANCED)
x2 y2
+ 1.
=
a2 b2 M()
P(a cos  b sin 

Figure 11.15

Sol: Follow the procedure for finding the locus starting from the parametric equation of the tangent.
The equation of the tangent at any point (acos φ, bsin φ) is
x cos φ y sin φ
+ 1 
= ….(i)
a b
Let M (α,β) be the foot of the perpendicular drawn from the centre (0, 0) to the tangent (i).
α cos φ β sin φ
x2 + y 2 =
C2 M is on the tangent, + 1 
= …(ii)
a b
β  b cos φ 
x2 + y 2 =
C2 CM ⊥ PM, − =−1
α  asin φ 

cos φ sin φ 1
φ asin φ α
or bβ cos= ∴ = = .
aα bβ a2 α2 + b2β2
α aα β bβ
Putting in (ii), . + . 1
=
a a α +b β
2 2 2 2 b a α + b2β2
2 2

or α2 + β2= a2 α2 + b2β2 ∴ (α2 + β2 )2 = a2 α2 + b2β2

∴ The equation of the required locus is (x2 + y 2 )2 = a2 x2 + b2 y 2 .


M a them a ti cs | 11.21

12. NORMAL TO AN ELLIPSE

12.1 Equation of Normal in Different Forms

Following are the various forms of equations of the normal to an ellipse.

x2 y2 a2 x b2 y
(a) Point form: The equation of the normal at (x1 , y1 ) to the ellipse + 1 is
= − =a2 − b2
a2 b2 x1 y1

x2 y2
(b) Parametric form: The equation of the normal to the ellipse + 1 at (acos φ, bsin φ ) is x − 2y + 4 =.
= 0
a2 b2

x2 y 2
(c) Slope form: If m is the slope of the normal to the ellipse + 1 , then the equation of normal is
=
x + 2y + a =.
0 a2 b2

 ±a2  mb2 
The co-ordinates of the point of contact are.  , .
 2 2 2 
 a +b m a2 + b2m2 

12.2 Number of Normal and Co-normal Points


On a given ellipse exactly one normal can be drawn from a point lying on ellipse. If the point is not lying on the
given ellipse, at most 4 lines which are normal to the ellipse at the points where they cut the ellipse. Such points
on the ellipse are called co-normal points. In this section, we shall learn about the co-normal points and various
relations between their eccentric angles.
Conormal points are the points on ellipse, whose normals to the ellipse pass through a given point are called
co-normal points.

12.3 Properties of Eccentric Angles of Conormal Points


x2 y 2
Property 1: The sum of the eccentric angles of the co-normal points on the ellipse + 1 is an odd multiple
=
2 2
of π. a b

x2 y2
Property 2: If θ1 , θ2 and θ3 are eccentric angles of three co-normal points on the ellipse + 1 , then
=
a2 b2
sin(θ1 + θ2 ) + sin(θ2 + θ3 ) + sin(θ3 + θ1 ) = 0

Property 3: Co-normal points lie on a fixed curve called Apollonian Rectangular Hyperbola

(a 2
)
− b2 xy + b2kx − a2hy =
0

x2 y2
Property 4: If the normal at four points P(x1 , y1 ) , Q(x2 , y 2 ) , R(x3 , y 3 ) and S(x 4 , y 4 ) on the ellipse + 1 are
=
a2 b2
1 1 1 1 
concurrent, then (x1 + x2 + x3 + x 4 )  + + + 4.
 =
 x1 x2 x3 x 4 
1 1 . 2 2 | Ellipse

Illustration 28: If the normal at an end of the latus rectum of an ellipse passes through one extremity of the minor,
show that the eccentricity of the ellipse is given by e4 + e2 − 1 =0 .  JEE MAIN)

Sol: Subtitute the point (0, ±b) in the equation of the normal and simplify it.
x2 y2
Let + 1 be the ellipse. The coordinates of an end of the latus rectum are (ae, b2 / a) .
=
a2 b2
a2 x b2 y
The equation of normal at (ae, b2 / a) is − a2 b2
=−
ae b2 / a
It passes through one extremity of the minor axis whose coordinates are (0, ±b).
∴ ± ab = a2 − b2

⇒ a2b2 = (a2 − b2 )2 ⇒ a2 .a2 (1 − e2 ) = (a2e2 )2 ⇒ 1 − e2 = e4 ⇒ e4 + e2 − 1 = 0

Illustration 29: Any ordinate MP of an ellipse meets the auxiliary circle in Q. Prove that the locus of the point of
intersection of the normal P and Q is the circle x2 + y 2 =(a + b)2 . (JEE MAIN)

Sol: Consider a point on the ellipse and find the intersection of the ordinate with the circle. Next find the intersection
of the normal at P and Q and eliminate the parameter θ .
x2 y2
Let P(acos θ, bsin θ) be any point on the ellipse = 1 , and let Q (acos θ, asin θ) be the corresponding point
+
a2 b2
on the auxiliary circle x2 + y 2 =
a2 . The equation of the normal at P (acos θ, bsin θ) to the ellipse is
ax sec θ − by cosec θ= a2 − b2  …(i)
The equation of the normal at Q (acos θ, asin θ) to the circle x2 + y 2 =
a2 is
=y x tan θ  …(ii)
Let (h , k) be the point of intersection of (i) and (ii). Then,
ahsec θ − bk cosecθ= a2 − b2  …(iii)
and, P(acos θ, bsin θ)  …(iv)

Eliminating θ from (iii) and (iv), we get

k2 h2
ah 1 + − bk 1 + = a2 − b2
h2 k2

⇒ (a − b) h2 + k 2 =a2 − b2 ⇒ h2 + k 2 =(a + b)2

Hence, the locus of (h, k) is x2 + y 2 =(a + b)2 .

Illustration 30: If the length of the major axis intercepted between the tangent and normal at a

sec θ cosec θ 1
 2π   2π  x2 y 2
point sec  θ +  cosec  θ +  1 on the ellipse 2 + 2 =1 is equal to the semi-major axis, prove that the
 3   3  a b
 2π   2π 
sec  θ −  cosec  θ −  1
 3   3 

e {sec θ(sec θ − 1)}1/2 .


eccentricity of the ellipse is given by= (JEE MAIN)
M a them a ti cs | 11.23

Sol: Obtain the points of intersection of the tangent and the normal and then use the distance formula.
x2 y2
The equation of the tangent and normal to the ellipse + 1 at the point P
=
a2 b2

sec θ cosec θ 1
 2π   2π 
sec  θ +  cosec  θ +  1 are given by
 3   3 
 2π   2π 
sec  θ −  cosec  θ −  1
 3   3 

x y
cos θ + sin θ =1  …(i)
a b

θ (a2 − b2 ) 
and, ax sec θ − by cosec= …(i) respectively.
Suppose (i) and (ii) meet the major axis i.e. y = 0 at Q and R respectively. Then, the coordinates of Q and R are given
by

 a2 − b2 
Q(asec θ,0) and R  cos θ,0  ∴ QR = a [Given]
 a 
 
a2 − b2
⇒ asec θ − cos θ = a ⇒ a2 − (a2 − b2 )cos
=2
θ a2 cos θ ⇒ a2 − a2e2 cos
=2
θ a2 cos θ
a
2
⇒ 1 − e2 cos2 θ = cos θ ⇒ e2 cos2 θ = 1 − cos θ ⇒ e= e {sec θ(sec θ − 1)}1/2
sec θ(sec θ − 1) ⇒=

x2 y2
Illustration 31: If ω is one of the angles between the normals to the ellipse + 1 at the points whose
=
a2 b2
π 2cot ω e2
eccentric angles are θ and + θ , then prove that = . (JEE ADVANCED)
2 sin2θ 1 − e2
Sol: Evaluate the equation of the normal at the two points and then use the formula of the angle between two lines.
x2 y2 π
The equation of the normal to the ellipse + 1 at the points whose eccentric angles are θ and
= + θ are
a2
b 2 2
ax sec θ − by cosecθ= a − b and, −ax cosecθ − by secθ = a2b2 respectively. Since ω is the angle between these
2 2

(a / b)tan θ + (a / b)cot θ
two normals, therefore, tan ω =
1 − (a2 / b2 )

ab(tan θ + cot θ) 2ab 2ab


⇒ tan ω = ⇒ tan ω = ⇒ tan ω =
2 2
2
b −a 2
(sin2θ)(b − a ) (a − b2 )sin2θ
2

2a2 1 − e2 2cot ω e2
⇒ tan ω = ⇒ =−
a2e2 sin2θ sin2θ 1 − e2

Illustration 32: If the tangent drawn at point (t2 ,2t) on the parabola y 2 = 4x is the same as the normal drawn at
a point ( 5 cos θ, 2sin θ) on the ellipse 4x2 + 5y 2 =
20 , find the values of t and θ . (JEE ADVANCED)

Sol: Write the equation for the tangent and normal in terms of the parameter. Compare the two equations to get
the values of t and θ .
1 1 . 2 4 | Ellipse

The equation of the tangent at (t2, 2t) to the parabola y2 = 4x is


 1
2ty = 2 (x + t) ⇒ ty =x + t2  ±2 3, ±   … (i)
 7

The equation of the normal at point ( 5 cos θ, 2sin θ) on the ellipse 4x2 + 5y 2 =
20 is
⇒ ( 5 sec θ)x − ( 2cosecθ)y − 1 = 
0 … (ii)
5 sec θ − 2cosec θ −1
It is given that (i) and (ii) represent the same line. Therefore,= = 2
1 −t t
2cosec θ 1 2 1
⇒ t= and t = − ⇒
= t cot θ and t =
− sin θ
5 sec θ 2cosec θ 5 2

2 1
⇒ cot θ = − sin θ ⇒ 4 cos θ = − 5 sin2 θ ⇒ 4 cos θ = − 5(1 − co s2 θ)
5 2

⇒ 5 cos2 θ − 4 cos θ − 5 = 0 ⇒ 5 cos2 θ − 5cos θ + cos θ − 5 = 0

⇒ 5 cos θ(cos θ − 5) + (cos θ − 5)= 0 ⇒ (cos θ − 5)( 5 cos θ + 1)= 0

 1 
θ cos−1  −
⇒=  [cosθ ≠ − 5]
 5

1 1 1 1 1
Putting cosθ = − in t =
− sin θ we get t =− 1− =−
5 2 2 5 5

 1  1
Hence,
= θ cos−1  −  and t = − .
 5 5

Illustration 33: The normal at a point P on the ellipse x2 + 4y 2 =16 meets the x-axis at Q. If M is the mid point of
the line segment PQ, then the locus of M intersects the latus rectums of the given ellipse at the points.
 (JEE ADVANCED)
 3 5 2   3 5 19   1  4 3 
(a)  ± , ±  (b)  ± ,±  (c)  ±2 3, ±  (d)  ±2 3, ± 
 7 7   2 4   7   7 
     

Sol: Put y = 0 in the equation of the normal to get the point Q in terms of θ . Get the locus of the mid-point as
required. In the last step solve the equation of the locus and the latus rectum.
x2 y 2
Equation of the ellipse is + 1
=
16 4
Y
Equation of the normal at P(4 cos θ, 2sin θ) to the ellipse is
θ 42 − 22 ⇒ 2x sec θ − y cosec θ = 6
4x sec θ − 2y cosec = P(4cos 2sin)
It meets x-axis at Q(3cos θ,0) . If (h, k) are the coordinates of M, then M
O Q X
4 cos θ + 3cos θ 2sin θ + 0
h = ,k
2 2

2h
⇒ =
cosθ ,=
sin θ k
7
2
4h2 x2 y Figure 11.16
⇒ + k2 =
1 Locus of M is ⇒ + 1.
=
49 (7 / 2)2 1
M a them a ti cs | 11.25

Latus rectum of the given ellipse is x =±ae =± 16 − 4 =±2 3

12 × 4 1 1
So locus of M meets the latus rectum at points for which y 2 =1− = ⇒y=±
49 49 7
 1
And hence the required points are  ±2 3, ±  .
 7

13. CHORD BISECTED AT A GIVEN POINT


Q(x2, y2)
2 2
x y
The equation of the chord of the ellipse + 1,
=
2
a b2 P (x1, y1)
whose mid point is (x1 , y1 ) is T = S1
xx1 yy1
where T = + − 1 R(x3, y3)
2
a b2
Figure 11.17
x12 y12
S1 = + −1 .
a2 b2

x2 y2
Illustration 34: Find the locus of the midpoint of a focal chord of the ellipse + 1 .
= (JEE MAIN)
a2 b2
Sol: In the equation T = S1, substitute x = ae and y = 0.
x2 y2
Let (h, k) be the midpoint of a focal chord of the ellipse + 1 . Then, the equation of the chord is
=
2 2
a2 b2
hx ky h k
+ −1 = + − 1 [Using : T = S1]
2 2 2
a b a b2

hx ky h2 k2
or, + = + It passes through the focus (ae, 0) of the ellipse.
a2 b2 a2 b2

hae h2 k2 xe x2 y 2
∴ +0= + . Hence, the locus of (h, k) is = + .
a2 a2 b2 a a2 b2

x2 y2
Illustration 35: Find the locus of the mid-point of the normal chords of the ellipse + 1.
=
a2 b2
 (JEE ADVANCED)
Sol: Similar to the previous question.
Let (h, k) be the mid point of a normal chord of the given ellipse. Then, its equation is
hx ky h2 k2
+ −1 = + − 1 [Using: T = S1]
a2 b2 a2 b2

hx ky h2 k2
or + = +  …(i)
a2 b2 a2 b2

If (i) is a normal chord, then it must be of the form

ax sec θ − by cosecθ= a2 − b2  …(ii)

h2 k2
+
h k 2
b2
∴ = = a
a3 sec θ −b3 cosecθ a2 − b2
1 1 . 2 6 | Ellipse

a3  h2 k 2  −b3  h2 k 2 
⇒ cos θ
=  =
+  , sin θ  + 
h(a2 − b2 )  a2 b2  k(a2 − b2 )  a2 b2 

Eliminating θ from the above relations, we get


2 2 2
a6  h2 k 2  b6  h2 k 2   a6 b6   h2 k 2 
 +  +  +  1
= ⇒ +  +  =(a2 − b2 )
h2 (a2 − b2 )2  a2 b2  k 2 (a2 − b2 )2  a2 b2   h2 k 2   a2 b2 
  
2
 a6 b6   x2 y 2 
Hence, the locus of (h, k) is  +  +  =(a2 − b2 )2 .
 x2 y 2   a2 b2 
  

14. DIAMETERS
Definition: A chord through the centre of an ellipse is called a diameter of the ellipse.
x2 y2
(y mx + c) of slope m of the ellipse
The equation of the diameter bisecting the chords = + 1
=
a2 b2
b2
is y = − x , which is passing through (0, 0)
a2m
Y

y=mx+c

X’ X

2
-b
Diameter y = x
a2m
Y’
Figure 11.18

Conjugate diameter: Two diameters of an ellipse are said to be conjugate diameters if each bisects all chords
parallel to the other. The coordinates of the four extremities of two conjugate diameters are
P(acos φ, bsin φ) ; P'( −acos φ, − bsin φ)

Q( −asin φ, bcos φ) ; Q '( −acos φ, − bsin φ)


Y

Q A
B P
o
90
 X
X’
C
P’ Q’

A’ B’

Y’
Figure 11.19
M a them a ti cs | 11.27

−b2
If y = m1 x and y = m2 x are two conjugate diameters of an ellipse, then m1m2 = .
a2
(a) Properties of diameters:
(i) The tangent at the extremity of any diameter is parallel to the chords it bisects or parallel to the conjugate
diameter.
(ii) The tangents at the ends of any chord meets on the diameter which bisects the chord.

(b) Properties of conjugate diameters:


(i) The eccentric angles of the ends of a pair of conjugate diameters of an ellipse differ by a right angle, i.e.,
π
φ − φ' =
2 (a cos’, b sin’)
D P(a cos, b sin)

A’ A
C

P’ D’
Figure 11.20

(ii) The sum of the squares of any two conjugate semi-diameters of an ellipse is constant and equal to the
sum of the squares of the semi axes of the ellipse i.e., CP2 + CD2 =a2 + b2 .

D P(a cos, b sin)

A’ A
S C S’

P’ D’

Figure 11.21

(iii) The product of the focal distances of a point on an ellipse is equal to the square of the semi-diameter
which is conjugate to the diameter through the point i.e., SP.S'P = CD2 .
(iv) The tangents at the extremities of a pair of conjugate diameters form a parallelogram whose area is
constant and equal to the product of the axes i.e., Area of parallelogram = (2a)(2b) = Area of rectangle
contained under major and minor axes.
Y
Q
M
D P
R’ R
X’ X
C
P’ D’
Q’
Y’

Figure 11.22

(v) The polar of any point with respect to an ellipse is parallel to the diameter to the one on which the point
lies. Hence obtain the equation of the chord whose mid point is ( x1 , y1 ) , i.e., chord is T = S1 .
(vi) Major and minor axes of ellipse is also a pair of conjugate diameters.
1 1 . 2 8 | Ellipse

(c) Equi-conjugate diameters: Two conjugate diameters are called equi-conjugate, if their lengths are equal i.e.,
(CP)2 = (CD)2 .

(a2 + b2 )
∴(CP) = (CD) = for equi-conjugate diameters.
2

x2 y2
Illustration 36: If PCP’ and DCD’ form a pair of conjugate diameters of the ellipse + 1 and R is any point
=
a2 b2
c2 , then prove that PR 2 + DR 2 + P'R 2 + D'R 2 = 2(a2 + b2 + 2c2 ) .
on the circle x2 + y 2 = (JEE MAIN)

Sol: Using the definition of conjugate diameters, get the coordinates of the point P, P’, Q and Q’. Starting from the
L.H.S. prove the R.H.S.
Let R(h, k) be any point on the circle x2 + y 2 =
c2 . Then h2 + k 2 =
c2  …(i)
Since PCP’ and DCD’ form a pair of conjugate diameters, the coordinates of the extremities are:
P(acos θ, bsin θ) , P'( −acos θ, − bsin θ) D( −asin θ, bcos θ) , D'(asin θ, − bcos θ)

∴ PR 2 + DR 2 + P'R 2 + D'R 2 = (h − acos θ)2 + (k − bsin θ)2 + (h + asin θ)2 + (k − bcos θ)2

 +(h + acos θ)2 + (k + bsin θ)2 + (h − asin θ)2 + (k + bcos θ)2

= 4(h2 + k 2 ) + 2a2 + 2b2

= 2a2 + 2b2 + 4c2 [Using (i)]

= 2(a2 + b2 + 2c2 )

x2 y2
Illustration 37: CP and CD are conjugate semi-diameters of the ellipse + 1.
=
a2 b2
x2 y21
Show that the locus of the mid-point of PD is the ellipse + = . (JEE MAIN)
a2
b22

Sol: Consider two points which lie on two conjugate diameters. Find the mid point of these two points and eliminate
the parameter θ to get the locus of the mid point.
Let P (acosθ, bsinθ), D (-asinθ, bcosθ) and (h, k) be the mid-point of PD. Then,
2h acos θ − asin θ and =
= 2k b sin θ + b cos θ

2h 2k 4h2 4k 2
⇒ = cos θ − sin θ and = sin θ + cos θ ⇒ + = (cos θ − sin θ)2 + (sin θ + cos θ)2
a b a2 b2

h2 k2 1 x2 y 2 1
⇒ + = . Hence, the locus of (h, k) is ⇒ + =.
a2 b2 2 a2 b2 2

Illustration 38: If y = x and 3y + 2x =


0 are the equations of a pair of conjugate diameters of an ellipse, then the
eccentricity of the ellipse is  (JEE MAIN)

2 1 1 2
(a) (b) (c) (d)
3 3 2 5

Sol: Use the condition of conjugacy of diameters in an ellipse to find the eccentricity.
x2 y2 2
Let the equation of the ellipse be + 1 . Slope of the given diameters are m1 = 1 , m2 = − .
=
a2
b2 3
M a them a ti cs | 11.29

2 b2
⇒ m1m2 =− =− [Using the condition of conjugacy of two diameters]
3 a2
2 1 1
3b2 = 2a2 ⇒ 3a2 (1 − e2 ) = 2a2 ⇒ 1 − e2 = ⇒ e2 = ⇒e=
3 3 3

Illustration 39: Show that the locus of the point of intersection of tangents at the end-point of the conjugate
x2 y2
diameters of the ellipse + 1 is another ellipse of the same eccentricity. 
= (JEE ADVANCED)
a2 b2
Sol: Using two points at the end points of the conjugate diameters of an ellipse, write the equation of the tangent.
Solve the two equations to eliminate the parameter θ .

x2 y2
Let CP and CD be two conjugate semi-diameters of the ellipse = 1 . Then, the eccentric angles of P and D
+
a2 b2
π
are θ and + θ respectively. So, the coordinates of P and D are (acos θ, bsin θ) and ( −asin θ, bcos θ) respectively.
2
The equation of the tangents at P and D are
x y
cos θ + sin θ =1  …(i)
a b

−x y
and sin θ + cos θ =1  …(ii)
a b
h k −h k
Let (h, k) be the point of intersection (i) and (ii). Then, cos θ + sin θ =1 and sin θ + cos θ =1
a b a b
2 2
h k   h k  h2 k 2
⇒  cos θ + sin θ  +  − sin θ + cos θ  = 1 + 1 ⇒ + = 2
a b   a b  a2 b2

h2 k2
Hence, the locus of (h, k) is + 2 which represents an ellipse of eccentricity e, given by
=
a2 b2

2b2 b2
e1 = 1 − =1−
2a2 a2
x2 y2
Clearly, it is same as the eccentricity of + 1.
=
a2 b2

Illustration 40: If α and β are the angles subtended by the major axis of an ellipse at the extremities of a pair of
conjugate diameters, prove that cot2 α + cot2 β = constant. (JEE MAIN)

Sol: Using the co-ordinates of the co-ordinates of the end points of a diameter, find the angle subtended by the
major axis. Repeat the same process for the other end of the diameter. Then find the value of cot2 α + cot2 β and
prove that it is independent of the parameter.
x2 y 2
Let CP and CD be a pair of conjugate semi-diameters of the ellipse + 1 . Then, the coordinates of P and D
=
are (acos θ, bsin θ) and ( −asin θ, bcos θ) respectively. a2 b2

bsin θ b θ
m1 = Slope of AP = = − cot
acos θ − a a 2

bsin θ b θ
m2 = Slope of A’P = = tan
acos θ + a a 2
1 1 . 3 0 | Ellipse

m1 − m2 −(b / a)cot(θ / 2) − (b / a)tan(θ / 2)


∴ tan α = ⇒ tan α =
1 + m1m2 1 − (b2 / a2 )

ab
 θ θ  2ab  1
⇒ tan α
=  cot + tan  ⇒ tan α = 2 
2
a −b  2
2 2  a − b2  sin θ

π   2ab  1
Replacing θ by  + θ  , we get tan β = 
2   a2 − b2  cos θ
2 2
2
 a2 − b2 
2
 a2 − b2 
∴ cot α
= + cot β   (sin2 θ=
+ cos2 θ) =  Constant.
 2ab   2ab 
   

Illustration 41: Find the locus of the points of intersection of normals at two points on an ellipse which are
extremities of conjugate diameters.  (JEE MAIN)

Sol: Solve the equation of the normal at the extremities of conjugate diameters.

x2 y2
Let PP’ and QQ’ be two conjugate diameters of the ellipse + 1.
=
a2 b2
π
Let the eccentric angle of the point P be ' φ ' . Then the eccentric angle of Q is ' φ + ' .
2
∴ = P (acos φ, bsin φ) Q’+ 
2
D P’’
  π  π  
=Q acos  φ +  , bsin  φ +  
  2  2  
O
x − acos φ y − bsin φ
The equation of the normal =
at P (acos φ, bsin φ) is = P’ D’
2
(acos φ) / a (bsin φ) / b2
Figure 11.23
or ax sec φ − bycosec φ= a2 − b2  …(i)
Similarly, the equation of the normal at Q is

 π  π 2 2
ax sec  φ +  − bycosec  φ + = a −b
 2   2 

or −ax co secφ − by secφ= a2 − b2  …(ii)

The locus of the point of intersection of (i) and (ii) is obtained by eliminating φ from them. Now we have
ax sec φ − by cosec φ − (a2 − b2 ) = 0

by secφ + ax cosec φ + (a2 − b2 ) = 0

By cross multiplication,

sec φ cosec φ a2 − b2
= =
−by + ax −by − ax a2 x2 + b2 y 2

a2 x2 + b2 y 2 1
∴ cos φ = .
ax − by a − b2
2

a2 x2 + b2 y 2 1
sin φ = .
−(ax + by) a − b2
2
M a them a ti cs | 11.31

Squaring and adding,


2
(a2 x2 + b2 y 2 )2  1 1   a2 x2 + b2 y 2  2(a2 x2 + b2 y 2 )
1  +  =  .
(a2 − b2 )2  (ax − by)2 (ax + by)2   a2 − b2  (a2 x2 − b2 y 2 )2

⇒ 2(a2 x2 + b2 y 2 )3 =
(a2 − b2 )2 .(a2 x2 − b2 y 2 )2 .

15. POLE AND POLAR


Let P(x1, y1) be any point inside or outside the ellipse. A chord through P intersects the ellipse at A and B respectively.
If tangents to the ellipse at A and B meet at Q(h, k) then locus of Q is called polar of P with respect to the ellipse
and point P is called the pole.
Q(h,k) A Q(h,k) A
A’
Polar P(x₁, y₁) B Polar
Pole A’
B B’
B’ P(x₁, y₁)
Pole Q’

(a) (b)

Figure 11.24

Note: If the pole lies outside the ellipse then the polar passes through the ellipse. If the pole lies inside the ellipse
then the polar lies completely outside the ellipse. If the pole lies on the ellipse then the polar becomes the same
as the tangent.
x2 y 2 xx1 yy1
Equation of polar: Equation of polar of the point (x1, y1) with respect to ellipse + =1 is given by 2 + 2 = 1,
a2 b2 a b
i.e., T = 0
x2 y2  −a2l −b2m 
Coordinates of Pole: The pole of the line lx + my + n =0 with respect to ellipse + 1 is P 
= , .
a2 b2  n n 

Properties of pole and polar:

(a) If the polar of P (x1 , y1 ) passes through Q(x2 , y 2 ) , then the polar of Q(x2 , y 2 ) goes through P (x1 , y1 ) and
x1 x2 y1 y 2
such points are said to be the conjugate points. Condition for conjugate points is + 1.
=
2
a b2
(b) If the pole of line l1 x + m1 y + n1 =
0 lies on another line l2 x + m2 y + n2 =
0 , then the pole of the second line will
lie on the first and such lines are said to be conjugate lines.
(c) Pole of a given line is the same as the point of intersection of tangents at its extremities.
(d) Polar of focus is directrix.

x2 y2
Illustration 42: Obtain the locus of poles of tangents to the ellipse + 1 with respect to concentric ellipse
=
a2 b2
x2 y2
+ 1 . 
= (JEE MAIN)
α2 β2
x2 y 2
Sol: Taking a point (h , k), write the equation of the polar w.r.t. the ellipse + 1 . In the next step put the
=
condition for polar to be the tangent to the other given ellipse. α2 β2
1 1 . 3 2 | Ellipse

hx ky  β2h  β2
The equation of the polar is + 1⇒ y=
= −  x+
α2 β2  α2k  k
 
2 2
x2 y2  β2   −β2h 
This touches + 1 . Therefore,
= =   a2   + b2
a2 b 2  k   2 
   αk 

β4 β4h2 a2h2 b2k 2


⇒ = a2 + b2 ⇒ + = 1
k2 α 4k 2 α4 β4

a2 x2 b2 y 2
Hence, the locus of (h, k) is ⇒ + 1.
=
α4 β4

x2 y2
Illustration 43: Find the locus of the mid-points of the chords of the ellipse = 1 whose poles are on the +
a2 b2
auxiliary circle or the tangents at the extremities of which intersect on the auxiliary circle. (JEE ADVANCED)

Sol: Compare the equation of the chord and the tangent to get the point which lies on the auxiliary circle. Substitute
the point in the equation of the circle to get the required locus.
Let (h, k) be the mid-point of a chord of the ellipse. Then, its equation is
hx ky h2 k2
+ = +  …(i)
a2 b2 a2 b2
x2 y2
Let (x1, y1) be its pole with respect to the ellipse + 1 . Then, the equation of the polar is
=
a2 b2
xx1 yy1
+ 1 
= …(ii)
2
a b2

Clearly, (i) and (ii) represent the same line. Therefore,


x1 y1 1 h k
= = ⇒ x1 = , y1 =
h k 2 2
(h / a ) + (k / b ) 2 2 2 2 2
(h / a ) + (k / b ) 2
(h / a ) + (k 2 / b2 )
2 2

2
 h2 k 2 
It is given that (x1, y1) lies on auxiliary circle. Therefore x12 + y12 2 2
= a ⇒ h +k = a  + 2
 . 2
 a2 b2 
 
2
 x2 y 2 
Hence the locus of (h, k) is x2 + y=
2
a2  +  .
 a2 b2 
 

16. SUBTANGENT AND SUBNORMAL


Y
Let the tangent and normal at P(x1, y1) meet the x-axis at A and B
respectively. Length of subtangent at P(x1, y1) to the ellipse A
P(x₁, y₁)
x2 y2 a2
+ 1 is DA = CA − CD = − x1
=
a2 b2 x1 X’ X
C B D A
2 2
x y
Length of sub-normal at P(x1 , y1 ) to the ellipse + 1 is
=
2
a b2
 b2  b2
BD =CD − CB =x1 −  x1 − x  = x1 =(1 − e2 )x 1 . Y’
 a2 1 2
  a Figure 11.25
M a them a ti cs | 11.33

PLANCESS CONCEPTS

Misconception: As there is no y1 term involved in the above results, don’t think that the lengths are
x12 y12
independent of y1. Always remember that + 1
=
a2 b2
Vaibhav Krishnan (JEE 2009, AIR 22)

PROBLEM-SOLVING TACTICS

x2 y2 m2 (a2 − b2 )2
•• If the line=
y mx + c is a normal to the ellipse + 1 then c2 =
= is the condition of normality
a2 b2 a2 + b2m2
of the line to the ellipse.

••  he tangent and normal at any point of an ellipse bisect the external and angles between the focal
T
radii to the point. It follows from the above property that if an incident light ray passing through the
focus (S) strikes the concave side of the ellipse, then the reflected ray will pass through the other
focus (S’).

•• If SM and S’M’ are perpendicular from the foci upon the tangent at any point of the ellipse, then SM. S’M’ = b2
and M, M’ lie on the auxiliary circle.
x2 y 2
•• If the tangent at any point P on the ellipse + 1 meets the major axis in T and minor axis in T’, then CN.
=
2 2
CT = a , CN’. CT’ = b
2 2 a b

Where N and N’ are the feet of the perpendicular from P on the respective axis.

•• If SM and S’ M’ are perpendicular from the foci S and S’ respectively upon a tangent to the ellipse, then CM
and CM’ are parallel to S’P and SP respectively.

FORMULAE SHEET

 1. The general equation of second order ax2 + 2hxy + by 2 + 2gx + 2fy + c =0 represents an ellipse
 a h g
 
if ∆ ≠ 0, h2 < ab. where  ∆ = h b f 
 g f c 

 2. The sum of the focal distance of any point on an ellipse is a constant and is equal to the length of the major
axis of the ellipse i.e. SP + S’P = 2a.
x2 y2
 3. Standard equation of an ellipse is + 1
=
a2 b2
Where a = length of semi-major axis,
b = length of semi-minor axis
1 1 . 3 4 | Ellipse

 4.

 x2 y 2 
Ellipse  2 + 2 = 1
Imp. Terms  a b 

For a > b For b > a


Centre (0, 0) (0, 0)
Vertices (±a, 0) (0, ±b)
Length of major axis 2a 2b
Length of minor axis 2b 2a
Foci (±ae , 0) (0 , ± be)
Equation of directrices x = ±a/e y = ±b/e
Relation in a, b and e b2 = a2 (1 – e2) a2 = b2 (1 – e2)
Length of latus rectum 2b2 2a2
a b

Ends of latus rectum  b2   a2 


 ±ae, ±   ± , ± be 
 a   b 
  

Parametric equations (a cos φ, b sinφ) (a cos φ, b sin φ)

(0 ≤ φ < 2π)
Focal radii SP = a – ex1 SP = b –ey1
S’P = a + ex1 S’P = b + ey1
Sum of focal radii (SP + S’P =) 2a 2b
Distance between foci 2ae 2be
Distance between directrices 2a/e 2b/e
Tangents at the vertices x = –a, x = a y = b, y = –b

 5. The equations=x acos φ,


= y bsin φ taken together are called the parametric equations of the ellipse

x2 y2
+ 1 , where φ is the parameter.
=
a2 b2

 6. (i) If the centre of the ellipse is at (h, k) and the axes are parallel to the coordinate axes, then its equation is

(x − h)2 (y − k)2
+ 1.
=
a2 b2
(lx + my + n)2 (mx − ly + p)2
(ii) If the equation of the ellipse is + 1 , where lx + my + n =
= 0 and mx − ly + p =0
a2 b2
lx + my + n mx − ly + p
are perpendicular lines. Substitute = X and = Y , to put the equation in the standard form.
2 2
l +m l2 + m2
x2 y2
 7. If P(acos α, bsin α ) and Q(acos β, bsin β) are any two points on the ellipse + 1 , then the equation of a
=
a2 b2
x α+β y α+β α −β
chord joining these two points is cos   + sin  =cos  .
a  2  b  2   2 
M a them a ti cs | 11.35

x2 y 2 x2 y2
 8. The point P(x1 , y1 ) lies outside, on, or inside the ellipse + 1 according to 1 + 1 − 1 > 0, =
= 0 or < 0
2 2 2 2
respectively. a b a b

x2 y2
 9. The line =
y mx + c intersects the ellipse + 1 on two distinct points if a2m2 + b2 > c2 , on one point
=
2 2
a b
2 x2 y 2
if=c a m + b and does not intersect if a m2 + b2 < c2 . For an ellipse
2 2 2
+ 2
1 , the auxiliary circle is
=
x2 + y 2 =a2 . a2 b2

x2 y2 xx1 yy1
 10. The equation of the tangent to the ellipse + 1 at the point (x1 , y1 ) is
= + 1 . The equation
=
2 2 2
a b a b2

mx ± a2m2 + b2 and the point of contact is


of tangent to the ellipse having its slope equal to m is y =
 ±a2m   b2 x y
 ,  . The equation of the tangent at any point (acos φ, bsin φ) is cos φ + sin φ =1 .
 2 2 2  a b
 a m +b a2m2 + b2 
2 2
x y
Point of intersection of the tangents to the ellipse + 1 at the points P(acos θ1 , bsin θ1 ) ,
=
2
a b2
 acos ( (θ1 + θ2 ) / 2 ) b sin ( (θ1 + θ2 ) / 2 ) 
and Q(acos θ2 , bsin θ2 ) is  , .
 cos ( (θ − θ ) / 2 ) cos ( (θ − θ ) / 2 ) 
 1 2 1 2 

 11. Equation of pair of tangents drawn from an outside point P(x1 , y1 ) is SS1 = T2 .

x2 y2
 12. For an ellipse + 1 , the equation of director circle is x2 + y 2 = a2 + b2 .
=
2 2
a b
x2 y2 a2 x b2 y
 13. The equation of normal to the ellipse + 1 at the point (x1 , y1 ) is
= − =a2 − b2 . The equation of
a2 b 2 x1 y 1
2 2
x y
normal to the ellipse + ) a2 − b2 .
1 at any point (acos φ, bsin φ) is (ax sec φ − bycosecφ=
=
2 2
a b
x2 y2
 14. If m is the slope of the normal to the ellipse + 1 , then the equation of the normal is
=
a2 b2
m(a2 − b2 )  ±a2 ±mb2 
y mx ±
= . The co-ordinates of the point of contact are  , .
 2 
a2 + b2m2  a +b m
2 2
a2 + b2m2 

 15. The properties of conormal points are


x2 y 2
(i) Property 1: The sum of the eccentric angles of the co-normal points on the ellipse + 1 is an odd
=
multiple of π . a2 b2

x2 y 2
(ii) Property 2: If θ1 , θ2 and θ3 are eccentric angles of three co-normal points on the ellipse + 1,
=
then sin(θ1 + θ2 ) + sin(θ2 + θ3 ) + sin(θ3 + θ1 ) = 0 . a2 b2

(iii) Property 3: Co-normal points lie on a fixed curve called an Apollonian Rectangular

(
Hyperbola a2 − b2 xy + b2kx − a2hy =
0 )
(iv) Property 4: If the normal at four points P(x1y1), Q(x2y2), R(x3y3) and S(x4y4) on the

x2 y2 1 1 1 1 
ellipse + 1 are concurrent, then (x1 + x2 + x3 + x 4 )  +
=
x
+ + 4.
 =
a2 b2  1 x 2 x3 x 4 
1 1 . 3 6 | Ellipse

 16. If SM and S’M’ are perpendiculars from the foci upon the tangent at any point of the ellipse, then
SM × S'M' = b2 and M, M’ lie on the auxiliary circle.

x2 y2
 17. If the tangent at any point P on the ellipse + 1 meets the major axis at T and minor axis at T’, then
=
a2 b2
CN × CT = a2, CN’ × CT’ = b2. Where N and N’ are the feet of the perpendiculars from P on the respectively axis.

x2 y2
 18. The equation of the chord of the ellipse + 1 , whose mid point is (x1 , y1 ) , is T = S1 .
=
a2 b2
x2 y2 xx1 yy1
 19. The chord of contact from a point P(x1 , y1 ) to an ellipse + 1 is T = 0 is
= + 1.
=
2 2 2
a b a b2
x2 y2
 20. The equation of the diameter bisecting the chords =
(y mx + c) of slope m of the ellipse + 1
=
a2 b2
b2
is y = − x.
a2m
x2 y2 −b2
 21. If m1 and m2 are the slopes of two conjugate diameters of an ellipse + 1 , then m1m2 =
= .
a2 b2 a2

 22. The eccentric angle of the ends of a pair of conjugate diameters of an ellipse differ by a right angle,
π
i.e., φ − φ ' = .
2
 23. The sum of the squares of any two conjugate semi-diameters of an ellipse is constant and is equal to the sum
of the squares of the semi axes of the ellipse i.e., CP2 + CD2 = a2 + b2.

 24. The product of the focal distances of a point on an ellipse is equal to the square of the semi-diameter which
is conjugate to the diameter through the point i.e., SP × S'P = CD2.

 25. The tangents at the extremities of a pair of conjugate diameters form a parallelogram whose area is constant
and equal to the product of the axes.
i.e. Area of the parallelogram = (2a)(2b) = Area of the rectangle contained under major and minor axes.

 26. Two conjugate diameters are called equi-conjugate, if their lengths are equal i.e., (CP)2 = (CD)2

(a2 + b2 )
∴(CP) = (CD) = for equi-conjugate diameters.
2
x2 y2  −a2l −b2m 
 27. Equation of the polar of the point (x1, y1) w.r.t. an ellipse + 1 is P 
= , .
a2 b2  n n 

x2 y2  −a2l −b2m 
 28. The pole of the line lx + my + n =0 with respect to the ellipse + 1 is P 
= , .
a2 b2  n n 

x1 x2 y1 y 2
 29. Condition for a conjugate point is + 1.
=
a2 b2
x2 y2 a2
 30. The length of a sub tangent at P(x1 , y1 ) to the ellipse + 1 is
= − x1 .
a2 b2 x1

x2 y2 b2
 31. The length of a sub normal at P(x1 , y1 ) to the ellipse + 1 is
= x1= (1 − e2 )x1 .
a2 b2 a2
M a them a ti cs | 11.37

Solved Examples

JEE Main/Boards The equation of directrices are given by X = ±


a
e
Example 1: Find the centre, the eccentricity, the foci, 9
i.e. x + 1 =±
the directrices and the lengths and the equations of the 2
axes of the ellipse 7 11
i.e. x = ,x= −
2 2
5x + 9y + 10x − 36y − 4 =
0 2 2
The equation of the axes are given by
Sol: Rewrite the equation in the standard form and
compare them to get the centre, eccentricity etc. X = 0, Y = 0
i.e. x + 1= 0, y − 2= 0
5x2 + 9y 2 + 10x − 36y − 4 =,
0 the given equation can
be written as i.e. x =
−1, y =
2

5(x2 + 2x) + 9(y 2 − 4y) =


4 Length of the axes being 2a, 2b

5(x + 1)2 + 9(y − 2)2 =


45 i.e, 6, 2 5 .

(x + 1)2 (y − 2)2
+ 1
=
9 5 Example 2: If the chord through point θ1 and θ2 on an
Shift the origin to O' ≡ ( −1,2) x2 y2
ellipse + 1 intersects the major axis at (d, 0)
=
∴ X= x + 1 ; Y= y − 2 a2 b2
θ1 θ2 d−a
X2 Y2 prove that tan tan = .
∴ + 1
= …(i) 2 2 d+a
9 5
Sol: Substitute the point (d , 0) in the equation of the
This is in standard form
chord to prove the given result.
∴ =a 3,=
b 5
Equation of the chord joining the points θ1 and θ2 is
2 2
a −b 4 2 x  θ + θ2  y  θ1 + θ2   θ − θ2 
e2
∴= = ⇒e= cos  1 cos  1
a2 9 3  + sin   = 
a  2  b  2   2 
2 a 9 Since (d, 0) lies on it
Also ae = 3 . = 2 and = .
3 e 2
P(1)
Now for an ellipse in the standard form we have Centre
a R(d,)
≡ (0, 0); foci ≡ (±ae, 0); directrices x = ± ; axes x = 0,
e O
y = 0, length of major axis = 2a, length of minor axis Q(1)
= 2b.
Now for (i) the centre is given by X = 0, Y = 0
d  θ + θ2   θ1 − θ2 
⇒ x + 1= 0, y − 2= 0 ∴ cos  1  = cos  
a  2   2 
ax by
i.e. Centre + c
= cos ( (θ1 − θ2 ) / 2 )
3 4 d
=
Foci are given by X =
±ae, Y =
0 cos ( (θ1 + θ2 ) / 2 ) a

i.e. x + 1 =±2 and y − 2 =0 Applying componendo and dividendo, we get

i.e. = y 2 and x =
x 1,= −3, y =
2 d − a cos ( (θ1 − θ2 ) / 2 ) − cos ( (θ1 + θ2 ) / 2 )
=
∴ Foci ≡ (1,2); ( −3, 2) d + a cos ( (θ1 − θ2 ) / 2 ) + cos ( (θ1 + θ2 ) / 2 )
1 1 . 3 8 | Ellipse

2sin(θ1 / 2)sin(θ2 / 2)
=
2cos(θ1 / 2)cos(θ2 / 2)
θ θ
tan 1 tan 2
2 2
Equation of normal at Q ( )
5 cos φ,2sin φ is

cos φ x 5 sin φ − sin φ cos φ 


2y= …(ii)
2 2
x y
Equation (i) and (ii) represent the same line. Comparing
Example 3: A tangent to the ellipse = 1 +
a b2 the coefficients in equations (i) and (ii).
2

touches it at the point P in the first quadrant and meets t 1 t2


the x and y axes in A and B respectively. If P divides AB = =
2cos φ 5 sin φ − sin φ.cos φ
in the ratio 3 : 1, find the equation of the tangent at P.
2 cos φ
Sol: Consider a point in the parametric form and obtain ⇒ t = cot φ, t2 = −
5 5
the points A and B. Now use the condition that the
point P divides AB in the ratio 3:1. 4 cos φ
cot2 φ = −
Let P ≡ (acos θ,bsin θ) : 5 5
π
 4 cos φ 
0<θ<  …(i) ⇒ cos φ  + 5 =
0
 sin2 φ 
2  
Equation of the tangent at P(θ) is (cos φ ≠ 0 t ≠ 0)
x y
cos θ + sin θ =1 (x − 5)2 y 2
a b ⇒ + 1
=
9 25
y


0,
b
 B
⇒ ( )
5 1 − cos2 φ + 4 cos φ =0
sin 1
P 1
3
R(d,) A ⇒ cos φ = −
x 5

cos ,0
a  1 
∴ φ= 2nπ ± cos−1  −  where 0 ≤ φ ≤ 2π .
 5
∴ Corresponding values of t are given by
 a   b  cos φ 1
∴A≡ ,0  and B ≡  0,  t2 =
− = .
 cos θ   sin θ 5
5
Now P divides segment AB in the ratio 3 : 1 1
∴ t= ±
 a 3b  5
∴ P≡ ,   …(ii)
 4 cos θ 4 sin θ 
Example 5: Show that the sum of the squares of the
By (i) and (ii), we have x2 y2
perpendiculars on any tangent to += 1 from 2
1 3 a2 b2
cos
= θ ; sin
= θ
∴ 2 2 points on the minor axis, each of which is at a distance

∴ Equation of tangent at P is bx + a 3y =
2ab . a2 − b2 from the centre, is 2a2 .

Example 4: If the tangent drawn at a point (t2 ,2t); t ≠ 0 Sol: Use the standard equation of a tangent in terms of
m and then proceed accordingly,
on the parabola y 2 = 4x is the same as the normal
drawn at a point ( 5 cos φ,2sin φ ) on the ellipse The general equation of a tangent to the ellipse is

4x2 + 5y 2 =
20 , find the value of t and φ . mx ± a2m2 + b2 
y= …(i)

Sol: Write the equation of the tangent and the normal Let the points on the minor axis be P(0,ae) and
using ‘t’ and ‘ φ ’ and compare.
b2 a2 (1 − e2 )
Q(0, − ae) as =
2 2
Equation of the tangent at P(t ,2t) to y = 4x is
Length of the perpendicular from P on (i) is
yt= x + t2  …(i)
M a them a ti cs | 11.39

ae ± a2m2 + b2 mx ± a2m2 + b2 . It passes through (1, 2)


y=
P1 = 5 2 5
( 2 − m)
2
1 + m2 ∴ = m +
3 2

−ae ± a2m2 + b2 ⇒ 4m2 + 24m − 9 =0


Similarly, P2 = Angle between the tangents is
1 + m2
m1 − m2 4 36 + 9 12
2 tan θ = = =−
P12 + P22
Hence, = 2 2 2 2
{a e + (a m + b )}2 1 + m1m2 −5 5
1 + m2
 12 
2 2 2 2 2 2 2 θ tan−1  −
∴= 
= {(a − b ) + a m =
+ b } 2a  5
1 + m2
Example 8: The locus of the foot of the perpendicular
Example 6: Find the equation of the ellipse having its drawn from the centre to any tangent to the ellipse
centre at the point (2,–3), one focus at (3,–3) and one x2 y2
vertex at (4, –3). + 1 is
=
a2 b2
Sol: Use the basic knowledge of the major axis, centre (A) A circle (B) An ellipse
and focus to get the equation of the ellipse.
(C) A hyperbola (D) None of these.
C ≡ (2, −3) , S ≡ (3, −3) and A ≡ (4, −3)
Sol: Find the foot of the perpendicular from the centre
Now, CA = 2
to any tangent and eliminate the parameter.
∴ a=2
Equation of a tangent to the ellipse is
Again CS = 1
1 1 mx ± a2m2 + b2 
y= …(i)
∴ ae = 1 ⇒e= =
a 2 Equation of the line through the centre (0, 0)
We know that perpendicular to (i) is
2
b= a2 − a2e2  −1 
y =  x  …(ii)
m
⇒ b= 3
Eliminating m from (i) and (ii) we get the required locus
∴ Equation of ellipse is of the foot of the perpendicular as
( x − 2) ( y + 3)
2 2
x2 x2
+ 1
= − ± a2
y= + b2
22
( 3)
2 y y2

⇒ (x2 + y 2 )2 = a2 x2 + b2 y 2
⇒ 3 ( x − 2 ) + 4 ( y + 3) =
2 2
12
which does not represent a circle, an ellipse or a
⇒ 3x2 + 4y 2 − 12x + 24y + 36 =
0 hyperbola.

Example 7: Show that the angle between pair of Example 9: The ellipse x2 + 4y 2 = 4 is inscribed in a
tangents drawn to the ellipse 3x2 + 2y 2 =
5 from the rectangle aligned with the coordinate axes, which in
  turn is inscribed in another ellipse that passes through
12
point (1, 2) is tan−1  − . the point (4, 0). The equation of the ellipse is
 5
(A) 4x2 + 48y 2 =
48 (B) 4x2 + 6y 2 =
48
Sol: Starting from the standard equation of a tangent
in terms of m, satisfy the point (1 ,2) and get the values (C) x2 + 16y 2 =
16 (D) x2 + 12y 2 =
16
of m. Using the value of m, find the angle between the
two tangents. Let the equation of the tangents be Sol: Consider the standard equation of the ellipse. Use
the two points given in the question to find the value
of ‘a’ and ‘b’.
1 1 . 4 0 | Ellipse

x2 y2 ⇒ 10(h2 + k 2 − 6h − 8k + 25) = (h + 3k − 3)2


Let the equation of the required ellipse be + 1.
=
2 2
a b Locus of (h, k) is
Given that it passes through (4, 0)
9x2 + y 2 − 6xy − 54x − 62y + 241 =
0.
⇒ a=4
It also passes through (2, 1), one of the vertex of
rectangle. Example 11: If an ellipse slides between two
perpendicular straight lines, then the locus of its centre is
4 1
⇒ + 1
= (A) A parabola (B) An ellipse
2
4 b2
4 (C) A hyperbola (D) A circle
⇒ b2 = and the required equation is
3
Sol: Use the concept of a Director Circle.
x2 3y 2
+ 1
= Let 2a, 2b be the length of the major and minor axes
16 4
respectively of the ellipse. If the ellipse slides between
⇒ x2 + 12y 2 =
16 two perpendicular lines, the point of intersection
P of these lines being the point of intersection of
y
perpendicular tangents lies on the director circle of the
ellipse. This means that the centre C of the ellipse is
P(2,1)
a2 + b2 from P. Hence
B
always at a constant distance
A x the locus of C is a circle
O (4,0)
x2 y 2
+ 1
=
9 4

Example 10: Tangents are drawn from the point P(3, 4) B


C
2 2
x y
to the ellipse + =1 touching the ellipse at point
o
90
9 4 P A
A and B. The equation of the locus of the point whose
distances from the point P and the line AB are equal is

(A) 9x2 + y 2 − 6xy − 54x − 62y + 241 =


0 Example 12: If α, β are the eccentric angles of the
x2 y 2
(B) x2 + 9y 2 + 6xy − 54x + 62y − 241 =
0 extremities of a focal chord of the ellipse + 1,
=
16 9
α β
(C) 9x2 + 9y 2 − 6xy − 54x − 62y − 241 =
0 then tan   tan   =
2
  2
(D) x2 + y 2 − 2xy + 27x + 31y − 120 =
0
Sol: Equate the slope of the line joining the focus and
Sol: Write the equation of the chord of contact w.r.t. the two points.
point P. Then follow the standard procedure to find the
9 7
locus. The eccentricity e = 1 − = .
16 4
AB being the chord of contact of the ellipse from
P(3, 4) has its equation Let P (4 cos α , 3sin α ) and Q (4 cos β, 3sin β) be a focal

3x 4y chord of the ellipse passing through the focus at ( 7 ,0) .


+ = 1 ⇒ x + 3y = 3
9 4
3sin β 3sin α
Then =
If (h, k) is any point on the locus, then 4 cos β − 7 4 cos α − 7

h + 3k − 3 sin(α − β) 7
(h − 3)2 + (k − 4)2 = ⇒ =
1+9 sin α − sin β 4
M a them a ti cs | 11.41

cos[(α − β) / 2)] 7 PQ = 8 + 2 = 10
⇒ =
cos[(α + β) / 2] 4 ∴ Area of quadrilateral ABCD

α
⇒ tan   tan
=
β
 
7 − 4 8 7 − 23
= .
1
2
1
2
(
= (AB + CD)PQ = 8 2 + 3 2 10 =55 2 sq. units. )
2 2 7+4 9
Example 2: Show that the angle between the tangents
x2 y2
JEE Advanced/Boards to the ellipse =1 (where a > b ), and the circle
+
a2 b2
Example 1: Common tangents are drawn to the x2 + y 2 =
ab at their points of intersection in the first
parabola y2 = 4x and the ellipse 3x2 + 8y2 = 48 touching  (a − b) 
quadrant is tan−1  .
the parabola A and B and the ellipse at C and D. Find
 ab 
the area of the quadrilateral ABCD.
Sol: We find the point of intersection of the ellipse and
Sol: Write the standard equation of the parabola in the the circle. Then we find the slope of the tangents to the
slope. Use the condition for the line to be a tangent and circle and the ellipse and hence the angle.
obtain the value of m. We then find the points of contact
At the points of intersection of ellipse and circle,
with the ellipse and parabola and then find the area.
1 ab − y 2 y2
Let =
y mx + be a tangent to the parabola y 2 = 4x . + 1
=
m a2 b2
x2 y2 1  1 1  b ab2
It will touch the ellipse + 1 , if= 16m2 + 6
= ⇒ y2  −  = 1− ⇒ y2 =
42 ( 6)2
m2  b2 a2  a a+b
c2 a2m2 + b2 ]
[Using:=
a b
∴ y = ±b and x = ±a
⇒ 16m4 + 6m2 − 1 =0 a+b a+b
1  a b
⇒ (8m2 − 1)(2m2 + 1) =
0 ⇒ m= ± b a 
8 P , 
 a+b a + b 

We know that a tangent of slope m touches the
 a 2a  lies in first quadrant
parabola y 2 = 4ax at  ,  . So, the coordinates of
 m2 m  Equation of tangent at P to the circle is
the points of contact of the common tangents of slope
xa b yb a
1 + ab
=
m= ± to the parabola y 2 = 4x are A(8, 4 2 ) and a+b a+b
2 2
a
B(8, –4 2 ). Its slope is: m1 = −
b
We also know that a tangent of slope m touches the Equation of the tangent at P to the ellipse is
x 2
y 2  a2m b2 
ellipse + 1 at  
= ,± . xa b yb a
a2 b2   + 1
=
 a2m2 + b2 a2m2 + b2  a2
a+b b 2
a+b
Therefore, the coordinates of the points of contact of b3/2
1 Its slope in m2 = −
common tangents of slope m ± to the ellipse are a3/2
2 2
If α is the angle between these tangents, then
 3   3 
C  −2,  and D  −2, − .
 2  2 m2 − m1 −(b3/2 / a3/2 ) + (a1/2 / b1/2 )
tan α
= =
1 + m1m2 1 + (b3/2 / a3/2 )(a1/2 / b1/2 )
Clearly AB || CD. So, the quadrilateral ABCD is a
trapezium. a2 − b2 a−b
= .
We have, AB = 8 2 , CD = 3 2 and the distance a 1/2 1/2
b (a + b) ab
between AB and CD is
1 1 . 4 2 | Ellipse

Example 3: Any tangent to an ellipse is cut by the Then P, Q, and R have coordinates given by:
tangents at the extremities of the major axis at T and
T’. Prove that the circle on TT’ as the diameter passes   2π   2π  
P(acos θ, bsin θ) Q acos  θ +  , bsin  θ +   and
through the foci.   3   3 

Sol: We find out the point of intersection of the tangent A

with the axis and then use these points to find the P
equation of the circle.
x2 y 2
Let the equation of the ellipse be + = 1. O
a2 b2 Q R
The extremities A and A’ of the major axis are A (a, 0), B C
A’ (–a, 0). Equations of tangents A and A’ are x = a and
x y   4π   4π 
x = –a. Any tangent to the ellipse is cos θ + sin θ =1 . R acos  θ +  , bsin  θ +   respectively.
a b   3   3 
The points of intersection are
Normals at P, Q, R to ellipse are concurrent, if the
 b(1 − cos θ)   b(1 + cos θ)  determinants of the coefficients is zero. i.e., if
T  a,  , T'  −a, 
 sin θ   sin θ  sin(θ1 + θ2 ) + sin(θ2 + θ3 ) + sin(θ3 + θ1 ) = 0
The equation of the circle on TT’ as diameter is
 2π   6π   4π 
∴ sin  2θ +  + sin  2θ +  + sin  2θ + 
 b   bcos θ  
2 2
 3   3   3 
(x 2
− a2 ) 
+ y −

 −  =
sin θ   sin θ  
0
  
= sin ( 2θ ) + sin  2θ +
2π   4π 
 + sin  2θ + 
 3   3 
b b2 (1 − cos2 θ)
⇒ x2 − a2 + y 2 − 2y + =0 = 0 for all values of θ
sin θ sin2 θ
∴ The normals are concurrent.
2by
⇒ x2 + y 2 − + b2 − a2 =
0
sin θ
Example 5: Prove that the sum of the eccentric angles
2by of the extremities of a chord which is drawn in a given
⇒ x2 + y 2 − a2e2
=
sin θ direction is constant and equal to twice the eccentric
angle of the point, at which the tangent is parallel to
Foci S (ae, 0) and S’ (–ae, 0) lie on this circle.
the given direction.

Example 4: Let ABC be an equilateral triangle inscribed Sol: Consider two points on the ellipse and evaluate
in the circle x2 + y 2 =
a2 . Suppose perpendiculars from the slope of the chord. If the slope is constant prove
that the sum of the angles is constant.
x2 y2
A, B, C to the major axis of the ellipse = 1,+
a2 b2 Slope of chord AB = m
(a > b), meets the ellipse at P, Q, R respectively so that b(sin α − sin β)
= −
P, Q, R lie on the same side of the major axis as are the a(cos α − cos β)
corresponding points A, B, C. Prove that the normals to
the ellipse drawn at the points P, Q, R are concurrent. 2b cos((α + β) / 2). sin((α − β) / 2) b α+β
= = − cot  .
2asin((α + β) / 2).sin((β − α ) / 2) a  2 
Sol: Find the points of intersection of the perpendicular
and the ellipse. Then apply the condition for the α+β
∴ = constant if m is constant
normals at these three points to be concurrent. 2
x y
Let A, B, C have coordinates (acos θ, asin θ) , Eq. of a tangent is cos θ + sin θ =1
a b
  2π   2π   b
acos  θ +  , asin  θ +  , Slope of this tangent is − cot θ .
  3   3  a
b α+β
  4π   4π  − cot θ , then θ =
Now if m =
acos  θ +  , asin  θ +   respectively. a 2
  3   3 
M a them a ti cs | 11.43

So, the slopes are equal. They are parallel to each other. 1 1   1 1 
Hence proved. get x2  −  + y 2  − = 0
 a2 p2   b2 q2 
   
Example 6: P and Q are two points of the ellipse S P
2 2
x y
+ 1 such that sum of their ordinates is 3. Prove
= C (0,0)
25 9
Q R
that the locus of the intersection of the tangents at P
and Q is 9x2 + 25y 2 =.
150y Above equation will represent a pair of conjugate
diameters of the first ellipse if
Sol: Find the relation between the ordinate and use it
to find the locus. b2
m1m2 = −
a2
If (h, k) is the point of intersection of tangents at θ and
φ , then

((1 / a ) − (1 / p )) = − b
2 2
2

h cos ( (θ + φ) / 2 ) k sin ( (θ + φ) / 2 )
= ;
((1 / b ) − (1 / q )) a
2 2 2

a cos ( (θ − φ) / 2 ) b cos ( (θ − φ) / 2 ) 1 1   1 1 
⇒ a2  −  + b2  −  = 0
 a2 p2   b2 q2 
h2 k2 1    
∴ + =  …(i)
a 2
b2
cos ( (θ − φ) / 2 )
2

a2
+
b2
2
=
p2 q2
We are given that sum of ordinates is 3.

∴ b(sin θ + sin φ) = 3 Example 8: The points of intersection of the two


ellipses x2 + 2y2 ‒ 6x ‒ 12y + 23 = 0 and
θ+φ θ−φ
⇒ 2sin cos = 1  …(ii) 4x2 + 2y2 ‒ 20x ‒ 12y + 35 = 0.
2 2
8  1 47
(A) Lie on a circle centred at  ,3  and of radius .
k sin ( (θ + φ) / 2 ) 1  3  3 2
Now, = =
b cos ( (θ − φ) / 2 ) 2cos ( (θ − φ) / 2 )
2
 8 
(B) Lie on a circle centred at  − ,3  and of radius
2k 1  3 
∴ =  …(iii) 1 47
b cos ( (θ − φ) / 2 )
2 .
3 2
h2 k22k 1 47
Hence from (i) and (iii) we get + = (C) Lie on a circle centred at (8, 9) and of radius .
a 2
b2b 3 2

x2 y 2 2y (D) Are not cyclic.


∴ Locus of (h, k) is + =
25 9 3
Sol: Use the concept of the curve passing through the
⇒ 9x + 25y =
150y 2 2 intersection of two ellipses.
Equation of any curve passing through the intersection
Example 7: If the points of intersection of the ellipses of the given ellipse is
4x2 + 2y 2 − 20x − 12y + 35 +
x2 y2 x2 y2
+ = 1 and + = 1 are the extremities of the
a2 b2 p2 q2 0
λ(x2 + 2y 2 − 6x − 12y + 23) =
conjugate diameters of the first ellipse, then prove that Which represents a circle is
2 2
a b 4 + λ = 2 + 2λ ⇒ λ = 2
+ 2.
=
2 2
p q and the equation of the circle is thus,
6x2 + 6y 2 − 32x − 36y + 81 =
0
Sol: Use the condition for the pair of lines to represent
conjugate diameters.  16  81
⇒ x2 + y 2 −   x − 6y + 0
=
Subtracting in order to find points of intersection, we  3 6
1 1 . 4 4 | Ellipse

8  Example 11: Equation of the diameter of the ellipse E


Centre of the circle is  ,3  and the radius is conjugate to the diameter represented by L is
3 
2 (A) 9x + 2y =
0 (B) 2x + 9y =
0
8 2 81
  + (3) − (C) 4x + 9y =
0 (D) 4x − 9y =
0
3 6
Sol: Use the condition of conjugate diameters to find
128 + 162 − 243 1 47
= . the slope and hence write the equation of the line.
18 3 2
Let y = mx be the diameter conjugate to the diameter
Paragraph for Questions 9 to 12 L : y = 2x of the ellipse E, then

x2 y 2 4  b2 
C : x2 + y 2 =
9, E: + 1 , L : y = 2x
= 2m = −  mm' = − 2 

9 4 9  a 

Example 9: P is a point on the circle C, the perpendicular 2


⇒ m= − and the equation of the conjugate
PQ to the major axis of the ellipse E meets the ellipse at 9
MQ  2
M, then is equal to diameter is y =  −  x or 2x + 9y =
0.
PQ  9
1 2
(A) (B)
3 3 Example 12: If R is the point of intersection of the line
1 L with the line x = 1, then
(C) (D) None of these
2 (A) R lies inside both C and E
Sol: Proceed accordingly using parametric coordinates.. (B) R lies outside both C and E
Let the coordinates of P be (3cos θ, 3sin θ) then the (C) R lies on both C and E
eccentric angle of M, the point where the ordinate PQ
(D) R lies inside C but outside E
through P meets the ellipse is θ and the coordinates of
MQ 2sin θ 2 Sol: Use the position of a point w.r.t a circle.
M are (3cos θ, 2sin θ) , = = .
PQ 3sin θ 3 Coordinates of R are (1, 2)

Example 10: If L represents the line joining the point C (1, 2) = 1 + 22 − 9 < 0
P and C to its centre O and intersects E at M, then the 1
equation of the tangent at M to the ellipse E is ⇒ R lies inside C; E (1, 2) = +1 −1 > 0
9

(A) x + 3y =
3 5 (B) 4x + 3y =5 ⇒ R lies outside E.

(C) x + 3y + 3 5 =
0 (D) 4x + 3 + 5 =0 Example 13: If CF is perpendicular from the centre C
x2 y 2
Sol: Find the point of intersection of the line L and E. of the ellipse + =1 on the tangent at any point
a2 b2
Write the equation of the tangent at M. P, and G is the point where the normal at P meets the
Line L: y = 2x meets the circle C : x2 + y 2 =
9 at points minor axis, then (CF × PG)2 is equal to
3
for which x2 + 4x2 =
9 ⇒ x =± . Sol: Consider a parametric point on the ellipse and
5
proceed to find CF and PG.
3 6
Coordinates of P are ( ± ,± ) Equation of the tangent at P(7 cos θ, 5sin θ) on the
5 5
x y
3 4 ellipse is cos θ + sin θ =1 , then
⇒ Coordinates of M are ( ± ,± ) 7 5
5 5
72 × 52 25 × 49
Equation of the tangent at M to the ellipse E is (CF)2 = =
2 2 2
5 cos θ + 7 sin θ 2
25cos2 θ + 49 sin2 θ
x( ±3) y( ±4)
+ 1 ; x + 3y =
= ±3 5 .
9 5 4 5
M a them a ti cs | 11.45

Equation of the normal at P is 49


= (25cos2 θ + 49 sin2 θ)
7x 5y 25
− =72 − 52
cos θ sin θ So, (CF.PG)
= 2
(49)
= 2
2401 .
 −24 sin θ 
Coordinates of G are  0, 
 5 
2
2 2  24 sin θ 
(PG)
= (7 cos θ) +  5sin θ + 
 5 

JEE Main/Boards

Exercise 1
x2 y 2
Q.10 The common tangent of + =1 and α lies
Q.1 Find the equation of the ellipse whose vertices are 25 4
(5, 0) and (–5, 0) and foci are (4, 0) and (–4, 0). in 1st quadrant. Find the slope of the common tangent
and length of the tangent intercepted between the axis.
Q.2 Find the eccentricity of the ellipse 9x2 + 4y2 ‒ 30y = 0.
Q.11 Find a point on the curve x2 + 2y 2 = 6 whose
7 is minimum.
distance from the line x + y =,
Q.3 Find the equations of the tangents drawn from the
point (2, 3) to the ellipse 9x2 + 16y 2 =
144 .
Q.12 Find the equations to the normals at the ends of
the latus recta and prove that each passes through an
Q.4 Find the eccentric angle of a point on the ellipse 4 2
end of the minor axis if e + e = 1.
x2 y 2
+ = 2 at a distance 3 from the centre.
5 4
Q.13 Find the co-ordinates of those points on the
x2 y 2
Q.5 Obtain equation of chord of the ellipse ellipse + = 1 , tangent at which make equal
4x2 + 6y2 = 24 which has (0, 0) as its midpoint. a2 b2
angles with the axes. Also prove that the length of the
perpendicular from the centre on either of these is
Q.6 Find the foci of the ellipse
1 2
25(x + 1)2 + 9(y + 2)2 =
225 . (a + b2 ) .
2

Q.7 Find the eccentricity of the ellipse if


Q.14 Prove that in an ellipse, the perpendicular from
(a) Length of latus rectum = half of major axis a focus upon any tangent and the line joining the
centre of the ellipse to the point of contact meet on the
(b) Length of latus rectum = half of minor axis.
corresponding directrix.
Q.8 Find the condition so that the line x + my + n =0
x 2
y 2 Q.15 The tangent and normal at any point A of an
may be a normal to the ellipse + 1.
=
a 2
b2 x2 y2
ellipse + =1 cut its major axis in points P and Q
a2 b2
Q.9 If the normal at the point P(θ) to the ellipse
respectively. If PQ = a, prove that the eccentric angle of
5x2 + 14y2 = 70 intersects it again at the point Q(2θ),
the point P is given by e2 cos2 φ + cos φ − 1 = 0 .
2
show that cos θ = − .
3
1 1 . 4 6 | Ellipse

Q.16 A circle of radius r is concentric with the ellipse Q.25 The co-ordinates of the mid-point of the variable
x2 y 2 1
+ = 1 . Prove that the common tangent is chord=y (x + c) of the ellipse 4x2 + 9y 2 =
36 are
a2 b2 2
r 2 − b2
inclined to the major axis at an angle tan−1 . Q.26 A triangle ABC right angled at ‘A’ moves so that it
a2 − r 2 x2 y 2
walsys circumscribes the ellipse + =1 . The locus
Q.17 Show that the locus of the middle points of a2 b2
of the point ‘A’ is
x2 y 2
those chords of the ellipse + = 1 which are
a2 b2
drawn through the positive end of the minor axis is
Exercise 2
x2 y 2 y
+ = .
a2 b2 b Single Correct Choice Type

x2 y2
Q.18 Tangents are drawn from a point P to the circle Q.1 The equation + +1 =0 represents an
ellipse, if 2 −r r −5
x2 + y 2 =r 2 so that the chords of contact are tangent
to the ellipse a2 x2 + b2 y 2 =
r 2 . Find the locus of P. (A) r > 2 (B) 2 < r < 5 (C) r > 5 (D) r ∈ {2,5}

Q.19 Show that the tangents at the extremities of x2 y2


Q.2 The eccentricity of the ellipse + 1 is
=
x2 y 2 a2 b2
all chords of the ellipse + = 1 which subtend
a2 b2 5 3 2 5
a right angle at the centre intersect on the ellipse (A) (B) (C) (D)
6 5 3 3
x2 y2 1 1
+ = + .
a 4
b 4 2
a b2 a2
Q.3 If tan
tanθθ11.. tan
tanθθ2 =− then the chord joining two
b2
x2 y 2
points θ1 and θ2 on the ellipse + =1 will subtend
Q.20 Find the length of the chord of the ellipse a2 b2
x2 y 2 1 2 a right angle at:
+ =1 whose middle point is  ,  .
25 16 2 5
(A) Focus (B) Centre

Q.21 Prove that the circle on any focal distance as (C) End of the major axis (D) End of the minor axis
diameter touches the auxiliary circle.
Q.4 If the line =y 2x + c be a tangent to the ellipse
2 2
x y x 2
y 2
Q.22 Let P be a point on the ellipse = 1 , 0<b<a.
+ + 1 , then c is equal to
=
2
a b2 8 4
Let the line parallel to y-axis passing thorugh P meet
(A) ±4 (B) ±6 (C) ±1 (D) ±8
the circle x2 +y2 = a2 at the point Q such that P and Q
are on the same side of the x-axis. For two positive real
numbers r and s, find the locus of the point R on PQ Q.5 If the line 3x + 4y = − 7 touches the ellipse
such that PR : RQ = r : s as P varies over the ellipse. 3x2 + 4y 2 =
1 then, the point of contact is
0
Q.23 Consider the family of circles x2 +y2 = r2 , 2<r<5.  1 1   1 −1 
(A)  ,  (B)  , 
In the first quadrant, the common tangent to a circle of  7 7  3 3
this family and the ellipse 4x2 + 25y2 = 100 meets the
coordinate axes at A and B, then find the equation of  1 −1   −1 −1 
(C)  ,  (D)  , 
the locus of the mid-point of AB.  7 7  7 7
x2 y2
Q.24 A tangent to the ellipse =1 , meets the
+ Q.6 The point of intersection of the tangents at the
2 2 a2 b2 x2 y 2
x y point P on the ellipse + = 1 and its corresponding
ellipse + =a + b in the points P and Q. Prove that
a 2
b2 a2 b2
the tangents at P and Q are at right angles. point Q on the auxiliary circle meet on the line:
M a them a ti cs | 11.47

a
(A) x = (B) x = 0 (C) y = 0 (D) None of these (C) ay =bx − a4 + a2b2 + b 4
e
(D) by
by==ax++ a4 + a2b2 + b 4
ax
Q.7 The equation of the normal to the ellipse
x2 y 2
+ =1 at the positive and of latus rectum Q.13 In the ellipse the distance between its foci is 6 and
a2 b2
its minor axis is 8. Then its eccentricity is
(A) x + ey + e2a =
0 (B) x − ey − e3a =
0 4 1 3
2 (A) (B) (C) (D) None of these
(C) x − ey − e a =
0 (D) None of these 5 52 5

x2 y 2
Q.14 Equation of a tangent to the ellipse + 1
=
Q.8 The normal at an end of a latus rectum of the ellipse 25 16
x2 y 2 which cuts off equal intercepts on the axes is-
+ = 1 passes through an end of the minor axis, if:
a2 b2
(A) x + y − 41 =
0 (B) x − y + 9 =
0
4 2 3 2
1
(A) e + e = (B) e + e =
1 (C) x + y − 9 =
0 (D) None of these
2
(C) e + e =
1 (D) e3 + e =
1
Q.15 An ellipse has OB as a semi minor axis. FBF’ are
Q.9 If CF is perpendicular from the centre of the ellipse its foci, and the angle FPF’ is a right angle. Then the
x2 y 2 eccentricity of the ellipse, is
+ = 1 to the tangent at P and G is the point
a2 b2 (A)
1
(B)
1
where the normal at P meets the major axis, then the 3 2
product CF.PG is: 1
(C) (D) None of these
2
(A) a2 (B) 2b2 (C) b2 (D) a2 − b2

Q.16 The length of the latus rectum of the ellipse


0 is a common tangent to y 2 = 4x
Q.10 x − 2y + 4 = 9x2 + 4y 2 =
1 , is
x2 y 2
and + =1 . Then the value of b and the other 3 8 4 8
4 b2 (A) (B) (C) (D)
2 3 9 9
common tangent are given by:

(A) b = 3 ; x + 2y + a =0 (B) b = 3; x + 2y + 4 =
0 Q.17 If the distance between a focus and corresponding
1
(C) b = 3 ; x + 2y − 4 =
0 (D) b = 3 ; x − 2y − 4 =
0 directrix of an ellipse be 8 and the eccentricity be ,
then length of the minor axis is 2
16
Q.11 An ellipse is such that the length of the latus (A) 3 (B) 4 2 (C) 6 (D)
3
rectum is equal to the sum of the lengths of its semi
principal axes. Then: x2 y 2
Q.18 Let ‘E’ be the ellipse + =1 & ‘C’ be the
9 4
(A) Ellipse bulges to a circle
circle x2 + y 2 =9 . Let P and Q be the points (1, 2) and
(B) Ellipse becomes a line segment between the two foci (2, 1) respectively. Then:
(C) Ellipse becomes a parabola (A) Q lies inside C but outside E
(D) None of these (B) Q lies outside both C and E
(C) P lies inside both C and E
Q.12 Which of the following is the common tangent to
(D) P lies inside C but outside E
x2 y2 x2 y2
the ellipses, + 1 and
= + 1?
=
a2 + b2 b2 a2 a2 + b2

(A) ay =bx + a4 − a2b2 + b 4

(B) by =ax − a4 + a2b2 + b 4


1 1 . 4 8 | Ellipse

Q.19 The line, x + my + n =0 will cut the ellipse Q.25 The equation 2x2 + 3y2 ‒ 8x ‒ 18y + 35 = K
x2 y2 represents:
+ 1 in poins whose eccentric angle differe by
=
a2 b2 (A) A point if K = 0
π
if: (B) An ellipse if K < 0
2
(C) A hyperbola if K < 0
(A) x2 2 + b2n2 =
2m2 (B) a2m2 + b2 2 =
2n2 (D) A hyperbola if K > 0
(C) a2 2 + b2m2 =
2n2 (D) a2n2 + b2m2 =
22
Previous Years’ Questions
Q.20 The locus of point of intersection of tangents to
x2 y 2 Q.1 If P = (x, y), F1 = (3, 0), F2 = (‒3, 0) and 16x2 + 25y2 = 400,
an ellipse + =1 at two points the sum of whose
a2 b2 then PF1 + PF2 equals  (1998)
eccentric angles is constant is: (A) 8 (B) 6 (C) 10 (D) 12
(A) A hyperbola (B) An ellipse
(C) A circle (D) A straight line Q.2 The number of values of c such that the straight
x2
line y = 4x + c touches the curve + y2 =
1 is (1998)
Q.21 Q is a point on the auxiliary circle of an ellipse. P 4
is the corresponding point on ellipse. N is the foot of (A) 0 (B) 2 (C) 1 (D) ∞
perpendicular from focus S, to the tangent of auxiliary
circle at Q. Then Q.3 The line passing through the extremity A of the
major axis and extremity B of the minor axis of the
(A) SP = SN (B) SP = PQ
ellipse x2 + 9y2 = 9 meets its auxiliary circle at the point
(C) PN = SP (D) NQ = SP M. Then the area of the triangle with vertices at A, M
and the origin O is (2009)
Q.22 A tangent to the ellipse 4x2 + 9y 2 =
36 is cut by 31 29 21 27
the tangent at the extremities of the major axis at T (A) (B) (C) (D)
10 10 10 10
and T’. The circle on TT’ as diameter passes thorugh
the point y2
x2
Q.4 Let P be a variable point on the ellipse = 1 +
(A) (0, 0) (B) ( ±5,0) (C) ( ± 5,0) (D) ( ±3,0) a2 b2
with foci F1 and F2. If A is the area of the triangle PF1F2,
Q.23 Q is a point on the auxiliary circle corresponding then the maximum value of A is……………. (1994)
x2 y 2
to the point P of the ellipse + =1 . If T is the foot
a2 b2 Q.5 An ellipse has OB as a semi minor axis. F and F’ are
of the perpendicular dropped from the focus S onto the its foci and the angle FBF’ is a right angle. Then, the
tangent to the auxiliary circle at Q then the ∆ SPT is: eccentricity of the ellipse is…………. (1997)

(A) Isosceles (B) Equilateral


x2 y2
(C) Right angled (D) Right isosceles Q.6 Let P be a point on the ellipse =11, 0<b<a.
= +
a2 b2
Let the line parallel to y-axis passing through P meet the
x2 y 2 circle x2 + y2 = a2 at the point Q such that P and Q are on
Q.24 = y mx + c is a normal to the ellipse, + 1
=
2 2
if c2 is equal to a b the same side of x-axis. For two positive real numbers r
ans s, find the locus of the point R on PQ such that PR :
(a2 − b2 )2 2 2 2
(a − b ) RQ = r : s as P varies over the ellipse.  (2001)
(A) (B)
a2m2 + b2 am2 2
Q.7 Find the equation of the common tangent in
(a2 − b2 )2 m2 2 2 2 2
(a − b ) m 1st quadrant to the circle x2 + y2 = 16 and the ellipse
(C) (D)
a2 + b2m2 a2m2 + b2 x2 y 2
+ = 1 . Also find the length of the intercept of the
25 4
tangent between the coordinate axes.  (2005)
M a them a ti cs | 11.49

Q.8 A focus of an ellipse is at the origin. The directrix Q.12 The equation of the circle passing through the
is the line x = 4 and the eccentricity is 1/2. Then the x2 y 2
length of the semi−major axis is  (2008) foci of the ellipse + =1 , and having centre at
16 9
8 2 4 5 (0, 3) is  (2013)
(A) (B) (C) (D)
3 3 3 3
(A) x2 + y 2 − 6y − 7 =0 (B) x2 + y 2 − 6y + 7 =0

Q.9 The ellipse x2 + 16y2 = 16 is inscribed in a rectangle (C) x2 + y 2 − 6y − 5 =0 (D) x2 + y 2 − 6y + 5 =0


aligned with the coordinate axes, which in turn in
inscribed in another ellipse that passes through the Q.13 The locus of the foot of perpendicular drawn from
point (4, 0). Then the equation of the ellipse is (2009) the centre of the ellipse x2 + 3y 2 =
6 on any tangent to
(A) x2 + 16y 2 =
16 (B) x2 + 12y 2 =
16 it is  (2014)

( )
2
(C) 4x2 + 48y 2 =
48 (D) 4x2 + 64y 2 =
48 (A) x2 + y 2 =6x2 + 2y 2

(B) ( x + y )
2
Q.10 Equation of the ellipse whose axes are the axes of 2 2
=6x2 − 2y 2
coordinates and which passes through the point (-3, 1)
and has eccentricity is 
(C) ( x − y )
(2011) 2 2
2
=6x2 + 2y 2

(A) 5x2 + 3y 2 − 48 =
0 (B) 3x2 + 5y 2 − 15 =
0
(D) ( x − y )
2
2 2
=6x2 − 2y 2
(C) 5x2 + 3y 2 − 32 =
0 (D) 3x2 + 5y 2 − 32 =
0

Q.11 An ellipse is drawn by taking a diameter of the circle Q.14 The area (in sq.units) of the quadrilateral formed
(x ‒ 1)2 + y2 = 1 as its semiminor axis and a diameter of the by the tangents at the end points of the latera recta to
the Ellipse  (2015)
circle x2 + (y ‒ 2)2 = 4as its semi-major axis. If the centre
of the ellipse is the origin and its axes are the coordinate 27 27
(A) (B) 18 (C) (D) 27
axes, then the equation of the ellipse is  (2012) 4 2
(A) 4x2 + y 2 =
4 (B) x2 + 4y 2 =
8

(C) 4x2 + y 2 =
8 (D) x2 + 4y 2 =
16

JEE Advanced/Boards

Exercise 1 Q.4 An ellipse passes through the points (‒3, 1) and


(2, ‒2) and its principal axis are along the coordinate
Q.1 Find the equation of the ellipse with its centre (1, 2), axes in order. Find its equation.
focus at (6, 2) and containing the point (4, 6).
Q.5 If any two chords be drawn through two points on
the major axis of an ellipse equidistant from the centre,
Q.2 The tangent at any point P of a circle x2 + y 2 = a2
α β γ δ
meets the tangent at a fixed point A (a, 0) in T and T is show that tan . tan . tan . tan = 1 where α,β, γ, δ
joined to B, the other end of the diameter through A, 2 2 2 2
prove that the locus of the intersection of AP and BT is are the eccentric angles of the extremities of the chords.
1
an ellipse whose eccentricity is .
2 Q.6 (a) Obtain the equations of the tangents to the
ellipse 5x2 + 9y 2 =
45 , perpendicular to 3x + 4y =.
11
Q.3 The tangent at the point α on a standard ellipse
ax by
meets the auxiliary circle in two points which subtends (b) Prove that the straight line + c will be a
=
a right angle at the centre. Show that the eccentricity of
3 4
x2 y2
the ellipse is (1 + sin2 α )−1/2 . normal to the ellipse + 1 , if 5c = a2e2 .
=
2 2
a b
1 1 . 5 0 | Ellipse

Q.7 Prove that the equation to the circle having double Q.16 Prove that the length of the focal chord of the
x2 y 2 x2 y 2
contact with the ellipse + = 1 at the ends of a ellipse + = 1 which is inclined to the major axis
a2 b2 a2 b2
2ab2
latus rectum, is x2 + y 2 − 2ae3 x = a2 (1 − e2 − e4 ) . at angle θ is .
a2 + sin2 θ + b2 cos2 θ

Q.8 Find the equations of the lines with equal intercepts x2 y2


Q.17 The tangent at a point P on the ellipse = 1 +
x2 y 2 a2 b2
on the axis & which touch the ellipse + =1.
16 9 intersects the major axis in T & N is the foot of the
  perpendicular from P to the same axis. Show that the
16
Q.9 The tangent at P  4 cos θ, sin θ  to the ellipse circle on NT as diameter intersects the auxiliary circle
 11  orthogonally.
2 2
16x + 11y = 256 is also a tangent to the circle
x2 + y 2 − 2x − 15 =
0 . Find the θ . Find also the equation x2 y2
Q.18 The tangents from (x1 , y1 ) to the ellipse = 1 +
to the common tangent. a2 b2
intersect at right angles. Show that the normals at the
4 y x
Q.10 A tangent having slope − to the ellipse
3 points of contact meet on the line = .
x 2
y 2 y1 x1
+ 1 , intersects the axis of x and y in points A
=
18 32
and B respectively. If O is the origin, find the area of Q.19 Find the locus of the point the chord of contact of
triangle OAB. x2 y 2
the tangent drawn from which to the ellipse + = 1
a2 b2
Q.11 ‘O’ is the origin & also the centre of two concentric touches the circle x2 + y 2 =,
C2 where c < b < a .
circles having radii of the inner & the outer circle as ‘a’
and ‘b’ respectively. A line OPQ is drawn to cut the inner Q.20 Find the equation of the common tangents to the
circle in P & the outer circle in Q. PR is drawn parallel x2 y2 x2 y2
to the y-axis & QR is drawn parallel to the x-axis. Prove ellipse + =1 and + 1.
=
a2 + b2 b2 a2 a2 + b2
that the locus of R is an ellipse touching the two circles.
If the foci of this ellipse lie on the inner circle, find the
ratio of inner outer radii & find also the eccentricity of Q.21 P and Q are the corresponding point on a standard
the ellipse. ellipse and its auxiliary circle. The tangent at P to the
ellipse meets the major axis in T. prove that QT touches
the auxiliary circle.
Q.12 ABC is an isosceles triangle with its base BC twice
its altitude. A point P moves within the triangle such
Q.22 If the normal at the point P(θ) to the ellipse
that the square of its distance from BC is half the
rectangle contained by its distances from the two sides. x2 y 2
+ = 1 , intersects it again at the point Q(2θ) ,
Show that the locus of P is an ellipse with eccentricity 14 5
2
2 show that cos θ = −   .
passing through B & C. 3
3
x2 y2
Q.23 A straight line AB touches the ellipse + 1
=
Q.13 Find the equations of the tangents drawn from a2 b2
the point (2, 3) to the ellipse, 9x2 + 16y 2 =
144 . & the circle x2 + y 2 = r 2 ; where a > r > b . PQ is a focal
chord of the ellipse. If PQ be parallel to AB and cuts
Q.14 Common tangents are drawn to the parabola the circle in P & Q, find the length of the perpendicular
y 2 = 4x & the ellipse 3x2 + 8y 2 = 48 touching the drawn from the centre of the ellipse to PQ. Hence show
parabola at A and B and the ellipse at C & D. Find the that PQ = 2b.
area of the quadrilateral.
x2 y2
Q.24 If the tangent at any point of an ellipse +
= 1
a2 b2
Q.15 If the normal at a point P on the ellipse of semi makes an angle α with the major axis and an angle β
axes a, b & centre C cuts the major & minor axes at G with the focal radius of the point of contact then show
and g, show that a2 (CG)2 + b2 .(Cg)2 =(a2 − b2 )2 . Also
cos β
prove that CG = e2CN , where PN is the ordinate of P. that the eccentricity of the ellipse is given by: e = .
cos α
M a them a ti cs | 11.51

Q.25 An ellipse is drawn with major and minor axes Q.5 An ellipse is described by using an endless string
of lengths 10 and 8 respectively. Using one focus as which passes over two pins. If the axes are 6 cm and 4
centre, a circle is drawn that is tangent to the ellipse, cm, the necessary length of the string and the distance
with no part of the circle being outside the ellipse. The between the pins respectively in cm, are
radius of the circle is_________.
(A) 6, 2 5 (B) 6, 5

Q.26 Point ‘O’ is the centre of the ellipse with major (C) 4,2 5 (D) None of these
axis AB and minor axis CD. Point F is one focus of the
ellipse. 1 f OF = 6 & the diameter of the inscribed
Q.6 If F1 & F2 are the feet of the perpendiculars from the
circle of triangle OCF is 2, then the product (AB) (CD) =
_____________________________. x2 y 2
foci S1 and S2 of an ellipse + = 1 on the tangent at
5 3
any point P on the ellipse, then (S1F1). (S2F2) is equal to:
Exercise 2 (A) 2 (B) 3 (C) 4 (D) 5

Single Correct Choice Type


Q.7 a & b are positive real numbers, such that a > b . If
the area of the ellipse ax2 + by 2 = 3 equals area of the
Q.1 The equation to the locus of the middle point of 2 2
ellipse (a + b)x + (a − b)y = 3 , then a/b is equal to
x2 y 2
the portion of the tangent to the ellipse + =1
16 9 5 +1 5 +1 6 −1 5 −1
included between the co-ordinate axes is the curve. (A) (B) (C) (D)
4 2 2 4

(A) 9x2 + 16y 2 =


4x2 y 2 (B) 16x2 + 9y 2 =
4x2 y 2
Q.8 The locus of image of the focus of the ellipse
(C) 3x2 + 4y 2 =
4x2 y 2 (D) 9x2 + 16y 2 =
x2 y 2 x2 y 2
+ = 1(a > b) with respect to any of the tangent
a2 b2
Q.2 P & Q are corresponding points on the ellipse to the ellipse is
x2 y 2
+ = 1 and the auxiliary circle respectively. The
16 9 (A) (x ± ae)2 =y 2 + 4a2 2 2 2
(B) (x ± ae) = 4a − y
normal at P to the ellipse meets CQ in R where C is
centre of the ellipse. Then (CR) is (C) (x ± ae)2 =y 2 − 4a2 (D) (x ± ae)2 =
4a2

(A) 5 units (B) 6 units (C) 7 units (D) 8 units


Q.9 The normal at a variable point P on an ellipse

Q.3 The equation of the ellipse with its centre at (1, 2), x2 y2
+ = 1 of eccentricity e meets the axes of the
focus at (6, 2) and passing through the point (4, 6) is a2 b2
ellipse in Q and R then the locus of the mid-point of QR
(x − 1)2 (y − 2)2 (x − 1)2 (y − 2)2 is a conic with an eccentricity e’ such that:
(A) + 1 (B)
= + 1
=
45 20 20 45
(A) e’ is independent of e
2
(x − 1) 2
(y − 2) (x − 1) 2
(y − 2) 2 (B) e ‘ = 1
(C) + 1 (D)
= + 1
=
25 16 16 25 (C) e ‘ = e
(D) e ‘ = 1/e
Q.4 A line of fixed length (a+b) moves so that its ends
are always on two fixed perpendicular straight lines. The Q.10 A circle has the same centre as an ellipse & passes
locus of the point which divided this line into portions through the foci F1 and F2 of the ellipse, such that the
of lengths a and b is: two curves intersect in 4 points. Let ‘P’ be any one of
(A) An ellipse (B) An hyperbola their point of intersection. If the major axis of the ellipse
is 17 and the area of the triangle PF1F2 is 30, then the
(C) A circle (D) None of these distance between the foci is:
(A) 11 (B) 12
(C) 13 (D) None of these.
1 1 . 5 2 | Ellipse

Q.11 The arc of the rectangle formed by the α β 1−e


(C) tan tan =
perpendiculars from the centre of the standard ellipse 2 2 1+e
to the tangent and normal at its point whose eccentric
angle is π / 4 is: α β a2 − b2
(D) tan =tan [a − a2 − b2 ] when a > b
2
(a − b )ab 2 2
(a + b )ab2 2 2 b 2
(A) (B)
2 2 2 2
a +b a −b
Q.16 The equation of the common tangents to the
(a2 − b2 ) (a2 + b2 )
(C) (D) ellipse x2 + 4y 2 =
8 and the parabola y 2 = 4x are
ab(a2 + b2 ) (a2 − b2 )ab
(A) 2y − x =4 (B) 2y + x =4
Q.12 Co-ordinates of the vertices B and C of a triangle (C) 2y + x + 4 =
0 (D) 2y + x =0
ABC are (2, 0) and (8, 0) respectively. The vertex A is
x2 y 2
B C Q.17 The distance of a point on the ellipse + 1,
=
varying in such a way that 4 tan tan = 1 . Then locus 6 2
of A is 2 2
from its centre is 2. Then the eccentric angle is:
(x − 5)2 y 2 (x − 5)2 y 2
(A) + 1 (B)
= + 1
= (A) π / 4 (B) 3π / 4 (C) 5π / 4 (D) 7π / 4
25 16 16 25

(x − 5)2 y 2 (x − 5)2 y 2 Q.18 The tangents at any point F on the standard ellipse
(C) + 1 (D)
= + 1
=
25 9 9 25 with foci as S and S’ meets the tangents at the vartices
A and A’ in the points V and V’, then:

Multiple Correct Choice Type (A) (AV).(AV') = b2

(B) (AV).(A' V') = a2


Q.13 Identify the statement which are True
(A) The equation of the director circle of the ellipse, 900
(C) ∠V'SV =
5x2 + 9y 2 =
45 is x2 + y 2 =
14 (D) V’S’ VS is a cyclic quadrilateral
(B) The sum of the focal distances of the point (0, 6) on
x2 y 2
the ellipse +
25 36
= 1 is 10 Previous Years’ Questions
(C) The point of intersection of anytangent to a parabola
and the perpendicular to it from the focus lies on the Q.1 If a > 2b > 0 , then positive value of m for which
tangent at the vertex
y =mx − b a + m2 is a common tangent to x2 + y 2 =
b2
2
(D) The line through focus and (at12 ,2at1 )y = 4ax ,
and (x − a)2 + y 2 =
b2 is  (2002)
meets it again in on the point (at22 , 2at2 ) , if t1 t2 = −1 .
2b a2 − 4b2
(A) (B)
Q.14 The angle between pair of tangents drawn to the a2 − 4b2 2b
ellipse 3x2 + 2y 2 =
5 from the point (1, 2) is
2b b
12 6 (C) (D)
(A) tan −1
(B) tan −1
a − 2b a − 2b
5 5
 x2
−1 12 −1 5 Q.2 Tangent is drawn to ellipse + y2 =
1 at
(C) tan (D) π − cot −  27
5  12 
  (3 3 cos θ, sin θ) (where θ ∈ (0, π / 2) ). Then the value
x 2
y 2 of θ such that the sum of intercepts on axes made by
Q.15 If P is a point of the ellipse + 1 , whose
= this tangent is minimum, is (2003)
2 2
a b
foci are S and S’. Let ∠PSS' =
α and ∠PS'S =
β , then π π π π
(A) (B) (C) (D)
(A) PS + PS’ = 2a, if a > b 3 6 8 4

(B) PS + PS’ = 2b, if a < b


M a them a ti cs | 11.53

Q.3 The normal at a point P on the ellipse x2 + 4y2 = 16 Q.8 Let d be the perpendicular distance from the centre
meets the x-axis at Q. if M is the mid point of the line x2 y2
segment PQ, then the locus of M intersects the latusrectum of the ellipse + = 1 to the tangent drawn at a
a2 b2
of the given ellipse at the points. (2009)
point P on the ellipse. If F1 and F2 are the two foci of the
 3 5 2  3 5 19  ellipse, then show that  (1995)
(A)  ± , ±  (B)  ± ,± 
 2 7   2 4   b2 
  (PF1 − PF2 )2 = 4a2  1 − 
 d2 
 1  4 3 
(C)  ±2 3, ±  (D)  ±2 3, ± 
 7  7 

Q.9 A tangent to the ellipse x2 + 4y2 = 4 meets the
1 ellipses x2 + 2y2 = 6 at P and Q. Prove that tangents at P
Q.4 An ellipse has eccentricity and one focus at the and Q of ellipse x2 + 2y2 = 6 are at right angles. (1997)
2
1 
point P  , 1  . Its one directrix is the common tangent,
2 
Q.10 Find the coordinates of all the points P on the ellipse
nearer to the point P, to the circle x2 + y 2 =
1 and the
hyperbola x2 − y 2 = 1 . The equation of the ellipse, in x2 y 2
+ = 1 , for which the area fo the triangle PON is
the standard form is…………….  (1996) a2 b2
maximum, where O denotes the origin and N be the foot
of the perpendicular from O to the tangent at P. (1999)
Paragraph Based Questions 5 to 7

Tangents are drawn from the point P(3, 4) to the ellipse Q.11 Let ABC be an equilateral triangle inscribed in the
x2 y 2
+ = 1 touching the ellipse at points A and B. circle x2 + y 2 =
a2 . Suppose perpendiculars from A, B,
9 4
(2010) x2 y 2
C to the major axis of the ellipse + 1 , (a > b)
=
a2 b2
Q.5 The coordinates of A and B are
meets the ellipse respectively at P, Q, R so that P, Q, R lie
on the same side of the major axis as A, B, C respectively.
(A) (3, 0) and (0, 2)
Prove that, the normals to the ellipse drawn at the
 8 2 161  points P, Q and R are concurrent.  (2000)
 9 8
(B)  − ,  and  − , 
 5 15   5 5
 Q.12 Prove that, in an ellipse, the perpendicular from
 8 2 161  a focus upon any tangent and the line joining the
(C)  − ,  and (0, 2) centre of the ellipse of the point of contact meet on the
 5 15 
 corresponding directrix.  (2002)
 9 8
(D) (3,0) and  − , 
 5 5 Q.13 Let P(x1, y1) and Q(x2, y2), y1<0,y2<0, be the end
points of the latus rectum of the ellipse x2 + 4y2 = 4. The
Q.6 The orthocenter of the triangle PAB is equations of parabolas with latus rectum PQ are (2008)

 8  7 25   11 8   8 7 (A) x2 + 2 3 y =
3+ 3 (B) x2 − 2 3 y =
3+ 3
(A)  5,  (B)  ,  (C)  ,  (D)  , 
 7 5 8   5 5  25 5  (C) x2 + 2 3 y =
3− 3 (D) x2 − 2 3 y =
3− 3

Q.7 The equation of the locus of the point whose


Q.14 An ellipse intersects the hyperbola 2x2 ‒ 2y2 = 1
distances from the point P and the line AB are equal is
orthogonally. The eccentricity of the ellipse is reciprocal
(A) 9x2 + y 2 − 6xy − 54x − 62y + 241 =
0 of that of the hyperbola. If the axes of the ellipse are
along the coordinates axes, then  (2009)
(B) x2 + 9y 2 + 6xy − 54x + 62y − 241 =
0
(A) Equation of ellipse is x2 + 2y 2 =
2
2 2
(C) 9x + 9y − 6xy − 54x − 62y − 241 =
0
(B) The foci of ellipse are ( ± 1,0 )
(D) x2 + y 2 − 2xy + 27x + 31y − 120 =
0
1 1 . 5 4 | Ellipse

(C) Equation of ellipse is x2 + 2y2 = 4 Q.19 A vertical line passing through the point (h, 0)

(
(D) The foci of ellipse are ± 2,0 ) intersects the ellipse
x2 y 2
4
+
3
= 1 at the points P and

Q. Let the tangents to the ellipse at P and Q meet


Q.15 The line passing through the extremity A of the
major axis and extremity B of the minor axis of the at the point R. If ∆ (h) = area of the triangle PQR,
ellipse x2 + 9y 2 = 9 meets its auxiliary circle at the
=∆1 max ∆ (h) and ∆2 = min = ∆ (h) , then
point M. Then the area of the triangle with vertices at 1/2 ≤ h ≤ 1 1/2 ≤ h ≤ 1
A, M and the origin O is  (2009)
8
31 29 21 27 ∆1 − 8 ∆2 =_____  (2013)
(A) (B) (C) (D) 5
10 10 10 10
x2 y 2
Q.20 Suppose that the foci of the ellipse + =1
Q.16 Match the conics in column I with the statements/ 9 5
expressions in column II.  (2009) are (f1, 0) and (f2, 0) where, f1 > 0 and f1 < 0. Let P1 and
P2 be two parabolas with a common vertex at (0, 0) and
Column I Column II with foci at (f1, 0) and (2f2, 0), respectively. Let T1 be a
tangent to P1 which passes through (2f2, 0) and T2 be a
(A) Circle (p) T
 he locus of the point (h, k) for
tangent to P2 which passes through (f1, 0). Then m1 is
which the line hx + ky = 1 touches
2 2
the slope of T1 and m2 is the slope of T2, then the value
the circle x + y = 4
 1 
of  + m22  is  (2015)
2
(B) Parabola (q) P oints z in the complex plane m 
satisfying z + 2 − z − 2 = ±3
Q.21 Consider the hyperbola H : x2 − y 2 = 1 and a circle
(C) Ellipse (r) P
 oints of the conic have S with center N ( x2 ,0 ) . Suppose that H and S touch each
parametric representation other at a point P ( x1 , y1 ) with x1 > 1 and y1 > 0 . The
 1 − t2  2t common tangent to H and S at P intersects the x-axis at
= x = 3 ,y
 1 + t2  1+t 2
point M. If (l, m) is the centroid of the triangle ∆ PMN ,
 
then the correct expression(s) is(are)  (2015)
(D) Hyperbola (s) T  he eccentricity of the conic lies
in the interval 1 ≤ x < ∞ dl 1
(A) = 1− for x1 > 1
dx1 3x12
(t) P
 oints z in the complex plane
dm x1
satisfying Re ( z + 1 ) = z + 1
2 2
(B) = for x1 > 1
dx1 3  x2 − 1 
 1 
 
Q.17 Equation of a common tangent with positive slope dl 1
(C) = 1+ for x1 > 1
to the circle as well as to the hyperbola is (2010) dx1 3x2
1

(A) 2x − 5 y − 20 =
0 (B) 2x − 5 y + 4 =
0 dm 1
(D) = for y1 > 0
dx1 3
(C) 3x − 4y + 8 =0 (D) 4x – 3y + 4 = 0
Q.22 If the tangents to the ellipse at M and N meet at
x2 y 2 R and the normal to the parabola at M meets the x-axis
Q.18 The ellipse E1 : + = 1 is inscribed in a
9 4 at Q, then the ratio of area of the triangle MQR to area
rectangle R whose sides are parallel to the coordinate of the quadrilateral MF1NF2 is  (2016)
axes. Another ellipse E2 passing through the point (0, 4) (A) 3 : 1 (B) 4 : 5
circumscribes therectangle R. The eccentricity of the
ellipse E2 is  (2012) (C) 5 : 8 (D) 2 : 3

2 3 1 3
(A) (B) (C) (D)
2 2 2 4
M a them a ti cs | 11.55

PlancEssential Questions
JEE Main/Boards JEE Advanced/Boards

Exercise 1 Exercise 1
Q.6 Q.13 Q.16 Q.3 Q.5 Q.7
Q.19 Q.21 Q.22 Q.9 Q.12 Q.15
Q.23 Q.25 Q.27 Q.17 Q.26

Exercise 2 Exercise 2
Q.6 Q.8 Q.11 Q.1 Q.4 Q.6
Q.14 Q.15 Q.21 Q.10 Q.13
Q.23 Q.25
Previous Years’ Questions
Previous Years’ Questions Q.1 Q.4 Q.11
Q.2 Q.5 Q.6 Q.12

Answer Key

JEE Main/Boards 2 14
Q.10 slope = ± ; length =
3 3
Exercise 1
Q.11 (2, 1)

Q.1 9x2 + 25y 2 =


225 a2 x b2 y a2 x b2 y
Q.12 – = a2 – b2 or − = a2 – b 2
Q.2 5 /3 ae  b2  −ae  b2 
±  ± 
 a   a 
Q.3 y − 3= 0, x + y= 5    
 a2 b2 
Q.4 π / 4, 3π / 4 Q. 13  ± ,± 
 a2 + b2 a2 + b2 

Q.5 All lines passing through origin
x2 y2
Q.6 (‒1, 2) and (‒1, ‒6) Q.18 + r2
=
a2 b2
1 3
Q.7 (a) e = (b) e =
2 2 7
Q.20 41
5
a2 b2(a2 − b2 )2
Q.8 + =
2 2
 m n2
1 1 . 5 6 | Ellipse

x2 y 2 (r + s)2
Q.22 + 1
= Q.23 4x=
2 2
y 25y 2 + 4x2
a2 (ra + sb)2

Q.25 − 9c/25, 8c/25 Q.26 x2 + y 2 = a2 + b2 , a director circle

Exercise 2
Single Correct Choice Type

Q.1 B Q.2 D Q.3 B Q.4 B Q.5 D Q.6 C

Q.7 B Q.8 A Q.9 C Q.10 A Q.11 A Q.12 B

Q.13 C Q.14 A Q.15 B Q.16 C Q.17 D Q.18 D

Q.19 C Q.20 B Q.21 A Q.22 C Q.23 A Q.24 C

Q.25 A

Previous Years’ Questions


1
Q.1 C Q.2 B Q.3 D Q.4 b a2 − b2 Q.5
2
2 2 2
x y (r + s) 14
Q.6 + 1
= Q.7 Q.8 A Q.9 B
2 2
a (ar + bs) 3

Q.10 D Q.11 D Q.12 A Q.13 A Q.14 D

JEE Advanced/Boards

Exercise 1

Q.1 4x2 + 9y 2 − 8x − 36y − 175 =


0 Q.4 3x2 + 5y 2 =
32

Q.6 (a) 4x − 3y + 3 21 =
0 ; 4x − 3y − 3 21 =
0 Q.8 x + y − 5= 0, x + y + 5= 0

π 5π
Q.9 θ = or ; 4x ± 33y − 32 =
0 Q.10 24 sq. units
3 3
1 1
Q.11 , Q.13 y −=
3 0 & x +=
y 5
2 2
x2 y21
Q.14 55 2 sq. units Q.19 + =
4 4
a b c2

±ax a4 + a2b2 + b 4
Q.20 by = Q.23 r 2 − b2

Q.25 2 units Q.26 65


M a them a ti cs | 11.57

Exercise 2
Single Correct Choice Type

Q.1 A Q.2 C Q.3 A Q.4 A Q.5 A Q.6 B

Q.7 B Q.8 B Q.9 C Q.10 C Q.11 A Q.12 A

Multiple Correct Choice Type

Q.13 A, C, D Q.14 C, D Q.15 A, B, C Q.16 A, C Q.17 A, B, C, D Q.18 A, C, D

Previous Years’ Questions


( x − (1 / 3)) ( y − 1)
2 2

Q.1 A Q.2 B Q.3 C Q.4 + 1


=
19 1 12

 ±a2 ±b2 
Q.5 D Q.6 C Q.7 A Q.10  , 
 2 2 
 a +b a2 + b2 

Q.13 B, C Q.14 A, B Q.15 D Q.16 A → p; B → s, t; C → r; D → q, s

Q.17 B Q.18 C Q.20 D Q.21 A, B, D Q.22 C

Solutions

JEE Main/Boards  15  225


2

9x + 4  y −  −
2 0
=
 4  4
Exercise 1
2 2
 15   15 
or 9x2 + 4  y −  =  
Sol 1: 2a = 10  4   2 
⇒a=5 2
 15 
4 4 2 y − 
ae = 4 ⇒ e = = x 4 
a 5 or + 1
=
2 2
5  15 
\b2 = a2(1 – e2)    
2  4 
 16 
b2 = 25  1 −  =9
 25  15 5
∴a= ,b=
\b = ± 3 4 2
x2 y 2
Equation of ellipse is + 1 or 9x2 + 25y2 = 225
= b2 25 16 4 5
25 9 e2 = 1 − =1– × =1– =
a2 4 225 9 9
Sol 2: 9x2 + 4y2 – 30y = 0 5
∴e =
 15  3
9x2 + 4  y 2 − y  = 0
 2 
1 1 . 5 8 | Ellipse

x2 y 2 and another point as(–acosα, – bsinα)


Sol 3: + 1
=
16 9 ∴ – acosα = a cosβ ⇒ β = π +
P(2, 3) So there are infinite value of α which will satisfy this
condition therefore all line passing through origin will
\a2 = 16 or a = ± 4
be chord to the given ellipse.
b2 = 9 or b = ± 3
x2 y2 Sol 6:
\ Equation is + 1
=
(4)2 (3)2 (x + 1)2 (y + 2)2
+ 1
= (-1, 3)
\ Equation tangent will be 9 25
a2 = 25
2 2 2
y = mx + a m +b
b2 = 9
y = mx + 16m2 + 9 b2 = a2(1 – e2)
9 = 25(1 – e2) (-4, -2) (2, -2)
As this line passes through (2, 3) (-1, -2)
4
⇒e=
\3 – 2m = 16m2 + 9 5
y coordinate of foci
⇒ 9 + 4m2 – 12m = 16m2 + 9 (-1, -7)
 4
or 12m + 12m = 0
2 = – 2 ± 5 × 
 5 
12(m + 1)m = 0 = – 2 ± 4 = (2, 6)

⇒ m = 0, – 1 ∴ foci is at (–1, 2), (–1, –6)

\ y = 3 or y = – x + 5 2b2
Sol 7: Length of latus rectum =
a
i.e. y – 3 = 0 or x + y = 5
2b2
(a) If =a
a
x2 y 2 x2 y 2
Sol 4: + 2 ⇒
= + 1
= ⇒ 2b2 = a2
5 4 10 8
b2 1 1 1
let φ be eccentric angle ∴ = or e2 =1 − =
a2 2 2 2
∴ Any point on ellipse will be (a cosφ, b sinφ) 1
∴ e=
2
\P= ( 10 cos φ, 8 sin φ )
2b2
(b) if =b
∴ a
( 10 cos φ)2 + ( 8 sin φ)2 =3
⇒ 2b = a
⇒10cos2φ + 8sin2φ = 9
∴ 4b2 = a2
or 2cos2φ = 1 b2 1
or −
1 a 2 4
or cos2φ =
2 1 3
∴ e2 = 1 − =
1 π 3π 4 4
\cosφ = ± or φ = ,
2 4 4 3
∴ e=
2
x2 y 2
Sol 5: + 1
= x2 y2
4 6 Sol 8: + 1
=
a2 b2
Since midpoint of chord is (0, 0)
lx + my + n = 0
∴ Take one point as (a cosα, b sinα)
M a them a ti cs | 11.59

 n
y=– x− y = mx ± 25m2 + 4
m m
x2 y 2
Equation of normal equation of tangent for + 1 is
=
16 16
(a2 − b2 )m'
⇒ y = m’x  y = mx ± 4 m2 + 1
2 2 2
a + b m'
\25m2 + 4 = 16m2 + 16
2 2
-l (a − b )m' n 2
∴ =m' ;  =– or 9m2 = 12 or m = ±
m 2
a + b (m')2 2 m
3
2 4
(a2 − b2 )
 equation tangent is y = ± x ± 4 +1
n m 3 3
∴ = 
m 2 2 4 7
a2 + b2 ×
 y=± x±
m2 3 3

a2m2 + b2 2  4 7
⇒ =  (a2 – b2) y intercept =
m n 3

2m2 x intercept = 2 7
∴ a m + b l = (a – b )
2 2 2 2 2 2 2
2
n
16
a 2
(a − b ) b 2 2 2 2 ∴ Length = 4×7 + ×7
⇒ + = 3
2 m2 n2
7 × 28 7×2 14
Sol 9: Normal of P(a cosθ, b sinθ) = = =
3 3 3
x2 y 2
+ 1
=
14 5 x2 y 2
Sol 11: Point on curve + 1
=
a2 x b2 y 6 3
− = a2 – b2
acos θ b sin θ is ( 6 cos θ, 3 sin θ)

14x 5y ∴ Distance of point from line x + y – 7 = 0 is


– = 14 – 5 = 9
cos θ sinθ
6 cos θ + 3 sin θ − 7
= f(θ)
As this passes through (a cos2θ, b sin2θ) 2
3
14 cos2θ 5sin2θ ∴ f’(θ) = – 3 sinθ + cosθ = 0
∴ – =9 2
cos θ sin θ
1
\14[2cos2θ – 1] – 5 × 2 cos2θ = 9cosθ ⇒ tanθ =
2
\18cos2θ – 9cosθ – 14 = 0
2 1
9 ± 81 + 4 × 14 × 18 or cosθ = and sinθ =
cosθ = 3 3
36
 2. 6 3 
9 ± .33 24 42 ∴ Point is  , 
= =– , 
36 36 36  3 3 
2 P = (2, 1)
\cosθ = – is only possible solution
3
 b2 
Sol 12: End of latus rectum is  ae, ± 
x2 y 2 a 
Sol 10: Equation of tangent for + 1
= 
25 4  b 
2
or  −ae, ± 
y = mx ± a2m2 + b2  a 

1 1 . 6 0 | Ellipse

∴ Equation of normal is 1 2
= (a + b2 )
2
a2 x b2 y
– = a2 – b2
−ae  b2  Hence proved.
± 
 a 
 
a2 x b2 y Sol 14: Let P = (acosθ, bsinθ)
or − = a2 – b2
−ae  b2  b
±  Slope of tangent = –
 a  atanθ
 
if they passed through (0, b) ∴ Slope of normal to tangent

⇒  ab = a2 – b2 atan θ
=
2
b
 b2  b2
⇒  − 1 = ∴ Equation of line
 a2  a2
  a
FN = (y) = tanθ(x – ae)  …(i)
⇒ (–e ) = 1 – e
2 2 b

⇒ e4 + e2 =1 And equation of CP
b
y= tan θx …(ii)
x2 y2 a 
Sol 13: + 1
=
a2 b2 bx a
∴= (x − ae)
Let point be (a cosφ, b sinφ) a b

i.e. (a cosφ, b sin φ)  b2 − a2  a2e


∴  x = −
x cos θ y sin θ  ab  b
∴ Equation of tangent is + =1  
a b
a
Tangent is inclined at equal angle to axis \x=
e
∴ m = ± 1 or b \ The two lines intersect on directrix
− cot θ = | ±1 |
a
b Sol 15: Normal : axsecθ – by cosecθ = a2 – b2
or tanθ =
a
x y
a Tangent : cosθ + sinθ = 1
∴ cosθ = ± , a b
a2 + b2
for y = 0
b
sinθ = ± point of intersection of normal is
a2 + b2
a2 − b2 ae2
x= = = ae2cosθ
 ±a2 b2  asec θ sec θ
\ Points are  ,± 
 a2 + b2 a2 + b2  Point of intersection of tangent is
 
And equation of tangent is a
x = asecθ =
cos θ
x  a2  y  b2  ae2
 ±  +  ± =1 – + asecθ = a
a2  a2 + b2  b2  a2 + b2  sec θ
x y or – e2 + sec2 θ = secθ
or + ±1 =0
a2 + b2 a2 + b2
or e2cos2θ – 1 = – cosθ
\ Distance of tangent from origin is
⇒ e2cos2θ + cosθ – 1 = 0
±1
 1   1  x2 y2
 2 +  Sol 16: + 1
=
 a + b2   a2 + b2  a2 b2
M a them a ti cs | 11.61

x2 + y2 = r2 ellipse,. To the points of intersection of ellipse and (i) is


obtained be making homogenous equation of (i) and
equation of tangent at circle ⇒ xcosθ + y sinθ = r then ellipse & is given by
or equation of tangent at ellipse is y =mx + a2m2 + b2 x2 y2  hx by 
2
+ –  +  =0
2
if it is a tangent to circle, then perpendicular from (0, 0) a2 b2 a b2 
is equal to r.
 1 h2   1 k2  −2hk
\x2  −  + y2  2 − 4  2 2 xy = 0 …(ii)
2
a m +b 2 2  a2 a4  b b  ab
= |r|    
m2 + 1
It chord of contact subtends 90º at origin then the lines
or a m + b = m r + r
2 2 2 2 2 2 separated by (ii) should be ⊥

or (a2 – r2)m2 = r2 – b2 1 h2 1 k2
⇒ − + − 0
=
a2 a4 b2 b4
r 2 − b2
∴m= h2 k2 1 1
a2 − r 2 or + = +
4 4 2
a b a b2
 r 2 − b2  (r 2 − b2 )
∴ tanθ =   or θ = tan–1 1 2
 a2 − r 2  (a2 − r 2 ) Sol 20: Equation of chord whose middle point is  , 
 
is 2 5
2 2
1 2 1 2
Sol 17: Let (h, k) be midpoints of chords, x× y×    
2 + 5 =   + 5
2
∴ Equation of chord with midpoint (h. , k) is 25 16
25 16
xh 4k h2 k2
+ −1 = + −1 x y 1
a2 b2 a2 b2 ∴ + =
50 40 50
It passes through (0, b) 4
y=– (x – 1)
h2 k2 k x2 y 2 y 5
\ Equation is: + =or + = 2
a2 b2 b a2 b2 b 4 2
2   (x − 1)
x  5
∴ + 1
=
Sol 18: Let P be (h, k) 25 16
∴ Chord of contact is
⇒ x2 + x – 12 = 0
xh + yk – r2 = 0
∴ x1 = – 4 and x2 = 3
hx r 2
or y = – + ∴ Length of chord
k k
c2 = a2m2 + b2
L= (x1 − x2 )2 + (y1 − y 2 )2
r4 r2 b2 r2
or = × + 16 7
k2 a2 k2 b2 = (x1 − x2 ) 1 + m2 = 7 1 + = 41
25 5
x2 y2
or r2 = + is locus of P
a2 b2 Sol 21: Let F = (ae, 0) & p = (acosθ, bsinθ)
Radius of circle
Sol 19: We have to find the locus of P(h, k) such that the
1 2 2
chord to contact subtends 90º at centre the equation of = b sin θ + a2 (e − cos θ)2
chord of contact is 2
1 2 2
hx ky = b sin θ + a2e2 − 2aecos θ + a2 cos2 θ
+ 1
= …(i) 2
a2 b2  1 2
= a − b2 cos2 θ + a2 cos2 θ − 2aecos θ
the equation of the straight line joining the centre of 2
1 1 . 6 2 | Ellipse

1 2 2 a2m2 + b2
= a e cos2 θ − 2aecos θ + a2 ± =r
2
m2 + 1
1 1
= (a − aecos θ)2 = (a – ae cosθ)
2 2 or a2m2 + b2 = r2(m2 + 1)
Radius of auxillary circle = a m2(25 – r2) = r2 – 4
1
\r1 – r2 = (a + ae cosθ) r2 − 4
2 m=–
Centre of circle with FP as diameter 2r − r 2
since tangent lies in first quadrant m < 0
 ae + cos θ b sin θ 
=C=  ,  (r 2 − 4)
 2 2  \m2 =
25 − r 2
Distance between centre equation of tangent is

a(e + cos θ)2 b2 sinb2 θ (r 2 − 4) 25(r 2 − 4)


= + y=– x + +4
4 4 25 − r 2 25 − r 2
1 2
= a (e + cos θ)2 + b2 sin2 θ r2 − 4 2 / r2
2 y=– x+
25 − r 2 25 − r 2
1
= (a + aecos θ)2
2 ∴ Midpoint is
1  1 21r 2 1 21r 2 
= (a + ae cos θ)
2  , 
 2 r 2 − 4 2 25 − r 2 
 
∴ The two circle touch each other internally.
21r 2 21r 2
∴ 2x = ; 2y =
Sol 22: Let the co-ordinate of P be (a cos θ, bsinθ) the r2 − 4 25 − r 2
coordinates of Q are (acosθ, bsinθ)
21r 2 21r 2
Let R(h, k) be a point on PQ such that PR : RQ = r : s ∴ 4x2 = ; 4y2 =
2
r −4 25 − r 2
then h = acosθ and 25 4
∴ + 1
=
2
rasin θ + sbsin θ 4x 4y 2
R=
r+s or 25y2 + 4x2 = 4x2y2
h (r + s)k
⇒ cosθ = & sinθ =
a ra + sb Sol 24: The given ellipses are
2
h (r + s)2 k 2 x2 y2
⇒   + 1
= + 1
=
a (ra + sb)2 a2 b2
x2 y2
x2 (r + s)2 y 2 + 1
=
or + 1
= a(a + b) b(a + b)
a2 (ra + sb)2
chord of contact of P(x1, y1) w. r. t. ellipse is
x2 y 2
Sol 23: Equation of ellipse is + 1
= xx1 yy1
25 4 + 1
=
Tangent to ellipse is a(a + b) b(a + b)

y = mx ± a2m2 + b2 or lx + my = n

x2 y2
or mx – y ± a2m2 + b2 = 0 It is touches + 1
=
a2 b2
it is tangent to circle a2l2 + b2m2 = n2
M a them a ti cs | 11.63

a2 × x12 b2 xy1 2 4 5
+ =1 ∴ e= 1− =
2
a (a + b) 2 2
b (a + b) 2 9 3

x12 + y12 = (a + b)2


Sol 3: (B) Let P be (q1) and Q = (q2)
Locus of P is
b
Slope OP = tanq1
x2+ y2 = (a + b)2 a
b
= a(a + b) + b(a + b) and slope OQ = tanq2
a
Which is the director circle.
b2  −a2 
∴ MOP × MOQ = ×  =–1
Sol 25: Let mid-point = (h, k) a2  b2 

∴ Equation of ellipse is ∴ It subtends 90º at centre

4xh + 9ky – 36 = 4h2 + 9k2 – 36


Sol 4: (B) c2 = a2m2 + b2
And equation of chord is
∴ c2 = 8 × 4 + 4
x – 2y + c = 0
4h ak  4h2 + 9k 2  9 \c=± 36 = ± 6
∴ =
− =
−  ⇒h=– k
1 2  c  8
 
Sol 5: (D) Let (x1, y1) be point of contact to ellipse
8 2 9k
4× k + 9k 2 +k
9k 64 1 16 \3xx1+ 4yy1=1 is equation of tangent at (x1, y1)
– =– ⇒+ =
2 c 2 c 3x1 4y1 1
∴ = = −
\25K = 8c 3 4 7
8c 9c  1 1 
\k = ;h=– ∴ (x1, y1) =  − ,− 
25 25
 7 7

Sol 26: ∴ From A two ⊥ tangents can be drawn to


ellipse Sol 6: (C) x-axis

∴ A is the director circle  b2 


Sol 7: (B) The positive end of latus rectum is  ae, 
 a 
i.e. x2 + y2 = a2 + b2 
acosθ = ae ∴ cosθ = e
b2 b
Exercise 2 bsinθ = & sinθ =
a2 a
Single Correct Choice Type ax by
equation of normal is − = a2 – b2
cos θ sin θ
x y2 ax by
Sol 1: (B) + =1 ∴ − × a = a2(e2)
r −2 5−r e b
∴ r – 2 > 0 and 5 – r > 0 or x – ey = ae3

∴ r ∈ (2, 5) ∴ x – ey – e3a = 0

Sol 2: (D) 4x2 + 8x + 9y2 + 36y + 4 = 0 Sol 8: (A) Consider normal at positive end of
latus rectum from above equation of normal is
⇒ (2x + 2)2 + (3y + 6)2 = 36 x – ey – e3a = 0
It passes through (0, –b)
(x + 1)2 (y + 2)2
⇒ + =1 ∴ be – e3a = 0
9 4
⇒ b – e2a = 0
1 1 . 6 4 | Ellipse

b a aa2 a2
∴ = e2 Sol
⇒ y12:
=(B)
± ⇒ a2±⋅ x+±a2 a+2 b⋅ 2 + a2 + b2
x ±y =
a b bb2 b2
⇒ 1 – e2 = e4 or e2 + e4 = 1
a a4 +aa2b2 + ab44 + a2b2 + b 4
⇒ y =± x ±y =±
⇒ x±
b b b2 b2
Sol 9: (C) Tangent at P is
a 1 a 1
x cos θ y sin θ ⇒ y =± x ±y =±a4 +x a±2b2 + ab44 + a2b2 + b 4

+ –7=0 b b b b
a b
4
1 ⇒ yb =± ax
⇒ ±yb a= a2b±
±+ ax 2
+ ab44 + a2b2 + b 4
∴ CF =
cos2 θ sin2 θ
+
a2 b2 Sol 13: (C) 2ae = 6
P = (acosθ, bsinθ)
2b = 8
Equation of normal at
∴ ae = 3 & b = 4
ax by
P= − = a2 – b2 \a2 – b2 = 9 & b2 = 16
cos θ sin θ
\a2 = 25
 (a2 − b2 )cos θ 
∴ G= ,0 
 a  16 3
  e= 1− =
25 5
b4 2 sin2 θ cos2 θ
\PG= b2 sin2 θ + cos2 θ = b +
Sol 14: (A) Slope of ellipse = – 1
a2 b2 a2
∴ Equation is y = – x ± 25 + 16
∴ PG. CF = b2
∴x+y=± 41
x
Sol 10: (A) y = + 2
2
Sol 15: (B) FBF’ is 90º
∴ c2 = a2m2 + b2
We know that BF = BF’ = a
1
⇒ 4 = 4 × + b2 ⇒ b = 3
4 ∴ 2a2 = 4a2e2
∴ The other common tangent has slope – m 1 1
∴ e2 = ⇒e=
1 2 2
∴c= =–2
1
− Sol 16: (C) Ellipse is
2
1
∴ Equation is y = – x –2 x2 y2
2 + 1
=
2 2
or x + 2y + 2 = 0 1 1
   
3 2
2b2 since a < 0
Sol 11: (A) =a+b
a 2a2 2×1 4
∴ Latus rectum is = =
or 2b2 = a2 + ab b 1 9

2
a2 + ab – 2b2 = 0 a
Sol 17: (D) – ae = 8
e
b ± b2 + 8b2 1
a=– e=
2 2
b + 3b 3a 16
a=– =b ∴ = 8 ;a =
2 2 3
∴ Ellipse bulges to circle
M a them a ti cs | 11.65

1 b2 a
= 1− ae −
4 ∴ SN = cos θ = |aecosθ – a|
a2
16 16 3 1 + tan2 θ
\b2 = a2 (1 – l2) = × ×
3 3 4
8 SP = (ae − acos θ)2 + b2 sin2 θ
b=
3
16 = a2 (cos2 θ + a2 − b2 cos2 θ − 2a2ecos θ)
∴ Length of minor axis = 2b =
3
= (a cos θ)2 + a2 − 2a2ecos θ = |aecosθ – a|
Sol 18: (D) E(P) > 0 , E(Q) < 0
∴ SP = SN
C(P) < 0 and C(Q) < 0
x y
∴ P lies inside C but outside E Sol 22: (C) Equation of tangent is cos θ + sinθ = 1
3 2
T is x = 3 and T’ x = – 3
Sol 19: (C) Let P = (acosθ, bsinθ) ∴ Point of intersection of tangent & T let say
 π  π   2(1 − cos θ)   θ
Q =  acos  + θ  ,bsin  + θ   P =  3, −  =  3,2 tan 
 2  2   sin θ   2
= (– asinθ, bcosθ)
 2(1 + cos θ)   θ
P’ =  −3,  =  −3,2cot 
alcosθ +mbsinθ + n = 0  sin θ   2 
And ⇒ alcosθ + mbsinθ = – n → 1 – alsinθ + mbcosθ
∴ Equation of circle is
+ n = 0 & –alsinθ + mbcosθ = – n(2)
 θ  θ
squaring and adding 1 and 2 (x + 3) (x – 3) +  y − 2 tan   y − 2cot  =0
 2  2
we get a2l2 + m2b2 = 2n2 ∴ When y = 0
x2 – 5 = 0
Sol 20: (B) Q1 + Q2 = C
∴x=± 5
Point of intersection of tangent at (q1) and (q2) is
 ∴ It always passes through ( ± 5, 0)
 θ + θ2   θ1 + θ2  
 acos  1  b sin   i.e. it always passes through focus.
  2 ,  2 
(x, y) = 
   
 cos  θ1 − θ2  cos  θ1 + θ2  
  Sol 23: (A) Q = (acosθ, asinθ)
  2   2 
P = (acosθ, bsinθ)
x y
∴ =
c C ∆ SPT is an isosceles triangle.
acos   b sin  
2
  2
Sol 24: (C) Equation of normal in slope form is
\ Locus of P is a straight line
m(a2 − b2 )
y = mx 
Sol 21: (A) P = (acosθ, bsinθ) a2 + b2m2
Q = (acosθ, asinθ)
m2 (a2 − b2 )2
equation of tangent at Q ∴ c2 =
a2 + b2m2
is (y – asinθ) = – 1tanθ (x – acosθ)
a Sol 25: (A) ( 2x − 2 2)2 + ( 3y − 3 3)2 = k
x + ytanθ – = 0 & (ae, 0)
cos θ
2(x − 2)2 3(y − 3)2
∴ + 1
=
k k
1 1 . 6 6 | Ellipse

For ellipse k > 0  12 9 


On solving equation (i) and (ii), we get M  − , 
For a point k = 0  5 5
1 27
Now, area of ∆AOM= .OA × MN= sq. unit
2 10
Previous Years’ Questions
x2 y2
Sol 4: Given, + 1
=
Sol 1: (C) Given, 16x2 + 25y 2 =
400 a2 b2

x2 y 2 Foci F1 and F2 are (—ae, 0) and (ae, 0) respectively. Let


⇒ + 1
= P(x, y) be any variable point on the ellipse. The area A
25 16
of the triangle PF1F2 is given by
Here,
= a2 25,
= b2 16
y
2 2
But a (1 − e )
P(x, y)
16 25(1 − e2 )
⇒=
16 x’ x
⇒ = 1 − e2 F1 O (ae,0) F2
25 (-ae,0)
16 9 3
⇒ e2 =1− = ⇒ e=
25 25 5
π
Now, foci of the ellipse are ( ±ae, 0) ≡ ( ±3,0) θ=
6
3
We have 3 = a. 1
5 = ( − y)( −ae × 1 − ae × 1)
⇒ a=5 2
1
Now, PF1 + PF2 =
major axis = 2a − y( −2ae) =
= aey
2
= 2 × 5 = 10
x2
= ae.b 1 −
a2
Sol 2: (B) For ellipse, condition of tangency is
c2 = a2m + b2 So, A is maximum when x = 0
x2 ⇒ Maximum of A
Given line is =
y 4x + c and curve + y2 =
1
4
b2 a2 − b2
⇒ ma − b 1 + m 2
=1 − b 1 + m 2 = abe = ab 1 − ab
= = b a2 − b2
2 2
a a

⇒ c=
± 65 =65 or − 65 Sol 5: Since, angle FBF’ is right angled

So, number of values are 2. ∴ (slope of FB). (slope of F’B) = –1

y
Sol 3: (D) Equation of auxiliary circle is x2 + y 2 =
9 …(i)
B(0, b)
x y
Equation of AM is + = 1  …(ii)
3 1
y x’ x
F’ O F
(-ae,0) (ae,0)

) 125 , 95)
M
B(0,1)
y’
x’ x
N O A(3,0)
 0 − b  0 − b 
⇒   = −1
 ae − 0   −ae − 0 
M a them a ti cs | 11.67

b2  (1) is tangent to circle x2 + y 2 =


16
⇒ = −1 ⇒ b2 = a2e2
−a2e2
( ab − bx )
2

1 ⇒ x2 + 16
=
⇒ a2 (1 − e2 ) =
a2e2 ⇒ e2 = a
2
1 a2b2 + b2 x2 − 2ab2 x
⇒ e= ⇒ x2 + 16
=
2 a2
PR r b2 2b2
Sol 6: Given, = ⇒ x 2 + b2 + x2 − .x =
16
RQ s y a2 a

 b2  2b2
Q (a cos, a sin) ⇒ x2  1 + − x + b2 − 16 =
0
 a2  a
R (a cos, )  
P (a cos, a sin)
x’ x For unique solution
O (0,0)

4b2  b2 

a2
− 4  1 +  12 − 16 =

 a2 
0 ( )
y’ b4 b4 16b2
⇒ = b2 − 16 + −
⇒ α − bsin θ = r a2 a2 a2
asin θ − α s
⇒ αs − b sin= b2
θ.s rasin θ − αr ⇒ b2 − 16 =
16
a2
⇒ αs +=
αr rasin θ + b sin θ.s
⇒ a2b2 − 16a2 =
16b2
⇒ α(s + r) = sin θ(ra + bs)

⇒ α=
sin θ(ra + bs) (
⇒ a2b2 = 16 a2 + b2  ) (ii) ... (ii)
r+s
x2 y 2
Similarly (i) is tamest to ellipse + 1 are will get
=
Let the coordinate of R be (h, k) 25 4
the relatiesn
⇒=h acos θ
b2 4a2 + 25b2 
a2= (3) ... (iii)
(ar + bs)sin θ
and k = α =
r+s Solving (i) (ii) we get a = 2 7
h k(r + s)
⇒ cos
= θ , sin
= θ 7
a ar + bs b=4
3
On squaring and adding, we get
x y
⇒ Eq. of tangent +
h2 k 2 (r + s)2 2 7 7
sin2 θ + cos2 =
θ + 4 =1
a2
(ar + bs) 2 3
14
h2 k 2 (r + s)2 Distances = a2 + b2 =
1
⇒= + 3
a2 (ar + bs)2
Focus (S = 6, 2)
x2 y 2 (r + s)2
Hence, locus of R is + 1.
=
a2 (ar + bs)2

Sol 7: In 1st quadrat eq. of target will be of fly from

x y if x int ercept = q


+ =1 
a b  y int ercept = b
1 1 . 6 8 | Ellipse

Sol 8: (A) Major axis is along x-axis. Sol 12: (A)

P(2,1)

1
v’ v
A 2 A 2 (4,0)
r

a
− ae =
4
e
9 7
 1 a=4,b =3,e =1 − ⇒
a 2 −  =
4 16 4
 2

a= .
8 Focii is ( ± ae, 0 ) (
⇒ ± 7 ,0 )
3
( ae)
2
=r + b2
Sol 9:
x2 y 2 7+9 = 4
x2 + 4y 2 = 4 ⇒ + = 1 ⇒ a = 2,b = 1 ⇒ P = ( 2,1 )
4 1
Now equation of circle is ( x − 0 ) + ( y − 3) =
2 2
16
x y2 x2 y 2
Required Ellipse is 2 + 2 =1 ⇒ 2 + 2 =1 x2 + y 2 − 6y − 7 =0
a b 4 b
(2, 1) lies on it
Sol 13: (A)
4 1 1 1 3 4
⇒ + =1 ⇒ =1 − = ⇒ b2 = x2 y2
16 b2 b 2 4 4 3 Here ellipse is + = a2 6,b
1 , where= = 2
2
a2 b2
x2 y2 x2 3y 2 Now, equation of any variable tangent is
∴ + =1 ⇒ + =1 ⇒ x2 + 12y 2 =16
16  4  16 4
  mx ± a2m2 + b2 
y= ….(i)
3
where m is slope of the tangent
3 3a2
2
Sol 10: (D) b = a 1 − e 2
( 2
)  2
= a  1 −  = a2 =
2

 5 5 5
So, equation of perpendicular line drawn from
−x
x2 y2 9 5 centre to tangent is y =  ….(ii)
+ =1 ⇒ + =1 m
a2 b2 a2 3a2 Eliminating m, we get

(x )
2
2
32 + y2 = a2 x2 + b2 y 2
a2 =
3
( )
2
2 32 ⇒ x2 + y 2 = 6x2 + 2y 2
b =
5
∴ Required equation of ellipse 3x2 + 5y 2 − 32 =
0
x2 y 2
Sol 14: (D) + 1
=
9 5
Sol 11: (D) Semi minor axis b = 2
 b2 
Semi major axis a = 4 =a 3,=
b 5  ae, 
 a 

x2 y2
Equation of ellipse = + =1 b2 5  5
a2
b 2 = ,  2, 
a 3  3
x2 y 2
⇒ + 1
=
16 4 5 2
e= 1− =
9 3
⇒ x2 + 4y 2 =
16 .
M a them a ti cs | 11.69

1 9  It passes through (a, 0)


Area = 4  × × 3  =27
2 2  \h=a
∴ T = (a, k)
2x 5y
+ =1
9 3' 5 TB is
(0,3) 2x y
+ =1 (k − 0)
9 3 y= × (x + a)
2a
\2ay = kx + ka ⇒ kx – 2ay + ka = 0
 
9
2
,0 and ax + ky = a2
Let point of intersection be (x1, y1)
2a2k 2a3 − ak 2
y1 = and x1 =
k 2 + 2a2 2a2 − k 2

JEE Advanced/Boards x1 =
a(2a2 − k 2 )
2a2 + k 2
Exercise 1
(x1)2 + 2(y1)2 = a2

Sol 1: Let X = x – 1 x 2 y 12 
∴ 1 + a2 
=
And Y = y – 2  a2  a 2 
   
 
∴ Centre = (0, 0)   2 

Focus (F1) (5, 0) b2 1 1


\ Eccentricity is = 1− = 1− =
F2 = (–5, 0) and point on ellipse a2 2 2

= (3, 4)
Sol 3: Equation of auxiliary circle is
F1P + F2P = 2a x2 + y2 = a2  …. (i)
= 80 + 20 = 2a Equation of tangent at P(α) is
3 20 = 2a x cos α y sin α
+ 1
= …(ii)
a b 
∴ a=3 5
Equation of pair of lines OA, OB is obtained by making
homogenous equation of i w. r. t. (ii)
ae = 5
2
x y 
\a2 – b2 = 25 ∴ x + y = a  a cos α + b sin α 
2 2 2
 
∴ b2 = 20
2xyasin α cos α
b= 2 5 ∴(1 – cos2α)x2 –
b
x2 y2 x2 y 2
∴ Equation of ellipse is + 1 or
= + 5
=
a2 b2 9 4  a2 

+y  1 − sin2 α  =
2 0
2 
(x − 1)2 (y − 2)2  b 
⇒ + =5
9 4 But ∠AOB = 90º
4x2 + 9y2 – 8x – 36y – 175 = 0 ∴ coeff of x2 + coeff of y2 = 0
a2
Sol 2: Let T = (h, k) ∴ 1 – cos2α + 1 – sin2α = 0
b2
∴ AP is a2 − b2
1= sin2 α
2
xh + yk = a 2 b
1 1 . 7 0 | Ellipse

a2e2 γ δ α β c − a −(c + a)
1= sin2α tan tan tan tan = × =1
2 2 2 2 c + a −(c − a)
a2 (1 − e2 )
1 3
⇒ e2 = or e = (1 + sin2α)–1/2 Sol 6: (a) m of line = –
(1 + sin2 α ) 4
4
∴ Slope of line ⊥ to given line =
Sol 4: (–3, 1) = (a cosa1, bsina1) 3
(a1 – α) = (a cosa2, bsina2) x2 y2
Equation of ellipse is + 0
=
(3)2 5
9 1 4 4
∴ + 1&
= + 1
= 2
a2 b2 a2 b2 4 4
∴ Equation of tangent is y = x ± 9×  +5
3 3
32 4x
∴ =3 y= ± 21
a2 3
32 32 ∴ 3y = 4x ± 3 21
∴ a2 = & b2 =
3 5
(b) Equation of normal to the ellipse is axsecθ – by
x2 y 2 cosecθ = a2 – b2
∴ Equation of ellipse is + =1
32 32
ax by
3 5 or − =a2 − b2
∴ 3x2+ 5y2 = 32 cos θ sin θ
ax by
the normal given is + c
=
Sol 5: Let α and β form a chord which interests the 3 4
major axis at (c, 0)
3 4 a2 − b2
∴ Equation of chord is ∴= =
cos θ − sin θ c

x α+β y ( α + β) α −β (3c)2 (4c)2


cos   + sin = cos   ∴ + 1
=
a  2  b 2  2  (a2 − b2 )2 (a2 − b2 )2
c ( α + β) α −β
cos = cos   ∴ 5c = a2 – b2 or 5x = a2e2
a 2  2 
α+β x2 y 2
cos  Sol 7: Equation of ellipse is + 1 the ends of
=
 a2
b 2
 2 =a latus rectum are

α −β c
cos    b2   b2 
 2  L1  ae,  & L2  ae, − 
 9   9 
 
α+β α −β
cos   + cos   For double contact the centre of circle should lie on normal
 2   2  = a+c
⇒ of L1 & L2. By symmetry y-coordinates of centre = 0
α+β α −β a−c
cos   − cos  
 2   2  Equation of normal at LI is

α β a2 x b2 y
2coscos − = a2 – b2
⇒ 2 2 = a+c ae b2 / a
α β a−c
−2sin sin For centre y = 0
2 2
a2  b2 
α β c−a ∴ x centre =  1 − 2  = ae3
⇒ tan tan = a  a 
2 2 c+a
Similarly let γ and δ intersect major axis at (– c, 0) ∴ Equation of circle is
2
 b2 
γ δ −c − a (x – ae ) + y = (ae – ae ) + 
3 2 2 3 2

∴ tan tan =  9 
2 2 a−c  
M a them a ti cs | 11.71

 b2 
2 4
y=– x±8
(x – ae ) + y = (ae – ae ) + 
3 2 2 3 2 
 3
 9 
3
∴A=± ×8=±6&B=±8
x – 2ae + a e + y = (ae – ae ) + a (1 – e )
2 3 2 6 2 3 2 2 2 2
4
∴ x2 – 2ae 3 + y2 = a2(1 – e2 – e4) 1 1
∴ Area of A = A ×B = × 6 × 8 = 24
2 2

Sol 8: Since lines have equal intercept on axis


Sol 11: P = (bcosθ, bsinθ)
∴ Slope = – 1
Q = (acosθ, asinθ)
∴ Equation is y = – x ± a2m2 + b2
R = (acosθ, bsinθ)
y=–x± 25
x2 y2
∴ + 1 is locus of R which is
=
or x + y ± 5 = 0 are the equation of tangents. a2 b2

 16  An ellipse since focus lie on inner circle


Sol 9: P =  4 cos θ, sin θ  = (acosθ, bsinθ)
 11  Q = (a cos, a sin)
∴ Equation of tangent is P

x cos θ y sin θ R =(a cos, b sin)


+ 1
= b
4 16
11
It is also tangent to circle
x2 + y2 – 2x – 15 = 0 ∴ b = ae
c = (1, 0) and r = 4  b2 
⇒ b2 = a2  1 − 
∴ Distances from center = radius  a2 

cos θ b2 1 b 1
−1 ∴ = ⇒ =
∴ 4 =4 2 2 a
a 2
cos2 θ 11sin2 θ
+
16 256 b2 1
And e = 1− =
2
(cos θ − 4) 2
11sin θ 2 a 2
⇒ = cos2θ +
16 16
Sol 12: Let B = (a, 0) C = (–a, 0) & A = (a, 0)
(cosθ – 4) = 16 cos θ + 11sin θ
2 2 2
Equation of line AB is
cos2θ – 8cosθ + 16 = 11 + 5cos2θ x + y = a and that of AC is y = x + a
4cos2θ + 8cosθ – 5 = 0 Let P= (x, y) distance from BC = y

4cos2θ + 10cosθ – 2cosθ – 5 = 0 And area of PRAS = PR x PS


1 (x + y − a) (x − y + a)
∴ cosθ = = ×
2 2 2
π 5π
∴ θ = or θ = A
3 3
S R’
2 2 2
Sol 10: y = mx ± a m +b P

2
4 4
y=– x± 18 ×   + 32
3 3
C B
1 1 . 7 2 | Ellipse

according to Q. so coordinate of point of contact are A(8, 4 2) and


1 (x2 − (y − a)2 )
y2 = ± B(8, − 4 2) we also know that tangent of slope m
2 2
x2 y 2
∴ ± 4y2 = x2 – y2 – 2ay – a2 touches the ellipse + 1 at
=
a2 b2
When it is +4y2 it forms a hyperbola.
 a2m b2 
When it is –4y2 it forms an ellipse  ,± 
 2 2 2 
 a m +b a2m2 + b2 
∴ x2 + 3y2 – 2ay – a2 = 0
 3   −3 
 & D  −2,
2 ∴ C =  −2,
 1 4a2 
x2 + 3  y −  =  2  2
 3 3

 1
2
AB || CD ∴ Quadrilateral is trapezium
y − 
x2  3 1
∴ + =1 Area = × h (AB + CD)
4a 2
4a2 2
3 9 1  6 
= × 10 ×  8 2 +  = 55 2 sq. units
b2 1 2 2 2  2
\ e2 = 1 – =1– = ⇒e=
a2 3 3 3
Sol 15: Equation of normal at P(acosθ, bsinθ) is
Sol 13: Let equation of tangent be ax by
− = a2 – b2
cos θ mnθ
y = mx ± 16m2 + 9 is passes through (2, 3)
 cos θ 2 
2 G=  (a − b2 ), 0 
∴ (3 –2m) = ± 16m + 9  a 
⇒ 4m2 – 12m + 9 = 16m2 + 9  − sin θ 2 
And g =  0, (a − b2 ) 
 b 
⇒ 12m2 + 12m = 0
∴ a2CG2 + b2 (Cg)2
m = 0 or m = – 1
cos2 θ sin2 θ
Rechecking we get when m = 0 C > 0 & when m = – 1 = a2 × (a2 – b2)2 + b2 (a2 – b2)2
C>0 a2 b2
= (a2 – b2)2
∴ Equation of tangent is
 b2 
y = 3 and y = – x + 5 or x + y = 5. CG = acosθ  1 − 
 a2 

1
Sol 14: Let y = mx + be tangent to parabola y2 = 4x. = acosqe2 = e2 × abscissa of P
m
x2 y2
It will touch ellipse + 1 if
= Sol 16: ∴ (ae + rcosθ, rsinθ) lies on ellipse
42 ( 6 )2
1 (ae + r cos θ)2 r 2 sin2 θ
= 16m2 + 6 ∴ + =1
m2 a2 b2
⇒ 16m4 + 6m2 – 1 = 0  cos2 θ sin2 θ  2ecos θ
∴ +  r2 + r + e2 – 1 = 0
 a2 b 2  a
⇒ (8m2 – 1) (2m2 + 1) = 0  

1 To find the  chord we have to find (r1 – r2) as r1 +ve


m=± and r2 < 0
2 2
we know that a tangent at slope m touches parabola
 a 2a 
at  , 
 m2 m 
M a them a ti cs | 11.73

Y Sol 18: Let P = (acosα), bsinα) and Q = (a cosβ, bsinβ)


Q P(x,y) Since tangents at P & Q are ⊥ is
−b −b
 ∴ × −1
=
X’
O M
X atan atan β

b2
x2 + y2 =1 ∴ tanatanβ= –
a2 b2 a2
Y’ the point of intersection to tangents is

∴ (r1 – r2)2  α+β α+β


 acos   bcos  
  2 ,  2 
4e2 cos2 θ 4(e2 − 1)
= –  α −β α −β 
 cos2 θ sin2 θ 
2  cos2 θ sin2 θ   cos   cos   
2
a  2 + 2   2 + 2    2   2  
 a b   a b 

Find the point of interaction of normal from their
2 2 4 2 equations.
4e a b cos θ
=
(b cos2 θ + a2 sin2 θ)2
2 You can easily show that
slop ON = slope OT
2 2 2 2 2 2 2
4a b (b cos θ + a sin θ)(e − 1) y x
– ∴ N lies on =
(b2 cos2 θ + a2 sin2 θ)2 y1 x1

 b2  Sol 19: Let (h, k) be the point the chord of contact is


4  1 − 2  a2b 4 cos2 θ + 4b 4 (b2 cos2 θ + a2 sin2 θ)
 a  xh ky
=  + –1=0
(b2 cos2 θ + a2 sin2 θ)2 a2 b2

It touches circle x2 + y2 = c2
4a2b 4
= | −1 |
(b2 cos2 θ + a2 sin2 θ)2 ∴ =c
h2 k2
+
2ab2 a4 b4
∴ (r1 – r2) =
(b2 cos2 θ + a2 sin2 θ)
 h2 k 2 
∴ 1 = c  4 + 4 
2

x cos θ y sin θ a b 
Sol 17: The tangent at P is + 1
=
a b x2 y2 1
where p = (acosθ, bsinθ) on + = is locus of P.
4 4
a b c2
 a 
∴ T= ,0  & N = (acosθ, 0)
 cos θ 
Sol 20: Equation of tangent to ellipse
equation of circle with TN as diameter is
x2 y2
 a  + 1
=
x −  (x – acosθ) + y = 0 a2 + b2 b2
2

 cos θ 
y = mx ± (a2 + b2 )m2 + b2
 1 
⇒ x2 – a  + cos θ  x + y2 + a2 = 0
 cos θ  It is also tangent to the ellipse,
Equation of auxiliary circle is x + y – a = 0 2 2 2
x2 y2
+ =1
a 1  a2 a2 + b2
2gg1 + 2ff1= 2  + cos θ  ×0+2×0×0 = 0
2  cos θ  \c2 = a2 m2 + (a2+ b2)
c1 + c2 = a2 – a2 = 0 ∴ (a2+ b2)m2 + b2 = a2m2 + (a2+ b2)
The two circle as orthogonal
1 1 . 7 4 | Ellipse

a2 a Length of ⊥ from centre to PQ (LP)


∴ m2 = m=±
b 2 b
r 2 − b2
∴ Tangents are + × ae
+mae a2 − r 2
= =
by = ± ax ± a4 + a2b2 + b 4 m2 + 1 a2 − b2
a2 − r 2
Sol 21: P = (acosθ, bsinθ) &
r 2 − b2 1
Q = (acosθ, asinθ) =+ ×a× a2 − b2 = r 2 − b2
2
a −b 2 a
tangent at P is

x cos θ y sin θ PQ = 2 r 2 − LP2 = 2 b2 = 2b


+ 1
=
a b
 a  P
∴T=  ,0  
 cos θ 
T
90- 90-

asin θ 1 180-
Slope QT = =– S’ P S
a tanθ
acos θ −
cos θ
∴ QT ⊥ OQ ∴ QT is tangent to the auxiliary circle
Sol 24:
Sol 22: Normal at P(θ) is ∠SPN = 90 – b
ax secθ – bycosecθ = a2 – b2 ∠S’PN = ∠SPN. As normal bisects angle between S’P
and SP
It passes through Q(acos2θ, bsin2θ)
∠SPS’ = 180 – 2β
a2 × cos2 θ b2 × 2sin θ cos θ
∴ − = a2 – b2 ∠PSS’ = 180 – α + b
cos θ cos θ sin θ
∠PS’S = α + β – 180
∴ a2(2cos2θ – 1) – 2b2cos2θ = (a2 – b2)cosθ.
Applying sine rule on DSPS’
∴ 18cos2θ – 9cosθ – 14 = 0
sin ∠PS'S sin ∠PSS sin ∠S'PS
18cos2θ –21cosθ – 12cosθ – 14 = 0 = =
PS PS' SS'
\3cosθ(6cosθ – 7) + 2(6cosθ – 7) =0 sin(α + β − 180°)

2 PS
∴ cosθ = –
3 sin(180º −(α − β)) sin(180º −2β)
= =
PS' 2ae
Sol 23: Let equation of tangent to ellipse be
PS + PS’ = 2a
y = mx + a2m2 + b2 sin(α − β) − sin(α + β) sin2β
⇒ =
Now it touches circle 2a 2ae
2sin β cos β cos β
∴ c2 = r2(m2 + 1) ∴ e= =
2sin β cos α cos α
∴ a2m2 + b2 = r2m2 + 1
Sol 25: Let P = (acosθ, bsinθ)
2 r 2 − b2
m = 5x 4y
a2 − r 2 normal is − 9
=
cos θ sin θ
Equation of PQ is y = m(x – ae) It passes through (ae, 0) = (3, 0)
or mx – y – mae = 0 when sinθ ≠ 0
M a them a ti cs | 11.75

15 5 Sol 2: (C) P = (acosθ, bsinθ)


= 9 ∴ cosθ = × not possible
cos θ 3 Q = (acosθ, asinθ)
When sinθ = 0 equation of normal is y = 0 which passes normal at P is
through (3, 0)
ax by
∴ Radius = a – ae = 5 – 3 = 2 − = a2 – b2
cos θ sin θ
4x 3y
Sol 26: PF = QF and PC = RC ⇒ − 7
=
cos θ sin θ
∴ Ae – 1 + b – 1 = a2e2 + b2 Equation of CQ is y = tanqx
x
(ae + b – 2)2 = (a2e2 + b2) ∴ = 7 ⇒ x = 7cosθ & y = 7sinθ s
cos θ
∴ 4 – 4ae – 4b + 2aeb = 0
∴ R = (7cosθ, 7sinθ)
2 = 2ae + 2b – aeb
∴  (CR) = 7
ae = 6
C
∴ 2 = 12 + 2b – 6b Sol 3: (A) Lines that C = (1, 2)
5 Patting through P = (4, 6)
∴b= P
2
1  Centre and focus have same y coordie
al =a –b
2 2 2 2 R
1 ∴ This will be ellipse where major axis is horizontal so.
25
( x − n) (y − k)
2 2
36 = a2 – O F
4
Q Eqn. will be + 1
=
a2 b2
16a 13
a2 = ∴a= Centre is (h, k)
4 2
The left focus (h ‒ c, k)
AB. CD = 4ab = 65
Right focus (h + c, k)
Where c2 = a2 ‒ b2
Exercise 2 On putting values

( x − 1) ( y − 2)
2 2

Single Correct Choice Type + 1 


= (1 ) (i)
a2 b2
x2 y 2 Patting through (9, 6)
Sol 1: (A) Tangent to ellipse + 1 is
=
16 9
x y 32 y2 9 16
4
cosθ + sinθ = 1
3
⇒ + 1 ⇒
= 2
+ 1
= (2)
a2
b 2 a b2  (ii)
Let P be point of intersection of x-axis & Q be Let Q be
The given frees (6, 2) must be right focuses
the point on tangent and y-axis
⇒ (h + c,k ) =
( 6,2 )
 4   3 
∴ P = , 0  & Q =  0, 
 cos θ   sin θ  ⇒ h=
+c 6 =
k 2

Let M = (x, y) ⇒=c 5 (=


 h 1)

x=
2
& y=
3 Now, C2 = a2 − b2 ⇒ a2 − b2 = 25  (3) (iii)
cos θ 2sin θ
Solving (ii) and (iii)
4 9
∴ + 1
= Substituting in (i)
x2 4y 2
( x − 1) ( y − 2)
2 2
\16y2 + 9x2 = 4x2y2 + 1
=
45 20
1 1 . 7 6 | Ellipse

Sol 4: (A) Let the lines be x-axis & y-axis Sol 8: (B) Let P = (h, k)
Let P = (h, 0) and Q = (0, k) Foot of perpendicular from focus to any tangent of the
ellipse lies on its auxiliary circle.
h2 + k2 = (a + b)2
∴ Midpoint of P & S lies on auxiliary circle
 bh ak 
(x, y) =  , 
 (a + b) (a + b)   h ± ae k 
∴ M=  , 
2 2  2 2
x y
  +  = 1 2 2
b  a  h±ae   k  2
⇒   +   =a .
 2  2
∴ Equation of P is an ellipse
2 2
 x±ae   y  2
Sol 5: (A) Now sum of distance of points from two
⇒   +   =a .
 2  2
foci = constant = 2a for an ellipse
∴ Necessary length of string = 2a = 6 Sol 9: (C) Equation of normal is
a = 3 and b = 2 ax secθ – by cosecθ = a2 – b2
Distance between pins = 2ae
 a2 − b2 
Q=   cosθ
b2 6× 5  a 
 
= 6 1− = = 2 5
a 2 3
 a2 − b2 
R=–   sinθ
Sol 6: (B) It is a known property that  b 
 
SF1·SF2 = b2 = 3
 a2 − b2 (a2 − b2 ) 
M = (x, y) =  × cos θ, − sin θ 
 2a 2b 
Sol 7: (B) Ellipse 1 is  
∴ Locus of M is
x2 y2
+ 1
=
2 2 (a2 − b2 )2
 3  3 (ax)2 + (by)2 =
    4
 a  b
   
x2 y2
∴ + 0
=
And E2 is  a2 − b2 
2
 a2 − b2 
2

   
x2
+
y2
1
=  2a   2b 
2 2
 3   3  coeff of y is > coeff of x.
   
 a+b   a−b  2
     a2 − b2 
 
2a  b2
are of ellipse = pab e’ = 1 –  =1– =0
2
 a2 − b2  a2
∴ pa1b1 = pa2b2  
 2b 
3 3
⇒ =
ab a2 − b2 Sol 10: (C) Equation of ellipse is
⇒ a – b = ab
2 2
x2 y2
+ 1 and of a circle is
=
a a
2
a2 b2
⇒   −   −1 =0
b b x2 + y2 = a2 – b2

∴ a 1+ 5 a2 − b2 − y 2 y2
= ∴ + 1
=
b 2 a2 b2
M a them a ti cs | 11.77

⇒ (a2 – b2)y2 = a2b2 – a2b2 + b4 2ae = 6


4
b \a = 5
∴ y2 =
2 2
a −b b2
a
∴1 − =
b 2 a 25
2
y=
a2 − b2 \b = 4
(x − 5)2 y 2
\ Equation of ellipse can be + 1
=
17 25 16
2a = 17∴ a =
2
Multiple Correct Choice Type
And aexy = 30
b2 x2 y 2
∴ a2 − b2 × = 30 Sol 13: (A, C, D) (A) ellipse is + 1
=
a2 − b2 9 5
Equation of director circle is x2 + y2 = a2 + b2
∴ Distance between foci
⇒ x2 + y2 = 14
289
= 2ae = 2 a2 − b2 = 2 − 30 (B) Sum of focal distances = 2b(when b > 0)
4
and 2a when a > 0
13
= 2× 13
=
2 ∴ S = 2 × 6 = 12
(C) Free (a known property of parabola)
Sol 11: (A) Equation of tangent is
(D) Line passes through focus.
x cos θ y sin θ
+ −1= 0 2at2 − 0 2at1
a b =
at22 − a at12 − a
x y
⇒ + –1=0
2a 2b (slope of PF = QF)
1
∴ P’ = ∴ t2( t12 – 1) = t1( t22 – 1)
1 1
2
+ t1t2(t2 – t1) + (t2 – t1) = 0
2a 2b2

Equation of normal is 2ax – 2by = a2 – b2 ∴ t1 = t2 or t1t2 = – 1

But points are distinct


a2 − b2
∴ P2 =
2(a2 + b2 ) ∴ t1t2 = – 1

∴ Area of rectangle = P1P2


Sol 14: (C, D) Equation of tangent is
2 2 2 2
2ab (a − b ) (a − b )ab
= + = 5 2 5
2 2 2 2 (a2 + b2 ) y = mx ± m +
a +b 2(a + b ) 3 2
It passes through (1, 2)
Sol 12: (A) If for an ellipse S & S’ are focus, then
5 2 5
(m – 2)2 = m +
 PSS'   PS'S  1−e 3 2
tan   × tan   =
 2   2  1+e 5 2 5
m2 – 4m + 4 = m +
3 2
∴ Centre of ellipse = (5, 0)
2 2 3
1−e 1 m + 4m − =0
= 3 2
1+e 4
3 4m2 + 24m – 9 = 0
∴ 5e = 3e =
5
1 1 . 7 8 | Ellipse

angle between tangents α+β α −β


2sin   cos  
1
=  2   2 
m1 − m2 (m1 + m2 )2 − 4m1m2 e α+β α+β
tanθ = = 2sin   cos  
1 + m1m2 1 + m1m2  2   2 

9 α −β
62 + ×4 cos  
4 3 5 12 1
=  2 
= = ×4 =
9 5 5 e α+β
1− cos  
4  2 
12 α −β α+β
\θ = tan–1 cos   − cos  
5 1−e  2   2 
 5 =
12 1+e α −β α+β
other angle = π – tan–1 = π – cot–1  12  cos   + cos  
5    2   2 

 α β 1−e (1 − e)2


∴ tan tan = =
2 2 1+e 1 − e2

b2 b2
1+1− −2 1− 2 2 2 2
If angle between lines is E0 = a2 a2 = 2a − b − 2a a − b
b2 b2
∴ π + θ can also be considered angle between lines,
a2
∴ D is also correct
Sol 16: (A, C) Equation of tangent to parabola is
x2 y2
Sol 15: (A, B, C) + 1
= 1
a2 b2 y = mx +
m
(A) and (B) are true
for ellipse c2 = a2 m2+ b2
By sine rule in DPSS’ then
1
∴ = 8m2 + 2
S'P 2
SP SS' m
= =
sin(β) sinα sin( π − (α + β)) 8m4 + 2m2 – 1 = 0

8m4 + 4m2 – 2m2 – 1 = 0


P(acos , bsin)
1 1
∴ m2 = ⇒m=±
4 2
 
∴ Equation of tangents
1
S’ (-ae, 0) S (ae, 0) are y = x +2 or x – 2y + 4 = 0
2
1
SP S'P SS' and y = – x − 2 or 2y + x + 4 = 0
= = 2
sin β sin α sin(α + β)

SP + S'P SS' Sol 17: (A, B, C, D) a2cos2θ + b2sin2 θ= 4


=
sin α + sin β sin(α + β) 6cos2θ + 2sin2θ = 4
2a 2ae 4cos2θ = 2
∴ =
sin α + mβ sin(α + β) 1
cosθ = ±
2
1 sin β + sin α
∴ = π 3π 5π 7π
e sin ( α + β ) ∴ Eccentric angle is , , ,
4 4 4 4
M a them a ti cs | 11.79

Sol 18: (A, C, D) Equation of tangent is x cos θ y sin θ


∴ Equation of tangent is + 1
=
3 3 1
x cos θ y sin θ
+ 1
= v’ Thus, sum of intercepts
a b
3 3 1 
A =(a, 0) A’
v
=  + =f(θ) (say)
A  cos θ sin θ 
 
 b(1 − cos θ) 
∴ V=  a, 
 sin θ  3 3 sin3 θ − cos3 θ
=⇒ f '(θ) = Put f '(θ) 0
sin2 θ cos2 θ
A’ = (–a, 0)
1
sin3 θ
⇒= cos3 θ
3/2
 b(1 + cos θ)  3
∴ V’ =  −a, 
 sin θ  1 π
⇒ tan θ = , i.e., θ =
3 6
b2 (1 − cos2 θ)
AV × A’V =
sin2 θ π
and
= at θ , f ''(0) > 0
We know that V’V subtend a right angle at each of the 6
foci π
∴ Hence, tangent is minimum at θ = .
6
∴ VV’ SS’ lie on a circle with VV’ as diameter.
Sol 3: (C) There are two common tangents to the circle
x2 + y 2 =
1 and the hyperbola x2 − y 2 =
1 . These are
Previous Years’ Questions x = 1 and x = –1. But x = 1 is nearer to the point P(1/2, 1).
Therefore, directrix of the required ellipse is x = 1.
Sol 1: (A) Given, y =mx − b 1 + m2 touches both the
circles, so distance from centre = radius of both the 3
∴e=
circles. 2
3
∴ x =±4 × =±2 3 ( x = ±ae)  …(ii)
ma − 0 − b 1 + m2 2
=b 4
On solving equation (i) and (ii), we get × 12 + y 2 =
1
m2 + 1 49
48 1 1
⇒ y2 =1− = ⇒y=±
−b 1+m 2 49 49 7
and =b  1
m2 + 1 ∴ Required points  ±2 3, ±  .
 7

⇒ ma − b 1 + m2 =− b 1 + m2 Sol 4: Now, If Q(x, y) is any point on the ellipse, then its


distance from the focus is

⇒ m2a2 − 2abm 1 + m2 + b2 = b2 (1 + m2 ) QP = (x − 1 / 2)2 + (y − 1)2


and its distance from the directrix is | x − 1 | by definition
⇒ ma − 2b 1 + m2 =
0 of ellipse, QP
= e | x −1 |
2 2
y
⇒m
= a 4b2 (1 + m2 )
P ) 12 ‘ 1 )
2b
⇒ m=
a − 4b2
2

x’ x
-1 1
Sol 2: (B) Given tangent is drawn at (3 3 cos θ, sin θ)
O

x2 y 2
to + 1.
=
27 1
y’
1 1 . 8 0 | Ellipse

2 ⇒ x + 3y = 3 …(i)
 1 2 1
⇒  x −  + (y − 1) = x −1 Equation of the straight line perpendicular to AB
 2 2
5.
through P is 3x − y =

 1
2
1 Equation of PA is x − 3 =0.
⇒  x −  + (y − 1)2 = (x − 1)2
 2 4 The equation of straight line perpendicular to PA
 9 8 8
1 1 2 through B  − ,  is y = .
⇒ x2 − x + + y 2 − 2y +=
1 (x − 2x + 1)  5 5 5
4 4
 11 8 
Hence, the orthocenter is  ,  .
⇒ 4x2 − 4x + 1 + 4y 2 − 8y + 4 = x2 − 2x + 1  5 5

⇒ 3x2 − 2x + 4y 2 − 8y + 4 =
0 1
Sol 7: (A) Equation of AB is y − 0 =− (x − 3)
3
 1 1
2
2
⇒ 3  x −  −  + 4(y − 1)2 =
0 x + 3y − 3 =0 x + 3y − 3 = 10[(x − 3)2 + (y − 4)2 ]
 3 9
 
(Look at coefficient of x2 and y 2 in the answers).
2
 1 1
⇒ 3  x −  + 4(y − 1)2 =
 3 3 Sol 8: Let the coordinates of point P be (acosθ, bsinθ).
2
Then equation fo tangent at P is
 1
x −  x y
3 (y − 1)2 cos θ + =1 …(i)
⇒  + 1
= a bsin θ 
1/9 1 / 12
We have , d = length of perpendicular from O to the
tangent at P.
Comprehenesion Type
y
Sol 5: (D) Figure is self explanatory.

P(3,4) P (a cos, b sin)


D
B x’ x
F1(-ae,0) O F2(ae,0)
F
A(3,0) x

y’
0 + 0 −1
Sol 6: (C) Equation of AB is d=
cos2 θ sin2 θ
+
P(3, 4)
a2 b2

1 cos2 θ sin2 θ
) )
9 8
5‘5 B A(3, 0)

=
d a2
+
b2
1 cos2 θ sin2 θ

= +
d2 a2 b2
8  b2 
y −0
= 5 (x − 3) We have, to prove (PF1 − PF2 )2 = 4a2  1 −  .
 d2 
9 
− −3  b 
2
5 Now, RHS = 4a2  1 − 
 d2 
8 
= (x − 3) 4a2b2
−24 = 4a2 −
1 d2
⇒ y=
− (x − 3)
3
M a them a ti cs | 11.81

 cos2 θ sin2 θ  Suppose the tangents of P and Q meets in A(h,k).


= 4a2 − 4a2b2  +  Equation of the chord of contact of the tangents
 a2 b2 
 through A(h,k) is
= 4a2 − 4b2 cos2 θ − 4a2 sin2 θ hx ky
+ = 1 …(iv)
6 3 
= 4a2 (1 − sin2 θ) − 4b2 cos2 θ
But Eqs. (iv) and (ii) represent the same straight line, so
= 4a2 cos2 θ − 4b2 cos2 θ comparing Eqs. (iv) and (ii), we get
= 4 cos2 θ(a2 − b2 ) h/6 k /3 1
= = ⇒
= h 3cos θ and=
k 3sin θ
cos θ / 2 sin θ 1
= 4 cos2 θa2e2  e =
a − (b / a)2 
  Therefore, coordinates of A are (3cos θ, 3sin θ) .
Now, the joint equation of the tangents at A is given
Again,
= PF1 e acos θ + a / e
by T2 = SS1
= a ecos θ + 1 2
 hx ky 
i.e,  + − 1
= a(ecos θ + 1 ) ( − 1 ≤ cos θ ≤ 1 and 0 < e < 1)  6 3 

Similarly, PF2 =a(1 − ecos θ)  x2 y 2   h2 k 2 


=  + − 1 + − 1  …(v)
 6 3  6 3 
Therefore, LHS = (PF1 − PF2 )2   

= [a(ecos θ + 1) − a(1 − ecos θ)]2 In equation (v).

= (aecos θ + a − a − aecos θ)2 h2 1  h2 k 2 


Coefficient of x2 = −  + − 1

36 6  6 3 
= (2aecos
= θ)2 4a2e2 cos2 θ 

Hence, LHS = RHS. h2 h2 k 2 1 1 k 2


= − − + = −
36 36 18 6 6 18

Sol 9: Given, x2 + 4y 2 =
4 k 2 1  h2 k 2 
And coefficient of y 2 = −  + − 1
x2 y 2 
9 3 6 3 
or + 1
= …(i) 
4 1  k 2 h2 k 2 1 h2 1
= − − + =− +
Equation of any tangent to the ellipse on (i) can be 9 18 9 3 18 3
written as
Again, coefficient of x2 + coefficient of y 2
x
cos θ + y sin θ =1 …(ii) 1 2 1 1
2  = − (h + k 2 ) + +
18 6 3
Equation of second ellipse is
1 1
y =− (9 cos2 θ + 9 sin2 θ) +
A 18 2
9 1
= − + =0
Q 18 2
3
P
1 Which shows that two lines represent by equation (v)
x’
-2 O 2
x are at right angles to each other.
6
- 6
-1
- 3

y’
x2 + 2y 2 =
6

x2 y 2
⇒ + 1
= …(iii)
6 3 
1 1 . 8 2 | Ellipse

Sol 10: Let the coordinates of P be (acos θ, bsin θ) . sec2 θ(a2 tan2 θ + b2 − 2a2 tan2 θ)
Equations of tangents at P is =
y (a2 tan2 θ + b2 )2

sec2 θ(atan θ + b)(b − atan θ)


N =
P (a2 tan2 θ + b2 )2
x
x’
O For maximum or minimum, we put
M
f '(θ)= 0 ⇒ b − atan θ = 0
K
[sec2 θ ≠ 0. atan θ + b ≠ 0, 0 < θ < π / 2]
y’
x y ⇒ tan θ =b / a
cos θ + sin θ =1
a b
> 0, if 0 < θ < tan−1 (b / a)
Again, equation of normal at point P is Also, f '(θ) 
−1
< 0, if tan (b / a) < θ < π / 2
ax sec θ − by cosec θ= a2 − b2
Therefore, f(θ) has maximum, when
Let M be foot of perpendicular from O to PK, the normal
at P. b b
= θ tan−1  = ⇒ tan θ
1 a a
Area of ∆OPN = (Area of rectangle OMPN)
2 b a
Again
= sin θ =− cos θ
1 a2 + b2 a2 + b2
= ON. OM
2
By using symmetry, we get the required points
Now,
1 ab  ±a2 ±b2 
ON = =  , .
 2 2 
cos2 θ sin2 θ b2 cos2 θ + a2 sin2 θ  a +b a2 + b2 
+
a2 b2
( ⊥ from O, to line NP) Sol 11: Let the coordinates of A ≡ (acos θ, b sin θ) , so
that the coordinates of
a2 − b2
and OM = =B {acos(θ + 2π / 3), asin(θ + 2π / 3)}
a2 sec2 θ + b2cosec2 θ
and
= C {acos(θ + 4 π / 3), asin(θ + 4 π / 3)}
2 2
(a − b ).cos θ.sin θ
= According to the given condition, coordinates
a2 sin2 θ + b2cos2 θ of P are (acos θ, bsin θ) and that of Q are
{acos(θ + 2π / 3), bsin(θ + 2π / 3)} and that of R are
Thus area of {acos(θ + 4 π / 3), bsin(θ + 4 π / 3)} .

ab(a2 − b2 ).cos θ.sin θ y


∆OPN =
2(a2 sin2 θ + b2cos2 θ)
A (a cos,  sin)
B
2 2
ab(a − b )tan θ
= P
2(a2 tan2 θ + b2 ) x’ x
Q R O
tan θ
Let f(θ) = (0 < θ < π / 2)
a tan2 θ + b2
2

C
2 2 2 2 2 2
sec θ(a tan θ + b ) − tan θ(2a tan θ sec θ)
f '(θ) = y’
(a2 tan2 θ + b2 )2
[It is given that P, Q, R are on the same side of x-axis as
A, B and C].
M a them a ti cs | 11.83

Equation of the normal to the ellipse at P is The equation of line perpendicular to tangent is,
ax by x sin θ y cos θ
− =a2 − b2 or ax sin θ − by cos θ − =
λ
cos θ sin θ b a
1 Since, it passes through the focus (ae, 0), then
= (a2 − b2 )sin2θ ……(i)
2  aesin θ
− 0 =λ
Equation of normal to the ellipse at Q is b

 2π   2π  ⇒ λ =aesin θ
ax sin  θ +  − by cos  θ + = b
 3   3 
x sin θ y cos θ aesin θ
∴ Equation is − =  …(i)
1 2  4π  b a b
(a − b2 )sin  2θ +  …(ii)
2  3  Equation of line joining centre and point of contact
Equation of normal to the ellipse at R is P(acos θ, b sin θ) is
b
 4π   4π  =y (tan θ)x …(ii)
ax sin  θ +  − by cos  θ + = a
 3   3  
Point of intersection Q of Eqs. (i) and (ii) has x coordinate
1 2  8π  a
(a − b2 )sin  2θ +  …(iii) .
2  3  e

a
 4π   2π   2π  Hence, Q lies on the corresponding directrix x = .
But sin  θ + =  sin  2π + θ −  = sin  θ −  e
 3   3   3 
x2 y 2
 4π   2π   2π  Sol 13: (B, C) + 1
=
and cos  θ + =  cos  2π + θ −  = cos  θ − 3  4 1
 3   3   
 8π   4π   4π 
(
b2 a2 1 − e2
= )
and sin  2θ + =  sin  4 π + 2θ −  = sin  2θ − 
 3   3   3  3
⇒e=
Now, eq. (iii) can be written as 2
 1  1
ax sin(θ − 2π / 3) − by cos(θ − 2π / 3) = ⇒ P  3, −  and Q  − 3, − 
 2  2
1 2
(a − b2 )sin(2θ − 4 π / 3) …(iv) (given y1 and y 2 less than 0).
2 
For the liens (i), (ii) and (iv) to be concurrent, we must Co-ordinates of mid-point of PQ are
have the determinant  1
R ≡  0, −  .
 2
1 2
asin θ −bcos θ (a − b2 )sin2θ
2 PQ = 2 3 = length of latus rectum.
 2π   2π  1 2  4π 
∆1 asin  θ +
=  −bcos  θ +  (a − b2 )sin  2θ + =  0 ⇒ two parabola are possible whose vertices are
 3   3  2  3 
 2π   2π  1 2  4π  
asin  θ −  −bcos  θ −  (a − b2 )sin  2θ −  3 1  3 1
 3   3  2  3   0, − −  and  0, − .
 2 2  
  2 2
Thus, line (i), (ii) and (iv) are concurrent. Hence the equations of the parabolas are

x2 − 2 3y =
3+ 3
Sol 12: Any point on the ellipse
x2 y2 And x2 + 2 3y =
3− 3 .
+ 1 be P (acos θ, bsin θ) .
=
a2 b2
Sol 14: (A, B) Ellipse and hyperbola will be confocal
The equation of tangent at point P is given by;

x cos θ y sin θ ⇒ ( ± ae,0 ) ≡ ( ± 1,0 )


+ 1
=
a b
1 1 . 8 4 | Ellipse

 1  2 sec θx − 3tan θy =6
⇒±a× ,0  ≡ ( ± 1, 0 )
 2  It is also tangent to circle x2 + y 2 − 8x =
0
1
⇒ a =2 and e =
2 8 sec θ − 6
⇒ 4
=
2 2
(
⇒ b = a 1 − e ⇒b = 1 2
) 2 4 sec2 θ + 9 tan2 θ

∴ Equation of ellipse
x2 y 2
+ 1
=
( 8 sec
= θ − 6)
2
(
16 13 sec2 θ − 9 )
2 1
⇒ 12 sec2 θ + 8 sec θ − 15 = 0
Sol 15: (D) Equation of line AM is x + 3y −3 = 0 5 3 5
⇒ sec θ = and − but sec ≠
3 6 2 6
Perpendicular distance of line from origin =
10 3 5
⇒ sec θ = − and ⇒ tan θ =
9 9 2 2
Length of AM = 2 9 − =2×
10 10 ∴ Slope is positive
1 9 3 27 Equation of tangent = 2x − 5 y + 4 = 0
⇒ Area = ×2× × = sq. units
2 10 10 10
Sol 18: (C)
Sol 16: A → p; B → s, t; C → r; D → q, s y

1  h2  (0, 4)
(p) = 4  1 + 
k2  k 2 
 (-3, 2) y=2
x=3
⇒= (
1 4 k 2 + h2 ) x = -3
x
2
1 (-3, -2) y = -2 (3, -2)
∴ h2 + k 2 =
  which is a circle.
2
(q) If z − z1 − z − z 2 =where
k k < z1 − z 2 the locus is
a hyperbola.

(r) Let t = tan α Equation of ellipse is

⇒x
= 3 cos 2 α and sin2 α =y ( y + 2)( y − 2) + λ ( x + 3)( x − 3) =0
x 4
or cos 2 α = and sin 2 α =y It passes through (0, 4) ⇒ λ =
3 3
x2 2 x2 y 2
∴ y
+= sin2 2α + cos2=
2α 1 which is an ellipse. Equation of ellipse is + 1
=
3 12 16
(s) If eccentricity is 1, ∞ ) , then the conic can be a 1
e= .
parabola (if e = 1) and a hyperbola if e∈ (1, ∞ ) . 2
(t) Let z =
x + iy; x, y ∈ R
Sol 19: (9)
⇒ ( x + 1 ) − y 2 = x2 + y 2 + 1
2

x2 y 2
⇒ y2 =
x ; which is a parabola. + 1
=
4 3

Sol 17: (B) Let equation of tangent to ellipse 3


=y 4 − h2 at x = h
2
sec θ tan θ
x− y=
1
3 2 Let R ( x1 , 0 )
M a them a ti cs | 11.85

xx1 4
PQ is chord of contact, so =1 ⇒ x = d 1
4 x1 = 1−
dx1 3x12
which is equation of PQ, x = h
4 4 dm 1 dm 1 dy1 x1
So =h ⇒ x1 = = = , =
x1 h dy1 3 dx1 3 dx1 3 x2 − 1
1
1
∆ (h) = area of ∆ PQR = × PQ × RT
2 Sol 22: (C)
1 2 3 3
( )
3/2
= × 4 − h2 × ( x1 − h) = 4 − h2 x×3 y 6
2 2 2h Equation of tangent at M is + 1
=
2×9 8

=∆ ' (h)
(
− 3 4 + 2h2 ) 4 − h2
Put y = 0 as intersection will be on x-axis.
∴ R ≡ ( 6, 0 )
2
2h
which is always decreasing. Equation of normal at M is
3
45 5 1 3  3
So ∆1 = maximum of ∆ (h) = at h = 3
X +=
y 2 + 
8 2 2 2  2 
9
∆2 = minimum of ∆ (h) =at h = 1 3 7
2 Put y = 0, x = 2 + =
2 2
8 8 45 5 9 7 
So ∆1 − 8 ∆2 = × − 8. = 45 − 36= 9 ∴ Q ≡ , 0
5 5 8 2 2 
1  7 5
∴ Area ( ∆ MQR ) = ×  6 −  × 6 = 6 sq. units.
Sol 20: (D) 2  2 4
Area of quadrilateral
The equation of P1 is y 2 − 8x =
0 and P2 is y 2 + 16 x =
0

Tangent to y 2 − 8x =
0 passes through (-4, 0) (MF1NF2 ) =
2 × Area ( ∆ F1 F2 M)
1
2 1 = 2× ×2× 6 = 2 6
⇒ 0= m1 ( −4 ) + ⇒ = 2 2
m1 m12
5/4 5
∴ Required Ratio = =
Also tangent to y 2 + 16 x =
0 passes through 2 8
(2, 0)
4
⇒ 0= m2 × 2 − ⇒ m22 = 2
m2
1
⇒ + m22 =
4
m12

Sol 21: (A, B, D)


1 
Tangent at P, xx1 − yy1 =1 intersects x axis at M  , 0 
 x1 
y y −0
Slope of normal = − 1 =1
x1 x1 − x2

⇒ x2= 2x1 ⇒ N ≡ ( 2x1 ,0 )

1
3x1 +
x1 y
For=
centroid  = ,m 1
3 3
2017-18 100 &
op kers
Class 11 T
By E ran culty
-JE Fa r
IIT enior emie .
S fP r es
o titut
Ins

MATHEMATICS
FOR JEE MAIN & ADVANCED
SECOND
EDITION

Exhaustive Theory
(Now Revised)

Formula Sheet
9000+ Problems
based on latest JEE pattern

2500 + 1000 (New) Problems


of previous 35 years of
AIEEE (JEE Main) and IIT-JEE (JEE Adv)

5000+Illustrations and Solved Examples


Detailed Solutions
of all problems available

Topic Covered Plancess Concepts


Tips & Tricks, Facts, Notes, Misconceptions,
Hyperbola Key Take Aways, Problem Solving Tactics

PlancEssential
Questions recommended for revision
12. HYPERBOLA

1. INTRODUCTION
A hyperbola is the locus of a point which moves in the plane in such a way that Z
the ratio of its distance from a fixed point in the same plane to its distance
X’ P
from a fixed line is always constant which is always greater than unity.
M
The fixed point is called the focus, the fixed line is called the directrix. The
constant ratio is generally denoted by e and is known as the eccentricity of the Directrix
hyperbola. A hyperbola can also be defined as the locus of a point such that S (focus)
the absolute value of the difference of the distances from the two fixed points Z’
(foci) is constant. If S is the focus, ZZ′ is the directrix and P is any point on the
hyperbola as show in figure. Figure 12.1

SP
Then by definition, we have = e (e > 1).
PM
Note: The general equation of a conic can be taken as ax2 + 2hxy + by 2 + 2gx + 2fy + c =0
This equation represents a hyperbola if it is non-degenerate (i.e. eq. cannot be written into two linear factors)

 a h g
 
∆ ≠ 0, h2 > ab. Where  ∆ = h b f 
 g f c 

PLANCESS CONCEPTS

1. T he general equation ax2 + 2hxy + by 2 + 2gx + 2fy + c =0 can be written in matrix form as
 a h g  x 
a h   x    
 x y     + 2gx + 2fy + c =0 and 
 x y 1  h b f   y  = 0

h b  y
g f c  1 
  
Degeneracy condition depends on the determinant of the 3x3 matrix and the type of conic depends on
the determinant of the 2x2 matrix.
2. Also the equation can be taken as the intersection of z =ax2 + 2hxy + by 2 and the plane
z= − ( 2gx + 2fy + c )
Vaibhav Gupta (JEE 2009, AIR 54)
1 2 . 2 | Hyperbola

2. STANDARD EQUATION OF HYPERBOLA


Let the center O of the hyperbola be at the origin O and the foci F1 and F2 be on the x-axis.
The coordinates of foci F1 and F2 are (–c, 0) and (c, 0).
By the definition of hyperbola,
Distance between a point P and focus F1 – Distance between P and focus F2 = constant (say 2a)

PF1 – PF2 = 2a; (x + c)2 + (y − 0)2 − (x − c)2 + (y − 0)2 = 2a

⇒ (x + c)2 + (y)2 = 2a + (x − c)2 + (y)2

Squaring both the sides, we get (x + c)2 + y2 = 4a2 + 2(2a). (x − c)2 + (y)2 + (x – c)2 + y2

⇒ x2 + 2cx + c2 + y2 = 4a2 + 4a (x − c)2 + (y)2 + x2 – 2cx + c2 + y2

⇒ 4cx = 4a2 + 4a (x − c)2 + (y)2 ⇒ cx = a2 + a . (x − c)2 + (y)2 ⇒ cx – a2 = a (x − c)2 + (y)2




Squaring again, we get 
Y
c2x2 – 2a2cx + a4 = a2[(x – c)2 + y2] P(x,y)

c2x2 – 2a2cx + a4 = a2[x2 – 2cx + c2 + y2] = a2x2 – 2a2cx + a2c2 + a2y2


X’ X
O
c2x2 –a2x2 – a2y2 = a2c2 – a4 F1(-c,0) F2(c,0)
x2 y2
⇒ (c2 – a2)x2 – a2y2 = a2(c2 – a2) ⇒ − =1 Y’
a2 c2 − a2
x2 y2 Figure 12.2
⇒ − = 1 (taking b2 = c2 – a2)
a2 b2
x2 y2
Hence, any point P(x, y) on the hyperbola satisfies the equation − =1
a2 b2

3. TERMS ASSOCIATED WITH HYPERBOLA


(a) Focus: The two fixed points are called the foci of the hyperbola and are denoted by F1 and F2. The distance
between the two foci F1 and F2 is denoted by 2c.

2c
F1 F2
Focus Focus

Figure 12.3

(b) Centre: The midpoint of the line joining the foci is called the center of the hyperbola.

Center

Figure 12.4
M a them a ti cs | 12.3

(c) Transverse-Axis: The line through the foci is called the transverse axis. Length of the transverse axis is 2a.

Transverse Axis

2a

Figure 12.5

(d) Conjugate-Axis: The line segment through the center and perpendicular to the transverse axis is called the
conjugate axis. Length of the conjugate axis is 2b.

Conjugate Axis
2b

Figure 12.6

(e) Vertices: The points at which the hyperbola intersects the transverse axis are called the vertices of the
hyperbola. The distance between the two vertices is denoted by 2a.

Vertices

Figure 12.7

c
(f) Eccentricity: Eccentricity of the hyperbola is defined as and it is denoted by e. And e is always greater than
1 since c is greater than 1. a

a2
(g) Directrix: Directrix is a line perpendicular to the transverse axis and cuts it at a distance of from the centre.
c
a2 a2
i.e. x=± or y=±
c c

a2 a2
a
2 x= x=
y= c c
c 2 O Directrix
a O
y=
c Directrix

Figure 12.8
1 2 . 4 | Hyperbola

(h) L ength of The Latus Rectum: The Latus rectum of a hyperbola is a line segment perpendicular to the
transverse axis and passing through any of the foci and whose end points lie on the hyperbola. Let the length
of LF be  . Then, the coordinates of L are (c,  )
L(c,)
x2 y 2
Since, L lies on hyperbola − = 1.
a2 b2

latus rectum
c2 2
Therefore, we have − =1 O
a2 b2
 b2  L’
2 c2 c2 − a2 b4 b2
⇒ − −1 = ⇒  =b   =
2 2
⇒ Figure 12.9
b2 a2 a2  a2  a2 a
 
b2 b2 2b2
Latus rectum LL′ = LF + L′F = + =
a a a
x2 y 2
(i) Focal Distance of a Point: Let P(x, y) be any point on the hyperbola − = 1 as shown in figure. Then by
definition, a2 b2

We have SP = e.PM and S¢P = e.PM′


P(x,y)
 a O
⇒ SP = e.NK = e (CN – CK) = e  x −  = ex – a and S¢P = e(NK′)
 e F1 F2
 a
= e(CN + CK′) = e  x −  = ex + a
 e
⇒ S¢P – SP = (ex + a) – (ex – a) = 2a = length of transverse axis Figure 12.10

Illustration 1: Find the equation of the hyperbola, where the foci are (±3, 0) and the vertices are (±2, 0).
 (JEE MAIN)
Sol: Use the relation c2 = a2 + b2, to find the value of b and hence the
equation of the hyperbola.
O
We have, foci = (±c, 0) = (±3, 0) ⇒ c = 3
F1 4 F2
and vertices (±a, 0) = (±2, 0)
a=2
But c2 = a2 + b2 ⇒ 9 = 4 + b2 ⇒ b2 = 9 – 4 = 5 ⇒ b2 = 5 Figure 12.11

Here, the foci and vertices lie on the x-axis, therefore the equation of the
hyperbola is of the form
x2 y2 x2 y 2
− = 1 ⇒ − =1
a2 b2 4 5

Illustration 2: Find the equation of the hyperbola, where the vertices are (0, ±5) and the (0,5)
foci are (0, ±8).  (JEE MAIN)

Sol: Similar to the previous question.


We have, vertices (0, ±a) = (0, ±5) ⇒
a = 5
foci (0, ±c) = (0, ±8) ⇒ c=8
But, we know that c2 = a2 + b2 ⇒ 64 = 25 + b2
⇒ b2 = 64 – 25 = 39 (0,-5)
Here, the foci and vertices lie on the y-axis, therefore the equation of
Figure 12.12
M a them a ti cs | 12.5

y2 x2 y 2 x2
hyperbola is of the form − = 1. i.e., − =1
a2 b2 25 39
which is the required equation of the hyperbola.

Illustration 3: If circle c is a tangent circle to two fixed circles c1 and c2, then show that the locus of c is a hyperbola
with c1 and c2 as the foci. (JEE MAIN)

C1 C2

r1 r2

r r

C(h, a)

Figure 12.13

Sol: Refer to the definition of a hyperbola.


cc1 = r + r1; cc2 = r + r1
cc1 – cc2 = r1 – r2 = constant

Illustration 4: Find the equation of the hyperbola whose directrix is 2x + y = 1 and focus, (1, 2) and eccentricity 3 .
 (JEE MAIN)

Sol: Use the definition of the hyperbola to derive the equation.


Let P(x, y) be any point on the hyperbola. Draw PM perpendicular from P on the directrix.
They by definition SP = e PM 2
 2x + y − 1 
⇒ (SP) = e (PM) ⇒ (x – 1) + (y – 2) = 3 
2 2 2 2 2
 ⇒ 5(x2 – y2 – 2x – 4y + 5) = (4x2 + y2 + 1 + 4xy – 2y – 4x)
 4 +1 
⇒ 7x2 – 2y2 + 12x + 14y – 22 = 0
which is the required hyperbola.

Illustration 5: Find the equation of the hyperbola when the foci are at (± 3 5 , 0), and the latus rectum is of
length 8.  (JEE ADVANCED)

Sol: Use the formula for the length of the latus rectum to get a relation between a and b. Then use the foci and the
relation between a and b to get the equation of the hyperbola.
Here foci are at (± 3 5 , 0) ⇒ c= 3 5
2b2
Length of the latus rectum = =8
a
⇒ b2 = 4a … (i)
We know that
c2 = a2 + b2
F1 F2
O
(3 5)2 = a2 + 4a
(-3 5,0) 4 (3 5,0)
45 = a2 + 4a
a2 + 4a – 45 = 0
Figure 12.14
1 2 . 6 | Hyperbola

(a + 9)(a – 5) = 0
a = –9, a = 5 (a cannot be –ve)
Putting a = 5 in (i), we get
b2 = 5 × 4 = 20 ⇒ b2 = 20
Since, foci lie on the x-axis, therefore the equation of the hyperbola is of the form
x2 y2 x2 y 2
− = 1 i.e., − =1
a2 b2 25 20

⇒ 20x2 – 25y2 = 500 ⇒ 4x2 – 5y2 = 100

Which is the required equation of hyperbola.

Illustration 6: Find the equation of the hyperbola when the foci are at (0, ± 10 ), and passing through (2, 3)
 (JEE ADVANCED)
Sol: Start with the standard equation of a hyperbola and use the foci and the point (2 , 3) to find the equation.
Here, foci are at (0, ± 10 )
⇒ c= 10 Here the foci lie at the y-axis.
So the equation of the hyperbola is of the form
y2 x2
−= 1 … (i)
a2 b2
Point (ii, iii) lies on (i).

9 4 9 4 9 b2 + 4 9b2
So − = 1 ⇒ =1+ ⇒ = a2 =  … (ii)
a2 b2 a2 b2 a2 b2 b2 + 4

We know that 
c2 = a2 + b2
9b2
⇒ 10 = + b2 
b2 + 4 (0, 10) F2

9b2 + b 4 + 4b2 (2,3)


⇒ 2
= 10
b +4

⇒ 10b2 + 40 = b4 + 13b2
⇒ b4 + 3b2 – 40 =0
⇒ (b2 + 8) (b2 – 5) = 0
⇒ b2 + 8 = 0, b2 – 5 = 0 F1
(0, 10)
⇒ b2 = –8 & b2 = 5 (b2 = –8 not possible)
⇒ b2 = 5 in (ii), we get Figure 12.15

9 ×5 45
a2 = = =5
5+4 9
Again putting a2 = 5 and b2 = 5 in (i), we get
y 2 x2
− = 1 ⇒ y2 – x2 = 5
5 5
Which is the required equation of the hyperbola.
M a them a ti cs | 12.7

Illustration 7: An ellipse and hyperbola are confocal i.e., having same focus and conjugate axis of hyperbola &
1 1
minor axis of ellipse. If e1 and e2 are the eccentricities of the hyperbola and ellipse then find + .
e1 e22
2

 (JEE ADVANCED)
Sol: Consider the standard equation of an ellipse and hyperbola by taking the
eccentricity as e1 and e2 respectively. Find the relation between the eccentricities
by using the condition that they have the same focus.
x2 y2 x2 y2
Let − 1 and
= + 1⇒
= ae1 = Ae2 and B = b 
a2 b2 A2 B2 e1 e2

⇒ B2 = b2 ⇒ A2(e22 – 1) = a2(1 – e1)

a2e12 1 1
∴ (e22 − 1) = a2(1 – e12) ∴ + =2
e22 e12 e22
Figure 12.16

Illustration 8: Find the equation of a hyperbola if the distance of one of its vertices from the foci are 3 and 1. Find
all the possible equations. (JEE ADVANCED)

Sol: Consider two cases when the major axis is parallel to the X – axis and the minor axis is parallel to the Y-axis
and vice versa.
Case I: ae – a = 1
ae + a = 3 A2 A1
⇒ e = 2 S2 S1
(-ae, 0) (ae, 0)
⇒ a = 1
⇒ b2 = 3
Figure 12.17
x2 y 2
Equation of hyperbola is − =1
1 3
Case II b(e – 1) = 1
b(e + 1) = 3
⇒ e = 2, b = 1, a2 = 3
y 2 x2
Equation of hyperbola is − 1
=
3 1

4. CONJUGATE HYPERBOLA
The hyperbola whose transverse and conjugate axes are respectively the
conjugate and transverse axis of a given hyperbola is called the conjugate B1
hyperbola of the given hyperbola. The hyperbola conjugate to the (0, b) A
A2
hyperbola 1
(-a, 0) (a, 0)
x2 y2 y2 x2 (0,-b)
− = 1 is
=1 −
a2 b2 a2 b2 B2
The eccentricity of the conjugate hyperbola is given by a2 = b2(e2 – 1)
2a2 Figure 12.18
and the length of the latus rectum is
b
Condition of similarity: Two hyperbolas are said to be similar if they have the same value of eccentricity.
Equilateral hyperbola: If a = b or L(T.A.) = L(C.A) then it is an equilateral or rectangular hyperbola.
1 2 . 8 | Hyperbola

5. PROPERTIES OF HYPERBOLA/CONJUGATE HYPERBOLA

x2 y 2 x2 y2
Equation of the Hyperbola − =1 − −1
=
a2 b 2 a2 b2

F1
y
A

x’ x x x’
A’ O A O
Figure F1 F2

y’ A’

Figure 12.19 F2

y’

Figure 12.20

Centre (0, 0) (0, 0)

Vertices (±a, 0) (0, ±b)

Transverse axis 2a 2b

Conjugate axis 2b 2a

Relation between a, b, c c2 = a2 + b2 c2 = a2 + b2

Foci (±c, 0) (0, ±c)

c c
Eccentricity e= e‘=
a b

Length of latus rectum


2b 2 2a2
a b

PLANCESS CONCEPTS

•• If e1 and e2 are the eccentricities of a hyperbola and its conjugate hyperbola then

1 1
+ 1
=
e12 e22
b2 a2
e12 = 1 + e22 = 1 +
a2 b2

1 1
∴ + =1
e12 e22
•• The foci of a hyperbola and its conjugate hyperbola are CONCYCLIC and form vertices of square.
M a them a ti cs | 12.9

(0,be) F3

(0, b)
F2 F1
(-ae,0) (ae,0)

F4
(0,-be)

Figure 12.21

Anvit Tawar (JEE 2009, AIR 9)

6. AUXILIARY CIRCLE
A circle described on the transverse axis as diameter is an auxiliary circle and its equation is x2 + y2 = a2
Any point of the hyperbola is P ≡ (a sec θ, b tan θ)
P, Q are called corresponding point and θ is eccentric angle of P.

P(a sec, b tan


Q
a
 N
(-a,0) (a,0)

2 2 2
x +y =a

Figure 12.22

PLANCESS CONCEPTS

1. If O ∈ (0, π/2), P lies on upper right branch.


2. If O ∈ (π/2, π), P lies on upper left branch.
3. If O ∈ (π, 3π/2), P lies on lower left branch.
4. If O ∈ (3π/2, 2π), P lies on lower right branch.
Vaibhav Krishnan (JEE 2009, AIR 22)

7. PARAMETRIC COORDINATES
x2 y2
Let P(x, y) be any point on the hyperbola = 1. Draw PL perpendicular from P on OX and then a tangent

a2 b2
LM from L to the circle described on A′A as diameter.
1 2 . 1 0 | Hyperbola

Then, x = CL = CM sec θ = a sec θ


x2 y2
Putting x = a sec θ in − = 1, we obtain y = b tan θ
a2 b2
x2 y2
Thus, the coordinates of any point on the hyperbola = 1 are (a sec θ, b tan θ), where θ is the parameter

a2 b2
such that 0 ≤ θ ≤ 2π. These coordinates are known as the parametric coordinates. The parameter θ is also called the
eccentric angle of point P on the hyperbola.
x2 y2
The equation x = a sec θ and y = b tan θ are known as the parametric equations of the hyperbola − = 1.
a2 b2
Note: (i) The circle x2 + y2 = a2 is known as the auxiliary circle of the hyperbola.
x2 y2
Let P (a sec θ1, b tan θ1) and Q (a sec θ2, b tan θ2) be two points on the hyperbola − = 1.
a2 b2
Then the equation of the chord PQ is
b tan θ2 − b tan θ1 x  θ − θ2  y  θ1 + θ2   θ1 + θ2 
y – b tan q1 = (x – a sec q1) ⇒ cos  1  − sin   = cos  
asec θ2 − asec θ1 a  2  b  2   2 

Illustration 9: Find the eccentricity of the hyperbola whose latus rectum is half of its transverse axis. (JEE MAIN)

Sol: Establish the relation between a and b and then use the eccentricity formula.
x2 y2 2b2
Let the equation of the hyperbola be − 1 . Then transverse axis = 2a and latus rectum =
=
a2 b2 a
2
2b 1
According to the question = (2a)
a 2
⇒ 2b2 = a2 ⇒ 2a2(e2 – 1) = a2 ⇒ 2e2 – 2 = 1 ⇒ e2 = 3/2 ∴e= 3/2

Illustration 10: If the chord joining two points (a sec θ1, b tan θ1) and (a sec θ2, b tan θ2) passes through the focus
x2 y 2 θ θ 1−e
of the hyperbola − = 1, then prove that tan 1 tan 2 = . (JEE ADVANCED)
a 2
b 2 2 2 1+e

Sol: Obtain a relation between the two given eccentric angles by substituting y
the point in the equation of chord. P(x,y)
The equation of the chord joining (a sec θ1, b tan θ1) and (a sec θ2, b tan θ2) is M

x  θ − θ2  y  θ1 + θ2   θ1 + θ2  x’ L x
cos  1  − sin 
  = cos 
  C(0,0) A (x,0)
a  2  b  2   2 
(-x,0) A

 θ − θ2   θ1 + θ2 
If it passes through the focus (ae, 0) then e cos  1  = cos  
 2   2  y’
cos ( (θ1 − θ2 ) / 2 )
⇒ =1/e
cos ( (θ1 + θ2 ) / 2 ) Figure 12.23

θ1 θ2 1−e
using componendo dividendo rule we get tan tan = .
2 2 1+e

8. POINT AND HYPERBOLA


x2 y2 x12 y12
The point (x1, y1) lies outside, on or inside the hyperbola − = 1 according to − –1 ‘<’ or ‘=’ or ‘>’ 0
a2 b2 a2 b2
Proof: Draw PL perpendicular to x-axis. Suppose it cuts the hyperbola at Q(x1, y2).
M a them a ti cs | 12.11

Clearly, PL > QL
y12 y 22 y12 y 22 x12 y12 x12 y 22 x12 y12
⇒ y1 > y2 ⇒ > ⇒ − <− ⇒ − < − ⇒ − <1
b2 b2 b2 b2 a2 b2 a2 b2 a2 b2
 x2 y 2 
 Q(x1 , y 2 ) lies on 2 − 2 = 1
x12 y12  a b 
⇒ − – 1 < 0
a2 b2  2
x1 y 22 
 − =1 
 a2 b2 

x2 y2 x12 y12
Thus the point (x1, y1) lies outside the hyperbola − = 1. If − –1<0
a2 b2 a2 b2
Similarly, we can prove that the point (x1, y1) will lie inside or on the hyperbola according to

x12 y12
− – 1 > 0 or, = 0.
a2 b2
x12 y12
P lies outside/on/inside − – 1 < 0/ = 0 / > 0
a2 b2

Illustration 11: Find the position of the points (7, –3) and (2, 7) relative to the hyperbola 9x2 – 4y2 = 36.
 (JEE MAIN)
Sol: Use the concept of position of a point w.r.t. the hyperbola.
x2 y 2
The equation of the given hyperbola is 9x2 – 4y2 = 36 or, − = 1. Now,
4 9
72 ( −3)2 41 22 72 49 −49
− −1= > 0 and, − ⇒ 1– ⇒ 1= < 0.
4 9 4 4 9 9 9
Hence, the point (7, –3) lies inside the parabola whereas the point (2, 7) lies outside the hyperbola.

Illustration 12: Find the position of the point (5, –4) relative to the hyperbola 9x2 – y2 = 1.  (JEE MAIN)

Sol: Use the concept of position of a point


Since 9(5)2 – (4)2 = 1 = 225 – 16 – 1 = 208 > 0. So the point (5, –4) inside the hyperbola 9x2 – y2 = 1.

9. LINE AND HYPERBOLA


x2 y2
Consider a line y = mx + c and hyperbola − = 1.
a2 b2
x2 y2
Solving y = mx + c and − =1
a2 b2
⇒ b2x2 – a2(mx + c)2 = a2b2 ⇒ (b2 – a2m2) x2 – 2a2cmx – a2(b2 + c2) = 0;

D>0 D=0 D<0


Secant Tangent Neither secant
2
c = a m –b 2 2 2 nor tangent

condition of tangency
1 2 . 1 2 | Hyperbola

x2 y2
⇒ y = mx + a2m2 − b2 is tangent to the hyperbola − =1.
a2 b2

PLANCESS CONCEPTS

No. of tangents drawn to a hyperbola passing through a given point (h, k)


Let y = mx+c be tangent to the hyperbola
⇒ c2 = a2m2 – b2
(h,k)
Since line passes through (h, k)
⇒ (k – mh)2 = a2m2 – b2 
⇒ (h2 – a2)m2 – 2hkm + k2 + b2 = 0
Hence a maximum of 2 tangents can be drawn to the
hyperbola passing through (h, k)
2hk k 2 + b2 Figure 12.24
m1 + m 2 = m1m2 =
2 2 2 2
h −a h −a

if m1m2 = – 1 x2 + y 2 = a2 − b2
Shrikant Nagori (JEE 2009, AIR 30)

Illustration 13: Common tangent to y2 = 8x and 3x2 – y2 = 3. (JEE MAIN)

Sol: Start with the standard equation of a tangent to a parabola and apply the condition for it be a tangent to
3x2 – y2 = 3.
2 x2 y 2
Tangent to the parabola is of the form y = mx + . For this line to be tangent to − 1 c2 = a2m2 – b2
=
m 1 3
4
⇒ = m2 – 1 ⇒ m2 = 4 ∴ ± y = 2x + 1 are the common tangents.
2
m

10. TANGENT
x2 y2 xx1 yy1
Point Form: The equation of tangent to the hyperbola − = 1 at (x1, y1) is
= 1. −
2 2 2 2
a b a b
x2 y 2
Slope Form: The equation of tangents of slope m to the hyperbola − = 1 are given by
a2 b2
y = mx ± a2m2 − b2
 a2m b2 
The coordinates of the points of contact are  ± ,± 
 
 a2m2 − b2 a2m2 − b2 

x2 y2 x y
Parametric Form: The equation of a tangent to the hyperbola − = 1 at (a sec θ, b tan θ) is sec θ − tan θ =1
a 2
b 2 a b
Note:
(i) The tangents at the point P (a sec θ1, b tan θ1) and Q (a sec θ2, b tan θ2) intersect at the point R
 acos((θ1 − θ2 ) / 2) bsin((θ1 + θ2 ) / 2) 
 ,  .
 cos((θ1 + θ2 ) / 2) cos((θ1 + θ2 ) / 2) 

(ii) If | θ1 + θ2 | = π, then the tangents at these points (θ1 & θ2) are parallel.
M a them a ti cs | 12.13

(iii) There are two parallel tangents having the same slope m. These tangents touch the hyperbola at the extremities
of a diameter.
x2 y2
(iv) Locus of the feet of the perpendicular drawn from focus of the hyperbola − = 1 upon any tangent is its
a2 b2
auxiliary circle i.e. x + y = a and the product of these perpendiculars is b2.
2 2 2

(v) The portion of the tangent between the point of contact & the directrix subtends a right angle at the
corresponding focus.
(vi) The foci of the hyperbola and the points P and Q in which any tangent meets the tangents at the vertices are
concyclic with PQ as the diameter of the circle.

x2 y 2
Illustration 14: Prove that the straight line lx + my + n = 0 touches the hyperbola − = 1 if a2l2 – b2m2 = n2.
2 2
 a b (JEE MAIN)
Sol: Apply the condition of tangency and prove the above result.
The given line is lx + my + n = 0 or y = –l/m x – n/m
Comparing this line with y = Mx + c ∴M = – l/m and c = –n/m ….(i)

x2 y2
This line (i) will touch the hyperbola − = 1 if c2 = a2M2 – b2
a2 b2

n2 a2l2
⇒ = – b2 or a2l2 – b2m2 = n2. Hence proved.
m2 m2

Illustration 15: Find the equations of the tangent to the hyperbola x2 – 4y2 = 36 which is perpendicular to the line
x – y + 4 = 0.  (JEE MAIN)

Sol: Get the slope of the perpendicular line and use it to get the equation of the tangent.
Let m be the slope of the tangent. Since the tangent is perpendicular to the line x – y = 0
m × 1 = –1
⇒ m = –1
x2 y 2
Since x2 – 4y2 = 36 or − =1
36 9
x2 y2
Comparing this with − =1 ∴ a2 = 36 and b2 = 9
a2 b2
So the equation of the tangents are y = (–1)x ± 36 × ( −1)2 − 9
⇒ y = –x ± 27 ⇒ x+y± 3 3 =0

x2 y2
Illustration 16: If two tangents drawn from any point on hyperbola x2 – y2 = a2 – b2 to the ellipse + = 1 make
a2 b2
angles θ1 and θ2 with the axis then tan θ1 . tan θ2 .  (JEE ADVANCED)

Sol: Establish a quadratic in m, where m is the slope of the two tangents. Then use the sum and product of the
roots to find tan θ1 . tan θ2 .
Let c2 = a2 – b2
Any tangent to the ellipse y = mx ± a2m2 + b2

c tan θ = mc secθ ± a2m2 + b2 )


1 2 . 1 4 | Hyperbola

c2(tan θ – m secθ)2 = a2m2 + b2


(c2 sec2θ – a2)m2 + (……)m + c2tan2θ – b2 = 0 ⇒ tanθ1.tanθ2 =product of the roots.

c2 tan2 θ − b2
= = 1.
c2 sec2 θ − a2

11. NORMAL
x2 y2 a2 x b2 y
Point Form: The equation of the normal to the hyperbola − = 1 at (x1, y1) is + = a2 + b2.
a2 b2 x1 y1

x2 y 2
Parametric Form: The equation of the normal at (a sec θ, b tan θ) to the hyperbola − = 1 is a x cosθ + b y
cotθ = a2 + b2. a2 b2

x2 y2
Slope Form: The equation of a normal of slope m to the hyperbola − = 1 is given by
a2 b2
m(a2 + b2 )  a2 b2m 
y = mx  at the points  ± , 
 
a2 − b2m2  a2 − b2m2 a2 − b2m2 

Note:

(i) At most four normals can be drawn from any point to a hyperbola.
(ii) Points on the hyperbola through which, normal through a given point pass are called co-normal points.
(iii) The tangent & normal at any point of a hyperbola bisect the angle between the focal radii. This illustrates the
reflection property of the hyperbola as “An incoming light ray” aimed towards one focus is reflected from
the outer surface of the hyperbola towards the other focus. It follows that if an ellipse and a hyperbola have
the same foci, they cut at right angles at any of their common points.

Light Ray
Y Tangent
Q 
P

X
S’ S

Figure 12.25

x2 y2 x2 y2
(iv) The hyperbola − = 1 and the hyperbola − = 1 (a > k > b > 0) are confocal and therefore
a2 b2 a2 − k 2 k 2 − b2
orthogonal.
(v) The sum of the eccentric angles of co-normal points is an odd multiple of π.
x2 y2
(vi) If θ1, θ2 and θ3 are eccentric angles of three points on the hyperbola − = 1. The normals at which are
a2 b2
concurrent, then sin(θ1 + θ2) + sin(θ2 + θ3) + sin(θ3 + θ1) =0

x2 y2
(vii) If the normals at four points P(x1, y1), Q(x2, y2), R(x3, y3) and S(x4, y4) on the hyperbola − = 1 are concurrent,
a2 b2
1 1 1 1 
then (x1 + x2 + x3 + x4)  + + +  = 4.
 x1 x2 x3 x 4 
M a them a ti cs | 12.15

x2 y2
Illustration 17: How many real tangents can be drawn from the point (4, 3) to the hyperbola − = 1. Find the
equation of these tangents and the angle between them. 16 9 (JEE MAIN)

Sol: Use the concept of Position of a Point w.r.t. the hyperbola to find the number of real tangents.
Given point P = (4, 3)
x2 y 2
Hyperbola S ≡ − =1=0
16 9
16 9
∵ S1 ≡ – – 1 = -1 < 0
16 9
⇒ Point P ≡ (4, 3) lies outside the hyperbola
∴ Two tangents can be drawn from the point P(4, 3). Equation of a pair of tangents is SS1 = T2.
2
 x2 y 2   4x 3y 
⇒  − − 1  (–1) ≡  − − 1
 16 9   16 9 
 
2
x 2
y x 2
y2 xy x 2y 4
⇒ − + +1= + +1– + ⇒ 3x2 – 4xy – 12x + 16y = 0 and θ = tan–1  
16 9 16 9 6 2 3 3

Illustration 18: Find the equation of common tangents to hyperbolas


x2 y2 y2 x2
H1: − = 1 ; H2: − = 1 (JEE MAIN)
a2 b2 a2 b2
Sol: Compare the equation of the common tangents to H1 and H2 and compare the two equations to find the value
of m.
Tangent to H1

y = mx ± a2m2 − b2

x2 y2
H2: − =1
( −b2 ) ( −a2 )
a2m2 – b2 = (–b2) m2 – (–a2)
∴ a2(m2 – 1) = b2(1 – m2)
m=±1 Figure 12.26

Equation of common tangents are ± y = x + a2 − b2

Illustration 19: If the normals at (xr, yr); r = 1, 2, 3, 4 on the rectangular hyperbola xy = c2 meet at the point Q(h, k),
prove that the sum of the ordinates of the four points is k. Also prove that the product of the ordinates is –c4.
 (JEE ADVANCED)

Sol: Write the equation of the normal in the parametric form and then use the theory of equations.
 c
Any point on the curve xy = c2 is  ct, 
 t
 c
The equation of the normal to the hyperbola at the point  ct,  is
 t
c −1
y– = (x – ct).
t  dy 
  c
 dx ct,
t

c2 dy −c2
Here, xy = c2 ; or y = ∴ =
x′ dx x2
1 2 . 1 6 | Hyperbola

 dy  c2 1
∴   = 2 2 = − 2
 dx ct, c c t t
t
 c
∴ The equation of the normal at  ct,  is
 t
c
y– = t2(x – ct) or ty – c = t3(x – ct) or ct4 – t3x + ty – c = 0
t
The normal passes through (h, k). So
ct4 – t3h + tk – c = 0  … (i)
c
Let the roots of (i) be t1, t2, t3, t4. Then xr = ct, yr =
tr
∴ sum of ordinates = y1 + y2 + y3 + y4

c c c c t t t +t t t +t t t +t t t
= + + + = c 2 3 4 3 4 1 4 1 2 1 2 3
t1 t2 t3 t 4 t1 t2 t3 t 4

−k / c
=c. = k, {from roots of the equation (i)} and, product of the ordinates
−c / c
c c c c c4 c4
= y1y2y3y4 = . . . = = = –c4.
t1 t2 t3 t 4 t1 t2 t3 t 4 −c / c

Hence proved.

x2 y2
Illustration 20: The perpendicular from the centre on the normal at any point of the hyperbola − = 1 meet
a2 b2
at R. Find the locus of R. (JEE ADVANCED)

Sol: Solve the equation of the normal and the equation of line perpendicular to it passing through the origin.
Let (x1, y1) be any point on the hyperbola.

x12 y12
So, − = 1 … (i)
a2 b2
x − x1 y − y1 x1 y1
The equation of the normal at (x1, y1) is = or (y − y1 ) + (x − x1 ) = 0 … (ii)
x1 y1 a 2
b2

a2 y1 a2 b2
‘m’ of the normal = −
b2 x1

∴ The equation of the perpendicular from the centre (0, 0) on (ii) is


b2 x1
y= .x … (iii)
a2 y1
The intersection of (ii) and (iii) is R and the required locus is obtained by eliminating x1, y1 from (i), (ii) and (iii).
x1 y1
From (iii), =
= t (say)
a y b2 x
2

Putting in (ii), yt(y – b2xt) + xt(x – a2yt) = 0


or (x2 + y2)t – (a2 + b2)xyt2 = 0.
But t ≠ 0 for then (x1, y1) = (0, 0) which is not true.

x2 + y 2 x2 + y 2 a2 (x2 + y 2 )
∴ t= ; ∴ x1 = a2 y =
xy(a2 + b2 ) xy(a2 + b2 ) x(a2 + b2 )
M a them a ti cs | 12.17

x2 + y 2 b2 (x2 + y 2 )
and y1 = b2 x =
xy(a2 + b2 ) y(a2 + b2 )

1 a4 (x2 + y 2 )2 1 b 4 (x2 + y 2 )2
∴ from (i), . − . =1
a2 x2 (a2 + b2 ) b2 y 2 (a2 + b2 )2

 a2 b2 
or {x2 + y2)2.  −  = (a2 + b2)2.
 x2 y 2 
 

x2 y2
Illustration 21: A normal to the hyperbola = 1 meets the axes in M and N and lines MP and NP are

a2 b2
drawn perpendicular to the axes meeting at P. Prove that the locus of P is the hyperbola a2x2 – b2y2 = (a2 + b2)2.

 (JEE ADVANCED)

Sol: Find the co-ordinates of the point M and N and then eliminate the parameter between the ordinate and
abscissae.
x2 y 2
The equation of normal at the point Q(a sec φ, b tan φ) to the hyperbola − = 1 is
a2 b2
ax cosφ + by cot φ = a2 + b2  … (i)
 a2 + b2   a2 + b2 
The normal (i) meets the x-axis in M  sec ϕ, 0  and y-axis in N  0, tan ϕ 
 a   b 
   
∴ Equation of MP, the line through M and perpendicular to axis, is

 a2 + b2  ax
x=   sec ϕ or sec φ = 2  … (ii)
 a  (a + b2 )
 
and the equation of NP, the line through N and perpendicular to the y-axis is

 a2 + b2  by
y=   tan φ or tan φ = 2  … (iii)
 b  (a + b2 )
 
The locus of the point is the intersection of MP and NP and will be obtained by eliminating φ from (ii) and (iii), so
we have sec2φ – tan2φ = 1

a2 x2 b2 y 2
⇒ − = 1 or a2x2 – b2y2 = (a2 + b2)2 is the required locus of P.
(a2 + b2 )2 (a2 + b2 )2

Illustration 22: Prove that the length of the tangent at any point of hyperbola intercepted between the point of
contact and the transverse axis is the harmonic mean between the lengths of perpendiculars drawn from the foci
on the normal at the same point.  (JEE ADVANCED)

Sol: Proceed according to the question to prove the above statement.


P1 S1G e2 x1 − ae ae2 − aecos θ
= = =
P TG e2 x1 − acos θ ae2 − acos2 θ

P1 e(e − cos θ) P e + cos θ cos θ


∴ = ⇒ = =1+
P (e − cos θ)(e + cos θ) P1 e e

P cos θ P P 1 P 2
Similarly we get = 1− ∴ + =2 ⇒ + =
P1 e P1 P2 P1 P2 P
Hence Proved.
1 2 . 1 8 | Hyperbola

12. DIRECTOR CIRCLE


The locus of the intersection point of tangents which are at right angles is known as the Director Circle of the
hyperbola. The equation to the director circle is: x2 + y2 = a2 – b2.
If b2 < a2 this circle is real.
If b2 = a2 (rectangular hyperbola) the radius of the circle is zero & it reduces to a point circle at the origin. In this
case the centre is the only point from which the tangents at right angles can be drawn to the curve.
If b2 > a2, the radius of the circle is imaginary, so that there is no such circle & so no pair of tangents at right angles
can be drawn to the curve.
Or we can say that
If L(T.A) > L(C.A) ⇒ circle is real.
If L(T.A) < L(C.A) ⇒ No real locus, Imaginary circle.
If L(T.A) = L(C.A) ⇒ point circle

13. CHORD

13.1 Chord of Contact


It is defined as the line joining the point of intersection of tangents drawn from any point. The equation to the
x2 y 2 xx yy
chord of contact of tangent drawn from a point P(x1, y1) to the hyperbola − = 1 is 1 − 1 = 1.
2 2 2
a b a b2
13.2 Chord Bisected at a Given Point
x2 y2 xx1 yy1 x12 y12
The equation of the chord of the hyperbola − = 1, bisected at the point (x1, y1) is − –1= −
a2 b2 a2 b2 a2 b2
– 1 (T = S1' ), where T and S1' have their usual meanings.

13.3 Chord of Hyperbola (Parametric Form)


x α+β y α+β α −β
Note: Chord of ellipse cos   + sin   = cos  
a  2  b  2   2   Q()
For a hyperbola it is P()

x α −β y α+β α+β


cos   − sin  =cos  
a  2  b  2   2 
d α −β α+β Figure 12.27
Passing through (d, 0) cos   = cos  
a  2   2 
d cos ( (α + β) / 2 )
=
a cos ( (α − β) / 2 )

d+a −2cos α / 2 cos β / 2


=
d−a 2cos α / 2 sin β / 2

a−d α β
= tan tan
a+d 2 2
1−e α β
if d = ae ⇒ = tan tan
1+e 2 2
M a them a ti cs | 12.19

PLANCESS CONCEPTS

Point of intersection of tangents at P(α) and Q(β) can be obtained by


comparing COC with the chord at P(α) & Q(β) P()
Q()
Equation of PQ
xh yk
COC ⇒ − =1
2
a b2
x α −β y α+β α+β
PQ ⇒ cos   − sin   = cos  
a  2  b  2   2  R(h,k)
cos ( (α − β) / 2 ) sin ( (α + β) / 2 ) Figure 12.28
∴ h=a , k =b
cos ( (α + β) / 2 ) cos ( (α + β) / 2 )

Nitish Jhawar (JEE 2009, AIR 7)

x2 y 2
Illustration 23: If tangents to the parabola y2 = 4ax intersect the hyperbola − = 1 at A and B, then find the
locus of point of intersection of tangents at A and B.  a2 b2 (JEE MAIN)

Sol: The point of intersection of the tangents at A and B is nothing but the point for which AB is the chord of
contact. Use this information to find the locus.
Let P ≡ (h, k) be the point of intersection of tangent at A and B
xh yk
∴ Equation of the chord of contact AB is = 1  − ….(i)
2
a b2
Which touches the parabola. Equation of the tangent to the parabola y2 = 4ax
y = mx – a/m ⇒ mx – y = –a/m ….(ii)
equation (i) and (ii) must be same
m −1 −a / m h b2 ak
∴ = = ⇒ m= and m = −
((h / a )) ( −(k / b ))
2 2 1 k a2 b2

hb2 ak b4
∴ = − ⇒ locus of P is y2 = – x.
ka2 b2 a3

Illustration 24: A point P moves such that the chord of contact of a pair of tangents from P to y2 = 4x touches the
rectangular hyperbola x2 – y2 = 9. If locus of ‘P’ is an ellipse, find e.  (JEE MAIN)

Sol: Write the equation of the chord of contact to the parabola w.r.t. a point (h , k). Then solve this equation with
the equation of the hyperbola.
A
P (h,k)

Figure 12.29

2x 2h 4h2 4
yy1 = 2a(x + x1) ; yk = 2(x + h) ⇒ y= + ; = 9. –9
k k k 2
k2
1 2 . 2 0 | Hyperbola

x2 y 2 4 5
4h = 36 – 9k
2 2 + 1
= e2 = 1 – e=
9 4 9 3

x2 y2
Illustration 25: Find the locus of the mid-point of focal chords of the hyperbola = 1.  (JEE MAIN) −
a2 b2
Sol: Use the formula T = S1 to get the equation of the chord and substitute the co-ordinates of the focus.
Let P ≡ (h, k) be the mid-point
xh yk h2 k2
∴ Equation of the chord whose mid-point (h, k) is given − –1= − – 1 since it is a focal chord.
2 2 2
a b a b2
∴ It passes through the focus, either (ae, 0) or (–ae, 0)
ex x2 y 2
∴ Locus is ± = −
a a2 b2

x2 y2
Illustration 26: Find the condition on ‘a’ and ‘b’ for which two distinct chords of the hyperbola − =1
2a2 2b2
passing through (a, b) are bisected by the line x + y = b. (JEE ADVANCED)

Sol: Consider a point on the line x + y = b and then find a chord with this point as the mid-point. Then substitute
the point in the equation of the chord to get the condition between ‘a’ and ‘b’.
Let the line x + y = b bisect the chord at P(α, b – α)
∴ Equation of the chord whose mid-point is P(α, b – α) is:

xα y(b − α ) α2 (b − α )2
− = −
2a2 2b2 2a2 2b2
α (b − α ) α2 (b − α )2
Since it passes through (a, b) ∴ − = −
2a 2b 2a2 2b2
1 1  1 1
α2  − + α −  = 0 ⇒ a = b
2 
b a
2
a b 

x2 y 2
Illustration 27: Locus of the mid points of the focal chords of the hyperbola − = 1 is another hyperbola
whose eccentricity is e. a2 b2 (JEE ADVANCED)

Sol: Use the formula T = S1 and proceed further.


A
xh yb h2 k2
T = S1 ; − = −
a2 b2 a2
b2 m(h,k)
eh h2 k 2 g(ae,0)
It passes through focus ⇒ = −
a a2 b2
x2 ex y2 1 2 y2 P
⇒ − = ⇒ [x − eax] =
a2 a b2 a2 b2
Figure 12.30
( x − (ea / 2))
2
1  e2a2  y 2
2 2 2
ea  y e
⇒  x −  − = ⇒ − =
a2  2  4  b2 a 2
b 42

Hence the locus is a hyperbola of eccentricity e.
x2 y2
Illustration 28: Find the locus of the midpoint of the chord of the hyperbola − = 1 which subtends a right
a2 b2
angle at the origin. (JEE ADVANCED)
M a them a ti cs | 12.21

Sol: Use the formula T = S1 and then homogenise the equation of the hyperbola using the equation of the chord
to find the locus.
Let (h, k) be the mid-point of the chord of the hyperbola. Then its equation is
hx ky h2 k2
− =  − … (i)
a2 b2 a2 b2
The equation of the lines joining the origin to the points of intersection of the hyperbola and the chord (i) is
obtained by making a homogeneous hyperbola with the help of (i)

((hx / a ) − (ky / b ))
2
2 2
x 2
y2
∴ − =
((h / a ) − (k / b ))
2
a2 b2 2 2 2 2

2 2
1  h2 k 2  2 1  h2 k 2  2 h2 2 k 2 2 2hk
⇒  −  x − 2  2 − 2  y = 4 x + 4 y − 2 2 xy  … (ii)
a2  a2 b2  b  a b  a b ab

The lines represented by (ii) will be at right angles if the coefficient of x2 + the coefficient of y2 = 0
2 2 2
1  h2 k 2  h2 1  h2 k 2  k2  h2 k 2   1 1  h2 k 2
⇒  −  − −  −  − =0 ⇒  −  − = +
a2  a2 b2  a4 b2  a2 b2  b 4  a2 b2   a2 b2  a4 b 4
 
2
 x2 y 2  1 1  x2 y 2
hence, the locus of (h, k) is  −   2 − 2 = 4 + 4
 a2 b 4  a b  a b
 

14. DIAMETER
The locus of the mid-points of a system of parallel chords of a hyperbola is called a diameter. The point where a
diameter intersects the hyperbola is known as the vertex of the diameter.

14.1 Equation of Diameter


The equation of a diameter bisecting a system of parallel chords of slope m of the hyperbola

x2 y2 b2
− = 1 is y = x.
a2 b2 a2m

14.2 Conjugate Diameters


Two diameters of a hyperbola are said to be conjugate diameters if each bisects the chords parallel to the other.
x2 y2
Let y = m1x and y = m2x be conjugate diameters of the hyperbola − = 1.
a2 b2
Then, y = m2x bisects the system of chords parallel to y = m1x. So, its equation is

b2
y= x … (i)
a2m
b2 b2
Clearly, (i) and y = m2x represent the same line. Therefore, m2 = ⇒ m 1m 2 =
a2m1 a2
x2 y2 b2
Thus, y = m1x and y = m2x are conjugate diameters of the hyperbola − = 1, if m1m2 =
a2 b2 a2
1 2 . 2 2 | Hyperbola

PLANCESS CONCEPTS

•• In a pair of conjugate diameters of a hyperbola, only one meets the hyperbola on a real point.

x2 y2
•• Let P(a sec θ, b tan θ) be a point on the hyperbola = 1 such that CP and CD are conjugate

a2 b2
diameters of the hyperbola. Then, the coordinates of D are (a tan θ, b sec θ)

•• If a pair of conjugate diameters meet the hyperbola and its conjugate in P and D respectively then
CP2 – CD2 = a2 – b2. 
Shivam Agarwal (JEE 2009, AIR 27)

15. POLE AND POLAR


Let P(x1 , y1 ) be any point inside the hyperbola. A chord through P intersects the hyperbola at A and B respectively.
If tangents to the hyperbola at A and B meet at Q(h, k) then the locus of Q is called the polar of P with respect to
the hyperbola and the point P is called the pole.
If P(x1 , y1 ) is any point outside the hyperbola and tangents are drawn, then the line passing through the contact
points is polar of P and P is called the pole of the polar.
Note: If the pole lies outside the hyperbola then the polar passes through the hyperbola. If the pole lies inside the
hyperbola then the polar lies completely outside the hyperbola. If pole the lies on the hyperbola then the polar
becomes the same as the tangent.

x2 y 2
Equation of polar: Equation of the polar of the point (x1 , y1 ) with respect to the hyperbola − = 1 is given by
xx1 yy1 a2 b2
− = 1 , i.e., T = 0
a2 b2
x2 y 2  −a2l b2m 
Coordinates of Pole: The pole of the line lx + my + n = 0 with respect to hyperbola + =1 is P  , .
a2 b2  n n 

Properties of pole and polar:

 1. If the polar of P (x1 , y1 ) passes through Q(x2 , y 2 ) , then the polar of Q(x2 , y 2 ) goes through P (x1 , y1 ) and such
x x y y
points are said to be conjugate points. Condition for conjugate points is 1 2 − 1 2 = 1.
a2 b2
 2. If the pole of line l1 x + m1 y + n1 = 0 lies on another line l2 x + m2 y + n2 =0 , then the pole of the second line will
lie on the first and such lines are said to be conjugate lines.
 3. Pole of a given line is the same as the point of intersection of the tangents at its extremities.
 4. Polar of focus is its directrix.

16. ASYMPTOTES
An asymptote to a curve is a straight line, such that distance between (0,b)
the line and curve approaches zero as they tend to infinity.
In other words, the asymptote to a curve touches the curves at infinity
(-a,0) (a,0)
i.e. asymptote to a curve is its tangent at infinity.
The equations of two asymptotes of the hyperbola
(0,-b)
x2 y2 b x y
− = 1 are y = ± x or ± = 0
a2
b 2 a a b Figure 12.31
M a them a ti cs | 12.23

x2 y2
Combined equation of asymptote = 0

a2 b2
Note: If the angle between the asymptotes of the hyperbola is θ, then its eccentricity is sec θ.

PLANCESS CONCEPTS
x2 y2 x2 y2
•• The combined equation of the asymptotes of the hyperbola − = 1 is − = 0.
a2 b2 a2 b2
•• When b = a, the asymptotes of the rectangular hyperbola x2 – y2 = a2 are y = ±x, which are at right
angles.
•• A hyperbola and its conjugate hyperbola have the same asymptotes.
•• The equation of the pair of asymptotes differ from the hyperbola and the conjugate hyperbola by the
same constant, i.e. Hyperbola – Asymptotes = Asymptotes – Conjugate hyperbola
•• The asymptotes pass through the centre of the hyperbola.
•• The bisectors of the angles between the asymptotes are the coordinates axes.
•• The asymptotes of a hyperbola are the diagonals of the rectangle formed by the lines drawn through
the extremities of each axis parallel to the other axis.
•• Asymptotes are the tangents to the hyperbola from the centre.
x2 y 2
•• The tangent at any point P on − = 1 with the centre C meets asymptotes at Q, R and cut off
2
∆CQR of constant area = ab. a b2

•• The parts of the tangent intercepted between the asymptote is bisected at the point of contact.
•• If f(x, y) = 0 is an equation of the hyperbola then the centre of the hyperbola is the point of intersection
∂f ∂f
of = 0 and = 0.
∂x ∂y
Ravi Vooda (JEE 2009, AIR 71)

Illustration 29: Find the asymptotes of xy – 3y – 2x = 0. (JEE MAIN)

Sol: Proceed according to the definition of asymptotes.


Since the equation of a hyperbola and its asymptotes differ in constant terms only
∴ Pair of asymptotes is given by xy – 3y – 2x + λ = 0 ...(i)
where λ is any constant such that represents two straight lines
∴ abc + 2fgh – af2 – bg2 – ch2 = 0
⇒ 0 + 2x – 3/2x – 1 + 1/2 – 0 – 0 – λ (1/2)2 = 0
∴ λ=6
From (i) the asymptotes of given hyperbola are given by xy – 3y – 2x + 6 = 0 or (y – 2) (x – 3) = 0
∴ Asymptotes are x – 3 = 0 and y – 2 = 0

Illustration 30: Find the equation of that diameter which bisects the chord 7x + y – 20 = 0 of the hyperbola
x2 y 2
− = 1. (JEE ADVANCED)
3 7
1 2 . 2 4 | Hyperbola

Sol: Consider a diameter y = mx and solve it with the equation of the hyperbola to form a quadratic in x. Find the
midpoint of the intersection of the chord and hyperbola. Use this point to find the slope of the diameter.
The centre of the hyperbola is (0, 0). Let the diameter be y = mx … (i)
The ends of the chord are found by solving
7x + y – 20 = 0  … (ii)
2 2
x y
and − = 1  … (iii)
3 7
x2 1
Solving (ii), (iii) we get − (20 − 7x)2 = 1
3 7
or 7x2 – 3(400 – 280x + 49x2) = 21 or 140x2 – 840x + 1221 = 0
Let the roots be x1, x2
840
Then x1 + x2 = = 6  ... (iv)
140

If (x1, y1), (x2, y2) be ends then 7x1 + y1 – 20 = 0, 7x2 + y2 – 20 = 0

Adding, 7(x1 + x2) + (y1 + y2) – 40 = 0

or 42 + y1 + y2 – 40 = 0, using (iv) ; ∴ y1 + y2 = –2

 x + x 2 y1 + y 2   6 −2 
∴ The middle point of the chord =  1 , =  ,  = (3, –1)
 2 2  2 2 
1 1
This lies on (i). So –1 = 3m ; ∴ m = − ∴ the equation of the diameter is y = − x.
3 3

Illustration 31: The asymptotes of a hyperbola having centre at the point (1, 2) are parallel to the lines
2x + 3y = 0 and 3x + 2y = 0. If the hyperbola passes through the point (5, 3) show that its equation is (2x + 3y – 8)
(3x + 2y + 7) = 154. (JEE ADVANCED)

Sol: With the information given, find out the equation of the asymptotes and then use the fact that the point (5, 3)
lies on the hyperbola to find the equation of the hyperbola.
Let the asymptotes be 2x + 3y + λ = 0 and 3x + 2y + µ = 0. Since the asymptote passes through (1, 2) then λ = –8
and µ = –7
Thus the equation of the asymptotes are 2x + 3y – 8 = 0 and 3x + 2y – 7 = 0
Let the equation of the hyperbola be (2x + 3y – 8) (3x + 2y – 7) + v =0 … (i)
It passes through (5, 3), then (10 + 9 – 8) (15 + 6 – 7) + v = 0
⇒ 11 × 14 + v = 0
∴ v = – 154
putting the value of v in (i) we obtain (2x + 3y – 8) (3x + 2y – 7) – 154 =0
which is the equation of the required hyperbola.

17. RECTANGULAR HYPERBOLA


A hyperbola whose asymptotes are at right angles to each other is called a rectangular hyperbola.
x2 y2 b
The equation of the asymptotes of the hyperbola − = 1 are given by y = ± x.
a2
b 2 a
M a them a ti cs | 12.25

The θ angle between these two asymptotes is given by


 (b / a) − ( −(b / a) )  2b / a 2ab
tan θ   = =
 1 + (b / a) ( −(b / a) )  2
1−b / a 2
a − b2
2
 
2ab π
If the asymptotes are at right angles, then θ = π/2 ⇒ tan θ = tan π/2 ⇒ = tan ⇒ a = b.
a −b 2 2 2
Thus, the transverse and conjugate axes of a rectangular hyperbola are equal and the equation of the hyperbola
is x2 – y2 = a2.

Remarks: Since the transverse and conjugate axis of a rectangular hyperbola are equal. So, its eccentricity e is
given by

b2 b2
e= 1+ = 1+ = 2
a2 a2

17.1 With Asymptotes as Coordinate Axes


Equation of the hyperbola referred to the transverse and conjugate axes along the axes of co-ordinates, the
equation of the rectangular hyperbola is x2 – y2 = a2 ….(i)
The asymptotes of (i) are y = x and y = –x. Each of these two asymptotes is inclined at an angle of 45º with the
transverse axis. So, if we rotate the coordinate axes through an angle of –π/4 keeping the origin fixed, then the axes
coincide with the asymptotes of the hyperbola and, we have
X+Y Y−X
x = X cos(–π/4) – Y sin(–π/4) = and y = X sin(–π/4) + Y cos(–π/4) =
2 2
2 2
X+Y Y−X
Substituting the values of x and y in (i), we obtain the   −  =a
2

 2   2 
a2 a2
⇒ XY = ⇒ XY = c2, where c2 =
2 2
Thus, the equation of the hyperbola referred to its asymptotes as the coordinates axes is
a2
xy = c2, where c2 =
2
Remark: The equation of a rectangular hyperbola having coordinate axes as its asymptotes is xy = c2.
If the asymptotes of a rectangular hyperbola are x = α, y = β, then its equation is
(x – α) (y – β) = c2 or xy – ay – bx + λ = 0 ; ( λ ≤ αβ )
17.2 Tangent

Point Form
x y
The equation of the tangent at (x1, y1) to the hyperbola xy =c2 is xy1 + yx1 = 2c2 or, + = 2.
x1 y1
Parametric Form
 c x
The equation of the tangent at  ct,  to the hyperbola xy = c2 is + yt = 2c.
 t  t
 c  c  2ct1 t2 2c 
Note: Tangent at P  ct1 ,  and Q  ct2 ,  to the rectangular hyperbola xy = c2 intersect at  . 
 t1   t2   t1 + t2 t1 + t2 
1 2 . 2 6 | Hyperbola

17.3 Normal
Point Form
The equation of the normal at (x1, y1) to the hyperbola xy = c2 is xx1 – yy1 = x12 – y12
Parametric Form
 c y c
The equation of the normal at  ct,  to the hyperbola xy = c2 is xt − = ct2 −
 t  t t2
Note:
 c
(i) The equation of the normal at  ct,  is a fourth degree equation in t. So, in general, at most four normals
 t
can be drawn from a point to the hyperbola xy = c2.
(ii) The equation of the polar of any point P(x1, y1) with respect to xy = c2 is xy1 + yx1 = 2c2.
(iii) The equation of the chord of the hyperbola xy = c2 whose midpoint (x,y) is xy1 + yx1 = 2x1y1 or, T = S′. where
T and S′ have their usual meanings.
(iv) The equation of the chord of contact of tangents drawn from a point (x1, y1) to the rectangular xy = c2 is xy1
+ yx1 = 2c2.

Illustration 32: A, B, C are three points on the rectangular hyperbola xy = c2, find
(i) The area of the triangle ABC
(ii) The area of the triangle formed by the tangents A, B and C (JEE ADVANCED)

Sol: Use parametric co-ordinates and the formula for the area to get the desired result.

 c c  c
Let co-ordinates of A, B and C on the hyperbola xy = c2 be  ct1 ,  .  ct2 ,  and  ct3 ,  respectively
 t1   t2   t3 
 c c c 
 ct1 ct2 ct3 
1 t1 t2 t3  c2 t1 t2 t2 t3 t3 t1
(i) Area of triangle ABC =  + + = − + − + −
2 c c c  2 t2 t1 t3 t2 t1 t3
 ct2 ct3 ct1 
 t2 t3 t1 

c2 c2
= | t32 t3 − t22 t3 + t1 t22 − t32 t1 + t2 t32 − t12 t2 | = | (t − t )(t − t )(t − t ) |
2t1 t2 t3 2t1 t2 t3 1 2 2 3 3 1

(ii) Equation of tangents at A, B, C are x + yt12 – 2ct1 = 0, x + yt22 – 2ct2 = 0 and x + yt32 – 2ct3 = 0
2
1 t12 −2ct1
1
∴ Required Area = 1 t22 −2ct2  … (i)
2 | C1C2C3 |
1 t32 −2ct3

1 t12 1 t12 1 t12


where C1 = , C2 = − and C3 =
1 t32 1 t32 1 t32

∴ C1 = t32 – t22, C2 = t12 – t32 and C3 = t22 – t12

1 (t1 − t2 )(t2 − t3 )(t3 − t1 )


From (i) = 4c2 (t1 − t2 )2 (t2 − t3 )2 (t3 − t1 )2 = 2c2
2 | (t32 − t22 )(t12 − t32 )(t22 − t12 ) | (t1 + t2 )(t2 + t3 )(t3 + t1 )

(t1 − t2 )(t2 − t3 )(t3 − t1 )


∴ Required area is, 2c2
(t1 + t2 )(t2 + t3 )(t3 + t1 )
M a them a ti cs | 12.27

PROBLEM SOLVING TACTICS

( x − h) (y − k)
2 2

(a) In general convert the given hyperbola equation into the standard form − 1 and compare it
=
a2 b2
2 2 2 2
x y x y
with − 1 . Then solve using the properties of the hyperbola
= − 1 . So, it is advised to remember
=
2 2 2
a b a b2
the standard results.
x2 y 2
(b) Most of the standard results of a hyperbola can be obtained from the results of an ellipse + 1 just by
=
changing the sign of b2. a2 b2

FORMULAE SHEET

HYPERBOLA

(a) Standard Hyperbola:

Hyperbola x2 y2 x2 y2
− 1
= − + 1
=
Imp. Terms a2 b2 a2 b2
x2 y2
or − −1
=
a2 b2

Centre (0, 0) (0, 0)


Length of transverse axis 2a 2b
Length of conjugate axis 2b 2a
Foci (±ae, 0) (0, ±be)
Equation of directrices x = ± a/e y = ± b/e
Eccentricity
 a2 + b2   a2 + b2 
e=   e=  
 a2   b2 
   

Length of L.R. 2b2/a 2a2/b


Parametric co-ordinates (a sec φ, b tan φ) (a tan φ, b sec φ)

0 ≤ φ < 2π 0 ≤ φ < 2π
Focal radii SP = ex1 – a SP = ey1 – b

S¢P = ex1 + a S¢P = ey1 + b


|S¢P – SP| 2a 2b
Tangents at the vertices x = –a, x = a y =–b, y = b
Equation of the transverse axis y=0 x=0
Equation of the conjugate axis x=0 y=0
1 2 . 2 8 | Hyperbola

S(0,be)
Y

N B L
M′ M P(x,y) B (0, b) b
y=

Rectum
Z a

X′ X X′ X
A′ Z′ C Z A S(ae,0) C b
S′(–ae,0) y=−

Latus
(–a,0) (a,0) a
a B′(0, –b)
a x=
N′ x=– B′ e L′

S’(0,–be)
e
Y′
Y′

Figure 12.32: Hyperbola Figure 12.33: Conjugate Hyperbola

(b) Special form of hyperbola: If (h , k) is the centre of a hyperbola and its axes are parallel to the co-ordinate
(x − h)2 (y − k)2
axes, then the equation of the hyperbola is − =1
a2 b2

(c) Parametric equations of a hyperbola: The equation x = a sec φ and y = b tan φ are known as the parametric
equation of the standard hyperbola
x2 y2
− = 1.
a2 b2
x2 y2 x12 y12 xx1 yy1
S
If = − , then S1 = − −1 ; T = − −1
2 2 2 2 2
a b a b a b2

(d) Position of a point and a line w.r.t. a hyperbola: n The point (x1, y1) lies inside, on or outside the hyperbola
x2 y2 x12 y12
− = 1 according to–1 being >, = or < zero. −
a2 b2 a2 b2
The line y = mx + c intersects at 2 distinct points, 1 point or does not intersect with the hyperbola according as c2
>, = or < a2m2 – b2.

(e) Tangent:
x2 y2
(i) Point form: The equation of tangent to the hyperbola − = 1 at (x1, y1) is
a2 b2
xx1 yy1
− = 1.
2
a b2
x2 y 2
(ii) Parametric form: The equation of tangent to the hyperbola − = 1 at parametric coordinates (a sec φ, b
2 2
x y a b
tan φ) is sec φ − φ =11.
a b
x2 y 2
(iii) Slope form: The equation of the tangents having slope m to the hyperbola − = 1 are
a2 b2
y = mx ± a2m2 − b2 and the co-ordinates of points of contacts are

 a2m b2 
± ,± 
 
 a2m2 − b2 a2m2 + b2 
x2 y2
(f) Equation of a pair of tangents from an external point (x1, y1) to the hyperbola − = 1 is SS1 = T2.
2 2
a b
M a them a ti cs | 12.29

(g) Normal:
x2 y2
(i) Point form: The equation of the normal to the hyperbola − = 1 at (x1, y1) is
2 2
a2 b2
a x b y
+ = a2 + b2.
x1 y1

(ii) Parametric form: The equation of the normal at parametric coordinates (a sec θ, b tanθ) to the hyperbola

x2 y2
− = 1 is ax cos θ + by cot θ = a2 + b2.
a2 b2
2 2
(iii) Slope form: The equation of the normal having slope m to the hyperbola x − y = 1 is
a2 b2
m(a2 + b2 )
y = mx 
a2 − b2m2
x2 y 2
(iv) Condition for normality: y = mx + c is a normal to the hyperbola − = 1 if
2 2 2 a2 b2
m(a + b )
c2 =
(a2 − m2b2 )
 a2 mb2 
(v) Points of contact: Co-ordinates of the points of contact are  ± , .
 
 a2 − b2m2 a2 − b2m2 
x2 y2
(h) The equation of the director circle of the hyperbola = 1 is given by x2 + y2 = a2 – b2.

a2 b2
(i) Equation of the chord of contact of the tangents drawn from the external point (x1, y1) to the hyperbola is

xx1 yy1
given by − = 1.
2
a b2
x2 y2
( j) The equation of the chord of the hyperbola − = 1 whose mid point is (x1, y1) is T = S1.
a2 b2

(k) Equation of a chord joining points P(a sec f1, b tan f1) and Q (a sec f2, b tan f2) is

x  φ − φ2  y  φ1 + φ2   φ + φ2 
cos  1  − sin  cos  1
 = 
a  2  b  2   2 

(l) Equation of the polar of the point (x1, y1) w.r.t. the hyperbola is given by T = 0.

x2 y2  a2  b2m 
The pole of the line lx + my + n = 0 w.r.t. − = 1 is  − , 
a2 b2  n n 

x2 y2 b2
(m) The equation of a diameter of the hyperbola − = 1 corresponding to the chords of slope m is y = x
a2 b2 a2m
b2
(n) The diameters y = m1x and y = m2x are conjugate if m1m2 =
a2
(o) Asymptotes:
•• Asymptote to a curve touches the curve at infinity.
x2 y2 b
•• The equation of the asymptotes of the hyperbola − = 1 are y = ± x.
a2
b 2 a
1 2 . 3 0 | Hyperbola

•• The asymptote of a hyperbola passes through the centre of the hyperbola.

x2 y2 x2 y2
•• * The combined equation of the asymptotes of the hyperbola − = 1 is − =0
a2 b2 a2 b2
x2 y2 a2
•• * The angle between the asymptotes of − = 1 is 2 tan–1
or 2 sec–1 e.
a2 b2 b2
•• A hyperbola and its conjugate hyperbola have the same asymptotes.
•• The bisector of the angles between the asymptotes are the coordinate axes.
•• Equation of the hyperbola – Equation of the asymptotes = constant.

(p) Rectangular or Equilateral Hyperbola:


•• A hyperbola for which a = b is said to be a rectangular hyperbola, its equation is x2 – y2 = a2.
•• xy = c2 represents a rectangular hyperbola with asymptotes x = 0, y = 0.
•• Eccentricity of a rectangular hyperbola is 2 and the angle between the asymptotes of a rectangular
hyperbola is 90º.
c
•• Parametric equation of the hyperbola xy = c2 are x = ct, y = , where t is a parameter.
t
•• Equation of a chord joining t1, t2 on xy = c2 is x + y t1t2 = c(t1 + t2)
x y
•• Equation of a tangent at (x1, y1) to xy = c2 is + = 2.
x1 y1
•• Equation of a tangent at t is x + yt2 = 2ct
•• Equation of the normal at (x1, y1) to xy = c2 is xx1 – yy1 = x12 – y12.
•• Equation of the normal at t on xy = c2 is xt3 – yt – ct4 + c = 0.
(i.e. Four normals can be drawn from a point to the hyperbola xy = c2)
•• If a triangle is inscribed in a rectangular hyperbola then its orthocentre lies on the hyperbola.
•• Equation of chord of the hyperbola xy = c2 whose middle point is given is T = S1.
 2ct1 t2 2c 
•• Point of intersection of tangents at t1 and t2 to the hyperbola xy = c2 is  , 
 t1 + t2 t1 + t2 

Solved Examples

JEE Main/Boards The equation of the hyperbola is


(x − 1)2 (y − 4)2
− =1
Example 1: Find the equation of the hyperbola whose a2 b2
foci are (6, 4) and (–4, 4) and eccentricity is 2. 5
The distance between the foci is 2ae = 10; ∴ a =
2
Sol: Calculate the value of ‘a’, by using the distance 25 75
b = a (e – 1) =
2 2 2
(4 – 1) =
between the two foci and eccentricity. Then calculate 4 4
the value of ‘b’. Using these two values find the equation Hence the equation of the hyperbola is
of the hyperbola.
(x − 1)2 (y − 4)2
Let S, S′ be the foci and C be the centre of the hyperbola − =1
25 75
S, S′ and C lie on the line y = 4. The co-ordinates of the
4 4
centre are (1, 4).
M a them a ti cs | 12.31

Example 2: Obtain the equation of hyperbola whose [(a secθ + ae)2 + a2tan2q]
asymptotes are the straight lines x + 2y + 3 = 0 & 3x +
= a4[(sec2θ + tan2θ + e2)2 – 4e2sec2q]
4y + 5 = 0 and which passes through the point (1, –1)
= a4[(2sec2θ – 1 + 2)2 – 4.2 sec2q]
Sol: Use the following formula:
= a4[(2sec2θ + 1)2 – 8sec2q]
Equation of hyperbola – Equation of asymptotes =
= a4[(2sec2θ – 1)2]
constant.
∴ SP.S’P = a2(2sec2θ – 1)
The equation of the hyperbola, is
= a2(sec2θ + tan2θ)
(x + 2y + 3)(3x + 4y + 5) = k, k being a constant.
= CP2.
This passes through the point (1, –1)
∴ (1 + 2(–1) + 3)(3(1) + 4(–1) + 5) = k Example 5: Find the equation of the hyperbola
⇒ k=2×4=8 conjugate to the hyperbola
∴ The equation of the hyperbola is 2x2 + 3xy – 2y2 – 5x + 5y + 2 = 0
(x + 2y + 3)(3x + 4y + 5) = 8 Sol: Use the formula:
Equation Hyperbola + Conjugate Hyperbola
Example 3: If e and e′ are the eccentricities of two
hyperbolas conjugate to each other, = 2(Asympototes)
1 1 Let asymptotes be
show that + = 1.
2 2
e e′ 2x2 + 3xy – 2y2 – 5x + 5y + λ = 0
Sol: Start with the standard equation of two hyperbolas The equation above represents a pair of lines if
and eliminate ‘a’ and ‘b’.
abc + 2fgh – af2 – bg2 – ch2 = 0
x2 y2 y2 x2
Let − = 1 and − =1 ∴ λ = –5
a2 b2 b2 a2
Equation Hyperbola + Conjugate Hyperbola
be the two hyperbolas with eccentricities e and e′
respectively = 2(Asympototes)
2
1 a ∴ Conjugate Hyperbola
b2 = a2(e2 – 1) ⇒ =
e2 a2 + b2 = 2(Asymptotes) – Hyperbola
2
1 b 2x2 + 3xy – 2y2 – 5x + 5y – 8 = 0
a2 = b2(e¢2 – 1) ⇒ =
2 2 2
e′ a +b
1 1 a2 b2 Example 6: If (5, 12) and (24, 7) are the foci of a hyperbola
∴ + = + =1 passing through the origin then the eccentricity of the
e2 e′2 (a2 + b2 ) (a2 + b2 )
hyperbola is

Example 4: If any point P on the rectangular hyperbola Sol: Use the definition of the hyperbola S¢P – SP = 2a.
x2 – y2 = a2 is joined to its foci S, S′ show that SP.S¢P = Let S(5, 12) and S′(24, 7) be the two foci and P(0, 0) be
CP2, where C is the centre of the hyperbola. a point on the conic then

Sol: The eccentricity of a rectangular hyperbola is 2 . SP = 25 + 144 = 169 = 13;


Consider a parametric point on the hyperbola and
simplify the LHS. S¢P = (24)2 + 72 = 625 = 25

Any point on the rectangular hyperbola and SS′ = (24 − 5)2 + (7 − 12)2 = 192 + 52 = 386
x2 – y2 = a2 is P(a secθ, a tanθ) ; eccentricity of a
since the conic is a hyperbola, S¢P – SP = 2a, the length
rectangular hyperbola is 2 .
of transverse axis and SS′ = 2ae, e being the eccentricity.
S is (ae, 0), S′ is (–ae, 0) and C is (0, 0) SS′ 386
⇒ e= =
(SP) .(S’P) = [(a secθ – ae) + a tan q] ×
2 2 2 2 2
S′P − SP 12
1 2 . 3 2 | Hyperbola

Example 7: An equation of a tangent to the hyperbola. Equation of the normal at (x, y) is


16x2 – 25y2 – 96x + 100y – 356 = 0 which makes an angle
dx
π/4 with the transverse axis is Y–y= − (X – x) which meets the x-axis at G
dy
Sol: Write the equation of the hyperbola in the standard  dy  dy
 0, x + y  , then x + y = ± 2x
form and compare to get the equation of the tangent.  dx  dx
Equation of the hyperbola can be written as dy
⇒ x+y = 2x ⇒ y dy = x dx
dx
X2/52 – Y2/42 = 1 ….(i)
⇒ x2 – y2 = c
where X = x – 3 and Y = y – 2.
or y dy = –3x dx
Equation of a tangent which makes an angle π/4, with
the transverse axis X = 0 of (i) is ⇒ 3x2 + y2 = c

π π Thus the curve is either a hyperbola or an ellipse.


Y = tan X ± 25 tan2 − 16
4 4
Example 10: Find the centre, eccentricity, foci and
⇒ y–2=x–3± 25 − 16 directrices of the hyperbola
⇒ y–2=x–3±3 16x2 – 9y2 + 32x + 36y – 164 = 0
⇒ y = x + 2 or y = x – 4. Sol: Represent the equation of the hyperbola in the
standard form and compare.
Example 8: If the normal at P to the rectangular Here,
hyperbola x2 – y2 = 4 meets the axes of x and y in G and
g respectively and C is the centre of the hyperbola, then 16x2 + 32x + 16 – (9y2 – 36y + 36) – 144 = 0
prove that Gg=2PC. or 16(x + 1)2 – 9(y – 2)2 = 144

Sol: In the equation of a normal, find the point of (x + 1)2 (y − 2)2


∴ − =1
intersection with the axes and find the coordinates of 9 16
G and g. Putting x + 1 = X and y – 2 = Y, the equation becomes
Let P(x1, y1) be any point on the hyperbola x2 – y2 = 4 X2 Y2
− =1
then equation of the normal at P is 9 16
y1 which is in the standard form.
y – y1 = − (x − x1 )
x1
Q b2 = a2(e2 – 1), here a2 = 9 & b2 = 16
⇒ x1y + y1x = 2x1y1.
16 25 5
Then coordinates of G are (2x1, 0) and of g are (0, 2y1) ∴ e2 – 1 = ⇒ e2 = , i.e., e =
9 9 3
so that
Now, centre = (0, 0)X,Y = (–1, 2)
PG = (2x1 − x1 )2 + y12 = x12 + y12 = PC
 5 
foci = (±ae, 0)X,Y =  ±3. , 0  = (±5, 0)x, Y
Pg = x12 + (2y1 − y1 ) 2
= x12 + y12 = PC  3 X, Y
and = (–1 ± 5, 2) = (4, 2), (–6, 2)
Gg = (2x1 )2 + (2y1 )2 = 2 x12 + y12 = 2PC Directrices in X, Y coordinates have the equations

Hence proved. a 3
X± =0 or x+1± =0
e 5/3
Example 9: The normal to the curve at P(x, y) meets the 9
i.e., x + 1 ± =0
x-axis at G. If the distance of G from the origin is twice 5
the abscissa of P, then the curve is- 14 4
∴ x= − and x =
5 5
Sol: Similar to the previous question.
M a them a ti cs | 12.33

JEE Advanced/Boards standard form of a tangent to a hyperbola and compare.


Equation of the chord PQ to the hyperbola is
2 2
x y
Example 1: S is the focus of the hyperbola − 1.
= x  θ − θ2  y  θ1 + θ2   θ1 + θ2 
2
a b2 cos  1  − sin   = cos  
a  2  b  2   2 
M is the foot of the perpendicular drawn from S on a
tangent to the hyperbola. Prove that the locus of M is
x y  θ + θ2   θ1 + θ2 
x2 + y2 = a2. cos α − sin  1  = cos  
a b  2   2 
Sol: Use the definition of an auxiliary circle.
b cos α b cos ( (θ1 + θ2 ) / 2 )
Let M = (x1, y1) be any point on the locus. y= x−  … (i)
a sin ( (θ1 + θ2 ) / 2 ) sin ( (θ1 + θ2 ) / 2 )
Let the equation of the corresponding tangent to the
x2 y 2 For line y = mx + c to be a tangent to
hyperbola − = 1 be
a2 b2
x2 y2
y = mx ± 2
a m −b 2 2 − = 1 we have
a2 b2
(the sign is chosen according to the position of M) c2 = a2m2 – b2
But M(x1, y1) lies on it
x2 cos2 α y2
2 2 2 − = 1  … (ii)
∴ y1 = mx1 ± a m −b  … (i) a2 b2
Segment SM is perpendicular to the given tangent. If (i) is tangent to (ii), then, we must have
1
∴ Slope of segment SM is −  bcos ( (θ1 + θ2 ) / 2 ) 
2
m  θ + θ2 
  = b2 cot2  1
 sin ( (θ + θ ) / 2 )   2 
and S ≡ (ae, 0)  1 2   
1
∴ Equation of SM is (y – 0) = − (x − ae) which is true.
m
But M(x1, y1) lies on it
1 Example 3: Show that the portion of the tangent to
y1 = − (x − ae)  … (ii) x2 y 2
m 1 the hyperbola − = 1 intercepted between the
a2 b2
From (i), (y1 – mx1) = ± a2m2 − b2
asymptotes is bisected at the point of contact. Also
From (ii), (my1 + x1) = ae show that the area of the triangle formed by this
tangent and the asymptotes is constant.
Squaring and adding we get the required locus of M
Sol: Calculate the point of intersection of the tangent
y12(1 + m2) + x12(1 + m2) = a2e2 + a2m2 – a2(e2 – 1) and the asymptotes and then prove the statement.
∴ x12 + y12 = a2 x2 y2
− = 1 … (i)
Note: This is the equation of the auxiliary circle a2 b2
equation of the tangent at P(x1, y1) is

Example 2: PQ is the chord joining the points θ1 and xx1 yy1


− = 1 … (ii)
2
x2 y 2 a b2
θ2 on the hyperbola − =1. If θ1 – θ2 = 2α, where Y
a2 b2 Q
α is a constant, prove that PQ touches the hyperbola P
O
2 2 2
x cos α y
− =1
2
a b2 R
Sol: Write the equation of the chord passing through
the points q1 and q2. Represent this equation in the Equation of the asymptotes are
1 2 . 3 4 | Hyperbola

x y Similarly, the equation of altitude through Q


− = 0  … (iii) perpendicular to RP is
a b
 c 
x y y + ct1t2t3 = t3t1  x +  … (iii)
and + = 0 … (iv) t1 t2 t3 
a b 
If Q and R are the points of intersection of the tangent Solving (ii) and (iii), we get
at P with the asymptotes, then solving the equation (ii)
and (iii), we get  c 
∴ Orthocentre =  − , − c t1 t2 t3 

 a b   t1 t2 t3 
Q =  , 
 (x1 / a) − (y1 / b) (x1 / a) − (y1 / b)  These co-ordinates satisfy (i)

Solving the equation (ii) and (iv), we get Hence proved.

 a −b 
R =  ,  Example 5: Find the equation of the hyperbola,
 (x1 / a) + (y1 / b) (x1 / a) + (y1 / b)  whose eccentricity is 5/4, whose focus is
(a, 0) and whose directrix is 4x – 3y = a. Find also the
The midpoint of QR has coordinate (x1, y1) which is also
coordinates of the centre and the equation to other
the point of contact of the tangent.
directrix.
Area of ∆OQR =
Sol: Use the basic definition of a hyperbola.
 a  −b 
    (4x − 3y − a)2
1  (x1 / a) − (y1 / b)   (x1 / a) + (y1 / b) 
  (x – a)2 + (y – 0)2 = e2
25
2  b  a 
−     x2 – 2ax + a2 + y2 =
 
 (x1 / a) − (y1 / b)   (x1 / a) + (y1 / b) 
25 1
(16x2 + 9y2 + a2 – 24xy – 8ax + 6ay)×
=ab sq. units 16 25
7y2 + 24xy – 24ax – 6ay + 15a2 = 0  … (i)
Example 4: Prove that if a rectangular hyperbola
circumscribes a triangle it also passes through the S’
Directrix
orthocentre of the triangle.
M 4x-3y=a
Sol: Take three points on the hyperbola and find C
Directrix
the coordinates of the orthocentre. Prove that the P
orthocentre satisfies the equation of the hyperbola.
S’(a,0)
Let the equation of the curve referred to its asymptotes Transverse axis
be xy = c2  ….(i)
Let the angular points of the triangle be P, Q and R and Differentiating with respect to ‘x’
let their co-ordinates be 24y – 24a = 0  … (ii)
 c  c Differentiating with respect to ‘y’
P ≡  ct1 , , Q ≡  ct2 ,  and
 t1   t2 
14y + 24x – 6a = 0  … (iii)
 c
R ≡  ct3 ,  respectively. Solving (ii) and (iii)
 t3 
C ≡ (–a/3, a)
Equation of QR is x + t2t3 y = c (t2 + t3)
Transverse axis is
The equation of altitude through P and perpendicular
3x + 4y = 3a
to QR is
‘P’ is the point of intersection of the transverse axis and
c
y– = t2t3(x – ct1) the directrix:
t1
 13a 9a 
 ∴ P≡  ,  ‘C’ is mid point of MP
c   25 25 
i.e. y + c t1t2t3 = t2t3  x +  … (ii)
 t1 t2 t3 
M a them a ti cs | 12.35

 −89a 41a  Circle with foci (ae, 0) and (–ae, 0) as diameter is


∴ M=  , 
 75 25  (x – ax)(x + ae) + (y – 0)(y – 0) = 0
Equation of the other directrix 4x – 3y = λ, passes i.e. x2 + y2 = a2e2 = a2 + b2 … (i)
through the ‘M’
[∵ a2e2 = a2 + b2]
∴ 12x – 9y + 29a = 0
Let the chord of contact of P(x1, y1) touch the circle (i).
Equation of the chord of contact of P is [T = 0]
Example 6: Find the centre, eccentricity, foci, directrices
and the length of the transverse and conjugate axes of xx1 yy1
− =1
the hyperbola, whose equation is (x – 1)2 – 2(y – 2)2 + a 2
b2
6 = 0. i.e. b2x1x – a2y1y – a2b2 = 0 … (ii)
Sol: Represent the equation of the hyperbola in the This equation is tangent to the circle if
standard form and proceed.
a2b2
= ± (a2 + b2 )
(b 4 x12 + a4 y12 )
The equation of the hyperbola can be re-written as
(x − 1)2 (y − 2)2 Hence locus of P (x1, y1) is (b4x2 + a4y2) (a2 + b2) = a4b4.
− + =1
( 6 )2 ( 3)2
1
Example 8: An ellipse has eccentricity and one
Y2 X2 2
⇒ − =1 1 
( 3)2 ( 6 )2 focus at the point P  , 1  . One of its directrices is
2 
Where Y = (y – 2) and X = (x – 1) … (i) the common tangent, to the circle x2 + y2 = 1 and the
hyperbola x2 – y2 = 1, nearer to P. The equation of the
∴ Centre: X = 0, Y = 0 i.e. (1, 2)
ellipse in the standard form is.
So a = 3 and b = 6
Sol: he circle x2 + y2 = 1 is the auxiliary circle of the
so transverse axis = 2 3 ,
hyperbola x2 – y2 = 1 and they touch each other at the
and conjugate axis = 2 6 points (±1, 0). Use the definition of the ellipse to get
the final equation.
Also b2 = a2(e2 – 1)
The common tangent at these points are x = ±1.
⇒ 6 = 3(e2 – 1) i.e. e = 3 1 
Since x = 1 is near to the focus P  , 1  , this is the
In (X, Y) coordinates, foci are (0, ±ae) 2 
directrix of the required ellipse.
i.e. (0, ±3)
2 2
∴ foci are (1 + 0, 2 ± 3) x -y =1
i.e. (1, 5) and (1, –1)
Equations of directrices Y = ±a/e 2 2
x +y =1
∴ Directrices are y – 2 = ±1 x=1
⇒ y = 3, y = 1. Therefore, by definition, the equation of the ellipse is
2 2 2
 1 1   x −1
 x −  + ( y − 1) =
2
Example 7: Prove that the locus of a point whose chord    
of contact touches the circle described on the straight  2 2  1 
2
x2 y2  1
line joining the foci of the hyperbola − = 1 as the ⇒ 9  x −  + 12(y – 1)2 = 1.
a2 b2  3
x2 y2 1
diameter is + = .
a4 b4 (a2 + b2 ) Example 9: Prove that the angle subtended by any
chord of a rectangular hyperbola at the centre is the
Sol: Check if the line T = 0 is a tangent to the circle with
supplement of the angle between the tangents at the
two foci as the end points of the diameter.
ends of the chord.
1 2 . 3 6 | Hyperbola

Sol: Using the equation of chord, find the angle x2 y2


subtended at the centre and at the intersection of the Example 10: If a chord of ellipse + =1
a2 b2
tangents.
x2 y2
touches the hyperbola − = 1, show that the
Let P(x1, y1) and Q(x2, y2) be two ends of a chord of the a2 b2
rectangular hyperbola
2
 x2 y 2  x2 y 2
x2 – y2 = 1  …..(i) locus of its middle point is  +  = − .
R  a2 b2  a2 b2
 
Sol: Apply the condition of tangency in the equation
O of the chord.

P Q Let M(α, β) be the middle point of the chord PQ of the


ellipse
y1 x2 y2
Now, ‘m’ of OP = + = 1 ….(i)
x1
a2 b2
y2
‘m’ of OQ = The equation of the chord is
x2
xα yβ α2 β2
y1 y 2 + = +
− a2 b2 a2 b2
x1 x2 x2 y1 − x1 y 2
∴ tan θ = = ,
y y x1 x2 + y1 y 2 xb2 α b2  α2 β2 
1+ 1 . 2 ⇒ y=
− +  + 
x1 x2 a2β β  a2 b2 

Where ∠POQ = θ, This line is tangent to hyperbola if


The equations of tangents at P and Q are c2 a2m2 − b2
=
2 2
xx1 – yy1 = 1 and xx2 – yy2 = 1.  b2  α2 β2    b2 α 
x1 x2 ⇒   +  = a2   − b2
 β  a2 b2    a2β 
Their slopes are y and y .     
1 2
2
x1 x2  α2 β2  α2 β2
− ⇒  +  = −
y1 y 2 x1 y 2 − x2 y1  a2 b2  a2 b2
∴ tan φ = =  
x1 x2 y1 y 2 + x1 x2
1+ . ∴ The equation of the required locus of the middle
y1 y 2 point (α, β) is
∴ tan θ and tan φ are equal in magnitude but opposite 2
in sign  x2 y 2  x2 y 2
 2 + 2  =2 − 2
a b  a b
∴ tan θ = – tan φ = tan (π – φ) 
∴ θ + φ = π. Hence, proved.

JEE Main/Boards

Exercise 1 Q.3 Show that the line 21x + 5y = 116 touches the
hyperbola 7x2 – 5y2 = 232 and find the co-ordinates of
the point of contact.
Q.1 Find the centre, eccentricity and foci of the
hyperbola 9x2 – 16y2 – 18x – 64y – 199 = 0
Q.4 Find the locus of the middle points of the portion
Q.2 Find the equation to the tangent to the hyperbola x2 y 2
of the tangents to the hyperbola − = 1 included
4x2 – 3y2 = 13 at the point (2,1). a2 b2
between the axes.
M a them a ti cs | 12.37

Q.5 A point P moves such that the tangents PT1 and at the point Q(h, k), prove that
PT2 from it to the hyperbola 4x2 – 9y2 = 36 are mutually (i) x1 + x2 + x3 + x4 = h (ii) y1 + y2 + y3 + y4 = k
perpendicular. Find the equation of the locus of P.
(iii) x1x2x3x4 = y1y2y3y4 = –c4

Q.6 Find the equations of the two tangents to the


Q.17 Find the locus of the points of intersection of two
hyperbola xy = 27 which are perpendicular to the
x2 y 2
straight line 4x – 3y = 7. tangents to a hyperbola − = 1, if sum of their
a2 b2
slopes is a constant λ.
Q.7 Find the equation of the hyperbola which has
3x – 4y + 7 = 0 and 4x+3y+1 = 0 for its asymptotes and x2 y2
Q.18 A variable tangent to the hyperbola − =1
which passes through the origin. a2 b2
meets the transverse axis at Q and to the tangent at the
Q.8 Find the equation of chord of contact of tangents vertex (a, 0) at R. Show that the locus of the mid point of QR is
drawn from the point (–5, 2) to the hyperbola xy = 25. x(4y2 + b2) = ab2.

Q.9 Find the eccentric angle of the point lying in fourth Q.19 A tangent to the parabola x2 = 4ay meets the
quadrant on the hyperbola x2 – y2 = 4 whose distance hyperbola xy = k2 in two points P and Q. Prove that the
from the centre is 12 units. middle point of PQ lies on a parabola.

Q.10 Find the acute angle between the asymptotes of Q.20 Show that the locus of the middle points of the
4x2 – y2 = 16. normal chords of the rectangular hyperbola x2 – y2 = a2
is (y2 – x2)3 = 4a2x2y2.

Q.11 If the tangent and normal to a rectangular


hyperbola cut off intercepts a1 and a2 on one axis and x2 y 2
Q.21 Given a hyperbola − = 1 and circle x2 + y2 = 9.
b1 and b2 on the other axis, shows that a1a2 + b1b2 = 0. 9 4
Find the locus of mid point of chord of contact drawn
Q.12 Show that the area of the triangle formed by the from a point on the hyperbola to the circle.
two asymptotes of the rectangular hyperbola xy = c2 and
2 Q.22 A rectangular hyperbola whose centre is C, is cut
1  x − y1 
2 2
the normal at (x1, y1) on the hyperbola is  1  . by a circle of radius r in four points P, Q, R, S. Prove that
2 c 
  CP2 + CQ2 + CR2 + CS2 = 4r2.

Q.13 PN is the ordinate of any point P on the hyperbola


Q.23 The normal at the three points P, Q, R on a
x2 y 2
− = 1. If Q divides AP in the ratio a2: b2, show that rectangular hyperbola, intersect at a point S on the
2 2
a b curve. Prove that the centre of the hyperbola is the
NQ is perpendicular to A¢P where A′A is the transverse centroid of the triangle PQR.
axis of the hyperbola.
Q.24 A parallelogram is constructed with its sides
Q.14 A normal to the hyperbola x2 – 4y2 = 4 meets the x parallel to the asymptotes of a hyperbola and one of
and y axes at A and B respectively. Find the locus of the its diagonals is a chord of the hyperbola, show that the
point of intersection of the straight lines drawn through other diagonal passes through the centre.
A and B perpendicular to the x and y axes respectively.
Q.25 If the straight line y = mx + 2c −m touches the
Q.15 In any hyperbola, prove that the tangent at any hyperbola xy = c2 then the co-ordinates of the point
point bisects the angle between the focal distances of contact are (……………..,)
the point.
Q.26 If the normal to the rectangular hyperbola xy = c2
Q.16 If the normals at four points Pi(xi, yi) at the point ‘t’ meets the curve again at ‘t1’ then t3t1 has
i = 1, 2, 3, 4 on the rectangular hyperbola xy = c2 meet the value equal to …………………………….
1 2 . 3 8 | Hyperbola

Exercise 2 Q.9 Which of the following pair may represent the


eccentricities of two conjugate hyperbola for all
Single Correct Choice Type α ∈ (0, π/2) ?
(A) sin α, cos α (B) tan α, cot α
Q.1 The line 5x + 12y = 9 touches the hyperbola
(C) sec α, cosec α (D) 1 + sinα, 1 + cosα
x2 – 9y2 = 9 at the point-
(A) (–5, 4/3) (B) (5, –4/3) x2 y 2
Q.10 The number of normals to the hyperbola −
(C) (3, –1/2) (D) None of these = 1 from an external point is- a2 b2

(A) 2 (B) 4 (C) 6 (D) 5


Q.2 The length of the latus rectum of the hyperbola
x2 y 2 Q.11 A rectangular hyperbola circumscribe a triangle
− = –1 is-
a2 b2 ABC, then it will always pass through its-
2a2 2b2 b2 a2 (A) Orthocentre (B) Circum centre
(A) (B) (C) (D)
b a a b
(C) Centroid (D) Incentre

Q.3 The area of the square whose sides are the  c


directrixes of the hyperbola x2 – y2 = a2 and its conjugate Q.12 If the normal at  ct,  on the curve xy = c2 meets
 t
hyperbola, is-
the curve again at t′ then-
(A) a2 (B) 2a2 (C) 4a2 (D) 8a2
−1 1
(A) t′ = (B) t′ =
t 3 t
Q.4 The number of possible tangents which can
be drawn to the curve 4x2 – 9y2 = 36, which are 1 −1
(C) t′ = (D) t¢2 =
perpendicular to the straight line 5x + 2y – 10 = 0 is - t 2
t2
(A) Zero (B) 1 (C) 2 (D) 4
Q.13 The centre of the hyperbola 9x2 – 16y2 – 36x +
96y – 252 = 0 is-
Q.5 If m is a variable, the locus of the point of intersection
(A) (2, 3) (B) (–2, –3) (C) (–2, 3) (D) (2, –3)
x y x y 1
of the lines − = m and + = is a/an -
3 2 3 2 m
Q.14 The tangents from (1, 2 2 ) to the hyperbola 16x2
(A) Parabola (B) Ellipse – 25y2 = 400 include between them an angle equal to-
(C) Hyperbola (D) None of these π π π π
(A) (B) (C) (D)
6 4 3 2
Q.6 The eccentricity of the hyperbola with its principal
axes along the co-ordinate axes and which passes Q.15 The number of points from where a pair of
through (3, 0) and ( 3 2 , 2) is- perpendicular tangents can be drawn to the hyperbola,
x2sec2α – y2cosec2α = 1, α ∈ (0, π/4) is-
1 13 5 2
(A) (B) (C) (D) (A) 0 (B) 1 (C) 2 (D) Infinite
3 3 3 3

Q.7 The eccentricity of the conic represented by x2 y2


Q.16 If hyperbola = 1 passes through the focus

x2 – y2 – 4x +4y + 16 = 0 is- b2 a2
2 2
x y
(A) 1 (B) 2 (C) 2 (D) 1/2 of ellipse + = 1 then eccentricity of hyperbola is-
2
a b2
2
Q.8 An ellipse and a hyperbola have the same centre (A) 2 (B) (C) 3 (D) None of these
origin, the same foci and the minor-axis of the one is 3
the same as the conjugate axis of the other. If e1, e2 be
1 1 x2 y2
their eccentricities respectively, then + = Q.17 If the curves + = 1, (a > b) and x2 – y2 = c2
2 2
e1 e22
2 a b
(A) 1 (B) 2 (C) 4 (D) None of these cut at right angles then-
M a them a ti cs | 12.39

(A) a2 + b2 = 2c2 (B) b2 – a2 = 2c2 Previous Years’ Questions


(C) a2 – b2 = 2c2 (D) a2b2 = 2c2
x2 y2
2 2 Q.1 The equation − = 1, | r |<1 represents-
x y 1 1−r 1+r
Q.18 Two conics − = 1 and x2 = − y intersect
if - a 2
b 2 b  (1981)

1 1 (A) An ellipse (B) A hyperbola


(A) 0 < b ≤ (B) 0 < a <
2 2 (C) A circle (D) None of these

(C) a2 < b2 (D) a2 > b2


Q.2 Let P(a sec θ, b tan θ) and Q(a sec φ, b tan φ), where
π x2 y 2
θ + φ = , be two points on the hyperbola − =
Q.19 The locus of the mid points of the chords passing 2 a2 b2
x2 y 2 1. If (h, k) is the point of the intersection of the normals
through a fixed point (α, β) of the hyperbola, −
a2 b2 at P and Q, then k is equal to- (1999)
= 1 is-
α β a2 + b2  a2 + b2  a2 + b2  a2 + b2 
(A) A circle with centre  ,  (A) (B) −   (C) (D) − 
a  a  b  b 
 2 2    
α β
(B) An ellipse with centre  , 
 2 2 Q.3 If x = 9 is the chord of contact of the hyperbola x2
– y2 = 9, then the equation of the corresponding pair of
α β
(C) A hyperbola with centre  ,  tangents is- (1999)
 2 2
(A) 9x2 – 8y2 + 18x – 9 = 0
α β
(D) Straight line through  ,  (B) 9x2 – 8y2 – 18x + 9 = 0
 2 2
(C) 9x2 – 8y2 – 18x – 9 = 0
Q.20 If the eccentricity of the hyperbola
(D) 9x2 – 8y2 + 18x + 9 = 0
x2 – y2sec2a = 5 is 3 times the eccentricity of the
ellipse x2sec2a + y2 = 25, then a value of α is- x2 y2
Q.4 For hyperbola −= 1, which of the
(A) π/6 (B) π/4 (C) π/3 (D) π/2 cos2 α sin2 α
following remains constant with change in ‘α’ ?(2003)
Q.21 For all real values of m, the straight line y = mx + (A) Abscissa of vertices (B) Abscissa of foci
2
9m − 4 is a tangent to the curve- (C) Eccentricity (D) Directrix
(A) 9x + 4y = 36
2 2
(B) 4x + 9y = 36
2 2

Q.5 If the line 2x + 6 y = 2 touches the hyperbola x2


(C) 9x2 – 4y2 = 36 (D) 4x2 – 9y2 = 36
– 2y2 = 4, then the point of contact is- (2004)

Q.22 Locus of the middle points of the parallel chords (A) (–2, 6 ) (B) (–5, 2 6 )
with gradient m of the rectangular hyperbola xy = c2 is-
1 1 
(A) y + mx = 0 (B) y – mx = 0 (C)  ,  (D) (4, − 6 )
2 6 
(C) my – mx = 0 (D) my + x = 0
x2 y 2
Q.6 If e1 is the eccentricity of the ellipse + = 1 and
16 25
Q.23 The locus of the middle points of chords of e2 is the eccentricity of the hyperbola passing through
hyperbola 3x2 – 2y2 + 4x – 6y = 0 parallel to y = 2x is- the foci of the ellipse and e1e2 = 1, then equation of the
hyperbola is- (2006)
(A) 3x – 4y = 4 (B) 3y – 4x + 4 = 0
x2 y 2 x2 y 2
(C) 4x – 4y = 3 (D) 3x – 4y = 2 (A) − = 1 (B) − = –1
9 16 16 9
x2 y 2
(C) − = 1 (D) None of these
9 25
1 2 . 4 0 | Hyperbola

x2 y2
Q.7 A hyperbola, having the transverse axis of length Q.9 Let P(6, 3) be a point on the hyperbola −
2sinθ, is confocal with the ellipse 3x2 + 4y2 = 12. Then a2 b2
its equation is-  (2007) = 1. If the normal at the point P intersect the x-axis at

(A) x2cosec2θ – y2sec2θ = 1 (B) x2sec2θ – y2cosec2θ = 1 (9, 0), then the eccentricity of the hyperbola is- (2011)
(C) x2sin2θ – y2cos2θ = 1 (D) x2cos2θ – y2sin2θ = 1 5 3
(A) (B) (C) 2 (D) 3
2 2
Q.8 Consider a branch of the hyperbola
x2 – 2y2 – 2 2 x – 4 2y–6=0 Q.10 The eccentricity of the hyperbola whose length
with vertex at the point A. Let B be one of the end points of the latus rectum is equal to 8 and the length of its
of its latus rectum. If C is the focus of the hyperbola conjugate axis is equal to half of the distance between
nearest to the point A, then the area of the triangle ABC its foci, is:  (2016)
is-  (2008) 4 2 4
(A) (B) (C) 3 (D)
2 3 3 3 3
(A) 1 − sq. unit (B) − 1 sq. unit
3 2
2 3
(C) 1 + sq. unit (D) + 1 sq. unit
3 2

JEE Advanced/Boards

Exercise 1 Q.7 Find the equation of the tangent to the hyperbola


x2 – 4y2 = 36 which is perpendicular to the line
Q.1 Find the equation to the hyperbola whose directrix x – y + 4 = 0.
is 2x + y = 1 focus (1, 1) and eccentricity 3 . Find also
the length of its latus rectum.
Q.8 If θ1 and θ2 are the parameters of the extremities of
a chord through (ae, 0) of a hyperbola x2/a2 – y2/b2 = 1,
Q.2 The hyperbola x2/a2 – y2/b2 = 1 passes through the
then show that
point of inter-section of the lines. 7x + 13y – 87 = 0 and
5x – 8y + 7 = 0 and the latus rectum is 32 3 /5. Find ‘a’ θ1 θ2 e −1
tan tan + = 0.
& ‘b’. 2 2 e+1

Q.3 For the hyperbola x2/100 – y2/25 = 1, prove that the Q.9 If C is the centre of hyperbola x2/a2 – y2/b2 = 1, S, S′
(i) eccentricity = 5 /2 its foci and P a point on it. Prove that SP.S´P = CP2 – a2 + b2.
(ii) SA.S′A = 25, where S and S′ are the foci and A is the
vertex. Q.10 Tangents are drawn to the hyperbola 3x2 – 2y2 =
25 from the point (0, 5/2). Find their equations.
Q.4 Find the centre, the foci, the directrices, the length
of the latus rectum, the length and the equations of the Q.11 If the tangent at the point (h, k) to the hyperbola
axes and the asymptotes of the hyperbola x2/a2 – y2/b2 = 1 cuts the auxiliary circle in points whose
16x2 – 9y2 + 32x + 36y – 164 = 0. ordinates are y1 and y2 then prove that 1/y1 + 1/y2 = 2/k.

Q.5 If a rectangular hyperbola have the equation, Q.12 Tangents are drawn from the point (α, β) to the
xy = c2, prove that the locus of the middle point of the hyperbola 3x2 – 2y2 = 6 and are inclined at angles θ and
chords of constant length 2d is (x2 + y2)(xy – c2) = d2xy. φ to the x-axis. If tanθ . tanφ = 2, prove that β2 = 2α2 – 7.

Q.6 If m1 and m2 are the slopes of the tangents to the


Q.13 Find the number of normal which can be drawn
hyperbola x2/25 – y2/16 = 1 through the point (6, 2),
find the value of (i) m1 + m2 and (ii) m1m2.
M a them a ti cs | 12.41

from an external point on the hyperbola


x2 − y 2
= 1.
Exercise 2
2 2
a b
Single Correct Choice Type
Q.14 The perpendicular from the centre upon the
x2 y 2
normal on any point of the hyperbola x2/a2 – y2/b2 = 1 Q.1 Locus of middle point of all chords of −
4 9
meets at R. Find the locus of R.
= 1. Which are at distance of ‘2’ units from vertex of
parabola y2 = – 8ax is-
Q.15 If the normal at a point P to the hyperbola
2
x2/a2 – y2/b2 = 1 meets the x-axis at G, show that  x2 y 2  xy  x2 y 2   x2 y 2 
SG = e. SP, S being the focus of the hyperbola. (A)  + = (B)  −  = 4  + 
 4 9  6  4 9   16 81 
    
2
Q.16 Show that the area of the triangle formed by  x2 y 2   x2 y 2 
(C)  +  = +  (D) None of these
the lines x – y = 0, x + y = 0 and any tangent to the  4 9   9 4 
 
hyperbola x2 – y2 = a2 is a2.
x2 y2
Q.2 Tangents at any point on the hyperbola =1 −
Q.17 Find the locus of the middle point of the chords a2 b2
of hyperbola 3x2 – 2y2 + 4x – 6y = 0 parallel to y = 2x. cut the axes at A and B respectively. If the rectangle
OAPB (where O is origin) is completed then locus of
Q.18 The line y = mx + 6 is tangent to the hyperbola point P is given by-
x2 y2 a2 b2 a2 b2
− 1 at certain point. Find the value of m.
= (A) − = 1 (B) + =1
102 72 x2 y2 x2 y2
a2 b2
Q.19 A point P divides the focal length of the hyperbola (C) − = 1 (D) None of these
9x2 – 16y2 = 144 in the ratio S¢P: SP = 2: 3 where S and S′ y2 x2
are the foci of the hyperbola. Through P a straight line
is drawn at an angle of 135º to the axes OX. Find the Q.3 The locus of the foot of the perpendicular from the
points of intersection of the line with the asymptotes of centre of the hyperbola xy = c2 on a variable tangent is-
the hyperbola. (A) (x2 – y2)2 = 4c2xy (B) (x2 + y2)2 = 2c2xy
(C) (x2 – y2) = 4x2xy (D) (x2 + y2)2 = 4c2xy
Q.20 Find the equation of tangent to the hyperbola
x2 – 2y2 = 18 which is perpendicular to the line y = x. Q.4 The point of intersection of the curves whose
parametric equation are x = t2 + 1, y = 2t and x = 2s,
Q.21 If a chord joining the points P(a secθ, a tan θ) and y = 2/s is given by-
Q(a sec φ, a tan φ) on the hyperbola x2 – y2 = a2 is a normal
(A) (1, –3) (B) (2, 2) (C) (–2, 4) (D) (1, 2)
to it at P, then show that tan φ = tan θ (4 sec2θ – 1).

x2 y2
Q.22 Find the equations of the tangents to the Q.5 P is a point on the hyperbola = 1 , N is the

hyperbola x2 – 9y2 = 9 that are drawn from (3, 2). Find a2 b2
foot of the perpendicular from P on the transverse axis.
the area of the triangle that these tangents form with
The tangent to the hyperbola at p meets the transverse
their chord of contact.
axis at T. If O is the centre to the hyperbola, the OT.ON
is equal to-
Q.23 Let ‘p’ be the perpendicular distance from the
centre C of the hyperbola x2/a2 – y2/b2 = 1 to the tangent (A) e2 (B) a2 (C) b2 (D) b2/a2
drawn at point R on the hyperbola. If S and S′ are the
Q.6 The equation to the chord joining two point (x1, y1)
two foci of the hyperbola, then show that
and (x2, y2) on the rectangular hyperbola xy = c2 is-
 b2 
(RS + RS′)2 = 4a2  1 + . x y x y
 p2  (A) + = 1 (B) + =1
 x1 + x2 y1 + y 2 x1 − x2 y1 − y 2
x y x y
(C) + = 1 (D) + =1
y1 − y 2 x1 − x2 y1 − y 2 x1 − x2
1 2 . 4 2 | Hyperbola

Q.7 The eccentricity of the hyperbola whose latus (C) x = et + e–t & y = et – e–t
rectum is 8 and conjugate axis is equal to half the t
(D) x2 – 6 = 2cot & y2 + 2 = 4cos2 2
distance between the foci, is-
4 4 2
(A) (B) (C) (D) None of these Q.14 Circles are drawn on chords of the rectangular
3 3 3 hyperbola xy = a2 parallel to the line y = x as diameters.
All such circles pass through two fixed points whose
Q.8 The equation to the chord of the hyperbola x2 – y2 = 9 co-ordinates are-
which is bisected at (5, –3) is-
(A) (c, c) (B) (c, –c) (C) (–c, c) (D) (–c, –c)
(A) 5x + 3y = 9 (B) 5x – 3y = 16
(C) 5x + 3y = 16 (D) 5x – 3y = 9 Q.15 If the normal at (xi, yi) i = 1, 2, 3, 4 to its rectangular
hyperbola xy = 2 meet at the point (3, 4), then-
dx 3y (A) x1 + x2 + x3 + x4 = 3 (B) y1 + y2 + y3 + y4 = 4
Q.9 The differential equation = represents a
dy 2x
family of hyperbolas (except when it represents a pair (C) x1x2x3x4 = –4 (D) y1y2y3y4 = –4
of lines) with eccentricity-
3 5 2 5
Q.16 If (5, 12) and (24, 7) are the foci of a conic passing
(A) (B) (C) (D) through the origin then the eccentricity of conic is-
5 3 5 2
(A) 386 / 12 (B) 386 / 13 (C) 386 / 25 (D) 386 / 38
Multiple Correct Choice Type
Q.17 The value of m for which y = mx + 6 is a tangent
Q.10 Equation of a tangent passing through (2, 8) to x2 y2
the hyperbola 5x2 – y2 = 5 is- to the hyperbola − = 1 is-
100 49
(A) 3x – y + 2 = 0 (B) 3x + y – 14 = 0
 17   17   20   20 
(C) 23x – 3y – 22 = 0 (D) 3x – 23y + 178 = 0 (A)   (B) −   (C)   (D) −  
 20   20   17   17 
Q.11 The equation 16x2 – 3y2 – 32x + 12y – 44 = 0
represent a hyperbola - x2 y2
Q.18 The equation + = 1 represents-
12 − k k − 8
(A) The length of whose transverse axis is 4 3
(A) A hyperbola if k < 8
(B) The length of whose conjugate axis is 8
(B) An ellipse if 8 < k < 12, k ≠ 10
(C) Those centre is (1, 2)
(C) A hyperbola if 8 < k < 12
19
(D) Those eccentricity is (D) Circle if k = 10
3

Q.12 A common tangent to 9x2 – 16y2 = 144 and x2 + Q.19 Equations of a common tangent to the two
y2 = 9 is- x2 y 2 y 2 x2
hyperbolas − =1& − = 1 is-
3 15 2 15 a2 b2 a2 b2
(A) y = x+ (B) y = 3 x+
7 7 7 7
(A) y = x + a2 − b2 (B) y = x – a2 − b2
3 2 15
(C) y = 2 x + 15 7 (D) y = 3 x− (C) y = – x + a2 − b2 (D) y= – x – a2 − b2
7 7 7
Q.13 Which of the following equation in parametric
Q.20 The equation of the tangent lines to the hyperbola
form can represent a hyperbola, profile, where ‘t’ is a
x2 – 2y2 = 18 which are perpendicular the line y = x are-
parameter
a 1 b 1 (A) y = –x + 7 (B) y = –x + 3
(A) x =  t +  & y =  t − 
2 t 2 t (C) y = –x – 4 (D) y = –x – 3
tx y x ty
(B) − +t=0& + –1=0
a b a b
M a them a ti cs | 12.43

Q.21 The co-ordinate of a focus of the hyperbola et + e − t et − e − t


9x2 – 16y2 + 18x + 32y – 151 = 0 are- Q.4 For any real t, x = ,y= is a point
2 2
(A) (–1, 1) (B) (6, 1) (C) (4, 1) (D) (–6, 1) on the hyperbola x2 – y2 = 1. Find the area bounded by
this hyperbola and the lines joining its centre to the
points corresponding to t1 and –t2.  (1982)
Q.22 If (a sec θ, b tan θ) & (a sec φ, b tan φ) are the ends of
x2 y 2 θ ϕ
a focal chord of − =1, then tan tan equal to- Q.5 Tangents are drawn from any point o the hyperbola
a2
b 2 2 2
x2 y 2
e −1 1−e e+1 e+1 − = 1 to the circle x2 + y2 = 9. Find the locus of
(A) (B) (C) (D) 9 4
e+1 1+e 1−e e −1 mid point of the chord of contact.  (2005)

Q.23 If the normal at P to the rectangular hyperbola


x2 – y2 = 4 meets the axes in G and g and C is the centre Paragraph 6 to 7:
of the hyperbola, then- x2 y 2
The circle x2 + y2 – 8x = 0 and hyperbola − =1
(A) PG = PC (B) Pg = PC (C) PG = Pg (D) Gg = PC 9 4
intersect at the points A and B.  (2010)

Previous Years’ Questions Q.6 Equation of a common tangent with positive slope
to the circle as well as to the hyperbola is-
Q.1 If the circle x2 + y2 = a2 intersects the hyperbola
xy = c2 in four points P(x1, y1), Q(x1, y1), R(x3, y3), S(x4, y4), (A) 2x – 5 y – 20 = 0 (B) 2x – 5y+4=0
then- (1998) (C) 3x – 4y + 8 = 0 (D) 4x – 3y + 4 = 0
(A) x1 + x2 + x3 + x4 = 0 (B) y1 + y2 + y3 + y4 = 0
Q.7 Equation of the circle with AB as its diameter is-
(C) x1x2x3x4 = c4 (D) y1y2y3y4 = c4
(A) x2 + y2 – 12x + 24 = 0 (B) x2 + y2 + 12x + 24 = 0
Q.2 An ellipse intersects the hyperbola 2x2 – 2y2 = 1 (C) x2 + y2 + 24x – 12 = 0 (D) x2 + y2 – 24x – 12 = 0
orthogonally. The eccentricity of the ellipse is reciprocal
to that of the hyperbola. If the axes of the ellipse are
Q.8 The line 2x + y = 1 is tangent to the hyperbola
along the coordinate axes, then  (2009)
x2 y 2
(A) Equation of ellipse is x2 + 2y2 = 2 − = 1. If this line passes through the point of
a2 b2
(B) The foci of ellipse are (± 1, 0) intersection of the nearest directrix and the x-axis, then
(C) Equation of ellipse is x2+2y2 = 4 the eccentricity of the hyperbola is ……..  (2010)

(D) The foci of ellipse are (± 2 , 0)


Q.9 Consider a branch of the hyperbola
2 2
x y x2 − 2y 2 − 2 2x − 4 2y − 6 =with
0 vertex at the point
Q.3 Let the eccentricity of the hyperbola − =1
2 2
a b A. Let B be one of the end points of its latus rectum. If
be reciprocal to that of the ellipse x + 4y = 4. If the
22 C is the focus of the hyperbola nearest to the point A,
hyperbola passes through a focus of the ellipse, then then the area of the triangle ABC is (2008)
 (2011) 2 3 2 3
x 2
y 2 (A) 1 − (B) −1 (C) 1 + (D) +1
(A) The equation of the hyperbola is − =1 3 2 3 2
3 2
(B) A focus of the hyperbola is (2, 0)
5
(C) The eccentricity of the hyperbola is
3
(D) The equation of the hyperbola is x2 – 3y2 = 3
1 2 . 4 4 | Hyperbola

x2 y2
Q.10 Match the conics in column I with the statements/ Q.12 Let P(6, 3) be a point on the hyperbola −
= 1.
expressions in column II. (2009) a2 b2
If the normal at the point P intersects the x-axis at (9, 0),
then the eccentricity of the hyperbola is (2011)
Column I Column II
(A) Circle (p) T
 he locus of the point (h, k) for
5 3
(A) (B) (C) 2 (D) 3
which the line hx + ky = 1 touches 2 2
the circle x2 + y 2 = 4
x2 y2
(B) Parabola (q) P
 oints z in the complex plane Q.13 Let the eccentricity of the hyperbola = 1

a2 b2
satisfying z + 2 − z − 2 =
±3 be reciprocal to that of the ellipse x2 + y 2 =4 . If the
(C) Ellipse (r) P
 oints of the conic have parametric hyperbola passes through a focus of the ellipse, then
representation  (2011)
2 2
 1 − t2  x y
2t (A) The equation of the hyperbola is − =1
=x 3
= ,y 3 2
 1 + t2  1 + t2
 
(B) A focus of the hyperbola is (2, 0)
(D) Hyperbola (s) The eccentricity of the conic lies in 5
the interval 1 ≤ x < ∞
(C) The eccentricity of the hyperbola is
3
(t) P
 oints z in the complex plane (D) The equation of the hyperbola is x2 − 3 y 2 =
3
satisfying

(
Re z + 1 )
2 2
=z +1 x2 y 2
Q.14 Tangents are drawn to the hyperbola − = 1,
9 4
parallel to the straight line 2x −y = 1. The points of
Q.11 The line 2x + y = 1 is tangent to the hyperbola contact of the tangents on the hyperbola are (2012)
x2 y 2
− = 1 . If this line passes through the point of  9 1   9 1 
a2 b2 (A)  ,  (B) − ,− 
2 2 2   2 2 2 
intersection of the nearest directrix and the x-axis, then  
the eccentricity of the hyperbola is (2010)
( )
(C) 3 3, − 2 2 (D) −3 3, 2 2 ( )

PlancEssential Questions
JEE Main/Boards JEE Advanced/Boards

Exercise 1 Exercise 1
Q.7 Q.12 Q.15 Q.21 Q.5 Q.11 Q.12 Q.15
Q.24 Q.25 Q.27 Q.18 Q.22 Q.25

Exercise 2 Exercise 2
Q.3 Q.11 Q.18 Q.19 Q.3 Q.6 Q.8 Q.11
Q.17 Q.23
Previous Years’ Questions
Q.2 Q.6 Q.8
Previous Years’ Questions
Q.2 Q.3 Q.4 Q.8
M a them a ti cs | 12.45

Answer Key

JEE Main/Boards Q.8 2x – 5y = 50


π
Exercise 1 Q.9 7 rad.
4

Q.1 C(1, 2), e = 5/4, (6, 2) and (–4, 2) 4


Q.10 tan–1
3
Q.2 8x – 3y – 13 = 0
Q.14 4x2 – y2 = 25
Q.3 (6, –2)
Q.17 λ(x2 – a2) = 2xy
Q.4 a2y2 – b2x2 = 4x2y2
81y 2
Q.5 x2 + y2 = 5 Q.21 9x2 – = (x2 + y2)2 is the required locus.
4
Q.6 3x + 4y ± 36 = 0 c
Q.25 , c −m
Q.7 12x2 – 7xy – 12y2 + 31x + 17y = 0 −m

Q.26 –1

Exercise 2
Single Correct Choice Type

Q.1 B Q.2 A Q.3 B Q.4 A Q.5 C Q.6 B


Q.7 B Q.8 B Q.9 C Q.10 B Q.11 A Q.12 A
Q.13 A Q.14 D Q.15 D Q.16 C Q.17 C Q.18 B
Q.19 C Q.20 B Q.21 D Q.22 A Q.23 A

Previous Years’ Questions


Q.1 B Q.2 D Q.3 B Q.4 B Q.5 D Q.6 B
Q.7 A Q.8 B Q.9 B Q.10 B

JEE Advanced/Boards
Exercise 1
48
Q.1 7x2 + 12xy – 2y2 – 2x + 4y – 7 = 0 ; Q.2 a2 = 25/2 ; b2 = 16

5
Q.6 (i) 24/11 (ii) 20/11 Q.7 x + y ± 3 3 = 0

Q.10 3x + 2y – 5 = 0 ; 3x – 2y + 5 = 0 Q.14 (x2+y2)2 (a2y2–b2y2)= x2y2(a2+b2)2

x2 y 2 y 2 x2  4 3
Q.18 − = 1 and − = 1 Q.19 (–4, 3) and  − , − 
16 9 16 9  7 7
5 3
Q.22 y = x + ; x – 3 = 0; 8 sq. unit
12 4
1 2 . 4 6 | Hyperbola

Exercise 2
Single Correct Choice Type

Q.1 B Q.2 A Q.3 D Q.4 B Q.5 B Q.6 A

Q.7 C Q.8 C Q.9 B

Multiple Correct Choice Type

Q.10 A,C Q.11 B, C, D Q.12 B, D Q.13 A, C, D Q.14 A, D Q.15 A, B, C, D


Q.16 A, D Q.17 A, B Q.18 A, B, D Q.19 A, B, C, D Q.20 B, D Q.21 C, D
Q.22 B, C Q.23 A, B, C

Previous Years’ Questions


x2 y 2 (x2 + y 2 )2
Q.1 A, B, C, D Q.2 A, B Q.3 B, D Q.4 t1 Q.5 − =
9 4 81
Q.6 B Q.7 A Q.8 2 Q.9 B Q.10 A → p; B → s, t; C → r; D → q, s
Q.11 2 Q.12 B Q.13 B, D Q.14 A, B

Solutions

JEE Main/Boards 2.x



1.y
= 1 ⇒ 8x – 3y = 13
13 / 4 13 / 3
Exercise 1  21   116 
Sol 3: We have y =  −  x +  
 5   5 
Sol 1: 9(x2 – 2x + 1) – 16(y2 – 4y + 4) – 199 – 9 + 64 = 0
[y = mx + c form]
9(x – 1)2 – 16(y – 2)2 = 144
Now, y = mx + c is tangent when
(x − 1)2 (y − 2)2
− =1 a2m2 – b2 = c2
16 9
2
so a = 16 = 4 & b = 9 =3 232  21   232 
So .  −  
7  5   5 
b2 32 5
so e2 = 1+ = 1+ ⇒e =
a 2
4 2 4 63 × 232 232 × 5 (116)2 (116)2
= − =
25 25 25 25
Now centre would be where
So LHS = RHS
x – 1 = 0 and y – 2 = 0
Hence, the given line is tangent
⇒ x = 1 and y = 2
Now tangent
5
and focii distance = ae = 4 × (in x-direction) x.x1 y.y1
4 ⇒ − =1
282 / 7 232 / 5
focii = (1 + 5, 2) and (1 – 5, 2)
Now, comparing with the given tangent
(6, 2) and (–4, 2)
21 × 232 5 × 232 116
= =
x.x1 y.y1 x1 × 7 −5y1 1
Sol 2: Tangent ⇒ − =1
2
a b2 ⇒ x1 = 6 and y1 = –2
M a them a ti cs | 12.47

x y ⇒ c = ± 3 × 3 = ±9
Sol 4: Tangent = sec θ − tan θ = 1
a b
3 3
Now the tangent cuts the axes at (a cosθ, 0) and y = − x + 9 or y = − x – 9
4 4
(0, b cot θ)
b
acos θ bcot θ equation of asymptotes ⇒ y = ± x
mid points ⇒ = h and k = a
2 2
a b Sol 7: Equation (3x – 4y + 7)(4x + 3y + 1) + c = 0
⇒ = sec θ and = tan θ
2h 2k ⇒ 12x2 – 12y2 – 7xy + 31x + 17y + (7 + c) = 0
a2 b2 a2 b2 Now, it passes through origin
⇒ − =1 ⇒ − =4
4h2 4k 2 h2 k2 ⇒ 7 + c = 0 ⇒ c = –7
Replacing h and k, we get locus as ⇒ equation = 12x2 – 12y2 – 7xy + 31x + 17y = 0

a2 b2 5
− =4 ⇒ a2 y 2 − b2 x2 =4x2 y 2 Sol 8: xy = 25 ⇒ parametric ⇒ 5t & y =
2 2
h k t
dy dy / dt ( −5) / t2 −1
= = =
Sol 5: We have tangents dx dx / dt 5 t2
⇒ y = mx ± a2m2 - b2 ⇒ y = mx ± 9m2 − 4
(-5,2)
2
⇒ (y – mx) = ( 9m − 4 )
2 2
(5t,5/t)
y + m x – 4mxy = 9m – 4
2 2 2 2

⇒ (9 – x2)m2 + (4xy)m (4 + y2) = 0

Now h, k would satisfy this

⇒ (9 – h2)m2 + (4hk)m (4 + k2) = 0


−1 (5 / t − 2)
−(4 + k 2 ) Now slope = 2
=
So, m1m2 = =1 t (5t + 5)
9 − h2
⇒ 4 + k2 = 9 – h2 ⇒ h2 + k2 = 5 −1 (5 − 2t)
⇒ =
t 5t(t + 1)
Hence, the locus is x2 + y2 = 5
⇒ –5(t + 1) = t(5 – 2t)
4
Sol 6: We have m1 = (given line) ⇒ 2t2 – 10t – 5 = 0
3
3 Now chord of contact
Given m1. m2 = –1 ⇒ m2 = −
4
(5 / t1 − 5 / t2 )x −x
3 ⇒ y= +c = +c
So y = − x + c (5t1 − 5t2 ) t1 .t2
4
5 −5t1
⇒ Now putting this in the equation Now, = +c
t1 t1 .t2
 3  3
x.  − x + c  = 27 ⇒ − x2 + cx = 27 1 1  −x t + t 
 4  4 ⇒ c = 5 + ⇒ y = +5 1 2 
3 2  t1 t2  t1 + t2  t1 .t2 
⇒ x − cx + 27 = 0
4 +x 5.[5]
⇒ y= +
has only one solution ⇒ D = 0 ( +5 / 2) −5 / 2
⇒ b2 – 4ac = 0 2x
⇒ y= – 10 ⇒ 5y = 2x – 50
3 5
c2 – 4 × × 27 = 0
4
1 2 . 4 8 | Hyperbola

Sol 9: 4. sec2θ + 4 tan2θ = 12 [i. e. , q1 = q2]


⇒ sec2θ + tan2θ = 3 (1 – 1)(a2 + b2) = 0
⇒ 2tan2θ = 2
Sol 12: The asymptotes are x = 0, y = 0
⇒ tan θ = ±1
c
Let, x = ct, y = ,
⇒ θ = tan (–1) [from 4 quadrant]
–1 th
t
7π 1
⇒ θ= tangent slope = −
4 t2
Now normal at x1, y1
x2 y 2
Sol 10: − =1 ⇒ b has slope = t2
4 16
x y x y (y − c / t)
asymptotes ⇒ − = 0 and + = 0 so, = t2
2 4 2 4 (x − ct)
2
⇒ y = 2x and y = –2x  y − y1   x1 
⇒   =  
(m1 − m2 )  x − x1   c 
Now angle ⇒ tan–1
1 + m1 .m2
 x2 
[2 − ( −2)] 4 ⇒ y – y1 =  1  . (x – x1)
= tan–1 = tan–1  c2 
 
1−4 3
x12  x3 
y= . x +  y1 − 1 
Sol 11: Equation of hyperbola c2  c2 

⇒ ax cosq1 + by cot q1 = a2 + b2
[a cos θ, b cot q]
Equation of tangent
x y
⇒ sec θ2 − tan θ2 = 1
a b
[a secq2, b tanq2]
Now putting
Intersection of tangents
x13
x=0⇒ y y1 −
=
⇒ (a cos q2, 0) and (0, –b cotq2) c2
Intersection of normal and putting y = 0,
 sec θ (a2 + b2 )   (a2 + b2 )  x12  x3 
1
⇒  ,0  and  0, . tan θ1  .x =  1 − y1 
 a   a  2 c 2 
    c  
acos θ2 .sec θ1 .(a2 + b2 ) (x13 − y1c2 )
Now, a1. a2 + b1. b2= x=
a x12
(a2 + b2 )
+ (–b). cotq2× tanq1 y1
b x x1 −
= .c2
x12
= [cosq2. secq1 – cotq2. tanq1] (a2 + b2)

 cos θ2 cos θ2 .sin θ1  2 1  x3   y 


=  −  (a + b ]
2 Area =  y1 − 1   x1 − 1 c2 
 cos θ1 cos θ1 .sin θ2  2  c2   x12 

Now if the point is same: 1  y 2 .c2 x14 


=  y1 .x1 + x1 .y1 − 1 − 
2  x 2
c2 
 1
M a them a ti cs | 12.49

⇒ sec2θ – tan2θ = 1
1  c x x1 .y1 x1 y12 .c2 
2 2 4
=  − − 
2 
 c2 c2 (c2 / y1 )2  a2h2 b2k 2
⇒ − =1
(a2 + b2 )2 (a2 + b2 )2
2
4 4 2 2
1 x1 y1 1 x1 − y1 a2 x2 b2 y 2
= + − 2(x1 .y1 )2 = ⇒ − =1
2 c2 c2 2 c (a2 + b2 )2 (a2 + b2 )2

⇒ a2x2 – b2y2 = (a2 + b2)2


Sol 13:
here, a2 = 4, b2 = 1
2
b
2
a ⇒ 4x2 – y2 = 25
A’ A N
(-a) (a,0) (asec,0)
Sol 15: Tangent

P (a sec, b tan)

 ab2 + a3 sec θ 2 
Q=  ,a b tan θ 
 a2 + b2  O T S’
  S

slope of NQ

a2b tan θ a2b tan θ − 0


= = Coordinates of S’ = (ae, 0)
ab2 + a3 sec θ ab2 − ab2 sec θ
− asec θ S = (‒ae, 0)
a2 + b2
P = (a sec θ, b tan θ)
atan θ atan θ
= = Tangent at P cut the x axis at point T.
b − bsec θ b(1 − sec θ)
x sec θ y tan θ
(b tan θ − 0) Eq. of target at P = − =1
slope of A⇒ P = a b
a(sec θ + 1)
 a −b 
b tan θ.atan θ tan2 θ Coordinates of T =  , 
⇒ m 1. m2 = = = –1  sec θ tan θ
ab(1 − sec2 θ) − tan2 θ
=
T ( aCosθ, −b cot θ )
⇒ Hence proved.
⇒ ST = ae + acosθ
Sol 14: ⇒ S' T =ae − acos θ
(h,k) ST ae + acos θ e + cos θ
(0,k) =
⇒ =
ST ae − acos θ e − cos θ

Similarly on evaluating PS & Ps


(h,0)
PS PS
⇒ =
ax cosθ + by cot θ = a2 + b2 PS' PS'

x y S' T × −h
Which should be same as + =1 ∴ Area of ∆ PTS' = P
h k 2
S' T × h
 
Area of ∆PTS =
acos θ bcot θ a2 + b2 2
⇒ = =
1/h 1/k 1 Using fine rule: S O T S’

PS'× PT sin φ
a2 + b2 a2 + b2 Area of ∆ PTS' =
⇒ h= ,k= 2
acos θ bcot θ
PS'× PT sin θ
ah bk Area of ∆ PTS' =
⇒ = secθ, =tanθ 2
2 2
a +b a + b2
2
1 2 . 5 0 | Hyperbola

Area of ∆PTS' PS sin θ ST k2 + m2h2 – 2m. kh = a2m2 – b2


= =
Area of ∆PTS' PS'Sinφ S'T (a2 – h2)m2 + 2m. kh – (b2 + k2)

For θ = d the conditions necessary are met & PT bisect −2kh


Now m1 + m2 = λ =
the angle sps’ a2 − h2

c −2kh −2xy
Sol 16: Let x = ct and y = ⇒ a2 – h2 = ⇒ a2 − x2 =
t λ λ

dy dy / dt −1 / t −1
then, = = =
dx dx / dt t t2 Sol 18: x = a secθ, b = tan θ
so normal = t 2
x.sec θ y
− . tan θ = 1
(y − c / t) c a b
thus, = t2 ⇒ y – = t2x – ct3
(x − ct) t
⇒ ty – c = t3x – ct4 ⇒ ct4 – t3x + ty – c = 0 R

this satisfies h, k
Q
thus,ct4 – ht3 + kt – c = 0
(acos,0)
h h
h
thus, ∑ ti = ⇒ ∑ c.ti =h
i=1 c i=1
Now coordinate of Q ⇒ x = a cosθ.
⇒ x1 + x2 + x3 + x4 = h
Coordinates of R
similarly we have t1. t2. t3. t4 = –1 and
−k (sec θ − 1)b
(t1. t2. t3) + t2. t3. t4 + t3. t4. t1 + t4. t1. t2 = ⇒ =y
e tan θ
h
dividing by ∏ ti both sides Now
i=1
2h = a + a cosθ = a(1 + cosθ). . (i)
c c c c
⇒ + + + =k ⇒ Σy i =
k (sec θ − 1)b
t1 t2 t3 t 4 and2k = 0 +
tan θ
(iii) = –1
(1 − cos θ)b
2k =
⇒ –c4 = c4. sin θ
–c4 = x1. x2. x3. x4 Now
(1 − cos θ)2 b2 + b2 sin2 θ
And = –1 4k2 + b2 =
sin2 θ
c c c c
⇒ –c4 = . . . ⇒ y1. y2. y3. y4 = –c1
t1 t2 t3 t 4 (2 − 2cos θ)2 b2
(4k2 + b2) =
sin2 θ
a.sec θ.x b. tan.y
Sol 17: tangent ⇒ – =1 2(1 − cos θ)b2 2.(b2 )
2
a b 2 = = from (i)
sin2 θ (1 + cos θ)
x y
sec θ − tan θ =1 2b2
a b ⇒ 4k2 + b2 =
(2h / a)
tangent ⇒ y = mx ± a2m2 − b2
⇒ ab2 = h(4k2 + b2)
tangent passes through h, k
Sol 19: Let x = 2at
k = mh + a2m2 − b2
y = at2
(k – mh) = a m – b
2 2 2 2
dy 2at
then = =t
dx 2a
M a them a ti cs | 12.51

thus equation of tangent ⇒ 2ahk = (h2 – k2) ( k 2 − h2 )


y – at2 = t. (x – 2at)
⇒ 4a2h2k2 = (k2 – h2)3
y – at = xt – 2at
2 2

⇒ at2 – xt + y = 0 Sol 21: (3 secθ, 2tanθ) = point on hyperbola


k2 Now equation of the chord of contact is hx + ky = h2 + k2
Now x = in the above eqn
y and also3secθ x + 2 tanθ. y = 9
 k2  h h2 + k 2 k
⇒ at2 –   t + y = 0 so = =
 y  3sec θ 9 2 tanθ
 
⇒ y2 + yat2 – k2t = 0 3h 9k
⇒ secθ = and tan θ =
2 2
h +k 2(h + k 2 )
2
Now the2k = –at2
⇒ sec2θ – tan2θ = 1
k2
and similarly, y = gives
x 9h2 81k 2
⇒ − =1
k2 (h2 + k 2 )2 4(h2 + k 2 )2
xat – x t +
2 2
=0
x
81k 2
x t – at x – k = 0
2 2 2 9h2 − =(h2 + k 2 )2
4
at2
so x1 + x2 = = at ⇒ 2h = at
t
x2 y2
2 Sol 22: Let − 1 be the hyperbola then its
=
4h −ak a2 b2
so = at2 = –2k ⇒ h2 =
a 2 y2 x2
conjugate hyperbola is − 1.
=
Thus, it is a parabola. b2 a2

If e1 and e2 are their eccentricities, then


Sol 20: (a sec, a tan)
=(
b2 a2 e12 − 1 and
= a2 b2 e22 − 1 ) ( )
1 a2 1 b2
So = and =
e12 (a 2
+ b2 ) e22 (a 2
+ b2 )
1 1 1 1
So + 1, ⇒
= = 1−
2
dy a.sec2 θ 1 e1 2
e2 2 e1 e22
Now =
dx atan θ.sec θ sinθ
⇒ slope of normal = –sinθ Sol 23: Let the hyperbola be xy = c2.
Now hx – ky = –h2 – k2 (chord of equation) Let the x1, y1 be point where the other 3 normals
intersect.
−h
slope = = sinθ
k Now, equation of normal

k 2 − h2 −h  c
cosθ = and tan θ = ⇒  y −  = t2(x – ct)
k  t
k 2 − h2
⇒ ty – c = t3x – ct4
 a.k −ah 
so points A  ,  ⇒ ct4 – t3x + ty – c = 0
 2 2 
 k −h k 2 − h2 
Thus, passes through
this satisfies the line
h.ak ahk
⇒ + =(h2–k2)
2 2 2 2
k −h k −h
1 2 . 5 2 | Hyperbola

(x1, y1) or (cx1, c/t1) ⇒ normal = t2

So ct4 – t3. x1 + ty1 – c = 0


Now slope= t2
x1 c c
Now Sti = & product of roots ti = –1 −
c t t1 −1
= =
− y1 c(t − t1 ) t. t1
⇒ Sxi = x1 & St1. t2. t3 =
c ⇒ t1 . t3 = –1
Σ1 y1
⇒ x2 + x3 + x4 = 0 =
ti c
Exercise 2

xc = 0⇒ y1+y2+y3+y4=y1 Single Correct Choice Type
y2+y3+y4 = 0
Sol 1: (B) We have x. x1 – 9y. y1 = 9

5 12 9
so = =
yc = 0 x1 −9y1 9
Thus, the centroid of PQR is (0, 0) −4
⇒ y1 = and x1 = 5
3
Sol 24: Equations of normal at the points
x2 y2
Sol 2: (A) − = –1
x2 y2 a2 b2
on the hyperbola − 1, are
=
a2 b2 2a2
latus rectum =
b
ax cos φ + by cot φ= a2 + b2  … (i)
Sol 3: (B)
2 2
and ax cos φ + by cot φ= a + b

a
i.e. ax sin θ + by tan θ= a2 + b2  … (ii)
e
π
 θ+φ =
2 4a2
Solving (i) and (ii), y = k = −
(a 2
+ b2 ). area =
2a 2a
×
e e
=
e2
b
for rectangular hyperbola e = 2
Sol 25: Tangent to the hyperbola xy = c at (ct, c/t) will 2
area = 2a 2

1 2c
be of the form y = − x+ x2 y 2
t 2 t Sol 4: (A) − =1
9 4
1 2x
y = mx + 2c −m ⇒ t = y= +C
−m 5
 c   x2 − 9 
∴ Point is  ,c −m  4x2 4Cx
4  = + C2 +
 −m   9  25 5
 
c 16 4Cx
Sol 26: x = ct and y = 4x2 × − − 4 − C2 = 0
t  225 5
dy −1 64x2 – 180Cx – 180 – 45C2 = 0
Now =
dx t2 D=0
t1
180C2 = 4×64(–180 – 45C2)
M a them a ti cs | 12.53

⇒ C2 = 64(–4 – C2) Ellipse Hyperbola

⇒ C2 < 0 no possible tangent x2 y2 x2 y2


⇒ + = 1⇒ − =1
a12 b2 a22 b2
x y
Sol 5: (C) − =m
3 2 Now a1e1 = a2e2
x y 1 2 b2 2 2
+ = also e1 = 1 – b = ( e2 –1)a22
3 2 m 2
a1
1 1 1 1
1 + = +
⇒ y2 = + m2 – 2 2
e1 e2 2
b 2
b2
m2 1− 2 1+ 2
a1 a2
3 1
⇒ x= m + 
2 m a12 a22
= + … (i)
a12 − b2 a22 + b2
4x 2 1 
⇒ = m2 + +2
9 m 2 Also we have,

4x2 a1e1 = a2e2


⇒ – y2 = 4
9 ⇒ a12 – b2 = a22 + b2 … (ii)
2 2
x y ⇒ a12 + a22 = 2(a22 + b2) … (iii)
⇒ − =1
9 4
Now from (i)
2 2 2
x y 9x 4
Sol 6: (B) − =1 − =1 1 1 a12 + a22 2(a22 + b2 )
a 2
b 2
a2
b 2 ⇒ + = = =2
e12 e22 a22 + b2 a22 + b2
4
⇒ a2 = 32⇒ =1
b2
Sol 9: (C)
⇒ a = 3⇒ b2 = 4
a2 + b2 b2 + a2
We have e1 = and e2 =
x2 y 2 a2 b2
⇒ − 1
=
9 4
1 1
⇒ + =1
2 e12 e22
b 4 13
⇒ e= 1+ =1+ =
2
a 9 3
Sol 10: (B) Equation of normal at any point x′, y′ of the

Sol 7: (B) (x – 2)2 – (y – 2)2 + 16 = 0 x2 y2


curve − = 1 … (i)
(y − 2)2 (x − 2)2 a2 b2
⇒ − =1
16 16 a2 (x − x′) b2 (y − y′)
+ =0
16 x′ y′
⇒ e= 1+ = 2
16 a2 x b2 y
⇒ − a2 − b2 + =0
x′ y′
Sol 8: (B)
a2 x b2 y
+ = a2 + b2 (h, k) satisfy this)
x′ y′

a2h b2k
⇒ + = a2 + b2
x1 y1

⇒ a2h. y1 + b2k1x1 = (a2 + b2)(x1, y1)  … (iii)


thus, (x1, y1) lies on curve (iii) and curve (i) these two
1 2 . 5 4 | Hyperbola

points intersect at 4 points. ⇒ m2(h2 – cos2α) – 2kh. m + (k2 + sin2α)

Now we have m1. m2 = –1


Sol 11: (A) A rectangular hyperbola circumscribing a
triangle ABC always passes through the or the centre. h2 − cos2 α
= –1
k 2 + sin2 α
dy (c / t2 ) −1 ⇒ h2 + k2 = cos2α – sin2α
Sol 12: (A) We have = =
dx c t2 h2 + k2 = cos2α
so normal slope = t2
Now, Sol 16: (C) We have b = ae

b2
b = a 1−
a2 b

ae
b2 1
⇒ =
a2 2
c / t − c / t′
We have t2 =
ct − ct′ a2
ehyperbola = 1+ = 1+2 = 3
b2
(t′ − t)( −1) −1
⇒ t2 = ⇒ t′ =
t.t′.(t − t′) t3
Sol 17: (C) Let any tangent of (x1, y1)
x.x1 y.y1
Sol 13: (A) 9(x2 – 4x + 16) – 16(y2 – 6y + 9) then + = 1 [1st tangent]
2
a b2
–252 + 144 – 144 = 0
and x. x1 – y. y1 = c2 [2nd tangent]
⇒ 9(x – 2)2 – 16(y – 3)2 = 252
Now, m1. m2 = –1
⇒ Centre ⇒ (2, 3)
 −b2   x1   x1 
⇒   = –1
x2 y 2  a2   y   y 
Sol 14: (D) − =1   1  1 
25 16 (4)
tangents ⇒ −b2  x1 
2  x2 
1
⇒ .   = –1 ⇒ +b2  2  = + a2 … (i)
-5 5 2  2 
a  y1   y 
y = mx + 25m2 − 16  1
(-4)
⇒ (y – mx) = 25m – 16
2 2
x12 y12
Now + = 1 & x12 – y12 = c2
⇒ the point (1, 2 2 ) satisfy this a2 b2

⇒ (1 – 2 2 m)2 = 25m2 – 16 y12 + c2 y12 1 1  c2 


⇒ + = 1 ⇒ y12  +  = 1 − 
⇒ 1 + 8m2 –4 2 m = 25m2 – 16
a2 b2  a2 b2   a2 

⇒ 17m2 + 4 2 m – 17 = 0 [a2 − c2 ] b2 [a2 − c2 ]


y12 = =
1 1 a2 + b2
⇒ m1. m2 = –1 a2  2 + 2 
a b 

Sol 15: (D) y = mx + a2m2 − b2 [a2 − c2 ]b2


And x12 = c2 +
a2 [b2 + a2 ]
⇒ y = mx + cos2 α.m2 − sin2 α
a2b2
(k – mh) = cos α. m – sin α
2 2 2 2

b2c2 + a2c2 + a2b2 − c2b2 a2 (b2 + c2 )


= =
⇒ k + m h – 2mkh
2 2 2
b2 + a2 (a2 + b2 )
= cos2α. m2 – sin2α
M a them a ti cs | 12.55

a2 (b2 + c2 ) 1
so b2. × = a2
2 2
(a + b ) b (a − c2 )
2 2

a2 + b2 (h,k)
⇒ a2 – b2 = 2c2 (ct2,c/t2)

y2 y
Sol 18: (B) + +1=0
b 2
ba2 Now, 2h = c(t1 + t2)
2
 1  4 ×1 c c c.(t1 + t2 )
a2 – 4ac ⇒  − ≥0 and 2k = + =
2  t1 t2 t1 .t2
 ba  b2
1 4 1 4 c / t2 − c / t1 −1
⇒ − ≥ 0[b2 > 0] ⇒ − ≥0 m= =
b a 2 4
b 2
a 4 1 ct2 − ct1 t1 t2

1 1 1 −c.(t1 + t2 )
⇒ ≥4⇒ ≥2⇒ ≥ a2 m=
a 4 2
a 2 t1 .t2c(t1 + t2 )

−2k
hx ky h2 k2 m=
Sol 19: (C) − = − 2h
a2 b2 a2 b2 ⇒ k + mh = 0 ⇒ y + mx = 0
2 2
hα kβ h k
− = −
a 2
b 2
a 2
b2 Sol 23: (A) Let (h, k) be the midpoints of chords having
slope 2
2 2
h α  k β  α2 β2 2 1
⇒  −  − −  = −
⇒ tan θ =2 ⇒ sin θ = and co s θ =
 a 2a   b 2b  4a2 4b2 5 5
h α β Let the two endpoints of the chord be a distance r from
⇒ Centre: = and k = (h, k)
a 2a 2
⇒ endpoints of the chord are
x2 y2
Sol 20: (B) − =1
5 5cos2 α (h + r cos θ, k + r sin θ ) and (h − r cos θ, k − r sin θ )
 r 2r   r 2r 
b2 2 h
=+ ,k+ h
 and =− ,k− 
so e1 = 1+ = 1 + cos α 5 5  5 5
a2 

x2 y2 Plugging in the equation of the hyperbola


+ =1
25cos α 2 25 2 2
 r   2r   r   2r 
3h +  − 2k +  + 4 h +  − 6 k + =0 …(i)
e2 = 1 − cos2 α  5  5  5  5
and
1 + cos2α = b. (1 – cos2α) 2 2
 r   2r   r   2r 
1 1 3h −  − 2k −  + 4 h −  − 6 k − =0 …(ii)
⇒ cos2α = ⇒ cos α = 5 5 5 5
2 2    
Subtracting eqn. (ii) from (i),
Sol 21: (D) a2 = 9 and b2 = 4
12 hr 8kr 8r 24r
x2 y 2 − + − 0
=
− =1 5 5 5 5
9 4
⇒ 3h − 2k − 4 =
0
4x2 – 9y2 = 36
⇒ required locus is
Sol 22: (A) We have equation 3x – 4 y = 4.
1 2 . 5 6 | Hyperbola

a2 + b2  a2 + b2 
Previous Years’ Questions Thus, y = –
b
i.e., k = − 
 b 

 
Sol 1: (B) Given equation is
Sol 3: (B) Let (h, k) be point whose chord of contact
x2 y2 with respect to hyperbola x2 – y2 = 9 is x = 9.
− = 1, where | r | < 1
1−r 1+r We know that, chord of contact of (h, k) with respect to
⇒ 1 – r is (+ve) and 1 + r is (+ve) hyperbola x2 – y2 = 9 is T = 0
x2 y2 ⇒ h.x + k(–y) – 9 = 0
∴ Given equation is of the form − =1
2 2
a b ∴hx – ky – 9 = 0
Hence, it represents a hyperbola when | r | < 1. But it is the equation of the line x = 9.
This is possible when h = 1, k = 0 (by comparing both
Sol 2: (D) Firstly we obtain the slope of normal to equations).
x2 y2 Again equation of pair of tangents is T2 = SS1.
− = 1 at (a secθ, b tanθ)
a2 b2 ⇒ (x – 9)2 = (x2 – y2 – 9)(t2 – 02 – 9)
On differentiating w.r.t. x, we get ⇒ x2 – 18x + 81 = (x2 – y2 – 9)(–8)
2x 2y dy dy b2 x ⇒ x2 – 18x + 81 = –8x2 + 8y2 + 72
− × =0⇒ =
a 2
b 2 dx dx a2 y
⇒ 9x2 – 8y2 – 18x + 9 = 0
Slope, for normal at the point (a sec θ, b tan θ) will be

a2b tan θ a Sol 4: (B) Given equation of hyperbola is


− = − sin θ
2
b asec θ b x2 y2
− =1
∴ Equation of normal (asecθ, b tanθ) is cos2 α sin2 α

a Here, a2 = cos2α and b2 = sin2a


y – b tanθ = − sinθ(x – a secθ)
b x2 y2
[We, comparing with standard equation − = 1]
⇒ (a sinθ)x + by = (a2 + b2) tanq a2 b2
We know, foci = (± ae, 0)
⇒ ax + b cosecθ = (a2 + b2) secθ …(i)
x2 y 2
Similarly, equation of normal to − =1 at (a secθ, where ae = a2 + b2 = cos2 α + sin2 α =1
btanθ) is a2 b2
⇒ foci = (± 1, 0)
ax + b cosecφ = (a2 + b2) secθ  …(ii)
whereas vertices are (± cos α, 0)
On subtracting eqs.(ii) from (i), we get
1
b(cosecθ – cosecφ)y eccentricity, ae = 1 or e =
cos α
= (a2 + b2)(secθ – secφ) Hence, foci remain constant with change in ‘α’.
2 2
a +b sec θ − sec ϕ
⇒ y=
b cosecθ − cosecϕ Sol 5: (D) The equation of tangent at (x1, y1) is xx1 – 2yy1

sec θ − sec ϕ = 4, which is same as 2x + 6 y=2


But
cosecθ − cosecϕ x1 –2y1 4
∴ = =
2 6 2
sec θ − sec( π / 2 − θ)
= ⇒ x1 = 4 and y1 = – 6
cosecθ − cosec( π / 2 − θ)

(

φ + θ = π/2) Thus, the point of contact is (4, – 6 )

sec θ − cosecθ x2 y 2
= = –1 Sol 6: (B) The eccentricity of + = 1 is
cosecθ − sec θ 16 25
M a them a ti cs | 12.57

16 3
e1 = 1− = Sol 9: (B) Equation of normal to hyperbola at (x1, y1) is
25 5
a2 x b2 y
5 + = (a2 + b2)
∴e2 = (∵e1e2 = 1) x1 y1
3 a2 x b2 y
∴At (6, 3) ⇒ + = (a2 + b2)
⇒ Foci of ellipse (0, ±3) 6 3
⇒ Equation of hyperbola is It passes through (9, 0)
x2 y 2 a2 .9
− = –1 ⇒ = a2 + b2
16 9 6
3a2 a2
⇒ – a2 = b2 ⇒ =2
Sol 7: (A) The given ellipse is 2 b2
x2 y 2 b2 1
+ = 1 ⇒ a = 2, b = 3 ∴e2 = 1 + =1+
4 3 a2 2
1
∴3 = 4(1 – e2) ⇒ e = 3
2 ⇒ e=
2
1
∴ae = 2 × =1
2 1 ae
Sol 10: (B)
= 2b . ( 2ae )=
⇒b
Hence, the eccentricity e1, of the hyperbola is given by 2 2

a2e2
1 = e1sin θ ⇒ e1 = cosec q
(
⇒ a2 e2 − 1 = ) 4
⇒ 3e2 = 4 ⇒ e =
2
3
⇒ b2 = sin2θ(cosec2θ – 1) = cos2q
x2 y2
Hence, equation of hyperbola is − =1
sin2 θ cos2 θ JEE Advanced/Boards
or x2cosec2θ – y2sec2θ = 1
Exercise 1
Sol 8: (B) Given equation can be rewritten as

(x − 2)2 (y + 2)2 SP
− =1 Y Sol 1: = 3
4 2 PM
B
For point A(x, y) (x − 1)2 + (y − 1)2
X’ X = 3
2 A (2x + y − 1)
3 C
e= 1+ = 5
4 2
Y’
Squaring
x– 2 =2⇒ x=2+ 2
5[(x – 1)2 + (y – 1)2] = 3(2x + y – 1)2
For point C(x, y)
⇒ 7x2 – 2y2 + 12xy + 4y – 2x – 7 = 0
x– 2 = ae = 6

x= 6+ 2 x2 y2
Sol 2: − =1
a2 b2
Now,
7x + 13y = 87
AC = 6+ 2 –2– 2 = 6 –2
5x – 8y = –7
b2 2
andBC = = =1 87 − 7x 5x + 7
a 2 ⇒ =
Area of ∆ABC 13 8

1 3 ⇒ 8. 87 – 7. 13 = 121 x
= × ( 6 − 2) × 1 = – 1 sq. unit
2 2 ⇒ 121x = 605
1 2 . 5 8 | Hyperbola

x = 5, y = 4 Centre (–1, 2)
2b2 32 2 16 5
= e= 1+ =
a 5 9 3
5b2 = 16 2 a
foci = (–1 + ae, 2) = (4, 2)
25 16
− =1
a2 b2 = (–1 – ae, 2) = (–6, 2)
25 16 9 4
− =1 Directrix x + 1 = ⇒ x=
a2
a 2 5 5
−9 −14
25 2 – 5a = a2 2 x+1= ⇒ x=
5 5
a2 2 + 5a – 25 2 =0 2b2 2.16 32
Latus rectum = = =
a 3 3
−5 ± 25 + 200 5 −10
a= = or Length of major axis = 2×4 = 8
2 5 2 2
Length of minor axis = 2×3 = 6
Now, 5b = 16 2
2a Equation of axis is y = 2
5
⇒ a>0⇒ a=
2 Sol 5: P1(ct1, c/t1) P2(ct2, c/t2)
x2 y2 2h
Sol 3: − =1 t1 + t2 =
100 25 c

1 5 1 1 2k 2h 2k h
e= 1+ = + = ⇒ = t1t2=
4 2 t1 t2 c ct1 t2 c k
2
 5  1 1
S = (ae,0) =  × 10,0  = ( 5 5 , 0) c (t1 – t2) + c  −  = 4d2
2 2 2
 2 
   t1 t2 
S′ = (–ae, 0) = (– 5 5 , 0) 1 1
2

(t1 + t2)2–4t1t2+  + 
A = (10, 0)  t1 t2 
SA = (10 – 5 5 ) 4 4d2
– =
t1 t2 c2
S′A = (10 + 5 5 )
2 2
SA. S′A = 100 – 75 = 25  2h   2k  4 4d2
  +   – 4t1t2 – =
 c   c  t1 t2 c2
Sol 4:
(2h)2 + (2k)2 h k  4d2
− 4 +  =
c2 k h c2

(h2 + k 2 ) (h2 + k 2 ) 2d2


2 −2 =
c2 kh c2

16x2 – 9y2 + 32x + 36y – 164 = 0 (h2 + k2)hk – c2(h2 + k2) = d2kh

16x2 + 32x – 9y2 + 36y = 164 (h2 + k2)(hk – c2) = d2kh

16(x + 1)2 – 9(y – 2)2 Hence proved.

= 164 + 16 – 36 = 144
Sol 6: y – 2 = m(x – 6)
(x + 1)2 (y − 2)2
− =1 y = mx + 2 – 6m
9 16
M a them a ti cs | 12.59

x2 (mx + 2 − 6m)2 Using componendo rule we get


−1 =
25 16
1−e θ  θ 
= tan  1  tan  2 
16(x – 25) = 25(m x + 4 + 36m + 4mx – 24m – 12m x)
2 2 2 2 2
1+e  2   2 
x2 (16 – 25m2) + x(–100m + 300m2) – 400 – 100 – 900
m2 + 600 m = 0 Sol 9: e = (0, 0)
(300m – 100m) = 4(16 – 25m )(–900m + 600m – 500)
2 2 2 2
S(ae, 0)
100(3m2 – m)2 = 4(16 – 25m2)(–9m2 + 6m – 5) S′(–ae, 0)
25(9m4 + m2 – 6m3) = –144m2 + 96m – 80 + 225m4 –
P = (a secθ, b tanθ)
150m3 + 125m2
SP. S⇒ P =
25m2 = –19m2 + 96m – 80
44m2 – 96m + 80 = 0 ((asec θ − ae)2 + b2 tan2 θ)((asec θ + ae)2 + b2 tan2 θ)

11m2 – 24m + 20 = 0
= (a2 sec2 θ + a2e2 + b2 tan2 θ)2 − (2a2esec θ)2
24
m 1 + m2 =
11 = a2sec2θ + b2tan2θ – (a2 + b2)
20
m 1m 2 = = CP2 – (a2 + b2)
11
5
Sol 10: y – = mx
Sol 7: y = –x + c 2
x2 – 4(c – x)2 = 36 5
y = mx +
2
x2 – 4(c2 + x2 –2cx) = 36
 25 
3x2 – 8cx + 4c2 + 36 = 0 3x2 – 25 = 2  m2 x2 + + 5mx 
 4 
⇒ x + y = ±3 3
75
64c2 = 12(4c2 + 36) x2(3 – 2m)2 – 10mx – =0
2
16c2 = 12(4c2 + 36)
 75 
100m2 = 4(3 – 2m2)  − 
4c2 = 3c2 + 27  2 
c2 = 2 ⇒ c = ±3 3 50m2 = 150m2 – 225

100m2 = 225
Sol 8: Equation of chord
9 3
x  θ − θ2  y   θ1 + θ2    θ1 + θ2  m2 = ; m = ±
cos  1  −  sin    = cos  4 2
a   
 2  b   2   2 
2y = 3x + 5 or 2y + 3x = 5
If it pass through (ae, 0)
θ + θ2  y −k b2h
cos  1  Sol 11: =
 2  x −h a2k
e=
θ − θ2 
cos  1  ⇒ x2 + y2 = a2
 2  2
 a2k 
θ − θ2  h + 2 (y − k) + y2 = a2
cos  1   bh 
1  2 
=
e  θ1 + θ2  a4 k 2 2a2k
cos   h2+ (y2 + k2 – 2ky)+ (y – k) + y2 = a2
 2  4 2
b h b2
1 2 . 6 0 | Hyperbola

−h
 a4 k 2   −2k 3a4 2a2k  Slope of normal =
k
y2
 4 2
+ 1  + y  4 2 + 2  + h2 – a2
 b h   b h b 
[slope of OR =
k
]
h
a4 k 4 2a2k 2
+ − =0 that has equation:
b 4h2 b2
a2 x b2 y
2a2k 2k 3a4 + = a2 + b2
− x1 y1
y1 + y 2 b2 h2b 4
=
y1 y 2 a4k 4 2a2k 2 a2 y1 −a2 b − tan θ −a
h2 − a2 + 4 2 − ⇒ slope = – × = × = sin θ
b h b2 b 2 x1 b 2 asec θ b

2a2h2kb2 − 2k 3a4 2a2ka2b2 2 h a bh


= = = so + = + sinθ ⇒ = sin θ
4 4 2 2 4
h b − a h b + a k − 2a k b h 4 4 2 2 2 2
k ab2 4 2 k k b ak

a2k 2 − b2h2
x2 y 2 ⇒ cos θ =
Sol 12: − = 1; y – β = m(x – α) ax
2 3
x2 − 2 1 bh
= (mx – mα + β)2 and tanθ =
α 3
a k − b2h2
2 2

2
3x= − 6 2(m2 x2 + m2 α2 + β2 − 2m2 αx − 2mα + 2mxβ) Putting
x2 (3 − 2m2 ) + 2x(2m2 α − 2mβ) − 6
bh a2 x
−2m2 α2 − 2β2 + 4mαβ = 0 x1 = b − , y1 = asec θ =
a2k 2 − b2h2 a2k 2 − b2h2
(4m2 α − 4mβ=
)2 4(3 − 2m)2 (4mαβ − 2m2 α2 − 2β2 − 6)
2m2 (mα − β)2= (3 − 2m2 )(2mαβ − m2 α2 − β2 − 3) a2 x b2 y
in equation + =a2 + b2 and simplifying, we get
2m4 α2 + 2m2β2 − 4m3 αβ = −4m3 αβ + 2m4 α2 + x1 y1
6mαβ − 3m2 α2 − 3β2 + 9 + 2m2β2 + 6m2
locus as (x2 + y 2 )2 (a2 y 2 − b2 y 2 )= x2 y 2 (a2 + b2 )2
2 2 2
m (3α − 6) − 6mαβ + 3β + 9 = 0
3β2 + 9
= 2 ⇒ β2 + 3 = 2α2 − 4 Sol 15:
3α2 − 6
β2 = 2α2 − 7 (a sec, a tan)
G
Sol 13: Equation of any normal to the hyperbola is S(ae,0)

m(a2 + b2 )
y mx −
=
a2 − b2 m2

⇒ (a2 − b2 m2 ) (y − mx)=
2
m2 (a2 + b2 )2 a2 x b2 y
Normal: + = a2 + b2
x1 y1
If it passes through the point (x1, y1), then
⇒ ax. cosθ + by. cotθ = a2 + b2
(a2 − b2 m2 ) (y1 − mx1 )2 = m2 (a2 + b2 )2
Now for coordinates of G ⇒ put y = 0 in above equation
It is a 4 degree equation in m, so it gives 4 values of m.
corresponding to these 4 values, four normal can be (a2 + b2 )
⇒ x= .sec θ
drawn from the point (x1, y1). a

b2 b2 a2 + b2
Sol 14: also e2 = 1 + ⇒ e= 1+ =
a2 a2 a
(h,k)
(a sec, b tan)
Now
R
2
 (a2 + b2 )sec θ 
O SG = 
2
− a2 + b2 
 a 
M a them a ti cs | 12.61

and SP2 = ( a2 + b2 − asec θ)2 + (b tan θ)2


1 0 0
SP2 = a2 + b2 + a2sec2θ – 2a. a2 + b2 secθ + b2tan2θ 1 a a
= 1 −
2 sec θ + tan θ sec θ + tan θ
(a2 + b2 )
⇒ e2SP2 = [(a2 + b2) + a2 sec2θ – a a
a2 1
sec θ − tan θ sec θ + tan θ
2a a2 + b2 secθ + b2tan2q]
1
= = (2a2 ) a2
 (a + b )
2 2 2 2
=  + (a2 + b2 )sec2 θ –
2
 a
Sol 17: Let P(x1, y1) be the middle point of the chord of
2 2
2 a + b sec θ b (a + b ) 2 2 2
the hyperbola 3x2 – 2y2 + 4x – 6y = 0
+ tan2θ 
a a2  ∴ Equation of the chord is T = s1

 ⇒ 3xx1 – 2yy1 + 2(x + x1) – 3(y + y1)
b2 
= (a2 + b2 ) + (a2 + b2 ) ×  sec2 θ
 a2  ⇒ 3x12 − 2y12 + 4x1 − 6y1
⇒ (3x1 + 2) x − (2 y1 + 3) y + 2 x1 − 3y1
(a2 + b2 )2 b2 (a2 + b2 ) −2 a2 + b2 sec θ 
+ – 
a2 a2 a  ⇒ 3x12 − 2y12 + 4x1 − 6y1

If this chord is parallel to line y = 2x, then
 (a2 + b2 )sec2 θ 2 a2 + b2 sec θ  3x + 2
=  + (a2 + b2 ) −  m1 = m2 ⇒ − =2
 a2 a  − (2 y, 3)
 
⇒ 3x1 – 4y1 = 4
2
 (a + b )sec θ
2 
2
e2SP2 =  − a2 + b2  Hence, the locus of the middle point (x1, y1) is 3x – 4y = 4
 a 

e2SP2 = SG2 ⇒ eSP = SG x2 y2


Sol 18: Eq. of Hyperbola = − = 1
100 49
Sol 16: Equation of any tangent to x2 – y2 = a2 or mx ± a2m2 − 49
y=
Eqn. of tangent =  …(i)
2 2
x −y
= 1 is
a2 a2 =⇒ y mx I 100m2 − 49

x y mx + 6 
Given that = ….(ii)
tan θ = ⊥ or x sec θ − y tan θ = a  … (i)
a Equating (i) and (ii)
Equation of other two sides of the triangle are
⇒ 100m2 − 49 =
6

x – y = 0  ...(ii) ⇒ 100m2 49 =
36
17
⇒ 100m2 = 85 =
x + y = 0  … (iii) 100 20

⇒ m =17
Solving (ii) and (iii), (iii) and (i), (i) and (ii) in pairs, 20
the co-ordinates of the vertices of the triangle
 a a 
are (0, 0);  ,  and Sol 19:
 sec θ + tan θ sec θ + tan θ 
 a a 
 ,  3x/4
 sec θ − tan θ sec θ − tan θ 
S’ 2x P 3x S
∴ Area of triangle =
1 2 . 6 2 | Hyperbola

Now, a = 4, b = 3 θ− 0
= a cos  … (i)
9 25 5 2
⇒ e2 = 1 + = ⇒ e=
16 16 4
And normal to the hyperbola at P(a sec θ, a tan θ) is
so coordinates of S = (5, 0) and given by
S′ = (–5, 0) x y
+ 2a 
= … (ii)
sec θ tan θ
 3 × (–5) + 2 × 5 0 × 3 + 2 × 0 
so P =  ,  = (–1, 0)
 3+2 3+2  Note that equation (i) and (ii) are the same lines
comparing these lines, we get
Now slope of line through P ⇒ –1
⇒ y = –x + C θ− 0 θ− 0 θ− 0
cos − sin a cos
= 2 = 2 2
⇒ 0 = 1 + c ⇒ c = –1
1 1 1
so line through P = y = – x – 1 sec θ tan θ 2a
x y Solving above and simplifying, we get
Now asymptotes ⇒  −  = 0
4 3 0 tan θ (4 sec2 θ − 1)
tan =
x y
and  +  = 0
4 3 x2
Sol 22: − y2 = 1
9
Point of intersection ⇒
now, line: y = mx + 9m2 − 1
x (x + 1) x (x + 1)
+ =0 − =0
4 3 4 3 ⇒ 2 = 3m + 9m2 − 1
7x + 4 = 0x = –4
−4 ⇒ (2 – 3m)2 = 9m2 − 1
x= y=3
7
⇒ 9m2 – 6m × 2 + 4 = 9m2 – 1
−3
and y =
7 (one m = ∞)
A
 −4 −3 
 ,  and (–4, 3) S = 12 m
 7 7 
5
⇒ m= (-3,0) (3,0)
12
x2 y 2 C
Sol 20: Eq. of Hyperbola; x2 − 2y 2 =
18 ⇒ − 1
=
18 9 so one tangent ⇒ x = 3 B
5 3
and one is y = x+
mx ± m2a2 − b2
Eq. of tangent y = 12 4

⇒ y = mx ± m2 .18 − 9 12y = 5x + 9
Now tangent at B
∴ this is perpendicular to y = x
x.x1
⇒ the value of m =-1 – y. y1 = 1
9
⇒ y = -1 x ± 18 − 9 ⇒ same –5x + 12y = 9
−5 12 9 −4
y =−x ± 9 ⇒ = = ⇒ y1 =
x1 / 9 − y1 1 3
y =−x ± 3
x1 = –5

Sol 21: The chord joining the points P(a sec θ, a tan θ) 1
so ∆ = × AC × height
θ− 0 θ− 0 2
and given by x cos − y sin
2 2 1
= × 2 × [(3 – (–5)] = 8 sq. unit
2
M a them a ti cs | 12.63

Sol 23:
h2 k 2
R (a sec, b tan) −
Now d = 4 9 =2
2
h k2
+
S’ (-a,0) (a,0) S a2+b2 16 81
22 P
)- a+b ,0)
Sol 2: (A) Equation of tangent,
x.sec θ y tan θ
We have − =1
a b
(S ⇒ R – SR)2 =S ⇒ R2 + SR2 – 2S ⇒ R. RS = (2a)2 h.sec θ
so for h ⇒ =1
⇒ (S⇒ R + SR)2 = (S⇒ R – SR)2 + 4S⇒ R. SR a
⇒ h = a cos θ O A
= 4a2 + 4. S⇒ R × SR  …(i) (h,0)
and h′ = –b cotθ
Now, tangent B P
2 2
x.sec θ y. tan θ  a   −b  (k,0)
 =1
⇒   − (h,k)
⇒ − =1
a b h  k 

1 a2 b2
⇒ − 1
=
sec2 θ tan2 θ h2 k2
+
a2 b2
h−0 k −0 −( −2c t)
a2b2 Sol 3: (D) = =
⇒ P = 2
 …(ii) 1 t 2
1 + t2
b2 sec2 θ + a2 tan2 θ
2ct 2ct3
2 2 2 2 2 h= ,k=
SR =2
(asec θ − a + b ) + b tan θ 1 + t4 1 + t4

SR2 = a2sec2θ + a2 + b2 + b2tan2θ – 2asecθ. a2 + b2 k


= t2
h
3/2
= (a2 + b2)sec2θ + a2 – 2asecθ. a2 + b2 k 
2c  
k= h
SR = ( a2 + b2 . secθ – a) k2
1+ 2
h
similarly S⇒ R = ( a2 + b2 . secθ – a) k 
3
4c2  
k2 = h
SR. S⇒ R = (a2 + b2)sec2θ – a2 2
 k2 
= b2sec2θ + a2tan2θ  1 + 2 
 h 
a2b2
SR. S⇒ R = [(from (ii)] k 2 (h2 + k 2 )2 4c2k 3
P2 =
h4 h3
putting in (i)
 b2  (x2 + y2)2 = 4c2 xy
4a2b2
(S⇒ R + SR) = 4a + 2 2
= 4a  1 + 2 
2

P2  P 
Sol 4: (B) We have
Exercise 2 2s = t2 + 1and2t = 2/s
⇒ t = 1/s
Single Correct Choice Type
1
⇒ 2s = +1
hx ky h2 k 2 s2
Sol 1: (B) − = −
4 9 4 9 ⇒ 2s3 = 1 + s3
1 2 . 6 4 | Hyperbola

⇒ 2s3 – s – 1 = 0 y(t1 t2 ) x
− =1
⇒ (s – 1) (2s + s + 1) = 0
2 c(t1 + t2 ) c(t1 + t2 )

⇒ s=1 Now c(t1 + t2) = x1 + x2


2
y= = 2 c(t1 + t2 )
s and = y1 + y2
t1 .t2
x 2s
= = 2
⇒ (2, 2) y x
⇒ + =1
y1 + y 2 x2 + x1
Sol 5: (B)
(a sec, b tan)
Sol 7: (C) We have 2b = ae
T b e
O N ⇒ =
a 2
e2
So e2 = 1 +
4
4
We hae NP = a sec θ and tangent slope: ⇒ e2 =
3
dy b.sec2 θ b 2
= = ⇒ e=
dx a.sec θ. tan θ asinθ 3
x.sec θ y.b tan θ
so − =1 Sol 8: (C) (5)x – (–3)y = (5)2 – (–3)2
a b
5x + 3y = 16
so at y = 0

x = a cosθ Sol 9: (B) We have,


so OT = a cosθ
2∫ x.dx = 3∫ y.dy
so OT × ON = a cosθ. a secθ = a 2
3y 2
⇒ x2 = +c
c / t1 − c / t2 2
Sol 6: (A) We have, slope =
ct1 − ct2 3y 2
⇒ x2 – =c
2
(t2 − t1 ) −1 2/3 2 5
= = ⇒ e2 = 1 + =1+ =
t1 .t2 (t1 − t2 ) t1 .t2 1 3 3
−x
so y = +N Multiple Correct Choice Type
t1 .t2

−x x2 y 2
⇒ y= +N Sol 10: (A, C) − =1
t1 .t2 1 5

this satisfies, tangent ⇒ y = mx ± 1m2 − 5

c −c ⇒ (8 – 2m)2 = m2 – 5
= +N
t1 t2
⇒ 4m2 + 64 – 32m = m2 – 5
1 1 
⇒ N = c +  ⇒ 3m2 – 32m + 69 = 0
 t1 t2 
t + t  ⇒ 3m2 – 23m – 9m + 69 = 0
−x
Now, y = +c.  1 2
t1 .t2  t1 .t2  ⇒ m(3m – 23) – 3(3m – 23) = 0
M a them a ti cs | 12.65

23  t 
⇒ m = 3 or m = and y2 + 2 = 2  sin2 − 1  +2
3  2 
Now y = 3x + 2(A) y2 = 2cos t

23x (23)2 − 45 ⇒ x2 – y2 = 6
or3y = ±
3 3
b x
⇒ 3y = 23x ± 22 (B)t = 1 − 
y a

Sol 11: (B, C, D) 16(x2 – 2x) – 3(y – 4y) = 44 x b  x b x


so .   .  1 −  −  1 −  = 0
16(x – 1) – 3(y – 2) = 44 + 16 – 12
2 2 a y  a y a

(x − 1)2 (y − 2)2 bx.(a − x) y b(a − x)


⇒ − =1
3 16 − + =0
2
a y b ay
Conjugate = 2b = 2 × 4 = 8
Centre = (1, 2) x ba2y

b2 16 12 b2x. (a – x) – a2y2 + ab2(a – x) = 0


and e2 = 1 + =1+ ⇒ e=
a2 9 3
ab2x – b2x2 – a2y2 + a2b2 – ab2x = 0

x2 y 2 x2 y2
− ⇒ + =1
Sol 12: (B, D) =0 a2 b2
16 9
Now tangent
Sol 14: (A, D) We have equation of circle
2 2 2
1 ⇒ y = mx ± a m −b (x – x1)(x – x2) + (y0 – y1)(y – y2) = 0
Now,
y = mx ± 16m2 − 9
x1 = ct1&y1 = c/t1
tangent 2 ⇒ y = mx ± 3 m2 + 1
x2 = ct2&y2 = c/t2
so16m – 9 = 9(m + 1)
2 2

c / t2 − c / t1 −1
⇒ 7m2 = 18 so slope ⇒ =
ct2 − ct1 t1 .t2
2 Now, slope = 1
⇒ m = ±3
7
1
⇒ –t1.
2 18 t2
soy = 3 x ± 3 +1
7 7
−1
⇒ t1 =
2 16 t2
y= 3 x±
7 7
putting this above

Sol 13: (A, C, D) c c


(x – ct1)(x – ct2) + (y – )(y – )=0
t1 t2
2 2
 2x   2y  c c
(A)   −   = 4 (x – ct1)(x + ) + (y – )(y + ct1)=0
 a   b  t1 t1
x2 y2 1 
⇒ − =1 x2 – c2 + 2c  − t1  x + y2
2 2
a b  t1 
(C) x2 – y2 = 4  1
– c2 + cy. t1 − 
(D) x2 – 6 = 2 cos t  t1 
1 2 . 6 6 | Hyperbola

1 
(x2 + y2 – 2c2) + c[x – y]  − t1  = 0 Now AO + BO = 2a
 t1 
52 + 122 + 242 + 72 = 13 + 25 = 38
Now when x = y & x2 + y2 = 2c2
this is satisfied for So2ae = 192 + 52

x=c&y=c 38e = 386


x = –c & y = –c
⇒ e = (0)(if ellipse)

Sol 15: (A, B, C, D) x = 2 t and y = 2 /t BO – AO = 2a(hyperbola)

Now slope of normal = t2 ⇒ 242 + 72 − 52 + 122 = 2a


 c ⇒ 25 – 13 = 2a
so  y −  = t2(x – ct)
 t
⇒ 12 = 2a
ty – c = t3x – ct4
⇒ ct4 – t3x + ty – c = 0 So2ae = 386 ⇒ e = e = 386 / 12

passes through (3,4)


Sol 17: (A, B)
Now ct4 – 3t3 + 4t – c = 0
3
thus, Sti =
c
⇒ c. Sti = 3
⇒ Sxi = 3(A) Now,

Also πti = –1 6= 100m2 − 49


−4
St1. t2. t3 =
c ⇒ 36 + 49 = 100 m2
Σt1 .t2 .t3 4 1 85 17
⇒ = + × ⇒ ± =m ⇒ m=±
πti c ( −1) 100 20
1
⇒ c. Σ =4
ti Sol 18: (A, B, D)

⇒ Syi = 4(B) k < 8 and k > 12 hyperbola (A)


8 < k < 12 ellipse and
Now πti = –1
if k = 10 circle
⇒ π(cti) = –c4 = –( 2 )4 = –4
1 Sol 19: (A, B, C, D) y = mx + a2bm2 − b2
and =–1
πti
and y = mx + a2 − b2m2
c
⇒ π   = –c4 = –4(C) & (D)
 ti  so a2m2 − b2 = a2 − b2m2

⇒ a2m2 – b2 = a2 – b2m2
Sol 16: (A, D) B
(24,7) ⇒ a2(m2 – 1) = (m2 – 1)(–b2)
A ⇒ m = ±1
(5,12)
So, y = x ± a2 − b2 or y = –x ± a2 − b2

O
M a them a ti cs | 12.67

x2 y 2
Sol 20: (B, D) − =1 a2 x b2 y
18 9 Normal: + = a2 + b2
Now m = –1 x1 y1

So y = mx ± a2m2 − b2 x1 .(a2 + b2 )
⇒ g ⇒ x1 = x =
a2
⇒ y = –x ± 18( +1) − 9
sec θ.(a2 + b2 )
x= = 4 secθ
⇒ y=–x±3 a

⇒ y = –x ± 3 tan θ(a2 − b2 )
and G ⇒ y = ⇒ 4 tanθ
b
⇒ x + y = 3 and x + y = –3
PC = 2 sec2 θ + tan2 θ
Sol 21: (C, D) 9(x2 + 2y) – 16(y2 – 2y) = 151
sec2 θ.(a2 + b2 )2 tan2 θ(a2 + b2 )
Og = +
9(x + 1) – 16(y – 1) = 151 + 9 – 16
2 2
a2 b2

(x + 1)2 (y − 1)2 sec2 θ tan2 θ 8


− =1 = (a + b )
2 2 + = sec2 θ + tan2 θ
16 9 a2 b2 2
b2 9 25 5
Now e2 = 1 + =1+ = ⇒ e=
a 2 16 16 4 PG = (2sec θ)2 + (2 tan θ)2 =2. sec2 θ + tan2 θ
So distance from centre
5 Pg = (4 sec θ − 2sec θ)2 + (2 tan θ)2
ae = 4 × =5
4
⇒ (–1 + 5, 1) and (–1–5, 1) = 2 sec2 θ + tan2 θ

(4, 1) and (–6, 1)


Previous Years’ Questions
Sol 22: (B, C) Equation of chord connecting the points
(a secθ, b secθ) and (a tanφ, b tanφ) is Sol 1: (A, B, C, D) It is given that

x θ−φ y θ+φ θ+φ x2 + y2 = +a2 … (i)


cos   − sin  =cos   ... (i)
a  2  b  2   2  and xy = c2 … (ii)
If it passes through (ae, 0); we, have
We obtain x2 = c4/x2 = a2
θ−φ θ+φ
e cos   = cos   ⇒ x4 – a2x2 + c4 = 0 … (iii)
 2   2 

θ+φ Now, x1, x2, x3, x4 will be roots of Eq. (iii)


cos  θ φ

 2  1 − tan 2 . tan 2 Therefore, Sx2 = x1+x2+2x3+x4 = 0
⇒ e
= =
θ−φ θ φ
cos   1 + tan 2 . tan 2 and product of the roots x1x2x3x4=c4
 2 
θ φ 1−e Similarly, y1 + y2 + y3 + y4 = 0 and y3y2y1y4 = c4
⇒ tan . tan =
2 2 1+e Hence, all options are correct.
1+e
Similarly if (i) passes through (-ae, 0), tan.tan =
1−e Sol 2: (A, B) Given, 2x2 – 2y2 = 1
Sol 23: (A, B, C)
x2 y2
G ⇒ − = 1 … (i)
P (2 sec,2 tan) 1 1
   
2 2
(-2,0) C (2,0) g
Eccentricity of hyperbola = 2 So eccentricity of ellipse
1 2 . 6 8 | Hyperbola

= 1/ 2 ⇒ b2 = 1
2 2
x y
Let equation of ellipse be + = 1 (a > b) Equation of ellipse is x2 + 2y2 = 2
2
a b2
Coordinate of foci
1 b2
∴ = 1−  
2 a2 1
(±ae, 0) =  ± 2. ,0  = (± 1, 0)
 2 
b2 1 Hence, option (A) and (B) are correct.
⇒ = ⇒ a2 = 2b2
a 2 2
If major axis is along y-axis, then
∴x2 + 2y2 = 2b2 … (ii) 1 a2
= 1−
Let ellipse and hyperbola intersect as 2 b2
 1 1 
A sec θ, tan θ  ⇒ b2 = 2a2
 2 2 
∴2x2 + y2 = 2a2
On differentiating Eq. (i),
2x
⇒ y′ = −
dy dy x y
4x – 4y =0⇒ =
dx dx y −2
⇒ y′ 1 1 
=
2 2
a x b y  sec θ , tan θ  sin θ
∴ At (6, 3) ⇒ + = (a2+b2)  2 2 
6 3
As ellipse and hyperbola are orthogonal
It passes through (9, 0)
−2
a2 .9 ∴ .cosecθ = –1
⇒ = a2 + b2 sin θ
6 π
⇒ cosec2θ = 1 ⇒ θ = ±
3a 2
a 2 4
⇒ − a2 = b2 ⇒ =2
2 b2 ∴ 2x2 + y2 = 2a2
b2 1 3 1
∴e2 = 1 + =1+ ⇒ ⇒ 2+ = 2a2
a 2 2 2 2
5
dy sec θ ⇒ a2 =
= = cosecq 4
dx at A tan θ
5
⇒ 2x2 + y2 = , corresponding foci are (0, ± 1)
and differentiating Eq. (ii) 2
dy
2x + 4y =0 x2 y 2
dx Sol 3: (B, D) Here, equation of ellipse + =1
4 1
dy x 1 b2 1 3
= − = − cosecq ⇒ e2 = 1 – =1– =
dx at A 2y 2 a 2 4 4
3
Since, ellipse and hyperbola are orthogonal ∴e = and focus (± ae, 0)
2
1
∴ − cosec2θ = –1
2 ⇒ (± 3 , 0)

⇒ cosec2θ = 2 ⇒ θ = ± π x2 y2
For hyperbola − =1
4 a2 b2
 1   1  b2
∴ A  1,  or  1, −  e12 = 1 +
 2  2 a2
∴ From Eq. (i), 1 4
where, e12 = =
e 2 3
2
 1  b 4 2
1 + 2  = 2b
2
⇒ 1+ =
 2 a2 3
M a them a ti cs | 12.69

t
b2 1 e
2t1
−e
−2t1
1  e2t e−2t 1
∴ =  … (i) = −  − − 2t 
a2 3 4 2  2 2  0
and hyperbola passes through (± 3 ,0)
2t −2t1
e 1 −e 1 2t1 −2t
3 = − (e − e 1 − 4t1 ) =t1
⇒ =1 4 4
a2
⇒ a2 = 3 … (ii)
Sol 5: Let any point on the hyperbola is (3secθ, 2tanθ)
From Eqs.(i) and (ii), we get
∴ Chord of contact of the circle x2+y2 = 9 with respect
b2 = 1 … (iii) to the point (3sectθ, 2tanθ) is,
2 2
x y (3secθ)x + (2tanθ)y = 9 … (i)
∴ Equation of hyperbola is − =1
3 1 Let (x1, y1) be the mid point of the chord of contact
Focus is ( ± ae1, 0)
⇒ Equation of chord in mid point form is
 2 
⇒  ± 3. ,0  ⇒ (± 2, 0) xx1 + yy1 = x12 + y12 … (ii)
 3 
Since, Eqs. (i) and (ii) are identically equal
∴ (B) and (D) are correct answers.
3sec θ 2 tan θ 9
∴ = =
 et1 + e− t1 et1 − e− t1  x1 y1 x1 + y12
2

Sol 4: Let P =  , 
 2 2  9x1
  ⇒ secθ =
3(x12 + y12 )
 e− t1 + et1 e− t1 − et 
and Q =  ,  9y1
 2 2  and tanθ =
 
2(x12 + y12 )
We have to find the area of the region bounded by the
curve x2 – y2 = 1 & the lines joining the centre x = 0, Thus, eliminating ‘θ’ from above equation, we get

y = 0 to the points (t1) and (–t1) 81x12 81y12


− =1
y 9(x12 + y12 )2 4(x12 + y12 )2

P(f1)
( sec2θ – tan2θ = 1)

x2 y 2 (x2 + y 2 )2
x’
A
x ∴Required locus is − =
-1 C 1 N 9 4 81

Q(-f1) Sol 6: (B) Equation of tangents to hyperbola having


y’ slope m is
Required area y = mx + 9m2 − 4  … (i)
 t −t
e 1 +e 1  Equation of tangent to circle is
 
 ∫1
= 2 area of ∆PCN = 2 ydy
 y = m(x – 4) + 16m2 + 16  … (ii)
 
2
 1  et1 + e− t1 Eqs. (i) and (ii) will be identical for m = satisfy.
  et1 − e− t1  t dx  5
= 2    − ∫ 1 y .dt 
 2  2 

2  1 dy 
  ∴ Equation of common tangent is 2x – 5 y + 4 = 0.
 2t1 −2t 2 
t e −e 
t −t
e −e 1
= 2 −∫ 1  .dt 
 8 0    x2 y 2
  2   Sol 7: (A) The equation of the hyperbola is − =1
9 4
2t1 −2t1
e −e 1 t1 2t and that of circle is x2 + y2 – 8x = 0
(e + e−2t − 2)dt
2 ∫0
= −
4
1 2 . 7 0 | Hyperbola

x2 x2 − 8x
For their points of intersection + =1 Sol 10: A→p; B→s, t; C→ r; D→ q, s
9 4
⇒ 4x2 + 9x2 – 72x = 36 1  h2 
(p) = 4  1 + 
⇒ 13x2 – 72x – 36 = 0 k2  k 2 

⇒ 13x2 – 78x + 6x – 36 = 0
⇒ 13x(x – 6) = 6(x – 6) = 0
(
1 4 k 2 + h2
⇒= )
2
13 1
⇒ x = 6, x = − ∴ h2 + k 2 =
  which is a circle.
6 2
13
x= − not acceptable (q) If z − z1 − z − z 2 =where
k k < z1 − z 2 the locus is
6
a hyperbola.
Now, for x = 6, y = ± 2 3
(r) Let t = tan α
Required equation is (x – 6)2 + (y + 2 3 )(y – 2 3 )=0
⇒ x2 – 12x + y2 + 24 = 0 ⇒x
= 3 cos 2 α and sin2 α =y
x
⇒ x2 + y2 – 12x + 24 = 0 or cos 2 α = and sin 2 α =y
3
a  x2
Sol 8: On substituting  ,0  in y = –2x + 1, ∴ y 2 sin2 2α + cos2=
+= 2α 1 which is an ellipse.
e  3
(s) If eccentricity is 1, ∞ ) , then the conic can be a
2a
we get 0 = − +1
e parabola (if e = 1) and a hyperbola if e∈ (1, ∞ ) .
a 1 (t) Let z =
x + iy; x, y ∈ R
⇒ =
e 2
⇒ ( x + 1 ) − y 2 = x2 + y 2 + 1
2

Also, y = –2x + 1 is tangent to hyperbola


⇒ y2 =
x ; which is a parabola.
∴1 = 4a2 – b2
y
1 Sol 11: y =−2x + 1
⇒ = 4 – (e2 – 1) F
5 x 2a

4
= 5 – e2
F1
) ) 1
2 ,0
0=
− +1 
e
e2 a 1
⇒ e4 – 5e2 + 4 = 0 ⇒ =
e 2
⇒ (e2 – 4)(e2 – 1) = 0 e = 2a
x = a/e
⇒ e = 2, e = 1 c2 a2m2 − b2
=

e = 1 gives the conic as parabola. But conic is given as ⇒ 1= 4a2 − b2


hyperbola, hence e = 2.
⇒ 1 + b2 − 4a2 =0

(x − 2 ) − (y + 2 )
2 2

Sol 9: (B) Hyperbola is 1


= e2= 1 +
b2
= 1+
( 4a − 1)2

4 2 a2 a2
=a 2,=
b 2
1
3 e2 = 1 + 4 −
e= a2
2

Area =
1
(
a= e − 1) ×
(
b2 1 3 − 2 × 2
=
) ( 3− 2 ) e2= 5 −
e2
1

2 a 2 2 2
 3  ⇒ e4 − 5e2 + 4 =
0
⇒ Area =  − 1 .
 2


 ( )(
⇒ e2 − 1 e2 − 4 =
0 )
M a them a ti cs | 12.71

i.e., 2x − y =±4 2
e2 − 1 ≠ 0 e =2
x y x y
⇒ − 1 and −
= − 1
=
Sol 12: (B) Equation of normal is 2 2 4 2 2 2 4 2
2
a2 xx1 yy1
( y − 3) = −a2 ( x − 6 ) ⇒ 2
= 1⇒e =
3
2
. Comparing it with
9

4
1
=
2b 2b
 9 1 
2 2 We get point of contact as  , 
x y 2 2 2 
Sol 13: (B, D) Ellipse is − 1
= 
3 2  9 1 
and  − ,− .
 2 2 2 
2
1= (
22 1 − e2 ⇒=
e ) 2
3 

∴ Eccentricity of the hyperbola is


2 2 4 
⇒ b= a2  − 1  ⇒ 3b=
2
a2
3  3 
Foci of the ellipse are ( )
3,0 and − 3,0 . ( )
Hyperbola passes through ( 3,0 )
3
=1 ⇒ a2 =3 and b2 = 1
a2
∴ Equation of hyperbola is x2 − 3y 2 =
3

Focus of hyperbola is (ae, 0)


 2 
( ae ,0 ) ≡  3× , 0  ≡ ( 2, 0 )
 3 

Sol 14: (A, B) Slope of tangent = 2

The tangents are y = 2x ± 9 × 4 − 4


2017-18 100 &
op kers
Class 11 T
By E ran culty
-JE Fa r
IIT enior emie .
S fP r es
o titut
Ins

MATHEMATICS
FOR JEE MAIN & ADVANCED
SECOND
EDITION

Exhaustive Theory
(Now Revised)

Formula Sheet
9000+ Problems
based on latest JEE pattern

2500 + 1000 (New) Problems


of previous 35 years of
AIEEE (JEE Main) and IIT-JEE (JEE Adv)

5000+Illustrations and Solved Examples


Detailed Solutions
of all problems available

Topic Covered Plancess Concepts


Tips & Tricks, Facts, Notes, Misconceptions,
Relations and Functions Key Take Aways, Problem Solving Tactics

PlancEssential
Questions recommended for revision
13. R E L AT I O N S A N D
FUNCTIONS

1. INTRODUCTION TO SETS
A collection of well-defined objects that are distinct are known as sets.
Well-defined object clearly defines if the object belongs to a given collection or not.

Well-defined collections
(a) Odd natural numbers less than 10, i.e., 1, 3, 5, 7, 9.
(b) Rivers of India.
(c) Vowels in the English alphabet a, e, i, o, u.

Not well-defined collections


(a) Collection of bright students in class XI of a Nucleus Academy.
(b) Collection of renowned mathematicians of the world.
(c) Collection of beautiful girls of the world.
(d) Collection of fat people.

PLANCESS CONCEPTS

•• The terms objects, elements and member of a set are synonymous.


•• Sets are generally denoted by capital letters A, B, C, …, X, Y, Z.
•• The elements of a set are represented by small letters a, b, c, …, x, y, z.
•• If a is an element of a set A, then we can say that a belongs to A. The Greek symbol∈ represents ‘belongs
to’. Thus we write a ∈ A.
If b is not an element of a set A, then we write b ∉ A.
Uday Kiran G (JEE 2012, AIR 102)
1 3 . 2 | Relations and Functions

2. SOME IMPORTANT SYMBOLS AND THEIR MEANINGS

Symbol Meaning
⇒ Implies
∈ Belongs to
A ⊂ B A is subset of B
⇔ Implies and is implied by
∉ Does not belong to
| or : or S.t. Such that
∀ For all
∃ There exists
iff If and only if
& And

Symbol Meaning
a/b a is divisor of b
N Set of natural numbers
W Set of whole numbers
I or Z Set of integers
Q Set of rational numbers
Qc Set of irrational numbers
R Set of real numbers
C Set of complex numbers

3. REPRESENTATION OF SETS
The following two methods are used to represent sets:
(a) Roster or Tabular form
(b) Set builder form

3.1 Roster or Tabular Form


In the roster form, all the elements of a set are enclosed within braces { } and each element is separated by a
comma.
Few examples are listed as follows:
(a) The set of all even positive number less than 7 is {2, 4, 6}.
(b) The set of vowels in the English alphabets is {a, e, i, o, u}.
(c) The set of odd natural numbers is represented as {1, 3, 5, …}. The three dots (ellipses) denote that the list is
endless.
Note: (i) In the roster form, the elements of a set are not repeated, i.e. all the elements are taken as distinct, e.g.
“SCHOOL” ⇒ {S, C, H, O, L}
M a them a ti cs | 13.3

(ii) The order in which the element of a set is written is immaterial.


E.g. The set {1, 2, 3} and {2, 1, 3} are same.

3.2 Set Builder Form


In the set builder form, all the elements of a set possess a common property. This common property does not
match with any element outside this set.
E.g. (i) all the elements ‘a, e, i, o, u’ possess a common property, i.e. each alphabet is a vowel which none other
letters possessing this property.
This can be represented in set builder form as follows:

V = {x : x is a vowel in English alphabet}

Property possessed by element of set


Colon  such that
Symbol which denotes elements of
set any other also can use.
Braces  The set of all
Figure 13.1

i.e. The set of all x such that x is a vowel in English alphabet.


(ii) A = {x: x is a natural number and 3 < x < 10}
The set of all x such that x is a natural and 3 < x < 10. Hence the numbers 4, 5, 6, 7, 8 and 9 are the elements of set A.

4. SUBSET
Set A is a subset of B if B has all the elements of A, denoted by A ⊂ B (read as A is subset of B).

4.1 Number of Subset


If a set X contains n elements {x1, x2, …, xn}, then total number of subsets of X = 2n.
Proof: Number of subsets of set X is equal to the number of selections of elements taking any number of them at
a time out of the total n elements and it is equal to 2n.
∵ nC0 + nC1 + nC2+..... + nCn = 2n.

4.2 Types of Subset


Set A is said to be a proper subset of set B if all elements of subset A is present in set B and at least one element
in set B is not an element of subset A, i.e. A ⊂ B and A ≠ B.
The set A itself is an improper subset of A.
E.g., If X = {x1, x2, …, xn}, then total number of proper sets = 2n – 1 (excluding itself). The statement A ⊂ B can be
written as B ⊃ A, then B is called the superset of A.

5. TYPE OF SETS
5.1 Null or Empty Set
Any set is called empty or null set if no elements are present in that set. It is denoted by { }.
1 3 . 4 | Relations and Functions

Few examples are as follows:


(a) Set of odd numbers divisible by 2 is null set, as we know that odd numbers are not divisible by 2 and hence
the resultant set is a null set.
(b) Set of even prime numbers is not a null set because 2 is a prime number divisible by 2.
(c) {y: y is a point common to any two parallel lines} is null set because two parallel lines do not intersect.

5.2 Finite and Infinite Set


Any set is said to be finite set, if finite number of elements are present in it.
Few examples are listed as follows:
(a) { } [null set is a finite set]
(b) {a, e, i, o, u}
(c) {Jan, Feb,… Dec} etc.
Any set is said to be infinite set if the number of elements are not finite.
Few examples are as follows:
(a) S = {men living presently in different parts of the world} is non-countable. Therefore it is an infinite set.
(b) S = {x: x ∈ R} is infinite set.
(c) S = {x: 2 < x > 3, x ∈ R} is infinite set.

5.3 Equal and Equivalent Sets


Any two sets A and B are said to equal set if all elements of set A are in B and vice versa.
E.g. {a, e, i, o, u} = A and B = {e, i, a, u, o} then A = B.
Any two sets A and B are said to be equivalent sets if their number of elements in both the sets are same.
E.g. A = {1, 2, 3} and B = {a, b, c} are equivalent set, i.e. both the sets have three elements (A ≈ B).

PLANCESS CONCEPTS

1. Number of elements in any set is said to be cardinality/cardinal number of that set.


E.g. 1. A = {1, 2, 3, 4}, then cardinality of set A is 4.
2. B = {a, e, i, o, u}, then cardinality of set B is 5.
2. Any set does not change if one or more elements of the set are repeated.
E.g. 1. A = {June, Nov., April, Sept}
2. B = {June, Nov., June, Sept., April, Sept.} are equal.
Shivam Agarwal (JEE 2009, AIR 27)

5.4 Singleton Set


A set with single element is called a singleton set, i.e. n(X) = 1,
E.g. {x: x ∈ N, 1 < x < 3}, { { } }, i.e. set of null set, {π} is a set containing alphabet φ.
M a them a ti cs | 13.5

5.5 Universal Set


It is a set which includes all the sets under consideration, i.e. it is a super set of each of the given set. Thus, a set
that contains all sets in a given context is called the universal set. It is denoted by U.
E.g. If A = {1, 2, 3}, B = {2, 4, 5, 6} and C = {1, 3, 5, 7}, then U = {1, 2, 3, 4, 5, 6 7} is a universal set which contains all
elements of sets A, B and C.

5.6 Disjoint Set


Two sets are said to be disjoint if they have no elements in common, i.e. if A and B are two sets, then A ∩ B = φ.
If A ∩ B ≠ φ, then A and B are said to be intersecting or overlapping sets.
E.g. (i) If A = {1, 2, 3}, B= {4, 5, 6} and C = {4, 7, 9}, then A and B are disjoint sets, and B and C are intersecting sets.
(ii) Set of even natural numbers and odd natural numbers are disjoint sets.

5.7 Complementary Set


Complementary set of A is a set containing all those elements of universal except the element in set A. It is denoted
by A , AC or A’. So AC = {x: x ∈ U and x ∉ A}, e.g. If set A = {1, 2, 3, 4, 5} and universal set U = {1, 2, 3, 4, …, 50},
then A = {6, 7, …, 50}
Note: All disjoint sets are not complementary sets and vice versa.

5.8 Power Set


The collection of all subsets of set A is called the power set of A and is denoted by P(A) i.e., P(A) = {x: x is a subset
n
of A}. If X = {x1 x2 x3 ….. xn} then n(P(x)) = 2n; n(P(P(x))) = 22 .

Illustration 1: Which of the two sets are equal?


(i) A = {4, 8, 12, 18}, B = {8, 4, 12, 16}
(ii) A = {2, 4, 6, 8, 10} B = {x : x is positive even integer and x ≤ 10}
(iii) A = {x: x is a multiple of 10} B = {x: x is a multiple of 5 and x ≥ 10} (JEE MAIN)

Sol: Refer to the definition of different types of sets in the above section.
(i) The elements 4, 8, 12 belong to both sets A and B. But 16 ∈ B and 18 do not belong to both A and B. So A ≠ B .
(ii) All elements present in set A is present in set B, and vice versa. Therefore A = B.
(iii) Given
A = {10, 20, 30, 40, …} and B = {10, 15, 20, 25, 30, …}
Since 15, 25, 35 are not multiples of 10, B does not belong to A.
∴ A ≠B.
Illustration 2: From the following sets, select equal sets:
A = {2, 4, 8, 12}, B = {1, 2, 3, 4}, C = {4, 8, 12, 14}, D = {3, 1, 4, 2}, E = {–1, 1}
F = {0, a}, G = {1, –1}, H = {0, 1}  (JEE MAIN)

Sol: Similar to the previous question.


Since numbers 2, 4, 8, 12 belongs to set A, but 8, 12 ∉ B, 2 ∉ C, 8, 12, ∉ D, 2, 4, 8, 12, ∉ E, F, G and H.
Therefore, A ≠ B, A ≠ C, A ≠ D, A ≠ E, A ≠ F, A ≠ G, A ≠ H
1 3 . 6 | Relations and Functions

Elements 1, 2, 3, 4 belong to set B but 1, 2, 3 ∉ C; 2, 3, 4 ∉ F, G and H. Only D has all elements of B.


Therefore, B ≠ C; B ≠ E; B ≠ F; B ≠ G; B ≠ H and B = D.
Elements 4, 8, 12, 14 ∉ C; elements 8, 12, 14 ∉ D, E, F, G and H.
Therefore, C ≠ D; C ≠ E ; C ≠ F ; C ≠ G ; C ≠ H.
Repeating this procedure completely, we find only one equal set E ≠ F, D ≠ E, D ≠ F, D ≠ G,
D ≠ H, E ≠ H G ≠ H, F ≠ G, F ≠ H.
Finally, we have two equal sets from given sets, i.e. B = D and E = G.

6. VENN DIAGRAMS
The diagrams drawn to represent sets and their relationships are called Venn diagrams
or Euler–Venn diagrams. Here we represents the universal U as set of all points within
U
rectangle and the subset A of the set U is represented by a circle inside the rectangle. B A
If a set A is a subset of a set B, then the circle representing A is drawn inside the circle
representing B. If A and B are not equal but they have some common elements, then we
represent A and B by two intersecting circles. Figure 13.2

E.g., If A is subset of B then it is represent diagrammatically in Fig. 13.2. Let U be the


universal set, A is a subset of set B. Then the Venn diagram is represented as follows:
If A is set, then the complement of A is represent as follows (Refer Fig. 13.3):

U
A
A

Figure 13.3

7. SET OPERATIONS
7.1 Union of Sets
If A and B are two sets, then union (∪) of A and B is the set of all elements belonging to set A and set B. It is also
defined as A ∪ B = {x : x ∈ A or x ∈ B}. It is represented by shaded area in following figures.
U U
A B A B

Figure 13.4 Figure 13.5

7.2 Intersection of Sets


If A and B are two sets, then intersection (∩) of A and B is the set of elements which belongs to both A and B in common, i.e.
A ∩ B = {x: x ∈ A and x ∈ B} represented with shaded area in Venn diagram (see Fig. 13.6)

U
A B

Figure 13.6
M a them a ti cs | 13.7

7.3 Difference of Two Sets


If A and B are two sets, then the difference of A and B is the set of elements which belongs to A and not B.
Thus, A – B = {x: x ∈ A and x ∉ B}
A – B ≠ B – A.
It is represented through the Venn diagrams in Fig. 13.7.

U
A B
A- B
Figure 13.7

7.4 Symmetric Difference of Two Sets


Set of those elements which are obtained by taking the union of the difference of A and B: (A-B) and the difference
of B and A: (B-A) is known as the symmetric difference of two sets A and B denoted by (A ∆ B).
Thus A ∆ B = (A – B) ∪ (B – A)
Venn diagram is represented in Fig. 13.8. U
A B

AB
Figure 13.8

8. NUMBER OF ELEMENTS IN DIFFERENT SETS


If A, B and C are finite sets and U be the finite universal set, then
(a) n(A ∪ B) = n(A) + n(B) – n (A ∩ B)
(b) n(A ∪ B) = n(A) + n(B) (if A and B are disjoint sets)
(c) n(A − B) = n(A) – n (A ∩ B)
(d) n(A ∆ B) = n[(A – B) ∪ (B – A)] = n(A) + n(B) – 2n (A ∩ B)
(e) n(A ∪ B ∪ C) = n(A) + n(B) + n(C) – n (A ∩ B) – n (B ∩ C) – n (A ∩ C) + n(A ∩ B ∩ C)
(f) n(A’ ∪ B’) = n (A ∩ B)’ = n(∪) – n (A ∩ B)
(g) n(A’ ∩ B’) = n (A ∪ B)’ = n(∪) – n (A ∪ B)

9. ALGEBRAIC OPERATIONS ON SETS


9.1 Idempotent Operation
For any set A, we have (i) A ∪ A = A and (ii) A ∩ A = A

9.2 Identity Operation


For any set A, we have
(a) A ∪ ϕ = A
(b) A ∩ A = A, i.e. φ and U are identity elements for union and intersection, respectively.
1 3 . 8 | Relations and Functions

9.3 Commutative Operation


For any set A and B, we have
(a) A ∪ B = B ∪ A and (ii) A ∩ B = B ∩ A
i.e. union and intersection are commutative.

9.4 Associative Operation


If A, B and C are any three sets, then
(a) (A ∪ B) ∪ C = A ∪ (B ∪ C)
(b) (A ∩ B) ∩ C = A ∩ (B ∩ C)
i.e. union and intersection are associative.

9.5 Distributive Operations


If A, B and C are any three sets, then
(a) A ∪ (B ∩ C) = (A ∪ B) ∩ (A ∪ C)
(b) A ∩ (B ∪ C) = (A ∩ B) ∪ (A ∩ C)
i.e. union and intersection are distributive over intersection and union, respectively.

10. DE MORGAN’S PRINCIPLE


If A and B are any two sets, then
(a) (A ∪ B)’ = A’ ∩ B’
(b) (A ∩ B)’ = A’ ∪ B’
Proof: (a) Let x be an arbitrary element of (A ∪ B)’. Then x ∈ (A ∪ B)’⇒x ∉ (A ∪ B)
⇒ x ∉ A and x ∉ B ⇒ x ∈ A’ ∩ B’
Again let y be an arbitrary element of A’ ∩ B’. Then y ∈ A’ ∩ B’
⇒ y ∈A’ and y∈ B’ ⇒ y ∉ A and y ∉ B
⇒ y ∉(A ∪ B) ⇒ y ∈(A ∪ B)’
\ A’ ∩ B’ ⊆ (A ∪ B)’
Hence (A ∪ B)’ = A’ ∩ B’
Similarly (b) can be proved.

RELATIONS

1. ORDERED PAIR
A pair of elements written in a particular order is called an ordered pair. Let A and B be two sets. If a ∈ A, b ∈ B, then
elements (a, b) denotes an ordered pair, with first component a and second component b. Here, the order in which
the elements a and b appear is important. The ordered pair (1, 2) and (2, 1) are different, because they represent
different points in the co-ordinate plane.
Equality of ordered pairs: Ordered pair (a1, b1) is equal to (a2, b2) iff a1 = a2 and b1 = b2.
M a them a ti cs | 13.9

2. CARTESIAN PRODUCT OF SETS

2.1 Cartesian Product of Two Sets


The Cartesian product of two sets A and B is the set of all those ordered pair whose first co-ordinate is an element
of set A and the second co-ordinate is an element of set B.
It is denoted by A × B and read as ‘A cross B’ or ‘product set of A and B’.
i.e. A × B = {(a, b): a ∈ A and b ∈ B}
E.g. Let A = {1, 2, 3}
B = {3, 5}
Then A × B = {(1, 3), (1, 5), (2, 3), (2, 5), (3, 3), (3, 5)} and
B × A = {(3, 1), (3, 2), (3, 3), (5, 1), (5, 2), (5, 3)}
Hence A × B ≠ B × A
Thus “Cartesian product of sets is not commutative”.
E.g. A = {1, 2}, B= {a, b}
A × B = {(1, a), (1, b), (2, a), (2, b)}

Properties of Cartesian product


(a) A × B ≠ B × A (non-commutative)
(b) n(A × B) = n(A) n(B) and n(P(A × B)) = 2n(A)n(B)
(c) A = ϕ and B = ϕ ⇔ A × B = ϕ
2
(d) If A and B are two non-empty sets with n elements in common, then (A × B) and (B × A) have n element in
common.
(e) A × (B ∪ C) = (A × B) ∪ (A × C)
(f) A × (B ∩ C) = (A × B) ∩ (A × C)
(g) A × (B − C) = (A × B) − (A × C)

Illustration 3: If A = {2, 4} and B = {3, 4, 5}, then find (A ∩ B) × (A ∪ B) (JEE MAIN)

Sol: Use Cartesian Product of two Sets.


A ∩ B = {4} and A ∪ B = {2, 3, 4, 5}
\ (A ∩ B) × (B ∪ A) = {(4, 2), (4, 3), (4, 4), (4, 5)}

2.2 Cartesian Product of More than Two Sets


The Cartesian product of n sets A1, A2, …, An is denoted by A1 × A2 × … × An and is defined as
A1 × A2 × … × An = { (x1 x2, …, xn): xi∈ A, where i = 1, 2, …, n}

Note:
(i) Elements of A × B are called 2-tuples.
(ii) Elements of A × B × C are called 3-tuples.
(iii) Elements of A1 × A2 × … × An are also called n-tuples.
1 3 . 1 0 | Relations and Functions

E.g. P = {1, 2}, from the set P × P × P.


P × P × P = {(1, 1, 1), (1, 1, 2), (1, 2, 1), (1, 2, 2), (2, 1, 1), (2, 1, 2), (2, 2, 1), (2, 2, 2)}

2.3 Number of Elements in the Cartesian Product


If A and B are two finite sets then
n(A × B) = n(A) · n(B)
Thus, if A and B have m elements and n elements, respectively, then A × B has mn elements.
Proof: Let A = {x1, x2, x3, …, xn}
and B = {y1, y2, y3, …, yn}
Then A × B = {(x1, y1), (x2, y2) … (x1, yn),
(x2, y1), (x2, y2) … (x2, yn),

(xm, y1), (xm, y2) … (xm, yn)}
Clearly each row has n ordered pairs and there are m such rows. Therefore A × B has mn elements.
Similarly, n(A × B × C) = n(A) · n(B) · n(C)

Illustration 4: If n(A) = 7, n(B) = 8 and n(A ∩ B) = 4, then match the following columns: (JEE MAIN)
(i) n(A ∪B) (a) 56
(ii) n(A × B) (b) 16
(iii) n((A × B) × A) (c) 392
(iv) n((A × B) ∩ (B × A)) (d) 96
(v) n((A × B) ∪ (B × A)) (e) 11

Sol: Use the formula studied in the above section.


(i) n (A ∪ B) = n (A) + n (B) – n (A ∩ B) = 7 + 8 – 4 = 11
(ii) n(A × B) = n(A) n(B) = 7 * 8 = 56 = n(B × A)
(iii) n((A × B) × A) = n(B × A) · n(A) = 56 × 7 = 392
(iv) n((A × B) ∩ (B × A)) = (n(A ∩ B))2 = 42 = 16
(v) n((A × B) ∪ (B × A)) = n(A × B) + n(B × A) – n(A × B) ∩ (B × A) = 56 + 56 – 16 = 96

2.4 Representation of Cartesian Product


Let A = {1, 2, 3} and B = {a, b}

1 a
2
3 b

Figure 13.9

Each element of set A to each element of set B is represented by lines from A to B in Fig. 13.9.
M a them a ti cs | 13.11

3. RELATION
A relation R from one set to another say from set X to set Y (R : X → Y) is a correspondence between set X to set Y
through which some or more elements of X are associated with some or more elements of Y. Therefore a relation
R, from a non-empty set X to another non-empty set Y, is a subset of X × Y, i.e. R: X → Y is a subset of A × B.
Every non-zero subset of A × B is defined as a relation from set A to set B. Therefore, if R is a relation from A → B
then R = {(a, b) | (a, b) ∈ A × B and a R b}.
If A and B are two non-empty sets and R: A → B be a relation such that R: {(a, b) | (a, b) ∈ R, and a ∈ A and b ∈ B},
then
(a) ‘b’ is an image of ‘a’ under R.
(b) ‘a’ is a pre-image of ‘b’ under R.
For example consider sets X and Y of all male and female members of a royal family of the kingdom Ayodhya. X =
{Dashrath, Ram, Bharat, Laxman, Shatrughan} and Y = {Koshaliya, Kaikai, Sumitra, Sita, Mandavi, Urmila, Shrutkirti}.
A relation RH is defined as “husband of” from set X to set Y.

RH

Dashrath Kaushlya
Sumitra
Ram Kakai
Bharat Urmila
Laxman Sita
Shatrughan Madhvi
Sharukirti
x y

Figure 13.10

RH = {(Dashrath, Koshaliya), (ram, sita), (Bharat, Mandavi), (Laxman, Urmila), (Shatrughan, Shrutkirti), (Dashrath,
Kakai), (Dashrath, Sumitra)}
(a) If a is related to b, it is symbolically written as a R b.
(b) It is not necessary for each and every element of set A to have an image in set B, and set B to have a pre-
image in set A.
(c) Elements of set A having image in B are not necessarily unique.
(d) Number of relations between A and B is the number of subsets of A × B.
Number of relations = no. of ways of selecting a non-zero subset of A × B.
= mnC1 + mnC2 + … +mnCmn = 2mn – 1.

3.1 Domain
Domain of a relation is a collection of elements of the first set participating in the correspondence, i.e. it is set of
all pre-images under the relation. For example, from the above example, domain of RH = {Dashrath, Ram, Bharat,
Laxman, Shatrughan}.

3.2 Codomain
All elements of any set constitute co-domain, irrespective of whether they are related with any element of
correspondence set or not, e.g. Y = {Koshaliya, Kakai, Sumitra, Sita, Mandavi, Urmila, Shrutkirti} is co-domain of RH.
1 3 . 1 2 | Relations and Functions

3.3 Range
Range of relation is a set of those elements of set Y participating in correspondence, i.e. set of all images. Range of
RH = {Koshaliya, Kakai, Sumitra, Sita, Mandavi, Urmila, Shrutkirti}.

Illustration 5: A = {1, 2, 3, 4, 5} and B = {2, 4, 5}. Relation between A and B is defined as a R b ⇒ a and b are
relatively prime or co-prime (i.e., HCF is 1). Find domain and range of R. (JEE MAIN)

Sol: Write the elements of the Relation and then write the domain and range.
R = {(1, 2), (1, 4), (1, 5), (2, 5), (3, 2), (3, 4), (3, 5), (4, 5), (5, 2), (5, 4)}
Domain of R {1, 2, 3, 4, 5}
Range of R {2, 4, 5}

Illustration 6: A = {Jaipur, Patna, Kanpur, Lucknow} and B = {Rajasthan, Uttar Pradesh, Bihar}
a R b ⇒ a is capital of b, a ∈ A and b ∈ B. Find R. (JEE MAIN)

Sol: Use the concept / definition studied above.


R = {(Jaipur, Rajasthan), (Patna, Bihar), (Lucknow, Uttar Pradesh)}

Illustration 7: If A = {1, 3, 5, 7}, B = {2, 4, 6, 8}


Relation is a R b ⇒ a > b, a ∈ A, b ∈ B. Find domain and range of R. (JEE MAIN)

Sol: Similar to the Illustration 3.


R = {(3, 2), (5, 2), (5, 4), (7, 2) (7, 4), (7, 6)}
Domain = {3, 5, 7}
Range = {2, 4, 6}

4. REPRESENTATION OF A RELATION
4.1 Roster Form
In this form we represent set of all ordered pairs (a, b) such that (a, b) ∈ R where a ∈ A, b ∈ B.

4.2 Set Builder Form


Relation is denoted by the rule which co-relates the two set. This is similar to set builder form in sets.

4.3 Arrow-Diagram (Mapping)


It is a pictorial notation of any relation.
E.g. Let A = {–2, –1, 4} and B = {1, 4, 9}
A relation from A to B, i.e. a R b, is defined as a < b.

1. Roster form
R = {(0-2, 1), (–2, 4), (–2, 9), (–1, 1), (–1, 4), (–1, 9), (4, 9)}
M a them a ti cs | 13.13

2. Set builder notation


R = {(a, b): a ∈A and b ∈B, a is less than b}

3. Arrow-diagram (mapping)

A B

1 a
2
3 b

Figure 13.11

5. TYPES OF RELATION
5.1 Reflexive Relation
R: X → Y is said to be reflexive iff x R x ∀ x ∈ X, i.e. every element in set X must be a related to itself. Therefore ∀
x ∈ X; (x, x) ∈ R, then relation R is called as reflexive relation.

5.2 Identity Relation


Consider a set X. Then the relation I = {(x, x): x ∈ X} on X is called the identity relation on X i.e.
a relation I on X is identity relation if every element of X related to itself only. For example, y = x.
Note: All identity relations are reflexive but all reflexive relations are not identity.

5.3 Symmetric Relation


R: X → Y is said to be symmetric iff (x, y) ∈ R ⇒ (y, x) ∈ R.
For example, perpendicularity of lines in a plane is symmetric relation.

5.4 Transitive Relation


R: X → Y is transitive iff (x, y) ∈ R and (y, z) ∈ R ⇒ (x, z) ∈ R,
i.e. x R y and y R z ⇒ x R z.
For example, the relation “being sister of” among the numbers of a family is always transitive.
Note:
(i) Every null relation is a symmetric and transitive relation.
(ii) Every singleton relation is a transitive relation.
(iii) Universal and identity relation are reflexive, symmetric as well as transitive.

5.5 Antisymmetric Relation


Let A be any set. A relation R on set A is said to be an antisymmetric relation iff (a, b) ∈ R and
(b, a) ∈ R ⇒ a = b for all a, b ∈ A. For example, relations “being subset of”; “is greater than or equal to” and
“identity relation on any set A” are anti-symmetric relations.
1 3 . 1 4 | Relations and Functions

5.6 Equivalence Relation


A relation R from a set X to set Y (R → X → Y) is said to be an equivalence relation iff it is reflexive, symmetric as well
as transitive. The equivalence relation is denoted by, e.g. relation “is equal to” equality similarity and congruence of
triangle, parallelism of lines are equivalence relation.

5.7 Inverse Relation


If relation R is defined from A to B then inverse relation would be defined from B to A, i.e.
R: A → B ⇒ a R b, where a ∈ A, b ∈ B.
R–1: B →A ⇒ b R a, where a ∈ A, b ∈ B.
Domain of R = Range of R–1
and range of R = Domain of R–1.
\ R–1 = {b, a} | (a, b) ∈ R}
For example, a relation R is defined on the set of 1st ten natural numbers.
\ N = {1, 2, 3, …., 10} and a, b ∈ N.

N N

Figure 13.12

a R b ⇒ a + 2b = 10
R = {(2, 4), (4, 3), (6, 2) (8, 1)}
R–1 = {(4, 2), (3, 4), (2, 6) (1, 8)}
For example, a relation defined on the set of lines.
 1. aR b ⇒ a || b
It is a symmetric relation because if line ‘a’ is || to ‘b’, the line ‘b’ is || to ‘a’.
where (a, b) ∈ L {L is a set of || lines}
 2. L1RL2 L1⊥ L2 It is a symmetric relation
L1 , L2 ∈ L {L is a set of lines}
 3. a R b ⇒ ‘a’ is brother of ‘b’ is not a symmetric relation as ‘b’ may be sister of ‘a’.
 4. a R b ⇒ ‘a’ is cousin of ‘b’. This is a symmetric relation. If R is symmetric.
 5. R = R–1.
 6. Range of R = Domain of R.

FUNCTIONS

1. INTRODUCTION
The concept of function is of fundamental importance in almost all branches of Mathematics. It plays a major role
to solve real world problems in mathematics. As a matter of fact, functions are some special type of relations.
M a them a ti cs | 13.15

General definition:
Definition 1: Consider two sets A and B and let there exist a rule or manner or correspondence ‘f‘ which associates
to each element of A to a unique element in B. Then f is called a function or mapping f
from A to B. It is denoted by symbol f and represented by f: A →B (read as ‘f’ is a A B
function from A to B or ‘f maps A to B’).
If element a ∈ A is associated with an element b ∈ B, then b is called the ‘f image of a’
or ‘image of a under f’ or ‘the value of function f at a’. Also a is called the pre-image of
b or argument of b under the function f. We write it as
Figure 13. 13
b = f(a) or f: a → b or f: (a, b)
Function as a set of ordered pairs:
A function f: A → B can be expressed as a set of ordered pairs in which each ordered pair is such that its first
element belongs to A and second element is the corresponding element of B.
As such a function f: A → B can be considered as a set of ordered pairs (a, f(a)), where a ∈ A and f(a) ∈ B, which is
the f image of a. Hence, f is a subset of A × B.
Definition 2: A relation R from a set A to a set B is called a function if f
A B
(i) each element of A is associated with some element of B.
(ii) each element of A has unique image in B.
a b
Thus a function ‘f’ from set A to set B is a subset of A × B in which element a belonging
to A appears in only one ordered pair belonging to f. Hence, a function f is a relation
from A to B satisfying the following properties: Figure 13. 14

(i) f ⊂A × B (ii) ∀ a ∈A ⇒(a, (f(a)) ∈ f and (iii) (a, b) ∈f and (a, c) ∈f ⇒b = c.

PLANCESS CONCEPTS

Every function is a relation, but every relation may not necessarily be a function.

f f f f
A B A B A B A B
x1 y1 x1 y1
X1 Y1 y2 x2 y2
X1 x2 y3 x3 y3
Y1 x3
X2 Y2 y4 x4 y4
x4 y5 x5 y5
It is a function Not a function It is a function Not a function

Figure 13.15

Easy way to differentiate function from relation is as follows :


1. Consider every element in set A is a guest and every element in set B is hosting a function at the same
time and invited every element from A.
2. None of the elements can be at two functions simultaneously.
3. So if an element is attending two functions at the same time, then it is just a relation and if an element
is attending only one function then it is said to be a function.

Chen Reddy Sundeep Reddy (JEE 2012, AIR 63)


1 3 . 1 6 | Relations and Functions

2. RELATION VS FUNCTION
L
y
y
y = f(x)
y2 C y = f(x)
y3
y1 y2 B
y1 A
x
O x1 x2 x3
(a) x0 x
O
(b)
Figure 13.16 Figure 13.17

These figures show the graph of two arbitrary curves. In figure 16, any line drawn parallel to y-axis would meet the
curve at only one point. That means each element of X would have only one image. Thus figure 16 (a) represents
the graph of a function.
In figure 17, certain line parallel to y-axis (e.g., line L) would meet the curve in more than one point (A, B and C).
Thus element x0 of X would have three distinct images. Thus, this curve does not represent a function.
Hence, if y = f(x) represents a function, then lines drawn parallel to y-axis through different point corresponding to
points of set X should meet the curve at one point.
x2 y2
Equation of an ellipse + 1 is a relation, which is a combination of two functions
=
a2 b2

x2 x2
=y b 1− and y = – b 1 − . Similarly, the equation of the parabola y2 = x is a combination of two functions
a 2
a2
as shown in Fig. 13.18.

y y
2
x
y=b 1- 2
a y= x

x’ x x’ x
O O
2
x y =- x
y = -b 1- 2
a

y’ y’
Figure 13.18

3. DOMAIN, CO-DOMAIN AND RANGE OF A FUNCTION


Let f: A → B, then the set A is known as the domain of f and the set B is known as co-domain of f.
The set of all f images of elements of A is known as the range of f. Thus:
Domain of f = {a |a ∈ A, (a, f(a)) ∈ f} Range of f = {f(a) ∀ a ∈ A and f(a) ∈ B}
It should be noted that range is a subset of co-domain. If only the rule of function is given then the domain of the
function is the set of those real numbers, where function is defined.
Note: If domain of f(x) is D1 and domain of g(x) is D2 then domain of f(x) + g(x) = D1∩ D2.
M a them a ti cs | 13.17

f B
A c
o
d d
o o
m m
a a
i i Elements of range
n n

Domain
Figure 13.19

4. COMMON FUNCTIONS
4.1 Polynomial Function
If a function f is defined by f(x) = a0xn + a1xn–1 + a2xn–2 + …. an–1x + an where n is a non-negative integer and a0, a1,
a2, …., an are real numbers and a0 ≠ 0, then f is called a polynomial function of degree n.
2

2
f(x) = x + 1 f(x) = x + 1

Figure 13.20 Figure 13.21

PLANCESS CONCEPTS

(a) A polynomial of degree one with no constant term is called an odd linear function i.e., f(x) = ax, a ≠ 0
(b) There are two polynomial functions, satisfying the relation ; f(x) f(1/x) = f(x) + f(1/x)
They are :
(i) f(x) = xn + 1 and (ii) f(x) = 1 –xn, where n is positive integer.
(c) Domain of a polynomial function is R
(d) Range for odd degree polynomial is R whereas for even degree polynomial range is a subset of R.
(i) If f(x) + f(y) = f(xy) then f(x) = k log x
(ii) If f(x) · f(y) = f(x + y) then f(x) = akx
(iii) If f(x) + f(y) = f(x+ y) then f(x) = kx
Rohit Kumar (JEE 2012, AIR 79)

4.2 Algebraic Function


y is an algebraic function of x, if it is a function that satisfies an algebraic equation of the form P0(x) = yn + P1(x) yn–1
+ …. + Pn–1(x) y + Pn(x) = 0 where n is a positive integer and P0(x),
1 3 . 1 8 | Relations and Functions

P1(x) ……. are polynomials in x e.g., x3 + y3 – 3xy = 0 or


y = | x | is an algebraic function, since it satisfies the equation y2 – x2 = 0.

PLANCESS CONCEPTS

All polynomial functions are Algebraic but not the converse. A function that is not algebraic is called
Transcendental function.
Anvit Tawar (JEE 2012, AIR 9)

4.3 Fractional/Rational Function


g(x)
It is a function of the form y = f(x) = , where g(x) and h(x) are polynomial and h(x) ≠ 0.
h(x)

4.4 Exponential/Logarithmic Function


A function f(x) = ax = ex ln a (a > 0, a ≠ 1, x ∈ R) is called an exponential function. The inverse of exponential function
is called the logarithmic function.
i.e. g(x) = logax.

PLANCESS CONCEPTS

(a) f(x) = ex domain is R and range is R+.


(b) f(x) = e1/x domain is R – {0} and range is R+ – {1}.
(c) f(x) and g(x) are inverse of each other and their graphs are as shown.

+ +
x
f(x)=a ,a  1
(0, 1) x
45o (0, 1) f(x)=a ,0 < a < 1
(1, 0) (1, 0)
x
x

y=

g(x)=logax, a < 1 < 1


y=

g(x)=logax, a  1

Figure 13.22

Shivam Agarwal (JEE 2009, AIR 27)

4.5 Absolute Value Function


A function y = f(x) = | x | is called the absolute value function or modulus function. It is defined as y = | x | =
 x, if x ≥ 0

−x, if x < 0
M a them a ti cs | 13.19

y=-x y=x
x
O

Figure 13.23

Note: (a) f(x) = | x |, domain is R and range is R+∪ {0}.


1
(b) f(x) = , domain is R – {0} and range is R+.
f|x|

4.6 Signum Function


 1, x > 0
 x
A function y = f(x) = sgn(x) is defined as follows: y = f(x) =  0, x = 0 If is also written as sgn x = | x |/ x or ;x
≠ 0; f(0) = 0 −1, x < 0 |x|

Note: Domain is x ∈ R and its range = {–1, 0, 1}
d
sgn x = | x | when x ≠ 0
dx
= 0 when x = 0
y
y=1 if x > 0

x
O y=sgn x

y=-1 if x > 0

Figure 13.24

4.7 Greatest Integer Function or Step Function


The function y = f(x) = [x] is called the greatest integer function, where [x] denotes the greatest integer less than
or equal to x.

Note:
graph of y =[x] y
(a) –1 ≤ x < 0⇒ [x] = –1 3
0≤ x <1⇒ [x] = 0 2
1 ≤ x <2⇒ [x] = 1 1
2≤ x <3⇒ [x] = 2 etc. x
-3 -2 -1 1 2 3
(b) f(x) = [x], domain is R and range is I. -1
1  1 
(c) f(x) = domain is R – [0, 1) and range is  n ∈ I − {0}  -2
[x]  n 
-3
Properties of greatest integer function:
(a) [x] ≤x < [x] + 1 and x – 1 < [x] ≤ x, 0 ≤ x – [x] < 1. Figure 13.25

(b) [x + m] = [x] + m if m is an integer.


(c) [x] + [y] ≤[x + y] ≤[x] + [y] + 1.
(d) [x] + [–x] = 0 if x is an integer= –1 otherwise.
1 3 . 2 0 | Relations and Functions

4.8 Fractional Part Function


It is defined as g(x) = {x} = x – [x].
For example, the fractional part of the number 2.1 is 2.1 – 2 = 0.1 and the fractional part of –3.7 is 0.3. The period
of this function is 1 and graph of this function is as shown.
y
Note: (a) f(x) = {x}, domain is R and range is [0, 1) graph of y =[x]
1
1
(b) f(x) = , domain is R – I, range is (1, ∞)
{x}
{x + n} = {x}, where n ∈ I -1 1 2 3

5. TRANSFORMATION OF CURVES Figure 13.26

(a) Given graph of a function y = f(x) and we have to draw the graph of y = f(x – a) [means replacing x by x–a, a
> 0] then shift the entire graph through a distance a units in positive direction of x-axis.
y

y=sin(x-2) -2+2 1 +2 2+2
2 3+2

x
-4+2 0 2


-3+2 -+2 -1 + 2 +2
2

Figure 13.27

(b) Given graph of a function y = f(x), draw a graph of y = f(x + a) [means replacing x by x + a, a > 0]. Shift the
entire graph through in negative direction of x-axis.
y 3
-2
y=sin(x+2) 1 2
-
-2
2 0 x
-2
2-2
-3
-2 --2 
+2 -1 -2
2 2

Figure 13.28

(c) Given graph of a function y = f(x), draw a graph of y = af(x) [means replacing y by y/a, a >0]. Then multiply all
the values by a on y-axis.
y
y=2sinx 2

-/2 3
0  x
-2 -  2
2
-2
Figure 13.29
M a them a ti cs | 13.21

(d) Given graph of a function y = f(x), draw a graph of y = f(ax) [means replacing x by ax, a >0]. Then divide all
the values by a on x-axis.
y
y=sin(2x)
- -3
4 2
 x
-3 0
2
- - -1  
4 2 4 2
Figure 13.30

(e) Given graph of a function y = f(x), draw the graph of y = f(x) + a [means replacing y by y – a, a >0]. Then shift
the entire graph in positive direction of y-axis.

y
y=(sin x)+2 3

2
-4+2 2
1
-3 -2 - 0  2 2
2
Figure 13.31

(f) Given graph of a function y = f(x), draw the graph of y = f(x) –a [means replacing y by y + a, a > 0]. Then shift
the entire graph in negative direction of y-axis.

y=(sinx)-2 y 

-4 -3 -2 - 2 0 2 - 2 3 4
x
1

-2

-3
Figure 13.32

(g) Given graph of a function y = f(x), draw the graph of y = f(–x) [means replacing x by –x]. Then take the
reflection of the entire curve in y-axis.

y
y=sin (-x) 1
- 
3 2 2 3
-3 -2 - - 
3
-1
Figure 13.33

(h) Given graph of a function y = f(x), draw the graph of y = –f(x) [means replacing y by –y]. Then take the
reflection of the entire curve in x-axis.
1 3 . 2 2 | Relations and Functions

y
y= -sin x 1
- 
3 2 2 3
-4 -3 -2 - - 
2

-1
Figure 13.34

(i) Given graph of a function y = f(x), draw the graph of y = f(|x|) [means replacing x by |x|]
(a) Remove the portion of the curve, on left-hand side of y-axis.
(b) Take the reflection of right-hand size on the left-hand side.

y
y=sin x 1

-2 3
-  x
-4 -3 0 2 4

-1

Figure 13.35

( j) Given graph of a function y = f(x), draw the graph of y = | f(x) |.The projection of the curve lying below x-axis
will go above the axis.

y
y = sin x 1

x
-3 -2 - 0  2 3

Figure 13.36

(k) Given graph of a function y = f(x), draw the graph of |y| = f(x) [means replacing y by |y|]. Then remove a portion
of the curve below x-axis and then take the reflection of the upper part on the lower part.

y = sin x 1
-2 
x
-4 -3 - 0 2 3
-1
Figure 13.37

6. TIPS FOR PLOTTING THE GRAPH OF A RATIONAL FUNCTION


(a) Examine whether denominator has a root or not. If no root, then graph is continuous and f is non-monotonic.
x
For example, f(x) = ⇒ for DrD < 0, Dr will never be zero.
2
x − 5x + 9
f(x) is discontinuous, only when dominator has roots and hence non-monotonic.
M a them a ti cs | 13.23

x2 + 2x − 3 (x + 3)(x − 1)
For example, f(x) = =
2
x + 2x − 8 (x + 4)(x − 2)
D = 0 at x = –4, 2.

y=2
x
-4 -3 2

Figure 13.38

(b) If numerator and denominator has a common factor (say x = a) it would mean removable discontinuity at x = a,
(x − 2)(x − 1)
E.g. f(x) = . Such a function will always be monotonic, i.e. either increasing or decreasing and
(x + 3)(x − 2)
removable discontinuity at x = 2. y

2 y=1
x
-3 -1 -3

Figure 13.39

(c) Compute point where the curve cuts both x-axis and y–axis by putting y = 0 and x = 0, respectively, and mark
points accordingly.
y
1

f(x) = x – 1
x-
)=
f(x

x = 0, y = –1
1
y = 0, x = 1 O x

Figure 13.40
dy
(d) Compute and find the intervals where f(x) increases or decreases and also where it has horizontal
tangent. dx
y
y = x2 – 3x + 2
3
x=
2 x
1
1
y=
4

Figure 13.41

(e) In regions where curves is monotonic, compute y if x →∞ or x → –∞ to find whether y is asymptotic or not.
x −1
f(x) =
x −3 y x=3

y=1
1/3
x
0

Figure 13.42
1 3 . 2 4 | Relations and Functions

(f) If denominator vanishes, say at x = a and (x – a) is not a common factor between numerator and denominator,
then examine Lim and Lim to find whether f approaches ∞ or –∞. Plot the graphs of the following function.
x →a− x →a+

x
Illustration 8: Draw the graph of functions f(x) =
 nx
Sol: Calculate the domain of the given function. Then use the derivative of the given
function to trace the given curve.
1. nx − x·(1 / x) e
Domain of f(x) is x ∈ R (0, 1) ∪ (1, ∞) , f’(x) =
( nx)2 0 1 e
f’(x) = 0 at x = e
also as x approaches zero f(x) approaches zero from negative side and x approaches ∞
f(x) approaches +∞. Figure 13.43

From the graph we can observe the range of f(x) is (–∞, 0) ∪ [e, ∞).

7. TO FIND DOMAIN AND RANGE


To calculate domain or range of a function, the following points are considered.

7.1 Domain
(a) Expression under even root (i.e. square root, fourth root, etc.) ≥ 0 and denominator ≠ 0.
(b) If domain of y = f(x) and y = g(x) are D1 and D2 respectively, then the domain of f(x) + g(x) or
f(x) · g(x) is D1 ∩ D2.
f(x)
(c) Domain of is D1∩D2 – {g(x) = 0}.
g(x)

(d) Domain of f(x) = D1 ∩ {x: f(x) ≥ 0}


(e) Expression inside logarithm should be positive, i.e. for logaE to exist, E should positive.

7.2 Range
(a) For the real valued function, real values of y for x ∈ domain of f are the range of the function.
Therefore, find domain of f and then impose restriction upon y, for the values of x in domain.
(b) If f is a continuous real valued function, then the range of function = [minimum f, maximum f].

Method of finding range:


(a) Range of the function in restricted domain
For the range of y = f(x) in the interval [a, b], retain the portion of the curve y = f(x) below the lines x = a and x =
b. Then the required range is the projection of y-axis.
(b) Range of composite function
To find the range of f(g(x)), first find the range of g(x), say A. Then find the range of f(x) in domain A.

(c) If f = (A sin x + B cos x) + C then range of function is  − A2 + B2 + C, A 2 + B2 + C 


  .
(d) Range of periodic function can be found only for the interval whose length is a period of a function.
(e) Similarly for odd functions, if range on right-hand side on x-axis is (α, β). Then range on left-hand side of
x-axis will be (–α, -β), to get the final range, union of both these.
M a them a ti cs | 13.25

(f) Change of variable


y = f(g(x)); to get the range of f(g(x)) substitute g(x) at t and then find the range of f(t) the domain of f(t).

PLANCESS CONCEPTS

Whenever we substitute the variable t for g(x), care should be taken that the corresponding condition
on t should be written immediately. Further analysis of the function will be according to the condition.

Akshat Kharaya (JEE 2009, AIR 235)

Illustration 9: Find domain and range of f(x) = cot −1 log 4 (5x2 − 8x + 4) .  (JEE MAIN)
5
Sol: Use the definition of Domain and Range.

Consider the quadratic expression P(x) = 5x2 –8x + 4.
For the above quadratic, D < 0 ⇒ the expression always positive. /2

\ e expression always ∈ R. /4

Pmaximum = ∞, Pminimum = 4/5. 1

Figure 13.44
\ Range of log 4 (5x2 − 8x + 4) is (–∞, 1]
5
Now, draw the graph of cot–1(t) for t (–∞, 1]
π 
From the graph we can observe that range of f(x) is  , π 
4 

x2 − x + 1
Illustration 10: Draw a graph of f(x) = and also evaluate its domain and range.  (JEE MAIN)
x2 + x + 1
Sol: Find derivative of the given function and use the techniques of curve tracing.
For drawing the graph of f(x) first find out those point where f’(x) = 0
2(x2 − 1) y
f’(x) =
(x2 + x + 1)2 3
f’(x) = 0 at x = –1, 1
(0, 1)
1 x
f(1) = ; f(–1) = 3 1/3
3 -1
0 1
Also when x approach ±∞, f approaches 1.
1  Figure 13.45
From the graph, domain is x ∈ R and range  , 3
3 
ax + b
Note: Graph of f(x) = is always monotonic.
cx + d

 1 
Illustration 11: If f(x) = sin–1 x2 + ln x − [x]  + cot   . Then find its domain and range. ( JEE ADVANCED)
  2
 1 + 2x 
Sol: Follow the steps discussed above.
1 3 . 2 6 | Relations and Functions

Domain of function is {–1, 1} – {0}, because x – [x] = 0 for integral value of x; hence, middle term will not be defined.
Also [{f}] = 0, whenever f is meaningful.
 1
 cot −1 x = tan−1 
Therefore value of f(x) = sin–1x2 + tan–1 (1 + 2 x2)  x
 when x > 0 
 
Function is continuous and is even.
π
Least value of the function will occur when x → 0 and is
4.
π 3π 7π
Maximum value = Lim f(x) = sin–1 1 + tan–1 (1 + 2 ) = + =
x →±1 2 8 8
 π 7π 
Therefore, range of f(x) is  ,  .
4 8 

8. DOMAIN AND RANGE OF COMMON FUNCTION

A. ALGEBRAIC FUNCTION

Function Domain Range

R, if n is odd
(i) xn, (n ∈ N) R = set of real numbers
R+ ∪ {0}, if n is even

1 R – {0}, if n is odd
(ii) , (n ∈ N) R – {0}
x n R+, if n is even

R, if n is odd R, if n is odd
(iii) x1/n (n ∈ N)
R+ ∪ {0}, if n is even R+ ∪ {0}, if n is even

1 R – {0}, if n is odd R – {0}, if n is odd


(iv) , (n ∈ N)
x1/n R+, if n is even R+, if n is even

B. TRIGONOMETRIC FUNCTION

Function Domain Range


(i) sin x R [–1, 1]
(ii) cos x R [–1, 1]

π
(iii) tan x R – (2k + 1) ,k∈I R
2

π
(iv) sec x R – (2k + 1) ,k∈I (–∞, –1] ∪ [1, ∞)
2

(v) cosec x R – kπ, , k ∈ I (–∞, –1] ∪ [1, ∞)


(vi) cot x R – kπ, , k ∈ I R
M a them a ti cs | 13.27

C. INVERSE TRIGONOMETRIC FUNCTION

Function Domain Range

(i) sin–1 x [–1, 1]  π π


− , + 
 2 2 
(ii) cos–1 x [–1, 1] [0, π]

(iii) tan–1 x R  π π
− , 
 2 2

(iv) sec–1 x (–∞, –1] ∪ [1, ∞) π


[0, π]–  
2

(v) cosec–1 x (–∞, –1] ∪ [1, ∞)  π π


 − ,  – {0}
 2 2
(vi) cot–1 x R (0, π)

D. EXPONENTIAL FUNCTION

Function Domain Range


(i) ex R R+
(ii) e1/x R – {0} R+ – {1}
(iii) ax, a > 0 R R+
(iv) a1/x, a > 0 R – {0} R+ – {1}

E. LOGARITHIMIC FUNCTION

Function Domain Range


(i) logax, (a > 0)(a ≠ 1) R+ R

1
(ii) logxa =
loga x R+ – {1} R – {0}

(a > 0) (a ≠ 1)

F. INTEGRAL PART FUNCTION

Function Domain Range


(i) [x] R I
1
(ii) 1 
 x  R – [0, 1)  , n ∈ I − {0} 
n 

G. FRACTIONAL FUNCTION

Function Domain Range


(i) {x} R [0, 1)

1
(ii) R–I (1, ∞)
{x}
1 3 . 2 8 | Relations and Functions

H. MODULUS FUNCTION

Function Domain Range


(i) |x| R R+ ∪ {0}

1 R – {0} R+
(ii)
|x|

I. SIGNUM FUNCTION

Function Domain Range

|x|
sgn(x) = R {–1, 0, 1}
x

J. CONSTANT FUNCTION

Function Domain Range


f(x) = c R {c}

9. EQUAL OR IDENTICAL FUNCTION


Two functions f and g are said to be equal if the following conditions are satisfied:
(i) The domain of f is equal to the domain of g
(ii) The range of f is equal to the range of g and
(iii) f(x) = g(x), for every x belonging to their common domain,
1 x
E.g. f(x) =and g(x) are identical functions.
x x2
Few examples of equal functions are listed as follows:

(i) f(x) = ln x2; g(x) = 2ln x (N.I.)

(ii) f(x) = sin–1(3x – 4x3); g(x) = 3 sin–1 x (N.I.)


π
(iii) f(x) = sec–1x + cosec–1x; g(x) = (N.I.)
2
(iv) f(x) = cot2x.cos2x : g(x) = cot2x – cos2x (I)

(v) f(x) = Sgn(x2 + 1) ; g(x) = sin2x + cos2x (I)

(vi) f(x) = tan2x.sin2x; g(x) = tan2x – sin2x (I)

(vii) f(x) = sec2x – tan2x; : g(x) = 1 (N.I.)

(viii) f(x) = tan(cot–1x); g(x) = cot(tan–1 x) (I)

(ix) f(x) = x2 − 1 ; g(x) = x − 1 x + 1 (N.I.)

(x) f(x) = tan x. cot x ; g(x) = sin x . cosec x (N.I.)


x
(xi) f(x) = e n e ; g(x) = ex (I)
1 − cos2x
(xii) f(x) = ; g(x) = sin x (N.I.)
2
M a them a ti cs | 13.29

( x )
2
(xiii) f(x) = x2 ; g(x) = (N.I.)

(xiv) f(x) = log(x + 2) + log(x – 3); g(x) = (x2 – x – 6) (N.I.)


1
(xv) f(x) = ; g(x) = x −2 (I)
|x|
(xvi) f(x) = x | x | ; g(x) = x2 sgn x (I)

x2n − 1
(xvii) f(x) = Limit ; g(x) = sgn(| x | –1) (I)
n→∞ x2n + 1

(xviii) f(x) = sin(sin–1 x); g(x) = cos(cos–1 x) (I)


1 x
(xix) f(x) = ; g(x) = (N.I.)
1 1+x
1+
x
(xx) f(x) = [{x}]; g(x) = {[x]} (I) (note that f(x) and g(x) are constant functions)
−1 x
(xxi) f(x) = e n cot ; g(x) = cot–1 x (I)
−1 x
(xxii) f(x) = e n sec ; g(x) = sec–1 x (N.I.) Identical if x ∈ ( −∞ , − 1] ∪ (1, ∞ ) (xxiii)

(xxiii) f(x) = (fog)(x); G(x) = (gof)(x) where f(x) = ex; g(x) = ln x (N.I.)

10. CLASSIFICATIONS OF FUNCTIONS


10.1 One–One Function
One–one function (Injective mapping)
A function f : A → B is said to be a one–one function or injective mapping if different elements of A have different
f images in B. Thus for x1, x2 ∈ A and f(x1) ∈ A.
f(x2) ∈ B, f(x1) = f(x2) ⇔ x1 = x2 or x1≠ x2⇔f(x1) ¹f(x2)
3 −x
Example: f1 → R → R, f(x) = x + 1; f(x) = e ; f(x) = logx
nx
Diagrammatically an injective mapping is shown as follows:

A B A B
a 1 a 1
b 2 OR b 2
c 3 c 3

Figure 13.46

Note:
(i) If there is increase or decrease of continuous function in whole domain, then f(x) is one–one.
(ii) If any line parallel to x-axis cuts the graph of the function only at one point, then the function is one–one.

10.2 Many–One Function (Not Injective)


A function f: A → B is said to be a many–one function if two or more elements of A have the same f image in B.
Thus for f: A → B is many–one if for x1, x2∈ A and f(x1) = f(x2) but x1≠ x2
1 3 . 3 0 | Relations and Functions

For example, f1→ R → R, f(x) = [ x ]; f(x) = | x |; f(x) = ax2 + bx = c; f(x) = sin x.


Diagrammatically a many–one mapping is shown as follows:
A B A B
x1 x1
y1 y1
x2 OR x2
x3 y2 x3 y2

Figure 13.47

Note:
(i) If any continuous function has at least one local maximum or local minimum, then f(x) is many–one. In other
words, if a line parallel to x-axis cuts the graph of the function at least at two points, then f is many–one.
(ii) If a function is one–one, it cannot be many–one and vice versa.
One–one + Many–one = Total number of mapping.

10.3 Onto Functions


If the function f : A → B is such that each element in B (co-domain) is the f image of at least one element in A, then
we say that f is a function of A ‘onto’ B. Thus f: A → B is surjective ∀ b ∈ B, ∃ some a ∈ A such that f(a) = b.
Diagrammatically surjective mapping is shown as follows:
A B A B
x1 y1 y1
x2 y2 x1
x3 y3 OR x2 y2
x4 y4 x3 y3

Figure 13.48

Note: if range is equal to co-domain, then f(x) is onto.

10.4 Into Functions


If f: A → B is such that there exists at least one element in co-domain which is not the image of any element in
domain, then f(x) is into.
Diagrammatically into function is shown as follows:
A B A B
x1 y1 x1
y1 y2
x2 y2 OR x2
y
x3 y3 x3 y2 4

Figure 13.49

Note: If a function is onto, it cannot be into and vice versa. A polynomial of degree even and odd defined from R
→ R will always be into and onto, respectively.
Thus a function can be one of these four types:
M a them a ti cs | 13.31

(i) one–one onto (injective & surjective) (I ∩ S)


A B
x1 y1
x2 y2
x3 y3

Figure 13.50

(ii) one–one onto (injective but not surjective) (I ∩ S)

A B
x1 y1
x2 y2 y4
x3 y3

Figure 13.51

(iii) Many–one onto (surjective but not injective) (S ∩ I)

A B
x1
y1
x2
x3 y2

Figure 13.52

(iv) Many–one onto (neither surjective nor injective) (I ∩ S)


A B
x1
y1
x2
y3
x3 y2

Figure 13.53

Illustration 12: Let A be a finite set. If f: A → A is an onto function, then show that f is one–one. (JEE MAIN)

Sol: Use the definition of one-one and onto function.


Let A = {a1, a2, …, an}. To prove that f is a one–one function, we will have to show that f(a1), f(a2), …, f(an) are distinct
elements of A. We have,
Range of f = {f(a1),· f(a2), …, f(an)}
Since f : A → A is a onto function. Therefore,
Range of f = A.
⇒f = {f(a1), f(a2), …, f(an)} = A
But, A is a finite set consisting of n elements.
Therefore, f(a1), f(a2), f(a3), …, f(an) are distinct element of A.
Hence, f : A → A is one–one.
1 3 . 3 2 | Relations and Functions

Illustration 13: Let C and R denote the set of all complex and all real numbers respectively. Then show that f: C →
R given by f(z) = | z |, for all z ∈ C is neither one–one nor onto. (JEE MAIN)

Sol: Using two complex conjugate numbers, we can prove that the given function is not one-one. For the second
part, use the fact that modulus of a number cannot be negative.
Injectivity: We find that z1 = 1 – i and z2 = 1 + i are two distinct complex numbers such that | z1 | = | z2 |, i.e. z1¹z2
but f(z1) = f(z2).
It is clear that different elements may have the same image. So, f is not an injection.
Surjectivity: f is not a surjection, because negative real numbers in R do not have their pre-image in C. In other
words, for every negative real number there is no complex number z ∈ C such that f(z) = | z | = a. So, f is not a
surjection.
Illustration 14: For function f: A →A, fof = f. Prove that f is one–one if and only if f is onto. (JEE MAIN)

Sol: Starting with the relation given in the question fof = f and use the definition of one-one and onto function.
Suppose f is one–one.
Then, (fof) (x) = f(x)
f(f(x)) = f(x)
f(x) = x (f is one–one)
Thus, f(x) = x, ∀ x ∈ A.
for each x A, there is an x ∈ A such that f(x) = x
∴ f is onto.
Now suppose f is onto.
Then, for each y ∈ A, there is an x ∈ A such that f(x) = y.
Let x1, x2 ∈ A and let f(x1) = f(x2).
Then there exist y1, y2 ∈ A such that x1 = f(y1)·x2 = f(y2)
f(f(y1)) = f(f(y2))
(fof)(y1) = (fof)(y2)
f(y1) = f(y2) (fof = f)
x1 = x2
∴ f is one–one.

Illustration 15: Show that the function f: R → R given by f(x) = x3 + x is a bijection.  (JEE ADVANCED)

Sol: Consider two elements x and y in the domain and prove that f(x) = f(y) implies x = y. Use the definition of onto
to prove that the function f is a bijection.
Injectivity: Let x, y ∈ R such that
f(x) = f(y)
⇒ x3 + x = y3 + y
⇒ x3 – y3 + (x – y) = 0
⇒ (x – y) (x2 + xy + y2 + 1) = 0
⇒ x – y = 0 [Qx2 + xy + y2≥ 0 for all x, y ∈R. x2 + xy + y2 + 1 ≥ 1 for all x,y ∈ R]
⇒ x=y
M a them a ti cs | 13.33

Thus, f(x) = f(y)


⇒ x = y for all x, y ∈ R
So, f is an injective map.
Surjectivity: Let y be an arbitrary element of R then
f(x) = y ⇒ x3 + x = y
⇒ x3 + x – y = 0
We know that an odd degree equation has at least one real root. Therefore, for every real value of y, the equation
x3 + x – y = 0 has a real root α, such that
a3 + α – y = 0
a3 + α = y
f(α) = y
Thus, for every y ∈ R, there exist α ∈ R such that f(α) = y.
So, f is a surjective map.
Hence, f : R → R is a bijection.

n + 1, if n is even
Illustration 16: Let f: N ∪ {0} → N ∪ {0} be defined by f(n) =  . Show that f is a bijective function.
  n-1, if n is odd (JEE MAIN)
27
Sol: Divide the solution in three cases. (0, 17)
Case I – When both the numbers are even, Case II – When both the numbers are odd & Case
III – When one is even and other is odd. -1 -3
Let f(n) = f(m).
-37
Case I : both n, m are even.
Then, n + 1 = m + 1. So, n = m. Figure 13.54

Case II : Both n, m are odd.


Then, n – 1 = m – 1, which implies m = n.
Case III : n is even and m is odd. Then, f(n) is odd and f(m) is even. So, f(n) ≠ f(m)
In any case, f(n) = f(m) implies n = m. Thus, f is one–one.
Now f(2n) = 2n + 1, f(2n + 1) = 2n for all n ∈ N. So, f is onto. Hence f is a bijective function.
Illustration 17: Draw the graph of the function under the following condition and also check whether the function
is one–one and onto or not, f :R → R, f(x) = 2x3 – 6x2 – 18x + 17 (JEE MAIN)

Sol: Find the derivative of given function and understand the nature of the curve. Also find the values of f(x) at
some particular values of x to trace the given curve.
Domain of the function is x ∈ R
Here, f’(x) = 6x2 – 12x – 18 = 6(x2 – 2x – 3) = 6(x + 1) (x – 3)
f’(x) = 0 at x = –1, 3
f(∞) = ∞
f(–∞) = −∞
f(0) = 17, f(–1) = 27, f(3) = –37
From the graph we can say that f is many one onto function.
1 3 . 3 4 | Relations and Functions

11. COMPOSITE FUNCTIONS


Let f: A → B and g: B → C be two functions. Then the function gof: A → C defined by
(gof) (x) = g (f(x)) ∀ x ∈A is called the composite of the two functions f and g. Diagrammatically
it is shown as follows:
x f(x)
 → f  → g → g(f(x))
Thus the image of every x ∈A under the function gof is g-image of the f-image of x.
Note that gof is defined only if x ∈ A, f(x) is an element of the domain of g so that we can take its g-image. Hence,
for the product gof, the range of f must be a subset of the domain of g. In general, gof not equal to fog.

12. PROPERTIES OF COMPOSITE FUNCTIONS


(i) The composite of functions is not commutative, i.e. gof ≠ fog.
(ii) The composite of functions is associative, i.e. if f, g, h are functions such that fo(goh) and (fog)oh are defined,
then fo(goh) = (fog)oh.
Associativity: f: N → I0. f(x) = 2x
1
1
g: I0 → Q, g(x) = ; h: Q → R h(x) = e x ⇒ (hog)of = ho(gof) = e2x
x
(iii) The composite of two bijections is a bijection, i.e. if f and g are two bijections such that gof is defined, then
gof is also a bijection.
Proof: Let f : A → B and g: B → C be two bijections. Then gof exists such that
gof: A → C
We have to prove that gof is one–one and onto.
One–one: Let a1, a2 ∈ A such that (gof)(a1) = (gof)(a2), then
(gof)(a1) = (gof)(a2) ⇒ g[f(a1)] = g[f(a2)]
⇒ f(a1) = f(a2) [∵ g is one–one]
⇒a1 = a2 [∵ f is one–one]
\ gof is also one–one function.
Onto: Let c ∈ C, then
c ∈ C ⇒ ∃ b ∈ B s.t. g(b) = c [∵ g is onto]
and b ∈ B ⇒ ∃ a∈A s.t. f(a) = b [∵ f is into]
Therefore, we see that
c ∈ C ⇒ ∃ a ∈ A s.t. gof(a) = g[f(a)] = g(b) = c
i.e. every element of C is the gof image of some element of A. As such gof is onto function. Hence gof being
one–one and onto is a bijection.

Illustration 18: Let f: R → R, g: R → R be defined by f(x) = x2 + 3x + 1, g(x) = 2x – 3. Find fog and gof.(JEE MAIN)

Sol: Use the concept of composite functions.


We have (fog)(x) = f(g(x)) = f(2x – 3)= (2x – 3)2 + 3(2x – 3) + 1= 4x2 – 6x+ 1
and (gof)(x) = g(f(x)) = g(x2 + 3x + 1)= 2(x2 + 3x + 1) – 3= 2x2 + 6x – 1
Hence fog ≠ gof.
M a them a ti cs | 13.35

1
Illustration 19: If (x) = and g(x) = 0 are two real functions, show that fog is not defined. (JEE MAIN)
x2
Sol: Find the domain of fog(x).
We have,
Domain (f) = R – {0}, Range (f) = R – {0}
Domain (g) = R and Range (g) = {0}
Clearly, Range (g) ⊄ Domain (f)
∴ Domain (fog) = {x: x ∈ Domain (g) and g(x) ∈ Domain (f)}
⇒ Domain (fog) = {x: x ∈ R and g(x) ∈ Domain (f)}
⇒ Doman (fog) = f
[∵ g(x) = 0 ∉ Domain (f) for any x ∈ R]
Hence, fog is not defined.

1 1 2x + 1 1 3
Illustration 20: If f(x) = , x ≠ − , then show that, f(f(x)) = , provided that x ≠ − , − . (JEE MAIN)
2x + 1 2 2x + 3 2 2
Sol: Check the domain of f(x) and use the concept of composite functions.
1
We have, f(x) =
2x + 1
 1
Clearly, domain (f) = R – − 
 2
1 1 1−y
Let, y = ⇒2x + 1 = ⇒x =
2x + 1 y 2y
1
Since x is a real number distinct from − , y can take any non-zero real value.
2
So, Range ( f ) = R – {0}
We observe that range
1 
( f ) = R – {0} ⊄ domain ( f ) = R –  
2 
∴ Domain (fof) = {x: x ∈ domain ( f ) and f(x) ∈ Domain ( f )}



Domain (fof) = x : x ∈ R − −

{ }1
2 { }
1 
and f(x) ∈ R − − 
2 
 1 1
⇒ Domain (fof) = x : x ≠ − and f(x) ≠ − 
 2 2
 1 1 1
⇒ Domain (fof) = x : x ≠ − and ≠− 
 2 2x + 1 2
 1 3 
⇒ Domain (fof) = x : x ≠ − and x ≠ − 
 2 2
 1 3
R − − , − 
 2 2 
 1  1 2x + 1
Also, fof(x) = f(f(x)) = f   = =
 2x + 1  2 (1 / (2x + 1) ) + 1 2x + 3

 1 3 2x + 1
Thus, fof : R – − , −  → R is defined by fof(x) =
 2 2 2x + 3
1 3 . 3 6 | Relations and Functions

2x + 1 1 3
Hence, f(f(x)) = for all x ∈ R. x ≠− , − .
2x + 3 2 2

Illustration 21: If f(x) = log100x  2log10 x + 2  and g(x) = {x}. If the function (fog)(x) exists then find the
range of g(x).   −x  (JEE ADVANCED)
 

Sol: Find the domain of f(x) and use the given information that fog(x) exists.
To define f(x), the following condition must hold good:
1 g f
(i) 100 x > 0 and 100x ≠ 1⇒ x ≠
100
1 1
(ii) x > 0 and log10x + 1 < 0 ⇒ 0 < x < and x ≠
10 100
 1   1 1  fog
∴ Domain of f(x) is  0,  ∪ , 
 100   100 10  .
Figure 13.55
Here, g(x) = {x}, range of g(x) is [0, 1).

But, (fog) (x) exists ⇒ range of g(x) ⊂ domain of f(x).

 1   1 1 
∴ Range of g(x) is  0,  ∪ , 
 100   100 10  .

Illustration 22: Consider functions f and g such that composite gof is defined and is one–one. Should f and g
necessarily be one–one?  (JEE MAIN)

Sol: Take an example to prove it.


Consider f: {1, 2, 3, 4} → {1, 2, 3, 4, 5, 6} defined as f(x) = x, ∀ x = 1, 2, 3, 4 and
g: {1, 2, 3, 4, 5, 6} ®{1, 2, 3, 4, 5, 6} as g(x), for x = 1, 2, 3, 4 and g(5) = g(6) = 5.
Then, gof(x) = x, ∀ x = 1, 2, 3, 4, which shows that gof is one–one. But g is clearly not one–one.

14. INVERSE OF A FUNCTION


Let f: A → B be a one–one and onto function, then there exists a unique function.
g: B → A such that f(x) = y ⇔ g(y) = x, ∀ x ∈ A and y ∈ B. Then g is said to be inverse of f. Thus
g = f–1: B → A = {(f(x), x) | (x, f(x)) ∈ f}.

14.1 Properties of Inverse Functions


(i) The inverse of a bijection is unique.
Proof: Let f : A → B be a bijection, and let g: B → A and h: B → A be two inverse functions of f. Also let a1, a2 ∈ A and
b ∈ B, such that g(b) = a1 and h(b) = a2. Then
g(b) = a1⇒f(a1) = b
h(b) = a2⇒f(a2) = b
Since f is one–one, f(a1) = f(a2) ⇒a1 = a2⇒g(b) = h(b), b ∈B.
(ii) If f: A → B is a bijection and g: B → A is the inverse of f, then fog = IB and gof = IA, where IA and IB are identity
functions on the sets A and B, respectively.
M a them a ti cs | 13.37

Note that the graphs of f and g are the mirror images of each other in the line y = x. As shown in the figure given
below a point (x’, y’) corresponding to y = x2(x ≥ 0) changes to (y’ x’) corresponding to y = x , the changed form
of x = y.

y y y
x= y x= y
2
y=x

x
y=

y=
2
y=x

0 0 0
x x x
(i) (ii) (iii)
Figure 13.56

(iii) The inverse of a bijection is also a bijection.


Proof: Let f : A → B be a bijection and g : B → A be its inverse. We have to show that g is one–one and onto.
One-one: Let g(b1 ) = a2 and g(b2 ) = a2 ; a1 ,a2 ∈ A and b1 ,b2 ∈ B
Then g(b1) = g(b2) ⇒ a1 = a2
⇒ f(a1) = f(a2) [∵ f is bijection]
⇒ b1 = b2 [∵ g(b1) = a1 ⇒ b1 = f(a1); g(b2) = a2 ⇒ b2 = f(a2)]
Which proves that g is one–one.
Onto: Again, if a ∈ A, then
a ∈ A ⇒ ∃ b ∈ B s.t. f(a) = b(by definition of f)
⇒ ∃ b ∈ B s.t. a = g(b) [∵ f(a) = b ⇒ a = g(b)]
Which proves that g is onto. Hence g is also a bijection.
(iv) If f and g are two bijections f : A → B, g : B → C then inverse of gof exists and (gof)–1 = f–1og–1.
Proof: since f : A → B and g : B → C are two bijections.
∴ gof: A → C is also a bijection.
[By theorem the composite of two bijection is a bijection.]
As such gof has an inverse function (gof)–1 : C → A. We have to show that
(gof)–1 = f–1og–1
Now let a ∈ A, b ∈ B, c ∈ C such that f(a) = b and g(b) = c.
So (gof) (a) = g[f(a)] = g(b) = c
Now f(a) = b ⇒ a = f–1(b) .…(i)
g(b) = c ⇒ b = g–1(c)  .… (ii)
(gof)(a) = c ⇒ a = [gof)–1(c)  .… (iii)
Also (f–1og–1) (c) = f–1[g–1(c)] [by definition]
= f (b) [by (ii)]
–1

= a [by (i)]
= (gof) (c)
–1
[by (iii)]
\ (gof)–1 = f–1og–1, which proves the theorem.
1 3 . 3 8 | Relations and Functions

PLANCESS CONCEPTS

In the line y = x, the graphs of f and g are the mirror images of each other. As shown in the following
figure, a point (x’, y’) corresponding to y = ln x(x >0) changes to (y’, x’) corresponding to y = ex, the
changed form of x = ey.
The inverse of a bijection is also a bijection.
If f and g are two bijections f: A → B, g: B → C, then inverse of gof exists and (gof)–1 = f–1og–1.

x
e
y y= =x

x
y

n
y= n x

y=
x
y=e
x x x
1 1

Figure 13.57

Nitish Jhawar (JEE 2009, AIR 7)

 x, x <1
 2
Illustration 23: Find inverse of the function f(x) =  x , 1 ≤ x ≤ 4  (JEE MAIN)

8 x , x>4
Sol: Put f(x) = y and solve for x.
Graph of f(x)
 8 x
 y if y < 1 16
B

Using the above graph f (y) = x =  y if 1 ≤ y ≤ 16
–1
A 2
1 x
 2 x
 y if y > 16
164
 0 1 4
 x if x < 1

or f–1(x) =  x if 1 ≤ x ≤ 16 . Figure 13.58
 2
 x if x > 16
 16

Illustration 24: Let f: N → R be a function defined as f(x) = 4x2 + 12x + 15. Show that f : N → S, where S is the range
of f, is invertible. Find the inverse of f. (JEE MAIN)

Sol: Put 4x2 + 12x + 15 = y and solve for x. Then put f-1(y) in place of x.

Let y be an arbitrary element of range f. Then y = 4x2 + 12x + 15, for some x in N, which implies that y = (2x + 3)2
( y − 6 − 3)
+ 6. This gives x = , as y ≥ 6.
2
( y − 6 − 3)
Let us define g: S → N by g(y) = .
2

Now, gof(x) = g(f(x)) = g(4x2 + 12x + 15) = g((2x + 3)2 + 6)


M a them a ti cs | 13.39

((2x + 3)2 + 6 − 6) − 3 (2x + 3 − 3)


= = =x
2 2
 y −6 −3 2 y −6 −3 
and fog(y) = f   =  + 3 + 6
 2   2 
   
= ( y − 6 − 3 + 3)2 + 6 = ( y − 6 )2 + 6 = y – 6 + 6 = y

Hence, gof = IN and fog = IS. This implies that f is invertible with f–1 = g.

2x − 5
Illustration 25: For the function f: R – {4} → R – {–2} : f(x) = . Find
4−x
(a) zero’s of f(x) (b) range of f(x)
(c) intervals of monotonocity (d) f–1(x)
(e) local maxima and minima if any (f) interval when f(x) is concave upward and concave downward
2
(g) asymptotes (h) ∫ f(x) dx
1

(i) nature of function whether one–one or onto ( j) graph (JEE MAIN)

Sol:
(a) 5/2 (b) (–∞, –2) ∪ (–2, ∞)

4x + 5
(c) ↑ in its domain i.e., (–∞, 4) ∪ (4, ∞) (d) f–1(x) =
x+2
(e) no, (f) (–∞, 4) upwards and (4,∞) downwards

 2
(g) (g) y = –2 (h) −  2 + 3l n 
 3

(i) both one-one and onto
y 

0 5/2
x
( j) -1 4
-5/4
y=-2
-2
-
Figure 13.59

Illustration 26: f: R → R, f(x) = 3x + 2. Find the inverse of f, if it exists. (JEE MAIN)

Sol: Check whether the given function is one-one onto. Put 3x + 2 = y and solve for x. Then put f-1(y) in place of x.
∀ x1, x2∈ A, f(x1) = f(x2) ⇒ 3x1 + 2 = 3x2 + 2 ⇒ x1 = x2


f: R → R is one–one.
y −2
Now let y ∈ R. Then y = 3x + 2 ⇒ x =
3
y −2
Hence, for every y ∈ R, there is a corresponding x = ∈ R such that y = f(x). Hence, the range of f is R and so
f is onto, and the inverse of f exists. 3
1 3 . 4 0 | Relations and Functions

Again, y = f(x) ⇔ x = f–1(y)


y −2 x−2
∴ f–1 : R → R, f–1(y) = or equivalently, f–1(x) = .
3 3

15. DIFFERENT TYPES OF FUNCTIONS


15.1 Homogeneous Functions
A function is said to be homogeneous with respect to any set of variables when each of its terms is of the same
degree with respect to those variables.
For example 5x2 + 3y2 – xy is homogeneous in x and y. Symbolically if,
f(tx, ty) = tn. f(x,y), then f(x, y) is homogeneous function of degree n.
Examples of Homogeneous function:
x − y cos x
f(x, y) = is not a homogeneous function and
y sinx + x
x y y x y
f(x, y) = ln + ln ; x2 − y 2 + x; x + ycos are homogeneous function of degree one.
y x x y x

15.2 Bounded Function


A function is said to be bounded if | f(x) | ≤ M, where M is a finite quantity.

15.3 Implicit and Explicit Function


A function defined by an equation not solved for the dependent variable is called an implicit function. For example,
the equation x3 + y3 = 1 defines y as an implicit function. If y has been
y- x
expressed in terms of only x, then it is called explicit function.
Examples on implicit and explicit function (x, y) = 0 y- x
2 y- x
 1. x 1 + y + y 1 + x =0 ; explicit y=x
y- x
f1(x)
x f2(x) A
y= − or y = x (rejected) f2(x)
1+x
O
 2. y2 = x represents two separated branches y=f3(x)

 3. x3 + y3 – 3xy = 0 folium of desecrates Figure 13.60

15.4 Odd and Even Function


A function f(x) defined on the symmetric interval (–a, a) then,
If f(–x) = f(x) ∀ x in the domain of ‘f’ then f is said to be an even function.
E.g., f(x) = cos x; g(x) = x2 + 3;
If f(–x) = –f(x) ∀ x in the domain of ‘f’ then f is said to be an odd function.
E.g., f(x) = sin x; g(x) = x3 + x.

Examples on odd and even functions:

Odd Even Neither odd nor even

2x + 1
1. n  x + 1 + x2  1. x 1. 2x3 – x + 1k
  x
2 −1
M a them a ti cs | 13.41

1−x
2. n 2. 3
(1 − x)2 + 3 (1 + x)2 2. sin x + cos x
1+x

3. x sin2x – x3 3. constant

4. 1 + x + x2 − 1 + x + x2 4. x2 – | x |

1 + 2Kx (1 + 2x )2
5. 5.
Kx
1−2 2x

PLANCESS CONCEPTS
(a) f(x) – f(–x) = 0 ⇒ f(x) is even and f(x) + f(–x) = 0 ⇒ f(x) is odd.
(b) A function may neither be odd nor even.
(c) Inverse of an even function is not defined and an even function cannot be strictly monotonic.
(d) Every even function is symmetric about the y-axis and every odd function is symmetric about the
origin.
(e) Every function can be expressed as the sum of an even and an odd function.
E.g.,

2x + 2–x 2x – 2–x
f(x) = f(x) + f(–x) + f(x) – f(–x) 2= +
x

2 2 2 2
EVEN ODD EVEN ODD
(f) The only function which is defined on the entire number line and is even and odd at the same time
is f(x) = 0.
(g) If f and g are either both even or odd, then the function f · g will be even. But if any one of them is
odd then f · g will be odd.

f(x) g(x) f(x)+g(x) f(x) – g(x) f(x) . f(x) / (gof) / (fog) (x)
g(x) g(x) (x)
odd odd Odd odd even even odd odd
even even Even even even even even even
odd even neither odd nor even neither odd nor even odd odd even even
even odd neither odd nor even neither odd nor even odd odd even even

Shrikant Nagori (JEE 2009, AIR 30)

15.5 Periodic Functions


A function f(x) is called periodic if there exists a positive number T (T > 0) called the period of the function such that
f(x + T) = f(x), for all values of x within the domain of x.
E.g., Both the functions sin x and cos x are periodic over 2π, and tan x is periodic over π.
Examples on periodic function
2x 4x
(i) f(x) = cos – sin (15π)
3 5
1 3 . 4 2 | Relations and Functions

(ii) f(x) = cos (sin x) (π)

(iii) f(x) = sin(cos x) (2π)


π
(iv) f(x) = sin4x + cos4x  
2
(v) f(x) = x – [x] = {x} (one)

PLANCESS CONCEPTS

•• f(T) = f(0) = f(–T), where ‘T’ is the period.


•• Inverse of a periodic function does not exist.
•• Every constant function is always periodic, with no fundamental period.
•• If f(x) has a period T and g(x) also has a period T then it does not mean that f(x) + g(x) must have a
period T. For example, f(x) = | sin x | + | cos x |
1
•• If f(x) has a period p, then and f(x) also has a period p.
f(x)
•• If f(x) has a period T then f(ax + b) has a period T/a (a >0).
Proof : Let f(x + T) = f(x) and f[a (x + T’) + b] = f(ax + b)
f(ax + b + aT’) = f(ax + b)
T
f(y + aT’) = f(y) = f(y + T) ⇒ T = aT’ ⇒ T’ =
a
Vaihbav Gupta (JEE 2009, AIR 54)

15.6 Special Functions


If x, y are independent variables, then
(a) f(xy) = f(x) + f(y) ⇒ f(x) = k ln x or f(x) = 0
(b) f(xy) = f(x) · f(y) ⇒ f(x) = xn , n∈ R
(c) f(x + y) = f(x) · f(y) ⇒ f(x) = akx
(d) f(x + y) = f(x) + f(y) ⇒ f(x) = kx, where k is a constant

Illustration 27: Which of the following function(s) is(are) bounded on the intervals as indicated
1
x −1 1
(A) f(x) = 2 on (0, 1) (B) g(x) = x cos on (–∞,∞)
x
(C) h(x) = xe–x on(0, ∞) (D) l(x) = arc tan 2x on (–∞,∞)

Sol: Check for the continuity of the given functions. If the function is continuous then to find the value of f(x) at
the boundary points.
1 1
1 1/e
Limit f(x) Limit
(A) = = 2h−1 ; Limit
= f(x) Limit
= 2 −h 0
+
x →0 h→0 2 x →I− h→0 1

 1 Figure 13.61
⇒ f(x) ∈  0,  ⇒ bounded
 2
M a them a ti cs | 13.43

x
(C) Limit
= xe− x Limit
= he−h 0; Limit
= x e− x Limit
= 0
h→0 h→0 x →∞ x →∞ ex

x ex − xex  1
⇒ also y = ⇒ y’ = ex(1–x)⇒ h(x) =  0, 
 e
x 2x
e e

FORMULAE SHEET

Table: Domain and range of some standard functions-

Functions Domain Range

Polynomial function R R

Identity function x R R

Constant function K R (K)

R0 R0
1
Reciprocal function
x

X2, x (modulus function) R R + ∪ {x}

x3 , x x R R

R {-1,0,1}
x
Signum function
x

X+ x R R + ∪ {x}

x- x R R − ∪ {x}

[x] (greatest integer function) R 1

x-{x} R [0,1]

(0, ∞) [0,∞]
x

ax(exponential function) R R+

Log x(logarithmic function) R+ R


1 3 . 4 4 | Relations and Functions

Inverse Trigo Functions Domain Range

sin–1x (-1,1]
 −π π 
 , 
 2 2

cos–1x [-1,1] [0, π ]

tan–1x R
 −π π 
 , 
 2 2

cot–1x R (0, π )

sec–1x R-(-1,1)
π
[0, π ]-  
2 

cosec–1x R-(-1,1)
 −π π 
 ,  -{0}
 2 2

Inverse function: f–1 exists iff f is both one–one and onto.

f-1:B→A, f-1(b)=a ⇒ f(a)=b

Even and odd function: A function is said to be

(a) Even function if f(x)=f(x) and


(b) Odd function if f(–x)= –f(x)

Properties of even & odd function:

(a) The graph of an even function is always symmetric about y-axis.


(b) The graph of an odd function is always symmetric about origin.
(c) Product of two even or odd function is an even function.
(d) Sum & difference of two even (odd) function is an even (odd) function.
(e) Product of an even or odd function is an odd function.
(f) Sum of even and odd function is neither even nor odd function.
(g) Zero function, i.e. f(x) = 0, is the only function which is both even and odd.
(h) If f(x) is an odd (even) function, then f ’(x) is even (odd) function provided f(x) is differentiable on R.
(i)A given function can be expressed as sum of even and odd function.

1 1
(x)
i.e. f=
2
f ( x ) + f ( −x )  + f ( x ) − f ( −x )  =even function + odd function.
  2 

Increasing function: A function f(x) is an increasing function in the domain, D if the value of the function does not
decrease by increasing the value of x.

Decreasing function: A function f(x) is a decreasing function in the domain, D if the value of function does
not increase by increasing the value of x.
M a them a ti cs | 13.45

Periodic function: Function f(x) will be periodic if a +ve real number T exists such that

f ( x +=
T ) f ( x ) , ∀× ∈ Domain.

There may be infinitely many such real number T which satisfies the above equality. Such a least +ve number
T is called period of f(x).

(i) If a function f(x) has period T, then period of f(xn+a)=T/n and period of (x/n+a)=nT.

(ii) If the period of f(x) is T1 & g(x) has T2 then the period of f(x) ± g(x) will be L.C.M. of T1& T2 provided it satis-
fies definition of periodic function.

(iii) If period of f(x) & f(x) are same T, then the period of af(x)+bg(x) will also be T.

Function Period

sin x, cos x 2π

sec x, cosec x

tan x, cot x π

sin (x/3) 6π

tan 4x π /4

cos 2 π x 1

cos x π

sin4x+cos4x π /2

x−π
2 cos   6π
 3 

sin3 x + cos3x 2 π /3

Sin3 x +cos4x 2π

sinx

sin5x

tan2 x − cot2 x π

x-[x] 1

[x] 1
1 3 . 4 6 | Relations and Functions

Solved Examples

JEE Main/Boards Sol: Use algebra of Sets.


A – B = {2, 4, 6} and B – A = {9, 11, 13}
Example 1: If a set A = {a, b, c}, then find the number
of subsets of the set A and also mention the set of all Example 6: Let A be the set of all students of a
the subsets of A. boys school Show that the relation R in A given by
R = {(a, b) : a is sister of b} is the empty relation and
Sol: Use the formula for the no. of subsets and list it
R’ = {(a, b) : the difference between heights of a and b is
down.
less than 3 meters} is the universal relation.
Since n(A) = 3.
Sol: Show that the relation R = φ and the relation R’ is
∴ Number of subsets of A is 23 = 8 and set of all those
true for any two.
subsets is P(A) named as power set.
Since the school is boys school, no student of the
P(A): {φ, {a}, {b}, {c}, {a, b}, {b, c}, {a, c}, {a, b, c}}
school can be sister of any student of the school. Hence,
R = φ , showing that R is the empty relation. It is also
Example 2: Show that n {P[P(φ)]} = 4 obvious that the difference between heights of any two
students of the school has to be less than 3 metres. This
Sol: We have P(φ) = {f} shows that R’ = A × A is the universal relation.
∴ P(P(φ)) = {φ, {f}}
⇒ P[P(P)] = {φ, { φ }, {{ φ }}, {φ, { φ }} Example 7: Let T be the set of all triangles in a plane
with R a relation in T given by R = {(T1, T2)} : T1 is
Hence n {P[P(φ)]} = 4. congruent to T2. Show that R is an equivalence relation.

Example 3: If A = {x: x = 2n + 1, n ∈ Z} and B = {x: x = Sol: Prove that R satisfies the conditions for reflexive,
2n, n ∈ Z}, then find A ∪ B. symmetric and transitive relation.
R is reflexive, since every triangle is congruent to itself.
Sol: Write the two sets and then take union. Further (T, T1) ∈ R2.
A ∪ B = {x: x is an odd integer} ∪ {x: x is an even integer} ⇒ T1 is congruent to T2 ⇔ T2 is congruent to T1. (T2, T1)
= {x: x is an integer} = Z. ∈ R, Hence R is symmetric.
Moreover, (T1, T2), (T2, T3) ∈ R.
Example 4: If A = {x: x = 3n, n ∈ Z} andB = {x: x = 4n, n
∈ Z}, then find A ∩ B. ⇒ T1 is congruent to T2 and T2 is congruent to T.
⇒ T1 is congruent to T3 ⇒ (T1, T3) ∈ R. Therefore, R is
Sol: Clearly, set A is a multiple of 3 and the other set is a
an equivalence relation.
multiple of 4. Hence, the intersection of the two would
be set having multiples of 12.
Example 8: Let L be the set of all lines in a plane
We have and R be the relation in L defined as R = {(L1, L2): L1
x ∈ A ∩ B ⇔ x = 3n, n ∈ Z and x = 4n, n ∈ Z is perpendicular to L2}. Show that R is symmetric but
neither reflexive nor transitive.
⇔ x is a multiple of 3 and x is a multiple of 4.
⇒ x is a multiple of 3 and 4 both. Sol: Use the definition of Symmetric relation.
⇔ x is a multiple of 12 ⇔ x = 12n, n ∈ Z. R is not reflexive as the line L1 cannot be perpendicular
to itself, i.e. (L1, L1) ∉ R. R is symmetric as(L1, L2) ∈ R.
Hence, A ∩ B = {x: x = 12n, n ∈ Z}.
⇒ L1 is perpendicular to L2.
Example 5: If A = {2, 3, 4, 5, 6, 7} andB = {3, 5, 7, 9, 11, ⇒ L2 is perpendicular to L1.
13}, then find A – B andB – A.
⇒ (L2, L1) ∈ R.
M a them a ti cs | 13.47

R is not transitive. Indeed, if L1 is perpendicular to showing that R is reflexive.


L2 and L2 is perpendicular to L3, then L1 can never be
Similarly, (a, b) ∈ R ⇒ f(a) = f(b)
perpendicular to L3. In fact, L1 is parallel to L3, i.e. (L1, L3)
∈ R (L2 , L3) ∈ R but (L1, L3) ∈ R. ⇒ f(b) = f(a) ⇒ (b, a) ∈ R
Therefore, R is symmetric.
Example 9: Show that the relation R in the set {1, 2, 3}
Further, (a, b) ∈ R and (b, c) ∈ R.
given by R = {(1, 1), (2, 2), (3, 3), (1, 2), (2, 3)} is reflexive
but neither symmetric nor transitive. ⇒ f(a) = f(b) and f(b) = f(c)
⇒ f(a) = f(c)
Sol: R is reflexive, since (1, 1), (2, 2) and (3, 3) lie in R.
Also R is not symmetric, as (1, 2) ∈ R but (2, 1) ∉ R. (a, c) ∈ R, which implies that R is transitive.
Similarly, R is not transitive, as (1, 2) ∈ R and (2, 3) ∈ R Hence, R is an equivalence relation.
but (1, 3) ∉ R.

Example 13: Let R be a relation from Q into Q defined


Example 10: Show that the relation R in the set Z by R = {(a, b): a, b ∈ Q and a – b ∈ Z}. Show that,
of integers given by R = {(a, b): 2 divides a – b} is an
equivalence relation. (i) (a, a) ∈ R for all a ∈ Q.
(ii) (a, b) ∈ R implies (b, a) ∈ R.
Sol: Similar to example 7.
(iii) (a, b) ∈ R, (b, c) ∈ R implies (a, c) ∈ R.
R is reflexive, as 2 divides (a – a) for all a ∈ Z. Further, if
(a, b) ∈ R, then 2 divides a – b]. Sol: Do it by yourself.
Therefore, 2 divides b – a. Hence, (b, a) ∈ R, which (i) Since a – a = 0 ∈ Z, it follows that (a, a) ∈ R, for all
shows that R is symmetric. Similarly if (a, b) ∈ R and a∈Q
(b, c) ∈ R, then a – b and b – c are divisible by 2.
(ii) (a, b) ∈ R implies a – b ∈ Z. So, b – a ∈ Z. Therefore
Now, a – c = (a – b) + (b – c) is even. So, (a – c) is (b, a) ∈ R.
divisible by 2. This shows that R is transitive. Thus, R is
an equivalence relation in Z. (ii) (a, b) ∈ R, (b, c) ∈ R implies a – b ∈ Z,
b – c ∈ Z. So a – c = (a – b) + (b – c) ∈ Z. Therefore,
Example 11: Let X = {1, 2, 3, 4, 5, 6, 7, 8, 9}. Let R1 be a (a, c) ∈ R.
relation in X given by R1 = {(x, y): x – y is divisible by 3}
and R2 be another relation on X given by R2. = {(x, y): Functions
{x, y} ⊂ {1, 4, 7} or {x, y} ⊂ {2, 5, 8} or {x, y} ⊂ {3, 6, 9}}.
Show that R1 = R2. Example 14: Show that f : N → N, given by
x + 1, if x is odd
Sol: Prove that R1 is a subset of R2 and vice versa. f(x)=  Is both one–one and onto.
x – 1, if x is even
Note that the characteristic of sets {1, 4, 7}, {2, 5, 8} and
{3, 6, 9} is that difference between any two elements of Sol: Use the definition, and consider the case when the
these sets is a multiple of 3. Therefore (x, y) ∈ R1. two are either even or odd.
⇒ x – y is multiple of 3 ⇒ {x, y} ⊂ {1, 4, 7} or {x, y} ⊂ Suppose f(x1) = f(x2). Note that if x1 is odd and x2 is even,
{2, 5, 8} or {x, y} ⊂ {3, 6, 9} ⇒ (x, y) ∈ R2. Hence R1⊂ R2. then we will have x1 + 1 = x2–1, i.e., x2–x1 = 2 which is
impossible. Similarly the possibility of x1 being even and
Similarly, {x, y} ∈ R2⇒ {x, y} ⊂ {1, 4, 7} or {x, y} ⊂ {2, 5, 8}
x2 being odd can also be ruled out, using the similar
or {x, y} ⊂ {3, 6, 9}
argument. Therefore, both x1 and x2 must be either odd
⇒ x – y is divisible by 3 ⇒ {x, y} ∈ R1. This shows that or even. Suppose both x1 and x2 are odd. Then
R2 ∈ R1. Hence, R1 = R2.
f(x1) = f(x2)⇒ x1 + 1 = x2 + 1
⇒ x1 = x2.
Example 12: Let f: X → Y be a function. Define a relation
R in X given by R = {(a, b): f(a) = f(b)}. Examine if R is an Also any odd number 2r + 1 in the co–domain N is the
equivalence relation. image of 2r + 2 in the domain N and any even number
2r in the co–domain N is the image of 2r – 1 in the
Sol: For every a ∈ X, (a, a) ∈ R, since f(a) = f(a), domain N. Thus f is onto.
1 3 . 4 8 | Relations and Functions

Example 15: Which of the following functions are gof(x) = (x + 1)2, –2 ≤ x < 1
even/odd ?
= (x + 1)2 – 2 ≤ x ≤ 1
(i) f(x) = sin x + cos x

(ii) f(x) = 1 + x + x2 − 1 − x + x2 Example 18: If f(x) = (2 + (x – 3)3)1/3, find f–1.

Sol: A function is even if f(-x) = f(x) and odd if f(-x) = -f(x). Sol: Take (2 + (x – 3)3)1/3 = y and solve for x. Then put
f-1(y) in place of x.
(i) f(x) sin x + cos x
y = [(2 + (x – 3)3]1/3
f(–x) = – sin x + cos x
y3 = 2 + (x – 3)3 ; (x – 3)3 = y3 – 2
f(–x) ¹f(x) ∀ x ∈ R;
x – 3 = (y3 – 2)1/3 ; x = 3 + (y3 – 2)1/3
hence, f is not an even function.
g(y) = x = 3 + (y3 – 2)1/3 is the inverse function.
f( −x) ≠ −f(x) ∀ x ∈ R ; hence f is not an odd function.
\ f is neither even nor odd. Example 19: Let f(x) = x2 + x be defined on the interval
(ii) f(x) = 1 + x + x2 − 1 − x + x2 [0, 2]. Find the odd and even extensions of f(x) in the
interval [–2, 2].
f(–x) = 1 − x + x2 − 1 + x + x2
=–f(x) Sol: The definition is given for 0 ≤ x ≤ 2, so in order to
find the even and odd extension, define the function
∴ f is an odd function. for -2 ≤ x < 0.
2 + x : x ≥ 0 Odd extension
Example 16: Let f(x) =  .
2 − x : x < 0  f(x), 0≤x≤2  x2 + x, 0 ≤ x ≤ 2
Find f of(x) f(x) =  =  2
 − f( − x) −2≤x<0 −x + x, −2 ≤ x < 0
Sol: Use the concept of composite functions.
Even extension
f: R → R
 f(x), 0 ≤ x ≤ 2 x2 + x, 0 ≤ x ≤ 2
fof(x) = f(f(x)) f(x) =  =  2
f( −x), −2 ≤ x < 0  x − x, −2 ≤ x < 0
Let x ≥ 0; fof(x) = f(2 + x) = 2 + (2 + x) as 2 + x ≥ 0
\ fof(x) = 4 + x when x ≥ 0
Example 20: The value of n ∈ I for which the function
Let x < 0; fof(x) = f(2 – x) = 2 + (2 – x) sinnx
f(x) = has 4p as its period is
(as 2 –x ≥ 0) = 4 – x sin(x / n)
(A) 2 (B) 3 (C) 5 (D) 4
 4 + x if x ≥ 0
∴ fof(x) = 
 4 − x if x < 0 Sol: For n = 2,
sin2x
We have = 4(cos x/2) cos x.
Example 17: Let, sin(x / 2)
The period of cos x is 2p and that of cos(x/2) is 4p.
f(x) = x + 1,x ≤ 1= 2x + 1, 1 < x ≤ 2
sin2x
g(x) = x2, –1 £x < 2 = x + 2, 2 ≤ x ≤ 3 Hence the period of is 4p.
sin(x / 2)
Find fog and gof.
sin3x sin5x sin 4x
Also, the period of , and
Sol: Similar to the previous example. sin(x / 3) sin(x / 5) sin(x / 4)
cannot be 4p.
f{g(x)} = g(x) + 1, g(x) ≤ 1
= 2g(x) + 1, 1 < g(x) ≤ 2
Example 21: Find the range of the following functionf(x)
⇒ f{g(x)} = x2 + 1, –1 ≤ x ≤ 1
3
= 2x2 + 1, 1 < x ≤ 2 ; g{f(x)} = {f(x)}2, =
2 − x2
– 1 £f(x) < 2= f(x) + 2, 2 £f(x) £3
M a them a ti cs | 13.49

Sol: First find the domain of the given function and JEE Advanced/Boards
then proceed to find the values f(x) can take..
3 Example 1: Let A = {1, 2, 3}. Then show that the
Let y = = f(x)  … (i)
2−x 2 number of relations containing (1, 2) and (2, 3) which
are reflexive and transitive, but not symmetric, is four.
The function y is not defined for x = ± 2
2y − 3 Sol: Try all the possibilities and prove that the number
From (i), x2 =
y of such relations can be four.
2y − 3
since for real x, x2≥ 0, We have 0 The smallest relation R1 containing (1, 2) and (2, 3)
y
≥ which is reflexive and transitive, but not symmetric, is
\ y ≥ 3/2 or y < 0 (Note that y ≠ 0)
{(1, 1), (2, 2), (3, 3), (1, 2), (2, 3), (1, 3)}. Now, if we add
Hence the range of the function is the pair (2, 1) to R1 to get R2, then the relation R2 will
[–∞, 0] ∪ [3/2, ∞) be reflexive and transitive, but not symmetric. Similarly,
we can obtain R3 and R4 by adding (3, 2) and (3, 1),
respectively, to R1 to get the desired relations. However,
Example 22: Knowing the graph of y = f(x) draw we cannot add any two pairs out of (2, 1), (3, 2) and
f(x)+ | f(x) | f(x)− | f(x) | (3, 1) to R1 at a time, as by doing so, we will be forced
y= and y =
2 2 to add the remaining third pair in order to maintain
transitivity and in the process, the relation will become
Sol: Use the basics of curve tracing. symmetric also which is not required. Thus, the total
Let graph number of desired relations is four.

y
y= f(x) Example 2: Show that the number of equivalence
y
y=f(x) relation in the set {1, 2, 3} containing (1, 2) and (2, 1)
is two.
x x
O O
Sol: Similar to the previous one.
y
y y- f(x)+ f(x) The smallest equivalence relation R1 containing (1, 2)
2 and (2, 1) is {(1, 1), (2, 2), (3, 3), (1, 2), (2, 1)}. Now we are
x
O left with only four pairs namely (2, 3), (3, 2), (1, 3) and (3,
x f(x)- f(x)
O y= 1). If we add any one, say (2, 3) to R1, then for symmetry
2
we must add (3, 2) also and now for transitivity we are
forced to add (1, 3) and (3, 1). Thus, the only equivalence
Example 23: Draw the following graphs :
relation bigger than R1 is the universal relation. This
(i) | y | = cos x (ii) | y | = sin x shows that the total number of equivalence relations
containing (1, 2) and (2, 1) is two.
Sol: Similar to the previous example.
(i) | y | = cos x Example 3: Let R be a relation on the set A of ordered
pairs of positive integers defined by (x, y) R (u, v) if and
y only if xv = yu. Show that R is an equivalence relation.

Sol: Prove that the relation is reflexive, symmetric and


transitive.
-3/2 -/2 /2 3/2
Clearly, (x, y) R (x, y), ∀ ∈ (x, y) ∀ ∈ A, since xy = yx. This
shows that R is reflexive.
(ii) | y | = sin x Further, (x, y) R (u, v) ⇒ xv = yu ⇒ uy = vx and hence
(u, v) R (x, y).
y
This shows that R is symmetric.
Similarly, (x, y) R (u, v) and (u, v) R (a, b)
x ⇒ xv = yu and ub = va
- 0 
1 3 . 5 0 | Relations and Functions

a a a h(x) = 0, if x ≤ 0= x, if x ≥ 0,
⇒ xv = yu ⇒ xv b = yu
u u v u
then find the composite function ho(fog) and determine
⇒ xb = ya and hence (x, y) R (a, b). Thus, R is transitive. whether the function fog is invertible and the function
Thus, R is an equivalence relation. h is identity function.

Sol: Use the concept of composite functions and check


Example 4: If R1 and R2 are equivalence relation in set if the function is on-one onto.
A, show that R1 ∩ R2 is also an equivalence relation.
Here f(x) = x2 – 1 for all x.
Sol: Refer to the type of relation. and g(x) = x2 + 1 for all x
Since R1 and R2 are equivalence relations, (a, a) ∈ R1 and \f{g(x)} = {g(x)}2 – 1 = x2 + 1 – 1 = x2 for all x
(a, a) ∈ R2 ∀ a ∈ A. This implies that (a, a) ∈ R1 ∩ R2, ∀
a, showing R1 ∩ R2, is reflexive. Further, (a, b) ∈ R1 ∩ R2 \h{f(g(x))} = h(x)2 = x2
⇒ (a, b) ∈ R1 and (a, b) ∈ R2 ⇒ (b, a) ∈ R1 and (b, a) ∈ R2 because x2 ≥ 0 [from definition of h(x)]
⇒ (b, a) ∈ R1 ∩ R2, hence R1 ∩ R2 is symmetric.
Now, f{g(x)} = x2 for all x.
Similarly, (a, b) ∈ R1 ∩ R2 and (b, c) ∈ R1 ∩ R2
As x2≥ 0, (fog) (x) cannot be negative.
⇒ (a, c) ∈ R1 and (a, c) ∈ R2
So fog is not invertible.
⇒ (a, c) ∈ R1 ∩ R2. This shows that R1 ∩ R2 is transitive.
Again, h(x) = x for x ≥ 0.
Thus, R1 ∩ R2 is an equivalence.
But, by definition h(x) ≠ x for x < 0.
Example 5: If A and B be two sets containing 3 and Hence, h is not the identity function.
6 elements, respectively, what can be the minimum
number of elements in A ∪ B? Also find the maximum Example 7: Let f : R → R be given by
number of elements in A ∪ B.
f(x) = (x +1)2 – 1, x ≥ –1. Show that f is invertible. Also,
Sol: Vary the number of elements in the intersection of find the set S = {x: f(x) = f–1(x)}.
the two sets and find the maximum and the minimum.
Sol: Check if the function is one-one and onto.
We have,
In order to show that f(x) is invertible, it is sufficient to
n(A ∪ B) = n(A) + n(B) – n(A ∩ B). show that f(x) is a bijection.
This shows that n(A ∪ B) is minimum or maximum f is an injection. For any x, y ∈ R satisfying
according as n(A ∩ B) is maximum or minimum, x ≥ –1, y ≥ –1.
respectively.
We have f(x) = f(y)
Case 1: When n(A ∩ B) is minimum, i.e. n(A ∩ B) = 0.
This is possible only when A ∩ B = φ. In this case, ⇒ (x + 1)2 – 1 = (y + 1)2 –1

n(A ∪ B) = n(A) + n(B) – 0 ⇒ x2 + 2x = y2 + 2y

= n(A) + n(B) = 3 + 6 = 9 ⇒ x2 – y2 = –2(x – y)

So, maximum number of elements in A ∪ B is 9. ⇒ (x – y)(x + y) = –2(x – y)

Case 2: When n(A ∩ B) is maximum. ⇒ (x – y) [x + y + 2] = 0

This is possible only when A ⊆ B. In this case, n(A ∩ B) ⇒ x – y or x + y + 2 = 0


=3 ⇒ x = y or x = y = –1
∴ n(A ∪ B) = n(A) + n(B) – n(A ∩ B) Thus, f(x) = f(y) ⇒ x = y for all
= (3 + 6 – 3) = 6 x ≥ –1, y ≥ –1
So, minimum number of elements in A ∪ B is 6. So, f(x) is an injection.
f is a surjection: For all y ≥ –1 there exists.
Example 6: If f, g, h are functions from R to R such that
x = –1 + y +1 –1 such that f(x) = y
f(x) = x2 – 1, g(x) = x2 + 1 , ≥
M a them a ti cs | 13.51

 π
So, f(x) is a surjection. = 2y– 3 = sin  π − 
 4
Hence, f is bijection. Consequently, it is invertible.
 π
f(x) = f–1(x) ⇒ f(x) = x But –1 ≤ sin  π −  1
 4

(x + 1)2 – 1 = x ⇒ x = 0, –1 ∴ –1 ≤ 2 – 3 ≤ 1
y

⇒ 2 ≤ 2y ≤ 4 ⇒ 2 ≤ 2y ≤ 22
Example 8: Show that the function
Hence y ∈ [1, 2]
f: R → R defined by f(x) = 3x3 + 5 for all x ∈ R is a
bijection. Hence range of f(x) is [1, 2]

Sol: Similar to the previous example. Example 10: Find the period of the following functions
Injectivity: Let x, y be any two elements of R (domain). (i) f(x) = tan 2x
Then, f(x) = f(y)
(ii) f(x) = sin4x + cos4x
⇒ 3x3 + 5 = 3y3 + 5; x3 = y3⇒ x = y
(iii) f(x) = x – [x] + | cos px |
Thus, f(x) = f(y)
+ | cos 2 px | + ….. + | cos npx |
⇒ x = y for all x, y ∈ R. so, f is an injective map. Surjectivity:
Let y be an arbitrary element of R(co-domain). Sol: Proceed according to the section 15.5.
Then, f(x) = y (i) f(x) = tan 2x has period π /2as tan x has period π.
1/3
y −5  y −5
⇒3x3 + 5 = y ⇒ x3 = ;x=   (ii) f(x) = sin4x + cos4x
3  3 
= (sin2x + cos2x)2 – 2 sin2x cos2x
Thus we find that for all y ∈ R (co-domain) there exists
 y −5
1/3 1 1 3 1
=1– sin2 2x= 1 – (1 – cos 4x) = – cos 4x
x =   ∈ R (domain) such that f(x) = 2 4 4 4
 3 
3 Since cos x has period 2π,
  y − 5 1/3   y − 5 1/3 
f 
=   3    +5 2π π
 3    3   ∴ cos 4x has period =
    4 2.
=y–5+5=y 3 1 π
f(x) = + cos 4x has period
∴ 4 4 2.
This shows that every element in the co-domain has its
pre-image in the domain. So, f is a surjection. Hence, f (iii) x – [x] has period 1
is a bijection. | cos x | has period p
π
Example 9: Find the range of the following functions: | cos px | has period =1
π
 sinx − cos x + 3 2  π 1
f(x) = log2   | cos 2px | has period =
 2  2π 2
 
π 1
| cos npx | has period =
Sol: Put f(x) = y and simplify. Then use the range of the nπ n
trigonometric function to find the answer.
 1 1 1
 sinx − cos x + 3 2  l.c.m. 1, 1, , , ....,  = 1
⇒ f(x) = log2    2 3 n
 2 
  ∴ f(x) has period 1.
  π 
= log2  sin  π −  + 3  = y(let)
  4  Example 11: Find the domain of the following functions
 π
∵ 2y = sin  π −  + 3 (i) f(x) = x −1 + 6 − x
 4
(ii) f(x) = log1/2 (x2 + 4x + 3)
1 3 . 5 2 | Relations and Functions

1 Here, x2 – 3x + 4 = x ⇒ (x – 2)2 = 0
(iii) f(x) =
| x | −x x = 2 is the solution for the equation.

Sol: Use the fact that the quantity inside the square Example 13: Draw the graph of following function and
root should be positive. For (ii), use the condition for also find their domain and range.
logarithm to be defined.
(x + 2)(x − 1)
(i) x − 1 is defined only if x – 1 ≥ 0 (i) f(x) =
x(x + 1)
6 − x is defined only if 6 – x ≥ 0
x2 − 5x + 4
∴ f(x) is defined. (ii) f(x) =
x2 + 2x − 3
∀ x ∈ {x : x – 1 ≥ 0} ∩ {x : 6 – x ≥ 0}
Domain of f is = [1, 6] Sol: Use the method of curve tracing.
(x + 2)(x − 1)
(ii) f(x) = log1/2 (x2 + 4x + 3) (i) f(x) = , domain of f(x) is
x(x + 1)
f is defined if x2 + 4x + 3 > 0
x ∈ R – {–2, 1}
i.e. if (x + 1) (x + 3) > 0
2(2x + 1)
f’(x) =
i.e. if x > – 1 or x < –3 [x(x + 1)]2
Domain of f is (–∞, –3) ∪ (–1, ∞)
f’(x) = 0 at x = –1/2
(iii) f is defined if | x | – x > 0
Here, f(–∞) = 1– = f(+∞)
If x ≥ 0 then | x | = x
f(–1–) = –∞ =f(0–)f(–1+) = +∞ = f(0+)
If x < 0 then | x | = – x > 0 and | x | > x.
Range is (–∞, 1) ∪ (9, ∞)
∴Domain of f = {x: x < 0}
x2 − 5x + 4 (x − 4)(x − 1)
(ii) f(x) = =
Example 12: A function 2
x + 2x − 3 (x + 3)(x − 1)

3  7  Domain of f(x) is R – {1, –3}


f:  , ∞  →  , ∞  defined as
2  4  y
f(x) = x – 3x + 4. Solve the equation
2
1
f(x) = f–1(x).
-3 -3 1 4 x
Sol: It is equivalent to solving for f(x) = x.
4 -1
3 
Domain of f(x) is  , ∞  and its co-domain gives as
2 
7 .
 , ∞
4  f’(x) >0 ⇒ f(–∞) = 1 = f(+∞)

We can plot the graph of f(x) for the above domain as f(–3–) = +¥ ⇒ f(–3+) = –∞
 3 
f(4) = 0 Range is R – − , 1
 4 
y
7
(0, 9)
4 3 (0, 1) y=1
x
2
-1
-2 -1/2 0 (1, 0)
For solving f(x) = f–1(x), we can solve f(x) = x or f–1(x) = x -
Any one of the above equation can be solved depending
on the fact that easiest equation is given priority.
M a them a ti cs | 13.53

JEE Main/Boards

Exercise 1 1 + x 
Q.3 If f(x) = log   , prove that
1 − x 
Sets and Relations
 2x 
f   = 2f(x).
 1 + x2 
Q.1 If R be a relation and N defined by x + 2y = 8 then
find the domain of R. x2 + 2x + 1
Q.4 Find the domain of the function f(x) =
x2 − 8x + 12
Q.2 R is a relation from {11, 12, 13} to {8, 10, 12} defined
by y = x – 3 then find R–1. Q.5 Find the domain of definition of the function
1
y= + x+2
Q.3 Let A be the set of all students of a boy’s school. log10 (1 − x)
Let relation R in set A is given by R = {(a, b) ∈ A × A is
a sister of b}. Can we say that R is an empty relation? x
Give reason. Q. 6 Find the range of the function y = .
1 + x2

Q.4 Let A = {1, 2} and B = {1, 3} and R be a relation from


Q.7 Find the domain and range of the function
set A to set B defined as R = {(1, 1), (1, 3), (2, 1), (2, 3)}.
Is R a universal relation? Explain.  1  
 x, 2
: x ∈ R, x ≠ ±1   .
 1 − x  
Q.5 Let R be a relation in the set of natural numbers
N, defined by R = {(a, b) ∈ N × N : a < b}. Is relation R
Q.8 Find the domain and range of the function
reflexive? Explain.
1
y= .
Q.6 Let A be any non-empty set and P(A) be the power 2 − sin3x
set of A. A relation R defined on P(A) by X R Y ⇔ X ∩ Y
= X, Y ∈ P(A). Examine whether R is symmetric. Q.9 If f: R → R is defined by f(x) = x3 + 1 and g : R → R
f
is defined by g(x) = x + 1, then find f + g, f – g, f · g,
Q.7 Let A = {a, b, c} and R is a relation in A given by R = g
and α f(a ∈ R).
{(a, a), (a, b), (a, c), (b, a), (c, c)}. Is R symmetric? Explain.

Q.8 Given a relation R = {(yellow, black), (cat, dog), (red, Q.10 Let f: R → R is defined by f(x) = x and g : R → R is
green)}. White R–1. defined by g(x) = | x |. Find
(i) f + g (ii) f – g
Q.9 Let A = {1, 3, 5}, B = {9, 11} and let R = {(a, b)∈ A ×
(iii) f · g (iv) a f, a ∈ R
B: a – b is odd}. Write the relation r.
(v) f/g
Q.10 Let A = {a, b, c} and relation R in the set A be given
by R = {(a, c), (c, a)}. Is relation R symmetric? Explain. Q.11 Let f be the exponential function and g be
the logarithmic function defined by f(x) = ex and
Functions g(x) = logex. Find
2 tanx π (i) (f + g) (1) (ii) (f – g) (1) (iii) (f · g) (1)
Q.1 If f(x) = , then find f   .
4
2
1 + tan x
Q.12 If f(x) = cos[p2]x + cos[–p2]x, where [x] denotes the
|x|
Q.2 If f(x) = , x ≠ 0, prove that π
x integral part of x, write the value of f   .
| f(α) – f(–α)| = 2 α, a ≠ 0. 2
1 3 . 5 4 | Relations and Functions

Q.13 ({x}, represents fractional part function) (i) f(x) = x2 (ii) g(x) = | x |
(i) Domain of the function 1
(iii) h(x) = (iv) u(x) = 4 − x2
1 3 − x2
f(x) = ln (1 – {x}) + sinx + + 4 − x2 is _______.
2 Q.22 Consider the following rules:
(ii) Range of the function cos(2 sin x) is ______.
(i) f: R → R : f(x) = logex
(iii) Period of the function
(ii) g: R → R : g(x) = x
 πx  1
f(x) = sin   + {x} + tan2(px) is _______. (iii) h: A → R : h(x) = , where A = R – {–2, 2}
 3 
2
x −4
Which of them are functions? Also find their range, if
Q.14 Let A = {1, 2, 3}, B = {3, 6, 9, 10}. Which of the they are function.
following relations are functions from A to B? Also find
their range if they are function.
x2 − 4
f = {(1, 9), (2, 3), (3, 10)} Q.23 Let f: R – {2} → R be defined by f(x) =
x−2
g = {(1, 6), (2, 10), (3, 9), (1, 3)} and g: R → R be defined by g(x) = x + 2. Find whether
h = {(2, 6), (3, 9)} f = g or not.
u = {(x, y): y = 3x, x ∈ A}
Q.24 Let f = {(1, 1), (2, 3), (0, –1), (–1, –3)} be a linear
Q.15 Let A = {a, b, c, d}. Examine which of the following function from Z into Z and g(x) = x. Find f + g.
relation is a function on A?
Q.25 Find f + g, f – g, f · g, f/g and a f (a ∈ R) if
(i) f = {(a, a), (b, c), (c, d), (d, c)}
1
(ii) g = {(a, c), (b, d), (b, c)} (i) f(x) = , x ≠ –4 and g(x) = (x + 4)3
x+4
(iii) h = {(b, c), (d, a), (a, a)}
(ii) f(x) = cos x, g(x) = ex.

Q.16 (i) Let f = {(1, 1), (2, 3), (0, –1),


Q.26 If f(x) = x, g(x) = | x |, find (f+g)(–2),
(–1, –3)} be a function from Z to Z defined by f
f(x) = ax + b for some integers a, b determine a and b. (f – g) (2), (f·g)(2),   (–2), and 5f(2).
g
(ii) Let f = {(1, 1), (2, 3), (0, –1), (–1, –3)} be a linear
function from Z to Z, find f(x). Q.27 Define the function f: R → R by y = f(x) =x2.
Complete the table given below:
Q.17 Function f is given by f = {(4, 2),
(9, 1), (6, 1), (10, 3)}. Find the domain and range of f. x –4 –3 –2 –1 0 1 2 3 4
y=f(x)=x2
Q.18 If A = {(–3, –2, –1, 0, 1, 2, 3)} and f(x) = x2 – 1
defines f: A → R. Then find range of f. Q.28 Define the real valued function on
f: R – {0} → R as f(x) = 1/x
Q.19 Find the domain and range of the following
functions. Complete the figure given below :

(i) f(x) = x (ii) f(x) = 2 – 3x x –2 –1.5 –1 –0.5 0 1 1 2 2


(iii) f(x) = x – 1
2
(iv) f(x) = x + 2
2
(v) f(x) = x −1
1
2
y=f(x)=
x + 3x + 5 x1
Q.20 Find the domain of the function f(x) =
x2 − 5x + 4
Find the domain and range of f.
Q.21 Find the domain of the definition and range of the
function defined by the rules: Q.29 If f(x + 3) = x2 – 1, write the expression for f(x).
M a them a ti cs | 13.55

Exercise 2 Q.7 If A = {2, 3} and B = {1, 2}, then A × B is equal to


(A) {(2, 1), (2, 2), (3, 1), (3, 2)}
Sets and Relations
(B) {(1, 2), (1, 3), (2, 2), (2, 3)}
Single Correct Choice Type
(C) {(2, 1), (3, 2)}
Q.1 Let A = {1, 2, 3, 4}, and let R = {(2, 2),(3, 3),(4, 4), (D) {(1, 2), (2, 3)}
(1, 2)} be a relation on A. Then R is
(A) Reflexive (B) Symmetric Q.8 If A = {1, 2, 3} and B = {1, 4, 6, 9} and R is a relation
from A to B defined by ‘x is greater than y’. The range
(C) Transitive (D) None of these
of R is

Q.2 The void relation on a set A is (A) {1, 4, 6, 9} (B) {4, 6, 9}

(A) Reflexive (C) {1} (D) None of these

(B) Symmetric and transitive


Q.9 Let R = {(1, 3), (4, 2), (2, 4),(2, 3), (3, 1)} be a relation
(C) Reflexive and symmetric on the set A = {1, 2, 3, 4}. The relation R is
(D) Reflexive and transitive
(A) Transitive (B) Not symmetric
(C) Reflexive (D) A function
Q.3 For real number x and y, we write x R y ⇔ x – y + is
an irrational number. Then the relation R is
Q.10 If A, B and C are these sets such that A ∩B = A ∩
(A) Reflexive (B) Symmetric C and A ∪ B = A ∪ C, then
(C) Transitive (D) None of these (A) A = B (B) A = C
(C) B = C (D) A ∩ B = f
Q.4 Let R be a relation in N defined by
R = {(1 + x, 1 + x2) : x ≤ 5, x ∈ N}. Q.11 The relation R = {(1, 1), (2, 2), (3, 3), (1, 2), (2, 3),
Which of the following is false (1, 3)} on the set A = {1, 2, 3} is
(A) R = {(2, 2), (3, 5), (4, 10), (5, 17), (6, 25)} (A) Reflexive but not symmetric
(B) Domain of R = {2, 3, 4, 5, 6} (B) Reflexive but not transitive
(C) Range of R = {2, 5, 10, 17, 26} (C) Symmetric and transitive
(D) Neither symmetric nor transitive
(D) None of these

Q.12 Let A be the set of all children in the world and R


Q.5 The relation R = {(1, 1), (2, 2), (3, 3), (1, 2), (2, 3), be a relation in A defined by x R y if x and y have same
(1, 3)} on the set A = {1, 2, 3} is sex. Then R is
(A) Reflexive but not symmetric (A) Not reflexive (B) Not symmetric
(B) Reflexive but not transitive (C) Not transitive (D) An equivalence relation
(C) Symmetric and transitive
Q.13 Let A = {2, 3, 4, 5} and R = {(2 2), (3, 3), (4, 4),
(D) Neither symmetric nor transitive
(5, 5), (2, 3), (3, 2), (3, 5), (5, 3)} be a relation on A. Then R is

Q.6 Let A = {2, 3, 4, 5} and let R = {(2, 2), (3, 3), (4, 4), (5, (A) Reflexive and transitive
5), (2, 3), (3, 2), (3, 5), (5, 3)} be a relation in A. Then R is (B) Reflexive and symmetric
(A) Reflexive and transitive (C) An equivalence relation
(B) Reflexive and symmetric (D) None of these
(C) Reflexive and anti-symmetric
(D) None of these
1 3 . 5 6 | Relations and Functions

Q.14 Let L be the set of all straight lines in the xy-plane. (B)
y
Two lines l1 and l2 are said to be related by the relation
R if l1 is parallel to l2. Then the relation R is
(A) Reflexive (B) Symmetric (B) a x
O b
(C) Transitive (D) Equivalence

Q.15 Given the relation R = {(2, 3), (3, 4)} on the set
{2, 3, 4}. The number of minimum number of ordered
pair to be added to R so that R is reflexive and symmetric (C) y
(A) 4 (B) 5 (C) 7 (D) 6

Q.16 The minimum number of elements that must (C) -a a x


-b O b
be added to the relation R = {(1, 2),(2, 3)} on the set
{1, 2, 3}, so that it is equivalence is
(A) 4 (B) 7 (C) 6 (D) 5
(D) y
Functions

Single Correct Choice Type (D) x


-a a
O
Q.1 If f(x+ay, x–ay) = ay then f(x, y) is equal to:

x2 − y 2 x2 + y 2
(A) (B)
4 4
(C) 4xy (D) None of these 1  | sinx | sinx 
Q.4 Period of the function f(x) =  +  is
2  cos x | cos x | 
Q.2 The set of values of ‘a’ for which f: R → R f(x) = ax + (A)π /2 (B) p
cos x is bijective is
(C) 2 p (D) 4p
(A) [–1, 1] (B) R – {–1, 1}
(C) R – (–1, 1) (D) R – {0} Q.5 Let f : R → R be a function defined by
2x2 − x + 5
Q.3 The graph of function f(x) is as shown, adjacently f(x) = then f is
7x2 + 2x + 10
(A) One–one but not onto
y=f(x) (B) Onto but not one–one
(C) Onto as well as one–one
a b
(D) Neither onto nor one–one

1  1 
1 Q.6 If f(x) = cos  π2  x + sin  π2  x, [x] denoting the
Then the graph of is 2  2 
f(| x |)
greatest integer function, then
(A) y
π 1
(A) f(0) = 0 (B) f   =
3 4
(A) a b π
O
x (C) f   = 1 (D) f(π) = 0
2
M a them a ti cs | 13.57

x 4 − x3 + 3x2 − 2x + 2 Q.6 If g{f(x)} = | sin x| and f{g(x)} = (sin x )2, then


Q.7 Let f(x)= ln x and g(x) = .
2
2x − 2x + 1  (1998)
The domain of the composite function fog(x) is
(A) f(x) = sin2x, g(x) = x
(A) (–∞, ∞) (B) (0, ∞)
(B) f(x) = sinx, g(x) = | x |
(C) (0, ∞) (D) (1, ∞)
(C) f(x) = x2, g(x) = sin x

Previous Years’ Questions (D) f and g cannot be determined

Q.1 Let f(x) = | x – 1 |. Then,  (1983) Q.7 If f(x) = 3x – 5, then f–1(x)  (1998)
(A) f(x ) = [f(x)]
2 2
1
(A) Is given by
(B) f(x + y) = f(x) + f(y) 3x − 5
x +5
(C) f(| x |) = | f(x) | (B) Is given by
3
(D) None of the above
(C) Does not exist because f is not one–one
1 1 x  (D) Does not exist because f is not onto
Q.2 If f(x) = cos(log x), then f(x).f(y) – f   + f(xy)
2 2 y 
has the value (1983) Q.8 If the function f : [1, ∞) → [1, ∞) is defined by
1 f(x) = 2x(x–1), then f–1(x) is  (1999)
(A) –1 (B)
2
x(x −1)
1 1
(C) –2 (D) None of these (A)   (B) (1 + 1 + 4log2 x )
2 2

Q.3 The domain of definition of the function (1983) 1


(C) (1 − 1 + 4log2 x ) (D) Not defined
1 2
y= + x + 2 is
log10 (1 − x)
Q.9 Let f(θ) = sin θ (sin θ + sin 3θ). Then f(θ)  (2000)
(A) (–3,–2) excluding –2.5
(A) ≥ 0 only when q≥ 0 (B) ≤ 0 for all real q
(B) [0, 1] excluding 0.5
(C) ≥ 0 for all real q (D) ≤ 0 only when θ≤ 0
(C) (–2, 1) excluding 0
(D) None of the above Q.10 The domain of definition of the function y(x) is
given by the equation 2x + 2y = 2, is  (2000)
Q.4 Which of the following functions is periodic? (1983) (A) 0 < x ≤ 1 (B) 0 ≤ x ≤ 1
(A) f(x) = x – [x] where [x] denotes the greatest integer (C) –∞< x ≤ 0 (D) –∞< x < 1
less than or equal to the real number x
1 Q.11 Let f :N → Y be a function defined as f (x) = 4x +
(B) f(x) = sin for x 0, f(0) = 0
x
(C)f(x) = x cos x
3, where Y = {y ∈ N : y = 4x + 3 for some x ∈ N} . Show
that f is invertible and its inverse is  (2008)
(D) None of the above
3y + 4 y +3
(A) g ( y ) = (B) g ( y )= 4 +
(x − a)(x − b) 3 4
Q.5 For real x, the function will assume all
(x − c) y +3 y −3
real values provided  (1984) (C) g ( y ) = (D) g ( y ) =
4 4
(A) a > b > c (B) a < b < c
(C) a > c < b (D) a ≤ c ≤ b Q.12 For real x, let f ( x ) = x3 + 5x + 1 , then (2009)
(A) f is one-one but not onto R
(B) f is onto R but not one-one
1 3 . 5 8 | Relations and Functions

(C) f is one-one and onto R Q.16 The domain of the function


(D) f is neither one-one nor onto R 1
f (x) = is  (2011)
Statement-I (assertion) and statement-II (reason). x −x

Each of these questions also have four alternative (A) ( 0,∞ ) (B) ( −∞ ,0 )
choices, only one of which is the correct answer. You
have to select the correct choice (C) ( −∞ , ∞ ) − {0} (D) ( −∞ , ∞ )

Q.13 Let f ( x )= ( x + 1)
2
− 1, x ≥ − 1
Q.17 If f :R → R is a function defined by
Statement-I : The set
 2x − 1 
f ( x )  x  cos 
{ x : f ( x= }
) f −1 ( x )= {0, − 1}
=
 2 
π,

where [x] denotes the greatest integer function, then f


Statement-2 : f is a bijection.  (2009) is (2012)
(A) Statement-I is true, statement-II is true; statement-II (A) Continuous for every real x
is a correct explanation for statement-I
(B) Discontinuous only at x = 0
(B) Statement-I is true, statement-II is true; statement-II
(C) Discontinuous only at non-zero integral values of x
is nota correct explanation for statement-I
(D) Continuous only at x = 0
(C) Statement-I is true, statement-II is false
(D) Statement-I is false, statement-II is true
Q.18 Consider the function
f ( x ) = x − 2 + x − 5 , x ∈ R . (2012)
Q.14 Let f ( x ) = x x and g ( x ) = sinx  (2009)
Statement-I : gof is differentiable at x = 0 and its Statement-I: f’(4) = 0
derivative is continuous at that point. Statement-II : gof
Statement-II: f is continuous in [2, 5], differentiable in
is twice differentiable at x = 0.
(2, 5) and f(2) = f(5).
(A) Statement-I is true, statement-II is true; statement-II
(A) Statement-I is false, statement-II is true
is a correct explanation for statement-I
(B) Statement-I is true, statement-II is true; statement-II
(B) Statement-I is true, statement-II is true; statement-II
is a correct explanation for statement-I
is nota correct explanation for statement-I
(C) Statement-I is true, statement-II is true; statement-II
(C) Statement-I is true, statement-II is false
is not a correct explanation for statement-I
(D) Statement-I is false, statement-II is true
(D) Statement-I is true, statement-II is false

Q.15 Consider the following relations:


Q.19 If a ∈ R and the equation
( x, y ) | x, are real numbers and x 
( ) ( )
2
R=  −3 x −  x  + 2 x −  x  + a2 =
0
= wy for some rational number w 
(where [x] denotes the greatest integer ≤ x ) has no
 m p  m,n, p and q are integers such that n,  integral solution, then all possible values of a lie in the
S =  ,  
 n q  q ≠ 0 and qm = pn  interval (2014)

Then  (2010) (A) (-2, -1) (B) ( − ∞ , − 2 ) ∪ ( 2, ∞ )


(A) Neither R nor S is an equivalence relation
(C) ( −1, 0 ) ∪ ( 0, 1 ) (D) (1, 2)
(B) S is an equivalence relation but R is not an
equivalence relation
(C) R and S both are equivalence relations
(D) R is an equivalence relation but S is not an
equivalence relation
M a them a ti cs | 13.59

fk ( x )
Q.20 Let=
1
k
(
sink x + cosk x ) where x ∈ R and 1
Q.21 If f ( x ) + 2f   =3x, x ≠ 0 , and
x
k ≥ 1 . Then f4 ( x ) − f6 ( x ) equals. (2014)
S= {
x ∈R : f ( x ) = }
f ( −x ) ; then S:  (2016)
1 1 1 1 (A) Contains exactly one element
(A) (B) (C) (D)
4 12 6 3
(B) Contains exactly two elements.
(C) Contains more than two elements.
(D) Is an empty set.

JEE Advanced/Boards

Exercise 1 Q.11 Two finite sets have m and n elements respectively.


The total number of subsets of first set is 56 more than
Sets and Relations the total number of subsets of the second set. Calculate
the values of m and n
Q.1 Is set a collection of objects or a collection of well-
defined objects which are distinct and distinguishable? Q.12 If A = {x | x/2 ∈ Z, 0 ≤ x ≤ 10}. B = { x | x is one digit
prime} C= {x | x/3 ∈ N, x ≤ 12}, Then what is the value
Q.2 Is the set { x : x ∈ N, x is prime and 3 < x < 5} is void of A ∩ (B ∪ C) ?
or non-void?
Q.13 If n(A) = 10, n(B) = 15 and n(A ∪ B) = x, then what
Q.3 A = {a, e, i, o, u} and B = {i, o} then A ⊂ B or B ⊂ A? is the range of x?

Q.4 A set is defined as A = {x : x is irrational and 0.1 < x Q.14 Among 1000 families of a city, 40% read newspaper
< 0.101} then comment on whether A is null set or A is A, 20% read newspaper B, 10% read newspaper C, 5%
finite set or A is infinite set. read both A and B, 3% read both B and C, 4% read A
and C and 2% read all three newspapers. What is the
number of families which read only newspaper A?
Q.5 Which of these: f, {}, {2, 3} and {f} is a singleton Set?

Q.15 If for three disjoint sets A, B, C; n(A) = 10, n(B) =


Q.6 Two points A and B in a plane are related if
6 and n(C) = 5, then what is the value of n(A ∪ B ∪ C) ?
OA = OB, where O is a fixed point. Then comment
whether this relation is reflexive, symmetric, transitive
or equivalence? Q.16 If A and B are disjoint, then what is the value of
n(A ∪ B)?
Q.7 If A = {2, 3} and B = {-2, 3}, then what is the value
of A ∪ B? Q.17 If X and Y are two sets, then what is the value of
X ∩ (Y ∪ X)c ?
Q.8 Given the sets A = {1, 2, 3}, B = {3, 4}, C = {4, 5, 6},
then what is the value of A ∪(B ∩ C)? Q.18 Let n(U) = 700, n(A) = 200, n(B) = 300 and n(A ∩
B) = 100, then what is the value of n(Ac∩Bc)?
Q.9 If Na = {an : n ∈ N}, then what is the value of N6∩
N 8? Q.19 What is the value of set (A ∩Bc)c∪ (B ∩ C) ?

Q.10 Is it true that both I = { x : x ∈ R and x2 + x + 1 = 0}, Q.20 Sets A and B have 3 and 6 elements respectively.
II = { x : x ∈ R and x2– x + 1 = 0} are empty sets? What can be the minimum number of elements in A ∪ B?
1 3 . 6 0 | Relations and Functions

Q.21 In a class of 100 students, 55 students have passed


in Mathematics and 67 students have passed in physics, 1 − 5x
(iv) f(x) =
no student fails. Calculate the number of student who 7− x − 7
have passed in Physics only?
1
(v) y = + x+2
log10 (1 − x)
Q.22 Let X = {1, 2, 3, 4, 5, 6} be an universal set. Sets A,
B, C in the universal set X be defined by A = {1, 2, 3}, B
= {2, 4,5} and C = {3, 4, 5, 6}, then what is the value of (vi) f(x) = log100x  2log10 x + 1 
 −x 
(A – B) ∪ (B – A), (A – B) – C and A ∩ C’?  
1
(vii) f(x) = y = + x+2
Q.23 If A, B and C are any three sets, then is A × (B ∪ C) log10 (1 − x)
equal to (A × B) ∪ (A × C) or (A × B) ∩(A × C)?
 5x − x2 
(viii) y = log10  
Q.24 If A, B and C are any three sets, then is A × (B ∩ C)  4 
 
equal to (A × B) ∩ (A ×C) or (A ∩ B) × (A ∩C)?
1
(ix) f(x) = x2 − | x | +
Q.25Let A = {a, b, c, d}, B = {b, c, d, e}. Then what is the 9 − x2
value of [(A × B) ∩ (B × A)]?
(x) f(x) = (x2 − 3x − 10)  n2 (x − 3)
Q.26 In the set A= {1, 2, 3, 4, 5}, a relation R is defined
by R = {(x, y) | x, y ∈ A and x < y}. Then is R reflexive or (xi) f(x) = (sin x + cos x)2 − 1
transitive or symmetric?
cos x − (1 / 2)
(xii) f(x) =
Q.27 Let R be a relation on the set N of natural numbers 6 + 35x − 6x2
defined by nRm ⇔ is a factor of m (i.e., n | m). Then is
R symmetric? (xiii) f(x) = log1/3 (log4 ([x]2 − 5))

Q.28 If R is a relation from a finite set A having m 1 1


(xiv) f(x) = + log(2|x|−5) (x2 − 3 x + 10) +
elements to a finite set B having n elements, then what [x] 1− | x |
will be the number of relations from A to B ?
(xv) f(x) = log7log5log3log2(2x3 + 5x2 – 14x)
Q.29 Let L denote the set of all straight lines in a plane.
Let a relation R be defined by α R β ⇔ α ⊥ β, α, b∈ L. (xvi) f(x) = cos2x + 16 − x2
Then is R symmetric?
(xvii) f(x) = ln ( x2 − 5x − 24 − x − 2)
Functions
(xvii) y = log10 (1 – log10 (x2 – 5x +16))
Q.1 Find the domain of the definitions of the following  2 x +1
functions: (Read the symbol [*] and {*} as greatest (xix) f(x) = log4  2 − 4 x − 
 x + 2 
integers and fractional part functions respectively.) 

x−2 1−x
(i) y = + Q.2 Find the domain and range of the following
x+2 1+x
functions. (Read the symbols [*] and {*} as greatest
1 integers and fractional part function respectively)
(ii) y = x2 − 3x + 2 +
3 + 2x − x2
(i) y = log 5 ( 2(sinx − cosx) + 3 )
1
(iii) y = x + 3 – log10 (2x – 3) 2x
x−2 (ii) y =
1 + x2
M a them a ti cs | 13.61

x2 − 3x + 2 (1 + 2x )2
(iii) f(x) = (h) f(x) =
x2 + x − 6 2x
x x x
(iv) f(x) = (i) f(x) = + +1
1+ | x | x
e −1 2

(v) y = 2−x + 1+x ( j) f(x) = [(x + 1)2]1/3 + [(x – 1)2]1/3

(vi) f(x) = log(cosec x – 1)(2 – [sin x] – [sin x]2) Q.8 Find the period for each of the following functions:
x +1 (a) f(x) = sin4x + cos4x
(vii) f(x) =
x−2 (b) f(x) = | sin x | + | cos x |
3 2
Q.3 Classify the following functions f(x) defined in (c) f(x) = cos x – sin x
R → R as injective, surjective, both or none. 5 7

x2 + x + 1 Q.9 Write explicitly, function of y defined by the


(a) f(x) =
x2 + 2x + 3 following equations and also find the domains of
definition of the given implicit functions:
(b) f(x) = (x2 + 5x + 9) (x2 + 5x + 1)
(a) 10x + 10y = 10 (b) x + | y | = 2y
1
Q.4 Let f(x) = . Let f2(x) denote f[f(x)] and f3(x)
1−x Q.10 Find out for what integral values of n the number
denotes f[f{f(x)}]. Find f3n(x) where n is a natural number. 3π is a period of the functions: f(x) = cosnx · sin(5/n)x.
Also state the domain of this composite function.
Q.11 Compute the inverse of the functions:
x
Q.5 The function f(x) is defined as follows : on each of
(a) f(x) = ln (x + x2 + 1 ) (b) f(x) = 2 x −1
the intervals n ≤ x < n + 1, where n is a positive integer,
 1 10 x − 10− x
f(x) varies linearly, and f(n) = –1, f  n +  = 0. Draw the (c) y =
 2
graph of the function. 10 x + 10− x

n
Q.12 Show if f(x) = a − xn , x > 0, n ≥ 2, n ∈N, then
Q.6 (a) For what values of x is the inequality |f(x) + ϕ (x)|
(fof) (x) = x. Find also the inverse of f(x).
< |f(x)| + |φ(x)| true if, f(x) = x – 3, and ϕ(x) = 4 – x.
(b) For what values of x is the inequality | f(x) –ϕ (x)| > |
f(x) | – | φ(x) | true if, f(x) = x, and ϕ (x) = x – 2.  1   9 
Q.13 f :  − , ∞  →  − , ∞  ,
 2   4 
Q.7 Find whether the following functions are even or Defined as f(x) = x + x – 2. Find f–1(x) and solve the
2

odd or none: equation f(x) = f–1(x).


(a) f(x) = log(x + 1 + x2 )
(ax + 1) Q.14 f(x) is defined for x <0 as
(b) f(x) =
x
a −1
x2 + 1 x < −1
(c) f(x) = x4 – 2x2 f(x) = 
3
 − x −1 ≤ x < 0
(d) f(x) = x2 – | x |
Define f(x) for x ≥ 0, if f is
(e) f(x) = x sin2x – x3 (a) Odd (b) Even
(f) f(x) = K, where K is constant
1 − x 
(g) f(x) = ln  
1 + x 
1 3 . 6 2 | Relations and Functions

 1 Q.4 The relation R defined in A = {1, 2, 3} by aRb if | a2


Q.15 If f(x) = max  x,  for x > 0 where max(a, b) – b2 | ≤ 5. Which of the following is false
 x
(A) R = {(1, 1), (2, 2), (3, 3), (2, 1), (1, 2), (2, 3), (3, 2)}
denotes the greater of the two real numbers a and b.
1 (B) R–1 = R
Define the function g(x) = f(x) f   and plot its graph.
x (C) Domain of R = {1, 2, 3}
(D) Range of R = {5}
x2 + 2x + c
Q.16 Show that the function f(x) = attains
x2 + 4x + 3c
any real value if 0 < c ≤ 1. Q.5 Let a relation R in the set N of natural numbers be
defined as (x, y) ∈ R if and only if x2 – 4xy + 3y2 = 0 for
Q.17 (a) Find the domain and range of the function all x, y ∈N. The relation R is
(A) Reflexive
f(x) = log2 (x2 − 2 x + 2)
(B) Symmetric
2x + 3 (C) Transitive
(b) f : R – {2} → R – {2} ; f(x) = , find whether f(x)
is bijective or not. x−2
(D) An equivalence relation
x(x −2)+1
Q.18 Prove that function f(x) = 1 + 2 4 is many Q.6 Let R {(3, 3), (6, 6), (9, 9), (12, 12),
one. (6, 12), (3, 9), (3, 12), (3, 6)} be a relation on the
set A = {3, 6, 9, 12}. The relation is

Exercise 2 (A) An equivalence relation


(B) Reflexive and symmetric only
Sets and Relations
(C) Reflexive and transitive only
Single Correct Choice Type (D) Reflexive only

Q.1 Let R be a relation on the set N of natural numbers


defined by nRm⇔ n is a factor of m (i.e. n | m). Then R is Q.7 Let R = {(1, 3), (4, 2), (2, 4), (2, 3), (3, 1)} be a relation
on the set A = {1, 2, 3, 4}. The relation R is
(A) Reflexive and symmetric
(A) Reflexive (B) Transitive
(B) Transitive and symmetric
(C) Not symmetric (D) A function
(C) Equivalence
(D) Reflexive, transitive but not symmetric Q.8 Let N denote the set of all natural numbers and R
be the relation on N × N defined by (a, b) R (c, d) if ad(b
Q.2 Let R be a relation defined in the set of real numbers + c) = bc(a + d), then R is
by a R b ⇔ 1 + ab> 0. Then R is (A) Symmetric only
(A) Equivalence relation (B) Reflexive only
(B) Transitive (C) Transitive only
(C) Symmetric (D) An equivalence relation
(D) Anti–symmetric
Q.9 Let W denote the words in the English dictionary.
Q.3 Which one of the following relations on R is Define the relation R by R = {(x, y) ∈ W × W | the words
equivalence relation x and y have at least one letter in common}. Then R is

(A) x R1 y ⇔ | x | = | y | (A) Reflexive, symmetric and not transitive

(B) x R2 y ⇔x ≥ y (B) Reflexive, symmetric and transitive


(C) Reflexive, not symmetric and transitive
(C) x R3 y ⇔ x | y
(D) Not reflexive, symmetric and transitive
(D) x R4 y ⇔ x < y
M a them a ti cs | 13.63

Q.10 Let R be the real line. Consider the following Q.6 The value of x in [–2π, 2p], for which the graph of
subsets of the plane R × R :
1 + sinx
the function y = – sec x and
S = {(x, y) : y = x + 1 and 0 < x < 2} 1 − sinx
T = {(x, y) : x – y is an integer} 1 − sinx
y=– + sec x, coincide are
Which one of the following is true? 1 + sinx

(A) Both Sand T are equivalence relations on R  3π   π π   3π 


(A)  −2π, −  ∪  − ,  ∪  , 2π 
(B) S is an equivalence relation on R but T is not  2   2 2  2 
(C) T is an equivalence relation on R but S is not  3π π   π 3π 
(B)  − , −  ∪  , − 
(D) Neither S nor T is an equivalence relation on R  2 2 2 2 

(C) [–2 π, 2p]


Multiple Correct Choice Type
 π 3π 
(D) [–2 π, 2p] – ± , ± π, ± 
Q.11 Let X = {1, 2, 3, 4, 5} and Y = {1, 3, 5, 7, 9}. Which  2 2
of the following is/are relations from X to Y
(A) R1 = {(x, y) | y = 2+ x, x ∈ X, y ∈ Y} Q.7 The period of the function
(B) R2 = {(1, 1),(2, 1), (3, 3), (4, 3), (5, 5)}  x
f(x) = sin  cos  + cos(sin x) equal
(C) R3 = {(1, 1),(1, 3), (3, 5), (3, 7), (5, 7)}  2 
π
(D) R4 = {(1, 3),(2, 5), (2, 4), (7, 9)} (A) (B) 2π (C) p (D) 4p
2
x
Functions Q.8 Let f : R → R f(x) = . Then f(x) is
1+ | x |

Q.1 Domain of the function, (A) Injective but not surjective


2 1 (B) Surjective but not injective
f(x) = sin–1 x − x + sec–1  x  + ln x is
  (C) Injective as well as surjective
(A) [0, 1) (B) (0, 1] (C) (0, 1) (D) None of these (D) Neither injective nor surjective

Q.2 In the square ABCD with side AB = 2, two points M Multiple Correct Choice Type
and N are on the adjacent sides of the square such that
MN is parallel to the diagonal BD. If x is the distance
Q.9 Let f : I → R (where I is the set of positive integers)
of MN from the vertex A and f(x) = Area (∆AMN) then
range of f(x) is be a function defined by, f(x) = x , then f is

(A) (0, 2] (B) (0, 2] (C) (0, 2 2 ] (D) (0, 2 3 ] (A) One–one (B) Many one
(C) Onto (D) Into
Q.3 If ‘f’ and ‘g’ are bijective functions and gof is defined
the, gofis : Q.10 The function f(x) = log x is defined for x
x2
(A) Injective (B) Surjective belonging to
(C) Bijective (D) Into only (A) (–∞, 0) (B) (0, 1) (C) (1, ∞) (D) (0, ∞)

Q.4 If y = 5[x] + 1 = 6[x – 1] – 10, where [.] denotes the


x x − 2 x −1
greatest integer function, then [x + 2y] is equal to Q.11 If f(x) = , then
x −1 −1
(A) 76 (B) 61 (C) 107 (D) 189
(A) f(x) = –x if x < 2
Q.5 f : R → R, f(x) = ax3 + ex is one–one onto, then ‘a’ (B) 2f(1.5) + f(3) is non negative integer
belongs to the interval
(C) f(x) = x if x > 2
(A) (–∞, 0) (B) (–∞, 0] (C) [0, ∞) (D) (0, ∞) (D) None
1 3 . 6 4 | Relations and Functions

Q.12 Which of the following function(s) is(are) bounded 2x − 1


Q.2 If S is the set of all real x such that is
on the intervals as indicated 2x + 3x2 + x
3
1
positive, then S contains  (1986)
(A) f(x) = 2 x −1 on (0, 1)
1  3  3 1
(B) g(x) = x cos on (–∞, ∞) (A)  −∞ , −  (B)  − , − 
x  2  2 4
(C) h(x) = xe–x on (0, ∞)  1 1 1 
(C)  − ,  (D)  , 3 
(D)  (x) = arc tan 2 on (–∞, ∞)
x  4 2 2 

Q.13 Which of the following function(s) is/are periodic? Q.3 Let g(x) be a function defined on [–1, 1]. If the area
of the equilateral triangle with two of its vertices at
(A) f(x) – x – [x]
(0, 0) and [x, g(x)] is 3 / 4 , then the function g(x) is
(B) g(x) = sin(1/x), x ≠ 0 and g(0) = 0  (1989)
(C) h(x) = x cos x (A) g(x) = ± 1 − x2 (B) g(x) = 1 − x2
(D) w(x) = (sin x)
(C) g(x) = – 1 − x2 (D) g(x) = 1 + x2

Q.14 On the interval [0, 1], f(x) is defined as,


 x Q.4 If f(x) = cos[p2] x +cos [–p2]x, where [x] stands for
if x ∈ Q
f(x) =  . the greatest integer function, then  (1991)
1 − x if x ∉ Q
Then for all x ∈ R the composite function f[f(x)] is π
(A) f   = –1 (B) f(π) = 1
2
(A) A constant function
π
(B) An identity function (C) f(–π) = 0 (D) f   =1
4
(C) An odd linear polynomial
b−x
Q.5 Let f : (0, 1)→ R be defined by f(x) = , where
(D) 1 + x 1 − bx
b is a constant such that 0 < b < 1. Then,  (2014)
Q.15 Identify the pair(x) of functions which are identical. (A) f is not invertible on (0, 1)
1
1 − x2 (B) f ≠ f–1 on (0, 1) and f’(b) =
(A) y = tan (cos–1 x) : y = f '(0)
x
1
(B) y = tan (cos–1) : y = 1/x (C) f = f–1 on (0, 1)and f’(b) =
f '(0)
x (D) f–1 is differentiable on (0, 1)
(C) y = sin (arc tan x) : y =
2
1+x
(D) y = cos (arc tan x) : y = sin (arc cot x) Match the Columns

Q.6 Match the condition/expression in column I with
Previous Years’ Questions statement in column II
Let the functions defined in column I have domain
x+2  π π
Q.1 If y = f(x) = , then  (1984) (1992)
x −1  − ,  and range (–∞, ∞) 
 2 2
(A) x = f(y)
(B) f(1) = 3
Column I Column II
(C) y increases with x for x < 1
(A) 1 + 2x (p) Onto but not one–one
(D) f is a rational function of x (B) tan x (q) One–one but not onto
(r) One–one and onto
(s) Neither one–one nor onto
M a them a ti cs | 13.65

Q.7 Let f(x) =


x2 − 6x + 5
2
 (2007) ) x3 + ex/2 and g ( x ) = f −1 ( x ) ,
Q.10 If the function f ( x=
x − 5x + 6
then the value of g' (1 ) is  (2009)
Column I Column II
Q.11 Let f be a real-valued function defined on the
(A) If –1< x < 1, then f(x) satisfies (p) 0< f(x)< 1 x
interval (−1, 1) such that e− x f ( x ) =
2 + ∫ t 4 + 1dt , for
(B) If 1 < x < 2, then f(x) satisfies (q) f(x) < 0
0
(C) If 3 < x < 5, then f(x) satisfies (r) f(x) > 0 all x ∈ ( −1, 1 ) and f −1 be the inverse function of f. Then

(D) If x > 5, then f(x) satisfies (s) f(x) < 1 ( f ) ' (2) is equal to
−1
(2010)

(A) 1 (B) 1/3 (C) 1/2 (D) 1/e


Q.8 Match the statements/expressions in column I with
the values given in column II. (2009) Q.12 For any real number, let [x] denote the largest
integer less than or equal to x. Let f be a real valued
Column I Column II function defined on the interval [-10, 10] by

(A) T
 he number of solutions of the equation (p) 1  x −  x  if  x  is odd,
f (x) = 
0 in the interval  0, 
xesin x − cos x =
 π 1 +  x  − x if  x  is even
 2
10
π
(B) Value(s) of k for which the planes (q) 2
kx + 4y + z = 0, 4x + ky + 2z = 0 and
Then the value of
10 ∫ f ( x ) cos π x dx is  (2010)
−10
2x + 2y + z = 0 intersect in a straight
line   ˆ
(C) Value(s) of k for which |x – 1| + |x – 2| (r) 3 Q.13 Let a =−ˆi − k, i 2 ˆj + 3kˆ be three
ˆ bˆ =− ˆi + j and c =+
+ |x + 1| + |x + 2| = 4k has integer     
given vectors. If r is a vector such that r × b = c × b and
solution(s)   
r. a = 0 , then the value of r.b is  (2011)
(D) If y’ = y + 1 and y(0) = 1 then value(s) (s) 4
of y(ln2)
π π  x∈
(t) 5 Q.14 Let f ( x ) = sin  sin  sinx   for all R and
6 2 
π
g(x) = sin x for all x∈
R. Let (f o g)(x) denote f(g(x))
Q.9 Match the statements/expressions in column I with 2
the values given in column II.  (2009) and (g o f)(x) denote g(f(x)). Then which of the following
is (are) true?  (2015)
Column I Column II  1 1
(A) Range of f is  − , 
(A) R
 oot(s) of the expression 2sin2θ +sin2 θ − 2 (p)
π  2 2
6
 1 1
(B) Points of discontinuity of the function (B) Range of f o g is  − , 
(q)
π  2 2
 6x   3x  4 f (x) π
f ( x ) =   cos   , where [y]
π π (C) lim =
x →0 g(x) 6
denotes the largest integer less than or
equal to y (D) There is an x∈
R such that (g o f)(x) = 1
(C) Volume of the parallelopiped with
π
its edges represented by the vectors (r)
3
ˆi + ˆj , ˆi + 2 ˆj and ˆi + ˆj + π kˆ
 
(D) Angle between vectors a and b where π
   (s)
a,b and c are unit vectors satisfying 2
   
a+ b+ 3 c =0
(t) π
1 3 . 6 6 | Relations and Functions

Q.15 Match the statements given in column I with the intervals/union of intervals given in column II (2011)

Column I Column II

  2iz  
(A) The set Re 
2 
z 1, z ≠ 1 is
: z is a complex number,= (p) ( − ∞, − 1) ∪ (1, ∞ )
  1 − z  

 8 3 x −2 
(B) The domain of the function f ( x ) = sin−1 
()  is (q) ( − ∞, 0 ) ∪ ( 0, ∞ )
 
1−3 ( )
2 x −1
 

(C) 1 tan θ 1 (r) 2, ∞ )


 π
If f ( θ ) =− tan θ 1 tan θ , then the set f ( θ ) :0 ≤ θ <  is
 2
−1 − tan θ 1

(D) If f ( x ) = x3/2 ( 3x − 10 ) , x ≥ 0 , then f (x) is increasing in (s) ( − ∞, − 1 ∪ 1, ∞ )


(t) ( − ∞,0 ∪ 2, ∞ )

Q.16 The function f : 0,3 → 1,29  , defined by


Q.20 Let f1 : R → R,f2 : 0, ∞ ) → R,f3 : R → R and
f ( x ) = 2x3 − 15x2 + 36 + 1 , is (2012)
f4 : R → 0, ∞ ) be defined by (2014)
(A) One–one and onto (B) Onto but not one–one
| x | if x < 0 sin x if x < 0
; f2 ( x ) x2 ; f3 ( x ) = x
(C) One–one but not onto (D) Neither one–one nor onto
= f1 ( x ) = x if x ≥ 0
e if x ≥ 0 
Q.17 Consider the statements: and
P: There exists some x ∈IR such that f ( x ) + 2x = 2 1 + x2 ( ) (
f f ( x )
2 1 ) if x < 0
Q: There exists some x ∈IR such that 2f ( x ) +=
1 2x (1 + x ) f4 ( x ) = 
( ( ))
f2 f1 f1 ( x ) − 1 if x ≥ 0
Then  (2012)

(A) Both P and Q ate true (B) P is true and Q is true


List I List II
(C) P is false and Q is true (D) Both P and Q are false
(1) f4 is (p) Onto but not one-one

Q.18 Let f : (-1 , 1) → IR be such that (2) f3 is (q) N


 either continuous nor one-one
2  π  π π (3) f2 o f1 is (r) Differentiable but not one-one

= f ( cos 4 θ ) for θ∈ 0,  ∪  ,  .
2
2 − sec θ  4 4 2 (4) f2 is (s) Continuous and one-one
1
Then the value(s) of f   is (are)  (2012)
3 Codes:

1 2 3 4
Q.19 If the function e−1 f ( x ) assumes its minimum in
1 (A) r p s q
the interval [0, 1] at x = , which of the following is
4 (B) p r s q
true ? (2013) (C) r p q s
1 3 1 (D) p r q s
(A) f ' ( x ) < f ( x ) , <x< (B) f ' ( x ) > f ( x ) ,0 < x <
4 4 4
1 3
(C) f ' ( x ) < f ( x ) ,0 < x < (D) f ' ( x ) < f ( x ) , < x < 1
4 4
M a them a ti cs | 13.67

 π π Q.22 Let f :f R
:  → R be a continuous odd function,
Q.21 let f :  − ,  → R be given by
 2 2 1
which vanishes exactly at one point and f (1 ) = .
( )
3
f ( x ) − log ( sec x + tanx ) . Then  (2014) x 2
Suppose that F ( x ) = ∫ f ( t ) dt for all x ∈  −1,2 and
(A) f (x) is an odd function −1
x
F(x) 1
(B) f (x) is a one-one function G( x) = ∫ t f ( f ( t ) ) dt for all x∈ −1,2. If lim
x →1 G (x)
=
14
,
−1
(C) f (x) is an onto function 1
then the value of f   is  (2015)
(D) f (x) is an even function 2

PlancEssential Questions
JEE Main/Boards JEE Advanced/Boards
Exercise 1 Exercise 1
Sets and Relations Sets and Relations
Q.4 Q.11 Q.14 Q.16 Q.6 Q.11 Q.14 Q.21

Q.28 Q.30
Functions
Q.5 Q.13 Q.19 Q.22 Functions
Q.26 Q.28 Q.31 Q.9 Q.11 Q.15 Q.19

Exercise 2 Exercise 2
Sets and Relations
Sets and Relations
Q.1 Q.5 Q.9
Q.2 Q.4 Q.8

Functions Functions
Q.3 Q.6 Q.8 Q.11 Q.15

Previous Years’ Questions Previous Years’ Questions


Q.4 Q.10 Q.5 Q.7
1 3 . 6 8 | Relations and Functions

Answer Key

JEE Main/Boards
Exercise 1
Sets and Relations
Q.1 {2, 4, 6} Q.2 {(8, 11), (10, 13)}
Q.3 Empty relation Q.4 yes
Q.5 No Q.6 yes
Q.7 No Q.8 R–1 = {(black, yellow), (dog, cat), (green, red)}
Q.9 Empty relation Q.10 Yes

Functions
Q.1 1 Q.2 2 α, a ≠ 0 Q.3 2f(x)
Q.4 f = R– {–2, –6}
 1 1
Q.5 [–2, 0) ∪ (0, 1) Q.6  − , 
 2 2

1 
Q.7 x ∈ R – {–1, 1} ; y ∈ (–∞, 0) ∪ [1, ∞) Q.8  , 1
3 
Q.9 αx2 + α
 0, x ≤ 0 2x, x ≤ 0
Q.10 (i)  (ii) 
2x, x ≥ 0  0, x ≥ 0

−x , x ≤ 0
2
(iii)  (iv) ax
2
 x , x ≥ 0

f f(x) x −1, x < 0


(v)   (x) = = = 
g g(x) |x|  1, x > 0
Q.11 (i) e    (ii) e    (iii) 0

 −π 
Q.12 ‒1 Q13 (i) x∈  , 2 (ii) [cos 2, 1] (iii) 6
6 

Q.14 f, u are function g, h are not function. Range f = {3, 9, 10}, Range u = (3, 6, 9)
Q.15 Only f is a function from A to A
Q.16 (i) a = 2, b = –1 (ii) f(x) = 2x – 1
Q.17 Domain = {4, 6, 9, 10}. Range = {1, 2, 3}
Q.18 Range f = {–1, 0, 3, 8}
Q.19 (i) Dom. f = R; Range of f = R (ii) Dom. f = R; Range of f = R
(iii) Dom f = R; Range of f = [1, ∞) = {x : x ≥ –1} (iv) Dom f = R; Range of f = [2, ∞) = {x : x ≥ 2}
(v) Dom. f = [1, ∞); Range f = [0, ∞) = {x : x ≥ 0}
M a them a ti cs | 13.69

Q.20 Domain of f = R – {1, 4}


Q.21 (i) Dom. f = R, Range f = [0, ∞) , (ii) Dom. g = R, Range g = [0, ∞)

(iii) Dom h = R – { − 3, 3} , Range h = (–∞,0) ∪ [1/3, ∞), (iv) Dom u = [–2, 2], Range u = [0, 2]

Q.22 f and g are not functions as they are not defined form negative values of x. h is function. Range
 1
h =  −∞ , −  ∪ [0, ∞).
 4

Q.23 f ≠ g as dom. f ≠ dom. g.

Q.24 f + g = {(1, 2), (2, 5), (0, –1), (–1, –4)}

1 + (x + 4)4 1 − (x + 4)4 1 α
Q.25 (i) ; ; (x + 4)2; ; x ≠ –4; (ii) cos x + ex; cos x – ex; ex cos x; ex cos x; a cos x
x+4 x+4 (x + 4) 4 x +4

Q.26 0; 4; 4; –1; 10

Q.27

x –4 –3 –2 –1 0 1 2 3 4
y=f(x)=x2 16 9 4 1 0 1 4 9 16

Domain of f = {x : x ∈ R} = R
Range of f = {x : x ≥ 0, x ∈ R} = [0, ∞)
Graph of y = f(x) i.e., y = x2 is as shown in the following figure.

Q.28 Domain f = R – {0}, Range f = R – {0}

Q.29 f(x) = (x – 3)2 – 1

Exercise 2

Sets and Relation


Q.1 C Q.2 B Q.3 B Q.4 A Q.5 A Q.6 B

Q.7 A Q.8 C Q.9 B Q.10 C Q.11 A Q.12 D


Q.13 B Q.14 D Q.15 B Q.16 B

Functions
Q.1 A Q.2 C Q.3 C Q.4 C Q.5 D Q.6 C Q.7 A

Previous Years’ Questions


Q.1 D Q.2 D Q.3 C Q.4 A Q.5 D Q.6 A

Q.7 B Q.8 B Q.9 C Q.10 D Q.11 D Q.12 C

Q.13 C Q.14 C Q.15 B Q.16 B Q.17 A Q.18 B

Q.19 C Q.20 B Q.21 B


1 3 . 7 0 | Relations and Functions

JEE Advanced/Boards
Exercise 1
Sets and relations
Q.8 {1, 2, 3, 4} Q.11 6, 3 Q.14 330 families Q.16 n (A ∪ B) = n(A) + n(B)

Q.25 9

Functions
Q.1 (i) defined no where (ii) –1 < x ≤ 1 and 2 ≤ x < 3
3
(iii) < x < 2 and 2 < x <¥ (iv) (–∞, –1) ∪ [0, ∞)
2
 1   1 1 
(v) –2 ≤ x < 0 and 0 < x < 1 (vi)  0, ∪ , 
 10   10 10 
(vii) (–2 ≤ x <1)-{0} (viii) 1 ≤ x ≤ 4
(ix) (–3, –1]∪ {0} ∪ [1, 3) (x) {4} ∪ [5, ∞)
  1 
(xi) nπ,  n +  π  , n ∈ I
  2 

 1 π   5π   π 5π 
(xii)  − ,  ∪  , 6  ∪ 2Kπ + , 2Kπ +  , k ∈ I – {0}
 6 3  3   3 3
 1
(xiii) [–3, –2) ∪ [3, 4) (xiv) f (xv)  −4, −  ∪ (2, ∞)
 2

 5π −3π   π π   3π 5π 
(xvi)  − ,  ∪ − ,  ∪  , 
 4 4   4 4  4 4 

(xvii) (–∞, –3] (xviii) 2 < x < 3 (xix) [0, 1)

Q.2 (i) D : x ∈ R, R : [0, 2]


(ii) D = R; range [–1, 1]
(iii) D: {x | x ∈ R; x ≠ –3; x ≠ 2}, R: {f(x) | f(x) ∈ R, f(x) ≠ 1/5: f(x) ≠ 1}
(iv) D:R;R: (–1, 1)
(v) D: –1 ≤ x ≤ 2 R: [ 3, 6]
 π π 5π 
(vi) D : x ∈ (2nπ, (2n + 1)π) – 2nπ + , 2nπ + , 2nπ + , n ∈ I  and R : loga2 : a ∈ (0, ∞) – {1} ⇒ Range is (–∞,∞) – {0}
 6 2 6 
(vii) x ∈ R – {2}, f(x) ∈ R – {1}

Q.3 (a) Neither surjective nor injective (b) Neither injective nor surjective

Q.4 f3n(x) = x; Domain = R – {0, 1}

Q.5 2 Q.6 (a) x < 3 or x > 4 (b) x < 2

Q.7 (a) Odd (b) Odd (c) Even (d) Even (e) Odd (f) Even (g) Odd (h) Even
(i) Neither odd nor even ( j) Even
M a them a ti cs | 13.71

Q.8 (a) π/2 (b) π/2 (c) 70p

Q.9 (a) y = log(10 – 10x), –∞< x < 1 (b) y = x/3 when –∞< x < 0 and y = x when 0 ≤ x < +∞

ex − e− x log2 x 1 1+x
Q.10 ±1, ±3, ±5, ±15 Q.11 (a) (b) (c) log
2 log2 x − 1 2 1−x

Q.12 f–1(x) = (a – xn)1/n Q.13 x = ± 2

 −(x2 + 1) x >1 (x2 + 1) x >1


Q.14 (a) f(x) =  (b) 
3 3
 x x ∈ (0, 1]  −x x ∈ (0, 1]

1
 if 0 < x ≤ 1
Q.15 g(0) =  x2
 x2 if x >1

Q.17 (a) Domain x ∈ R, Range [0, ∞) (b) yes

Exercise 2
Sets and Relations
Single Correct Choice Type

Q.1 D Q.2 C Q.3 A Q.4 D Q.5 A Q.6 C Q.7 C

Q.8 D Q.9 A Q.10 C

Multiple Correct Choice Type

Q.11 A, B, C

Functions
Single Correct Choice Type

Q.1 C Q.2 B Q.3 A Q.4 D Q.5 D Q.6 A Q.7 D

Q.8 A

Multiple Correct Choice Type

Q.9 A, D Q.10 B, C Q.11 B, C Q.12 A, C, D Q.13 A, D Q.14 B, C Q.15 A, C, D

Previous Years’ Questions


Q.1 A, D Q.2 A, D Q.3 B, C Q.4 A, C Q.5 A Q.6 A → q; B → r

Q.7 A → p; B → q; C → q; D → p Q.8 A→ p; B → q, s; C→ q, r, s, t; D → r

Q.9 A→ q, s; B→ p, r, s, t; C→ t; D→ r Q.11 B Q.12 D Q.14 A, B, C

Q.15 A → s; B → t; C→r; D → r Q.16 B Q.17 C Q.18 A, B


1 3 . 7 2 | Relations and Functions

Q.19 C Q.20 D Q.21 A, B, C


Solutions

JEE Main/Boards Sol 5: R = {(a, b) ∈ N × N: a < b}


Reflective relation → R: A → B is said to be reflective
Exercise 1 iff a R a ∀ a ∈ A (here A, B ⇒ N)
Sets and Relations R = {(a, a) ∈ N × N: a < a}
we know a < a is universal false
Sol 1: Relation R: x + 2y = 8 (defined in N)
so R is not a reflexive relation.
x = 8 – 2y
x, y ∈ N
Sol 6: P(A) ⇒ Power set of A
so for
X R Y ⇒ X ∩ Y = X , Y ∈ P(A)
=y 1=x 8 – 2(1)
= 6
 X R Y ↔ – X ∩ Y = X, Y
=y 2=x 8 – 2(2)
= 4  naturalnumbers
=y 3= = 2 
x 8 – 2(3) YRX ↔ Y ∩ X
We know X ∩ Y = Y ∩ X
y = 4 x = 8 – 2(4) = 0
so XRY → YRX
so domain of R = {2, 4, 6}
so R is symmetric.

Sol 2: Given relation R Sol 7: A = {a, b, c}


R: {11, 12, 13} → {8, 10, 12} and R = {(a, a),(a, b), (a,c),(b,a),(c, c)}
y=x–3 R–1 = {(a, a), (a, b),(c, a),(b, a),(c, c)}
for R–1 x = y + 3 R ≠ R–1
{8, 10, 12} → {11, 12, 13} so R is not symmetric relation.
for y=8x =10 + 3 = 11 Element of
y=10x =10 + 3 = 13 {11, 12, 13} Sol 8: R = {(yellow, black), (cat, dog), (red, green)}
y=12x =12 + 3 = 15 R→x→y
R–1 → {(8, 11), (10, 13)} R–1 → y → x
So R–1 = {(black, yellow), (dog, cat), (green, red)}
Sol 3: R = {(a, b) ∈ A × H: a is sister of b}
Domain and range both (a, b) are set {A (a, b) ∈ A × A} Sol 9: A = {1, 3, 5}, B = {9, 11}
And it is given that A is only boy’s school a and b both R = {(a, b) ∈ A × B: a – b is odd}
are boys. So It is not possible that a is sister of b.
A – b will be odd when a, b both are not even and not
odd.
Sol 4: Given A = {1, 2}, B = {1, 3}
In A, B all elements are odd
R: A → B {(1, 1),(1, 3),(2, 1),(2, 3)}
so (a – b) ∈ π
Universal relation: Each element A B
of A is related to every element R is empty relation.
1 1
of B.
So R is universal relation.
2 2 Sol 10: A = {a, b, c}
R = {(a, c), (c, a)}
R–1 = {c, a},(a, c)
M a them a ti cs | 13.73

R ≡ R–1 relation R and R–1 both are same x + 2 is not defined for x + 2 < 0, x < −2
so R is symmetric.
log(1 − x) ≠ 0 ⇒ x ≠ 0

Functions So domain will be x ∈ ( −2,0)  (0,1)

2 tanx π 2 tan π / 4 x


Sol 1: f(x) = ⇒ f = Sol 6: Function y =
1 + tan x 2
 4  1 + tan2 ( π / 4) 1 + x2
1
y=
We know that tan (π/4) = 1 1 
 + x
π 2(1) x 
f  = 1
=
 4  1 + (1)
2
 1
 x +  is always greater than 2
 x
x
Sol 2: f(x) = From arithmetic mean > G.M
x 1
x+
If x > 0 f(x) = 1
For x > 0 ; x > x1 
 
−x 2 x
If x < 0; f(x) = = −1
x 1 1
For x < 0 ; x + < −2, x + > 2
Case I α > 0 x x
f(α ) =1 f( −α ) =− 1 −1 1
So range will be ≤y ≤
2 2
1 − ( −1) =2
 −1 1 
y∈ , 
Hence proved  2 2

1 + x  1
Sol 3: f(x) = log   Sol 7: f(x) = domain x ∈ R − { −1,1}
1 − x  1 − x2
2
 1 + x2 + 2x  1 + x 
= log   = log  x2 ≥ 0 ⇒ So the range y ∈ ( −∞ ,0) ∪ [1, ∞ )
 1 + x2 − 2x  
  1 − x  1
Sol 8: y = ⇒ 2 − sin3x ≠ 0
 2x  2 − sin3x
 2x  1 + 
f = log  1 + x2  = 2log  1 + x  = 2f(x) sin3x ≠ 2
2   2x 
1 + x  1 − x 
1 −  So the domain x ∈ R
 1 + x2 
x2 + 2x + 1 −1 ≤ sin3x ≤ 1
Sol 4: f(x) =
2
x + 8x + 12 1 
Range y ∈  ,1 
3 
x2 + 2x + 1 (x + 1)2
f(x) =
( ) 2
x2 + 8x + 16 − 16 + 12 (x + 4) − 4 Sol 9: f + g = x3 + 1 + x + 1 = x3 + x + 2 R → R
f − g = x3 + 1 − x − 1 = x3 − x R → R
(x + 4)2 − 4 ≠ 0
f.g= (x3 + 1)= x 4 + 1 + x + x3
(x + 4) ≠ ± 2
f x3 + 1
x ≠ − 2 and x ≠ − 6 = x ≠ −1
g x +1
Domain x ∈ R − { −2, −6}
α
= f ax2 + α
1
5: y
Sol= + x+2
log(1 − x) Sol 10: f : R → R f(x) = x
Logarithm is not defined for (1 − x) ≤ 0, x ≥ 1 g : R → R g(x) = x
1 3 . 7 4 | Relations and Functions

if x ≥ 0 g(x) = x y = cos(2sinx) ∈ [cos2,1]

 πx 
if x ≤ 0 g(x) = −x f(x) sin   + {x} + tan2 πx
(iii) =
 3 
2x, x > 0  0, x ≥ 0 πx
f +g= ; f −g = sin has time period = 6
  3
 0, x ≤ 0 2x, x ≤ 0
{ x } has time period = 1
 x2 x ≥ 0
f ⋅g =  2 tan2 πx has time period = 1
−x x ≤ 0
LCM (6,1,1) = 6
1 x≥0
f 
= −1 x≤0 Sol 14: f
g 
not defined x ≠ 0 A B
1 3
af = ax 6
2 9
3 10
Sol 11: f(x) = ex
Function
g(x) = loge x
Range {3, 9, 10}
(f + g)(1) =e(1) + loge(1) = e
A B
(f − g)(1) =e(1) − loge(1) = e 3
1
(1) 6
f.g(1)
= loge(1)
e= 0 g 2
9
3 10
22 2 2
Sol 12: π =
 9.8 ⇒ [ π ] =9 and [ −π ] =−10
not a function as 3 has two values in Range.
f(x)= cos9x + cos( −10)x= cos9x + cos10x h
A B
π π π
f  =
cos9   + cos10 =−1
2
  2
  2 1 3
2 6
1 3 9
Sol 13: (i) f(x)= n(1 − {x}) + sinx + + 4 − x2
2 10
1 − {x} > 0 (Always true)
1 not a function as 1 has no value
AND sinx + ≥0
2 u
1 A B
sinx ≥ −
2 1 3
 π 7π  2 6
x ∈ 2nπ − ,2nπ + 
 6 6  3 9
10
And 4 − x2 ≥ 0
x2 ≤ 4 putting x =1 y = 3
x ∈[ −2,2] x = 2y = 6

 −π  x = 3y = 3
So domain is x ∈  ,2
 6  So, it is a function and Range ∈ {3,6,9}
(ii) Range cos(2sinx)
−1 ≤ sin ≤ 1

−2 ≤ 2sinx ≤ 2
M a them a ti cs | 13.75

Sol 15: (i) Yes, it is because it cover all A and each x ∈ [1, ∞ )
corresponds to one value.
Range y ≥ 0 (Because value of under root
(ii) No, as it gives two value at x = b
(iii) No, as x=c has no image. is always non neg.) y∈ [0, ∞)

x2 + 3x + 5 x2 + 3x + 5
Sol 16: (i) f(x) = ax + b Sol
= 20: f(x) =
x2 − 5x + 4 (x − 4)(x − 1)
at x 1,=
= f(1) 1 (x − 4)(x − 1) ≠ 0
1= a + b  … (i) x ≠ 4 x ≠1
at x 2,=
= f(2) 3 Domain x ∈ R − {1, 4}

3 = 2a + b  … (ii) Sol 21: (i) Domain =R


Solving 1 & 2
Range y ∈ [0, ∞ )
a=2
(ii) Domain =R
b = −1
Range y ∈ [0, ∞ )
(ii) Let f(x)= a × +b
(iii) Domain 3 − x2 ≠ 0
Some question as part (i)
x≠± 3
So the answer will be f(x)
= 2x − 1
x ∈ R − { 3, − 3}
Sol 17: Domain = {4,9,6,10} Range x2 ≥ 0
Range = {2,1,3} 3 − x2 ≤ +3  1 
y ∈ ( −∞ ,0) ∪  , ∞ 
= x2 − 1
Sol 18: f(x) 3 

x2 ≥ 0 (iv)Domain 4 − x2 ≥ 0
f(0) =−1 = 4 ≥ x2
f(1)= 0= f( −1) x2 ≤ 4
f(2)= 3= f( −2) x ∈ [ −2,2]
f(3)= 8= f( −3) Range u(2) = 0
So the range will be { −1,0,3,8} u(0) = 2
y ∈ [0,2]
Sol 19: (i) Domain x ∈ R
Range y ∈ R Sol 22: (i) f : R → R : f(x) =
logx
(ii) Domain x ∈ R Not a function
Range y ∈ R Because f(x) is not defined for negative values of x.
(iii) Domain x ∈ R (ii) g : R → R : g(x) =
x
x2 ≥ 0 Not a function
So the range y ∈ [ −1, ∞ ) As f(x) is not defined for negative values.
(iv) Domain x ∈ R 1
(iii) h : A → R : h(x) ; A = R − { −2,2}
2
2
x ≥0 x −4
h(x) is defined for all values of x in A set.
So the range y ∈ [2, ∞ )
So it is a function.
(v) Domain x − 1 ≥ 0
x ≥1
1 3 . 7 6 | Relations and Functions

Sol 23: f : R − {2} → R f.g =


(2) (2)
= 2
4
x2 − 4 (x − 2)(x + 2) f
f(x) = = (x + 2) for x ≠ 2
= ( −2) =−1
x−2 (x − 2) g
g : R → R g(x) =
x + 2 at x ∈ R 5f(2)
= 5(2)
= 10
f≠g
As their domain are not same. Sol 27:

x –4 –3 –2 –1 0 1 2 3 4
Sol 24: f = a × +b (Suppose)
f(x) = x 16 9 4 1 0 1 4 9 16
2

f(1)=1=a+b  …(i)
f(2)=3= 2a + b  …(ii) Sol 28:
a = 2 b = −1
x –2 –15 –1 –0.5 0 0.5 1 1.5 2
f(x)
= 2x − 1
f(x) = 1/x –1/2 –2/3 –1 –2 X 2 1 2/3 1/2
g(x) = ×
f + g = 3x − 1 Domain R–{0}
at x = 1 (f + g)(1) =
2 Range ( −∞ ,0) ∪ (0, ∞ )
(f + g)(2) =
5
Sol 29: f(x + 3) = x2 − 1 = (x + 3)2 − 9 − 6x =
(f + g)(0) =
−1
(x + 3)2 − 6(x + 3) + 8 − 10
(f + g)( −1) =4
So f + g [(1,2)(2,5)(0, −1)( −1, −4)}
=

1
Sol 25: (i) f=
+g
x+4
+ (x + 4)3 x ≠ −4 Exercise 2
1 Sets and Relations
f=
−g − (x + 4)3 x ≠ −4
x+4
Single Correct Choice Type
f.g= (x + 4)2 ; x ≠ −4
Sol 1: (C) A = {1, 2, 3, 4}
f 1
= ; x ≠ −4 R = {(2, 2), (3, 3), (4, 4), (1, 2)}
g (x + 4)4
→ A = {1, 2, 3, 4}
(ii) f(x) cos
= = x g(x) ex
and (1, 1) ∉ R so R is not reflexive
g cos x + ex
f += → (1, 2) ∈ R but (2, 1) ∉ R
x
f −=
g cos x − e So R is not symmetric
f.g = ex (cos x) → (1,2) ∈ R, (2, 2) ∈ R
cos x (1, 2) ∈ R
f /g=
x
e So R is transitive.
αf =α cos x
Sol 2: (B) Void relation: A → A
Sol 26: f(x) = x g(x) = x
It is also called empty relation
If x ≥ 0 g(x) =
x
A relation R is void relation, if no element of set A is
If x < 0 g(x) =
−x related to any element of A.
(f + g)( −2) = x − x = 0 (x, y) ⇒ x is not related to y
(f − g)(2) = 2 (2) = 4 ∴ (y, x) ⇒ y is not related to x
M a them a ti cs | 13.77

∴ Symmetric R is not symmetric


(x, x) ⇒ x is any how will be related to itself → (1, 2) ∈ R and (2, 3) and also (1, 3) ∈ R
∴ So it can’t be reflexive So R is transitive.
(x, y)(y, z)
Sol 6: (B) R = {(2, 2), (3, 3), (4, 4), (5, 5), (2, 3), (3, 2), (3, 5),
⇒ x is not related to y
(5, 3)}
y is not related to z
in set A = {2, 3, 4, 5}
∴ This is transitive.
→ for set A = {2, 3, 4, 5}
(2, 2), (3, 3), (4, 4), (5, 5) ∈ R
Sol 3: (B) x R y ↔ x – y is an irrational number if
0 = x – x is an irrational no. (say z) R is reflexive
so R is not reflexive → (2, 3) (3, 2) ∈ R
→ if x – y is a irrational number and (3, 5) (5, 3) ∈ R
than y – x is also an irrational number R is symmetric (x, x) ∈ R
→ (x, y) ∈ R and (y, z) ∈ R So R is symmetric
say(z1) and say(z2) → (3, 5) ∈ R, (5, 3) ∈ R and also (3, 3) ∈ R
z 1 + z2 = x – y + y – z = x – z [(a, b) ∈ R , (b, c) ∈ R for transitive (a, c) ∈ R]
Its not necessary that z1 + z2 will be an irrational no. R so R is not transitive as (2, 3), (3, 5) ∈ R
is not transitive.
but (2, 5) ∉ R

Sol 4: (A) R = {(1 + x, 1 + x2): x ≤ 5, x ∈ N}


Sol 7: (A) A = {2, 3}B = {1, 2}
(A)R = {(2, 2), (3, 5), (4, 10), (5, 17), (6, 25)}
then A × B = {(2, 1) (2, 2), (3, 1)(3, 2)}
1+x =2 
(2, 2)→ x = 1 x ≤ 5 Sol 8: (C) A = {1, 2, 3}, B = {1, 4, 6, 9}
2
1+x = 2 
(x, y) ∈ R ∴ x > y
1+x =3 
(3, 5)→  x = 2, x ≤ 5 R: A → B
2
1+x = 5
⇒ 2 > 1, 3 > 1 ⇒ R = {(2, 1), (3, 1)}
1+x = 4  So range = {1}
(4, 10)→  x = 3, x ≤ 5
1 + x2 =10 
Sol 9: (B) R = {(1, 3), (4, 2), (2, 4), (2, 3), (3, 1)}
1+x = 5 
(5, 17)→  x = 4, x ≤ 5 on the set A = {1, 2, 3, 4}
1 + x2 =17 
→ for set A
1+x =6  (1, 1), (2, 2), (3, 3), (4, 4) ∉ R
(6, 25)→ 2  no solution
1+x = 25
R is not reflexive
Option A is false. → (2, 3) ∈ R but (3, 2) ∉ R
R is not symmetric
Sol 5: (A) R = {(1, 1), (2, 2), (3, 3),(1, 2), (2, 3), (1, 3)}
→ (1, 3) ∈ R, (3,1) ∈ R but
on the set A = {1, 2, 3}
(1, 1) ∉ R so
→ (1, 1), (2, 2), (3, 3) ∈ R
R is not transitive.
R is reflexive
→ (1, 2) ∈ R but (2, 1) ∉ R
1 3 . 7 8 | Relations and Functions

Sol 10: (C) of R


R is reflexive ⇒ (x, y) ∈ R
and (y, x) ∈ R
AB
A B
⇒ (2, 3) ∈ R ⇒ (3, 2) ∈ R
(5, 3) ∈ R ⇒ (3, 5) ∈ R
Given A ∩ B = A ∩ C  … (i) or say R–1 = R
and A ∪ B = A ∪ C … (ii) so R is symmetric.
we know that A ∪ B = A + B – A ∩ B
from (i) and (ii) Sol 14: (D) L = get of all times in x-y plane
R = {l1, l2 = l1 is parallel to l2}
A∪C=A+B–A∩C … (iii)
⇒ if (l1, l2) ∈ R
but A ∪ C = A + C – A ∩ C  … (iv)
so (l1, l2) ∈ R (l2 is also parallel to l1)
From (iii) – (iv)
symmetric
O=B–C
B=C
l1 l2
Sol 11: (A) R = {(1, 1), (2, 2), (3, 3), (1, 2), (2, 3), (1, 3)}
⇒ (l1, l2) ∈ R because a line is parallel to itself reflexive
Domain → {1, 2, 3}
⇒ (l1, l2) ∈ R, (l2, l3) ∈ R ⇒ (l1, l3) ∈ R
→ there are pairs such that
l1, l2 are parallel and (l2 and l3) are parallel, so l1 and l3 also
(a, a) → (1, 1),(2, 2), (3,3) are parallel R transitive
So R is reflexive → R–1 ≠ R
So R is not symmetric
l1 l2 l3
→ transitive if (x, y) ∈ R, (y, z) ∈ R
∴ R is equivalent
⇒ (x, z) ∈ R here (1, 2) ∈ R, (2, 3) ∈ R and (1, 3) is also
solution of R. Sol 15: (B) Relation R = {(2, 3), (3, 4)}
so R is transitive. on the set assume A = {2, 3, 4}
⇒ For reflexive → (x, x) ∈ R, x ∈ A
Sol 12: (D) A → set of all children in the world x R y if x
and y have same sex. (2, 2), (3, 3), (4, 4)
should be pair of R
So → x and x have same sex → Reflexive if x and y have 3pairs
same sex. So y and x also have same sex → symmetric
⇒ For symmetric → given (2, 3), (3, 4) ∈ R to be R →
x and y have same sex, y and z have same sex so it is
symmetric (3, 2) and (4, 3) should be pair of R
clear that x, y, z have same sex transitive
Total added pair is 5{(2, 2),(3, 3),(4, 4),(3, 2),(4,3)}
So R is an equivalence relation.

Sol 16: (B) R = {(1, 2), (2, 3)} on the set A (assume)
Sol 13: (B) A set A = {2, 3, 4, 5}
So A = {1, 2, 3}
Given relation R={(2, 2), (3, 3), (4, 4), (5, 5), (2, 3),(3, 2),
(3, 5) (5, 3)} → Reflexive (x, x) ∈ R, x ∈ A
⇒ if (x, y) ∈ R and x = y (1, 1), (2, 2), (3, 3) must be added
(x, y) ∈ R
→ symmetric (x, y) ∈ R for (y, x) ∈ R
Here x ∈ A
x = 2, 3, 4, 5 and (2, 2), (3, 3), (4, 4), (5, 5) are in solution (2, 1), (3, 2) must be added
M a them a ti cs | 13.79

→ Transitive (1, 2) ∈ R and (2, 3) ∈ R


1  sinx sinx 
Sol 4: =
(C) f(x)  + 
so (1, 3) must be added for transitive relation 2  cos x cos x 

If sinx > 0, cos > 0
Now → R = {(1, 1), (2, 2), (3, 3), (1, 2), (2, 3), (1, 3),(2, 1),
(3, 2)} F(x)=tanx

Now for symmetric pair (1, 3), pair (3, 1) must be added If sinx>0, cosx<0

Total added pairs → F(x) =0

7 {(1, 1)(2, 2)(3, 3)(2, 1)(3, 2)(1, 3) (3, 1)} If sinx<0, cosx<0 Time period = 2π

Functions F(x) = –tanx


If sinx<0, cosx>0
Single Correct Choice Type
F(x) = 0
Sol 1: (A) f(x + ay, x − ay) =×
a y
2x2 − x + 5
Put x + ay =
b Sol 5: (D) d(x) =
7x2 + 2x + 10
x − ay =
c
Not onto as range is not R. the function
2x= b + c
Does not goes to infinite at any x.
2ay= b − c
b−c 2 2 7 25 
y=  7x − x + 
2a 7 2 2 
f(x) =
 b + c  b − c  7x2 + 2x + 10
f(b,c) = a   
 2   2a  2 2 11 15 
 7x + 2x + 10 − x + 
2 2 7 2 2 
(b + c)(b − c) b − c =
=f(b,c) = 7x2 + 2x + 10
4 4

x2 − y 2 2 11x − 15
f(x, y) = = −
4 7 49x2 + 14x + 70

Quadratic cannot be one-one function


Sol 2: (C) f(x)
= ax + cos x
So f(x) is not one-one function
It is bijective so it must be strictly increasing or
decreasing 1  1 
= cos  π2  x + sin  π2  x
Sol 6: (C) f(x)
f ' (x) = a − sinx 2  2 
π 2
a ≤ −1 or a ≥ 1  =4
 2 
x ∈ R − ( −1 , 1)
f(x) cos 4x + sin 4x
=
Sol 3: (C) f(x) f(0)=1

a π 1 3
f(x) f  = + ≠4
b ab 3
  2 2

π
f  = 1
2
1 a f( π) =1
)f x ) b a b
1 3 . 8 0 | Relations and Functions

x 4 − x3 + 3x2 − 2x + 2 Sol 4: (A) Clearly, f(x) = x – [x] = {x} which has period 1
(A) f(x) =
Sol 7:= nx & g(x)
1
2x2 − 2x + 1 and sin , x cos x are non–periodic function.
x
 x 4 − x3 + 3x2 − 2x + 2 
fog(x) = n   x2 − (a + b)x + ab
 2x2 − 2x + 1 
  Sol 5: (D) Let y =
x−c
 x 4 − x3 + 3x2 − 2x + 2  ⇒ yx – cy = x2 – (a + b)x + ab
= n  
 2x2 − 2x + 1  ⇒ x2 – (a + b + y)x + (ab + cy) = 0
 
2x2 − 2x + 1 ≠ 0 for real roots, D ≥ 0

 1 1
2 ⇒ (a + b + y)2 – 4(ab + cy) ≥ 0
⇒ 2 x −  + > 0
 2 2 ⇒ (a + b)2 + y2 + 2(a + b) y – 4ab – 4cy ≥ 0

Let say h(x) = x 4 − x3 + 3x2 − 2x + 2 ⇒ y2 + 2(a + b – 2c) y + (a – b)2 ≥ 0


By hit and trial method, h(x) has no root. It is always which is true for all real value of y.
≥ 0 so domain x ∈ ( −∞ , ∞ )
⇒D≤0
⇒ 4(a + b – 2c)2 – 4(a – b)2 ≤ 0
Previous Years’ Questions
⇒ (a + b – 2c + a – b)(a + b – 2c – a + b) ≤ 0
Sol 1: (D) Given, f(x) = | x – 1 | ⇒ (2a – 2c) (2b – 2c) ≤ 0
∴ f(x ) = | x – 1 | and {f(x)} = (x – 1)
2 2 2 2
⇒ (a – c) (b – c) ≤ 0
⇒ f(x2) ≠ {f(x)}2, hence (a) is false ⇒ (c – a) (c – b) ≤ 0

Also, f(x + y) = | x + y – 1 | and f(x) = | x – 1 |, f(y) = | ⇒ c must lie between a and b


y – 1| Also, i.e. a ≤ c ≤ b or b ≤ c ≤ a
⇒ f(x + y) ≠ f(x) = f(y), hence (b) is false.
Sol 6: (A) Let f(x) = sin2x and g(x) = x
f(| x | ) = || x | – 1 | = | f(x) | = || x – 1|| = | x – 1 |
Now, fog(x) = f[g(x)] = f( x ) = sin2 x
∴ f(| x |) ≠ | f(x) |, hence c is false.
and gof(x) = g[f(x)] = g(sin2 x) = sin2 x = | sin x |

Sol 2: (D) Given, f(x) = cos(log x) Again let f(x) = sin x, g(x) = | x |

1 x  fog(x) = f[g(x)] = f(| x |) = sin | x | ≠ (sin x )2


∴ f(x) . f(y) – f   + f(xy)
2 y  When f(x) = x2, g(x) = sin x
1
= cos(log x). cos (log y) – [cos (log x – log y) + cos sin
fog(x) = f[g(x)] = f(sin x −1 x +1
) = (sin x )2
(log x + log y)] 2
and (gof) (x) = g[f(x)] = g(x2) = sin x2
1
= cos (log x) . cos (log y) – [(2cos(log x) . cos (log y)] = sin | x | ≠ | sin x |
2
= cos(log x) . cos (log y) – cos (log x) . cos (log y) = 0 Therefore, (a) is the answer.

Sol 3: (C) For domain of y, Sol 7: (B) Given, f(x) = 3x – 5(given)


1– x >0, 1 – x ≠ 1and x + 2 > 0 Let y = f(x) = 3x – 5 ⇒ y + 5 = 3x
⇒ x < 1, x ≠ 0 and x > –2 y +5
⇒x=
3
⇒ –2 < x < 1 excluding 0 ⇒ x ∈ (–2, 1) – {0}
y +5 x +5
f–1(y) = ⇒ f–1(x) =
3 3
M a them a ti cs | 13.81

Sol 8: (B) Let y =2x(x – 1), where y ≥ 1 as x ≥ 1 y −3


Sol 11: (D) Function is increasing
= x = g( y )
4
Taking log2 on both sides, we get

log2y = log2 2x(x–1) Sol 12: (C) Given f ( x ) = x3 + 5x + 1


Now f ' ( x=
) 3x2 + > 0, ∀ x ∈ R
⇒ log2 y = x(x – 1)
∴ f ( x ) is strictly increasing function
⇒ x2 – x – log2 y = 0
∴ It is one-one
1 ± 1 + 4log2 y
⇒x= Clearly, f(x) is a continuous function and also increasing
2
on R,
For y ≥ 1, log2y ≥ 0 ⇒ 4 log2y ≥ 0 ⇒ 1 + 4 log2y ≥ 1
Lt f ( x ) = − ∞ and Lt f ( x ) = ∞
⇒ 1 + 4log2 y ≥ 1 x→− ∞ x→∞

∴ f ( x ) takes every value between − ∞ and ∞ .


⇒ – 1 + 4log2 y ≤ –1
Thus, f(x) is onto function.

⇒1– 1 + 4log2 y ≤ 0
Sol 13: (C) There is no information about co-domain
But x ≥ 1 therefore f(x) is not necessarily onto.

So, x = 1 – 1 + 4log2 y is not possible.


Sol 14: (C) f ( x ) = x x and g ( x ) = sin x
1
Therefore, we take x = (1 + 1 + 4log2 y )
− sin x
2
2 ,x<0
1
gof ( x ) sin
= = xx ( )
 2
⇒ f–1(y) = (1 + 1 + 4log2 y )  sinx ,x≥0
2
−2 x cos x2 ,x<0
1
⇒ f (x) = (1 + 1 + 4log2 x )
–1 ∴ ( gof ) ' ( x ) = 2
2  2 x cos x ,x≥0

Sol 9: (C) It is given, Clearly, L ( gof ) ' ( 0 )= 0= R ( gof ) ' ( 0 )

f(θ) = sin θ (sin θ + sin 3 θ) ∴ g of is differentiable at x = 0 and also its derivative is


continuous at x = 0
= (sin θ + 3 sin θ – 4 sin3 θ) sin q
−2cos x2 + 4x2 sinx2 , x<0
= (4 sin θ – 4 sin3 θ) sin q Now, ( gof ) " ( x ) = 
2 2 2
 2cos x − 4x sin x ,x≥0
= sin2 θ (4 – 4 sin2 θ)
∴ L ( gof ) " ( 0 ) =
−2 and R ( gof ) " ( 0 ) = 2
= 4sin2θ cos2θ = (2sin θ cos θ)2
= (sin 2 θ)2 ≥ 0 ∴ L ( gof ) " ( 0 ) ≠ R ( gof ) " ( 0 )

Which is true for all θ. ∴ gof ( x ) is not twice differentiable at x = 0.

Sol 10: (D) Given that 2x + 2y = 2 ∀ x, y ∈ R Sol 15: (B)

But 2x, 2y > 0, ∀ x, y ∈ R xRy need not implies yRx


m p
Therefore, 2x = 2 – 2y < 2 S: S ⇔ qm =
pn
s q
⇒ 0 < 2x < 2 m m
S reflexive
Taking log on both sides with base 2, we get n n

log2 0 < log2 2x < log2 2 m p p m


S ⇒ S symmetric
n q q n
⇒ –∞ < x < 1
1 3 . 8 2 | Relations and Functions

m p p r
S , S
n q q s
=
Sol 20: (B) fk ( x )
1
k
(sink x + cosk x )
⇒ qm
= pn, ps= rq f4 ( x ) − f6 ( x=
) 1
4
( ) (
1
sin4 x + cos4 x − sin6 x + cos6 x
6
)
rn transitive.
⇒ ms =
1 1
S is an equivalence relation. = 1 − 2sin2 x cos2 x  − 1 − 3sin2 x cos2 x 
4   6 

1 1 1
Sol 16: (B) = − =
4 6 12
1
⇒ x − x > 0 ⇒ x > x ⇒ x is negative
1
x −x Sol 21: (B) f ( x ) + 2f   =
3x
x
x ∈ ( − ∞ ,0 ) S : f(x) = f(–x)
1
 f ( x ) + 2f   =
3x  … (i)
Sol 17: (A) x
 2x − 1   1 1 1 3
f ( x )  x  cos  =
=  π  x  cos  x −  π x→ f   + 2f ( x ) =  … (ii)
 2   2 x x x

=  x  sin π x is continuous for every real x. 3 2


(1) – 2 × (2) −3f ( x ) =
3x − f ( x )= −x
x x
Now, f (x) = f (-x)
Sol 18: (B) f ( x ) =
7 − 2x; x < 2
2 2 4
=3 ; 2 ≤ x ≤ 5 ∴ − x =− + x = 2x
x x x
=2x − 7; x > 5 2
x ⇒ x=
= ± 2
x
f(x) is constant function in [2, 5]
Exactly two elements.
f is continuous in [2, 5] and differentiable in
(2, 5) and f(2) = f(5)
by Rolle’s theorem f’ (4) = 0 JEE Advanced/Boards
∴ Statement-II and statement-I both are true and
statement-II is correct explanation for statement-I. Exercise 1
Sets and Relations
Sol 19: (C) − {x} + 2 {x} + a2 =
2
0

Now, −3{x} + 2 {x}


2 Sol 1: Sets → a collection of well-defined objects which are
distinct and distinguishable

Sol 2: Set {x: x ∈ N, x is prime and 3 < x < 5} there is


only one natural number 4 which follow 3 < x < 5, but
4 is not a prime no. therefore the set is void.

1 Sol 3: A = {a, e, i, Q, 4}, B {i,Q}


2/3
So B is subset of A
1

Sol 4: A = { x: x is irrational and 0. 1 < x < 0. 101 between 0.1


and 0. 101 there is infinite number which are irrational
to have no integral roots 0 < a2 < 1 so A is infinite set}

∴ a ∈ ( −1, 0 ) ∪ ( 0, 1 )
M a them a ti cs | 13.83

Sol 5: Singleton set → also known as a unit set which is Sol 9: Na = {an: n ∈ N}
with exactly on element (i.e., {0}) {π} is singleton set
N6 ∩ N8 → those element which are common in both
sets N6 and N8
Sol 6: In a plane there are two points A and B such that
OA = OB, O is a fixed point It means elements which are divisible by 6 and 8 both.

→ if OA = OB (assume it is a relation R = {(A, B): ⇒ LCM (6, 8) = 24


OA = OB})
so N24 = N6 ∩ N8
(B, A) ∈ R R is symmetric
which are divisible by 24 (6 and 8 both)
→ OA = OA (always true)
so (A, A) ∈ R R is reflexive Sol 10: (A){x: x ∈ R and x2 + x + 1 = 0}
→ if (A, B) ∈ R⇒ OA = OB  …… (i) x2 + x +1 = 0 ⇒ (x2 + 2x + 1) – x = 0
and (B, C) ∈ R⇒ OB = OC …… (ii) (x + 1)2 – x = 0
from equation (i) and (ii)
(x + 1)2 = x
C no solution
A
(A) is empty set
o
(B){x: x ∈ R and x2 – x + 1 = 0}
B
x2 – x + 1 = 0
OA = OB = OC
⇒ OA = OC +1 ± (1)2 – 4(1)(1)
x= ax2+ bx + c = 0
(A, C) ∈ R ⇒ R is transitive 2(1)

⇒ R is an equivalence relation 1 ± –3 b ± b2 – 4ac


x= x=–
2 2a
A B
Sol 7: no solution

2 3 -2 (B) is empty set

(C) {x: x ∈ R and x2 + 2x + 1 ≤ 0}


A = {2, 3}; B = {– 2, 3}
x2 + 2x + 1
So A ∪ B = {–2, 2, 3}
(x + 1)2

Sol 8: A = {1, 2, 3}, B = {3, 4} Which is zero at (x + 1) = 0

C = {4, 3, 6} ⇒x=–1

{4 is common is both set B and C} So (C) is not empty set

B ∩ C = {4} (D) {x: x ∈ R and x2 – 2x + 1 ≥ 0

and A ∪ (B ∩ C) x2 – 2x + 1 ≥ 0

= {1, 2, 3} ∪ ({4}) (x – 2)2 ≥ 0

= {1, 2, 3, 4} (x – 2)2 is always greater than or equal to ‘0’

A B C (x – 2)2 ≥ 0 always true for any x ∈ R

1,2 3 -2 4 5,6
1 3 . 8 4 | Relations and Functions

Sol 11: Total no. of subsets of M is 56 more than the Then A ∪ B will have
total no. of subsets of N so we know no. of total subsets
→A∪B=A+B–A∩B
of any set A which have n element = 2n
so 2m – 2n = 56 n(A ∪ B) = n(A) + n(B) – n(A ∩ B) ‘

m and n are integer = 10 + 15 – n(A ∩ B)


64 is the just bigger no. in 2 terms after 56
n
= 25 – n(A ∩ B)
so assume n(A ∩ B) can be zero to n(A)
2m = 64 = 26
A B C
so 64 – 2n = 56
A
2n = 64 – 56 = 8 or
8 = 23
n = 3 which is integer n(A  B)=0 n(A  B)=n(A)
n(A ∩ B)=0 n(A ∩ B) = n(A)
(m, n) = (6, 3)
so 15 ≤ n(A ∪ B) ≤ 25
Sol 12: Given A = {x | x/2 ∈ z, 0 ≤ x ≤ 10}
Sol 14: Total set = 1000 families of a city
B = {x | x is one digit prime}
40% read newspaper A
C = {x|x/3 ∈ N, x ≤ 12}
20% read newspaper B
x
(A) ⇒ ∈ z,0 ≤ x ≤ 10 5% read newspaper both A and B(A∩B)
2
x = 2z, z is integer, 0 ≤ x ≤ 10 10% read newspaper C
3% read newspaper both B and C(B∩C)
x = 0, 2, 4, 6, 8, 10
4% read newspaper both A and C(A∩C)
(B) x is one digit prime no.
2% read newspaper both all (A∩B∩C)
X = 2, 3, 5, 7
A B
x
(C) ∈ N, x ≤ 12
3
2%
x = 3N, x ≤ 12
C
x = 3, 6, 9, 12

B ∪ C = {3, 6, 9, 12} ∪ {2, 3, 5, 7}


Families which read only news paper A
= {2, 3, 5, 6, 7, 9, 12} =A–A∩B–A∩C+A∩B∩C
A ∩ (B ∩ C) = {0, 2, 4, 6, 8, 10} 40 – 5 – 4 + 2 = 33 %

∩ {2, 3, 5, 6, 7, 9, 12} = {2, 6} 33% of 1000 = 330 families

Sol 13: Sol 15:


A B
A B

n(A) = 10 
n(A) = 10 
n(B) = 6  all are disjoint
n(B) = 15 n(C) = 5 
M a them a ti cs | 13.85

So n(A ∪ B ∪ C) = n(A) + n(B) + n(C) Sol 19:


= 10 + 6 + 5 = 21
Other terms terns zero because there is no joints A AB
between any two sets
C
Sol 16: A and B are disjoint
Let (A ∩ B) ∪ (B ∩ C)
A B
(A ∩ BC)C = (AC ∪ B)
so A ∩ B = f
⇒ (AC ∪ B) ∪ (B ∩ C)
n(A ∩ B) = 0
We know that
∴ n (A ∪ B) = n(A) + n(B)
(B ∩ C) ⊂ (B)

Sol 17: so (AC ∪ B) ∪ (B ∩ C) = AC ∪ B

X Y
S Sol 20:
XY

B
A
3 6

YX
Given n(A) = 3
X ∩ (Y ∪ X)C
n(B) = 6
so in (Y ∪ X)C, there is no elements of X
we know
so X ∩ (Y ∪ X)C is π
n(A ∪ B) = n(A) + n(B) – n(A ∩ B)

Sol 18: for minimize n(A ∪ B), n(A ∩ B) should be maximum


which can be n(A) (lower in both)
U A B so n(A ∪ B) = 3 + 6 – 3 = 6

Sol 21: Let → class of 100 students


55 students have passed in mathematics (M)
67 students have passed in physics (P)
n(U) = 700
no student fail
n(A) = 200
so n(M ∩ P) = – n(M ∪ P) + n(M) + n(P)
n(B) = 300, n (A ∩ B) = 100
= – 100 + 55 + 67 = 22
Ac ∩ B c
no of students which are passed in physics only
We know
= n(P) – n(M ∩ P) = 67 – 22 = 45
n(A ∪ B) = n(A) + n(B) – n(A ∩ B)

(A ∪ B) = AC ∩ BC Sol 22: X = {1, 2, 3, 4, 5, 6} universal set


So n (A ∪ B) =n(AC∩BC)=n(U)–n(A∪B) A = {1, 2, 3}

n(A ∪ B) = 200 + 300 – 100 = 400 B = {2, 4, 5}

n(AC ∩ BC) = 700 – 400 = 300 C = {3, 4, 5, 6}


1 3 . 8 6 | Relations and Functions

So A – B = {1, 2, 3} – {2, 4, 5} = {1, 3} so from equation (i) and (ii)


B – A = {2, 4, 5} – {1, 2, 3} = {4, 5} x<y<z
(A – B) ∪ (B – A) = {1, 3) ∪ {4, 5} = {1, 3, 4, 5} so x < z

(A – B) – C = {1, 3} – {3, 4, 5, 6} = {1} ∴ (x, z) ∈ R R is transitive

C’ = X – C = {1, 2}
Sol 27: n R m ↔ n is a factor or of m (i. e. n/m)
A ∩ C′ = {1, 2, 3} ∩ {1, 2} = {1, 2}
Same as exercise –III question III

Sol 23:
Sol 28: n(A) = m
A x (B  C) n(B) = n and R: A → B
then total no. of relations form A to B is
(2m)n = 2mn
(AxB)  (AxC)
Sol 29: L ⇒ set of all straight lines in a plane
A × (B ∪ C) ⇒ (A × B) ∪ (A × C)
Relation R → α R β ↔ α ^β, α, β ∈ L any line never
perpendicular to itself
Sol 24: A × (A ∩ C)
so (α, α) ∉ R α ⊥ α (false)
(A × B) ∩ (A × C)
R is not reflexive
Sol 25: A = {a, b, c, d} → if (α, β) ∈ R ⇒ α ⊥ b
B = {b, c, d , e} so β ⊥ a
for → (A × B) ∩ (B × A) → (β, α) ∈ R
A × B = {(ai, bi)} R is symmetric
= {(a, b)(a, c)(a, d)(a, e), (b, b)(b, c)(b, d) (b, e)(c, b)(c, c) 
(c, d)(c, e) (d, b)(d, c)(d, d) (d, e)}

B × A = {(b,a) (b,b) (b,c) (b,d) (c,a) (c,b) (c,c) (c,d) (d,a)
(d,b) (d,c) (d,d) (e,a) (e,b) (e,c) (e,d)}
(A × B) ∩ (B × A) = {(b,b) (b,c) (b,d) (c,b) (c,c) (c,d) (d,b) → if (α, β) ∈ R ⇒ α ⊥ b
(d,c) (d,d)}
and (β, γ) ∈ R ⇒ β ⊥ g
total elements = 9
so a||γ ⇒ (α ⊥ γ) → false
Sol 26: Given A = {1, 2, 3, 4, 5} ⇒ (α, γ) ∉ R R is not transitive.
R = {x, y}|x, y ∈ A and x < y|

X < x always false
so (x, x) ∉ R 

R is not reflexive → if (x, y) ∈ R ⇒ x < y


so y < x always false 
(y, x) ∉ R
R is not symmetric
⇒ if (x, y) ∈ R ⇒ x < y  ……. (i)
and (y, z) ∈ R ⇒ y < z  ……. (ii)
M a them a ti cs | 13.87

Functions 1 − 5x
(iv) y =
x−2 1−x 7− x − 7
Sol 1:=
(i) y +
x+2 1+x 1 − 5x
≥0
x + 2 ≠ 0 x ≠ −2 7− x − 7
5x & 7x are always greater than zero.
x + 1 ≠ 0 x ≠ −1
x−2 Case I 1 − 5x ≥ 0 & 7− x − 7 < 0
≥0
x+2
5x ≤ 1 & 7 − x > 7
Case I x − 2 ≥ 0 & x + 2 > 0
× ≤ 0 & × < −1
x ≥ 2 & x > −2 x ∈ [2, ∞]
x ∈ (–∞, –1)
Or Case II x − 2 ≤ 0 & X + 2 < 0
Or Case II 1 − 5x ≤ 0 & 7− x − 7 < 0
x ≤ 2 & x < −2
5x ≥ 1 & 7 − x < 7
x ∈ ( −∞ , −2)
x ≥ 0 & x ≥ −1
1−x
And ≥0
1+x x ∈ [ 0, ∞ )
Case II 1 − x ≥ 0 & 1 + x > 0 So the answer is x ∈ ( −∞ , −1) ∪ [0, ∞ )
x ≤ 1 & x > −1 ⇒ x ∈ ( −1,1] 1
(v) y
= + x+2
log10 (1 − x)
Or Case II (1 − x) ≤ 0 & 1 + x < 0
(1 − x) > 0 & 1 − x ≠ 1
x ≥ 1 & x > −1
x <1 x ≠0
So the range domain will be x ∈ ( −1,1] ∩ [2, ∞ )
And x + 2 > 0
x∈φ
x ≥ −2 x ∈ [ −2, ∞ )
1
(ii) y= x2 − 3x + 2 + So the domain x ∈ [ −2,1) − {0}
3 + 2x − x2
1  2log10 x + 1 
y= (x − 2)(x − 1) + (vi) f(x) = log100x  
 −x 
(3 + x)(x + 1)
100 x ≠ 0,1 100 x > 0
(x − 2)(x − 1) ≥ 0 ⇒ x ∈ ( −∞ ,1] ∪ [2, ∞ )
1
And (3 − x)(x + 1) > 0 ⇒×∈ ( −1,3) x ≠ 0, x >0
100
The domain x ∈ ( −1,1 ], ∪ [ 2,3 ) 2log10 x + 1
And >0
1 −x
(iii) y =x+ − log10 (2x − 3)
(x − 2)1/3 2log10 x + 1
<0
×≥0 x
x > 0 So
And x − 2 ≠ 0
2log10 x + 1 < 0
x≠2
1 1
And 2x − 3 > 0 log10 x < − ⇒ 0 <x<
2 10
3
x>  1   1 
2 So domain x ∈  0,  − 
3   10   10 
So the domain x ∈  , ∞  − {2}
2 
1 3 . 8 8 | Relations and Functions

1 (xi) f(x)= (sinx)2 + cos2 x + 2sinx cos x − 1


(vii)=
f(x) + x+2
log(1 − x)

x+2 ≥ 0 &x ≥ −2 = 2sinx cos x = sin2x

And (1 − x) > 0 & 1 − x ≠ 1 sin2x ≥ 0

x <1 & x ≠ 0 2x ∈[2nπ,(2n + 1)π]

x ∈( −∞ ,1) − {0}
 1
x ∈ [nπ,  n +  π]
 2
So domain [ −2,1) − {0}
cos x − 1 / 2
 5x − x2  (xii) f(x) =
(viii) y = log   6 + 35x − 6x2
 4 
 
cos x − 1 / 2 −1
 5x − x2  = x ≠ 6,
log  ≥0 (6 − x)(6x + 1) 6
 4 
 
−1
5x − x2 Case I: Let’s say < x<6
≥1 6
4 1
5x − x2 − 4 ≥ 0 So cos x − >0
2
−1
x2 − 5x + 4 ≤ 0 Case II: x <
6
(x − 4)(x − 1) ≤ 0 −1
So cos x <0
2
x ∈ [1, 4]
1
cos x <
2 1 2
(ix) f(x)
= x= x +
9 − x2
9 − x2 > 0

x2 < 9
6
x ∈( −3 ,3) /3
And x2 − x ≥ 0 -1/6

1 + 2Kx
1 + x + x2 − 1 + x + x2 Case III: x > 6
1 − 2Kx
x ∈ [1, ∞ ) 1
cos x ≤
2
if x < 0 x (x + 1) ≥ 0
From group analysing solution
x ∈ ( −∞ , − 1]
 −1 π   5π 
 ,  ∪  ,6  ∪
So the domain x ∈ ( −3, − 1] ∪ [1,3)  6 3  3 

(x) (x2 − 3x − 10) n2 (x − 3)  π 5π 


2kπ + ,2kπ +  ,k ∈ I − {0}
 3 3
x − 3 > 0 at x = 4

x > 3 f(x) = 0 (xiii) f(x)


= log1 (log 4 [x]2 − 5)
3
And x2 − 3x − 10 ≥ 0 (x − 5)(x + 2) ≥ 0
2
log3 (log4 [x] − 5) ≥ 0
x ∈ ( −∞ , −2] ∪ [5, ∞ )
log4 [x]2 − 5) ≤ 1
So domain x ∈ (5, ∞ ) ∪ {4}
M a them a ti cs | 13.89

[x]2 ≤ 9 2
And x − 5x − 24 > 0
[x]∈ [ −3,3] x2 − 3x − 8x − 24 > 0
And [x]2 − 5 > 0 (x + 3)(x − 8) > 0
[x]2 > 5 ⇒ x ∈[ −∞ , −3] > 0
2 2
And [x] − 5 ≠ 1 [x] − 5 > 1 x ∈[ −∞ , −3] ∪ [8, ∞ )
2 2
[x] ≠ 6 [x] > 6 [x]= 3, − 3 Case I if x + 2 > 0 ⇒ x > −2
2 2
And [x] − 5 ≠ 4 [x] ≠ 9 The x2 − 5x − 24 > (x + 2)2
[x] ≠ ± 3 −5x − 24 > 4 + 4 x

So domain x ∈ [ −3,2) ∪ [3, 4) 9x + 28 < 0

1 1 −28
(xiv) f(x) = + log(2{x} −5) (x2 − 3x + 10) + x<
[x] 1− x 9

[x] ≠ 0 x ∈[ 0,1) So no answer

2{x} − 5 < 0 So log2{x} −5 (x2 − 3x + 10) is not defined. So Case II if x + 2 < 0 then always true in interval
x∈φ
x2 − 5x − 24 > x + 2
=(xv) f(x) log7 log5 log3 log2 (2x2 + 5x2 − 14x) x <− 2
log5 log3 log2 (2x3 + 5x2 − 14x) > 0 So the domain x ∈( −∞ , −3]
3 2
log3 log2 (2x + 5x − 14x) > 1 (xvii) y= log(1 − log10 (x2 − 5x + 16)
log2 (2x3 + 5x2 − 14x) > 3 1 − log10 (x2 − 5x + 16) > 0 AND x2 − 5x + 16 > 0
2x3 + 5x2 − 14x > 8
log10 (x2 − 5x + 16) < 1 AND is the always positive as its
2x3 + 5x2 − 14x − 8 > 0 −b 5
minimum value of= x = is positive.
2a 2
(x − 2)(2x2 + 9x + 4) > 0
x2 − 5x + 6 < 0
(x − 2)(2x + 4)(2x + 1) > 0
(x − 3)(x − 2) < 0
 −1 
x ∈  −4,  ∪ (2, ∞ ) x ∈( 2,3 )
 2 
So the domain 2<x<3
(xvi) f(x)
= cos2x + 16 − x2
2 x +1
(xix) f(x) = log4 (2 − (x)1/ 4 −
2
cos2x > 0 and 16 − x ≥ 0 x+2
 π π 2 x +1
2x ∈ 2nπ − ,2nπ +  & x ∈ [ − 4, 4 ] 2 − (x)1/ 4 − >0
 2 2 x+2
 π π
x ∈ nπ − ,nπ +  & x ∈ [ − 4 , 4] 2 x +1
 4 4 (x)1/ 4 − >2
x+2
 5π 3π   π π   3π 5π 
So domain ∈  − , −  ∪  − ,  ∪  ,  (x)3/ 4 + 2(x)1/ 4 + 2(x)2/ 4 + 1 < 2(x)2/ 4 + 4
 4 4   4 4  4 4 
x ≥ 0 so put x = t 4
2
(xvii) =
f(x) n( x − 5x − 24 − x − 2)
t3 + 2t + 2t2 + 1 < 2t2 + 4

x2 − 5x − 24 − x − 2 > 0 t3 + 2t − 3 < 0
1 3 . 9 0 | Relations and Functions

(t − 1)(t2 + t + 3) < 0 (x − 2)(x − 1)


Range f(x) =
(x − 2)(x + 3)
t2 + t + 3
4
Is always greater then zero. For x ≠ 2 f(x) =
1−
x+3
So (t − 1) < 0
x = 2 f(x)not defined
t <1
1
x1/ 4 < 1 ⇒ 0 ≤ x < 1 Range f(x)∈ R − { } − {1}
5
1
2: (i) y log
Sol = 5 ( 2(sinx − cos x) + 3 ) at x = 2 ⇒   at x = ∞ [1]
5
2(sinx − cos x) + 3 > 0 x
(iv) f(x) = domain x ∈ R
1+ x
x 1
2 2(sinx cos 450 − cos x sin 450 ) > −3 for x > 0 f(x) = = 1−
1+x 1+x
2sin(x − 450 ) > −3 0 < f (x) < 1

−3 x 1
sin(x − 450 ) > so the domain x ∈ R for x < 0 =
f(x) =
2 1−x 1
−1
  π  x
Range
= f(x) log 5 2sin  x −  + 3 −1 < f(x) < 0
  4 
 π So f(x) ∈( −1,1)
+ 1 ≤ 2sin  x −  + 3 ≤ 5
 4
(v) y = 2−x + 1+x
  π 
log 5 (1) ≤ log 5  2sin  x −  + 3  ≤ log 5 (5)
  4  z − x ≥ 0 and 1 + x ≥ 0
0 ≤ f(x) ≤ 2 x ≤ 2 and x ≥ −1

2x Domain x ∈ [ −1,2]
(ii) y =
2
1+x
Range 2 − x decreases with increment in x 1+x
Domain x ∈ R Increase with increment in x
2
Range y = Since both are linear function root, so we can say that
1 it’s maximum will be at middle point of boundary
x+
x defined.
1
x+ ≥ 2 for x ≥ 0 from arithmetic mean > geometric 1 1
x So f(x) ≤ 2 −   + 1 +  
mean 2
  2
1 3 3
x+ ≤ −2 for x ≤ 0 from A.M >G.M f(x) ≤ + =6
x 2 2
So [-1,1]
f(x) ≥ 3 ( at boundary point )
x2 − 3x + 2
(iii) f(x) =
x2 + x − 6 3 ≤ f(x) ≤ 6
2
Domain x + x − 6 ≠ 0
(vi)
= f(x) log(cos ec x −1) (2 − [sinx] − [sinx]2 )
2
x + 3x − 2x − 6 ≠ 0
[sinx]
= 0 or − 1
(x + 3) (x − 2) ≠ 0
2 − [sinx] − [sinx]2 =
2
x ≠ 2, − 3
M a them a ti cs | 13.91

cosecx − 1 > 0 & cosec x − 1 ≠ 1  5  11   25 


2 2
5
=  x +  −   x +  − 
x −1 + 6 − x  2 4   2 4
  
1
It has two roots so not injective nor Surjective (as the
| x | −x Ranges not R)
π π
x ≠ 2nπ + ,(2n + 1) − 1
6 6 Sol 4: f(x) =
1−x
π  1  1
x ∈ (2nπ(2n + 1)π) − {2nπ + } f2 (x) = f[f(x)] = f 
2  =
1 − x   1 
1−
So domain 
1 − x 
3  7 
 , ∞ →  , ∞   1 
2  4  f3 (x) = f f {f(x)} = f f  
Range loga 2;  1 − x 

a ∈ (0, ∞ ) − {1} ⇒ Range ( −∞ , ∞ ) − {0}  1 


1− 
x +1 =
1
= 1 − x 
(vii) f(x) = 1  1 
x−2 1− − 
 1  1 − x 
Domain x ∈ R − {2} 1− 
1 − x 
3
Range f(x)= 1 +  1
x−2 Sol 5: f(n) =
−1, f n +  =
0
 2
f(x)∈( −∞ , ∞ ) − {1}
f(n + x) = a(n + x) + b where 0 ≤ x < 1
Sol 3: Injective = one to one mapping
x = 0, f(n) =−1 =ax + b ....(i)
Surjective = onto function
1  1  1
x= , f  n +  =0 =a  n +  + b
x2 + x + 1 (x + 2) 2  2   2
(a) f(x) = = 1−
x2 + 2x + 3 x2 + 2x + 3 a
− = ax + b …(ii)
(x + 2) 2
= 1−
(x + 1)2 + 1 a
− =−1
2
It’s range is not R. So not surjective not injective
a=2
f(x) =x3 − 6x2 + 11x − 6 b =−1 − ax =−1 − 2x
Range ∈ R onto function ⇒ surjective
So f(n + x)= 2(n + x) − 1 − 2x
2
f(x) = (x − 1)(x − 5x + 6)
f(n + x) = 2x − 1 where 0 ≤ x < 1, n ∈ R +
2
0 3
1

=(x − 1)(x − 3)(x − 2)


Sol 6: (a) (x − 3) + (4 − x) < x − 3 + 4 − x
Not one-one as at x = 1, 2, 3 f(x) = 0
Not injective ⇒ 1 < x+3 + 4−x
2 2
(b) f(x) = (x + 5x + 9)(x + 5x + 1)

 5
2
25   5
2
25 
=  x +  + 9 −   x +  + 1 −  3 4
 2 4   2 4
   3 4
1 3 . 9 2 | Relations and Functions

x+3 + 4−x >1 (f) f(x)= k= f( −x)

Even
1 + x 
(g) n  = f( −x)
1 − x 
3 4
 1−x
f( −x) = − n  − =−f(x)
 1+x
x ∈ ( −∞ ,3) ∪ (4, ∞ )
Odd function
(b) x − (x − 2) > x − x − 2 (1 + 2x )2
(h) f(x) =
2 > x − x−2 2x
(2 + 1)2
x
f( −x)
= = f(x)
x − x−2 < 2 2x
If x > 2 not true Even
−x x
If 0 ≤ x ≤ 2 2x − 2 < 2 f( −x)
(i) = − + 1 ≠ f(x) ≠ −f(x)
1 2
−1
x ≤ 2 x ∈ [0,2] ex
If x < 0 − 2 < 2 always true So neither odd nor even
( −∞ ,2)
( j) f(x) = [(x + 1)2 ]1/3 + [(x − 1)2 ]1/3

= log  −x + 1 + x2 
Sol 7: (a) f( −x) 1/3 1/3
  f( −x)= (1 − x)2  + ( −x − 1)2 
   
= log  1 + x2 − x   1 + x2 +=
f( −x) + f(x) x  0 1/3 1/3
   = (x − 1)2  + (1 + x)2  = f(x)
   
f(x) = −f(x) Even function
So odd function
f(x) sin4 x + cos4 x
Sol 8: (a)=
ax + 1
(b) f(x) =
ax − 1 (sin2 x)2 + ((cos2 )3 x)2 + 2sin2 x
f(x) =
1
+1 cos2 x − 2sin2 cos2 x
x 1 + ax
f( −x) =a = =−f(x)
( )
2
1 ax
− 1 f(x) =sin2 x + cos2 x − 2sin2 x cos2 x
−1
ax
(sin2x)2
f(x)= 1 −
Odd function 2
= x 4 − 2x2
(c) f(x) sin2x ⇒ Time period ⇒ π

f( −x) = x 4 − 2x2 = f(x) (sin2x)2 ⇒ Timeperiod ⇒ π / 2

Even π
So f(x) has Time period
2
(d) f(x)= x2 − | x |
f(x) | sinx | + | cos x |
(b)=
f( −x) = x2 − | x |
π
Case-I 0 < x <
Even function 2

(e) f( −x) =−x sin2 x + x3 = −f(x) f(x)


= sinx + cos x
π
Odd function Case-II < x < π f(x)
= sinx − cos x
2

Case-III π < x < f(x) =
−(sinx + cos x)
2
M a them a ti cs | 13.93

3π  2 
Case-IV < x < 2π f(x) − sinx + cos x Sol 11: f(x) = n  x + x + 1 
2  
sin x cos x
y = n  x + x2 + 1 
 

ey − x= x2 + 1

Since combining graphs, we can see | sinx | + | cos x | e2y + x2 − 2ey (x) = x2 + 1
π
has Time period. (1 − e2y ) e2y − 1
2 x=
− =
2ey 2ey
3 2x
(c)
= f(x) cos x − sin
5 7 1 y 1 
2π e − y 
2 e 
cos3x has time period
3
1 1 x 1 
3x 10π f −= (x) e − x 
cos has time period 2 e 
5 3
g ( x ) = sinx
2π x
has time period × (7)
2 (b) f(x) = 2 x −1
 10π  L.C.M.(10π,7π) x
L.C.M.  ,7=
 = 70π log2 y =
 3  H.C.F.(3,1) x −1

Sol 9: (a) 10x + 10 y =


10 1
log2 y= 1 +
x −1
=y log(10 − 10x )
1
Domain x ≤ 1 (x − 1) =
log2 y − 1
(b) ( −∞ , ∞ ) − {0}
1
x= 1 +
If y > 0 y = x log2 y − 1
x
If y < 0 y = log2 y
3 x=
log2 y − 1
5 
f(x) cosnx ⋅ sin  x 
Sol 10:= log2 y
n  f −1 (x) =
log2 x − 1
 5 1
= 2cosnx ⋅ sin x 
 n 2 10x − 10− x
(c) y =
10x + 10− x
 5  5  1
= sin  − x  x + sin  + n  x  102x − 1
 n  x  2 y=
102x + 1
 
 2x 2π 
L.C.M.  ,  = 3π y + 1 2(102x )
 5 − n 5 + n =
y −1 −2
 n n 
 y +1
log10 −  
 2nπ 2nπ
L.C.M.  ,
 y +1
= 102x  y −1 
 = 3π x
⇒=
 5 − n2 5 + n2  y −1 2

2nπ 1  x +1
= 3π ⇒ f −1 (x) = log  
H.C.F.(5 − n2 ,5 + n2 ) 2 1 − x 
1 3 . 9 4 | Relations and Functions

 1
= (a − xn )1/n
Sol 12: f(x) Sol 15: f(x) = max  x, 
 x
1/n
 n
fof(x) =a − (a − x)n  
   
1
1/n -1
 n f(x)
=a − (a − xn )  1
    -1

to f(x) = x
If g is inverse of f then fog(x)=x from 1 1 
f = f(x)
= max  , x 
x  x 
Above we can say f −1 (x) = f(x)
1
Sol 13: f(x) = x2 + x − 2  , 0< x <1
x
2 x, x >1
 1 9 f(x) 
y = x +  − 1 ,
 2 4 x < −1
x
x, 0 > x > −1
1 9 
x =− + y−
2 4 1
f   = f (x)
1 9 x
f −1 (x) =y =− + x−
2 4
1
f(x) = f −1 (x)  if 0< x ≤1
g(x)  x2
⇒ fof(x) = x x2 if x >1

(x2 + x − 2)2 + (x2 + x − 2) − 2 =x

Solving this equation we get


x= ± 2

x + 1, x < −1
2
Sol 14: f(x) 
3
x − 1, x ≤ x < 0 x2 + 2x + c
Sol 16: f(x) =
(a) If f(x) is odd x2 + 4x + 3c

x2 + 1, x < −1 (x + 1)2 + c − 1


 f(x) =
+ x3 , −1 ≤ x < 0 (x + 2)2 + 3c − 4
f(x) 
3
+ x , 1≥x≥0 (c − 1) ≤ 0
 2
−(x + 1), x > 1 (36 − 4) ≤ 0
(b) Even function So the value of f(x) is always F if x ∈ R

x2 + 1, x < −1 f(x)


Sol 17: (a)= log2 (x2 − 2x + 2)

+ x3 , −1 ≤ x < 0
f(x) 
3
−x , 1≥x≥0 log2 (x − 1)2 + 1)
 2
x + 1, x >1
log2 (x − 1)2 + 1 ≥ 0
 
(x − 1)2 ≥ 0
M a them a ti cs | 13.95

That is always true so domain is R → if a R b → 1 + ab > 0


(x − 1)2 + 1 ≥ 1 1 + ab = 1 + ba
so 1 + ba also greater than zero
log2 [(x − 1)2 + 1] ≥ 0
so (b, a) ∈ R
Range f(x) ≥ 0
R is symmetric
2x + 3
(b) f(x) = → if (a, b) ∈ R and (b, c) ∈ R
x−2
2(x − 2) + 7 7 ⇒ 1 + ab > 0,1 + bc > 0
= = 2+
x−2 12 Its not necessary to 1 + ac > 0
f(x) ≠ 7 f(x) ∈ R − {2} R is not transitive.

Bijective function
Sol 3: (A) (A)x R, y ↔ |x| = |y|

Sol 18: −1 ≤ x(x − 2) < ∞ |x| = |x| reflexive


|x| = |y| ⇒ |y| = |x| symmetric
x(x − 2) + 1
1≤ <∞ |x| = |y| and |y| = |z|
4
x(x −2)+1 equivalence relation
1 ≤2 4 <∞ so |x| = |y| = |z| ⇒ |x| = |z| transitive
2 ≤ f(x) < ∞
At x=0 & x=2 value of f(x) is same. Sol 4: (D) Relation R defined in A = {1, 2, 3}
So many one function.
such that a R b ⇒ |a2 – b2| ≤ 5
(1, 2) ⇒ |(12 – 22)| = 3≤ 5
Exercise 2 (1, 3) ⇒ |12 – 32| = 8≤ 5
(2, 3) ⇒ |22 – 32| = 5 ≤ 5
Sets and Relations
and (a, a) = |a2 – a2| = 0 ≤ 5
Single Correct Choice Type
and (a, b) = (b, a) ⇒ |a2 – b2| = |b2 – a2|
Sol 1: (D) n R m ↔ n is a factor of m so R = {(1, 2), (2, 1), (2, 3), (3,2), (1, 1), (2, 2), (3, 3)}
→ every natural no. is a factor of itself so R–1 = R
R is reflexive
Domain = {1, 2, 3}
→ if n is factor of m, its not necessary that m is also
factor of n Range = {1, 2, 3} , so

R is not symmetric Option D is false (Range = {5})


→ if n is factor Q m
Sol 5: (A) (x, y) ∈ R ↔ x2 – 4xy + 3y2 = 0
and m is factor Q 
so n is also a factor of  for all x, y ∈ N
(i.e., 3 R 6 and 6 R 18 and 3 R 18 is true) → for (x, x) ∈ R ↔ x2 – 4x2 + 3x2 = 0
=0
Sol 2: (C) a R b ↔ 1 + ab > 0
R is reflexive
for a R b ⇒ 1 + ab > 0
→ if (x, y) ∈ R ⇒ x2 – 4xy + 3y2 = 0
⇒ ab > – 1
then for (y, x) ∈ R ⇒ y2 – 4xy + 3x2 should be zero
→ a R a ⇒ 1 + a2 > a
but 3x2 – 4xy + y2
it is always true
= x2 – 4xy + 3y2 + 2x2 – 2y2
so R is reflexive
1 3 . 9 6 | Relations and Functions

= 0 + 2x2 – 2y2 ⇒ cb(d + a) = da(c + b)


→ (y, x) ∈ R ⇒ x2 – y2 = 0 Which is equation (ii)
its not solution for all (x, y) ∈ N ⇒ (a, b) R (c, d) Þ (c, d) R (a, b)
so R is not symmetric so R is symmetric relation
→ (x, y) ∈ R x2 – 4xy + 3y2 = 0 → assume (a, b) R (c, d) and (c, d) R (e, f)

(y, z) ∈ R → y2 – 4zy + 3z2 = 0 ad(b + c) = bc(a + d) … (i)


cf(d + e) = de(c + f)  … (ii)
but we cannot find x2 – 4zx + 3z2 = 0 from above
equations, so R is not transitive. for (a, b), (e, f)
⇒ af(b + e) = be(a + f)

Sol 6: (C) R = {(3, 3), (6, 6), (9, 9),(12, 12), (6, 12), (3, 9), If (e, f) = (c, d)
(3, 12), (3, 6)} then (c, d) R (e, f) is always true
on the set A = {3, 6, 9, 12} (R is reflexive)
→ for set A so in equation (i) (c, d) → (e, f)
(3, 3), (6, 6), (9, 9), (12, 12) ∈ R af (b + e) = be (a + f)
R is reflexive so ((a, b), (e, f)) ∈ R
→ (6, 12) ∈ R but (12, 6) ∉ R ∴ R is transitive.
So R is not symmetric
Sol 9: (A) W = all wards in the English dictionary
→ (3, 6) ∈ R and (6, 12) ∈ R and also (3, 12) ∈ R
R = {(x, y) ∈ w × w the words x and y have at least one
R is transitive. letter in common}
→ a word have all letter common to itself R is reflexive
Sol 7: (C) R = {(1, 3), (4, 2), (2, 4), (2, 3), (3, 1)}
→ if x and y have one letter common soy and x is same
on the set A = {1, 2, 3, 4} condition
→ for set A (x, y) ∈ R → (y, x) ∈ R
(1, 1), (2, 2), (3, 3), (4, 4) ∉ A R is symmetric
R is not reflexive → if (x, y) ∈ R, (y, z) ∈ R
→ (2, 3) ∈ R , (3, 2) ∉ R x and y have one letter common
So R is not symmetric (1,3)∈ R, (3,1) ∈ R y and z have one letter common
But (1, 1) ∉ R its not mean that it is necessary to x and z have one
so R is not transitive. letter common
R is not transitive.
Sol 8: (D) R: N × N
(a, b) R (c, d) if ad(b + c) = bc(a + d) Sol 10: (C) R → real line
⇒ for (a, b)R(a, b) ⇒ ab(b+a) = ba(a+b) Given subset S = {(x, y): y = x + 1 and 0 < x < 2}
Which is true so R is reflexive and T = {(x, y): x – y is an integer}
→ for symmetric for S y = x + 1
(a, b) R (c, d) ⇒ (c, d) R (a, b) but x ≠ x + 1
ad(b + c) = bc(a + d) s is not reflexive
cb(d + a) = da(c + b) ... (ii) so s is not equivalence
ad(b + c) = bc(a + d)
M a them a ti cs | 13.97

for T (x, y) ∈ T ⇒ x – y is an integer x – x = 0 is an integer in (2, 4), 4 is not belongs to y


T is reflexive So R is not relation from x to y.
→ if (x, y) ∈ T → x – y is an integer
so + (y – x) also an integer Functions
so (y, x) ∈ T Sol 1: (C)
T is symmetric 1
f(x) sin−1 (x − x2 ) + sec−1   + n x − 1
=
→ if (x, y) ∈ T and (y, z) ∈ T x
x −1 ≠ ⇒ x ≠ 1
x – y = z1  … (i);
y – z = z2 … (ii) AND (x − x2 ) ≤ 1
(assume z1 and z2 are integer)
x − x2 ≤ 0
(i) + (ii)
x(x − 1) ≤ 0 ⇒ x ∈ [0,1]
x – y + y – z = z1 + z 2
x – z = z1 + z2 x(x − 1) ≤ 1 always true in internal [0,1]
∴ sum of two integer is also an integer
1 1
And ≥ 1 or ≤ −1 x ≠ 0
so x – z is an integer x x
(x, z) ∈ T
0 ≤ x ≤ 1 or 1 ≤ x π 0
T is transitive
So domain x ∈ (0,1)
⇒ T is an equivalence relation

Sol 2: (B)
Multiple Correct Choice Type
D C
Sol 11: (A, B, C) x = {1, 2, 3, 4, 5}, y= {1, 3, 5, 7, 9} N o
(A) R1 = {(x, y) | y = 2 + x , x ∈ X, y ∈ Y } P
B
R = {(1, 3) (3, 5)(5, 7)} A M

(B) R2 = {(1, 1), (2, 1)(3, 3), (4, 3), (5, 5)} AC = 2 2
is satisfied R: x → y BD = 2 2
1 1  AP = x
   
2 1 
3  3  → a subset of y PC
= 2 2 − x ⇒ MN
= 2x
   
4 3 1
( ∆AMN)Area =× base × height
5 5  2
   
1
(C) R3 = {(1, 1), (1, 3), (3, 5), (3, 7), (5, 7)} =× (AP) × (MN)
2
All elements are from x 1
= x2 (for x < 2)
× (2x)(x) =
1  1  2
    f( −1) =−1 f( −2) =−8 ∆AMN is maximum
1  3
3 5 
    f(x) =maximum= ( 2)2 = 2
3 7
5  7 So the range is (0,2)
   
all element are from y
(D) R4 = {(1, 3),(2, 5), (2, 4), (7, 2)}
1 3 . 9 8 | Relations and Functions

Sol 3: (A) Bijective x


Sol 8: (A) f(x) =
1+ x
gof If x>0
f → one-one function g → one-one 1
0 ≤ f(x) =−
1 < 1 one-one is the interval.
function so gof will be one-one function 1+x
f & g → onto function if x<0

gof(x) will be onto only of if domain of g = range of f x


f(x) =
1−x
Sol 4: (D) y= 5[x] + 1= 6[x − 1] − 10  −x   −x 
= − = −1  1 − 
1 − x   1−x
5I + 1 = 6I − 6 − 10
1
0 ≥ f(x)
= − 1 > −1
I = 17 1−x

x ∈ [17,18) So f(x) is injective (one-one)

y 5(17) +=
= 1 86 Multiple Correct Choice Type
[x + 2y] =
189
Sol 9: (A, D)
= f(x) x f : I →R
= ax3 + ex
Sol 5: (D) f(x) F is not onto function.
'
f= (x) 2ax2 + ex But f is one-one function.

For being a one-one f ' (x) ≥ 0 or f ' (x) ≤ 0


Sol 10: (B, C) f(x) = log x
x2
f(x) is always greater for any x. if a ≥ 0
x ≠ 0, x > 0 x ≠ 1
a ∈ (0, ∞ ) if a ≠ 0 then f is not onto function.
1 1
log
= 2 = log x
(x) 2 x 2
Sol 6: (A)
x ∈ (0, ∞ ) − {1}
1 + sinx 1 − sinx
− sec x =
− + sec x x is defined for (0, 1) and (1, ∞)
1 − sinx 1 + sinx

1 + sinx 1 − sinx x x − 2 x −1
+ 2sec x
= Sol 11: (B, C) y =
1 − sinx 1 + sinx
x −1 −1
2 x −1 ≥ 0 ⇒ x ≥ 1
= 2sec x
2
1 − sin x
And x −1 ≠ 1 ⇒ x ≠ 2
2
= 2sec x
cos2 x And x − 2 x − 1 ≥ 0

=sec x sec x if cos x ≠ 0 x ≥2 x −1


Case I if x ≥ 0 x2 ≥ 4 (x − 1)
 −3π   −π π   3π 
x ∈  −2π, ∪  ,  ∪  ,2π 
 2   2 2   2  x2 − 4x + 4 ≥ 0
(x − 2)2 ≥ 0 which is always true.
 x
Sol 7:
= (D) f(x) sin  cos  + cos(sinx) Case II if x ≤ 0 then not true
 2 
x So the domain is x ∈ [1, ∞ ) − {2}
cos has time period = 4π
2
sin x has time period= 2π
So the common time period = 4π
M a them a ti cs | 13.99

 Sol 13: (A, D) (A) f(x) =x − [x] ={x}


2 / 3 3 1 
−2 Periodic
 2 2  3 3−2 2
2f(1.5) + f(3) 2 
= + 1
 1  2 −1 (B) g(x)= sin   , x ≠ 0 & g(0) = 0
−1 x
 2 
1 1
is not periodic so sin is also not.
 3 3 − 2 2  (3 3 − 2 2) x x
2 +
 2 2  2 −1 (C) h(x) = x cos x
 
Not periodic C
= 0 = non negative (D) w(x) = sin x
Putting x − 1 = 2
t for x>2 Periodic

(t2 + 1) t2 + 1 − 2t x if x ∈ Q
f(x) = = t2 + 1 = x Sol 14: (B, C) f(x) = 
t −1 1 − x if x ∉ Q
Putting for x<2, it is not always defined. x ∈ [ 0,1 ]
1
0 ≤ f(x) < 1 [x ∈ [0,1] − {1}]
Sol 12: (A, C, D) (A) f(x) = 2 x −1
Decreasing function At x=1, f(x) = 1

So at boundary conditions [f(x)] = 0 for x ∈ [0,1)

f(x) = 21/x −1 at n 1;=


= [f(x)] 1
1
At f(x) = 21/x −1 f(x) = f([f(x)]) =1 ,for x ∈ [0,1)
2
At x= 1 − h f(x) = 0 −1 1 − x2
C, D) (A) y tan(cos
Sol 15: (A,= = x) (identical)
x
Bounded
1
1
(B) g(x) = x cos 1 − x2
x θ
1 x
cos will oscillate between [-1,1] for any is not
x Domain of both function, are not same at x=0
bounded so g(x) is also not y = tan−1 (cos x) is defined while the order is not.
(C) h(x)‒x ≥ 0 in (0, ∞) 1
(B) y = tan(cos−1 x) (is not identical) y =
x
h'(x) = e‒x ‒xe‒x
−1 x
=(C) y sin(tan
= (x)) (identical)
If x > 1 then h (x) is decreasing
1 + x2
x < 1 then h (x) is increasing
(D) y = cos(tan−1 x) ; y = sin(cot −1 x) (identical)
at x = 0 ,h (x) = 0
1
1 y=
at x = 1 , h (x) =
e 1 + x2
at x = ∞ h (x) = 0 ( so it is bounded)
(D) (x) = tan−1 (2x )
2x is strictly increasing and it is positive.
(x) is bounded as = x − ∞ , and (x) = 0
= x‒ ∞, (x) = π / 2
1 3 . 1 0 0 | Relations and Functions

Previous Years’ Questions π


∴ f   = cos

+ cos 5π = –1
2
  2
x+2
Sol 1: (A, D) Given, y = f(x) =
x −1 f(π) = cos 9π + cos 10π = –1 + 1 = 0
⇒ yx – y = x + 2 ⇒ x(y – 1) = y + 2 f(–π) = cos 9π + cos 10π = –1 + 1 = 0
y+2 π 9π 10π 1 1
⇒x= ⇒ x = f(y) f   = cos + cos = +0=
y −1 4 4 4
  2 2
Here, f(1) does not exist, so domain ∈ R – {1}. Hence, (a) and (c) are correct options.
dy (x − 1)·1(x + 2)·1
= b−x
dx (x − 1)2 Sol 5: (A) Here, f(x) = , where
1 − bx
3
=– 0 < b < 1, 0 < x < 1
(x − 1)2
For function to be invertible it should be one–one onto.
⇒ f(x) is decreasing for all x ∈ R – {1}
∴ Check Range:
Also, f is rational function of x. b−x
Let f(x) = y ⇒ y =
Hence, (a) and (d) are correct options. 1 − bx
⇒ y – bxy = b – x
2x − 1
Sol 2: (A, D) Since, >0 ⇒ x(1 – by) = b – y
2x3 + 3x2 + x
(2x − 1) b−y
⇒ >0 ⇒x= , where 0 < x < 1
2 1 − by
x(2x + 3x + 1)
(2x − 1) b−y
⇒ >0 ∴0< <1
x(2x + 1)(x + 1) 1 − by
+ – + – + b−y b−y
> 0 and <1
–∞ –1 -1/2 0 ½ 1 − by 1 − by

Hence, the solution set is, 1


⇒ y < b or y > … (i)
b
x ∈ (–∞, –1) ∪ (–1/2, 0) ∪ (1/2, ∞)
(b − 1)(y + 1) 1
< 0 –1 < y <  … (ii)
3 1 − by b
Sol 3: (B, C) Since, area of equilateral triangle = (BC)2
4  1
3 3 2 From Eqs. (i) and (ii), we get y ∈  −1,  ⊂ co–domain
⇒ = .[x + g2(x)] ⇒ g2(x) = 1– x2  b 
4 4 Thus, f(x) is not invertible.
A
Sol 6: A → q; B → r
y = 1 + 2x is linear function therefore, it is one–one and
its range is (– π + 1, π + 1). Therefore, (1 + 2x) is one–
one but not onto so (A) → (q) Again, see the figure.
B C
(0, 0) (x, g(x)) y
y=1+2x
⇒ g(x) = 1 − x2 or − 1 − x2

Hence, (b) and (c) are the correct options.


x’ - O  x
2 2
Sol 4: (A, C) Since, f(x) = cos[π2]x + cos[–π2]x

⇒ f(x) = cos(9)x + cos(–10)x,

(using [π2] = 9 and [–π2] = –10) y’


M a them a ti cs | 13.101

It is clear from the graph that y = tan x is one–one and We must have
onto, therefore (B) → (r)
( )
2 ( 8 − k ) + 2 ( 4 − 2k ) + k 2 − 16 =
0

Sol 7: A→ p; B → q; C→ q; D → p ⇒k=
2, 4 .
(x − 1)(x − 5) (C) Let f(x) = |x + 2| + |x + 1| + |x −1| + |x −2|
Given, f(x) =
(x − 2)(x − 3)
⇒ k can take value 2, 3, 4, 5.
The graph of f(x) is shown
dy
y (D) ∫ y + 1 = ∫ dx
⇒ f ( x ) =2ex − 1 ⇒ f (ln 2 ) =
3
y=1

Sol 9: A→ q, s; B→ p, r, s, t; C→ t; D→ r
x’ x
0 1 2 3 5
2sin2 θ + 4 sin2 θ cos2 θ = 2

y’ (
sin2 θ + 2sin2 θ 1 − sin2 θ =1)
1
3sin2 θ + 2sin2 θ − 1 = 0 ⇒ sin θ =± , ±1
(A) If – 1 < x < 1 ⇒ 0 < f(x) < 1 2
π π
(B) If 1 < x < 2 ⇒ f(x) < 0 ⇒θ= , .
4 2
(C) If 3 < x < 5 ⇒ f(x) > 0 3x
(B) Let y =
π
(D) If x > 5 ⇒ f(x) < 1 1 π 
⇒ ≤ y ≤3∀ x∈ , π
2 6 
Sol 8: A→ p; B → q, s; C→ (q, r, s, t); D → r
Now f(y) = [2y] cos[y]
(A) f ' ( x ) > 0, ∀ x ∈ ( 0, π 2 )
1 3
Critical points are=y = , y 1,
= y = ,y 3
f (0) < 0 and f ( π 2 ) > 0 2 2

so one solution. π π π 
⇒ points of discontinuity  , , , π  .
6 3 2 

-4x 4x 1 1 0
(C) 1 2 0 = π ⇒ volume of parallelepiped = π
1 1 π
4-2x 2x-4
6
 
(D) a + b =3

-2 -1 1 2
⇒ 2 + 2 cos α = 3

⇒ 2 + 2cos α =3
(B) Let (a, b, c) is direction ratio of the intersected line, π
⇒α = .
then 3
ak + 4b + c = 0 1
Sol 10: f (=
0 ) 1, f ' (=
x ) 3x2 + ex /2
2
4a + kb + 2c = 0
a b c
( )
⇒ f ' g ( x ) g' ( x ) =
1
= =
8 − k 4 − 2k k 2 − 16 1
0 ⇒ g' (1 ) = =
Put x = 2.
f ' (0)
1 3 . 1 0 2 | Relations and Functions

x
Sol 11: (B) e− x f ( x ) =
2 + ∫ t 4 + 1dt  … (i) (  π π
)
⇒ g g ( x ) ∈  − ,  ∀ x ∈R
0  2 2

(
f f −1 ( x ) = x ) ((  π π
))
Also, g g g ( x ) ∈  − ,  ∀ x ∈R
 2 2
⇒f f ( −1
( x ))' ( f −1
)
(x) ' =
1 ⇒ f (2) ' =( 1
−1
)
f ' f (2)
−1
( )  1 1
Hence, f (x) and f g ( x ) ∈  − ,  ( )
⇒ f (0) =
2⇒ f −1
(2) =
0  2 2

(f ( 2) ) ' = f ' 10
−1

( ) f (x)
1
3

sin  g g ( x )  1 g g ( x )
. 3
( ) ( )
lim = lim
x →0 g(x) x →0 1 g(x)
e− x ( f ' ( x ) − f ( x ) ) = x4 + 1
3
g g(x) ( )
Put x = 0
π 
⇒ f ' (0) − 2 =1 ⇒ f ' (0) = 3 sin  sin x 
π 2 π
⇒ lim .  =
(f −1
(2)) ' = 1 / 3 x →0 6 π
2
sinx
6

 {x} , 2n − 1 ≤ x < 2n  π  1  π  1 
Sol 12: (D) f ( x ) =  ( )
Range of g f ( x ) ∈  − sin   , sin   
1 − {x} , 2n ≤ x < 2n + 1  2  2  2  2 

Clearly f(x) is a periodic function with period = 2 (


⇒ g f (x) ≠1 . )
Hence f(x). cos π x is also periodic with period = 2
10 2 Sol 15: A → s; B → t; C→r; D → r
π2
∫ f ( x ) cos ( π x ) dx =
π2 ∫ f ( x ) cos ( π x ) dx
10 −10 0 2i ( x + iy ) 2i ( x + iy )
=(A) z =
1
(( )
π ∫ 1 − {x} + {x} cos ( π x ) dx
= 2
) 1 − ( x + iy )
2
1 − x2 − y 2 + 2ixy ( )
0
1 1 Using 1 − x2 =
y2
 π sin π x cos π x 
=2 π2 ∫ ( − x cos π x )dx =−2 π2  + 
 π π2  0 2ix − 2y 1
0 Z= = − .
 2 
2
2y − 2ixy y
=−2 π2  −  =4 1
 π2  1
 −1 ≤ y ≤ 1 ⇒ − ≤ − 1 or − ≥ 1 .
y y
    (B) For domain
Sol 13: r. × b = c × b
8.3x −2
taking cross with a −1 ≤ ≤1
1−3 ( )
2 x −1
     
( ) (
a × r. × b = a c × b ) 3x − 3x −2
⇒ − 1≤ −≤1
        
( )
a.b r − ( a. r ) b =a × c × b ( ) 1 − 32x −2

 3x − 3x −2
⇒r = − 3 ˆi + 6 ˆj + 3 kˆ Case I: −1 ≤ 0
1 − 32x −2
 
r .b =3 + 6 =9

(3 − 1)(3 − 1) ≥ 0
x x −2

(x)
Sol 14: (A, B, C) Given g=
π
2
sinx ∀ x ∈R (3 − 1) 2x −2

1  ⇒ x ∈ ( − ∞ , 0  ∪ (1, ∞ )
f ( x ) = sin  g g ( x ) 
3 
( )
M a them a ti cs | 13.103

3x − 3x −2 (1 − x )
2
Case – II: + 1≥ 0 sin2 x = x2 − 2x + 1 + 1
2x
1−3 −2
=(1 − x ) + 1
2


(3 x −2
)(
− 1 3x + 1 )≥0 ⇒ (1 − x ) cos2 x =
2
−1
(3 .3
x x −2
−1 ) Which can never be possible
⇒ x ∈ ( − ∞ ,1 ) ∪ 2, ∞ )
P is not true
So, x ∈ ( − ∞ ,0 ) ∪ 2, ∞ ) .
⇒ Let H (=
x ) 2f ( x ) + 1 − 2x (1 + x )
(C) R1 → R1 + R 3
H (0) = 2 f (0) + 1 – 0 = 1
0 0 2
f ( θ ) =− tan θ 1 tan θ H (1) = 2 f (1) + 1 – 4 = -3
−1 − tan θ 1 ⇒ So H (x) has a solution

(
= 2 tan2 θ=
+ 1 2sec2 θ . ) So Q is true.

3 1/2
f '(x) ( x ) (3x − 10 ) + ( x ) × 3
3/2
= Sol 18: (A, B)
(D) 2
15 1/2 1 1
= ( x ) ( x − 2) cos 4 θ= ⇒ 2 cos2 2 θ − 1=
2 3 3
Increasing, when x ≥ 2 . 2 2
⇒ cos2 2 θ = ⇒ cos 2 θ = ±
3 3
Sol 16: (B) f ( x ) = 2x3 − 15x2 + 36x + 1 2 1 + cos 2 θ 1
Now f ( cos 4 θ ) = = =1+
2
2 − sec θ cos 2 θ cos 2 θ
f ' ( x ) = 6x2 − 30x + 36

(
= 6 x2 − 5x + 6 ) 1
⇒ f  =
3
 

3
2
=6 ( x − 2 )( x − 3 )

f (x) is increasing in [0, 2] and decreasing in Sol 19: (C) Let, g ( x ) = e− x f ( x )


[2, 3] As g" ( x ) > 0 so g’ (x) is increasing.
f (x) is many one
So, for x < 1 / 4, g' ( x ) < g' (1 / 4 ) =
0
f (0) = 1
f (2) = 29 ( )
⇒ f ' ( x ) − f ( x ) e− x < 0

f (3) = 28 ⇒ f ' ( x ) < f ( x ) in ( 0, 1 / 4 )


Range is [1, 29]
Hence, f (x) is many-one-onto.
 x2 ,x<0
Sol 20: (D) f2 ( f1 ) = 
2x
Sol 17: (C) e x≥0
f4R → 0, ∞ )
f ( x ) + 2x = 1 (1 − x ) sin2 x + x2 + 2x
2

(
f f ( x )
2 1 ), x<0
 f ( x ) + 2x = 2 1 + x ( 2
) f4 ( x ) = 
( )
f2 f1 ( x ) − 1 , x ≥ 0
⇒ (1 − x ) sin2 x + x2 + 2x =2 + 2x2
2
 x2 , x<0
=
2x
e − 1 , x ≥ 0
1 3 . 1 0 4 | Relations and Functions

Sol 21: (A, B, C)


 π π
(log ( sec x + tanx ) )
3
f (x)
= ∀ x ∈ − , 
 2 2
f ( −x ) =−f ( x ) , hence f (x) is odd function

 π π
Let g (x) = sec x + tan x ∀ x ∈  − , 
 2 2
 π π

= g' ( x ) sec x ( sec x + tanx ) > 0 ∀ x ∈  − , 
 2 2
⇒ g ( x ) is one-one function

( ))
3
Hence log g ( x )
e ( is one-one function.

 π π
And g ( x )∈ ( 0, ∝ ) ∀ x ∈ − , 
 2 2

( )
⇒ log g ( x ) ∈R . Hence f (x) is an onto function.

1
Sol
= 22: G (1 ) t f ( f ( t ) ) dt
∫= 0
−1
f (- x) = - f (x)
1
Given f (1 ) =
2
F ( x ) − F (1 )
F(x) x= −1 f (1 ) 1
lim
= lim =
x → 1 G ( x ) x → 1 G ( x ) − G (1 )
( )
f f (1 ) 14
x −1
1/2 1
⇒ =
f (1 / 2 ) 14

1
7.
⇒ f  =
2
2017-18 100 &
op kers
Class 11 T
By E ran ulty
-JE Fac
IIT enior emier
S f Pr tes.
o titu
Ins

MATHEMATICS
FOR JEE MAIN & ADVANCED
SECOND
EDITION

Exhaustive Theory
(Now Revised)

Formula Sheet
9000+ Problems
based on latest JEE pattern

2500 + 1000 (New) Problems


of previous 35 years of
AIEEE (JEE Main) and IIT-JEE (JEE Adv)

5000+Illustrations and Solved Examples


Detailed Solutions
of all problems available

Topic Covered Plancess Concepts


Tips & Tricks, Facts, Notes, Misconceptions,
Limits, Continuity and Key Take Aways, Problem Solving Tactics
Differentiability
PlancEssential
Questions recommended for revision
14. LIMITS, CONTINUITY
AND DIFFERENTIABILITY

1. LIMITS

1.1 Intuitive Idea of Limits


(a) Suppose we are travelling from Kashmiri gate to Connaught Place by metro, which will reach Connaught
place at 10 a.m. As the time gets closer and closer to 10 a.m. the distance of the train from Connaught place
gets closer and closer to zero (here we assume that the train is not delayed). If we consider the time as an
independent variable denoted by t and the distance remaining as a function of time, say f(t), then we say that
f(t) approaches zero as t approaches zero. We can say that the limit of f(t) is zero as t approaches zero.
(b) Let a regular polygon be described in a circle of given radius. We notice the following points from the geometry.
(i) The area of the polygon cannot be greater than the area of the circle however large
the number of sides may be.
(ii) As the number of sides of the polygon increases indefinitely, the area of the polygon
continuously approaches the area of the circle.
(iii) Ultimately the difference between the area of the circle and the area of the polygon
can be made as small as we please by sufficiently increasing the number of sides of
the polygon.
We can say that the limit of the area of the polygon inscribed in a circle is the area of the Figure 14.1
circle as the number of sides increases indefinitely.

1.2 Meaning of x → a
Let x be a variable and a be constant. Since x is a variable; we can change its value at our pleasure. It can be
changed in such a way that its value comes nearer and nearer to a. Then we say that x approaches a and it is
denoted by x → a:
x a- a+ x

Figure 14.2

We know that |x – a| is the distance between x and a on the real number line and 0 < |x – a| if x ≠ a, “x tends to a”
means
(a) x ≠ a, i.e. 0 < |x – a|,
1 4 . 2 | Limits, Continuity and Differentiability

(b) x takes up values nearer and nearer to a, i.e. the distance |x – a| between x and a becomes smaller and smaller.
One may ask “how much smaller”? The answer is, as much as we please. It may be less than 0.1 or 0.00001
or 0.0000001 and so on. In fact, we may choose any positive number δ. However small it may be, |x – a| will
always be less than δ. The above discussion leads up to the following definition of x → a.
Let x be a variable and a be a constant.

Definition: Given a number δ > 0 however small, if x takes up values, such that 0 < |x – a| < δ. Then x is said to tend
to a, and is symbolically written as x → a

Note. If x approaches a from values less than a, i.e. from the left side of a, we write x → a–. If x approaches a from
values greater than a, i.e. from the right side of a, we write x → a+.
But x → a means both x → a– and x → a+. So x approaches or tends to a means x approaches a from both sides
right and left.

Neighbourhood of point a: The set of all real numbers lying between a – δ and a + δ is called the neighbourhood
of a. Neighbourhood of a = (a – δ, a + δ) ; x ∈ (a – δ, a + δ)

1.3 Limit of a Function


Let us take some examples to find the limit of various functions:

x2 − 4
Illustration 1: Consider the function f(x) = . We investigate the behaviour of f(x) at the point
x−2
x = 2 and near the point x =2. (JEE MAIN)

Sol: Here as f(2) = 0 , therefore try to evaluate the value of f(x) when x is very near to 2.

4−4 0
f(2)
= = , which is meaningless. Thus f(x) is not defined at x = 2.
2−2 0

Now we try to evaluate the value of f(x) when x is very near to 2 for some values of x less than 2 and then for x
greater than 2.

(1.9)2 − 4 −0.39 (2.1)2 − 4 0.41


f(1.9)
= = = 3.9 f(2.1)
= = = 4.1
1.9 − 2 −0.1 2.1 − 2 0.1

(1.99)2 − 4 −0.0399 (2.01)2 − 4 0.0401


f(1.99)
= = = 3.99 = f(2.01) = = 4.01
1.99 − 2 −0.01 2.01 − 2 0.01

(1.999)2 − 4 −0.003999 (2.001)2 − 4 0.004001


f(1.999)
= = = 3.999 = f(2.001) = = 4.001
1.999 − 2 −0.001 2.001 − 2 0.001

3 3.9 3.99 3.999 4 4.001 4.01 4.1

x=1 1.9 1.99 1.999 2 2.001 2.01 2.1

Figure 14.3

It is clear that as x gets nearer and nearer to 2 from either side, f(x) gets closer and closer to 4 from either side.
M a them a ti cs | 14.3

When x approaches 2 from the left hand side the function f(x) tends to a definite number 4. Thus we say that as x
tends to 2 the left hand limit of the function f exists and equals to the definite number 4. Similarly, as x approaches 2
from the right hand side, the function f(x) tends to a definite number, 4.
Again we say that as x approaches 2 from the right hand side of 2, the right hand limit of f exists and equals to 4.

−1,if x < 0
Illustration 2: Discuss the limit of the function f(x) =  at x = 0  (JEE MAIN)
 1, if x > 0

Sol: Sketch its graph when x is very near to 0 for range of x from – 1 to 1.

−1, x < 0
We have, f(x) = 
+1, x > 0

Let us sketch its graph

x -1 -0.5 -0.1 -0.01 -0.001 0.001 0.01 0.1 0.5 1


f(x) -1 -1 -1 -1 -1 1 1 1 1 1

(i) As x approaches zero from the left of zero, f(x) remains at –1. And we say that the left hand limit of f exists at
x = 0 and equals to –1. lim f(x) = −1
x →0 −

(ii) As x approaches zero from the right of zero, f(x) remains at 1. So we say that the right hand limit of f at x = 0
exists and equals to +1. lim f(x) = +1
x →0 +

(iii) Left hand limit of f(x) (at x = 0) ≠ Right hand limit of f(x) {at x = 0}.
So the lim f(x) does not exist.
x →0

1
Illustration 3: Discuss the limit of the function f(x) = at x = 0 and its graph  (JEE MAIN)
x
Sol: Same as above illustration.
1 1
We have, f(x) = Let us draw the graph of the given function f(x) = .
x x

x -1 -0.1 -0.01 -0.001 -0.0001 0.0001 0.001 0.01 0.1 1


f(x) -1 -10 -100 -1000 -10000 10000 1000 100 10 1

(i) As x approaches zero from the left of zero the graph never approaches a finite number so we say that the left
hand limit of f at x = 0 does not exist i.e. lim f(x) does
= −1 not exist
x →0 −
(ii) As x approaches zero from the right of zero, the graph again does not approach a finite number. Again we say
that the right hand limit of f at x = 0 does not exist. lim f(x) =does
+1 not exist
x →0 +
(iii) At x=0 left hand limit of f ≠ right hand limit of f
Hence, the limit of f(x) does not exist.
1 4 . 4 | Limits, Continuity and Differentiability

1
Y=
x
8
7
6
5
4
3
2
1
-8 -7 -6 -5 -4 -3 -2 -1
X’ X
0
11 2 3 4 5 6 7 8
2
3
4
5
6
7
8
Y’
Figure 14.4

PLANCESS CONCEPTS

(a) The fact that the limit of f(x) exists at x = a means that the graph of f(x) approaches the same value
from both sides of x = a.
(b) The fact that f(x) is continuous at x = a means that there is no break in the graph as x moves from
a– to a+.
Vaibhav Krishnan (JEE 2009, AIR 22)

1.4 Different Cases of Limits


Right hand limit is the limit of the function as x approaches a from the positive side.
Left hand limit is the limit of the function as x approaches a from the negative side.
Note: A function will have a limiting value only if its right hand limit equals its left hand limit.

Illustration 4: Discuss the limits of f(x) = |x| at x = 0 and draw its graph.  (JEE MAIN)

Sol: We have f(x) = |x|, therefore f(x) is equals to x for x > 0 and – x for x < 0.

Increasing x Decreasing x

x -3 -2 -1 0 1 2 3
f(x) 3 2 1 0 1 2 3
Limit
Decreasing f(x) Decreasing f(x)
 x, x > 0
We have f(x) = |x| Þ f(x) = 
−x, x < 0
M a them a ti cs | 14.5

Let us draw its graph.


f(x)

(-3, 3) (3, 3)
3

(-2, 2) 2 (2, 2)

(-1, 1) 1 (1, 1)

X’ X
0
-3 -2 -1 1 2 3

Figure 14.5

(i) As x approaches zero from left of zero, f(x) = 0. And we say that left hand limit of f(x) exists, and is equal to
zero.
lim− f(x) = 0
x →0

(ii) As x approaches zero from the right of zero f(x) is equal to zero. So we say that the right hand limit of f at x =
0 exists, and is equal to zero. lim f(x) = 0 Here =lim f(x) = lim f(x) 0
x →0 + x →0 − x →0 +
Hence we say that the limit of f (x) at x = 0 exists and equals to zero.

Illustration 5: Evaluate: lim x + 3  (JEE MAIN)


x →3
Sol: Simply taking value of x very near to 3 we can obtain value of the function at these points.
Let us compute the value of function f(x) for x very near to 3. Some of the points near and to the left of 3 are 2.9,
2.99, 2.999.
Values of the function are given in the table below. Similarly, some of the numbers near and right of 3 are
3.001,3.01,3.1. Value of the function at these points are also given in the table.

Increasing x Decreasing x

x 2.9 2.99 2.999 3 3.001 3.01 3.1


f(x) 5.9 5.99 5.999 6 6.001 6.01 6.1

Limit
Increasing f(x) Decreasing f(x)

From the table we deduce that the value of f(x) at x = 3 should be greater than 5.999 and less than 6.001.
It is reasonable to assume that the value of function f(x) at x = 3 from the left of 3 is 5.999.

lim f(x) ≈ 5.999  ... (i)


x →3−
Similarly, when x approaches x = 3 from the right, f(x) should be 6.001

lim f(x) ≈ 6.001 ... (ii)


x →3+ 
From (i) and (ii), we conclude that the limit is equal to 6.
1 4 . 6 | Limits, Continuity and Differentiability

lim
= f(x) lim
= f(x) lim
= f(x) 6
x →3− x →3+ x →3

3x2 − x − 10
Illustration 6: Evaluate: lim  (JEE MAIN)
x →2 x2 − 4

Sol: Similar to above problem.


Let us compute the value of function f(x) for x very near to 2. Some of the points near and to the left of 2 are 1.9,
1.99, 1.999.
Values of the function are given in the table below. Similarly, some of the numbers near and to the right of 2 are
2.001, 2.01, 2.1. Values of the function at these points are also given in the table.

Increasing x Decreasing x

x 1.9 1.99 1.999 2 2.001 2.01 2.1


f(x) 2.743 2.749 2.7499 2.75 2.750 2.7506 2.756
Limit
Increasing f(x) Decreasing f(x)

From the table we deduce that the value of f(x) at x = 2 should be greater than 2.7499 and less than 2.750.
It is reasonable to assume that the value of function f(x) at x = 3 from the left of 2 is 2.7499.

∴ lim f(x) ≈ 2.7499  .... (i)


x → 2−

Similarly, when x-approaches x = 2 from the right f (x) should be 2.750. lim f(x) ≈ 2.750  ....(ii)
x → 2+

From (i) and (ii), we conclude that the limit is equal to 2.75. lim
= f(x) lim
= f(x) lim
= f(x) 2.75
x → 2− x → 2+ x →2

PLANCESS CONCEPTS

• If Lim f ( a − b )= Lim + ( a + b ) , then we can say that both the left hand limit and right hand limits exist
x→a x → a+
at x = a, and irrespective of the value of the function at a, i.e f(a), the function does not have a limit at

x = a, that is Lim f ( x ) does not exist


x →a

• If both the left hand limit and right hand limit of f(x) at x = a exist and at least one of them is not equal
to f(a), then the limit of f at x = a does not exist.
Shrikant Nagori (JEE 2009, AIR 30)

1.5 Working Rule for Evaluation of Left and Right Hand Limits

Right hand limit of f(x), when x → a = lim f(x)


x →a+
Step I. Put x = a + h and replace a by a. +

Step II. Simplify lim f(a + h) .


h→0

Step III. The value obtained in step 2 is the right hand limit of f(x) at x = a.
M a them a ti cs | 14.7

Similarly for evaluating the left hand limit put x = a – h.


Evaluate the left-hand and right-hand limits of the following function at x = 1.

 5x − 4, if 0 < x ≤ 1
f(x) =  2
 4x − 3x, if 1 < x < 2

Illustration 7: Does lim f(x) exist? (JEE MAIN)


x →1

Sol: By taking left hand limit and right hand limit we can conclude that the given limit exist or not.
Left hand limit = lim f(x) = lim (5x − 4) [∵ f(x) = 5x – 4, if 0 < x ≤ 1]
x →1− x →1−

= lim[5(1 − h) − 4] = lim 5 − 5h − 4  = 5 − 4 = 1 [Put x = 1 – h]


h→0 h→0

Right hand limit = lim f(x) = lim (4x2 − 3x) [∵ f(x) = 4x2 – 3x, if 1 < x < 2]
x →1+ x →1+
2
= lim[4(1 + h) − 3(1 + h)] [Put x = 1 + h]
h→0

= lim[4 + 8h+ 4h2 − 3 − 3h] = lim[1 + 5h+ 4h2 ] = 1


h→0 h→0

∴ At x = 1, Left hand limit = Right hand limit ⇒ lim f(x) exists and it is equal to 1.
x →1

Illustration 8: Evaluate the left hand and right-hand limits of the following function at x = 1  (JEE MAIN)
1 + x2 , if 0 ≤ x ≤ 1
f(x) =  does lim f(x) exist ? 
 2 − x, if x > 1 x →1

Sol: By putting x = 1 – h and 1 + h, we can conclude left hand limit and right hand limit respectively. If both are
equal then the given limit exist otherwise not exist.
Left hand limit = lim f(x)
x →1−
= lim (1 + x2 ) [∵ f(x) = 1 + x2, if 0 ≤ x ≤ 1]
x →1−

= lim[1 + (1 − h)2 ] = 1 + 1 = 2 [Put x = 1 – h]


h→0

Right hand limit = lim f(x) = lim (2 − x) [∵ f(x) = 2 – x, if x > 1]


x →1+ x →1+

= lim[2 − (1 + h)] = 2 − 1 = 1 (Put x = 1 + h)


h→0

Therefore, At x = 1, Left hand limit ≠ Right hand limit ⇒ lim f(x) does not exist.
x →1

| x − 6 |
 , x≠6
Illustration 9: Evaluate the right hand limit of the function f(x) =  x − 6 at x = 6  (JEE MAIN)
 0, x = 6

Sol: Here Right hand limit of the given function f(x) at x = 6 is lim f(6 + h) .
h→0
Right hand limit of f(x) at x = 6 = lim f(x) , = lim f(6 + h) ,
x →6 + h→0

|6+h−6 | |h| h
= lim , = lim = lim , = lim 1 = 1
h→0 6 + h − 6 h→0 h h→0 h h→0
1 4 . 8 | Limits, Continuity and Differentiability

| x − 4 |
 , x≠4
Illustration 10: Evaluate the left hand limit of the function: f(x) =  x − 4 at x = 4  (JEE MAIN)
 0, x = 4

Sol: Here Left hand limit of the given function f(x) at x = 4 is lim f(4 − h) .
h→0

| 4 −h− 4 | | −h |
Left hand limit of f(x) at x = 4 = lim f(x) = =
lim f(4 − h) lim = lim
x→4 − h→ 0 h→ 0 4 −h− 4 h→ 0 −h
h
= lim =lim − 1 =−1
h→0 −h h→0

Illustration 11: Evaluate the left hand and right hand limits of the following function at x = 2:  (JEE MAIN)
2x + 3, if x ≤ 2
f(x) =  Does lim f(x) exist? 
 x + 5, if x > 2 x →2

Sol: Similar to illustration 7.


Left hand limit = lim f(x) = lim (2x + 3) (∵ f(x) = 2x + 3, if x ≤ 2)
x → 2− x → 2−

= lim[2(2 − h) + 3] (Put x = 2 – h) = 4 + 3 = 7
h→0

Right hand limit = lim f(x) = lim f(x + 5) (∵ f(x) = x + 5, if x > 2)


x → 2+ x → 2+

= lim(2 + h + 5) = 7 (put x = 2 + h)
h→0

Therefore, the left hand limit = right hand limit [at x = 2] ⇒ lim f(x) exists and it is equal to 7.
x →2

mx2 + n, x < 0

Illustration 12: For what integers m and n does lim f(x) exist, if f(x)= nx + m, 0 ≤ x ≤ 1  (JEE ADVANCED)
x →0
 2 x >1
nx + m,
Sol: As the given limit exist, therefore its Left hand limit must be equal to its Right hand limit.

lim f(x)
Limit at x = 0 ; = lim (mx2 + n) = lim m(0 − h)2 + n =
n
x →0 − x →0 −h h→0  

lim =
f(x) lim (nx +=
m) lim[n(0 + h) + m] = m
x →0 + x →(0 +h) h→0

Limit exists if lim=


f(x) lim f(x)=
⇒n m
x →0 − x →0 +

a + bx, x < 1

Illustration 13: Suppose
= f(x) = 4, x 1 and if lim f(x) = f(1) .What are possible values of a and b?
x →1
 b − ax, x > 1 (JEE Advanced)

Sol: Here =
lim f(x) lim
= f(x) lim f(x) .
x →1− x →1+ x →1

a + bx, x < 1

We have,
= f(x) = 4, x 1 Left hand lim= f(x) lim [a + bx] = lim[a + b(1 − h)] =a + b
x →1− x →1− h→0
b − ax, x > 1

M a them a ti cs | 14.9

Right hand lim


= f(x) lim [b − a x] = lim[b − a(1 + h)] =b − a
x →1+ x →1+ h→0

lim f(x)
= lim
= a 0,b
f(x) lim f(x) ⇒ a + b = b – a = 4 ⇒ = = 4
x →1− x →1+ x →1

 x− | x |
 , if x ≠ 0
Illustration 14: If f(x) =  x show that lim f(x) does not exist  (JEE ADVANCED)
 2, if x = 0 x →0

Sol: Here if left hand limit is not equal to the right hand limit of the given function then the lim f(x) does not exist.
x →0
(0 − h)− | 0 − h |
Left hand limit of f at x = 0 = lim f(x) = lim f(0 − h) =
lim [Putting x = 0 – h]
x →0 − h→0 h→0 (0 − h)

−h − h −2h
= lim = lim = lim = 2 2
h→0 −h h→0 −h h→0

(0 + h)− | 0 + h |
Right hand limit of f at x = 0 = lim f(x) = lim f(0 + h) =
lim [Putting x = 0 + h]
x →0 + h→0 h→0 (0 + h)

h− | h | h−h 0
= lim = lim = lim
= lim= 0 0
h→0 h h→0 h h→0 h h→0

Here, left hand limit of f (at x = 0) ≠ right hand limit of f(at x = 0). Therefore lim f(x) does not exist at x = 0
h→0

PLANCESS CONCEPTS

If f(x) denotes the greatest integer function then lim f(x)


= [0]
= 0 this representation is wrong.
x →0
The correct form is
lim f(x)
= lim [0 +=
h] 0
x →0 + h→0

lim f(x) =lim [0 − h] =−1


x →0 − h→0

Hence the limit doesn’t exist. Remember that the limit must be applied only after complete simplification.

Nitish Jhawar (JEE 2009, AIR 7)

1.6 Value of a Function at a Point and Limit at a Point

Case I: lim f(x) and f(a) both exist but are not equal.
x →a
 x2 − 1
 , x ≠1 x2 − 1
Example: f(x) =  x − 1 ; lim f(x)
= lim = lim(x + 1)
= 2 , f(x) exists at x = 1
x →1 x →1 x − 1 x →1
 0, x =1

f(1) = 0, value of f also exists at x = 1. But lim f(x) ≠ f(1) .
x →1

Case II: lim f(x) and f(a) both exist and are equal.
x →a
1 4 . 1 0 | Limits, Continuity and Differentiability

2
Example: f(x) = x2 ; lim
= f(x) lim(x
= ) 1 limit exists, and f(1) = (1)2 = 1; ⇒ value of f also exists. ⇒ lim f(x) = f(1)
x →1 x →1 x →1

1.7 Properties of Limits


Let f and g be two real functions with common domain D, then.

(a) lim(f + g)(x)= lim f(x) + lim g(x)


x →a x →a x →a

(b) lim(f − g)(x)= lim f(x) − lim g(x)


x →a x →a x →a

(c) lim(c ⋅ f)(x) =


c lim f(x) [c is a constant]
x →a x →a

(d) lim(fg)(x)
= lim f(x) ⋅ lim g(x)
x →a x →a x →a

f lim f(x)


(e) lim   (x) = x →a
x →a  g  lim g(x)
x →a

( )
n
lim ( f(x) ) = lim f(x)
n
(f)
x →a x →a

( )
lim g(x)
lim ( f(x) )
g(x)
(g) = lim f(x) x →a
x →a x →a

(h) If f(x)g(x) xa then lim f(x) ≤ lim g(x)


x →a x →a

PLANCESS CONCEPTS

lim f(x) =  , then lim | f(x) | = |  |


x →a x →a

The converse of this may not be true i.e.

lim | f(x) | =
|  |⇒
/ lim f(x) =

x →a x →a

lim f(x) = A > 0 and lim g(x) = B then lim f(x)g(x) = AB


x →a x →a x →a
Shivam Agarwal (JEE 2009, AIR 27)

1.8 Cancellation of Common Factor

f(x) f(x) 0
Let lim . If by substituting x = a, reduces to the form , then (x–a) is a common factor of f(x) and g(x).
x →a g(x) g(x) 0
So we first factorize f(x) and g(x) and then cancel out the common factor to evaluate the limit.

Working Rule:
f(x)
To find out lim , where lim f(x) = 0 and lim g(x) = 0 .
x →a g(x) x →a x →a

Step I: Factorize f(x) and g(x).

Step II: Cancel the common factor (s).


M a them a ti cs | 14.11

Step 3: Use the substitution method to obtain the limit.

Important formulae for factorization

(a) (a2 – b2) = (a – b) (a + b)


(b) a3 – b3 = (a – b) (a2 + ab + b2)
(c) a3 + b3 = (a + b) (a2 – ab + b2)
(d) a4 – b4 = (a2 – b2) (a2 + b2) = (a – b) (a + b) (a2 + b2)
(e) If f(α) = 0, then x – α is a factor of f(x)

x3 − 1
Illustration 15: Evaluate lim .  (JEE MAIN)
x →1 x − 1

Sol: Factorize the numerator and denominator.


x3 − 1 0
If we put x = 1, the expression assumes the indeterminate form . Therefore (x – 1) is a common factor of
x −1 0
(x3 – 1) and (x – 1). Factorising the numerator and denominator, we have,

x3 − 1  0 
lim  Form
x →1 x − 1  0 

(x − 1)(x2 + x + 1)
= lim = lim(x2 + x + 1) [After cancelling (x–1)] = 12 + 1 + 1 =3
x →1 (x − 1) x →1

4x2 − 1
Illustration 16: Evaluate lim  (JEE MAIN)
x → 2x − 1
1
2

Sol: Similar to above illustration, By factorizing numerator and denominator we can evaluate given limit.

1 4x2 − 1 0
If we put x = , the expression assumes the indeterminate from .
2 2x − 1 0

 1
Therefore  x −  i.e. (2x – 1) is a common factor of (4x2 – 1) and (2x – 1). Factorising the numerator and denominator,
we have,  2

4x2 − 1 0  (2x − 1)(2x + 1)


lim  Form = lim1 [∵ a2 – b2 = (a – b) (a + b)]
x → 2x − 1 0 (2x − 1)
1  x→
2 2

1
= lim(2x + 1) [After cancelling (2x – 1)]= 2 × +1 =1+1 = 2
x→
1 2
2

x2 − 9x + 20
Illustration 17: Evaluate lim .  (JEE MAIN)
x →5 x2 − 6x + 5

Sol: Take (x – 5) common from numerator and denominator to solve this problem.
x2 − 9x + 20 0
If we put x = 5, the expression assumes the indeterminate form .
2
x − 6x + 5 0

Therefore (x – 5) is a common factor of the numerator and denominator both. Factorising the numerator and
1 4 . 1 2 | Limits, Continuity and Differentiability

0  (x − 4) (x − 5) (x − 4)
denominator, we have  Form = lim = lim
0  x →5 (x − 1) (x − 5) x →5 (x − 1)

5−4 1
[After cancelling (x – 5)] = =
5 −1 4

x3 − 7x2 + 14x − 8
Illustration 18: Evaluate lim  (JEE MAIN)
x →2 x2 + 2x − 8

Sol: Same as above illustration.

x3 − 7x2 + 14x − 8 8 − 28 + 28 − 8 0
When x = 2 the expression assumes the form =
2
x + 2x − 8 4+ 4−8 0

Therefore (x – 2) is a common factor of the numerator and denominator.


Factorising the numerator and denominator, we get

(x − 1)(x − 2)(x − 4) (x − 1)(x − 4) (2 − 1)(2 − 4) 1 × ( −2) −2 1


lim = lim = = = = −
x →2 (x − 2)(x + 4) x → 2 (x + 4) (2 + 4) 6 6 3

x 4 − 3x3 + 2
Illustration 19: Evaluate lim  (JEE MAIN)
x →1 x3 − 5x2 + 3x + 1

Sol: Same as above illustration.

x 4 − 3x3 + 2 0
On putting x = 1 in , we get .
3 2
x − 5x + 3x + 1 0

It means (x – 1) is the common factor of the numerator and denominator. Factorising the numerator and denominator,
we get
(x − 1)(x3 − 2x2 − 2x − 2) (x3 − 2x2 − 2x − 2)
lim = lim [Cancellation of (x – 1)]
2
x →1 (x − 1)(x − 4x − 1) x →1 x2 − 4x − 1

13 − 2 × 12 − 2 × 1 − 2
= [Substitution method]
12 − 4 × 1 − 1

1 − 2 − 2 − 2 −5 5
= = =
1 − 4 −1 −4 4

Note: When the degree of the polynomial is higher, then it is difficult to factorize. So, we apply L’Hôpital’s rule

1.9 L’Hôpital’s Rule


L’Hôpital’s rule states that for functions f and g which are differentiable:

f '(x) f(x) f '(x)


If lim f(x) = lim g(x) = 0 or ±∞ and lim has a finite value then lim = lim
x →c x →c x →c g'(x) x →c g(x) x →c g'(x)

0 ∞
Note: Most common indeterminate forms are , ,0 x ∞ , ∞ − ∞ ,00 ,1∞ and ∞0
0 ∞
M a them a ti cs | 14.13

PLANCESS CONCEPTS

f(x)
Evaluation of limits using L’Hôpital’s rule is applicable only when becomes
g(x)
0 ∞
of the form or .
0 ∞
0 ∞ 0 ∞
If the form is not or simplify the given expression till it reduces to the form or and then
apply the rule. 0 ∞ 0 ∞

To apply L’Hôpital’s rule differentiate the numerator and the denominator separately.

What you may overlook is the fact that the LH rule is applicable only when the modified limit (obtained
by differentiating the numerator and denominator) also exists. Let’s consider an example to illustrate this
x2 + sinx ∞
point. Consider the limit lim . We see that the limit is of the indeterminate form . Applying
2 ∞
x
x →∞
x2 + sinx 2x + cos x ∞
the LH rule two times in succession, we obtain: lim = lim (again form)
x →∞ x 2 x →∞ 2x ∞
2 − sinx 1
= lim = 1 − lim sinx
x →∞ 2 2 x →∞

Which does not exist. However, only a few moments of consideration are required to conclude that the
limit must exist, because the numerator is x2 + sinx, and since x tends to infinity, the term sinx can be
ignored in comparison to x2 (as sinx only ranges from -1 to 1); the denominator is x2 and so the limit must
be 1. Why did the LH rule go wrong?
Ravi Vooda (JEE 2009, AIR 71)

1.10 Theorems
xn − an
 1. Let n be any positive integer. Then. lim = nan−1
x →a x − a

xn − an (a + h)n − an 1 
First Proof: Putting x = a + h, we get = = (a + h)n − an 
x−a a+h−a h 
1
= (an +n C1 an−1h + .... + hn ) − an  [Using the Binomial theorem]
h 

1 n
= [ C1 an–1 h + nC2 an–2 h2 + .... + hn] = nC1 an–1 + nC2 an–2 h + .... + hn–1
h
xn − an
∴ lim = lim  nC1an−1 + nC2an−2h + ....hn−1  = nan–1 (as nC1 = n)
x →a x − a h→0  

Second Proof: We know that,


xn − an
xn – an = (x – a) (xn–1 + axn–2 + ...an–2x + an–1) Therefore, =xn−1 + axn−2 + ....an−2 x + an−1
(x − a)
xn − an
⇒ lim =an−1 + a(an−2 ) + ....an−2a + an−1 = an–1 + an–1 + .... an–1 + an–1 (n terms) = n an–1
x →a (x − a)

xm − am
Illustration 20: Evaluate: lim  (JEE MAIN)
x →a xn − an
1 4 . 1 4 | Limits, Continuity and Differentiability

Sol: By using L’ Hospital rule, we can solve this problem.

xm − am
We have, lim
x →a xn − an

0   xm − am x − a   xm − am xn − an   xm − am   xn − an 
 form = xlim  ⋅ =  lim  ÷  = lim   ÷ lim  
0  →a  x − a x − a  x →a  x − a x − a  x →a  x − a  x →a  x − a 
n n

mam−1 m m−1−n+1 m m−n


= mam−1 ÷ nan−1 = = a = a
na n−1 n n

x5 − 32
Illustration 21: Evaluate: lim  (JEE MAIN)
x →2 x3 − 8

Sol: Dividing numerator and denominator by (x – 2) we can evaluate given limit.

x5 − 32 0
When x = 2, the expression assumes the indeterminate form .
3
x −8 0

x5 − 32 x5 − 25 (x5 − 25 ) / (x − 2)  x5 − 25   x3 − 23 
Now lim = lim = lim = lim   ÷ lim  
x →2 x3 − 8 x →2 x 3 − 23 x →2 (x3 − 23 ) / (x − 2) x →2  x − 2  x →2  x − 2 
   

 xn − an 
= 5 × 25−1 ÷ 3 × 23−1  lim = nan−1 
 x →a x − a 
5 × 245 20
= 5 × 24 ÷ 3 × 22 = = × 22 =
3× 2 32 3

xn − 2n
Illustration 22: Find all possible values of n, if lim = 80 , n ∈ N (JEE MAIN)
x →2 x − 2

Sol: Simply using L’Hospital rule we can obtain value of n.

xn − 2n  xn − an 
We have lim = 80 ⇒ n ⋅ 2n–1 = 80 ∴ lim nan−1 
=
x →2 x − 2
 x →a x − a 
⇒ n ⋅ 2n–1 = 5×25–1 ⇒ n = 5
5 5
(x + 2) 2 − (a + 2) 2
Illustration 23: lim  (JEE ADVANCED)
x →a x−a
Sol: Put x + 2 = y and a + 2 = b and after that solve this by using L’hospital rule.
Putting x + 2 = y and a + 2 = b, we get
5 5
5 3
y 2 − b2 5 −1 5  xn − an 
= lim = b 2 = b 2  lim = nan−1 
y →b y − b 2 2 y →a x − a
 
3
5
= (a + 2) 2
2

sinx
Illustration 24: Prove that lim =1
x →0 x
M a them a ti cs | 14.15

Sol: Proof by geometry


Draw a circle of radius unity and with its centre at O. Let ∠AOB = x radians
D


C
Join AB. Draw AC ⊥ OA at A. Produce OB to meet AC at C. Draw BD ⊥ OA
From the figure tan x
sin x
Area of DOAB < Area of sector OAB < Area of DOAC
1  x  1
⇒ (OA)(BD) <   π(OA)2 < (OA) ⋅ (AC)
x
A
2 2 π 2
B


  1

1 1 1  BD AC 
⇒ (OA)(OB)sinx < (OA)2 x < (OA) ⋅ (OA)tanx =
 sinx,tanx
= 
2 2 2  OB OA 

1 1 1
⇒ sinx < x < tanx  OA = OB 
2 2 2
sinx
⇒ sin x < x < tanx ⇒ sinx < x <
cos x
x 1
⇒ 1< < (Dividing by sin x)
sinx cos x
x 1
⇒ lies between 1 and
sinx cos x
When x → 0, cos x = 1
x
∴ When x → 0, lies between 1 and 1
sinx
x sinx
∴ lim = 1 or lim = 1 Proved
x →0 sinx x →0 x

Proof by algebra

lim
sinx
= lim 
 x − (x3 / 3!) + (x5 / 5!) − .... 
 = lim
( )
x 1 − (x2 / 3!) + (x 4 / 5!)..... 
= lim  1 −
x2 x 4 
+ .....  =
1
x →0 x x →0  x  x →0 x x →0  3! 5! 
  

sin2 5x
Illustration 25: Evaluate: lim  (JEE MAIN)
x →0 x2
sin θ
Sol: As we know lim = 1 , Therefore we can reduce given equation by multiplying and dividing by 25.
x →0 θ

sin2 5x sin5x sin5x  sin5x   sin5x 


We have, lim = lim ⋅ = lim 5   × lim 5  
x →0 x 2 x →0 x x x →0  5x  x →0  5x 

= 5(1) × 5(1) =
25

(a + x)2 sin(a + x) − a2 sina


Illustration 26: Evaluate: lim  (JEE ADVANCED)
x →0 x

Sol: By using algebra we can reduce given limit in the form of lim f(x) + lim g(x) + lim h(x) , and then by solving we
x →0 x →0 x →0
will get the result.

(a + x)2 sin(a + x) − a2 sina (a2 + 2ax + x2 )sin(a + x) − a2 sina


lim = lim
x →0 x x →0 x
1 4 . 1 6 | Limits, Continuity and Differentiability

a2 sin(a + x) + (2ax + x2 )sin(a + x) − a2 sina


= lim
x →0 x

a2 ( sin(a + x) − sina) 2ax sin(a + x) x2 sin(a + x)


= lim + lim + lim
x →0 x x →0 x x →0 x

2cos ( a + (x / 2) ) sin(x/ 2)
= a2 lim + lim 2asin(a + x) + lim x sin(a + x)
x →0 x x →0 x →0

 x sin(x/ 2)
= a2 lim cos  a +  lim + lim 2asin(a + x) + lim x sin(a + x)
x →0  2  x → 0 (x / 2) x →0 x →0

= a2 cos (a + 0) + 2a sin (a + 0) + 0 sin (a + 0)

= a2 cos a + 2 a sin a

cot 2x − cosec2x
Illustration 27: Evaluate: lim  (JEE ADVANCED)
x →0 x
Sol: By using trigonometric formulae, we can evaluate this problem.

We have, lim
cot 2x − cosec2x
= lim
((cos2x) / (sin2x)) − (1 / (sin2x)) = lim cos2x − 1
x →0 x x →0 x x →0 x ⋅ sin2x

−2sin2 x sinx sinx 1 lim 1


x →0
= lim = − lim − lim
= ⋅ lim = −1 ⋅ =−1
x →0 x ⋅ 2sinx ⋅ cos x x →0 x ⋅ cos x x →0 x x →0 cos x lim cos x
x →0
tanx − sinx
Illustration 28: Evaluate the following limits: lim  (JEE ADVANCED)
x →0 x3
Sol: Same as above problem.

We have, lim
tanx − sinx
= lim
((sinx) / (cos x)) − sinx = lim
sinx − sinx cos x
3 3
x →0 x x →0 x x →0 x3 cos x

sinx(1 − cos x) sinx (1 − cos x) 1 sinx 2sin2 (x / 2) 1


= lim =lim ⋅ ⋅ = lim ⋅ lim ⋅ lim ⋅
x →0 x3 ⋅ cos x x →0 x x 2 cos x x →0 x x →0 x 2 x →0 cos x

2
2  sin(x/ 2)  1  1  1  1
= 1 ⋅ lim   ⋅ xlim = (1)     =
x →0 4  (x / 2)  →0 cos x 2
   1 2

1 − cos x
Illustration 29: Evaluate: lim  (JEE ADVANCED)
x →0 x2
sinθ
Sol: Reduce given equation in the form of by using trigonometric formula.
θ
2
1 − cos x 2sin2 (x / 2) 2  sin(x/ 2)  1
We have, lim = lim = lim   =
x →0 x2 x →0 x2 4 x →0  (x / 2)  2

Illustration 30: Prove that: | sin x | ≤ | x |, holds for all x.  (JEE ADVANCED)

π
Sol: We know that sin x < x < tanx therefore by applying the cases, 0 ≤ x ≤ 1 < , −1 ≤ x ≤ 0 and |x| ≥ 1, we can
prove this illustration. 2
M a them a ti cs | 14.17

We know that sin x < x < tanx


π
If 0 ≤ x ≤ 1 < ⇒ |sin x| = sin x ≤ x ≤ |x|
2
If −1 ≤ x ≤ 0 ⇒ |sin x| = – sin x ⇒ sin (–x) ≤ –x ≤ |x|
If neither of the two cases hold, i.e. if |x| ≥ 1 then |sin x| ≤ 1 ≤ |x| ⇒ |sin x| ≤ |x|

1.11 Trigonometric Limits


Now, we will learn to evaluate the trigonometric limits when the variable tends to a non-zero number.
Woking Rule:
Let the variable tend to a. (x → a).
Step 1: Replace x by a + h, where h → 0.
Step 2: Now the problem is transformed in h where h → 0. Use the method already discussed in the previous
exercise.

sin( π − x)
Illustration 31: Evaluate: lim  (JEE MAIN)
x →π π( π − x)
Sol: By replacing x by π + h .
sin( π − x) 1 sin( π − x) 1 sinh 1  x → π 
lim
We have,= = lim lim =  
x →π π( π − x) π x →π ( π − x) π h→ 0 h π  ⇒ π − x → 0 

1 + cos x
Illustration 32: Evaluate: lim  (JEE MAIN)
x →π tan2 x
Sol: Simply replacing x by π + h and using trigonometric limit we can solve above problem.

1 + cos x 1 + cos( π + h)
lim = lim [Putting x = +h]
2
x →π tan x h →0 tan2 ( π + h)
1 − cosh 2sin2 (h / 2)
= lim [ tan ( + h) = tan h] = lim cos2 h

2 2
h →0 tan h h →0 sin h
2sin2 (h / 2) cos2 h 1 ×1 1
= lim = cos2 h lim
= =
h →0
( 2 2
4 sin (h / 2)cos (h / 2) h → 0
) 2
2cos (h / 2) 2 ×(1) 2

sinx − sina
Illustration 33: Evaluate: lim  (JEE ADVANCED)
x →a x− a
Sol: Same as above illustration replace x by a +h.

sinx − sina sin(a + h) − sina


We have, lim = lim (Putting x = a + h)
x →a x− a h→0 a+h − a

2cos ( a + (h / 2) ) sin(h / 2)  h  sin(h / 2)


= lim ( a + h + a) = lim 2cos  a +  [ a + h + a]
h→0 (a + h) − a h→0  2 h

= 2cos a lim 
h
 1 sin(h / 2) 
→0  2 (h / 2) 
 hlim
→0
( a+h + a )  h 
 h → 0 ⇒ → 0 
 2 
2

1 
= 2 cos a   (1)[ a + 0 + a] = 2 a cosa
2
1 4 . 1 8 | Limits, Continuity and Differentiability

1 + cos(2 x)
Illustration 34: Evaluate: lim  (JEE ADVANCED)
x→
π ( π − 2 x)2
2

π
Sol: Replace x by + h and then by using trigonometric formulae’s we can evaluate above problem.
2
1 + cos(2 x) 1 + cox2x 1 + cos2 ( ( π / 2) + h)  0   π 
We have, lim lim = lim  form  Put x= + h
0 2
2 2 2
x→
π ( π − 2 x)
x → ( π − 2x)
π h→0
 π − 2 ( ( π / 2) + h)    
2 2  
2
1 + cos( π + 2h) 1 − cos2h 0  2sin2 h 1  sinh  1 2 1
= lim = lim  form  = hlim = lim   = (1) =
h→0 ( π − π − 2h)2 h→0 4h2  0  → 0 4h 2 2 h→ 0  h  2 2

cos x − cosa
Illustration 35: Evaluate: lim  (JEE ADVANCED)
x →a cot x − cot a

Sol: By using trigonometric limit method we can solve this.

cos x − cosa cos(a + h) − cosa cos(a + h) − cosa


We have,
= lim lim
= lim [Put x = a+ h]
x →a cot x − cot a h→0 cot(a + h) − cot a h→0 ( (cos(a + h) / (sin(a + h) ) − ( (cosa) / (sina) )

2sin ( (a − a − h) / 2 ) sin ( (a + a + h) / 2 )
= lim sin(a + h)sina
h→0 cos(a + h)sina − cosa sin(a + h)

2sin ( −(h / 2) ) sin ( (2a + h) / 2 )


= lim sin(a + h)sina
h→0 sin(a − (a+ h))
h
−2sin sin ( (2a + h) / 2 ) 2sin(h/ 2)sin ( (2a + h) / 2 )
= lim 2 sin(a + h)sina = lim sin(a + h)sina
h→0 − sinh h→0 2sin(h/ 2)cos(h/ 2)

sin ( (2a + h) / 2 ) sin ( (2a + 0) / 2 )


= lim sin(a + h)sina = sin(a + 0)sina
h→0 cos(h/ 2) (1)

= sin a ⋅ sin a ⋅ sin a = (sin a)3 = sin3 a

2 − 3 cos x − sinx
Illustration 36: Evaluate: lim  (JEE ADVANCED)
x→
π (6x − π)2
6
π
Sol: Replace x by +h.
6

2 − 3 cos x − sinx 2 − 3 cos ( ( π / 6) + h) − sin ( ( π / 6) + h)  π 


We have, lim = lim Put x= + h
x→
π (6x − π) 2 h→0
6 ( ( π / 6) + h) − π 
2
 6 
6  

2 − 3 ( cos( π / 6)cosh− sin( π / 6)sinh) − sin( π / 6)cosh+ cos( π / 6)sinh


= lim
h→0 2
 π + 6h − π 

= lim
( ) (
2 − 3 ( 3 / 2)cosh− (1 / 2)sinh − (1 / 2)cosh+ ( 3 / 2)sinh )
h→0 36h2

2 − (3 / 2)cosh+ ( 3 / 2)sinh− (1 / 2)cosh− ( 3 / 2)sinh


= lim
h→0 36h2
M a them a ti cs | 14.19

2
2 − 2cosh 1 − cosh 2sin2 (h / 2) 1  sin(h/ 2) 
= lim
= = lim = lim = lim  
h→0 36h2 h→0 18h 2 h→0 18h 2 9 h→0  h 

2
1  sin(h/ 2)  1
=  hlim  =
9x4  → 0 (h / 2)  36

sinx − cos x
Illustration 37: Evaluate: lim  (JEE ADVANCED)
x→
π x − ( π / 4)
4
π
Sol: Replace x by + h.
4

sinx − cos x put x = ( π / 4) + h 


We have, lim  
x→
π x − ( π / 4)  As x → ( π / 4) ⇒ h → 0 
4

 cos ( ( π / 4) + h) 
sin ( ( π / 4) + h) − cos ( ( π / 4) + h)  
= lim = sin ( ( π / 4) + h + ( π / 2) ) 
h→0 ( π / 4) + h − ( π / 4)  
= sin (h + (3π / 4) ) 

sin ( ( π / 4) + h) − sin (h + (3π / 4) ) 2cos (h + ( π / 2) ) sin ( −( π / 4) )


= lim = lim
h→0 h h→0 h

( − sinh) sin ( −( π / 4) )  π  sinh  π 1


= lim = lim 2sin   = 2sin (1) =×
2 =2
h→0 h h→0 4 h  4 2

Note: All of the above illustrations can be solved using L’Hôpital’s rule.

1.12 Infinite Functions


Now, we will discuss the evaluation of the limits of two functions
(a) Exponential function (b) Logarithmic function

(a) Exponential function


1 1 1 1 1
Consider the series 1 + + + + + ..... + + .....∞
1! 2! 3! 4! n!
This infinite series is denoted by e. The domain of a function f(x) = ex, x ∈ R is R and the range is the set of positive
real numbers

(b) Logarithmic function


Let ey = x then it can be written as loge x = y
The domain of f(x) = loge x is R⊕ and the range is R.
The graph of the logarithmic of a function is in the adjoining figure.
Some Important Functions
 n(n − 1)x2 n(n − 1)(n − 2) 3 
(i) (1 + x)n = 1 + nx + + x + ....  [| x | < 1]
 2! 3! 
1 4 . 2 0 | Limits, Continuity and Differentiability

x 2 x3 xn
(ii) ex = 1 + x + + + ..... + ......
2! 3! 3!
x2
(iii) ax =
1 + x(loga) + (loga)2 + ..........
2!
x 2 x3 x 4
(iv) log(1 + x) =x − + − + ........ (|x|<1)
2 3 4
x 2 x3 x 4
(v) log(1 − x) =−x − − − − ......
2 3 4
x3 x5 x7
(vi) sinx =x − + − + ......
3! 5! 7!
x2 x 4 x6
(vii) cos x =1 − + − + ......
2! 4! 6!

x3 2x5
(viii) tanx =x + + + ......
3 15
x2
(ix) ax = 1 + x(loge a) + (loge a)2 + ........
2!
1 x3 1 3 x5
(x) sin –1 x = x + + + ....( −1 < x < 1)
2 3 24 5
x3 x5
(xi) tan−1 x =x − + ..... (–1 < x < 1)
3 5

PLANCESS CONCEPTS

Using these expansions is helpful where the limits are in the indeterminate form. But selecting the
number of terms to use in the expansion varies with problems.

Vaibhav Gupta (JEE 2009, AIR 54)

ex − 1
Theorem 1: Prove that lim =1
x →0 x
Proof: We know that

x x 2 x3 x 4 x x 2 x3 x 4
ex =1 + + + + + .... ⇒ ex − 1 = + + + + ....
1! 2! 3! 4! 1! 2! 3! 4!

ex − 1 x x 2 x3 ex − 1  x x 2 x3 
⇒ =1 + + + + .... ⇒ lim = lim  1 + + + + ....  = 1
x 2! 3! 4! x →0 x x →0  2! 3! 4! 
 
Hence proved.
log(1 + x)
Theorem 2: Prove that lim =1
x →0 x
loge (1 + x)
Proof: Let = y . then,
x
exy − 1 exy − 1
⇒ loge(1 + x) = xy ⇒ 1 + x = exy ⇒ exy –1 = x ⇒ =1 ⇒ ⋅y =
1
x xy
M a them a ti cs | 14.21
Now taking limit, when x → 0

exy − 1
lim ⋅ lim y =
1 [since x → 0 ⇒ xy → 0]
xy →0 xy x →0

 exy − 1 
1  lim
⇒ 1 ⋅ lim y = = 1
x →0

xy →0 xy 
loge (1 + x)
⇒ lim y = 1 ⇒ lim =1
x →0 xy →0 x

Hence proved

Note: If no base of log is mentioned, then it is taken for granted that the base is e.
∴ log a is same as loge a

ex − x − 1
Illustration 38: Evaluate: lim  (JEE MAIN)
x →0 x

ex − 1 ex − 1
Sol: As we already prove that lim = 1 , Therefore by writing given limit as lim − 1 we can easily solve
above problem.
x →0 x x →0 x

ex − x − 1 ex − 1
lim = lim −1 = 1 −1 = 0
x →0 x x →0 x

ax − bx
Illustration 39: Evaluate: lim  (JEE MAIN)
x →0 x

Sol: Add and subtract 1 with numerator.

ax − bx (ax − 1) − (bx − 1)
lim = lim
x →0 x x →0 x
 ax − 1 bx − 1  ax − 1 bx − 1 a
= lim  − = lim − lim = loga − logb =
log
x →0  
 x x  x → 0 x x → 0 x b

ex − e− x
Illustration 40: Evaluate: lim  (JEE ADVANCED)
x →0 x
ex − 1
Sol: Reduce given limit in the form of lim .
x →0 x
1
ex −
ex − e− x ex = lim e
2x
−1 1 e2x − 1 1
lim = lim lim = 2 lim = 2(1) = 2
x →0 x x →0 x x →0 x x →0 e x x →0 2x e0

esin x − 1
Illustration 41: Evaluate lim  (JEE ADVANCED)
x →0 x

Sol: By multiplying and dividing by sinx .

esin x − 1 esin x − 1 sinx


lim
= lim ×
x →0 x x →0 sinx x

esin x − 1 sinx esin x − 1


= lim =× lim lim = ×1 1
x →0 sinx x →0 x sin x →0 sinx
1 4 . 2 2 | Limits, Continuity and Differentiability

PLANCESS CONCEPTS

lim f(x) = 1 and lim g(x) = ∞ then


x →a x →a

g(x) lim g(x) f(x)−1


lim f(x) = e x →a
x →a 

lim an = 0, if |a| < 1 Does not exist if |a| ≥ 1.


n→∞

A common mistake made by students pertains to indeterminate limits. Consider a function f(x) = g(x)
h(x). We are given that lim g(x) = 0 . What is the value of lim f(x) ? Many students would say that it
x →0 x →0
is 0. However, the actual answer depends on lim h(x) = 0 . If it is not finite, then the limit of f(x) is
x →0
indeterminate. The point we are trying to make is that in calculating the limit of a function which is the
product of two or more functions, if one of the function tends to 0, then that does not make it necessary
for the entire limit to be 0 as well. Similar, remarks hold for other indeterminate forms. For example, if f(x)
= g(x)h(x), and g(x) → 1 as x → a, it does not necessarily imply that f(x) → 1 as x → a, because if lim h(x)
x →a
is not finite, then the limit on f(x) is indeterminate.

Akshat Kharaya (JEE 2009, AIR 235)

2. CONTINUITY
2.1 Introduction
The word ‘continuous’ means without any break or gap. If the graph of a function has no break or gap or jump, then
it is said to be continuous. A function which is not continuous is called a discontinuous function.
Ex.
(i) (ii) (iii) (iv)
y y y y
y = [x]
y=x
x x x x
0 -2 -1 12 3
y = sin x f(x) = 1/x
(Continuous function) (Continuous function) (Discontinuous at x = 0) (Discontinuous) At Every integer

Figure 14.6

Following are examples of some continuous functions:


(i) f(x) = x (Identity function)

(ii) f(x) = c (Constant function)

(iii) f(x) = a0xn + a1xn–1 + ....... + an (Polynomial function)

(iv) f(x) = sin x, cos x (Trigonometric function)

(v) f(x) = ax, ex, e–x (Exponential function)


M a them a ti cs | 14.23

(vi) f(x) = log x (Logarithmic function)

(vii) f(x) = sinh x, coshx, tanhx (Hyperbolic function)

(viii) f(x) = |x|, x + |x|, x – |x|, x |x| (Absolute value functions)

Following are example of some discontinuous functions:


No. Functions Points of discontinuity

(i) [x] Every Integer

(ii) x – [x] Every Integer


1
(iii) x = 0
x
π 3π
(iv) tan x, sec x x= ± , ± . ...........
2 2
(v) cot x, cosec x x= 0, ± π, ± 2π....
1 1
(vi) sin ,cos x=0
x x
(vii) e1/x x = 0

(viii) coth x, cosech x x=0

2.2 Continuity of a Function at a Point


A function f(x) is said to be continuous at a point x = a, if
(a) (i) f(a) exists
(b) (ii) lim f(x) exists and is finite
x →a

So lim f(x) = lim f(x)


x →a+ x →a−
(c) (iii) lim f(x) = f(a) or
x →a

Function f(x) is continuous at x = a

If lim
= f(x) lim
= f(x) f(a)
x →a+ x →a−

i.e. If right hand limit at ‘a’ = left hand limit at ‘a’ = value of the function at ‘a’.

If lim f(x) does not exist or lim f(x) ≠ f(a), then f(x) is said to be discontinuous at x = a
x →a x →a

2.3 Continuity from Left and Right


Function f(x) is said to be

(i) Left continuous at x = a if, lim f(x) = f(a) i.e. f(a−) = f(a)
x →a−

(ii) Right continuous at x = a if, lim f(x) = f(a) i.e. f(a+) = f(a)
x →a+

Thus, a function f(x) is continuous at a point x = a, if it is left continuous as well as right continuous at x = a.
1 4 . 2 4 | Limits, Continuity and Differentiability


 x + a 2 sinx 0≤x<π/4

 π π
Illustration =
42: f(x)  2x cot x + b ≤x≤ is continuous in [0, p]. Find a and b. (JEE MAIN)
 4 2
 π
acos2x − bsinx 2
<x≤π

π π
Sol: By checking left continuous and right continuous for x = and x = we can obtain value of a and b.
4 2
π π  π−  π π π π
f  = +b; f =  +a ⇒ + a = +b ⇒ a−b =
4 2 4  4 4 2 4
 

π  π+  π π
f  = b ; f  =−a − b ⇒ a − b =b ⇒ 2b =−a ⇒ a = , b = −
2 2  6 12
 

2x2 + 2, x ≤ 2
Illustration 43: Examine the continuity of the function f(x) =  , at the point x = 2 (JEE MAIN)
 2x, x>0

Sol: By obtaining Left hand limit and right hand limit we will get to know that given function is continuous or not.

f(2) = 22 + 2 = 6  ... (i)


L.H.L. f(2= ) lim((2 − h)2 +=
1) 5  ... (ii)
h→0

+
R.H.L. f(2= ) lim 2(2 +=
h) 4  ... (iii)
h→0

f(2 – 0) ≠ f(2 + 0) ≠ f(2)


∴ f(x) is not continuous at x = 2

 x + λ, x < 3

Illustration 44:
= If f(x) = 4, x 3 , is continuous at x = 3, then the value of λ is (JEE MAIN)
3x − 5, x > 3

Sol: As given function is continuous at x = 3, hence its left hand limit will be equal to its right hand limit f(x) is
continuous at x = 3

f(3)
∴= lim
= f(x) lim f(x) ⇒ 4 = lim((3 − h) + λ ) = 3 + λ ⇒ λ = 1
x →3− x →3+ h→0

2.4 Continuity of a Function in an Interval


(a) A function f(x) is said to be continuous in an open interval (a, b), if it is continuous at every point in (a, b). For
example, function y = sin x, y = cos x, y = ex are continuous in (–∞, ∞)
(b) A function f(x) is said to be continuous in the closed interval [a, b], if it is:
(i) Continuous at every point of the open interval (a, b)
(ii) Right continuous at x = a, i.e. RHL|x=a = f(a)
(iii) Left continuous at x = b, i.e. LHL|x=b = f(b)
M a them a ti cs | 14.25

2.5 Reasons of Discontinuity


Y
(a) Limit does not exist i.e. lim f(x) ≠ lim f(x) 2
x →a− x →a+
(b) f(x) is not defined at x = a 1

(c) lim f(x) ≠ f(a)


x →a
-2 -1 0 1 2
Geometrically, the graph of the function will exhibit a break at x = a, if the
function is discontinuous at x = a. The graph as shown is discontinuous at x = -1
1, 2 and 3.
-2
Y’
3 DIFFERENTIABILITY Figure 14.6

3.1 Meaning of a Derivative


The instantaneous rate of change of a function with respect to the dependent variable is called the derivative. Let ‘f’
be a given function of one variable and let Dx denote a number (positive or negative) to be added to the number
x. Let Df denote the corresponding change of ‘f ‘, then
∆f f(x + ∆x) − f(x)
Df = f(x + Dx) – f(x) ⇒ =
∆x ∆x
∆f
If approaches a finite value as Dx approaches zero, this limit is the derivative of ‘f’ at the point x. The derivative
∆x
of a function ‘f’ is denoted by symbols such as
df d dy df ∆f f(x + ∆x) − f(x)
f '(x), , (f(x)) or if y = f(x) by or y ' =
⇒ lim
= lim
dx dx dx dx ∆x →0∆x ∆x →0 ∆x
The process of finding derivative of a function is called differentiation. We also use the phrase differentiate f(x) with
respect to x which means to find f ‘(x).

PLANCESS CONCEPTS

The fact that f(x) is differentiable at x = a means that the graph is smooth as x moves from a– to a to a+.
Derivative of an even function is always an odd function.
Anvit Tawar (JEE 2009, AIR 9)

3.2 Existence of Derivative at x = a

(a) Right hand derivative:


The right hand derivative of f(x) at x = a, denoted by f ‘ (a+) is defined as:
f(a + h) − f(a)
f '(a+ ) = lim , provided the limit exists & is finite. (h>0)
h→0 h

(b) Left hand derivative:


The left hand derivative of f(x) at x = a, denoted by f ‘ (a–) is defined as:
f(a − h) − f(a)
f '(a− ) = lim , provided the limit exists & is finite. (h > 0)
h→0 −h
Hence f(x) is said to be derivable or differentiable at x = a if
1 4 . 2 6 | Limits, Continuity and Differentiability

f’(a+) = f’(a–) = finite quantity and it is denoted by f’(a);


Right hand and left hand derivative at x = a is also denoted by Rf’(a) and Lf’(a) respectively.

PLANCESS CONCEPTS

• If a function is not differentiable but is continuous at a point, it geometrically implies there is a sharp
corner or a kink at that point.
• If a function is differentiable at a point, then it is also continuous at that point.
• If a function is continuous at point x = a, then nothing can be guaranteed about the differentiability
of that function at that point.
• If a function f(x) is not differentiable at x = a, then it may or may not be continuous at x = a
• If a function f(x) is not continuous at x = a, then it is not differentiable at x = a
• If the left hand derivative and the right hand derivative of f(x) at x = a are finite (they may or may not
be equal), then f(x) is continuous at x = a.

Chinmay S Purandare (JEE 2012, AIR 698)

 ax + b x ≤ −1
Illustration 45: f(x) =  3 is differentiable for xÎR find ‘a’ & ‘b  (JEE ADVANCED)
ax + x + 2b x > −1

Sol: By equating left hand limit to its right hand limit we can obtain ‘a’ & ‘b.

f( −1 − h) − f( −1) (a( −1 − h) + b) − (b − a) −ah


f '( −1− ) =
lim = lim = lim =a
h→0 −h h→0 −h h→0 −h

f( −1 + h) − f( −1) (a(h− 1)3 + (h − 1) + 2b) − (b − a)



= f '( −1+ ) lim = lim
h→0 h h→0 h


h→0 
(
= lim  a h2 − 3h + 3 + 1 + )
( − a− 1 + b + a) 

You might also like

pFad - Phonifier reborn

Pfad - The Proxy pFad of © 2024 Garber Painting. All rights reserved.

Note: This service is not intended for secure transactions such as banking, social media, email, or purchasing. Use at your own risk. We assume no liability whatsoever for broken pages.


Alternative Proxies:

Alternative Proxy

pFad Proxy

pFad v3 Proxy

pFad v4 Proxy